You are on page 1of 1362

Krok 2 Medicine 2005 1

1. The physician must undertake 6. Define the basic registration document


measures for primary prophylaxis of iron at the profound study of a case rate wi-
deficiency anemia. Which of the following th temporary lost labor ability at the
categories of patient are subject to such industrial enterprise:
primary prophylactic measures?
A. A card of the personal account of a case
A. Pregnant women rate
B. Patients after 60 B. "The Report on reasons of a temporary
C. All children lost labor ability"
D. Patients after operation C. The sick-leave certificate
E. Workers of industrial enterprises D. A ambulatory medical card
E. The inpatient medical record
2. A patient with unstable angi-
na pectoris was given the following 7. A patient, aged 25, complains of pain
complex treatment: anticoagulants, ni- in the I finger on the right hand. On
trates, /alpha-adrenoblockers. However examination: the finger is homogeneously
on the third day of treatment the pain sti- hydropic, in bent position. On attempt to
ll romains. Which in vestigation shoud be unbend the finger the pain gets worse.
carried out to establish diagnosis? Acute pain appears on touching with
the probe in ligament projection. What
A. Coronarography decease is the most likely?
B. Stress-echocardiogram
C. Test with dosed physical exercises A. Thecal whitlow (ligament panaritium)
D. Esophageal electrocardiac stimulator B. Subcutaneous panaritium
E. Myocardial scintigraphy C. Articular (joint) panaritium
D. Bone panaritium
3. The 28 y.o. woman applied to doctor E. Paronychia
because of limited loss of the hair.
In the anamnesis - she had frequent 8. A sample of milk was taken for testi-
headache indisposition, arthromyalgia, ng from a 5 ton milk batch. Lab analysis
fever, irregular casual sexual life, drug showed the following: fat content 2%,
user. RW is negative. What examination specific density- 1,04 g/cm3 , acidity 210 C,
must be done first? reductase probe – weak positive. What
way the product is to be used in? What
A. Examination for HIV would you advise?
B. Examination for neuropathology
C. Examination for gonorrhea A. Sell but inform customers about milk
D. Examination for fungi quality
E. Examination for trichomoniasis B. Write the product off for animal feeding
C. Utilize technically
4. What methods of the collecting of the D. Sell without limitations
information is preferable for study of E. Annihilate the product
housing conditions of students of medical
HIGH SCHOOL for a training period? 9. Patient with thyrotoxicosis is in the
2-beds hospital ward of therapeutic
A. Questioning department. The area of the ward is 18
B. Interviewing
C. Selecting of materials m2 , height 3 m, ventilation rate 2,5/hr.
D. A method of the directed selection Air temperature - 200 , relative humidity
E. Statistical 45%, air movement velocity 0,3 m/s, light
coefficient 1/5, noise level 30 dB. Make
5. Choose a method of a graphic hygienic evaluation of the conditions.
representation of monthly information
about number of the registered cases A. Discomfortable microclimate
of acute intestinal infection and their B. Non-effective ventilation
comparisons to the average monthly C. Poor lighting
values, obtained for 5 previous years: D. High level of noise
E. All conditions are OK
A. The linear diagram
B. The radial diagram 10. 33 y.o. woman works as the secretary.
C. The sector diagram Her diet contains 150 g of protein (includi-
D. The figured diagram ng 100 g of animal protein), 200 g of fat,
E. The curvilinear disgram 600 g of carbohydrates. What pathology
Krok 2 Medicine 2005 2

can develop from this diet? dened, deformed, location of segments


ST and of wave T is discordant. The wave
A. Obesity Р is not changed, superimposes QRST,
B. Schizophrenia natural conformity between Р and QRS is
C. Paradontosis not present. What kind of arrhythmia is
D. Common cold present?
E. Uterine fibromyoma
A. Paroxismal ventricular tachycardia
11. A 9 y.o. girl has an average height and B. Sinus tachycardia
harmonic growth development. She was C. Atrial flutter
ill with acute respiratory infection for five D. Ventricular extrasystole
times. Define the group of her health. E. Atrial tachycardia
A. 2nd group 16. Ambulance was called to a 48 y.o.
B. 1st group man. From the words of relatives he has
C. 3rd group had three episodes of lost consciousness
D. 4th group and attacks during the day. On exami-
E. 5th group nation: the following fit is observed: pati-
ent is unconscious, fell on the floor, tonic
12. The child is 11 m.o. He suffers from and then clonic convuesions of trunk and
nervous-arthritic diathesis. The increased extremities happened. The attack lasted
synthesis of what acid is pathogenic at for 4 minute, ended by involuntary uri-
nervous-arthritic diathesis? nation. What type of attack was observed?
A. Uric acid A. Major epileptic seizure
B. Acetic acid B. Vegetatic crisis
C. Phosphoric acid C. Absence
D. Hydrochloric acid D. Episode of hysteria
E. Sulfuric acid E. Fainting
13. A 35 y.o. woman was admitted to 17. A 2 y.o. girl has been ill for 3
thoracic surgery department with fever up days. Today she has low–grade fever,
to 400 C, onset of pain in the side caused by severe catarrhal symptoms, non-abundant
deep breathing, cough with considerable maculopapular rash on her buttocks and
quantity of purulent sputum and blood enlarged occipital glands. What is your di-
with bad smell. What is the most likely di- agnosis?
agnosis?
A. Rubella
A. Abscess of the lung B. Scarlet fever
B. Complication of liver echinococcosis C. Measles
C. Bronchiectatic disease D. Adenoviral infection
D. Actinomycosis of lungs E. Pseudotuberculosis
E. Pulmonary tuberculosis
18. Male 30 y.o., noted growing fingers and
14. A 52 y.o. man has recurrent transi- facial scull, changed face. Complains of
ent ischemic attacks. Auscultation of the poor eyesight, weakness, skin darkeni-
carotid arteries detects murmur. What di- ng, loss of body weight. X-ray shows
agnostic method is necessary to apply fi- broadening of sella turcica, thinning of
rst? tuberculin sphenoidale, signs of increased
intracranial pressure. What diagnosis can
A. Ultrasound dopplerography you make?
B. CTof the brain
C. MRI of the brain A. Adenoma of hypophysis
D. Cerebral angiography B. Encephalitis of truncus
E. Electroencephalography C. Optico - hiasmatic arachnoiditis
D. Adrenal gland tumor
15. A 67 y.o. patient complains of palpi-
E. Tumor of pondo-cerebellar corner
tation, dizziness, noise in ears, feeling of
shortage of air. Objectively: pale, damp 19. A patient complains of a tormental
skin. Vesicular respiration, respiratory (agonizing) cough with expectorati-
rate- 22 per min, pulse- 200 bpm, AP- on of up to 600 ml/daily purulent
100/70 mm Hg. On ECG: heart rate- chocolatecolor sputum with a decay smell.
200 bmp, ventricular complexes are wi-
Onset of illness was abrupt, t0 - 390 C, fever
Krok 2 Medicine 2005 3

of irregular type. There is the area of A. Subacute bacteria endocarditis


darkening with a cavity in a center on X- B. Recurrence of rheumatic fever
ray film, with irregular contours and level C. Thrombocytopenia purpure
of liquid. What disease is the question? D. Mitral stenosis
E. Aortic stenosis
A. Gangrene of lung
B. Tuberculosis 23. A patient with nosocomial pneumonia
C. Bronchiectatic illness has signs of collapse. Which of the followi-
D. Pneumonia complicated by an abscess ng pneumonia complication is the most li-
E. Lobar pneumonia kely to be accompanied with collapse?
20. A 24 y.o. patient complains of nausea, A. Septic shock
vomiting, headache, shortness of breath. B. Exudative pleuritis
He had an acute nephritis being 10 C. Bronchial obstruction
y.o. Proteinuria was found out in uri- D. Toxic hepatitis
ne. Objectively: a skin is grey-pale, the E. Emphysema
edema is not present. Accent of II tone
above aorta. BP 140/100-180/100 mm Hg. 24. A 27 y.o. man complains of pain in epi-
Blood level of residual N2 - 6,6 mmol/L, gastrium which is relieved by food intake.
creatinine- 406 mmol/L. Day’s diuresis- EGDFS shows antral erosive gastritis,
2300 ml, nocturia. Specific density of uri- biopsy of antral mucous presents Hеli-
ne is 1009, albumin- 0,9 g/L, WBC- 0-2 in cobacter Pylori. What can be diagnosed in
f/vis. RBC.- single in f/vis., hyaline casts this case?
single in specimen. Your diagnosis? A. Gastritis of type B
A. Chronic nephritis with violation of B. Gastritis of A type
kidney function C. Reflux - gastritis
B. Feochromocitoma D. Menetrier’s disease
C. Hypertensive illness of the II degree E. Rigid antral gastritis
D. Nephrotic syndrome 25. A 62 y.o. patient with DM-2. Diabetes
E. Stenosis of kidney artery is being compensated by diet and Manini-
21. A 33 y.o. male patient was admi- lum. Patient has to undergo an operation
tted to a hospital. A patient is pale, at for inguinal hernia. What the tactics of
an attempt to stand up he complains of hypoglycemic therapy should be used?
strong dizziness. There was vomiting like A. Prescribe fast-acting insulin
coffee-grounds approximately hour ago. B. Give Glurenorm in place of Maninilum
BP- 90/60 mm Hg., pulse- 120 b/min. In C. Continue with the current therapy
anamnesis, a patient has suffered from D. Prescribe long-acting insulin
ulcer of the stomach, painless form during E. Prescribe guanyl guanidines
4 years. An ulcer was exposed at gastrofi-
beroscopy. Your diagnosis: 26. A 32 y.o. patient complains of severe
weakness, tremor of extremities. On
A. Ulcer of stomach, complicated with physical examination, there is loss of body
bleeding weight, wet and warm skin. The thyroid
B. Ulcer of duodenum, complicated with gland is enlarged up to the 3rd degree,
bleeding painless, elastic. Ps- 108 bpm. BP- 160/55
C. Erosive gastritis mm Hg. The rest is in norm. What can be
D. Acute pleurisy diagnosed?
E. Acute myocardial infarction, abdominal
form A. Diffuse toxic goitre of the 3rd degree,
thyrotoxicosis of the average degree
22. A 40 y.o. patient of rheumatic heart B. Diffuse euthyroid goitre of the 3rd
disease complains of anorexia, weakness degree
and loss of weigth, breathless and swelling C. Chronic autoimmune thyroiditis,
of feet. On examination: t0 - 390 C, pulse is hypertrophic type
100/min. As ucultation: diastolic murmur D. Chronic fibrous thyroiditis
in the mitral area. Petechical lesion a E. Toxiferous adenoma of the thyroid
round clavicle; spleen was palpable, tooth gland
extraction one month ago.
27. A 30 y.o. female with rheumatoid
arthritis of five years duration complains
Krok 2 Medicine 2005 4

of pain in the frist three fingers of her right


hand over past 6 weeks. The pain seems A. Leptospirosis
especially severe at night often awakening B. Yersiniosis
her from sleep.The most likelly cause is? C. Salmonellosis
D. Brucellosis
A. Carpal tunnel syndrome E. Trichinellosis
B. Atlanto-axial sublaxation of cervical
spine 32. A 2,9-kg term male infant is born to
C. Sensory peripheral neuropathy a mother who developed polyhydramnios
D. Rheumatoid vasculitis at 34 weeks’ gestation. At birth, the Apgar
E. Rheumatoid arthritis without compli- scores were 9 and 9. The infant develops
cation choking and cyanosis with the first feed. In
addition, is unable to place a nasogastric
28. A 38 y.o. patient was urgently admi- tube. What is the most likely diagnosis?
tted to the hospital with complaints of
sudden weakness, dizziness, loss of consci- A. Esophageal atresia
ousness, body weight loss, nausea, vomi- B. Choanal atresia
ting, severe pain in epigastric area, di- C. Laryngomalacia
arrhea, skin hyperpigmentation. What is D. Tracheal atresia
the most probable diagnosis? E. Respiratory distress syndrome
A. Addisonic crisis 33. Full term newborn has developed
B. Acute gastroenteritis jaundice at 10 hours of age. Hemolytic
C. Meningoencephalitis disease of newborn due to Rh-
D. Scleroderma incompatibility was diagnosed. 2 hours
E. Pellagra later the infant has indirect serum bili-
rubin level increasing up to 14 mmol/L.
29. An unconscious patient presents wi- What is most appropriate for treatment of
th moist skin, shallow breathing. There hyperbilirubinemia in this infant?
are signs of previous injection on the
shoulders and hips. BP- 110/70 mm Hg. A. Exchange blood transfusion
Tonus of skeletal muscles and reflexes B. Phototherapy
are increased. Cramps of muscles of the C. Phenobarbital
extremities are seen. What is the most li- D. Intestinal sorbents
kely disorder? E. Infusion therapy
A. Hypoglycemic coma 34. At term of a gestation of 40 weeks
B. Hyperglycemic coma height of standing of a uterine fundus is
C. Hyperosmolar coma less then assumed for the given term. The
D. Hyperlactacidotic coma woman has given birth to the child in wei-
E. Stroke ght of 2500 g, a length of a body 53 cm,
with an assessment on a scale of Apgar of
30. 7 y.o. boy with chronic sinusitis and 4-6 points. Labor were fast. The cause of
rercurent pulmonary infections has chest such state of the child were:
X-ray demonstrating a right-sided cardi-
ac silhouette. What is the most likely di- A. Chronic fetoplacental insufficiency
agnosis? B. Delay of an intra-uterine fetation
C. Placental detachment
A. Kartagener syndrome D. Infection of a fetus
B. Cystic fibrosis (mucoviscidosis) E. Prematurity
C. Bronchiolitis obliterans
D. Laryngotracheomalacia 35. A 42 y.o. man died in a road acci-
E. α-antitrypsin deficiency dent after the haemorrhage on the site,
due to acute hemorrhagic anaemia. What
31. A patient was admitted to the hospital minimum percentage volume of the total
on the 7th day of the disease with complai- blood loss would result in death at acute
nts of high temperature, headache, pain haemorrhage?
in the muscles, especially in calf muscles.
Dermal integuments and scleras are A. 25-30%
icteric. There is hemorrhagic rash on the B. 6-9%
skin. Urine is bloody. The patient was fi- C. 10-14%
shing two weeks ago. What is the most D. 15-20%
likely diagnosis? E. 35-50%
Krok 2 Medicine 2005 5

36. What preparations are used for treatment is indicated to the patient?
prevention of fungal infection?
A. Surgical drainage of the pleural cavity
A. Fluconozol, Orungol, Nisoral B. Antiinflammation therapy
B. Rubomycin, Bleomycin, Mytomycin C C. Symptomatic therapy
C. Cytosar, Cormyctin, Lomycitin D. Pleural puncture
D. Captopril, Enalapril E. Thoracotomy
E. Isoniazid, Ftibazid, Pyrazinamid
41. A youth, aged 15, from childhood
37. What developes most often after acci- suffers from atopic dermatitis and allergy
dental intake of Hydrochloric acid? to the shellfish. In the last 3 months
after acquiring aquarium fish, rhini-
A. Cardiac insufficiency tis, conjunctivitis, itching in the nose
B. Cushing’s syndrome developed. What level of immunologic
C. Kutling’s syndrome index should be defined in this case?
D. Deylads’s syndrome
E. Acute pancreatitis A. IgE
B. IgJ
38. A woman, aged 40, primigravida, with C. IgM
infertility in the medical history, on the D. IgA
42-43 week of pregnancy. Labour acti- E. Circulating immunocomplexes
vity is weak. Longitudinal presentation
of the fetus, I position, anterior positi- 42. The patient has developed pain in
on. The head of the fetus is engaged to the axillary area, rise of temperature
pelvic inlet. Fetus heart rate is 140 bmp, developed 10 hours ago. On examinati-
rhythmic, muffled. Cervix dilation is 4 cm. on: shaky gait is marked, the tongue is
On amnioscopy: greenish colour of amni- coated by white coating. The pulse is
otic fluid and fetal membranes. Cranial frequent. The painful lymphatic nodules
bones are dense, cranial sutures and small are determined in the axillary area. The
fontanel are skin is erythematous and glistering over
the lymphatic nodules. What is the most
A. Caesarean section probable diagnosis?
B. Amniotomy, labour stimulation, fetal
hypoxia treatment A. Bubonic plague
C. Fetal hypoxia treatment, in the II period B. Acute purulent lymphadenitis
- forceps delivery C. Lymphogranulomatosis
D. Fetal hypoxia treatment, conservative D. Anthrax
delivery E. Tularemia
E. Medication sleep, amniotomy, labour
stimulation 43. A 17 y.o. patient complains of acute
pain in the knee joint and t0 – 380 C. He
39. 30 y.o. woman, had mild trauma of was ill with angina 3 weeks ago. Objecti-
5th finger of the left hand 15 days ago. vely: deformation and swelling of the
She has treated her self at home. She knee joints with skin hyperemia. Small
presents to the hospital due to deteriorati- movement causes an acute pain in the joi-
on of the condition and temperature rise. nts. Which diagnose is the most correct?
Objectively: hyperemia and swelling on
the ventral surface of finger. Restricted A. Rheumatism, polyarthritis
movements of the finger. X-ray of the left B. Systemic lupus eritematodes
hand: an early stage of оsteomyolitis of C. Reactive polyarthritis
the fifth finger could not be excluded. The D. Infectious-allergic polyarthritis
diagnosis: panaris of 5th finger of the left E. Rheumarthritis
hand. What form of panaris has occurred
in the patient? 44. A 38 y.o. woman was hospitalized
to the surgical unit with vomiting and
A. Bony acute abdominal pain irradiating to the
B. Hypodermic spine. On laparocentesis hemmorhagic
C. Paronychia fluid is obtained. What disease should be
D. Tendon type suspected?
E. Joints type
40. A 36 y.o. patient is diagnosed with ri-
ght sided pneumothorax. What method of
Krok 2 Medicine 2005 6

A. Acute pancreatitis
B. Renal colic A. Acute purulent maxillary sinusitis
C. Acute enterocolitis B. Acute purulent frontitis
D. Perforated gastric ulcer C. Acute purulent ethmoiditis
E. Acute appendicitis D. Acute purulent sphenoiditis
E. -
45. The girl is 12 y.o. Yesterday she was
overcooled. Now she complains on pain 49. A woman, primagravida, consults
in suprapubic area, frequent painful uri- a gynecologist on 05.03.2002. A week
nation by small portions, temperature is ago she felt the fetus movements for
37, 80 C. Pasternatsky symptom is negati- the first time. Last menstruation was on
ve. Urinalysis: protein- 0,033 g/L, WBC– 10.01.2002. When should she be given
20-25 in f/vis, RBC– 1-2 in f/vis. What di- maternity leave?
agnosis is most probable?
A. 8 August
A. Acute cystitis B. 25 July
B. Dysmetabolic nephropathy C. 22 August
C. Acute glomerulonephritis D. 11 July
D. Acute pyelonephritis E. 5 September
E. Urolithiasis
50. An infant aged 1 year on the third day
46. The girl of 11 y.o. She is ill for 1 of common cold at night developed inspi-
month. She has "butterflytype rash on ratory stridor, hoarse voice and barki-
face (spots and papules), pain and swelli- ng cough. Physical examination revealed
ng of small joints on arms and legs, signs of suprasternal and intercostal chest retracti-
stomatitis (small-sized ulcers in mouth). ons. There is a bluish skin discoloration
CBC: Нb– 80 g/L, RBC– 2, 9 ∗ 1012 /L, moistly seen over the upper lip. The respi-
ratory rate is 52 per min and pulse- 122
WBC– 15 ∗ 109 /L, ESR- 40 mm/hour. Uri-
nalysis: protein– 0,33 g/L. What is the bpm. The body temperature is 37, 50 C.
most probable diagnosis? What disease does the infant have?

A. Systemic lupus erythematosus A. Acute infectious croup due to viral


B. Juvenile rheumatoid arthritis, systemic laryngotracheitis
type B. Acute laryngitis
C. Periarteriitis nodosa C. Bronchopneumonia without complicati-
D. Acute rheumatic fever ons
E. Dermatomyositis D. Acute bronchiolitis with respiratory
distress
47. The 56 y.o. patient has worked at the E. Acute epiglottitis
aluminium plant more than 20 years. Wi-
thin 3 last years he has developed looseni- 51. A newborn aged 3 days with hyperbi-
ng of teeth, bone and joint pains, piercing lirubinemia (428 mkmol/L) developed
pains in heart area, vomiting. The prelimi- following disorders. From beginning
nary diagnosis is: there were severe jaundice with poor
suckling, hypotomia and hypodynamia.
A. Fluorine intoxication Little bit later periodical excitation,
B. Mercury intoxication neonatal convulsions and neonatal primi-
C. Lead intoxication tive reflexes loss are noted. Now physi-
D. Phosphorus intoxication cal examination reveals convergent squi-
E. Manganese intoxication nt, rotatory nystagmus and setting sun eye
sign. How to explain this condition?
48. A 38 y.o. woman complains of a
purulent discharge from the left nostril. A. Encephalopathy due to hyperbilirubi-
The body temperature is 37, 50 C. The pati- nemia
ent is ill during a week and associates her B. Skull injury
illness with common cold. Pain on palpati- C. Brain tumour
on of her left cheek reveals tenderness. D. Hydrocephalus
The mucous membrane in the left nasal E. Spastic cerebral palsy
cavity is red and turgescent. The purulent 52. A 52 y.o. hard smoker patient complai-
exudates is seen in the middle meatus in ns of persistent cough with purulent
maxillary. What is the most probable di- sputum discharge especially in mornings,
agnosis? dyspnea provoked even by slight physi-
Krok 2 Medicine 2005 7

cal exertion, wheezing chest, tahypnoe, lower extremities X-Ray shows the thi-
general weakness. He considers himself ckening of flat bones. In the long bones
to be ill during 12 years. The overwri- there is a hyperostosis along the bone
tten conditions appear 3-4 times per year axis.The blood test has not revealed any
usually after common cold and have inflammation activity. Serum calcium is
tendency to progress. What disease do you normal. What disease do you consider in
think about first of all? this case?
A. Chronic obstructive lung disease A. Paget’s disease
B. Bronchial asthma B. Hyperparathyoid dystrophy
C. Mucoviscidosis (cystic fibrosis) C. Chronic osteomyelitis
D. Bronchoectatic disease D. Myeloma
E. Aspergillosis E. Mottled disease (marble disease)
53. 3 weeks ago the patient was ill wi- 57. The woman who has delivered twi-
th tonsillitis. Clinical examination reveals ns has early postnatal hypotonic uterine
edema, arterial hypertension, hematuria, bleeding reached 1,5% of her bodywei-
proteinuria (1,8 g/per day), granular and ght. The bleeding is going on. Conservati-
erythrocital casts. What is the preliminary ve methods to arrest the bleeding have
diagnosis? been found ineffective. The conditions of
patient are pale skin, acrocyanosis, oli-
A. Glomerulonephritis guria. The woman is confused. The pulse
B. Cystitis is 130 bpm, BP– 75/50 mm Hg. What is the
C. Pyelonephritis further treatment?
D. Intestinal nephritis
E. Renal amyloidosis A. Uterine extirpation
B. Supravaginal uterine amputation
54. 47 y.o. patient complains of intensi- C. Uterine vessels ligation
ve skin itching, jaundice, bone pain. The D. Inner glomal artery ligation
skin is hyperpigmentated. There is multi- E. Putting clamps on the uterine cervix
ple xanthelasma palpebrae. The liver is
+6 cm enlarged, hard with acute edge. 58. A 26 y.o. woman complains of a mild
The blood analysis revealed total bili- bloody discharge from the vagina and pain
rubin 160 mkmol/L, direct – 110 mkmol/L, in the lower abdomen. She has had the last
AST (asparate aminotransferase)- 2,1 menstruation 3,5 months ago. The pulse is
mmol/L per hour, ALT– 1,8 mmol/L, 80 bpm. The blood pressure (BP) is 110/60
alkaline phosphotase- 4,6 mmol/L per mm Hg and body temperature is 36, 60 C.
hour, cholesterol– 9,2 mmol/L, antimi- The abdomen is tender in the lower parts.
tochondrial antibodies M2 in a high titer. The uterus is enlarged up to 12 weeks of
What is the probable diagnosis? gestation. What is your diagnosis?
A. Primary biliary liver cirrhosis A. Inevitable abortion
B. Primary liver cancer B. Incipient abortion
C. Chronic viral hepatitis B C. Incomplete abortion
D. Acute viral hepatitis B D. Complete abortion
E. Alcoholic liver cirrhosis E. Disfunctional bleeding
55. The complications of acute cholecysti- 59. 18 y.o. woman complains of pain in
tis which require surgical intervention are the lower abdomen. Some minutes before
as follows EXCEPT: she has suddenly appeared unconscious at
home. The patient had no menses within
A. Jaundice last 3 months. On examination: pale skin,
B. Empyema of the gall-bladder the pulse- 110 bpm, BP- 80/60 mm Hg.
C. Emphysematous gall-bladder The Schyotkin’s sign is positive. Hb- 76
D. Gall-bladder perforation g/L. The vaginal examination: the uterus
E. Cholangitis conditioned by the presence is a little bit enlarged, its displacement is
of stones in the bile tract painful. There is also any lateral swelling
56. The 67 y.o. patient had 5 recurrent of indistinct size. The posterior fornix of
fractures of the lower extremities without the vagina is tendern and overhangs insi-
considerable cause within 5 years. O- de. What is the most probable diagnosis?
shaped deformity of the legs in the knee
joints has appeared. The skull, pelvis and
Krok 2 Medicine 2005 8

A. Impaired extrauterine pregnancy through out. What disease should a doctor


B. Ovarian apoplexy consider first of all?
C. Twist of cystoma of right uterine adnexa
D. Acute salpingoophoritis A. Carbuncle
E. Acute appendicitis B. Furuncle
C. Acute skin cellulitis
60. A 20 y.o. pregnant woman with 36 D. Carbuncle associated with anthrax
weeks of gestation was admitted to the E. Skin abscess
obstetrical hospital with complains of pain
in the lower abdomen and bloody vagi- 63. A 19 y.o. man was admitted to the
nal discharge. The general condition of reception department in 20 minutes after
the patient is good. Her blood pressure being wounded with the knife to the left
is 120/80 mm Hg. The heart rate of the chest. The patient is confused. The heart
fetus is 140 bpm, rhythmic. Vaginal exami- rate is 96 bpm and BP- 80/60 mm Hg.
nation: the cervix of the uterus is formed There are the dilated neck veins, sharply
and closed. The discharge from vagina is diminished apical beat and evident heart
bloody up to 200 ml per day. The head of enlargement What kind of penetrative
the fetus is located high above the minor chest wound complications has developed
pelvis entry. A soft formation was defined in patient?
through the anterior fornix of the vagina.
What is the probable diagnosis? A. Pericardium tamponade
B. Massive hemothorax
A. Placental presentation C. Open pneumothorax
B. Premature placental separation D. Closed pneumothorax
C. Uterine rupture E. Valve-likes pneumothorax
D. Threatened premature labor
E. Incipient abortion 64. A 35 y.o. patient complains of a
difficult swallowing, pain behind the
61. In the gynecologic office a 28 y.o. breastbone. He can eat only liquid food.
woman complains of sterility within three While swallowing sometimes he has
years. The menstrual function is not attacks of cough and dyspnea. Above
impaired. There were one artificial aborti- mentioned complaints are progressing. It
on and chronic salpingo-oophoritis in her is known that the patient has had a chemi-
case history. Oral contraceptives were not cal burn of esophagus one month ago.
used. Her husband’s analysis of semen is What complication does the patient have?
without pathology. From what diagnostic
method will you start the workup in this A. Corrosive esophagitis and stricture
case of sterility? B. Esophagitis
C. Esophageal diverticula
A. Hysterosalpingography D. Cardiac achalasia
B. Hormone investigation E. Cardiac insufficiency
C. Ultra sound investigation
D. Diagnostic scraping out of the uterine 65. An employee of a private company
cavity was ill with acute respiratory viral infecti-
E. Hysteroscopia on. Consulted a district doctor, who
determined the fact of temporary loss of
62. A 33 y.o. patient was admitted to the working ability, but refused to issue a sick-
reception room of the Central District list, arguing that the patient worked in
Hospital. He complains of a severely pai- the private and not state-owned company.
nful swelling localized on posterior neck, Should the sick-list be issued to the
fever up to 38, 40 C and general weakness. employees of private companies?
In anamnesis: diabetes mellitus within 5
years. On physical examination on the A. Issued regardless of company’s
posterior neck surface there is an infiltrate ownership
elevated above surrounding skin. The ti- B. Issued only to empties of state-owned
ssues affected by swelling are tense and companies
blue reddish discoloration in central area. C. Issued only on condition of payment
There are also several purulent necrotic guarantee by the company’s proprietor
pustules which are connected with each D. Issued a medical certificate of a set form
other and form a large skin necrosis. A E. Issued a medical certificate of a free
thinned necrotic skin of this swelling has form
holes looking like sieve, pus discharges 66. A worker was temporarily off work
Krok 2 Medicine 2005 9

because of illness during 16 days, was squeering substernal pain which had
under out-patient treatment. The doctor appeared 2 hours ago and irradiated
in charge issued a sick-list first for 5 days, to the left shoulder, marked weakness.
then prolonged it for 10 days. Who can On examination: pale skin, cold sweat.
further prolong the sick-list to this pati- Pulse- 108 bpm, AP- 70/50 mm Hg, heart
ent? sound are deaf, vesicular breathing, soft
abdomen, painless, varicouse vein on the
A. The doctor in charge of the case left shin, ECG: synus rhythm, heart rate is
together with the head of department 100 bmp, ST-segment is sharply elevated
B. Deputy head physician on the working in II, III aVF leads. What is the most li-
ability expertise kely disorder?
C. The doctor in charge of the case with
the permission of the head of department A. Cardiogenic shock
D. Working ability expertise committee B. Cardiac asthma
E. The head of department C. Pulmonary artery thromboembolia
D. Disquamative aortic aneurizm
67. A 13 y.o. patient was treated E. Cardiac tamponade
in dermatological hospital for atopic
dermatitis exacerbation. He was di- 71. In treatment and prevention establi-
scharged in the condition of clinical remi- shments, regardless of their organisati-
ssion. What recommendations should the onal and proprietary form, the rights of
doctor give to prevent exacerbations? the patients should be observed. Which of
these rights is the most significant?
A. Use of neutral creams to protect skin
B. Frequent skin washing with detergents A. The right to the protection of the
C. Systematic use of local corticosteroids patient’s interests
D. Systematic skin disinfection B. The right to the free choice
E. Avoidance of skin insolation C. The right to the information
D. The right to be heard
68. A full-term new-born suffered ante- E. The right to the protection from
and intranatal hypoxia, was born in incompetence
asphyxia (Apgar score 2-5 points). After
birth baby’s excitation is progressing, 72. A military unit stopped for 3-day’s rest
occurs vomiting, nystagmus, spasms, squi- in inhabited locality after a long march.
nt, spontaneous Babinski and Moro’s The sanitary-epidemiological reconnai-
reflexes. What is the most probable locati- ssance found several water sources. It is
on of the intracranial haemorrhage in this necessary to choose the source complyi-
case? ng with the hygienic standards for potable
water in the field
A. Subarachnoid hemorrhages
B. Small hemorrhages in brain tissue A. Artesian well water
C. Subdural hemorrhages B. Spring water
D. Periventricular hemorrhages C. River water
E. Haemorrhages in ventricles of brain D. Rain water
E. Water from melted snow
69. A patient, aged 16, complains of
headache, mainly in the frontal and 73. The district pediatrician is charged wi-
temporal areas, superciliary arch, appeari- th the analysis of infant mortality. What is
ng of vomiting at the peak of headache, taken for the unit of observation in infant
pain during the eyeballs movement, joint’s mortality investigation?
pain. On examination: excited, t0 - 390 ,
Ps- 110/min. Tonic and clonus cramps. A. A baby dead at the age up to 12 months
Uncertain meningeal signs. What is the B. A baby dead at the age up to 1 months
most likely diagnosis? C. A baby dead at the age over 28 days
D. A baby dead at the age up to 6 days
A. Influenza with cerebral edema mani- E. A baby dead at birth
festations
B. Influenza, typical disease duration 74. Chief district pediatrician has to carry
C. Respiratory syncytial virus out analysis of infant mortality rate. What
D. Parainfluenza should he take as a unit of the observati-
E. Adenovirus infection on?

70. A 64 y.o. patient has developed of


Krok 2 Medicine 2005 10

A. Child death case at the age up to 1 year out a study of morbidity rate for populati-
B. Child death case at the age up to the on which had been served at the polycli-
first month nics for the last 5 years. What statistical
C. Child death case after 28 days of life values can help in calculations of diseases
D. Child death case during first 7 days of level dissamination?
life
E. Child death case on labor A. Relative values
B. Standart values
75. A 43 y.o. patient complains of mass C. Average values
and, pain in the right breast, elevation D. Absolute values
of temperature to 37, 20 C during 3 last E. Dynamic row
months. Condition worsens before the
menstruation. On examination: edema 79. A 37 y.o. woman is suffering from
of the right breast, hyperemia, retracted squeezing substernal pain on physical
nipple. Unclear painful infiltration is exertion. On examination: AP- 130/80
palpated in the lower quadrants. What is mm Hg, heart rate=pulse rate 72 bpm,
the most probable diagnosis? heart boarders are dilated to the left side,
aortic systolic murmur. ECG- signs of the
A. Cancer of right mammary gland left venticle hypertrophy. What method
B. Right side acute mastitis of examination is the most informative in
C. Right side chronic mastitis this case?
D. Premenstrual syndrome
E. Tuberculosis of right mammary gland A. Echocardiography
B. Phonocardiography
76. A patient, aged 40, has been ill duri- C. Coronarography
ng approximately 8 years, complains of D. Sphygmography
pain in the lumbar part of the spine E. X-ray
on physical excertion, in cervical and
thoracal part (especially when coughi- 80. A child, aged 4, has being ill for 5 days,
ng), pain in the hip and knee joints on suffers from cough, skin rash, t0 - 38, 20,
the right. On examination: the body is fi- facial hydropy, photosensitivity, conjuncti-
xed in the forward inclination with head vitis. On the face, neck, upper part of the
down, gluteal muscles atrophy. Spine chest there is bright maculopapular rash
roentgenography: ribs osteoporosis, longi- with areas of merging. Hyperemic throat.
tudinal ligament ossification. What is the Seropurulent nasal discharge. In lungs
most likely diagnosis? there are dry crackles. What is the most
probable preliminary diagnosis?
A. Ancylosing spondyloarthritis
B. Tuberculous spondylitis A. Measles
C. Psoriatic spondyloarthropatia B. Adenovirus infection
D. Spondyloarthropatia on the background C. Scarlet fever
of Reiter’s disease D. Rubella
E. Spread osteochondrosis of the vertebral E. Enterovirus exanthema
column
81. There were registered 500 cases of
77. A worker, aged 38, working in the urolithiasis per 10000 inhabitants. What
slate production during 15 years, complai- kind of statictical indices is presented?
ns of expiratory exertional dyspnea, dry
cough. On examination: deafening of the A. Prevalence rate
percutory sounds in interscapular regi- B. Correlation coefficient
on, rough breath sounds, dry dissemi- C. Index of visualization
nated rales. On fingers’ skin - greyish D. Incidence rate
warts. Factory’s sectorial doctor suspects E. Index of compliance
asbestosis. Which method is the most 82. At year-end hospital administration
informative for diagnosis verification? has obtained the following data: annual
A. Thorax roentgenography number of treated patients and average
B. Bronchoscopy annual number of patient-used beds.
C. Spirography What index of hospital work can be
D. Bronchoalveolar lavage calculated based upon this data?
E. Blood gases examination
78. Deputy of chief medical officer carried
Krok 2 Medicine 2005 11

A. Bed turnover A. Right shin osteomielitis


B. Bed resources of the hospital B. Right shin tuberculosis
C. Average annual bed occupacy C. Right shin syphilis
D. Average duration of patients presence D. Bone cyst of tibia
in the hospital E. Right shin trauma
E. Average bed idle time
87. A 40 y.o. patient was admitted to
83. A 52 y.o. patient fell from 3 m hight on the gasteroenterology with skin itchi-
the ground with the right lumbar area. ng, jaundice, discomfort in the right
He complains of pain here. There is mi- subcostal area, generalized weakness. On
crohematuria in the urea. On urography examination: skin is jaundice, traces of
kidney’s functioning is satisfactory. What scratches, liver is +5 cm, splin is 6x8
is the most probable diagnosis? cm. In blood: alkaline phosphatase - 2,0
mmol/(hour*L), general bilirubin - 60
A. Kidney’s contusion mkmol/L, cholesterol - 8,0 mmol/L. What
B. Subcapsular kidney’s rupture is the leading syndrome in the patient?
C. Multile kidney’s ruptures
D. Paranephral hematoma A. Сholestatic
E. Kidney’s abruption B. Сytolytic
C. Mesenchymal inflammatory
84. A 3 y.o. child with weight deffici- D. Asthenic
ency suffers from permanent moist cough. E. Liver-cells insufficiency
There was a history of some pneumoni-
as with obstruction. On examination: di- 88. A 43 y.o. patient had cholecystectomy
stended chest, dullness on percussion over 6 years ago due to chronic calculous
the lower parts of lungs. On auscultation: cholecystitis. Lately he has suffered from
a great number of different moist rales. pain in the right subcostal area and
Level of sweat chloride is 80 mol/L. What recurrent jaundice. During last 2 weeks
is the most probable diagnosis? jaundice hasn’t gone. Stenoutic papillitis
0,5 cm in length has been revealed. What
A. Mucoviscidosis (cystic fibrosis) is the best way of treatment?
B. Bronchial asthma
C. Recurrent bronchitis A. To perform endocsopic papillosphi-
D. Bronchiectasis ncterotomy
E. Pulmonary hypoplasia B. To treat conservatively: antibiotics,
spasmolytics, antiinflammatory drugs
85. A 14 y.o. girl complains of profuse C. To perform external choledoch drain
bloody genital discharges during 10 D. To perform choledochoduodenostomy
days after suppresion of menses for 1,5 E. -
months. Similiar bleedings recur since 12
years on the background of disordered 89. A 12 y.o. child with acute
menstrual cycle. On rectal examination: glomerulonephritis presented with
no pathology on the internal genitalia. In hypertensive syndrom during first days of
blood: Нb– 70 g/L, RBC- 2, 3 ∗ 1012/L, Ht– the disease. What is the role of angiotesin
20. What is the most probable diagnosis? II in the pathogenesis?
A. Juvenale bleeding, posthemorrhagic A. Intensifies production and secretion of
anemia aldosterone
B. Werlholf’s disease B. Increases heart output
C. Polycyst ovarian syndrome C. Infibits deppresive action of
D. Hormonoproductive ovary tumor prostaglandins
E. Noncomplete spontaneous abortion D. Increases erythropoetin production
E. Increases renine level
86. A 14 y.o. patient suddenly fell ill when
high fever, acute pain in the right shin. 90. On physiologic-sanitary examinati-
In two weeks X-ray showed translucent on of railway department work it was
spaces (destructive focuses) with unevel revealed that loaders work is of III degree
countours in the middle third of tibia di- of difficulty. They unload vagons with
aphysis. Along the bone edge there was a sand, manually break coagulated mass by
narrow line of shadow (periostitis) 1-2 mm shovel and shift it. What criteria was used
from the surface. What is the most likely to evaluate work of loaders?
diagnosis?
Krok 2 Medicine 2005 12

A. Maximun load weigh which is shifted


B. Value of static loading for the shift A. To place suture on the uterus cervix
C. Time of active activities, % to the shift B. To administer tocolytic therapy
duration C. To interrupt pregnancy
D. Time of passive observation, % to the D. To administer hormonal treatment
shift duration E. To perform amniocentesis
E. Intellectual efforts
95. A district pediarician has carried out
91. A 51 y.o. woman complains of dull pain infant mortality rate analysis in his area.
in the right subcostal area and epigastric What data has been used?
area, nausea, appetite decline during 6
months. There is a history of gastric peptic A. Mortality of children under 1 y.o.
ulcer. On examination: weight loss, pulse structured by age, sex, causes
is 70 bpm, AP is 120/70 mm Hg. Diffuse B. Mortality of children under 1 y.o.,
tenderness and resistance of muscles on natimortality
palpation.There is a hard lymphatic node C. Hospital mortality of children,
1x1cm in size over the left clavicle. What structured by age
method of investigation will be the most D. Mortality of district adolescents
useful? E. Mortality of district newborn

A. Esophagogastroduodenoscopy with 96. A 26 y.o. male patient with


biopsy postoperative hypothyroidism take
B. Ultrasound examination of abdomen thyroxine 100 mg 2 times a day. He has
C. pH-metry developed tachycardia, sweating, irritabi-
D. Ureatic test lity, sleep disorder. Determine further
E. Stomach X-ray treatment tactics.

92. A man, aged 68, complains of ti- A. To decrease thyroxine dosage


redness, sweating, enlargement of cervi- B. To increase thyroxine dosage
cal, submaxillary and axillary lymph C. To administer betablockers
nodes. Blood tests: WBC - 35 ∗ D. To add mercasolil to the treatment
109 /L, lymphocytes - 60%, Botkin and E. To administer sedatives
Gumprecht bodies, level of haemoglobin
and quantity of thrombocytes is normal. 97. Body temperature of a 12 y.o. girl
Myelogram showed 40% of lymphocytes. increased up to 39−400 C in 4-5 hours after
What is the most probable diagnosis? she had taken 2 pills of aspirin. Complains
of general discomfort, dizziness, sudden
A. Chronic lympholeucosis appearance of red spots on the skin wi-
B. Chronic myeloleucosis th blister formation or exfoliation of the
C. Lymphogranulomatosis epidermis with erosive surface. Lesions
D. Acute leucosis on the skin looked like burns of II degree.
E. Tuberculous lymphadenitis Nikolsky syndrome is positive. What is the
most probable diagnosis?
93. Examination of placenta, which has
just been born, reveals defect 2x3 cm in A. Acute epidermical necrolysis
size. There is no bleeding. What tactics is B. Pemphigus vulgaris
the most reasonable? C. Polymorphic exudative erythema
D. Bullous dermatitis
A. Manual uretus cavity revision E. Duhring’s disease
B. Administration of uterotonic medicines
C. External uterus massage 98. A patient complains of pathological
D. Parturient supervision lump, appearing in the right inguinal regi-
E. Instrumental uterus cavity revision on on exercise. The lump is round-shaped,
4 cm in diameter, on palpation: soft elastic
94. A 27 y.o. gravida with 17 weeks of consistency, positions near the medial part
gestation was admitted to the hospital. of Poupart’s ligament. The lump is si-
There was a history of 2 spontaneous tuated inwards from the spermatic cord.
miscarriages. On bimanual examination: What is the most probable preliminary di-
uterus is enlarged to 17 weeks of gestati- agnosis?
on, uterus cervix is shortened, isthmus
allows to pass the finger tip. The diagnosis
is isthmico-cervical insufficiency. What is
the doctor’s tactics?
Krok 2 Medicine 2005 13

A. Right-sided direct inguinal hernia A. Anovular menstrual cycle


B. Right-sided oblique inguinal hernia B. Chronic adnexitis
C. Right-sided femoral hernia C. Abnormalities in genital development
D. Varicose veins of the right hip D. Immunologic infertility
E. Lipoma of the right inguinal area E. Genital endometriosis
99. A 52 y.o. woman complains of 103. A 43 y.o. woman complains of contact
weakness, painful itching after washing hemorrhages during the last 6 months.
and bathing, sensation of heaviness in the Bimanual exam: cervix of the uterus is
head. On examination: hyperemia of skin enlarged, restricted in mobility. Mirrors
of face, neck, extremities. АP- 180/100 showed the following: cervix of the uterus
mm Hg. Speeln is 4 cm below the rib is in the form of cauliflower. Chrobak and
arch edge. What is the most probable di- Schiller tests are positive. What is the most
agnosis? probable diagnosis?
A. Erythremia A. Cancer of cervix of the uterus
B. Essential hypertension B. Polypus of the cervis of the uterus
C. Dermatomyositis C. Cervical pregnancy
D. Allergic dermatitis D. Nascent fibroid
E. Systemic sclerodermia E. Leukoplakia
100. A 55 y.o. patient complains of 104. A local doctor has to prepare a
distended abdomen and rumbling, report about the health condition of the
increased winds evacuation, liguid foamy population of his region. What medical
feces with sour smell following the diary indexes of population health condition
products consumption. What is the correct should he use?
name of this syndrome?
A. Morbidity, disabilities, demographic,
A. Syndrome of fermentative dyspepsia physical development
B. Syndrome of decayed dyspepsia B. Social welfare, satisfaction of life quality
C. Syndrome of fatty dyspepsia C. Way of life, genetic, pollution
D. Dyskinesia syndrome D. Average treatment duration, compli-
E. Malabsorption syndrome cations
E. Average longevity
101. In an inhabited locality there is an
increase of diphtheria during the last 3 105. A boy, aged 9, is examined: height-
years with separate outbursts in families. 127 cm (-0,36), weight - 28,2 kg (+0,96),
What measure can effectively influence chest circumference- 64,9 cm (+0,66), lung
the epidemic process of diphtheria and vital capacity - 1520 ml (-0,16). What is the
reduce the morbidity rate to single cases? integrated assessment of the child’s physi-
cal development?
A. Immunization of the population
B. Hospitalization of patients A. Harmonious
C. Detection of carriers B. Disharmonious
D. Early diagnostics C. Markedly disharmonious
E. Disinfection in disease focus D. Excessive
E. Below the average
102. A 27 y.o. woman turns to the materni-
ty welfare centre because of infertility. 106. A 14 y.o. child suffers from of
She has had sexual life in marriage for 4 vegetovascular dystonia of pubertal peri-
years, doesn’t use contraceptives. She di- od. He has developed sympathoadrenal
dn’t get pregnant. On examination: geni- crisis. What medicine should be used for
tal development is without pathology, crisis reduction?
uterus tubes are passable, basal (rectal)
temperature is one-phase during last 3 A. Obsidan
menstrual cycles. What is the infertility B. No-shpa
cause? C. Amisyl
D. Euphyline
E. Corglicone
107. A woman complains of high
temperature to 380 C, mild pain in the
throat during 3 days. On examinati-
Krok 2 Medicine 2005 14

on: angle lymphatic nodes of the jaw 111. A 18 y.o. male patient complains of
are 3 cm enlarged, palatinel tonsils are pain in knee and ankle joints, temperature
enlarged and coated with grey plaque whi- elevation to 39, 50 C. He had a respiratory
ch spreads to the uvula and frontal palati- disease 1,5 week ago. On examination:
nel arches. What is the most probable di- temperature- 38, 50C, swollen knee and
agnosis? ankle joints, pulse- 106 bpm, rhythmic,
AP- 90/60 mm Hg, heart borders wi-
A. Larynx dyphtheria thout changes, sounds are weakened, soft
B. Infectious mononucleosis systolic apical murmur. What indicator is
C. Vincent’s angina connected with possible etiology of the
D. Agranulocytosis process?
E. Oropharyngeal candidosis
A. Antistreptolysine-0
108. A 36 y.o. patient was admitted to B. 1-antitrypsine
the hospital with sharp pain in substernal C. Creatinkinase
area following occasional swallowing D. Rheumatic factor
of a fish bone. On esophagoscopy the E. Seromucoid
foreign body wasn’t revealed. The pain
increased and localized between scapulas. 112. A 19 y.o. patient was admitted to the
In a day temperature elevated, conditi- hospital with acute destructive appendici-
on became worse, dysphagia intensified. tis. He sufferes from hemophilia B-type.
What complication has developed? What antihemophilic medicine should
be inclended in pre-and post-operative
A. Perforation of esophagus with mediasti- treatment plan?
nitis
B. Esophageal hemorrhage A. Fresh frosen plasma
C. Obstruction of esophagus B. Cryoprecipitate
D. Pulmonary atelectasis C. Fresh frosen blood
E. Aspirative pneumonia D. Native plasma
E. Dried plasma
109. A child from the first non-
complicated pregnancy but complicated 113. A 24 y.o. male patient was transferred
labor had cephalhematoma. On the to the chest surgery department from
second day there developed jaundice. general surgical department with acute
On the 3th day appeared changes of post-traumatic empyema of pleura. On
neurologic status: nystagmus, Graefe’s si- the X-ray: wide level horizontal of fluid
gn. Urea is yellow, feces- golden-yellow. on the right. What method of treatment
Mother’s blood group is (II)Rh−, child- should be prescribed?
(II)Rh+. On the third day child’s Hb is
200 g/L, RBC- 6, 1 ∗ 1012 /L, bilirubin in A. Punction and drainage of pleural cavity
blood - 58 mk mol/L due to unconjugated B. Decortication of pleura
bilirubin, Ht- 0,57. What is the child’s C. Pneumoectomy
jaundice explanation? D. Thoracoplasty
E. Lobectomy
A. Brain delivery trauma
B. Physiologic jaundice 114. A 28 y.o. homeless male was admi-
C. Hemolytic disease of newborn tted to the hospital because of initial di-
D. Bile ducts atresia agnosis "influenza". Roseolo-petechiae
E. Fetal hepatitis rash has appeared on the trunk and
internal surfaces of the limbs on the fifth
110. A gravida with 7 weeks of gestation day. Temperature is 410 C, euphoria, face
is referred for the artificial abortion. On and sclera’s hyperemia, tongue tremor,
operation while dilating cervical canal wi- tachycardia, splenomegaly, excitement.
th Hegar dilator №8 a doctor suspected What is the most probable diagnosis?
uterus perforation. What is immediate
doctors tactics to confirm the diagnosis? A. Typhus
B. Alcogolic delirium
A. Probing of uterus cavity C. Leptospirosis
B. Bimanual examination D. Measles
C. Ultrasound examination E. Abdominal typhoid
D. Laparoscopy
E. Metrosalpingography 115. A patient complains of intense pressi-
ng pain in the pharynx, mainly to the
Krok 2 Medicine 2005 15

right, impossibility to swallow even li- 119. A 30 y.o. male patient complains of
quid food. The illness started 5 days ago. itching of the skin which intensifies in the
The patient’s condition is grave. Body evening. He has been ill for 1,5 months.
temperature - 38, 90 C, speech is difficult, On examination: there is rash with paired
voice is constrained, difficulties in opening papules covered with bloody crusts on the
the mouth. Submaxillary glands to the ri- abdomen, hips, buttocks, folds between
ght are painful, enlarged. What is the most the fingers, flexor surfaces of the hand.
probable diagnosis? There are traces of line scratches. What
additional investigations are necessary to
A. Peritonsillar abscess make diagnosis?
B. Diphtheria
C. Pharyngeal tumour A. Examination of rash elements scrape
D. Vincent’s disease B. Determination of dermographism
E. Phlegmonous tonsillitis C. Serologic blood examination
D. Blood glucose
116. In a 65 y.o. female patient a tumor E. Examination for helmints
13х8 сm in size in the umbilical area
and above is palpated, mild tenderness 120. A child was delivered severely
on palpation, unmovable, pulsates. On premature. After the birth the child has
ausculation: systolic murmur. What is the RI symptoms, anasarca, fine bubbling
most probable diagnosis? moist rales over the lower lobe of the right
lung. Multiple skin extravasations, bloody
A. Abdominal aortic aneurism foam from the mouth have occured after
B. Stomach tumor the 2 day. On chest X-ray: atelectasis of
C. Arterio-venous aneurism the lower lobe of the right lung. In blood:
D. Tricuspid valve insufficiency Hb-100 g/L, Ht- 0,45. What is the most
E. Mitral insufficiency probable diagnosis?
117. A 25 y.o. woman complains of profuse A. Edematous-hemorrhagic syndrome
foamy vaginal discharges, foul, burning B. Disseminated intravascular clotting
and itching in genitalia region. She has syndrome
been ill for a week. Extramarital sexual C. Pulmonary edema
life. On examination: hyperemia of vagi- D. Hyaline membrane disease
nal mucous, bleeding on touching, foamy E. Congenital pneumonia
leucorrhea in the urethral area. What is
the most probable diagnosis? 121. A 58 y.o. male patient is examined
by a physician and suffers from general
A. Trichomonas colpitic weakness, fatigue, mild pain in the left
B. Gonorrhea subcostal area, sometimes frequent pai-
C. Chlamydiosis nful urination. Moderate splenomegaly
D. Vagina candidomicosis has been revealed. Blood test: neutrophi-
E. Bacterial vaginosis lic leukocytosis with the progress to
myelocyte; basophil- 2%; eosinophil-
118. A 18 y.o. woman consulted a 5%. There is a urate crystales in urine,
gynecologist with complaints of the pain erythrocyte- 2-3 in the field of vision.
in the lower part of the abdomen, fever What is the preliminary diagnosis?
up to 37, 50C, considerable mucopurulent
discharges from the genital tract, colic A. Chronic myeloleucosis
during urinating. After mirror and vagina B. Leukemoid reaction
examination the results are the followi- C. Lymphogranulomatosis
ng: the urethra is infiltrated, cervix of D. Hepar cirrhosis
the uterus is hyperemic, erosive. The E. Urolithiasis
uterus is painful, ovaries are painful, thi-
ckened, free. Bacterioscopy test showed 122. A 2 m.o. child was delivered at term
diplococcus. What diagnosis is the most with weight 3500 g and was on the mi-
probable? xed feeding. Current weight is 4900 g.
Evaluate the current weight of child.
A. Recent acute ascending gonorrhea
B. Trichomoniasis
C. Candydomycosis
D. Chronic gonorrhea
E. Chlamydiosis
Krok 2 Medicine 2005 16

A. Corresponding to the age all muscle groups, generalized convulsi-


B. 150 g less than necessary ons every 10-15 min. What is the most
C. Hypotrophy of I grade probable diagnosis?
D. Hypotrophy of II grade
E. Paratrophy of I grade A. Tetanus
B. Tetania
123. A 2 m.o. breast-fed child suffers from C. Meningoencephalitis
cheek skin hyperemia, sporadic papulous D. Hemorrhagic stroke
elements on the skin of the chest and E. Epilepsy
back following the apple juice introducti-
on. The child is restless. What is the initial 127. A 18 y.o. female student complains
pediatritian’s tactics? of dyspnea during the intensive exertion.
The condition became worse half a year
A. Clarify mother’s diet and exlude obli- ago. On examination: pulse rate is 88 bpm,
gate allergens accelerated, AP- 180/20 mm Hg, pale skin,
B. Refer to prescribe dermathologist heart borders are dilated to the left and
C. Administer general ultraviolet irradiati- up. There is systolic-diastolic murmur in
on the 2hd intercostal space, S2 at pulmonary
D. Treat with claritine artery is accentuated. ECG has revealed
E. Apply ointment with corticosteroids to both ventricles hypertrophy. Thoracic X-
affected skin areas ray has revealed pulsation and protrusi-
on of the left ventricle, lung trunk. What
124. A 43 y.o. woman complains of severe doctor’s tactics should be?
pain in the right abdominal side irradi-
ating in the right supraclavicular area, A. Cardiosurgeon consultation
fever, dryness and bitterness in the mouth. B. Dispensary observation
There were multiple vomitings without C. Administration of therapeutic treatment
relief. Patient relates the onset of pain to D. Continuation of investigation
the taking of fat and fried food. Physi- E. Exemption from physical exercises
cal examination: the patient lies on the
right side, pale, dry tongue, tachycardia. 128. A 35 y.o. woman is suspected of
Right side of abdomen is painful during aplastic anemia. The bone marrow puncti-
palpation and somewhat tense in right on has been administered with the di-
hypochondrium. What is the most likely agnostic purpose. What changes in the
diagnosis? marrow punctatum are suggested?
A. Perforative ulcer A. Replacement of marrow elements with
B. Acute cholecystitis adipose tissue
C. Acute bowel obstruction B. Replacement of marrow elements with
D. Acute appendicitis fibrous tissue
E. Right-sided renal colic C. Prevalence of megaloblasts
D. Presence of blast cells
125. On the 5-th day of the respiratory E. Absolute lymphocytosis
disease a 24 y.o. man has developed
progressive headaches systemic dizziness, 129. A girl, aged 13, consults the school
feeling of seeing double, paresis of mi- doctor on account of moderate bloody
mic muscles on the right, choking while discharge from the genital tracts, which
swallowing. Acute viral encephalitis has appeared 2 days ago. Secondary sexual
been diagnosed. What is the main directi- characters are developed. What is the
on of urgent therapy? most probable cause of bloody discharge?
A. Zovirax A. Menarche
B. Glucocorticoids B. Juvenile haemorrhage
C. Ceftriaxon C. Haemophilia
D. Lasix D. Endometrium cancer
E. Hemodesis E. Werlhof’s disease
126. A 65 y.o. woman complains of compli- 130. A 55 y.o. male patient complains of
cated mouth opening following foot weakness during 2 months, pain in the
trauma 10 days ago. Next day she ate with right side of the thorax, cough, blood-
difficulties, there were muscles tension of streaked sputum. On X-ray: intensive tri-
back, the back of the head and abdomen. angle shadow in the area of lower lobe
On the third day there was tension of that is connected to mediastinum. What is
Krok 2 Medicine 2005 17

the most likely disorder in the lungs? A. Aortic stenosis


B. Defect of interventricular septum
A. Central cancer of lungs C. Defect of interatrial septum
B. Tuberculosis of lungs D. Coarctation of the aorta
C. Bronchiectasia E. Open aortic duct
D. Pulmonary infarction
E. Pleuropneumonia 135. At’s planned to construct multifield
a new hospital in one of the cental city
131. In a forest summer camp children districts. What building type is the most
have variable procedures to harden their appropriate in this case?
organisms. What procedure has the most
hardening power? A. Centralized and blocked
B. Centralized
A. Contrast shower C. Decentralized
B. Morning exercises on the fresh air D. Mixed
C. Hygienic shower E. Blocked
D. Walking on the fresh air
E. Bath with hydromassage 136. A 60 y.o. patient experiences acute air
insufficiency following of the venoectomy
132. There is a dynamic growth of due to subcutaneous vein thrombophlebi-
number of congenital abnormalities such tis 3 days ago. Skin became cianotic, wi-
as central paralysis, newborns blindness, th grey shade. Marked psychomotor exci-
idiocy among the population that lives tement, tachypnea, substernal pain. What
near to pesticides production enterprise. postoperative complication has occured?
Compounds of which pollutant can cause
the development of this pathology? A. Thromboembolia of pulmonary artery
B. Hemorrhagia
A. Mercury C. Hypostatic pneumonia
B. Strontium D. Myocardial infarction
C. Cadmium E. Valvular pneumothorax
D. Iron
E. Chrome 137. A 1,5 y.o. child fell ill acutely with
high temperature 380 C, headache, fati-
133. A 7 y.o. child had elevation of gue. The temperature declined on the
temperature tol 400 C in anamnesis. For fifth day, muscular pain in the right leg
the last 3 months he presents fusiform occured in the morning, there were no
swelling of fingers, ankle joints and knee movements and tendon reflexes, sensiti-
joint, pain in the upper part of the sternum vity was reserved. What is the initial di-
and cervical part of the spinal column. agnosis?
What is the most probable diagnosis?
A. Polyomyelitis
A. Juvenile rheumatic arthritis B. Viral encephilitis
B. Rheumatism C. Polyartropathy
C. Toxic synovitis D. Osteomyelitis
D. Septic arthritis E. Hip joint arthritis
E. Osteoarthrits
138. On observation of sanitary conditions
134. A 5 y.o. child with stigmas of of studying at the technical university it
dysembryogenesis (small chin, thick li- was necessary to evaluate the visual regi-
ps, opened mouth, hyperthelorismus) has men of students, who study from 9 a.m to
systolic murmur in the second intercostal 3 p.m. What index of natural light will be
to the right of the sternum. The murmur the most informative?
passes to the neck and along the sternum
left edge. The pulse on the left brachi- A. Natural light coefficient
al artery is weakened. BP on the right B. Light coefficient
arm is 110/60 mm Hg, on the left - 100/60 C. Depth of study room
mm Hg. ECG results: hypertrophy of the D. Time of the room insolation
right ventricle. What defect is the most E. Presence of mixed (upper-lateral) light
probable? 139. A 24 y.o. woman presents with
prolonged fever, nocturnal sweating. She’s
lost weight for 7 kg during the last 3
months. She had irregular intercourses.
Krok 2 Medicine 2005 18

On examination: enlargement of all nation: heart boarders are dilated to the


lymphaden groups, hepatolienal syndrom. left side, sounds are muffled, pulse- 76
In blood: WBC- 2, 2 ∗ 109 /L. What is the bmp, rhythmic, AP- 155/80 mm Hg, ECG:
most likely diagnosis? the left type, the rest of signs are normal.
What additional examination is necessary
A. HIV-infection to confirm the diagnosis?
B. Lymphogranulomatosis
C. Tuberculosis A. Bicycle ergometry
D. Infectious mononucleosis B. Echocardiography
E. Chroniosepsis C. Blood lipoproteins
D. General blood count
140. A female rheumatic patient experi- E. Transaminases of blood
ences diastolic thoracic wall tremor (di-
astolic thrill), accentuated S1 at apex, 144. A 35 y.o. male patient suffers
there is diastolic murmur with presystolic from chronic glomerulohephritis and
intensification, opening snap, S2 accent at has been on hemodialysis for the last
pulmonary artery. What rind of heart di- 3 years. He has developed irregulari-
sorder is observed? ties in the heart activity, hypotension,
progressive weakness, dyspnea. On ECG:
A. Mitral stenosis bradycardia, 1st degree atrioventicular
B. Aortic valve insufficiency block, high sharpened T-waves. Before
C. Pulmonary artery stenosis he had severely disturbed the drinking
D. Mitral valve insufficiency and diet regimen. What is the most likely
E. Opened arterial duct cause of these changes?
141. A 31 y.o. woman has suffered from A. Hyperkaliemia
systemic sclerodermia for 14 years. She B. Hyperhydratation
was treated in hospitals many times. C. Hypokaliemia
She complains of periodical dull pain in D. Hypernatremia
the cardiac area, palpitation, dyspnea, E. Hypocalcemia
headache, eyelids swelling, weight loss,
pain and limbs deformities. Which organ’s 145. A 20 daily y.o. female patient
disorder worsens the prognosis? is suffering from chronic bronchitis.
Recently there has been production about
A. Kidneys 0,5 L of purulent sputum with maximum
B. Heart discharge in the morning. Fingers are li-
C. Lungs ke "drum sticks", there are "watching
D. Gastro-intestinal tract glass"nails. What is the most probable di-
E. Skin and joints agnosis?
142. A 70 y.o. male patient with mi- A. Bronchiectasia
ld headaches complains of speech di- B. Pneumonia
sorder, weakness in right limbs. There C. Chronic bronchitis
was a history of miocardial infarction and D. Gangrene of lungs
arrhythmia. On nu eroligical examinati- E. Tuberculosis
on there are elements of motor aphasia,
central paresis of VII and XII cranial 146. A 18 y.o. woman complains of
nerves pairs on the right side, cental type weakness, dizziness, loss of appetite,
of hemiparesis and hemihyperesthisia on menorrhagia. There are petechiae on the
the same side. What is the most probable skin of the upper extremities. Blood test:
diagnosis? Hb– 105 g/L; RBC- 3, 2 ∗ 1012 /L; coloured
index– 0,95; thromb.– 20 ∗ 109/L. The sedi-
A. Ischemic stroke mantation time according to Lee White
B. Hemorrhagic stroke is 5 ; hemorrhagia duration according to
C. Transitory ischemic attack Duke is 8 , "pinch and tourniquet"test is
D. Epidural hematoma positive. What is the most probable di-
E. Cerebral tumor agnosis?
143. A 52 y.o. male patient suffers from
squeezing pain attacks in substernal area
which irradiates to the left hand and
occurs occasionally and on physical exerti-
on. He has had it for 1 year. On exami-
Krok 2 Medicine 2005 19

A. Idiopathic thrombocytopenic purpura concentric hypertrophy of the left ventri-


B. Hemophilia cle. Ultrasound examination of the ki-
C. Hemorrhagic diathesis dneys reveals thinned cortical layer. Urine
D. Iron deficiency anemia analysis shows proteinuria of 3,5 g/day.
E. Marchiafava-Micheli’s disease What is the probable diagnosis?
147. A 30 y.o. primipara has intensive labor A. Essential arterial hypertension
pushings with an interval of 1-2 min and of B. Chronic pyelonephritis
50 sec duration. There is a appearing of C. Chronic glomerulonephritis
the fetus head. Perineum is of 4 cm hei- D. Polycystic disease of the kidneys
ght, has turned pale. What should be done E. Cushing’s disease
in this case?
152. A 28 y.o. primagravida, pregnancy
A. Episiotomy is 15-16 weaks of gestation, presents to
B. Perineum protection the maternity clinics with dull pain in
C. Perineotomy the lower part of the abdomen and in
D. Vacuum extraction of the fetus lumbar area. On vaginal examination:
E. Observation uterus cervix is 2,5 cm, external isthmus
allows to pass the finger tip. Uterus body
148. A 28 y.o. male patient was admi- is enlarged according to the pregnancy
tted to the hospital because of high term. Genital discharges are mucous, mi-
temperature 390 C, headache, generalized ld. What is the diagnosis?
fatigue, constipation, sleep disorder for 9
days. There are sporadic roseolas on the A. Threatened spontaneous abortion
abdomen, pulse- 78 bpm, liver is enlarged B. Spontaneous abortion which has begun
for 2 cm. What is the most probable di- C. Stopped pregnancy
agnosis? D. Hydatid molar pregnancy
E. Placenta presentation
A. Abdominal typhoid
B. Typhus 153. A primapara with pelvis size 25-28-
C. Sepsis 31-20 cm has active labor activity. Waters
D. Brucellosis poured out, clear. Fetus weight is 4500 g,
E. Leptospirosis the head is engaged to the small pelvis
inlet. Vasten’s sign as positive. Cervix
149. A 40 h.o. child age has hyperosthesia, of uterus is fully dilated. Amniotic sac
CNS depression, dyspepsia. Sepsis is is absent. The fetus heartbeat is clear,
suspected. What should the differential rhythmic, 136 bpm. What is the labor tacti-
diagnosis be made with? cs?
A. Hypoglycemia A. Caesarean section
B. Hypocalcemia B. Vacuum extraction of the fetus
C. Hyperbilirubinemia C. Obstetrical forseps
D. Hyperkaliemia D. Conservative tactics of labor
E. Hypomagnesemia E. Stimulation of the labor activity
150. A 20 y.o. patient with bronchial 154. A 41 y.o. man complains of acute
asthma experiences dyspnea attacks 3-4 pain in the right side of the thorax and
times a week. Nocturnal attacks are 1 time sudden increase of dyspnea following
a week. FEV1- 50% of necessary figures, the lifting of heavy object. The patient’s
during the day it’s variations is 25%. What condition is serious: lips and mucous are
is the severity of bronchial asthma condi- cyanotic, breathing rate is 28 per min,
tion? pulse- 122 bpm., AP- 80/40 mm Hg. There
is tympanitis on percussion and weakened
A. Moderate severity condition breathing on auscultaion on the right. S2 is
B. Mild condition accentuated over pulmonary artery. What
C. Serious condition is the urgent measure on the prehospital
D. Asthmatic status stage?
E. Intermittent flow
151. A 40 y.o. man complains of headache
in occipital area. On physical exami-
nation: the skin is pale; face and hand
edema, BP- 170/130 mm Hg. On EchoCG:
Krok 2 Medicine 2005 20

A. Air aspiration from the pleural cavity labor?


B. Epinephrine introduction
C. Euphilline introduction A. Application of obstetrical forceps
D. Call for cardiologic team B. Stimulation of the labor activity with
E. Oxygen inhalation oxitocyne
C. Ceasarian section
155. A 6 y.o child complains of thirst, D. Application of craniodermal forceps by
polyuria, increased appetite for 2 months Ivanov’s
with weight loss for 3 kg. There has been E. Application of obstetrical cavity forceps
nocturnal enuresis during last week. On
examination: hyperglycemia 14 mol/L. 159. A 31y.o. patient has had mental
The diagnosis is diabetis mellitus I type. disorder for a long time. He suffers
What is the genesis of this disease? from insomnia for a long time. He has
developed fears, suicidal thoughts, tried
A. Autoimmune to hang himself. His mood is depressed,
B. Viral he refuses from treatment. What measures
C. Bacterial are the most expedient for the prevention
D. Neurogenic of suicide?
E. Virus-bacterial
A. Admission to the mental hospital
156. A 74 y.o. female patient complai- B. Admission to the nu erological
ns of pain, distended abdomen, nausea. department
She suffers from heart ichemia, post- C. Out-patient treatment
infarction and diffusive cardiosclerosis. D. Psychotherapeutic conversation
On examination: grave condition, di- E. Strict home supervision
stended abdomen, abdominal wall fai-
ls to take active part in breathing. On 160. A 43 y.o. woman complains of shooti-
laparoscopy: some cloudy effusion, one ng heart pain, dyspnea, irregularities in
of the bowel loops is dark-blue. What is the heart activity, progressive fatigue duri-
the most probable diagnosis? ng 3 weeks. She had acute respiratory
disease a month ago. On examination:
A. Mesenterial vessels thrombosis AP- 120/80 mm Hg, heart rate 98 bpm,
B. Volvulus heart boarders +1,5 cm left side, sounds
C. Acute intestinal obstruction are muffled, soft systolic murmur at apex
D. Ichemic abdominal syndrome and Botkin’s area; sporadic extrasystoles.
E. Erysipelas Liver isn’t palpated, there are no edema.
Blood test: WBC- 6, 7∗109/L, sedimentati-
157. A 56 y.o. man, who has taken on rate- 21 mm/hour. What is the most
alcoholic drinks regularly for 20 years, probable diagnosis?
complains of intensive girdle pain in the
abdomen. Profuse nonformed stool 2- A. Acute myocarditis
3- times a day has appeared for the last B. Climacteric myocardiodystrophia
2 years, loss of weight for 8 kg for 2 C. Ichemic heart disease, angina pectoris
years. On examination: abdomen is soft, D. Rheumatism, mitral insufficiency
painless. Blood amylase - 12g/L. Feces E. Hypertrophic cardiomyopathy
examination-neutral fat 15 g per day,
starch grains. What is the most reasonable 161. A 52 y.o. male patient has become
treatment at this stage? ill gradually. There is pain in the left si-
de of the thorax during 2 weeks, elevati-
A. Pancreatine on of temperature till 38 − 390 C. On
B. Contrykal examination: left chest side falls behind
C. Aminocapron acid in breathing movement no voice tremor
D. Levomicytine over the left lung. Dullness that is more
E. Imodium intensive in lower parts of this lung.
158. A 30 y.o. woman has second labor Right heart border is deviated outside.
which lasts for 14 hours. The fetus Sharply weakened breathing over the left
heartbeat is mufflet, arrhythmic, 100 bpm. lung, no rales. Heart sounds are mufflet,
On vaginal examination: complete cervix tachycardia. What is the most probable
dilatation, fetus head is in the area of diagnosis?
small pelvis outlet. Sagital suture is in the
direct size. The small fontanelle is at the
symphis. What is the further tactics of the
Krok 2 Medicine 2005 21

A. Exudative pleuritis Pain occured 2 days ago. On exami-


B. Spotaneous pneumothorax nation: Vis OD- 0,03, congested injecti-
C. Atelectasis of lung on of the eye ball, significant cornea
D. Cirrhotic tuberculosis edema, front chamber is deep, pupil
E. Infarction-pneumonia is narrow, athrophic iris, there is optic
nerve excavation on the eye fundus,
162. A mother of a newborn child suffers intraocular pressure- 38 mm Hg. Vis OS-
from chronic pyelonephritis. She had 0,8 unadjustable. The eye is calm, healthy.
acute respiratory viral disease before Intraoccular pressure- 22 mm Hg. What is
the labor. Labor in time, with prolonged the most probable diagnosis?
period without waters. A child had
erythematous eruption on the 2 day, A. Acute glaucoma attack
then there were seropurulent vesicles B. Right eye’s uveitis
about 1cm. Nikolsky’s symptom is posi- C. Right eye’s keratitis
tive. Erosions have occured after vesi- D. Eye nerve’s neuritis
cle rupture. The child is flabby. The E. Maculodystrophy
temperature is subfebrile. What is the
most probable diagnosis? 166. A 41 y.o. woman has suffered from
nonspecific ulcerative colitis during 5
A. Newborn pemphigus years. On rectoromanoscopy: marked
B. Vesiculopustulosis inflammatory process of lower intesti-
C. Pseudofurunculosis nal parts, pseudopolyposive changes of
D. Sepsis mucous. In blood: WBC- 9, 8 ∗ 109 /L,
E. Ritter’s dermatitis RBC- 3, 0 ∗ 1012 /L, sedimentation rate-
52 mm/hour. What medication provides
163. A child was born at 34 weeks of pathogenetic treatment of this patient?
gestation in bad condition. The cardi-
nal symptoms show respiratoty disorders: A. Sulfasalasine
sound prolonged expiration, additional B. Motilium
muscles taking part in breathing, crepitati- C. Vikasolum
on rales on the background of the rough D. Linex
breath sounds. Assesment according to E. Kreon
Silverman’s scale was 0, in 3 hours- 6 with
presence of clinical data. What diagnostic 167. A 49 y.o. female patient presents wi-
method can determine pneumopathy’s th acute attacks of headache associated
type in the child? with pulsation in temples, increasing AP
to 280/140 mm Hg. Pheochromocytoma
A. Chest X-ray is suspected. What is the mechanism of
B. Blood test hypertensive crisis in this patient?
C. Blood gases
D. Proteinogram A. Increasing of catecholamines
E. Immunologic investigation concentration
B. Increasing of aldosterone level in blood
164. During intramuscular DTP vaccinati- C. Increasing of plasma renin activity
on in clinic, a 3 m.o. child developed si- D. Increasing of vasopressin excretion
gns of laryngospasm, paleness of skin, E. Increasing of thyroxine excretion
cyanosis of lips, "cock cry", stop of respi-
ration, tension of the whole body with 168. To replace the blood loss replacement
overturned backward head. Allergologi- 1000 ml of the same group of Rhesus-
cal history of the child is not complicated. compatible donated blood was transfused
What is the most probable diagnosis? to the patient. The blood was conserved
by sodium citrate. At the end of
A. Spasmophilia, tonic spasms hemotransfusion there appeared exci-
B. Anaphylactic shock, clonic spasms tement, pale skin, tachycardia, muscles
C. Meningoencephalitic reaction, clonic cramps in the patient. What complicati-
and tonic spasms on should be suspected?
D. Cerebral haemorrhage, tonic spasms
E. Meningism, clonic and tonic spasms A. Citrate intoxication
B. Citrate shock
165. A 60 y.o. man complains of signifi- C. Allergic reaction
cant pain in the right eye, photophobia, D. Anaphylactic shock
lacrimation, reduced vision of this eye, E. Pyrogenous reaction
headache of the right part of the head.
Krok 2 Medicine 2005 22

169. A 20 y.o. patient suddely felt ill 12 A. Arteriosclerosis obliterans


hours ago. There was pain in epigactric B. Obliterating endarteritis
area, nausea, sporadic vomiting. He had C. Hemoral arthery thombosis
taken alcohol before. In few hours the D. Raynauld’s disease
pain localized in the right iliac area. On E. Buerger’s disease (thromboangiitis
examination: positive rebound tenderness obliterans)
symptoms. WBC- 12, 2 ∗ 109/L. What is the
most probable diagnosis? 173. A patient had macrofocal myocardial
infarction. He is overweight for 36%, AP
A. Acute appendicitis is 150/90 mm Hg, blood sugar- 5,9 mmol/L,
B. Acute pancreatitis general cholesterol- 4,9 mmol/L, uric acid-
C. Perforated ulcer 0,211 mmol/L. Which risk factor should be
D. Rightside kidney colic urgently eradicated during the secondary
E. Acute cholecystitis prevention?
170. A patient, aged 58, was fishing in A. Obesity
the winter. On return home after some B. Arterial hypertension
time felt some pain in the feet. Consulted C. Hyperglycemia
a doctor. On examination: feet skin was D. Hypercholesterolemia
pale, then after rewarming became red, E. Hyperuricemia
warm to the touch. Edema is not signifi-
cant, limited to the toes. All types of sensi- 174. A 2 y.o. boy was admitted to the
tivity are preserved. No blisters. What hospital with weight loss, unstable di-
degree of frostbite is observed? scharges, anorexia, following the semoli-
na’s introduction (since 5 months). The
A. I degree child is adymanic, flabby, pale dry skin,
B. II degree subcutaneous layer is emaciated. Di-
C. III degree stended and tensed abdomen, tympani-
D. IV degree tis on percussion of the upper part of
E. V degree the abdomen, splashing sounds, feces
are foamy, of light color, foul. On
171. A 24 y.o. emotionally-labile woman coprocytogram: a lot of neutral fat. What
presents with irritation, depressed mood, is the cause of the disease?
palpitation, shooting pain in the heart
area, generalized fatigue following A. Celiakia (celiac disease)
the divorce. On examination: palm B. Mucoviscidosis (cystic fibrosis)
hyperhydrosis, pulse rate- 72-78 bpm, labi- C. Intestinal dysbacteriosis
le, heart without changes. ECG is normal. D. Chronic enteritis
What is the most probable pathology in E. Disaccharidase insufficiency
this case?
175. On medical observation a doctor
A. Neurasthenia identified girl (162 cm tall and 59 kg wei-
B. Ipochondric neurosis ght) who complained loss of ability to see
C. Compulsive neurosis surrounding objects clearly in the evening.
D. Schizophrenia On examination: dry skin, hyperkeratosis.
E. Depressive neurosis Her daily ration includes the following vi-
tamines: vitamine А– 0,5 mg, vit.1 – 2,0
172. A 98 y.o. male patient complai- mg, vit.2 – 2,5 mg, vit.6 – 2 mg, vit.С– 70
ns of pain in the left lower limb whi- mg. What is the hypovitaminosis type?
ch intensifies on walking, feeling of cold
and numbness in both feet. He has been A. A-hypovitaminosis
ill for 6 years. On examination: pale B. B1 -hypovitaminosis
dry skin, hyperkeratosis. Hairy coveri- C. B2 -hypovitaminosis
ng is poorly developed on the left shin. D. B6 -hypovitaminosis
"Furrow symptom "is positive on the left. E. C-hypovitaminosis
Pulse on foot arteries and popliteal artery
isn’t palpated, on the femoral artery it’s 176. A woman in labor, on vaginal
weak. On the right limb the artery pulsati- inspection: cervix dilation is up to 2 cm,
on is reserved. What is the most probable fetal bladder is intact. Sacral cavity is
diagnosis? free, sacral promontory is reachable only
with a bent finger, the inner surface of
the sacrococcygeal joint is accessible for
Krok 2 Medicine 2005 23

inspection. The head of the fetus presents. with bread, shoes in her underwear in
Sagittal suture occupies the transverse di- her bosom as well as "invaluable books".
ameter of pelvic inlet, the small fontanel What is the most probable diagnosis?
to the left, on the side. What labor stage is
this? A. Senile dementia
B. Atherosclerotic (lacunar) dementia
A. Cervix dilation stage C. Presenile melancholia
B. Preliminary stage D. Behaviour disorder
C. Prodromal stage E. Dissociated personality (psychopathy)
D. Expulsion of fetus stage
E. Placental stage 181. A 29 y.o. patient was admitted to
the hospital with acute girdle pain in epi-
177. A 30 y.o. patient had deep burn gastric area, vomiting in 1 hour after the
covering 30% of body 30 days ago. Now meal. On examination: pale, acrocyanosis.
he presents with continued fever, loss of Breathing is frequent, shallow. Abdomi-
appetite, night sweats. Burned surface nal muscles are tensed, positive Schotkin-
weakly granulates. What is the stage of Blumberg’s symptom. What is the maxi-
burn disease? mal term to make a diagnosis?
A. Septicotoxemia A. In 2 hours
B. Primary burn shock B. In 0,5 hours
C. Secondary burn shock C. In 1 hour
D. Acute burn toxemia D. In 3 hours
E. Convalescence E. In 6 hours
178. A 45 y.o. woman complains of contact 182. A 33 y.o. patient was admitted to
bleedings during 5 months. On speculum the hospital with stopped recurrent peptic
examination: hyperemia of uterus cervix, ulcer bleeding. On examination he is
looks like cauliflower, bleeds on probi- exhausted, pale. Нb– 77 g/L, Нt– 0,25.
ng. On bimanual examination: cervix is Due to anemia there were two attempts of
of densed consistensy, uterus body isn’t blood transfution of identical blood group
enlarged, mobile, nonpalpable adnexa, ()Rh+. Both attempts were stopped
parametrium is free, deep fornixes. What because of anaphylactic reaction. What
is the most likely diagnosis? blood transfution environment is desi-
rable in this case?
A. Cancer of cervix of uterus
B. Cancer of body of uterus A. Washed erythrocytes
C. Fibromatous node which is being born B. Freshcitrated blood
D. Cervical pregnancy C. Erythrocyte mass (native)
E. Polypose of cervix of uterus D. Erythrocyte emulsion
E. Erythrocyte mass poor for leucocytes
179. A female, aged 20, after smoking and thrombocytes
notices a peculiar inebriation with the
feeling of burst of energy, elation, irreality 183. A 19 y.o. boy was admitted to the
and changing of surroundings: the world hospital with closed abdominal trauma.
gets full of bright colours, the objects On operation multiple ruptures of spleen
change their dimensions, people’s faces and small intestine were revealed. AP
get cartoon features, loss of time and is falling, it is necessary to perform
space judgement. What is the most likely hemotransfusion. Who can determine
diagnosis? patient’s blood group and rhesus compati-
bility?
A. Cocainism
B. Morphinism A. A doctor of any speciality
C. Barbiturism B. A laboratory physician
D. Nicotinism C. A surgeon
E. Cannabism D. A traumotologist
E. An anaesthesilogist
180. A 75 y.o patient can not tell the
month, date and season of the year. After 184. A 27 y.o. woman suffers from
long deliberations she manages to tellher pyelonephritits of the only kidney. She
name. She is in irritable and dissatisfi- presents to the maternity welfare centre
ed mood. She always carries a bundle because of suppresion of menses for 2,5
with belongings with her, hides a parcel months. On examination pregnancy 11
Krok 2 Medicine 2005 24

weeks of gestation was revealed. In uri- in the gastroenterological unit. 2 weeks


ne: albumine 3,3 g/L, leucocytes cover the later developed constant pain, increasing
field of vision. What is doctor’s tactics in and resistant to medication. The abdomen
this case? is painful in epigastric area, moderate
defence in pyloroduodenal area. Which
A. Immediate pregancy interruption complication development aggravated the
B. Pregnancy interruption after urine patient’s state?
normalization
C. Maintenance of pregnancy till 36 weeks A. Malignisation
D. Pregnancy interruption at 24-25 weeks B. Penetration
E. Maintenance of pregnancy till delivery C. Perforation
term D. Haemorrhage
E. Stenosis
185. A 35 y.o. female patient was admitted
to the surgical department with symptoms 189. A 54 y.o. male patient suffers from
of ulcerative gastric hemorrhage. It’s been dyspnea during mild physical exertion,
the third hemorrhage for the last 2 years. cough with sputum which is excreted
After conservative treatment vomiting wi- with diffculty. On examination: diffuse
th blood stopped, hemoglobin elevated cyanosis. Is Barrel-chest. Weakened vesi-
from 60 till 108 g/L. General conditi- cular breathing with prolonged expirati-
on became better. But profuse vomiti- on and dry whistling rales. AP is 140/80
ng with blood reoccured in 2-3- hours. mm Hg, pulse is 92 bpm, rhythmic. Spi-
Hemoglobin decreased to 93,1 g/L then to rography: vital capacity (VC)/predicted
58,1 g/L. What is the tactics of treatment? vital capacity- 65%, FEV1/FVC– 50%.
Determine the type of respiratory insuffi-
A. Urgent surgery ciency (RI).
B. Deferred surgery
C. Conservative treatment A. RI of mixed type with prevailing
D. Conservative treatment with following obstruction
surgery B. RI of restrictive type
E. Taylor’s treatment C. RI of obstructive type
D. RI of mixed type with prevailing resri-
186. A victim of a road accident, aged 44, ction
is operated on account of intraperitoneal E. There is no RI
haemorrhage. In which case can the pati-
ent’s blood from the abdominal cavity be 190. A patient aged 18 with a cranial injury
used for autotransfusion? was in comatose state during several
hours. In post-comatose period gets ti-
A. Stomach rupture red quickly, non-productive in dialog - in
B. Bladder rupture the beginning answers 2-3 questions, then
C. Liver rupture gets tired and can not understand the poi-
D. Splenic rupture nt of the question. Which psychotropic
E. Small intestines rupture should be given to the patient to prevent
187. A man, aged 30, complains of intense psychoorganic syndrome?
pain, reddening of skin, edema in the A. Nootropics
ankle-joint area, fever up to 390 . Sudden B. Neuroleptics
onset of the illness. In the past there were C. Stimulators
similar attacks lasting 5-6 days without D. Tranquillisers
residual changes in the joint. The skin E. Antidepressants
over the joint is hyperemic without defi-
nite borders and without infiltrative bank 191. A 25 y.o. patient was admitted with
on the periphery. What is the most likely chest trauma. Clinical and X-ray exami-
diagnosis? nation have revealed tense pneumothorax
on the left. What emergency treatment
A. Gout should be undertaken?
B. Infectional arthritis
C. Rheumatoid arthritis A. Pleural cavity drainage
D. Erysipelatous inflammation B. Intravenous infusions
E. Osteoarthritis C. Oxigenotherapy
D. Intubation
188. A patient, aged 25, suffering from E. Analgetics
stomach ulcer. Had a course of treatment
Krok 2 Medicine 2005 25

192. A 38 y.o. patient complains of pain in 197. A 2,5 m.o. child presents with muscle
lumbar part of spinal column with irradi- hypotonia, sweating, alopecia of the back
ation to the back surface of the left leg of the head. The child is prescribed
following the lifting of a heavy object. massage, curative gymnastics and vitamin
Pain is increasing on change of the body D. What is the dosage and frequency of
position and in vertical position. positi- vitamin D administration?
ve stretching symptoms were revealed on
examination. What is an initial diagnosis? A. 3000 IU daily
B. 500 IU daily
A. Intervertebral ligaments disorder C. 1000 IU daily
B. Spinal cord tumor D. 500 IU every other day
C. Arachnomielitis E. 1000 IU every other day
D. Polyneuritis
E. Myelopathy 198. A 43 y.o. male complains of stomach
pain, which relieves with defecation,
193. A child is being discharged from and is accompanied by abdominal wi-
the surgical department after conservati- nds, rumbling, the feeling of incomplete
ve treatment of invagination. What evacuation or urgent need for bowel
recommendations should doctor gi- movement, constipation or diarrhea
ve to mother to prevent this disease in alternation. These symptoms have
recurrence? lasted for over 3 months. No changes in
laboratory tests. What is the most likely
A. Strict following of feeding regimen diagnosis?
B. Common cold prophilaxis
C. Feces observation A. Irritable bowel syndrome
D. Gastro-intestinal disease prevention B. Spastic colitis
E. Hardening of the child C. Colitis with hypertonic type dyskinesia
D. Chronic enterocolitis, exacerbation
194. A male patient presents with swollen phase
ankles, face, eyelids, elevated AP- 160/100 E. Atonic colitis
mm Hg, pulse- 54 bpm, daily loss of
albumine with urine- 4g. What therapy 199. After delivery and revision of
is pathogenetic in this case? placenta there was found the defect
of placental lobe. General condition of
A. Corticosteroids woman is normal, uterine is firm, there is
B. Diuretics moderate bloody discharge. Inspection of
C. NSAID birth canal with mirrors shows absence
D. Calcium antagonists of lacerations. What is the following
E. Antibiotics necessary action?
195. During dynamic investigation of a A. Manual exploration of the uterine
patient the increase of central venous cavity
pressure is combined with the decrease of B. External massage of uterus
arterial pressure. What process is proved C. Use of uterine contracting agents
by such combination? D. Urine drainage, cold at lower abdomen
E. Use of hemostatic medications
A. Increase of bleeding speed
B. Developing of cardiac insufficiency 200. A patient, aged 81, complains of
C. Shunting constant urinary excretion in drops, feeli-
D. Depositing of blood in venous channel ng of fullness in the lower abdomen.
E. Presence of hypervolemia On examination: above pubis there is a
spherical protrusion, over which there is
196. A male patient complains of a dullness of percussion sound, positi-
heartburn which gest stronger while ve suprapubic punch. What symptom is
bending the body, substernal pain duri- observed in this patient?
ng swallowing. There is a hiatus hernia on
X-ray. What disoeder should be expected A. Paradoxal ischuria
at gastroscopy? B. Urinary incontinence
C. Dysuria
A. Gastroesophageal reflux D. Enuresis
B. Chronic gastritis E. Pollakiuria
C. Gastric peptic ulcer
D. Acute erosive gastritis
E. Duodenal peptic ulcer
Krok 1 Medicine 2006 1

1. A 47 y.o. woman complains of having A. Acute pericarditis


paroxysmal headaches for the last 5 years. B. Pulmonary embolism
The pain is one-sided, intense, localised C. Tietze’s syndrome
in frontal region of head, accompani- D. Dissecting aortic aneurysm
ed by nausea and stomach discomfort, E. Dressler’s syndrome
begins one of a sudden. Onset is usually
preceded by vision reduction. Anamnesis 4. A 54 y.o. man was admitted to the hospi-
gives evidence of periodical AP rise, but tal with complaints of sudden intense
at the moment the woman doesn’t take headache in occipital region and vomi-
any medicines. Inbetween the onsets of ting. In the medical hystory: moderate
headache her state is satisfactory. Objecti- arterial hypertension, the patient was
vely: high-calorie diet (body weight index taking hydrochlorothiazide. Three days
ago he consulted a therapeutist about
- 29), AP- 170/95 mm Hg. Neurologic
intense headache that was suppressed
state has no pecularities. What is the most by an analgetic. Objectively: consci-
probable diagnosis?
ousness is confused, left pupil is mydri-
A. Migraine atic. Evident photophobia and tension of
B. Chronic subdural hematoma neck muscles. Left-side hemiparesis with
C. Epilepsy increased muscle tonus and reflexes. Body
D. Benign intracranial hypertension temperature is low, rash is absent. AP-
E. Hypertensive encephalopathy 230/130 mm Hg, Ps- 50 bpm, BR- 12/min.
What is your preliminary diagnosis?
2. A 7 y.o. boy suddenly felt pain in
his right knee, it became edematic. The A. Acute subdural hematoma
day before he took part in a cross- B. Myasthenia
country race. Family anamnesis has no C. Disseminated sclerosis
data about hemophilia and bleeding si- D. Migraine
ckness. Objectively: body temperature is E. Acute bacterial meningitis
37, 50 . The knee is painful, hot to the 5. A 76 y.o. woman complains of progressi-
touch, edematic with local tissue tensi- ng swallowing disorder, mostly she has
on over it. Blood count: Нb- 123 g/L, had problems with solid food for the
leukocytes - 5, 6 ∗ 109 /L, thrombocytes - last 6 weeks. Sometimes she has regurgi-
354 ∗ 109/L, prothrombin time - 12 seconds tation of solid masses. Swallowing is
(normally 10-15 seconds), partly acti- not painful. She lost 6 kg. 10 years ago
vated thromboplastin time - 72 seconds she had myocardiac infarction, she takes
(normally 35-45 seconds). Hemorrhage ti- constantly aspirine and prolonged ni-
me is normal, VIII:C factor is 5% of norm. trates. She consumes alcochol in moderate
What is the most probable diagnosis? proportions, smokes. Objectively: icteric
skin, neck has no pecularities, lymph
A. Hemophilia A nodes are not enlarged. Thorax has no
B. Hemophilia B changes, cardiovascular system has no evi-
C. Schoenlein-Henoch disease dent changes. Liver is +3 cm. What is the
D. Vitamin K deficiency preliminary diagnosis?
E. Thrombocytopenia
A. Cancer of esophagus
3. On the 3rd day after the acute anteri- B. Diaphragmatic hernia
or myocardial infarction a 55 y.o. pati- C. Diffuse constriction of esophagus
ent complains of dull ache behind his D. Myasthenia
breast bone, that can be reduced by bendi- E. Esophageal achalasia
ng forward, and of dyspnea. Objectively:
AP- 140/180 mm Hg, heart sounds are 6. A 38 y.o. man complains of having
dull. ECG results: atrial fibrillation wi- occasional problems with swallowing of
th frequence of ventricular contractions both hard and fluid food for many months.
at the rate of 110/min, pathological Q Sometimes he feels intense pain behind
wave and S-T segment raising in the ri- his breast bone, epecially after hot dri-
ght chest leads. The patient refused from nks. There are asphyxia onsets at night.
thrombolisis. What is the most probable He has not put off weight. Objectively:
diagnosis? his general condition is satisfactory, skin is
of usual colour. Examination revealed no
changes of gastrointestinal tract. X-ray pi-
cture of thorax organs presents esophagus
Krok 1 Medicine 2006 2

dilatation with level of fluid in it. What is pains in the bottom of abdomen irradiati-
the preliminary diagnosis? ng to the anus, nausea, giddiness, bloody
dark discharges from sexual tracts for one
A. Esophagus achalasia week, the delay of menses for 4 weeks. Si-
B. Myastenia gns of the peritoneum irritation are positi-
C. Cancer of esophagus ve. Bimanual examination: borders of the
D. Esophagus candidosis uterus body and its appendages are not
E. Gastroesophageal reflux determined because of sharp painfullness.
The diverticulum and painfullness of the
7. A woman is admitted to the maternity back and dextral fornixes of the vagina
hospital with stopped birth activity and are evident. What is the most probable di-
mild bloody discharges from the vagina. agnosis?
The condition is serious, the skin is pale,
consciousness is confused. AP- 80/40 mm A. Broken tubal pregnancy
Hg. The palpitation of the fetus is not B. Apoplexy of the ovary
determined. In medical hystory there was C. Acute right-side adnexitis
a Cesarean section a year ago. Make a di- D. Torsion of the crus of the ovary tumour
agnosis: E. Acute appendicitis
A. Hysterorrhesis 11. Name a statistical observation unit
B. Presentation of the cord for determination of blood sugar impact
C. Placental presentation on the healing of wound’s surface in a
D. Abjointing of the mucous fuse from postoperative period:
cervix of the uterus
E. Premature expultion of the amniotic A. The patient in a postoperative period
waters B. An amount of blood sugar
C. Blood analysis
8. A 35 y.o. woman consulted a doctor D. The patient who has a wound surface
about occasional pains in paraumbilical E. The patient who was discharged on an
and iliac region that reduce after defecati- after-care
on or passage of gases. Defecation takes
place up to 6 times a day, stool is not 12. Choose a method of graphic
solid, with some mucus in it. Appetite is representation of monthly information
normal, she has not put off weight. Fi- about the number of registered cases
rst such symptoms appeared 1,5 year ago, of acute intestinal infection and their
but colonoscopy data reveals no organic comparison to the average monthly
changes. Objectively: abdomen is soft, a values, obtained for 5 last years:
little bit painful in the left iliac region.
Blood and urine are normal. What is the A. The linear diagram
preliminary diagnosis? B. The radial diagram
C. The sector diagram
A. Irritable bowels syndrome D. The figured diagram
B. Celiac disease E. Curvilinear diagram
C. Crohn’s disease
D. Pseudomembranous colitis 13. A patient, aged 25, complains of pain
E. Dispancreatism in the I finger on the right hand. On
examination: the finger is homogeneously
9. A 60 y.o. man complains of sense hydropic, in bent position. On attempt
of heaviness in the region of scrotum. to unbend the finger the pain is getti-
Objectively: scrotum edema in the left ng worse. Acute pain appears during
part. Testicle is of normal size, but there is the probe in ligament projection. What
a soft, scrotum limited edema over it that decease is the most likely?
can be pressed and disappears when the
patient lies down. What is the preliminary A. Thecal whitlow (ligament panaritium)
diagnosis? B. Subcutaneous panaritium
C. Articular (joint) panaritium
A. Varicocele D. Bone panaritium
B. Inguinal lymphadenopathy E. Paronychia
C. Ectopic testicle
D. Inguinal hernia 14. An employee has been invalid for 6
E. Varicosity of subcutaneous veins months as a result of a hip fracture. Who
has the right to authorize the issue of the
10. A 26 y.o. woman complains of sudden medical sick-list for the last 2 months?
Krok 1 Medicine 2006 3

the hospital with complaints of poor night


A. MSEC vision. Objectively: increased darkness
B. Head physician of the polyclinic adaptation time, Bitot’s spots on conjucti-
C. DCC va. The patient’s skin is dry, scales off,
D. DCC together with the head physician folliculitis signs of the face skin are
of a polyclinic present. What is the cause of this disease?
E. Deputy head physician on working
capacity A. Retinole deficit
B. Thiamine deficit
15. An employee had an abortion by C. Biotin deficit
medical indications on the 6.03.2001 and D. Folic acid deficit
she stayed in a hospital till 17.03.2001. E. Napthtochynones deficit
What term is the medical sick-list issued
for? 20. What guarantees against the
preconceived attitude to the physician in
A. For 12 days cases of professional law violations do you
B. For 3 days know?
C. For 4 days
D. For 10 days A. Sanction of public prosecutor, inquiry
E. For 11 days by preliminary investigator of prosecutor’s
office, committee of experts
16. A 5 tons milk batch was sampled. B. Draw up a statement about forensic
The lab analysis revealed: fat content 2%, medical examination
specific density - 1,04 g/cm3 , acidity - 210 Т, C. Conduct an inquiry by preliminary
reductase probe - weak-positive. What investigator of police department
way is the product to be used in? D. Utilisation copy of medical documents
E. Conduct forensic medical examination
A. Sell but inform customers about milk by district forensic medicine expert
quality
B. Discard for animal feeding 21. A 23 y.o. patient has a gunshot wound
C. Technical utilization of his left arm.The bones of the arm are
D. Sell without limitations not damaged. What appropriate surgical
E. Do the product away care must be provided to the patient?
17. Patient with thyreotoxicosis is in A. Primary surgical processing with a
the 2 beds hospital ward of therapeutic flowing suction
department. The area of the ward is 18 B. Wound saturing
m2 , height 3 m, ventilation rate 2,5/hr. Air C. Wound saturing and drying
temperature - 200 , relative humidity - D. Wound drying with towel gauzes
45%, air movement velocity - 0,3 m/s, li- E. Aseptic dressing of the wound
ght coefficient - 1/5, noise level - 30 dB.
Do hygienic evaluation of the conditions 22. A 34 y.o. patient 3 hours ago was bi-
meet the standards? tten by a dog. He has got a non-bleeding
wound in his left arm caused by the dog’s
A. Discomfortable microclimate bite. What surgical care would you provi-
B. Non-effective ventilation de to the patient?
C. Poor lighting
D. High level of noise A. Wound bathing with detergent water
E. All conditions meet the requirements and antiseptic application
B. Aseptic bandage
18. A 33 y.o. patient, works as a secretary. C. Cream bandage
Her diet contains 150 g of protein (includi- D. Complete suturing of the wound
ng 100 g of animal protein), 200 g of fat, E. Incomplete suturing of the wound
600 g of carbohydrates. What pathology 23. A 37 y.o. patient complains of pain in
can result from this diet? the right arm which increases during moti-
A. Obesity on, raised body temperature up to 390 C.
B. Schizophrenia In the right cubital fossa there is a trace of
C. Paradontosis injection, hyperemia and thickening along
D. Common cold the vein. Your diagnosis?
E. Uterine fibromyoma
19. A 15 year old adolescent was taken to
Krok 1 Medicine 2006 4

A. Phlebit
B. Phlegmon A. Incision biopsy
C. Abscess B. Yaks’s reaction
D. Inflammation of lymph C. Radioisotope diagnostics
E. Erysipelas D. Termography
E. Glass-print
24. A 35 y.o. woman was admitted to
thoracic surgery department with fever up 29. A 74 y.o. patient has been ill with beni-
to 400 C, onset of pain in the side caused by gn prostate hyperplasy for the last 5 years.
deep breathing, cough with considerable 4 days ago, after alcochol consumpti-
quantity of purulent sputum and blood on, there was an acute retention of uri-
with bad smell. What is the most likely di- nation. At the pre-admission stage his
agnosis? urinary bladder was catheterized with
metallic catheter. Examination revealed:
A. Abscess of the lung right epididymis is enlarged, thick and pai-
B. Complication of liver echinococcosis nful, there are purulent discharges from
C. Bronchiectatic disease urethra. What way of emergency care
D. Actinomycosis of lungs must be chosen?
E. Pulmonary tuberculosis
A. Trocar or open epicystostomy
25. A laboratory obtained a milk sample B. Transuretral resection or prostatectomy
sent for analysis. Analysis gave the C. Introduction of permanent urethral
following data: color - whitish, smell - has catheter
no pecularities, taste - typical for milk, D. Microwave thermotherapy of prostate
density - 1,038, acidity - Turner’s 350 , fat E. Placing of intraprostatic stent
- 3,2%. What is the quality level of this
milk? 30. A 10 y.o. boy was ill with angina 2
weeks ago, has complaints of joint pain
A. The milk is of poor quality and stiffness of his left knee and ri-
B. The milk is of high quality ght elbow. There was fever (38, 50 ) and
C. The milk is of reduced quality ankle disfunction, enlargement of cardiac
D. The milk is falsificated dullness by 2 cm, tachycardia, weakness
E. The milk is nominally qualified of the 1st sound, gallop rhythm, weak
systolic murmur near apex. What di-
26. For the persons who live in a hot area agnosis corresponds with such symptoms?
after an accident at a nuclear object, the
greatest risk within the first decade is A. Acute rheumatic fever
represented by cancer of: B. Systemic lupus erythematosis
C. Juvenile rheumatoid arthritis
A. Thyroid gland D. Reiter’s disease
B. Skin E. Reactive arthritis
C. Reproduction system organs
D. Breast 31. A baby boy was born in time, it was his
E. Lungs mother’s 1st pregnancy. The jaundice was
revealed on the 2nd day of life, then it
27. A 40 y.o. patient was diagnosed: progressed. The adynamia, vomiting and
1. Medular thyroid gland cancer. 2. hepatomegaly were presented. The indi-
Feochromocytoma. What operation rect bilirubin level was 275 mcmol/L, the
should be performed at first? direct bilirubin level - 5 mcmol/L, Hb- 150
g/L. Mother’s blood group - 0(I), Rh+,
A. Operation on account of child’s blood group - A(II), Rh+. Make a
feochromocytoma diagnosis.
B. Operation on thyroid gland
C. Krail’s operation A. Hemolytic disease of newborn (АВО
D. Subtotal resection of thyroid gland and incompatibility), icteric type
fascicular resection of limphatic nodes B. Jaundice due to conjugation disorder
E. Vanach’s operation C. Hepatitis
D. Physiological jaundice
28. A 28 y.o. woman consulted a doctor
with the complaints of enlargening in size E. Hemolytic disease of newborn (Rh -
of an inborn pigment nevus, it was also incompatibility)
wetting and itching. What test should not 32. Ambulance was called to a 48 y.o. man.
be used for diagnostics in this case? According to the relatives he has had
Krok 1 Medicine 2006 5

three episodes of lost consciousness and A. Epidermal


attacks during the day. Patient is unconsci- B. Dust
ous, fell on the floor, tonic and then C. Pollen
clonic convulsions of trunk and extremi- D. Itch mite
ties happened. The attack lasted for 4 E. Chemical
minutes, ended by involuntary urination.
What type of attack was observed? 37. A 40 y.o. patient with rheumatic heart
disease complains of anorexia, weakness
A. Major epileptic seizure and loss of weigth, breathlessness and
B. Vegetatic crisis swelling of feet. The patient had tooth
C. Absence extraction one month ago. On examinati-
D. Episode of hysteria on: t0 - 390 C, Ps- 100/min. Auscultation: di-
E. Fainting astolic murmur in the mitral area. Petechi-
al lesion are round of clavicle; spleen was
33. A 2 y.o. girl has been ill for 3 palpable.
days. Today she has low grade fever,
severe catarrhal presentations, slight A. Subacute bacteria endocarditis
maculopapular rash on her buttocks and B. Recurrence of rheumatic fever
enlarged occipital lymph nodes. What is C. Thrombocytopenia purpura
your diagnosis? D. Mitral stenosis
E. Aortic stenosis
A. Rubella
B. Scarlet fever 38. A 62 y.o. patient suffers from DM-2.
C. Measles Diabetes is being compensated by diet
D. Adenoviral infection and Maninilum. Patient has to undergo an
E. Pseudotuberculosis operation on inguinal hernia. What tactics
of hypoglycemic therapy should be used?
34. A 7 y.o. girl has mild form of varicella.
Headache, weakness, vertigo, tremor of A. Prescribe fast-acting insulin
her limbs, ataxia, then mental confusion B. Give Glurenorm in place of Maninilum
appeared on the 5th day of illness. Meni- C. Continue with the current therapy
ngeal signs are negative. Cerebrospinal D. Prescribe long-acting insulin
fluid examination is normal. How can you E. Prescribe guanyl guanidines
explain these signs?
39. A 33 y.o. woman has been suffering
A. Encephalitis from DM (diabetes mellitus) for 5 years.
B. Meningitis For the last 3 years she has been taki-
C. Meningoencephalitis ng more than 100 units of insulin per
D. Myelitis day. Body weight has increased up to 10
E. Neurotoxic syndrome kg. Fasting blood glucose is 13 mmol/L,
glucoseuria - 3%. Generalized microangi-
35. A mother with an infant visited the opathy. By increasing the dose of insulin
pediatrician for expertise advice. Her the parameters of glycemia do not change.
baby was born with body weight 3,2 kg The diagnosis is:
and body length 50 cm. He is 1 year old
now. How many teeth the baby should A. DM 1st type, severe form,
have? decompensation, insulin resistant
B. DM 2nd type, severe form,
A. 8 decompensation
B. 10 C. DM st type, severe form,
C. 12 subcompensation, Somoji phenomenon
D. 20 D. DM 2nd type, moderate form, Zabrodi
E. 6 phenomenon
E. DM 1st type, severe form,
36. A boy of 7 y.o. had an attack of asthma decompensation, allergic reaction to
and distant whistling rales after playing insulin
with a dog. In the medical hystory: atopic
dermatitis caused by eating eggs, chicken, 40. A 32 y.o. man is divorced, has an
beef. What group of allergens is the reason irregular sexual life. He complains of falli-
of the development of bronchial astma ng out of hair in the region of eyelashes,
attacks? eyebrows, scalp. Objectively: diffuse
alopecia is observed, eyebrow margin is
absent, eyelashes are stair-like (Pinkus’ si-
Krok 1 Medicine 2006 6

gn). What examination should be carried 45. A 28 y.o. man fell seriously ill, he feels
out first of all? chill, has got a fever, body temperature
raised up to 38, 50, paroxysmal pain in
A. Wasserman test, IFT the left iliac region, frequent defecation
B. T.pallidum Immobilization Test (TPI) in form of fluid bloody and mucous mass.
C. Detection of the nasal mucous Abdomen palpation reveals painfulness
membrane for Micobacterium Leprae in its left half, sigmoid colon is spasmed.
Hansen What is the most probable diagnosis?
D. Consultation of neuropathist
E. CBC A. Acute dysentery
B. Amebiasis
41. A triad of symptoms (“stearing spot”, C. Colibacillosis
“terminal film”, “blood dew”) have been D. Nonspecific ulcerative colitis
revealed on examination of a patient. E. Malignant tumors of large intestine
What disease should you think about?
46. The disease of a 21 y.o. patient began
A. Psoriasis with raise of temperature up to 39, 00,
B. Lichen ruber planus headache, chill, repeated vomiting. Rigi-
C. Vasculitis dity of occipital muscles is determined.
D. Seborrhea The analysis of liquor revealed: cytosis -
E. Ritter’s disease 1237 in 1 ml, including: 84% of neutrophi-
ls, 16% of lymphocytes. On bacterioscopy:
42. A 4 y.o. child attends the kindergarten. gram-negative cocci are found in liquor.
Complains of poor appetite, fatigue.
What is the most probable disease?
Objective examination: skin and mucous
membrane are pale, child is asthenic. In A. Meningococcal infection: purulent
the hemogram: hypochromatic anemia meningitis
1st, leucomoide reaction of the eosinophi- B. Meningococcal infection: serous meni-
le type. What pathology must be excluded ngitis
first of all? C. Secondary purulent meningitis
D. Serous meningitis
A. Helminthic invasion
B. Lymphoprolipherative process E. Infectious mononucleosis
C. Hypoplastic anemia 47. The family doctor examined a patient
D. Duodenal ulcer and diagnosed an acute bleeding of an
E. Atrophic gastritis intestine. What is professional tactics of
the doctor in this situation?
43. A 36 y.o. woman is in the 12-th week
of her first pregnancy. She was treated for A. The urgent hospitalization in surgical
infertility in the past. She contacted a chi- department
ld who fell ill with rubella 2 days after B. To inject intravenously the ami-
their meeting. Woman doesn’t know if she nocapronic acid
has ever been infected with rubella. What C. The urgent hospitalization in
is the adequate tactics? therapeutic department
D. Treatment at a day time hospital
A. Monitoring of the specific IgG IgM with
E. Treatment at home
the ELISA
B. Fetus wastage 48. A 40 y.o. woman is ill with rheumatic
C. Immunoglobulin injection disease with composite mitral disease with
D. Cyclovin prescription prevalence of the stenosis of left venous
E. Interferon prescription foramen.Complains of palpitation, fati-
gability, progressing dyspnea, attacks of
44. A 32 y.o. woman has got the Laiel’s
dyspnea and hemoptysis. Now she cannot
syndrome after taking the biceptol. What be engaged even in the easy activities.
immunotrope medicines are to be prescri-
What tactics is the most expedient?
bed in this situation?
A. Mitral comissurotomia
A. Steroid immunosupressants B. Conduction of current bicilino-
B. Non-specific immune modulators
prophilaxis
C. Specific immune modulators
C. Prescription of anticoagulants
D. Interferons D. Prescription of venous vasodilatators
E. Non-steroid immunosupressants
E. -
Krok 1 Medicine 2006 7

49. A man, 42 years old, died in a road suppressed by 1 tablet of nitroglycerin.


accident after the hemorrhage on the spot, What is the most likely diagnosis?
because of acute hemorrhagic anemia.
What minimum percent of the whole A. Unstable angina pectoris
blood volume could result in death by B. Angina pectoris of a new onset
acute hemorrhage? C. Myocarditis
D. Radiculitis
A. 25-30% E. Stable angina pectoris of the III functi-
B. 6-9% onal class
C. 10-14%
D. 15-20% 54. A 41 y.o. woman complains of
E. 35-50% weakness, fatigue, fever up to 380 C, rash
on the face skin, pain in the wrists and
50. A 50 y.o. male patient was taken the elbows. On physical examination:
to the emergency department with di- erythematous rash on the cheeks with
ffuse abdominal pain and signs of cardi- "butterfly"look, the wrists and elbow joi-
ovascular collapse. On arrival he had BP- nts are involved symmetrically, swollen,
95/60 mm Hg, Ps- 120/min, diuresis - 20 sensitive, friction rub over the lungs,
ml/h, HgB- 100 g/L, RBC- 2, 1 ∗ 1012 /L. the heart sounds are weak, regular, HR-
The patient needs introduction of: 88/min, BP- 160/95 mm Hg. CBC shows
anemia, leucopenia, lymphopenia; on uri-
A. Crystalloid and colloid ne analysis: proteinuria, leukocyturia,
B. Crystalloid and red blood cells casts. What is the main mechanism of di-
C. Crystalloid and 5% dextrose sease development?
D. 5% dextrose and red blood cells
E. 5% dextrose and colloid A. Production of antibodies to double-
stranded DNA
51. A 58 y.o. man complained of severe B. Production of myocytes antibodies
inspiratory dyspnea and expectorati- C. Production of antibodies to endothelial
on of frothy and blood-tinged sputum. cells
He has been suffering from essential D. Production of myosin antibodies
hypertension and ischemic heart disease. E. Production of antimitochondrial anti-
On examination: acrocyanosis, "bubbli- bodies
ng"breathing, Ps- 30/min, BP- 230/130 mm
Hg, bilateral rales. Choose medicines for 55. A 56 y.o. woman has an acute onset of
treatment. fever up to 390 C with chills, cough, and
pain on respiration in the right side of
A. Morphine, furosemide, nitroprusside her chest. On physical examination: HR-
sodium 90/min, BP- 95/60 mm Hg, Ps- 26/min.
B. Theophylline, prednisolon There is dullness over the right lung. On
C. Albuterol, atropine, papaverine X-ray: infiltrate in the right middle lobe
D. Strophanthine, potassium chloride, of the lung en palpation. What is the di-
plathyphylline agnosis?
E. Cordiamine, isoproterenol
A. Community-acquired lobar pneumonia
52. A patient has got a sudden attack with moderate severity
of severe substernal pain at night. On B. Community-acquired
examination: confusion, pallor of the skin, bronchopneumonia
acrocyanosis, cold sweat, BP- 80/50 mm C. Acute pleurisy
Hg, Ps- 120/min, irregular and weak pulse. D. Acute lung abscess
What condition are these symptoms typi- E. Hospital-acquired lobar pneumonia
cal for?
56. A 29 y.o. woman is critically ill. The
A. Cardiogenic shock illness is presented by high fever, chills,
B. Acute left-side heart failure sweating, aching pain in lumbar area, di-
C. Acute right-side heart failure scomfort during urination and frequent
D. Radicular syndrome voiding. Pasternatsky’s sign is positive in
E. Acute vascular insufficiency both sides. On lab examination: WBC-
53. A 61 y.o. man complained of sneezing 20 ∗ 109 /L; on urine analysis: protein -
and substernal pain on exertion. In the 0,6g/L, leukocyturia, bacteriuria. Your
last 2 weeks such pain appeared at rest, preliminary diagnosis.
with increased frequency, and couldn’t be
Krok 1 Medicine 2006 8

A. Acute pyelonephritis A. Essential hypertension, 2nd stage


B. Exacerbation of chronic pyelonephritis B. Essential hypertension, 1st stage
C. Acute glomerulonephritis C. Symptomatic hypertension
D. Acute cystitis D. Cardiomyopathy
E. Nephrolithiasis E. Ischemic heart disease
57. A 45 y.o. man has complained of having 61. A 30 y.o. man complains of intense
epigastric and right subcostal aching pain, pain, skin reddening in the region of ankle
pruritus, indigestion, dark color of the uri- joint, temperature rise up to 390 . He
ne and acholic stool, fever and significant fell ill suddenly. In the past there were
weight loss for 1 month. On examinati- such onsets that lasted for 5-6 days and
on: jaundice, presence of Curvuasier’s si- didn’t cause any residual changes of the
gn. US scan did not reveal stones in the joint. The skin over the joint is hyperemic,
gallbladder and choledochus. What is the without distinct outlines and infiltrative
most likely diagnosis? bank at the periphery. What is the most
probable diagnosis?
A. Cancer of the pancreas head
B. Gallbladder stones A. Gout
C. Chronic pancreatitis B. Infectional arthritis
D. Chronic cholangitis C. Rheumatoid arthritis
E. Chronic hepatitis D. Erysipelatous inflammation
E. Osteoarthrosis
58. A 27 y.o. man complained of aching
epigastric pain right after meal, heartburn 62. A 6 y.o. asthmatic child was taken
and nausea. Stomach endoscopy revealed to the emergency hospital because of
a large amount of mucus, hyperemia and severe coughing and wheezing for the last
edema of mucous membrane in gastric 24 hours. Physical examination reveals
fundus with areas of atrophy. Make a di- that the child is excitable, has intercostal
agnosis. and suprasternal retractions, expiratory
wheezing throughout all lung fields, RR-
A. Chronic gastritis of type A 60/min. Initial treatment may include the
B. Chronic gastritis of type B prescription of:
C. Peptic ulcer of stomach
D. Chronic gastritis of type C A. Subcutaneous epinephrine
E. Menetrier’s disease B. Parenteral phenobarbital
C. Intravenous fluids in the first 2 h to
59. A 25 y.o. woman complained of fati- compensate water deficiency
gue, hair loss and brittle nails. The exami- D. N-acetyl cysteine and cromolyn by
nation revealed pallor of skin, Ps- 94/min, inhalation
BP- 110/70 mm Hg. On blood count: E. Parenteral gentamicyn
Hb- 90 g/L, RBC- 3, 5 ∗ 1012 /L, C.I.- 0,7;
ESR- 20 mm/h. Serum iron level was 8,7 63. A surgical department admitted a
mcmol/L. What treatment would you ini- newborn boy with foamy discharges from
tiate? nose and mouth, cyanosis attacks. X-ray
examination: blind end of esophagus is
A. Ferrous sulfate orally at the level of the II thoracic vertebra,
B. Iron dextrin injections gastric air bubble is under the left cupula
C. Vitamin B12 intramuscularly of diaphragm. What is the most probable
D. Blood transfusion diagnosis?
E. Packed RBCs transfusion
A. Esophagus atresia, tracheo-esophageal
60. A 60 y.o. woman has had increased BP fistula
up to 210/110 mm Hg for the last 7 years. B. Total esophagus atresia
On examination: heart apex is displaced C. Paraesophageal hernia
to the left. There are signs of left ventri- D. Esophagus atresia without a fistula
cular hypertrophy on ECG. What is the E. Bronchoesophageal fistula
most probable diagnosis?
64. A 30 y.o. man complains of sharp
pain in the right ear, hearing loss, hi-
gh temperature for three days. Objecti-
vely: right ear whispering language - 0,5
m, external ear is intact, otoscopically
- eardrum protrusion, hyperemia and
Krok 1 Medicine 2006 9

swelling, loss of landmarks. What disease cal history registers gradual onset of fever,
is it? malaise, loss of weight. There was nothing
typical about the kind of fever which has
A. Acute purulent otitis media been present for more than 7-10 days and
B. Acute mastoiditis changed quickly. Physical examination di-
C. Chronic secretory otitis media dn’t give evident results. What is the only
D. Chronic purulent otitis media most important examination for excludi-
E. Eustachian tube disfunction ng miliary tuberculosis?
65. A youth, aged 15, from childhood A. Chest X-ray
suffers from atopic dermatitis and allergy B. Liver or bone marrow biopsy
to the shellfish. In the last 3 months C. Tuberculin skin testing
after acquiring aquarium fish rhini- D. Sputum smear and culture of m.
tis, conjunctivitis, itching in the nose tuberculosis
developed. Level of what immunologic E. Bronchoscopy
index should be defined in this case?
69. A patient has got pain in the axillary
A. IgE area, rise of temperature developed 10
B. IgJ hours ago. On examination: shaky gait is
C. IgM evident, the tongue is coated with white
D. IgA coating. The pulse is frequent. The painful
E. Circulating immunocomplexes lymphatic nodes are revealed in the axi-
llary area. The skin is erythematous and
66. A pregnant woman (35 weeks), aged glistering over the lymphatic nodes. What
25, was admitted to the hospital because is the most probable diagnosis?
of bloody discharges. In her medical hi-
story there were two artificial aborti- A. Bubonic plague
ons. In a period of 28-32 weeks there B. Acute purulent lymphadenitis
was noted the onset of hemorrhage and C. Lymphogranulomatosis
USD showed a placental presentation. D. Anthrax
The uterus is in normotonus, the fetus E. Tularemia
position is transversal (Ist position). The
heartbeats is clear, rhythmical, 140 bpm. 70. A 17 y.o. patient complains of acute
What is the further tactics of the pregnant pain in the knee joint and t0 - 380 C. He was
woman care? ill with angina 3 weeks ago. Objectively:
deformation and swelling of the knee joi-
A. To perform a delivery by means of nts with skin hyperemia. Small movement
Cesarean section causes an acute pain in the joints. Which
B. To perform the hemotransfusion and to diagnose is the most correct?
prolong the pregnancy
C. To introduct the drugs to increase the A. Rheumatism, polyarthritis
blood coagulation and continue observati- B. Systemic lupus erythematodes
on C. Reactive polyarthritis
D. Stimulate the delivery by intravenous D. Infectious-allergic polyarthritis
introduction of oxytocin E. Rheumatoid arthritis
E. To keep the intensity of hemorrhage
under observation and after the bleeding 71. A girl is 12 y.o. Yesterday she was
is controlled to prolong the pregnancy overcooled. Now she is complaining on
pain in suprapubic area, frequent painful
67. Studying of pulmonary tuberculosis urination by small portions, temperature
incidence provided data about patients’ is 37, 80 C. Pasternatsky symptom is
socioeconomic living conditions and bad negative. Urine analysis: protein - 0,033
habits. What method allows to estimate g/L, WBC- 20-25 in f/vis, RBC- 1-2 in f/vis.
the impact of these factors on tuberculosis What diagnosis is the most probable?
incidence?
A. Acute cystitis
A. Calculation of correlation coefficient B. Dysmetabolic nephropathy
B. Calculation of correspondence index C. Acute glomerulonephritis
C. Calculation of regression coefficient D. Acute pyelonephritis
D. Standardized index calculation E. Urolithiasis
E. Calculation of reliability coefficient
72. A 56 y.o. patient has worked at the
68. A 15 y.o. girl was examined. Her medi- aluminium plant over 20 years. Within 3
Krok 1 Medicine 2006 10

last years he has got loosening of teeth, A. Primary adrenocortical insufficiency


bone and joint pains, piercing pains in B. Pheochromocytoma
heart area, vomiting. The preliminary di- C. Primary hyperaldosteronism
agnosis is: D. Congenital adrenocortical hyperplasia
E. Diabetes insipidus
A. Fluorine intoxication
B. Mercury intoxication 76. A 52 y.o. hard smoker complains of
C. Lead intoxication persistent cough with purulent sputum
D. Phosphorus intoxication discharge especially in the mornings,
E. Manganese intoxication dyspnea provoked even by slight physi-
cal exercises, wheezing chest, tachypnoe,
73. A 38 y.o. woman complains of a general weakness. He considers himself to
purulent discharge from the left nostril. be ill for 12 years. The foresaid presentati-
The body temperature is 37, 50 C. The pati- ons appear 3-4 times per year usually
ent has been ill for a week and associ- after a common cold and have tendency
ates her illness with common cold. There to progress. What disease do you think
are a pain and tenderness on palpation of about first of all?
her left cheek. The mucous membrane in
the left nasal cavity is red and turgescent. A. Chronic obstructive lung disease
The purulent exudate is seen in the mi- B. Bronchial asthma
ddle meatus in maxillary. What is the most C. Mucoviscidosis (cystic fibrosis)
probable diagnosis? D. Bronchoectatic disease
E. Aspergillosis
A. Acute purulent maxillary sinusitis
B. Acute purulent frontitis 77. 3 weeks ago a patient was ill with
C. Acute purulent ethmoiditis tonsillitis. Clinical examination reveals
D. Acute purulent sphenoiditis edema, arterial hypertension, hematuria,
E. - proteinuria (1,8 g/per day), granular and
erythrocital casts. What is the preliminary
74. Condition of a parturient woman has diagnosis?
been good for 2 hours after live birth:
uterus is thick, globe-shaped, its bottom A. Glomerulonephritis
is at the level of umbilicus, bleeding is B. Cystitis
absent. The clamp put on the umbilical C. Pyelonephritis
cord remains at the same level, when the D. Intestinal nephritis
woman takes a deep breath or she is bei- E. Renal amyloidosis
ng pressed over the symphysis with the
verge of hand, the umbilical cord drows 78. 47 y.o. patient complains of intensive
into the vagina. Bloody discharges from skin itching, jaundice, bone pain. The skin
the sexual tracts are absent. What is the is hyperpigmentated. There are multiple
doctor’s further tactics? xanthelasma palpebrae. The liver is +6
cm enlarged, solid with acute edge. The
A. To do manual removal of afterbirth blood analysis revealed total bilirubin -
B. To apply Abduladze method 160 mkmol/L, direct - 110 mkmol/L, AST-
C. To apply Crede’s method 2,1 mmol/L, ALT- 1,8 mmol/L, alkaline
D. To do curettage of uterine cavity phosphotase - 4,6 mmol/L, cholesterol-
E. To introduct oxitocine intravenously 9,2 mmol/L, antimitochondrial antibodies
M2 in a high titer. What is the probable
75. A 34 y.o. patient has been suffering diagnosis?
from pulmonary tuberculosis for 7 years;
he complains of muscle weakness, weight A. Primary biliary liver cirrhosis
loss, diarrhea, frequent urination. Objecti- B. Primary liver cancer
vely: hyperpigmentation of skin, gums, C. Chronic viral hepatitis B
internal surface of cheeks. AP- 90/58 mm D. Acute viral hepatitis B
Hg. Blood count: RBC- 3, 1 ∗ 1012 /L, E. Alcoholic liver cirrhosis
Hb- 95 g/L, C.I.- 0,92; leukocytes - 9, 4 ∗
109 /L, eosinophils - 7, segmentonuclear 79. In the gynecologic office a 28 y.o.
leukocytes - 45, stab neutrophils - 1, woman complains of sterility within three
years. The menstrual function is not
lymphocytes - 40, monocytes - 7, Na+ - impaired. There were one artificial aborti-
115 mmol/L, + - 7,3 mmol/L. What is the on and chronic salpingo-oophoritis in her
preliminary diagnosis? case history. Oral contraceptives were not
used. Her husband’s analysis of semen
Krok 1 Medicine 2006 11

is without pathology. What diagnostic probable diagnosis?


method will you start from the workup
in this case of sterility? A. Schizophrenia
B. Depressive disorder
A. Hysterosalpingography C. Schizoid personality disorder
B. Hormone investigation D. Autistic personality disorder
C. Ultra sound investigation E. Pick’s disease
D. Diagnostic scraping out of the uterine
cavity 83. A worker was temporarily off work
E. Hysteroscopia because of illness for 16 days, was under
out-patient treatment. The doctor in
80. A patient has undergone an operati- charge issued a sick-list first for 5 days,
on on account of perforated ulcer of then prolonged it for 10 days. Who can
stomach, terminal phase of diffuse peri- further prolong the sick-list of this pati-
tonitis and endotoxic shock. In the post- ent?
operative period he is prescribed arti-
ficial pulmonary ventilation with 60% A. The doctor in charge of the case
oxygen inhalation. Blood gases: 2 - 70- together with the head of department
78 mm Hg, hypoxemy doesn’t ecrease, B. Working ability expertise committee
CVP (central venous pressure) - 150- C. The doctor in charge of the case with
180 mm of water column, AP- 90/60 mm the permission of the head of department
Hg (against the backgound of taking big D. Deputy head physician on the working
doses of dopamine). Radiogram shows di- ability expertise
ffuse pulmonary infiltration. What cause E. The head of department
the refractory arterial hypoxemia? 84. The results of a separate diagnostic
A. Respiratory distress syndrome curettage of the mucous of the uterus’
B. Bilateral pneumonia cervix and body made up in connecti-
C. Pneumothorax on with bleeding in a postmenopausal
D. Mendelson’s syndrome period: the scrape of the mucous of the
E. Pulmonary edema cervical canal revealed no pathology, in
endometrium - the highly differentiated
81. According to results of medical adenocarcinoma was found. Metastases
and pedagogical observation during are not found. What method of treatment
the gymnastics lesson in the 9th grade is the most correct?
there was plotted a phisiological curve
characterized by gradual increasing of A. Surgical treatment and
pulse rate during the opening part of hormonotherapy
lesson, 80% increase during the main B. Surgical treatment + chemotherapy
part; the curve has 4 waves. How can the C. Surgical treatment and radial therapy
lesson’s structure be assessed? D. Radial therapy
E. -
A. Long interval between exercises
B. The lesson’s structure is correct 85. A 27 y.o. woman complains of havi-
C. Exercise stress is insufficient ng the disoders of menstrual function for
D. Exercise stress is excesive 3 months, irregular pains in abdomen.
E. Exercise stress is adequate On bimanual examination: in the dextral
appendage range of uterus there is an
82. A patient is 16 y.o. In the last year elastic spherical formation, painless, 7
his behaviour has gradually changed: he cm in diameter. USI: in the right ovary
secluded himself, was not interested in - a fluid formation, 4 cm in diameter,
communication with friends, in learning. unicameral, smooth. What method of
He became indifferent towards relatives, treatment is the most preferable?
motivelessly rude, was speaking or laughi-
ng to himself. He answers the questions A. Prescription of an estrogen-gestogen
formally correctly, laconically. Considers complex for 3 months with repeated
himself to be absolutely healthy but a li- examination
ttle tired, says, he’s thinking about writing B. Operative treatment
a book "Projection of humanity on the C. Dispensary observation of the patient
plane of Universe". He always has a copy- D. Anti-inflammatory therapy
book with which is full of a great many E. Chemotherapeutic treatment
of the same daggers. What is the most 86. A 40 y.o. patient complains of yellowi-
Krok 1 Medicine 2006 12

sh discharges from the vagina. Bimanual A. In-patient treatment with common


examination: no pathological changes. regimen
The smear contains Trichomonas vaginalis B. First out-patient treatment, then out-
and blended flora. Colposcopy: two hazy patient treatment
fields on the front labium, with a negative C. Out-patient treatment
Iodum test. Your tactics: D. In-patient treatment with bed rest
E. -
A. Treatment of specific colpitis and with
the subsequent biopsy 90. A 15 y.o. boy was twice attacked
B. Diathermocoagulation of the cervix of by bees, as a result he had severe
the uterus anaphylactic shock. What is the most
C. Specific treatment of Trichomonas effective prophylaxis method?
colpitis
D. Cervix ectomy A. Desensibilisation by means of bee
E. Cryolysis of cervix of the uterus venom extract
B. Prescription of corticosteroids for
87. A full-term new-born suffered from summer
ante- and intranatal hypoxia, was born C. Long-term prophylactic treatment with
in asphyxia (Apgar score 2-5 points). antihistamines
After birth baby’s excitation is progressi- D. Limitation of outside staying during
ng, occurs vomiting, nystagmus, spasms, summer months
squint, spontaneous Babinski and Moro’s E. Protective clothing
reflexes. What is the most probable locati-
on of the intracranial hemorrhage in this 91. A healthy 75 y.o. woman who leads
case? a moderately active way of life went
through a preventive examination that
A. Subarachnoid hemorrhages revealed serum concentration of common
B. Small hemorrhages in brain tissue cholesterol at the rate of 5,1 mmol/L
C. Subdural hemorrhages and HDL (high-density lipoproteins)
D. Periventricular hemorrhages cholesterol at the rate of 70 mg/dl.
E. Hemorrhages in ventricles of brain ECG reveals no pathology. What dietary
recommendation is the most adequate?
88. A 37 y.o. primigravida woman has
been having labor activity for 10 hours. A. Any dietary changes are necessary
Labor pains last for 20-25 seconds every B. Decrease of cholesterol consumption
6-7 minutes. The fetus lies in longitude, C. Decrease of saturated fats consumption
presentation is cephalic, head is pressed D. Decrease of carbohydrates consumpti-
upon the entrance to the small pelvis. on
Vaginal examination results: cervix of E. Increase of cellulose consumption
uterus is up to 1 cm long, lets 2 transverse
fingers in. Fetal bladder is absent. What is 92. In treatment and prevention establi-
the most probable diagnosis? shments, regardless of their organisational
and proprietary form, the rights of pati-
A. Primary uterine inertia ents should be observed. Which of these
B. Secondary uterine inertia rights are the most significant?
C. Normal labor activity
D. Discoordinated labor activity A. The right to the protection of the
E. Pathological preliminary period patient’s interests
B. The right to the free choice
89. A patient at a doctor complains of C. The right to the information
temperature rise up to 38, 20 , edema in D. The right to be heard
the region of his upper lip. Objectively: E. The right to the protection from
upper lip is evidently edematic, in the incompetence
middle of edema there is a cone-shaped 93. A patient has complained of great
swelling. Skin and mucous membrane weakness for 6 years. He fell seriously
over it are dark-red. Diagnosis: labial ill, the illness is accompanied by body
furuncle. A surgeon cut the furuncle, temperature rise, indisposition, pain in
treated the wound with hydrogen peroxi- joints and along the legs muscles. Objecti-
de solution and applied a bandage with vely: violet-bluish erythema around eyes
hypertensive solution. What therapeutic and over knee joints. HR- 120/min, heart
regimen should be recommended? sounds are weak. Blood count: leukocytes
- 12 ∗ 109/L, ESR- 40 mm/h. What is the
Krok 1 Medicine 2006 13

most probable diagnosis? tive; spleen isn’t palpable. Urine is dark,


stool is partly acholic. What is the most
A. Dermatomyositis probable diagnosis?
B. Systemic lupus erythematosus
C. Rheumathoid arthritis A. Virus A hepatitis
D. Atopic dermatitis B. Leptospirosis
E. Reactive polyarthritis C. Infectious mononucleosis
D. Hemolytic anemia
94. A military unit stopped for 3-day’s rest E. Intestinal yersiniosis
in inhabited locality after a long march.
The sanitary-epidemiological reconnai- 98. A full-term newborn child has a di-
ssance found several water sources. It is agnosis Rh-factor hemolytic disease of
necessary to choose the source complying newborn. Bilirubin rate is critical. The chi-
with the hygienic standards for drinking ld’s blood group is В(III), his mother’s
water in the field blood group - А(II). The child has indi-
cation for hemotransfusion. What donor
A. Artesian well water blood must be chosen?
B. Spring water
C. River water A. Blood group В(III), Rh (-)
D. Rain water B. Blood group А(II), Rh (-)
E. Water from melted snow C. Blood group B(III), Rh (+)
95. A district doctor was commisioned D. Blood group А(II), Rh (+)
with a task to work out a plan of E. Blood group О(I), Rh (-)
treatment-and-prophylaxis actions for the 99. At year-end hospital administration
population of his district. What actions
has obtained the following data: annual
of secendary prophylaxis must he include number of treated patients and average
into this plan? annual number of beds used for patient’s
A. Prevention of disease complications treatment. What index of hospital work
B. Disease prevention can be calculated based upon this data?
C. Elimination of disease causes A. Bed turnover
D. Improvement of population’s living
B. Bed resources of the hospital
conditions
C. Average annual bed occupancy
E. Rehabilitation actions D. Average duration of patients presence
96. A 43 y.o. patient complains of formati- in the hospital
on and pain in the right mammary gland, E. Average bed idle time
rise of temperature up to 37, 20 C duri- 100. A 52 y.o. patient fell from 3 m hei-
ng the last 3 months. Condition worsens ght on the flat ground with the right
before the menstruation. On examinati- lumbar area. He complains of pain in
on: edema of the right breast, hyperemia, this area. There is microhematuria in the
retracted nipple. Unclear painful infiltrati- urea. Excretory urography revealed that
on is palpated in the lower quadrants. kidney’s functioning is satisfactory. What
What is the most probable diagnosis? is the most probable diagnosis?
A. Cancer of the right mammary gland A. Kidney’s contusion
B. Right-side acute mastitis B. Subcapsular kidney’s rupture
C. Right-side chronic mastitis C. Multiple kidney’s ruptures
D. Premenstrual syndrome D. Paranephral hematoma
E. Tuberculosis of the right mammary E. Kidney’s abruption
gland
101. A 3 y.o. child with weight defficiency
97. A 20 y.o. patient was admitted to suffers from permanent moist cough. In
the hospital with complaints of having history there are some pneumonias with
skin and sclera icteritiousness, dark uri- obstruction. On examination: distended
ne, single vomiting, appetite loss, body chest, dullness on percussion over the
temperature rise up to 380  for 2 days. lower parts of lungs. On auscultation: a
Three weeks ago he went in for fishing and great number of different rales. Level of
shared his dishes with friends. Objecti- sweat chloride is 80 mmol/L. What is the
vely: the patient is flabby, t0 - 36, 80 , skin most probable diagnosis?
and scleras are icteritious, liver sticks from
under the costal margin by 3 cm, it is sensi-
Krok 1 Medicine 2006 14

A. Mucoviscidosis (cystic fibrosis) - 4,2 g. Biochemical blood analysis reveals


B. Bronchial asthma permanent hypoproteinemia (43,2 g/L),
C. Recurrent bronchitis hypercholesterolemia (9,2 mmol/L). What
D. Bronchiectasis variant of glomerulonephritis is the most
E. Pulmonary hypoplasia probable?
102. A 14 y.o. girl complains of profuse A. Nephrotic
bloody discharges from genital tracts duri- B. Nephritic
ng 10 days after suppresion of menses for C. Isolated urinary
1,5 month. Similiar bleedings recur since D. Hematuric
12 years on the background of disordered E. Mixed
menstrual cycle. On rectal examination:
no pathology of the internal genitalia. In 106. A man, aged 68, complains of
blood: Нb- 70 g/L, RBC- 2, 3 ∗ 1012 /L, Ht- tiredness, sweating, enlargement of
20. What is the most probable diagnosis? cervical, submaxillary and axillary
lymph nodes. Blood test: WBC- 35 ∗
A. Juvenile bleeding, posthemorrhagic 109 /L, lymphocytes - 60%, Botkin and
anemia Gumprecht bodies, level of haemoglobin
B. Werlholf’s disease and quantity of thrombocytes is normal.
C. Polycyst ovarian syndrome Myelogram showed 40% of lymphocytes.
D. Hormonoproductive ovary tumor What is the most probable diagnosis?
E. Noncomplete spontaneous abortion
A. Chronic lympholeucosis
103. A 33 y.o. patient with first time B. Chronic myeloleucosis
detected diabetes mellitus keeps the di- C. Lymphogranulomatosis
et and maintains glycemia at the level of D. Acute leucosis
10,0 mmol/L after meal. He keeps himself E. Tuberculous lymphadenitis
from insulinotherapy. What examination
is the most important for differentiation 107. Examination of a 32 y.o. patient who
of the 1st (insulin-dependent) and the 2nd had apendectomy because of gangrenous
(insulin-independent) types of diabetes? appendicitis revealed clinical picture of
pelvic abscess. What is the best way to
A. Determination of insular cells antibodi- expose pelvic abscess of this patient?
es
B. Glucose-tolerant test A. Through the rectum
C. Glycemia examination on an empty B. Through the postoperative wound
stomach C. By means of the sacral approach
D. Estimation of glucolized blood D. Through the anterior abdominal wall
hemoglobin E. Through the obturator foramen
E. Fructosamine estimation
108. Examination of a just born placenta
104. A 43 y.o. patient had cholecystectomy reveals defect 2x3 cm large. Hemorrhage
6 years ago because of chronic calculous is absent. What tactic is the most
cholecystitis. Lately he has been suffering reasonable?
from pain in the right subcostal area and
recurrent jaundice. Jaundice hasn’t gone A. Manual uretus cavity revision
for the last 2 weeks. Stenoutic papillitis B. Prescription of uterotonic medicines
0,5 cm long has been revealed. What is C. External uterus massage
the best way of treatment? D. Parturient supervision
E. Instrumental uterus cavity revision
A. To perform endocsopic papillosphi-
ncterotomy 109. A 3 y.o. girl has had a temperature
B. To treat conservatively: antibiotics, rise up to 380 , rhinitis, dry superficial
spasmolytics, antiinflammatory drugs cough, flabbiness, appetite loss. Palpati-
C. To perform external choledoch drainage on didn’t reveal any changes over her
D. To perform transduodenal papillosphi- lungs. Percussion sound has a wooden
ncterotomy resonance, auscultation revealed puerile
E. To perform choledochoduodenostomy breathing, no rales. In blood: leukopenia,
lymphocytosis, increased ESR. What is
105. A 7 y.o. boy has been treated in a the most probable diagnosis?
hospital for a month. At the time of admi-
ssion he had evident edemata, proteinuria
- 7,1 g/L, protein content in the daily urine
Krok 1 Medicine 2006 15

A. Acute simple tracheitis probable water pollution?


B. Acute obstructive bronchitis
C. Recurrent bronchitis, acute condition A. 30 m
D. Acute simple bronchitis B. 25 m
E. Bilateral microfocal pneumonia C. 20 m
D. 15 m
110. A 12 y.o. girl took 2 pills of aspirine E. 10 m
and 4 hours later her body temperature
raised up to 39 − 400 . She complai- 114. A 43 y.o. woman complains of contact
ns of general indisposition, dizziness, hemorrhages during the last 6 months. Bi-
sudden rash in form of red spots and bli- manual examination: cervix of the uterus
sters. Objectively: skin lesions resemble of is enlarged, its mobility is reduced. Mi-
second-degree burns, here and there with rrors showed the following: cervix of
erosive surface or epidermis peeling. Ni- the uterus is in the form of cauliflower.
kolsky’s symptom is positive. What is the Chrobak and Schiller tests are positive.
most probable diagnosis? What is the most probable diagnosis?
A. Acute epidermal necrolisis A. Cancer of cervix of the uterus
B. Pemphigus vulgaris B. Polypus of the cervis of the uterus
C. Polymorphous exudative erythema C. Cervical pregnancy
D. Bullous dermatitis D. Nascent fibroid
E. Duhring’s disease E. Leukoplakia
111. A patient complains of pathological 115. A boy, aged 9, was examined: height -
lump, appearing in the right inguinal regi- 127 cm (-0,36), weight - 28,2 kg (+0,96),
on on exercise. The lump is round-shaped, chest circumference - 64,9 cm (+0,66),
4 cm in diameter, on palpation: soft elastic lung vital capacity - 1520 ml (-0,16). What
consistency, is positioned near the medi- is the complex assessment of the child’s
al part of Poupart’s ligament. The lump is physical development?
situated inwards from the spermatic cord.
What is the most probable preliminary di- A. Harmonious
agnosis? B. Disharmonious
C. Apparently disharmonious
A. Right-sided direct inguinal hernia D. Excessive
B. Right-sided oblique inguinal hernia E. Below the average
C. Right-sided femoral hernia
D. Varicose veins of the right hip 116. A 14 y.o. child suffers from
E. Lipoma of the right inguinal area vegetovascular dystonia of pubertal peri-
od. He has got sympathoadrenal atack.
112. A 54 y.o. patient has been sufferi- What medicine should be used for atack
ng from osteomyelitis of femoral bone reduction?
for over 20 years. In the last month
there appeared and gradually progressed A. Obsidan
edemata of lower extremities. Urine B. No-shpa
analysis: proteinuria - 6,6 g/L. Blood C. Amisyl
analysis: disproteinemia in form of D. Euphyline
hypoalbuminemia, increase of α2 - and E. Corglicone
γ-globulins, ESR- 50 mm/h. What is the
most probable diagnosis? 117. A 16 y.o. teenager complains of
weakness, dizziness, sense of heaviness in
A. Secondary renal amyloidosis the left hypochondrium. Objectively: skin
B. Acute glomerulonephritis and visible mucous membranes are icteric.
C. Myelomatosis Steeple skull. Liver +2 cm, the lower pole
D. Chronic glomerulonephritis of spleen is at the level of navel. Blood
E. Systemic lupus erythematosus test: RBC- 2, 7 ∗ 1012/L, Hb- 88 g/L, WBC-
5, 6 ∗ 109 /L, ESR- 15 mm/h. What is the
113. A mineshaft is situated on the terri- most probable reason of bilirubin level
tory of homestead land, it is 20 m away change?
from the house, 10 m - from the toilet and
15 m - from the neighbour’s house. What
is the smallest distance that, according
to the sanitary code, should be establi-
shed between the well and the source of
Krok 1 Medicine 2006 16

A. Increase of unconjugated bilirubin Laparoscopy results: ovaries, peritoneum


B. Increase of conjugated bilirubin of rectouterine pouches and pararectal fat
C. Increase of unconjugated and are covered with "cyanotic spots". What
conjugated bilirubin is the most probable diagnosis?
D. Decrease of conjugated bilirubin
E. Decrease of unconjugated bilirubin A. Widespread form of endometriosis
B. Polycystic ovaries
118. A 28 y.o. woman consulted a C. Chronic salpingitis
doctor about edematic face, moderate D. Genital organs tuberculosis
legs edemata; occasionally her urine E. Ovarian cystoma
has colour of "meat slops". When she
was a teenager she often fell ill wi- 122. A pregnant woman in her 8th week
th angina. Objectively: skin is pallor, was admitted to the hospital for artifici-
body temperature is 36, 80, Ps- 68/min, al abortion. In course of operation during
rhythmic. AP- 170/110 mm Hg. What uri- dilatation of cervical canal of uterus by
ne changes are the most probable? means of Hegar’s dilator № 8 the doctor
suspected uterus perforation. What is the
A. Proteinuria, hematuria, cylindrouria immediate tactics for confirmation of this
B. Increase of relative density, hematuria, diagnosis?
bacteriuria
C. Decrease of relative density, protei- A. Uterine probing
nuria, some urinary sediment B. Bimanual examination
D. Erythrocyturia and urinozuria C. US examination
E. Decrease of relative density, proteinuria D. Laparoscopy
E. Metrosalpingography
119. A 58 y.o. woman had voluminous
bleeding from the ruptured varicous node 123. A 19 y.o. patient was admitted to the
on the left crus. What first aid should be hospital with acute destructive appendici-
provided? tis. He suffers from hemophilia B-type.
What antihemophilic medicine should
A. Heightened position of extremity, be included in pre- and post-operative
compressive sterile bandage treatment plan?
B. Distal tourniquet
C. Proximal turniquet A. Fresh frozen plasma
D. Troyanov-Trendelenburg operation B. Cryoprecipitate
E. Z-shaped stitch on the raptured varicose C. Fresh frozen blood
node D. Native plasma
E. Dried plasma
120. It is determined that 30 of a 100
women with risk factor had preterm labor, 124. A 28 y.o. patient without permanent
and of a 100 women without risk factor 5 residence was admitted to the hospital
women had preterm labor. What method with the preliminary diagnosis influenza.
of statistic data processing should the On the fith day of illness he got a
doctor use in order to estimate reliabili- maculopapular petechial rash on his
ty of differences between the compared body and internal surfaces of extremiti-
groups? es. Body temperature is 410 , euphoria,
face hyperemia, sclera reddening, tongue
A. Student’s criterion calculation tremor, tachycardia, splenomegaly, exci-
B. Average computing tement. What is the most probable di-
C. Relative numbers calculation agnosis?
D. Standardization method
E. Correlation analysis A. Epidemic typhus
B. Delirium alcoholicum
121. A patient was admitted to the hospi- C. Leptospirosis
tal with complaints of occasional pai- D. Measles
ns at the bottom of abdomen that get E. Typhoid fever
worse during menses, weakness, indisposi-
tion, nervousness, some dark bloody di- 125. An 18 y.o. woman consulted a
scharges from vagina on the day before gynecologist about the pain in the lower
and the day after menses. Bimanual part of abdomen, fever up to 37, 50 C,
examination results: body of womb is considerable mucopurulent discharges
enlarged, appendages cannot be determi- from the genital tracts, painful urinati-
ned, posterior fornix has tuberous surface. on. Vaginal examination with mirrors: the
Krok 1 Medicine 2006 17

urethra is infiltrated, cervix of the uterus present. There are positive Ortner’s and
is hyperemic, erosive. The uterus is pai- Hubergrits-Skulski’s symptoms. Urine di-
nful, ovaries are painful, thickened; forni- astase is 320 g/h. What diagnosis is the
xes are free. Bacterioscopy test revealed most probable?
diplococcus. What diagnosis is the most
probable? A. Chronic pancreatitis
B. Acute cholangitis
A. Recent acute ascending gonorrhea C. Chronic cholecystitis
B. Trichomoniasis D. Acute cholecystitis
C. Candydomycosis E. Cancer of pancreas
D. Chronic gonorrhea
E. Chlamydiosis 130. A 48 y.o. patient has been staying in
the emergency department for 2 days on
126. A 28 y.o. patient complains of pain account of acute anteroseptal myocardial
in legs during walking, chill of feet and infarction. In the course of examination
toes. He has been ill for a year. Objecti- he suddenly "snored". There was a tonic
vely: leg skin is pale, cool; turgor is contraction of skeletal muscles; eye pupi-
decreased, hypotrichosis. Femoral and ls dilatated. Pulse on a.carotis is absent.
popliteal artery pulsation is weak, it is What is the immediate tactics?
palpable on the foot arteries only after
nitroglycerine test. Rheographic index is A. Electric defibrillation
<1. What is the most probable diagnosis? B. Saphar’s triple airway maneuver
C. ECG record
A. Obliterating endarteritis D. Intracardiac introduction of adrenalin
B. Chronic thrombophlebitis with atropine
C. Obliterating aterosclerosis E. Precardiac stroke
D. Raynaud’s disease
E. Buerger’s disease 131. A 31 y.o. woman has complained for
3 years of pain and swelling of radiocarpal
127. A 2 m.o. child was delivered in ti- and metacarpophalangeal articulations,
me with weight 3500 g and was on the morning stiffness that lasts up to 1,5
mixed feeding. Current weight is 4900 g. hours. Two weeks ago she felt pain, swelli-
Evaluate the current weight of the child: ng and reddening of knee joints, body
temperature raised up to 37, 50. Exami-
A. Corresponding to the age nation of her internal organs revealed no
B. 150 g less than necessary pathologic changes. Her diagnosis was
C. Hypotrophy of the I grade rheumatoid arthritis. What changes in X-
D. Hypotrophy of the II grade ray pictures of her joints are the most
E. Paratrophy of the I grade probable?
128. A woman consulted a doctor on A. Constriction of joint space, usura
the 14th day after labor about sudden B. Constriction of joint space, subchondral
pain, hyperemy and induration of the left osteosclerosis
mammary gland, body temperature ri- C. Cysts in subchondral bone
se up to 390 , headache, indisposition. D. Multiple marginal osteophytes
Objectively: fissure of nipple, enlargement E. Epiphysis osteolysis
of the left mammary gland, pain on
palpation. What pathology would you thi- 132. 8 hours after a road accident an
nk about in this case? unconscious victim with closed crani-
ocerebral trauma was admitted to the
A. Lactational mastitis hospital. Objectively: anisocoria, wound
B. Lacteal cyst with suppuration in the parietal region - 3,0х1,0 cm,
C. Fibrous adenoma of the left mammary neck muscles rigidity, Ps- 58/min, tense.
gland Convulsive syndrome. What is the most
D. Breast cancer important indication for the immediate
E. Phlegmon of mammary gland surgical procedure?
129. A 60 y.o. woman complains A. Intracranial hemorrhage
of unbearable pains in the right B. Anisocoria
hypochondrium. In the medical hystory: C. Unconsciousness
acute pancreatitis. Body temperature is D. Wounds
38, 20 . Objectively: sclera icteritiousness. E. Intensification of convulsions
No symptoms of peritonium irritation are
Krok 1 Medicine 2006 18

133. A girl, aged 13, consulted the school


doctor on account of moderate bloody A. Correlation coefficient
discharge from the genital tracts, which B. Student’s coefficient
appeared 2 days ago. Secondary sexual C. Coefficient of variation
characters are developed. What is the D. Representation error
most probable cause of bloody discharge? E. Sygmal deviation
A. Menarche 138. A 24 y.o. woman consulted a doctor
B. Juvenile hemorrhage about continued fever, night sweating.
C. Haemophilia She lost 7 kg within the last 3 months.
D. Endometrium cancer She had casual sexual contacts. Objecti-
E. Werlhof’s disease vely: enlargement of all lymph nodes,
hepatolienal syndrome. Blood count:
134. A pregnant woman was registered leukocytes - 2, 2 ∗ 109 /L. What disease can
in a maternity welfare clinic in her be suspected?
11th week of pregnancy. She was bei-
ng under observation during the whole A. HIV-infection
term, the pregnancy course was normal. B. Lymphogranulomatosis
What document must the doctor give the C. Tuberculosis
pregnant woman to authorize her hospi- D. Infectionous mononucleosis
talization in maternity hospital? E. Chroniosepsis
A. Exchange card 139. A 31 y.o. patient has been suffering
B. Appointment card for hospitalization from systemic scleroderma for 14 years.
C. Individual prenatal record She has been treated in hospital many ti-
D. Medical certificate mes. She complains of occasional dull pain
E. Sanitary certificate in the heart region, palpitation, dyspnea,
headache, eye-lid edemata, weight loss
135. In course of observation of sanitary and deformation of extremities joints.
conditions of studying at the technical uni- What organ affection worsens the disease
versity it was necessary to evaluate the vi- prognosis?
sual regimen of students, who study from
9 a.m to 3 p.m. What index of natural light A. Kidneys
will be the most informative? B. Heart
C. Lungs
A. Natural light coefficient D. Gastrointestinal tract
B. Light coefficient E. Skin and joints
C. Depth of study room
D. Time of the room insolation 140. A 2 month old child who was born wi-
E. Presence of mixed (upper-lateral) light th body weight 5100 g has jaundice, hoarse
cry, umbilical hernia, developmental lag.
136. A 70 y.o. man is ill with ischemic His liver is +2 cm, spleen isn’t enlarged.
heart disease. His mood is evidently Stool and urine are of normal color. In
depressed, anxious. As a result of conti- anamnesis: delayed falling-away of umbi-
nious sleeplessness he has got fears, suici- lical rest. Blood count: Hb- 120 g/L,
dal thoughts. He would sit for a long time RBC- 4, 5 ∗ 1012 /L, ESR- 3 mm/h. General
in the same pose, answer after a pause, in level of serum bilirubin - 28 mcmol/L,
a low, monotonous voice. His face has a unconjugated bilirubin - 20 mcmol/L,
look of suffering, pain, fear. What is the conjugated bilirubin - 8 mcmol/L. What
main psychopathologic syndrome? disease would you think about first of all?
A. Depressive syndrome A. Congenital thyreoid deficiency
B. Paranoid syndrome B. Congenital hepatitis
C. Asthenic syndrome C. Hemolitic anemia
D. Phobic syndrome D. Conjugated jaundice
E. Obsessive syndrome E. Cytomegalovirus infection
137. 200 patients suffering from essenti- 141. The doctors in maternity hospi-
al hypertension were examined in order tal made a newborn boy the followi-
to obtain data about patients’ arterial ng diagnosis: congenital heart disease
pressure and age. What statistic value (interventricular septal defect). At the
should be applied in order to measure age of 2 months the boy has got a
relation between these characteristics? dyspnea. Objectively: BR- up to 60/min,
Krok 1 Medicine 2006 19

tachycardia up to 170/min, liver is 3 cm of consciousness, pain in the region of


below the costal margin. What medicines heart. Objectively: HR- 40/min, sounds
must be immediately prescribed? are rhytmic, the 1st sound is dull, occasi-
onally very intensive. AP- 180/90 mm
A. Cardiac glycosides Hg. What is the most probable reason of
B. Nonsteroidal antiinflammatory drugs hemodynamic disorders?
C. Potassium preparations
D. β-adrenoceptor blockers A. III degree atrioventricular heart block
E. Glucocorticoids B. I degree atrioventricular heart block
C. Bradysystolic form of the atrial fibri-
142. A 50 y.o. woman who suffers from llation
chronic pyelonephritis was prescribed a D. Sinus bradycardia
combination of antibiotics for the period E. Complete block of the left branch of
of exacerbation - gentamicin (80 mg 3 ti- His bundle
mes a day) and biseptol (960 mg twice a
day). What consequences may be caused 146. A 32 y.o. patient who has been staying
by such a combination of antibiotics? in a hospital on account of acute abscess
of his right lung suddenly felt pain after
A. Acute renal insufficiency coughing in the right half of thorax, he got
B. Glomerulosclerosis heavy breathing, cyanosis. What compli-
C. Chronic renal insufficiency cation is the most probable?
D. Antibiotic combination is optimal and
absolutely safe A. Pyopneumothorax
E. Acute suprarenal insufficiency B. Infarction-pneumonia
C. Myocardial infarction
143. A 60 y.o. patient complains of having D. Esophagus perforation
passing reduction of strength in his left E. Exudative pleurisy
extremities for a month. Some time later
he has got persistent weakness of extremi- 147. Fluorography of a 45 y.o. man
ties in the mornings. Objectively: AP- revealed some foci of small intensity
140/90 mm Hg, conscious, central paresis with nondistinct outlines on the top of
of the VII and XII pair of left-side crani- his right lung. The patient doesn’t feel
al nerves, central hemiparesis and hemi- worse. He has been smoking for many
hyperesthesia also on the left side. What years. Objectively: vesicular resonance
medicines should be chosen for the di- over lungs, respiration is vesicular, rales
fferentiated treatment of the patient? are absent. Blood count is not changed.
What is the most probable diagnosis?
A. Anticoagulants
B. Hemostatics A. Focal pulmonary tuberculosis
C. Hypotensive B. Peripheral cancer of lung
D. Diuretics C. Eosinophilic pneumonia
E. Corticosteroids D. Bronchiolitis
E. Disseminated pulmonary tuberculosis
144. A 52 y.o. male patient suffers from
squeezing pain attacks in substernal 148. An 18 y.o. patient was admitted to the
area which irradiates to the left hand hematologic department with complai-
and occurs occasionally and on physi- nts of headache, general weakness, poor
cal exercises. He has had it for 1 year. appetite, body temperature rise up to
On examination: heart borders are 390 , neck swelling. Objectively: skin and
enlargement to the left side, sounds are mucous membranes are extremely pale,
muffled, Ps- 76 bpm, rhythmic, AP- 155/80 lymph nodes on the both sides of neck
mm Hg, ECG: the left type, the rest signs are up to 1 cm large, painless. Liver is
are normal. What additional examination enlarged +1 cm, painless, spleen +0,5 cm,
is necessary to confirm the diagnosis? t0 - 380 . Blood count: Нb- 98g/L, RBC-
2, 9 ∗ 1012 /L, leukocytes - 32 ∗ 109 /L, stab
A. Veloergometry neutrophils - 0%, segmental leukocytes
B. Echocardiography - 28%, monocytes - 2%, lymphocytes -
C. Lipoprotein test 39%, blasts - 31%, reticulocytes - 31%,
D. General blood count thrombocytes - 120∗109/L, ESR- 36 mm/h.
E. Transaminases of blood What form of leukosis does the patient
145. A 70 y.o. patient complains of have?
weakness, dizziness, short periods of loss
Krok 1 Medicine 2006 20

A. Acute lymphoblastic leukosis Bouchard’s nodes, painful, stiff. X-ray pi-


B. Acute myeloblastic leukosis cture of joints: joint spaces are constri-
C. Chronic lympholeukosis cted, there are marginal osteophytes,
D. Chronic myeloleukosis subchondral sclerosis. What is the most
E. Undifferentiated leukosis probable diagnosis?
149. A mother of a 5 y.o. girl consulted A. Osteoarthritis
a doctor about doughter’s involuntary B. Reiter’s disease
urination at night, nightmares, sleep di- C. Bechterew’s disease
sorders, slow gaining of body weight. D. Rheumatic arthritis
Objectively: malnutrition, intellectual E. Psoriatic arthritis
development is good, the girl can read and
explains common situations quite adultly. 153. A 42 y.o. man who has been ill with
Her skin is very pale, liver is enlarged in duodenal ulcer for 20 years complains of
size. Her mother suffers from holetithi- getting a sense of heaviness in stomach
asis. What type of diathesis is the most after meal, foul-smelling eructation, vomi-
probable in the child’s case? ting, weight loss. Objectively: his state is
relatively satisfactory, tissue turgor is di-
A. Gouty diathesis minished. On palpation the belly is soft,
B. Urine acid diathesis there are no symptoms of peritenium irri-
C. Exudative diathesis tation, "splashing sounds"in epigastrium.
D. Allergic diathesis Defecation - once in 3 days. What compli-
E. Lymphohypoplastic diathesis cation corresponds with the patient’s state
and described clinical presentations?
150. After manual reposition and appli-
cation of plaster splint a patient with A. Ulcerative pyloric stenosis
fractures of forearm bones had an edema B. Concealed ulcer perforation
of hand and fingers, he felt pain and C. Stomach cancer
lack of sensitivity. What tactics should the D. Ulcer penetration
doctor choose? E. Chronic pancreatitis
A. To cut the bandage that fastens the 154. An 18 y.o. girl complains of weakness,
splint dizziness, loss of appetite, menorrhagia.
B. To prescribe analgetics and diuretics There are many-coloured petechiae on
C. To remove the plaster the skin of the upper extremities. Blood
D. It’s a natural phenomena, the edema test: Hb- 105 g/l; RBC- 3, 2 ∗ 1012 /L; C.I.-
will dissipate in a day 0,95; thromb.- 20 ∗ 109/L. The sedimentati-
E. To repeat reposition on time according to Lee White is 5 ;
151. A 35 y.o. male patient suffers hemorrhagia duration according to Duke
from chronic glomerulonephritis and is 8 , "pinch and tourniquet"test is positi-
has been treated with hemodialysis for ve. What is the most probable diagnosis?
the last 3 years. He has got irregulari- A. Idiopathic thrombocytopenic purpura
ties in the heart activity, hypotension, B. Hemophilia
progressive weakness, dyspnea. On ECG: C. Hemorrhagic diathesis
bradycardia, 1st degree atrioventricular D. Iron deficiency anemia
block, high sharpened T-waves. Some ti- E. Marchiafava-Micheli’s disease
me before he had seriously broken the
water consumption and dietary pattern. 155. A 46 y.o. patient complains of skin
What is the most likely cause of these itch, sweating, especially at night, body
changes? temperature rise up to 38, 60. Objecti-
vely: chest skin has marks of scratchi-
A. Hyperkaliemia ng, supraclavicular lymph nodes are as
B. Hyperhydratation big as a pigeon egg, they are not matted
C. Hypokaliemia together with skin. What test is the most
D. Hypernatremia reasonable?
E. Hypocalcemia
A. Punction of an enlarged lymph node
152. A 60 y.o. patient complains of pain in B. Common blood count
interphalangeal joints of hand that gets C. Plan radiography of thorax
worse during working. Objectively: di- D. Immunogram
stal and proximal joints of the II-IV fi- E. Whole protein and protein fractions
ngers are defigured, with Heberden’s and
Krok 1 Medicine 2006 21

156. A 32 y.o. patient lives in an endemic 160. A 1,5 y.o. child fell seriously ill: chi-
echinococcous region. For the last 6 ll, body temperature rise up to 40, 10,
months he has been having pain in the ri- then rapid dropping to 36, 20 , skin is
ght hypochondrium, temperature rise. An covered with voluminous hemorrhagic
echinococcus liver affection is suspected. rash and purple cyanotic spots. Extremiti-
What type of examination will be the most es are cold, face features are sharpened.
informative in this case? Diagnosis: meningococcosis, fulminant
form, infection-toxic shock. What antibi-
A. USI examination otic must be used at the pre-admission
B. Plan radiography of abdominal cavity stage?
C. Biochemical laboratory analysis
D. Angiography A. Soluble Levomycetine succinate
E. Liver scanning B. Penicillin
C. Lincomycin
157. A 30 y.o. primigravida woman has got D. Gentamycin
intensive labor pains every 1-2 minutes E. Sulfamonometoxin
that last 50 seconds. The disengagement
has started. The perineum with the height 161. A 41 y.o. patient complains of acute
of 4 cm has grown pale. What actions are pain in the right side of the thorax and
necessary in this situation? sudden progress of dyspnea following
the lifting of a heavy object. The pati-
A. Episiotomy ent’s condition is grave: lips and mucous
B. Perineum protection membranes are cyanotic, BR- 28/min, Ps-
C. Perineotomy 122 bpm., AP- 80/40 mm Hg. There is
D. Vacuum extraction of fetus tympanitis on percussion and weakened
E. Expectant management breathing on auscultaion on the right. S2
158. A 50 y.o. patient was admitted to the sound is accentuated above pulmonary
hospital with complaints of blood in urine. artery. What is the main urgent measure
There was no pain or urination disorders, on the prehospital stage?
hematuria has lasted for 3 days. Objecti- A. Air aspiration from the pleural cavity
vely: kidneys are not palpable, suprapubic B. Epinephrine introduction
region has no pecularities, external geni- C. Euphilline introduction
tals have no pathology. Rectal exami- D. Call for cardiologic team
nation revealed: prevesical gland is not E. Oxygen inhalation
enlarged. Cytoscopy revealed no changes.
What disease would you think about first 162. A 38 y.o. woman suffers from
of all? paroxysmal AP rises up to 240/120 mm
Hg accompanied by nausea, vomiting,
A. Cancer of kidney tachycardia, excessive sweating. During
B. Tuberculosis of urinary bladder the onset blood is hyperglycemic. After
C. Varicocele the onset there is voluminous urination.
D. Kidney dystopy Kidneys sonography revealed accessory
E. Necrotic papillitis mass bordering upon the upper pole of the
159. A patient complains of skin pai- right kidney, presumably it belongs to the
nfullness and reddness of the right adrenal gland. What laboratory test will
gastrocnemius muscle. Objectively: body allow to make a more precise diagnosis?
temperature is 38, 50 , enlarged and pai- A. Estimation of catecholamine and vani-
nful inguinal lymph nodes on the right. llylmandelic acid excretion with urine
Skin of extremity is edematic, hyperemic, B. Estimation of insulin and C-peptide
covered with eruption in form of vesi- content in blood
cles containing dark fluid; its palpation is C. Estimation of glomerular filtration rate
painful. There is distinct border between D. Estimation of thyroxin and thyrotropic
normal and hyperemic skin. What is the hormon in blood
most probable diagnosis? E. Estimation of renin content in blood
A. Erysipelas, hemorrhagic form 163. A 48 y.o. farmer was admitted to
B. Anthrax, dermal form the hospital with complaints of headache,
C. Herpetic infection nausea, vomiting, cough with sputum,
D. Chickenpox breath shortage, weak sight, sweating, sali-
E. Crus phlegmon vation. He was cultivated the garden with
Krok 1 Medicine 2006 22

phosphoorganic pesticides. Blood count: veless weakness, rapid fatigability, loss of


RBC- 4, 1 ∗ 1012 /L, Нb- 136 g/L, C.I.- 0,9, appetite, aversion to meat. Two days ago
leukocytes - 13, 0 ∗ 109/L, ESR- 17 mm/h. she had stomach bleeding. Objectively:
His diagnosis is acute intoxication with temperature - 37, 50, BR- 20/min, Ps-
phosphoorganic pesticides. What is the 96/min, AP- 110/75 mm Hg. On palpation
most important diagnostic criterion for in epigastrium - pain and muscle tension.
this pathology? Blood count: Hb- 82 g/L, ESR- 35 mm/h.
What examination will allow to make a
A. Low level of choline esterase diagnosis?
B. Reticulocytosis
C. Leukocytosis A. Cytologic
D. Anemia B. Radiography
E. Thrombocytopenia C. Endoscopy
D. Stomach content examination
164. A 74 y.o. female patient complains of E. Coprology
pain, abdominal distension, nausea. She
suffers from heart ichemia, post-infarction 168. A 42 y.o. woman suffers from mi-
and atherosclerotic cardiosclerosis. On cronodular cryptogenic cirrhosis. Duri-
examination: grave condition, distended ng the last week her condition has been
abdomen, abdominal wall fails to take worsening: she had spasms, consciousness
active part in breathing. On laparoscopy: aberration, jaundice. What examination
some cloudy effusion in abdominal cavity, may account for the worsening of the pati-
one of the bowel loops is dark-blue. What ent’s condition?
is the most probable diagnosis?
A. Estimation of serum ammonia
A. Mesenterial vessels thrombosis B. Estimation of cholesterol esters
B. Volvulus C. Estimation of α-fetoprotein content
C. Acute intestinal obstruction D. Estimation of alanine aminotransferase
D. Ichemic abdominal syndrome and aspartate aminotransferase
E. Erysipelas E. Estimation of alkaline phosphatase level
165. A 30 y.o. woman is in her second 169. A mother of a newborn child suffers
labor that has been lasting for 14 hours. from chronic pyelonephritis. She had
Fetal heartbeats are muffled, arrhythmic, acute respiratory viral disease before
100/min. Vaginal examination results: the labor. Labor in time, with prolonged
cervical dilatation is complete, fetal head period without waters. A child had
is close to the exit of small pelvis. Sagi- erythematous eruption on the 2 day,
ttal suture has the direct diameter, small then there were seropurulent vesicles for
crown is close to the pubis. What is the about 1cm large. Nikolsky’s symptom
further tactics of labor management? is positive. Erosions have occured after
vesicle rupture. The child is flabby. The
A. Use of obstetrical forceps temperature is subfebrile. What is the
B. Labor stimulation by means of oxitocine most probable diagnosis?
C. Cesarean section
D. Craniodermal forceps A. Newborn pemphigus
E. Use of cavity forceps B. Vesiculopustulosis
C. Pseudofurunculosis
166. A 31 y.o. patient has had mental di- D. Sepsis
sorder for a long time. He suffers from E. Ritter’s dermatitis
insomnia for a long time. He has got fears,
suicidal thoughts, tried to hang himself. 170. A 41 y.o. woman has suffered
His mood is depressed, he refuses from from nonspecific ulcerative colitis for
treatment. What measures are the most 5 years. On rectoromanoscopy: evident
expedient for the prevention of suicide? inflammatory process of lower intesti-
nal parts, pseudopolyposive changes of
A. Admission to the mental hospital mucous membrane. In blood: WBC-
B. Admission to the neurological 9, 8∗109/L, RBC- 3, 0∗1012/L, sedimentati-
department on rate - 52 mm/hour. What medication
C. Out-patient treatment provides pathogenetic treatment of this
D. Psychotherapeutic conversation patient?
E. Strict supervision at home
167. A 63 y.o. woman complains of moti-
Krok 1 Medicine 2006 23

A. Sulfasalasine
B. Motilium A. Celiakia (celiac disease)
C. Vikasolum B. Mucoviscidosis (cystic fibrosis)
D. Linex C. Intestinal dysbacteriosis
E. Kreon D. Chronic enteritis
E. Disaccharidase insufficiency
171. A 49 y.o. female patient was admi-
tted to the hospital with acute attacks 175. A child with chronic cardialtis, cardial
of headache accompanied by pulsation insufficiency IIА that is being treated with
in temples, increasing AP up to 280/140 digoxin has got progressing bradycardia,
mm Hg. Pheochromocytoma is suspected. nausea, vomiting. dizziness, sleep di-
What mechanism of hypertensive atack sorders. ECG results: extrasystole, PQ-
does this patient have? 0,18. What is the most probable cause of
this condition?
A. Increasing of catecholamines
concentration A. Cardiac glucosides overdose or
B. Increasing of aldosterone level in blood intolerance
C. Increasing of plasma renin activity B. Pulmonary edema
D. Increasing of vasopressin excretion C. Atrioventricular heart block of the I
E. Increasing of thyroxine excretion degree
D. Acute enteric infection
172. A child was born with body weight E. Hypokaliemia
3250 g and body length 52 cm. At the age
of 1,5 month the actual weight is suffici- 176. Examination of a hanged man’s
ent (4350 g), psychophysical development corpse revealed the following: livores
corresponds with the age. The child is mortis disappear when pressed, restore
breast-fed, occasionally there are regurgi- in 50 seconds, rigor mortis is moderately
tations. What is the cause of regurgitati- evident only in masseteric muscles, neck
ons? muscles and fingers. Body temperature is
31, 00. What is the prescription of death
A. Aerophagia coming?
B. Pylorostenosis
C. Pylorospasm A. 6-7 hours
D. Acute gastroenteritis B. 1-2 hours
E. Esophageal atresia C. 16-24 hours
D. 8-10 hours
173. A 32 y.o. woman has been sufferi- E. 10-18 hours
ng for 5 months from pain in lumbar
region, low grade fever, frequent urinati- 177. In course of herniotomy of a 12 y.o.
on. Urine analysis: moderate proteinuria, patient doctors revealed a testicle in the
leukocytes occupy the whole field of sight, hernial sac. What type of hernia is it?
bacteriuria. Blood analysis: leukocytosis,
increased ESR. What is the most probable A. Congenital oblique inguinal hernia
diagnosis? B. Acquired oblique inguinal hernia
C. Direct inguinal hernia
A. Chronic pyelonephritis D. Femoral hernia
B. Acute glomerulonephritis E. Obturator hernia
C. Chronic glomerulonephritis
D. Acute pyelonephritis 178. Vaginal inspection of a parturient
E. Urolithiasis woman revealed: cervix dilation is up to
2 cm, fetal bladder is intact. Sacral cavi-
174. A 2 y.o. boy was admitted to ty is free, sacral promontory is reachable
the hospital with weight loss, unstable only with a bent finger, the inner surface
feces, anorexia, following the semolina’s of the sacrococcygeal joint is accessible
introduction (since 5 months). The chi- for examination. The fetus has cephalic
ld is adynamic, flabby, his skin is pale presentation. Sagittal suture occupies the
and dry, subcutaneous fat layer is emaci- transverse diameter of pelvic inlet, the
ated. Distended and tensed abdomen, small fontanel to the left, on the side.
tympanitis on percussion of the upper What labor stage is this?
part of abdomen, splashing sounds,
feces are foamy, of light color, foul. On
coprocytogram: a lot of neutral fat. What
is the most probable cause of the disease?
Krok 1 Medicine 2006 24

A. Cervix dilatation stage ng he remembered this condition. What


B. Preliminary stage psychopathologic syndrome is it?
C. Prodromal stage
D. Stage of fetus expulsion A. Delirium
E. Placental stage B. Oneiric syndrome
C. Twilight disorder of consciousness
179. Ambulace brought to the hospi- D. Hallucinatory paranoid syndrome
tal a patient with acute respiratory vi- E. Maniacal syndrome
ral infection. The illness began suddenly
with temperature rise up to 39, 90. He 183. A 19 y.o. boy was admitted to the
complains of headache in frontotemporal hospital with closed abdominal trauma. In
lobes, pain in eyeballs, aching of the course of operation multiple ruptures of
whole body, nose stuffiness, sore throat, spleen and small intestine were revealed.
dry cough. At home he had a nasal AP is falling rapidly, it is necessary
hemorrhage twice. What type of acute to perform hemotransfusion. Who can
respiratory viral infection is it? determine the patient’s blood group and
rhesus compatibility?
A. Influenza
B. Adenoviral infection A. A doctor of any speciality
C. Parainfluenza B. A laboratory physician
D. RS-infection C. A surgeon
E. Enterovirus infection D. A traumatologist
E. An anaesthesilogist
180. A 52 y.o. patient complains of
dyspnea caused even by moderate physi- 184. A 20 y.o. patient complains of
cal activity, cough with hardly secreted amenorrhea. Objectively: hirsutism,
sputum. He has been ill for 12 years. obesity with fat tissue prevailing on the
Objectively: BR- 26/min. Lung exami- face, neck, upper part of body. On the
nation: tympanitis, diminished vesicular face there are acne vulgaris, on the
breathing with prolonged expiration, di- skin - striae cutis distense. Psychological
sseminated dry rales. In the past he and intellectual development is normal.
was taking only theopecym or ami- Gynecological condition: external geni-
nophylline inravenously. Prescribe him tals are moderately hairy, acute vaginal
the basic treatment after the exacerbati- and uterine hypoplasia. What diagnosis is
on is suppressed: the most probable?
A. Atrovent A. Itsenko-Cushing syndrome
B. Alupent B. Turner’s syndrome
C. Inhacort C. Stein-Levental’s syndrome
D. Tilade D. Shichan’s syndrome
E. Aminophylline E. Babinski-Froehlich syndrome
181. Plan radiography of the patient’s 185. A man, aged 30, complains of intense
abdominal cavity reveals some hemi- pain, reddening of skin, edema in the
spherical lucent areas situated above di- ankle-joint area, fever up to 390 . There
stinct horizontal levels. What is the cause was acute onset of the illness. In the past
of such X-ray picture? there were similar attacks lasting 5-6 days
without residual changes in the joint. The
A. Intestinal obstruction skin over the joint is hyperemic without
B. Perforative ulcer definite borders and without infiltrative
C. Meteorism bank on the periphery. What is the most
D. Cancer of large intestine likely diagnosis?
E. Price’s disease
A. Gout
182. A patient of a somatic hospital has B. Infectional arthritis
got psychomotor agitation as a result C. Rheumatoid arthritis
of high fever: he tried to run about the D. Erysipelatous inflammation
department; thought that some water was E. Osteoarthritis
running down the walls, he pretended to
see rats and cockroaches on the floor. 186. A 5 y.o. girl was by accident closed
Claimed he were in a hostel, recognized in a dark room for several minutes. When
his "aquaintances". After introduction of the door was opened, the child was standi-
sedative drugs he fell asleep. In the morni- ng motionless in the middle of the room
Krok 1 Medicine 2006 25

staring at one point, her face had a look of birth canal with mirrors shows absence of
terror, she didn’t respond to any stimuli. 40 lacerations and raptures. What action is
minutes after her state changed into cryi- nesessary?
ng. On the next day she could remember
nothing of this incident. What is the most A. Manual exploration of the uterine
probable mechanism of this reaction? cavity
B. External massage of uterus
A. Psychogenic C. Use of uterine contracting agents
B. Endogenic D. Urine drainage, cold on the lower
C. Exogenously organic abdomen
D. Endogenically organic E. Use of hemostatic medications
E. Conditioned reflex
191. A 25 y.o. patient complains of body
187. A 42 y.o. patient was admitted 3 hours temperature rise up to 370 , pain at the
after a trauma with evident subcutaneous bottom of her abdomen and vaginal di-
emphysema of the upper part of his scharges. Three days ago, when she was
body, dyspnea, tachycardia 120/min. X-ray in her 11th week of pregnancy, she had
examination revealed no pneumothorax, an artificial abortion. Objectibely: cervix
significant dilatation of mediastinum to of uterus is clean, uterus is a little bit
the both sides. What emergency care is enlarged in size, painful. Appendages
needed? cannot be determined. Fornixes are deep,
painless. Vaginal discharges are sangui-
A. Drainage of anterior mediastinum nopurulent. What is the most probable
B. Pleural cavity punction diagnosis?
C. Pleural cavity drainage
D. Toracoscopy A. Postabortion endometritis
E. Toracotomy B. Hematometra
C. Pelvic peritonitis
188. An 8 y.o. boy was ill with B hepatitis D. Postabortion uterus perforation
one year ago. In the last 2 months he has E. Parametritis
complaints of undue fatiguability, sleep di-
sorder, appetite loss, nausea, especially in 192. A 67 y.o. patient complains of
the mornings. Skin isn’t icterious, liver and dyspnea, breast pain, common weakness.
spleen are 1 cm below the costal margins, He has been ill for 5 months. Objecti-
painless. Alanine aminotransferase activi- vely: t0 - 37, 30 , Ps- 96/min. Vocal tremor
ty is 2,2 mcmol/L. How can this condition over the right lung cannot be determined,
be estimated? percussion sound is dull, breathing cannot
be auscultated. In sputum: blood diffusi-
A. Development of chronic hepatitis vely mixed with mucus. What is the most
B. Recurrance of viral hepatitis type B probable diagnosis?
C. Biliary dyskinesia
D. Residual effects of old viral hepatitis A. Cancer of lung
type B B. Macrofocal pneumonia
E. Development of liver cirrhosis C. Bronchoectatic disease
D. Focal pulmonary tuberculosis
189. A 2,5 m.o. child has got muscle E. Exudative pleuritis
hypotony, sweating, occipital alopecia.
Along with massage and therapeutic 193. People who live in the radiation
exercises the child was prescribed vi- polluted regions are recommended to
tamin D. What dosage and frequency are include pectins into their dietary intake
correct? for the radioactive nuclides washout.
What products are the main source of
A. 3000 IU every day pectins?
B. 500 IU every day
C. 1000 IU every day A. Fruit and vegetebles
D. 500 IU every other day B. Bread
E. 1000 IU every other day C. Milk
D. Meat
190. After delivery and revision of E. Macaroni
placenta there was found the defect
of placental lobe. General condition of 194. A 25 y.o. pregnant woman in her
woman is normal, uterus is firm, there is 34th week was taken to the maternity
moderate bloody discharge. Inspection of house in grave condition. She complains
Krok 1 Medicine 2006 26

of headache, visual impairment, nausea. probable diagnosis?


Objectively: solid edemata, AP- 170/130
mm Hg. Suddenly there appeared fibri- A. Premenstrual syndrome
llary tremor of face muscles, tonic and B. Neurasthenia
clonic convulsions, breathing came to C. Renal disease
a stop. After 1,5 minute the breathing D. Mastopathy
recovered, there appeared some bloody E. Disease of cardiovascular system
spume from her mouth. In urine: protein
- 3,5 g/L. What is the most probable di- 198. A 15 y.o. patient has a developmental
agnosis? lag, occasionally he has skin yellowi-
ng. Objectively: spleen is 16х12х10 cm,
A. Eclampsia cholecystolithiasis, skin ulcer of the
B. Epilepsy lower third of left crus. Blood count:
C. Cerebral hemorrhage RBC- 3, 0 ∗ 1012 /L, Hb- 90 g/L, C.I.-
D. Cerebral edema 1,0; microspherocytosis, reticulocytosis.
E. Stomach ulcer Total serum bilirubin is 56 mcmol/L,
unconjugated - 38 mcmol/L. What therapy
195. The Transcarpathian region is will be the most appropriate?
characterized by constant high (over
80%) air moisture. Population of this regi- A. Splenectomy
on feels an intense cold in winter when the B. Spleen transplantation
temperature is temperately low. What way C. Portacaval shunt
of heat emission becomes more active? D. Omentosplenopexy
E. Omentohepatopexy
A. Convection
B. Irradiation 199. In a city with population 400000
C. Evaporation people 5600 fatal cases were recorded,
D. Conduction including 3300 cases because of blood
E. Radiation circulation diseases, 730 - because
of tumors. What index will allow to
196. A 51 y.o. patient complains of having characterize mortality from blood ci-
intensive bloody discharges from vagina rculation diseases in this city?
for 15 days after delay of menstruation
for 2,5 months. In anamnesis: disorders A. Intensive index
of menstrual function during a year, at B. Extensive index
the same time she felt extreme irritability C. Relative intensity index
and had sleep disorders. US examinati- D. Visuality index
on results: uterus corresponds with age E. Correlation index
norms, appendages have no pecularities, 200. During the medical examination a
endometrium is 14 mm thick. What is the port crane operator complained of dizzi-
doctor’s tactics? ness, nausea, sense of pressure against
A. Diagnostic curettage of uterine cavity tympanic membranes, tremor, dyspnoea,
B. Conservative treatment of bleeding cough. He works aloft, the work is
C. Hysterectomy connected with emotional stress. Workers
D. Supravaginal amputation of uterus are affected by vibration (general and
without appendages local), noise, ultrasound, microclimate
E. TORCH-infection test that warms in summer and cools in winter.
What factor are the worker’s complaints
197. An 18 y.o. patient complains of connected with?
painfulness and swelling of mammary
glands, headaches, irritability, edemata A. Infrasound
of lower extremities. These symptoms B. Noise
have been present since the begin of C. Vibration
menarche, appear 3-4 days before regular D. Intensity of work
menstruation. Gynecological examination E. Altitude work
revealed no pathology. What is the most
Krok 2 Medicine 2007 1

1. An 8 y.o. child presents with low- cal management should be:


grade fever, arthritis, colicky abdomi-
nal pain, and a purpuric rash limited to A. Ensuring that the airway is open and
the lower extremities. laboratory studies the patient is oxygenating
reveal a guaiac-positive stool, a urinalysis B. Inserting a tongue blade
with red blood cell (RBC) casts and mild C. Administering an instravenous bolus of
proteinuria, and a normal platelet count. 50% dextrose
The most likely diagnosis is: D. Injecting 5 mg of diazepam followed by
a loading dose of phenytoin
A. Henoch-Schonlein’s vasculitis E. Inducing pentobarbital coma
B. Systemic lupus erythematosus (SLE)
C. Rocky Mountain spotted fever 6. Which gestational age gives the
D. Idiopathic thrombocytopenic purpura most accurate estimation of weeks of
E. Poststreptococcal glomerulonephritis pregnancy by uterine size?

2. A 32 y.o. woman consulted a A. Less that 12 weeks


gynecologist about having abundant B. Between 12 and 20 weeks
long menses within 3 months. Bimanual C. Between 21 and 30 weeks
investigation: the body of the uterus is D. Between 31 and 40 weeks
enlarged according to about 12 weeks E. Over 40 weeks
of pregnancy, distorted, tuberous, of
dense consistence. Appendages are not 7. A number of viable fetuses per 1000
women at the age between 15 and 44 is
palpated. Histological test of the uterus
determined by:
body mucosa: adenocystous hyperplasia
of endometrium. Optimal medical tactics: A. Genital index
B. Reproductive level
A. Surgical treatment
C. Birth rate
B. Hormonetherapy
C. Phytotherapy D. Perinatal rate
E. Obstetric rate
D. Radial therapy
E. Phase by phase vitamin therapy 8. A 21 y.o. man complains of having
morning pains in his back for the last three
3. A woman was hospitalised with full- months. The pain can be relieved during
term pregnancy. In survey: the uterus
the day and after physical exercises. Physi-
is morbid, the abdomen is tense, heart
sounds of the fetus are not auscultated. cal examination revealed reduced mobili-
ty in the lumbar part of his spine, increase
What is the most probable complication
of muscle tonus in the lumbar area and
of pregnancy?
sluch during moving. X-ray pattern of spi-
A. Premature detachment of the normally ne revealed bilateral sclerotic changes in
posed placenta the sacrolumbal part. What test will be
B. Preterm labour the most necessary for confirming a di-
C. Back occipital presentation agnosis?
D. Acute hypoxia of a fetus
A. HLA-B27
E. Hydramnion
B. ESR
4. By the end of the 1st period of physi- C. Rheumatoid factor
ological labour the clear amniotic waters D. Uric acid in blood plasma
were given vent. Contractions lasted 35-40 E. Antinuclear antibodies
sec every 4-5 min. Palpitation of the fetus 9. A 44 y.o. man has acute disarthria,
is 100 bpm. The AP is 140/90 mm Hg. Di-
right-sided Horner’s syndrome, hiccup,
agnosis:
right-sided ataxia, loss of pain sensation
A. Acute hypoxia of the fetus of his face to the right and of his body to
B. Labors before term the left. The man is conscious. Computer
C. Premature detachment of normally tomography of brain is normal. The most
posed placenta reasonable measure will be:
D. Back occipital presentation
E. Hydramnion
5. A 51 y.o. women was taken to the
emergency department in convulsive
status epilepticus. The first means of medi-
Krok 2 Medicine 2007 2

A. Introduction of direct coagulants and A. Echocardiogram


observation B. Catheterization of left pleural cavi-
B. Endarterectomy of the right carotid ty while the outer end of catheter is
artery submerged in water
C. Endarterectomy of the left carotid C. Continued parenteral introduction of
artery fliud in order to raise AP
D. Out-patient observation D. Introduction of peripheral vasodi-
E. Surgical cerebral decompression latators in order to reduce CVP
E. Introduction of loop diuretics in order
10. A 65 y.o. man who has problems with to reduce CVP
urination as a result of benign prostate
gland adenoma dveloped fever and chi- 13. A young man has painful indurati-
ll, hypotension, sinus tachycardia. Skin ons in the peripapillary regions of both
is warm and dry. Clinical blood analysis mammary glands. The most reasonable
revealed absolute granulocytopenia. action will be:
These hemodynamic changes are most li-
kely to be caused by: A. To leave these indurations untouched
B. To remove them
A. Endotoxemia with activation of C. To cut and drain them
complement system D. To take an aspirate for bacterial
B. Secondary reflex vasodilatation as a inoculation and cytology
result of lowered cardiac output E. To administer steroids locally
C. Secondary circulation insufficiency with
retained systolic function as a result of 14. A 34 y.o. woman in her 29-th week
peripheral vasoconstriction of pregnancy, that is her 4-th labor to
D. Reflex vagus stimulation with lowered come, was admitted to the obstetric
cardiac output department with complaints of sudden
E. Secondary endothelial changes as a and painful bloody discharges from vagi-
result of bacterial lesion na that appeared 2 hours ago. The di-
scharges are profuse and contain grumes.
11. A woman in her 39-th week of Cardiac funnction of the fetus is rhytmic,
pregnancy, the second labor, has regular 150 strokes in the minute, uterus tone is
birth activity. Uterine contractions take normal. The most probable provisional
place every 3 minutes. What criteria diagnosis will be:
describe the beginning of the II labor
stage the most precisely? A. Placental presentation
B. Detachment of normally located
A. Cervical dilatation no less than 4 cm placenta
B. Cervical smoothing over 90% C. Vasa previa
C. Duration of uterine contractions more D. Bloody discharges
than 30 seconds E. Disseminated intravascular coagulation
D. Presenting part is in the lower region of syndrome
small pelvis
E. Rupture of membranes 15. A patient is 65 y.o. He has been a
smoker for 40 years. Hew has lost 10
12. A 20 y.o. man has a stab knife wound kg during the last 3 months. Complai-
in the left half of thorax close to ni- ns of pain in the epigastric area after
pple. AP is 90/60 mm Hg, Ps- 130/min, taking meals, diarrhea, jaundice. Physi-
BR- 32/min. During inspiration there is cal examination revealed enlarged, pai-
increase of pulse wave in the region of nless gallbladder. Feces are light-coloured
jugular vein, decrease of peripheral arteri- and clay-like. Blood analysis revealed
al pulse and reduction of AP. Respiratory increased level of whole and direct bili-
murmurs are unchanged. X-ray pattern of rubin, alkaline phosphotase and glutami-
thorax organs has no pecularities. After nepyruvate transferase. Clinical urine
introduction of 2 l of isotonic solution the analysis showed positive bilirubin reacti-
AP stayed low, CVP raised up to 32 cm on and negative urobilinogene reaction.
of water column. The first step in further Where is the initial process that caused
treatment of the patient will be: these changes?
Krok 2 Medicine 2007 3

A. In pancreas measure will be to introduct:


B. In common bile duct
C. In liver A. Lidocain
D. In duodenum B. Flecainid
E. In gallbladder C. Amyodaron
D. Propafenone
16. A 75 y.o. man has acute pain in E. Veropamil
the paraumbilical region accompanied
by vomiting and feeling of abdominal 20. A 23 y.o. woman who suffers from
swelling in approximately 30 minutes insulin-dependent diabetes was admitted
after meals. He lost 10 kg during the last to the acute care department with mental
months because he doesn’t eat in order confusion, inadequate anxious behavi-
to avoid pain. Abdomen examination our, hyperhidrosis, excessive salivation,
reveals no changes in the periods between tachycardia. What examination will be a
pain attacks. Above the right femoral primary task?
artery a murmur can be auscultated, peri-
pheral pulsation in the lower extrimities is A. Blood test for sugar
weak. X-ray examination of stomach and B. Clinical blood analysis
colonoscopy reealed no changes. What is C. Plasma electrolytes test
the leading factor of this pathogenesis? D. Gaseous composition of arterial blood
E. Blood urea and creatinine test
A. Ischemia
B. Psychogenic changes 21. A 40 y.o. woman has changes of
C. Neoplastic process mammary gland. What are the most often
D. Inflammation symtomps that precede the malignizati-
E. Transient obstruction on?

17. A 75 y.o. woman with coronary heart A. Skin induration with inverted nipple
disease constantly takes warfarin. She B. Painful movable induration
was taken to the acute care department C. Painless movable induration
with complaints of sudden sensation of D. Bloody discharges from the nipple
weakness in the left half of her body and E. Pure discharges from the nipple
eyeball deviation to the left. What exami-
nation of the patient will be a primary 22. A patient who takes diuretics has
task? developed arrhythmia as a result of
cardiac glycoside overdose. What is the
A. Computer tomography of brain treatment tactics in this case?
B. Magnetic resonance tomography of
brain A. Increased potassium concentration in
C. Electroencephalogram blood
D. Ultrasonic examination of carotid B. Increased sodium consentration in
arteries blood
E. Spinal punction C. Reduced magnesium concentration in
blood
18. A 65 y.o. patient has acute pain, D. Increased calcium level in blood
paresthesia, paleness of his left extremi- E. -
ty. Pulse in the a. dorsalis pedis is absent.
There is skin coldness and paleness 23. Name a statistical observation unit
that gradually spreads upwards. These for determination of influence amount
symptoms are most likely to be the evi- of bloodsugar on the healing of wound’s
dence of: surface in a postoperative period:

A. Arterial occlusion A. The patient in a postoperative period


B. Thrombophlebitis of superficial veins B. An amount of bloodsugar
C. Hernia of lumbar disc C. Blood analysis
D. Thrombophelebitis of deep veins D. The patient who has a wound surface
E. - E. The patient who was discharged on an
after-care
19. A 58 y.o. patient developed acute
myocardium infarction 4 hours ago, now 24. The parameter of infantile mortality
he is in the acute care department. for the last year was - 16,3, in present year
ECG registers short paroxysms of ventri- - 15,7. Name a kind of the diagram that can
cular tachycardia. The most appropriate be used for a graphic representation of it:
Krok 2 Medicine 2007 4

50%, level of noise 30 dB. What occupati-


A. Stylar onal hazard is the principal one under
B. Linear these conditions?
C. Intrastylar
D. Sector A. Air pollution with anesthetic
E. Radial B. Improper occupational microclimate
C. High level of noise
25. A 25 y.o. patient complains of pain D. Mental overfatigue
in the I finger on the right hand. On E. Compelled working pose
examination: the finger is homogeneously
hydropic, in bent position. On attempt 30. What guarantees against the
to unbend the finger the pain is getti- preconceived attitude to the physician in
ng worse. Acute pain appears during cases of professional law violations do you
the probe in ligament projection. What know?
decease is the most likely?
A. Sanction of public prosecutor, inquiry
A. Thecal whitlow (ligament panaritium) by preliminary investigator of prosecutor’s
B. Subcutaneous panaritium office, committee of experts
C. Articular (joint) panaritium B. Draw up a statement about forensic
D. Bone panaritium medical examination
E. Paronychia C. Conduct an inquiry by preliminary
investigator of police department
26. A 9 y.o. child with diagnosis "chronic D. Utilisation copy of medical documents
tonsillitis"stands dispanserization control. E. Conduct forensic medical examination
Within 1 year of observation there was by district forensic medicine expert
one exacerbation of disease. Physical
condition is satisfactory. The general state 31. A 34 y.o. patient 3 hours ago was bi-
is not infringed. Define group of health: tten by a dog. He has got a non-bleeding
wound in his left arm caused by the dog’s
A. III (a) bite. What surgical care would you provi-
B. II-d de to the patient?
C. I-st
D. III (b) A. Wound bathing with detergent water
E. III (c) and antiseptic application
B. Aseptic bandage
27. An engineer-chemist at the age of 47 C. Cream bandage
often fells ill with an occupational skin di- D. Complete suturing of the wound
sease. Who makes a decision to transfer E. Incomplete suturing of the wound
him to other job accepts?
32. A 37 y.o. patient complains of pain in
A. DCC the right arm which increases during moti-
B. A head physician on, raised body temperature up to 390 C.
C. The attending physician In the right cubital fossa there is a trace of
D. The chief of shop injection, hyperemia and thickening along
E. MSEC the vein. Your diagnosis?
28. A 5 tons milk batch was sampled. A. Phlebit
The lab analysis revealed: fat content 2%, B. Phlegmon
specific density - 1,04 g/cm3 , acidity - 210 Т, C. Abscess
reductase probe - weak-positive. What D. Inflammation of lymph
way is the product to be used in? E. Erysipelas
A. Sell but inform customers about milk 33. A 38 y.o. woman was hospitalized to
quality the surgical unit with acute abdominal
B. Discard for animal feeding pain irradiating to the spine and vomiti-
C. Technical utilization ng. On laparocentesis hemmorhagic fluid
D. Sell without limitations is obtained. What disease is suspected?
E. Do the product away
29. An anestesiologist gives narcosis to the
patient, he uses a non-reversive contour.
Anesthetic is halothane. Air temperature
in the operation room is 210 , humidity
Krok 2 Medicine 2007 5

A. Acute pancreatitis
B. Renal colic A. Determination of the level of
C. Acute enterocolitis Gonadotropins
D. Perforative gastric ulcer B. USI of organs of a small pelvis
E. Acute appendicitis C. Progesteron assay
D. Computer tomography of the head
34. A 40 weeks pregnant woman in intri- E. Determination of the contents of
nsic obstetric investigation: the cervix of a Testosteron-Depotum in Serum of blood
uterus is undeveloped. The oxytocin test
is negative. Upon inspection at 32 weeks 38. For the persons who live in a hot area
it is revealed: AP- 140/90 mm Hg, protei- after an accident at a nuclear object, the
nuria 1 g/l, peripheric edemata. Reflexes greatest risk within the first decade is
are normal. Choose the most correct tacti- represented by cancer of:
cs of guiding the pregnant:
A. Thyroid gland
A. Laborstimulation after preparation B. Skin
B. Strict bed regimen for 1 month C. Reproduction system organs
C. Complex therapy of gestosis for 2 days D. Breast
D. Cesarean section immediately E. Lungs
E. Complex therapy of gestosis for 7 days
39. A 34 y.o. woman in the 10-th week
35. A 41 y.o. patient was admitted to of gestation (the second pregnancy)
the intensive care unit with hemorrhagic consulted a doctor of antenatal clinic with
shock due to gastric bleeding. He has a hi- purpose of statement on the dyspensary
story of hepatitis B during the last 5 years. record. In the previous pregnancy there
The source of bleeding are esophageal vei- took place hydramnion, the child’s birth
ns. What is the most effective method for weight was 4086. What method of exami-
control of the bleeding? nation is necessary for carrying out, first
of all?
A. Introduction of obturator nasogastric
tube A. The test for tolerance to glucose
B. Intravenous administration of pituitrin B. Determination of the contents of
C. Hemostatic therapy fetoproteinum
D. Operation C. Bacteriological investigation of di-
E. Administration of plasma scharge from the vagina
D. A cardiophonography of fetus
36. A woman had the rise of temperature E. USI of the fetus
up to 390  on the first day after labour.
The rupture of fetal membranes took 40. Patient 22 y.o., was admitted to trauma
place 36 hours before labour. The investi- center with complains of pain in the
gation of the bacterial flora of cervix left ankle joint, which increased while
of the uterus revealed hemocatheretic movements and weight bearing. On the
streptococcus of group A. The uterus clinical examination it was found, that the
body is soft, tender. Discharges are bloody, patient had the closed fracture of medial
mixed with pus. Specify the most probable malleolus without displacement. In which
postnatal complication: position the foot has to be fixed in plaster
cast?
A. Metroendometritis
B. Thrombophlebitis of pelvic veins A. At right angle with varus positioning of
C. Infected hematoma the foot
D. Infection of the urinary system B. In position of planter flexion of foot
E. Apostatis of junctures after the episi- C. In position of pronation
otomy D. In position of supination
E. In position of dorsal flexion of foot
37. A 24 y.o. patient 13 months after
the first labour consulted a doctor about 41. The 10 y.o. boy has complains on
amenorrhea. Pregnancy has concluded headache, weakness, fever 400 , vomiting,
by a Cesarean section concerning to a expressed dyspnea, pale skin with flush on
premature detachment of normally posed right cheek, lag of right hemithorax respi-
placenta hemorrhage has made low fi- ratory movement, dullness on percussi-
delity 2000 ml owing to breakdown of on over low lobe of right lung, weakness
coagulability of blood. Choose the most of vesicular respiration in this zone. The
suitable investigation: abdomen is painless and soft at palpati-
Krok 2 Medicine 2007 6

on. Which disease lead to these symptoms A. Encephalitis


and signs? B. Meningitis
C. Meningoencephalitis
A. Pneumonia croupousa D. Myelitis
B. Intestinal infection E. Neurotoxic syndrome
C. Acute appendicitis
D. Acute cholecystitis 46. A 7 y.o. girl fell ill abruptly: fever,
E. Flu headache, severe sore throat, vomiti-
ng. Minute bright red rash appear in
42. The patient with acute respiratory vi- her reddened skin in 3 hours. It is
ral infection (3-rd day of disease) has more intensive in axillae and groin.
complaints on pain in lumbar region, Mucous membrane of oropharynx is
nausea, dysuria, oliguria. Urinalysis - hyperemic. Greyish patches is on the
hematuria (100-200 RBC in eyeshot tonsills. Submaxillary lymph nodes are
spot), specific gravity - 1002. The blood enlarged and painful. What is your di-
creatinin level is 0,18 mmol/L, potassium agnosis?
level - 6,4 mmol/L. Make the diagnosis:
A. Scarlet fever
A. Acute interstitial nephritis B. Measles
B. Acute renal failure C. Rubella
C. Acute glomerylonephritis D. Pseudotuberculosis
D. Acute cystitis E. Enteroviral infection
E. Acute renal colic
47. The child has complains of the "ni-
43. The 7 m.o. infant is suffering from ght"and "hungry"abdominal pains. At fi-
acute pneumonia which was compli- broscopy in area a bulbus ofa duodenum
cated by cardiovascular insufficiency and the ulcerrative defect of 4 mms diameter
respiratory failure of II degree. The is found, the floor is obtected with a fi-
accompanied diagnosis is malnutrition brin, (H.p +). Administer the optimum
of II degree. Choose the best variant of schemes of treatment:
therapy:
A. Omeprasole - Trichopolum - Clari-
A. Ampiox and Amicacin tromicin
B. Macropen and Penicillin B. De-nol
C. Penicillin and Ampiox C. Maalox - Ranitidin
D. Gentamycin and Macropen D. Vicalinum - Ranitidin
E. Ampiox and Polymixin E. Trichopolum
44. A 2 y.o. girl has been ill for 3 48. 6 m.o. infant was born with body’s
days. Today she has low grade fever, mass 3 kg and length 50 cm. He is given
severe catarrhal presentations, slight natural feeding. How many times per day
maculopapular rash on her buttocks and the infant should be fed?
enlarged occipital lymph nodes. What is
your diagnosis? A. 5
B. 7
A. Rubella C. 6
B. Scarlet fever D. 8
C. Measles E. 4
D. Adenoviral infection
E. Pseudotuberculosis 49. A 40 y.o. patient with rheumatic heart
disease complains of anorexia, weakness
45. A 7 y.o. girl has mild form of varicella. and loss of weight, breathlessness and
Headache, weakness, vertigo, tremor of swelling of feet. The patient had tooth
her limbs, ataxia, then mental confusion extraction one month ago. On examinati-
appeared on the 5th day of illness. Meni- on: t0 - 390 C, Ps- 100/min. Auscultation: di-
ngeal signs are negative. Cerebrospinal astolic murmur in the mitral area. Petechi-
fluid examination is normal. How can you al lesion around the clavicle; spleen was
explain these signs? palpable.
Krok 2 Medicine 2007 7

A. Subacute bacteria endocarditis "terminal film", "blood dew") have been


B. Recurrence of rheumatic fever revealed on examination of a patient.
C. Thrombocytopenia purpura What disease should you think about?
D. Mitral stenosis
E. Aortic stenosis A. Psoriasis
B. Lichen ruber planus
50. A patient with nosocomial pneumonia C. Vasculitis
presents signs of collapse. Which of D. Seborrhea
the following pneumonia complications E. Ritter’s disease
is most likely to be accompanied with
collapse? 55. Patient 27 y.o. was hospitalized to the
psychiatric hospital for the 4-th time duri-
A. Septic shock ng 2 years. Heard voices commenting on
B. Exudative pleuritis his actions, had delusions of persecution
C. Bronchial obstruction (was sure that the Mafia wanted to ki-
D. Toxic hepatitis ll him). After a course of treatment with
E. Emphysema neuroleptics was discharged from hospital
with the diagnosis of schizophrenia, state
51. A 38 y.o. patient has been treated in a
of remission. The secondary prevention of
hospital. A fever of 390 C, chest pain which the relapses of schizophrenia requires:
is worsened by breathing, cough, browni-
sh sputum appeared on the 7-th day of the A. Supportive treatment with neuroleptics
treatment. Chest X- ray shows left lower of prolonged action
lobe infiltrate. Which of the following is B. Long-term hospitalization
the treatment of choice for this patient? C. Psychiatric observation
D. Participation in a self-help group
A. Cephalosporins of the III generation E. Psychoanalytic treatment
B. Penicillin
C. Erythromycin 56. A 42 y.o. woman works at
D. Tetracycline the factory on the fabrication of
E. Streptomycin mercury thermometers, complains of the
headache, swoons, reduction of memory,
52. A 62 y.o. patient suffers from DM-2. small and frequent flutter of fingers of
Diabetes is being compensated by diet drawn hands, the eyelids and the tongue,
and Maninilum. Patient has to undergo an bleeding gums, gingivitis. What preparati-
operation on inguinal hernia. What tactics on is it nessesary to use for the elimination
of hypoglycemic therapy should be used? of mercury from the organism?
A. Prescribe fast-acting insulin A. Unithiol
B. Give Glurenorm in place of Maninilum B. Pentoxil
C. Continue with the current therapy C. Magnesium sulphate
D. Prescribe long-acting insulin D. Sodium hydrate of carbon
E. Prescribe guanyl guanidines E. Seduxen
53. A patient of 32 y.o. complains of severe 57. The disease began acutely. The
weakness, tremor of extremities. Objecti- frequent watery stool developed 6 hours
ve examination: body weight loss, wet and ago. The body’s temperature is normal.
warm skin. The thyroid gland is enlarged Then the vomiting was joined. On exami-
up to the 3-rd degree, painless, elastic. Ps- nation: his voice is hoarse, eyes are deeply
108/min. BP- 160/55 mm Hg. There are no sunken in the orbits. The pulse is frequent.
other abnormalties. The diagnosis is: Blood pressure is low. There is no urine.
A. Diffuse toxic goiter of the 3-rd degree, What is the preliminary diagnosis?
thyrotoxicosis of the average degree A. Cholera
B. Diffuse euthyroid goiter of the 3-rd B. Toxic food-borne infection
degree C. Salmonellosis
C. Chronic autoimmune thyroiditis, D. Dysentery
hypertrophic type E. Typhoid fever
D. Chronic fibrous thyroiditis
E. Toxiferous adenoma of the thyroid 58. A 28 y.o. man fell seriously ill, he feels
gland chill, has got a fever, body temperature
raised up to 38, 50, paroxysmal pain in
54. A triad of symptoms ("stearing spot", the left iliac region, frequent defecation
Krok 2 Medicine 2007 8

in form of fluid bloody and mucous mass.


Abdomen palpation reveals painfulness A. Doxazosin
in its left half, sigmoid colon is spasmed. B. Labetalol
What is the most probable diagnosis? C. Phetolamine
D. Propranolol
A. Acute dysentery E. Isoproterenol
B. Amebiasis
C. Colibacillosis 63. A 30 y.o. man presents with a history
D. Nonspecific ulcerative colitis of recurrent pneumonias and a chronic
E. Malignant tumors of large intestine cough production of foul smelling, pi-
rulent sputum, ocassionally glood tinged,
59. The patient was admitted to the which is worse in the morning and on lying
hospital on the 7-th day of the disease down. On physical examination, the pati-
with complaints of high temperature, ent appears chronically ill with clubbing
headache, pain in the muscles, especi- of fingers, inspiratory rales at the base of
ally in calf muscles. The dermal lungs posteriorly. Most likely diagnosis:
integuments and scleras are icteric. There
is hemorrhagic rash on the skin. Urine A. Bronchoectasis
is bloody. The patient went fishing two B. Chronic bronchitis
weeks ago. What is the diagnosis? C. Disseminated pulmonary tuberculosis
D. Pulmonary neoplasm
A. Leptospirosis E. Chronic obstructive emphysema
B. Yersiniosis
C. Salmonellosis 64. The family doctor examined a patient
D. Brucellosis and diagnosed an acute bleeding of an
E. Trichinellosis intestine. What is the doctor’s professi-
onal tactics in this situation?
60. A 43 y.o. patient was admitted to
the hospital with complaints of high A. The urgent hospitalization in to the
temperature of the body and severe surgical department
headache. On examination: carbuncle B. Aminocapronic acid intravenously
is revealed on the forearm. There are C. The urgent hospitalization in to tthe
intense edema around it, insignificant herapeutic department
pain, regional lymphadenitis. The patient D. Treatment at a day time hospital
is a worker of cattle-ranch. What disease E. Treatment at home
is it necessary to think about first?
65. A 19 y.o. girl was admitted to
A. Anthrax Emergency Department: unconsionsness
B. Carcinoma of skin state, cyanosis, myotic pupil, superficial
C. Erysipelas breathing - 12/min. BP- 90/60 mm Hg, Ps-
D. Erysipeloid 78/min. Choose the action corresponding
E. Eczema to this clinical situation:
61. A 16 y.o. female presents with abdomi- A. Controlled respiration
nal pain and purpuric spots on the B. Gastric lavage
skin. Laboratory investigations reveals a C. Oxygen inhalation
normal platelet count, with haematuria D. Caffeine injection
and proteinuria.The most likely diagnosis: E. Cordiamine injection
A. Schonlein-Henoch purpura 66. A 58 y.o. man complaines of severe
B. Haemolytic uraemic syndrome inspiratory dyspnea and expectorati-
C. Thrombotic thrombocytopenic purpura on of frothy and blood-tinged sputum.
D. Heavy metal poisoning He has been suffering from essential
E. Sub acute bacterial endocarditis hypertension and ischemic heart disease.
On examination: acrocyanosis, "bubbli-
62. A 60 y.o. asthmatic man comes for a ng"breathing, Ps- 30/min, BP- 230/130 mm
check up and complains that he is havi- Hg, bilateral rales. Choose medicines for
ng some difficulty in "starting to urinate". treatment.
Physical examination indicates that the
man has blood pressure of 160/100 mm
Hg, and a slight enlarged prostate. Whi-
ch of the following medications would be
useful in treating both of these conditions:
Krok 2 Medicine 2007 9

A. Morphine, furosemide, nitroprusside A. Ultrasound examination of the


sodium gallbladder
B. Theophylline, prednisolon B. Liver function tests
C. Albuterol, atropine, papaverine C. X-ray examination of the gastrointesti-
D. Strophanthine, potassium chloride, nal tract
plathyphylline D. Ultrasound study of the pancreas
E. Cordiamine, isoproterenol E. Blood cell count
67. A 41 y.o. woman complains of 70. A 25 y.o. woman complained of fati-
weakness, fatigue, fever up to 380 C, rash gue, hair loss and brittle nails. The exami-
on the face skin, pain in the wrists and nation revealed pallor of skin, Ps- 94/min,
the elbows. On physical examination: BP- 110/70 mm Hg. On blood count: Hb-
erythematous rash on the cheeks with 90 g/L, RBC- 3, 5·1012/L, C.I.- 0,7; ESR- 20
"butterfly"look, the wrists and elbow joi- mm/h. Serum iron level was 8,7 mcmol/l.
nts are involved symmetrically, swollen, What treatment would you initiate?
sensitive, friction rub over the lungs,
the heart sounds are weak, regular, HR- A. Ferrous sulfate orally
88/min, BP- 160/95 mm Hg. Hematology B. Iron dextrin injections
shows anemia, leucopenia, lymphopenia; C. Vitamin B12 intramuscularly
on urinalysis: proteinuria, leukocyturia, D. Blood transfusion
casts. What is the main mechanism of di- E. Packed RBCs transfusion
sease development?
71. A 38 y.o. woman is seriously ill. She
A. Production of antibodies to double- complains of frequent paroxysms of expi-
stranded DNA ratory dyspnea. The last paroxysm lasted
B. Production of myocytes antibodies over 12 hours and failed to respond to
C. Production of antibodies to endothelial theophylline. The skin is palish gray, moi-
cells st, RR of 26/min. On auscultation, breath
D. Production of myosin antibodies sounds are absent over some areas. Your
E. Production of antimitochondrial anti- preliminary diagnosis?
bodies
A. Bronchial asthma, status asthmaticus
68. A 56 y.o. woman has an acute onset of B. Chronic obstructive bronchitis
fever up to 390 C with chills, cough, and C. Atopic bronchial asthma, respiratory
pain on respiration in the right side of failure of the III degree
her chest. On physical examination: HR- D. Bronchiectasis, respiratory failure of
90/min, BP- 95/60 mm Hg, RR- 26/min. the II-III degree
There is dullness over the right lung on E. Ischemic heart disease, pulmonary
percussion. On X-ray: infiltrate in the ri- edema
ght middle lobe of the lung. What is the 72. A 19 y.o. girl admitted to the hospi-
diagnosis? tal complained of pain in the knee and
A. Community-acquired lobar pneumonia fever of 38, 60 C. She is ill for 2 weeks after
with moderate severity acute tonsillitis. On exam, hyperemia and
B. Community-acquired swelling of both knees, temperature is
bronchopneumonia 37, 40C, HR- 94/min, BP- 120/80 mm Hg,
C. Acute pleurisy and heart border is displaced to the left;
D. Acute lung abscess S1 is weak, systolic murmur is present.
E. Hospital-acquired lobar pneumonia Total blood count shows the following:
Hb- 120 g/L, WBC- 9, 8 · 109 /L, ESR of
69. A 50 y.o. woman for 1 year complai- 30 mm/L. ECG findings: the rhythm is
ned of attacks of right subcostal pain after regular, PQ = 0,24 sec. What is a causative
fatty meal. Last week the attacks have agent of the disease?
repeated every day and become more pai-
nful. What diagnostic study would you A. Beta-hemolytic streptococci
recommend? B. Viral-bacterial association
C. Autoimmune disorder
D. Staphylococci
E. Ricchetsia
73. A 42 y.o. woman complains of dyspnea,
edema of the legs, and tachycardia during
Krok 2 Medicine 2007 10

small physical exertion. Heart borders are


displaced to the left and S1 is accentuated, A. 18-45 days after conception
there is diastolic murmur on apex. The li- B. The first 7 days
ver is enlarged by 5 cm. What is the cause C. 10-14 days after conception
of heart failure? D. 90-120 days after conception
E. The third trimester
A. Mitral stenosis
B. Mitral regurgitation 78. A 7 d.o. boy is admitted to the hospital
C. Tricuspid stenosis for evaluation of vomiting and dehydrati-
D. Tricuspid regurgitation on. Physical examination is otherwi-
E. Aortic stenosis se normal except for minimal hyperpi-
gmentation of the nipples. Serum sodi-
74. A 54 y.o. woman complains of increasi- um and potassium concentrations are 120
ng fatigue and easy bruising of 3 weeks’ meq/L and 9 meq/L respectively. The most
duration. Physical findings included pale, likely diagnosis is:
scattered ecchymoses and petechiae and
mild hepatosplenomegaly. In blood: RBC- A. Congenital adrenal hyperplasia
2, 5 · 1012 /L; Hb- 73 g/L; HCT- 20%; PLT- B. Pyloric stenosis
23 · 109/L; and WBC- 162 · 109/L with 82% C. Secondary hypothyroidism
blasts, that contained Auric rods; peroxi- D. Panhypopituitarism
dase stain was positive. What is the most E. Hyperaldosteronism
probable diagnosis? 79. A 7 y.o. boy has crampy abdomi-
A. Acute leukemia nal pain and a rash on the back of
B. Chronic leukemia his legs and buttocks as well as on
C. Thrombocytopenia the extensor surfaces of his forearms.
D. Hemolytic anemia Laboratory analysis reveals proteinuria
E. Megaloblastic anemia and microhematuria. He is most likely to
be affected by:
75. A 39 y.o. woman complaines of
squeezed epigastric pain 1 hour after A. Anaphylactoid purpura
meal and heartburn. She had been ill B. Systemic lupus erythematosus
for 2 years. On palpation, there was C. Poststreptococcal glomerulonephritis
moderate tenderness in pyloroduodenal D. Polyarteritis nodosa
area. Antral gastritis was revealed on E. Dermatomyositis
gastroscopy. What study can establish 80. A 27 y.o. patient has been havi-
genesis of the disease? ng for almost a year fatigue, hyperhi-
A. Revealing of Helicobacter infection in drosis, heaviness in the left hypochondri-
gastric mucosa um, especially after meals. Objectively:
B. Detection of autoantibodies in the spleen and liver enlargement. In blood:
serum erythrocytes - 3, 2 · 1012 /l, Hb- 100
C. Gastrin level in blood g/l, colour index - 0,87, leukocutes -
D. Examination of stomach secretion 100 · 109 /l, basophils - 7%, eosinophils -
E. Examination of stomach motor function 5%, myelocytes - 15%, juveniles - 16%,
stab neutrophils - 10%, segmentonuclear
76. A male, 50 y.o., has a black flat mole leukocytes - 45%, lymphocytes - 2%,
on the skin of the leg for 10 years. Si- monocytes - 0%, reticulocytes - 0,3%,
nce 4 months ago the shapes of the thrombocytes - 400 · 109 /l, ESR- 25 mm/h.
mole become irregular. What diagnostic What is the most probable diagnosis?
methods should be used?
A. Chronic myeloleukosis
A. Excision biopsy B. Chronic lympholeukosis
B. Fine needle biopsy C. Acute leukosis
C. Smear for microscopic examination D. Erythremia
D. Incision biopsy E. Hepatocirrhosis
E. Thermography
81. A 28 y.o. woman comes to the
77. The highest risk of congenital anomali- Emergency Room with a slightly
es probably occurs when human embryos reddened, painful "knot", 8 cm above
or fetuses are exposed to ionizing radi- the medial malleolus. Examination in
ation. During which part of gestational the standing position demonstrates a di-
period does it occur? stended vein above and below the mass.
Krok 2 Medicine 2007 11

There are no other abnormalities on nd of complications should we think about


physical examination. The most likely di- first of all?
agnosis is:
A. Intestinal haemorrhage
A. Superficial venous thrombosis B. Thrombophlebitis
B. Early deep vein thrombosis C. Meningitis
C. Insect bite D. Nephroso-nephritis
D. Cellulitis E. Hepatite
E. Subcutaneous hematoma
86. The patients has sustained blunt
82. A patient with a history of coronary trauma to the chest. Which of the followi-
artery disease and atrial fibrillation has ng would most likely be the cause of acute
the onset of sudden pain and weakness of cardiopulmonary collapse?
the left leg. Examination reveals a cool,
pale extremity with absent pulses below A. Pneumothorax
the groin and normal contralateral leg. B. Hemothorax
The most likely diagnosis is: C. Pulmonary contusion
D. Rib fractures
A. Arterial embolism E. Acute adult respiratory distress
B. Arterial thrombosis syndrome (ARDS)
C. Acute thrombophlebitis
D. Cerebrovascular accident 87. A patient has got pain in the axillary
E. Dissecting aortic aneurysm area, rise of temperature developed 10
hours ago. On examination: shaky gait is
83. A 30 y.o. man complains of sharp evident, the tongue is coated with white
pain in the right ear, hearing loss, hi- coating. The pulse is frequent. The painful
gh temperature for three days. Objecti- lymphatic nodes are revealed in the axi-
vely: right ear whispering language - 0,5 llary area. The skin is erythematous and
m, external ear is intact, otoscopically glistering over the lymphatic nodes. What
- eardrum protrusion, hyperemia and is the most probable diagnosis?
swelling, loss of landmarks. What disease
is it? A. Bubonic plague
B. Acute purulent lymphadenitis
A. Acute purulent otitis media C. Lymphogranulomatosis
B. Acute mastoiditis D. Anthrax
C. Chronic secretory otitis media E. Tularemia
D. Chronic purulent otitis media
E. Eustachian tube disfunction 88. A 56 y.o. patient has worked at the
aluminium plant over 20 years. Within 3
84. A 22 y.o. man complains of acute last years he has got loosening of teeth,
throat pain, increasing upon swallowing bone and joint pains, piercing pains in
during 3 days. Body temperature 38, 30 , heart region, vomiting. The provisional
neck lymph nodules are slightly enlarged diagnosis is:
and painful. Pharyngoscopically - tonsi-
lar hyperemia, enlargement and edema, A. Fluorine intoxication
tonsils are covered by round yellow fi- B. Mercury intoxication
brinous patches around crypts openings. C. Lead intoxication
Beta-haemolytic streptococcus in swab D. Phosphorus intoxication
analysis. What is the diagnosis? E. Manganese intoxication

A. Acute membranous tonsilitis 89. A 34 y.o. patient has been suffering


B. Acute follicular tonsilitis from pulmonary tuberculosis for 7 years;
C. Pharyngeal diphtheria he complains of muscle feebleness, weight
D. Infectious mononucleosis loss, diarrheas, increased frequency of uri-
E. Pharyngeal candidosis nation. Objectively: hyperpigmentation of
skin, gums, internal cheek surfaces. AP is
85. A patient is staying in the hospital with 90/58 mm Hg. Blood count: erythrocutes
the diagnosis of abdominal typhus. Duri- - 3, 1 · 1012 /L, Hb- 95 g/L, C.I.- 0,92;
ng the 3-d week from the beginning of the leukocytes - 9, 4 · 109 /L, eosinophils
disease the patient stopped keeping di- - 7, segmentonuclear leukocytes - 45,
et and confinement to bed. As a result stab neutrophils - 1, lymphocytes - 40,
the body temperature and rapid pulse monocytes - 7, Na+ - 115 mmole/L, + -
decreased and melena appeared. What ki- 7,3 mmole/L. What is the preliminary di-
Krok 2 Medicine 2007 12

agnosis? A. IHD. Unstable angina


B. Cardialgia due to spine problem
A. Primary insufficiency of adrenal cortex C. IHD. Functional Class II angina
B. Pheochromocytoma D. Myocarditis
C. Primary hyperaldosteronism E. Myocardial dystrophy
D. Congenital adrenal hyperplasia
E. Diabetes insipidus 94. A full-term child survived antenatal
and intranatal hypoxia, it was born in
90. 3 weeks ago a patient was ill with asphyxia (2-5 points on Apgar score).
tonsillitis. Clinical examination reveals After birth the child has progressi-
edema, arterial hypertension, hematuria, ng excitability, there are also vomi-
proteinuria (1,8 g/per day), granular and ting, nystagmus, spasms, strabismus,
erythrocital casts. What is the preliminary spontaneous Moro’s and Babinsky’s
diagnosis? reflexes. What localization of intracrani-
al hemorrhage is the most probable?
A. Glomerulonephritis
B. Cystitis A. Subarachnoid hemorrhage
C. Pyelonephritis B. Small cerebral tissue hemorrhages
D. Intestinal nephritis C. Subdural hemorrhage
E. Renal amyloidosis D. Periventricular hemorrhages
E. Hemorrhages into the brain ventricles
91. A patient complains of feeling heavi-
ness behind his breast bone, periodical 95. The patient 25 y.o. was admitted on the
sensation of food stoppage, dysphagy. 1st day of the disease with complaints of
During the X-ray examination bari- double vision in the eyes, heavy breathi-
um contrast revealed a single saccular ng. The day before the patient ate home-
outpouching of anterodextral esophagus made mushrooms. On objective exami-
wall with regular contours and rigidly nation: paleness, widened pupils, disorder
outlined neck. What is the most probable of swallowing, bradycardia, constipation
diagnosis? are marked. What is the diagnosis?
A. Esophageal diverticulum A. Botulism
B. Cancer of esophagus B. Yersiniosis
C. Hiatal hernia C. Leptospirosis
D. Varix dilatation of esophageal veins D. Salmonellosis, gastrointestinal form
E. Esophageal polyp E. Lambliasis
92. A 40 y.o. patient complains of yellowi- 96. A healthy 75 y.o. woman who leads
sh discharges from the vagina. Bimanual a moderately active way of life went
examination: no pathological changes. through a preventive examination that
The smear contains Trichomonas vaginalis revealed serum concentration of common
and blended flora. Colposcopy: two hazy cholesterol at the rate of 5,1 mmol/l
fields on the front labium, with a negative and HDL (high-density lipoproteins)
Iodum test. Your tactics: cholesterol at the rate of 70 mg/dl.
ECG reveals no pathology. What dietary
A. Treatment of specific colpitis and with recommendation is the most adequate?
the subsequent biopsy
B. Diathermocoagulation of the cervix of A. Any dietary changes are necessary
the uterus B. Decrease of cholesterol consumption
C. Specific treatment of Trichomonas C. Decrease of saturated fats consumption
colpitis D. Decrease of carbohydrates consumpti-
D. Cervix ectomy on
E. Cryolysis of cervix of the uterus E. Increase of cellulose consumption
93. A 52 y.o. patient with previously functi- 97. Laparotomy was performed to a 54
onal Class II angina complains of 5 days y.o. woman on account of big formation
of intensified and prolonged retrosternal in pelvis that turned out to be one-sided
pains, decreased exercise tolerance. Angi- ovarian tumor along with considerable
na is less responsive to Nitroglycerinum. omental metastases. The most appropri-
Which of the following diagnosis is most ate intraoperative tactics involves:
likely?
Krok 2 Medicine 2007 13

A. Ablation of omentum, uterus and both A. Artesian well water


ovaries with tubes B. Spring water
B. Biopsy of omentum C. River water
C. Biopsy of an ovary D. Rain water
D. Ablation of an ovary and omental E. Water from melted snow
metastases
E. Ablation of omentum and both ovaries 102. A 10 y.o. boy with hemophilia has
with tubes signs of acute respiratory viral infection
with fever. What of the mentioned anti-
98. Heart auscultation of a 16 y.o. boy wi- febrile medications are contraindicated to
thout clinical symptoms revealed accent this patient?
of the S II and systolic murmur above
the pulmonary artery. Heart sounds are A. Acetylsalicylic acid
resonant, rhythmic. What is the most B. Analgin
probable diagnosis? C. Pipolphen
D. Paracetamol
A. Functional murmur E. Panadol extra
B. Stenosis of pulmonary artery valve
C. Insufficiency of pulmonary artery valve 103. A full-term newborn child has a di-
D. Nonclosure of Botallo’s duct agnosis newborn’s Rh-factor hemolytic di-
E. Defection of interatrial septum sease. Bilirubin rate is critical. The child’s
blood group is B(III), his mother’s blood
99. A 74 y.o. patient has been sufferi- group - A(II). The child has indication for
ng from hypertension for 20 years. He hemotransfusion. What donor blood must
complains of frequent headache, dizzi- be chosen?
ness, he takes enalapril. Objectively:
accent of the SII above aorta, Ps- 84 bpm, A. Blood group B(III)Rh−
rhythmic, AP- 180/120 mm Hg. What B. Blood group A(II)Rh−
group of hypotensive medications could C. Blood group B(III)Rh+
be additionally prescribed under consi- D. Blood group A(II)Rh+
deration of the patient’s age? E. Blood group O(I)Rh−
A. Thiazide diuretics 104. At year-end hospital administration
B. Loop diuretics has obtained the following data: annual
C. β-adrenoceptor blockers number of treated patients and average
D. α-adrenoceptor blockers annual number of beds used for patient’s
E. Central sympatholytics treatment. What index of hospital work
can be calculated based upon this data?
100. In treatment and prevention establi-
shments, regardless of their organisational A. Bed turnover
and proprietary form, the rights of pati- B. Bed resources of the hospital
ents should be observed. Which of these C. Average annual bed occupancy
rights are the most significant? D. Average duration of patients presence
in the hospital
A. The right to the protection of the E. Average bed idle time
patient’s interests
B. The right to the free choice 105. A 52 y.o. patient fell from 3 m hei-
C. The right to the information ght on the flat ground with the right
D. The right to be heard lumbar area. He complains of pain in
E. The right to the protection from this area. There is microhematuria in the
incompetence urea. Excretory urography revealed that
kidney’s functioning is satisfactory. What
101. A military unit stopped for 3- is the most probable diagnosis?
day’s rest in inhabited locality after a
long march. The sanitary-epidemiological A. Kidney’s contusion
reconnaissance found several water B. Subcapsular kidney’s rupture
sources. It is necessary to choose the C. Multiple kidney’s ruptures
source complying with the hygienic D. Paranephral hematoma
standards for drinking water in the field E. Kidney’s abruption
conditions.
106. A 3 y.o. child with weight defficiency
suffers from permanent moist cough. In
history there are some pneumonias with
Krok 2 Medicine 2007 14

obstruction. On examination: distended stool becomes fatty. Reduced production


chest, dullness on percussion over the of what factor is the most probable cause
lower parts of lungs. On auscultation: a of steatorrhea?
great number of different rales. Level of
sweat chloride is 80 mmol/L. What is the A. Lipase
most probable diagnosis? B. Tripsin
C. Acidity of gastric juice
A. Mucoviscidosis (cystic fibrosis) D. Amylase
B. Bronchial asthma E. Alkaline phosphatase
C. Recurrent bronchitis
D. Bronchiectasis 111. A 54 y.o. woman has been ill with
E. Pulmonary hypoplasia osteomyelitis of femoral bone for over
20 years. During the last month there
107. A 14 y.o. girl complains of profuse appeared and have been steadily increasi-
bloody discharges from genital tracts duri- ng edemata of lower extremities. Urine
ng 10 days after suppresion of menses for analysis revealed: proteinuria - 6,6 g/l.
1,5 month. Similiar bleedings recur since Blood analysis: disproteinemia in form
12 years on the background of disordered of hypoalbuminemia, raise of α2 - and γ-
menstrual cycle. On rectal examination: globulines, ESR- 50 mm/h. What is the
no pathology of the internal genitalia. In most probable diagnosis?
blood: Нb- 70 g/L, RBC- 2, 3 · 1012 /L, Ht-
20. What is the most probable diagnosis? A. Secondary renal amyloidosis
B. Acute glomerulonephritis
A. Juvenile bleeding, posthemorrhagic C. Myelomatosis
anemia D. Chronic glomerulonephritis
B. Werlholf’s disease E. Systematic lupus erythematosus
C. Polycyst ovarian syndrome
D. Hormonoproductive ovary tumor 112. A 43 y.o. woman complains of contact
E. Noncomplete spontaneous abortion hemorrhages during the last 6 months. Bi-
manual examination: cervix of the uterus
108. A 43 y.o. patient had cholecystectomy is enlarged, its mobility is reduced. Mi-
6 years ago because of chronic calculous rrors showed the following: cervix of
cholecystitis. Lately he has been suffering the uterus is in the form of cauliflower.
from pain in the right subcostal area and Chrobak and Schiller tests are positive.
recurrent jaundice. Jaundice hasn’t gone What is the most probable diagnosis?
for the last 2 weeks. Stenoutic papillitis
0,5 cm long has been revealed. What is A. Cancer of cervix of the uterus
the best way of treatment? B. Polypus of the cervis of the uterus
C. Cervical pregnancy
A. To perform endocsopic papillosphi- D. Nascent fibroid
ncterotomy E. Leukoplakia
B. To treat conservatively: antibiotics,
spasmolytics, antiinflammatory drugs 113. After a long periode of subfebrility
C. To perform external choledoch drainage a patient registered increase of dyspnea,
D. To perform transduodenal papillosphi- pain in the right hypochondrium, leg
ncterotomy edemata. Objectively: neck veins are
E. To perform choledochoduodenostomy edematic. Ps is 120 bpm, sometimes it di-
sappears during inspiration. Heart sounds
109. Prevalence of a disease in region N are very weakened. ECG showed low-
amounted 1156 occurences per 1000 of voltage waves of ventricular complex. A
inhabitants. What of the mentioned indi- month ago there was raise of ST V1 − V4
ces characterizes the disease prevalence? segment. Cardiac silhouette is enlarged,
roundish. What is the most probable di-
A. Intensive agnosis?
B. Extensive
C. Ratio A. Exudative pericarditis
D. Visual index B. Small-focal myocardial infarction
E. Standardized C. Postinfarction cardiosclerosis
D. Metabolic postinfection myocardi-
110. A patient suffers from chronic opathy
recurrent pancreatitis with evident di- E. Primary rheumatic carditis
sturbance of exocrinous function. After
intake of rich spicy food and spirits his 114. A 14 y.o. child suffers from
Krok 2 Medicine 2007 15

vegetovascular dystonia of pubertal peri- nts including back. AP is 100/20 mm Hg.


od. He has got sympathoadrenal atack. What is the most probable diagnosis?
What medicine should be used for atack
reduction? A. Opened arterial duct
B. Interventricular septal defect
A. Obsidan C. Isolated stenosis of pulmonary arterial
B. No-shpa orifice
C. Amisyl D. Interatrial septal defect
D. Euphyline E. Valvar aortic stenosis
E. Corglicone
118. A patient was admitted to the hospi-
115. A patient consulted a doctor about tal with complaints of periodical pain
acure respiratory viral infection. The pati- in the lower part of abdomen that gets
ent was acknowledged to be off work. The worse during menses, weakness, malai-
doctor issued him a medical certificate for se, nervousness, dark bloody smears
5 days. The patient is not recovering. What from vagina directly before and after
measures should the doctor take in order menses. Bimanual examination revealed
to legalize the further disability of pati- that uterus body is enlarged, appendages
ent? cannot be palpated, posterior fornix has
tuberous surface. Laparoscopy revealed:
A. To prolong the medical certificate at his ovaries, peritoneum of rectouterine pouch
own discretion but no more than for 10 and pararectal fat have "cyanotic eyes".
days in total What is the most probable diagnosis?
B. To prolong the medical certificate at his
own discretion but no more than for 6 days A. Disseminated form of endometriosis
in total B. Polycystic ovaries
C. To prolong the medical certificate C. Chronic salpingitis
together with department superintendent D. Tuberculosis of genital organs
D. To send the patient to the medical E. Ovarian cystoma
consultative commission
E. To send the patient to the medical social 119. A 19 y.o. patient was admitted to the
expert comission hospital with acute destructive appendici-
tis. He suffers from hemophilia B-type.
116. A 58 y.o. patient complains What antihemophilic medicine should
of weakness, leg edemata, dyspnea, be included in pre- and post-operative
anorexia. He has been suffering from treatment plan?
chronic bronchitis for many years. During
the last 5 years he has been noting intensi- A. Fresh frozen plasma
fied discharge of sputum that is often B. Cryoprecipitate
purulent. Objectively: RR- 80/min, AP- C. Fresh frozen blood
120/80 mm Hg. Disseminated edemata, D. Native plasma
skin is dry and pale, low turgor. In urine: E. Dried plasma
intense proteinuria, cylindruria. Specify
the most probable pathological process in 120. A 28 y.o. patient who has no
kidneys: permanent residence was admitted to
the hospital with preliminary diagnosis
A. Renal amyloidosis "influenza", on the 5-th day of di-
B. Chronic glomerulonephritis sease there are appeared maculopapular
C. Chronic pyelonephritis and petechial rash on his body and
D. Interstitial nephritis internal surfaces of his extremities. Body
E. Acute glomerulonephritis temperature is 410 , euphoria, hyperemic
face, scleras reddening, tongue tremor,
117. A 4 y.o. boy was admitted to the tachycardia, splenomegaly, excitement.
hospital with complaints of dyspnea, What is the most probable diagnosis?
rapid fatigability. His anamnesis regi-
sters frequent respiratory diseases. On A. Spotted fever
percussion: heart borders are dilatated B. Delirium alcoholicum
to the left and upwards. On auscultation: C. Leptospirosis
amplification of the SII above pulmonary D. Measles
artery, a harsh systolodyastolic "machi- E. Typhoid fever
ne"murmur is auscultated between the II
and the III rib to the left of breast bone, 121. An 18 y.o. woman consulted a
this murmur is conducted to all other poi- gynecologist about the pain in the lower
Krok 2 Medicine 2007 16

part of abdomen, fever up to 37, 50C, A. Corresponding to the age


considerable mucopurulent discharges B. 150 g less than necessary
from the genital tracts, painful urinati- C. Hypotrophy of the I grade
on. Vaginal examination with mirrors: the D. Hypotrophy of the II grade
urethra is infiltrated, cervix of the uterus E. Paratrophy of the I grade
is hyperemic, erosive. The uterus is pai-
nful, ovaries are painful, thickened; forni- 125. A 46 y.o. patient complains of coli-
xes are free. Bacterioscopy test revealed cky pain in the right lumbar region that is
diplococcus. What diagnosis is the most irradiating to the lower part of abdomen,
probable? nausea. She didn’t have such pains before.
Survey radiograph of abdominal cavity
A. Recent acute ascending gonorrhea organs didn’t reveal any pathological stai-
B. Trichomoniasis ns. Ultrasonic sonogram revealed in the
C. Candydomycosis enlarged right renal pelvis a hyperechoic
D. Chronic gonorrhea mass approximately 1,5 cm large that gi-
E. Chlamydiosis ves rise to an "ultrasonic track". What is
the most probable diagnosis?
122. An infant is 2 d.o. It was full-term
born with signs of intrauterine infection, A. Renal calculus
that’s why it was prescribed antibiotics. B. Benign tumor of kidney
Specify, why the gap between antibiotic C. Renal cyst
introductions to the new-born children is D. Renal tuberculosis
longer and dosage is smaller compared to E. Malignant tumor of kidney
the older children and adults?
126. A woman consulted a doctor on
A. The newborns have a lower level of the 14-th day after labor about sudden
glomerular filtration pain, hyperemy and induration of the left
B. The newborns have lower concentration mammary gland, body temperature ri-
of protein and albumins in blood se up to 390 , headache, indisposition.
C. The newborns have reduced activity of Objectively: fissure of nipple, enlargement
glucuronil transferase of the left mammary gland, pain on
D. The newborns have diminished blood palpation. What pathology would you thi-
pH nk about in this case?
E. The newborns have bigger hematocrit
A. Lactational mastitis
123. A district doctor keeps the record of B. Lacteal cyst with suppuration
reconvalescents after infectious diseases, C. Fibrous adenoma of the left mammary
people who are disposed to frequent gland
and long-lasting diseases, patients with D. Breast cancer
chronic pathologies. What category of E. Phlegmon of mammary gland
patients should belong to the III health
group? 127. A 42 y.o. patient complains of
weakness, heartbeat, nasal hemorrhages,
A. People with chronic diseases cutaneous hemorrhages. His condition
B. People disposed to frequent and long- has been worsening progressively for a
lasting diseases month. Objectively: grave condition, the
C. People with chronic pathologies and extremities and body skin has spotted and
disposed to frequent and long-lasting petechial hemorrhages, lymph nodes are
diseases not palpable, Ps- 116/min, liver is +2 cm
D. Reconvalescents after infectious di- enlarged, spleen is not palpable. Blood
seases and patients with chronic pathologi- has evident pancytopenia. What disease
es should you think about first of all?
E. All above mentioned categories
A. Hypoplastic anemia
124. A 2 m.o. child was delivered in ti- B. Acute leukosis
me with weight 3500 g and was on the C. Werlhof’s disease
mixed feeding. Current weight is 4900 g. D. Hemorrhagic vasculitis
Evaluate the current weight of the child: E. Acute agranulocytosis
128. A 63 y.o. patient was operated on
account of big multinodular euthyroid
goiter. Despite of techical difficulties a
forced subtotal resection of both parts
Krok 2 Medicine 2007 17

of the thyroid gland was performed. A. Glucocorticoids


On the 4-th day after the operation the B. Antibiotics
woman had cramps of face muscles and C. Nootropics
upper extremities, stomach ache. Positive D. Desensitizing medications
Chvostek’s and Trousseau’s signs. What is E. Vitamins
the most probable cause of such conditi-
on? 133. A 70 y.o. man is ill with ischemic
heart disease. His mood is evidently
A. Insufficiency of parathyroid glands depressed, anxious. As a result of conti-
B. Postoperative hypothyroidism nious sleeplessness he has got fears, suici-
C. Thyrotoxic crisis dal thoughts. He would sit for a long time
D. Injury of recurrent nerve in the same pose, answer after a pause, in
E. Tracheomalacia a low, monotonous voice. His face has a
look of suffering, pain, fear. What is the
129. A girl 13 y.o. consulted the school main psychopathologic syndrome?
doctor on account of moderate bloody
discharge from the genital tracts, which A. Depressive syndrome
appeared 2 days ago. Secondary sexual B. Paranoid syndrome
characters are developed. What is the C. Asthenic syndrome
most probable cause of bloody discharge? D. Phobic syndrome
E. Obsessive syndrome
A. Menarche
B. Juvenile hemorrhage 134. A 32 y.o. patient has been suffering
C. Haemophilia from systematic scleroderma for 14 years.
D. Endometrium cancer She was repeatedly exposed to treatment
E. Werlhof’s disease in the in-patient department. Complains
of periodical dull cardiac pain, dyspnea,
130. A 30 y.o. victim of fire has thermal headache, eyelid edemata, weight loss,
burns of III-A and III-B degree that pain and deformation of extremities joi-
amount 20% of total skin coverlet. AP nts. What organ’s lesion deteriorates the
is 110/70 mm Hg, HR- 120/min. What prognosis for the disease?
transfusion means shoul be used for blind
infusion correction during transportation? A. Kidneys
B. Heart
A. Salines C. Lungs
B. Polyglucine D. Gastrointestinal tract
C. 10% glucose solution E. Skin and joints
D. Fresh frozen plasma
E. Albumin 135. A 2 m.o. child with birth weight
5100 g has jaundice, hoarse cry, umbi-
131. In course of observation of sanitary lical hernia, physical development lag.
conditions of studying at the technical uni- Liver is +2 cm enlarged, spleen is not
versity it was necessary to evaluate the vi- enlarged. In anamnesis: delayed falling-
sual regimen of students, who study from away of umbilical cord rest. In blood:
9 a.m to 3 p.m. What index of natural light Hb- 120 g/L, erythrocytes - 4, 5 · 1012 /L,
will be the most informative? ESR- 3 mm/h. Whole serum bilirubin is 28
mcmole/L, indirect - 20 mcmole/L, direct
A. Natural light coefficient - 8 mcmole/L. What is the most probable
B. Light coefficient diagnosis?
C. Depth of study room
D. Time of the room insolation A. Congenital hypothyreosis
E. Presence of mixed (upper-lateral) light B. Congenital hepatitis
C. Hemolitic anemia
132. A 25 y.o. man who has been suffering D. Conjugated jaundice
from disseminated sclerosis for 4 years E. Cytomegalovirus infection
complains of increasing unsteadyness,
weakness of his lower extremities, uri- 136. A 13 y.o. girl complains of having
nary retention. Objectively: central temperature rises up to febrile figures for
tetraparesis. Cerebellar ataxia. Disturbed a month, joint ache, periodical skin rash.
function of pelvic organs. What is the most Examination revealed steady enhanci-
appropriate therapy in this case? ng of ESR, LE-cells. What is the most
probable diagnosis?
Krok 2 Medicine 2007 18

A. Systematic lupus erythematosus A. Cereals - wheat, oats


B. Juvenile rheumatoid arthritis B. Milk and dairy produce
C. Systematic scleroderma C. Fruit
D. Acute lymphoblast leukosis D. Animal protein
E. Rheumatics E. Digestible carbohydrates
137. A 50 y.o. woman who suffers from 141. Fluorography of a 45 y.o. man
chronic pyelonephritis was prescribed a revealed some little intensive foci with
combination of antibiotics for the period indistinct outlines on the top of his ri-
of exacerbation - gentamicin (80 mg 3 ti- ght lung for the first time. The patient
mes a day) and biseptol (960 mg twice a doesn’t feel worse. He has been smoking
day). What consequences may be caused for many years. Objectively: pulmonary
by such a combination of antibiotics? sound above lungs on percussion, respi-
ration is vesicular, no rales. Blood count
A. Acute renal insufficiency is unchanged. What is the most probable
B. Glomerulosclerosis diagnosis?
C. Chronic renal insufficiency
D. Antibiotic combination is optimal and A. Focal pulmonary tuberculosis
absolutely safe B. Peripheral cancer of lung
E. Acute suprarenal insufficiency C. Eosinophilic pneumonia
D. Bronchopneumonia
138. A patient has been suffering from E. Disseminated pulmonary tuberculosis
morning cough accompanied by discharge
of small amount of sputum, dyspnea for 8 142. An 8 y.o. boy complains of constant
years. He has been smoking for 10 years. cough along with discharge of greenish
Objectively: cyanosis, prolonged expirati- sputum, dyspnea during physical activi-
on, dry rales. What is the most probable ties. At the age of 1 year and 8 months
diagnosis? he fell ill for the first time with bilateral
pneumonia that had protracted course.
A. Chronic obstructive bronchitis Later on there were recurrences of the
B. Chronic non-obstructive bronchitis disease 5-6 times a year, during the remi-
C. Idiopatic fibrosing alveolitis ssion periods there was constant producti-
D. Multiple bronchiectasis ve cough. What examination results will
E. Bronchial asthma be the most important for making a final
diagnosis?
139. A 70 y.o. patient complains of
weakness, dizziness, short periods of A. Bronchography
unconsciousness, pain in the cardiac area. B. Roentgenography of thorax organs
Objectively: HR- 40 bpm, heart sounds C. Bacterial inoculation of sputum
are rhythmic, the S1 is dull, periodically D. Bronchoscopy
amplified. AP is 180/90 mm Hg. What is E. Spirography
the most probable cause of hemodynamic
disturbances? 143. A 35 y.o. patient who suffers
from chronic glomerulonephritis and has
A. Atrioventricular block type III been hemodialysis-dependent for the
B. Atrioventricular block type I last three years developed intermissions
C. Bradysystolic form of ciliary arrhythmia of heart activity, hypotension, increasi-
D. Sinus bradycardia ng weakness, dyspnea. ECG showed
E. Complete left bandle-branch block bradycardia, atrioventricular block type
I, high pointed waves T. The day before
140. A child is 1 y.o. Within the last months the flagrant violation of diet took place.
after the begining of supplemental feeding What is the most probable cause of these
the child has appetite loss, diarrhea with changes?
massive defecation, sometimes vomiting.
Objectively: body temperature is normal. A. Hyperkaliemia
Body weight is 7 kg. Evident pallor of B. Hyperhydratation
skin, leg edemata, enlarged abdomen. C. Hypokaliemia
Coprogram shows a lot of fatty acids and D. Hypernatriemia
soaps. The child was diagnosed with celi- E. Hypocalciemia
ac disease and prescribed gluten-free diet.
What shoul be excluded from the dietary 144. A 60 y.o. patient cpmplains of
intake in this case? weakness, dizziness, heaviness in the
upper part of abdomen, paresthesia of
Krok 2 Medicine 2007 19

toes and fingers. Objectively: skin icteri- 148. A 43 y.o. patient complains of peri-
tiousness, tongue is crimson, smooth. odical pain attacks in the right half of her
Hepatomegaly. In blood: Hb- 90 g/l, face. The attack is characterized by spasm
erythrocytes - 2, 3 · 1012 /l, reticulocytes of mimetic muscles of the right face’s half,
- 0,2%; color index - 1,2, macrocytosis; reddening of skin on this side. Blood has
Jolly’s bodies, Cabot’s ring bodies. What no pathologies. She was diagnosed with
medication is the most appropriate for right-sided trifacial neuralgia. What medi-
treatment? cation should be prescribed?
A. Vitamin 12 A. Finlepsin
B. Feroplex B. Prednisolone
C. Packed red blood cells C. Actovegine
D. Prednisolone D. Analgine
E. Dyspherol E. Indometacin
145. A 30 y.o. primigravida woman has got 149. A 35 y.o. patient experienced a strong
intensive labor pain every 1-2 minutes that nervous stress that resulted in formati-
lasts 50 seconds. The disengagement has on of reddened and edematic areas on
started. The perineum with the height of the back surface of her hands with further
4 cm has grown pale. What actions are formation of small inflammated nodules,
necessary in this situation? vesicles and then erosions accompanied
by profuse discharge of serous liquid. The
A. Episiotomy process is also accompanied by intense
B. Perineum protection itching. What is the most probable di-
C. Perineotomy agnosis?
D. Vacuum extraction of fetus
E. Expectant management A. Common eczema
B. Allergic dermatitis
146. A doctor of the general practice has C. Microbial eczema
registered the following death causes for D. Common contact dermatitis
the previous year: the first place was taken E. Toxicodermia
by cardiovascular diseases (60%), the
second - by tumors (18%), then - traumas 150. A 42 y.o. patient lifted a heavy object
(8,3%) etc. What diagrams will provide that resulted in acute pain in the right half
the most substantial information about of his chest, increased dyspnea. The pati-
the registered ocurrences? ent’s condition is grave: cyanosis of lips
and mucous membranes, RR is 28 pm, Ps-
A. Pie diagram 122 bpm. On percussion there is tympani-
B. Cartogram tis above the right half of chest, on
C. Line diagram auscultation - stongly diminished breath
D. Circle diagram sounds; accent of the II heart sound above
E. Column diagram the pulmonary artery. AP is 80/40 mm Hg.
What is the main emergency action at the
147. A patient complains of nycturia, pre-admission stage?
constant boring pain in perineum and
suprapubic region, weak urine jet, A. Air aspiration from the pleural cavity
frequent, obstructed and painful urinati- B. Adrenaline introduction
on. He has been ill for several months, C. Aminophylline introduction
pain in perineum appeared after getting D. Calling a cardiological brigade
problems with urination. Rectal exami- E. Oxygen inhalation
nation revealed that prostate is enlarged
(mostly owing to the right lobe), dense, 151. A 74 y.o. patient complains of
asymmetric, its central sulcus is flattened, abdomen pain and sweling, nausea. She
the right lobe is dense, painless, tuberous. suffers from ischemic heart disease,
What disease can it be? postinfarction and atherosclerotic cardi-
osclerosis. Objectively: the patient is in
A. Cancer of prostate grave condition, abdomen is swollen,
B. Prostate sclerosis abdominal wall doesn’t take active part in
C. Urolithiasis, stone of the right lobe of respiration. Laparoscopy revealed a small
prostate amount of muddy effusion in abdominal
D. Prostate tuberculosis cavity, one of the loops of small intestine
E. Chronic congestive prostatitis is dark-cyan. What is the most probable
diagnosis?
Krok 2 Medicine 2007 20

ness, asthenovegetative syndrome. What


A. Thrombosis of mesenteric vessels a severity degree of chronic intoxication
B. Twisted bowels with benzol corresponds with described
C. Acute intestinal obstruction symptoms?
D. Ischemic abdominal syndrome
E. Erysipelas A. Minor
B. Moderate
152. A 10 y.o. child who is at oligoanuretic C. Severe
stage of acute renal insufficiency has got D. Disease is not connected with work
sensations of pricking in the mucous conditions
membrane of oral cavity and tongue, E. -
extremities numbness, reduced reflexes,
respiratory disturbance, arrhythmia. What 156. A 30 y.o. man was always reserved
are these symptoms caused by? by nature. He never consulted psychiatri-
sts. He complains of headache, sensation
A. Hyperkaliemia "as if something bursts, moves, bubbles
B. Hyponatremia under his skin". Objectively: no pathology
C. Hyperazotemia was revealed. What is the most probable
D. Acidosis psychopathologic symptom in this case?
E. Alkalosis
A. Cenestopathy
153. A 30 y.o. woman has the 2-nd B. Paresthesia
labour that has been lasting for 14 hours. C. Hallucination
Hearbeat of fetus is muffled, arrhythmic, D. Hypersthesia
100/min. Vaginal examination: cervix of E. Dysmorphopsia
uterus is completely opened, fetus head
is level with outlet from small pelvis. 157. A 45 y.o. man complains of having
Saggital suture is in the straight diameter, intensive pain in the epigastric region 1,5-
small crown is near symphysis. What is the 2 hours later after food intake. He has
further tactics of handling the delivery? been suffering from ulcer for 11 years.
Objectively: t0 - 36, 50 , RR- 16/min, Ps-
A. Use of obstetrical forceps 70 bpm, AP- 120/80 mm Hg. On palpati-
B. Stimulation of labour activity by on: local painfulness in the right epigastric
oxytocin region. What parameters of intragastric
C. Cesarean section Ph-meter in the region of stomach body
D. Cranio-cutaneous (Ivanov’s) forceps are the most typical for this patient’s di-
E. Use of cavity forceps sease?
154. A 75 y.o. man with IHD A. рН = 1,0-2,0
(atherosclerotic cardiosclerosis, atrial fi- B. рН = 3,0-4,0
brillation, cardiac insufficiensy stage 2 B; C. рН = 4,0-5,0
chronic pyelonephritis) was prescribed di- D. рН = 5,0-6,0
goxin. During the first 6 days the digoxin E. рН = 6,0-7,0
dose amounted 0,25 mg twice per day that
let to abatement of dyspnea, edemata and 158. A 13 y.o. teenager who suffers from
cyanosis. But on the 7-th day the pati- hemophilia A was taken to the hospi-
ent developed nausea and bradycardia. tal after a fight at school. His diagnosis
What is the most probable cause of di- is right-sided hemarthros of knee joint,
goxin intoxication? retroperitoneal hematoma. What should
be primarily prescribed?
A. Disturbed elimination of the medicati-
on by kidneys A. Fresh frozen plasma
B. Too long intake of saturating dose B. Aminocapronic acid
C. Exceeding daily saturating dose C. Washed thrombocytes
D. Disturbed metabolism of digoxin in D. Placental albumin
liver E. Dry plasma
E. Treatment complex doesn’t include
unitiole 159. A 3 m.o. child fell seriously ill,
body temperature rised up to 37, 80 C,
155. A patient who has been contacti- there is semicough. On the 3-rd day the
ng with benzol for 6 years has a cough grew worse, dyspnea appeared. On
nonevident leukopenia, moderate reti- percussion: tympanic sound above lungs,
culocytosis, gingival hemorrhage, dizzi- on auscultation: a lot of fine moist and
Krok 2 Medicine 2007 21

wheezing rales during expiration. What is 9, 8 · 109/L, RBC- 3, 0 · 1012/L, sedimentati-


the most probable diagnosis? on rate - 52 mm/hour. What medication
provides pathogenetic treatment of this
A. Acute respiratory viral infection, patient?
bronchiolitis
B. Acute respiratory viral infection, A. Sulfosalasine
bronchopneumonia B. Motilium
C. Acute respiratory viral infection, C. Vikasolum
bronchitis D. Linex
D. Acute respiratory viral infection, E. Kreon
bronchitis with asthmatic component
E. Acute respiratory viral infection, focal 164. A 49 y.o. female patient was admi-
pneumonia tted to the hospital with acute attacks of
headache accompanied by pulsation in
160. A worker at a porcelain factory who temples, AP rised up to 280/140 mm Hg.
has been in service for 10 years complains Pheochromocytoma is suspected. What
of cough, dyspnea, ache in his chest. What mechanism of hypertensive atack does
occupational disease are these complaints this patient have?
most typical for?
A. Increasing of catecholamines
A. Silicosis concentration
B. Multiple bronchiectasis B. Increasing of aldosterone level in blood
C. Chronic dust bronchitis C. Increasing of plasma renin activity
D. Occupational bronchial asthma D. Increasing of vasopressin excretion
E. Chronic cor pulmonale E. Increasing of thyroxine excretion
161. Mother of a newborn child suffers 165. A child was born with body weight
from chronoc pyelonephritis. She survi- 3250 g and body length 52 cm. At the age
ved acute respiratory viral infection di- of 1,5 month the actual weight is suffici-
rectly before labour. Delivery was at term, ent (4350 g), psychophysical development
the period before discharge of waters was corresponds with the age. The child is
prolonged. On the 2-nd day the child breast-fed, occasionally there are regurgi-
got erythematous rash, later on - vesi- tations. What is the cause of regurgitati-
cles about 1 cm large with seropurulent ons?
content. Nikolsky’s symptom is positive.
Dissection of vesicles results in erosions. A. Aerophagia
The child is inert, body temperature is B. Pylorostenosis
subfebrile. What is the most probable di- C. Pylorospasm
agnosis? D. Acute gastroenteritis
E. Esophageal atresia
A. Impetigo neonatorum
B. Vesicular pustulosis 166. A patient with hepatic cirrhosis drank
C. Pseudofurunculosis some spirits that resulted in headache,
D. Sepsis vomiting, aversion to food, insomnia,
E. Ritter’s dermatitis jaundice, fetor hepaticus, abdominal
swelling. What complication of hepatic ci-
162. Medical examination of a man rrhosis is meant?
revealed "geographic tongue". This mi-
crosymptom is the evidence of the followi- A. Hepatocellular insufficiency
ng vitamin deficiency: B. Hemorrhage from varicosely dilatated
veins of esophagus
A. Vitamins of B group C. Portal hypertension
B. Vitamin A D. Acute stomach ulcer
C. Vitamin C E. Thrombosis of mesenteric vessels
D. Vitamin D
E. Vitamin P P 167. On the 4-th day of injections a 60
y.o. patient felt pain and tissue induration
163. A 41 y.o. woman has suffered in the left buttock. Objectively: the skin
from nonspecific ulcerative colitis for in the superexternal quadrant of the left
5 years. On rectoromanoscopy: evident buttock is red and hot, palpation reveals a
inflammatory process of lower intesti- painful infiltrate 6х6 cm large with softeni-
nal parts, pseudopolyposive changes of ng in the centre. Body temperature is
mucous membrane. In blood: WBC- 37, 90. What action is necessary to di-
Krok 2 Medicine 2007 22

agnose an abscess? A. Combined


B. Complex
A. Punction C. Associated
B. Biopsy D. Adjacent
C. Ultrasonic examination E. Mixed
D. X-ray investigation
E. Clinical blood analysis 172. X-ray pattern of thorax organs
revealed a large intensive inhomogeneous
168. A 39 y.o. patient complains of havi- opacity with indistinct outlines on the ri-
ng dyspnea during physical activity, crus ght side at the level of the 4-th rib. In the
edemata, palpitation, heart intermissions. centre of this opacity there is a horizontal
Objectively: HR is 150 bpm, atrial fibri- level and clearing of lung tissue above
llation. Heart is both ways enlarged. Heart it. What disease does this X-ray pattern
sounds are muted. Liver is 6 cm below the correspond with?
costal margin. Echocardiogram reveals di-
latation of heart chambers (end diastolic A. Abscess of the right lung
volume of left ventricle is 6,8 cm) is 29% B. Peripheral cancer
EF, valve apparatus is unchanged. What is C. Tuberculoma of the right lung
the most probable diagnosis? D. Right-sided pneumothorax
E. -
A. Dilated cardiomyopathy
B. Exudative pericarditis 173. A 30 y.o. patient has got multiple
C. Restrictive cardiomyopathy body skin rash consisting of small pai-
D. Hypertrophic cardiomyopathy red elements that are scattered on the
E. Thyreotoxic cardiomyopathy skin diorderly and mostly focally, they are
accompanied by itch. The rash appeared
169. The major repair of a hospital a few days after attending sport centre
included renewal of colour design of and sauna. What is the most probable di-
hospital premises because it is of great agnosis?
psychological and aesthetical importance;
and so the walls of patient wards will be A. Scab
painted under consideration of: B. Eczema
C. Contact dermatitis
A. Windows orientation D. Allergic dermatitis
B. Hospital profile E. Neurodermitis
C. Diseases of patients who will be staying
in these wards 174. A 56 y.o. patient ill with
D. Wall reflection coefficient cholecystectomy suddenly had an intense
E. Creation of cozy atmosphere hemorrhage. She needs blood transfusi-
on. Her blood group is (V )Rh− .
170. A 10 y.o. child has average indi- Hemotransfusion station doesn’t dispose
ces of body length and her chest ci- of this group. What group of donors can
rcumference exceeds average indices, be involved?
body weight index is heightened due to
lipopexia. Functional characteristics of A. Donors of rare blood groups
physical development are below average. B. Donors of active group
Physical development of this child can be C. Relatives
estimated as: D. Emergency donors
E. Reserve donors
A. Disharmonic
B. Average 175. A 33 y.o. patient was admitted to the
C. Below average hospital with stopped repeated ulcerati-
D. Harmonic ve bleeding. He was pale and exhausted.
E. Deeply disharmonic Blood count: Нb- 77 g/l, Нt- 0,25. In
view of anemia there were made two
171. Atmospheric air of an industri- attempts of blood transfusion of the same
al centre is polluted with the following group - ()Rh+ . In both cases the
wastes of metallurgical plants: sulphuric, transfusion had to be stopped because
nitric, metal, carbon oxides that have of development of anaphylactic reaction.
negative influence upon the inhabitants’ What transfusion medium would be advi-
health. The effct of these hazards can be sable in this case?
characterized as:
Krok 2 Medicine 2007 23

A. Washed erythrocytes rash. Body temperature is of hectic nature.


B. Fresh citrate blood What is the most probable diagnosis?
C. Erythrocytic mass (native)
D. Erythrocytic suspension A. Sepsis
E. Erythrocytic mass poor in leukocytes B. Hemorrhagic disease of newborn
and thrombocytes C. Hemolytic disease of newborn
D. Thrombocytopathy
176. A 19 y.o. boy was admitted to the E. Omphalitis
hospital with closed abdominal trauma. In
course of operation multiple ruptures of 180. Elderly people develop tumors more
spleen and small intestine were revealed. frequently. One of the main causes is:
AP is falling rapidly, it is necessary
to perform hemotransfusion. Who can A. Decreased activity of cellular immunity
determine the patient’s blood group and B. Decreased intensity of antibodies
rhesus compatibility? formation
C. Increased disfunctioning of mitoses
A. A doctor of any speciality D. Increased activity of cellular immunity
B. A laboratory physician E. Increased intensity of antibodies
C. A surgeon formation
D. A traumatologist
E. An anaesthesilogist 181. A 30 y.o. man complains of intense
pain, reddening of skin, edema in the
177. A 20 y.o. patient complains of ankle-joint area, fever up to 390 . There
amenorrhea. Objectively: hirsutism, was acute onset of the illness. In the past
obesity with fat tissue prevailing on the there were similar attacks lasting 5-6 days
face, neck, upper part of body. On the without residual changes in the joint. The
face there are acne vulgaris, on the skin over the joint is hyperemic without
skin - striae cutis distense. Psychological definite borders and without infiltrative
and intellectual development is normal. bank on the periphery. What is the most
Gynecological condition: external geni- likely diagnosis?
tals are moderately hairy, acute vaginal
and uterine hypoplasia. What diagnosis is A. Gout
the most probable? B. Infectional arthritis
C. Rheumatoid arthritis
A. Itsenko-Cushing syndrome D. Erysipelatous inflammation
B. Turner’s syndrome E. Osteoarthritis
C. Stein-Levental’s syndrome
D. Shichan’s syndrome 182. A 7 y.o. boy was admitted to the
E. Babinski-Froehlich syndrome hospital. He complains of unpleasant
sensations in the heart region, pain in the
178. A 14 y.o. girl got a bad mark at the epigastrium, dizziness, vomiting. Objecti-
lesson of math as well as teacher’s repri- vely: evident paleness of skin, dyspnea,
mand that made her cry for a long time. jugular pulse. Heart borders are within
At the end of the lesson she suddenly lost the normal range. Heart sounds are clear,
consciousness and fell down. Objectively: HR- 170/min, small pulse. AP- 90/50 mm
skin is of pale pink colour, Ps- 100 bpm, Hg. EKG showed: paroxysm of ventri-
satisfactory, AP- 110/70 mm Hg. Eyeli- cular tachycardia. The paroxysm can be
ds are closed and don’t give away to the suppressed by:
tries of their opening. There are no evi-
dent injuries. What is the provisional di- A. Lidocain
agnosis? B. Morphine
C. Enalapril
A. Hysteric syncope D. Nifedipine
B. Epilepsy E. Strophanthine
C. Vagotonic syncope
D. Long QT syndrome 183. A child is 2 m.o. Inguinofemoral folds
E. Sympathicotonic collapse contain acutely inflamed foci with distinct
borders in form of spots that are slightly
179. A newborn child has purulent di- above the surrounding areas due to skin
scharges from the umbilical wound, skin edema. The rash has appeared during the
around the umbilicus is swollen. Objecti- week. Vesiculation and wetting are absent.
vely: the child’s skin is pale, of yellow- What is the most probable diagnosis?
greyish colour, generalized hemorrhagic
Krok 2 Medicine 2007 24

A. Napkin-area dermatitis 188. After delivery and revision of


B. Infantile eczema placenta there was found the defect
C. Dermatomycosis of placental lobe. General condition of
D. Psoriasis woman is normal, uterus is firm, there is
E. Complicated course of scabies moderate bloody discharge. Inspection of
birth canal with mirrors shows absence of
184. A 38 y.o. patient lifted a heavy object lacerations and raptures. What action is
that resulted in pain in the lumbar part of nesessary?
spine irradiating to the posterior surface
of his left leg. The pain increases during A. Manual exploration of the uterine
changing body position and also in the cavity
upright position. Examination revealed B. External massage of uterus
positive symptoms of tension. What is the C. Use of uterine contracting agents
preliminary diagnosis? D. Urine drainage, cold on the lower
abdomen
A. Pathology of intercostal disks E. Use of hemostatic medications
B. Spinal cord tumor
C. Arachnomyelitis 189. A 67 y.o. patient complains of
D. Polyneuritis dyspnea, breast pain, common weakness.
E. Myelopathy He has been ill for 5 months. Objecti-
vely: t0 - 37, 30 , Ps- 96/min. Vocal tremor
185. A 35 y.o. patient was admitted to over the right lung cannot be determined,
the local hospital a week after a road percussion sound is dull, breathing cannot
accident with clinical picture of clotted be auscultated. In sputum: blood diffusi-
hemothorax. What is the most appropri- vely mixed with mucus. What is the most
ate treatment tactic for prevention of probable diagnosis?
acute pleural empyema?
A. Lung cancer
A. Surgical removal of clotted hemothorax B. Macrofocal pneumonia
B. Treatment by pleural punctions C. Bronchoectatic disease
C. Complex conservative therapy D. Focal pulmonary tuberculosis
D. Passive drainage of pleural cavity E. Exudative pleuritis
E. Active drainage of pleural cavity
190. Poorly refined wastes of an industrial
186. An aircraft factory processes materi- plant are usually thrown into the river that
als with use of lasers. It is determined that supplies drinking water. It causes peri-
the device radiates in the light spectrum shing of some microorganisms, disturbs
and that levels of laser radiation at the processes of water self-purification and
workplaces exceed the alarm level. Speci- worsens its quality that can have negati-
fy, what organs will be affected in the first ve influence upon people’s health. How
place? is this effect of environmental factors
A. Eyes called?
B. Skin A. Indirect
C. Liver B. Direct
D. Spleen C. Associated
E. Kidneys D. Complex
187. At a machine-building plant the casts E. Combined
are cleaned by means of abrasion machi- 191. A 22 y.o. patient complains of having
nes that are a source of local vibration. boring pain in the right iliac region for
What are the most efficient preventive one week, morning sickness, taste change.
measures for preventing harmful effect of Delay of menstruation is 3 weeks. Objecti-
vibration on workers’ organisms?
vely: AP- 110/70 mm Hg, Ps- 78/min, t0 -
A. Use of gloves that reduce vibration 37, 00. Bimanual examination revealed
B. Preliminary and periodical medical that uterus is a little enlarged, soft,
examinations movable, painless. Appendages palpation:
C. Hand massaging a painful formation 3х4 cm large on the
D. Warm hand baths right, it is dense and elastic, moderately
E. Giving sanitary instructions to the movable. What is the most probable di-
workers agnosis?
Krok 2 Medicine 2007 25

A. Progressing tubal pregnancy 195. On the 15-th day after a minor trauma
B. Interrupted tubal pregnancy of the right foot a patient felt malai-
C. Cyst of the right ovary se, fatigability, irritability, headache, high
D. Uterine pregnancy body temperature, feeling of compressi-
E. Acute appendicitis on, tension and muscular twitching of his
right crus. What disease can it be?
192. A 15 y.o. patient has developmental
lag, periodical skin yellowing. Objectively: A. Tetanus
spleen is 16х12х10 cm large, holecistoli- B. Anaerobic gas gangrene
thiasis, skin ulcer on the lower third of C. Erysipelas
his left crus. Blood count: erythrocytes D. Acute thrombophlebitis
- 3, 0 · 1012 /L, Hb- 90 g/L, C.I.- 1,0, mi- E. Thromboembolism of popliteal artery
crospherocytosis, reticulocytosis. Blood
bilirubin - 56 mmole/L, indirect bili- 196. 10 years ago a patient had a fracture
rubin - 38 mmole/L. Choose the way of in the middle one-third of his left femoral
treatment: bone, and during the last 7 years he
has been having acute inflammation in
A. Splenectomy the area of old fracture accompanied by
B. Spleen transplantation formation of a fistula through which some
C. Portocaval anastomosis pus with small fragments of bone tissue is
D. Omentosplenopexy discharged. After a time the fistula closes.
E. Omentohepatopexy What complication of the fracture is it?
193. A 30 y.o. parturient woman was taken A. Chronic osteomyelitis
to the maternity house with complaints of B. Bone tuberculosis
having acute, regular labour pains that last C. Soft tissue phlegmon
25-30 seconds every 1,5-2 minutes. Labour D. False joint
activity began 6 hours ago. Uterus is in E. Trophic ulcer
higher tonus, head of the fetus is above
the opening into the small pelvis. Fetal 197. Medical examination of a 43 y.o. man
heartbeat is 136/min. P.V: cervical dilatati- revealed objectively pailness of skin and
on is 4 cm, uterine fauces is spasming at mucous membranes, smoothness of li-
a height of parodynia. Head is level with ngual papillas, transverse striation of nails,
opening into the small pelvis, it is being fissures in the mouth corners, tachycardia.
pushed off. What is the most probable di- Hemoglobin content amounts 90 g/l; there
agnosis? are anisocytosis, poikilocytosis. The most
probable causative agent of this condi-
A. Discoordinated labour activity tion is deficiency of the following mi-
B. Secondary powerless labour activity croelement:
C. Pathological preliminary period
D. Primary powerless labour activity A. Iron
E. Normal labour activity B. Copper
C. Zinc
194. A primigravida woman appealed to D. Magnesium
the antenatal clinic on the 22.03.03 wi- E. Selenium
th complaints of boring pain in the lower
part of abdomen. Anamnesis registered 198. The total area of ground intended
that her last menstruation was on the for building of a regional hospital is 2,0
4.01.03. Bimanual examination revealed hectare. What is the highest possible
that uterine servix is intact, external capacity of the in-patient hospital that can
fauces is closed, uterus is enlarged up be built upon this ground?
to the 9-th week of pregnancy, movable,
painless. What complication can be A. 100 beds
suspected? B. 200 beds
C. 400 beds
A. Risk of abortion in the 9-th week of D. 800 beds
pregnancy E. Over 1000 beds
B. Abortion that started in the 9-th week
of pregnancy 199. A 33 y.o. woman survived two
C. Hysteromyoma operations on account of extrauteri-
D. Vesicular mole ne pregnancy, both uterine tubes were
E. - removed. She consulted a doctor with a
question about possibility of having a chi-
Krok 2 Medicine 2007 26

ld. What can be advised in this case? document should be used for registration
of this disease?
A. Extracorporal fertilization
B. Insemination with her husband’s semen A. Urgent report on infectious disease
C. Substitutional maternity B. Statistic coupon for registration of final
D. Artifical fertilization with donor’s diagnoses
semen C. Outpatient’s card
E. Induction of ovulation D. Inpatient’s card
E. Statistic card of the patient who left
200. A worker diagnosed with "acute in-patient hospital
dysentery"was sent to the infectious
department by a doctor of aid post. What
Krok 2 Medicine 2008 1

1. An 8 year old child has low-grade A. Atony of the uterus


fever, arthritis, colicky abdominal pain B. Injury of cervix of the uterus
and a purpuric rash llocalized on the lower C. Hysterorrhexis
extremities. laboratory studies reveal a D. Delay of the part of placenta
guaiac-positive stool, urinalysis with red E. Hypotonia of the uterus
blood cell (RBC) casts and mild protei-
nuria, and a normal platelet count. The 5. On the first day after labour a woman
most likely diagnosis is: had the rise of temperature up to 39o C.
Rupture of fetal membranes took place
A. Henoch-Schonlein’s vasculitis 36 hours before labour. Examination of
B. Systemic lupus erythematosus (SLE) the bacterial flora of cervix of the uterus
C. Rocky Mountain spotted fever revealed hemocatheretic streptococcus of
D. Idiopathic thrombocytopenic purpura A group. The uterus body is soft, tender.
E. Poststreptococcal glomerulonephritis Discharges are bloody, with admixtures of
pus. Specify the most probable postnatal
2. A woman complains of having slight complication:
dark bloody discharges and mild pains in
the lower part of abdomen for several A. Metroendometritis
days. Last menses were 7 weeks ago. B. Thrombophlebitis of veins of the pelvis
The pregnancy test is positive. Bimanual C. Infectious hematoma
investigation: the body of the uterus indi- D. Infective contamination of the urinary
cates for about 5-6 weeks of pregnancy, system
it is soft, painless. In the left appendage E. Apostasis of sutures after the episi-
there is a retort-like formation, 7 х 5 cm otomy
large, mobile, painless. What examination
is necessary for detection of fetus locali- 6. A woman of a high-risk group (chronic
zation? pyelonephritis in anamnesis) had vagi-
nal delivery. The day after labour she
A. Ultrasound complained of fever and loin pains,
B. Hysteroscopy frequent urodynia. Specify the most
C. Hromohydrotubation probable complication:
D. Colposcopy
E. Cystoscopy A. Infectious contamination of the urinary
system
3. A pregnant woman in her 40th week of B. Thrombophlebitis of veins of the pelvis
pregnancy undergoes obstetric examinati- C. Infectious hematoma
on: the cervix of uterus is undeveloped. D. Endometritis
The oxytocin test is negative. Examinati- E. Apostasis of sutures after episiotomy
on at 32 weeks revealed: AP 140/90 mm
Hg, proteinuria 1 g/l, peripheral edemata. 7. A woman in her 39th week of
Reflexes are normal. Choose the most pregnancy, the second labour, has regular
correct tactics: birth activity. Uterine contractions take
place every 3 minutes. What criteria
A. Labour stimulation after preparation describe the beginning of the II labor
B. Absolute bed rest for 1 month stage the most precisely?
C. Complex therapy of gestosis for 2 days
D. Caesarian section immediately A. Cervical dilatation by no less than 4 cm
E. Complex therapy of gestosis for 7 days B. Cervical smoothing over 90%
C. Duration of uterine contractions over
4. A 26 year old woman had the second 30 seconds
labour within the last 2 years with D. Presenting part is in the lower region of
oxytocin application. The child’s wei- small pelvis
ght is 4080 gr. After the placent bi- E. Rupture of fetal bladder
rth there were massive bleeding, si-
gns of hemorrhagic shock. Despite the 8. A 24 years old primipara was hospitali-
introduction of contractive agents, good sed with complaints about discharge of
contraction of the uterus and absence the amniotic waters. The uterus is tonic
of any cervical and vaginal disorders, on palpation. The position of the fetus is
the bleeding proceeds. Choose the most longitudinal, it is pressed with the head
probable cause of bleeding: to pelvic outlet. Palpitation of the fetus
is rhythmical, 140 bpm, auscultated on
the left below the navel. Internal exami-
nation: cervix of the uterus is 2,5 cm
Krok 2 Medicine 2008 2

long, dense, the external os is closed, light A. Droperidolum of 0,25% - 2,0 ml


amniotic waters out of it. Point a correct B. Dibazolum of 1% - 6,0 ml
component of the diagnosis: C. Papaverine hydrochloride of 2% - 4,0
ml
A. Antenatal discharge of the amniotic D. Hexenalum of 1% - 2,0 ml
waters E. Pentaminum of 5% - 4,0 ml
B. Early discharge of the amniotic waters
C. The beginning of the 1st stage of labour 12. A 40 year old woman has changes of
D. The end of the 1st stage of labour mammary gland. What are the most often
E. Pathological preterm labour symtomps that precede the malignizati-
on?
9. A 29 year old patient underwent
surgical treatment because of the beni- A. Skin induration with inverted nipple
gn serous epithelial tumour of an ovary. B. Painful movable induration
The postoperative period has elapsed wi- C. Painless movable induration
thout complications. What is it necessary D. Bloody discharges from the nipple
to prescribe for the rehabilitational peri- E. Pure discharges from the nipple
od:
13. A patient who takes diuretics has
A. Hormonotherapy and proteolytic developed arrhythmia as a result of
enzymes cardiac glycoside overdose. What is the
B. Antibacterial therapy and adaptogens treatment tactics in this case?
C. Lasertherapy and enzymotherapy
D. Magnitotherapy and vitamin therapy A. Increased potassium concentration in
E. The patient does not require further blood
care B. Increased sodium consentration in
blood
10. A patient was delivered to a surgical C. Reduced magnesium concentration in
department after a road accident with a blood
closed trauma of chest and right-sided rib D. Increased calcium concentration in
fracture. The patient was diagnosed wi- blood
th right-sided pneumothorax, it is indi- E. -
cated to perform drainage of pleural cavi-
ty. Pleural puncture should be made in: 14. An onset of severe preeclampsia at 16
weeks gestation might be caused by:
A. In the 2nd intercostal space along the
middle clavicular line A. Hydatidiform mole
B. In the 6th intercostal space along the B. Anencephaly
posterior axillary line C. Twin gestation
C. In the 7th intercostal space along the D. Maternal renal disease
scapular line E. Interventricular defect of the fetus
D. In the projection of pleural sinus 15. A 9 year old girl with a history of
E. In the point of the greatest dullness on intermittent wheezing for several years
percussion is brought to the pediatrician. The child
11. A primagravida with pregnancy of 37- has been taking no medications for some
38 weeks complains of headache, nausea, time. Physical examination reveals agitati-
pain in epigastrium. Objective: the skin is on and perioral cyanosis. Intercostal and
acyanotic. Face is hydropic, there is short suprasternal retractions are present. The
fibrillar twitching of blepharons, muscles breath sounds are quiet, and wheezing is
of the face and the inferior extremities. audible bilaterally. The child is admitted
The look is fixed. AP- 200/110 mm Hg; to the hospital. Appropriate interventi-
sphygmus of 92 bpm, intense. Respiration ons might include all of the following
rate is 32/min. Heart activity is rhythmi- EXCEPT:
cal. Appreciable edemata of the inferior A. Prescribe nebulized cromolyn sodium
extremities are present. Urine is cloudy. B. Prescribe intravenous aminophylline
What medication should be administered? C. Administer supplemental oxygen
D. Prescribe intravenous corticosteroids
E. Prescribe nebulized metaproterenol
16. Routine examination of an otherwise
healthy child with a history of bronchial
Krok 2 Medicine 2008 3

asthma reveals AP of 140/90 mm Hg. The ldhood. Examination reveals a lift at the
most likely cause of the hypertension is: left sternal border, split S1 , and fixed spli-
tting of S2 . There is a grade 3/6 midsystolic
A. Renal disease pulmonic murmur and a 1/6 middiastolic
B. Theophylline toxicity tricuspid murmur at the lower left sternal
C. Chronic lung disease border. Chest x-ray shows right ventricular
D. Coarctation of the aorta enlargement and prominent pulmonary
E. Obesity arteries. ECG demonstrates atrial fibri-
llation with a right bundle branch block.
17. The average body lenth of newborn The most likely diagnosis is:
boys is 50,9 cm at a sigma 1,66; and
average mass - 3432 at a sigma 5,00. What A. Arterial septal defect
criterion is necessary to compare degree B. Coarctation of the aorta
of variability of these signs? C. Patent ductus arteriosus
D. Tetralogy of Fallot
A. Coefficient of variation E. Ventricular septal defect
B. Sigma
C. Limit 22. Generalized low voltage on an ECG
D. Amplitude (QRS deflection < 5 mm in limb leads and
E. Coefficient of association < 10 mm in precordial leads) may be a
18. What method is applied to establish marker for all of the following disorders
rate of correlation between age of men EXCEPT:
and their mortality due to myocardial A. Hyperthyroidism
infarction? B. Pericardial effusion
A. Method of grade correlation (Spirman) C. Cardiac transplant rejection
B. Correlation ratio D. Amyloidosis
C. The quadrate method (Pirson) E. Coronary artery disease
D. The Indirect method (Student) 23. Five days after a total hip replacement
E. Method of graduated correlation a 72 year old woman becomes acutely
(Armler) short of breath, diaphoretic and
hypotensive. Both lung fields are clear to
19. Adenosine triphosphate may be auscultation and percussion, but exami-
expected to convert which of the followi- nation of the neck reveals mild jugular
ng arrhythmias to sinus rhythm? venous distension with prominent A
A. Paroxysmal supraventricular waves. Heart sounds are normal. ECG
tachycardia shows sinus tachycardia with a new right
B. Paroxysmal ventricular tachycardia bundle branch block and minor nonspeci-
C. Atrial fibrillation fic ST − T wave changes. The most likely
D. Atrial flutter diagnosis is:
E. Ventricular fibrillation A. Pulmonary thromboembolism
20. A 48 year old man complains of fati- B. Acute myocardial infarction
que and shortness of breath. His Ht is C. Aortic dissection
32%, and Hb - 103 g/l. Peripheral blood D. Pericarditis
smear reveals macrocytosis. Serum vi- E. Aspiration
tamin B12 level is 90 pg/ml (normal is 24. A child is 6 years old. Within one year
170 to 940); serum folate level is 6 ng/ml of observation he had URI that lasted 8
(normal is 2 to 14). Possible causes to days. Physical state is satisfactory. Specify
consider include all of the following hi health group:
EXCEPT:
A. I
A. Colonic diverticulitis B. II
B. Vegetarianism C. III (a)
C. Regional enteritis D. III (b)
D. Pancreatitis E. III (c)
E. Fish tapeworm infection
25. Which of the following IS NOT TYPI-
21. A 56 year old man complains of fati- CAL for Addison’s disease?
gue, dyspnea on exertion and palpitati-
ons. He has had a murmur since chi-
Krok 2 Medicine 2008 4

A. High serum Na+ A. Nasogastric decompression and


B. High serum K + parenteral alimentation
C. Elevated BUN B. Retrocolic gastrojejunostomy
D. Dilute urine C. Duodenojejunostomy
E. Elevated hematocrit D. Immediate exploration
E. Tube duodenostomy
26. A 38 year old man, previously in good
health, suddenly develops severe abdomi- 31. A 25 year old man has a sudden onset
nal pain radiating from the left loin to of chest pain on the right and dyspnea. His
groin and accompanied by nausea, perspi- trachea is deviated to the left. All of the
ration and the need for frequent urinati- following would be anticipated EXCEPT:
on. He is restless, tossing in bed but has
no abnormal findings. The most likely di- A. Pleural friction rub on the left
agnosis is: B. Absence of rhonchi
C. Absence of rales
A. Leftsided renal colic D. Hyperresonance over the right chest
B. Herpes zoster E. Distant breath sounds on the right
C. Sigmoid diverticulitis
D. Torsion of the left testicle 32. A 5 tons milk batch was sampled.
E. Retroperitoneal haemorrhage The lab analysis revealed: fat content 2%,
specific density - 1,04 g/cm3 , acidity - 210 Т,
27. A 40 year old woman has a self- reductase probe - weak-positive. What
detected hard breast mass. The procedure way is the product to be used in?
of choice for confirming the diagnosis is:
A. Sell but inform customers about milk
A. Excision biopsy quality
B. Mammography B. Discard for animal feeding
C. Thermography C. Technical utilization
D. Ultrasonography D. Sell without limitations
E. Aspiration biopsy with cytology E. Do the product away
28. A 38 year old man was admitted to a 33. What guarantees against the
hospital from his working place on July preconceived attitude to the physician in
19 because of hip fracture. He was invalid cases of professional law violations do you
till November 19. Requires prolongation know?
of treatment. Who decides on the issue of
further temporary invalidity? A. Sanction of public prosecutor, inquiry
by preliminary investigator of prosecutor’s
A. Specialized (traumatologic) MSEC office, committee of experts
B. DCC B. Draw up a statement about forensic
C. The head physician of a polyclinic medical examination
D. Interregional general MSEC C. Conduct an inquiry by preliminary
E. Regional MSEC investigator of police department
D. Utilisation copy of medical documents
29. During investigation for chronic, E. Conduct forensic medical examination
severe, epigastric pain, a 40 year old by district forensic medicine expert
alcoholic man is found to have multiple
areas of narrowing alternating with di- 34. If a child has adherent fingers on his
latation ("chain of lakes"appearance) of right hand, then what will be your di-
the main pancreatic duct. The operation agnosis?
of choice is:
A. Syndactyly
A. Lateral pancreaticojejunostomy B. Polydactyly
B. Distal pancreaticojejunostomy C. Macrodactyly
C. Sphincterotomy D. Ectrodactyly
D. Distal pancreatectomy E. Ectromelia
E. Total pancreatectomy
35. A 16 year old patient with complai-
30. The treatment of choice for duodenal nts of frequent pain in the abdomen was
obstruction caused by secondary diagnosed with melanoma, examination
duodenal hematoma that developed a few revealed also pigmentation of the mucosa
days after blunt abdominal injury is: and skin, polyp in the stomach and large
intestine. It is know that the patient’s
Krok 2 Medicine 2008 5

mother has an analogous pigmentation A. The test for tolerance to glucose


and has been often treated for anemia. B. Determination of the contents of
What disease is suspected? fetoproteinum
C. Bacteriological examination of di-
A. Peytz-Egers’s polyposis scharges from vagina
B. Chron’s disease D. A cardiophonography of fetus
C. Tuberculosis of the intestine E. US of fetus
D. Adolescent polyposis
E. Hirschprung’s disease 40. A patient with acute purulent oti-
tis media complicated by mastoiditis was
36. A 41 year old patient was admitted to admitted to a hospital. Roentgenogram of
the intensive care unit with haemorrhagic mastoid processes showed the shadiowi-
shock due to gastric bleeding. He has a hi- ng of the cellular system on the lesion,
story of hepatitis B during the last 5 years. absence of bone septa was present. What
The source of bleeding are esophageal vei- are the necessary therapeutic +actions at
ns. What is the most effective method for the second stage of mastoiditis?
control of the bleeding?
A. Mastoidotomy
A. Introduction of obturator nasogastric B. Paracentesis of the drum
tube C. Radical operation on the middle ear
B. Intravenous administration of pituitrin D. Tympanoplasty
C. Hemostatic therapy E. Cateterization of the Eustachian tube
D. Operation
E. Administration of plasma 41. A 22 year old patient was admitted to
trauma center with complaints of pain in
37. It is suspected that a 34 year old pati- the left ankle joint that was getting worse
ent has an abscess of Douglas pouches. during moving and weight bearing. On the
What diagnostic method is to be chosen? clinical examination it was found that the
patient had the closed fracture of medial
A. Digital examination of rectum malleolus without displacement. In which
B. Rectoromanoscopy position should the foot be fixed in plaster
C. Laparoscopy cast?
D. Percussion and auscultation of stomach
E. R-scopy of abdominal cavity A. At right angle with varus positioning of
the foot
38. A 52 year old man has recurrent transi- B. In position of planter flexion of foot
ent ischemic attacks. Auscultation of the C. In position of pronation
carotid arteries detected murmur. What D. In position of supination
diagnostic method is to be applied in the E. In position of dorsal flexion of foot
first place?
42. A patient with acute respiratory viral
A. Ultrasound dopplerography infection (3rd day of disease) complains
B. CT of the brain of pain in lumbar region, nausea, dysuria,
C. MRI of the brain oliguria. Urinalysis - hematuria (100-200
D. Cerebral angiography
E. Electroencephalography RBC in eyeshot spot), specific gravity -
1002. The blood creatinin level is 0,18 mi-
39. A 34 year old woman in the 10th llimole/l, potassium level - 6,4 millimole/l.
week of gestation (the second pregnancy) Make the diagnosis:
consulted a doctor of antenatal clinic in A. Acute interstitial nephritis
order to be registered there. In the previ- B. Acute renal failure
ous pregnancy hydramnion was observed, C. Acute glomerylonephritis
the child’s birth weight was 4086 g. What D. Acute cystitis
examination method should be applied in E. Acute renal colic
the first place?
43. A 3 month old infant suffering
from acute segmental pneumonia has
dyspnea (respiration rate - 80 per mi-
nute), paradoxical breathing, tachycardia,
total cyanosis. Respiration / pulse ratio is
1:2. The heart dullness under normal size.
Such signs characterise:
Krok 2 Medicine 2008 6

A. Encephalitis
A. Respiratory failure of III degree B. Meningitis
B. Respiratory failure of I degree C. Meningoencephalitis
C. Respiratory failure of II degree D. Myelitis
D. Myocarditis E. Neurotoxic syndrome
E. Congenital heart malformation
48. A 14 year old patient complains of
44. A 3 year old child has been sufferi- chest pain, temperature up to 38, 5o C,
ng from fever, cough, coryza, conjuncti- breathlessness. He had acute tonsillitis
vitis for 4 days. He has been taking 2 weeks ago. He is in grave condition.
sulfadimethoxine. Today it has fever up The skin is pale. Heart borders are di-
to 39o C and maculopapular rash on its lated, heart sounds are quiet. Above total
face. Except of rash the child’s skin has no heart area you can hear pericardium fricti-
changes. What is your diagnosis? on sound. Electrocardiogram: the descent
of QRS voltage, the inversion T. The liver
A. Measles is enlarged by 3 cm. ESR - 4 mm/h, ASL
B. Allergic rash - 0 - 1260, C-reactive protein +++. Your
C. Rubella diagnosis:
D. Scarlet fever
E. Pseudotuberculosis A. Rheumatic pancarditis
B. Rheumatic pericarditis
45. A 2 year old girl has been ill for C. Rheumatic myocarditis
3 days. Today she has low grade fever, D. Rheumatic endocarditis
severe catarrhal presentations, slight E. Septic endocarditis
maculopapular rash on her buttocks and
enlarged occipital lymph nodes. What is 49. A woman born a child. It was her fith
your diagnosis? pregnancy but the first delivery. Mother’s
blood group is A(II)Rh− , newborn’s -
A. Rubella A(II)Rh+ . The level of indirect bilirubin
B. Scarlet fever in umbilical blood was 58 micromole/l,
C. Measles hemoglobin - 140 g/l, RBC- 3, 8·1012/l. In 2
D. Adenoviral infection hours the level of indirect bilirubin turned
E. Pseudotuberculosis 82 micromole/l. The hemolytic disease
46. A 3 year old boy fell ill abruptly: of newborn (icteric-anemic type, Rh-
fever up to 39o C, weakness, vomi- incompatibility) was diagnosed. Choose
tng. Haemorrhagic rash of various size the therapeutic tactics:
appeared on his lower limbs in 5 hours.
Meningococcemia with infective - toxic A. Replacement blood transfusion
shock of the 1 degree was diagnosed. (conservative therapy)
What medications should be admini- B. Conservative therapy
stered? C. Blood transfusion (conservative
therapy)
A. Chloramphenicol succinate and predni- D. Symptomatic therapy
sone E. Antibiotics
B. Penicillin and prednisone
C. Penicillin and immunoglobulin 50. A mother consulted a pediatrician
D. Chloramphenicol succinate and about her son. Her son was born with
interferon body mass of 3 kg and length of 48 cm.
E. Ampicillin and immunoglobulin He’s 1 year old now. What is the required
normal mass?
47. A 7 year old girl has mild form
of varicella. Headache, weakness, verti- A. 10,5 kg
go, tremor of her limbs, ataxia, then B. 9,0 kg
mental confusion appeared on the 5th C. 11,0 kg
day of illness. Meningeal signs are negati- D. 12,0 kg
ve. Cerebrospinal fluid examination is E. 15,0 kg
normal. How can you explain these signs?
51. A 2 month old healthy infant with
good appetite is given artificial feeding
since he turned 1 month old. When is
it recommended to start the corrective
feeding (fruit juice)?
Krok 2 Medicine 2008 7

hypervolaemic type of essential


A. 4,0 months hypertension. Which of the following
B. 1,5 months medications is to be prescribed either as
C. 2,0 months monotherapy or in complex with other
D. 3,0 months antihypertensive drugs?
E. 1,0 months
A. Hypothiazid
52. A nine year old child is at a hospi- B. Dibazol
tal with acute glomerulonephritis. Cli- C. Clonidine
nical and laboratory examinations show D. Kapoten
acute condition. What nutrients must not E. Nifedipin
be limited during the acute period of
glomerulonephritis? 57. A 62 year old patient complains of
rest dyspnea, heart pains. 3 years ago
A. Carbohydrates he had myocardial infarction. Physical
B. Salt examination: orthopnea, acrocyanosis,
C. Liquid swollen cervical veins. Ps - 92, total heart
D. Proteins enlargement, the liver is enlarged by 7 cm,
E. Fats shin edema. What is the stage of chronic
heart failure (CHF)?
53. Examination of a 3-month-old child
revealed scrotum growth on the right. This A. CHF- 2 B
formation has elastic consistency, its size B. CHF- 1
decreases during sleep and increases when C. CHF- 2 А
the child is crying. What examination will D. CHF- 0
be helpful for making a correct diagnosis? E. CHF- 3
A. Palpation of the thickened cord crossing 58. A patient, aged 49, complains of fever
the pubical tubercule (sign of the silk of 37, 5o C, heart pain, dyspnea. S1 is
glove) clapping; S2 is accentuated in the aortic
B. Diaphanoscоpy area; opening snap, presystolic murmur
C. Palpation of the external inguinal ring can be auscultated. What is the most effi-
D. Puncture of the scrotum cient examination for valvular disorder
E. Examination of the formation in assessment?
Trendelenburg’s position
A. Echocardiography+Doppler-
54. A rounded well-defined shadow was Echocardiography
found in the costo-vertebral angle on B. Phonocardiography
the chest roentgenogram of an otherwise C. Ballistocardiogram
healthy 9 year old girl. Make a prelimi- D. Chest X-ray
nary diagnosis: E. ECG
A. Ganglioneuroma 59. A patient with nosocomial pneumonia
B. Sympatoblastoma presents signs of collapse. Which of
C. Ganglioneuroblastoma the following pneumonia complications
D. Sympatogonioma is most likely to be accompanied by
E. Sarcoma of the vertebra collapse?
55. A 52 year old patient with history of A. Septic shock
functional Class II angina complains of B. Exudative pleuritis
having intense and prolonged retrosternal C. Bronchial obstruction
pains, decreased exercise tolerance for D. Toxic hepatitis
5 days. Angina is less responsive to ni- E. Emphysema
troglycerine. What is the most probable
diagnosis? 60. A patient, aged 48, complains of heavi-
ness in the right hypochondrium, itchi-
A. IHD. Unstable angina ng of the skin. Repeatedly he had been
B. Cardialgia due to spine problem treated in infectious diseases hospital
C. IHD. Functional Class II angina because of icterus and itch. Objectively:
D. Myocarditis meteorism, ascitis, dilation of abdomi-
E. Myocardial dystrophy nal wall veins, protruding navel, spleen
enlargement. Diagnosis is:
56. A 52 year old patient has
Krok 2 Medicine 2008 8

A. Liver cirrhosis
B. Cancer of the liver A. Hypothalamus
C. Cancer of the head of pancreas B. Frontal lobes
D. Gallstones C. Pituitary
E. Viral hepatitis B D. Hippocampus
E. Corpus callosum
61. A 27 year old man complains of pai-
ns in epigastrium which are relieved by 66. The observed patient’s movements
food intake. EGDFS shows antral erosive are retarded, she answers no questions.
gastritis, biopsy of antral mucous presents Sometimes she spontaneously stays in
Hеlicobacter Pylori. Diagnosis is: strange postures. It is possible to set her
body and limbs into different positions
A. Gastritis of type B artificially. If the psychiatrist lifts her arm
B. Gastritis of type A or leg, so that she remains standing on the
C. Reflux-gastritis other leg, the patient can stay in such a
D. Menetrier’s gastritis position for quite a long time. Name the
E. Rigid antral gastritis probable disorder:
62. A 62 year old patient suffers from A. Catatonic stupor, schizophrenia
DM-2. Diabetes is being compensated by B. Depressive stupor, bipolar disorder
diet and Maninilum. The patient has to C. Apathetic stupor, schizophrenia
undergo an operation on inguinal hernia. D. Psychogenic stupor, stress disorder
What tactics of hypoglycemic therapy E. Dissociative stupor, dissociative
should be chosen? psychosis
A. Prescribe fast-acting insulin 67. A 50 year old patient has been
B. Give Glurenorm instead of Maninilum admitted to the clinics with atrophic
C. Continue with the current therapy gastritis. Blood count: erythrocytes -
D. Prescribe long-acting insulin 3, 8 · 1012 /l, Hb - 68 g/l, c.i. - 1,
E. Prescribe guanyl guanidines macroanisocytosis, poikilocytosis. There
is megaloblastic type of haemopoesis.
63. Prophylactic photoroentgenography A number of leukocytes, reticulocytes
examination of a 25 year old man revealed and thrombocytes is lreduced. Which
focal shadowings of small and medium pathology is suspected?
intensity with irregular contours in the 1st
and 2nd segments of the right lung. Which A. B12 -deficiency anemia
clinical form can be suspected? B. Irondeficiency anemia
C. Hemolytic anemia
A. Focal D. Post-hemorrhagic anemia
B. Disseminated E. Thalassaemia
C. Miliary
D. Fibro-cavernous 68. A 28 year old patient was admi-
E. Tuberculoma tted to the clinic with complaints of the
temperature rise up to 39, 0oC, headache,
64. A triad of symptoms ("stearing spot", weakness, constipation on the 9th day
"terminal film", "blood dew") has been of the disease. On examination: single
revealed on examination of a patient. roseolas on the skin of the abdomen are
What disease should you think about? present. The pulse rate is 78 bpm. The li-
A. Psoriasis ver is enlarged by 2 cm. What is the most
probable diagnosis?
B. Lichen ruber planus
C. Vasculitis A. Typhoid fever
D. Seborrhea B. Leptospirosis
E. Ritter’s disease C. Brucellosis
D. Sepsis
65. A female 28 years old patient became
depressed, her mood is melancholic; E. Malaria
this state is accompanied by hypobulia, 69. A patient has been in a hospital. The
hypokinesia, slow speed of thinking. Her beginning of the disease was gradual:
attitude towards her past, present and nausea, vomiting, dark urine, аcholic
future is pessimistic. The pathogenetic stools, yellowness of the skin and scleras.
mechanism of this state is supposed to The liver is protruded by 3 cm. Jaundice
involve dysfunction in the: progressed on the 14th day of the disease.
Krok 2 Medicine 2008 9

The liver diminished in size. What compli- necessary in this clinical situation:
cation of viral hepatitis caused deteriorati-
on of the patient’s condition? A. Controlled respiration
B. Gastric lavage
A. Hepatic encephlopathy C. Oxygen inhalation
B. Meningitis D. Caffeine injection
C. Relapse of viral hepatitis E. Cordiamine injection
D. Cholangitis
E. Infectious-toxic shock 74. A pregnant woman may be diagnosed
with hepatitis if it is confirmed by the
70. An 18 year old patient was admitted presence of elevated:
to a hospital with complaints of headache,
weakness, high temperature, sore throat. A. SGOT (ALT)
Objectively: enlargement of all groups of B. Sedimentation rates
lymph nodes was revealed. The liver is C. WBCs
enlarged by 3 cm, spleen - by 1 cm. In D. Alkaline phosphatase
blood: leukocytosis, atypical lymphocytes E. BUN
- 15%. What is the most probable di-
agnosis? 75. An endometrial adenocarcinoma that
has extended to the uterine serosa would
A. Infectious mononucleosis be classified as stage:
B. Acute lymphoid leukosis
C. Diphtheria A. IIIA
D. Angina B. IC
E. Adenoviral infection C. IIA
D. IIB
71. A 75 year old man who has been E. IV AB
suffering from diabetes for the last six
months was found to be jaundiced. He 76. A 56 year old woman has an acute
was asymptomatic except for weight loss onset of fever up to 39o C with chills,
at the rate of 10 pounds in 6 months. cough, and pain on respiration in the right
Physical examination revealed a hard, side of her chest. On physical examinati-
globular, right upper quadrant mass that on: HR - 90/min, BP- 95/60 mm Hg, RR-
moves during respiration. A CT scan 26/min. There is dullness over the right
shows enlargement of the head of the lung on percussion. On X-ray: infiltrate in
pancreas, with no filling defects in the li- the right middle lobe of the lung. What is
ver. The most likely diagnosis is: the diagnosis?

A. Carcinoma of the head of the pancreas A. Community-acquired lobar pneumonia


B. Infectious hepatitis of moderate severity
C. Haemolytic jaundice B. Community-acquired
D. Malignant biliary stricture bronchopneumonia
E. Metastatic disease of liver C. Acute pleuritis
D. Acute lung abscess
72. A 33 year old male patient was E. Nosocomial lobar pneumonia
brought to Emergency Department wi-
th the signs of cardiovascular collapse: BP 77. A 26 year old man was admitted to
- 60/30 mm Hg, Ps - 140 bpm, the skin is the hospital because of stabbing back pain
pale and moist, diuresis 20 ml/h, Hb - 80 on inspiration and dyspnea. Examination
g/l, red blood cell count - 2, 5 · 1012 /l. The results: BT of 37o C, Ps of 24/min, HR of
reduction of blood volume averages: 92/min, vesicular breath sounds. There is
a dry, grating, low-pitched sound heard on
A. 30-40% both expiration and inspiration in the left
B. 10-15% inferior lateral part of the chest. What is
C. 15-20% the most likely diagnosis?
D. 20-25%
E. 25-30% A. Acute fibrinous pleuritis
B. Myocarditis
73. A 19 year old girl was admitted to C. Pneumonia
emergency department: unconsciousness, D. Acute bronchitis
cyanosis, myotic pupils are present, E. Pneumothorax
superficial breathing is 12/min. BP is 90/60
mm Hg, Ps- 78/min. Choose the action 78. A 34 year old woman fell ill 3 months
Krok 2 Medicine 2008 10

ago after cold exposure. She complai- A. Chronic mercury intoxication


ned of pain in her hand and knee joints, B. Neuroinfection
morning stiffness and fever up to 38oC. C. Parkinson syndrome
Interphalangeal, metacarpophalangeal D. Acute mercury intoxication
and knee joints are swollen, hot, with E. Mercury encephalopathy
reduced ranges of motions; ESR of 45
mm/h, CRP (+++), Vaaler-Rouse test of 82. A 6 week old child is admitted because
1:128. What group of medicines would you of tachypnea. Birth had been uneventful,
recommend the patient? although conjunctivitis developed on the
third day of life and lasted for about
A. Nonsteroidal anti-inflammatory drugs 2 weeks. Physical examination reveals
B. Cephalosporines tachypnea, bilateral inspiratory crackles
C. Tetracyclines and single expiratory wheezing. Bilateral
D. Sulfonamides pneumonia is evident on chest X-ray. The
E. Fluorchinolones child is afebrile and has no history of fever.
White blood cell count is 15 · 109 /l, with
79. A 50 year old woman complained of 28% of eosinophils. The most likely cause
attacks of right subcostal pain after fatty of this child’s symptoms is:
meal she has been suffering from for a
year. Last week the attacks repeated every A. Clamydia trachomanis
day and became more painful. What di- B. Pneumocystis carinii
agnostic study would you recommend? C. Mycoplasma pneumoniae
D. Visceral larva migrans
A. Ultrasound examination of the E. Varicella
gallbladder
B. Liver function tests 83. A 42 year old woman complai-
C. X-ray examination of the gastrointesti- ns of dyspnea, edema of the legs and
nal tract tachycardia during minor physical exerti-
D. Ultrasound study of the pancreas on. Heart borders are displaced to the left
E. Blood cell count and S1 is accentuated, there is diastolic
murmur on apex. The liver is enlarged by
80. A 25 year old woman complained 5 cm. What is the cause of heart failure?
of edema on her face and legs, rise
of blood pressure up to 160/100 mm A. Mitral stenosis
Hg and weakness. She fell ill 3 weeks B. Mitral regurgitation
after recovering from angina. Urinalysis C. Tricuspid stenosis
data: protein of 0,5 g/l, erythrocytes D. Tricuspid regurgitation
of 17-20/field, leukocytes of 2-3/field, E. Aortic stenosis
erythrocyte casts. What treatment should
be initiated after specifying the diagnosis? 84. A 60 year old man complains of
fever, significant weight loss, bone and
A. Penicillin OS joint pain, bleeding gums. Examinati-
B. Heparin on revealed paleness, lymphadenopathy,
C. Ceftriaxone hepato- and splenomegaly. CBC: WBC
D. Dipyridamole - 270 · 109 /l with 13% lymphocytes,
E. Ciprofloxacine 1% monocytes, 21% basophiles,
29% neutrophils, 9% blasts, 12%
81. A fitter of a metallurgic factory promyelocytes, 12% myelocytes, 2%
with occupational exposure to high metamyelocytes, 1% eosinophils. ESR -
concentrations of mercury fumes for 16 22 mm/h. Name the drug for treatment:
years presents instability of pulse and
blood pressure, general hyperhydrosis, A. Myelosan
asymmetric innervations of facial muscles B. Prednisolone
and tongue, positive subcortical reflexes, C. Cytosar
hand tremor on physical examination. A D. Vinblastine
dentist revealed paradontosis and chronic E. Blood transfusion
stomatitis. What is the most probable di-
agnosis? 85. A 54 year old woman complains
of increasing fatigue and easy bruising
of 3 weeks’ duration. Physical findings
included pale, scattered ecchymoses and
petechiae and mild hepatosplenomegaly.
Blood count: RBC- 2, 5 · 1012 /l; Hb - 73
Krok 2 Medicine 2008 11

g/l; Ht - 20%; PLT- 23 · 109 /l; and WBC- A. Acute purulent otitis media
162 · 109 /l with 82% blasts, that contained B. Acute mastoiditis
Auric rods; peroxidase stain was positive. C. Chronic secretory otitis media
What is the most probable diagnosis? D. Chronic purulent otitis media
E. Eustachian tube disfunction
A. Acute leukemia
B. Chronic leukemia 90. Among the following causes of acute
C. Thrombocytopenia renal failure, the one that would be classi-
D. Hemolytic anemia fied as "postrenal"is:
E. Megaloblastic anemia
A. Calculi
86. The highest risk of congenital anomali- B. Cardiac failure
es probably occurs when human embryos C. Septicemia
or fetuses are exposed to ionizing radiati- D. Rhabdomyolysis
on. At what stage of gestational period E. Acute glomerulonephritis
does it occur?
91. A patient complains about strong
A. 18-45 days after conception dyspnea that is getting worse during
B. The first 7 days physical activity. Presentations appeared
C. 10-14 days after conception suddenly 2 hours ago at work: acute chest
D. 90-120 days after conception pain on the left, cough. The pain was
E. The third trimester abating, but dyspnea, dizziness, pallor,
cold sweat and cyanosis were progressi-
87. A 28 year old woman was admi- ng. Vesicular respiration is absent, X-ray
tted to the emergency room with a sli- picture shows a shadow on the left. What
ghtly reddened, painful "knot"8 cm above pathology might be suspected?
the medial malleolus. Examination in
the standing position demonstrates a di- A. Spontaneous left-sided pneumothorax
stended vein above and below the mass. B. Pulmonary infarction
There are no other abnormalities on C. Pleuritis
physical examination. The most likely di- D. Left-sided pneumonia
agnosis is: E. Pulmonary abscess

A. Superficial venous thrombosis 92. During an operation for presumed


B. Early deep vein thrombosis appendicitis the appendix was found to
C. Insect bite be normal; however, the terminal ileum
D. Cellulitis is evidently thickened and feels rubbery,
E. Subcutaneous hematoma its serosa is covered with grayish-white
exudate, and several loops of apparently
88. A 25 year old patient complains of normal small intestine are adherent to it.
pain in the I finger on his right hand. On The most likely diagnosis is:
examination: the finger is homogeneously
hydropic, in bent position. On attempt A. Crohn’s disease of the terminal ileum
to unbend the finger the pain is getti- B. Perforated Meckel’s diverticulum
ng worse. Acute pain appears during C. Ulcerative colitis
the probe in ligament projection. What D. Ileocecal tuberculosis
decease is the most likely? E. Acute ileitis

A. Thecal whitlow (ligament panaritium) 93. A 50 year old woman with a 2-year
B. Subcutaneous panaritium history of mild, diffuse, tender thyroid
C. Articular (joint) panaritium enlargement complains of 10 pound wei-
D. Bone panaritium ght gain and fatigue. What is the most
E. Paronychia probable diagnosis?

89. A 30 year old man complains of acute A. Hashimoto’s thyroiditis


pain in his right ear, hearing loss, hi- B. Riedel’s thyroiditis
gh temperature for three days. Objecti- C. Subacute thyroiditis
vely: right ear whispering language - 0,5 D. Suppurative thyroiditis
mm, external ear is intact, otoscopically E. Papillary thyroid carcinoma
- eardrum protrusion, hyperemia and 94. A severely traumatized patient who
swelling, loss of landmarks. What disease has been receiving prolonged parenteral
is it? alimentation develops diarrhea, mental
Krok 2 Medicine 2008 12

depression, alopecia and perioral and ation of heart and mediastinum shadows
periorbital dermatitis. Administration of towards the lesion. What is the most
which of the following trace elements is probable diagnosis?
most likely to reverse these complicati-
ons? A. Cenral cancer of lung
B. Lung abscess
A. Zinc C. Lung cyst
B. Iodine D. Peripheral cancer of lung
C. Selenium E. Atelectasis
D. Silicon
E. Copper 99. A 68 year old patient has been sufferi-
ng from chronic pancreatitis for 35 years.
95. A patient has got pain in the axillary During the last 5 years he has been
area, rise of temperature developed 10 observing abatement of pain syndrome,
hours ago. On examination: shaky gait is abdominal swelling, frequent defecations
evident, the tongue is coated with white up to 3-4 times a day (feces are greyi-
deposit. The pulse is frequent. The painful sh, glossy, with admixtures of undigested
lymphatic nodes are revealed in the axi- food), progressing weight loss. Change of
llary area. The skin over the lymph nodes symptom set is caused by joining of:
is erythematous and glistering. What is the
most probable diagnosis? A. Exocrine pancreatic insufficiency
B. Endocrine pancreatic insufficiency
A. Bubonic plague C. Syndrome of lactase deficiency
B. Acute purulent lymphadenitis D. Irritable bowels syndrome
C. Lymphogranulomatosis E. Chronic enterocolitis
D. Anthrax
E. Tularemia 100. A 60 year old patient has been
suffering from arterial hypertension for
96. A 56 year old patient has worked at 15 years. After recurrent stroke she
the aluminium plant over 20 years. Wi- started complaining about amotivati-
thin 3 last years he has been experiencing onal bad mood, problems with attenti-
loosening of teeth, bone and joint pains, on concentration; she forgets to close
piercing pains in heart region, vomiting. the entrance door, cannot recall events
The provisional diagnosis is: of the last day. Computer tomography
shows areas of postinfarction changes in
A. Fluorine intoxication the cortical postfrontal regions. What is
B. Mercury intoxication the most probable diagnosis?
C. Lead intoxication
D. Phosphorus intoxication A. Vascular dementia
E. Manganese intoxication B. Alzheimer’s disease
C. Huntington’s chorea
97. An excited patient constantly tries D. Pick’s disease
to look into the next room because he E. Dissociative amnesia
is sure of his friends’ presence. He clai-
ms to hear his friends and other people 101. A 40 year old patient complains
talking although no-one is there. He tri- of yellowish discharges from the vagi-
es to persuade his doctor that people in na. Bimanual examination revealed no
the next room are disputing about "his pathological changes. The smear contains
punishment", repeats aloud phrases he Trichomonas vaginalis and blended flora.
has allegedly heard from the next room. Colposcopy revealed two hazy fields on
What state is it? the frontal labium, with a negative Iodine
test. Your tactics:
A. Acute hallucinosis
B. Verbal illusions A. Treatment of specific colpitis and
C. Delusion subsequent biopsy
D. Confabulations B. Diathermocoagulation of the cervix of
E. Obsessional ideas the uterus
C. Specific treatment of Trichomonas
98. Survey radiograph of chest of a 62 colpitis
year old smoker who often suffers from D. Cervix ectomy
"pneumonias"showed a triangle shadow E. Cryolysis of cervix of the uterus
in the right lung, its vertex is pointing
towards the lung root. It also showed devi- 102. A patient complained about
Krok 2 Medicine 2008 13

problems with pain and tactile sensitivi- permissible?


ty, pain in the nail bones at the end of
the working day. He works at a plant with A. 0,1%
mechanical devices. What pathology can B. 0,05%
be suspected? C. 0,15%
D. 0,2%
A. Vibration disease E. 0,3%
B. Caisson disease
C. Noise disease 107. Study of actual diet of an adult
D. Overwork symptoms revealed the following: proteins make
E. Hypovitaminosis of B1 up 16% of energy value of daily rati-
on, fats - 25%, carbohydrates - 59%.
103. A 25 year old patient complains Evaluate compliance of protein, fat and
about weakness, dizziness, appearance of carbohydrate share in the energy value of
haemorrhagic skin rash. She has been daily ration with the recommended shares
suffering from this for a month. Blood of these nutrients?
count: erythrocytes: 1, 0 · 1012 /l, Hb- 37 g/l,
colour index - 0,9, leukocytes - 1, 2 · 109 /l, A. Carbohydrate share is insufficient, there
is excess of proteins
thrombocytes - 42 · 109 /l. What diagnostic B. Fat share is insufficient
method will be the most effective? C. Carbohydrate share is insufficicent
A. Sternal puncture D. Carbohydrate share is excessive
B. Spleen biopsy E. Nutrient content complies with the
C. Liver biopsy recommended shares of energy value
D. Coagulogram 108. A healthy 75 year old woman who
E. Abdominal ultrasound leads a moderately active way of life
104. A 28 year old woman had the second went through a preventive examinati-
labour and born a girl with manifestati- on that revealed serum concentration
ons of anemia and progressing jaundice. of common cholesterol at the rate of
The child’s weight was 3 400 g, the length 5,1 millimol/l and HDL (high-density li-
was 52 cm. The woman’s blood group is poproteins) cholesterol at the rate of
B (III) Rh− , the father’s blood group is 70 mg/dl. ECG reveals no pathology.
A (III) Rh+ , the child’s blood group is B What dietary recommendation is the most
adequate?
(III) Rh+ . What is the cause of anemia?
A. Any dietary changes are necessary
A. Rhesus incompatibility B. Decrease of cholesterol consumption
B. Antigen A incompatibility C. Decrease of saturated fats consumption
C. Antigen B incompatibility D. Decrease of carbohydrates consumpti-
D. Antigen AB incompatibility on
E. Intrauterine infection E. Increase of cellulose consumption
105. A 25 year old patient was admitted on 109. Periodical survey of a worker
the 1st day of the disease with complai- of a chemicals plant revealed a mali-
nts of double vision in the eyes, heavy gnant neoplasm on the urinary bladder.
breathing. The day before the patient This occupational disease was the most
ate home-made mushrooms. On objective probably caused by contact with the
examination: paleness, mydriatic pupils, following industrial poison:
difficult diglutition, bradycardia, consti-
pation. What is the diagnosis? A. Benzidine
B. Vinyl chloride
A. Botulism C. Nickel carbonyl
B. Yersiniosis D. Asbestos
C. Leptospirosis E. Arsenic
D. Salmonellosis, gastrointestinal form
E. Lambliasis 110. A 52 year old patient was admi-
tted to a hospital because of hi-
106. Maximum permissible concentration gh hemorrhagic diathesis of mucous
of carbon dioxide in the air is considered membranes, massive skin haemorrhages
to be a sanitary index of air purity in a in form of ecchymoses and spots, nasal
classroom. What concentration of carbon and stomachal haemorrhages. After clini-
dioxide in the air is accepted as maximum cal examinations her illness was diagnosed
Krok 2 Medicine 2008 14

as thrombocytopenic purpura. What is the Stool is liquid in form of rice water. What
most probable cause of this disease? is the most probable diagnosis?
A. Generation of antithrombocytic anti- A. Cholera
bodies B. Acute dysentery
B. Disturbed hemostasis C. Salmonellosis
C. Deficit of the VIII factor of blood D. Escherichiosis
coagulation E. Intestinal amebiasis
D. Inherited insufficiency of plasm factors
of blood coagulation 115. At year-end hospital administration
E. Iron deficit in blood serum, bone has obtained the following data: annual
marrow and depot number of treated patients and average
annual number of beds used for patients’
111. A child with tetralogy of Fallot is treatment. What index of hospital work
most likely to exhibit: can be calculated on the base of this data?
A. Increased pressure in the right ventricle A. Bed turnover
B. Increased pulmonary blood flow B. Bed resources of the hospital
C. Increased pulse pressure C. Average annual bed occupancy
D. Normal pressure gradient across the D. Average duration of patients’ presence
pulmonary valve in the hospital
E. Normal oxygen tension (P aO2) in the E. Average bed idle time
left ventricle
116. A 3 year old child with weight deffici-
112. In treatment and prevention establi- ency suffers from permanent moist cough.
shments, regardless of their organisational In history there are some pneumonias
and proprietary form, the rights of pati- with obstruction. On examination: di-
ents should be observed. Which of these stended chest, dullness on percussion over
rights is the most significant? the lower parts of lungs. On auscultation:
a great number of different rales. Level of
A. The right to the protection of the sweat chloride is 80 millimol/l. What is the
patient’s interests most probable diagnosis?
B. The right to the free choice
C. The right to the information A. Mucoviscidosis (cystic fibrosis)
D. The right to be heard B. Bronchial asthma
E. The right to the protection from C. Recurrent bronchitis
incompetence D. Bronchiectasis
E. Pulmonary hypoplasia
113. A military unit stopped for 3-day’s
rest in an inhabited locality after a 117. A 14 year old girl complains of
long march. The sanitary-epidemiological profuse bloody discharges from genital
reconnaissance found several water tracts during 10 days after suppresion of
sources. It is necessary to choose the menses for 1,5 month. Similiar bleedings
source complying with the hygienic recur since 12 years on the background
standards for drinking water in the field of disordered menstrual cycle. On rectal
conditions: examination: no pathology of the internal
genitalia. In blood: Нb - 70 g/l, RBC-
A. Artesian well water 2, 3 · 1012 /l, Ht - 20. What is the most
B. Spring water probable diagnosis?
C. River water
D. Rain water A. Juvenile bleeding, posthemorrhagic
E. Melt snow water anemia
B. Werlholf’s disease
114. A man in grave condition was deli- C. Polycyst ovarian syndrome
vered to the admission ward of a hospi- D. Hormonoproductive ovary tumor
tal on the 2nd day of illness. Examinati- E. Incomplete spontaneous abortion
on revealed body temperature of 36, 1oC,
sharpened features of face, dry skin that 118. A 43 year old patient had
makes a fold, aphonia, convulsive twitchi- cholecystectomy 6 years ago because of
ng of some muscle groups. Acrocyanosis chronic calculous cholecystitis. Lately he
is present. Heart sounds are muffled, Ps is has been suffering from pain in the ri-
102 bpm, AP is 50/20 mm Hg. Abdomen is ght subcostal area and recurrent jaundi-
soft, drawn-in, painless. Anuria is presnt. ce. Jaundice hasn’t gone for the last 2
Krok 2 Medicine 2008 15

weeks. Stenosing papillitis 0,5 cm long has meal. Exacerbations happen in spring
been revealed. What is the best way of and in autumn. The patient has food
treatment? intolerance of eggs and fish. Objecti-
vely: stomach palpation reveals pai-
A. To perform endocsopic papillosphi- nfulness in the gastroduodenal area.
ncterotomy Electrophasoduodenoscopy revealed a
B. To treat conservatively: antibiotics, 5 mm ulcer on the anterior wall of
spasmolytics, antiinflammatory drugs duodenum. Urease test is positive. What
C. To perform external choledoch drainage is the most probable leading mechanism
D. To perform transduodenal papillosphi- of disease development?
ncterotomy
E. To perform choledochoduodenostomy A. Chelicobacterial infection
B. Dietary allergy
119. A full-term infant is 3 days old. C. Autoantibody production
On the different parts of skin there are D. Reduced prostaglandin synthesis
erythemas, erosive spots, cracks, areas of E. Disorder of gastric motor activity
epidermis peeling. The infant has scalded
skin syndrome. Nikolsky’s symptom is 123. A 48 year old woman complains
positive. General condition of the infant about weakness, weight loss, appetite
is grave. Anxiety, hyperesthesia, febrile loss, headache. The patient had acute
temperature are evident. What is the most glomerulonephritis when she was young.
probable diagnosis? She has been suffering from arterial
hypertension since she was 25. She didn’t
A. Exfoliative dermatitis undergo systematic treatment, consulted
B. Phlegmon of newborn a doctor rarely. Examination revealed si-
C. Finger’s pseudofurunculosis gns of chronic renal insufficiency of the I
D. Impetigo neonatorum stage (creatinine - 0,43 millimole/l). What
E. Mycotic erythema dietary recommendations are the most
reasonable?
120. An outpatient hospital made record
of 11600 diseases within one year. Among A. Restriction of protein consumption
them influenza and ARD make up 5800, B. Restriction of fat consumption
circulatory system diseases - 3480, digesti- C. Restriction of carbohydrate consumpti-
on diseases - 1300, other diseases - 1020. on
What relative index can be calculated D. Diet with high content of "alkali-
according this data? ne"dishes
E. Consumption of higher amounts of
A. Extensive
liquid
B. Intensive
C. Visualization 124. A 50 year old woman complains
D. Correlation about dull cardiac pain, asphyxia, body
E. - temperature rise up to 38oC. She had
influenza a week ago. Objectively: Ps -
121. A 35 year old woman consulted 100 bpm, dropped-beat pulse during inspi-
a doctor about affection of arm skin ration. AP - 100/70 mm Hg, heart sounds
and lower third of forearm in form of a
are muffled. ECG: reduced voltage, ST
large edema, hyperemia, vesiculation and segment is above the isoline in all leads.
maceration. The disease developed after
X-ray picture shows extensively enlarged
using a laundry detergent "Lotos". The
cardiac silhouette. Palmus is of small
patient has been using it for a month. She amplitude. What is the most probable di-
hasn’t suffered from dermatological di-
agnosis?
seases before. What is the most probable
diagnosis? A. Exudative pericarditis
B. Myocardium infarction
A. Allergic dermatitis
C. Dilatation cardiomyopathy
B. Dermatitis simplex
D. Myocarditis
C. Toxicoallergic dermatitis E. Stenocardia
D. Microbial eczema
E. Localized neurodermatitis 125. Examination of a 4 month old child
revealed some lemon-yellow squamae wi-
122. A 32 year old patient complains th fatty crusts on the scalp. What is the
about heartburn and dull pain in the most probable diagnosis?
epigastrium that appear 2-3 hours after
Krok 2 Medicine 2008 16

would be the most adequate in this case?


A. Gneiss
B. Milk crust A. Parenteral introduction of polyvalent
C. Strophulus antibotulinic serum
D. Pseudofurunculosis B. Parenteral disintoxication
E. Infantile eczema C. Parenteral introduction of antibiotics
D. Gastric lavage
126. A 39 year old patient complai- E. Parenteral introduction of antitetanus
ned about morning headache, appeti- serum
te loss, nausea, morning vomiting, peri-
odic nasal haemorrhages. The patient had 130. A 19 year old patient was admi-
acute glomerulonephritis at the age of tted to a hospital with acute destructive
15. Examination revealed rise of arteri- appendicitis. He suffers from hemophilia
al pressure up to 220/130 mm Hg, skin of B type. What antihemophilic medicati-
haemorrhages on his arms and legs, pallor ons should be included in pre- and post-
of skin and mucous membranes. What operative treatment plan?
chemestry test has the greatest diagnostic
importance in this case? A. Fresh frozen plasma
B. Cryoprecipitate
A. Blood creatinine C. Fresh frozen blood
B. Blood bilirubin D. Native plasma
C. Blood sodium E. Dried plasma
D. Uric acid
E. Fibrinogen 131. An 18 year old woman consulted a
gynecologist about the pain in the lower
127. A 14 year old child suffers from part of abdomen, fever up to 37, 5o C,
vegetovascular dystonia of pubertal peri- considerable mucopurulent discharges
od. He has got sympathoadrenal atack. from the genital tracts, painful urinati-
What medicine should be used for attack on. Vaginal and speculum examination
reduction? results: the urethra is infiltrated, cervix
of the uterus is hyperemic, erosive. The
A. Obsidan uterus is painful, ovaries are painful, thi-
B. No-shpa ckened; fornixes are free. Bacterioscopy
C. Amysyl test revealed diplococcus. What diagnosis
D. Aminophylline is the most probable?
E. Corglicone
A. Recent acute ascending gonorrhea
128. A 23 year old man complains B. Trichomoniasis
about face edemata, headache, dizzi- C. Candydomycosis
ness, reduced urination, change of urine D. Chronic gonorrhea
colour (dark-red). These presentations E. Chlamydiosis
appeared after pharyngitis. Objectively:
face edemata, pale skin, temperature - 132. A 26 year old woman who delivered
37, 4oC; heart rate - 86/min, AP - 170/110 a child 7 months ago has been suffering
mm Hg. Heart sounds are muffled, the II from nausea, morning vomiting, sleepi-
sound is accentuated above aorta. What ness for the last 2 weeks. She suckles the
etiological factor is probable in this case? child, menstruation is absent. She hasn’t
applied any contraceptives. What method
A. β-hemolytic streptococcus should be applied in order to specify her
B. Staphylococcus aureus diagnosis?
C. Alpha-hemolytic streptococcus
D. Pyogenic streptococcus A. Ultrasonic examination
E. Saprophytic staphylococcus B. Roentgenography of small pelvis organs
C. Palpation of mammary glands and
129. A 12 year old girl complains about pressing-out of colostrum
abrupt weakness, nausea, dizziness, visi- D. Bimanual vaginal examination
on impairment. The day before she ate E. Speculum examination
home-made stockfish, beef. Examination
revealed skin pallor, a scratch on the left 133. A 2 month old full-term child was
knee, dryness of mucous membranes of born with weight 3500 g and was on the
oral pharynx, bilateral ptosis, mydriatic mixed feeding. Current weight is 4900 g.
pupils. The girl is unable to read a simple Evaluate the current weight of the child:
text (mist over the eyes). What therapy
Krok 2 Medicine 2008 17

A. Corresponding to the age Examination at an outpatient’s hospi-


B. 150 g less than necessary tal revealed paleness of skin, sleepiness.
C. Hypotrophy of the I grade Blood count: Hb - 95 g/l, erythrocytes
D. Hypotrophy of the II grade - 3, 5 · 1012 /l, reticulocytes - 90 /00 ,
E. Paratrophy of the I grade colour index - 0,7, osmotic stability of
erythrocytes - 0,44-0,33%, serum iron - 4,9
134. A woman consulted a doctor on micromole/l. What is the most probable
the 14th day after labour about sudden cause of anemia?
pain, hyperemy and induration of the left
mammary gland, body temperature ri- A. Iron deficit
se up to 39o C, headache, indisposition. B. Hemogenesis immaturity
Objectively: fissure of nipple, enlargement C. Infectious process
of the left mammary gland, pain on D. Erythrocyte hemolysis
palpation. What pathology would you thi- E. B12 deficit
nk about in this case?
138. A 13 year old girl consulted the
A. Lactational mastitis school doctor on account of moderate
B. Lacteal cyst with suppuration bloody discharge from the genital tracts,
C. Fibrous adenoma of the left mammary which appeared 2 days ago. Secondary
gland sexual characters are developed. What
D. Breast cancer is the most probable cause of bloody di-
E. Phlegmon of mammary gland scharge?
135. A 32 year old patient complai- A. Menarche
ns about pain in small joints of her B. Juvenile hemorrhage
hands, paresthesia at the tips of fi- C. Haemophilia
ngers, weakness, difficult diglutition. D. Endometrium cancer
She has been suffering from this for E. Werlhof’s disease
13 years. Objectively: face amimia,
shortening of nail bones, skin indurati- 139. An 8 year old girl complains about
ons in the area of shoulder girdle are joint pain, temperature rise up to 38o C,
present. Roentgenological examination dyspnea. Objectively: the left cardiac
of lungs revealed basal pneumosclerosis. border is deviated by 2,5 cm to the left,
Fibrogastroscopy revealed esophagus tachycardia, systolic murmur on the apex
constriction in its cardial part. Blood and in the V point are present. Blood
count: leukocytes - 9, 8 · 109 /l, ESR - 22 count: leukocytes - 20, 0 · 109 /l, ESR - 18
mm/h, γ-globulin - 22%. What is the most mm/h. What sign gives the most substanti-
probable diagnosis? al proof for rheumatism diagnosis?
A. Systemic scleroderma A. Carditis
B. Systemic lupus erythematosus B. Arthralgia
C. Rheumatoid arthritis C. Leukocytosis
D. Dermatomyositis D. Fever
E. Myxedema E. Accelerated ESR
136. A 30 year old woman ill with 140. During inspection of sanitary conditi-
influenza felt palpitation and dull cardi- ons of studying at a technical university it
ac pain during moderate physical exerci- was necessary to evaluate the visual regi-
se. Objectively: Ps - 96 bpm, AP - 100/60 men of students, who study from 9 a.m to
mm Hg. The first sound is quiet above 3 p.m. What index of natural light will be
the apex, soft systolic murmur is present. the most informative?
What complication is indicated by these
clinical presentations? A. Natural light coefficient
B. Light coefficient
A. Acute viral myocarditis C. Depth of study room
B. Acute allergic infectious myocarditis D. Time of the room insolation
C. Idiopathic myocarditis E. Presence of mixed (superolateral) light
D. Myocardiopathy
E. Neurocirculatory dystonia 141. A 70 year old man is suffering from
coronary heart disease. His mood is evi-
137. A 5 month old boy was born dently depressed, anxious. As a result
prematurely, he didn’t suffer from any of continious sleeplessness he has got
disease at the infant age and later on. fears, suicidal thoughts. He would sit for
Krok 2 Medicine 2008 18

a long time in the same pose, answer be of the greatest importance for confi-
after a pause, in a low, monotonous voi- rmation of provisional diagnosis?
ce. His face has a look of suffering, pain,
fear. What is the main psychopathologic A. Lumbar puncture
syndrome? B. Examination of eye fundus
C. Ultrasonic dopplerography
A. Depressive syndrome D. EEG
B. Paranoid syndrome E. Computer tomography
C. Asthenic syndrome
D. Phobic syndrome 145. A 10 year old girl complains about
E. Obsessive syndrome abdominal pain that is arising and getti-
ng worse after eating rough or spicy food.
142. A 37 year old farmer complains She complains also about sour eructati-
about general weakness, spastic pain in on, heartburn, frequent constipations,
the lower parts of his abdomen, mainly headache, irritability. She has been sufferi-
in the left iliac area, frequent defecati- ng from this for 12 months. Objectively:
ons up to 18 times a day, feces contain the girl’s diet is adequate. Tongue is moist
admixtures of mucus and blood. The with white deposit at the root. Abdomen
illness began abruptly 3 days ago with is soft, painful in its epigastric part. What
chill, fever, headache. General condition study method will help to make a di-
is moderately severe, body temperature is agnosis?
37, 8oC. Sigmoid colon is spasmed and pai-
nful. What is the most probable diagnosis? A. Esophagogastroduodenoscopy
B. Intragastral pH-metry
A. Dysentery C. Fractional examination of gastric juice
B. Amebiasis D. Contrast roentgenoscopy
C. Nonspecific ulcerative colitis E. Biochemical blood analysis
D. Yersiniosis
E. Salmonellosis 146. A 35 year old patient who suffers
from chronic glomerulonephritis and has
143. A 3 year old child fell acutely ill, been hemodialysis-dependent for the last
body temperature rose up to 39, 5oC, three years developed intermissions of
the child became inert, there appeared heart activity, hypotension, progressi-
recurrent vomiting, headache. Exami- ng weakness, dyspnea. ECG showed
nation revealed positive meningeal bradycardia, atrioventricular block type
symptoms, after this lumbal puncture was I, high pointed waves T . The day before
performed. Spinal fluid is turbid, runs the flagrant violation of diet took place.
out under pressure, protein concentrati- What is the most probable cause of these
on is 1,8 g/l; Pandy reaction is +++, sugar changes?
concentration is 2,2 millimole/l, chloride
concentration - 123 millimole/l, cytosis is A. Hyperkaliemia
2, 35 · 109 (80% of neutrophils, 20% of B. Hyperhydratation
lymphocytes). What is the most probable C. Hypokaliemia
diagnosis? D. Hypernatriemia
E. Hypocalciemia
A. Purulent meningitis
B. Serous viral meningitis 147. A 34 year old coal miner who has
C. Serous tuberculous meningitis been in service for 10 years was di-
D. Subarachnoid haemorrhage scharged from an occupational disease
E. Brain tumour clinic after examination and treatment.
He was diagnosed with the I stage of
144. A 39 year old patient suffering anthracosilicosis, peribronchitis, 0 stage
from hypertension felt suddenly intense of respiratory failure. What expert deci-
headache in the region of occiput; there sion should be taken about his working
appeared recurrent vomiting. These capacity?
presentations has been lasting for 5 hours.
Objectively: Ps - 88 bpm, AP - 205/100
mm Hg, painfulness of occipital points,
rigidity of occipital muscles are present.
Kernig’s symptom is bilaterally positi-
ve. Subarachnoid haemorrhage is also
suspected. What diagnostic method will
Krok 2 Medicine 2008 19

A. He should be referred to the local medi- 151. A 24 year old patient complai-
cal expert comission for determination ns about general weakness, dizziness,
of his disability group for the period of body temperature rise up to 37, 5o C,
re-training sore throat, neck edema, enlargement
B. He can continue working as coal miner of submaxillary lymph nodes. Objecti-
provided that sanitary and hygienic condi- vely: mucous membrane of oropharynx
tions are observed is edematic and cyanotic, tonsils are
C. He should be given medical certificate enlarged and covered with films that
for the further out-patient treatment spread beyond the tonsils and cannot
D. He should stay on the sick-list to be easily removed. What is the leading
consolidate treatment results mechanism of this illness’ development?
E. He should be referred to the local medi-
cal expert comission for determination of A. Action of bacterial exotoxin
percentage of working capacity loss B. Action of bacterial endotoxin
C. Allergic
148. A 52 year old woman complains D. Accumulation of suboxidated products
about face distortion. It appeared 2 days E. Bacteriemia
ago after supercooling. Objectively: body
temperature is 38, 2oC. Face asymmetry is 152. A 38 year old man complains
present. Frontal folds are flattened. Left about mild pain and muscle weakness
eye is wider than right one and doesn’t of shoulder and pelvic girdles and back
close. Left nasolabial fold is flattened, that has been progressing for the last 3
mouth corner is lowered. Examination weeks. He has also significant problems
revealed no other pathology. Blood count: with getting up, going up and down the
leukocytes - 10 · 109 /l, ESR - 20 mm/h. stairs and shaving. It is suspected that the
What is the most probable diagnosis? patient is suffering from dermatomyosi-
tis. Blood count: Hb - 114 g/l, leukocytes
A. Facial neuritis - 10, 8 · 109 /l, eosin - 9%, ESR - 22 mm/h,
B. Trigeminus neuralgia C-reactive protein (++). Change of the
C. Hemicrania (migraine) following laboratory factor will be of the
D. Ischemic stroke greatest diagnostic importance:
E. Brain tumour
A. Creatine phosphokinase
149. A patient complained about general B. Ceruloplasmin
weakness, fever, painful rash on his trunk C. Sialic acids
skin . He has been suffering from this for 3 D. Antibodies to the native DNA
days. Objectively: lateral surface of trunk E. γ-globulins
on the left is hyperemic and edematic,
there are some groups of vesicles with 153. A 9 year old boy has been sufferi-
serous and haemorrhagic contents. What ng from diabetes mellitus for a year. He
is the most probable diagnosis? gets insulin injections (humulin R, NPH),
the dose makes up 0,4 units per 1 kg of
A. Herpes zoster body weight a day. Insulin is untroduced
B. Contact dermatitis simplex subcutaneously (into the shoulder) by
C. Contact allergic dermatitis means of a syringe. What measures
D. Microbial eczema should be taken in order to prevent li-
E. Herpetiform Duhring’s dermatosis podystrophy?
150. Internal obstetric examination of A. To change point of introduction
a parturient woman revealed that the B. To limit fats in the boy’s diet
sacrum hollow was totally occupied wi- C. To reduce insulin dose
th fetus head, ischiadic spines couldn’t be D. To apply periodically other types of
detected. Sagittal suture is in the strai- insulin
ght diameter, occipital fontanel is directed E. To administer antioxidants
towards symphysis. In what plane of small
pelvis is the presenting part of the fetus? 154. A 25 year old pediatrician fell
ill a week ago: body temperature rose
A. Plane of pelvic outlet up to 37, 6o C, there appeared a slight
B. Wide pelvic plane swelling on his neck. His illness was di-
C. Narrow pelvic plane agnosed as ARD, cervical lymphadenitis.
D. Plane of pelvic inlet Treatment course included erythromycin,
E. Over the pelvic inlet hot compress on the neck. In course of
Krok 2 Medicine 2008 20

treatment body tempearture rose up to lacunae. What is the most probable di-
39o C, there appeared headache, repeated agnosis?
vomiting, meningeal syndrome. What
studies are necessary for the final di- A. Chronic tonsillitis
agnosis? B. Chronic pharyngitis
C. Acute lacunar tonsillitis
A. Cerebrospinal puncture D. Paratonsillitis
B. Puncture of cervical lymph node E. Tonsillar tumour
C. Complete blood count
D. Sputum test for secondary flora 159. A 43 year old patient had right-
E. Roentgenological examination of lungs sided deep vein thrombosis of iliofemoral
segment 3 years ago. Now he is sufferi-
155. A 41 year old woman has suffered ng from the sense of heaviness, edema
from nonspecific ulcerative colitis for of the lower right extremity. Objectively:
5 years. On rectoromanoscopy: evident moderate edema of shin, brown indurati-
inflammatory process of lower intesti- on of skin in the lower third of shin, varix
nal parts, pseudopolyposive changes of dilatation of superficial shin veins are
mucous membrane. In blood: WBC- 9, 8 · present. What is the most probable di-
109 /l, RBC- 3, 0 · 1012/l, ESR - 52 mm/hour. agnosis?
What medication provides pathogenetic
treatment of this patient? A. Postthrombophlebitic syndrome, vari-
cose form
A. Sulfosalasine B. Acute thrombosis of right veins
B. Motilium C. Lymphedema of lower right extremity
C. Vikasolum D. Parkes-Weber syndrome
D. Linex E. Acute thrombophlebitis of superficial
E. Kreon veins
156. A 49 year old female patient was 160. A 23 year old patient fell ill 3
admitted to a hospital with acute attacks weeks ago when she noticed a very pai-
of headache accompanied by pulsation in nful induration in her axillary crease. 4-5
temples, AP rose up to 280/140 mm Hg. days later it burst and discharged a lot
Pheochromocytoma is suspected. What of pus. After that some new infiltrations
mechanism of hypertensive atack does appeared around the affected area. The
this patient have? patient has never suffered from skin di-
seases before. What is the most probable
A. Increase of catecholamine concentrati- diagnosis?
on
B. Increase of aldosterone level in blood A. Hydradenitis
C. Increase of plasma renin activity B. Furuncle
D. Increase of vasopressin secretion C. Mycosis
E. Increase of thyroxine secretion D. Herpes zoster
E. Streptococcal impetigo
157. A child was born with body weight
3250 g and body length 52 cm. At the age 161. A child is 1 day old. During delivery
of 1,5 month the actual weight is suffici- there had been problems with extraction
ent (4350 g), psychophysical development of shoulders. Body weight is 4300,0. Right
corresponds with the age. The child is arm hangs down along the body, hand is
breast-fed, occasionally there are regurgi- pronated, movement in the arm is absent.
tations. What is the cause of regurgitati- "Scarf"symptom is positive. What is the
ons? most probable diagnosis?
A. Aerophagia A. Total right-sided obstetric paralysis
B. Pylorostenosis B. Proximal right-sided obstetric paralysis
C. Pylorospasm C. Distal right-sided obstetric paralysis
D. Acute gastroenteritis D. Hemiparesis
E. Esophageal atresia E. Tetraparesis
158. Three weeks after acute angina 162. An outbreak of food poisoning was
the patient is still weak, inert, subfebri- recorded in an urban settlement. The
le, his retromaxillary lymph nodes are illness was diagnosed as botulism on the
enlarged. Tonsils are flabby, stick together grounds of clinical presentations. What
with arches, there are purulent plugs in foodstuffs should be chosen for analysis
Krok 2 Medicine 2008 21

in the first place in order to confirm the patients with coronary heart disease.
diagnosis? What parametric criterion (coefficient)
can be used for estimation of results vali-
A. Tinned food dity?
B. Potatoes
C. Pasteurized milk A. Student’s coefficient (t)
D. Boiled meat B. Sign criterion
E. Cabbage C. Conformity coefficient
D. Wilcoxon’s t-criterion
163. A 42 year old metalworker has E. Kolmogorov-Smirnov’s criterion
been working at the turning machine
for production of heavy large-size parts 167. It is planned to make complete
for 5 years. His work requires using isolation boxes in the infectious
of hand and pedal levers that involves department in order to prevent nosocomi-
considerable physical force. What means al airborne infections. The boxes consist
for osteoarthrosis prevention should be of a tambour, a ward and a lock chamber.
recommended? What structure should be also included in
a complete isolation box?
A. To limit physical work
B. To administer protein-and-carbohydrate A. Bathroom unit
diet B. Manipulation room
C. To administer protein-and-vitamin diet C. Doctor’s consulting room
D. To improve health at the Black sea coast D. Patient’s examination room
E. To go in for weightlifting E. Nursing room
164. A 30 year old patient undergoes 168. A 30 year old man complains of
treatment because of Werlhof’s disease. intense pain, reddening of skin, edema
Objectively: the patient is pale, there are in the ankle-joint area, fever up to 39o C.
petechial haemorrhages on the extension There was an acute onset of the illness. In
surfaces of forearms. Ps is 92 bpm, AP is the past there were similar attacks lasting
100/60 mm Hg. The lower edge of spleen 5-6 days without residual changes in the
is at a level with umbilicus. Blood count: joint. The skin over the joint is hyperemic
erythrocytes: 2, 8 · 1012 /l, Hb - 90 g/l, Ht - and ill-defined, without infiltrative bank
0,38, thrombocytes - 30 · 109 /l. The patient on the periphery. What is the most likely
is being prepared for splenectomy. What diagnosis?
transfusion medium should be chosen
in the first place for the preoperational A. Gout
preparation? B. Infectious arthritis
C. Rheumatoid arthritis
A. Thrombocytic mass D. Erysipelatous inflammation
B. Stored blood E. Osteoarthritis
C. Native erythrocytic mass
D. Erythrocytic suspension 169. Preventive examination of a 55 year
E. Washed erythrocytes old patient revealed diabetes mellitus.
The patient hasn’t received treatment for
165. A 19 year old boy was admitted to a it. Objectively: height is 170 cm, weight
hospital with closed abdominal trauma. In is 106 kg, skin humidity is normal. Ps is
course of operation multiple ruptures of 76 bpm, rhythmic, left border of relati-
spleen and small intestine were revealed. ve heart dullness is deviated by 1 cm to
AP is falling rapidly, it is necessary to the left from the middle clavicular line,
perform hemotransfusion. Who can speci- heart sounds are quiet, AP is 140/80 mm
fy the patient’s blood group and rhesus Hg. Glycemia on an empty stomach is 6,9
compatibility? millimole/l. Glucose rate in the daily uri-
ne is 0,5%, diuresis makes up 2,5 l. What
A. A doctor of any speciality treatment tactics should be chosen?
B. A laboratory physician
C. A surgeon A. To administer dietotherapy
D. A traumatologist B. To administer metformin
E. An anaesthesilogist C. To administer glibenclamid
D. To administer repaglinide
166. Clinical and statistical study E. To administer insulin
was devoted to the effect of a new
pharmacological medication upon the 170. The results of 5 year monitoring
Krok 2 Medicine 2008 22

allowed to estimate the level of envi- has Kussmaul’s respiration, there is also
ronmental influence upon health indi- smell of acetone in the air. Symptoms of
ces of popultaion. What statistic method peritoneum irritation are positive. Blood
should be chosen? sugar is at the rate of 33 millimole/l. What
emergency actions should be taken?
A. Calculation of correlation coefficient
B. Calculation of conformity coefficient A. Intravenous infusion of short-acting
C. Calculation of coefficient of difference insulin
validity B. Intravenous infusion of glucose along
D. Calculation of regression coefficient with insulin
E. Calculation of dynamic indices C. Introduction of long-acting insulin
D. Intravenous infusion of
171. In order to improve organism neohaemodesum along with glutamic
tolerance of boarding-school pupils acid
a doctor developed a program. The E. Intravenous infusion of sodium chloride
program is based upon the following pri- saline
nciples: graduality, consistency, indivi-
duality, coomplexity. What of the main 175. In order to study impact of microcli-
principles of organism tempering wasn’t mate upon the human organism it is
taken into account? necessary to make systematic observati-
on of air temperature over 3 days. Choose
A. Systematicness a device that will allow to make the most
B. Autodefense increase precise temperature records:
C. Increase of influence force
D. Increase of influence intensity A. Thermograph
E. Increase of resistance B. Alcohol thermometer
C. Mercury thermometer
172. A patient complains about D. August’s psychrometer
pyrosis and permanent pain behind his E. Assmann psychrometer
breastbone. When he bends forward
after eating there appears regurgi- 176. A 3 month old child has occiput
tation. Roentgenological examination alopecia, anxious sleep, excessive sweati-
revealed extrasaccular cardiofunctional ng. What disease might be suspected?
hernia of esophageal opening of di-
aphragm. Esophagoscopy revealed si- A. Rachitis
gns of reflux-esophagitis. What is the B. Spasmophilia
necessary tretment tactics? C. Anemia
D. Phosphate diabetes
A. Operation in a surgical department E. Chondrodystrophy
B. Conservative treatment in an outpati-
ents’ clinic 177. After delivery and revision of
C. Conservative treatment in the placenta there was found the defect of
therapeutic hospital placental lobule. General condition of
D. Conservative treatment in a surgical woman is normal, uterus is firm, there
department is moderate bloody discharge. Speculum
E. Treatment at a health resort inspection of birth canal shows absence of
lacerations and raptures. What action is
173. 15 minutes after the second vacci- nesessary?
nation with diphteria and tetanus toxoi-
ds and pertussis vaccine a 4 month old A. Manual exploration of the uterine
boy manifested symptoms of Quincke’s cavity
edema. What medication should be appli- B. External massage of uterus
ed for emergency aid? C. Introduction of uterine contracting
agents
A. Prednisolone D. Urine drainage, cold on the lower
B. Heparin abdomen
C. Adrenalin E. Introduction of hemostatic medications
D. Furosemide
E. Seduxen 178. A 7 year old boy has periods of loss of
attention that last 10-15 seconds and occur
174. During examination a patient is several times a day. During these peri-
unconscious, his skin is dry and hot, ods he stands still in one position, doesn’t
face hyperemia is present. The patient answer the questions or react when he is
Krok 2 Medicine 2008 23

called by his name. Neurological exami-


nation revealed no changes. What di- A. Lymphadenitis
agnostic method should be applied to B. Lymphangitis
specify his diagnosis? C. Phlegmon
D. Tetanus
A. Electroencephalography E. Erysipelas
B. Skull roentgenogram
C. Computer tomogram 183. A 52 year old patient complains
D. Echoencephalography about headache, weakness of his upper
E. Examination of evoked potentials left extremity. Neurological symptoms
become more intense during physical
179. A 52 year old patient complains stress of the left extremity. Pulsation on
about pain in the right part of her chest, the arteries of the left extremity is sharply
dyspnea, cough with a lot of foul-smelling dampened but it remains unchanged on
albuminoid sputum in form of "meat the carotid arteries. What is the most
slops". Objectively: the patient’s condi- probable diagnosis?
tion is grave, cyanosis is present, breathi-
ng rate is 31/min, percussion sound above A. Occlusion of the left subclavicular
the right lung is shortened, auscultation artery, steal syndrome
revealed different rales. What is the most B. Thoracal outlet syndrome
probable diagnosis? C. Raynaud’s syndrome
D. Takayasu’s disease
A. Lung gangrene E. Occlusion of brachiocephalic trunk
B. Lung abscess
C. Pleura empyema 184. A 57 year old patient abruptly lost the
D. Multiple bronchiectasis sight of one eye. Examination revealed
E. Chronic pneumonia that his sight made up 0,02 excentri-
cally, eye fundus has haemorrhages of
180. A 15 year old patient suffers from different forms and sizes ("squashed
headache, nasal haemorrhages, sense of tomato"symptom). Disc of optic nerve is
lower extremity coldness. Objectively: hyperemic. In anamnesis general vascular
muscles of shoulder girdle are developed, pathology is recorded. Direct-acting anti-
lower extremities are hypotrophied. coagulants were administered. What is the
Pulsation on the pedal and femoral arteri- most probable diagnosis?
es is sharply dampened. AP is 150/90 mm
Hg, 90/60 on the legs. Systolic murmur A. Thrombosis of central vein of retina
can be auscultated above carotid arteries. B. Hypertensive angiopathy
What is the most probable diagnosis? C. Hypertensive angioneuropathy
D. Embolism of central artery of retina
A. Aorta coarctation E. Diabetic retinopathy
B. Aorta aneurism
C. Aortal stenosis 185. A 28 year old patient complained
D. Aortal insufficiency about prolongation of intermenstrual
E. Coarctation of pulmonary artery periods up to 2 months, hirsutism.
Gynaecological examination revealed
181. A patient with frostbite of both feet that the ovaries were enlarged, pai-
was delivered to the admission ward. nless, compact, uterus had no peculari-
What actions should be taken? ties. Pelvic ultrasound revealed that the
ovaries were 4-5 cm in diameter and had
A. To apply a bandage, to introduce multiple enlarged follicles on periphery.
vasodilating medications Roentgenography of skull base showed
B. To administer cardiac medications that sellar region was dilated. What is the
C. To put feet into hot water most probable diagnosis?
D. To rub feet with snow
E. To apply an alcohol compress A. Stein-Leventhal syndrome
B. Algodismenorrhea
182. A patient has a stab wound on his ri- C. Sheehan’s syndrome
ght foot. On the fourth day after injury D. Premenstrual syndrome
the patient’s body temperature rose up E. Morgagni-Stewart syndrome
to 38o C, inguinal lymph nodes became
enlarged and painful, skin over them 186. A woman consulted a therapeuti-
reddened. What complication might be st about fatigability, significant weight
suspected? loss, weakness, loss of appetite. She has
Krok 2 Medicine 2008 24

been having amenorrhea for 8 months. thrice a year - in February, in April and in
A year ago she born a full-term child. December. How should these occurences
Haemorrhage during labour made up 2 be recorded?
l. She got blood and blood substitute
transfusions. What is the most probable A. It is necessary to fill in 3 statistic talons
diagnosis? signed (+)
B. It is necessary to fill in 3 statistic talons
A. Sheehan’s syndrome signed (−)
B. Stein-Leventhal syndrome C. It is necessary to fill in 1 statistic talon
C. Shereshevsky-Turner’s syndrome signed (+)
D. Homological blood syndrome D. It is necessary to fill in 1 statistic talon
E. Vegetovascular dystonia signed (+) and 2 statistic talons signed (−)
E. It is necessary to fill in 1 statistic talon
187. A 68 year old patient complains about signed (−)
acute pain in his right foot, toe edema
and darkening of skin of the IV toe. He 191. Researchers studied disease inci-
has been suffering from diabetes melli- dence of influenza and acute respiratory
tus for 15 years, doesn’t receive regular viral infection within the last 5 years.
treatment. What complication of diabetes What kind of graphic presentation should
mellitus is it? be used for the best visualization of this
data?
A. Gangrene of the IV toe on the right
foot A. Linear diagram
B. Panaritium B. Pie diagram
C. Haematoma C. Bar diagram
D. Erysipelas D. Histogram
E. Fracture of the IV toe on the right foot E. Radial diagram
188. Preventive examination of an 11 year 192. A 55 year old patient felt suddenly
old boy helped to determine his habi- sick in a hospital corridor, he was immedi-
tus type. It was established that the chi- ately examined by a doctor. Examination
ld’s shoulders were deviated and brought revealed that the patient’s skin was pale,
forward, with forward flexion of head, autonomous respiration was absent, pulse
the thorax was flattened, abdomen was on carotid arteries couldn’t be felt, pupi-
convex. The child’s backbone had signs of ls were mydriatic. What action should be
deepened cervical and lumbar curvatures. taken at the beginning of cardiac resusci-
What habitus is it? tation?
A. Kyphosis A. Precordial thump
B. Lordosis B. Mouth-to-mouth ventilation
C. Round-shouldered C. Closed-chest cardiac massage
D. Corrected D. Restoration of airway patency
E. Normal E. Defibrillation
189. During hygienic examination of a 193. A 24 year old patient complai-
hospital it was established that the area ned about putting on weight, limosis.
for each bed in a double ward was: in the Objectively: the patient’s constitution is
infectious department for children - 7 2 , of hypersthenic type, body weight index
in the infectious department for adults - is 33,2 kg/m2 , waist circumference is 100
8 2 , in the burns department - 9 2 , in cm. Correlation of waist circumference to
the radiological department - 10 2 , in the the thigh circumference is 0,95. What is
critical care department - 13 2 . In whi- the most probable diagnosis?
ch wards the area for each bed doesn’t
correspond with hygienic requirements?
A. In burns wards
B. In infectious wards for children
C. In infectious wards for adults
D. In radiological wards
E. In critical care wards
190. A 2 year old child has been ill with
acute respiratory viral infection of upper
Krok 2 Medicine 2008 25

A. Alimentary constitutional obesity of A. β-adrenoceptor blocker


the I stage, abdominal type B. Adenosine pyrophosphate inhibitor
B. Hypothalamic Itsenko-Cushing obesity C. Diuretic
of the II stage, gynoid type D. α-adrenoceptor blocker
C. Alimentary constitutional obesity of the E. Dihydropyridine calcium antagonist
III stage, gynoid type
D. Alimentary constitutional obesity of the 197. A 30 year old patient complains
II stage, abdominal type about inability to become pregnant over
E. Hypothalamic Itsenko-Cushing obesity 3 years of married life. The patient is of
of the I stage, abdominal type supernutrition type, she has hair along the
median abdominal line, on the internal
194. A parturient woman is 27 year old, thigh surface and in the peripapillary area.
it was her second labour, delivery was at Menses started at the age of 16, they are
term, normal course. On the 3rd day of infrequent and non-profuse. US revealed
postpartum period body temperature is that the uterus was of normal size, ovaries
36, 8oC, Ps - 72/min, AP - 120/80 mm Hg. were 4х5х5 cm large and had a lot of cystic
Mammary glands are moderately swollen, inclusions. What is the most probable di-
nipples are clean. Abdomen is soft and agnosis?
painless. Fundus of uterus is 3 fingers
below the umbilicus. Lochia are bloody, A. Polycystic ovaries
moderate. What is the most probable di- B. Ovarian cystoma
agnosis? C. Chronic oophoritis
D. Menstrual irregularity
A. Physiological course of postpartum E. Bilateral ovarian tumours
period
B. Subinvolution of uterus 198. A 4 month old child was admitted
C. Postpartum metroendometritis to a surgical department 8 hours after
D. Remnants of placental tissue after the first attack of anxiety. The attacks
labour happen every 10 minutes and last for 2-
E. Lactostasis 3 minutes, there was also one-time vomi-
ting. Objectively: the child’s condition is
195. Esophagus wall of a 72 year grave. Abdomen is soft, palpation reveals
old patient with severe concomitant a tumour-like formation in the right ili-
pathology was injured during urgent fi- ac area. After rectal examination the
broesophagogastroscopy. This resulted in doctor’s finger was stained with blood.
progressing of acute respiratory failure What is the most probable diagnosis?
and collapse of the left lung. What aid
should be rendered? A. Ileocecal invagination
B. Gastrointestinal haemorrhage
A. Drainage of pleural cavity by Bullaux C. Wilm’s tumour
method, mediastinum drainage, anti- D. Helminthic invasion
bacterial therapy E. Pylorostenosis
B. Buelau’s drainage of pleural cavity,
antibacterial therapy 199. During preventive examination a 16
C. Left-sided thoracotomy, closure of year old patient presented no problems.
esophagus and mediastinum wound Objectively: the patient has signs of
D. Left-sided thoracotomy, closure of malnutrition, he is asthenic, AP is 110/70
esophagus wound mm Hg, Ps is 80 bpm, cardiac border
E. Endoscopic closure of esophagus is normal, auscultation above the cardi-
wound, drainage ac apex reveals three sounds, cardi-
ac murmur is absent. ECG shows no
196. A 46 year old woman who has been pathological changes, phonocardiogram
suffering from hypertension for 5 years shows that the third sound comes 0,15 s
was diagnosed with hypertensive crisis. after the second one above the apex. How
She complains about palpitation, sense are these changes called?
of head pulsation; heart rate is 100/min,
AP is 190/100 mm Hg (haemodynamics A. III physiological sound
is of hyperkinetic type). What medication B. Fout-ta-ta-rou (reduplication of the 2nd
should be the medication of choice? sound)
C. Protodiastolic gallop rhythm
D. Atrial gallop rhythm
E. IV physiological sound
Krok 2 Medicine 2008 26

200. A 52 year old patient with dissemi- pathogenetic point of view?


nated vertebral osteochondrosis lifted a
significant load that resulted in lumbar A. Diclofenac
pain and pain along the sciatic nerve. B. Aspirin
Objectively: positive Lasegue’s sign on C. Analgin
the left, reduced Achilles reflex. What D. Novocaine
drug would be the most effective from the E. Spasmalgon
Krok 2 Medicine 2009 1

1. A woman complains of having slight A. Infectious contamination of the urinary


dark bloody discharges and mild pains in system
the lower part of abdomen for several B. Thrombophlebitis of veins of the pelvis
days. Last menses were 7 weeks ago. C. Infectious hematoma
The pregnancy test is positive. Bimanual D. Endometritis
investigation: the body of the uterus indi- E. Apostasis of sutures after episiotomy
cates for about 5-6 weeks of pregnancy,
it is soft, painless. In the left appendage 5. In the woman of 24 years about earli-
there is a retort-like formation, 7х5 cm er normal menstrual function, cycles
large, mobile, painless. What examination became irregular, according to tests of
is necessary for detection of fetus locali- function diagnostics - anovulatory. The
zation? contents of prolactin in blood is boosted.
Choose the most suitable investigation:
A. Ultrasound
B. Hysteroscopy A. Computer tomography of the head
C. Hromohydrotubation B. Determination of the level of
D. Colposcopy gonadotropins
E. Cystoscopy C. USI of organs of small pelvis
D. Progesterone assay
2. A pregnant woman in her 40th E. Determination of the contents of
week of pregnancy undergoes obstetric testosteron-depotum in blood serum
examination: the cervix of uterus is
undeveloped. The oxytocin test is negati- 6. A patient was delivered to a surgical
ve. Examination at 32 weeks revealed: department after a road accident with a
AP 140/90 mm Hg, proteinuria 1 g/l, peri- closed trauma of chest and right-sided
pheral edemata. Reflexes are normal. rib fracture. The patient was diagnosed
Choose the most correct tactics: with right-sided pneumothorax, it is indi-
cated to perform drainage of pleural cavi-
A. Labour stimulation after preparation ty. Pleural puncture should be made in:
B. Absolute bed rest for 1 month
C. Complex therapy of gestosis for 2 days A. In the 2nd intercostal space along the
D. Caesarian section immediately middle clavicular line
E. Complex therapy of gestosis for 7 days B. In the 6th intercostal space along the
posterior axillary line
3. A 26 year old woman had the second C. In the 7th intercostal space along the
labour within the last 2 years with scapular line
oxytocin application. The child’s weight is D. In the projection of pleural sinus
4080 g. After the placent birth there were E. In the point of the greatest dullness on
massive bleeding, signs of hemorrhagic percussion
shock. Despite the injection of contracti-
ve agents, good contraction of the uterus 7. A patient who takes diuretics has
and absence of any cervical and vaginal developed arrhythmia as a result of
disorders, the bleeding proceeds. Choose cardiac glycoside overdose. What is the
the most probable cause of bleeding: treatment tactics in this case?

A. Atony of the uterus A. Increased potassium concentration in


B. Injury of cervix of the uterus blood
C. Hysterorrhexis B. Increased sodium consentration in
D. Delay of the part of placenta blood
E. Hypotonia of the uterus C. Reduced magnesium concentration in
blood
4. A woman of a high-risk group (chronic D. Increased calcium concentration in
pyelonephritis in anamnesis) had vagi- blood
nal delivery. The day after labour she E. -
complained of fever and loin pains,
frequent urodynia. Specify the most 8. A 20 year old patient was delivered to
probable complication: the hospital in summer from the street wi-
th haemorrage from the brachial artery.
First medical aid involved aplication of
a tourniquet for provisional arrest of
bleeding. What is the maximal exposure
of the tourniquet?
Krok 2 Medicine 2009 2

A. 120 minutes A. Leftsided renal colic


B. 15 minutes B. Herpes zoster
C. 30 minutes C. Sigmoid diverticulitis
D. 60 minutes D. Torsion of the left testicle
E. 180 minutes E. Retroperitoneal haemorrhage
9. The average body lenth of newborn 13. A 40 year old woman has a self-
boys is 50,9 cm at a sigma 1,66; and detected hard breast mass. The procedure
average mass - 3432 at a sigma 5,00. What of choice for confirming the diagnosis is:
criterion is necessary in order to compare
degree of variability of these signs? A. Excision biopsy
B. Mammography
A. Coefficient of variation C. Thermography
B. Sigma D. Ultrasonography
C. Limit E. Aspiration biopsy with cytology
D. Amplitude
E. Coefficient of association 14. What is the maximum durati-
on of medical certificate in case of
10. Indicate the registration medical tuberculosis?
document for the patient, who 21.02. was
addressed to the doctor with diagnosis A. 2 months
ARVD for the first time in this year: B. Week
C. 2 weeks
A. The statistical coupon is to be filled in D. Month
and it is necessary to deliver on a sign (+) E. 10 months
B. The statistical coupon for registration
of final diagnosis is not necessary 15. The student has the following devices:
C. The statistical coupon is to be filled in, Geiger counter, Ebert counter, Krotov’s
but a sign (+) is not necessary to be put in apparatus, Mischuk device, Ebert device.
D. It is necessary to fill in the emergency What device can he use to assess air germ
notice on a case of a contagion pollution?
E. The necessary registration form is not A. Krotov’s apparatus
indicated B. Ebert’s counter
11. Five days after a total hip joi- C. Geiger’s counter
nt replacement a 72 year old woman D. Mischuk’s device
becomes acutely short of breath, di- E. Ebert’s device
aphoretic and hypotensive. Both lung 16. Student В. lives in the canalized
fields are clear to auscultation and house in the flat with complete set of
percussion, but examination of the neck
sanitary equipment (WC, bath, shower,
reveals mild jugular venous distension wi-
th prominent A waves. Heart sounds are local water heater). How much water
normal. ECG shows sinus tachycardia wi- consumption has he got?
th a new right bundle branch block and A. 160-200 l
minor nonspecific ST − T wave changes. B. 10-15 l
The most likely diagnosis is: C. 50-100 l
A. Pulmonary thromboembolism D. 300-400 l
B. Acute myocardial infarction E. 500 l
C. Aortic dissection 17. What guarantees against the
D. Pericarditis preconceived attitude to the physician in
E. Aspiration cases of professional law violations do
12. A 38 year old man, previously in you know?
good health, suddenly develops severe
abdominal pain radiating from the left
loin to groin and accompanied by nausea,
perspiration and the need for frequent
urination. He is restless, tossing in bed
but has no abnormal findings. The most
likely diagnosis is:
Krok 2 Medicine 2009 3

A. Sanction of public prosecutor, inquiry is transparent, other parts of eyeball


by preliminary investigator of prosecutor’s have no changes. Visus 0,9. Right eye
office, committee of experts is healthy, Visus 1,0. What additional
B. Draw up a statement about forensic method would you choose first of all?
medical examination
C. Conduct an inquiry by preliminary A. Staining test with 1% fluorescein
investigator of police department B. X-ray examination of orbit
D. Utilisation copy of medical documents C. Tonometria
E. Conduct forensic medical examination D. Gonioscopia
by district forensic medicine expert E. Cornea sensation-test
18. A 63 year old patient was diagnosed 23. A patient with acute purulent otitis
with purulent mediastinitis. What of the media complicated by mastoiditis was
below listed diseases are NOT the cause admitted to a hospital. Roentgenogram
of purulent mediastinitis? of mastoid processes showed the shadi-
owing of the cellular system on the lesion,
A. Cervical lymphadenitis absence of bone septa was present. What
B. Deep neck phlegmon are the necessary therapeutic actions at
C. Perforation of the cervical part of the the second stage of mastoiditis?
oesophagus
D. Perforation of the thoracic part of the A. Mastoidotomy
oesophagus B. Paracentesis of the drum
E. Iatrogenic injury of the trachea C. Radical operation on the middle ear
D. Tympanoplasty
19. A 36 year old patient was diagnosed E. Cateterization of the Eustachian tube
with right-sided pneumothorax. What
method of treatment is indicated to the 24. The most available and informative
patient? diagnostic method for closed trauma of
the urinary bladder is:
A. Surgical treatment: drainage of the
pleural cavity A. Retrograde cystography
B. Antiinflammation therapy B. Pelvic arteriography
C. Symptomatic therapy C. Cystography
D. Pleural puncture D. Sonography of the urinary bladder
E. Thoracotomy E. Palpation and percussion of abdomen
20. It is suspected that a 34 year old pati- 25. Female 45 year old patient was admi-
ent has an abscess of Douglas pouches. tted to the traumatological ward with the
What diagnostic method is to be chosen? closed fracture of the medial malleolus
with its displacement up to 3 mm. The
A. Digital examination of rectum foot is to be fixed with a plaster cast in
B. Rectoromanoscopy the following position:
C. Laparoscopy
D. Percussion and auscultation of stomach A. At right angle with varus positioning
E. R-scopy of abdominal cavity of the foot
B. In position of planter flexion of foot
21. A patient has restrained umbilateral C. In position of pronation
hernia complicated by phlegmon hernia, D. In position of supination
it is necessary to take following actions: E. In position of dorsal flexion of foot
A. Herniotomy by Mayo-Sapezhko 26. A 3 month old infant suffering from
B. Herniotomy by Mayo acute segmental pneumonia has dyspnea
C. Herniotomy by Sapezhko (respiration rate - 80 per minute),
D. Herniotomy by Lekser paradoxical breathing, tachycardia, total
E. Herniotomy by Grenov cyanosis. Respiration and pulse - ratio is
1:2. The heart dullness under normal size.
22. A 10 year old boy complains Such signs characterise:
about pain in his left eye and strong
photophobia after he had injured his
left eye with a pencil at school. Left
eye examination: blepharospasm, cili-
ary and conjunctival congestion, cornea
Krok 2 Medicine 2009 4

A. Respiratory failure of III degree


B. Respiratory failure of I degree A. Replacement blood transfusion
C. Respiratory failure of II degree (conservative therapy)
D. Myocarditis B. Conservative therapy
E. Congenital heart malformation C. Blood transfusion (conservative
therapy)
27. A 3 year old child has been sufferi- D. Symptomatic therapy
ng from fever, cough, coryza, conjuncti- E. Antibiotics
vitis for 4 days. He has been taking
sulfadimethoxine. Today it has fever up 31. Infant is 6,5 months now and is given
to 39o C and maculopapular rash on its natural feeding since birth. Body mass
face. Except of rash the child’s skin has was 3,5 kg, with length 52 cm at birth.
no changes. What is your diagnosis? How many times per day the supplement
(up feeding) should be given?
A. Measles
B. Allergic rash A. 2
C. Rubella B. 3
D. Scarlet fever C. 1
E. Pseudotuberculosis D. 0
E. 4
28. A 2 year old girl has been ill for
3 days. Today she has low grade fever, 32. A 12 year old child has the ulcer di-
severe catarrhal presentations, slight sease of stomach. What is the etiology of
maculopapular rash on her buttocks and this disease?
enlarged occipital lymph nodes. What is
your diagnosis? A. Intestinal bacillus
B. Helicobacter pylory
A. Rubella C. Salmonella
B. Scarlet fever D. Lambliosis
C. Measles E. Influenza
D. Adenoviral infection
E. Pseudotuberculosis 33. A nine year old child is at a hospi-
tal with acute glomerulonephritis. Cli-
29. A 3 year old boy fell ill abruptly: nical and laboratory examinations show
fever up to 39o C , weakness, vomi- acute condition. What nutrients must not
tng. Haemorrhagic rash of various si- be limited during the acute period of
ze appeared on his lower limbs within glomerulonephritis?
5 hours. Meningococcemia with infecti-
ve - toxic shock of the 1 degree was di- A. Carbohydrates
agnosed. What medications should be B. Salt
administered? C. Liquid
D. Proteins
A. Chloramphenicol succinate and predni- E. Fats
sone
B. Penicillin and prednisone 34. Examination of a 3-month-old chi-
C. Penicillin and immunoglobulin ld revealed scrotum growth on the right.
D. Chloramphenicol succinate and This formation has elastic consistency, its
interferon size decreases during sleep and increases
E. Ampicillin and immunoglobulin when the child is crying. What examinati-
on will be helpful for making a correct
30. A woman delivered a child. It was diagnosis?
her fifth pregnancy but the first deli-
very. Mother’s blood group is A(II)Rh− , A. Palpation of the thickened cord crossi-
newborn’s - A(II)Rh+ . The level of indi- ng the pubical tubercule (sign of the silk
rect bilirubin in umbilical blood was 58 glove)
micromole/l, haemoglobin - 140 g/l, RBC- B. Diaphanoscоpy
3, 8 · 1012 /l. In 2 hours the level of indi- C. Palpation of the external inguinal ring
rect bilirubin turned 82 micromole/l. The D. Punction of the scrotum
hemolytic disease of newborn (icteric- E. Examination of the formation in
anemic type, Rh-incompatibility) was di- Trendelenburg’s position
agnosed. Choose the therapeutic tactics:
35. A 52 year old patient with history
Krok 2 Medicine 2009 5

of functional Class II angina complai- A. Focal


ns of having intense and prolonged B. Disseminated
retrosternal pains, decreased exercise C. Miliary
tolerance for 5 days. Angina is less D. Fibro-cavernous
responsive to nitroglycerine. What is the E. Tuberculoma
most probable diagnosis?
40. A woman 26 years old has abused
A. IHD. Unstable angina alcohol for 7 years. She has psychologi-
B. Cardialgia due to spine problem cal dependence on alcohol, but no wi-
C. IHD. Functional Class II angina thdrawal syndrome. Drinks almost every
D. Myocarditis day approximately 50-100 g of wine. She
E. Myocardial dystrophy is in her 4-th week of pregnancy. Primary
prevention of fetal alcohol syndrome
36. A 52 year old patient has requires:
hypervolaemic type of essential
hypertension. Which of the following A. Treatment of alcoholism and full absti-
medications is to be prescribed either as nance from alcohol during all the period
monotherapy or in complex with other of pregnancy
antihypertensive drugs? B. Medical abortion
C. Decrease of alcohol use
A. Hypothiazid D. Participation in the A-ANON group
B. Dibazol E. Gyneacological observation
C. Clonidine
D. Kapoten 41. The observed patient’s movements
E. Nifedipin are retarded, she answers no questions.
Sometimes she spontaneously stiffens in
37. A 62 year old patient complains of strange postures. It is possible to set her
rest dyspnea, heart pains. 3 years ago body and limbs into different positions
he had myocardial infarction. Physical artificially. If the psychiatrist lifts her arm
examination: orthopnea, acrocyanosis, or leg, so that she remains standing on
swollen cervical veins. Ps - 92, total heart the other leg, the patient can stay in such
enlargement, the liver is enlarged by 7 a position for quite a long time. Name the
cm, shin edema. What is the stage of probable disorder:
chronic heart failure (CHF)?
A. Catatonic stupor, schizophrenia
A. CHF- 2 B B. Depressive stupor, bipolar disorder
B. CHF- 1 C. Apathetic stupor, schizophrenia
C. CHF- 2 А D. Psychogenic stupor, stress disorder
D. CHF- 0 E. Dissociative stupor, dissociative
E. CHF- 3 psychosis
38. A 27 year old man complains of pai- 42. The man, aged 42, applied to the
ns in epigastrium which are relieved by therapeutist with complaints of pricking
food intake. EGDFS shows antral erosive pains in scapulas area, dyspnea on physi-
gastritis, biopsy of antral mucous presents cal exertion, cough with discharge of
Hеlicobacter Pylori. Diagnosis is: small amount of sputum. During 10 years
he works in coal mining. On percussion-
A. Gastritis of type B box-note sound in the lower parts, on
B. Gastritis of type A auscultation- a harsh breathing. There
C. Reflux-gastritis were no changes in the heart. Possible
D. Menetrier’s gastritis diagnosis?
E. Rigid antral gastritis
A. Silicosis
39. Prophylactic photoroentgenography B. Tuberculosis of lungs
examination of a 25 year old man C. Silicatosis
revealed focal shadowings of small and D. Bronchiectatic disease
medium intensity with irregular contours E. Chronic bronchitis
in the 1st and 2nd segments of the ri-
ght lung. Which clinical form can be 43. A man, aged 37, working on the
suspected? collective farm on sowing, was admitted
to the infectious hospital with the clini-
cal symptoms: miosis, labored breathing,
Krok 2 Medicine 2009 6

sweating.What kind of poisoning is it and weakness, high temperature, sore throat.


what is the first aid? Objectively: enlargement of all groups of
lymph nodes was revealed. The liver is
A. Poisoning by POC. Treatment: atropine enlarged by 3 cm, spleen - by 1 cm. In
B. Poisoning by lead. Treatment: tetacine blood: leukocytosis, atypical lymphocytes
Calcii - 15%. What is the most probable di-
C. Poisoning by the methylic alcohol. agnosis?
Treatment: ethylic alcohol
D. Poisoning by vapours of mercury. A. Infectious mononucleosis
Treatment: unithiol B. Acute lymphoid leukosis
E. - C. Diphtheria
D. Angina
44. The 25 year old patient was admi- E. Adenoviral infection
tted on the 1st day of the disease wi-
th complaints of double vision in the 48. A 4 year old girl was playing with
eyes, difficult respiration. The day before her toys and suddenly she got an attack
the patient ate home-made mushrooms. of cough, dyspnea. Objectively: respirati-
On objective examination: paleness, wi- on rate - 45/min, heart rate - 130/min.
dened pupils, disorder of swallowing, Percussion revealed dullness of percutory
bradycardia, constipation are marked. sound on the right in the lower parts.
What is the diagnosis? Auscultation revealed diminished breath
sounds with bronchial resonance on the
A. Botulism right. X-ray pictue showed shadowing of
B. Yersiniosis the lower part of lungs on the right. Blood
C. Leptospirosis analysis revealed no signs of inflammati-
D. Salmonellosis, gastrointestinal form on. The child was diagnosed with foreign
E. Lambliasis body in the right bronchus. What compli-
cation caused such clinical presentations?
45. A 28 year old patient was admi-
tted to the clinic with complaints of the A. Atelectasis
temperature rise up to 39, 0oC , headache, B. Emphysema
weakness, constipation on the 9th day C. Pneumothorax
of the disease. On examination: single D. Bronchitis
roseolas on the skin of the abdomen are E. Pneumonia
present. The pulse rate is 78 bpm. The li-
ver is enlarged by 2 cm. What is the most 49. A 75 year old man who has been
probable diagnosis? suffering from diabetes for the last six
months was found to be jaundiced. He
A. Typhoid fever was asymptomatic except for weight loss
B. Leptospirosis at the rate of 10 pounds in 6 months.
C. Brucellosis Physical examination revealed a hard,
D. Sepsis globular, right upper quadrant mass that
E. Malaria moves during respiration. A CT scan
shows enlargement of the head of the
46. A patient has been in a hospital. The pancreas, with no filling defects in the
beginning of the disease was gradual: liver. The most likely diagnosis is:
nausea, vomiting, dark urine, аcholic
stools, yellowness of the skin and scleras. A. Carcinoma of the head of the pancreas
The liver is protruded by 3 cm. Jaundi- B. Infectious hepatitis
ce progressed on the 14th day of the di- C. Haemolytic jaundice
sease. The liver diminished in size. What D. Malignant biliary stricture
complication of viral hepatitis caused E. Metastatic disease of liver
deterioration of the patient’s condition?
50. A 60 year old man with unstable angi-
A. Hepatic encephlopathy na pectoris fails to respond to heparin,
B. Meningitis nitroglycerin, beta adrenegic blockers
C. Relapse of viral hepatitis and calcium channel antagonist. The best
D. Cholangitis management includes:
E. Infectious-toxic shock
47. An 18 year old patient was admitted to
a hospital with complaints of headache,
Krok 2 Medicine 2009 7

A. Coronary artery bypass grafting murmur on apex. The liver is enlarged by


B. Intravenous streptokinase 5 cm. What is the cause of heart failure?
C. Excercise testing
D. Oral aspirin A. Mitral stenosis
E. Antihypertensive therapy B. Mitral regurgitation
C. Tricuspid stenosis
51. A 22 year old woman complained D. Tricuspid regurgitation
of right subcostal aching pain, nausea, E. Aortic stenosis
and decreased appetite. She fell ill
2 months after appendectomy when 55. A 32 year old welder complains of
jaundice appeared. She was treated in weakness and fever. His illness started as
an infectious hospital. 1 year later above tonsillitis a month before. On exam, BT
mentioned symptoms developed. On of 38, 9o C , RR of 24/min, HR of 100/min,
exam: the subicteric sclerae, enlarged fi- BP of 100/70 mm Hg, hemorrhages
rm liver. Your preliminary diagnosis: on the legs, enlargement of the lymph
nodes. CBC shows Hb of 70 g/l, RBC of
A. Chronic viral hepatitis 2, 2 · 1012 /l, WBC of 3, 0 · 109/l with 32% of
B. Calculous cholecystitis blasts, 1% of eosinophiles, 3% of bands,
C. Gilbert’s disease 36% of segments, 20% of lymphocytes,
D. Acute viral hepatitis and 8% of monocytes, ESR of 47 mm/h.
E. Chronic cholangitis What is the cause of anemia?
52. A 25 year old woman complained A. Acute leukemia
of edema on her face and legs, rise B. Chronic lympholeukemia
of blood pressure up to 160/100 mm C. Aplastic anema
Hg and weakness. She fell ill 3 weeks D. Vitamin B12 deficiency anemia
after recovering from angina. Urinalysis E. Chronic hemolytic anemia
data: protein of 0,5 g/l, erythrocytes
of 17-20/field, leukocytes of 2-3/field, 56. A male patient, 60 years old, tobacco
erythrocyte casts. What treatment should smoker for 30 years, alcoholic, has
be initiated after specifying the di- dysphagia and weight loss since 4 months.
agnosis? Suggested diagnosis?
A. Penicillin OS A. Cancer of the esophagus
B. Heparin B. Esophageal achalasia
C. Ceftriaxone C. Hanter’s disease
D. Dipyridamole D. Esophagitis
E. Ciprofloxacine E. Esophageal diverticulum
53. A fitter of a metallurgic factory 57. Which of the following symptoms
with occupational exposure to high would occur only if a total-body acute
concentrations of mercury fumes for 16 radiation exposure exceeded 5.000 rad
years presents instability of pulse and (50 Gy)
blood pressure, general hyperhydrosis,
asymmetric innervations of facial muscles A. Hallucinations and impairment of
and tongue, positive subcortical reflexes, vision
hand tremor on physical examination. A B. Nausea and vomiting
dentist revealed paradontosis and chronic C. Diarrhea
stomatitis. What is the most probable di- D. Bleeding gums
agnosis? E. Epilation (hair loss)
A. Chronic mercury intoxication 58. In the 43rd week of gestation a
B. Neuroinfection long, thin infant was delivered. He is
C. Parkinson syndrome apneic, limp, pale, and covered with "pea
D. Acute mercury intoxication soup"amniotic fluid. The first step in the
E. Mercury encephalopathy resuscitation of this infant at delivery
should be:
54. A 42 year old woman complai-
ns of dyspnea, edema of the legs and
tachycardia during minor physical exerti-
on. Heart borders are displaced to the left
and S1 is accentuated, there is diastolic
Krok 2 Medicine 2009 8

A. Suction of the trachea under direct abating, but dyspnea, dizziness, pallor,
vision cold sweat and cyanosis were progressi-
B. Artificial ventilation with bag and mask ng. Vesicular respiration is absent, X-ray
C. Artificial ventilation with endotracheal picture shows a shadow on the left. What
tube pathology might be suspected?
D. Administration of 100% oxygen by
mask A. Spontaneous left-sided pneumothorax
E. Catheterization of the umbilical vein B. Pulmonary infarction
C. Pleuritis
59. 25 children at the age of 2-3 years D. Left-sided pneumonia
who don’t attend any child welfare insti- E. Pulmonary abscess
tutions should be observed by a district
pediatrician within the current year. How 63. Which of the methods of examination
many initial visits of this group of chi- is the most informative in the diagnostics
ldren should be planned? of a tube infertility?
A. 50 A. Laparoscopy with chromosalpi-
B. 20 ngoscopy
C. 40 B. Pertubation
D. 100 C. Hysterosalpingography
E. 200 D. Transvaginal echography
E. Bicontrast pelviography
60. A 25 year old patient complai-
ns of pain in the I finger on his ri- 64. A 38 year old man worked at roofi-
ght hand. On examination: the finger is ng and drain pipes production for 15
homogeneously hydropic, in bent positi- years. He seeks medical help for expi-
on. On attempt to unbend the finger the ratory breathlessness on exertion, and
pain is getting worse. Acute pain appears dry cough. On exam, wheezes above
during the probe in ligament projection. both lungs, grayish warts on fingers are
What decease is the most likely? seen. Factory physician has diagnosed
asbestosis. What method is the most
A. Thecal whitlow (ligament panaritium) important for this diagnosis?
B. Subcutaneous panaritium
C. Articular (joint) panaritium A. Chest X-ray
D. Bone panaritium B. Bronchoscopy
E. Paronychia C. Blood gas analysis
D. Spirography
61. A 26 year old manual worker E. Electrocardiography
complained of 3 weeks history of
fevers and fatigue, weight loss with 65. A patient has got pain in the axillary
no other symptoms. Physical findings: area, rise of temperature developed 10
Temperature 37, 6oC , Ps- 88 bpm, blood hours ago. On examination: shaky gait is
pressure 115/70 mm Hg, superficial lymph evident, the tongue is coated with white
nodes (occipital, submental, cervical, axi- deposit. The pulse is frequent. The pai-
llary) are enlarged, neither tender nor nful lymphatic nodes are revealed in the
painful. Rubella-like rash on the trunk axillary area. The skin over the lymph
and extremities. Herpes simplex lesi- nodes is erythematous and glistering.
ons on the lips. Candidosis of oral cavi- What is the most probable diagnosis?
ty. What infectious disease would you A. Bubonic plague
suspect? B. Acute purulent lymphadenitis
A. HIV infection C. Lymphogranulomatosis
B. Influenza D. Anthrax
C. Rubella E. Tularemia
D. Infectious mononucleosis 66. Examination of a 9 month old girl
E. Tuberculosis revealed skin pallor, cyanosis during exci-
62. A patient complains about strong tement. Percussion revealed transverse
dyspnea that is getting worse during dilatation of cardiac borders. Auscultati-
physical activity. Presentations appeared on revealed continuous systolic murmur
suddenly 2 hours ago at work: acute chest on the left from the breastbone in the
pain on the left, cough. The pain was 3-4 intercostal space. This murmur is
Krok 2 Medicine 2009 9

conducted above the whole cardiac regi- should be applied for the diagnosis confi-
on to the back. What congenital cardiac rmation?
pathology can be suspected?
A. Echocardiography
A. Defect of interventricular septum B. Roentgenography of chest
B. Defect of interatrial septum C. Phonocardiography
C. Coarctation of aorta D. ECG
D. Fallot’s tetrad E. Veloergometry
E. Pulmonary artery stenosis
71. A 28 year old parturient complai-
67. A 32 year old patient complains about ns about headache, vision impairment,
cardiac irregularities, dizziness, dyspnea psychic inhibition. Objectively: AP-
at physical stress. He has never suffered 200/110 mm Hg, evident edemata of
from this before. Objectively: Ps- 74 bpm, legs and anterior abdominal wall. Fetus
rhythmic. AP- 130/80 mm Hg. Auscultati- head is in the area of small pelvis. Fetal
on revealed systolic murmur above aorta, heartbeats is clear, rhythmic, 190/min.
the first heart sound was normal. ECG Internal investigation revealed complete
showed hypertrophy of the left ventri- cervical dilatation, fetus head was in the
cle, signs of repolarization disturbance area of small pelvis. What tactics of labor
in the I , V5 and V6 leads. Echocardiogram management should be chosen?
revealed that interventricular septum was
2 cm. What is the most probable di- A. Forceps operation
agnosis? B. Cesarean
C. Embryotomy
A. Hypertrophic cardiomyopathy D. Conservative labor management with
B. Aortic stenosis episiotomy
C. Essential hypertension E. Stimulation of labor activity
D. Myocardium infarction
E. Coarctation of aorta 72. A patient complained about problems
with pain and tactile sensitivity, pain
68. An 8 year old boy suffering from in the nail bones at the end of the
haemophilia was undergoing transfusion working day. He works at a plant with
of packed red cells. Suddenly he got pain mechanical devices. What pathology can
behind the breastbone and in the lumbar be suspected?
area, dyspnea, cold sweat. Objectively:
pale skin, heart rate - 100/min, AP- 60/40 A. Vibration disease
Hg; oliguria, brown urine. For treatment B. Caisson disease
of this complication the following drug C. Noise disease
should be administered: D. Overwork symptoms
E. Hypovitaminosis of B1
A. Prednisolone
B. Lasix 73. A 25 year old patient complains about
C. Adrenaline weakness, dizziness, haemorrhagic skin
D. Aminophylline rash. She has been suffering from this
E. Analgine for a month. Blood count: erythrocytes:
1, 0 · 1012 /l, Hb- 37 g/l, colour index - 0,9,
69. A maternity hospital registered 616 li- leukocytes - 1, 2 · 109 /l, thrombocytes -
ve births, 1 stillbirth, 1 death on the 5th 42 · 109 /l. What diagnostic method will be
day of life over a 1 year period. What the most effective?
index allows the most precise estimation
of this situation? A. Sternal puncture
B. Spleen biopsy
A. Perinatal mortality C. Liver biopsy
B. Crude mortality rate D. Coagulogram
C. Natality E. Abdominal ultrasound
D. Neonatal mortality
E. Natural increase 74. A 68 year old female patient complai-
ns about temperature rise up to 38, 3oC ,
70. After objective clinical examination a haematuria. ESR- 55 mm/h. Antibacteri-
12 year old child was diagnosed with mi- al therapy turned out to be ineffective.
tral valve prolapse. What complementary What diagnosis might be suspected?
instrumental method of examination
Krok 2 Medicine 2009 10

78. A healthy 75 year old woman who


A. Renal cancer leads a moderately active way of life
B. Polycystic renal disease went through a preventive examinati-
C. Renal amyloidosis on that revealed serum concentration
D. Urolithiasis of common cholesterol at the rate of
E. Chronic glomerulonephritis 5,1 millimole/l and HDL (high-density
75. A 48 year old female patient complai- lipoproteins) cholesterol at the rate of
ns about contact haemorrhage. Speculum 70 mg/dl. ECG reveals no pathology.
What dietary recommendation is the
examination revealed hypertrophy of most adequate?
uterus cervix. It resembles of cauliflower,
it is dense and can be easily injured. Bi- A. Any dietary changes are necessary
manual examination revealed that forni- B. Decrease of cholesterol consumption
ces were shortened, uterine body was C. Decrease of saturated fats consumption
nonmobile. What is the most probable D. Decrease of carbohydrates consumpti-
diagnosis? on
A. Cervical carcinoma E. Increase of cellulose consumption
B. Metrofibroma 79. A parturient complains about pain in
C. Endometriosis the mammary gland. Palpation revealed
D. Cervical pregnancy a 3х4 cm large infiltration, soft in the
E. Cervical papillomatosis centre. Body temperature is 38, 5oC .
76. A 34 year old female patient has What is the most probable diagnosis?
been suffering from anxious depressi- A. Acute purulent mastitis
on accompanied by panic attacks for B. Pneumonia
2 years. She has been a patient of a C. Pleuritis
psychotherapist. Treatment resulted in D. Retention of milk
incomplete remission. The patient had E. Birth trauma
to break off psychotherapy because of
moving to a new place of residence. Soon 80. A 52 year old patient was admi-
after this her condition grew significantly tted to a hospital because of hi-
worse, she was almost permanently anxi- gh hemorrhagic diathesis of mucous
ous, panic attacks turned up 5-6 times a membranes, massive skin haemorrhages
day and were accompanied by palpitati- in form of ecchymoses and spots, nasal
on, dyspnea, cold sweat, thanatophobia. and stomachal haemorrhages. After cli-
What drug group is the most appropriate nical examinations her illness was di-
for medicamental therapy? agnosed as thrombocytopenic purpura.
What is the most probable cause of this
A. Antidepressants disease?
B. Sedative neuroleptics
C. Antipsychotic neuroleptics A. Generation of antithrombocytic anti-
D. Lithium drugs bodies
E. Cardiotonics, respiratory analeptics B. Disturbed hemostasis
C. Deficit of the VIII factor of blood
77. 1,5 hour after start of gullet bougi- coagulation
enage a 48 year old patient suffering from D. Inherited insufficiency of plasm factors
corrosive stricture felt acute abdominal of blood coagulation
pain. Previously he had been sufferi- E. Iron deficit in blood serum, bone
ng from duodenal ulcer. Examination marrow and depot
revealed that abdomen was very tense
and painful; Ps- 110 bpm, painful si- 81. A 37 year old patient applied
alophagia, skin pallor. What is the most to a local therapeutist. As a result
probable diagnosis? of exacerbation of chronic obstructi-
ve bronchitis the patient had been
A. Perforation of abdominal part of temporarily disabled for 117 days within 1
esophagus year. What tactics will be legally correct?
B. Acute myocardium infarction
C. Strangulation of diaphragmal hernia
D. Perforation of duodenal ulcer
E. Thrombosis of mesenteric vessels
Krok 2 Medicine 2009 11

A. The patient should be referred to the


medicosocial expertise A. Tuberculous
B. The therapeutist should extend a B. Cancerous
medical certificate C. Staphylococcal
C. The patient should be referred to D. Viral
the medical consultation comission for E. Autoimmune
extension of medical certificate
D. The therapeutist should issue a new 85. At year-end hospital administration
medical certificate has obtained the following data: annual
E. The patient shoul be referred to the number of treated patients and average
sanatorium-and-spa treatment annual number of beds used for patients’
treatment. What index of hospital work
82. A 54 year old male patient complai- can be calculated on the base of this data?
ns about permanent dull pain in the
mesogastral region, weight loss, dark A. Bed turnover
blood admixtures in the feces, constipati- B. Bed resources of the hospital
ons. He put off 10 kg within a year. In C. Average annual bed occupancy
blood: erythrocytes: 3, 5·1012 /l, Hb- 87 g/l, D. Average duration of patients’ presence
in the hospital
leukocytes - 12, 6 · 109 /l, stab neutrophil E. Average bed idle time
shift, ESR- 43 mm/h. What is the most
probable diagnosis? 86. A female patient has been suffering
from pain in the right subcostal area, bi-
A. Cancer of transverse colon tter taste in the mouth, periodical bile
B. Gastric ulcer vomiting for a month. The patient put off
C. Chronic colitis 12 kg. Body temperature in the evening
D. Chronic pancreatitis is 37, 6oC . Sonography revealed that bi-
E. Stomach cancer le bladder was 5,5х2,7 cm large, its wall -
83. A 10 month old boy has been ill for 0,4 cm, choledochus - 0,8 cm in diameter.
5 days after consumption of unboiled Anterior liver segment contains a roundi-
milk. Body temperature is 38 − 39o C , sh hypoechoic formation up to 5 cm in
there is vomiting, liquid stool. The chi- diameter and another two up to 1,5 cm
ld is pale and inert. His tongue is covered each, walls of these formations are up to
with white deposition. Heart sounds are 0,3 cm thick. What is the most probable
muffled. Abdomen is swollen, there is diagnosis?
borborygmus in the region of ubbilicus, A. Alveolar echinococcus of liver
liver is enlarged by 3 cm. Stool is liquid, B. Liver cancer
dark-green, with admixtures of mucus, 5 C. Liver abscess
times a day. What is the most probable D. Cystous liver cancer
diagnosis? E. Paravesical liver abscesses
A. Salmonellosis 87. A 3 year old child with weight defi-
B. Staphylococcal enteric infection ciency suffers from permanent moi-
C. Escherichiosis st cough. In history there are some
D. Acute shigellosis pneumonias with obstruction. On exami-
E. Rotaviral infection nation: distended chest, dullness on
84. Examination of a 22 year old man percussion over the lower parts of lungs.
suffering from polyarthralgia and hi- On auscultation: a great number of di-
gh fever revealed right-sided exudati- fferent rales. Level of sweat chloride is
ve pleuritis. X-ray picture showed a 80 millimol/l. What is the most probable
homogenous shadow below the IV rib on diagnosis?
the right. In the II segment there were A. Mucoviscidosis (cystic fibrosis)
single dense focal shadows. Mantoux B. Bronchial asthma
test with 2 TU resulted in formation C. Recurrent bronchitis
of a papula 16 mm large. Pleural liquid D. Bronchiectasis
has increased protein concentration, Ri- E. Pulmonary hypoplasia
valta’s reaction is positive, there was
also increased number of leukocytes with 88. A 14 year old girl complains of
prevailing lymphocytes. What is the most profuse bloody discharges from genital
probable etiology of pleuritis? tracts during 10 days after suppresion of
Krok 2 Medicine 2009 12

menses for 1,5 month. Similiar bleedings The first heart sound is weakened on
recur since 12 years on the background the apex, auscultation revealed systolic
of disordered menstrual cycle. On rectal souffle. What is the most probable aeti-
examination: no pathology of the internal ological factor that caused this pathologi-
genitalia. In blood: Нb - 70 g/l, RBC- cal process?
2, 3 · 1012 /l, Ht - 20. What is the most
probable diagnosis? A. β -haemolytic streptococcus
B. Staphylococcus
A. Juvenile bleeding, posthemorrhagic C. Pneumococcus
anemia D. Virus
B. Werlholf’s disease E. Fungi
C. Polycyst ovarian syndrome
D. Hormonoproductive ovary tumor 92. A 50 year old locksmith was di-
E. Incomplete spontaneous abortion agnosed with typhoid fever. The pati-
ent lives in a separate apartment with all
89. A 46 year old patient is to be prepared facilities. Apart of him there are also 2
to the operation on account of stomach adults in his family. What actions should
cancer. Preoperative preparation involves be taken about persons communicating
infusion therapy. It was injected up to 3 l with the patient?
of solutions into his right lunar vein. On
the next day he got tensive pain in the A. Bacteriological study
region of his right shoulder. Examinati- B. Antibiotic prophylaxis
on of interior brachial surface revealed C. Isolation
an oblong area of hyperemia, skin edema D. Dispensary observation
and painful cord. What complication is it? E. Vaccination

A. Acute thrombophlebitis 93. A 39 year old patient complained


B. Vein puncture and edema of about morning headache, appetite loss,
paravenous cellular tissue nausea, morning vomiting, periodic nasal
C. Necrosis of paravenous cellular tissue haemorrhages. The patient had acute
D. Acute lymphangitis glomerulonephritis at the age of 15.
E. Phlegmon of paravenous cellular tissue Examination revealed rise of arterial
pressure up to 220/130 mm Hg, skin
90. A 58 year old female patient complai- haemorrhages on his arms and legs,
ns about periodical headache, dizziness pallor of skin and mucous membranes.
and ear noise. She has been sufferi- What biochemical index has the greatest
ng from diabetes mellitus for 15 years. diagnostic importance in this case?
Objectively: heart sounds are rhythmic,
heart rate is 76/min, there is diastolic A. Blood creatinine
shock above aorta, AP is 180/110 mm B. Blood bilirubin
Hg. In urine: OD- 1,014. Daily loss of C. Blood sodium
protein with urine is 1,5 g. What drug D. Uric acid
should be chosen for treatment of arteri- E. Fibrinogen
al hypertension? 94. A 43 year old patient was admitted
A. Ihibitor of angiotensin converting to the infectious diseases hospital wi-
enzyme th high body temperature and intense
B. β -blocker headache. The iIlness has lasted for 2
C. Calcium channel antagonist days. Examination revealed a carbuncle
D. Thiazide diuretic on his forearm. The area around it was
E. α-blocker apparently edematic and slightly painful.
Regional lymphadenitis and hepatoli-
91. A 25 year old patient had pharyngi- enal syndrome were also present. It is
tis 2 weeks ago. Now he complains known from the anamnesis that the pati-
about body temperature rise up to 38o C , ent works at a cattle-breeding farm. What
general weakness, dyspnea during walki- disease should be suspected in the first
ng, swelling and shifting pain in the arti- place?
culations. Objectively: cyanosis of lips,
rhythmic pulse of poor volume - 100 bpm.
Left cardiac border deviates outwards
from the mediaclavicular line by 1 cm.
Krok 2 Medicine 2009 13

A. Anthrax haemorrhagic elements have necrosis in


B. Erysipelas the centre. What is the most probable di-
C. Erysipeloid sease?
D. Skin cancer
E. Eczema A. Meningococcemia
B. Rubella
95. Study of morbidity with temporary di- C. Influenza
sability among workers of a machine bui- D. Haemorrhagic vasculitis
lding plant revealed that average durati- E. Scarlet fever
on of a case was 20 days. What diseases
influenced upon the index value? 99. A 13 year old girl was admitted to
the cardiological department because of
A. Chronic pain in the muscles and joints. Exami-
B. Acute nation of her face revealed an edematic
C. Subacute erythema in form of butterfly in the regi-
D. Preexisting diseases on of nose bridge and cheeks. What is the
E. Hard to determine most probable diagnosis?
96. A 14 year old child suffers from A. Systemic lupus erythematosus
vegetovascular dystonia of pubertal peri- B. Rheumatism
od. He has got sympathoadrenal atack. C. Dermatomyositis
What medicine should be used for attack D. Rheumatoid arthritis
reduction? E. Periarteritis nodosa
A. Obsidan 100. A 15 year old girl suddenly got
B. No-shpa arthralgia, headache, nausea, vomiting;
C. Amysyl pain and muscle tension in the lumbar
D. Aminophylline area; body temperature rose up to 38 −
E. Corglicone 39o C . Pasternatsky’s symptom was disti-
nctly positive on the right. In the uri-
97. A 52 year old male patient complai- ne: bacteriuria, pyuria. What is the most
ns about attacks of asphyxia, pain in his probable diagnosis?
left side during respiration. These mani-
festations turned up all of a sudden. It A. Acute pyelonephritis
is known from his anamnesis that he B. Renal colic
had been treated for thrombophlebitis C. Acute glomerulonephritis
of the right leg for the last month. In D. Pararenal abscess
the admission ward the patient suddenly E. Cystitis
lost consciousness, there was a sudden
attack of asphyxia and pain in his left 101. A 19 year old patient was admi-
side. Objectively: heart rate - 102/min, tted to a hospital with acute destructive
respiratory rate - 28/min, AP- 90/70 appendicitis. He suffers from hemophilia
mm Hg. Auscultation revealed diastolic of B type. What antihemophilic medicati-
shock above the pulmonary artery, gallop ons should be included in pre- and post-
rhythm, small bubbling rales above the operative treatment plan?
lungs under the scapula on the right,
pleural friction rub. What examination A. Fresh frozen plasma
method will be the most informative for B. Cryoprecipitate
a diagnosis? C. Fresh frozen blood
D. Native plasma
A. Angiography of pulmonary vessels E. Dried plasma
B. Echocardioscopy
C. Study of external respiration function 102. A 59 year old female patient applied
D. ECG to a maternity welfare clinic and complai-
E. Coagulogram ned about bloody discharges from the
genital tracts. Postmenopause is 12
98. A 4 month old child fell seriously ill: years. Vaginal examination revealed that
body temperature rose up to 38, 5o C , the external genital organs had signs of age
child became inert and had a single vomi- involution, uterus cervix was not erosi-
ting. 10 hours later there appeared rash ve, small amount of bloody discharges
over the buttocks and lower limbs in form came from the cervical canal. Uterus was
of petechiae, spots and papules. Some of normal size, uterine appendages were
Krok 2 Medicine 2009 14

unpalpable. Fornices were deep and pai- enlargement of the left mammary gland,
nless. What method should be applied for pain on palpation. What pathology would
the diagnosis specification? you think about in this case?
A. Separated diagnosic curretage A. Lactational mastitis
B. Laparoscopy B. Lacteal cyst with suppuration
C. Puncture of abdominal cavity through C. Fibrous adenoma of the left mammary
posterior vaginal fornix gland
D. Extensive colposcopy D. Breast cancer
E. Culdoscopy E. Phlegmon of mammary gland
103. A 26 year old woman who delivered 107. A 32 year old patient suffering
a child 7 months ago has been suffering from chronic viral hepatitis complains
from nausea, morning vomiting, sleepi- about dull pain in the right subcostal
ness for the last 2 weeks. She suckles the area, nausea, dry mouth. Objecti-
child, menstruation is absent. She hasn’t vely: liver dimensions are 13-21-11
applied any contraceptives. What method cm (according to Kurlov), spleen is
should be applied in order to specify her by 2 cm enlarged, aspartate ami-
diagnosis? notransferase is 3,2 micromole/l·h, alani-
ne aminotransferase - 4,8 millimole/l·h.
A. Ultrasonic examination Serological study revealed HBeAg , high
B. Roentgenography of small pelvis concentration of DNA HBV . What drug
organs should be chosen for treatment of this
C. Palpation of mammary glands and patient?
pressing-out of colostrum
D. Bimanual vaginal examination A. α-interferon
E. Speculum examination B. Acyclovir
C. Remantadinum
104. A boy is 1 year old. Previously he D. Arabinoside monophosphate
had purulent otitis. After that he started E. Essentiale-forte
complaining about pain in the upper thi-
rd of his left thigh, body temperature 108. A 5 month old boy was born
rose up to 39o C . Objectively: the thigh is prematurely, he didn’t suffer from any
swollen in its upper third, inguinal fold is disease at the infant age and later on.
smoothed. Extremity is in half-bent posi- Examination at an outpatient’s hospi-
tion. Active and passive movements are tal revealed paleness of skin, sleepiness.
impossible because of acute pain. What is Blood count: Hb - 95 g/l, erythrocytes
the most probable diagnosis? - 3, 5 · 1012 /l, reticulocytes - 90 /00 ,
colour index - 0,7, osmotic stability of
A. Acute haematogenic osteomyelitis erythrocytes - 0,44-0,33%, serum iron
B. Acute coxitis - 4,9 micromole/l. What is the most
C. Intermuscular phlegmon probable cause of anemia?
D. Osteosarcoma
E. Brodie’s abscess A. Iron deficit
B. Hemogenesis immaturity
105. A 2 month old full-term child was C. Infectious process
born with weight 3500 g and was on the D. Erythrocyte hemolysis
mixed feeding. Current weight is 4900 g. E. B12 deficit
Evaluate the current weight of the child:
109. A 45 year old woman complains
A. Corresponding to the age about unbearable pain attacks in the left
B. 150 g less than necessary part of face lasting 1-2 minutes. Such
C. Hypotrophy of the I grade attacks are provoked by mastication.
D. Hypotrophy of the II grade These symptoms appeared 2 months ago
E. Paratrophy of the I grade after exposure to cold. Objectively:the
106. A woman consulted a doctor on pain is localized in the ending points of
the 14th day after labour about sudden trigeminus on the left. A touch near nose
pain, hyperemy and induration of the wing provokes another pain attack and
left mammary gland, body temperature tonic spasm of face muscles. What is the
rise up to 39o C , headache, indisposi- most probable diagnosis?
tion. Objectively: fissure of nipple,
Krok 2 Medicine 2009 15

A. Neuralgia of trigeminus intense headache in the region of occi-


B. Neuralgia of glossopharyngeal nerve put; there appeared recurrent vomiting.
C. Arthritis of mandibular joint These presentations has been lasting for
D. Facial migraine 5 hours. Objectively: Ps - 88 bpm, AP -
E. Maxillary sinusitis 205/100 mm Hg, painfulness of occipital
points, rigidity of occipital muscles are
110. A 38 year old man was delivered present. Kernig’s symptom is bilaterally
to the hospital in unconscious state. The positive. Subarachnoid haemorrhage is
symptoms of illness turned up a day also suspected. What diagnostic method
before: headache, nausea, vomiting, to - will be of the greatest importance for
38, 5o C , dizziness, delusion. For the last confirmation of provisional diagnosis?
4 days he had been complaining of pain
and hearing loss in the left ear. Objecti- A. Lumbar puncture
vely: sopor, rigidity of occipital muscles, B. Examination of eye fundus
bilateral Kernig’s symptom, general C. Ultrasonic dopplerography
hyperesthesia, purulent discharges from D. EEG
the left ear. What is the most probable E. Computer tomography
diagnosis?
114. Immediately after delivery a woman
A. Secondary purulent meningitis had haemorrhage, blood loss exceeded
B. Primary purulent meningitis postpartum haemorrhage rate and was
C. Tuberculous meningitis progressing. There were no symptoms of
D. Subarachnoidal haemorrhage placenta detachment. What tactics should
E. Parenchymatous subarachnoidal be chosen?
haemorrhage
A. Manual removal of placenta and
111. A 70 year old man is suffering from afterbirth
coronary heart disease. His mood is evi- B. Uterus tamponade
dently depressed, anxious. As a result C. Instrumental revision of uterine cavity
of continious sleeplessness he has got walls
fears, suicidal thoughts. He would sit for D. Removal of afterbirth by Crede’s
a long time in the same pose, answer method
after a pause, in a low, monotonous voi- E. Intravenous injection of
ce. His face has a look of suffering, pain, methylergometrine with glucose
fear. What is the main psychopathologic
syndrome? 115. A 33 year old patient was delivered
to the infectious diseases department on
A. Depressive syndrome the 7-th day of disease. He complained
B. Paranoid syndrome about great weakness, high temperature,
C. Asthenic syndrome pain in the lumbar area and leg muscles,
D. Phobic syndrome icteritiousness, dark colour of urine,
E. Obsessive syndrome headache. The acute disease started with
chill, body temperature rise up to 40o C ,
112. A 23 year old female patient headache, pain in the lumbar area and
complains about periodical chill and sural muscles. Icterus turned up on the
body temperature rise up to 40o C , sense 4th day, nasal and scleral haemorrhages
of heat taking turns with profuse sweati- came on the 5th day. Fever has lasted for
ng. The patient has had already 3 attacks 6 days. Diuresis - 200 ml. What is the most
that came once in two days and lasted 12 probable diagnosis?
hours. She has lived in Africa for the last
2 months. Liver and spleen are enlarged. A. Leptospirosis
In blood: erythrocytes - 2, 5 · 1012 /l. What B. Typhoid fever
is the most probable diagnosis? C. Virus A hepatitis
D. Sepsis
A. Malaria E. Yersiniosis
B. Spotted fever
C. Sepsis 116. A 72 year old male patient complai-
D. Haemolytic anaemia ns about itch in his left shin, especially
E. Leptospirosis around a trophic ulcer. Skin is reddened
and edematic, there are some oozing lesi-
113. A 39 year old patient sufferi- ons, single yellowish crusts. The focus of
ng from hypertension suddenly gotb affection is well-defined. What is the most
Krok 2 Medicine 2009 16

probable diagnosis? ld revealed diffuse thyroid enlargement


of the II degree. Heart auscultation
A. Microbial eczema revealed dullness of heart sounds, heart
B. Allergic dermatitis rate was 64/min. The child has frequent
C. Seborrheic eczema constipations, anemia. Concentration of
D. Cutaneous tuberculosis thyreoglobulin antibodies is increased.
E. Streptococcal impetigo What disease might have caused such
symptoms?
117. A 29 year old female patient
complains about periodical right-sided A. Autoimmune thyroiditis
headache that is usually provoked by B. Diffuse toxic goiter
strong smells or excitement. The patient’s C. Thyroid carcinoma
mother suffers from the same disease. D. Thyroid hyperplasia
Objectively: examination of internal E. Endemic goiter
organs revealed no pathology. During the
attack there are general hyperesthesia 121. A 13 year old patient is suffering
and nausea, at the end of the attack from an acute disease with the followi-
polyuria is observed. Palpation of the ng symptoms: thirst, polyuria, weakness.
right temporal artery during the attack Objectively: his general condition is sati-
revealed tension and painfulness of the sfactory, there is no smell of acetone.
artery. Complete blood count and urine Glucose concentration in blood on an
analysis reveale dno pecularities. What is empty stomach is 32 micromole/l, in urine
the most probable diagnosis? - 6%, acetone +. What treatment should
be administered?
A. Migraine
B. Neuritis of the facial nerve A. Short-acting insulin
C. Neuralgia of trigeminus B. Long-acting insulin
D. Meniere’s syndrome C. Biguanides
E. Epilepsy D. Sulfonylurea
E. Diet
118. A 52 year old woman complains
about face distortion. It turned up 2 days 122. Surgical department admitted a 37
ago after supercooling. Objectively: body year old patient with a big crushed wound
temperature is 38, 2oC . Face asymmetry is of his left thigh 4 hours after he got
present. Frontal folds are flattened. Left this trauma. What is the main provisi-
eye is wider than right one and doesn’t on for successful prevention of gaseous
close. Left nasolabial fold is flattened, gangrene?
mouth corner is lowered. Examinati-
on revealed no other pathology. Blood A. Removal of necrotic tissues and timely
count: leukocytes - 10 · 109 /l, ESR - 20 surgical processing of the wound
mm/h. What is the most probable di- B. Injection of specific serum 3 000 U
agnosis? C. Injection of specific serum 30 000 U
D. Infiltration of soft tissues around the
A. Facial neuritis wound with antibiotic solution
B. Trigeminus neuralgia E. Wound lavage with 6% solution of
C. Hemicrania (migraine) hydrogen peroxide
D. Ischemic stroke
E. Brain tumour 123. A 40 year old patient was bitten by a
stray dog for about an hour ago. The bite
119. A 10 year old boy suffers from can be seen on the patient’s left shin in
chronic viral hepatitis type B with maxi- form of a wound 4х2х0,5 cm large. What
mal activity. What laboratory test can kind of aid would be recommended in
give the most precise characteristic of this case?
cytolysis degree?
A. Wound lavage with soapsuds, retension
A. Transaminase test sutures
B. Weltman’s coagulation test B. Aseptic bandage
C. Takata-Ara test C. Salve bandage
D. Prothrombin test D. Blind suture
E. Test for whole protein E. Retension sutures

120. Examination of a 12 year old chi- 124. A patient has got acute macrofocal
Krok 2 Medicine 2009 17

myocardial infarction complicated by A. Sulfosalasine


cardiogenic shock. The latter is progresi- B. Motilium
ng under conditions of weak general C. Vikasolum
peripheric resistance and lowered cardi- D. Linex
ac output. What antihypotensive drug E. Kreon
should be injected to the patient in the
first place? 128. A 38 year old female patient
complains about body stiffness in
A. Dopamine the morning, especially in the arti-
B. Noradrenaline culations of her upper and lower li-
C. Adrenaline mbs, that disappears 30-60 minutes
D. Mesatonum later after active movements. She has
E. Prednisolone also arthritis of metacarpophalangeal
and proximal phalangeal articulations,
125. A patient applied to the subfebrile temperature. ESR- 45 mm/h.
traumatology cenre and complained Roentgenography revealed osteoporosis
about a trauma of the lower third of the and erosion of articular surface of small
volar forearm surface caused by cut on hand and foot articulations. What is the
a piece of galss. Objectively: flexion of most probable diagnosis?
the IV and V fingers is impaired, sensi-
tivity of the interior dorsal and palmar A. Rheumatoid arthritis
surface of hand as well as of the IV finger B. Psoriatic arthropathy
is reduced. What nerve is damaged? C. Osteoarthrosis deformans
D. Systemic lupus erythematosus
A. Ulnar E. Reactive polyarthritis
B. Radial
C. Median 129. A child was born with body wei-
D. Musculoskeletal ght 3250 g and body length 52 cm. At
E. Axillary the age of 1,5 month the actual wei-
ght is sufficient (4350 g), psychophysical
126. It was noticed that a 7 year old pupil development corresponds with the age.
had been innattentive several times duri- The child is breast-fed, occasionally there
ng the lessons. The teacher also noticed are regurgitations. What is the cause of
that the child had been smacking his lips regurgitations?
and had vacant look. There were no falls
and convulsions. During such short peri- A. Aerophagia
ods of absence the child didn’t react to B. Pylorostenosis
calling his name. His mother notic such C. Pylorospasm
phenomena before but didn’t pay much D. Acute gastroenteritis
attention to them thinking that the child E. Esophageal atresia
was deep in thought. What type of epi-
leptic attack (according to the standard 130. Three weeks after acute angina
classification) is it? the patient is still weak, inert, subfebri-
le, his retromaxillary lymph nodes are
A. Absentia enlarged. Tonsils are flabby, stick together
B. Generalized tonoclonic epilepsy with arches, there are purulent plugs in
C. Simple partial epilepsy lacunae. What is the most probable di-
D. Complex partial epilepsy agnosis?
E. Jacksonian partial epilepsy
A. Chronic tonsillitis
127. A 41 year old woman has suffered B. Chronic pharyngitis
from nonspecific ulcerative colitis for C. Acute lacunar tonsillitis
5 years. On rectoromanoscopy: evident D. Paratonsillitis
inflammatory process of lower intesti- E. Tonsillar tumour
nal parts, pseudopolyposive changes of
mucous membrane. In blood: WBC- 131. A 23 year old patient fell ill 3 weeks
9, 8 · 109 /l, RBC- 3, 0 · 1012 /l, ESR - ago when she noticed a very painful
52 mm/hour. What medication provides induration in her axillary crease. 4-5 days
pathogenetic treatment of this patient? later it burst and discharged a lot of
pus. After that some new infiltrations
appeared around the affected area. The
patient has never suffered from skin di-
Krok 2 Medicine 2009 18

seases before. What is the most probable A. Nascent submucous fibromatous node
diagnosis? B. Abortion in progress
C. Cervical carcinoma
A. Hydradenitis D. Cervical myoma
B. Furuncle E. Algodismenorrhea
C. Mycosis
D. Herpes zoster 135. A child is 1 day old. During delivery
E. Streptococcal impetigo there had been problems with extracti-
on of shoulders. Body weight is 4300,0.
132. A 28 year old woman has bursti- Right arm hangs down along the body,
ng pain in the lower abdomen during hand is pronated, movement in the arm is
menstruation; chocolate-like discharges absent. "Scarf"symptom is positive. What
from vagina. It is known from the is the most probable diagnosis?
anamnesis that the patient suffers from
chronic adnexitis. Bimanual examinati- A. Total right-sided obstetric paralysis
on revealed a tumour-like formation of B. Proximal right-sided obstetric paralysis
heterogenous consistency 7х7 cm large C. Distal right-sided obstetric paralysis
to the left from the uterus. The formati- D. Hemiparesis
on is restrictedly movable, painful when E. Tetraparesis
moved. What is the most probable di-
agnosis? 136. A 42 year old man applied to a
hospital 10 minutes after he got stung
A. Endometrioid cyst of the left ovary by a bee and complained about face
B. Follicular cyst of the left ovary edema and difficult respiration. Objecti-
C. Fibromatous node vely: Ps- 98 bpm, AP- 130/80 mm Hg.
D. Exacerbation of chronic adnexitis A doctor on duty injected him 1 ml of
E. Tumour of sigmoid colon 1% dimedrol solution intramuscularly
and recommended to apply to his local
133. As a result of prophylactic medical therapeutist on the next day. What tacti-
examination a 35 year old woman was di- cs of treatment should be chosen for this
agnosed with alimentary and constituti- patient?
ve obesity of the III degree. It is known
from her anamnesis that the patient A. Intravenous introduction of predni-
doesn’t observe rules of rational nutri- solone and hospitalization
tion: she often overeats, the last food B. Intravenous introduction of calcium
intake is usually 10-15 minutes before chloride
going to bed, prefers fattening and ri- C. The patient needs no further medical
ch in carbohydrates food. What is the aid
main alimentary risk factor of obesity D. Hospitalization for observation
development? E. Intravenous introduction of calcium
chloride and hospitalization
A. Energetic unprofitableness of nutrition
B. Excess of carbohydrates 137. A 44 year old man has been worki-
C. Excess of fats ng in coke industry for 16 years. Dust
D. Lack of cellulose concentration at his workplace is 5-10
E. Violation of dietary pattern times more than maximum permissi-
ble concentration. Roentgenography of
134. A 40 year old female patient has lungs revealed changes that are typical
been observing excessive menstruati- for pneumoconiosis. What is the most
on accompanied by spasmodic pain probable type of pneumoconiosis in this
in the lower abdomen for a year. Bi- case?
manual examination performed during
menstruation revealed a dense formati- A. Anthracosis
on up to 5 cm in diameter in the cervi- B. Anthracosilicosis
cal canal. Uterus is enlarged up to 5-6 C. Silicatosis
weeks of pregnancy, movable, painful, of D. Asbestosis
normal consistency. Appendages are not E. Siderosis
palpable. Bloody discharges are profuse.
What is the most probable diagnosis? 138. Study of morbidity rate in a city
N revealed that population of different
administrative districts differed in age
structure. What statistic method allows
Krok 2 Medicine 2009 19

to eliminate influence of this factor upon the most probable diagnosis?


morbidity indices?
A. Poisoning with carbon monooxide
A. Standardization B. Poisoning with hydrocyanic acid
B. Wilcoxon’s t-criterion C. Poisoning with anilin colouring agents
C. Correlative regressive analysis D. Poisoning with methane
D. Analysis of dynamic series E. Poisoning with benzine
E. Calculation of average values
143. The amount of ultraviolet radiati-
139. An outbreak of food poisoning was on dose was measured in minutes. What
recorded in an urban settlement. The device was applied for measurement of
illness was diagnosed as botulism on the the biodose?
grounds of clinical presentations. What
foodstuffs should be chosen for analysis A. Gorbachev’s biodosimeter
in the first place in order to confirm the B. UV-meter
diagnosis? C. Actinometer
D. Radiometer
A. Tinned food E. Catathermometer
B. Potatoes
C. Pasteurized milk 144. A 63 year old patient complai-
D. Boiled meat ned about pain in the lumbar area. He
E. Cabbage underwent a course of physiological
treatment on account of radiculitis but
140. A 72 year old female patient has this led to no improvement of his conditi-
been treated for urolithiasis in the on. R-graphy of spinal column and pelvic
urological department. After atropine bones revealed osteoporosis and serious
injection she got acute pain in her left eye bone defects. Blood analysis revealed
and abrupt vision impairment. Objecti- moderate normochromic anaemia, uri-
vely: visual acuity of the left eye is 0,01, ne analysis revealed proteinuria. Whole
the eye is dense but painful on palpati- blood protein made up 10,7 g/l. What di-
on, cornea is opaque, there is cyanotic sease should be suspected?
induration of eyeball vessels. What is the
most probable diagnosis? A. Myelomatosis
B. Urolithiasis
A. Acute attack of primary glaucoma of C. Acute radiculitis
the left eye D. Metastases in bones
B. Acute iridocyclitis of the left eye E. Systemic osteoporosis
C. Secondary glaucoma of the left eye
D. Acute keratitis of the left eye 145. A sergeant was injured by a shell
E. Degeneration of the left eye cornea splinter in the left subcostal area. He
was bandaged with a first-aid pack on
141. Administration of a plant producing a battlefield. The patient was delivered
red lead paint intends to form a group of to the regiment medical aid station. He
medical specialists for periodical medi- complains about dizziness, weakness, thi-
cal examinations. What specialist must rst, abdominal pain. General condition is
be obligatory included into this group? grave, the patient is pale. Ps is 120 bpm.
Abdomen is soft, painful on palpation.
A. Neuropathologist The bandage is well fixed but a little bit
B. Gynaecologist soaked with blood. The patient should be
C. Psychiatrist evacuated to the medical battalion with
D. Dermatologist the following transport and in the followi-
E. Otolaryngologyst ng turn:
142. A 42 year old man works in a A. With medical vehicle in the first turn
boiler room. He complains about gi- B. With a passing car in the first turn
rdle headache and recurring vomiting. C. With medical vehicle in the second turn
There was also short-term consciousness D. With a passing car in the second turn
loss. Objectively: increase of tendon E. With a passing car in the third turn
reflexes, spontaneous myofibrillations.
AP is 150/80 mm Hg, Ps- 104 bpm. Vi- 146. On the 5th day after labor body
sible mucous membranes and cutaneous temperature of a parturient suddenly
surfaces have crimson colouring. What is rose up to 38, 7oC . She complains about
Krok 2 Medicine 2009 20

weakness, headache, abdominal pain, A. Noncarrying of pregnancy


irritability. Objectively: AP- 120/70 mm B. Intestinal obstruction
Hg, Ps- 92 bpm, to - 38, 7oC . Bimanual C. Fetus hypotrophy
examination revealed that the uterus was D. Premature placenta detachment
enlarged up to 12 weeks of pregnancy, E. Late gestosis
it was dense, slightly painful on palpati-
on. Cervical canal lets in 2 transverse fi- 150. It is planned to make complete
ngers, discharges are moderate, turbid, isolation boxes in the infectious
with foul smell. In blood: skeocytosis, department in order to prevent
lymphopenia, ESR- 30 mm/h. What is the nosocomial airborne infections. The
most probable diagnosis? boxes consist of a tambour, a ward and
a lock chamber. What structure should be
A. Endometritis also included in a complete isolation box?
B. Parametritis
C. Pelviperitonitis A. Bathroom unit
D. Metrophlebitis B. Manipulation room
E. Lochiometra C. Doctor’s consulting room
D. Patient’s examination room
147. A 19 year old boy was admitted E. Nursing room
to a hospital with closed abdominal
trauma. In course of operation multi- 151. A 3 year old boy has petechi-
ple ruptures of spleen and small intesti- al eruption. Examination revealed no
ne were revealed. AP is falling rapidly, other pathological changes. Thrombocyte
it is necessary to perform hemotransfusi- number is 20 · 109 g/l; haemoglobin and
on. Who can specify the patient’s blood leukocyte concentration is normal. What
group and rhesus compatibility? is the most probable diagnosis?
A. A doctor of any speciality A. Immune thrombocytopenic purpura
B. A laboratory physician B. Schonlein-Henoch disease
C. A surgeon C. Disseminated intravascular
D. A traumatologist coagulopathy
E. An anaesthesilogist D. Acute lymphoblastic leukemia
E. Systemic lupus erythematosus
148. A 36 year old man was delivered
to the surgical department an hour after 152. Indices that characterize populati-
a road accident. His condition is getti- on health include demographic indices.
ng worse: respiratory insufficiency is What environment is used for calculation
progressing, there are cardiac abnormali- of these indices?
ties. Clinical and roentgenological
investigations revealed mediastinal di- A. Population number
splacement. What process has caused this B. Employment number
complication? C. Number of hospitalized people
D. Number of patients
A. Valvular pneumothorax E. Number of population being liable to
B. Open pneumothorax preventive examination
C. Closed pneumothorax
D. Subcutaneous emphysema 153. An infant is full-term. Delivery was
E. Mediastinitis pathological, with breech presentation.
Examination of the infant revealed li-
149. An 18 year old primigravi- mited abduction of the right leg to 50o ,
da in her 27-28 week of pregnancy positive "clicking"symptom on the right,
underwent an operation on account of asymmetric inguinal folds. What is the
acute phlegmonous appendicitis. In the most probable diagnosis?
postoperative period it is necessary to
take measures for prevention of the A. Inborn dislocation of the right hip
following pegnancy complication: B. Inborn dislocation of both hips
C. Varus deformity of both femoral necks
D. Fracture of both femoral necks
E. Right hip dysplasia
154. A 30 year old man complains of
intense pain, reddening of skin, edema
Krok 2 Medicine 2009 21

in the ankle-joint area, fever up to 39o C . A. Injection of narcotic analgetics and


There was an acute onset of the illness. In powdered blood substitutes
the past there were similar attacks lasting B. Primary surgical pocessing
5-6 days without residual changes in the C. Administration of detoxicating blood
joint. The skin over the joint is hyperemic substitutes
and ill-defined, without infiltrative bank D. Necrotomy of burn surface,
on the periphery. What is the most likely haemotransfusion
diagnosis? E. Antibacterial and detoxicating therapy
A. Gout 158. A 54 year old female patient was
B. Infectious arthritis admitted to the hospital with evident
C. Rheumatoid arthritis acrocyanosis, swollen cervical veins,
D. Erysipelatous inflammation enlarged liver, ascites. Cardiac borders
E. Osteoarthritis are dilated. Heart sounds cannot be
auscultated, apical beat is undetectable.
155. The results of 5 year monitoring AP is 100/50 mm Hg. X-ray picture of
allowed to estimate the level of envi- chest shows enlarged heart shadow in
ronmental influence upon health indi- form of a trapezium. What pathology mi-
ces of popultaion. What statistic method ght have caused these symptoms?
should be chosen?
A. Cardiac tamponade
A. Calculation of correlation coefficient B. Exudative pleuritis
B. Calculation of conformity coefficient C. Complex heart defect
C. Calculation of coefficient of difference D. Acute cardiac insufficiency
validity E. Hiatal hernia
D. Calculation of regression coefficient
E. Calculation of dynamic indices 159. 15 minutes after the second vacci-
nation with diphteria and tetanus toxoi-
156. A 37 year old miner has lifted si- ds and pertussis vaccine a 4 month old
gnificant loads and afterwards felt pain boy manifested symptoms of Quincke’s
in the lumbar area irradiating to his edema. What medication should be appli-
left leg. He walks slowly and carefully. ed for emergency aid?
Lumbar lordosis is flattened. There is
also left-sided scoliosis and tension of A. Prednisolone
paravertebral muscles. Neri’s and Dejeri- B. Heparin
ne’s symptoms are positive, there is C. Adrenalin
Lasegue’s sign on the left from the angle D. Furosemide
of 350. What method will help to specify E. Seduxen
the diagnosis?
160. A 16 year old female patient
A. CT of lumbosacral part of vertebral underwent an operation on account of
column diffuse toxic goiter of the III-IV degree
B. Lumbal puncture 12 years ago. Now she has recurrence of
C. Renal sonography thyrotoxicosis. The patient was offered
D. Descending myelography operative intervention, but it is necessary
E. Electromyography first to localize the functioning gland ti-
ssue. What method should be applied for
157. A 47 year old male patient got a this purpose?
flame burn of trunk and upper extremiti-
es and was delivered to the hospital. The A. Gland scanning
patient is in grave condition, confused B. USI
mental state, with fever. AP- 80/50 mm C. Puncture aspiration biopsy
Hg, Ps- 118 bpm. It was locally stated that D. Roentgenography of neck
the patient got III B degree burns with E. Roentgenography of esophagus
total area of 20%. What medical actions
should be taken? 161. Examination of a 26 year old female
patient revealed a node in the right lobe
of thyroid gland. The node appeared
no earlier than 3 months ago. The pati-
ent associates this node with stress. She
doesn’t complain either about pain or
enlargement of the node. Ultrasonic
Krok 2 Medicine 2009 22

scanning revealed a 2x2,5 cm large node A. Delirious


in the inferior part of the right lobe of B. Oneiric
thyroid gland. What treatment should be C. Amentive
administered? D. Twilight state
E. Asthenic confusion
A. Surgical intervention
B. Conservative therapy 165. On the third day of life an infant’s
C. Dynamic observation skin got icteric colouring. The child was
D. No need for treatment born with body weight of 3,200 kg,
E. - body length of 52 cm. The child is acti-
ve. There is puerile respiration above
162. During examination a patient is the lungs. Respiratory rate is 36/min,
unconscious, his skin is dry and hot, heart sounds are rhythmic, heart rate is
face hyperemia is present. The patient 130/min. Abdomen is soft, liver comes
has Kussmaul’s respiration, there is also out from the edge of costal arch by 2 cm,
smell of acetone in the air. Symptoms of spleen is not palpable. Feces are in form
peritoneum irritation are positive. Blood of meconium. What is the most probable
sugar is at the rate of 33 millimole/l. What diagnosis?
emergency actions should be taken?
A. Physiologic jaundice
A. Intravenous infusion of short-acting B. Hemolytic disease of newborn
insulin C. Neonatal sepsis
B. Intravenous infusion of glucose along D. Minkowsky-Shauffard disease
with insulin E. Biliary tracts atresia
C. Introduction of long-acting insulin
D. Intravenous infusion of 166. A pediatrician talked to a mother of
neohaemodesum along with glutamic a 7 month old breast-fed boy and found
acid out that the child was fed 7 times a day.
E. Intravenous infusion of sodium chlori- How many times should the child of such
de saline age be fed?
163. A patient complains about evaginati- A. 5 times
ons in the region of anus that appear duri- B. 3 times
ng defecation and need to be replaced. C. 4 times
Examination with anoscope revealed 1x1 D. 6 times
cm large evaginations of mucosa above E. 7 times
the pectineal line. What is the most
probable diagnosis? 167. In order to study impact of microcli-
mate upon the human organism it is
A. Internal hemorrhoids necessary to make systematic observati-
B. Acute paraproctitis on of air temperature over 3 days. Choose
C. External hemorrhoids a device that will allow to make the most
D. Anal fissure precise temperature records:
E. -
A. Thermograph
164. A 34 year old patient was delivered B. Alcohol thermometer
to the hospital because of follicular tonsi- C. Mercury thermometer
llitis charactirized by high temperature. D. August’s psychrometer
The patient has been abusing alcohol E. Assmann psychrometer
for 12 years. In the evening on the day
of hospitalization he became anxious, 168. A prematurely born girl is now
couldn’t stay in bed, left his ward several 8 months old. She has dyspnea,
times and applied to the staff on duty tachycardia, hepatosplenomegaly, physi-
with different complaints. He reported cal developmental lag, limb cyanosis.
about seeing alot of spiders and flies in There is also parasternal cardiac hump,
his ward as well as abou hearing threats auscultation revealed systolodiastolic
from the corridor. He was exasperated by murmur in the II intercostal space on
the fact that other patients didn’t hear the left. AP is 90/0 mm Hg. What disease
them. He lost also spatial orientation. should be suspected?
What psychopathological syndrome is it?
Krok 2 Medicine 2009 23

A. Patent ductus arteriosus is the most probable diagnosis?


B. Coarctation of aorta
C. Stenosis of aortal valve A. Sheehan’s syndrome
D. Stenosis of pulmonary artery B. Stein-Leventhal syndrome
E. Nonclosure of interventricular septum C. Shereshevsky-Turner’s syndrome
D. Homological blood syndrome
169. A 72 year old patient complains E. Vegetovascular dystonia
about pain and bleeding during defecati-
on. Digital rectal investigation revealed 173. A 35 year old female patient sufferi-
a tumour of anal canal. After verificati- ng from cholelithiasis has broken her di-
on of the diagnosis the patient was di- et, and this caused an acute pain attack
agnosed with squamous cell carcinoma. in the right subcostal are. The pain eased
The secondary (metastatic) tumour will off on the third day, but the patient got
be most probably found in: progressing jaundice. What non-invasive
diagnostic method should be applied?
A. Lungs
B. Liver A. Endoscopic retrograde cholangi-
C. Pelvic bones opancreatography
D. Mediastinum B. Infusive cholecystocholangiography
E. Brain C. Test for bilirubin
D. Duodenal probing
170. A 52 year old patient complains E. Survey radiography of abdominal
about headache, weakness of his upper organs
left extremity. Neurological symptoms
become more intense during physical 174. A 68 year old patient complains
stress of the left extremity. Pulsation on about acute pain in his right foot, toe
the arteries of the left extremity is sharply edema and darkening of skin of the IV
dampened but it remains unchanged on toe. He has been suffering from diabetes
the carotid arteries. What is the most mellitus for 15 years, doesn’t receive
probable diagnosis? regular treatment. What complication of
diabetes mellitus is it?
A. Occlusion of the left subclavicular
artery, steal syndrome A. Gangrene of the IV toe on the right
B. Thoracal outlet syndrome foot
C. Raynaud’s syndrome B. Panaritium
D. Takayasu’s disease C. Haematoma
E. Occlusion of brachiocephalic trunk D. Erysipelas
E. Fracture of the IV toe on the right foot
171. A 25 year old woman applied to a
maternity welfare clinic and complained 175. A plot of land with total area of 2,0
about being unable to conceive within 3 hectare was intended for building of a
years of regular sexual life. Examinati- hospital. The maximal capacity of the
on revealed weight gain, male pattern of hospital will be:
hair distribution on the pubis, excessi-
ve pilosis of thighs. Ovaries were dense A. 100 beds
and enlarged, basal temperature was B. 200 beds
monophase. What is the most probable C. 400 beds
diagnosis? D. 800 beds
E. Over 1000 beds
A. Sclerocystosis of ovaries
B. Tubo-ovaritis 176. A 26 year old woman complai-
C. Adrenogenital syndrome ns about edemata, swelling and pai-
D. Premenstrual syndrome nfulness of mammary glands, headache,
E. Gonadal dysgenesis tearfulness, irritability. These signs turn
up 5 days before menstruation and di-
172. A woman consulted a therapeutist sappear after its start. What clinical
about fatigability, significant weight loss, syndrome is it?
weakness, loss of appetite. She has had
amenorrhea for 8 months. A year ago
she born a full-term child. Haemorrhage
during labour made up 2 l. She got blood
and blood substitute transfusions. What
Krok 2 Medicine 2009 24

A. Premenstrual syndrome A. Muscle biopsy


B. Postcastration syndrome B. ECG
C. Adrenogenital syndrome C. Echocardiogram
D. Climacteric syndrome D. Electromyography
E. Stein-Leventhal syndrome E. Determination of cortisol concentrati-
on in blood and urine
177. A 27 year old patient suffers from
haemophilia. He was admitted to the 180. A 33 year old patient has acute blood
hospital with melena and skin pallor. loss (erythrocytes - 2, 2·1012/l, Hb- 55 g/l),
Objectively: Ps- 110 bpm, AP- 100/60 mm blood group is A(II)Rh+ . Accidentally
Hg. In blood: Hb- 80 g/l, erythrocytes - the patient got transfusion of donor
2, 8 · 1012 /l. What medication should be packed red blood cells of AB(IV )Rh+
administered in the first place? group. An hour later the patient became
anxious, got abdominal and lumbar pain.
A. Cryoprecipitate Ps- 134 bpm, AP- 100/65 mm Hg, body
B. Stored blood temperature - 38, 6o C . After catheteri-
C. Packed red blood cells zation of urinary bladder 12 ml/h of
D. Dicinone dark-brown urine were obtained. What
E. Epsilon-aminocapronic acid complication is it?
178. A 38 year old patient complains A. Acute renal insufficiency
about inertness, subfebrile temperature, B. Cardial shock
enlargement of lymph nodes, nasal C. Allergic reaction to the donor red
haemorrhages, bone pain. Objecti- blood cells
vely: the patient’s skin and mucous D. Citrate intoxication
membranes are pale, palpation revealed E. Toxic infectious shock
enlarged painless lymph nodes;
sternalgia; liver was enlarged by 2 cm, 181. A parturient woman is 27 year old,
spleen - by 5 cm, painless. In blood: it was her second labour, delivery was
erythrocytes - 2, 7 · 1012 /l, Hb- 84 g/l, at term, normal course. On the 3rd day
leukocytes - 58 · 109 /l, eosinophils - of postpartum period body temperature
1%, stab neutrophils - 2%, segmented is 36, 8oC , Ps - 72/min, AP - 120/80 mm
neutrophils - 12%, lymphocytes - 83%, Hg. Mammary glands are moderately
lymphoblasts - 2%, smudge cells; ESR- swollen, nipples are clean. Abdomen is
57 mm/h. What is the most probable di- soft and painless. Fundus of uterus is
agnosis? 3 fingers below the umbilicus. Lochia
are bloody, moderate. What is the most
A. Chronic lymphatic leukemia probable diagnosis?
B. Chronic myeloleukemia
C. Acute lymphatic leukemia A. Physiological course of postpartum
D. Acute myeloleukemia period
E. Lymphogranulomatosis B. Subinvolution of uterus
C. Postpartum metroendometritis
179. A 36 year old female patient D. Remnants of placental tissue after
complains about general weakness, labour
edemata of her face and hands, rapid E. Lactostasis
fatigability during walking, difficult di-
glutition, cardiac irregularities. These 182. A patient suffering from
symptoms turned up 11 days after holi- gastroesophageal reflux has taken from
day at the seaside. Objectively: face time to time a certain drug that "reduces
erythema, edema of shin muscles. Heart acidity"for 5 years. This drug was
sounds are muffled, AP is 100/70 mm recommended by a pharmaceutist. The
Hg. In blood: ASAT activity is 0,95 following side effects are observed:
millimole/h·l, ALAT - 1,3 micromole/h·l, osteoporosis, muscle weakness, indi-
aldolase - 9,2 IU/l, creatine phosphoki- sposition. What drug has such following
nase - 2,5 micromole Р/g·l. What method effects?
of study would be the most specific?
Krok 2 Medicine 2009 25

A. Aluminium-bearing antacid got married a month ago. Objectively:


B. Inhibitor of proton pump general state is satisfactory. Lung exami-
C. 2 -blocker nation revealed vesicular respiration.
D. Metoclopramide Heart sounds are rhythmic, heart rate is
E. Gastrozepin 78/min, AP- 128/68 mm Hg. Abdomen is
soft, painful in the suprapubic area. Uri-
183. A 34 year old patient complai- ne contains 12-18 erythrocytes and 12-15
ns of profuse sweating at night, skin bacteria within eyeshot. What is the most
itching, weight loss (9 kg within the probable diagnosis?
last 3 months). Examination revealed
malnutrition, skin pallor. Palpation of A. Infection of inferior urinary tracts -
neck and inguinal areas revealed dense cystitis
elastic lymph nodes for about 1 cm in di- B. Urolithiasis
ameter, nonmobile, non-adhering to skin. C. Infection of superior urinary tracts -
What is the most probable diagnosis? pyelonephritis
D. Gonorrhoea
A. Lymphogranulomatosis E. Primary syphilis
B. Chronic lymphadenitis
C. Lymphosarcoma 187. Examination of placenta revealed
D. Burkitt’s lymphoma a defect. An obstetrician performed
E. Cancer metastases manual investigation of uterine cavi-
ty, uterine massage. Prophylaxis of
184. A department chief of an in-patient endometritis in the postpartum period
hospital is going to examine resident should involve following actions:
doctors as to observation of medical-
technological standards of patient servi- A. Antibacterial therapy
ce. What documentation should be B. Instrumental revision of uterine cavity
checked for this purpose? C. Haemostatic therapy
D. Contracting agents
A. Health cards of in-patients E. Intrauterine instillation of dioxine
B. Statistic cards of discharged patients
C. Treatment sheets 188. A 9 year old boy had acute respi-
D. Registry of operative interventions ratory viral infection. After it there
E. Annual report of a patient care insti- appeared polydipsia, polyuria, weakness,
tution nausea. Examination revealed the
following symptoms: mental confusion,
185. A woman is 34 years old, it is her dry skin, soft eyeballs, Kussmaul’s respi-
tenth labor at full term. It is known ration, acetone smell from the mouth,
from the anamnesis that the labor started muffled heart sounds, soft and painless
11 hours ago, labor was active, painful abdomen. Blood sugar was 19 millimole/l.
contractions started after discharge of What acute condition is it?
waters and became continuous. Suddenly
the parturient got knife-like pain in A. Ketoacidotic coma
the lower abdomen and labor activity B. Hyperosmolar coma
stopped. Examination revealed positive C. Cerebral coma
symptoms of peritoneum irritation, ill- D. Hepatic coma
defined uterus outlines. Fetus was easi- E. Acute renal insufficiency
ly palpable, movable. Fetal heartbeats
wasn’t auscultable. What is the most 189. A patient consulted a venereologi-
probable diagnosis? st about painful urination, reddening of
the external opening of urethra, profuse
A. Rupture of uterus purulent discharges from the urethra. He
B. Uterine inertia considers himself to be ill for 3 days. He
C. Discoordinated labor activity also associates the disease with a casual
D. Risk of uterus rupture sexual contact that took place for about a
E. II labor period week ago. If provisional diagnosis "acute
gonorrheal urethritis"will be confirmed,
186. A 22 year old female patient then bacteriological study of urethral di-
complains about frequent and painful uri- scharges will reveal:
nation, urge to urinate at night, enuresis,
pain in the suprapubic and lumbar area.
Her urine often has beer colouring. She
Krok 2 Medicine 2009 26

A. Gram-negative diplococci ting. Objectively: the child’s condition is


B. Gram-positive diplococci grave. Abdomen is soft, palpation reveals
C. Spirochaete a tumour-like formation in the right ili-
D. Proteus vulgaris ac area. After rectal examination the
E. Mycoplasma doctor’s finger was stained with blood.
What is the most probable diagnosis?
190. A 30 year old patient complai-
ns about inability to become pregnant A. Ileocecal invagination
over 3 years of married life. The pati- B. Gastrointestinal haemorrhage
ent is of supernutrition type, she has hair C. Wilm’s tumour
along the median abdominal line, on the D. Helminthic invasion
internal thigh surface and in the peripapi- E. Pylorostenosis
llary area. Menses started at the age of 16,
they are infrequent and non-profuse. US 194. Estimation of physical development
revealed that the uterus was of normal of a child involved dynamometry and
size, ovaries were 4х5х5 cm large and had estimation of body weight and length,
a lot of cystic inclusions. What is the most annual gain in body length, chest ci-
probable diagnosis? rcumference, number of permanent
teeth, secondary sexual characters, lung
A. Polycystic ovaries vital capacity. Which of the mentioned
B. Ovarian cystoma indices relates to the physiometric ones?
C. Chronic oophoritis
D. Menstrual irregularity A. Lung vital capacity, dynamometry
E. Bilateral ovarian tumours B. Body length and weight, chest ci-
rcumference
191. On the second day after preventi- C. Secondary sexual characters
ve vaccination a 2 year old boy got D. Number of permanent teeth
abdominal pain without clear localizati- E. Annual gain in body length
on, body temperature rose up to 38o C .
On the third day the child got red papular 195. During preventive examination a 16
haemorrhagic eruption on the extensor year old patient presented no problems.
surfaces of limbs and around the joints. Objectively: the patient has signs of
Knee joints were edematic and slightly malnutrition, he is asthenic, AP is 110/70
painful. Examination of other organs mm Hg, Ps is 80 bpm, cardiac border
and systems revealed no pathological is normal, auscultation above the cardi-
changes. What is the most probable di- ac apex reveals three sounds, cardi-
agnosis? ac murmur is absent. ECG shows no
pathological changes, phonocardiogram
A. Haemorrhagic vesiculitis shows that the third sound comes 0,15
B. Thrombocytopenic purpura s after the second one above the apex.
C. Meningococcemia How are these changes called?
D. Urticaria
E. DIC syndrome A. III physiological sound
B. Fout-ta-ta-rou (reduplication of the
192. On the 6th day of life a child got 2nd sound)
multiple vesicles filled with seropurulent C. Protodiastolic gallop rhythm
fluid in the region of occiput, neck and D. Atrial gallop rhythm
buttocks. General condition of the chi- E. IV physiological sound
ld is normal. What disease should be
suspected? 196. Estimation of community health
level involved analysis of a report on
A. Vesiculopustulosis diseases registered among the populati-
B. Impetigo neonatorum on of district under charge (reporting
C. Miliaria form 12). What index is calculated on the
D. Impetigo grounds of this report?
E. Epidermolysis bullosa
193. A 4 month old child was admitted
to a surgical department 8 hours after
the first attack of anxiety. The attacks
happen every 10 minutes and last for 2-3
minutes, there was also one-time vomi-
Krok 2 Medicine 2009 27

A. Common sickness rate A. Cortisol


B. Index of pathological affection B. Corticotropin (ACTH)
C. Index of morbidity with temporary C. Adrenaline
disability D. Noradrenaline
D. Index of hospitalized morbidity E. Adrostendion
E. Index of basic non-epidemic morbidity
199. A 67 year old female patient
197. A 37 year old male patient was complains about edemata of face and
admitted to the resuscitation department legs, pain in the lumbar area that is getti-
because of attacks of tonoclonic spasms ng worse at moving; great weakness,
repeating every half an hour. Between sometimes nasal haemorrhages, rise of
the attacks the patient remains unconsci- body temperature up to 38, 4o C . Objecti-
ous. AP is 120/90 mm Hg, Ps- 100 bpm. vely: painfulness of vertebral column
A day before the patient was at weddi- and ribs on palpation. Laboratorial study
ng and consumed alcohol. 5 years ago he revealed daily proteinuria of 4,2 g, ESR-
had a closed craniocerebral trauma and 52 mm/h. What changes of laboratory
brain contusion that later caused single indices are to be expected?
convulsive attacks accompanied by loss
of consciousness, but the patient didn’t A. Whole protein of blood serum - 101 g/l
undergo antiepileptic treatment. What B. Leukocytes - 15,3 g/l
drug should be injected for emergency C. Haemoglobin - 165 g/l
aid? D. Albumins - 65%
E. γ -globulins - 14%
A. Diazepam
B. Magnesium sulfate 200. A female patient consulted a doctor
C. Sodium oxybutyrate about gain in weight, chill, edemata,
D. Aminazine dry skin, sleepiness, problems with
E. Sodium thiopental concentration. Objectively: the patient’s
height is 165 cm, weight is 90 kg, gynoid
198. A 43 year old female patient was body proportions, to - 35, 8oC , ESR-
delivered to the hospital in grave condi- 58/min, AP- 105/60 mm Hg. Heart sounds
tion. She suffers from Addison’s di- are weakened, bradycardia is present.
sease. The patient had been regularly Other internal organs have no changes.
taking prednisolone but a week before Thyroid gland is not palpable. Mammary
she stopped taking this drug. Objecti- glands ooze milk droplets. Hormonal
vely: sopor, skin and visible mucous study revealed rise of TSH and prolactin
membranes are pigmented, skin and concentration, reduction of T4 . What
muscle turgor is lowered. Heart sounds factor caused obesity?
are muffled, rapid. AP- 60/40 mm Hg,
heart rate - 96/min. In blood: Na- A. Primary hypothyroidism
120 millimole/l, K- 5,8 micromole/l. B. Secondary hypothyroidism
Development of this complication is pri- C. Prolactinoma
marily caused by the deficit of the followi- D. Hypopituitarism
ng hormone: E. Adiposogenital dystrophy
Krok 2 Medicine 2010 1

1. A 40-year-old patient complains A. To continue the worker in office with


of colic pains in the lower abdomen a warning of dismissal in case of repeated
and profuse bloody discharges from the violation of labor discipline
genital tracts. Over the last 2 years she B. To discharge the worker, i.e. to satisfy
has been having menses for 15-16 days, demands of the collective
profuse, with clots, painful. In anamnesis C. To issue the sick list
there are 2 medical abortions. On bi- D. To shift the solution of this problem on
manual investigation: in the canal of the other officials or public organizations
uterine cervix some fibromatous nodes E. -
are palpable, they are 3 cm in diameter,
on the thin crus. Discharges are bloody, 5. 25 unorganized children in the age 2-3
moderate. Choose the correct treatment year will be observed on a pediatric distri-
tactics: ct it in the current year. What scheduled
number of initial visitations will make to
A. Operation: untwisting of the nodes this group of children?
B. Hormonal hemostasis
C. Step-by-step vitamin therapy A. 50
D. Supravaginal ablation of the uterus B. 20
without appendages C. 40
E. Hysterectomy without appendages D. 100
E. 200
2. 13 months after the first labor a 24-year-
old patient complained of amenorrhea. 6. A patient with high temperature came
Pregnancy ended in Caesarian secti- to a first-aid post in the evening. The fact
on because of premature detachment of temporary disability was established.
of normally positioned placenta which Indicate the order of examination in this
resulted in blood loss at the rate of 2000 case:
ml owing to disturbance of blood clotting. A. The night duty doctor should issue
Choose the most suitable investigation: a medical certificate, which will be
A. Estimation of gonadotropin rate subsequently used for issuing a sick list
B. USI of small pelvis organs from the date of the previous day
C. Progesteron assay B. The sick list for 1 day should be issued
D. Computer tomography of head C. The sick list for up to 3 days should be
E. Estimation of testosteron rate in blood issued
serum D. The sick list for 3 days should be issued
E. Any document shouldn’t be issued
3. A 20-year-old patient was delivered to
the hospital in summer from the street wi- 7. An 8-year-old boy fell ill acutely: he
th haemorrage from the brachial artery. presents with fever, weakness, headache,
First medical aid involved application of a abdominal pain, recurrent vomiting, then
tourniquet for provisional arrest of bleedi- diarrhea and tenesmus. Stools occur 12
ng. What is the maximal exposure of the times daily, are scanty, contain a lot of
tourniquet? mucus, pus, streaks of blood. His sigmoid
gut is tender and hardened. What is your
A. 120 minutes diagnosis?
B. 15 minutes
C. 30 minutes A. Dysentery
D. 60 minutes B. Salmonellosis
E. 180 minutes C. Cholera
D. Staphylococcal gastroenteritis
4. Head of a department and a trade- E. Escherichiosis
union group have appealed to the head
of a hospital about dismissal of the seni- 8. An infant was born with body mass 3 kg
or nurse who has 17 year record of servi- and body length 50 cm. Now he is 3 years
ce. The facts of charge were confirmed old. His brother is 7 years old, suffers from
and recognized by the nurse herself. This rheumatic fever. Mother asked the doctor
nurse lives with a daughter (who is di- for a cardiac check up of the 3-year-old
son. Where is the left relative heart border
vorced and unemployed) and a 9-month- located?
old grandson. Make an administrative
decision:
Krok 2 Medicine 2010 2

A. 1 cm left from the left medioclavicular A. Erythrocyte hemolysis


line B. Intravascular hemolysis
B. 1 cm right from the left medioclavicular C. Disturbance of the conjugative function
line of liver
C. Along the left medioclavicular line D. Bile condensing
D. 1 cm left from he left parasternal line E. Mechanical obstruction of the bile
E. 1 cm right from the left parasternal line outflow
9. A 14-year-old boy has rheumati- 13. A 4-month-old girl with blond hair
sm. Over the last 2 years he has had and blue eyes has "mousy"odor of
3 rheumatic attacks. What course of sweat and urine, delayed psychomotoric
rheumatism does the patient have? development. The most typical laboratory
data for this disorder is:
A. Prolonged
B. Acute A. Positive urine ferric chloride test
C. Subacute B. High level of oxyproline in urine
D. Latent C. High level of glycosaminoglycanes in
E. Persistent-reccurent urine
D. High concentration of chlorides in
10. The patient with aquired heart failure sweat
has diastolic pressure of 0 mm Hg. What E. Low level of thyroid gland hormones in
heart failure does the child have? blood
A. Aortal insufficiency 14. A man, aged 25, presents with faci-
B. Mitral stenosis al edema, moderate back pains, body
C. Aortal stenosis temperature of 37, 5o C, BP- 180/100 mm
D. Mitral insufficiency Hg, hematuria (up to 100 in v/f), protei-
E. Rheumatism nuria (2,0 g/l), hyaline casts - 10 in v/f,
11. An 18-month-old child was taken specific gravity - 1020. The onset of the
to a hospital on the 4-th day of the disease is probably connected with acute
disease. The disease began acutely wi- tonsillitis 2 weeks ago. The most likely di-
th temperature 39, weakness, cough, agnosis is:
breathlessness. He is pale, cyanotic, A. Acute glomerulonephritis
has had febrile temperature for over 3 B. Acute pyelonephritis
days. There are crepitative fine bubbli- C. Cancer of the kidney
ng rales on auscultation. Percussi- D. Urolithiasis
on sound is shortened in the right E. Chronic glomerulonephritis
infrascapular region. X-ray picture shows
non-homogeneous segment infiltration 8- 15. A 62-year-old patient has DM-2. Di-
10 mm on the right, the intensification of abetes is being compensated by diet and
lung pattern. Your diagnosis: Maninilum. Pаtient has to undergo an
operation for inguinal hernia. What tacti-
A. Segmentary pneumonia cs of hypoglycemic therapy should be
B. Grippe chosen?
C. Bronchitis
D. Bronchiolitis A. Prescribe the short-acting insulin
E. Interstitial pneumonia B. Give Glurenorm in place of Maninilum
C. To continue with the current therapy
12. A baby was born at 36 weeks D. Prescribe thelong-acting insulin
of gestation. Delivery was normal, by E. Prescribe guanylguanidine
natural way. The baby has a large
cephalohematoma. The results of blood 16. A neonate is 5 days old. What vacci-
count are: Hb- 120g/l, Er- 3, 5 · 1012 /l, total nation dose of BCG vaccine (in мg) is
serum bilirubin - 123 mmol/l, direct bili- necessary for vaccination of this child?
rubin - 11 mmol/l, indirect - 112 mmol/l.
What are causes of hyperbilirubinemia in A. 0,05 мg
this case? B. 0,025 мg
C. 0,075 мg
D. 0,1 мg
E. 0,2 мg
17. Patient, male, 16 years old was behind
Krok 2 Medicine 2010 3

other children in development since early 22. A 20-year-old woman has a 3-4 month
childhood and still has moderate mental history of bloody diarrhoea; stool exami-
retardation. He is short, has dismorphic nation proved negative for ova and parasi-
body, his face is round, flattened, his eyes tes; stool cultures negative for clostridi-
are narrow and slanted, and there are epi- um, campylobacter and yersinia; normal
cantial folds in the corners of his eyes. small bowel series; edema, hyperemia
There is only one transversal flexor line and ulceration of the rectum and sigmoid
on his palms. What is the probable eti- colon seen on sigmoidoscopic examinati-
ology of this state? on. Select the most likely diagnosis:
A. Chromosome abnormality A. Ulcerative colitis
B. Gene abnormality B. Gastroenteritis
C. Maternal alcohol abuse during C. Carcinoid syndrome
pregnancy D. Zollinger-Ellison syndrome
D. Pathological delivery E. Granulomatous colitis
E. Infection in mother during pregnancy
23. A 36-year-old alcoholic patient has ci-
18. A patient who works as a nightman rrhosis and pancreatic insufficiency due
was diagnosed with of chronic arseni- to recurrent pancreatitis. He complains of
ous intoxication. What form of anemia night blindness, decreased ability to taste
is characteristic for this disease? food, and dry skin with hyperpigmentati-
on. These complaints suggest deficiency
A. Haemolytic anemia of:
B. Aplastic anemia
C. Iron deficiency anemia A. Zinc
D. Hyper sideric anemia B. Copper
E. Normochromic anemia C. Selenium
D. Chromium
19. A patient, aged 52, works as a E. Manganese
street cleaner. He has been suffering
from pneumoconiosis for 2 years. What 24. A 60-year-old woman, mother of 6
treatment should be recommended? children, developed a sudden onset of
upper abdominal pain radiating to the
A. Alkaline inhalations back, accompanied by nausea, vomiting,
B. Broncholitics fever and chills. Subsequently, she noti-
C. Oil inhalations ced yellow discoloration of her sclera and
D. Sulfonilamides skin. On physical examination the pati-
E. Antibiotics ent was found to be febrile with temp. of
38, 9oC, along with right upper quadrant
20. A 28-year-old patient was hospitalized tenderness. The most likely diagnosis is:
with preliminary diagnosis "influenza".
Roseolous-petechial rash appeared on A. Choledocholithiasis
the 5th day of disease on the trunk. The B. Benign biliary stricture
temperature is 41oC. Hyperemia of face, C. Malignant biliary stricture
reddening of scleras, tremor of tongue, D. Carcinoma of the head of the pancreas
tachycardia, splenomegaly are present. E. Choledochal cyst
What is the most likely diagnosis?
25. A 30-year-old patient with complai-
A. Epidemic typhus nts of occipital headache, disturbed sleep
B. Measles with nightmares came to a policlinic. BP
C. Alcohol delirium was 150/95 mm Hg. He was diagnosed wi-
D. Leptospirosis th hypertensic crisis. The patient should
E. Typhoid fever be registered in the following dispensary
group for arterial hypertension survei-
21. The diagnostics of the AIDS epidemic llance:
initially was made in the USA by means
of: A. In the second
B. In the first
A. The epidemiological method C. In the fourth
B. The bacteriological method D. In the third
C. The virological method E. In the fifth
D. The viroscopic method
E. The serological method 26. A young patient who came to a poli-
Krok 2 Medicine 2010 4

clinic was diagnosed with the 1 stage rheumatic fever complains of fever up
of hypertension. How often should he to 38 − 39o C, abdominal pain, dyspnea,
undergo the medical check-up? tachycardia. Heart borders are displaced
to the left by 2 cm, systolic and di-
A. Twice a year astolic murmurs above aorta, BP of
B. Once a year 160/30 mm Hg. Petechial rash occurs after
C. 3 times a year measurement of blood pressure. Liver is
D. 4 times a year enlarged by 3 cm, spleen is palpable. Uri-
E. 5 times a year ne is brown-yellow. What is the most likely
diagnosis?
27. A 45-year-old male patient was admi-
tted to the intensive care unit because of A. Infectious endocarditis
myocardial infarction. An hour later the B. Rheumatic fever
ventricular facilitation occurred. Which of C. Acute hepatitis
the following should be administered? D. Acute nephritis
E. Aortic regurgitation
A. Defibrillation
B. External chest compression 32. A 30-year-old patient complains of
C. Lidocaine injection breathlessness, pain in the right rib arc
D. Adrenalin injection region, dry cough and the edema of legs.
E. Cardiac pacing He is ill for 2 months. He had been treated
for rheumatic fever without any effect.
28. A 47-year-old obese man complained On exam: cyanosis, edema of legs, BT of
of periodic attacks of acute arthritis in 36, 6oC, RR of 28/min, HR of 90/min, BP
the st left tarsophalangeal joint. Lab exam of 110/80 mm Hg, crackles above low parts
revealed increased serum rate of uric acid. of both lungs, heart borders are displaced
What is the diagnosis? to the left and to the right, weak sounds,
A. Gout arthritis systolic murmur above the apex. What is
B. Reiter’s disease the preliminary diagnosis?
C. Rheumatoid arthritis A. Dilated cardiomyopathy
D. Rheumatic arthritis B. Infectious endocarditis
E. Osteoarthritis C. Acute myocarditis
29. A 40-year-old woman who has worked D. Rheumatic fever, mitral stenosis
in weaving branch for 10 years complai- E. Acute pericarditis
ns of frequent headache, sleeplessness, 33. A full term infant was born after
irritability, fatigue, tiredness. Physical a normal pregnancy, delivery, however,
examination revealed instability of blood was complicated by marginal placental
pressure, internal organs are without detachment. At 12 hours of age the chi-
changes. What is the most likely di- ld, although appearing to be in good
agnosis? health, passes a bloody meconium stool.
A. Noise-induced disease For determining the cause of the bleedi-
B. Hypertension ng, which of the following diagnostic
C. Atopic bronchial asthma procedures should be performed first?
D. Asthenovegetative syndrome A. Barium enema
E. Encephalopathy B. An Apt test
30. A 45-year-old driver was admitted to C. Gastric lavage with normal saline
the hospital with 5 hour substernal pain. D. An upper gastrointestinal series
Nitroglycerin is not effective. He is pale, E. Platelet count, prothrombin time, and
heart sounds are regular but weak. HR - partial thromboplastin time
96 per minute, BP of 100/60 mm Hg. What 34. A newborn infant has mild cyanosis,
is the most likely diagnosis? diaphoresis, poor peripheral pule,
A. Acute myocardial infarction hepatomegaly and cardiomegaly. Respi-
B. Stable angina ratory rate is 60 breaths per minute, and
C. Pulmonary embolism heart rate is 230 beats per minute. The chi-
D. Acute myocarditis ld most likely has congestive heart failure
E. Acute left ventricular failure caused by:

31. A 33-year-old man with a history of


Krok 2 Medicine 2010 5

A. Paroxysmal atrial tachycardia test data - mild leukocytosis and eosi-


B. A ventricular septal defect and nophilia. What is the diagnosis?
transposition of the great vessels
C. Atrial flutter and partial atrioventricular A. Allergic rhinitis
block B. Foreign body in the nose
D. Hypoplastic left heart syndrome C. Infective rhinitis
E. A large atrial septal defect and valvular D. Atrophic rhinitis
pulmonary stenosis E. Acute sinusitis
35. A 6-year-old boy was brought to the 40. A 27-year old patient with malaria
emergency room with a 3-hour history of caused by P. falciparum was treated wi-
fever up to 39, 5o C and sore throat. The th Chloroquine (600 mg base followed by
child looks alert, anxious and has a mild 300 mg base in 6 hours, then 300 mg base a
inspiratory stridor. You should immedi- day for 2 days) without clinical and parasi-
ately: tologic responses to the treatment. What
is the most likely reason for the failure to
A. Prepare to establish an airway respond to the therapy?
B. Obtain an arterial blood gas and start
an IV line A. Chloroquine resistant strain of P. falci-
C. Order a chest x-ray and lateral view of parum
the neck B. Glucose-6-phosphate dehydrogenase
D. Examine the throat and obtain a culture deficiency in patient
E. Admit the child and place him in a mist C. Late recognition of infection due to P.
tent falciparum
D. Inappropriate route of administration
36. For a patient with a stab injury to the E. Hypersensitivity of the patient to
femoral artery, the treatment of choice is: Chloroquine
A. End- to-end anastomosis 41. Ten hours before initial observation
B. Interposition of autogenous vein graft patient had frequent feces and vomiti-
C. Ligation of both transected ends ng. Fecal and vomiting masses looked
D. Interposition of homologous arterial like rice-water. Nausea and abdomi-
graft nal pain weren’t observed. Hiccup and
E. Interposition of Dacron graft convulsions of lower limbs, temperature
37. Which of the following best summari- 35, 4oC, hoarse voice, greyish wry face,
acrocyanosis were observed. Respiratory
zes indications for operation on an
abdominal aortic aneurysm? rate 40/min, threadlike pulse 120/min,
blood pressure 40/0 mm/Hg, abdomen is
A. Any aneurysm greater than 5 cm in drawn. What treatment measures should
diameter be taken first of all?
B. Any abdominal aortic aneurysm
C. Only symptomatic aneurysm A. Intravenous rehydration
B. Antibiotic therapy
D. Only symptomatic aneurysm greater
C. Cardiac glycosides
than 5 cm in diameter
E. Only ruptured aneurysm D. Disintoxicational therapy
E. Antibotulinic serum injection
38. Which of the following symptoms
42. A nurse of the kindergarten was taken
is the most typical for the adenoid
hypertrophy? to the hospital with complaints of accute
pain in parumbilical region, convulsions
A. Nasal obstruction of lower limbs, multiple bile vomiting,
B. Deglutition disturbance frequent watery foul feces of green colour
C. Olfaction disturbance in huge amounts. At the same time all
D. Giddiness the staff in the kindergarden got ill. Two
E. Headache days ago all of them ate cottage cheese wi-
th sour cream. General condition of pati-
39. A 30-year-old woman complains of ents is of moderate severity. Temperature
nasal blockage, sneezing, watery nasal di- 38, 2oC. Heart tones: rhythmic and muted.
scharges. The body temperature is normal. Heart rate 95/min, arterial pressure: 160
Objectively - edema of nasal mucous mm/Hg. Abdomen is slightly swollen, pai-
membrane on both sides, especially of the nful. Liver +2 cm. What is the most likely
lower turbinate (with cyanosis). Blood diagnosis?
Krok 2 Medicine 2010 6

A. Acute pancreatitis
A. Salmonellosis B. Acute cholecystitis
B. Dysentery C. Acute appendicitis
C. Cholera D. Acute diverticulitis
D. Food toxic infection E. Mesenteric adenitis
E. Enterovirus infection
46. In which of the following di-
43. A 45-year-old woman, mother of four sorders does the pathophysiology of
children, comes to the emergency room portal hypertension involve presinusoidal
complaining of a sudden onset of the epi- intrahepatic obstruction?
gastric and right upper quadrant pain,
radiating to the back, accompanied by A. Congenital hepatic fibrosis
vomiting. On examination, tenderness is B. Alcoholic cirrhosis
elicited in the right upper quadrant, bowel C. Hemochromatosis
sounds are decreased, and laboratory data D. Budd-Chiari syndrome
shows leukocytosis, normal serum levels E. Cavernomatous transformation of the
of amylase, lipase, and bilirubin. The most portal vein
likely diagnosis is:
47. A 25-year-old man was admitted to a
A. Acute cholecystitis hospital with a 2 month history of cough
B. Perforated peptic ulcer disease and fever. A chest x-ray showed extensive
C. Myocardial infarction left upper lobe disease with a 2 cm cavi-
D. Sigmoid diverticulitis ty. All three points were strongly positive
E. Acute pancreatitis on direct smear and grew M. Tuberculosis,
fully sensitive to all first-line drugs. Pati-
44. A 50-year-old man comes to the ent must be treated with:
emergency room with a history of vomiti-
ng of 3 days’ duration. His past history A. Isoniazid + rifampicin + pyrazinamide
examination reveals that for about 20 B. Streptomycin + isoniazid
years he has been suffering from epigasric C. Isoniazid + ethambutol
pain lasting for 2 to 3 weeks, during early D. Kanamycin + ethambutol + pyrazinami-
spring and autumn. He remembers getti- de
ng relief from pain by taking milk and E. P-aminosalicylic acid + streptomycin
antacids. Physical examination showed a
fullness in the epigastric area with visi- 48. A 5-year-old boy was progressively
ble peristalsis, absence of tenderness, and getting worse compared to the previous
normal active bowel sounds. The most li- 2 months. A chest x-ray has shown ri-
kely diagnosis is: ght middle lobe collapse. A tuberculin
skin test was strongly positive. What is
A. Gastric outlet obstruction the most characteristic finding in primary
B. Small bowel obstruction tuberculosis?
C. Volvulus of the colon
D. Incarcerated umbilical hernia A. Hilar or paratracheal lymph node
E. Cholecystitis enlargement
B. Atelectasis with obstructive pneumonia
45. A 24-year-old law student is brought C. Cavity formation
to the emergency room complaining of D. Miliary tuberculosis
severe abdominal pain of 6-8 hours durati- E. Hematogenous dissemination leading
on. He had been to a party the night to extrapulmonary tuberculosis
before. The pain is in the epigastrium
radiating to the back and is accompani- 49. A 50-year-old man was examined in
ed by nausea. The patient had vomited the clinic for persistent cavitation and
twice prior to coming to the emergency sputum. He was treated with rifampi-
room. Clinical examination revealed that cin, isoniazid, ethambuthol. The most
the young man was anxious, with acute common toxic effect of ethambutol is:
condition, with a regular pulse rate of A. Optic neuritis
100/min, blood pressure of 100/68 mm B. Eighth cranial nerve damage
Hg, and body temperature of 38, 1oC. The C. Hepatic enzyme elevation
most likely diagnosis is: D. Peripheral neuropathy
E. Mental symptoms
50. A 3-year-old child has been admi-
Krok 2 Medicine 2010 7

tted to a hospital because of ostealgia on the left from the breastbone in the
and body temperature rise up to 39oC. 3-4 intercostal space. This murmur is
Objectively: the patient is in grave condi- conducted above the whole cardiac regi-
tion, unable to stand for ostealgia, there on to the back. What congenital cardiac
is apparent intoxication, lymph nodesare pathology can be suspected?
enlarged up to 1,5 cm. Liver can be
palpated 3 cm below the costal margin, A. Defect of interventricular septum
spleen - 2 cm below the costal margin. In B. Defect of interatrial septum
blood: RBCs - 3, 0·1012 /l, Hb- 87 g/l, colour C. Coarctation of aorta
index - 0,9, thrombocytes - 190 · 109 /l, D. Fallot’s tetrad
E. Pulmonary artery stenosis
WBCs - 3, 2 · 109 /l, eosinophils - 1, stab
neutrophils - 1, segmented neutrophils - 54. A 27-year-old patient with a history
0, lymphocytes - 87, monocytes - 2, ESR of ronchial asthma was stung by a bee.
- 36 mm/h. What examination should He had a sensation of chest compression,
be conducted in order to specify the di- breath shortage, difficult expiration, sense
agnosis? of heat in the upper half of body, dizziness,
apparent itch, convulsions. Objectively:
A. Sternal puncture noisy wheezing breath, AP - 90/60 mm
B. Ultrasound Hg, Ps- 110 bpm. Auscultation revealed
C. Lymph node puncture weak rhythmic heart sounds, rough respi-
D. Lymph node biopsy ration above lungs, sibilant rales. What
E. Computer tomography drug group should be administered in the
51. A 22-year-old girl has been complai- first place?
ning of having itching rash on her face A. Glucocorticoids
for 2 days. She associates this disease wi- B. Methylxanthines
th application of cosmetic face cream. C. Cardiac glycosides
Objectively: apparent reddening and D. Anticonvulsive
edema of skin in the region of cheeks, E. Analgetics
chin and forehead; fine papulovesicular
rash. What is the most likely diagnosis? 55. A 32-year-old patient complains of
cardiac irregularities, dizziness, dyspnea
A. Allergic dermatitis at physical stress. He has never suffered
B. Dermatitis simplex from this before. Objectively: Ps- 74 bpm,
C. Eczema rhythmic. AP- 130/80 mm Hg. Auscultati-
D. Erysipelas on revealed systolic murmur above aorta,
E. Neurodermatitis the first heart sound was normal. ECG
52. Apgar test done on a newborn gi- showed hypertrophy of the left ventri-
rl at 1st and 5th minute after birth gave cle, signs of repolarization disturbance in
the result of 7-8 scores. During the deli- the I, V5 and V6 leads. Echocardiogram
very there was a short-term difficulty wi- revealed that interventricular septum was
th extraction of shoulder girdle. After bi- 2 cm. What is the most likely diagnosis?
rth the child had the proximal extremity A. Hypertrophic cardiomyopathy
dysfunction and the arm couldn’t be raised B. Aortic stenosis
from the side. The shoulder was turned C. Essential hypertension
inwards, the elbow was flexed, there was D. Myocardium infarction
also forearm pronation, obstetric palsy of E. Coarctation of aorta
brachial plexus. What is the clinical di-
agnosis? 56. On the 21 day after appearance of vesi-
culous chickenpox rash a 7-year-old chi-
A. Duchenne-Erb palsy ld developed ataxia, nystagmus, intenti-
B. Trauma of thoracic spine on tremor, muscle hypotonia. Liquor
C. Right hand osteomyelitis analysis shows insignificant lymphocytic
D. Intracranial haemorrhage pleocytosis, slightly increased protein rate.
E. Trauma of right hand soft tissues What complication is it?
53. Examination of a 9-month-old girl
revealed skin pallor, cyanosis during exci-
tement. Percussion revealed transverse
dilatation of cardiac borders. Auscultati-
on revealed continuous systolic murmur
Krok 2 Medicine 2010 8

A. Encephalitis 60. A painter working at a motorcar


B. Purulent meningitis plant was diagnosed with acute intoxi-
C. Pneumonitis cation with amide compounds of benzene,
D. Acute nephritis moderate severity grade. The in-patient
E. Postherpetic neuralgia treatment resulted in considerable health
improvement. What expert decision
57. Clinic of a research instutute for should be made in this case?
occupational diseases examined a worker
who works at a concentration plant and A. The patient should be issued a sick list
diagnosed him with chronic dust bronchi- for out-patient treatment
tis. The case is investigated by a commi- B. The patient may get back to work
ssion including the representatives of: the providing he will keep to hygiene and
plant, clinic, territorial SES, department sanitary regulations
of Social Insurance Fund, trade union. C. The patient should be referred to the
According to the "regulation on investi- Medical and Social Expert Commission
gation of. . . ", the commission should for assigning the disability group because
be headed by the representative of the of an occupational disease
following authority: D. The patient should be referred to the
Medical and Social Expert Commission
A. Territorial SES for determination of percentage of work
B. Plant capicty loss
C. Social Insurance Fund E. -
D. Trade union
E. Clinic 61. A maternity hospital registered 616 li-
ve births, 1 stillbirth, 1 death on the 5th
58. Basing upon the data of laboratory day of life over a 1 year period. What
assessment of sanitary state of soil in index allows the most precise estimation
a certain territory, the soil was found of this situation?
to be low-contaminated according to
the sanitary indicative value; contami- A. Perinatal mortality
nated according to the coli titer; low- B. Crude mortality rate
contaminated according to the anaerobe C. Natality
titer (Cl. Perfringens). This is indicative of: D. Neonatal mortality
E. Natural increase
A. Fresh fecal contamination
B. Insufficient intensity of soil humification 62. A 44-year-old patient complai-
C. Old fecal contamination ns about difficult urination, sensati-
D. Constant entry of organic protein on of incomplete urinary bladder
contaminations emptying. Sonographic examination of
E. Insufficient insolation and aeration of the urinary bladder near the urethra
soil entrance revealed an oval well-defined
hyperechogenic formation 2x3 cm large
59. A 28-year-old patient underwent that was changing its position during the
endometrectomy as a result of incomplete examination. What conclusion can be
abortion. Blood loss was at the rate made?
of 900 ml. It was necessary to start
hemotransfusion. After transfusion of A. Concrement
60 ml of erythrocytic mass the patient B. Malignant tumour of the urinary
presented with lumbar pain and fever whi- bladder
ch resulted in hemotransfusion stoppage. C. Urinary bladder polyp
20 minutes later the patient’s conditi- D. Prostate adenoma
on got worse: she developed adynamia, E. Primary ureter tumour
apparent skin pallor, acrocyanosis,
profuse perspiration. to - 38, 5oC, Ps- 63. An emergency team has delivered to
110/min, AP- 70/40 mm Hg. What is the a hospital an unconscious patient found
most likely diagnosis? lying in the street in winter. Objectively:
the patient is pale, with superficial respi-
A. Hemotransfusion shock ration; bradycardia with heartrate 54/min,
B. Hemorrhagic shock to - 35, 0oC. AP- 100/60 mm Hg. Palpati-
C. Septic shock on of chest and abdomen revealed no
D. Anaphylactic shock peritoneal symptoms. There is a smell of
E. DIC syndrome alcohol from the patient’s mouth. What is
the most likely diagnosis?
Krok 2 Medicine 2010 9

A. Up to 4500
A. Hypothermia B. Up to 2500
B. Acute cardiovascular insufficiency C. Up to 3500
C. Apparent death D. Up to 5500
D. Frostbite of trunk and extremities E. Up to 7500
E. -
68. A patient who has been consumi-
64. A 28-year-old parturient complai- ng refined foodstuffs for a long time
ns about headache, vision impairment, complains about headache, fatiguability,
psychic inhibition. Objectively: AP- depression, insomnia, irritability. Objecti-
200/110 mm Hg, evident edemata of vely: muscle asthenia, pain and cramps in
legs and anterior abdominal wall. Fetus the gastrocnemius muscles, during walki-
head is in the area of small pelvis. Fetal ng the patient lands onto his heel first,
heartbeats is clear, rhythmic, 190/min. then on the external edge of foot. Cardi-
Internal examination revealed complete ovascular system exhibits tachycardia,
cervical dilatation, fetus head was in the hypoxia, dystrophic changes of myocardi-
area of small pelvis. What tactics of labor um. There are also gastrointestinal di-
management should be chosen? sorders. What is the most likely diagnosis?
A. Forceps operation A. Hypovitaminosis B1
B. Cesarean B. Hypovitaminosis B2
C. Embryotomy C. Hypovitaminosis B12
D. Conservative labor management with D. Hypovitaminosis B6
episiotomy E. Hypovitaminosis B15
E. Stimulation of labor activity
69. A 9-year-old boy has been sufferi-
65. A 35-year-old patient complains about ng from bronchoectasis since he was 3.
pain and morning stiffness of hand joi- Exacerbations occur quite often, 3-4 ti-
nts and temporomandibular joints that mes a year. Conservative therapy results
lasts over 30 minutes. She has had these in short periods of remission. The di-
symptoms for 2 years. Objectively: edema sease is progressing, the child has physi-
of proximal interphalangeal digital joi- cal retardation. The child’s skin is pale,
nts and limited motions of joints. What acrocyanotic, he has "watch glass"nail
examination should be administered? deformation. Bronchography revealed
saccular bronchiectases of the lower lobe
A. Roentgenography of hands of his right lung. What is the further
B. Complete blood count treatment tactics?
C. Rose-Waaler reaction
D. Immunogram A. Surgical treatment
E. Proteinogram B. Further conservative therapy
C. Physiotherapeutic treatment
66. A 68-year-old female patient complai- D. Sanatorium-and-spa treatment
ns about temperature rise up to 38, 3oC, E. Tempering of the child’s organism
haematuria. ESR- 55 mm/h. Antibacterial
therapy turned out to be ineffective. What 70. A 46-year-old patient once took
diagnosis might be suspected? part in elimination of breakdown at an
atomic power plant. Currently he is bei-
A. Renal cancer ng treated at an in-patient hospital. He
B. Polycystic renal disease was diagnosed with progressing vegetati-
C. Renal amyloidosis ve insufficiency. This disease relates to
D. Urolithiasis the following group of ionizing radiation
E. Chronic glomerulonephritis effects:
67. Bacterial analysis of air in a livi- A. Somato-stochastic
ng space in winter period by means of B. Somatic
Krotov’s apparatus revealed that total C. Genetic
number of microorganisms in 1m3 of air D. Hormesis
was 7200. What is the allowed number E. Heterosis
of microorganisms for the air to be
characterized as "pure"? 71. A child is 4 years old, has been
ill for 5 days. There are complaints of
cough, skin rash, to - 38, 2oC, face puffiness,
photophobia, conjunctivitis. Objectively:
Krok 2 Medicine 2010 10

there is bright, maculo-papulous, in some A. Physical


areas confluent rash on the face, neck, B. Chemical
upper chest. The pharynx is hyperemic. C. Biological
There are seropurulent discharges from D. Combined
the nose. Auscultation revealed dry rales E. Physicochemical
in lungs. What is the most likely di-
agnosis? 75. In autumn a 25-year-old patient
developed stomach ache that arose 1,5-2
A. Measles hours after having meals and at night. He
B. Adenoviral infection complains about pyrosis and constipation.
C. Scarlet fever The pain is getting worse after consumi-
D. Rubella ng spicy, salty and sour food, it can be
E. Enterovirus exanthema relieved by means of soda and hot-water
bag. The patient has been suffering from
72. A female patient has been sufferi- this disease for a year. Objectively: furred
ng from pain in the right subcostal area, moist tongue. Abdomen palpation reveals
bitter taste in the mouth, periodical bile epigastrial pain on the right, resistance
vomiting for a month. The patient put off of abdominal muscles in the same region.
12 kg. Body temperature in the evening What is the most likely diagnosis?
is 37, 6o C. Sonography revealed that bi-
le bladder was 5,5х2,7 cm large, its wall - A. Duodenal ulcer
0,4 cm, choledochus - 0,8 cm in diameter. B. Chronic cholecystitis
Anterior liver segment contains a roundi- C. Diaphragmatic hernia
sh hypoechoic formation up to 5 cm in D. Stomach ulcer
diameter and another two up to 1,5 cm E. Chronic pancreatitis
each, walls of these formations are up to
0,3 cm thick. What is the most likely di- 76. On the 4th day after suturing the
agnosis? perforative stomach ulcer a patient wi-
th pulmonary emphysema developed
A. Alveolar echinococcus of liver spontaneous pneumothorax. What is the
B. Liver cancer best place for pleural drainage?
C. Liver abscess
D. Cystous liver cancer A. The second intercostal space along the
E. Paravesical liver abscesses medioclavicular line
B. The eighth intercostal space along the
73. A 46-year-old patient is to be prepared posterior axillary furrow
to the operation on account of stomach C. The seventh intercostal space along the
cancer. Preoperative preparation involves anterior axillary furrow
infusion therapy. It was injected up to 3,0 D. The sixth intercostal space along the
l of solutions into his right lunar vein. On anterior axillary furrow
the next day he got tensive pain in the E. The fifth intercostal space along the
region of his right shoulder. Examination medioclavicular line
of interior brachial surface revealed a stri-
pe of hyperemia, skin edema and a painful 77. A 32-year-old male patient has been
cord. What complication is it? suffering from pain in the sacrum and
coxofemoral joints, painfulness and sti-
A. Acute thrombophlebitis ffness in the lumbar spine for a year. ESR
B. Vein puncture and edema of paravenous - 56 mm/h. Roentgenography revealed
cellular tissue symptoms of bilateral sacroileitis. The
C. Necrosis of paravenous cellular tissue patient is the carrier of HLA B27 anti-
D. Acute lymphangitis gen. What is the most likely diagnosis?
E. Phlegmon of paravenous cellular tissue
A. Ankylosing spondylitis
74. A children’s health camp received a B. Coxarthrosis
party of tinned food. External examinati- C. Rheumatoid arthritis
on of the tins revealed that they had deep D. Reiter’s disease
dents; formed a concavity when pressed E. Spondylosis
and didn’t immediately return to the ini-
tial state; rust was absent; the tins were 78. A 58-year-old female patient complai-
greased with inedible fat. Specify the bloat ns about periodical headache, dizziness
type: and ear noise. She has been suffering from
diabetes mellitus for 15 years. Objecti-
vely: heart sounds are rhythmic, heart rate
Krok 2 Medicine 2010 11

is 76/min, there is diastolic shock above A. Acute glomerulonephritis with nephri-


aorta, AP is 180/110 mm Hg. In urine: tic syndrome
OD- 1,014. Daily loss of protein with uri- B. Acute glomerulonephritis with
ne is 1,5 g. What drug should be chosen nephrotic syndrome
for treatment of arterial hypertension? C. Acute glomerulonephritis with
nephrotic syndrome, hematuria and
A. Ihibitor of angiotensin converting hypertension
enzyme D. Acute glomerulonephritis with isolated
B. β-blocker urinary syndrome
C. Calcium channel antagonist E. Nephrolithiasis
D. Thiazide diuretic
E. α-blocker 82. A 60-year-old female patient was
admitted to a hospital for acute
79. A 50-year-old locksmith was di- transmural infarction. An hour ago
agnosed with typhoid fever. The patient the patient’s contition got worse. She
lives in a separate apartment with all faci- developed progressing dyspnea, dry
lities. Apart of him there are also 2 adults cough. Respiratory rate - 30/min, heart
in his family. What actions should be taken rate - 130/min, AP- 90/60 mm Hg. Heart
about persons communicating with the sounds are muffled, diastolic shock on the
patient? pulmonary artery. There are medium moi-
st rales in the lower parts of lungs on the
A. Bacteriological study right and on the left. Body temperature -
B. Antibiotic prophylaxis 36, 4oC. What drug should be given in the
C. Isolation first place?
D. Dispensary surveillance
E. Vaccination A. Promedol
B. Aminophylline
80. A child was taken to a hospital wi- C. Dopamine
th focal changes in the skin folds. The D. Heparin
child was anxious during examination, E. Digoxin
examination revealed dry skin with soli-
tary papulous elements and ill-defined 83. A 52-year-old male patient complains
lichenification zones. Skin eruption was about attacks of asphyxia, pain in his ri-
accompanied by strong itch. The child ght side during respiration. These mani-
usually feels better in summer, his condi- festations turned up all of a sudden. It is
tion is getting worse in winter. The child known from his anamnesis that he had
has been artificially fed since he was 2 been treated for thrombophlebitis of the
months old. He has a history of exudative right leg for the last month. In the admissi-
diathesis. Grandmother by his mother’s si- on ward the patient suddenly lost consci-
de has bronchial asthma. What is the most ousness, there was a sudden attack of
likely diagnosis? asphyxia and pain in his side. Objecti-
vely: heart rate - 102/min, respiratory rate
A. Atopic dermatitis - 28/min, AP- 90/70 mm Hg. Auscultati-
B. Contact dermatitis on revealed diastolic shock above the
C. Seborrheal eczema pulmonary artery, gallop rhythm, small
D. Strophulus bubbling rales above the lungs under the
E. Urticaria scapula on the right, pleural friction rub.
What examination method will be the
81. 2 weeks after recovering from angi- most informative for a diagnosis?
na an 8-year-old boy developed edemata
of face and lower limbs. Objectively: the A. Angiography of pulmonary vessels
patient is in grave condition, AP- 120/80 B. Echocardioscopy
mm Hg. Urine is of dark brown colour. C. Study of external respiration function
Oliguria is present. On urine analysis: D. ECG
relative density - 1,015, protein - 1,2 g/l, E. Coagulogram
RBCs are leached and cover the whole
vision field, granular casts - 1-2 in the vi- 84. A primagravida in her 20th week
sion field, salts are represented by urates of gestation complains about pain in
(big number). What is the most likely di- her lower abdomen, blood smears from
agnosis? the genital tracts. The uterus has an
increased tonus, the patient feels the
fetus movements. Bimanual examination
Krok 2 Medicine 2010 12

revealed that the uterus size corresponded was regarded as a pubertal manifestati-
the term of gestation, the uterine cervix on, the girl didn’t undergo any treatment.
was contracted down to 0,5 cm, the The girl’s irritability gradually gave place
external orifice was open by 2 cm. The to a complete apathy. The girl got puffy
discharges were bloody and smeary. What face, soft tissues pastosity, bradycardia,
is the most likely diagnosis? constipations. Skin pallor and gland densi-
ty progressed, the skin got a waxen hue.
A. Incipient abortion What disease may be assumed?
B. Risk of abortion
C. Abortion in progress A. Autoimmune thyroiditis
D. Incomplete abortion B. Diffuse toxic goiter
E. Missed miscarriage C. Thyroid carcinoma
D. Subacute thyroiditis
85. On the 4th day after recovering from E. Juvenile basophilism
a cold a patient was hospitalized with
complaints of solitary spittings of mucoid 88. A newborn’s head is of doli-
sputum. On the 2nd day there was a si- chocephalic shape, that is front-to-back
ngle discharge of about 250 ml of purulent elongated. Examination of the occipi-
blood-streaked sputum. Objectively: the tal region of head revealed a labour
patient’s condition is moderately severe. tumour located in the middle between the
Respiratory rate - 28-30/min, Ps- 96 bpm, prefontanel and posterior fontanel. The
AP- 110/70 mm Hg. Respiration above the delivery tok place with the following type
left lung is vesicular, weak above the right of fetus head presentation:
lung. There are moist rales of different
types above the lower lobe and amphoric A. Posterior vertex presentation
breath near the angle of scapula. What is B. Anterior vertex presentation
the most likely diagnosis? C. Presentation of the bregma
D. Brow presentation
A. Acute pulmonary abscess E. Face presentation
B. Exudative pleuritis
C. Acute focal pneumonia 89. A 56-year-old patient with diffuse
D. Pleural empyema toxic goiter has ciliary arrhythmia with
E. Pyopneumothorax pulse rate 110 bpm, arterial hypertensi-
on, AP- 165/90 mm Hg. What preparati-
86. 350 workers of a mettalurgical plant on should be administered along with
had to undergo a yearly preventive exami- mercazolil?
nation. A territorial polyclinic carried out
preventive examination of 325 workers. A. Propranolol
As a result of it, 1 worker was recogni- B. Radioactive iodine
zed as temporarily disabled, 15 workers C. Procaine hydrochloride
underwent further rehabilitation at an D. Verapamil
after-work sanatorium, 10 workers were E. Corinfar
provided with diet meal. What index
characterizing the preventive work of the 90. Over a current year among workers of
polyclinic should be applied in this case? an institution 10% haven’t been ill a single
time, 30% have been ill once, 15% - twice,
A. Coverage of preventive medical exami- 5% - 4 times, the rest - 5 and more times.
nations What is the percentage of workers relati-
B. Frequency of case detection during ng to the I health group?
examinations
C. Percentage of people who underwent A. 55%
rehabilitation at an after-work sanatorium B. 10%
D. Percentage of people who were provi- C. 40%
ded with diet meal D. 60%
E. Percentage of temporarily disabled E. 22%
people 91. A 16-year-old boy was admitted to the
87. A 14-year-old girl has been presenti- hospital for the reason of intractable nasal
ng with irritability and tearfulness for haemorrhage and intolerable pain in the
about a year. A year ago she was also right cubital articulation. Objectively: the
found to have diffuse enlargement of the affected articulation is enlarged and exhi-
thyroid gland (II grade). This condition bits defiguration and skin hyperaemia.
There are manifestations of arthropathy
Krok 2 Medicine 2010 13

in the other articulations. Ps- 90 bpm; A. Insulinemia on an empty stomach


colour index - 1,0, WBC - 5, 6 · 109 /l, B. Daily glycemia variability
thrombocytes- 220 · 109 /l, ESR - 6 mm/h. C. Daily glucosuria variability
Lee-White coagulation time: start - 24’, fi- D. Glycemia test an hour after taking
nish - 27’10”. What drug will be the most meals
effective for this patient treatment? E. Glucose tolerance test

A. Cryoprecipitate 95. A 52-year-old male patient has an 18


B. Calcium chloride year history of diabetes mellitus. One year
C. Erythromass ago he had cystitis. The patient takes 0,005
D. Aminocapronic acid g of maninil thrice a day. Objectively: hei-
E. Vicasol ght - 176 cm, weight - 82 kg. Glycemia
variability on an empy stomach is at the
92. A 42-year-old woman complains about rate of 10,3-12,4 millimole/l. Analyses
bruises on her both legs and prolonged revealed proteinuria at the rate of 0,033
menstruation; general weakness, tinni- g/l. The most efficient way to prevent di-
tus cerebri. Objectively: multiple macular abetic nephropathy progress will be:
haemorrhages on the legs and body.
The patient presents with tachypnoe, A. To replace maninil with insulin
tachycardia, systolic murmur in all B. To increase maninil dosage
auscultatory points. AP- 75/50 mm Hg. C. To decrease daily caloric content
Blood count: RBC - 1, 9 · 1012 /l, Нb- 60 D. To supplement the present therapy with
g/l, colour index - 0,9, WBC - 6, 5 · 109 /l, insulin
thrombocytes - 20 · 109 /l, ESR- 12 mm/h. E. To administer antibacterial therapy
Duke bleeding time - 12 minutes. Bone 96. A 34-year-old female patient complai-
marrow analysis revealed plenty of juveni- ns about weakness, 12 kg weight loss
le immature forms of megacaryocytes within 6 months, sweating, palpitation,
without signs of thrombocyte pinch-off. irritability. Objectively: III grade thyroid
What is the most likely diagnosis? gland is elastic, diffuse enlargement is
A. True thrombocytopenic purpura present, there is also a node in the ri-
B. Type A haemophilia ght lobe. Cervical lymph nodes are not
C. Willebrand’s disease enlarged. What treatment tactics would
D. Acute megacaryoblastic leukemia be the most rational?
E. Tupe B haemophilia A. Operation after antithyroid therapy
93. A 43-year-old man who often contacts B. Radioactive iodine administration
with ethyl gasoline was admitted to C. Immediate surgical intervention
a hospital with complaints of general D. Conservative antithyroid therapy
weakness, dizziness, memory impairment, E. Immediate gamma-ray teletherapy
sleepiness at daytime and insomnia at ni- 97. On the next day after being taken by
ght, sense of hair in the mouth, colicky influenza a 46-year-old woman presented
pains in the right subcostal region. What with intensified headache, dizziness,
is the most likely diagnosis? nausea. Objectively: the patient is consci-
A. Chronic tetraethyl lead intoxication ous, psychomotor excitement is present;
B. Alcoholic delirium there is general hyperesthesia, moderate
C. Chronic mercury intoxication meningeal syndrome, nystagmus. Tendon
D. Chronic manganese intoxication teflexes are higher on the right, right
E. Chronic lead intoxication extremities display muscle weakness,
right-sided pathological Babinski’s sign is
94. A 30-year old woman taken by present. Liquor is transparent, pressure is
influenza has empty stomach glycemia at 220 mm of water column; cytosis is 46/3
the rate of 11,3 millimole/l, glucosuria at with prevailing lymphocytes. What is the
the rate of 25 g/l. The patient is 168 cm tall most likely diagnosis?
and weighs 67 kg. What test would be the
most informative for the diagnosis specifi- A. Influenzal meningoencephalitis
cation? B. Bacterial meningoencephalitis
C. Subarachnoidal haemorrhage
D. Parenchymatous subarachnoidal
haemorrhage
E. Ischemic stroke
Krok 2 Medicine 2010 14

98. After examination a 46-year-old pati- Over the last week the attacks occurred
ent was diagnosed with left breast cancer daily and became more painful. On the
T2N2M0, cl. gr. II-a. What will be the 3rd day of hospitalization the patient
treatment plan for this patient? presented with icteritiousness of skin and
scleras, light-colored feces and dark uri-
A. Radiation therapy + operation + ne. In blood: neutrophilic leukocytosis -
chemotherapy 13, 1 · 109 /l, ESR - 28 mm/h. What is the
B. Operation only most likely diagnosis?
C. Operation + radiation therapy
D. Radiation therapy only A. Chronic calculous cholecystitis
E. Chemotherapy only B. Chronic recurrent pancreatitis
C. Fatty degeneration of liver
99. A 34-year-old male patient was deli- D. Chronic cholangitis, exacerbation stage
vered to the neurological department wi- E. Hypertensive dyskinesia of gallbladder
th complaints of intense headache, double
vision, light and noise intolerance. The 103. A 30-year-old gravida consulted a
attack came suddenly while load lifting. gynecologist about bright red bloody di-
Objectively: torpor, moderate divergent scharges from the vagina in the 32 week of
strabismus, diplonia. Bilateral Kernig’s gestation. She was hospitalized with suspi-
symptoms. No paresises. Bloody liquor. cion of placental presentation. Under
What medication should be administered what conditions is it rational to conduct
in the first line? the internal examination for the diagnosis
specification?
A. Epsilon-aminocapronic acid
B. Acetylsalicilic acid A. In the operating room prepared for the
C. Heparin operation
D. Nicotinic acid B. In the examination room of antenatal
E. Glutamic acid clinic
C. In the admission ward of maternity
100. A patient suffering from acute hospital
posttraumatic pain received an injection D. In the delivery room keeping to all the
of morphine that brought him a significant aseptics regulations
relief. Which of the following mechani- E. The examination is not to be conducted
sms of action provided antishock effect of because of risk of profuse haemorrhage
morphine in this patient?
104. A 72-year-old male patient complai-
A. Stimulation of opiate receptors ns about itch in his left shin, especially
B. Block of central cholinergic receptors around a trophic ulcer. Skin is reddened
C. Stimulation of benzodiazepine receptors and edematic, there are some oozing lesi-
D. Inhibition of dopamine mediation ons, single yellowish crusts. The focus of
E. Intensification of GABA-ergic reactions affection is well-defined. What is the most
101. After supercooling a 38-year- likely diagnosis?
old woman developed muscle pain, A. Microbial eczema
body temperature rise up to 39oC, B. Allergic dermatitis
headache, dysuria, positive Pasternatsky’s C. Seborrheic eczema
symptome. In the urine: leukocyturia, D. Cutaneous tuberculosis
bacteriuria. In blood: Decrease in Hb rate E. Streptococcal impetigo
down to 103 g/l, left shift leukocytosis,
ESR acceleration up to 32 mm/h. Blood 105. A 30-year-old patient presented with
urea - 6,0 millimole/l. What is the most li- body temperature rise up to 38, 5o C, pain
kely diagnosis? in the small articulations of hands; face
edemata and erythema. In blood: RBCs -
A. Acute pyelonephritis 2, 6 · 1012 /l; Hb- 98 г/л; WBCs - 2 · 109 /l;
B. Renal tuberclosis ESR - 58 mm/h. In the urine: protein - 3,1
C. Acute glomerulonephritis g/l; RBCs - 10-15 in the vision field. What
D. Urolithiasis disease can be suspected in this case?
E. Acute cystitis
102. A 50-year-old patient complains
about having pain attacks in the right
subcostal area for about a year. He pain
arises mainly after taking fattening food.
Krok 2 Medicine 2010 15

A. Systemic lupus erythematosus A. Neuroleptic therapy


B. Sepsis B. Vitamin therapy
C. Systemic scleroderma C. Physiotherapy
D. Periarteritis nodosa D. Psychotherapy
E. Acute glomerulonephritis E. Reflex therapy
106. A 5-grade pupil complains about 110. A patient has got acute macrofocal
extensive skin rash accompanied by myocardial infarction complicated by
intensive itch, especially at night. Objecti- cardiogenic shock. The latter is progressi-
vely: there are small red papules set ng under conditions of weak general peri-
mostly in pairs in the region of interdigital pheric resistance and decreased cardiac
folds on both hands, on the flexor surface output. What antihypotensive drug should
of radicarpal articulations, abdomen and be injected to the patient in the first place?
buttock skin as well as internal surface
of thighs. In the centre of some papules A. Dopamine
vesicles or serohaemorrhagic crusts can B. Noradrenaline
be seen. There are multiple excoriations. C. Adrenaline
What is the most likely diagnosis? D. Mesatonum
E. Prednisolone
A. Scabies
B. Dermatitis 111. A patient came to the traumatology
C. Ringworm of body centre and complained about a trauma
D. Toxicoderma of the lower third of the volar forearm
E. Eczema surface caused by cut on a piece of glass.
Objectively: flexion of the IV and V fi-
107. A 43-year-old female patiet complai- ngers is impaired, sensitivity of the interi-
ns of eruption on her right leg skin, pain, or dorsal and palmar surface of hand as
weakness, body temperature rise up to well as of the IV finger is reduced. What
38o C. The disease is acute. Objectively: nerve is damaged?
there is an edema on the right leg skin in
the region of foot, a well-defined bright A. Ulnar
red spot in form of flame tips which feels B. Radial
hot. There are isolated vesicles in focus. C. Median
What is your provisional diagnosis? D. Musculoskeletal
E. Axillary
A. Erysipelas
B. Microbial eczema 112. A patient was delivered to the
C. Contact dermatitis admission ward 40 minutes after a
D. Toxicoderma minor trauma with complaints of
E. Haemorrhagic vasculitis pain in the thoracic spine. He has
a history of pulmonary tuberculosis.
108. Surgical department admitted a 37- Roentgenography results: suspected
year-old patient with a big crushed wound compression fracture of the VIII and
of his left thigh 4 hours after he got IX thoracic vertebrae. What pathologi-
this trauma. What is the main provisi- cal process in spine might have stimulated
on for successful prevention of gaseous pathological fracture of vertebral bodies?
gangrene?
A. Tuberculous spondylitis
A. Removal of necrotic tissues and timely B. Syphilis
surgical processing of the wound C. Malignant tumour metastasis
B. Injection of specific serum 3 000 U D. Osteomyelitis
C. Injection of specific serum 30 000 U E. Traumatic fracture
D. Infiltration of soft tissues around the
wound with antibiotic solution 113. On the next day after esophagoscopy
E. Wound lavage with 6% solution of and biopsy a 44-year-old female patient
hydrogen peroxide developed emphysema of both supraclavi-
cular regions, deglutition pain and pain
109. A 35-year-old patient was admitted to behind the breastbone, cyanosis. Body
the in-patient psychiatric facility for the fi- temperature - 39o C. In blood: WBCs -
rst time. He was diagnosed with paranoid 16 · 109 /l, left shift in the neutrophils. X-
schizophrenia. What is the most rational ray picture shows a limited shadow of
treatment method? the posterior mediastinum at a level wi-
th trachea bifurcation. The patient was
Krok 2 Medicine 2010 16

diagnosed with esophagus rupture. What breast. Objectively: in the upper internal
treatment tactics would be the most rati- quadrant of the left breast there is a
onal? neoplasm up to 2,5 cm in diameter, dense,
uneven, painless on palpation. Regional
A. Surgical: mediastinum drainage, gastric lymph nodes are not enlarged. What is the
fistula establishment most likely diagnosis?
B. Surveillance
C. Conservative antibacterial treatment A. Cancer
D. Radical surgical treatment with B. Cyst
esophagus suturing C. Fibroadenoma
E. Surgical: gastric fistula establishment D. Mastopathy
E. Lipoma
114. A 30-year-old woman with a long hi-
story of chronic pyelonephritis complains 118. A 40-year-old female patient
about considerable weakness, sleepiness, has been observing profuse menses
decrease in diuresis down to 100 ml per accompanied by spasmodic pain in
day. AP- 200/120 mm Hg. In blood: creati- the lower abdomen for a year. Bi-
nine - 0,62 millimole/l, hypoproteinemia, manual examination performed during
albumines - 32 g/l, potassium - 6,8 milli- menstruation revealed a dense formati-
mole/l, hypochromic anemia, increased on up to 5 cm in diameter in the cervical
ESR. What is the first step in the pati- canal. Uterus is enlarged up to 5-6 weeks
ent treatment tactics? of pregnancy, movable, painful, of normal
consistency. Appendages are not palpable.
A. Haemodialysis Bloody discharges are profuse. What is the
B. Antibacterial therapy most likely diagnosis?
C. Enterosorption
D. Haemosorption A. Nascent submucous fibromatous node
E. Blood transfusion B. Abortion in progress
C. Cervical carcinoma
115. A 10-year-old girl consulted a doctor D. Cervical myoma
about thirst, frequent urination, wei- E. Algodismenorrhea
ght loss. She has been observing these
symptoms for about a month. Objecti- 119. A 65-year-old patient complai-
vely: no pathology of internal organs was ns about pain in the lumbar spine,
revealed. What laboratory analysis should moderate disuria. He has been sufferi-
be carried out in the first place? ng from these presentations for about
half a year. Prostate volume is 45
A. Blood glucose analysis on an empty cm3 (there are hypoechogenic nodes in
stomach both lobes, capsule invasion). Prostate-
B. Glucose in urine test on the base of specific antigen is 60 ng/l. Prostate biopsy
daily diuresis revealed an adenocarcinoma. Which of
C. Acetone in urine test the supplemental examination methods
D. Glucose tolerance test will allow to determine the stage of
E. Glucosuric profile neoplastic process in this patient?
116. A 19-year-old patient complains A. Computer tomography of pelvis
about skin rash that appeared 2 days ago B. Roentgenography of lumbar spine
after eating smoked fish. The rash di- C. Excretory urography
sappears after 4-6 hours but then turns up D. Bone scintigraphy
again. It is accompanied by itch. Objecti- E. Roentgenography of chest
vely: trunk and upper limbs are covered
with multiple pink blisters as big as a pea 120. A 44-year-old man has been worki-
or a bean. What is the most likely di- ng in coke industry for 16 years. Dust
agnosis? concentration at his workplace is 5-10
times higher than maximum permissi-
A. Acute urticaria ble concentration. Roentgenography of
B. Allergic dermatitis lungs revealed changes that are typical
C. Quincke’s edema for pneumoconiosis. What is the most
D. Toxicodermia probable type of pneumoconiosis in this
E. Purigo case?
117. A 68-year-old patient consulted
a doctor about a tumour in her left
Krok 2 Medicine 2010 17

A. Anthracosis A. 4
B. Anthracosilicosis B. 1
C. Silicatosis C. 2
D. Asbestosis D. 3
E. Siderosis E. 5
121. Study of morbidity rate in a city 125. 2 days ago a patient presented with
N revealed that population of different acute pain in the left half of chest, gneral
administrative districts differed in age weakness, fever and headache. Objecti-
structure. What statistic method allows vely: between the 4 and 5 rib on the left
to eliminate influence of this factor upon the skin is erythematous, there are multi-
morbidity indices? ple groups of vesicles 2-4 mm in diameter
filled with transparent liquid. What diease
A. Standardization are these symptoms typical for?
B. Wilcoxon’s t-criterion
C. Correlative regressive analysis A. Herpes zoster
D. Analysis of dynamic series B. Pemphigus
E. Calculation of average values C. Herpes simplex
D. Streptococcal impetigo
122. A driver had been fixing a car in E. Herpetiform Duhring’s dermatosis
a closed garage and afterwards complai-
ned about headache, dizziness, nausea, 126. A 27-year-old military servant was
muscle asthenia, sleepiness. Objectively: delivered to the regimental aid post 1,5
pulse and respiratory rate elevation, hour after getting an abdominal injury wi-
excitement, hypertension, delirium of th an aerial bomb splinter. Objectively:
persecution. What is the most likely di- the patient is in grave condition, pale, Ps-
agnosis? 132/min, AP- 80/60 mm Hg. Subcostal
area has a bandage on from the first-
A. Intoxication with carbon oxide aid pack, the bandage is soaked with
B. Intoxication with ethyl gasoline blood. The abdomen is tense. Percussion
C. Posttraumatic encephalopathy revealed dullness in flat parts that changes
D. Hypertensive crisis its location with the change of body posi-
E. Asthenovegetative syndrome tion. There are symptoms of peritoneal
irritation. What department of the regi-
123. A worker who undergoes regular mental aid post should provide medical
medical check-up for duodenal ulcer care?
received a subsidized 24-day sanatori-
um voucher from his plant. The term of A. Dressing ward
annual leave of a worker is 24 calender B. Admission-and-sorting tent
days, it will take 4 days more to get to the C. Isolation ward
sanatorium and back home. What is the D. Evacuation
procedure of obtaining a 4-day sick-leave? E. In the manner of self-help
A. Medical Expert Commission issues a 127. A sergeant got injured with a shell
4-day sick list splinter in the left subcostal region. He
B. Medical Expert Commission issues a was bandaged by his fellow soldiers wi-
28-day sick list th an individual field dressing. Afterwards
C. The doctor in charge issues a 4-day sick he was delivered to the regimental medi-
list cal station. The patient complains of di-
D. The doctor in charge issues a health zziness, weakness, thirst, abdominal pain.
certificate and sanatorium patient’s file for His general condition is grave, he is pale,
28 days Ps- 120 bpm. The abdomen is painful, soft
E. Medical Expert Commission issues a on palpation. The dressing is well-fixed
4-day health certificate but slightly soaked with blood. The vi-
ctim should be evacuated to the collecti-
124. The total area of a ward at the ng battalion with the following type of
therapeutical department is 28 m2 . What transport and in the following turn:
is the maximum number of beds that can
be exploited in this ward?
Krok 2 Medicine 2010 18

A. With ambulance vehicle in the 1st turn


B. With a passing vehicle in the 1st turn A. Hypertonic solution
C. With ambulance vehicle in the 2nd turn B. Vishnevsky ointment
D. With a passing vehicle in the 2nd turn C. Ichthyol ointment
E. With a passing vehicle in the 3rd turn D. Chloramine
E. Dimexide
128. On the 5th day after labor body
temperature of a 24-year-old parturient 132. A 37-year-old patient was delivered
suddenly rose up to 38, 7oC. She complai- to a hospital in unconscious state. He
ns about weakness, headache, abdominal got a closed abdominal injury. He is
pain, irritability. Objectively: AP- 120/70 suspected to have internal haemorrhage,
mm Hg, Ps- 92 bpm, to - 38, 7oC. Bimanual intraabdominal injury. He is being
examination revealed that the uterus was prepared an emergency operation. By
enlarged up to 12 weeks of pregnancy, it means of standard sera a physician
was dense, slightly painful on palpation. determined his blood group and got the
Cervical canal lets in 2 transverse fingers, following results: 0(I) - no agglutinati-
discharges are moderate, turbid, with foul on, А(II) - no agglutination, В(III) - no
smell. In blood: skeocytosis, lymphopenia, agglutination. What is the patient’s blood
ESR - 30 mm/h. What is the most likely di- group?
agnosis?
A. 0(I)
A. Endometritis B. А(II)
B. Parametritis C. В(III)
C. Pelviperitonitis D. АВ(IV)
D. Metrophlebitis E. -
E. Lochiometra
133. A 36-year-old man was delivered
129. An 11-yearold girl was taken by an to the surgical department an hour
acute disease: she got pain in the lumbar after a road accident. His condition
region, nausea, vomiting, frequent uri- is getting worse: respiratory insuffici-
nation, body temperature 39o C. Objecti- ency is progressing, there are cardiac
vely: the abdomen is soft, painful on abnormalities. Clinical and roentgenologi-
palpation in the lumbar region. Common cal investigations revealed mediastinal di-
urine analysis revealed considerable splacement. What process has caused this
leukocyturia, bacteriuria. The urine complication?
contained colibacilli. What is the most li-
kely diagnosis? A. Valvular pneumothorax
B. Open pneumothorax
A. Acute pyelonephritis C. Closed pneumothorax
B. Acute appendicitis D. Subcutaneous emphysema
C. Chronic glomerulonephritis E. Mediastinitis
D. Acute vulvovaginitis
E. Acute glomerulonephritis 134. An 18-year-old primigravida in
her 27-28 week of gestation underwent
130. A patient is active, lively, fussy. He an operation on account of acute
opposes the examination. His speech phlegmonous appendicitis. In the
is fast and loud, his statements are postoperative period it is necessary to
spontaneous and inconsequent. Specify take measures for prevention of the
the psychopathological state of this pati- following pegnancy complication:
ent:
A. Noncarrying of pregnancy
A. Psychomotor agitation B. Intestinal obstruction
B. Catatonic agitation C. Fetus hypotrophy
C. Delirium D. Premature placenta detachment
D. Behavioral disorder E. Late gestosis
E. Paranoid syndrome
135. A 30-year-old patient presents with
131. A surgeon examined a 42-year-old sudden loss of consciousness, tonoclonic
patient and diagnosed him with right spasms, AP- 150/100 mm Hg, heart rate
forearm furuncle, purulo-necrotic stage. - 100/min. Before the attack he felt di-
The furuncle was lanced. At the hydration zziness, sense of irreality, inexplicable
stage the wound dressing should enclose fear. After the attack the patient failed
the following medication:
Krok 2 Medicine 2010 19

to remember it. What is the most likely A. Legionella


diagnosis? B. Mycoplasm
C. Streptococcus
A. Epilepsy D. Staphylococcus
B. Morgagni-Adams-Stokes syndrome E. Pneumococcus
C. Vestibular syncope
D. Transitory ischemic attack 140. A 47-year-old patient complains
E. Hyperventilation ischemic attack about cough with purulent sputum, pain
in the lower part of the left chest, peri-
136. A 26-year-old male patient complai- odical body temperature rise. She has
ns of piercing pain during breathing, been suffering from these presentations
cough, dyspnea. Objectively: to - 37, 3oC, for about 10 years. Objectively: "drumsti-
respiration rate - 19/min, heart rate=Ps ck"distal phalanges. What examination
- 92/min; AP- 120/80 mm Hg. Vesicular would be the most informative for making
respiration. In the inferolateral parts of a diagnosis?
chest auscultation in both inspiration and
expiration phase revealed noise that was A. Bronchography
getting stronger at phonendoscope pressi- B. Bronchoscopy
ng and can be still heard after cough. ECG C. Survey radiograph of lungs
showed no pathological changes. What is D. Pleural puncture
the most likely giagnosis? E. Bacteriological analysis of sputum
A. Acute pleuritis 141. A heat station working on solid fuel
B. Intercostal neuralgia is located in a residential district. On
C. Subcutaneous emphysema cloudy foggy days in december there was
D. Spontaneous pneumothorax an increase in diseases with upper airway
E. Pericarditis sicca affection and signs of general intoxicati-
on. There were also mortal cases among
137. HIV displays the highest tropism the elderly people. What is the most likely
towards the following blood cells: factor that provoked toxic effect?
A. T-helpers A. Suspended materials
B. T-suppressors B. High air humidity
C. T-killers C. Calm
D. Thrombocytes D. Low air temperature
E. Erythrocytes E. Temperature gradient
138. In terms of megacalorie (1000 kcal = 142. A 32-year-old female patient
4184 kJ) the ration of an adult includes consulted a doctor about a skin affecti-
30 g of proteins, 37 g of fats, 137 g of on of her body, face and extremities which
carbohydrates, 15 mg of vitamin C, 0,6 turned up after peroral intake of biseptol.
mg of thiamine (vitamin B1 ). The ration is The patient was diagnosed with drug toxi-
UNBALANCED as to the contents of: coderma. What treatment methods would
be the most effective for this patient?
A. Vitamin C
B. Proteins A. Extracorporal therapy
C. Fats B. Anti-inflammatory therapy
D. Carbohydrates C. Antihistamine therapy
E. Thiamine D. Disintoxication therapy
E. Immunotherapy
139. A 22-year-old patient is a clerk. His
working day runs in a conditioned room. 143. A 42-year-old woman has had
In summer he was taken by an acute di- hyperpolymenorrhea and progressing
sease with the following symptoms: fever, algodismenorrhea for the last 10 years.
dyspnea, dry cough, pleural pain, myalgia, Gynecological examination revealed no
arthralgia. Objectively: moist rales on the changes of uterine cervix; discharges are
right, pleural friction rub. X-ray picture moderate, of chocolate colour, uterus is
showed infiltration of the inferior lobe. In slightly enlarged and painful, appendages
blood: WBC - 11 · 109 /l, stab neutrophi- are not palpable, the fornices are deep
ls - 6%, segmented neutrophils - 70%, and painless. What is the most likely di-
lymphocytes - 8%, ESR - 42 mm/h. What agnosis?
is the ethiological factor pneumonia?
Krok 2 Medicine 2010 20

A. Uterine endometriosis diffuse toxic goiter of the III-IV degree


B. Uterine carcinoma 12 years ago. Now she has recurrence of
C. Subserous uterine fibromyoma thyrotoxicosis. The patient was offered
D. Endomyometritis operative intervention, but it is necessary
E. Adnexal endmetriosis first to localize the functioning gland ti-
ssue. What method should be applied for
144. On the tenth day after discharge this purpose?
from the maternity house a 2-year-old
patient consulted a doctor about body A. Gland scanning
temperature rise up to 39o C, pain in the B. USI
right breast. Objectively: the mammary C. Puncture aspiration biopsy
gland is enlarged, there is a hyperemized D. Roentgenography of neck
area in the upper external quadrant, in the E. Roentgenography of esophagus
same place there is an ill-defined indurati-
on, lactostasis, fluctuation is absent. 148. A 26-year-old woman complains of
Lymph nodes of the right axillary regi- having bloody discharges from the geni-
on are enlarged and painful. What is the tals for the last 14 days, abdominal pain,
most likely diagnosis? general fatiguability, weakness, weight
loss, body temperature rise, chest pain,
A. Lactational mastitis obstructed respiration. 5 weeks ago she
B. Abscess underwent induced abortion in the 6-7
C. Erysipelas week of gestation. Objectively: the patient
D. Dermatitis is pale and inert. Bimanual examination
E. Tumour revealed that the uterus was enlarges up
to 8-9 weeks of gestation. In blood: Hb- 72
145. A 47-year-old male patient got a g/l. Urine test for chorionic gonadotropin
flame burn of trunk and upper extremi- gave the positive result. What is the most
ties and was delivered to the hospital. The likely diagnosis?
patient is in grave condition, confused
mental state, with fever. AP- 80/50 mm A. Chorioepithelioma
Hg, Ps- 118 bpm. It was locally stated that B. Metroendometritis
the patient got III B degree burns with C. Uterus perforation
total area of 20%. What medical actions D. Uterine fibromyoma
should be taken? E. Uterine carcinoma
A. Injection of narcotic analgetics and 149. A 1,5-year-old child was taken by an
powdered blood substitutes acute disease: body temperature up to
B. Initial surgical d-bridement 39oC, frequent vomiting up to 5 times.
C. Administration of detoxicating blood Nervous system tests revealed positive
substitutes Kernig’s and Brudzinski’s signs. The gi-
D. Necrotomy of burn surface, ven symptoms relate to:
haemotransfusion
E. Antibacterial and detoxicating therapy A. Meningeal signs
B. Discoordination syndrome
146. A 54-year-old female patient was C. Motor disorder syndrome
admitted to the hospital with evident D. Encephalic syndrome
acrocyanosis, swollen cervical veins, E. Infectious toxicosis signs
enlarged liver, ascites. Cardiac borders
are dilated. Heart sounds cannot be 150. A pediatrician had a conversation wi-
auscultated, apical beat is undetectable. th a mother of a 7-month-old breast-fed
AP is 100/50 mm Hg. X-ray picture of boy and found out that the child was fed
chest shows enlarged heart shadow in 7 times a day. How many times should the
form of a trapezium. What pathology mi- child of such age be fed?
ght have caused these symptoms?
A. 5 times
A. Cardiac tamponade B. 3 times
B. Exudative pleuritis C. 4 times
C. Complex heart defect D. 6 times
D. Acute cardiac insufficiency E. 7 times
E. Hiatal hernia
151. 1 day ago a 19-year old male patient
147. A 16-year-old female patient got a trauma after a fall from 1,5 m hei-
underwent an operation on account of ght when he "bestrode"a wooden beam.
Krok 2 Medicine 2010 21

Urination is extremely difficult, there is A. Subungual panaritium


macrohematuria, urethrorrhagia. There is B. Erysipelas
a hematoma in the perineum and on the C. Paronychia
scrotum. Urethrography revealed flowing D. Erysipeloid
of contrast medium into the membranous E. Abscess
part of urethra. What is the treatment
tactics of choice? 155. Statistic of patients of common medi-
cal practice displays constant increase in
A. Epicystostomy elderly and old people number. What ki-
B. Urine bladder catheterization for urine nd of pathology is expected to prevail in
diversion the morbidity structure of population in
C. Urethra bougienage question?
D. Cold to the perineum, hemostatic
therapy A. Chronic pathology
E. Primary suturing of urethra B. Occupational pathology
C. Non-epidemic pathology
152. A 40-year-old patient underwent an D. Acute pathology
operation for a lumbar phlegmon. Body E. Infectious pathology
temparature rose again up to 38o C, he
got intoxication symptoms, there was an 156. A 30-year-old patient consulted a
increase of leukocyte number in blood. doctor about menstruation absence for
The wound that was nearly free from 2 years after labour, loss of hair, body
necrotic tissues and full of granulations weight loss. The labour was compli-
started to discharge pus, the granulations cated by a haemorrhage caused by uterus
turned pale. What complication dveloped hypotonia. Objectively: the patient is
in this patient? asthenic, external genitals are hypoplastic,
the uterus body is small and painless. The
A. Sepsis appendages are not palpaple. What is the
B. Putrid phlegmon most likely diagnosis?
C. Erysipelas
D. Allergic reaction A. Sheehan’s syndrome
E. Erysipeloid B. Ovarian amenorrhea
C. Turner’s syndrome
153. A 40-year-old female patient has a hi- D. Exhausted overy syndrome
story of rheumatism. She complains about E. Galactorrhea-amenorrhea syndrome
acute pain in her left eye, especially at
night, vision impairment, photophobia, 157. A 25-year-old patient complains
lacrimation. The patient cannot suggest of burning and pain during urination,
any reasons for the disease. Objectively: purulent discharges from the urethra that
weak pericorneal injection, flattening of turned up 2 days ago. Objectively: the
iris relief, iris discoloration. What is the urethral labia are edematic, hyperemic,
most likely diagnosis? there are profuse purulent discharges
from the urethra. Provisional diagnosis:
A. Iridocyclitis recent acute gonorrheal urethritis. What
B. Iritis supplemental data of clinical examinati-
C. Keratitis ons will alow to specify the diagnosis?
D. Choroiditis
E. Acute attack of glaucoma A. Microscopy of the urethral discharges
B. Serological blood analysis
154. Three days ago a boy underwent C. Common urine analysis
removal of a foreign body from under a D. Coccal cells detection during discharge
nail plate. 2 days later he felt acute pulsati- microscopy
ng pain at the end of the nail bone which E. Yeast cells detection during discharge
was getting worse at pressing. Nail fold microscopy
became hyperemic, body temperature
rose up to 37, 5oC, there was a change in 158. A 28-year-old patient has been admi-
nail plate colour. What is the most likely tted to the gynecological department
diagnosis? three days after a casual coitus. She
complains about pain in her lower
abdomen and during urination, profuse
purulent discharges from the vagina,
body temperature rise up to 37, 8oC. The
patient was diagnosed with acute bi-
Krok 2 Medicine 2010 22

lateral adnexitis. Supplemental examinati- ns about episodes of unconsciousness,


on revealed: the 4th degree of purity of spontaneous syncopes that are quickly
the vaginal secretion, leukocytes within over after a change of body position. A
the whole visual field, diplococcal bacteria syncope can be accompanied byquickly
located both intra- and extracellularly. elapsing bradycardia. There are no other
What is the etiology of acute adnexitis in complications of gestation. What is the
this patient? most likely reason for such condition?
A. Gonorrheal A. Postcava compresseion by the gravid
B. Colibacterial uterus
C. Chlamydial B. Pressure rise in the veins of extremities
D. Trichomonadal C. Pressure fall in the veins of extremities
E. Staphylococcal D. Vegetative-vascular dystonia (cardial
type)
159. A boy is 8 year old. His physi- E. Psychosomatic disorders
cal development is compliant with his
age. The child has had cardiac murmur 163. A 49-year-old woman complains
since birth. Objectively: skin and visi- about headache, head and neck going hot,
ble mucous membranes are of normal increased perspiration, palpitation, arteri-
colour. AP- 100/70 mm Hg. Auscultation al pressure rise up to 170/100 mm Hg,
revealed systolo-diastolic murmur and di- irritability, insomnia, tearfulness, memory
astolic shock above the pulmonary artery. impairment, rare and scarce menses, body
ECG shows overload of the left heart. weight increase by 5 kg over the last half
Roentgenoscopy shows coarsening of the a year. What is the most likely diagnosis?
lung pattern, heart shadow of normal
form. What is the most likely diagnosis? A. Climacteric syndrome
B. Premenstrual syndrome
A. Atrioseptal defect C. Vegetative-vascular dystonia
B. Pulmonary artery stenosis D. Arterial hypertension
C. Aorta coarctation E. Postcastration syndrome
D. Fallot’s tetrad
E. Patent ductus arteriosus 164. The correlation between the servi-
ce record and eosinophil concentration in
160. A 45-year-old patient with uroli- blood was studied in workers at dyeing
thiasis had an attack of renal colic. shops of textile factories. What index will
What is the mechanism of the attack be the most informative for the analysis of
development? this data?
A. Disturbed urine outflow from the A. Correlation factor
kidney B. Student’s criterion
B. Increase in relative density of urine C. Standardized index
C. Ureteric twists D. Fitting criterion
D. Destruction of glomerules E. Sign index
E. Renal artery spasm
165. A 38-year-old patient complains
161. A 25-year-old woman came to a about inertness, subfebrile temperature,
maternity welfare clinic and complained enlargement of lymph nodes, nasal
about being unable to conceive within 3 haemorrhages, ostealgia. Objectively: the
years of regular sexual life. Examinati- patient’s skin and mucous membranes
on revealed weight gain, male pattern of are pale, palpation revealed enlarged
hair distribution on the pubis, excessi- painless lymph nodes; sternalgia; liver
ve pilosis of thighs. Ovaries were dense was enlarged by 2 cm, spleen - by 5
and enlarged, basal temperature was cm, painless. In blood: erythrocytes -
monophase. What is the most likely di- 2, 7 · 1012 /l, Hb- 84 g/l, leukocytes - 58 ·
agnosis? 109 /l, eosinophils - 1%, stab neutrophi-
A. Sclerocystosis of ovaries ls - 2%, segmented neutrophils - 12%,
B. Tubo-ovaritis lymphocytes - 83%, lymphoblasts - 2%,
C. Adrenogenital syndrome smudge cells; ESR- 57 mm/h. What is the
D. Premenstrual syndrome most likely diagnosis?
E. Gonadal dysgenesis
162. A 32-year-old gravida complai-
Krok 2 Medicine 2010 23

A. Chronic lymphatic leukemia A. Acute renal insufficiency


B. Chronic myeloleukemia B. Cardial shock
C. Acute lymphatic leukemia C. Allergic reaction to the donor
D. Acute myeloleukemia erythrocyte mass
E. Lymphogranulomatosis D. Citrate intoxication
E. Toxic infectious shock
166. A 58-year-old patient complains
about sensation of numbness, sudden 169. A 45-year-old man has been exhi-
paleness of II-IV fingers, muscle ri- biting high activity for the last 2 weeks,
gidness, intermittent pulse. The pati- he became talkative, euphoric, had li-
ent presents also with polyarthralgia, ttle sleep, claimed being able "to save
dysphagia, constipations. The patient’s the humanity"and solve the problem
face is masklike, solid edema of hands is of cancer and AIDS, gave money the
present. The heart is enlarged; auscultati- starangers. What is the most likely di-
on revealed dry rales in lungs. In blood: agnosis?
ESR - 20 mm/h, crude protein - 85/l, γ-
globulines - 25%. What is the most likely A. Maniacal onset
diagnosis? B. Panic disorder
C. Agitated depression
A. Systemic scleroderma D. Schizo-affective disorder
B. Dermatomyositis E. Catatonic excitation
C. Rheumatoid arthritis
D. Systemic lupus erythematosus 170. A 67-year-old female patient sufferi-
E. Raynaud’s disease ng from the essential hypertension
suddenly at night developed headache,
167. A 36-year-old female patient dyspnea that quickly progressed to
complains of general weakness, edemata asphyxia. Objectively: the patient is pale,
of her face and hands, rapid fatiguability with sweaty forehead, AP- 210/140 mm
during walking, difficult diglutition, cardi- Hg, heart rate - 120/min, auscultation
ac irregularities. These symptoms turned revealed solitary dry rales and moist rales
up 11 days after a holiday at the seasi- in the lower parts. The shins are pastose.
de. Objectively: face erythema, edema of What kind of emergency aid would be the
shin muscles. Heart sounds are muffled, most efficient in this case?
AP is 100/70 mm Hg. In blood: ASAT
activity is 0,95 millimole/h·l, ALAT - 1,3 A. Nitroglycerin and furosemide
millimole/h·l, aldolase - 9,2 IU/l, creatine intravenously
phosphokinase - 2,5 millimole Р/g·l. What B. Enalapril and furosemide intravenously
method of study would be the most speci- C. Digoxin and nitroglycerin intravenously
fic? D. Labetalol and furosemide intravenously
E. Nitroglycerin intravenously and capoten
A. Muscle biopsy internally
B. ECG
C. Echocardiogram 171. A patient suffering from gastroesophageal
D. Electromyography reflux has taken from time to time a
E. Determination of cortisol concentration certain drug that "reduces acidity"over
in blood and urine 5 years. This drug was recommended
by a pharmaceutist. The following side
168. A 33-year-old patient has acute blood effects are observed: osteoporosis, muscle
loss: erythrocytes - 2, 2 · 1012 /l, Hb- 55 g/l, asthenia, indisposition. What drug has
blood group is A(II)Rh+ . Accidentally such following effects?
the patient got a transfusion of donor
A. Aluminium-bearing antacid
erythrocyte mass of AB(IV )Rh+ group.
An hour later the patient became anxi- B. Inhibitor of proton pump
C. H2 -blocker
ous, got abdominal and lumbar pain.
Ps- 134 bpm, AP- 100/65 mm Hg, body D. Metoclopramide
E. Gastrozepin
temperature - 38, 6oC. After catheterizati-
on of urinary bladder 12 ml/h of dark- 172. A 26-year-old patient complains
brown urine were obtained. What compli- about considerable muscle weakness, di-
cation is it? zziness, extended abdominal pain, nausea
and vomiting giving no relief. The disease
has been gradually developing within 6
months. There was progress of general
Krok 2 Medicine 2010 24

weakness, skin darkening. The patient fell A. Osteogenic sarcoma


into grave condition after an ARD: there B. Hyperparathyroid dystrophy
appeared abdominal pain and frequent C. Chronic osteomyelitis
vomiting. Objectively: the skin is dry with D. Myelomatosis
diffuse pigmentation. Heart sounds are si- E. Marble-bone disease
gnificantly weakened, heart rate - 60/min,
AP- 80/40 mm Hg. The abdomen is sli- 176. A 10-year-old girl was admitted to a
ghtly painful in the epigastrial region. In hospital with carditis presentations. It is
blood: WBCs - 8, 1 · 109 /l, glucose - 3,0 known from the anamnesis that two weeks
millimole/l. What is the most likely di- ago she had exacerbation of chronic tonsi-
agnosis? llitis. What is the most likely etiological
factor in this case?
A. Chronic adrenal insufficiency. Addi-
sonian crisis A. Streptococcus
B. Acute pancreatitis B. Staphylococcus
C. Toxic infectious shock C. Pneumococcus
D. Acute food poisoning D. Klebsiella
E. Acute cholecystitis E. Proteus

173. A puerpera is 25 years old, it is her 177. A department chief of an in-patient


second day of postpartum period. It was hospital is going to inspect resident
her first labour, it took place at full term. doctors as to observation of medical-
The lochia should be: technological standards of patient service.
What documentation should be checked
A. Bloody for this purpose?
B. Sanguino-serous
C. Mucous A. Health cards of in-patients
D. Purulent B. Statistic cards of discharged patients
E. Serous C. Treatment sheets
D. Registry of operative interventions
174. A 32-year-old patient consulted a E. Annual report of a patient care instituti-
doctor about being inable to get pregnant on
for 5-6 years. 5 ago the primipregnancy
ended in artificial abortion. After the vagi- 178. Workers of a laboratory produci-
nal examination and USI the patient was ng measuring devices (manometers,
diagnosed with endometrioid cyst of the thermometers etc) complain about a
right ovary. What is the optimal treatment mettalic taste in mouth, stomatitis,
method? dyspepsia, sleep disturbance, unsteady
walk, abrupt decrease in cardiac activi-
A. Surgical laparoscopy ty. These presentations must have been
B. Anti-inflammatory therapy caused by the intoxication with the
C. Conservative therapy with estrogen- following substance:
gestagenic drugs
D. Hormonal therapy with androgenic A. Mercury
hormones B. Lead
E. Sanatorium-and-spa treatment C. Manganese
D. Toluol
175. A 20-year-old man complains about E. Tetraethyl lead
pain arising in the lower third of femoral
bone under stress and at rest. He deni- 179. A full-term infant has respiratory
es having a trauma. Objectively: the rate of 26/min, heart rate of 90/min, blue
skin is of normal colour, deep palpati- skin, muscle hypotonia. During catheter
on reveals pastosity and pain, knee joi- suction of mucus and amniotic fluid from
nt motions are limited. X-ray picture the nose and mouth the child reacted wi-
of the meta-epyphisis of distal femur th a grimace. Low reflexes. Auscultati-
shows a destruction zone and spicules. In on revealed weakened vesicular respirati-
blood: immature cell forms, no signs of on above lungs. Heart sounds are loud.
inflammation. What is the most likely di- After 5 minutes the respiration became
agnosis? rhythmic, at the rate of 38/min, heart rate
of 120/min. What is the most likely di-
agnosis?
Krok 2 Medicine 2010 25

A. Asphyxia
B. Inborn pneumonia A. Gram-negative diplococci
C. Birth trauma B. Gram-positive diplococci
D. Bronchopulmonary dysplasia C. Spirochaete
E. Respiratory distress syndrome D. Proteus vulgaris
E. Mycoplasma
180. Examination of placenta revealed a
defect. An obstetrician performed manual 184. As a result of load lifting a 68-year-
investigation of uterine cavity, uterine old female felt acute pain in the lumbar
massage. Prophylaxis of endometritis in region, in a buttock, posterolateral surface
the postpartum period should involve of her right thigh, external surface of
following actions: the right shin and dorsal surface of foot.
Objectively: weakness of the anterior ti-
A. Antibacterial therapy bial muscle, long extensor muscle of the
B. Instrumental revision of uterine cavity right toes, short extensor muscle of the ri-
C. Haemostatic therapy ght toes. Low Achilles reflex on the right.
D. Contracting agents Positive Lasegue’s sign. What examinati-
E. Intrauterine instillation of dioxine on method would be the most effective for
specification of the diagnosis of discogenic
181. A 35-year-old man complains about compression of L5 root?
intense lumbar pain irradiating to the
inguinal area, external genitalia, thigh; A. Magnetic resonance scan
frequent urination, chill, nausea, vomi- B. Spinal column X-ray
ting. Objectively: positive Pasternatsky’s C. Electromyography
symptom. Urine analysis revealed that D. Angiography
RBCs and WBCs covered the total fi- E. Lumbar puncture
eld of microscope; the urine exhibited hi-
gh protein concentration. These clinical 185. A hospital admitted a patient wi-
presentations were most likely caused by th coarse breathing (obstructed inspirati-
the following pathology: on), skin cyanosis, tachycardia and arteri-
al hypertension. He has a histrory of
A. Urolithiasis, renal colic bronchial asthma. An hour ago he was
B. Cholelithiasis, biliary colic having salbutamol inhalation and forgot
C. Renal infarct to remove a cap that was aspired whi-
D. Intestinal obstruction le taking a deep breath. What measures
E. Osteochondrosis, acute radicular should the doctor take?
syndrome
A. To perform the Heimlich manoever
182. Head circumference of a 1-month- B. To perform conicotomy immediately
old boy with signs of excitement is 37 cm, C. To send for an anesthesiologist and wait
prefontanel is 2x2 cm large. After feedi- for him
ng the child regurgitates small portions D. To use an inhalation of β2 -adrenoceptor
of milk; stool is normal in its volume and agonist
composition. Muscle tone is within norm. E. To make a subcutaneous injection of
What is the most likely diagnosis? dexamethasone
A. Pylorospasm 186. A 28-year-old patient was delivered
B. Meningitis to the admission ward in the unconscious
C. Pylorostenosis state with generalized epileptic attacks
D. Microcephaly taking place every 15-20 minutes. Duri-
E. Craniostenosis ng transportation the patient was given
183. A patient consulted a venereologi- two injections of diazepam, magnesia
sulphate, but they failed to bring the pati-
st about painful urination, reddening of
the external opening of urethra, profuse ent to consciousness. What department
purulent discharges from the urethra. He should render emergency aid?
considers himself to be ill for 3 days. A. Resuscitation department
He also associates the disease with a B. Neurological department
casual sexual contact that took place for C. Surgcal department
about a week ago. If provisional diagnosis D. Therapeutic department
"acute gonorrheal urethritis"is confirmed, E. Psychiatric department
then bacteriological study of urethral di-
scharges will reveal: 187. On the second day after preventi-
Krok 2 Medicine 2010 26

ve vaccination a 2-year-old boy presented A. Posttraumatic osteomyelitis


with abdominal pain without clear locali- B. Hematogenic osteomyelitis
zation, body temperature rose up to 38oC. C. Wound abscess
On the third day the child got red papular D. Posttraumatic phlegmon
haemorrhagic eruption on the extensor E. Suture sinus
surfaces of limbs and around the joints.
Knee joints were edematic and slightly 191. Estimation of community health level
painful. Examination of other organs and involved analysis of a report on diseases
systems revealed no pathological changes. registered among the population of distri-
What is the most likely diagnosis? ct under charge (reporting form 12). What
index is calculated on the grounds of this
A. Haemorrhagic vasculitis report?
B. Thrombocytopenic purpura
C. Meningococcemia A. Common morbidity rate
D. Urticaria B. Index of pathological affection
E. DIC syndrome C. Index of morbidity with temporary
disability
188. On the 6th day of life a child got D. Index of hospitalized morbidity
multiple vesicles filled with seropurulent E. Index of basic non-epidemic morbidity
fluid in the region of occiput, neck and
buttocks. General condition of the chi- 192. A 49-year-old patient undergoes
ld is normal. What disease should be regular medical check-up for uterine fi-
suspected? bromyoma. Within the last year the uterus
has enlarged up to 20 weeks of gestation.
A. Vesiculopustulosis What is the rational way of treatment?
B. Impetigo neonatorum
C. Miliaria A. Surgical treatment
D. Impetigo B. Hormonal therapy
E. Epidermolysis bullosa C. Further surveillance
D. Embolization of uterine arteries
189. A 60-year-old patient complains E. Treatment with prostaglandin inhibitors
about asphyxia, palpitation, rapid fati-
guability. He has 8 year history of essential 193. A patient is being prepared for the
hypertension. Objectively: the left cardiac operation on account of varix dilatation
border is 2 cm deviated to the left from of lower extremities veins. Examination
the medioclavicular line, heart sounds of the patient’s soles revealed flour-like
are rhythmic and weak; there is diastolic desquamation along the skin folds. All
shock above aorta. AP- 170/100 mm Hg. the toenails are greyish-yellow, thickened
Liver - +2 cm; shin pastosity is present. and partially decayed. What dermatosis
ECG shows deviation of cardiac axis to should be suspected?
the left, left ventricle hypertrophy. Ejecti-
on fraction - 63%. What type of cardiac A. Rubromycosis
insufficiency is observed? B. Pityriasis versicolor
C. Candidosis
A. Diastolic D. Microsporia
B. Systolic E. Microbial eczema
C. Combined
D. It’s a norm 194. A 46-year-old patient was issued a
E. Unspecified 10-day sick list because of exacerbation of
chronic cholecystitis. The patient’s general
190. A 3-year-old male patient consulted condition got better, but the clinical mani-
a family doctor 2 months after he had festations of the disease are still present.
been operated for an open fracture of What authority is entitled to extend the
brachial bone. Objectively: the patient’s sick list?
condition is satisfactory, in the region of
the operative wound there is a fistula A. Medical Expert Commission
with some purulent discharge, redness, B. Family doctor
fluctuation. X-ray picture shows brachi- C. Deputy head doctor for terapeutic
al bone destruction with sequestra. What management
complication arose in the postoperative D. Deputy head doctor for medical-labour
period? expertise
E. Head doctor
Krok 2 Medicine 2010 27

195. A 43-year-old female patient was 198. A 52-year-old patient works as a


delivered to the hospital in grave condi- secretary and has 30 year record of servi-
tion. She suffers from Addison’s di- ce. She complains of spasms in her ri-
sease. The patient had been regularly ght hand during working and inability
taking prednisolone but a week before to type and write. Up to 80% of her
she stopped taking this drug. Objecti- work involves hand load. The patient has
vely: sopor, skin and visible mucous been presenting with these symptoms for
membranes are pigmented, skin and 2 years. Objectively: the right hand is
muscle turgor is decreased. Heart sounds tense, there is an increase in muscle tone,
are muffled, rapid. AP- 60/40 mm Hg, attempts to write cause spasms. Exami-
heart rate - 96/min. In blood: Na - 120 mi- nation revealed no pathological changes
llimole/l, K - 5,8 millimole/l. Development of CNS. What is the most likely diagnosis?
of this complication is primarily caused by
the deficit of the following hormone: A. Spastic form of coordination neurosis
B. Neuralgic form of coordination neurosis
A. Cortisol C. Paretic form of coordination neurosis
B. Corticotropin (ACTH) D. Hysteric neurosis
C. Adrenaline E. Chronic manganese intoxication
D. Noradrenaline
E. Adrostendion 199. During examination at a military
commissariat a 15-year-old teenager was
196. Forensic medical expertise of corpse found to have interval sysolic murmur on
of a newborn revealed: body weight 3500 the cardiac apex, diastolic shock above
g, body length 50 cm, the umbilical cord the pulmonary artery, tachycardia. Which
was smooth, moist, glossy, without any si- of the suuplemental examination methods
gns of drying. Hydrostatic tests were posi- will be the most informative for the di-
tive. The test results are the evidence of: agnosis specification?
A. Live birth A. Echocardiography
B. Stillbirth B. Electrocardigraphy
C. Primary atelectasis C. Roengenography
D. Secondary atelectasis D. Phonocardiography
E. Hyaline membrane disease E. Rheography
197. A maternity house has admitted a 200. A 67-year-old female patient
primagravida complaining of irregular, complains about edemata of face and legs,
intense labour pains that have been lasti- pain in the lumbar area that is getting
ng for 36 hours. The woman is tired, failed worse at moving; great weakness, someti-
to fall asleep at night. The fetus is in longi- mes nasal haemorrhages, rise of body
tudinal lie, with cephalic presentation. temperature up to 38, 4oC. Objectively:
The fetus heartbeat is clear and rhythmic, painfulness of vertebral column and ribs
145/min. Vaginal examination revealed on palpation. Laboratorial study revealed
that the uterine cervix was up to 3 cm daily proteinuria of 4,2 g, ESR- 52 mm/h.
long, dense, with retroflexion; the external What changes of laboratory indices are to
orifice was closed; the discharges were of be expected?
mucous nature. What is the most likely di-
agnosis? A. Whole protein of blood serum - 101 g/l
B. Leukocytes - 15,3 g/l
A. Pathological preliminary period C. Haemoglobin - 165 g/l
B. Uterine cervix dystocia D. Albumins - 65%
C. Primary uterine inertia E. γ-globulins - 14%
D. Physiological preliminary period
E. Secondary uterine inertia
Krok 2 Medicine 2011 1

1. A patient suddenly felt an acute 5. A schizophrenic patient considers hi-


chest pain irradiating to the left arm. mself to be "an outstanding scientist,
Objectively: the patient is excited, with a brilliant composer and an unrivalled
pale skin. Breathing rate - 38/min, AP artist". He complains that "family and
- 180/110 mm Hg. Later the patient lost friends are always jealous of him and
consciousness and fell down. Pulse on the want to poison him". Determine the
great vessels was absent, the pupils were psychopathological syndrome:
equally dilated. What is the most likely di-
agnosis? A. Paranoiac
B. Paranoid
A. Clinical death C. Manic
B. Agonal state D. Paratrophic
C. Coma E. Hebephrenic
D. Heart attack
E. Disorder of the cerebral circulation 6. Examination of a 9-month-old girl
revealed skin pallor, cyanosis during exci-
2. A 62-year-old patient complaining tement. Percussion revealed transverse
of enlargement of cervical, supraclavi- dilatation of cardiac borders. Auscultati-
cular and axillary lymph nodes, subfebri- on revealed continuous systolic murmur
le temperature for the last 3 months has to the left of the breastbone in the
been admitted to a hospital. In blood: 3-4 intercostal space. This murmur is
WBCs - 64 · 109 /l, lymphocytes - 72%. conducted above the whole cardiac regi-
What method of study should be used to on to the back. What congenital cardiac
specify the diagnosis? pathology can be suspected?
A. Myelogram A. Defect of interventricular septum
B. Lymphography B. Defect of interatrial septum
C. Lymphoscintigraphy C. Coarctation of aorta
D. X-rays D. Fallot’s tetrad
E. Thermography E. Pulmonary artery stenosis
3. A child is 2 years old. The child 7. A 42-year-old patient complains of back
complains of hoarse voice, dyspnea with pain, darkened urine, general weakness,
obstructed inspiration. The disease started dizziness that occurred after treating a
3 days ago from dry cough and nose stuffi- cold with aspirin and ampicillin. Objecti-
ness. Objectively: general condition is vely: the patient is pale, with subicteric
unbalanced, stridor is present. The child’s sclerae. HR - 98 bpm. Liver - +2 cm,
skin is pale. Body temperature is 37, 7oC. spleen - +3 cm. In blood: RBCs - 2, 6 ·
The palatine arches are hyperemic. There 1012 /l, Hb - 60 g/l, CI - 0,9, WBCs -
is no deposit. Heart sounds are rhythmic. 9, 4 · 109 /l, basophils - 0,5%, eosinophils
Auscultation of lungs reveals rough - 3%, stab neutrophils - 6% segmented
breathing sounds, crepitation is absent. neutrophils - 58%, lymphocytes - 25%,
Parainfluenza virus has been detected in monocytes - 7%, ESR - 38 mm/hour, reti-
nasopharynx lavage. What is the most li- culocytes - 24%. Total bilirubin - 38 milli-
kely diagnosis? mole/l. What complication occurred in the
patient?
A. Acute laryngotracheitis
B. Epiglottitis A. Acquired hemolytic anemia
C. Foreign body B. Toxic hepatitis
D. Diphtheria C. Cholelithiasis
E. Laryngospasm D. Agranulocytosis
E. Paroxysmal nocturnal hemoglobinuria
4. A teacher of a secondary school was
diagnosed with pulmonary tuberculosis. 8. A hospital has admitted a 52-year-
What is the maximum duration of his old patient with disseminated pulmonary
medical certificate? tuberculosis, complaints of acute pain in
the right half of chest, that developed
A. Ten months after going upstairs to the 3rd floor; cough,
B. Five months dyspnea at rest, marked cyanosis. What ki-
C. Four months nd of complication should suspected first
D. Two months of all?
E. A month
Krok 2 Medicine 2011 2

A. Spontaneous pneumothorax A. Prednisolone


B. Cardiac failure B. Lasix
C. Pulmonary failure C. Adrenaline
D. Pleuritis D. Aminophylline
E. Acute myocardial infarction E. Analgine
9. A 44-year-old patient has been admi- 13. A 16-year-old patient who has a hi-
tted to a hospital with complaints of dull, story of intense bleedings from minor
aching pain in the left lumbar region, the cuts and sores needs to have the roots
admixture of pus in the urine. Examinati- of teeth extracted. Examination reveals
on revealed a grade II staghorn calculus an increase in volume of the right knee
on the left. What method of treatment is joint, limitation of its mobility. There are
indicated for this patient? no other changes. Blood analysis shows
an inclination to anaemia (Hb- 120 g/l).
A. Surgery Before the dental intervention it is requi-
B. Contact lithotripsy red to prevent the bleeding by means of:
C. Distance lithotripsy
D. Conservative therapy A. Cryoprecipitate
E. Ascending litholysis B. Epsilon-aminocapronic acid
C. Fibrinogen
10. On the 21 day after appearance of vesi- D. Dried blood plasma
culous chickenpox rash a 7-year-old chi- E. Calcium chloride
ld developed ataxia, nystagmus, intenti-
on tremor, muscle hypotonia. Liquor 14. A 3-year-old child has been di-
analysis shows a low-grade lymphocytic agnosed with type I diabetes mellitus,
pleocytosis, slightly increased protein rate. hyperosmolar coma. The laboratory confi-
What complication is it? rmed the diagnosis. Which laboratory fi-
ndings are characteristic for such conditi-
A. Encephalitis on?
B. Purulent meningitis
C. Pneumonitis A. High hyperglycemia without ketonemia
D. Acute nephritis B. Hyperglycemia and ketonemia
E. Postherpetic neuralgia C. Hyperglycemia and glucosuria
D. Hyperglycemia and ketonuria
11. A 38-year-old male patient has been E. Hyperglycemia and high indicators of
taking alcohol for 3 years. 3 days after acid-base balance
a regular drinking period he felt anxi-
ety and fear. It appeared to him that he 15. A 58-year-old female patient came
was surrounded by spiders and worms, to the antenatal clinic with complai-
pursued by some "condemnatory voi- nts of bloody light-red discharges from
ces". His behaviour became aggressive. the genital tracts. Menopause is 12
The patient demonstrated correct self- years. Gynaecological examination found
awareness but impairment of temporal externalia and vagina to have age involuti-
and spatial orientation. What is the most on; uterine cervix was unchanged, there
likely diagnosis? were scant bloody discharges from uteri-
ne cervix, uterus was of normal size;
A. Delirium alcoholicum uterine appendages were not palpable;
B. Alcoholic paranoia parametria were free. What is the most
C. Alcoholic hallucinosis likely diagnosis?
D. Alcoholic encephalopathy
E. Pathologic intoxication A. Uterine carcinoma
B. Atrophic colpitis
12. An 8-year-old boy suffering from C. Abnormalities of menstrual cycle with
haemophilia was undergoing transfusion climacteric character
of packed red cells. Suddenly he felt pain D. Cervical carcinoma
behind the breastbone and in the lumbar E. Granulosa cell tumor of ovary
area, dyspnea, cold sweat. Objectively:
pale skin, heart rate - 100/min, AP - 60/40 16. A 3-year-old child was playing in a
mm Hg; oliguria, brown urine. For the playpen when he suddenly developed
treatment of this complication the followi- paroxysmal cough and shortness of
ng drug should be administered: breath. Objectively: dry cough, mixed
dyspnea. Lung auscultation revealed
some wheezes. Breathing sounds on the
Krok 2 Medicine 2011 3

right are diminished. The child doesn’t A. Renal cancer


mix with other children. Immunization B. Polycystic renal disease
is age-appropriate. What pathological C. Renal amyloidosis
condition can be suspected? D. Urolithiasis
E. Chronic glomerulonephritis
A. Foreign body in the respiratory tracts
B. Pneumonia 20. A 47-year-old patient complains of
C. Acute respiratory viral infection insomnia, heaviness over his entire body,
D. Pertussis constantly depressed mood. He considers
E. Bronchial asthma himself good-for-nothing, inadequate.
Believes that he is a burden to his fami-
17. A 44-year-old patient complai- ly, wants to die. The patient is depressed,
ns about difficult urination, sensati- inactive, has a hypomimic face with
on of incomplete urinary bladder sorrowful expression. He speaks qui-
emptying. Sonographic examination of etly and monotonely,gives short answers.
the urinary bladder near the urethra What is the most likely diagnosis?
entrance revealed an oval well-defined
hyperechogenic formation 2x3 cm large A. Major depressive disorder
that was changing its position during the B. Atherosclerotic depression
examination. What conclusion can be C. Initial stage of Alzheimer’s disease
made? D. Late-onset schizophrenia
E. Neurotic depression
A. Concrement
B. Malignant tumour of the urinary 21. Bacterial analysis of air in a livi-
bladder ng space in winter period by means of
C. Urinary bladder polyp Krotov’s apparatus revealed that total
D. Prostate adenoma number of microorganisms in 1m3 of
E. Primary ureter tumour air was 7200. What is the permissible
number of microorganisms for the air to
18. A 10-year-old child has been folowed- be characterized as "pure"?
up for the dilated cardiomyopathy. The
child presents with dyspnea, cardialgia. A. Up to 4500
There are dense, nonmobile edemata B. Up to 2500
on the lower extremities and sacrum. C. Up to 3500
Ps- 120/min. The cardiac borders are D. Up to 5500
extended transversely. Heart sounds are E. Up to 7500
muffled, there is blowing systolic murmur
at the apex and over the xiphoid process. 22. A 48-year-old patient got a job-related
Liver is 3 cm enlarged, urine output is injury of a hypodermic varicose vein on
reduced. The blood total protein - 58.6 g/l. his shin that was accompanied by the
In urine: protein - 0,025 g/l, WBCs - 2-4 in intensive phleborrhagia. Choose the opti-
the field of vision, RBCs - 2-3 in the field mal variant of first aid:
of vision. What is the main mechanism of
edema syndrome development: A. Pressure bandage and limb strapping
B. Application of Esmarch’s tourniquet
A. Venous congestion of greater circulati- above the injury
on C. Application of Esmarch’s tourniquet
B. Venous congestion of lesser circulation beneath the injury
C. Peripheral circulation disorder D. Occlusion of femoral artery in a typical
D. Secondary nephropathy development place
E. Hypoproteinemia E. Maximal limb flexion in knee joint

19. A 69-year-old female patient complai- 23. A patient who had eaten mushrooms
ns of temperature rise up to 38, 3oC, in the morning was delivered to the
haematuria. ESR - 55 mm/h. Antibacterial infectious diseases hospital at night. The
therapy turned out to be ineffective. What disease development was rapid. The pati-
diagnosis might be suspected? ent presented with stomach pain, frequent
diarrhea, intractable vomiting, burning
thirst, headache and dizziness. He died on
the third day. What mushrooms are most
likely to have caused mycetismus?
Krok 2 Medicine 2011 4

A. Deadly amanita A. Physical


B. Morels B. Chemical
C. Fly agarics C. Biological
D. Sulfur-tufts D. Combined
E. Russules E. Physicochemical
24. A 28-year-old patient complains of 28. 2 weeks after recovering from angina a
periodic compressing heart pain. His 29-year-old patient noticed face edemata,
brother died at the age of 34 from a weakness, decreased work performance.
cardiac disease with similar symptoms. There was gradual progress of dyspnea,
Objectively: the patients skin is pale. edemata of the lower extremities, lumbar
Heart borders display no significant devi- spine. Objectively: pale skin, weakening
ations. Heart sounds are loud, there is a of the heart sounds, anasarca. AP- 160/100
systolic murmur above all the points with mm Hg. In urine: the relative density -
a peak above the aorta. Echocardioscopy 1021, protein - 5 g/l, erythrocytes - 20-30
reveals thickening of the interventricular in the field of vision, hyaline cylinders -
septum in the basal parts, reduction of left 4-6 in the field of vision. What is the most
ventricular cavity. What drug should be likely diagnosis?
administered in order to prevent the di-
sease progression? A. Acute glomerulonephritis
B. Essential hypertension
A. Metoprolol C. Acute pyelonephritis
B. Digoxin D. Infectious allergic myocarditis
C. Nitroglycerin E. Myxedema
D. Captopril
E. Furosemide 29. A 56-year-old scientist experiences
constricting retrosternal pain several ti-
25. A 10-year-old boy underwent mes a day while walking for 100-150 m.
treatment in cardiological department for The pain lasts for up to 10 minutes and can
rheumatism, I acute attack of rheumatic be relieved by nitroglycerine. Objectively:
fever, active phase, II degree. The pati- the patient is overweight, heart borders
ent was discharged in satisfactory condi- exhibit no abnormalities, heart sounds are
tion. Which drug should be chosen for rhythmic, Ps- 78 bpm, AP- 130/80 mm Hg.
prevention of rheumatism recurrence? ECG contains low amplitude of T wave in
V4−5 . What disease might be suspected?
A. Bicillinum-5
B. Bicillinum-1 A. Stable FC III stenocardia
C. Erythromycin B. Instable stenocardia
D. Ampicillin C. Stable FC I stenocardia
E. Oxacillin D. Stable FC II stenocardia
E. Stable FC IV stenocardia
26. Examination of a 13-year-old boy
reveals that his body length is 147 сm 30. In autumn a 25-year-old patient
(+2), body weight - 38 kg (+1,5), ci- developed stomach ache arising 1,5-2
rcumference of chest - 72 cm (+0,2). Esti- hours after having meals and at night.
mate the harmonicity of the child’s physi- He complains of pyrosis and constipation.
cal development: The pain is getting worse after consumi-
ng spicy, salty and sour food, it can be
A. Disharmonious relieved by means of soda and hot-water
B. Harmonious bag. The patient has been suffering from
C. Above the average this disease for a year. Objectively: furred
D. Sharply disharmonious moist tongue. Abdomen palpation reveals
E. Supernormal epigastrial pain on the right, resistance
of abdominal muscles in the same region.
27. A children’s health camp received a What is the most likely diagnosis?
party of tinned food. External examinati-
on of the tins revealed that they had deep A. Duodenal ulcer
dents, could be easily concaved when B. Chronic cholecystitis
pressed and wouldn’t immediately return C. Diaphragmatic hernia
to the initial state; rust was absent; the tins D. Stomach ulcer
were greased with inedible fat. Specify the E. Chronic pancreatitis
bloat type:
31. A full-term infant is 3 days old.
Krok 2 Medicine 2011 5

On the different parts of skin there are suffering from pain in the sacrum and
erythemas, erosive spots, cracks, areas of coxofemoral joints, painfulness and sti-
epidermis peeling. The infant has scalded ffness in the lumbar spine for a year. ESR
skin syndrome. Nikolsky’s symptom is - 56 mm/h. Roentgenography revealed
positive. General condition of the infant symptoms of bilateral sacroileitis. The
is grave. Anxiety, hyperesthesia, febrile patient is the carrier of HLA B27 anti-
temperature are evident. What is the most gen. What is the most likely diagnosis?
probable diagnosis?
A. Ankylosing spondylitis
A. Exfoliative dermatitis B. Coxarthrosis
B. Phlegmon of newborn C. Rheumatoid arthritis
C. Finger’s pseudofurunculosis D. Reiter’s disease
D. Impetigo neonatorum E. Spondylosis
E. Mycotic erythema
36. A 58-year-old female patient complai-
32. A 7-year-old boy has been managed ns about periodical headache, dizziness
for a month. Immediately after hospi- and ear noise. She has been suffering from
talization there were apparent edemata, diabetes mellitus for 15 years. Objecti-
proteinuria - 7,1 g/l, daily urine protein vely: heart sounds are rhythmic, heart rate
- 4,2 g. Biochemical blood test shows is 76/min, there is diastolic shock above
persistent hypoproteinemia (43,2 g/l), aorta, AP is 180/110 mm Hg. In urine:
hypercholesterolemia (9,2 millimole/l). OD- 1,014. Daily loss of protein with uri-
The patient is most likely have the followi- ne is 1,5 g. What drug should be chosen
ng type of glomerulonephritis: for treatment of arterial hypertension?
A. Nephrotic A. Ihibitor of angiotensin converting
B. Nephritic enzyme
C. Isolated urinary B. β-blocker
D. Hematuric C. Calcium channel antagonist
E. Combined D. Thiazide diuretic
E. α-blocker
33. A secundipara has regular birth activi-
ty. Three years ago she had cesarean secti- 37. A 62-year-old patient has been deli-
on for the reason of acute intrauterine vered to the surgical department with
hypoxia. During parodynia she complai- complaints of sudden pain in the umbilical
ns of extended pain in the area of region irradiating to the back and groin,
postsurgical scar. Objectively: fetus pulse which was accompanied by a syncope.
is rhythmic - 140 bpm. Vaginal examinati- Objectively: the patient is in grave condi-
on shows 5 cm cervical dilatation. Fetal tion, umbilical region is tender on palpati-
bladder is intact. What is the tactics of on, bowel sounds are diminished. AP
choice? drop is present. USI reveals: free fluid in
the abdomen, thickening of the wall of
A. Cesarean section the abdominal aorta. The most likely di-
B. Augmentation of labour agnosis is:
C. Obstetrical forceps
D. Waiting tactics of labor management A. Rupture of abdominal aortic aneurism
E. Vaginal delivery B. Stomach ulcer perforation
C. Acute pancreatitis
34. A 54-year-old female patient D. Peritonitis
consulted a doctor about bloody di- E. Acute appendicitis
scharges from the genital tracts after
2 years of amenorrhea. USI and bi- 38. A therapeutist needs to analyze adult
manual examination revealed no genital health in the area of service. Which groups
pathology. What is the tactics of choice? of indicators will be included into this
analysis?
A. Fractional biopsy of lining of uterus and
uterine mucous membranes
B. Styptic drugs
C. Contracting drugs
D. Estrogenic haemostasia
E. Hysterectomy
35. A 32-year-old male patient has been
Krok 2 Medicine 2011 6

A. Demographic, sickness rates, disability A. 3,0 g/kg


B. Demographic, sickness rates, physical B. 2,0 g/kg
development C. 2,5 g/kg
C. Sickness rates, disability, death rates D. 3,5 g/kg
D. Birth rates, sickness rates, disability E. 4,0 g/kg
E. Sickness rates, death rates, physical
development 43. A 60-year-old female patient was
admitted to a hospital for acute
39. A 54-year-old patient has an over transmural infarction. An hour later
20-year history of femoral osteomyelitis. the patient’s contition got worse. She
Over the last month she has developed developed progressing dyspnea, dry
progressing edemata of the lower extremi- cough. Respiratory rate - 30/min, heart
ties. Urine test reveals: proteinuria at the rate - 130/min, AP- 90/60 mm Hg. Heart
rate of 6,6 g/l; in blood: dysproteinemia in sounds were muffled, there was also di-
form of hypoalbuminemia, increase in α2 - astolic shock on the pulmonary artery.
and γ-globulin rate, ESR - 50 mm/h. What The patient presented with medium moi-
is the most likely diagnosis? st rales in the lower parts of lungs on the
right and on the left. Body temperature -
A. Secondary renal amyloidosis 36, 4oC. What drug should be given in the
B. Acute glomerulonephritis first place?
C. Myelomatosis
D. Chronic glomerulonephritis A. Promedol
E. Systemic lupus erythematosus B. Aminophylline
C. Dopamine
40. An emergency team deliverd a 83- D. Heparin
year-old patient complaining of inability E. Digoxin
of her right leg to support the body after
falling on her right side. Objectively: the 44. A 62-year-old male has been hospi-
patient lies on a gurney, her right leg is talized in the intensive care unit with
rotated outwards, the outside edge of foot a continuous attack of retrosternal pain
touches the bed. There is positive straight that cannot be relieved by nitroglycerin.
leg raising sign. What is your provisional Objectively: AP- 80/60 mm Hg, heart rate
diagnosis? - 106/min, breathing rate - 22/min. Heart
sounds are muffled, a gallop rhythm is
A. Femoral neck fracture present. How would you explain the AP
B. Femoral diaphysis fracture drop?
C. Hip dislocation
D. Hip joint contusion A. Reduction in cardiac output
E. Cotyloid cavity fracture B. Reduction in peripheral resistance
C. Blood depositing in the abdominal
41. 3 days ago a 29-year-old patient cavity
presented with pulsating pain in the regi- D. Adrenergic receptor block
on of rectum, fever, general weakness. E. Internal haemorrhage
Objectively: local tenderness in the anal
region in the 6 o’clock position. Digital 45. A patient is on the sick leave for 4
investigation of rectum revealed a pai- months continuously from the date of
nful infiltration reaching the pectinate li- injury. The treatment is going to last for
ne. What is the most likely diagnosis? 1-2 months. Who has the right to extend
the duration of medical certificate for this
A. Acute periproctitis patient?
B. Acute anal fissure
C. Acute haemorrhoids A. Medical advisory commission after
D. Rectum tumour medico-social expert commission exami-
E. Acute prostatitis nation
B. Medical superintendent
42. A child is 7 months old. Birth wei- C. Medical advisory commission after
ght was 3450, the child is breastfed. inpatient treatment
Supplemental feeding was introduced on D. District doctor by agreement with a
time. Determine the daily protein requi- department chief
rements for the child: E. Medico-social expert commission
46. A child is 9 months old. The patient’s
body temperature is 36, 7oC, the skin is
Krok 2 Medicine 2011 7

pale, humid, there is pain in leg muscles. had to undergo a yearly preventive exami-
There is no extremities mobility, sensitivi- nation. A territorial polyclinic carried out
ty is present. The child has been diagnosed preventive examination of 325 workers.
with poliomyelitis. The causative agent of As a result of it, 1 worker was recogni-
this disease relates to the following family: zed as temporarily disabled, 15 workers
underwent further rehabilitation at an
A. Picornavirus after-work sanatorium, 10 workers were
B. Paramyxovirus provided with diet meal. What index
C. Tohovirus characterizing the preventive work of the
D. Adenovirus polyclinic should be applied in this case?
E. Rotavirus
A. Coverage of preventive medical exami-
47. Working conditions of a buildi- nations
ng company worker are characterized B. Frequency of case detection during
by cooling microclimate effect, silica- examinations
containing dust, caustic alkali (quicklime) C. Percentage of people who underwent
and noise. What medical expert should be rehabilitation at an after-work sanatorium
the chief of the commission that periodi- D. Percentage of people who were provi-
cally examines the workers of the menti- ded with diet meal
oned category? E. Percentage of temporarily disabled
people
A. Therapeutist
B. Ophthalmologist 51. A 14-year-old girl has been presenti-
C. Neurologist ng with irritability and tearfulness for
D. Dermatologist about a year. A year ago she was also
E. Otolaryngologist found to have diffuse enlargement of the
thyroid gland (II grade). This condition
48. A full-term baby (the 1st uncompli- was regarded as a pubertal manifestati-
cated pregnancy, difficult labour) had on, the girl didn’t undergo any treatment.
a cephalogematoma. On the 2nd day The girl’s irritability gradually gave place
there was jaundice, on the third the to a complete apathy. The girl got puffy
following changes in neurological status face, soft tissues pastosity, bradycardia,
appeared: nystagmus, Graefe syndrome. constipations. Skin pallor and gland densi-
Urine was yellow, feces were of golden- ty progressed, the skin became of a waxen
yellow colour. Mother’s blood group is A hue. What disease may be suspected?
(II) Rh− , the baby’s one - A (II) Rh+ . On
the third day the child’s Hb was 200g/l, A. Autoimmune thyroiditis
RBCs - 6, 1 · 1012 /l, blood bilirubin - 58 B. Diffuse toxic goiter
micromole/l at the expense of unbound C. Thyroid carcinoma
fraction. What caused the jaundice in the D. Subacute thyroiditis
child? E. Juvenile basophilism
A. Craniocerebral birth trauma 52. 4 hours after having meals a patient
B. Physiological jaundice with signs of malnutrition and steatorrhea
C. Neonatal anaemia experiences stomach pain, especially
D. Biliary atresia above navel and to the left of it. Diarrheas
E. Fetal hepatitis take turns with constipation lasting up to
3-5 days. Palpation reveals moderate pai-
49. After birth a child was pale and had nfulness in the choledochopancreatic regi-
arrhythmical breathing. Oxygen therapy on. The amylase rate in blood is stable.
didn’t have any effect. Pulse was weak X-ray reveals some calcifications located
and rapid. It was difficult to measure above navel. What is the most likely di-
arterial pressure accurately. There were no agnosis?
edemata. What is the most likely reason
for these symptoms? A. Chronic pancreatitis
B. Chronic gastroduodenitis
A. Asphyxia C. Duodenal ulcer
B. Congestive heart failure D. Zollinger-Ellison syndrome
C. Intracranial haematoma E. Chronic calculous cholecystitis
D. Intrauterine sepsis
E. Congenital pneumonia 53. A 56-year-old patient with diffuse
toxic goiter has ciliary arrhythmia with
50. 350 workers of a metalurgical plant
Krok 2 Medicine 2011 8

pulse rate of 110 bpm, arterial hypertensi- afternoon. Pain in the lumbar region,
on, AP- 165/90 mm Hg. What preparati- said about 10 years old. Objectively: pale
on should be administered along with skin, to - 37, 2oC, AP- 180/100 mm Hg,
mercazolil? minor costovertebral angle tenderness
(Pasternatsky symptom). In blood: RBCs
A. Propranolol - 3, 5 · 1012 /l, WBCs - 6, 5 · 109 /l, ESR -
B. Radioactive iodine 22 mm/h. In urine: the relative density -
C. Procaine hydrochloride 1010, leukocytes - 12-15 in the field of visi-
D. Verapamil on, erythrocytes - 2-3 in the field of vision.
E. Corinfar Urine bacterial count - 100000 in 1 ml.
54. On the 3rd day of life a baby What is the most likely diagnosis?
presented with haemorrhagic rash, A. Chronic pyelonephritis
bloody vomit, black stool. Examination B. Nephrolithiasis
revealed anaemia, extended coagulati- C. Polycystic renal disease
on time, hypoprothrombinemia, normal D. Chronic glomerulonephritis
thrombocyte rate. What is the optimal E. Amyloidosis
therapeutic tactics?
58. A 49-year-old male patient complains
A. Vitamin K of dyspnea of combined nature, cough,
B. Sodium ethamsylate shin edemata, abdomen enlargement due
C. Epsilon-aminocapronic acid to ascites. He has a 20-year history of
D. Fibrinogen chronic bronchitis. For the last 3 years
E. Calcium gluconate
he has been disabled (group II) because
55. A woman consulted a doctor on of cardiac changes. Objectively: mixed
the 14th day after labour about sudden cyanosis, edemata. Ps - 92/min, rhythmic,
pain, hyperemy and induration of the left AP - 120/70 mm Hg, respiration rate
mammary gland, body temperature ri- - 24/min. There is accentuation of the
se up to 39o C, headache, indisposition. second sound above the pulmonary artery.
Objectively: fissure of nipple, enlargement Auscultation reveals the box resonance
of the left mammary gland, pain on above the lungs. There are also dry
palpation. What pathology would you thi- rales over the entire surface of lungs.
nk about in this case? What is the mechanism of heart changes
development in this patient?
A. Lactational mastitis
B. Lacteal cyst with suppuration A. Euler-Liljestrand reflex
C. Fibrous adenoma of the left mammary B. Kitaev’s reflex
gland C. Bainbridge reflex
D. Breast cancer D. Cardiovascular reflex
E. Phlegmon of mammary gland E. Respiratory reflex

56. On the 5th day of a respiratory di- 59. A 43-year-old female patient
sease accompanied by fever a 24-year-old complains of unstable defecation with
man developed progressing headaches, frequent constipations, abdominal swelli-
systemic dizziness, double vision, faci- ng, headache, sleep disturbance. Body
al muscles paresis on the right, choking weight is unchanged. What disease are
from swallowing. He was diagnosed with these clinical presentations typical for?
acute viral encephalitis. Identify the main
tendency of the emergency treatment: A. Irritable colon syndrome
B. Chronic enteritis
A. Zovirax C. Chronic pancreatitis
B. Glucocorticoids D. Chronic atrophic gastritis
C. Ceftriaxone E. Colorectal cancer
D. Lasix
60. A 43-year-old man who often contacts
E. Haemodezum
with ethyl gasoline was admitted to
57. A 54-year-old male patient complai- a hospital with complaints of general
ns of aching pain in the lumbar regi- weakness, dizziness, memory impairment,
on, that is getting worse after standi- sleepiness at daytime and insomnia at ni-
ng in an upright position, physical ght, sensation of a hair in the mouth, coli-
exercise, supercooling. The patient also cky pains in the right subcostal region.
reports of experiencing weakness in the What is the most likely diagnosis?
Krok 2 Medicine 2011 9

A. Methotrexate
A. Chronic tetraethyl lead intoxication B. Chloroquine
B. Alcoholic delirium C. Prednisolone
C. Chronic mercury intoxication D. Diclofenac sodium
D. Chronic manganese intoxication E. Aspirin
E. Chronic lead intoxication
65. A 3-year-old child has been deli-
61. A 35-year-old patient has been in the vered to a hospital in soporose state wi-
intensive care unit for acute renal failure th considerable amyotonia, inhibition of
due to crush for 4 days. Objectively: the tendon and periosteal reflexes. Miosis and
patient is inadequate. Breathing rate - asthenocoria are also present. Corneal
32/min. Over the last 3 hours individual reflexes are preserved. Pulse is rapid and
moist rales can be auscultated in lungs. weak. AP- 80/50 mm Hg. The parents
ECG shows high T waves, right ventri- suspect the child of accidental taking
cular extrasystoles. CVP - 159 mm Hg. some tablets. Such clinical presentati-
In blood: the residual nitrogen - 62 mi- ons are typical for intoxication with the
llimole/l, K + - 7,1 millimole/l, Cl− - 78 mi- following tableted drugs:
llimole/l, Na+ - 120 millimole/l, Ht - 0,32,
Hb - 100 g/l, blood creatinine - 0,9 milli- A. Tranquilizers
mole/l. The most appropriate method of B. Antropine drugs
treatment would be: C. Antihypertensive drugs
D. Barbiturates
A. Hemodialysis E. Beta-2-adrenoceptor agonists
B. Plasma sorption
C. Hemosorption 66. A 23-year-old patient complains of a
D. Plasma filtration dull ache, sensation of heaviness and di-
E. Ultrafiltration stention in the epigastrium immediately
after meals, foul-smelling eructation; dry
62. A 47-year-old woman underwent a mouth, empty stomach nausea, diarrhea.
thyroid gland resection on ccount of Objectively: the skin is pale, the patient is
nodular euthyroid goiter. What preparati- of thin build. Abdomen is soft on palpati-
ons are most likely to prevent the disease on, there is epigastric pain. The liver does
recurrence? not extend beyond the costal arch. In
blood: Hb - 110 g/l, RBCs - 3, 4 · 1012 /l,
A. Thyroid hormones WBC count is normal. ESR - 16 mm/h.
B. Mercazolil What is the most informative study that
C. Thyrotropin will allow make a diagnosis?
D. Antistruminum (potassium iodide)
E. Radioactive iodine A. Esophageal gastroduodenoscopy
B. X-ray of digestion organs
63. A 30-year-old man was injured in a fire C. Study of gastric juice
and got thermic burns of III-A and III-B D. pH-metry
degree that affected 20% of the total skin. E. Duodenal probing
AP - 110/70 mm Hg, heart rate -120/min.
What transfusion means should be used 67. A 49-year-old patient complains of
for blind infusion before transportation? deglutition problems, especially with solid
food, hiccups, voice hoarseness, nausea,
A. Saline solutions regurgitation, significant weight loss (15
B. Polyglycine kg within 2,5 months). Objectively: body
C. 10% glucose solution weight is reduced. Skin is pale and
D. Fresh frozen plasma dry. In lungs: vesicular breathing, heart
E. Albumin sounds are loud enough, heart activity is
64. A patient has an over a year-old rhythmic. The abdomen is soft, painless
history of fast progressive rheumatoid on palpation. Liver is not enlarged. What
arthritis. X-raying confirms presence of study is required to make a diagnosis?
marginal erosions. What basic drug would A. Esophageal duodenoscopy along with
be the most appropriate in this case? biopsy
B. Clinical blood test
C. X-ray of digestive tract organs
D. X-ray in Trendelenburg’s position
E. Study of gastric secretion
Krok 2 Medicine 2011 10

68. A 60-year-old patient has been story of chronic glomerulonephritis with


admitted to a hospital with complai- latent course. Over the past six months
nts of dyspnea, tightness in the right she has developed general weakness,
subcostal area, abdomen enlargement. loss of appetite, low work performance,
These presentations have been progressi- nausea. The patient complains of
ng for a year. Heart auscultation reveals headache, pain in the joints. On exami-
presystolic gallop rhythm. Objectively: nation: anemia, blood urea - 34,5 mi-
swelling of the neck veins, ascites, llimole/l, blood creatinine - 0,766 milli-
palpable liver and spleen. What disease mole/l, hyperkalemia. What complication
requires differential diagnostics? has developed?
A. Constrictive pericarditis A. Chronic renal insufficiency
B. Hepatocirrhosis B. Acute renal insufficiency
C. Lung cancer with invasion to the pleura C. Nephrotic syndrome
D. Chronic pulmonary heart D. Renal amyloidosis
E. Pulmonary embolism E. Pyelonephritis
69. A 40-year-old patient, the forester, 72. A child is 1 year old. Ater the recent
complains of severe headache, body introduction of complementary feedi-
temperature rise up to 39, 5oC, trembling ng the child has presented with loss of
limbs. From the patient’s history we know appetite, diarrhea with large amounts
that he had seriously cut his hand duri- of feces and occasional vomiting, body
ng the dissection of a killed fox. Objecti- temperature is normal. Objectively: body
vely: depressed mood. The patient asks weight is 7 kg, the child is very pale, there
not to turn on the light or open the door. are edemata of both legs, abdomen is si-
Any noise causes apparent motor exci- gnificantly enlarged. Coprogram shows
tation. When he saw a carafe of water, many fatty acids and soaps. The child has
he developed convulsive throat spasms. been diagnosed with celiac disease and
What tactics should an emergency doctor administered the gluten-free diet. What is
choose? to be excluded from the ration?
A. Deliver the patient to the infectious A. Cereals - wheat and oats
disease hospital B. Milk and dairy products
B. Deliver the patient to the resuscitation C. Fruit
department D. Animal protein
C. Deliver the patient to the neurological E. High digestible carbohydrates
department
D. Deliver the patient to the psychiatric 73. Development of chronic venous
hospital insufficiency of lower extremities depends
E. Let him stay at home and consult a on the functional condition of so-called
psychiatrist musculovenous pump. This term refers to
the following group of muscles:
70. A 5-year-old child developed an acute
disease starting from body temperature A. Shin muscles
rise up to 38, 5o C, running nose, cough B. Abdominal wall muscles
and conjunctivitis. On the 4th day the chi- C. Buttock region muscles
ld presented with maculo-papular rash D. Thigh muscles
on face. Body temparature rose again E. Foot muscles
up to 39, 2oC. Over the next few days
the rash spread over the whole body and 74. After treating a field with pestici-
extremities. Mucous membrane of palate des a machine operator presents wi-
was hyperemic, there was whitish deposi- th great weakness, headache, nausea,
tion on cheek mucous membrane next vomiting, diarrhea, visual impairment,
to molars. What is your provisional di- watery eyes. Objectively: the patient is
agnosis? excited, hypersalivation, hyperhidrosis,
muscle fibrillation of tongue and eyelids
A. Measles are oberved. Pupils are narrowed, there
B. Acute viral respiratory infection is tachycardia, lung auscultation reveals
C. Yersinia moist small and medium bubbling rales.
D. Enterovirus diseases In blood: changed level of cholinesterase
E. Rubella activity. What is the most likely diagnosis?
71. A 28-year-old woman has a 12-year hi-
Krok 2 Medicine 2011 11

A. Intoxication with organophosphorous A. Reiter’s syndrome


pesticides B. Rheumatic arthritis
B. Intoxication with organochlorine pesti- C. Gout
cides D. Bechterew’s disease
C. Intoxication with organomercurial E. Rheumatoid arthritis
pesticides
D. Intoxication with arsenic-containing 78. Topographic percussion of lungs in
pesticides a patient who got a serious job-related
E. Intoxication with carbamic acid deri- barotrauma revealed that the lower lungs
vatives borders were located one rib below
normal, there was a significant increase
75. A 7-year-old child was brought to in both lungs height and Kronig’s isthmus.
a doctor for a check. The child has What disease should be suspected in the
a 4-year history of bronchial asthma, first place?
asthma attacks occur mainly in spri-
ng and summer. Allergy tests revealed A. Pulmonary emphysema
hypersensitivity to poplar seed tufts, fi- B. Exudative pleuritis
eld herbs. What recommendation should C. Chronic bronchitis
be given? D. Bronchial asthma
E. Pneumothorax
A. Specific hyposensitization
B. Physiotherapy 79. A 32-year-old patient lives in an area
C. Treatment at a health resort endemic for echinococcosis. In the last
D. Phytotherapy 6 months he reports of pain in the ri-
E. Needle reflexotherapy ght subcostal area, fever. He is suspected
to have liver echinococcosis. What study
76. A farmer hurt his right foot duri- would be the most informative in this
ng working in a field and came to the case?
emergency station. He doesn’t remember
when he got last vaccination and he has A. USI
never served in the army. Examination B. Survey radiography of abdominal cavity
of his right foot revealed a contaminated C. Biochemical laboratory examination
wound up to 5-6 cm long with uneven D. Angiography
edges. The further treatment tactics will E. Liver scanning
be:
80. A 30-year-old gravida consulted a
A. To make an injection of tetanus gynecologist about bright red bloody di-
anatoxin and antitetanus serum scharges from the vagina in the 32 week
B. To make an injection of tetanus of gestation. She was hospitalized wi-
anatoxin th a suspicion of placental presentation.
C. To make an injection of antitetanus Under what conditions is it rational to
serum conduct the internal examination in order
D. Surgical d-bridement only to make a diagnosis?
E. To administer an antibiotic
A. In the operating room prepared for the
77. A 35-year-old patient has been admi- operation
tted to a hospital for pain in the left B. In the examination room of antenatal
sternoclavicular and knee joints, lumbar clinic
area. The disease has an acute character C. In the admission ward of maternity
and is accompanied by fever up to 38oC. hospital
Objectively: the left sternoclavicular and D. In the delivery room keeping to all the
knee joints are swollen and painful. In aseptics regulations
blood: WBCs - 9, 5 · 109 /l, ESR - 40 mm/h, E. The examination is not to be conducted
CRP - 1,5 millimole/l, fibrinogen - 4,8 because of risk of profuse haemorrhage
g/l, uric acid - 0,28 millimole/l. Exami- 81. After a serious nervous stress a 35-
nation of the urethra scrapings reveals year-old patient has developed on the
chlamydia. What is the most likely di- dorsal surface of hands redness and
agnosis? swelling that were later replaced by
small inflammatory nodules, vesicles and
following erosion with a significant serous
discharge. The process is accompanied by
severe itching. What is the most likely di-
Krok 2 Medicine 2011 12

agnosis? 85. A 32-year-old patient has a 3-year hi-


story of asthma attacks, that can be hardly
A. True eczema stopped with berotec. Over a few last
B. Allergic dermatitis months he has experienced pain in the
C. Microbal eczema joints and sensitivity disorder of legs and
D. Simple contact dermatitis feet skin. Ps - 80/min, AP - 210/100 mm
E. Toxicoderma Hg. In blood: eosinophilia at the rate of
15%. What disease can be suspected in
82. A 36-year-old patient complains of this case?
skin rash that appeared a week ago and
doesn’t cause any subjective problems. A. Periarteritis nodosa
Objectively: palm and sole skin is covered B. Systemic lupus erythematosus
with multiple lenticular disseminated C. Systemic scleroderma
papules not raised above the skin level. D. Dermatomyositis
The papules are reddish, dense on palpati- E. Wegener’s disease
on and covered with keratinous squamae.
What is the provisional diagnosis? 86. A 46-year-old patient complains of
sudden palpitation, which is accompani-
A. Secondary syphilis ed by pulsation in the neck and head, fear,
B. Verrucosis nausea. The palpitation lasts for 15-20 mi-
C. Palmoplanar psoriasis nutes and is over after straining when
D. Palmoplanar rubrophytosis holding her breath. What kind of cardi-
E. Palm and sole callosity ac disorder may be suspected?
83. A 30-year-old patient complains of A. An attack of supraventricular
paroxysmal abdominal pain, frequent paroxysmal tachycardia
liquid stools up to 10 times a day. B. An attack of ventricular paroxysmal
Throughout the first 3 days he had tachycardia
a fever, since the 2nd day of disease C. An attack of atrial flutter
there were scant liquid stools mixed wi- D. An attack of ciliary arrhythmia
th mucus. On palpation: tenderness of E. An attack of extrasystolic arrhythmia
all colon segments. Sigmoid colon was
found spastic. What is your provisional 87. The patient complains of a painful
diagnosis? swelling in the chin region, malaise,
headache. Examination reveals an acutely
A. Acute dysentery inflamed cone-shaped dense node. The
B. Intestinal amebiasis skin over it is tense, red. In the center of
C. Salmonellosis the node there is an ulcer with overhangi-
D. Cholera ng edges and a necrotic core of a dirty-
E. Balantidiasis green colour. Submandibular lymph nodes
on the right are enlarged and painful.
84. A 38-year-old woman experiences epi- What is the most likely diagnosis?
sodic increases in arterial pressure up to
240/120 mm Hg, which is accompanied by A. Furuncle
nausea, vomiting, tachycardia, increased B. Tuberculosis
sweating, hyperglycemia. The attack is C. Carbuncle
usually followed by the excessive urinati- D. Tertiary syphilis (gummatous form)
on. Renal sonography reveals an additi- E. Parasitic sycosis
onal formation adjacent to the upper pole
of the right kidney and possibly belonging 88. A welder at work got the first-degree
to the adrenal gland. What laboratory test burns of the middle third of his right
will allow to clarify the diagnosis? shin. 5 days later the skin around the
burn became edematic and itchy. Objecti-
A. Determination of urinary excretion of vely: on a background of a well-defined
catecholamines and vanillylmandelic acid erythema there is polymorphic rash in
B. Blood test for insulin and C-peptide form of papules, vesicles, pustules, erosi-
C. Estimation of glomerular filtration ons with serous discharge. What is the
rate by measuring endogenous creatinine most likely diagnosis?
clearance
D. Blood test for thyroxine and
thyrotrophic hormone
E. Blood test for renin level
Krok 2 Medicine 2011 13

A. Microbal eczema some painless nodular elements tending


B. True eczema to peripheral growth and fusion. He has a
C. Toxicoderma 2-year history of this disease. Aggravation
D. Occupational eczema takes place mainly in spring. In anamnesis:
E. Streptococcal impetigo the patient’s father had similar skin lesi-
ons. Objectively: pathological elements
89. A 58-year-old patient has a 3-year hi- looke like guttate and nummular nodules,
story diabetes mellitus type II. He has plaques covered with white scales. What
been keeping to a diet and regularly taki- is your provisional diagnosis?
ng glyburide. He has been delivered to
a hospital on an emergency basis for A. Psoriasis
acute abdomen. Objectively: the patient B. Lichen ruber planus
is of supernutrition type. The skin is dry. C. Neurodermitis
In the lungs vesicular breathing can be D. Pityriasis rosea
auscultated. Heart sounds are regular, E. Seborrheic eczema
90/min. AP- 130/70 mm Hg. The symptom
of "wooden belly"is visible. Blood sugar 93. A 47-year-old patient came to see a
- 9,8 millimole/l. The patients has indi- doctor on the 7th day of disease. The di-
cation for laparotomy. What is the most sease developed very fast: after the chill
appropriate way of further treatment of body temperature rose to 40oC and lasted
diabetes? up to 7 hours, then dropped abruptly,
which caused profuse sweat. There were
A. To administer short insulin three such attacks occuringonce in two
B. To continue taking glyburide days. Two days ago the patient arrived
C. To administer Semilong to be taken in from Africa. Objectively: pale skin, subi-
the morning and insulin - in the evening cteric sclera, significantly enlarged liver
D. To administer 1 tablet of Glurenorm and spleen. What is the cause of fever
three times a day attacks in this disease?
E. To administer 1 tablet of Maninil three
times a day A. Erythrocytic schizogony
B. Tissue schizogony
90. An 8-year-old girl has been admitted C. Exotoxin of a causative agent
to the cardiology department. Objecti- D. Endotoxin of a causative agent
vely: there is a skin lesion over the E. Gametocytes
extensor surfaces of joints with atrophic
cicatrices, depigmentation, symmetrical 94. On the 2 nd day of illness a 27-year-
affection of skeletal muscles (weakness, old patient complains of the unbearable
edema, hypotrophy). What disease are headache, repeated vomiting. Objecti-
these changes most typical for? vely: the patient is in a grave condition.
He is conscious but adynamic. Lies in a
A. Dermatomyositis forced position with his head thrown back.
B. Systemic scleroderma There is no skin rash. Occipital muscles
C. Nodular periarteritis are evidently rigid, there are Kernig’s
D. Systemic lupus erythematosus and Brudzinski’s signs. to - 39, 5o C, Ps-
E. Reiter’s disease 120/min., AP- 130/80 mm Hg. What is the
reason for the leading syndrome of this
91. A 43-year-old female patiet complai- disease?
ns of eruption on her right leg skin, pain,
weakness, body temperature rise up to A. Liquor hypertension
38o C. The disease is acute. Objectively: B. Liquor hypotension
there is an edema on the right leg skin in C. Affection of the cranial nerve nuclei
the region of foot, a well-defined bright D. Haemorrhages into the adrenal glands
red spot in form of flame tips which feels E. Hyperthermy
hot. There are isolated vesicles in focus.
What is your provisional diagnosis? 95. On the 1st day of life a full-term girl
(2nd labour) weighing 3500g, with Apgar
A. Erysipelas score of 8 points, presented with jaundice.
B. Microbial eczema Indirect bilirubin of blood - was 80 mi-
C. Contact dermatitis cromole/l, 6 hours later - 160 micromole/l.
D. Toxicoderma What is the optimal method of treatment?
E. Haemorrhagic vasculitis
92. A 45-year-old patient complains of
Krok 2 Medicine 2011 14

A. Exchange blood transfusion A. X-ray of chest


B. Phototherapy B. Clinical blood test
C. Infusion therapy C. Determination of blood gas compositi-
D. Phenobarbital treatment on
E. Enterosorbents D. Proteinogram
E. Immunoassay
96. A 42-year-old female patient suffers
from micronodular cryptogenic cirrhosis. 100. A children’s surgical unit admi-
Over the last week her condition has tted a 1-month-old boy who had been
deteriorated: there appeared convulsi- prenatally diagnosed with the left-sided
ons, mental confusion, increased jaundice. pyelectasis. Such studies as drip infusion
What study may explain such aggravati- urography, cystography and USI allowed
on? to reveal initial hydronephrosis. There is
no information confirming the secondary
A. Determination of serum ammonia pyelonephritis. What tactics of this patient
B. Determination of cholesterol ethers management is most advisable?
C. Determination of alpha-phetoprotein
rate A. 6-month surveillance
D. Determination of ALAT and ASAT B. Urgent nephrostomy
E. Determination of alkaline phosphatase C. Anderson-Hynes operation
rate D. There is no need in further surveillance
and treatment
97. A 30-year-old patient was hospitalized E. Antibacterial therapy
with a diagnosis: intestinal obstruction.
During the surgery it was revealed that 101. A 24-years-old patient has chronic
the obstruction of the small intestine had glomerulonephritis.Urine test reveals the
been caused by a mass of helminths. What following: the relative density is 1010,
helminths are these? protein - 1,65 g/l, RBCs - 5-7 in the field
of vision, WBCs - 2-3 in the field of visi-
A. Ascarids on. Blood creatinine - 0,350 millimole/l.
B. Guinea worms Serum sodium - 148 millimole/l. What is
C. Filarial worms the main reason for hyperazotemia in this
D. Cysticerci patient?
E. Pinworms
A. Reduction of glomerular filtration rate
98. On the 5th day after a surgery for B. Reduction of tubular reabsorption rate
colon injury a patient complains of bursti- C. Increased proteinuria
ng pain in the postoperative wound, D. Reduction of renal blood flow
weakness, drowsiness, headache, fever up E. Sodium retention in the organism
to 40o C. Objectively: the skin around the
wound is swollen, there is gas crepitati- 102. After an accident a patient complains
on. The wound discharges are scarce foul- of pain in the hip joint. Objectively: the
smelling, of dark-gray colorl. What is the leg is in the position of flexion, adducti-
most likely diagnosis? on and internal rotation, significantly
contracted. There is elastic resistance to
A. Anaerobic clostridial wound infection passive adduction or abduction of the
B. Abscess extremity. Major trochanter is located hi-
C. Postoperative wound infection gh above the Roser-Nelaton line. A si-
D. Erysipelas gnificant lordosis is present. What is your
E. Phlegmon provisional diagnosis?
99. A child was born at a gestational A. Iliac dislocation of hip
age of 34 weeks in grave condition. The B. Femoral neck fracture with a di-
leading symptoms were respiratory di- splacement
stress symptoms, namely sonorous and C. Cotyloid cavity fracture with a central
prolonged expiration, involving additional dislocation of hip
muscles into respiratory process. The Si- D. Inferoposterior dislocation of hip
lverman score at birth was 0 points, in 3 E. Pertrochanteric fracture of hip
hours it was 3 points with clinical findings.
Which diagnostic study will allow to di- 103. As a result of prolonged exposure
agnose the form of pneumopathy? to the sun a 20-year-old patient has
developed low-grade fever, pain and
swelling in the knee and ankle joints,
Krok 2 Medicine 2011 15

erythema on her face and nose bridge, are hypoechogenic nodes in both lobes,
leukopenia and accelerated ESR. She capsule invasion). The rate of prostate-
has been provisionally diagnosed wi- specific antigen is 60 ng/l. Prostate biopsy
th systemic lupus erythematosus. What revealed an adenocarcinoma. Which of
pathognomonic laboratory data may the supplemental examination methods
confirm this diagnosis? will allow to determine the stage of
neoplastic process in this patient?
A. Antinuclear factor
B. Accelerated ESR A. Computer tomography of pelvis
C. C-reactive protein B. Roentgenography of lumbar spine
D. Anaemia C. Excretory urography
E. Lymphocytosis D. Bone scintigraphy
E. Roentgenography of chest
104. Examination of the corpse of a man
who died from hanging reveals: death 108. While staying in a stuffy room a 19-
spots disappear when pressed upon and year-old emotionally labile girl developed
restore after 50 seconds, rigor mortis is severe weakness, dizziness, blackout,
moderately expressed only in the masti- nausea and loss of consciousness without
catory muscles as well as neck and finger convulsions. Objectively: the patient is
muscles, body temperature is of 31o C. The unconscious, the skin is pale, extremities
time of death: are cold. AP - 90/60 mm Hg, Ps- 96/min,
deficient, breathing is shallow. Pupillary
A. 6-7 hours ago and tendon reflexes are present. There are
B. 1-2 hours ago no pathological signs. What is the most li-
C. 16-24 hours ago kely diagnosis?
D. 8-10 hours ago
E. 10-18 hours ago A. Syncope
B. Vegetovascular paroxysm
105. After contusion of the right eye a C. Epileptic attack
patient complains of sudden loss of vision D. Hysterical neurosis
with remaining light perception. Objecti- E. Transient ischemic attack
vely: the eye is not irritated. The cornea
is transparent. Pupil reacts to light. The 109. The air of a foundry worker’s working
pupil area is black. The fundus reflex is zone contains condensation aerosol with
absent. What is the most likely cause of dust particles sized 2 nm (90%), 2-5 nm
vision loss? (2%), over 5 nm(6%), below 2 nm (about
2%). Characterize the dust dispersivity:
A. Hemophthalmia
B. Retinal detachment A. Fine-dispersed
C. Traumatic cataract B. Median-dispersed
D. Acute occlusion of retinal vessels C. Coarsely dispersed
E. Optic nerve avulsion D. Ultrafine-dispersed
E. Mist
106. A 68-year-old patient consulted
a doctor about a tumour in her left 110. A 40-year-old female patient
breast. Objectively: in the upper internal complain of headache, dizziness, muscle
quadrant of the left breast there is a weakness, sometimes - cramps in the
neoplasm up to 2,5 cm in diameter, dense, extremities. She has been taking anti-
uneven, painless on palpation. Regional hypertensive medications for 10 years.
lymph nodes are not enlarged. What is the AP- 180/100 mm Hg. Blood potassium -
most likely diagnosis? 1,8 millimole/l, sodium - 4,8 millimole/l. In
urine: alkaline reaction, the relative densi-
A. Cancer ty - 1012, protein and sugar are not found,
B. Cyst WBCs - 3-4 in the field of vision, RBCs - 1-
C. Fibroadenoma 2 in the field of vision. Conn’s syndrome is
D. Mastopathy suspected. Which drug should be chosen
E. Lipoma for the treatment of arterial hypertensi-
107. A 65-year-old patient complains on?
of pain in the lumbar spine, moderate
disuria. He has been suffering from
these presentations for about half a
year. Prostate volume is 45 cm3 (there
Krok 2 Medicine 2011 16

A. Spironolactone A. Warming procedures


B. Propanolol B. Dehydrating drugs
C. Enalapril C. Analgetics
D. Hydrochlorothiazide D. Vitamins of B group
E. Clonidine E. Intravenous injection of aminophylline
111. A patient is 30 years old, works 115. The institutions which take part in
as a carpenter. Six months ago there carrying out medical examinations can be
appeared some behavioural changes: he prevention and treatment facilities, medi-
got interested in philosophy, began writi- cal board of Ministry of Defense, medi-
ng a treatise on the purpose of his human cal board of Ministry of Home Affai-
existence, quitted his job, stopped cari- rs, medico-social expert commissions,
ng about his children, went out carelessly forensic medical boards etc. What insti-
dressed, heard "voices in his head"that tutions are responsible for temporary di-
guided his behaviour. The patient claimed sability examination?
sure that he was an Ambassador of God
on Earth and was constantly feeling His A. Prevention and treatment facilities
influence. He is not critical about his di- B. Sanitary-and-prophylactic institutions
sease. What diagnosis can be assumed? C. Medico-social expert commissions
D. Medical boards of Ministry of Defense
A. Schizophrenia E. Medical boards of Ministry of Home
B. Alcocholic psychosis Affairs
C. Reactive psychosis
D. Somatogenic psychosis 116. 10 minutes after delivery a woman
E. Organic psychosis discharged placenta with a tissue defect
5х6 cm large. Discharges from the genital
112. A 27-year-old patient complains of tracts were profuse and bloody. Uterus
nasal haemorrhages, multiple bruises on tonus was low, fundus of uterus was
the anterior surface of the trunk and located below the navel. Examination
extremities, sudden weakness. In blood: of genital tracts revealed that the uteri-
Hb- 74 g/l, reticulocytes - 16%, RBCs - ne cervix, vaginal walls, perineum were
2, 5 · 1012 /l, platelets - 30 · 109 /l, ESR- 25 intact. There was uterine bleeding with
mm/h. What is the most effective measure following blood coagulation. Your actions
for the treatment of thrombocytopenia? to stop the bleeding:
A. Splenectomy A. To make manual examination of uterine
B. Iron preparations cavity
C. Hemotransfusion B. To apply hemostatic forceps upon the
D. Cytostatics uterine cervix
E. Vitamin B12 C. To introduce an ether-soaked tampon
into the posterior fornix
113. 2 days ago a patient presented with D. To put an ice pack on the lower
acute pain in the left half of chest, general abdomen
weakness, fever and headache. Objecti- E. To administer uterotonics
vely: between the 4 and 5 rib on the left
the skin is erythematous, there are multi- 117. While lifting a heavy load a 39-
ple groups of vesicles 2-4 mm in diameter year-old patient suddenly felt a severe
filled with transparent liquid. What diease headache, pain in the interscapular regi-
are these symptoms typical for? on,and started vomiting. Objectively: the
pulse is rhythmic, 60/min., AP - 180/100
A. Herpes zoster mm Hg. The patient is agitated. He
B. Pemphigus presents with photophobia, hyperacusis.
C. Herpes simplex There are positive Kernig’s and Brudzi-
D. Streptococcal impetigo nski’s signs on both sides. In blood: WBCs
E. Herpetiform Duhring’s dermatosis - 10 · 109 /l. CSF is bloody, cytosis is 240/3.
What is the most likely diagnosis?
114. After lifting a load a patient felt
undurable pain in the loin. He was di- A. Subarachnoid haemorrhage
agnosed with acute lumbosacral radiculi- B. Sympathoadrenal crisis
tis. Which of the following is contraindi- C. Acute hypertonic encephalopathy
cated for this patient? D. Meningococcal meningitis
E. Ischemic stroke
Krok 2 Medicine 2011 17

118. A surgeon examined a 42-year- A. Pneumonic plaque


old patient and diagnosed him with ri- B. Miliary tuberculosis
ght forearm furuncle at the purulo- C. Influenza
necrotic stage. The furuncle was lanced. D. Ornithosis
At the hydration stage the wound dressing E. Sepsis
should enclose the following medication:
122. HIV displays the highest tropism
A. Hypertonic solution towards the following blood cells:
B. Vishnevsky ointment
C. Ichthyol ointment A. T-helpers
D. Chloramine B. T-suppressors
E. Dimexide C. T-killers
D. Thrombocytes
119. After a 10-year-old child had been E. Erythrocytes
bitten by a bee, he was delivered to
a hospital. There were lip, face and 123. Educational rooms are illuminated
neck edemata. The patient felt hot and with various lighting fittings. What type of
short of breath. Objectively: breathi- lighting fittings is the most appropriate in
ng was laboured and noisy. There were respect of hygienic norms?
foamy discharges from the mouth, cough. A. Indirect light fittings
The skin was pale and cold. There B. Direct light fittings
was bradypnoea. Heart sounds were C. Semi-reflected light fittings
muffled and arrhythmic. Thready pulse D. Ambient light fittings
was present. What diagnosis was made by E. Combined light fittings
the expert in resuscitation?
124. A 24-year-old female patient
A. Anaphylactic shock complains of acute pain in the lower
B. Quincke’s edema abdomen that turned up after a physical
C. Bronchial asthma stress. She presents with nausea, vomiting,
D. Acute cardiovascular collapse dry mouth and body temperature 36, 6o C.
E. Cerebral coma She has a right ovarian cyst in history. Bi-
120. Half an hour after a 30-year-old manual examination reveals that uterus
woman had had some custard cake, is dense, painless, of normal size. The left
she experienced lancinating abdominal fornix is deep, uterine appendages aren’t
pain, nausea, vomiting. Objectively: body palpable, the right fornix is contracted.
temperature - 36, 0oC, pale skin, breathi- There is a painful formation on the right
ng rate - 20/min, Ps- 100/min. AP- 95/65 of uterus. It’s round, elastic and mobile.
mm Hg, loud cardiac sounds. Dry tongue. It is 7х8 cm large. In blood: leukocytosis
Abdomen was painful in its epigastrial with the left shit. What is the most likely
part, there were no signs of peritoneum diagnosis?
irritation. What is the first measure to be A. Ovarian cyst with pedicle torsion
taken? B. Right-sided pyosalpinx
A. Gastric lavage C. Subserous fibromyoma of uterus
B. Administration of an enterosorbent D. Acute metritis
C. Injection of Cerucal E. Extrauterine pregnancy
D. Intravenous rehydratation 125. A 39-year-old patient complains of
E. Antibiotic therapy a tumour on the anterior surface of her
121. A 45-year-old patient, a sailor, was neck. The tumour has been observed for
hospitalized on the 2nd day of the di- 2 years. It is nonmobile and has enlarged
sease. A week ago he returned from recently. The patient has a changed tone
India. Complains of body temperature of of voice, a sense of pressure. Objectively:
41o C, severe headache, dyspnea, cough in the left lobe of the thyroid gland a
with frothy rusty sputum. Objectively: 3 cm node is palpable; it is very dense,
the patient is pale, mucous membranes tuberous, painless. Cervical lymph nodes
are cyanotic, breathing rate - 24/min, are enlarged. Functional status of the
tachycardia is present. In lungs: dimini- thyroid gland is unchanged. What is the
shed breath sounds, moist rales over both most likely diagnosis?
lungs, crepitation. What is the most likely
diagnosis?
Krok 2 Medicine 2011 18

A. Thyroid gland cancer A. Bronchography


B. Nodular euthyroid goiter B. Bronchoscopy
C. Nodular hyperthyroid goiter C. Survey radiograph of lungs
D. Chronic lymphomatous Hashimoto’s D. Pleural puncture
thyroiditis E. Bacteriological analysis of sputum
E. Chronic fibrous Riedel’s thyroiditis
129. A 49-year-old patient complai-
126. A 22-year-old patient is a clerk. His ns of dyspnea, cough. There are no
working day runs in a conditioned room. sputum discharges. He has repeatedly
In summer he was taken by an acute di- used salbutamol and intal but with no
sease with the following symptoms: fever, effect. Objectively: he is only able to sit
dyspnea, dry cough, pleural pain, myalgia, while leaning on the table. Cyanosis of
arthralgia. Objectively: moist rales on the face, acrocyanosis are present. Breathi-
right, pleural friction rub. X-ray picture ng is shallow, laboured, in some parts it
showed infiltration of the inferior lobe. In cannot be auscultated; there are diffuse
blood: WBC - 11 · 109 /l, stab neutrophi- rales, expiration is significantly prolonged.
ls - 6%, segmented neutrophils - 70%, Heart sounds are muffled, tachycardia is
lymphocytes - 8%, ESR - 42 mm/h. What present. Ps - 112/min., AP - 110/70 mm
is the ethiological factor of pneumonia? Hg. Liver is located near the costal arch.
There are no peripheral edemata. What is
A. Legionella your provisional diagnosis?
B. Mycoplasm
C. Streptococcus A. Status asthmaticus
D. Staphylococcus B. Chronic obstructive bronchitis
E. Pneumococcus C. Bronchiale asthma, moderate gravity
D. Foreign object aspiration
127. A 53-year-old woman complained of E. Cardiac asthma
cardiac pain and rhythm intermissions.
She had experienced these presentations 130. A 3-year-old girl presents with
since childhood. The patient’s father had pertussis-like cough with thick sputum.
a history of cardiac arrhythmias. Objecti- There have been persistent changes in
vely: the patient was in grave condition, Ps lungs since the age of 6 months when she
- 220 bpm, AP - 80/60 mm Hg. ECG: heart was first diagnosed with acute pneumonia.
rate - 215/min, widening and deformati- Chloride concentration in the perspirati-
on of QRS complex accompanied by atri- on is 112 mEq/l. The child has been di-
oventricular dissociation; positive P wave. agnosed with mucoviscidosis. What is the
Some time later heart rate reduced down basis for autosomal recessive disease -
to 45/min, there was a complete dissociati- mucoviscidosis?
on of P wave and QRST complex. Which
of the following will be the most effective A. Inadequate transport of sodium and
treatment? chloride ions
B. α1 -antitrypsin deficiency
A. Implantation of the artificial pacemaker C. Deposition of calcium triphosphates
B. β-adrenoreceptor blocking agents and carbotates in the alveoles
C. Cholinolytics D. Pulmonary cysts
D. Calcium antagonists E. Pulmonary artery hypoplasia
E. Cardiac glycosides
131. A parturient woman is 23 years
128. A 47-year-old patient complains old. Vaginal obstetric examination reveals
about cough with purulent sputum, pain full cervical dilatation. There is no fetal
in the lower part of the left chest, peri- bladder. Fetal head is in the plane of
odical body temperature rise. She has pelvic outlet. Sagittal suture is in mesati-
been suffering from these presentations pellic pelvis, anterior fontanel is closer to
for about 10 years. Objectively: "drumsti- pubes. The fetal head diameter in such
ck"distal phalanges. What examination presentation will be:
would be the most informative for making
a diagnosis? A. Suboccipito-bregmaticus
B. Fronto-occipitalis recta
C. Biparietal
D. Suboccipitio-frontalis
E. Mento-occipitalis
132. A 42-year-old woman has had
Krok 2 Medicine 2011 19

hyperpolymenorrhea and progressing A. Determination of blood calcium and


algodismenorrhea for the last 10 years. phosphor
Gynaecological examination revealed no B. Determination of thyrotropic hormone
changes of uterine cervix; discharges are C. Determination of potassim
moderate, of chocolate colour, uterus is D. Determination of sodium
slightly enlarged and painful, appendages E. Determination of thyroid hormones
are not palpable, the fornices are deep
and painless. What is the most likely di- 136. A patient with bilateral hydrothorax
agnosis? has repeatedly undergone pleural
puncture on both sides. After a regular
A. Uterine endometriosis puncture the patient’s condition has
B. Uterine carcinoma become worse: he presents with fever,
C. Subserous uterine fibromyoma chest pain. The next day, the attending
D. Endomyometritis physician performing pleural puncture
E. Adnexal endmetriosis revealed some pus on the right. What
is the mechanism of acute right-sided
133. Examination of a 38-year-old pati- empyema development?
ent who had been hit with a blunt object
on the left side of chest revealed a A. Contact-and-aspiration
fracture of the X rib with fragments di- B. Lymphogenous
splacement, parietal pneumothorax. The C. Hematogenous
patient complains of pain in the left D. Implantation
subcostal area. Objectively: the patient E. Aerial
is pale, AP- 80/40 mm Hg, Ps- 138/min,
of poor volume. USI reveals fluid in the 137. A student analyzes noise level of
left abdomen. Splenic rupture is present. cold-pressing process. What device should
What treatment tactics should be chosen? be applied for this hygienic study?

A. Drainage of the left pleural cavity and A. Noise and vibration analyzer
laparotomy B. Noise analyzer
B. Immediate upper middle laparotomy C. Sound tester
and following drainage of the left pleural D. Actinometer
cavity E. Pyranometer
C. Immediate laparotomy and alcohol- 138. Environmental pollution is prevented
novocaine block of the X rib by mechanical separation of nontoxic
D. Anti-schock actions and laparotomy solid domestic waste. Specify the method
following the arterial pressure rise which can be used for mechanical utili-
E. Left-sided thoracotomy and immedi- zation of these wastes:
ately following laparotomy
A. Compressing of wastes into building
134. 15 minutes after the second vacci- blocks
nation with DTP vaccine a 4-month-old B. Hydrolysis
boy exhibited the symptoms of Quincke’s C. Burning as power-plant fuel
edema. What medication should be given D. Burial of wastes
for emergency aid? E. Waste neutralization in biothermal
A. Prednisolone boxes
B. Heparin 139. A 26-year-old woman complains of
C. Adrenalin having bloody discharges from the geni-
D. Furosemide tals for the last 14 days, abdominal pain,
E. Seduxen general fatiguability, weakness, weight
135. On the 2nd day after a surgery for loss, fever, chest pain, obstructed respirati-
toxic mixed goiter IV a 35-year-old pati- on. 5 weeks ago she underwent an induced
ent complains of heart pain. ECG shows abortion in the 6-7 week of gestation.
prolonged QT intervals. Chvostek’s and Objectively: the patient is pale and inert.
Trousseau symptoms cannot be clearly Bimanual examination revealed that the
defined. The patient is provisionally di- uterus was enlarged up to 8-9 weeks of
agnosed with latent tetany. What study gestation. In blood: Hb - 72 g/l. Urine
will allow to confirm the diagnosis? test for chorionic gonadotropin gave the
apparently positive result. What is the
most likely diagnosis?
Krok 2 Medicine 2011 20

A. Chorioepithelioma A. Poliomyelitis
B. Metroendometritis B. Diphtheria and tetanus
C. Uterus perforation C. Measles, rubella, parotitis
D. Uterine fibromyoma D. Pertussis
E. Uterine carcinoma E. Type B hepatitis
140. A 28-years-old woman complains of 144. A 40-year-old patient underwent an
nausea and vomiting about 10 times per operation for a lumbar phlegmon. Body
day. She has been found to have body temparature rose again up to 38o C, he
weight loss and xerodermia. The pulse is got intoxication symptoms, there was an
100 bpm. Body temperature is 37, 2oC. increase of leukocyte number in blood.
Diuresis is low. USI shows 5-6 weeks of The wound that was nearly free from
pregnancy. What is the most likely di- necrotic tissues and full of granulations
agnosis? started to discharge pus, the granulations
turned pale. What complication dveloped
A. Moderate vomiting of pregnancy in this patient?
B. Mild vomiting of pregnancy
C. I degree preeclampsia A. Sepsis
D. Premature abortion B. Putrid phlegmon
E. Food poisoning C. Erysipelas
D. Allergic reaction
141. A full-term baby was born with body E. Erysipeloid
weight of 3200 g, body length of 50 cm,
Apgar score - 8-10 points. What is the 145. After a car accident a 37-year-old
optimum time for the first breast-feeding? patient has an acute pain and bleeding in
the middle third of his right thigh. Objecti-
A. First 30 minutes vely: there is a wound on the anterior
B. First 6 hours surface of the right thigh with massive
C. First 24 hours bleeding, abnormal mobility at the level
D. First 48 hours of the middle third of the thigh. The first
E. After 48 hours aid is to be started with:
142. A 26-year-old patient with left lower A. Digital occlusion of the femoral artery
lobe pneumonia experiences an acute B. Injection of pain-killers
chest pain on the left during coughing. C. Tourniquet application
Objectively: diffuse cyanosis, widening of D. Immobilization of the extremity with a
the left half of chest. Percussion reveals transportation splint
high tympanitis. Auscultation reveals no E. Venipuncture and intravenous infusion
respiratory murmurs above the left half of polyglycine
of chest. There is a deviation of the right
cardiac border towards the midclavicular 146. A 9-year-old boy fell from a tree
line. What examination will be the most and hit the occipital region, there was a
informative? momentary loss of consciousness. Objecti-
vely: the child’s condition is satisfactory,
A. X-Ray he complains of the headache and dizzi-
B. Bronchoscopy ness. The X-ray of skull reveals a commi-
C. Bronchography nuted depressed fracture of occipital bone
D. Pneumotachometry in the region of inion. What treatment is
E. Spirography indicated for the patient?
143. A 3-year-old child has been taken A. Surgical intervention
to a pediatrician. He has no recent hi- B. Anti-inflammatory therapy
story of any diseases. Objective exami- C. Hemostatic therapy
nation revealed no pathology of the D. Therapeutic lumbar punctures
internal organs. The child needs the routi- E. Complex conservative treatment
ne immunization against the following di-
sease: 147. A 40-year-old female patient has a hi-
story of rheumatism. She complains about
acute pain in her left eye, especially at
night, vision impairment, photophobia,
lacrimation. The patient cannot suggest
any reasons for the disease. Objectively:
weak pericorneal injection, flattening of
Krok 2 Medicine 2011 21

iris relief, iris discoloration. What is the ent felt sharp abdominal pain irradiating
most likely diagnosis? to the right scapula, there was a single
vomiting. She has a history of rheumatoid
A. Iridocyclitis arthritis. Objectively: pale skin, AP-
B. Iritis 100/60 mm Hg, Ps- 60/min. Abdomen is
C. Keratitis significantly painful and tense in the epi-
D. Choroiditis gastrium and right subcostal areat, there
E. Acute attack of glaucoma are positive symptoms of parietal peri-
toneum irritation over the right costal
148. 14 days ago a 49-year-old patient arch, that is tympanitis. What is the tactics
was operated on for perforated appendi- of an emergency doctor?
citis and disseminated fibrinopurulent
peritonitis. The postoperative period was A. To take the patient to the surgical
uneventful. On the 9th day the patient hospital
presented with low-grade fever, abdomi- B. To inject pain-killers and watch the
nal pain, frequent liquid stools. USI of the dynamics
left mesogastrium reveals a fluid formati- C. To perform gastric lavage
on 9x10 cm large. In blood: leukocytosis D. To inject spasmolytics
with the left shift. What is your provisi- E. To take the patient to the rheumatologi-
onal diagnosis? cal department
A. Interloop abscess 152. A 72-year-old patient complains of
B. Abdominal cavity tumour pain and bleeding during defecation. Digi-
C. Liver abscess tal rectal investigation revealed a tumour
D. Left kidney cyst of the anal canal. After verification of the
E. Spleen abscess diagnosis the patient was diagnosed with
squamous cell carcinoma. The secondary
149. The Carpathian region is characteri- (metastatic) tumour will be most probably
zed by constant high humidity of
found in:
atmospheric air (over 80%). Inhabi-
tants of this region feel severe cold in A. Lungs
corresponding season at a medium low B. Liver
temperature. It’s caused by heat emission C. Pelvic bones
by: D. Mediastinum
E. Brain
A. Convection
B. Radiation 153. A patient has a stab wound on his ri-
C. Vaporization ght foot. On the fourth day after injury
D. Conduction the patient’s body temperature rose up
E. - to 38o C, inguinal lymph nodes became
enlarged and painful, skin over them
150. A 22-year-old female patient reddened. What complication might be
complains of dull pain in her right iliac suspected?
area that she has been experiencing for
a week, morning sickness and gustatory A. Lymphadenitis
change. She has a histrory of menstruati- B. Lymphangitis
on delay for 3 weeks. Objectively: AP- C. Phlegmon
80/50 mm Hg, pulse is 78 bpm, body D. Tetanus
temperature is 37o C. Bimanual examinati- E. Erysipelas
on reveals that uterus is enlarged, soft,
mobile and painless. Uterine appendages 154. A 25-year-old female patient
are palpable on the right, there is a dense, complains about having amenorrhea
elastic and moderately painful formation for 3 years. She associates it with di-
3x4 cm large. What is the most likely di- fficult labour complicated by massive
agnosis? hemorrhage. She also complains of loss
of weight, hair fragility and loss, lack of
A. Progressing fallopian pregnancy appetite and depression. Objective exami-
B. Interrupted fallopian pregnancy nation reveals no pathological changes of
C. Right ovarian cyst uterus and its appendages. What is the
D. Uterogestation desease pathogenesis?
E. Acute appendicitis
151. 3 hours ago a 65-year-old female pati-
Krok 2 Medicine 2011 22

A. Hypoproduction of gonadotropin A. Liver abscess


B. Hyperproduction of estrogens B. Pylephlebitis
C. Hyperproduction of androgens C. Hepatic cyst
D. Hypoproduction of progesterone D. Liver necrosis
E. Hyperproduction of prolactin E. Budd-Chiari syndrome
155. During a surgery on a 30-year-old 159. A 32-year-old gravida complains of
patient a dark ileo-ileal conglomerate was episodes of unconsciousness, spontaneous
discovered, the intussusceptum intesti- syncopes that are quickly over after a
ne was considered to be unviable. The change of body position. A syncope
intussuscipiens intestine was dilated to 7-8 can be accompanied by quickly elapsing
cm, swollen, full of intestinal contents and bradycardia. There are no other compli-
gases. What pathology led to the surgery? cations of gestation. What is the most li-
kely reason for such condition?
A. Invagination (combined) obstruction
B. Strangulation obstruction A. Postcava compresseion by the gravid
C. Obturation obstruction uterus
D. Paralytic obstruction B. Pressure rise in the veins of extremities
E. Spastic obstruction C. Pressure fall in the veins of extremities
D. Vegetative-vascular dystonia (cardiac
156. In the current year general practi- type)
tioners of the municipal polyclinic have E. Psychosomatic disorders
referred 11 patients with coronary artery
disease to the in-patient hospital. In 3 160. A 54-year-old female patient
cases the diagnosis wasn’t confirmed. consulted a gynaecologist about bloody
What managerial decision shoud be made discharges from the vagina for 1 month.
in such case? Last menstruation was 5 years ago.
Gynaecological examination revealed no
A. Analysis of each case of diagnostic pathological changes. What is the tactics
divergence of choice?
B. Analysis of medical check-up quality
C. Analysis of diagnostic examination A. Diagnostic fractional curettage of uteri-
quality ne cavity
D. Analysis of doctors’ skill level B. Colposcopy
E. Analysis of material and technical C. USI
basisof the polyclinic D. Cytosmear
E. Symptomatic therapy
157. It is required to analyze the level
of daylight illumination in a ward of 161. A factory worker has ARD compli-
therapeutics unit. What device should be cated by acute bronchitis. He receives
applied to estimate the level of daylight treatment in the outpatient setting. The
illumination? attending doctor has issued him a medical
certificate for 5 days and then extended its
A. Illuminometer duration by 5 more days. Patient can’t get
B. Anemometer down to work because of his health status.
C. Katathermometer Who should extend the duration of medi-
D. Actinometer cal certificate for this patient?
E. Psychrometer
A. A department chief
158. A 43-year-old patient had been admi- B. Medical superintendent
tted to a hospital with clinical presentati- C. Deputy medical superintendent in
ons of ischiorectal periproctitis. On the charge of temporary disability examinati-
12th day of treatment the patient’s condi- on
tion deteriorated: there was an increase D. Deputy medical superintendent in
in the rate of intoxication and hepatic charge of medical treatment
failure, the body temperature became E. Medical advisory commission
hectic, AP was 100/60 mm Hg. USI of
liver revealed a hydrophilic formation. 162. An ambulance delivered a 21-
In blood: WBCs - 19, 6 · 109 /l, RBCs.- year-old woman to the gynaecological
3, 0 · 1012 /l, Hb- 98 g/l. What complicati- department with complaints of colicky
on was developed? abdominal pain and bloody discharges
from the genital tracts. Bimanual exami-
nation revealed that uterus was soft,
Krok 2 Medicine 2011 23

enlarged to the size of 6 weeks of gestati- A. Correlation factor


on, a gestational sac was palpated in B. Student’s criterion
the cervical canal. Uterine appendages C. Standardized index
weren’t palpable. Fornices are free, deep D. Fitting criterion
and painless. Discharges from the genital E. Sign index
tracts are bloody and profuse. What is the
most likely diagnosis? 166. A 30-year-old female patient has
been delivered to the gynaecological
A. Abortion in progress department with complaints of acute
B. Cervical pregnancy pain in the lower abdomen and body
C. Threat of abortion temperature 38, 8o C. In history: sexual life
D. Incipient abortion out of wedlock and two artificial aborti-
E. Interrupted fallopian pregnancy ons. Gynaecological examination reveals
no changes of uterine. The appendages
163. According to the report of water are enlarged and painful on both si-
quality control, drinking city water has des. Vaginal discharges are purulent and
the following characteristics: turbidity profuse. What study is required to confirm
- 1,5 mg/m3 , odour - 3 points, metallic a diagnosis?
taste - 2 points, pale yellow colour, colour
index - 20o , temperature - 12o . Which of A. Bacteriological and bacterioscopic
these factors doesn’t comply with hygienic analysis
requirements? B. Hysteroscopy
C. Curettage of uterine cavity
A. Odour D. Colposcopy
B. Turbidity E. Laparoscopy
C. Colour index
D. Temperature 167. A 38-year-old patient complains
E. Taste of inertness, subfebrile temperature,
enlargement of lymph nodes, nasal
164. On the fifth day after a casual haemorrhages, bone pain. Objectively:
sexual contact a 25-year-old female pati- the patient’s skin and mucous membranes
ent consulted a doctor about purulent di- are pale, palpation revealed enlarged
scharges from the genital tracts and itch. painless lymph nodes; sternalgia; liver
Vaginal examination showed that vagi- was enlarged by 2 cm, spleen - by 5
nal part of uterine cervix was hyperemic cm, painless. In blood: erythrocytes -
and edematic. There was an erosive area 2, 7 · 1012 /l, Hb- 84 g/l, leukocytes - 58 ·
around the external orifice of uterus. 109 /l, eosinophils - 1%, stab neutrophi-
There were mucopurulent profuse di- ls - 2%, segmented neutrophils - 12%,
scharges from the cervical canal, uteri- lymphocytes - 83%, lymphoblasts - 2%,
ne body and appendages exhibited no smudge cells; ESR- 57 mm/h. What is the
changes. Bacterioscopic examination most likely diagnosis?
revealed bean-shaped diplococci that
became red after Gram’s staining. What A. Chronic lymphatic leukemia
is the most likely diagnosis? B. Chronic myeloleukemia
C. Acute lymphatic leukemia
A. Acute gonorrheal endocervicitis D. Acute myeloleukemia
B. Trichomonal colpitis E. Lymphogranulomatosis
C. Candidal vulvovaginitis
D. Clamydial endocervicitis 168. A 25-year-old victim of a road accient
E. Bacterial vaginism complains of chest pain, dyspnea. Objecti-
vely: the patient is in a grave condition,
165. The correlation between the servi- Ps- 120/min, AP- 90/70 mm Hg.There is
ce record and eosinophil concentration in pathological mobility of fragments of III-
blood has been studied in female workers V ribs on the right. Percussion reveals a
of dyeing shops of textile factories. What box sound over the right lung, breathi-
index will be the most informative for the ng sounds cannot be auscultated on the
analysis of this data? right. What examination should be admi-
nistered in the first place?
Krok 2 Medicine 2011 24

A. X-ray of chest organs


B. Bronchoscopy A. Bloody
C. Pleural puncture B. Sanguino-serous
D. USI of chest organs C. Mucous
E. Thoracoscopy D. Purulent
E. Serous
169. A 24-year-old patient complains
about putting on weight, limosis. Objecti- 173. A 32-year-old patient consulted a
vely: the patient’s constitution is of doctor about being inable to get pregnant
hypersthenic type, body weight index is for 5-6 years. 5 ago the primipregnancy
33,2 kg/m2 , waist circumference is 100 cm. ended in artificial abortion. After the vagi-
Correlation of waist circumference to the nal examination and USI the patient was
thigh circumference is 0,95. What is the diagnosed with endometrioid cyst of the
most likely diagnosis? right ovary. What is the optimal treatment
method?
A. Alimentary constitutional obesity of
the I stage, abdominal type A. Surgical laparoscopy
B. Hypothalamic Itsenko-Cushing obesity B. Anti-inflammatory therapy
of the II stage, gynoid type C. Conservative therapy with estrogen-
C. Alimentary constitutional obesity of the gestagenic drugs
III stage, gynoid type D. Hormonal therapy with androgenic
D. Alimentary constitutional obesity of the hormones
II stage, abdominal type E. Sanatorium-and-spa treatment
E. Hypothalamic Itsenko-Cushing obesity
of the I stage, abdominal type 174. A 6-year-old girl drank some
coloured fizzy drink which gave her
170. A 58-year-old patient complains a feeling of pressure in the throat.
about sensation of numbness, sudden 30 minutes later the child’s lips got
paleness of II-IV fingers, muscle ri- swollen, then edema gradually spread
gidness, intermittent pulse. The pati- over the whole face, laryngeal breathi-
ent presents also with polyarthralgia, ng became difficult. The child is exci-
dysphagia, constipations. The patient’s ted. Ps- 120/min, breathing rate - 28/min,
face is masklike, solid edema of hands is breathing is noisy, indrawing of intercostal
present. The heart is enlarged; auscultati- spaces is observed. What basic aid is
on revealed dry rales in lungs. In blood: most appropriate for the restoration of
ESR - 20 mm/h, crude protein - 85/l, γ- laryngeal breathing?
globulines - 25%. What is the most likely
diagnosis? A. Corticosteroids
B. Sedative drugs
A. Systemic scleroderma C. Tracheostomy
B. Dermatomyositis D. Antibacterial drugs
C. Rheumatoid arthritis E. Conicotomy
D. Systemic lupus erythematosus
E. Raynaud’s disease 175. A 60-year-old patient had eaten too
much fatty food, which resulted in sudden
171. A 45-year-old man has been exhibi- pain in the right subcostal area, nausea,
ting high activity for the last 2 weeks, he bilious vomiting, strong sensation of bi-
is talkative, euphoric, has little sleep, clai- tterness in the mouth. Two days later the
ms being able "to save the humanity and patient presented with jaundice, dark uri-
solve the problem of cancer and AIDS", ne. Objectively: sclera and skin are icteric,
gives money to starangers. What is the abdomen is swollen, liver is increased by 3
most likely diagnosis? cm, soft, painful on palpation, Ortner’s,
Kehr’s, Murphy’s, Zakharyin’s, Mayo-
A. Maniacal onset Robson’s symptoms are positive. Which
B. Panic disorder method should be applied for diagnosis in
C. Agitated depression the first place?
D. Schizo-affective disorder
E. Catatonic excitation
172. A parturient woman is 25 years old,
it is her second day of postpartum period.
It was her first full-term uncomplicated
labour. The lochia should be:
Krok 2 Medicine 2011 25

A. USI of gallbladder and biliary duct A. III


B. Fibrogastroduodenoscopy B. II
C. X-ray of abdominal organs C. I
D. Radionuclide scanning of liver and D. IV
gallbladder E. V
E. Diagnostic laparotomy
180. A 17-year-old patient complains of
176. A 20 year-old patient complains pain in the area of the left knee joint.
of nosebleeds, numbness of the lower Soft tissues of thigh in the affected regi-
extremities. Objectively: hyperaemia of on are infiltrated, joint function is limi-
face, on the upper extremities AP is160/90 ted. X-ray picture of the distal metaepi-
mm Hg, and 80/50 mm Hg on the lower physis of the left femur shows a destructi-
ones. Pulse on the popliteal and pedal on focus with periosteum detachment and
arteries is of poor volume, there is systolic Codman’s triangle found at the defect
murmur over the carotid arteries. What is border in the bone cortex. X-ray of chest
the most likely diagnosis? reveals multiple small focal metastases.
What treatment is indicated?
A. Aorta coarctation
B. Dissecting aortic aneurysm A. Palliative chemotherapy
C. Aortopulmonary window B. Radioiodine therapy
D. Ventricular septal defect C. Distance gamma-ray therapy
E. Atrial septal defect D. Disarticulation of the lower extremity
E. Amputation of the lower extremity
177. An 8-month-old baby has had
problems with nasal breathing and muco- 181. Head circumference of a 1-month-
purulent discharge from the nose for a old boy with signs of excitement is 37 cm,
week. Examination reveals a rhinedema, prefontanel is 2x2 cm large. After feeding
muco-purulent discharges from the mi- the child regurgitates small portions of mi-
ddle nasal meatus as well as on the lk; stool is normal in respect of its volume
back of pharynx. What disease are these and composition. Muscle tonus is within
symptoms most typical for? norm. What is the most likely diagnosis?
A. Ethmoiditis A. Pylorospasm
B. Sphenoiditis B. Meningitis
C. Maxillary sinusitis C. Pylorostenosis
D. Frontitis D. Microcephaly
E. Hemisinusitis E. Craniostenosis
178. A department chief of an in-patient 182. As a result of lifting a load a 62-year-
hospital is going to inspect resident old female felt acute pain in the lumbar
doctors as to observation of medical- region, in a buttock, posterolateral surface
technological standards of patient service. of her right thigh, external surface of
What documentation should be checked the right shin and dorsal surface of foot.
for this purpose? Objectively: weakness of the anterior ti-
bial muscle, long extensor muscle of the
A. Health cards of in-patients right toes, short extensor muscle of the ri-
B. Statistic cards of discharged patients ght toes. Low Achilles reflex on the right.
C. Treatment sheets Positive Lasegue’s sign. What examinati-
D. Registry of operative interventions on method would be the most effective for
E. Annual report of a patient care instituti- specification of the diagnosis of discogenic
on compression of L5 root?
179. While asessing the health status A. Magnetic resonance scan
of graduates of a secondary school, the B. Spinal column X-ray
doctor found one of them to have grade 3 C. Electromyography
tonsillar hypertrophy, chronic rhinitis and D. Angiography
vegetative-vascular dystonia. The organi- E. Lumbar puncture
sm functionality is reduced. This student
belongs to the following health group: 183. At first appointment with an
obstetrician-gynaecologist a pregnant
woman is referred to other medical speci-
alists. She must be obligatory examined
by the following specialists:
Krok 2 Medicine 2011 26

the gynecological unit with complaints of


A. Therapeutist and dentist pain in the lower abdomen and insigni-
B. Therapeutist and endocrinologist ficant bloody discharges from the genital
C. Dentist and phthisiatrician tracts for 3 hours. Last menstruation was 3
D. ENT and ophthalmologist months ago. Vaginal examination showed
E. Dentist and cardiologist that body of womb was in the 10th week
of gestation, a fingertip could be inserted
184. A hospital admitted a patient wi- into the external orifice of uterus, bloody
th coarse breathing (obstructed inspirati- discharges were insignificant. USI showed
on), skin cyanosis, tachycardia and arteri- small vesicles in the uterine cavity. What
al hypertension. He has a histrory of is the most likely diagnosis?
bronchial asthma. An hour ago he was
having salbutamol inhalation and forgot A. Grape mole
to remove a cap that was aspired whi- B. Abortion in progress
le taking a deep breath. What measures C. Incipient abortion
should the doctor take? D. Threat of spontaneous abortion
E. Incomplete abortion
A. Perform the Heimlich manoever
B. Perform conicotomy immediately 188. A 30-year-old male patient consulted
C. Send for an anesthesiologist and wait a family doctor 2 months after he had
for him been operated for an open fracture of
D. Use an inhalation of β2 -adrenoceptor brachial bone. Objectively: the patient’s
agonist condition is satisfactory, in the region of
E. Make a subcutaneous injection of the operative wound there is a fistula
dexamethasone with some purulent discharge, redness,
fluctuation. X-ray picture shows brachi-
185. A 19-year-old student has been hospi- al bone destruction with sequestra. What
talized on an emergency basis because complication arose in the postoperative
of severe dyspnea, pain in the left si- period?
de of chest. The patient got sick 3 days
ago. Objectively: body temperature - A. Posttraumatic osteomyelitis
38, 8oC. BH - 42/min., hypopnoe. There B. Hematogenic osteomyelitis
is dullness of percussion sound on the C. Wound abscess
right of the scapula middle, breathing D. Posttraumatic phlegmon
sounds cannot be auscultated. The left E. Suture sinus
border of heart is displaced outwards
by 3 cm. Embryocardia is preent, HR - 189. A 50-year-old male suburbanite
110/min. The right hypochondrium is pai- underwent treatment in rural outpati-
nful on palpation. What are the immedi- ent clinic for pneumonia. The treatment
ate treatment measures in this situation? didn’t have effect and the disease got
complicated by exudative pleuritis. What
A. Urgent puncture of pleural cavity prevention and treatment facility should
B. Administartion of antibiotics of penici- the patient be referred to for further aid?
llin group
C. Injection of lasix A. Central district hospital
D. Injection of cardiac glycosides B. Regional hospital
E. Transfer of the patient to the thoracic C. Phthisio-pulmonological dispensary
surgery department D. Municipal hospital
E. Tuberculosis dispensary
186. On the 6th day of life a child got
multiple vesicles filled with seropurulent 190. A patient is being prepared for the
fluid in the region of occiput, neck and operation on account of varix dilatation
buttocks. General condition of the chi- of lower extremities veins. Examination
ld is normal. What disease should be of the patient’s soles revealed flour-like
suspected? desquamation along the skin folds. All
the toenails are greyish-yellow, thickened
A. Vesiculopustulosis and partially decayed. What dermatosis
B. Impetigo neonatorum should be suspected?
C. Miliaria
D. Impetigo
E. Epidermolysis bullosa
187. A pregnant woman was delivered to
Krok 2 Medicine 2011 27

A. Rubromycosis bodies to Rh weren’t found at 32 weeks of


B. Pityriasis versicolor pregnancy. Redetermination of antibodies
C. Candidosis to Rh didn’t reveal them at 35 weeks of
D. Microsporia pregnancy as well. How often should the
E. Microbial eczema antibodies be determined hereafter?
191. A 58-year-old patient complains of a A. Once a week
headache in the occipital region, nausea, B. Once in two weeks
choking, opplotentes. The presentati- C. Once in three weeks
ons appeared after a physical exerti- D. Montly
on. Objectively: the patient is excited. E. There is no need in further checks
Face is hyperemic. Skin is pale. Heart
sounds are regular, the 2nd aortic sound 195. A patient is 50 years old, works as a
is accentuated. AP- 240/120 mm Hg, HR- builder with 20 years of service record.
92/min. Auscultation reveals some fine He was admitted to the hospital for chest
moisr rales in the lower parts of the lungs. pain, dry cough, minor dyspnea. Objecti-
Liver is not enlarged. ECG shows signs of vely: sallow skin, acrocyanosis, asbestos
hypertrophy and left ventricular overload. warts on the hands. In lungs - rough respi-
What is the most likely diagnosis? ration, diffuse dry rales. The x-ray pi-
cture shows intensification of pulmonary
A. Complicated hypertensic crisis pattern, signs of pulmonary emphysema.
B. Acute myocardial infarction, pulmonary What is the most likely diagnosis?
edema
C. Bronchial asthma exacerbation A. Asbestosis
D. Uncomplicated hypertensic crisis B. Lung cancer
E. Community-acquired pneumonia C. Pneumonia
D. Chronic obstructive bronchitis
192. A 43-year-old female patient was E. Tuberculosis
delivered to the hospital in grave condi-
tion. She has a history of Addison’s di- 196. A 14-year-old girl complains of pain
sease. The patient had been regularly in vaginal area and lower abdomen that
taking prednisolone but a week before last for 3-4 days and have been observed
she stopped taking this drug. Objecti- for 3 months about the same time. Each
vely: sopor, skin and visible mucous time pain is getting worse. Objectively:
membranes are pigmented, skin and mammary glands are developed, hairi-
muscle turgor is decreased. Heart sounds ness corresponds to the age. The virgi-
are muffled, rapid. AP- 60/40 mm Hg, nal membrane is intact, cyanotic and
heart rate - 96/min. In blood: Na - 120 mi- protruded. She has never had menstruati-
llimole/l, K - 5,8 millimole/l. Development on. She has been diagnosed with pri-
of this complication is primarily caused by mary amenorrhea. What is the reason of
the deficit of the following hormone: amenorrhea?
A. Cortisol A. Hymen atresia
B. Corticotropin (ACTH) B. Turner’s syndrome
C. Adrenaline C. Babinski-Frohlich syndrome
D. Noradrenaline D. Pregnancy
E. Adrostendion E. Sexual development delay
193. Forensic medical expertise of corpse 197. A 22-year-old vegetarian patient
of a newborn revealed: body weight 3500 with signs of malnutrition consulted a
g, body length 50 cm, the umbilical cord doctor about smell and taste distortion,
was smooth, moist, glossy, without any si- angular stomatitis. Objectively: marked
gns of drying. Hydrostatic tests were posi- blue sclerae. The patient was diagnosed
tive. The test results are the evidence of: with iron deficiency anemia. What is the
dominating clinical syndrome?
A. Live birth
B. Stillbirth A. Sideropenic
C. Primary atelectasis B. Anaemic
D. Secondary atelectasis C. Haemologic
E. Hyaline membrane disease D. Haemolytic
E. Myelodysplastic
194. A primigravida is 22 years old. She
has Rh(-), her husband has Rh(+). Anti- 198. Condition of a patient with purulent
Krok 2 Medicine 2011 28

otitis has abruptly deteriorated: he A. Postoperative lethality


presents with headache, vomiting, febri- B. Total lethality
le temperature, general hyperesthesia. C. Index of late hospitalization since a
There are meningeal signs, papilledemas. disease incursion
Focal symptoms are absent. Cerebrospi- D. Standardized lethality
nal fluid is turbid, pressure is high, there E. -
is albuminocytologic dissociation with
neutrophil predominance. What disease 200. A 10-year-old boy periodically
can be suspected? has short states (up to 10-15 seconds)
that can be characterized as a "sudden
A. Secondary purulent meningitis blackout"and are accompanied by gaze
B. Meningoencephalitis fixation in the upright position, absent-
C. Serous meningitis minded and vacant face expression, lack
D. Primary purulent meningitis of movements and following amnesia.
E. Subarachnoid haemorrhage Specify this state:
199. A municipal hospital reported on A. Absence
the number of operated patients includi- B. Obnubilation
ng fatal outcomes following the operati- C. Trance
ons. Which index of hospital work can be D. Fugue
calculated on the ground of this data? E. Thought block
Krok 2 Medicine 2012 1

1. Routine examination of a child with A. Developing of cardiac insufficiency


a history of bronchial asthma reveals AP B. Depositing of blood in venous channel
of 140/90 mm Hg. The most likely cause of C. Shunting
the hypertension is: D. Presence of hypervolemia
E. Increase of bleeding speed
A. Renal disease
B. Theophylline overdose 6. A neonate was born from the 1st
C. Chronic lung disease gestation on term. The jaundice was
D. Coarctation of the aorta revealed on the 2nd day of life, then it
E. Obesity became more acute. The adynamia, vomi-
ting and hepatomegaly were observed.
2. Head of a department and a trade- Indirect bilirubin level was 275µmol/L,
union group have appealed to the head direct bilirubin level - 5µmol/L, Hb- 150
of a hospital about dismissal of the seni- g/l. Mother’s blood group - 0(I), Rh+ , chi-
or nurse who has 17 year record of servi- ld’s blood group - A(II), Rh+ . What is the
ce. The facts of charge were confirmed most probable diagnosis?
and recognized by the nurse herself. This
nurse lives with a daughter (who is di- A. Hemolytic disease of the neonate (АВ0
vorced and unemployed) and a 9-month- incompatibility), icteric type
old grandson. Make an administrative B. Jaundice due to conjugation disorder
decision: C. Hepatitis
D. Physiological jaundice
A. To continue the worker in office with E. Hemolytic disease of the neonate (Rh -
a warning of dismissal in case of repeated incompatibility)
violation of labor discipline
B. To discharge the worker, i.e. to satisfy 7. An infant was born with body mass 3 kg
demands of the collective and body length 50 cm. Now he is 3 years
C. To issue the sick list old. His brother is 7 years old, suffers from
D. To shift the solution of this problem on rheumatic fever. Mother asked the doctor
other officials or public organizations for a cardiac check up of the 3-year-old
E. - son. Where is the left relative heart border
located?
3. Purulent mediastinitis is diagnosed at a
63-year-old patient. What diseases from A. 1 cm left from the left medioclavicular
the stated below CANNOT cause the line
purulent mediastinitis? B. 1 cm right from the left medioclavicular
line
A. Cervical lymphadenitis C. Along the left medioclavicular line
B. Deep neck phlegmon D. 1 cm left from he left parasternal line
C. Perforation of the cervical part of the E. 1 cm right from the left parasternal line
easophagus
D. Perforation of the thoracic part of the 8. A neonate is 5 days old. What vacci-
easophagus nation dose of BCG vaccine (in mg) is
E. Iatrogenic injury of the trachea necessary for vaccination of this child?
4. For the persons who live in a hot area A. 0,05 mg
after an accident at a nuclear object, the B. 0,025 mg
greatest risk within the first decade is C. 0,075 mg
represented by cancer of: D. 0,1 mg
E. 0,2 mg
A. Thyroid gland
B. Skin 9. A 60-year-old woman, mother of 6
C. Reproduction system organs children, developed a sudden onset of
D. Breast upper abdominal pain radiating to the
E. Lungs back, accompanied by nausea, vomiting,
fever and chills. Subsequently, she noti-
5. During dynamic investigation of a pati- ced yellow discoloration of her sclera and
ent the increase of central venous pressure skin. On physical examination the pati-
is combined with the decrease of arterial ent was found to be febrile with temp of
pressure. What process is proved by such 38, 9oC, along with right upper quadrant
combination? tenderness. The most likely diagnosis is:
Krok 2 Medicine 2012 2

A. Choledocholithiasis g/L, WBC- 20-25 in f/vis, RBC- 1-2 in f/vis.


B. Benign biliary stricture What diagnosis is the most probable?
C. Malignant biliary stricture
D. Carcinoma of the head of the pancreas A. Acute cystitis
E. Choledochal cyst B. Dysmetabolic nephropathy
C. Acute glomerulonephritis
10. 4 days ago a 32-year-old patient D. Acute pyelonephritis
caught a cold: he presented with sore E. Urolithiasis
throat, fatigue. The next morning he
felt worse, developed dry cough, body 14. A woman, primagravida, consults
temperature rose up to 38, 2o C, there a gynecologist on 05.03.2012. A week
appeared muco-purulent expectoration. ago she felt the fetus movements for
Percussion revealed vesicular resonance the first time. Last menstruation was on
over lungs, vesicular breathing weakened 10.01.2012. When should she be given
below the angle of the right scapula, fi- maternity leave?
ne sonorous and sibilant wheezes. What is
the most likely diagnosis? A. 8 August
B. 25 July
A. Focal right-sided pneumonia C. 22 August
B. Bronchial asthma D. 11 July
C. Acute bronchitis E. 5 September
D. Pulmonary carcinoma
E. Pulmonary gangrene 15. A 40-year-old female patient has been
hospitalized for attacks of asphyxia, cough
11. A 45-year-old woman, mother of four with phlegm. She has a 4-year history of
children, comes to the emergency room the disease. The first attack of asphyxia
complaining of a sudden onset of the epi- occurred during her stay in the countrysi-
gastric and right upper quadrant pain, de. Further attacks occurred while cleani-
radiating to the back, accompanied by ng the room. After 3 days of inpatient
vomiting. On examination, tenderness is treatment the patient’s condition has si-
elicited in the right upper quadrant, bowel gnificantly improved. What is the most
sounds are decreased, and laboratory data likely etiological factor?
shows leukocytosis, normal serum levels
of amylase, lipase, and bilirubin. The most A. Household allergens
likely diagnosis is: B. Pollen
C. Infectious
A. Acute cholecystitis D. Chemicals
B. Perforated peptic ulcer disease E. Psychogenic
C. Myocardial infarction
D. Sigmoid diverticulitis 16. A 3-year-old child has been admi-
E. Acute pancreatitis tted to a hospital because of ostealgia
and body temperature rise up to 39o C.
12. During an operation for presumed Objectively: the patient is in grave condi-
appendicitis the appendix was found to tion, unable to stand for ostealgia, there
be normal; however, the terminal ileum is apparent intoxication, lymph nodesare
is evidently thickened and feels rubbery, enlarged up to 1,5 cm. Liver can be
its serosa is covered with grayish-white palpated 3 cm below the costal margin,
exudate, and several loops of apparently spleen - 2 cm below the costal margin. In
normal small intestine are adherent to it. blood: RBCs - 3, 0·1012 /l, Hb- 87 g/l, colour
The most likely diagnosis is: index - 0,9, thrombocytes - 190 · 109 /l,
WBCs - 3, 2 · 109 /l, eosinophils - 1, stab
A. Crohn’s disease of the terminal ileum neutrophils - 1, segmented neutrophils -
B. Perforated Meckel’s diverticulum 0, lymphocytes - 87, monocytes - 2, ESR
C. Ulcerative colitis - 36 mm/h. What examination should
D. Ileocecal tuberculosis be conducted in order to specify the di-
E. Acute ileitis agnosis?
13. A girl is 12-year-old. Yesterday she was A. Sternal puncture
overcooled. Now she is complaining on B. Ultrasound
pain in suprapubic area, frequent painful C. Lymph node puncture
urination by small portions, temperature D. Lymph node biopsy
is 37, 8oC. Pasternatsky symptom is E. Computer tomography
negative. Urine analysis: protein - 0,033
Krok 2 Medicine 2012 3

17. A 22-year-old girl has been complai- A. Hypothermia


ning of having itching rash on her face B. Acute cardiovascular insufficiency
for 2 days. She associates this disease wi- C. Apparent death
th application of cosmetic face cream. D. Frostbite of trunk and extremities
Objectively: apparent reddening and E. -
edema of skin in the region of cheeks,
chin and forehead; fine papulovesicular 21. A 28-year-old parturient complai-
rash. What is the most likely diagnosis? ns about headache, vision impairment,
psychic inhibition. Objectively: AP-
A. Allergic dermatitis 200/110 mm Hg, evident edemata of
B. Dermatitis simplex legs and anterior abdominal wall. Fetus
C. Eczema head is in the area of small pelvis. Fetal
D. Erysipelas heartbeats is clear, rhythmic, 190/min.
E. Neurodermatitis Internal examination revealed complete
cervical dilatation, fetus head was in the
18. A 16-year-old patient who has a hi- area of small pelvis. What tactics of labor
story of intense bleedings from minor management should be chosen?
cuts and sores needs to have the roots
of teeth extracted. Examination reveals A. Forceps operation
an increase in volume of the right knee B. Cesarean
joint, limitation of its mobility. There are C. Embryotomy
no other changes. Blood analysis shows D. Conservative labor management with
an inclination to anaemia (Hb- 120 g/l). episiotomy
Before the dental intervention it is requi- E. Stimulation of labor activity
red to prevent the bleeding by means of:
22. A 35-year-old patient complains about
A. Cryoprecipitate pain and morning stiffness of hand joi-
B. Epsilon-aminocapronic acid nts and temporomandibular joints that
C. Fibrinogen lasts over 30 minutes. She has had these
D. Dried blood plasma symptoms for 2 years. Objectively: edema
E. Calcium chloride of proximal interphalangeal digital joi-
nts and limited motions of joints. What
19. A 44-year-old patient complai- examination should be administered?
ns about difficult urination, sensati-
on of incomplete urinary bladder A. Roentgenography of hands
emptying. Sonographic examination of B. Complete blood count
the urinary bladder near the urethra C. Rose-Waaler reaction
entrance revealed an oval well-defined D. Immunogram
hyperechogenic formation 2x3 cm large E. Proteinogram
that was changing its position during the
examination. What conclusion can be 23. A patient, aged 16, complains of
made? headache, mainly in the frontal and
temporal areas, superciliary arch, appeari-
A. Concrement ng of vomiting at the peak of headache,
B. Malignant tumour of the urinary pain during the eyeballs movement, joint’s
bladder pain. On examination: excited, to - 39o C,
C. Urinary bladder polyp Ps- 110/min. Tonic and clonus cramps.
D. Prostate adenoma Uncertain meningeal signs. What is the
E. Primary ureter tumour most likely diagnosis?

20. An emergency team has delivered to A. Influenza with cerebral edema mani-
a hospital an unconscious patient found festations
lying in the street in winter. Objectively: B. Influenza, typical disease duration
the patient is pale, with superficial respi- C. Respiratory syncytial virus
ration; bradycardia with heartrate 54/min, D. Parainfluenza
to - 35, 0oC. AP- 100/60 mm Hg. Palpati- E. Adenovirus infection
on of chest and abdomen revealed no
peritoneal symptoms. There is a smell of 24. A 29-year-old female patient complai-
alcohol from the patient’s mouth. What is ns of dyspnea, heaviness and chest pain
the most likely diagnosis? on the right, body temperature rise up to
37, 2oC. The disease is associated with a
chest trauma received 4 days ago. Objecti-
vely: skin is pale and moist. Ps- 90 bpm,
Krok 2 Medicine 2012 4

regular. Palpation reveals a dull sound on rates?


the right, auscultation reveals significantly
weakened vesicular breathing. In blood: A. Relative values
RBCs - 2, 8 · 1012 /l, colour index - 0,9, Hb- B. Standard values
100 g/l, WBCs - 8, 0 · 109/l, ESR - 17 mm/h. C. Average values
What results of diagnostic puncture of the D. Absolute values
pleural cavity can be expected? E. Dynamic series

A. Haemorrhagic punctate 29. A female patient has been sufferi-


B. Chylous liquid ng from pain in the right subcostal area,
C. Exudate bitter taste in the mouth, periodical bile
D. Transudate vomiting for a month. The patient put off
E. Purulent punctate 12 kg. Body temperature in the evening
is 37, 6oC. Sonography revealed that bi-
25. A 54-year-old drowned man was le bladder was 5,5х2,7 cm large, its wall -
rescued from the water and delivered 0,4 cm, choledochus - 0,8 cm in diameter.
to the shore. Objectively: the man is Anterior liver segment contains a roundi-
unconscious, pale, breathing cannot be sh hypoechoic formation up to 5 cm in
auscultated, pulse is thready. Resuscitati- diameter and another two up to 1,5 cm
on measures allowed to save the patient. each, walls of these formations are up to
What complications may develop in the 0,3 cm thick. What is the most likely di-
near future? agnosis?
A. Pulmonary edema A. Alveolar echinococcus of liver
B. Respiratory arrest B. Liver cancer
C. Encephalopathy C. Liver abscess
D. Cardiac arrest D. Cystous liver cancer
E. Bronchospasm E. Paravesical liver abscesses
26. A 46-year-old patient once took 30. A 55-year-old male had been treated
part in elimination of breakdown at an at the surgical department for acute lower-
atomic power plant. Currently he is bei- extremity thrombophlebitis. On the 7th
ng treated at an in-patient hospital. He day of treatment he suddenly developed
was diagnosed with progressing vegetati- pain in the left part of chest, dyspnea
ve insufficiency. This disease relates to and cough. Body temperature was 36, 1o C,
the following group of ionizing radiation respiratory rate - 36/min. The patient was
effects: also found to have diminished breath
sounds without wheezing. Ps- 140/min,
A. Somato-stochastic thready. AP- 70/50 mm Hg. The ECG
B. Somatic shows Q ÁÁ-S1 syndrome. What is the most
C. Genetic likely diagnosis?
D. Hormesis
E. Heterosis A. Pulmonary embolism
B. Myocardial infarction
27. A military unit stopped for 3-day’s rest C. Cardiac asthma
in an inhabited locality after a long march. D. Bronchial asthma
The sanitary-epidemiological reconnai- E. Pneumothorax
ssance found several water sources. It is
necessary to choose the source complying 31. A 16-year-old adolescent was vacci-
with the hygienic standards for drinking nated with DTP. In eight days there was
water in the field conditions: stiffness and pain in the joints, subfebri-
le temperature, urticarial skin eruption,
A. Artesian well water enlargement of inguinal, cervical lymph
B. Spring water nodes and spleen. What kind of allergic
C. River water reaction is observed?
D. Rain water
E. Melt snow water A. Immunocomplex
B. Hypersensitivity of immediate type
28. Deputy of chief medical officer carried C. Cytoxic
out a study of morbidity rate for populati- D. Hypersensitivity of delayed type
on which had been served at the polyclini- E. -
cs within the last 5 years. What statistical
values can help in calculation of morbidity 32. A 56-year-old scientist experiences
Krok 2 Medicine 2012 5

constricting retrosternal pain several ti- A. Ankylosing spondylitis


mes a day while walking for 100-150 m. B. Coxarthrosis
The pain lasts for up to 10 minutes and can C. Rheumatoid arthritis
be relieved by nitroglycerine. Objectively: D. Reiter’s disease
the patient is overweight, heart borders E. Spondylosis
exhibit no abnormalities, heart sounds are
rhythmic, Ps- 78 bpm, AP- 130/80 mm Hg. 36. A 58-year-old female patient complai-
ECG contains low amplitude of T wave in ns about periodical headache, dizziness
V4−5 . What disease might be suspected? and ear noise. She has been suffering from
diabetes mellitus for 15 years. Objecti-
A. Stable FC III stenocardia vely: heart sounds are rhythmic, heart rate
B. Instable stenocardia is 76/min, there is diastolic shock above
C. Stable FC I stenocardia aorta, AP is 180/110 mm Hg. In urine:
D. Stable FC II stenocardia OD- 1,014. Daily loss of protein with uri-
E. Stable FC IV stenocardia ne is 1,5 g. What drug should be chosen
for treatment of arterial hypertension?
33. In autumn a 25-year-old patient
developed stomach ache arising 1,5-2 A. Ihibitor of angiotensin converting
hours after having meals and at night. enzyme
He complains of pyrosis and constipation. B. β-blocker
The pain is getting worse after consumi- C. Calcium channel antagonist
ng spicy, salty and sour food, it can be D. Thiazide diuretic
relieved by means of soda and hot-water E. α-blocker
bag. The patient has been suffering from
this disease for a year. Objectively: furred 37. At the radiological unit of a hospi-
moist tongue. Abdomen palpation reveals tal gamma-devices of radiotherapy of
epigastrial pain on the right, resistance "Agat"type and other closed sources of
of abdominal muscles in the same region. ionizing radiation are used for treatment
What is the most likely diagnosis? of malignant neoplasms. What measures
are to be taken to protect personnel duri-
A. Duodenal ulcer ng working with radioactive sources of
B. Chronic cholecystitis such type?
C. Diaphragmatic hernia
D. Stomach ulcer A. Reduction of working time and screeni-
E. Chronic pancreatitis ng of the source
B. Capsulation of devices and organization
34. A secundipara has regular birth activi- of room ventilation
ty. Three years ago she had cesarean secti- C. Screening of the source and the use of
on for the reason of acute intrauterine means of individual protection of respirati-
hypoxia. During parodynia she complai- on organs
ns of extended pain in the area of D. The increase of distance to the source
postsurgical scar. Objectively: fetus pulse and individual hygiene compliance
is rhythmic - 140 bpm. Vaginal examinati- E. Systematical cleansing of surfaces
on shows 5 cm cervical dilatation. Fetal from the radioactive contamination and
bladder is intact. What is the tactics of shortening of working time
choice?
38. A 20-year-old adolescent lives in
A. Cesarean section the nidus of tuberculous infection. The
B. Augmentation of labour tuberculine Mantoux test with 2 TU
C. Obstetrical forceps was determined as hyperergic. What si-
D. Waiting tactics of labor management gns determine the hyperergic test of this
E. Vaginal delivery adolescent?
35. A 32-year-old male patient has been A. 6 mm papula, necrosis
suffering from pain in the sacrum and B. 20 mm papula
coxofemoral joints, painfulness and sti- C. 24 mm hyperemia
ffness in the lumbar spine for a year. ESR- D. 4 mm papula
56 mm/h. Roentgenography revealed E. 12 mm hyperemia
symptoms of bilateral sacroileitis. The
patient is the carrier of HLA B27 anti- 39. A survey radiograph of a miner
gen. What is the most likely diagnosis? (24 years of service record, the dust
concentration in the workplace is at the
rate of 260-280 mg/m3 with 15% of free
Krok 2 Medicine 2012 6

silica) shows lung changes that are typi- 43. To study physical development of chi-
cal for pneumoconiosis. What type of ldren and adolescents, anthropometric
pneumoconiosis is it? studies are widely used. Choose a physi-
ometric method of study from the below
A. Anthracosilicosis given.
B. Carboconiosis
C. Silicatosis A. Determination of vital capacity of lungs
D. Anthracosilicatosis B. Measurement of growth
E. Silicosis C. Determination of thorax form
D. Determination of vertebra form
40. A 5-year-old child had an attack E. Determination of body weight
of palpitation with nausea, dizziness,
generalized fatigue. On ECG: tachycardia 44. An electro-gas welding operator
with heartbeat rate of 220/min. Ventricle working at a machine workshop performs
complexes are deformed and widened. P welding and cutting of metal, which is
wave is absent. What medication is to be accompanied by intense UV-radiation.
prescribed to provide first aid? His welding station is equipped with
effective mechanical ventilation. What
A. Lydocain occupational disease is most likely
B. Isoptin to develop in an electro-gas welding
C. Seduxen operator?
D. Novocainamides
E. Strophantin A. Photoelectric ophthalmia
B. Heatstroke
41. A 57-year-old man complains of C. Vegetative-vascular dystonia
shortness of breath, swelling on shanks, D. Chronic overheating
irregularity in cardiac work, pain in the E. Pneumoconiosis
left chest half with irradiation to the
left scapula.Treatment is uineffective. On 45. A 60-year-old female patient had
physical exam: heart’s sounds are dimini- been admitted to a hospital for acute
shed, soft systolic murmur on the apex. transmural infarction. An hour later
Ps - 100/min, arrhythmical, BP - 115/75 the patient’s contition got worse. She
mm Hg. The liver is +2 cm, painful. developed progressing dyspnea, dry
Roentgenoscopy: enlargement of heart cough. Respiratory rate - 30/min, heart
shadow to all sides, pulsation is weak. rate - 130/min, AP- 90/60 mm Hg. Heart
Electrocardiogram (ECG): leftventricled sounds were muffled, there was also di-
extrasystolia, decreased voltage. What astolic shock on the pulmonary artery.
method of investigation is necessary to The patient presented with medium moi-
do to determine the diagnosis? st rales in the lower parts of lungs on the
right and on the left. Body temperature -
A. Echocardiography 36, 4oC. What drug should be given in the
B. Veloergometria first place?
C. X-ray kymography
D. ECG in the dynamics A. Promedol
E. Coronarography B. Aminophylline
C. Dopamine
42. A 27-year-old woman presents at D. Heparin
the maternity welfare centre because of E. Digoxin
infertility. She has had sexual life in marri-
age for 4 years, doesn’t use contraceptives. 46. A 65-year-old patient complains of
She hasn’t get pregnant. On examination: gradual worsening of the left eye vision
genital development is without pathology, during 10 months. On physical examinati-
uterine tubes are passable, basal (rectal) on: acuty of vision of the left eye is 0,01,
temperature is one-phase during last 3 not correctable. The eye is quiet, pupil
menstrual cycles. What is the infertility of the eye is grayish, reflex from the eye-
cause? ground is absent. Intraocular pressure is
18 mm/Hg. What is the most probable
A. Anovular menstrual cycle preliminary diagnosis?
B. Chronic adnexitis
C. Abnormalities in genital development
D. Immunologic infertility
E. Genital endometriosis
Krok 2 Medicine 2012 7

A. Senile cataract A. Acute pulmonary abscess


B. Open-angle glaucoma B. Exudative pleuritis
C. Disorder of blood circulation in retina C. Acute focal pneumonia
vessels D. Pleural empyema
D. Leukoma of the cornea E. Pyopneumothorax
E. Exfoliation of the retina
50. A 25-year-old woman complains of
47. A patient is on the sick leave for 4 profuse foamy vaginal discharges, foul,
months continuously from the date of burning and itching in genitalia region.
injury. The treatment is going to last for She has been ill for a week. Extramarital
1-2 months. Who has the right to extend sexual life. On examination: hyperemia
the duration of medical certificate for this of vaginal mucous, bleeding on touchi-
patient? ng, foamy leucorrhea in the urethral area.
What is the most probable diagnosis?
A. Medical advisory commission after
medico-social expert commission exami- A. Trichomonas colpitic
nation B. Gonorrhea
B. Medical superintendent C. Chlamydiosis
C. Medical advisory commission after D. Vagina candidomicosis
inpatient treatment E. Bacterial vaginosis
D. District doctor by agreement with a
department chief 51. A 50-year-old locksmith has a long-
E. Medico-social expert commission term record of work under the effect
of mercury vapors with concentration
48. A child from the first non-complicated exceeding MPC by 5-10 times. Clini-
pregnancy but complicated labor had cal examination revealed the lability of
cephalhematoma. On the second day vasomotors of skin, pulse and arterial
there developed jaundice. On the 3th day pressure; total hyperhydrosis; asymmetric
appeared changes of neurologic status: innervation of facial and lingual muscles,
nystagmus, Graefe’s sign. Urea is yellow, positive subcortical reflexes, intenti-
feces- golden-yellow. Mother’s blood on tremor. Against the background of
group is ü(II)Rh− , child- ü(II)Rh+ . increased emotional excitability the pati-
On the third day child’s Hb- 200 g/L, ent presents with lack of self-confidence,
RBC- 6, 1 · 1012 /L, bilirubin in blood - shyness. A dentist found him to have
58 mk mol/L due to unconjugated bili- parodontosis, chronic stomatitis. What di-
rubin, Ht- 0,57. What is the child’s jaundi- sease can be suspected?
ce explanation?
A. Chronic mercury intoxication
A. Brain delivery trauma B. Residual effects of neuroinfection
B. Physiologic jaundice C. Parkinson’s syndrome
C. Hemolytic disease of newborn D. Acute mercury intoxication
D. Bile ducts atresia E. Vascular encephalopathy
E. Fetal hepatitis
52. 4 hours after having meals a patient
49. On the 4th day after recovering from with signs of malnutrition and steatorrhea
a cold a patient was hospitalized with experiences stomach pain, especially
complaints of solitary spittings of mucoid above navel and to the left of it. Diarrheas
sputum. On the 2nd day there was a si- take turns with constipation lasting up to
ngle discharge of about 250 ml of purulent 3-5 days. Palpation reveals moderate pai-
blood-streaked sputum. Objectively: the nfulness in the choledochopancreatic regi-
patient’s condition is moderately severe. on. The amylase rate in blood is stable.
Respiratory rate - 28-30/min, Ps- 96 bpm, X-ray reveals some calcifications located
AP- 110/70 mm Hg. Respiration above the above navel. What is the most likely di-
left lung is vesicular, weak above the right agnosis?
lung. There are moist rales of different
types above the lower lobe and amphoric A. Chronic pancreatitis
breath near the angle of scapula. What is B. Chronic gastroduodenitis
the most likely diagnosis? C. Duodenal ulcer
D. Zollinger-Ellison syndrome
E. Chronic calculous cholecystitis
53. A 54-year-old patient complains
of weakness, weight loss despite the
Krok 2 Medicine 2012 8

unchanged appetite, frequent urinati- emergent therapy.


on, skin itch for six months. Some time
ago the patient underwent treatment for A. Zovirax
furunculosis. She hasn’t been examined B. Glucocorticoids
recently. Objectively: malnutrition, dry C. Cephtriaxon
skin with signs of scratching. Small lymph D. Lasix
nodes can be palpated in the axillary regi- E. Hemodesis
ons. Changes in the internal organs are
absenr. What testing must be administered 57. A 30-year-old patient was delivered
in the first place? to the admission ward of the infecti-
ous disease department. The disease had
A. Blood sugar test on an empty stomach started acutely on the background of
B. Complete blood count normal temperature with the appearance
C. Endoscopy of stomach of frequent, liquid, profuse stool without
D. Lymph node biopsy pathological impurities. Diarrhea was not
E. Blood sterility testing accompanied by abdominal pain. 12 hours
later there appeared recurrent profuse
54. A woman consulted a doctor on vomiting. The patient rapidly developed
the 14th day after labour about sudden dehydration. What is the most likely di-
pain, hyperemy and induration of the left agnosis?
mammary gland, body temperature ri-
se up to 39o C, headache, indisposition. A. Cholera
Objectively: fissure of nipple, enlargement B. Shigellosis
of the left mammary gland, pain on C. Staphylococcal food toxicoinfection
palpation. What pathology would you thi- D. Salmonellosis
nk about in this case? E. Campylobacteriosis

A. Lactational mastitis 58. A 67-year-old male complains


B. Lacteal cyst with suppuration of dyspnea on exertion, attacks of
C. Fibrous adenoma of the left mammary retrosternal pain, dizziness. He has no
gland history of rheumatism. Objectively: pale
D. Breast cancer skin, acrocyanosis. There are rales in the
E. Phlegmon of mammary gland lower parts of lungs. There is systolic thri-
ll in the II intercostal space on the right,
55. Several hours before, a 28-year- coarse systolic murmur conducted to the
old patient suddenly developed acute vessels of neck. AP- 130/90 mm Hg, heart
headache and repeated vomiting, then rate - 90/min, regular rhythm. The liver
lost consciousness. Objectively: focal extends 5 cm under the edge of costal
neurological symptoms were not found. arch, shin edemata are present. Specify
Pronounced meningeal symptoms were the assumed valvular defect:
revealed. AP - 120/80 mm Hg. Accordi-
ng to clinical and liquorological findi- A. Aortic stenosis
ngs the patient was diagnosed with B. Pulmonary artery stenosis
subarachnoid haemorrhage. After admi- C. Mitral insufficiency
nistration of dehydrants the patient’s D. Ventricular septal defect
condition somewhat improved. What is E. Tricuspid regurgitation
the main component of further emergency
care? 59. A 24-year-old female teacher complai-
ns of dizziness and heart pain irradiati-
A. Coagulants ng to the left nipple. Pain is not associ-
B. Anticoagulants ated with physical activity and cannot
C. Antiaggregants be relieved by nitroglycerin, it abates
D. Fibrinolytics after taking Valocordin and lasts an hour
E. Corticosteroids or more. The patient has a nearly 2-
year history of this disease. Objectively:
56. A 24-year-old man on the 5th day Ps- 76 bpm. AP- 110/70 mm Hg. Heart
of acute respiratory disease with high borders are norma, heart sounds are clear.
grade temperature started having strong The ECG shows respiratory arrhythmia.
headaches, systemic dizziness, sensati- Radiograph of the cervicothoracic spi-
on of double vision, paresis of mimic ne shows no pathology. Lungs, abdomen
muscles to the right, tickling by swallowi- are unremarkable. What changes in blood
ng. Diagnosis: Acute viral encephalitis. formula can be expected?
Determine the basic direction of the
Krok 2 Medicine 2012 9

A. No changes
B. Leukocytosis A. Consultation of an expert in narcology
C. Thrombocytopenia B. Prescription of medications the patient
D. Leukemic hiatus asks for
E. Increased ESR C. Additional consultation of surgeon
D. Treatment with antibiotics
60. A 51-year-old female patient complai- E. Consultation of infectious diseases
ns of frequent defecation and liquid doctor
blood-streaked stools with mucus admi-
xtures, diffuse pain in the inferolateral 63. A 1,5-year-old child fell ill acutely
abdomen, 6 kg weight loss over the with high temperature 38o C, headache,
previous month. Objectively: body fatigue. The temperature declined on the
temperature - 37, 4oC, malnutrition, skin fifth day, muscular pain in the right leg
is pale and dry. Abdomen is soft, sigmoid occured in the morning, there were no
is painful and spasmodic, makes a rumbli- movements and tendon reflexes, sensiti-
ng sound. Liver is dense, painful, extends vity was reserved. What is the initial di-
3 cm below the costal margin. What is the agnosis?
most likely diagnosis?
A. Polyomyelitis
A. Non-specific ulcerative colitis B. Viral encephilitis
B. Bacillary dysentery C. Polyartropathy
C. Sprue D. Osteomyelitis
D. Intestinal enzymopathy E. Hip joint arthritis
E. Helminthic invasion
64. Development of chronic venous
61. A 32-year-old female complains of di- insufficiency of lower extremities depends
zziness, headache, palpitation, tremor. For on the functional condition of so-called
the last several months she has been under musculovenous pump. This term refers to
outpatient monitoring for the increased the following group of muscles:
arterial pressure. Since recently such
attacks have become more frequent and A. Shin muscles
severe. Objectively: skin is covered with B. Abdominal wall muscles
clammy sweat, tremor of the extremities C. Buttock region muscles
is present. HR- 110/min, AP- 220/140 mm D. Thigh muscles
Hg. Heart sounds are weakened. In blood: E. Foot muscles
WBCs - 9, 8 · 109 /l, ESR - 22 mm/h. Blood 65. A 7-year-old child was brought to
glucose - 9,8 millimole/l. What disease is a doctor for a check. The child has
the most likely cause of this crisis? a 4-year history of bronchial asthma,
A. Pheochromocytoma asthma attacks occur mainly in spri-
B. Essential hypertension ng and summer. Allergy tests revealed
C. Preeclampsia hypersensitivity to poplar seed tufts, fi-
D. Primary hyperaldosteronism eld herbs. What recommendation should
E. Diabetic glomerulosclerosis be given?

62. A 19-year-old woman complains of A. Specific hyposensitization


pain in the abdomen and joints, asks B. Physiotherapy
for more analgetics and somnifacient C. Treatment at a health resort
injections. The patient was examined. D. Phytotherapy
Gynecological and urological pathologi- E. Needle reflexotherapy
es are absent. There are signs of previous 66. A farmer hurt his right foot duri-
punctures along superficial veins of the ng working in a field and came to the
extremities. The patient does not explain emergency station. He doesn’t remember
the origin of punctures. Tendon reflexes of when he got last vaccination and he has
upper and lower extremities are the same, never served in the army. Examination
quick. Photoreaction of the pupil of the of his right foot revealed a contaminated
eye is weak. The tongue is grey coated. wound up to 5-6 cm long with uneven
During communication the patient in edges. The further treatment tactics will
affectively not even-tempered. There is be:
diarrhea without pathologic inclusions.
What tactics is necessary to improve the
condition of this patient?
Krok 2 Medicine 2012 10

A. To make an injection of tetanus vely: there are small red papules set
anatoxin and antitetanus serum mostly in pairs in the region of interdigital
B. To make an injection of tetanus folds on both hands, on the flexor surface
anatoxin of radicarpal articulations, abdomen and
C. To make an injection of antitetanus buttock skin as well as internal surface
serum of thighs. In the centre of some papules
D. Surgical d-bridement only vesicles or serohaemorrhagic crusts can
E. To administer an antibiotic be seen. There are multiple excoriations.
What is the most likely diagnosis?
67. A 50-year-old patient complains about
having pain attacks in the right subcostal A. Scabies
area for about a year. He pain arises mai- B. Dermatitis
nly after taking fattening food. Over the C. Ringworm of body
last week the attacks occurred daily and D. Toxicoderma
became more painful. On the 3rd day of E. Eczema
hospitalization the patient presented with
icteritiousness of skin and scleras, light- 71. A 43-year-old female patiet complai-
colored feces and dark urine. In blood: ns of eruption on her right leg skin, pain,
neutrophilic leukocytosis - 13, 1 · 109 /l, weakness, body temperature rise up to
ESR- 28 mm/h. What is the most likely 38oC. The disease is acute. Objectively:
diagnosis? there is an edema on the right leg skin in
the region of foot, a well-defined bright
A. Chronic calculous cholecystitis red spot in form of flame tips which feels
B. Chronic recurrent pancreatitis hot. There are isolated vesicles in focus.
C. Fatty degeneration of liver What is your provisional diagnosis?
D. Chronic cholangitis, exacerbation stage
E. Hypertensive dyskinesia of gallbladder A. Erysipelas
B. Microbial eczema
68. In morgue there are dead bodi- C. Contact dermatitis
es with the following causes of death: D. Toxicoderma
electrotrauma; rupture of the spleen wi- E. Haemorrhagic vasculitis
th acute anemia. There is one unknown
person; one ethyl alcohol poisoned person 72. A 47-year-old patient came to see a
and one drowned man. What dead body doctor on the 7th day of disease. The di-
should the blood group be determined sease developed very fast: after the chill
for? body temperature rose to 40oC and lasted
up to 7 hours, then dropped abruptly,
A. All dead bodies of the unknown persons which caused profuse sweat. There were
B. Body of poisoned person three such attacks occuringonce in two
C. Body of drowned man days. Two days ago the patient arrived
D. Body of person with internal from Africa. Objectively: pale skin, subi-
hemorrhage cteric sclera, significantly enlarged liver
E. Body of person with a sudden death and spleen. What is the cause of fever
attacks in this disease?
69. A 36-year-old patient complains of
skin rash that appeared a week ago and A. Erythrocytic schizogony
doesn’t cause any subjective problems. B. Tissue schizogony
Objectively: palm and sole skin is covered C. Exotoxin of a causative agent
with multiple lenticular disseminated D. Endotoxin of a causative agent
papules not raised above the skin level. E. Gametocytes
The papules are reddish, dense on palpati-
on and covered with keratinous squamae. 73. On the 2 nd day of illness a 27-year-
What is the provisional diagnosis? old patient complains of the unbearable
headache, repeated vomiting. Objecti-
A. Secondary syphilis vely: the patient is in a grave condition.
B. Verrucosis He is conscious but adynamic. Lies in a
C. Palmoplanar psoriasis forced position with his head thrown back.
D. Palmoplanar rubrophytosis There is no skin rash. Occipital muscles
E. Palm and sole callosity are evidently rigid, there are Kernig’s
and Brudzinski’s signs. to - 39, 5o C, Ps-
70. A 5-grade pupil complains about 120/min., AP- 130/80 mm Hg. What is the
extensive skin rash accompanied by reason for the leading syndrome of this
intensive itch, especially at night. Objecti- disease?
Krok 2 Medicine 2012 11

ng the divorce. On examination: palm


A. Liquor hypertension hyperhydrosis, pulse rate- 72-78 bpm, labi-
B. Liquor hypotension le, heart without changes. ECG is normal.
C. Affection of the cranial nerve nuclei What is the most probable pathology in
D. Haemorrhages into the adrenal glands this case?
E. Hyperthermy
A. Neurasthenia
74. A patient has chronic heart fai- B. Ipochondric neurosis
lure of the II stage. He takes furosemi- C. Compulsive neurosis
de regularly three times a week. D. Schizophrenia
He had developed bronchopneumonia E. Depressive neurosis
and had been administered combined
pharmacotherapy. On the fifth day of 78. A 30-year-old woman with a long hi-
therapy the patient complained of hearing story of chronic pyelonephritis complains
impairment. What drug coadministered about considerable weakness, sleepiness,
with furosemide might have caused the decrease in diuresis down to 100 ml per
hearing loss? day. AP- 200/120 mm Hg. In blood: creati-
nine - 0,62 millimole/l, hypoproteinemia,
A. Gentamicin albumines - 32 g/l, potassium - 6,8 milli-
B. Linex mole/l, hypochromic anemia, increased
C. Nystatin ESR. What is the first step in the pati-
D. Tavegil ent treatment tactics?
E. Mucaltin
A. Haemodialysis
75. A 54-year-old female patient has been B. Antibacterial therapy
admitted to a hospital 12 days after the C. Enterosorption
beginning of acute pancreatitis. Objecti- D. Haemosorption
vely: the patient is in grave condition. The E. Blood transfusion
body temperature is hectic. Ps - 112 bpm.
The abdomen is swollen. Epigastrium 79. A 10-year-old girl consulted a doctor
palpation reveals a very painful infiltrati- about thirst, frequent urination, wei-
on falling in the localization of pancreas. ght loss. She has been observing these
Abdominal cavity contains fluid. There symptoms for about a month. Objecti-
is an edema of the left lumbar region. In vely: no pathology of internal organs was
blood: WBCs - 18·109 /l. What is the requi- revealed. What laboratory analysis should
red tactics of the patient’s treatment? be carried out in the first place?
A. Surgical treatment A. Blood glucose analysis on an empty
B. Further conservative treatment stomach
C. Peritoneal dialysis B. Glucose in urine test on the base of
D. Increase in antienzymes daily diuresis
E. Massive antibacterial therapy C. Acetone in urine test
D. Glucose tolerance test
76. To replace the blood loss replacement E. Glucosuric profile
1000 ml of the same group of Rhesus-
compatible donated blood was transfused 80. A 36-year-old female patient complai-
to the patient. The blood was conserved ns of bruises on the body, gingival
by sodium citrate. At the end of haemorrhage, general weakness. A month
hemotransfusion there appeared exci- ago she had a severe domestic poisoni-
tement, pale skin, tachycardia, muscles ng with some pesticide (the patient can
cramps in the patient. What complicati- not remember the name). She has a 7-
on should be suspected? year record of working in contact with
petroleum products, particularly benzene.
A. Citrate intoxication In blood: RBCs - 3, 2 · 1012 /l, WBCs -
B. Citrate shock 2, 7 · 109 /l, thrombocytes - 70 · 109 /l. What
C. Allergic reaction is the most likely pathology?
D. Anaphylactic shock
E. Pyrogenous reaction
77. A 24-year-old emotionally-labile
woman presents with irritation, depressed
mood, palpitation, shooting pain in the
heart area, generalized fatigue followi-
Krok 2 Medicine 2012 12

A. Benzene intoxication convulsions. Objectively: the patient is


B. Organophosphorus pesticide intoxicati- unconscious, the skin is pale, extremities
on are cold. AP - 90/60 mm Hg, Ps- 96/min,
C. Organochlorine pesticide Intoxication deficient, breathing is shallow. Pupillary
D. Mercury-containing pesticide intoxi- and tendon reflexes are present. There are
cation no pathological signs. What is the most li-
E. Chronic fatigue Syndrome kely diagnosis?
81. A 6-year-old child complains of A. Syncope
frequent liquid stool and vomiting. On the B. Vegetovascular paroxysm
2nd day of desease the child presented C. Epileptic attack
with inertness, temperature rise up to D. Hysterical neurosis
38, 2oC, Ps- 150 bpm, scaphoid abdomen, E. Transient ischemic attack
palpatory painful sigmoid colon, defecati-
on 10 times a day with liquid, scarce stool 85. A patient complains of impaired far vi-
with mucus and streaks of green. What is sion. Previously his eyes often turned red
a provisional diagnosis? and hurt. Objectively: the eyes are not irri-
tated, the cornea is transparent, anterior
A. Shigellosis chambers are median deep, their moisture
B. Salmonellosis is transparent. The iris of the right eye
C. Escherichiosis has not changed in colour, its pattern is
D. Intestinal amebiasis unchanged. The pupil is of irregular shape,
E. Yersiniosis scalloped. Biomicroscopy of the crystalli-
ne lens reveals the areas of opacity and
82. A 68-year-old patient consulted vacuoles. Make a diagnosis:
a doctor about a tumour in her left
mammary gland. Objectively: in the upper A. Complicated cataract of the right eye
internal quadrant of the left mammary B. Senile cataract of the right eye
gland there is a neoplasm up to 2,5 cm C. Diabetic cataract of the right eye
in diameter, dense, uneven, painless on D. Tetanic cataract of the right eye
palpation. Regional lymph nodes are not E. Radiation cataract of the right eye
enlarged. What is the most likely di-
agnosis? 86. A 26-year-old patient complains of
experiencing pain in the right hand for 4
A. Cancer days. The pain arose at the site of corns
B. Cyst on the palmar surface at the base of
C. Fibroadenoma the II and III fingers. Some time later
D. Mastopathy the dorsum of hand became edematic.
E. Lipoma I and III fingers are half-bent in the
interphalangeal joints, the hand looks like
83. A 65-year-old patient complains of "rake". What is the most likely diagnosis?
pain in the lumbar spine, moderate di-
suria. He has been suffering from these A. Phlegmon of the second interdigital
presentations for about half a year. space of the right hand
Prostate volume is 45 cm3 (there are B. Adenophlegmon of the right hand
hypoechogenic nodes in both lobes, C. Corn abscess of the right hand
capsule invasion). The rate of prostate- D. U-shaped phlegmon of the right hand
specific antigen is 60 ng/l. Prostate biopsy E. Tendovaginitis
revealed an adenocarcinoma. Which of
the supplemental examination methods 87. A 40-year-old female patient complain
will allow to determine the stage of of headache, dizziness, muscle weakness,
neoplastic process in this patient? sometimes - cramps in the extremities. She
has been taking antihypertensive medi-
A. Computer tomography of pelvis cations for 10 years. AP- 180/100 mm Hg.
B. Roentgenography of lumbar spine Blood potassium - 1,8 millimole/l, sodium
C. Excretory urography - 4,8 millimole/l. In urine: alkaline reacti-
D. Bone scintigraphy on, the relative density - 1012, protein and
E. Roentgenography of chest sugar are not found, WBCs - 3-4 in the fi-
eld of vision, RBCs - 1-2 in the field of visi-
84. While staying in a stuffy room a 19- on. Conn’s syndrome is suspected. Which
year-old emotionally labile girl developed drug should be chosen for the treatment
severe weakness, dizziness, blackout, of arterial hypertension?
nausea and loss of consciousness without
Krok 2 Medicine 2012 13

A. Spironolactone A. Splenectomy
B. Propanolol B. Iron preparations
C. Enalapril C. Hemotransfusion
D. Hydrochlorothiazide D. Cytostatics
E. Clonidine E. Vitamin B12
88. An 18-year-old patient presents no 92. A woman while working in vegetable
problems. Percussion reveals that heart garden developed severe pain in the loin.
borders are displaced to the right and Lasague’s and Nery tension signs are
left by 1 cm, there is a coarse systolic obviously marked on the right. Lumbar
murmur with its epicenter within the 4th lordosis is smoothed, movements are
intercostal space on the left. What is the harshly restrained in lumbar part of the
most informative examination to confirm spine. Right ankle (Achilles) reflex is
the clinical diagnosis? absent. What kind of disease can it be?
A. Ventriculography A. Lumbar-sacral radiculitis
B. ECG B. Lumbalgia
C. PCG C. Hepatic colic
D. Echocardiography D. Renal colic
E. Polycardiography E. Neuritis of femoral nerve
89. A 56-year-old patient complains of 93. After lifting a load a patient felt
having persistent chest pain on the ri- undurable pain in the loin. He was di-
ght for the last 2 months. The pain agnosed with acute lumbosacral radiculi-
is not associated with respiration. He tis. Which of the following is contraindi-
also complains of cough with blood- cated for this patient?
streaked sputum, weakness, decreased
performance, fatigue. Chest radiograph A. Warming procedures
shows a globular shade of 4x6 cm B. Dehydrating drugs
connected to the root of the lung in the C. Analgetics
lower part of the right lung. What is the D. Vitamins of B group
most likely diagnosis? E. Intravenous injection of aminophylline
A. Peripheral lung cancer 94. An unconscious 35-year-old patient
B. Metastasis has been delivered by an ambulance to
C. Lung abscess the intensive care unit. Objectively: the
D. Pneumonia patient is in semicoma. Moderate mydri-
E. Tuberculoma asis is present. The reaction of pupils to
light is reduced. The reaction to verbal
90. A 30-year-old patient had deep burn instructions is missing. AP is150/100 mm
covering 30% of body 30 days ago. Now Hg, there is tachycardia. Blood contains
he presents with continued fever, loss of methanol. What antidote should be admi-
appetite, night sweats. Burned surface nistered?
weakly granulates. What is the stage of
burn disease? A. Ethanol
B. Unithiol
A. Septicotoxemia C. Thiamine chloride
B. Primary burn shock D. Tavegil
C. Secondary burn shock E. Naloxone
D. Acute burn toxemia
E. Convalescence 95. A 36-year-old man was delivered
to the surgical department an hour
91. A 27-year-old patient complains of after a road accident. His condition
nasal haemorrhages, multiple bruises on is getting worse: respiratory insuffici-
the anterior surface of the trunk and ency is progressing, there are cardiac
extremities, sudden weakness. In blood: abnormalities. Clinical and roentgenologi-
Hb- 74 g/l, reticulocytes - 16%, RBCs - cal investigations revealed mediastinal di-
2, 5 · 1012 /l, platelets - 30 · 109 /l, ESR- 25 splacement. What process has caused this
mm/h. What is the most effective measure complication?
for the treatment of thrombocytopenia?
Krok 2 Medicine 2012 14

A. Valvular pneumothorax lymphocytes - 8%, ESR - 42 mm/h. What


B. Open pneumothorax is the ethiological factor of pneumonia?
C. Closed pneumothorax
D. Subcutaneous emphysema A. Legionella
E. Mediastinitis B. Mycoplasm
C. Streptococcus
96. A 26-year-old male patient complains D. Staphylococcus
of piercing pain during breathing, cough, E. Pneumococcus
dyspnea. Objectively: to - 37, 3oC, respi-
ration rate - 19/min, heart rate = Ps- 100. A 53-year-old woman complained
92/min; AP- 120/80 mm Hg. Vesicular of cardiac pain and rhythm intermissions.
respiration. In the inferolateral parts of She had experienced these presentations
chest auscultation in both inspiration and since childhood. The patient’s father had
expiration phase revealed noise that was a history of cardiac arrhythmias. Objecti-
getting stronger at phonendoscope pressi- vely: the patient was in grave conditi-
ng and can be still heard after cough. ECG on, Ps- 220 bpm, AP- 80/60 mm Hg.
showed no pathological changes. What is ECG: heart rate - 215/min, widening and
the most likely giagnosis? deformation of QRS complex accompani-
ed by atrioventricular dissociation; posi-
A. Acute pleuritis tive P wave. Some time later heart rate
B. Intercostal neuralgia reduced down to 45/min, there was a
C. Subcutaneous emphysema complete dissociation of P wave and
D. Spontaneous pneumothorax QRST complex. Which of the following
E. Pericarditis sicca will be the most effective treatment?
97. Educational rooms are illuminated wi- A. Implantation of the artificial pacemaker
th various lighting fittings. What type of B. β-adrenoreceptor blocking agents
lighting fittings is the most appropriate in C. Cholinolytics
respect of hygienic norms? D. Calcium antagonists
E. Cardiac glycosides
A. Indirect light fittings
B. Direct light fittings 101. A 49-year-old patient complai-
C. Semi-reflected light fittings ns of dyspnea, cough. There are no
D. Ambient light fittings sputum discharges. He has repeatedly
E. Combined light fittings used salbutamol and intal but with no
effect. Objectively: he is only able to sit
98. A 25-year-old patient complains of while leaning on the table. Cyanosis of
general weakness, dry cough, sweating, face, acrocyanosis are present. Breathi-
subfebrile temperature. Objectively: lung ng is shallow, laboured, in some parts it
auscultation reveals vesicular resiration cannot be auscultated; there are diffuse
with no wheezing. Fluorogram shows rales, expiration is significantly prolonged.
focal shadows of high intensity in the 1- Heart sounds are muffled, tachycardia is
2 segments of the right lung. Mantoux test present. Ps - 112/min., AP- 110/70 mm Hg.
gave a reaction of 16 mm of induration. Liver is located near the costal arch. There
What clinical form of tuberculosis is most are no peripheral edemata. What is your
likely? provisional diagnosis?
A. Focal A. Status asthmaticus
B. Infiltrative B. Chronic obstructive bronchitis
C. Disseminated C. Bronchiale asthma, moderate gravity
D. Tuberculoma D. Foreign object aspiration
E. Miliary E. Cardiac asthma
99. A 22-year-old patient is a clerk. His 102. A 42-year-old patient with acute
working day runs in a conditioned room. haemorrhage and class III blood loss
In summer he was taken by an acute di- underwent blood transfusion and got 1,8
sease with the following symptoms: fever, l of preserved blood and erythromass
dyspnea, dry cough, pleural pain, myalgia, of the same group and Rh. After the
arthralgia. Objectively: moist rales on the transfusion the patient complained of
right, pleural friction rub. X-ray picture unpleasant retrosternal sensations, his
showed infiltration of the inferior lobe. In arterial pressure dropped to 100/60 mm
blood: WBC - 11 · 109 /l, stab neutrophi- Hg, there appeared convulsions. Blood
ls - 6%, segmented neutrophils - 70%, serum calcium was at the rate of 1,7 milli-
Krok 2 Medicine 2012 15

mole/liter. What is the mechanism of this from the maternity house a 2-year-old
complication development? patient consulted a doctor about body
temperature rise up to 39o C, pain in the
A. Citrate binds calcium ions, right breast. Objectively: the mammary
hypocalcemia impairs myocardial function gland is enlarged, there is a hyperemized
B. Citrate is cardiotoxic and nephrotoxic area in the upper external quadrant, in the
C. Citrate causes the development of same place there is an ill-defined indurati-
metabolic acidosis on, lactostasis, fluctuation is absent.
D. The increased citrate rate causes Lymph nodes of the right axillary regi-
convulsions on are enlarged and painful. What is the
E. Citrate binds potassium causing severe most likely diagnosis?
hypokalemia
A. Lactational mastitis
103. A 27-year-old patient with a knife B. Abscess
stomach wound has been delivered to C. Erysipelas
a hospital 4 hours after injury. Objecti- D. Dermatitis
vely: the patient is in grave condition. E. Tumour
Ps- 120 bpm, weak. AP- 70/40 mm Hg.
Laparotomy revealed a lot of liquid blood 107. During the dynamic observation over
in the abdominal cavity. The patient has a parturient woman in the second stage of
been found to have bleeding from the labor it was registered that the fetal heart
mesenteric vessels of the small intestine. rate fell down to 90-100/min and didn’t
Damage to hollow organs has not been come to normal after contractions. Vagi-
revealed. What is the best way to restore nal examination revealed the complete
the blood loss? cervical dilatation, the fetal head filling
the entire posterior surface of the pubic
A. Autoblood reinfusiont symphysis and sacral hollow; the sagi-
B. Erythromass transfusion ttal suture lied in the anteroposterior di-
C. Transfusion of washed erythrocytes ameter of the pelvic outlet, the posteri-
D. Transfusion of fresh frozen plasma or fontanelle was in front under the
E. Rheopolyglucinum transfusion pubic arch. What plan for further labour
management should be recommended?
104. A 42-year-old woman has had
hyperpolymenorrhea and progressing A. Application of forceps minor
algodismenorrhea for the last 10 years. B. Caesarean section
Gynaecological examination revealed no C. Episiotomy
changes of uterine cervix; discharges are D. Application of cavity forceps
moderate, of chocolate colour, uterus is E. Stimulation of labour activity by
slightly enlarged and painful, appendages intravenous injection of oxytocin
are not palpable, the fornices are deep
and painless. What is the most likely di- 108. A female patient consulted a
agnosis? dermatologist about the rash on the trunk
and extremities. Objectively: interdigital
A. Uterine endometriosis folds, flexor surfaces of wrists and navel
B. Uterine carcinoma region are affected with pairs of nodulo-
C. Subserous uterine fibromyoma cystic eruptions and crusts. The rash is
D. Endomyometritis accompanied by skin itch that is getting
E. Adnexal endmetriosis stronger at night. What external treatment
should be administered?
105. The results of 5 year monitoring
allowed to estimate the level of envi- A. 20% benzyl benzoate emulsion
ronmental influence upon health indi- B. 5% sulfuric ointment
ces of popultaion. What statistic method C. 2% sulfuric paste
should be chosen? D. 5% naphthalan ointment
E. 5% tetracycline ointment
A. Calculation of correlation coefficient
B. Calculation of conformity coefficient 109. An ambulance had been called
C. Calculation of coefficient of difference to a 48-year-old man. According to
validity his relatives, the patient had had three
D. Calculation of regression coefficient attacks of unconsciousness accompanied
E. Calculation of dynamic indices by convulsions within 24 hours. On exami-
nation the doctor witnessed the followi-
106. On the tenth day after discharge ng attack: the patient lost consciousness
Krok 2 Medicine 2012 16

and fell to the floor, developed tonic, then pulse is present. Ps- 185 bpm, of poor
clonic convulsions of trunk and extremiti- volume. AP- 75/40 mm Hg. ECG taken
es. The attack lasted 1 minute and ended during an attack shows ectopic P waves,
with involuntary urination. Specify the ki- QRS wave is not deformed. At the end
nd of attack: of an attack a compensatory pause is
observed. The most likely cause of the
A. Epileptic seizure attack is:
B. Syncope
C. Vegetative crisis A. Paroxysmal atrial tachycardia
D. Coma B. Sinus tachycardia
E. Attack of hysteria C. Paroxysmal ventricular tachycardia
D. Complete AV-block
110. A patient is 60-year-old, retired, E. Atrial fibrillation
worked as deputy director of a research
institute. Behavioural changes appeared 2 113. A 1-month-old child became restless
years ago after the death of her husband: and presented with an increase in head
she stopped looking after herself and sweating. It’s known from the history that
leaving the house; then she stopped to the child has been fed with cow’s milk
clean the apartment and cook. Mental since birth (September 5). Examination
status: temporal disorientation. The pati- revealed craniotabes. A doctor admini-
ent does not understand many of the stered a course of UV radiation. Decide,
questions, is confused; does not know if the child needs ergocalciferol:
how to cook soup or fasten a button.
Her speech is characterized by stumbli- A. 2-2,5 months after the UVR withdrowal
ng and logoclonia. She does not recognize B. Does not need
doctors, fellow patients. She cries a lot but C. In combination with UVR
can not explain the reason for tears. What D. Immediately after the UVR withdrowal
is the mechanism of this pathology? E. A month after the UVR withdrowal
A. Atrophy of the cerebral cortex 114. Two hours ago a 38-year-old pati-
B. Atherosclerotic changes in cerebral ent got pain in his right shin. He was
vessels diagnosed with popliteal artery emboli-
C. Serotonin deficiency sm, acute arterial insufficiency of grade I.
D. Impaired conversion of dopamine to What is the most appropriate therapeutic
noradrenaline tactics?
E. Disorder of melatonin metabolism
A. Embolectomy
111. A 26-year-old patient has abused B. Destruction of the embolus by the
alcohol since the age of 16, needs a catheter
morning-after drink to cure hangover. He C. Resection of the popliteal artery
takes alcohol nearly every day, "a little at D. Bypass grafting
a time". Twice a week he gets severely E. Amputation at the middle of shin
drunk. The patient works as a motor
mechanic, over the last 2 years work 115. A child is 2 days old. He was born wi-
conflicts have become more frequent. th a weight of 2900 kg, body length of 50
What medical and tactical actions should cm. On examination the skin is intensely
be taken in this case? red, elastic, with preserved turgor. Pueri-
le respiration is present. Respiration rate
A. Voluntary consultation and treatment - 40/min, cardiac sounds are rhythmic,
at an addiction clinic sonorous. HR- 138/min. The abdomen is
B. Compulsory treatment soft. The liver extends 2 cm below the
C. Referral to treatment at an activity costal margin. Diuresis is sufficient. Stool
therapy centre is in form of meconium. What is the most
D. Referral to medical-social expert likely diagnosis?
commission for assessment of his working
ability A. Physiological erythema of the newborn
E. Consultation with a psychologist B. Toxic erythema of the newborn
C. Neonatal phlegmon
112. An 8-year-old girl periodically has D. Erysipelas
sudden short-term heart pain, sensation E. Exfoliative Ritter’s dermatitis
of chest compression, epigastric pain, di-
zziness, vomiting. Objectively: the patient 116. A 47-year-old female patient
is pale, respiratory rate - 40/min, jugular complains of leg heaviness, fatigue when
Krok 2 Medicine 2012 17

standing and walking. This feeling di- recommendations should doctor gi-
sappears when she takes a horizontal posi- ve to mother to prevent this disease
tion. Objectively: dilatation of the superfi- recurrence?
cial veins of the left shin and thigh with
pigmentation and trophic skin disorders. A. Strict following of feeding regimen
What functional test should the exami- B. Common cold prophilaxis
nation be started with? C. Feces observation
D. Gastro-intestinal disease prevention
A. Trendelenburg’s test E. Hardening of the child
B. Pratt test 2
C. Pratt test 1 121. A 40-year-old patient underwent an
D. Sheinis test operation for a lumbar phlegmon. Body
E. Perthes’ test temparature rose again up to 38o C, he
got intoxication symptoms, there was an
117. A 28-year-old patient complains of di- increase of leukocyte number in blood.
scomfort, acute pain in the lower third of The wound that was nearly free from
the left labia majora. The disease began necrotic tissues and full of granulations
suddenly after menstruation. Objectively: started to discharge pus, the granulations
body temperature is 38o C. The left labia turned pale. What complication dveloped
majora has a formation to 3 cm diameter, in this patient?
with hyperemic surface, extremely painful
to the touch, with symptoms of fluctuati- A. Sepsis
on. What is the most likely diagnosis? B. Putrid phlegmon
C. Erysipelas
A. Acute bartholinitis D. Allergic reaction
B. Vulvar cancer E. Erysipeloid
C. Vulvar fibroid
D. Bartholin gland cyst 122. An emergency situation at a chemical
E. Hypertrophy of the labia plant caused acute occupational intoxi-
cation. A doctor who revealed the case
118. A 40 week pregnant secundipara is of "acute occupational disease (intoxicati-
28 years old. Contractions are very active. on)"must notify the following authority:
Retraction ring is at the level of navel, the
uterus is hypertonic, in form of hourglass. A. Sanitary and epidemiological station
On auscultation the fetal heart sounds B. Plant administration
are dull, heart rate is 100/min. AP of the C. Trade union committee of the plant
parturient woman is 130/80 mm Hg. What D. Medical unit of the plant
is the most likely diagnosis? E. Ministry of Health of Ukraine
A. RIisk of hysterorrhexis 123. A 42-year-old builder consulted a
B. Mazolysis doctor about a foot injury with a nail
C. Disturbed labour that he got in the morning of the same
D. Complete hysterorrhexis day. The wound was washed with water.
E. Attack of eclampsia Three years ago he was vaccinated agai-
nst tetanus. Examination established sati-
119. A neonatologist examining a full- sfactory condition of the patient. The
term mature baby revealed the shorteni- left foot was slightly edematic, there was
ng and external rotation of the newborn’s a stab wound on the sole. In order to
lower extremity. Clinical examination prevent tetanus it is primarily required
revealed positive Ortolani sign, symptom to:
of non-vanishing pulse, additional skin
folds on the internal surface of thigh. A. Give an intravenous injection of 0,5 ml
What is the most likely diagnosis? of tetanus anatoxin
B. Give an intravenous injection of 1 ml of
A. Congenital hip dislocation tetanus anatoxin, 3000 IU of antitetanus
B. Dysplasia of the hip joint serum
C. Varus deformity of the femoral neck C. Give an intravenous injection of 3000
D. Femoral neck fracture IU of antitetanus serum
E. Fracture of the femoral shaft D. Treat the wound with suds
E. Administer a course of antibiotic
120. A child is being discharged from therapy
the surgical department after conservati-
ve treatment of invagination. What 124. A 6-year-old child has duodenal ulcer.
Krok 2 Medicine 2012 18

What antibacterial drug should be co- A. Hypoproduction of gonadotropin


administered together with metronidazole B. Hyperproduction of estrogens
and De-Nol in order to eradicate Heli- C. Hyperproduction of androgens
cobacter pylori infection? D. Hypoproduction of progesterone
E. Hyperproduction of prolactin
A. Amoxicillin
B. Tetracycline 128. Hygienic expertise of a sample taken
C. Oleandomycin from the batch of grain revealed 2% of
D. Biseptol grains infected with microscopic Fusari-
E. Sulfadimethoxinum um fungi. On the ground of laboratory
analyses this batch of grain should be:
125. Against the background of angina
a patient has developed pain in tubular A. Sold without restrictions
bones. Examination revealed generalized B. Tested for toxicity
enlargement of lymph nodes, hepatoli- C. Used for forage production
enal syndrome, sternalgia. In blood: RBCs D. Used for ethanol production
- 3, 6 · 1012 /l, Hb- 87 g/l, thrombocytes E. Destroyed
- 45 · 109 /l, WBCs - 13 · 109 /l, blasts - 129. Among the inhabitants of a
87%, stab neutrophils - 1%, segmented workmen’s settlement located near an
neutrophils - 7%, lymphocytes - 5%, ESR industrial plant the cases of nervous and
- 55 mm/h. What is the most likely di- endocrine system diseases as well as renal
agnosis? diseases became more frequent. Blood
A. Acute leukemia analyses revealed a decrease in sulfhydryl
B. Erythremia groups. These pathologies might have
C. Chronic lymphocytic leukemia been caused by the following substance
D. Chronic myeloid leukemia released into the environment:
E. Multiple myeloma A. Mercury
126. A 51-year-old patient complains B. Cadmium
of having intensive bloody discharges C. Boron
from vagina for 15 days after delay D. Chromium
of menstruation for 2,5 months. In E. Lead
anamnesis: disorders of menstrual functi- 130. It is required to analyze the level
on during a year, at the same time she of daylight illumination in a ward of
felt extreme irritability and had sleep di- therapeutics unit. What device should be
sorders. US examination results: uterus applied to estimate the level of daylight
corresponds with age norms, appendages illumination?
have no pecularities, endometrium is 14
mm thick. What is the doctor’s tactics? A. Illuminometer
B. Anemometer
A. Diagnostic curettage of uterine cavity C. Katathermometer
B. Conservative treatment of bleeding D. Actinometer
C. Hysterectomy E. Psychrometer
D. Supravaginal amputation of uterus
without appendages 131. During the medical examination
E. TORCH-infection test at school the schoolchildren had to
undergo plantography. After the analysis
127. A 25-year-old female patient of footprints platypodia was found in 30%
complains about having amenorrhea of pupils. What is the percentage of the
for 3 years. She associates it with di- flatfoot isthmus?
fficult labour complicated by massive
hemorrhage. She also complains of loss A. 65%
of weight, hair fragility and loss, lack of B. 30%
appetite and depression. Objective exami- C. 50%
nation reveals no pathological changes of D. 55%
uterus and its appendages. What is the E. 45%
desease pathogenesis?
132. A factory’s sectorial doctor selects
a group of persons who often fall ill for
thorough monitoring. At the same time
he takes into consideration the number of
Krok 2 Medicine 2012 19

etiologically related cases with temporary A. The whole carcass should be technically
disability in each of the employees over disposed
the last year. An employee falls into this B. Meat can be sold without any restricti-
group if the number of sickness cases is: ons
C. The carcass should be used for the
A. 4 or more production of canned meat
B. 1 or more D. Meat should be disinfected by boiling
C. 2 or more E. Meat should be disinfected by freezing
D. 3 or more
E. 6 or more 137. A 28-year-old female patient
complains of having haemorrhage from
133. Half an hour ago a 67-year-old patient the genital tracts for 1 month. 6 months
with a hernia picked up a heavy object, ago she had natural delivery and gave bi-
which caused acute pain in the region rth to a girl weighing 3100 g. Objectively:
of hernia protrusion, the hernia couldn’t the uterus is enlarged to 9-10 weeks, mobi-
be reduced. Objectively: the hernia in le, painless, of heterogenous consistency.
the right inguinal region is roundish, ti- Examination reveals vaginal cyanosis,
ght, moderately painful; during palpati- anaemia and body temperature rise up
on it was reduced back to the cavity, the to 37, 8oC. There is a significant increase
pain was gone. Specify the further medical in hCG concentration in the urine. What
tactics: is your provisional diagnosis?
A. Inpatient surveillance A. Uterine chorionepithelioma
B. Immediate hernioplasty B. Pregnancy
C. Immediate laparotomy C. Hydatidiform mole
D. Planned hernioplasty a month later D. Endometritis
E. Planned hernioplasty a year later E. Uterine fibromyoma
134. Production areas of a greenhouse 138. The correlation between the servi-
complex have the following microcli- ce record and eosinophil concentration in
mate parameters: air temperature - 42oC, blood was studied in workers at dyeing
humidity - 98%, air velocity - 0,05 shops of textile factories. What index will
mps, temperature of enclosing surfaces be the most informative for the analysis of
- 15o C. Characterize the microclimate of this data?
production areas:
A. Correlation factor
A. Overheated B. Student’s criterion
B. Comfortable C. Standardized index
C. Cooling D. Fitting criterion
D. Uncomfortable E. Sign index
E. Satisfactory
139. A 43-year-old male patient undergoi-
135. A 54-year-old female patient ng treatment for peptic ulcer complai-
consulted a gynaecologist about bloody ns of weakness, dizziness, coffee-ground
discharges from the vagina for 1 month. vomiting, melena. After administration of
Last menstruation was 5 years ago. haemostatics the patient’s condition has
Gynaecological examination revealed no not improved, fresh blood has shown up
pathological changes. What is the tactics in the vomit, skin bruises of different sizes
of choice? have appeared. In blood: thrombocytes -
50 · 109 /l, Lee-White clotting time - 35 mi-
A. Diagnostic fractional curettage of uteri- nutes, APTT - 80 seconds. In this case it is
ne cavity most rational to administer the following
B. Colposcopy preparation:
C. USI
D. Cytosmear A. Fresh frozen plasma
E. Symptomatic therapy B. Heparin
C. Fibrinogen
136. Sanitary-veterinary examination of a D. Rheopolyglucinum
cow carcass revealed measle contaminati- E. Vikasol
on (2-3 measles per 10 cm2 ). What tactics
should be chosen in respect of this meat 140. A girl of 3 months presents wi-
consumption? th rhinitis, dyspnea, dry cough. These
manifestations has been observed for
Krok 2 Medicine 2012 20

two days. Objectively: the child has oesophageal stricture a patient developed
pale skin, acrocyanosis, shallow respi- acute retrosternal pain getting worse
ration at the rate of 80/min. Percussi- when throwing the head back and
on reveals handbox resonance over the swallowing. Objectively: dilatation of
whole surface of lungs, there are a lot of fi- the neck veins, dropped beat pulse, si-
ne rales. What is the most likely diagnosis? gns of purulent intoxication, oliguria,
emphysema of the upper portion of chest.
A. Acute bronchiolitis What disease can be suspected?
B. Pneumonia
C. Mucoviscidosis A. Suppurative mediastinitis
D. Foreign body of the airway B. Thrombosis of the superior vena cava
E. Acute bronchitis C. Pleural empyema
D. Acute myocardial infarction
141. A 30-year-old patient has been admi- E. Spontaneous pneumothorax
tted to the intensive care unit for multiple
bee stings. The skin is covered with cold 145. A 32-year-old patient consulted a
sweat. The pulse is felt just on the carotid doctor about being inable to get pregnant
arteries, 110 bpm, respiration is 24/min, for 5-6 years. 5 ago the primipregnancy
rhythmic, weakened. Which drug should ended in artificial abortion. After the vagi-
be given in the first place? nal examination and USI the patient was
diagnosed with endometrioid cyst of the
A. Adrenalin hydrochloride intravenously right ovary. What is the optimal treatment
B. Prednisolone intravenously method?
C. Adrenalin hydrochloride
intramuscularly A. Surgical laparoscopy
D. Dopamine intravenously B. Anti-inflammatory therapy
E. Tavegil intravenously C. Conservative therapy with estrogen-
gestagenic drugs
142. A 58-year-old patient complains D. Hormonal therapy with androgenic
about sensation of numbness, sudden hormones
paleness of II-IV fingers, muscle ri- E. Sanatorium-and-spa treatment
gidness, intermittent pulse. The pati-
ent presents also with polyarthralgia, 146. The objective of a statistical study
dysphagia, constipations. The patient’s was to find out the extent of seeking
face is masklike, solid edema of hands is medical care by the population. For this
present. The heart is enlarged; auscultati- purpose 300 residents of the area were
on revealed dry rales in lungs. In blood: interviewed. Information was collected by
ESR- 20 mm/h, crude protein - 85/l, γ- means of a special questionnaire. What
globulines - 25%. What is the most likely method of collecting information was
diagnosis? used by researchers?
A. Systemic scleroderma A. Anamnestic
B. Dermatomyositis B. Immediate registration
C. Rheumatoid arthritis C. Immediate examination
D. Systemic lupus erythematosus D. Doing extracts
E. Raynaud’s disease E. -
143. A 45-year-old man has been exhi- 147. An 8-month-old baby has had
biting high activity for the last 2 weeks, problems with nasal breathing and muco-
he became talkative, euphoric, had li- purulent discharge from the nose for a
ttle sleep, claimed being able "to save week. Examination reveals a rhinedema,
the humanity"and solve the problem muco-purulent discharges from the mi-
of cancer and AIDS, gave money the ddle nasal meatus as well as on the
starangers. What is the most likely di- back of pharynx. What disease are these
agnosis? symptoms most typical for?
A. Maniacal onset A. Ethmoiditis
B. Panic disorder B. Sphenoiditis
C. Agitated depression C. Maxillary sinusitis
D. Schizo-affective disorder D. Frontitis
E. Catatonic excitation E. Hemisinusitis
144. After the pneumatic dilatation of 148. A city’s population is 400000 inhabi-
Krok 2 Medicine 2012 21

tants in 2005 there were registered 5600 A. Individual records of pregnant and
deaths, including 3300 cases caused by postpartum women
cardiovascular diseases, 730 - by tumours. B. Prenatal records
Which of the following indicators allows C. Medical records of outpatients
to characterize the share of the circulatory D. Labour and delivery record
system diseases as the cause of death in E. Neonatal record
the city?
153. Examination of placenta revealed a
A. Extensive index defect. An obstetrician performed manual
B. Intensive index investigation of uterine cavity, uterine
C. Index of relative intensity massage. Prophylaxis of endometritis in
D. Demonstrativeness index the postpartum period should involve
E. Ratio following actions:
149. It is planned to organize a rural A. Antibacterial therapy
outpatient clinic. The patients will be able B. Instrumental revision of uterine cavity
to visit the doctors of the following speci- C. Haemostatic therapy
alities: D. Contracting agents
E. Intrauterine instillation of dioxine
A. Therapeutist, dentist, pediatrician,
obstetrician-gynecologist 154. A 35-year-old patient complains
B. Therapeutist, pediatrician, neurologist of heartburn, sour eructation, burning,
C. Pediatrician, obstetrician-gynaecologist, compressing retrosternal pain and pain
ophthalmologist along the esophagus rising during forward
D. Pediatrician, therapeutist, bending of body. The patient hasn’t been
ophthalmologist examined, takes Almagel on his own initi-
E. Obstetrician-gynaecologist, therapeutist ative, claims to feel better after its taking.
Make a provisional diagnosis:
150. A 10-year-old girl was admitted to a
hospital with carditis presentations. It is A. Gastroesophageal reflux disease
known from the anamnesis that two weeks B. Functional dyspepsia
ago she had exacerbation of chronic tonsi- C. Cardiospasm
llitis. What is the most likely etiological D. Gastric ulcer
factor in this case? E. Duodenal ulcer
A. Streptococcus 155. All the joints on the left elbow of
B. Staphylococcus a newborn are extended, the whole arm
C. Pneumococcus hangs vertically along the trunk with the
D. Klebsiella forearm pronated. Active movements in
E. Proteus the elbow joint are absent but present in
the shoulder joint. The hand is flattened,
151. In a rural health district a child di- atrophied, cold to the touch, hangs
ed in the first month of life. In order to passively. Grasp reflex and hand-mouth
analyze this situation it was necessary reflex on the affected side are missing.
to complete expert evaluation of medi- Haemogram values are normal. What is
cal records. Which medical document was the most likely diagnosis?
analyzed in the first place?
A. Inferior distal obstetrical paralysis
A. Child developmental history record B. Osteomyelitis
B. Neonatal record C. Proximal obstetrical paralysis
C. Record of vaccinations D. Complete obstetrical paralysis
D. Medical record of an outpatient E. Hypoxic-ischemic encephalopathy
E. Child’s medical record
156. Head circumference of a 1-month-
152. The head of prenatal care department old boy with signs of excitement is 37 cm,
intends to complete expert evaluation of prefontanel is 2x2 cm large. After feeding
compliance with medical and technologi- the child regurgitates small portions of mi-
cal standards of pregnancy follow-up. lk; stool is normal in respect of its volume
What documents must be verified for this and composition. Muscle tonus is within
purpose? norm. What is the most likely diagnosis?
Krok 2 Medicine 2012 22

A. Pylorospasm A. To perform the Heimlich manoever


B. Meningitis B. To perform conicotomy immediately
C. Pylorostenosis C. To send for an anesthesiologist and wait
D. Microcephaly for him
E. Craniostenosis D. To use an inhalation of β2 -adrenoceptor
agonist
157. A 35-year-old victim of a road E. To make a subcutaneous injection of
accident has got an injury of the ri- dexamethasone
ght side of his chest. Objectively: respi-
ration rate - 28-30/min, respiration is 160. A patient undergoing treatment
shallow, restricted respiratory excursi- for the left-sided destructive pneumonia
on and acrocyanosis are present. Ps- presents with deterioration of his general
110 bpm, AP- 90/60 mm Hg. Respi- condition, progressing dyspnea, cyanosis.
ratory sounds over the right lung cannot Objectively: the left side of chest is not
be auscultated. Chest radiograph shows involved in respiration, breathing sounds
fractures of the VI-VII ribs on the right, cannot be auscultated. Radiograph shows
the right pleural cavity contains both air a shadow reaching the 5 rib with a hori-
and fluid, with the fluid at about the level zontal fluid level and a radiolucency above
of the V rib, the shadow of the mediasti- it, the mediastinum is displaced to the ri-
num is displaced to the left. What first aid ght. What is the medical tactics?
should be provided to the victim?
A. Thoracostomy
A. Puncture of the pleural cavity B. Open thoracotomy
B. Antibiotic administration C. Endotracheal intubation
C. Vagosympathetic blockade D. Infusion and antibacterial therapy
D. Artificial ventilation of lungs E. Emergency bronchoscopy
E. Urgent thoracotomy
161. A 10 week pregnant woman was
158. A 20-year-old patient complains of admitted to a hospital for recurrent pain
pain in the left lumbar region, arterial in the lower abdomen, bloody discharges
pressure rise up to 160/110 mm Hg. USI from the genital tracts. The problems
revealed that the structure and size of the turned up after ARVI. The woman was
right kidney were within age norms, there registered for antenatal care. Speculum
were signs of 3 degree hydronephrotic examination revealed cyanosis of vagi-
transformation of the left kidney. Doppler nal mucosa, clean cervix, open cervical
examination revealed an additional artery canal discharging blood and blood clots;
running to the lower pole of the kidney. the lower pole of the gestational sac was
Excretory urogram shows a narrowing visible. What tactics should be chosen?
in the region of ureteropelvic junction.
Specify the therapeutic tactics: A. Curettage of the uterus
B. Pregnancy maintenance therapy
A. Surgical intervention C. Expectant management, surveillance
B. Administration of spasmolytics D. Hysterectomy
C. Administration of ACE inhibitors E. Antiviral therapy
D. Kidney catheterization
E. Administration of β-blockers 162. A 29-year-old patient complains of
absent menstruation for a year, milk di-
159. A hospital admitted a patient wi- scharge from the nipples when pressed,
th coarse breathing (obstructed inspirati- loss of lateral visual fields. X-ray shows an
on), skin cyanosis, tachycardia and arteri- expansion of the sella turcica. What is the
al hypertension. He has a histrory of most likely cause of this condition?
bronchial asthma. An hour ago he was
having salbutamol inhalation and forgot A. Pituitary tumour
to remove a cap that was aspired whi- B. Mammary tumour
le taking a deep breath. What measures C. Functional disorder of the
should the doctor take? hypothalamic-pituitary-ovarian system
D. Ovarian tumor
E. Pregnancy
163. On the second day after preventi-
ve vaccination a 2-year-old boy presented
with abdominal pain without clear locali-
zation, body temperature rose up to 38o C.
Krok 2 Medicine 2012 23

On the third day the child got red papular A. Posttraumatic osteomyelitis
haemorrhagic eruption on the extensor B. Hematogenic osteomyelitis
surfaces of limbs and around the joints. C. Wound abscess
Knee joints were edematic and slightly D. Posttraumatic phlegmon
painful. Examination of other organs and E. Suture sinus
systems revealed no pathological changes.
What is the most likely diagnosis? 167. A 40-year-old female patient
complains of having a bulge on the anteri-
A. Haemorrhagic vasculitis or surface of neck for 5 years. Objecti-
B. Thrombocytopenic purpura vely: Ps- 72 bpm, arterial pressure - 110/70
C. Meningococcemia mm Hg, in the right lobe of thyroid gland
D. Urticaria palpation reveals a mobile 4x2 cm node,
E. DIC syndrome the left lobe is not palpable, the basal
metabolic rate is 6%. What is the most
164. A patient with uterine fibromyoma likely diagnosis?
sized up to 8-9 weeks of pregnancy
consulted a gynaecologist about acute A. Nodular euthyroid goiter
pain in the lower abdomen. Examination B. Nodular hyperthyroid goiter
revealed pronounced positive symptoms C. Riedel’s thyroiditis
of peritoneal irritation, high leukocytosis. D. Mixed euthyroid goiter
Vaginal examination revealed that the E. The median cervical cyst
uterus was enlarged up to 9 weeks of
pregnancy due to the fibromatous nodes, 168. A 14-year-old patient with signs of
one of which was mobile and extremely internal haemorrhage has been taken
painful. Appendages were not palpable. to a hospital after a fight. He has had
Discharges were mucous, coming in haemophilia A since childhood. He has
moderate amounts. What is the treatment been diagnosed with retroperitoneal
tactics? hematoma. What should be administered
in the first place?
A. Urgent surgery (laparotomy)
B. Surveillance and spasmolytic therapy A. Cryoprecipitate
C. Fractional diagnostic curettage of the B. Aminocapronic acid
uterine cavity C. Dried plasma
D. Surgical laparoscopy D. Platelet concentrate
E. Surveillance and antibacterial therapy E. Fresh blood

165. On the 6th day of life a child got 169. A patient is 14 years old. Cytochemi-
multiple vesicles filled with seropurulent cal study of punctate revealed 40% of
fluid in the region of occiput, neck and blasts, there was negative reaction to
buttocks. General condition of the chi- peroxidase and with Sudan black, positive
ld is normal. What disease should be reaction to glycogen. Specify the form of
suspected? acute leukemia:

A. Vesiculopustulosis A. Lymphoblastic
B. Impetigo neonatorum B. Myeloblastic
C. Miliaria C. Monoblastic
D. Impetigo D. Promyelocytic
E. Epidermolysis bullosa E. Undifferentiated

166. A 3-year-old male patient consulted 170. A 58-year-old patient complains of a


a family doctor 2 months after he had headache in the occipital region, nausea,
been operated for an open fracture of choking, opplotentes. The presentati-
brachial bone. Objectively: the patient’s ons appeared after a physical exerti-
condition is satisfactory, in the region of on. Objectively: the patient is excited.
the operative wound there is a fistula Face is hyperemic. Skin is pale. Heart
with some purulent discharge, redness, sounds are regular, the 2nd aortic sound
fluctuation. X-ray picture shows brachi- is accentuated. AP- 240/120 mm Hg, HR-
al bone destruction with sequestra. What 92/min. Auscultation reveals some fine
complication arose in the postoperative moisr rales in the lower parts of the lungs.
period? Liver is not enlarged. ECG shows signs of
hypertrophy and left ventricular overload.
What is the most likely diagnosis?
Krok 2 Medicine 2012 24

A. Complicated hypertensic crisis A. Echocardiography


B. Acute myocardial infarction, pulmonary B. ECG
edema C. Diuresis monitoring
C. Bronchial asthma exacerbation D. Monitoring of K+ , Na+ concentration
D. Uncomplicated hypertensic crisis in blood
E. Community-acquired pneumonia E. 24-hour monitoring of heart rhythm
171. A 59-year-old male patient with 174. Forensic medical expertise of corpse
essential hypertension of stage II is regi- of a newborn revealed: body weight 3500
stered with the dispensary department of g, body length 50 cm, the umbilical cord
a polyclinic. The patient regularly takes was smooth, moist, glossy, without any si-
ACE inhibitors and calcium antagonists. gns of drying. Hydrostatic tests were posi-
How often should a therapeutist examine tive. The test results are the evidence of:
this patient (except for exacerbation peri-
ods)? A. Live birth
B. Stillbirth
A. Every 3 months C. Primary atelectasis
B. Every 6 months D. Secondary atelectasis
C. Every 4 months E. Hyaline membrane disease
D. Once a year
E. Every 9 months 175. A 60-year-old patient complains of
nearly permanent sensation of heavi-
172. A 37-year-old patient complains of ness and fullness in the epigastrium,
pain in the lumbar spine that is getting that increases after eating, foul-smelling
stronger during walking; restricted mobi- eructation, occasional vomiting with food
lity, edema of the right side of abdomen. consumed 1-2 days ago, weight loss. 12
He has a history of focal tuberculosis. X- years ago he was found to have an ulcer
ray picture shows the destruction of the of pyloric channel. The patient has taken
adjacent surfaces of the 1-2 vertebral bodi- ranitidine for periodic hunger pain. The
es of the lumbar spine, vertebral body hei- patient’s condition has been deteriorati-
ght is decreased, intervertebral foramen ng over the last 3 months. Objectively:
is undetectable. Abdominal ultrasound splashing sound in the epigastrium is
reveals a 15x20 cm formation in the present. What kind of complication is it?
retroperitoneal space, there are echo si-
gnals of fluid presence. What is the most A. Pyloric stenosis
likely diagnosis? B. Penetration of gastric ulcer
C. Functional pyloric spasm
A. Tuberculous spondylitis of the lumbar D. Foreign body in the stomach (bezoar)
spine E. Malignization of gastric ulcer
B. Fracture of the 1-2 vertebral bodies of
the lumbar spine 176. A hospital admitted an 11-year-
C. Spinal metastases old boy diagnosed with medium-severe
D. Spondylolisthesis of the lumbar spine asthma, exacerbation period. In order
E. Osteochondrosis to arrest the attacks the boy was admi-
nistered broncholytic nebulizer therapy.
173. The condition of a 3-year-old child During the day the child’s condition
with acute non-rheumatic myocarditis has stabilized. What is the most appropriate
suddenly deteriorated: he presents with method for further monitoring of respi-
anxiety, acrocyanosis, peripheral edemata, ratory function in this patient?
dyspnea. Auscultation of lungs reveals
fine moist rales on both sides mainly A. Peak flowmetry
in the lower parts. AP- 65/40 mm Hg. B. Spirometry
HR- 150/min, heart sounds are muffled, C. Pneumotachometry
arrhythmic (extrasystole). Liver is +4 cm. D. Bronchodilatation tests
Oliguria is present. The child has been di- E. Veloergometry
agnosed with acute heart failure. Which
method of examination is most informati- 177. A full-term newborn was born wi-
ve for assessing the child’s status dynami- th body weight of 4000 g, body length of
cs? 57 cm. Reaction to the postnatal check
was absent. There was diffuse cyanosis,
heart rate of 80/min. What resuscitation
measures should be taken?
Krok 2 Medicine 2012 25

A. Start ALV with a mask a large portion of the esophagus. What is


B. Give 100% oxygen the most likely diagnosis?
C. Intubate the child and start ALV
D. Start tactile stimulation A. Esophageal carcinoma
E. Give an injection of naloxone B. Benign tumour
C. Esophageal achalasia
178. A 52-year-old patient works as a D. Peptic ulcer
secretary and has 30 year record of servi- E. Sideropenic dysphagia
ce. She complains of spasms in her ri-
ght hand during working and inability 182. A 27-year-old patient complai-
to type and write. Up to 80% of her ns of irritability, tearfulness, depressi-
work involves hand load. The patient has on, and sometimes aggressiveness,
been presenting with these symptoms for headache, nausea, vomiting, swelling
2 years. Objectively: the right hand is of the mammary glands. The menti-
tense, there is an increase in muscle tone, oned problems arise 5-6 days before
attempts to write cause spasms. Exami- menstruation and gradually progress until
nation revealed no pathological changes menstruation, 3 days after it the problems
of CNS. What is the most likely diagnosis? disappear. What is the most likely di-
agnosis?
A. Spastic form of coordination neurosis
B. Neuralgic form of coordination neurosis A. Premenstrual syndrome
C. Paretic form of coordination neurosis B. Premature pathological climacterium
D. Hysteric neurosis C. Secondary psychogenic amenorrhea
E. Chronic manganese intoxication D. Preclimacterium syndrome
E. Algomenorrhea
179. A multigravida with Rh-
isosensitization was found to have a 183. A 2-year-old child in a satisfactory
decrease in anti-Rh titer from 1:32 condition periodically presents with
to 1:8 at 33-34 weeks of gestation. moderate proteinuria, microhematuria.
Ultrasound revealed double contour USI results: the left kidney is
of head, ebnlargement of fetal liver, undetectable, the right one is enlarged,
placental thickness of 50 mm. The pati- there are signs of double pyelocaliceal
ent has indication for: system. What study is required to speci-
fy the diagnosis?
A. Premature delivery
B. Course of desensitizing therapy A. Excretory urography
C. Plasmapheresis B. Micturating cystography
D. Repeated (after 2 weeks) USI C. Retrograde urography
E. Administration of anti-Rh gamma D. Doppler study of renal vessels
globulin E. Radioisotope renal scan
180. A 22-year-old vegetarian patient 184. An emergency doctor has diagnosed
with signs of malnutrition consulted a a 32-year-old woman with generalized
doctor about smell and taste distortion, convulsive status epilepticus. The deteri-
angular stomatitis. Objectively: marked oration in the patient’s condition is caused
blue sclerae. The patient was diagnosed by a sudden gap in the epilepsy treatment.
with iron deficiency anemia. What is the Specify the doctor’s further tactics:
dominating clinical syndrome?
A. Hospitalization in the intensive care
A. Sideropenic unit
B. Anaemic B. Hospitalization in the department of
C. Haemologic neurology
D. Haemolytic C. Hospitalization in the department of
E. Myelodysplastic neurosurgery
D. Outpatient monitoring by a
181. A patient complains of retrosternal neuropathologist
pain, difficult swallowing, over 10 E. Outpatient monitoring by a
kg weight loss within three months, neurosurgeon
general weakness. In blood: hypochromic
anaemia, neutrophilic leukocytosis. In 185. A 75-year-old male patient complains
feces: weakly positive Gregersen’s reacti- of slight pain in the right iliac region. The
on. On esophagram a filling defect with abdominal pain arose 6 days ago and was
ill-defined serrated edges shows up along accompanied by nausea. Surgical exami-
Krok 2 Medicine 2012 26

nation revealed moist tongue, Ps- 76 bpm. The serum prolactin level is increased.
AP- 130/80 mm Hg. Abdomen was soft, What is the most likely diagnosis?
slightly painful in the right iliac region
on deep palpation, the symptoms of the A. Hyperprolactinemia
peritoneum irritation were doubtful. In B. Hypothyroidism
blood: RBCs - 4, 0 · 1012 /l, Hb- 135 g/l, C. Polycystic ovary syndrome
WBCs - 9, 5 · 109 /l, stab neutrophils - 5%, D. Pituitary adenoma
segmentonuclear - 52%, lymphocytes - E. Sheehan syndrome
38%, monocytes - 5%, ESR - 20 mm/h. 189. During self-examination a 22-year-
Specify the doctor’s further tactics: old patient revealed a mammary tumour.
A. Emergency operation for acute Palpation revealed a firm, painless, freely
appendicitis mobile formation up to 2 cm, peripheral
B. Hospitalization, dynamic surveillance lymph nodes were not changed. USI
C. Send the patient home results: in the superior external quadrant
D. Refer the patient to a district therapist of the right mammary gland there was a
E. Administration of additional examinati- big formation of increased echogenicity,
on: abdominal ultrasound, x-ray contrast sized 18x17 mm. The patient was provi-
study of the gastrointestinal tract sionally diagnosed with fibroadenoma.
What is a doctor’s further tactics?
186. A 56-year-old patient was undergoing
a surgery for suture repair of perforated A. Surgical removal of the tumour prior to
ulcer. During the operation the cardi- pregnancy
omonitor registered ventricular fibrillati- B. Dynamic follow-up
on. The first-priority measure should be: C. Surgical treatment after pregnancy
D. Radical mastectomy
A. Electrical defibrillation E. Nonsteroid anti-inflammatory drugs,
B. Injection of adrenalin oral contraceptives
C. Injection of lidocaine
D. Injection of atropine 190. A 42-year-old female lives in the
E. Injection of calcium chloride basement, is unemployed, undernouri-
shed. She complains of having general
187. On the 10th day postpartum a weakness, hair loss, brittle nails for six
puerperant woman complains of pain and months, likes to eat chalk. Objectively:
heaviness in the left mammary gland. the patient is emaciated, pale, has dry
Body temperature is 38, 8o C, Ps- 94 bpm. skin. Peripheral lymph nodes are not
The left mammary gland is edematic, enlarged. Liver is +1,5 cm. In blood: RBCs
the supero-external quadrant of skin - 1, 8 · 1012/l, Hb- 62 g/l, colour index - 0,78,
is hyperemic. Fluctuation symptom is reticulocytes - 0,5o /oo , ESR- 18 mm/h.
absent. The nipples discharge drops of Leukogram exhibits no pathology. What
milk when pressed. What is a doctor’s is a provisional diagnosis?
further tactics?
A. Nutritional iron deficiency anaemia
A. Antibiotic therapy, immobilization and B. Chronic hepatitis
expression of breast milk C. B12 -deficiency anaemia
B. Compress to both mammary glands D. Acquired haemolytic anaemia
C. Inhibition of lactation E. Congenital haemolytic anaemia
D. Physiotherapy
E. Opening of the abscess and drainage of 191. A 78-year-old patient complains of
the mammary gland severe pain in the lower abdomen, peri-
neum and rectum; intense urge to urinate
188. A 30-year-old female patient and inability to urinate without assistance.
complains of milk discharge from the Abdomen palpation reveals a moderately
mammary glands, 5-month absence of painful globular formation above the
menstruation. She had one physiologi- pubis, there is percussion dullness over
cal labour four years ago. Objectively: the formation. What is the most likely di-
mammary glands are normally developed. agnosis?
Bimanual examination reveals that the
uterus is decreased in size, the ovaries
are of normal size. MRI-scan shows no
cerebral pathologies. Concentration of
thyroid-stimulating hormone is normal.
Krok 2 Medicine 2012 27

A. Acute urinary retention ne medical examination he presents no


B. Chronic urinary retention problems. In blood: RBCs - 4, 5·1012/l, Hb-
C. Chronic incomplete urinary retention 80 g/l, WBCs - 2, 8 · 109 /l, thrombocytes -
D. Paradoxical ischuria 30 · 109 /l. Decide if this person can work
E. Cystitis with sources of ionizing radiation:
192. A 3-year-old child with ARVI had A. Working with radioactive substances
been administered biseptol, paracetamol, and other sources of ionizing radiation is
nazoferon. On the third day of treatment contraindicated
the baby’s condition deteriorated: B. The patient is allowed to work with
he developed sore throat, stomatitis, radioactive substances
conjunctivitis, hypersalivation, painful C. The patient can only work with radi-
dark red spots on the neck, face, chest oactive substances of low activity
and legs, then the spots were replaced wi- D. The patient can be allowed to work
th vesicles. Examination revealed lesions after an extended medical examination
of mucous membranes around the mouth E. The patient is allowed to work with
and anus. What is your provisional di- radioactive substances for the limited
agnosis? period of time
A. Stevens-Johnson syndrome 196. A patient who undergoes treatment
B. Atopic dermatitis at a tuberculosis clinic has complained
C. Chickenpox of having progressing headache for the
D. Serum sickness last 3 weeks. Neurological examination
E. Bullous dermatitis reveals rigidity of occipital muscles, no
193. Preventive examination of a 50-year- focal symptoms. What is your provisional
old woman revealed a dense tumour of diagnosis?
the right mammary gland up to 5 cm in di- A. Tuberculous meningitis
ameter without distinct outlines. The skin B. Chorea minor
over the tumour looked like lemon peel. C. Brain tumour
Palpation revealed a lymph node in the D. Myelitis
axillary region. What is the most likely di- E. Convexital arachnoiditis
agnosis?
197. A patient with chronic suppurati-
A. Breast cancer ve otitis has developed severe headache,
B. Lactocele vomiting, body temperature rise. The
C. Diffuse mastopathy meningeal symptoms are present. There
D. Mastitis are no focal neurological symptoms. The
E. Breast lipoma further tactics of a doctor should be:
194. A 20-year-old patient complains of A. Urgent hospitalization and diagnostic
breath shortness, continuous dull heart lumbar puncture
pain, irritability. Objectively: general B. Skull radiography
condition is satisfactory, the pulse is labi- C. Regular medical check-up
le, AP- 130/60 mm Hg. ECG shows D. Administration of anti-inflammatory
repolarization disorder. The patient has drugs
been diagnosed with cardiac-type neuroci- E. Referral for a consultation with
rculatory dystonia. The patient should otolaryngologist
receive treatment under the following
conditions: 198. A 12-year-old child had three attacks
of acute rheumatic fever accompani-
A. Outpatient treatment ed by carditis. Examination revealed
B. Inpatient treatment at the therapeutic the symptoms of chronic tonsillitis, mi-
department tral insufficiency, carious teeth. What
C. Inpatient treatment at the cardiology is the optimal method of secondary
department prophylaxis?
D. Inpatient treatment at the cardiac
surgery department
E. Inpatient treatment at the psychiatric
department
195. A 38-year-old male works within the
range of ionizing radiation. At a routi-
Krok 2 Medicine 2012 28

A. Year-round bicillin prophylaxis till the A. Physiological III sound


age of 25 B. The sound of the mitral valve opening
B. Course of cardiotrophic drugs twice a C. Protodiastolic gallop rhythm
year D. Pericardial diastolic sound
C. Year-round bicillin prophylaxis for 3 E. The sound of the tricuspid valve opening
years
D. Tonsillectomy 200. A patient’s condition is getting worse
E. Oral cavity sanitation towards evening: she becomes excited,
complains of "internal anxiety", "a wei-
199. Routine examination of a 16-year- ght on her heart", foreboding of evil -
old boy revealed the presence of three "something evil will happen to me or my
heart sounds on auscultation. The third family". The patient is sad, melancholic,
sound is low and occurs in early diastole, has poor appetite and sleep disorders.
there is no additional murmur. In history: Specify the kind of mental disorder:
pneumonia six months ago. The pati-
ent presents no problems. Examination A. Anxious depression
revealed hyposthenia, underdevelopment B. Somatized depression
of muscles. Laboratory and instrumental C. Endogenous depression
studies reveald no peculiarities. What is D. Hypochondriac depression
the origin of the additional heart sound? E. Agitated depression
Krok 2 Medicine 2013 1

1. A 28-year-old patient complains of


infertility. The patient has been marri- A. Right-sided tubal pregnancy
ed for 4 years, has regular sexual life B. Right ovary apoplexy
and does not use contraceptives but has C. Acute right-sided salpingoophoritis
never got pregnant. Examination revealed D. Pelvioperitonitis
normal state of the genitals, tubal patency. E. Incipient abortion
Basal body temperature recorded over the
course of 3 consecutive menstrual cycles 5. 5 weeks after hypothermia a 22-year-old
appeared to have a single phase. What is patient developed fever, weakness, muscle
the most likely cause of infertility? pain, inability to move independently.
Objectively: tenderness, induration of
A. Anovulatory menstrual cycle shoulder and shin muscles, restricted acti-
B. Immunological infertility ve movements, erythema on the anteri-
C. Genital endometriosis or surface of chest. There is a periorbi-
D. Chronic salpingoophoritis tal edema with heliotropic erythema.
E. Ovulatory menstrual cycle Gottron’s sign is present. What study is
required to verify the diagnosis?
2. A 40-year-old female patient has been
hospitalized for attacks of asphyxia, cough A. Muscle biopsy
with phlegm. She has a 4-year history of B. Aminotransferase activity
the disease. The first attack of asphyxia C. Pneumoarthrography
occurred during her stay in the countrysi- D. ASO titre
de. Further attacks occurred while cleani- E. Rheumatoid factor
ng the room. After 3 days of inpatient
treatment the patient’s condition has si- 6. A 24-year-old patient consulted a
gnificantly improved. What is the most doctor about enlarged submandibular
likely etiological factor? lymph nodes. Objectively: submandi-
bular, axillary and inguinal lymph nodes
A. Household allergens are enlarged. Chest radiograph shows
B. Pollen enlarged mediastinal lymph nodes. In
C. Infectious blood: RBCs - 3, 4 · 1012 /l, Hb- 100 g/l,
D. Chemicals colour index - 0,88, thrombocytes - 190 ·
E. Psychogenic 109 /l, WBCs - 7, 5 · 109/l, eosinophils -
8%, stab neutrophiles - 2%, segmented
3. A 13-year-old girl complains of peri- neutrophiles - 67%, lymphocytes - 23%,
odic prickly pain in the heart regi- ESR - 22 mm/h. What study is required to
on. Percussion revealed no changes of verify the cause of lymphadenopathy?
cardiac borders. Auscultation revealed
arrhythmic enhanced heart sounds, A. Open biopsy of lymph nodes
extrasystole at the 20-25 cardiac impulse. B. Ultrasound examination of the
ECG showed the sinus rhythm, impai- abdomen
red repolarization, single supraventricular C. Mediastinal tomography
extrasystoles at rest. What is the most li- D. Puncture biopsy of lymph nodes
kely diagnosis? E. Sternal puncture
A. Vegetative-vascular dysfunction 7. A patient operated for acute paraprocti-
B. Rheumatism tis undergoes antibacterial and detoxi-
C. Nonrheumatic carditis fication therapy, the local course of the
D. Myocardial degeneration disease has the positive dynamics. Since
E. Intoxication syndrome the operation the patient has had chi-
lls, pyrexia, tachycardia, euphoria for five
4. A 28-year-old patient has been taken days. The doctor suspected sepsis. What
to a hospital for acute pain in the lower study will confirm the diagnosis?
abdomen. There was a brief syncope.
The delay ofmenstruation is 2 months. A. Blood culture for a pathogen
Objectively: the patient has pale skin, B. X-ray of lungs
AP- 90/50 mm Hg, Ps- 110/min. Lower C. Liver ultrasound
abdomen is extremely painful. Vaginal D. Determining the rate of microbial
examination reveals uterus enlargement. contamination of wound
There is positive Promtov’s sign. Right E. Determining the rate of average-weight
appendages are enlarged and very pai- molecules
nful. Posterior vault hangs over. What is
the most likely diagnosis? 8. A painter working at a motorcar
Krok 2 Medicine 2013 2

plant has been diagnosed with A. Tuberculous meningitis


moderately severe intoxication with ami- B. Secondary purulent meningitis
de compounds of benzene. The in-patient C. Epidemic cerebrospinal meningitis
treatment resulted in a considerable D. Serous meningitis
health improvement. What expert deci- E. Pneumococcal meningitis
sion should be made in this case?
11. A 13-year-old girl has a 5-year histrory
A. The patient should be issued a sick list of pain in the right hypochondrium irradi-
for out-patient treatment ating to the right shoulder blade. The pain
B. The patient may get back to work attacks are usually associated with diet vi-
providing he will keep to hygiene and olations, they are short and can be easily
sanitary regulations relieved by antispasmodic drugs. During a
C. The patient should be referred to pain attack, palpation of the abdomen is
the medio-social expert commission for painful, the pain is most intensive in the
evaluation of percentage of work capicty projection of the gallbladder. What is the
loss most likely diagnosis?
D. The patient should be referred to
the medio-social expert commission for A. Biliary dyskinesia
attributing the disability group because of B. Chronic cholecystitis
an occupational disease C. Chronic gastroduodenitis
E. - D. Chronic pancreatitis
E. Duodenal ulcer
9. A 58-year-old female patient came
to the antenatal clinic complaining of 12. A 24-year-old patient got a puncture
bloody light-red discharges from the injury below the Poupart’s ligament
genital tracts. Menopause is 12 years. accompanied by intense arterial bleedi-
Gynaecological examination revealed ng. The best method to temporarily stop
age involution of externalia and vagi- the bleeding in the patient would be:
na; uterine cervix was unchanged, there A. Compression band
were scant bloody discharges from uteri- B. Esmarch’s tourniquet
ne cervix, uterus was of normal size; C. Maximum limb bending
uterine appendages were not palpable; D. Compressing a blood vessel with a
parametria were free. What is the most clamp
likely diagnosis? E. Wound suturing
A. Uterine carcinoma 13. A 64-year-old patient complains of
B. Atrophic colpitis severe pain in the right side of chest,
C. Abnormalities of menstrual cycle of dyspnea, dry cough which appeared
climacteric nature suddenly on exertion. Objectively: the ri-
D. Cervical carcinoma ght side of the chest lags behind in the act
E. Granulosa cell tumor of ovary of breathing. Percussion reveals tympanic
10. A 7-year-old boy had complai- sound. Auscultation reveals pronouncedly
ned of headache, nausea, fatigue for 3 diminished breath sounds on the right. Ps-
weeks. His condition gradually deteri- 100/min, weak, arrhythmic. AP- 100/50
orated, headache and general weakness mm Hg. Cardiac sounds are decreased.
progressed. The boy had bronchitis at What disease can be suspected in this pati-
the age of 3. His father has a history ent?
of pulmonary tuberculosis. Objectively: A. Right-sided pneumothorax
body temperature 37, 5oC, conscious, li- B. Right-sided hydrothorax
es supine, with the hip and knee flexed C. Right-sided dry pleurisy
to 90 degrees, nuchal rigidity +6 cm, D. Right-sided pleuropneumonia
partial ptosis of the right eyelid, the di- E. PATE
lated right pupil. General hyperalgesia
is present. Liquor: transparent, pressure 14. An 18-year-old patient since childhood
- 400 mm of water column, protein - suffers from bleeding disorder after mi-
1,5%, cytosis - 610/3 with predominant nor injuries. His younger brother also
lymphocytes, sugar - 1,22 mmol/l, chlori- has bleeding disorders with occasional
des - 500 mmol/l. What is the most likely haemarthrosis. Which laboratory test will
diagnosis? be informative for diagnosis verification?
Krok 2 Medicine 2013 3

A. Clotting time A. II degree flame burn of hands with an


B. Fibrinogen rate area of 4/
C. Blood clot retraction B. II degree flame burn of hands with an
D. Thrombocyte count area of 2/
E. Determination of prothrombin time C. IIIa degree flame burn of hands with an
area of 4/
15. A patient complains of fatigue, lack D. III degree flame burn of hands with an
of appetite, pain and burning sensation area of 4/
in the tongue, numbness of the distal li- E. IIb degree flame burn of hands with an
mbs, diarrhea. Objectively: pale skin with area of 2/
lemon-yellow tint, face puffiness, brown
pigmentation in the form of a "butterfly", 18. During the periodic medical exami-
bright red areas on the tongue. The li- nation an assembly fitter (works on
ver is 3 cm below the costal margin, soldering details) didn’t report any health
soft. Blood count: RBCs - 1, 5 · 1012 /l, problems. Closer examination revealed
colour index - 1,2, WBCs - 3, 8 · 109 /l, signs of asthenic-vegetative syndrome.
thrombocytes - 180 · 109 /l, eosinophils - Blood included red blood cells with
0%, stab neutrophils - 1%, segmented basophilic aggregations and a somewhat
neutrophils - 58%, lymphocytes - 38% higher number of reticulocytes, uri-
monocytes - 3%, RBC macrocytosis. ESR ne had a high concentration of delta-
- 28 mm/h. What diagnosis are these aminolevulinic acid. The complex of
presentation typical for? symptoms indicates the initial stage of
chronic intoxication with:
A. B12 -deficiency anemia
B. Iron deficiency anemia A. Lead
C. Aplastic anemia B. Manganese
D. Acute erythromyelosis C. Mercury
E. Chronic adrenal failure D. Tin
E. Ethanol
16. Medical examination of a 19-
year-old worker revealed generalized 19. A patient complains about sudden
lymphadenopathy mainly affecting the onsets of paroxysmal pain in the right
posterior cervical, axillary and ulnar lumbar region. 2 hours after the onset the
lymph nodes. There are multiple injecti- patient had hematuria. Plain radiograph
on marks on the elbow bend skin. The of the lumbar region shows no pathologi-
man denies taking drugs, the presence cal shadows. USI reveals pyelocaliectasis
of injection marks ascribes to influenza on the right, the left kidney is normal.
treatment. Blood count: RBCs- 3, 2·1012/l, What is the most likely diagnosis?
Hb- 100 g/l, WBCs- 3, 1 · 109 /l, moderate
lymphopenia. What study is required in A. Renal colic
the first place? B. Acute appendicitis
C. Bowel volvulus
A. ELISA for HIV D. Torsion of the right ovary cyst
B. Immunogram E. Right renal pelvis tumour
C. Sternal puncture
D. X-ray of lungs 20. Examination of a group of persons
E. Lymph node biopsy living on the same territory revealed
the following common symptoms: dark-
17. A patient got flame burns of both yellow pigmentation of the tooth enamel,
hands. On the dorsal and palmar surface diffuse osteoporosis of bone apparatus,
of hands the blisters are filled with serous ossification of ligaments and joints, functi-
fluid. The wrist joint region is hyperemic. onal disorders of the central nervous
The forearms were not injured. What is system. This condition may be caused
the provisional diagnosis? by the excessive concentration of the
following microelement in food or drinki-
ng water:
A. Fluorine
B. Copper
C. Nickel
D. Iodine
E. Cesium
Krok 2 Medicine 2013 4

21. A 50-year-old patient complains of A. Panzinorm forte


bursting pain in the left lower limb that B. Insulin
is getting worse on exertion, swelling in C. Gastrozepin
the region of shin and foot. Objectively: D. Contrycal
left shin and foot are doughy, skin of the E. No-spa
lower shin is indurated and has a bronze
tint, subcutaneous veins are dilated, there 25. A patient is on the sick leave for 4
is an ulcer with necrotic masses. What is months continuously from the date of
the most likely diagnosis? injury. The treatment is going to last for
1-2 months more. Who has the right to
A. Postthrombophlebitic syndrome extend the duration of medical certificate
B. Chronic arterial insufficiency for this patient?
C. Acute arterial thrombosis
D. Deep vein thrombosis of the lower A. Medical advisory commission after
limbs medico-social expert commission exami-
E. Gangrene of the lower extremity nation
B. Medical superintendent
22. A 12-year-old boy has a 6-year hi- C. Medical advisory commission after
story of insulin-dependent diabetes. The inpatient treatment
disease is labile. Since recently there D. District doctor by agreement with a
have been periodical rises in blood department chief
pressure. Microalbuminuria test gave E. Medico-social expert commission
positive results. The patient’s condition
corresponds with the following stage of 26. Full-term pregnancy. Body weight
diabetic nephropathy: of the pregnant woman is 62 kg. The
fetus has the longitudinal position, the
A. Stage III - early-stage nephropathy fetal head is pressed against the pelvic
B. Stage I - renal hypertrophy and inlet. Abdominal circumference is 100
hyperfunction cm. Fundal height is 35 cm. What is the
C. Stage II - histological changes in the approximate weight of the fetus?
kidneys
D. Stage IV - advanced clinical A. 3 kg 500 g
nephropathy B. 4 kg
E. V stage - chronic renal failure C. 2 kg 500 g
D. 3 kg
23. A 48-year-old patient was found to E. 4 kg 500 g
have diffuse enlargement of the thyroid
gland, exophthalmia, weight loss of 4 kg 27. A rural hospital serves 6200 people.
in 2 months, sweating. Objectively: HR- Preventive examinations were planned
105/min, AP- 140/70 mm Hg. Defecati- for 560 farm workers with different risk
on act is normal. What kind of therapy is factors. 400 workers underwent preventi-
recommended in this case? ve examination. 120 individuals were
found to have cardiovascular problems,
A. Mercazolil 90 of them were registered with a di-
B. Radioiodine spensary department for health care mai-
C. Propranolol ntenance. Which indicator should be used
D. Lugol’s solution to assess the organization of health care
E. Thyroxine maintenance at the hospital?
24. A 48-year-old male patient complains A. Share of workers who actually
of constant pain in the upper abdomen, underwent preventive examinations
mostly on the left, that is getting worse B. Share of newly diagnosed patients
after taking meals; diarrhea, weight loss. C. Share of persons subject to preventive
The patient is an alcohol abuser. 2 years examinations
ago he had acute pancreatitis. Blood D. Share of diseased persons
amylase is 4 g/h·l. Coprogram shows E. Incidence of cardiovascular diseases
steatorrhea, creatorrhea. Blood glucose
is 6,0 mmol/l. What treatment is indicated 28. A 20-year-old patient complai-
for this patient? ns of severe headache, double vision,
weakness, fever, irritability. Objectively:
body temperature is at the rate of 38, 1o C,
the patient is reluctant to contact, sensi-
tive to stimuli. There is ptosis of the
Krok 2 Medicine 2013 5

left eyelid, exotropia, anisocoria S>D, A. Posterior vertex presentation


pronounced meningeal syndrome. On B. Anterior vertex presentation
lumbar puncture the cerebrospinal fluid C. Presentation of the bregma
flowed out under a pressure of 300 mm D. Brow presentation
Hg, the fluid is clear, slightly opalescent. E. Face presentation
24 hours later there appeared the fibri-
nous film. Protein - 1,4 g/l, lymphocytes - 32. During medical monitoring of labor
600/3 per mm3 , sugar - 0,3 mmol/l. What and professional training of 6th grade
is the provisional diagnosis? schoolchildren the school doctor found
that lessons in carpentry workshop are
A. Tuberculous meningitis held on Friday as the fourth and fifth
B. Meningococcal meningitis lesson of a total of 90 minutes, the motor
C. Lymphocytic Armstrong’s meningitis density at the first lesson is 69%, and 65%
D. Syphilitic meningitis at the second. Does the lesson organizati-
E. Mumps meningitis on meet the hygienic requirements?

29. A 32-year-old patient has developed A. Meets the requirements


an acute condition after hypothermia: B. Does not meet the requirements in
temperature - 40oC, cough with 200 terms of training duration
ml of sputum per day. The sputum is C. Does not meet the requirements in
purulent, foul-smelling. To the right of terms of motor density at the 1st lesson
the lower lobe the mixed moist rales can D. Does not meet the requirements in
be auscultated. Blood tst results: WBCs - terms of motor density at the 2nd lesson
18, 0 · 109 /l, ESR - 45 mm/h. Radiographi- E. Does not meet the requirements as
cally: in the lower lobe of the right lung there are two consecutive lessons
there is a thick-walled cavity up to 6 cm
in diameter with a high horizontal level. 33. During coal extraction in a mine the
What is the most likely diagnosis? concentration of coal dust in the working
area is 450 mg/m3 (MPC is 10 mg/m3 ).
A. Lung abscess What occupational respiratory disease
B. Fibro-cavernous pulmonary may develop in miners?
tuberculosis
C. Lung cyst A. Anthracosis
D. Decomposing lung carcinoma B. Allergic nasopharyngitis
E. Infiltrative pulmonary tuberculosis C. Byssinosis
D. Siderosis
30. A family consists of 5 persons. E. Talcosis
The husband is a stope miner. His
spouse is a housewife. Their 20-year-old 34. A patient with suspected pheochromocytoma
daughter works as a kindergarten teacher. has normal blood pressure in the peri-
Their 18-year-old son is a student. The ods between the atacks and a tendency
grandmother is a pensioner, she has di- to tachycardia. Urine test revealed no
abetes. Which member of this family can pathology. It was decided to use a
be primarily classed among the group of provocative test with histamine. What
persons with a high risk of tuberculosis in medication should be prepared to provide
the planning of preventive examinations emergency care in case of a positive test
for tuberculosis? result?

A. Grandmother A. Phentolamine
B. Husband B. Pipolphen
C. Daughter C. Nifedipine
D. Son D. Mesatonum
E. Wife E. Prednisolone

31. A newborn’s head is of doli- 35. A 54-year-old patient complains of


chocephalic shape, that is front-to-back frequent painful urination, chills, fever
elongated. Examination of the occipital up to 38o C. Urine test results: protein -
region revealed a labour tumour located 0,33 g/L, WBCs - up to 50-60 in the field
in the middle between the prefontanel of vision, RBCs - 5-8 in the field of vi-
and posterior fontanel. Specify the type sion, gram-negative bacilli. Which of the
of fetal presentation: listed antibiotics should be preferred in
this case?
Krok 2 Medicine 2013 6

A. Ciprofloxacin A. Chronic type A gastritis


B. Oxacillin B. Chronic type B gastritis
C. Erythromycin C. Chronic type C gastritis
D. Tetracycline D. Menetrier disease
E. Tseporin E. Stomach cancer
36. The share of circulatory diseases was 40. A 45-year-old female patient complai-
15% in the total of registered diseases ns of frequent liquid stools with a lot of
among city population. What statistic indi- mucus, pus and blood; pain across the
cator is it? abdomen, loss of 7 kg within 6 months.
She has a 1-year history of non-specific
A. Extensive ulcerative colitis. What group of drugs
B. Intensive should be preferred for this patient?
C. Demonstrative
D. Ratio A. Corticosteroids
E. Average B. Antibacterial
C. Sulfonamides
37. During a regular medical examination D. Nitrofurans
at a metallurgical plant 20% of workers E. Polyenzymes
were found overweight (body weight was
5-14% higher than normal), and had early 41. A 24-year-old female teacher complai-
signs of obesity (grade I-II) with Quetelet ns of dizziness and heart pain irradiati-
index from 26 to 30. What products share ng to the left nipple. Pain is not associ-
must be reduced in the diet of this group ated with physical activity and cannot be
of people in the first place in order to relieved by nitroglycerin, it abates after
normalize their body weight? taking Valocordin and lasts an hour or
more. The patient has a nearly 2-year hi-
A. Bakery products story of this disease. Objectively: Ps- 76
B. Vegetables bpm. AP- 110/70 mm Hg. Heart borders
C. Fruit are normal, heart sounds are clear.
D. Milk and dairy products The ECG shows respiratory arrhythmia.
E. Meat and fish products Radiograph of the cervicothoracic spi-
ne shows no pathology. Lungs, abdomen
38. Several hours before, a 28-year- are unremarkable. What changes in blood
old patient suddenly developed acute formula can be expected?
headache and repeated vomiting, then
lost consciousness. Objectively: focal A. No changes
neurological symptoms were not found. B. Leukocytosis
Pronounced meningeal symptoms were C. Thrombocytopenia
revealed. AP - 120/80 mm Hg. Accordi- D. Leukemic hiatus
ng to clinical and liquorological findi- E. Increased ESR
ngs the patient was diagnosed with
subarachnoid haemorrhage. After admi- 42. A 60-year-old male patient, who works
nistration of dehydrants the patient’s as a construction worker, complains of
condition somewhat improved. What is pain in the right hip and knee joints,
the main component of further emergency that is getting worse on exertion. These
care? presentations have been observed for
the last 5 years. Objectively: the pati-
A. Coagulants ent is overnourished. Right knee joi-
B. Anticoagulants nt is moderately deformed. Examinati-
C. Antiaggregants on of other organs and systems revealed
D. Fibrinolytics no pathology. Blood tet results: WBCs -
E. Corticosteroids 8, 2 · 109 /l, ESR - 15 mm/h. Uric acid - 0,35
mmol/l. What is the most likely diagnosis?
39. Gastric juice analysis of a 42-year-
old male patient revealed absence of A. Deforming osteoarthritis
free hydrochloric acid at all stages. B. Reactive arthritis
Endoscopy revealed pallor, thinning of C. Gout
gastric mucosa, smoothed folds. Mi- D. Rheumatoid arthritis
croscopically the atrophy of glands with E. Reiter’s disease
intestinal metaplasia was found. What di-
sease is this situation typical for? 43. Chief physician of a polyclinic
encharged a district doctor with a task to
Krok 2 Medicine 2013 7

determine the pathological prevalence of appeared 5 days ago after a contact with
disease N in his district. What document a person having ARVI. Objectively: the
allows to estimate the disease prevalence child is in grave condition. Temperature
in the population of a medical district? of 38o C, cyanosis of nasolabial triangle is
present. RR- 54/min, nasal flaring while
A. Prophylactic examinations register breathing. There was percussion dullness
B. Statistic coupons (+) on the right below the scapula angle, and
C. Statistic coupons (-) tympanic sound over the rest of lungs.
D. Statistic coupons (+) and (-) Auscultation revealed bilateral fine moist
E. Vouchers for medical appointments rales predominating on the right. What is
the most likely diagnosis?
44. A 32-year-old female complains of di-
zziness, headache, palpitation, tremor. For A. Acute pneumonia
the last several months she has been under B. ARVI
outpatient observation for the increased C. Acute laryngotracheitis
arterial pressure. Since recently such D. Acute bronchitis
attacks have become more frequent and E. Acute bronchiolitis
severe. Objectively: skin is covered with
clammy sweat, tremor of the extremities 48. A 40-year-old patient complains of
is present. HR- 110/min, AP- 220/140 mm fever up to 39o C, cough with sputum and
Hg. Heart sounds are muffled. Blood test blood admixtures, dyspnea, weakness,
results: WBCs - 9, 8 · 109/l, ESR - 22 mm/h. herpetic rash on the lips. Objectively:
Blood glucose - 9,8 millimole/l. What di- respiration rate - 32/min. Under the
sease is the most likely cause of this crisis? shoulder blade on the right the increased
vocal fremitus and dullness of percussi-
A. Pheochromocytoma on sound were revealed. Auscultation
B. Essential hypertension revealed bronchial respiration. Blood
C. Preeclampsia count: WBCs - 14 · 109 /l, ESR - 35 mm/h.
D. Primary hyperaldosteronism What is the provisional diagnosis?
E. Diabetic glomerulosclerosis
A. Right-sided croupous pneumonia
45. A 55-year-old male has a 1,5-year B. Focal right-sided pneumonia
history of viral cirrhosis with symptoms C. Cavernous tuberculosis of the right lung
of portal hypertension. Over the last D. Lung cancer
month the weakness has progrssed, E. Exudative pleuritis
there appeared coffee ground vomit. Fi-
brogastroduodenoscopy revealed vari- 49. A patient underwent stomach resecti-
ceal esophageal haemorrhage. What drug on. During the operation, the left upper
should be used to reduce the pressure in limb of the patient was abducted and fi-
the portal vein? xed to the operating table for anesthetic
management. Postoperatively, the pati-
A. Vasopressin ent developed dysfunction of the upper
B. Reserpine extremity in form of "drop hand."This
C. Calcium gluconate symptom results from the damage of the
D. Dicynone following nerve:
E. Furosemide
A. Radial nerve
46. In an urban settlement situated on the B. Axillary nerve
riverbank an outbreak of hepatitis A was C. Ulnar nerve
registered. The disease might have water D. Median nerve
origin. This assumption can be confirmed E. Musculocutaneous nerve
by growth of the following indicators of
water quality: 50. In the morning a patient had nausea,
abdominal discomfort, single vomiti-
A. Number of coli-phages ng, dry mouth. In the evening, the
B. Escherichia coli index patient presented with the increasing
C. Oxidability general weakness, double vision, diffi-
D. Presence of benign leptospirosis cult swallowing of solid food. Objectively:
pathogen ptosis, mydriasis, anisocoria, absence of
E. Index of fecal coli-forms gag and pharyngeal reflex, dry mucous
membranes. The previous evening the
47. A 9-month-old child presents with patient had dinner with canned food
fever, cough, dyspnea. The symptoms
Krok 2 Medicine 2013 8

and alcohol. What is the presumptive di- tive airway pressure). Ro-gram of lungs
agnosis? shows reticular and nodular pattern, air
bronhogram. What is the most likely cause
A. Botulism of respiratory distress syndrome?
B. Food toxicoinfection
C. Intoxication with unknown poison A. Hyaline membrane disease
D. Acute ischemic stroke B. Segmental atelectasis
E. Poliomyelitis C. Bronchopulmonary dysplasia
D. Congenital pulmonary emphysema
51. An 8-year-old child complains of fever E. Edematous hemorrhagic syndrome
up to 38, 8o C, throat pain when swallowi-
ng, skin rash. Objectively: lacunar tonsi- 55. An 8-year-old child with a 3-year
llitis, circumscribed hyperaemia and history of diabetes was hospitalized in
enanthema of soft palate, pinoint-sized hyperglycemic coma. Specify the initial
skin rash, mostly in the folds and on dose of insulin to be administered:
the flexor surfaces of the extremities,
pale nasolabial triangle. Which antibiotic A. 0,1-0,2 U/kg of body weight per hour
should be administered in the first place? B. 0,05 U/kg of body weight per hour
C. 0,2-0,3 U/kg of body weight per hour
A. Penicillin D. 0,3-0,4U/kg of body weight per hour
B. Gentamicin E. 0,4-0,5 U/kg of body weight per hour
C. Ampicillin
D. Lincomycin 56. A 12-year-old girl undergoes
E. Tetracycline regular gastroenterological check-ups for
duodenal ulcer, biliary dyskinesia. What
52. A 28-year-old female patient with a is the recommended frequency of anti-
six-year history of Raynaud’s syndrome relapse treatment?
has recently developed pain in the small
joints of hands, difficult movement of food A. Twice a year
down the esophagus. What kind of disease B. Every 2 months
can you think of in this case? C. Every 3 months
D. Once a year
A. Systemic scleroderma E. Three times a year
B. Periarteritis nodosa
C. Rheumatoid arthritis 57. A 47-year-old patient came to see a
D. Systemic lupus erythematosus doctor on the 7th day of disease. The di-
E. Pseudotrichiniasis sease developed very fast: after the chill
body temperature rose up to 40o C and
53. A 16-year-old girl had had polyuria, lasted up to 7 hours, then it dropped
polydipsia for 2 months. She had lost abruptly, which caused profuse sweat.
8 kg with a good appetite. The patient There were three such attacks occuri-
was urgently hospitalized for abdominal ng once in two days. Two days ago the
pain and nausea. Examination revealed patient arrived from Africa. Objectively:
glycemia at the rate of 18 mmol/l, pale skin, subicteric sclera, significantly
glycosuria at the rate of 24 g/l. Insulin enlarged liver and spleen. What is the
and infusion of isotonic solutions of sodi- cause of fever attacks in this disease?
um chloride and glucose eliminated these
problems, including thirst. What is the A. Erythrocytic schizogony
most likely diagnosis? B. Tissue schizogony
C. Exotoxin of a causative agent
A. Diabetes mellitus type 1 D. Endotoxin of a causative agent
B. Diabetes mellitus type 2 E. Gametocytes
C. Renal glycosuria
D. Diabetes insipidus 58. On the 2nd day of illness a 27-
E. Secondary (symptomatic) diabetes year-old patient complains of unbearable
headache, repeated vomiting. Objecti-
54. A boy was born at 32 weeks gestati- vely: the patient is in a grave condition.
on. 2 hours after birth he developed He is conscious but adynamic. Lies in a
respiratory distress. The RD severity forced position with his head thrown back.
assessed by Silverman score was 5. The There is no skin rash. Nuchal muscles
respiratory disorders progressed, respi- are evidently rigid, there are Kernig’s and
ratory failure couldn’t be eliminated by Brudzinski’s signs. to - 39, 5oC, Ps- 120/min,
Martin-Bouyer CPAP (continuous posi- AP- 130/80 mm Hg. The leading syndrome
Krok 2 Medicine 2013 9

of this disease is caused by: Heart sounds are muffled, rhythmic.


ECG: Rv1+Sv5=15 mm. X-ray picture
A. Liquor hypertension shows the bulging of pulmonary artery
B. Liquor hypotension cone, right ventricle enlargement. What is
C. Affection of the cranial nerve nuclei the most likely cause of this condition?
D. Haemorrhages into the adrenal glands
E. Hyperthermy A. Pulmonary heart
B. Atherosclerotic cardiosclerosis
59. A 42-year-old female patient suffers C. Dilatation cardiomyopathy
from micronodular cryptogenic cirrhosis. D. Mitral stenosis
Over the last week her condition has E. Primary pulmonary hypertension
deteriorated: she developed convulsions,
mental confusion, progressing jaundice. 63. A 40-year-old woman with a history
What study may give reasons for such of combined mitral valve disease wi-
aggravation? th predominant stenosis complains of
dyspnea, asthma attacks at night, heart
A. Determination of serum ammonia problems. At present, she is unable to do
B. Determination of cholesterol ethers easy housework. What is the optimal tacti-
C. Determination of alpha-phetoprotein cs of the patient treatment?
D. Determination of ALAT and ASAT
E. Determination of alkaline phosphatase A. Mitral commissurotomy
B. Implantation of an artificial valve
60. A 60-year-old patient complai- C. Antiarrhythmia therapy
ns of recurrent pain in the proximal D. Treatment of heart failure
interphalangeal and wrist joints, their E. Antirheumatic therapy
periodic swelling and reddening that
have been observed for 4 years. X-ray 64. A 25-year-old patient complains of
picture represents changes in form of having dull heart pain for the last 10
osteoporosis, joint space narrowing and days, dyspnea on mild exertion, palpi-
single usuras. What is the most likely di- tations. The diasease developed 2 weeks
agnosis? ago after a respiratory infection. Objecti-
vely: acrocyanosis, AP- 90/75 mm Hg, Ps-
A. Rheumatoid arthritis 96/min. Cardiac borders appear to be shi-
B. Osteoarthritis fted to the left and right. Heart sounds
C. Gout are weak and have triple rhythm, there is
D. Pseudogout systolic murmur at the apex. ECG showed
E. Multiple myeloma sinus rhythm, complete left bundle branch
block. What is the most likely diagnosis?
61. A 28-year-old patient has been hospi-
talized for the pain in the epigastric regi- A. Infectious-allergic myocarditis
on. He has a 10-year history of duodenal B. Exudative pericarditis
ulcer (DU). Recently, the pain character C. Infective endocarditis
has changed: it became permanent, persi- D. Myocarditic cardiosclerosis
stent, irradiating to the back. There are E. Vegetative-vascular dystonia
general weakness, dizziness, fatigue. The
patient has put off weight. Objectively: 65. A child undergoes in-patient
HR- 68/min, AP- 120/80 mm Hg. What is treatment for acute staphylococcal
most likely cause of deterioration? destruction of the right lung. Unexpectedly
he develped acute chest pain on the ri-
A. Penetration ght, dyspnea, cyanosis. The right side of
B. Haemorrhage chest lags behind in the respiratory act.
C. Perforation of duodenal wall Percussion reveals dullness in the lower
D. Exacerbation of duodenal ulcer parts on the right, bandbox resonance in
E. Stenosis development the upper parts. Borders of the relati-
ve cardiac dullness are shifted to the
62. A 57-year-old male patient complai- left. What complication has most likely
ns of dyspnea on exertion, heaviness developed?
in the right hypochondrium and shin
edemata towards evening. Objectively:
temperature - 38, 1oC, HR- 20/min,
HR=Ps=92/min, AP- 140/90 mm Hg.
There is apparent kyphoscoliosis. In the
lungs single dry rales can be auscultated.
Krok 2 Medicine 2013 10

A. Right-sided pyopneumothorax ameter. Excretory urograms show normal


B. Pleural empyema condition on the left, and the deformati-
C. Spontaneous pneumothorax on of the superior renal calyces with sati-
D. Exudative pleuritis sfactory function on the right. What kind
E. Right lung abscess of disease can you think of?
66. Examination of a 13-year-old gi- A. Simple cyst of the right kidney
rl revealed acute glomerulonephritis, B. Multicystic kidney disease
nephritic syndrome at the initial stage wi- C. Multiple cysts of the right kidney
thout renal dysfunction. What is the main D. Tumour of the right kidney
drug of choice for the basic therapy of this E. Right hydronephrosis
patient?
70. While staying in a stuffy room a 19-
A. Antibiotic year-old emotionally labile girl developed
B. Prednisolone severe weakness, dizziness, blackout,
C. Curantyl nausea and loss of consciousness without
D. Heparin convulsions. Objectively: the patient is
E. Saluretic unconscious, the skin is pale, extremiti-
es are cold. AP- 90/60 mm Hg, Ps- 96/min,
67. A 28-year-old patient consulted deficient, breathing is shallow. Pupillary
a surgeon about pain, edema and and tendon reflexes are present. There are
hyperemia of the left side of his face, no pathological signs. What is the most li-
weakness, fever up to 39o C. These mani- kely diagnosis?
festations has been present for three days.
Objectively: there is an inflammatory infi- A. Syncope
ltrate of the left nasolabial fold 4x4 cm B. Vegetovascular paroxysm
large with a necrotic core in the center, the C. Epileptic attack
pronounced edema of the left side of face, D. Hysterical neurosis
moderate nuchal rigidity. What treatment E. Transient ischemic attack
is needed?
71. A 13-year-old boy complains of pain
A. Hospitalization, antibiotics, anti- in the upper third of his left thigh, body
coagulants, rest temperature rise up to 39oC. There is a
B. Antibiotics, surgery under general swelling in the upper third of thigh and
anesthesia inguinal fold smoothness. The extremity is
C. Wet-to-dry dressings with antiseptics, in a half-bent position. Active and passi-
aspirin ve movements are not possible because of
D. Physiotherapy, immunomodulators the sharp pain. What is the most likely di-
E. Antibiotics, angioprotectors, daily agnosis?
dressings
A. Acute hematogenous osteomyelitis
68. A 6-year-old child complains of B. Acute coxitis
frequent liquid stool and vomiting. On the C. Intermuscular phlegmon
2nd day of desease the child presented D. Osteosarcoma
with inertness, temperature rise up to E. Brodie’s disease
38, 2oC, Ps- 150 bpm, scaphoid abdomen,
palpatory painful sigmoid colon, defecati- 72. A 12-year-old cild has been hit on
on 10 times a day with liquid, scarce stool the stomach. The patient is in moderately
with mucus and streaks of green. What is grave condition, has a forced position in
a provisional diagnosis? bed. The skin is pale, Ps- 122/min. The
stress on the left costal arch causes a
A. Shigellosis slight pain. There are positive Weinert,
B. Salmonellosis Kulenkampff symptoms. Macroscopically
C. Escherichiosis the urine is unchanged. What is the most
D. Intestinal amebiasis likely diagnosis?
E. Yersiniosis
A. Spleen rupture, abdominal bleeding
69. A 48-year-old patient complains of B. Left kidney rupture, retroperitoneal
having dull pain in the right lumbar regi- hematoma
on for over three years. USI shows that C. Rupture of the pancreas
kidneys are of normal size, at the upper D. Liver rupture, abdominal bleeding
pole of the right kidney there is a fluid- E. Rupture of a hollow organ, peritonitis
containing formation up to 12 cm in di-
Krok 2 Medicine 2013 11

73. A 30-year-old multigravida has been vacuoles. Make a diagnosis:


in labour for 18 hours. 2 hours ago the
pushing stage began. Fetal heart rate is A. Complicated cataract of the right eye
clear, rhythmic, 136/min. Vaginal exami- B. Senile cataract of the right eye
nation reveals the completecervical di- C. Diabetic cataract of the right eye
latation, the fetal head in the pelvic outlet D. Tetanic cataract of the right eye
plane. Sagittal suture in line with obstetric E. Radiation cataract of the right eye
conjugate, the occipital fontanel is near
the pubis. The patient has been diagnosed 77. During her first visit to the prenatal
with primary uterine inertia. What is the clinic a pregnant woman was referred to
further tactics of labour management? other doctors for mandatory consultation.
The patient was refered to:
A. Outlet forceps
B. Labour stimulation A. Internist, dentist
C. Cesarean section B. Internist, surgeon
D. Skin-head Ivanov’s forceps C. Dentist, surgeon
E. Vacuum extraction of the fetus D. Surgeon, oculist
E. Therapist, oculist
74. A patient complains of frequent, bulky,
frothy stools with greenish mucus, crampi- 78. A 59-year-old male complains of heart
ng pain in the umbilical region, abdomi- pain, cough, fever up to 38o C. Three
nal murmur, body temperature at the weeks ago he suffered a heart attack.
rate of 39o C. The patient associates the Objectively: Ps- 86/min, rhythmic, blood
disease with consumption of soft-boiled pressure - 110/70 mm Hg. Auscultation
eggs. What is the most likely pathogen? reveals pericardial rub, rales beneath the
shoulder blade. Radiography reveals no
A. Salmonella pathology. Blood count: WBCs - 10 · 109 /l,
B. Yersinia ESR - 35 mm/h. ECG shows no dynamics.
C. Shigella It would be most reasonable to administer
D. Enteropathogenic E.Coli the drugs of the following pharmaceutical
E. Vibrio cholerae El Tor group:
75. X-ray picture of chest shows a density A. Glucocorticoids
and an abrupt decrease in the upper lobe B. Antibiotics
of the right lung. The middle and lower C. Direct anticoagulants
lobe of the right lung exhibit significant D. Nitrates and nitrites
pneumatization. The right pulmonary E. Fibrinolytics
hilum comes up to the dense lobe. In
the upper and middle parts of the left 79. A 40-year-old female patient complai-
pulmonary field there are multiple focal ns of headache, dizziness, muscle
shadows. In the basal region of the left weakness, occasional cramps in the
pulmonary field there are clear outlines of extremities. She has been taking anti-
two annular shadows with quite thick and hypertensive medications for 10 years.
irregular walls. What disease is this X-ray AP- 180/100 mm Hg. Blood potassium -
pattern typical for? 1,8 millimole/l, sodium - 4,8 millimole/l. In
urine: alkaline reaction, the relative densi-
A. Fibro-cavernous pulmonary ty - 1012, protein and sugar are not found,
tuberculosis WBCs - 3-4 in the field of vision, RBCs - 1-
B. Atelectasis of the right upper lobe 2 in the field of vision. Conn’s syndrome is
C. Abscessing pneumonia suspected. Which drug should be chosen
D. Peripheral cancer for the treatment of arterial hypertensi-
E. Pancoast tumour on?
76. A patient complains of impaired far vi- A. Spironolactone
sion. Previously his eyes often turned red B. Propanolol
and hurt. Objectively: the eyes are not C. Enalapril
irritated, the cornea is transparent, anteri- D. Hydrochlorothiazide
or chambers are median deep, their liquid E. Clonidine
is transparent. The iris of the right eye
has not changed in colour, its pattern is 80. A 27-year-old patient complains of
unchanged. The pupil is of irregular shape, nasal haemorrhages, multiple bruises on
scalloped. Biomicroscopy of the crystalli- the anterior surface of the trunk and
ne lens reveals the areas of opacity and extremities, sudden weakness. In blood:
Krok 2 Medicine 2013 12

Hb- 74 g/l, reticulocytes - 16%, RBCs - the patient is pale, mucous membranes
2, 5 · 1012 /l, platelets - 30 · 109 /l, ESR- 25 are cyanotic, breathing rate is 24/min,
mm/h. What is the most effective measure tachycardia is present. In lungs: dimini-
for the treatment of thrombocytopenia? shed breath sounds, moist rales over both
lungs, crepitation. What is the most likely
A. Splenectomy diagnosis?
B. Iron preparations
C. Hemotransfusion A. Pneumonic plaque
D. Cytostatics B. Miliary tuberculosis
E. Vitamin B12 C. Influenza
D. Ornithosis
81. After lifting a load a patient felt E. Sepsis
undurable pain in the loin. He was di-
agnosed with acute lumbosacral radiculi- 85. An 8-month-old baby has decreased
tis. Which of the following is contraindi- appetite, pale skin, enlarged right side
cated for this patient? of abdomen. Palpation the right side of
abdomen reveals a dense elastic tumour-
A. Warming procedures like formation 10x7 cm large. There is
B. Dehydrating drugs a positive ballotement sign. What is the
C. Analgetics most likely diagnosis?
D. Vitamins of B group
E. Intravenous injection of aminophylline A. Nephroblastoma (Wilms’ tumour)
B. Liver tumour
82. The institutions which take part in C. Congenital hydronephrosis
medical examinations can be prevention D. Colon pathology
and treatment facilities, medical board E. Intestinal tumour
of Ministry of Defense, medical board of
Ministry of Home Affairs, medico-social 86. An infant is 3 weeks old. Since bi-
expert commissions, forensic medical rth there has been observed periodical
boards etc. What institutions are responsi- vomiting within a few minutes after feedi-
ble for temporary disability examination? ng. The amount of vomitive masses does
not exceed that of previous feeding. The
A. Prevention and treatment facilities infant has age-appropriate body weight.
B. Sanitary-and-prophylactic institutions What is the most likely cause of this
C. Medico-social expert commissions symptom?
D. Medical boards of Ministry of Defense
E. Medical boards of Ministry of Home A. Pylorospasm
Affairs B. Esophageal chalasia
C. Adrenogenital syndrome
83. While lifting a heavy load a 39-year-old D. Pyloristenosis
patient suddenly felt a severe headache, E. Esophageal achalasia
pain in the interscapular region, and
started vomiting. Objectively: the pulse 87. A city somatic hospital with 300
is rhythmic, 60/min, AP- 180/100 mm Hg. beds consists of the main building whi-
The patient is agitated. He presents wi- ch houses the therapeutic and surgical
th photophobia, hyperacusis. There are departments. Several separate buildings
positive Kernig’s and Brudzinski’s signs house the maternity, pediatric and radi-
on both sides. In blood: WBCs - 10 · 109 /l. ologic departments that are connected
CSF is bloody, cytosis is 240/3. What is the to the main building by underground
most likely diagnosis? walkways and above-ground covered
skybridges. Specify the building system
A. Subarachnoid haemorrhage of the hospital:
B. Sympathoadrenal crisis
C. Acute hypertonic encephalopathy A. Central-unit
D. Meningococcal meningitis B. Centralized
E. Ischemic stroke C. Decentralized
D. Free
84. A 45-year-old patient, a sailor, was E. Combined
hospitalized on the 2nd day of the di-
sease. A week ago he returned from 88. A 13-year-old girl complains of fever
India. Complains of body temperature of up to 37, 4o C during the last 2 months
41o C, severe headache, dyspnea, cough after recovering from ARVI. Objectively:
with frothy rusty sputum. Objectively: malnutrition, diffuse grade II enlargement
Krok 2 Medicine 2013 13

of the thyroid gland feeling dense on provisional diagnosis?


palpation, exophthalmos, tachycardia.
What kind of pathological syndrome is A. Status asthmaticus
it? B. Chronic obstructive bronchitis
C. Bronchiale asthma, moderate gravity
A. Thyrotoxicosis D. Foreign object aspiration
B. Hypothyroidism E. Cardiac asthma
C. Hypoparathyroidism
D. Hyperparathyroidism 92. Blood typing resulted in positi-
E. Thymomegaly ve isohemagglutination reaction with
standard sera of А(II) and В(III) groups
89. A 53-year-old female patient complai- and negative reaction with sera of 0(I)
ns of cardiac pain and rhythm intermissi- and АВ(IV) groups. What is this result
ons. She has experienced these presentati- indicative of?
ons since childhood. The patient’s father
had a history of cardiac arrhythmias. A. Faulty standard sera
Objectively: the patient is in grave condi- B. The first blood group
tion, Ps- 220 bpm, AP- 80/60 mm Hg. C. The second blood group
ECG results: heart rate - 215/min, extensi- D. The third blood group
on and deformation of QRS complex E. The fourth blood group
accompanied by atrioventricular dissoci-
ation; positive P wave. Some time later 93. A patient complains of jerking,
heart rate reduced down to 45/min, there throbbing pain in the III finger on the
was a complete dissociation of P wave and right hand. The patient associates these
QRST complex. Which of the following pain onsets with an injury by a nail. The
will be the most effective treatment? finger skin is hyperemic and tense, palpati-
on with a bulbous-end probe reveals the
A. Implantation of the artificial pacemaker most painful area. What is the provisional
B. β-adrenoreceptor blocking agents diagnosis?
C. Cholinolytics
D. Calcium antagonists A. Subcutaneous felon
E. Cardiac glycosides B. Pandactylitis
C. Bone felon
90. On the 2nd day of life a full-term D. Paronychia
boy developed mild jaundice of skin and E. Articular felon
mucous membranes, the general condition
of the child is normal. Blood test results: 94. Sanitary examination of the burns unit
indirect hyperbilirubinemia - 120 mmol/l. for adults revealed that 4-bed wards had
The child’s blood group is A(II) Rh(+), an area of 28 m2 . What is the minimally
his mother’s blood group - B(III) Rh(+). required ward area for this department?
What is the doctor’s tactics of choice?
A. 40 m2
A. No drug therapy B. 24 m2
B. Treatment with enterosorbents C. 28 m2
C. Treatment with prednisolone D. 30 m2
D. Treatment with cholekinetics E. 52 m2
E. Blood transfusion
95. A patient presented to a hospital
91. A 49-year-old patient complains of with a carbuncle of the upper lip. The
dyspnea, cough. There are no sputum body temperature is 39o C. There is a
discharges. He has repeatedly used pronounced edema of the upper lip and
salbutamol and intal but with no effect. eyelids. What is the surgeon’s tactics of
Objectively: he is only able to sit whi- choice?
le leaning on the table. Cyanosis of
face, acrocyanosis are present. Breathi- A. Hospitalize in the surgical unit
ng is shallow, laboured, in some parts it B. Administer out-patient course of antibi-
cannot be auscultated; there are diffuse otics
rales, expiration is significantly prolonged. C. Disclose the carbuncle and administer
Heart sounds are muffled, tachycardia is out-patient treatment
present. Ps - 112/min, AP- 110/70 mm Hg. D. Administer physiotherapy
Liver is located near the costal arch. There E. Disclose the carbuncle and administer
are no peripheral edemata. What is your antibiotics
Krok 2 Medicine 2013 14

96. From urine of a 14-year-old boy with test results: ESR- 47 mm/h, total bili-
the exacerbation of secondary obstructive rubin - 86,1 mmol/l, direct bilirubin - 42,3
pyelonephritis Pseudomonas aeruginosa mmol/l. Total protein - 62 g/l, albumins
was isolated with a titer of 1000000 mi- - 40%, globulins - 60%, gamma globuli-
crobes per 1 ml. Which antibiotic is most ns - 38%. Viral hepatitis markers were
advisable to be administered in this case? not detected. The antibodies to smooth
muscle cells are present. On ultrasound
A. Ciprofloxacin the portal vein diameter was of 1 cm.
B. Ampicillin What is the most likely diagnosis?
C. Cefazolinum
D. Azithromycin A. Autoimmune hepatitis
E. Chloramphenicol B. Primary biliary cirrhosis
C. Gilbert’s syndrome
97. A 62-year-old male patient complai- D. Cholangiogenic hepatitis
ns of intense pain in the left leg that E. Hemachromatosis
suddenly arose three hours before, leg
numbness and coldness. During the year 100. A patient is 60 years old, retired,
there has been pain in the leg while walki- worked as deputy director of a research
ng, hypersensitivity to cooling. Objecti- institute. Behavioural changes appeared 2
vely: the left foot and shin have marbled years ago after the death of her husband:
skin, subcutaneous veins are collapsed. she stopped looking after herself and
The foot is cold, active movements of leaving the house; then she refused to
the foot and toes are preserved. Pulse is clean the apartment and cook. Mental
present only on the femoral artery. There status: temporal disorientation. The pati-
is rough systolic murmur above the artery. ent does not understand many of the
Make a provisional diagnosis: questions, is confused; does not know
how to cook soup or fasten a button.
A. Acute occlusion of the left femoral Her speech is characterized by stumbli-
artery ng and logoclonia. She does not recognize
B. Occlusive disease doctors, fellow patients. She cries a lot but
C. Stenosis of the left popliteal artery can not explain the reason for tears. What
D. Acute thrombophlebitis is the mechanism of this pathology?
E. Acute arterial thrombosis ileofemoralny
A. Atrophy of the cerebral cortex
98. A 28-year-old patient complai- B. Atherosclerotic changes in cerebral
ns of profuse, painful and prolonged vessels
menstruation. Before and after the C. Serotonin deficiency
menstrual period there is spotting lasting D. Impaired conversion of dopamine to
for 4-6 days. Vaginal examination reveals noradrenaline
that the uterus is enlarged correspondi- E. Disorder of melatonin metabolism
ng to 5-6 weeks of pregnancy, has limited
mobility, is painful. Appendages are not 101. A 70-year-old patient with a
palpable. On the 15th day of the menstrual strangulated inguinal hernia called a
cycle, the uterus was of normal size, pai- therapist in. The strangulation took place
nless. On account of stated problems and 10 hours ago. There are signs of intestinal
objective examination the patient has obstruction. The skin over the herniation
been diagnosed with internal endometri- is hyperemic. What is the tactics of choi-
osis. Which drug should be used for the ce?
efective treatment of this patient?
A. Emergency hospitalization to a surgical
A. Duphaston hospital
B. Synoestrolum B. Referral to a surgeon
C. Parlodel C. Cold to the hernia, analgesics, antibioti-
D. Ovidon cs
E. - D. Reduction of hernia after a narcotic
injection
99. A 24-year-old female patient complai- E. Reduction of hernia
ns of pain in the right hypochondrium
that is getting worse after taking meals; 102. An employee was on a business trip
nausea, fever up to 37, 7oC, icteric skin, to another city, where he fell ill and was
pain in the large joints. These presentati- hospitalized. The sick leave certificate can
ons have been observed for 8 months. be issued:
Objectively: hepatosplenomegaly. Blood
Krok 2 Medicine 2013 15

A. With the permission of the head doctor 106. Examination of a 38-year-old pati-
of the city hospital ent who had been hit with a blunt object
B. With the permission of the deputy head on the left side of chest revealed a
doctor after the disability examination fracture of the X rib with fragments di-
C. With the permission of the deputy chief splacement, parietal pneumothorax. The
doctor in charge of medical work patient complains of pain in the left
D. By doctor in charge and chief of subcostal area. Objectively: the patient
department is pale, AP- 80/40 mm Hg, Ps- 138/min,
E. By doctor in charge of poor volume. USI reveals fluid in the
left abdomen. Splenic rupture is present.
103. A 14-year-old boy with a history What treatment tactics should be chosen?
of chronic tonsillitis and sinusitis has
developed a feeling of heart irregulari- A. Drainage of the left pleural cavity
ties and additional pulse. HR- 83/min. followed by laparotomy
ECG results: regular impulses with no vi- B. Immediate upper median laparotomy
sible P wave that occur every two sinus followed by drainage of the left pleural
contractions, QRS complex is dramati- cavity
cally deformed and prolonged to over C. Immediate laparotomy and alcohol-
0,11 s, T wave is discordant followed by novocaine block of the X rib
a complete compensatory pause. Specify D. Anti-schock actions followed by
the arrhythmia type: laparotomy after the arterial pressure rise
E. Left-sided thoracotomy immediately
A. Trigeminal extrasystole followed by laparotomy
B. Bigeminal extrasystole
C. Partial AV-blockade 107. A baby is 3 months old. The mother
D. Complete AV-block consulted a pediatrician about lack of
E. Left bundle branch block breast milk. After several test weighings
it was found that the child had to receive
104. An 8-year-old girl periodically has supplementary feeding. What is the opti-
sudden short-term heart pain, sensation mal milk formula for this child?
of chest compression, epigastric pain, di-
zziness, vomiting. Objectively: the patient A. Malysh
is pale, respiratory rate - 40/min, jugular B. Milk formula № 2
pulse is present. Ps- 185 bpm, of poor C. Milk formula № 3
volume. AP- 75/40 mm Hg. ECG taken D. Whole cow’s milk
during an attack shows ectopic P waves, E. Malutka
QRS wave is not deformed. At the end
of an attack a compensatory pause is 108. Examination of a newborn revealed
observed. The most likely cause of the skin redness that appeared immediately
attack is: after birth and reached the maximum
intensity on the second day of life. What
A. Paroxysmal atrial tachycardia is your provisional diagnosis?
B. Sinus tachycardia
C. Paroxysmal ventricular tachycardia A. Simple erythema
D. Complete AV-block B. Toxic erythema
E. Atrial fibrillation C. Transient erythema
D. Erythema nodosum
105. A 45-year-old patient with acute E. Annular erythema
abscess of the left lung has suddenly
developed acute chest pain and dyspnea 109. A 27-year-old sexually active female
while coughing, tachycardia has increased. complains of numerous vesicles on the ri-
The control Ro-gram shows collapsed left ght sex lip, itch and burning. Eruptions
lung, the air in the left pleural cavity regularly turn up before menstruation and
and a horizontal fluid level. What is the disappear 8-10 days later. What is the most
mechanism of this complication? likely diagnosis?
A. Abscess burst into the pleural cavity A. Herpes simplex virus
B. Bullae rupture of the left lung B. Bartholinitis
C. Inflammation spread to the visceral C. Primary syphilis
pleura D. Cytomegalovirus infection
D. Atelectasis of the left lung E. Genital condylomata
E. Acute cardiovascular insufficiency
110. A 38-year-old patient has suddenly
Krok 2 Medicine 2013 16

developed pain in the left side of his chest, score was 4. Which of the following is a
suffocation. Objectively: moderately risk factor for a spinal cord injury?
grave condition, Ps- 100/min, AP- 90/60
mm Hg, breath sounds on the left cannot A. Difficult delivery of the head and
be auscultated. Chest radiography shows shoulders
the collapse of the left lung up to 1/2. B. Young age of the mother
What kind of treatment should be admi- C. Pernicious habits
nistered? D. Uterine inertia
E. Chronic hypoxia
A. Passive thoracostomy
B. Rest, resolution therapy 115. A 10-year-old child has been admi-
C. Pleural puncture tted to a hospital with a closed crani-
D. Operative therapy ocerebral injury with suspected cerebral
E. Active thoracostomy edema. The patient is in grave condition,
unconscious. The dyspnea, tachycardia,
111. A 40 week pregnant secundipara is hypertension are present. Muscle tone is
28 years old. Contractions are very active. increased, there is nystagmus, pupillary
Retraction ring is at the level of navel, the and oculomotor reactions are disturbed.
uterus is hypertonic, in form of hourglass. The mandatory component of intensi-
On auscultation the fetal heart sounds ve care is dehydration. What diuretic is
are dull, heart rate is 100/min. AP of the adequate in this case?
parturient woman is 130/80 mm Hg. What
is the most likely diagnosis? A. Furosemide
B. Mannitol
A. RIisk of hysterorrhexis C. Hydrochlorthiazide
B. Mazolysis D. Spironolactone
C. Disturbed labour E. Moduretic
D. Complete hysterorrhexis
E. Attack of eclampsia 116. A 57-year-old patient taken to the
surgical department by ambulance has
112. A 10-year-old patient has a hi- been provisionally diagnosed with acute
story of mild bronchial asthma. During intestinal obstruction. Acute pancreatitis
a regular check-up the patient should be is suspected. What is the most informative
recommended: method of study to verify the diagnosis?
A. To avoid allergenic food A. Plan radiography of stomach
B. To avoid body tempering procedures B. Biochemical blood analysis
C. Not to go in for sports C. Complete blood count and clinical
D. To avoid spa treatment urinalysis
E. Not to go to the seaside D. Fibrogastroduodenoscopy
E. Ultrasound
113. A 26-year-old patient with left lower
lobe pneumonia experiences an acute 117. After myocardial infarction, a 50-
chest pain on the left during coughing. year-old patient had an attack of asthma.
Objectively: diffuse cyanosis, extension of Objectively: bubbling breathing with
the left side of chest. Percussion reveals frequency of 32/min, cough with a lot
high tympanitis. Auscultation reveals no of pink frothy sputum, acrocyanosis,
respiratory murmurs above the left side swelling of the neck veins. Ps- 108/min,
of chest. There is a deviation of the right AP- 150/100 mm Hg. Heart sounds
cardiac border towards the midclavicular are muffled. Mixed moist rales can be
line. What examination will be the most auscultated above the entire lung surface.
informative? What drug would be most effective in this
situation?
A. X-Ray
B. Bronchoscopy A. Nitroglycerin intravenously
C. Bronchography B. Pentamin intravenously
D. Pneumotachometry C. Strophanthin intravenously
E. Spirography D. Dopamine intravenously
E. Aminophylline intravenously
114. A baby was born by a young smoker.
The labour was complicated by uterine 118. A baby born after fast labour has
inertia, difficult delivery of the baby’s palsy of hand muscles. Grasp reflex is
head and shoulders. The baby’s Apgar absent, as well as hand-to-mouth reflex.
Krok 2 Medicine 2013 17

Hand sensitivity is absent. What is the A. Emergency notification on infectious


most likely diagnosis? disease (form № 058/o)
B. Sick child care certificate
A. Dejerine-Klumpke palsy C. Infant’s record (report form № 112/o)
B. Duchenne-Erb’s palsy D. House call record (form № 031/o)
C. Total lesion of the brachial plexus E. Child care sick leave
D. Muscle paresis
E. Bernard-Horner syndrome 123. Hygienic expertise of a sample taken
from the batch of grain revealed that
119. Against the background of angina 2% of grains were infected with mi-
a patient has developed pain in tubular croscopic Fusarium fungi. On the ground
bones. Examination revealed generalized of laboratory analyses this batch of grain
enlargement of lymph nodes, hepatoli- should be:
enal syndrome, sternalgia. In blood: RBCs
- 3, 6 · 1012 /l, Hb- 87 g/l, thrombocytes A. Sold without restrictions
- 45 · 109 /l, WBCs - 13 · 109 /l, blasts - B. Tested for toxicity
87%, stab neutrophils - 1%, segmented C. Used for forage production
neutrophils - 7%, lymphocytes - 5%, ESR D. Used for ethanol production
- 55 mm/h. What is the most likely di- E. Destroyed
agnosis?
124. Examination of a 43-year-old man
A. Acute leukemia objectively revealed pallor of skin and
B. Erythremia mucous membranes, loss of tongue papi-
C. Chronic lymphocytic leukemia llae, transverse striation of fingernails,
D. Chronic myeloid leukemia cracks in the mouth corners, tachycardia.
E. Multiple myeloma Blood test results: Hb- 90 g/l, anisocytosis,
poikilocytosis. The most likely causative
120. In order to reduce weed growth on agent of this state is inadequate intake of:
agricultural land, some herbicides have
been used for a long time. In terms of A. Iron
environmental stability these herbicides B. Copper
are rated as stable. Specify the most likely C. Zinc
route of their entry into the human body: D. Magnesium
E. Selene
A. Soil-plants-humans
B. Soil-microorganisms-humans 125. Factory’s sectorial doctor chooses a
C. Soil-animals-humans group of chronically ill people. He takes
D. Soil-protozoa-humans into account the duration of etiologically
E. Soil-insects-humans related cases with a temporary disability
over the last year in each of the workers.
121. A selective population research study The employees will fall into this group if
was aimed at exploring the effect of air the duration is:
emissions from a metallurgical plant on
the obstructive bronchitis morbidity in a A. 40 days or more
city. The calculated correlation coefficient B. 60 days or more
was +0,79. Evaluate the strength and di- C. 20 days or more
rection of the relationship: D. 30 days or more
E. 10 days or more
A. Direct, strong
B. Inverse, strong 126. 6 people live in a modern flat wi-
C. Direct, average th the total area of 60 m2 . There are
D. Inverse, average TV-video equipment, radios, microwave
E. - ovens, computer. The residents of the
flat complain of bad health, occasional
122. A general practitioner visited a 2- headaches, arrhythmia, conjunctivitis.
year-old child and diagnosed him with What is the most likely cause of this condi-
measles. The child attends a nursery, has tion?
a 5-year-old sister. What document must
be filled in for the effective antiepidemic
measures in the given health locality?
Krok 2 Medicine 2013 18

A. Electromagnetic fields
B. Anthropotoxins A. Status epilepticus
C. Carbon dioxide B. Frequent generalized seizures
D. Tetrachlor plumbum C. Frequent jacksonian seizures
E. Formaldehyde D. Hysterical attacks
E. Frequent complex partial seizures
127. A man abused alcohol, drank away
the property and wages thus getting hi- 131. A 45-year-old patient complains
mself, his wife and two underage children of fever up to 40o C, general weakness,
into deep financial problems. He was regi- headache and spasmodic contraction of
stered in a local drug abuse clinic. His wife muscles in the region of a shinwound. The
asked a family doctor, what kind of petiti- patient got injured five days ago when ti-
on she could file in court: lling soil and didn’t seek medical attenti-
on. What kind of wound infection can be
A. On restriction of her husband’s civil suspected?
capacity
B. On recognition of her husband’s A. Tetanus
incapacity B. Anthrax
C. On recognition of her husband’s partial C. Erysipelas
incapacity D. Gram-positive
D. On recognition of her husband’s E. Gram-negative
incompetence
E. On recognition of her husband’s disabi- 132. A 60 year-old female has been sufferi-
lity ng weakness, dizziness, fatigue over the
last year. Recently she has also developed
128. A child is 12 years old. He complai- dyspnea, paresthesia. Objectively: skin
ns of a dull aching pain in the epigastri- and mucous membranes are pale and sli-
um and right hypochondrium, that is ghtly icteric. The tongue is smooth due
getting worse after taking fatty or fried to the loss of lingual papillae. Liver and
food, headache, weakness, nausea, low- spleen are located at the costal margin.
grade fever. Abdominal palpation reveals Blood count: Hb- 70 g/l, RBCs - 1, 7·1012/l,
a marked resistance of muscles in the right colour index - 1,2, macrocytes. Admini-
hypochondrium, positive Kerr’s, Ortner’s, ster the patient a pathogenetically justifi-
Murphy’s symptoms. What is the most li- ed drug:
kely diagnosis?
A. Vitamin B12
A. Chronic cholecystitis B. Vitamin B6
B. Acute appendicitis C. Ascorbic acid
C. Viral hepatitis D. Iron preparations
D. Acute gastritis E. Vitamin B1
E. Acute pancreatitis
133. Analysis of organization of medi-
129. A 3-month-old girl presents wi- cal care in a regional centre has shown
th rhinitis, dyspnea, dry cough. These that every year about 12% of patients
manifestations has been observed for receive inpatient care for diseases that
two days. Objectively: the child has don’t require round-the-clock monitori-
pale skin, acrocyanosis, shallow respi- ng and intensive care. What are the most
ration at the rate of 80/min. Percussi- appropriate organizational changes requi-
on reveals handbox resonance over the red to address this problem?
whole surface of lungs, massive fine rales.
What is the most likely diagnosis? A. Development of medical care forms
replacing the in-patient care
A. Acute bronchiolitis B. Changes to the statute of outpatient
B. Pneumonia clinics
C. Mucoviscidosis C. Development of primary care
D. Foreign body of the airway D. Restructuring of specialized care
E. Acute bronchitis E. Upgrading of hospital facilities
130. A patient had 4 generalized convulsi- 134. A newborn (mother’s I pregnancy)
ve seizures within a day. Between the sei- weighing 3500 g presents with jaundice,
zures the patient did not maintain clear lethargy, reduced reflexes. Objectively:
consciousness (was in a coma or stupor). second grade jaundice of skin with saffron
Specify his state: tint, liver - +2cm, spleen - +1 cm. Urine
Krok 2 Medicine 2013 19

and feces are yellow. Blood count: Hb- 100 kely diagnosis?
g/l, RBCs - 3, 2 · 1012 /l, WBCs - 18, 7 · 109 /l,
mother’s blood type - 0(I) Rh(+), baby’s A. Acute bartholinitis
blood type - А(II) Rh(-), bilirubin - 170 B. Labial furuncle
C. Acute vulvovaginitis
mmol/l, indirect fraction. ALT, AST rates
are normal. What disease is the child most D. Bartholin gland cyst
E. Carcinoma of vulva
likely to have?
138. A 28-year-old female patient has
A. Hemolytic disease of newborn, AB0-
been admitted to a hospital. She states
conflict
B. Perinatal hepatitis to be ill for 12 years. On examination she
has been diagnosed with bronchiectasis
C. Hemolytic disease of newborn, Rh-
conflict with affection of the left lower lobe of
lung. What is the optimal treatment tacti-
D. Biliary atresia
cs for this patient?
E. Physiologic jaundice
A. Left lower lobectomy
135. Doctors of a polyclinic conduct
a statistical research of the disease B. Left-sided pneumoectomy
C. Antibiotic therpy
outcomes in two groups of patients (those
registered with dispensary departments D. Active drainage of the left pleural cavity
E. Bronchopulmonary lavage
and unregistered ones), depending on
age and level of hygiene. What type of 139. 4 weeks after myocardial infarction a
statistical tables would be most suitable 56-year-old patient developed acute heart
for profound analysis of the interrelation pain, pronounced dyspnea. Objectively:
between the above-mentioned variables? the patient’s condition is extremely grave,
there is marked cyanosis of face, swelling
A. Cross tabulation and throbbing of neck veins, peripheral
B. Analytical pulse is absent, the carotid artery pulse is
C. Group rhythmic, 130 bpm, AP is 60/20 mm Hg.
D. Simple Auscultation of heart reveals extremely
E. Developing muffled sounds, percussion reveals heart
136. A 57-year-old female complains border extension in both directions. What
of having a sensation of esophageal is the optimal treatment tactis for this
compresion, palpitation, difficult breathi- patient?
ng during eating solid food, occasional A. Pericardiocentesis and immediate
vomiting with a full mouth, "wet pi- thoracotomy
llow"sign at night for the last 6 months. B. Oxygen inhalation
Objectively: body tempearture - 39oC, C. Puncture of the pleural cavity on the left
height - 168 cm, weight - 72 kg, Ps- 76/min, D. Conservative treatment, infusion of
АP- 120/80 mm Hg. X-ray revealed a adrenomimetics
considerable dilation of esophagus and E. Pleural cavity drainage
its constriction in the cardial part. What
pathology is most likely to have caused 140. A 15-year-old boy feels pain in the
dysphagia in this patient? region of the left knee joint. Objecti-
vely: the soft tissues in the affected region
A. Achalasia cardiae are infiltrated, the joint function is limi-
B. Primary esophagism ted. Radiography reveals a focus of bone
C. Hiatal hernia destruction in the distal metaepiphysial
D. Esophageal carcinoma segment of the left femur. The destruction
E. Reflux esophagitis is accompanied by periosteal detachment
137. A 37-year-old patient complains of and a defect formed within cortex of
acute pain in the region of genitals, Codman triangle bone. X-ray of chest
swelling of the labia, pain when walki- shows multiple microfocal metastases.
ng. Objectively: body temperature is What is the most likely pathology?
38, 7oC, Ps- 98/min. In the interior of A. Osteogenic sarcoma
the right labia there is a dense, painful B. Fibrosarcoma
tumour-like formation 5,0x4,5 cm large, C. Chondrosarcoma
the skin and mucous membrane of geni- D. Ewing’s sarcoma
tals is hyperemic, there are profuse foul- E. Juxtacortical sarcoma
smelling discharges. What is the most li-
Krok 2 Medicine 2013 20

141. On the first day after a surgery for di- tation: she makes stereotypic grimaces,
ffuse toxic goiter a patient developed di- exposed, masturbating in front of a loud
fficulty breathing, cold sweats, weakness. laugh, repeating the stereotypical abusive
Objectively: pale skin, body temperature shouts. The patient should be assigned:
- 38, 5o C, RR - 25/min, Ps- 110/min, AP-
90/60 mm Hg. What early postoperative A. Neuroleptics
complication occurred in the patient? B. Antidepressants
C. Tranquilizers
A. Thyrotoxic crisis D. Nootropics
B. Hypothyroid crisis E. Mood stabilizers
C. Postoperative tetany
D. Acute thyroiditis 145. A young woman with seborrhea adi-
E. Compression of the trachea by the posa has numerous non-itchy light brown
hematoma and white spots with clear outlines and
defurfuration on the torso and shoulder
142. A 49-year-old patient consulted a skin. What is the provisional diagnosis?
doctor about difficult swallowing, voice
hoarseness, weight loss. These symptoms A. Pityriasis versicolor (scaly skin disease)
have been gradually progressing for the B. Tinea corporis
last 3 months. Objectively: the patient is C. Seborrheic dermatitis
exhausted, there are enlarged supraclavi- D. Pityriasis rosea
cular lymph nodes. Esophagoscopy E. Vitiligo
revealed no oesophageal pathology. Whi-
ch of the following studies is most 146. 10 days after birth a newborn
appropriate in this case? developed a sudden fever up to 38, 1o C.
Objectively: the skin in the regi-
A. Computed tomography of chest, medi- on of navel, abdomen and chest is
astinum erythematous; there are multiple pea-
B. X-ray of lungs sized blisters with no infiltration at the
C. Multiplanar imaging of esophagus base; single bright red moist erosions wi-
D. Radioisotope investigation of chest th epidermal fragments on the periphery.
E. Ultrasound investigation of mediasti- What is your provisional diagnosis?
num
A. Epidemic pemphigus of newborn
143. A 45-year-old female patient B. Syphilitic pemphigus
complaining of general weakness, nausea C. Streptococcal impetigo
and vomiting hass been delivered to a D. Vulgar impetigo
hospital by the ambulance. Recently there E. Atopic dermatitis
has been a lack of appetite, weight loss.
Objectively: hyperpigmentation of skin, 147. An emergency physician arrived to
blood pressure at the rate of 70/45 mm Hg, provide medical care for a hangman taken
bradycardia. Additional studies revealed out of the loop by his relatives. The doctor
the reduced concentration of aldosterone revealed no pulse in the carotid arteri-
and cortisol in blood, decreased excretion es, absence of consciousness, spontaneous
of 17-ketosteroids and 17-oxyketosteroids breathing and corneal reflexes; cadaver
in the urine, hyponatremia, chloropenia, spots on the back and posterior parts of
hypokalemia. What therapeutic measures extremities. A person can be declared
are required? dead if the following sign is present:

A. To administer glucocorticoids, mi- A. Cadaver spots


neralocorticoids, and a diet with a high B. Absence of spontaneous breathing
content of cooking salt C. Absence of corneal reflexes
B. To prescribe a diet with a high content D. Pulselessness
of cooking salt E. Unconsciousness
C. To administer prednisolone 148. A 40-year-old patient is registered
D. To administer aldosterone in a narcological dispensary. Somatically:
E. To administer insulin skin is dramatically hyperemic, sclera are
144. A 23-year-old female patient has a injected, hyperhidrosis is present. AP-
mental disease since the age of 18, the 140/100 mm Hg, heart rate - 100/min.
course of disease has no remission peri- Mental state: autopsychic orientation is
ods. At a hospital the patient mostly intact, allopsychic orientation is distorted.
presents with non-purposeful foolish exci- The patient presents with motor anxiety.
Krok 2 Medicine 2013 21

There is a look of fear on his face. He Prostate-specific antigen rate is of 5 ng/ml.


refuses to talk about his problems and What is the most likely disease that caused
asks to release him immediately, because acute urinary retention?
he "may be killed."This state developed a
day after a regular drinking bout. What is A. Prostatic hyperplasia
your provisional diagnosis? B. Prostate carcinoma
C. Sclerosis of the prostate
A. Delirium tremens D. Tuberculosis of the prostate
B. Organic delirium E. Acute prostatitis
C. Paranoia
D. Alcoholic hallucinosis 152. A 10 week pregnant woman was
E. Alcoholic paranoid admitted to a hospital for recurrent pain
in the lower abdomen, bloody discharges
149. A 35-year-old female patient has gai- from the genital tracts. The problems
ned 20 kg weight within a year with the turned up after ARVI. The woman was
normal diet. She complains of chill, sleepi- registered for antenatal care. Speculum
ness, shortness of breath. The patient’s examination revealed cyanosis of vagi-
mother and sister are corpulent. Objecti- nal mucosa, clean cervix, open cervical
vely: height - 160 cm, weight - 92 kg, canal discharging blood and blood clots;
BMI - 35,9. Obesity is uniform, there are the lower pole of the gestational sac was
no striae. The face is amimic. The skin visible. What tactics should be chosen?
is dry. The tongue is thickened. Heart
sounds are muffled. HR- 56/min, AP- A. Curettage of the uterus
140/100 mm Hg. The patient has consti- B. Pregnancy maintenance therapy
pations, amenorrhea for 5 months. TSH- C. Expectant management, surveillance
28 mkME/l (normal rate - 0,32-5). Crani- D. Hysterectomy
ogram shows no pathology. What is the E. Antiviral therapy
etiology of obesity?
153. A patient with fibromyoma of uterus
A. Hypothyroid sized up to 8-9 weeks of pregnancy
B. Hypo-ovarian consulted a gynaecologist about acute
C. Hypothalamic-pituitary pain in the lower abdomen. Examination
D. Alimentary and constitutive revealed pronounced positive symptoms
E. Hypercorticoid of peritoneal irritation, high leukocytosis.
Vaginal examination revealed that the
150. A 26-year-old patient consulted a uterus was enlarged corresponding to
doctor abut sore throat, fever up to 9 weeks of pregnancy due to the fi-
38, 2oC. A week ago, the patient had angi- bromatous nodes, one of which was mobi-
na, didn’t follow medical recommendati- le and extremely painful. Appendages
ons. On examination, the patient had were not palpable. There were moderate
forced position of his head, trismus of mucous discharges. What is the optimal
chewing muscles. Left peritonsillar regi- treatment tactics?
on is markedly hyperemic, swollen. What
is the provisional diagnosis? A. Urgent surgery (laparotomy)
B. Surveillance and spasmolytic therapy
A. Left-sided peritonsillar abscess C. Fractional diagnostic curettage of the
B. Meningitis uterine cavity
C. Phlegmonous angina D. Surgical laparoscopy
D. Diphtheria of the pharynx E. Surveillance and antibacterial therapy
E. Tonsil tumour
154. A 40-year-old female patient
151. A 77-year-old patient complains of complains of having a bulge on the anteri-
inability to urinate, bursting pain above or surface of neck for 5 years. Objecti-
the pubis. The patient developed acute vely: Ps- 72 bpm, arterial pressure - 110/70
condition 12 hours ago. Objectively: mm Hg, in the right lobe of thyroid gland
full urinary bladder is palpable above palpation reveals a mobile 4x2 cm node,
the pubis. Rectal prostate is enlarged, the left lobe is not palpable, the basal
dense and elastic, well-defined, with no metabolic rate is 6%. What is the most
nodes. Interlobular sulcus is distinct. likely diagnosis?
Ultrasonography results: prostate volume
is 120 cm3 , it projects into the bladder
cavity, has homogeneous parenchyma.
Krok 2 Medicine 2013 22

A. Nodular euthyroid goiter headache in the occipital region, nausea,


B. Nodular hyperthyroid goiter choking, opplotentes. The presentati-
C. Riedel’s thyroiditis ons appeared after a physical exerti-
D. Mixed euthyroid goiter on. Objectively: the patient is excited.
E. The median cervical cyst Face is hyperemic. Skin is pale. Heart
sounds are regular, the 2nd aortic sound
155. A multigravida at 39 weeks of gestati- is accentuated. AP- 240/120 mm Hg, HR-
on has been delivered to a hospital having 92/min. Auscultation reveals some fine
a regular labour activity for 8 hours, the moist rales in the lower parts of the lungs.
waters burst an hour ago. She complains Liver is not enlarged. ECG shows signs of
of headache, seeing spots. AP is of 180/100 hypertrophy and left ventricular overload.
mm Hg. Urine test results: protein - 3,3 What is the most likely diagnosis?
g/l, hyaline cylinders. Fetal heart rate is
140/min, rhythmical. Vaginal examination A. Complicated hypertensic crisis,
reveals complete crevical dilatation, the pulmonary edema
fetal head is on the pelvic floor, sagittal B. Acute myocardial infarction, pulmonary
suture is in line with obstetric conjugate, edema
the occipital fontanel is under the pubis. C. Bronchial asthma exacerbation
What is the optimal tactics of labour D. Uncomplicated hypertensic crisis
management? E. Community-acquired pneumonia
A. Outlet forceps 159. A patient complains of being unable
B. Cavity forceps to get pregnant for 5 years. A complete
C. Cesarean section clinical examination gave the following
D. Vacuum extraction of the fetus results: hormonal function is not impai-
E. Conservative labour management red, urogenital infection hasn’t been
found, on hysterosalpingography both
156. After a contact with chemicals a tubes were filled with the contrast medi-
plant worker has suddenly developed um up to the isthmic segment, abdominal
stridor, voice hoarseness, barking cough, contrast was not visualized. The patient’s
progressing dyspnea. Objective exami- husband is healthy. What tactics will be
nation reveals acrocyanosis. What is your most effective?
provisional diagnosis?
A. In-vitro fertilization
A. Laryngeal edema B. Insemination with husband’s sperm
B. Laryngeal carcinoma C. ICSI within in-vitro fertilization
C. PATE program
D. Pulmonary atelectasis D. Hydrotubation
E. Pneumothorax E. Laparoscopic tubal plasty
157. A 58-year-old patient complains of 160. Six months ago, a 5-year-old child
general weakness, loss of 10 kg of wei- was operated for CHD. For the last 3
ght within 1,5 months, progressive pain weeks he has complained of fever, heart
in the lumbar region, increased blood pain, aching muscles and bones. Exami-
pressure up to 220/160 mm Hg, subfebri- nation results: "white-coffee"skin colour,
le temperature. Objectively: in the ri- auscultation revealed systolic murmur in
ght hypochondrium palpation reveals a the region of heart along with a noise in
formation with uneven surface and low the III-IV intercostal space. Examinati-
mobility; veins of the spermatic cord and on of fingertips revealed Janeway lesions.
scrotum are dilated. Blood test results: What is your provisional diagnosis?
Hb- 86 g/l, ESR- 44 mm/h. Urine test
results: specific gravity - 1020, protein - A. Infectious endocarditis
0,99 g/l, RBCs - cover the whole field of B. Sepsis
vision, WBCs - 4-6 in the field of vision. C. Nonrheumatic carditis
What is the provisional diagnosis? D. Acute rheumatic fever
E. Typhoid fever
A. Renal tumour
B. Urolithiasis 161. A 45-year-old female patient has
C. Acute pyelonephritis worked as a painter for 14 years. Contacts
D. Acute glomerulonephritis with synthetic paint result in face skin
E. Nephroptosis redness, swelling, intense itching, oozi-
ng lesions. The symptoms disappear after
158. A 58-year-old patient complains of a the exposure to chemical agents, but recur
Krok 2 Medicine 2013 23

even at the smell of paint. The symptom by pain syndrome. Objectively: body
intensity progresses with relapses. Make a temperature is at the rate of 36, 8oC, Ps-
provisional diagnosis: 78/min, abdomen is soft and painless,
the symptoms of peritoneal irritation are
A. Occupational eczema present, palpation reveals a dramatically
B. Simple contact dermatitis enlarged, tense gallbladder. What disease
C. Contact-type allergy are these symptoms typical for?
D. Urticaria
E. Toksikodermiya A. Cancer of the pancreatic head
B. Duodenal ulcer
162. In a cold weather, the emergency C. Acute cholecystitis
room admitted a patient pulled out of D. Chronic cholecystitis
the open water. There was no respiratory E. Lamblia-induced cholecystitis
contact with the water. The patient is exci-
ted, pale, complains of pain, numbness of 166. A 22-year-old vegetarian patient wi-
hands and feet, cold shiver. Breathing rate th signs of malnutrition consulted a doctor
is 22/min, AP- 120/90 mm Hg, Ps- 110/min, about smell and taste distortion, angular
rectal temperature is 34, 5oC. What kind stomatitis. Objectively: expressively blue
of warming is indicated for this patient? sclerae. The patient was diagnosed with
iron deficiency anemia. What is the domi-
A. Passive warming nating clinical syndrome?
B. Infusion of 37o C solutions
C. Hot compresses A. Sideropenic
D. Warm bath B. Anaemic
E. Hemodialysis with blood warming C. Haemologic
D. Haemolytic
163. Survey radiograph of a 52-year-old E. Myelodysplastic
worker of an agglomeration plant (28
years of experience, the concentration of 167. A 13-year-old girl was admitted to
metal dust is 22-37 mg/m3 ) shows mildly the gynecological department with heavy
pronounced interstitial fibrosis with di- bleeding, which appeared after a long
ffused contrast well-defined small nodular delay of menstruation. Shortly before,
shadows. The patient has no complaints. the girl suffered a serious psychotrauma.
Pulmonary function is not compromised. Her menarche occurred at the age of
What is the provisional diagnosis? 11, she has a 30-day cycle with 5 to
6 days of moderate, painless bleeding.
A. Siderosis The patient is somatically healthy, of
B. Silicosis normosthenic constitution with height
C. Anthraco-silicatosis of 160 cm, weight of 42 kg. The pati-
D. Silicatosis ent is pale. Rectoabdominal examination
E. Anthracosis revealed that the uterus was of normal si-
ze and consistency, anteflexio-versio, the
164. A week ago a 65-year-old patient appendages were not changed. What is
suffered an acute myocardial infarction, the most likely diagnosis?
his general condition deteriorated: he
complains of dyspnea at rest, pronounced A. Juvenile bleeding
weakness. Objectively: edema of the B. Ovarian cyst
lower extremities, ascites is present. Heart C. Hysteromyoma
borders are extended, paradoxical pulse is D. Girl is healthy
2 cm displaced from the apex beat to the E. Amenorrhea
left. What is the most likely diagnosis?
168. It is planned to build a multi-
A. Acute cardiac aneurysm disciplinary hospital with 500 beds in a
B. Recurrent myocardial infarction town. Specify the location of a polyclinic
C. Acute pericarditis within the medical centre:
D. Cardiosclerotic aneurysm
E. Pulmonary embolism
165. A 64-year-old patient has been
referred to planned hospitalization
for general weakness, poor appetite,
progressive jaundice which appeared over
3 weeks ago and wasn’t accompanied
Krok 2 Medicine 2013 24

A. At the main entrance A. Post-castration syndrome


B. In the garden and park area B. Premenstrual syndrome
C. In the centre of the territory near C. Early pathological menopause
medical buildings D. Secondary psychogenic amenorrhea
D. It is not allowed to place the polyclinic E. Physiological premenopause
within the centre territory
E. In the service zone 172. A 2-year-old child in a satisfactory
condition periodically presents with
169. A 48-year-old patient complains of moderate proteinuria, microhematuria.
weakness, subfebrile temperature, achi- USI results: the left kidney is
ng pain in the kidney region. These undetectable, the right one is enlarged,
presentations turned up three months there are signs of double pyelocaliceal
ago after hypothermia. Objectively: ki- system. What study is required to speci-
dneys are painful on palpation, there is bi- fy the diagnosis?
laterally positive Pasternatsky’s symptom.
Urine test res: acid reaction, pronounced A. Excretory urography
leukocyturia, microhematuria, minor B. Micturating cystography
proteinuria - 0,165-0,33 g/l. After the urine C. Retrograde urography
sample had been inoculated on conventi- D. Doppler study of renal vessels
onal media, bacteriuria were not found. E. Radioisotope renal scan
What research is most required in this
case? 173. Explosion of a tank with benzene at a
chemical plant has killed and wounded
A. Urine test for Mycobacterium a large number of people. There are
tuberculosis over 50 victims with burns, mechani-
B. Daily proteinuria cal traumas and intoxication. Specify
C. Nechiporenko urine test the main elements of medical care and
D. Zimnitsky urine test evacuation of population in this situation:
E. Isotope renography
A. Sorting, medical assistance, evacuation
170. A 22-year-old patient complains B. Sorting, evacuation, treatment
of amenorrhea for 8 months. Menarche C. Medical assistance, evacuation, isolation
occured at the age of 12,5. Since the age D. Isolation, rescue activity, recovery
of 18 the patient has a history of irregular E. Sorting, recovery, rescue activity
menstruation. The patient is nulligravi-
da. The mammary glands are developed 174. A 26-year-old patient with affective
properly, nipples discharge drops of mi- bipolar disorder has developed a condi-
lk when pressed. Gynecological study tion manifested by mood improvement,
results: prolactin level is 2 times higher behavioural and sexual hyperactivity,
than normal. CT reveals a bulky formati- verbosity, active body language, reduced
on with a diameter of 4 mm in the region need for sleep. Which of the following
of sella. What is the most likely diagnosis? drugs are most effective in this case?

A. Pituitary tumour A. Neuroleptics with a sedative effect


B. Lactational amenorrhea B. Antidepressants with an activating
C. Stein-Leventhal syndrome effect
D. Sheehan’s syndrome C. Neuroleptics with an activating effect
E. Pituitary basophilia D. Tranquilizers
E. Antidepressants with a sedative effect
171. A 38-year-old female patient
complains about hot flashes and feeli- 175. An emergency doctor has diagnosed
ng of intense heat arising up to 5 times a 32-year-old woman with generalized
a day, headaches in the occipital region convulsive status epilepticus. The deteri-
along with high blood pressure, palpitati- oration in the patient’s condition is caused
ons, dizziness, fatigue, irritability, memory by a sudden gap in the epilepsy treatment.
impairment. 6 months ago the patient Specify the doctor’s further tactics:
underwent extirpation of the uterus wi-
th its appendages. What is the most likely
diagnosis?
Krok 2 Medicine 2013 25

A. Hospitalization in the intensive care accompanied by nausea. Surgical exami-


unit nation revealed moist tongue, Ps- 76 bpm.
B. Hospitalization in the department of AP- 130/80 mm Hg. Abdomen was soft,
neurology slightly painful in the right iliac region
C. Hospitalization in the department of on deep palpation, the symptoms of the
neurosurgery peritoneum irritation were doubtful. In
D. Outpatient monitoring by a blood: RBCs - 4, 0 · 1012 /l, Hb- 135 g/l,
neuropathologist WBCs - 9, 5 · 109 /l, stab neutrophils - 5%,
E. Outpatient monitoring by a segmentonuclear - 52%, lymphocytes -
neurosurgeon 38%, monocytes - 5%, ESR - 20 mm/h.
176. A 19-year-old patient complains Specify the doctor’s further tactics:
of dyspnea on exertion. He often has A. Emergency operation for acute
bronchitis and pneumonia. Since chi- appendicitis
ldhood, the patient presents with cardi- B. Hospitalization, dynamic surveillance
ac murmur. Auscultation revealed splitti- C. Send the patient home
ng of the II sound above the pulmonary D. Refer the patient to a district therapist
artery, systolic murmur in 3 intercostal E. Administration of additional examinati-
space at the left sternal border. ECG on: abdominal ultrasound, x-ray contrast
showed right bundle branch block. What study of the gastrointestinal tract
is the provisional diagnosis?
180. A 55-year-old patient whose
A. Atrial septal defect menstruation stopped 5 years ago
B. Open ductus arteriosus complains of vaginal dryness, frequent
C. Aortarctia and painful urination. Gynecologist
D. Aortic stenosis revealed signs of atrophic colpitis. Urine
E. Mitral insufficiency analysis revealed no peculiarities. Whi-
177. 20 minutes after a normal delivery ch locally acting product will provide the
at 39 weeks a puerpera had a single proper therapeutic effect?
temperature rise up to 38o C. Objecti- A. Vaginal suppositories "Ovestin"
vely: the uterus is dense, located between B. Vaginal tablets "Tergynan"
the navel and the pubis, painless. Lochia C. Vaginal cream "Meratin Combi"
are bloody, of small amount. Breasts are D. Vaginal gel "Metronidazole"
moderately soft and painless. What is the E. Vaginal cream "Dalacin"
optimal tactics?
181. A 63-year-old patient with persistent
A. Further follow-up atrial fibrillation complains of moderate
B. Antibiotic therapy dyspnea. Objectively: peripheral edemata
C. Appointment antipyretic are absent, vesicular respiration is present,
D. Manual examination of the uterine heart rate - 72/min, AP- 140/90 mm Hg.
cavity What combination of drugs will be most
E. Expression of breast useful in the secondary prevention of
178. A 30-year-old patient got in a car heart failure?
accident. He is unconscious, pale, has A. Beta-blockers, ACE inhibitors
thready pulse. In the middle third of the B. Beta-blockers, cardiac glycosides
right thigh there is an extensive lacerati- C. Cardiac glycosides, diuretics
on with ongoing profuse external arteri- D. Cardiac glycosides, ACE inhibitors
al bleeding. What urgent actions must be E. Diuretics, beta-blockers
taken to save the life of the patient?
182. A 57-year-old patient had an attack
A. Tourniquet above the wound of the of retrosternal pain that lasted more than
right thigh 1,5 hours. Objectively: the patient is inert,
B. Tourniquet below the wound of the adynamic, has pale skin, cold extremities,
right thigh poor volume pulse, heart rate - 120/min,
C. Artificial lung ventilation AP- 70/40 mm Hg. ECG shows ST elevati-
D. Precordial thump on in leads II, III, aVF. What condition are
E. Plaster bar these changes typical for?
179. A 75-year-old male patient complains
of slight pain in the right iliac region. The
abdominal pain arose 6 days ago and was
Krok 2 Medicine 2013 26

A. Cardiogenic shock nodes were not changed. USI results: in


B. Arrhythmogenic shock the superior external quadrant of the right
C. Perforated gastric ulcer mammary gland there was a big formation
D. Acute pericarditis of increased echogenicity, sized 18x17 mm.
E. Acute pancreatitis The patient was provisionally diagnosed
with fibroadenoma. What is a doctor’s
183. An 8-year-old boy has a 2-year hi- further tactics?
story of blotchy itchy rash appearing
after eating citrus fruit. The first eruption A. Surgical removal of the tumour prior to
occurred at the age of 6 months after the pregnancy
introduction of juices to the baby’s diet. B. Dynamic follow-up
Father has a history of bronchial asthma, C. Surgical treatment after pregnancy
mother - that of allergic rhinitis. What is D. Radical mastectomy
the most likely diagnosis? E. Nonsteroid anti-inflammatory drugs,
oral contraceptives
A. Atopic dermatitis
B. Psoriasis 187. A patient is 31 years old. Double-
C. Pityriasis Rosea contrast barium swallow revealed a filli-
D. Urticaria ng defect on the posterior wall in the mi-
E. Quincke’s edema ddle segment of esophagus. The defect
looked like a well-defined oval 1,8x1,3 cm
184. On the 10th day postpartum a large. Mucosal folds adjacent to the defect
puerperant woman complains of pain and were intact, peristalsis and elasticity of
heaviness in the left mammary gland. the walls remained unchanged. Digestive
Body temperature is 38, 8o C, Ps- 94 bpm. tract problems were absent. What is the
The left mammary gland is edematic, provisional diagnosis?
the supero-external quadrant of skin
is hyperemic. Fluctuation symptom is A. Esophageal tumour
absent. The nipples discharge drops of B. Achalasia cardia
milk when pressed. What is a doctor’s C. Esophageal burn
further tactics? D. Diverticulum
E. Barrett’s esophagus
A. Antibiotic therapy, immobilization and
expression of breast milk 188. A 50-year-old patient has worked
B. Compress to both mammary glands at a chemical plant for 15 years. His
C. Inhibition of lactation work involved using xylene solvent. The
D. Physiotherapy patient was hospitalized with suspected
E. Opening of the abscess and drainage of chronic intoxication. He was found to
the mammary gland have anemic syndrome. What is the first-
priority measure of secondary anemia
185. A 25-year-old patient has been admi- prevention?
tted to the hospital with the followi-
ng problems: weakness, sweating, itchi- A. Job change
ng, weight loss, enlarged submandibular, B. Administration of glucocorticosteroids
cervical, axillary, inguinal lymph nodes. C. Administration of iron supplements
Objectively: hepatomegaly. Lymph node D. Including seafood into the diet
biopsy revealed giant Berezovsky-Reed- E. Including meat into the diet
Sternberg- cells, polymorphocellular
granuloma made by lymphocytes, reti- 189. A 49-year-old patient complains of
cular cells, neutrophils, eosinophils, fi- itching, burning in the external genitals,
brous tissue, plasma cells. What is the frequent urination. The symptoms has
most likely diagnosis? been present for the last 7 months. The
patient has irregular menstruation, once
A. Lymphogranulomatosis every 3-4 months. Over the last 2 years
B. Lymph node tuberculosis she presents with hot flashes, sweating,
C. Lymphoreticulosarcoma sleep disturbance. Examination revealed
D. Cancer metastases to lymph nodes no pathological changes of the internal
E. Macofollicular reticulosis reproductive organs. Complete blood
count and urinalysis showed no pathologi-
186. During self-examination a 22-year- cal changes. Vaginal smear contained 20-
old patient revealed a mammary tumour. 25 leukocytes in the field of vision, mixed
Palpation revealed a firm, painless, mobi- flora. What is the most likely diagnosis?
le formation up to 2 cm, peripheral lymph
Krok 2 Medicine 2013 27

A. Menopausal syndrome
B. Cystitis A. Amiodarone
C. Trichomonas colpitis B. Flecainide
D. Vulvitis C. Encainide
E. Bacterial vaginosis D. Moracizine
E. Digoxin
190. A 56-year-old female patient
complains of recurrent attacks of intensi- 194. A 38-year-old male works within the
ve pain irradiating along the ureters. Uri- range of ionizing radiation. At a routi-
ne test results: protein - 0,37 g/l, RBCs- ne medical examination he presents no
20-25 in the field of vision, WBCs - 12- problems. In blood: RBCs - 4, 5·1012/l, Hb-
14 in the field of vision. What method 80 g/l, WBCs - 2, 8 · 109 /l, thrombocytes -
of instrumental diagnostics is the most 30 · 109 /l. Decide if this person can work
informative for the diagnosis? with sources of ionizing radiation:
A. Intravenous urography A. Working with radioactive substances
B. USI of kidneys and other sources of ionizing radiation is
C. Computer tomography contraindicated
D. Radioisotope renography B. The patient is allowed to work with
E. Cystoscopy radioactive substances
C. The patient can only work with radi-
191. 2 weeks after labour a parturi- oactive substances of low activity
ent woman developed breast pain bei- D. The patient can be allowed to work
ng observed for 3 days. Examination after an extended medical examination
revealed body temperature at the rate E. The patient is allowed to work with
of 39o C, chills, weakness, hyperaemia, radioactive substances for the limited
enlargement, pain and deformity of the period of time
mammary gland. On palpation the infi-
ltrate was found to have an area of softeni- 195. A puerpera breastfeeding for 1,5
ng and fluctuation. What is the most likely weeks consulted a doctor about uniform
diagnosis? breast engorgement. Breasts are painful.
The body temperature is of 36, 6o C. Milk
A. Infiltrative-purulent mastitis expressing is difficult. What is the most li-
B. Phlegmonous mastitis kely diagnosis?
C. Lactostasis
D. Serous mastitis A. Lactostasis
E. Mastopathy B. Infiltrative mastitis
C. Purulent mastitis
192. A 22-day-old infant had developed D. Fibrocystic mastopathy
red subcutaneous nodules from 1,0 to 1,5 E. Gangrenous mastitis
cm large on the scalp. Later the nodules
suppurated, body temperature rose up 196. A 28-year-old male patient complains
to 37, 7o C, there appeared symptoms of of regurgitation, cough and heartburn that
intoxication, the regional lymph nodes occurs every day after a meal, when bendi-
grew bigger. Blood test results: anemia, ng forward or lying down. These problems
leukocytosis, neutrophilia, accelerated have been observed for 4 years. Objective
ESR. What is the most likely diagnosis? status and laboratory values are normal.
FEGDS revealed endoesophagitis. What
A. Pseudofurunculosis is the leading factor in the development of
B. Pemphigus this disease?
C. Vesiculopustulosis
D. Scalp phlegmon A. Failure of the inferior esophageal
E. - sphincter
B. Hypersecretion of hydrochloric acid
193. A 70-year-old patient consulted a C. Duodeno-gastric reflux
doctor about arrhythmic cardiac activity, D. Hypergastrinemia
dyspnea. Objectively: AP- 150/90 mm Hg, E. Helicobacter pylori infection
extrasystole arrhythmia (10-12 beats per
minute), left ventricular systolic dysfuncti- 197. A 24-year-old patient had been deli-
on (ejection fraction at the rate of 42%). vered to the thoracic department with
Which of antiarrhythmic drugs should a chest injury, a fracture of the IV, V,
be administered as initial therapy in this VI ribs on the right. Plan radiography
case? shows the fluid level in the pleural cavity
Krok 2 Medicine 2013 28

reaching the III rib on the right. Puncture had experienced antenatal fetal death due
blood contained clots. What is the optimal to hemolytic disease. What is the optimal
treatment tactics? tactics of pregnancy management?
A. Emergency thoracotomy A. Early delivery
B. Pleural puncture B. Delivery at 37 weeks of gestation
C. Thoracentesis and thoracostomy C. Screening for Rh-antibodies 2 weeks
D. Hemostatic therapy later and early delivery in case of further
E. Medical thoracoscopy titer rise
D. Introduction of anti-Rh (D)
198. A 10-year-old patient complains of immunoglobulin
skin itch that occurs at night. Objecti- E. Ultrasound for signs of hemolytic
vely: multiple paired papules, burrow disease of the fetus
tracks in the interdigital skin folds, on
the anterolateral surfaces of abdomen and 200. A 7-year-old female child has
buttocks. Specify the period of regular developed an acute condition. She
medical check-up for pupils of the class complains of a headache, two onsets of
where the patient learns: vomiting. Objectively: deferred reactions,
body temperature - 39, 3o C, pronounced
A. 2 months hyperesthesia, nuchal rigidity, positive
B. 2 weeks superior and inferior Brudzinski’s signs,
C. 5 days symmetric Kernig’s sign. What is the
D. 6 months provisional diagnosis?
E. 1 year
A. Meningitis
199. Examination of a Rh-negative B. Food toxicoinfection
pregnant woman at 32 weeks of gestation C. Craniocerebral trauma
revealed a four-time rise of Rh-antibody D. Toxic encephalopathy
titer within 2 weeks, the titer was 1:64. E. Encephalitis
In the first two pregnancies the patient
Krok 2 Medicine 2014 1

1. Head of a department and a trade-union A. Schede-type vertical suspension


group have appealed to the head of a hospital B. Closed reduction
about dismissal of the senior nurse who has C. Intramedullary osteosynthesis
17-year record of service. The facts of charge D. Use of Ilizarov apparatus
were confirmed and recognized by the nurse E. Periosteal osteosynthesis
herself. The nurse lives with a daughter (who is
5. What juice should be included in a complex
divorced and unemployed) and a 9-month-old drug and dietary therapy for patients with
grandson. Make an administrative decision: gastric ulcer or duodenal ulcer and increased
A. To continue the worker in office with a gastric juice acidity in order to accelerate the
warning of dismissal in case of repeated violati- ulcer healing?
on of labor discipline A. Potato, potato and carrot
B. To discharge the worker, i.e. to satisfy B. Apple, birch and apple
demands of the collective C. Pumpkin
C. To issue the sick leave D. Cabbage, cabbage and carrot
D. To embark other officials or public organi- E. Celery, parsley
zations with this problem
E. - 6. A 7-year-old child complains of itchi-
ng, papular erythematous rash, dry skin.
2. A 50-year-old female patient complains of Objectively: there is lichenification in the
aching pain in the lower abdomen. She has popliteal fossae and antecubital spaces.
a history of normal menstrual cycle. At the What immunologic indicator if found in the
age of 40, the patient underwent a surgery for blood serum will verify the diagnosis (atopic
gastric ulcer. Examination findings: abdomen
is soft, in the hypogastrium there is a well- dermatitis)?
defined nodular tumor of limited mobility. A. Total IgE
Vaginal examination findings: the cervix is B. Secretory IgA
clean, of cylindrical shape. Body of the uterus C. IgM
cannot be palpated separately. On both sides D. IgG
of the uterus palpation reveals tight tumors wi- E. IgD
th an uneven surface. The tumors are immobi-
le andl fill the whole pelvic cavity. What is the 7. A 7-year-old patient presents with body
most likely diagnosis? temperature rise up to 39o C , dry cough, pain
in the lateral abdomen. Objectively: there is
A. Krukenberg tumor cyanosis of the nasolabial triangle, inspiratory
B. Ovarian fibroid dyspnea with accessory muscle recruitment.
C. Ovarian granulosa cell tumor Percussion reveals pulmonary dullness; among
D. Bilateral pioovarium auscultation findings there are diminished
E. Subserous metrofibroma breath sounds, crepitant rales. Respiratory
3. A 38-year-old male patient complains of rate is of 50/min, HR- 120/min. Evaluate the
marked dyspnea that escalates with physical grade of respiratory failure in the patient:
exertion. The problems, namely acute chest A. II
pain on the left and cough, arose unexpectedly B. I
2 hours before at work. The pain abated, C. III
but there were progressing dyspnea, dizzi- D. IV
ness, pallor, cold sweat, cyanosis. Auscultation E. 0
reveals the absence of vesicular breath sounds,
radiograph shows a shadow on the left. What 8. A 43-year-old alcohol abuser had not
pathology can be suspected? consumed alcohol for the last two days. In
the evening he claimed to see rats and feel
A. Left-sided spontaneous pneumothorax like they bite his feet. The patient is disori-
B. Pulmonary infarction ented, agitated, all the time attempts to run
C. Pleurisy somewhere. Specify the psychopathological
D. Left-sided pneumonia syndrome:
E. Lung abscess
A. Delirious
4. A 2-year-old boy has been admitted to B. Amential
the casualty department for the pain and C. Oneiroid
deformity of his right thigh. Radiograph D. Choreatic
shows a femoral fracture with longitudinal E. Ganser’s syndrome
displacement. What method of treatment is
indicated for the patient? 9. A 67-year-old male patient complains of
rash, severe pain in the subscapular regi-
on on the right. Objectively: skin in the ri-
ght subscapular region is covered with li-
nearly arranged pink-red edematous lesions
that are somewhat infiltrated, and have clear
Krok 2 Medicine 2014 2

boundaries. On the lesion surface there are of intense pain in the right hypochondrium
vesicles with transparent exudate. What is the irradiating to the right supraclavicular region.
most likely diagnosis? Skin and sclerae are icteric. There is tension
and tenderness in the right hypochondrium
A. Herpes zoster on palpation. Body temperature is 38, 8o C .
B. Duhring dermatitis Blood test results: WBC- 11, 2 · 109 /l, total bi-
C. Erysipelas lirubin - 112 mmol/l (conjugated - 86 mmol/l,
D. Atopic dermatitis unconjugated - 26 mmol/l). What is the most
E. Impetigo likely diagnosis?
10. An 8-year-old boy was brought to the A. Cholangitis
admission department by his parents. Parents B. Acute pancreatitis
report that he has had pain in the right knee C. Acute appendicitis
for the last 9 months, recently mother has noti- D. Pancreatic tumor
ced some limitation of motion in his right leg, E. Perforated duodenal ulcer
and morning stiffness that doesn’t last till the
evening. What is the most likely diagnosis? 15. A 3-year-old girl has had an increase
in body temperature up to 38, 5o C for four
A. Juvenile rheumatoid arthritis days. The child refuses to eat. Over the last
B. Rheumatism two days, nose and mouth breathing has
C. Osteomyelitis of the knee joint become difficult. Mesopharyngoscopy reveals
D. Reactive arthritis hyperthermia and enlargement of tonsils, as
E. Traumatic arthritis well as hyperemia and bulging of the posteri-
11. A 13-year-old girl was admitted to the or wall of the oropharynx, which significantly
gynecology department for having a signifi- narrows the oropharyngeal lumen. What
cant bleeding from the genital tract for 10 complication of quinsy occurred in the pati-
days. The patient has a history of irregular ent?
menstrual cycle since menarche. Menarche A. Retropharyngeal abscess
occurred at the age of 11. Recto-abdominal B. Paratonsillar abscess
examination revealed no pathology. What is C. Parapharyngeal abscess
the provisional diagnosis? D. Phlegmon of the mouth floor
A. Juvenile uterine bleeding E. Laryngostenosis
B. Adenomyosis 16. Within a year, in a maternity hospital there
C. Injury of the external genitalia were 616 livebirths, one stillbirth, one infant
D. Werlhof’s disease died on the 5th day of life. What index can
E. Endometrial polyp most accurately describe this situation?
12. A 64-year-old male patient has a 35- A. Perinatal mortality
year history of chronic pancreatitis. In B. Total mortality
the last 5 years, he claims to observe the C. Birthrate
pain abatement, bloating, frequent bowel D. Infant mortality
movements up to 3-4 times a day, grayish, E. Natural increase
glossy stool with undigested food rests, the
progressive loss of body weight. Change of 17. In one of the surgical departments the
symptoms in the patient is due to overlay of: quality assurance testing of sterilization of
surgical instruments was performed. After
A. Exocrine pancreatic insufficiency an instrument had been treated with 1%
B. Endocrine pancreatic insufficiency phenolphthalein, the solution turned pink.
C. Lactase deficiency syndrome This indicates that the instrument has:
D. Irritable bowel syndrome
E. Chronic enterocolitis A. Synthetic detergent residues
B. Residual blood
13. During the doctor’s round, a 56-year-old C. Drugs residues
male patient with decompensated cirrhosis D. Residual tissue
complains of dizziness, palpitations, moving E. Disinfectant residues
black specks seen before the eyes, general
weakness. The patient is pale, Ps- 110/min, 18. A 21-year-old female patient consulted a
AP- 90/50 mm Hg. What complication is most gynecologist about itching, burning, watery
likely to have occurred in the patient? vaginal discharges with a fish-like smell.
Speculum examination revealed that the cervi-
A. Bleeding from esophageal varices cal and vaginal mucosa was of a normal pi-
B. Hepatocellular insufficiency nk color. Vaginal examination revealed no
C. Hepatic encephalopathy alterations of the uterus and appendages.
D. Acute coronary syndrome Gram-stained smears included clue cells.
E. Paroxysmal tachycardia What is the most likely pathology?
14. A 57-year-old female patient complains
Krok 2 Medicine 2014 3

A. Bacterial vaginosis (gardnerellosis) mitive judgments, is unabe to perform si-


B. Chlamydiosis mple arithmetic operations or explain simple
C. Gonorrhea metaphors. The patient is untidy, takes no
D. Trichomoniasis interest in anything, passive. Considers hi-
E. Candidiasis mself to be completely healthy. Qualify mental
condition of the patient:
19. A 9-year-old patient has measles. On
the 6th day after the rash appeared, the boy A. Total dementia
developed a condition manifested by dyspnea, B. Lacunar (dysmnestic) dementia
barking cough, stenotic respiration. Objecti- C. Somnolentia
vely: the rash on the face, neck and torso D. Korsakoff’s (amnesic) syndrome
turned brown. There is a branny desquamati- E. Hysterical pseudodementia
on. Respiratory rate is 22/min. What compli-
cation should be diagnosed? 23. A 13-year-old boy with hypertrophic cardi-
omyopathy complains of dyspnea on mini-
A. Laryngotracheitis mal exertion. EhoCG reveals asymmetric left
B. Bronchitis ventricular hypertrophy, signs of pulmonary
C. Pneumonia hypertension, dilatation of the left atrium. EF
D. Pharyngitis is 64%. The revealed alterations are indicative
E. Quinsy of:
20. A 26-year-old secundipara at 40 weeks of A. Diastolic heart failure
gestation arrived at the maternity ward after B. Systolic heart failure
the beginning of labor activity. 2 hours before, C. Primary pulmonary hypertension
bursting of waters occurred. The fetus was D. Primary arterial hypertension
in a longitudinal lie with cephalic presentati- E. Symptomatic arterial hypertension
on. Abdominal circumference was 100 cm,
fundal height - 42 cm. Contractions occurred 24. Study of natural illumination for a
every 4-5 minutes and lasted 25 seconds each. workplace in a secondary school classroom
Internal obstetric examination revealed cervi- revealed that the angle of sunlight incidence
cal effacement, opening by 4 cm. Fetal bladder was 25o , window opening angle - 3o , window-
was absent. Fetal head was pressed against the to-floor area ratio - 1:4, daylight ratio - 0,5%,
pelvic inlet. What complication arose in chi- depth ratio - 2. What indicators do not meet
ldbirth? hygienic standards?
A. Early amniorrhea A. Daylight ratio
B. Primary uterine inertia B. Window opening angle
C. Secondary uterine inertia C. Window-to-floor area ratio
D. Discoordinated labor D. Depth ratio
E. Clinically narrow pelvis E. Angle of incidence
21. Examination of a 35-year-old patient wi- 25. Bakers at bread production work in condi-
th rheumatism revealed that the right heart tions of high temperature and high heat radi-
border was 1 cm displaced outwards from the ation. What is used to increase the body’s
right parasternal line, the upper border was resistance to the unfavorable effects of these
on the level with inferior margin of the 1st harmful work environment factors?
rib, the left border was 1 cm in from the left
midclavicular line. Auscultation revealed atri- A. Vitamin preparations
al fibrillation, loud apical first sound, diastolic B. Milk
shock above the pulmonary artery. Echocardi- C. Pectin
ocopy revealed abnormal pattern of the mitral D. Therapeutic and preventive diet № 1
valve motion. What heart disease is characteri- E. Therapeutic and preventive diet number № 3
zed by these symptoms?
26. A 24-year-old male patient got a puncture
A. Mitral stenosis injury below the Poupart’s ligament, which
B. Mitral valve prolapse was accompanied by intense arterial bleedi-
C. Mitral valve insufficiency ng. The best method to temporarily stop the
D. Aortic stenosis bleeding in the patient would be:
E. Tricuspid valve insufficiency
A. Compression band
22. A 54-year-old male patient works as B. Esmarch’s tourniquet
an engineer. At the age of 35, he got C. Maximum limb bending
infected with syphilis and treated it with D. Compressing a blood vessel with a clamp
"traditional remedies". About 5 years ago, E. Wound suturing
he became forgetful, unable to cope with
work, told cynical jokes, bought useless thi- 27. 5 days before, a 26-year-old female patient
ngs, collected cigarette butts in the street. developed an acute condition. Objectively:
Objectively: the patient is indifferent, has marked headache, vomiting, weakness, poor
slow speech, dysarthria, can make only pri- appetite, temperature up to 39o C . Objecti-
Krok 2 Medicine 2014 4

vely: the patient is in a moderately grave agnosed with hidradenitis. What is the most
condition, excited. The face is hyperemic, likely causative agent of this disease?
sclerae are injected. The tongue is coated with
brown fur. The trunk and limbs are covered A. Staphylococci
with plentiful roseolous and petechial rash. B. Streptococci
Hepatosplenomegaly is present. Complement C. Proteus vulgaris
binding reaction with Rickettsia prowazekii D. Pseudomonas aeruginosa
is positive with the titer of 1:640. What drug E. Mixed infection
should be administered?
32. A 36-year-old female has a 7-year history of
A. Doxycycline pollen allergy. Over the last 2 years in August
B. Chloramphenicol and September (during ragweed flowering),
C. Penicillin the patient has had 2-3 asthma attacks that
D. Streptomycin could be treated with one dose of salbutamol.
E. Metronidazole Objectively: body temperature - 36, 5o C ,
respiratory rate - 18/min, Ps- 78/min, AP-
28. A 39-year-old female patient complains of 115/70 mm Hg. There is vesicular breathing
dyspnea when walking, palpitation, edemata above the lungs. Cardiac sounds are sonorous,
in the evening. The patient’s height is 164 cm, of regular rhythm. What drug would be most
weight - 104 kg. Objectively: overnutrition. effective to prevent asthma attacks during the
Heart sounds are weak, and tachycardia is critical season for the patient?
present. The menstrual cycle is not broken.
Blood sugar is 5,6 mmol/l, ACTH-response A. Intalum inhalation
tests revealed no alterations. X-ray of the B. Berotec inhalation
Turkish saddle revealed no pathology. What C. Atrovent inhalation
disease is it? D. Suprastin administration
E. Theopecum administration
A. Alimentary obesity
B. Climax 33. A study of the structure of death causes
C. Pituitary obesity in the urban population revealed that cardi-
D. Diabetes mellitus ovascular diseases accounted for 55,0% of all
E. Cushing’s syndrome (primary hypercorti- deaths. What statistic value represents these
solism) data?

29. A 26-year-old male patient complains of a A. Extensive index


rash on the upper lip skin, which arose on B. Intensive index
a background of influenza with high-grade C. Index of evidence
fever and is accompanied by pain and burning. D. Index of correlation
The rash has been present for 3 days. Objecti- E. Correspondence index
vely: the skin of the upper lip is edematic and
erythematous, grouped vesicles are filled with 34. A 25-year-old female patient complains of
serous fluid and have a rough surface. What is marked weakness, sleepiness, blackouts, di-
the most likely diagnosis? zziness, taste disorder. The patient has a hi-
story of menorrhagia. Objectively: the pati-
A. Herpetic vesicular dermatitis ent has marked weakness, pale skin, cracks in
B. Eczema the corners of mouth, peeling nails, systolic
C. Contact dermatitis apical murmur. Blood test results: RBC -
D. Dermatitis herpetiformis 3, 4 · 1012 /l, Hb- 70 g/l, color index - 0,75,
E. Erythema multiforme platelets - 140 · 109 /l, WBC- 6, 2 · 109 /l. What is
the most likely diagnosis?
30. A 6-year-old boy complains of paroxysmal
pain that occurs after a mental stress, consumi- A. Chronic posthemorrhagic anemia
ng cold drinks or ice cream. After clinical and B. Acute leukemia
instrumental examination the boy has been di- C. Acute posthemorrhagic anemia
agnosed with hypertensive biliary dyskinesia. D. B12 -deficiency anemia
The drugs of the following groups should be E. Werlhof’s disease
administered in the first place:
35. A 51-year-old female is a weaving factory
A. Antispasmodics and choleretics worker with 15 years of service record. During
B. Choleretics and cholekinetics a regular preventive examination she complai-
C. Sedatives and cholekinetics ned of frequent headaches, poor sleep, tingli-
D. Antioxidants ng in the heart, irritability, rapid fatigabili-
E. Antibiotics ty, hearing impairment. For years, the noise
level has exceeded the maximum allowable
31. A 15-year-old patient consulted a concentration by 10-15 dB. A year ago, the
dermatologist about a painful lump in patient underwent a course of treatment for
the armpit. Objectively: there is a walnut- essential hypertension. Specify the most likely
sized node, lymphadenitis, infiltration of the diagnosis:
surrounding tissues. The patient has been di-
Krok 2 Medicine 2014 5

A. Noise disease
B. Essential hypertension A. Hemolytic anemia
C. Neurasthenia B. Iron-deficiency anemia
D. Asthenic-vegetative syndrome C. Protein-deficiency anemia
E. Arteriosclerotic encephalopathy D. B12 -deficiency anemia
E. Hereditary elliptocytosis
36. A 49-year-old male patient who had been
scheduled for a surgery for gastric cancer 40. A lumbar puncture was performed for a
underwent preoperative infusion therapy. Up newborn suspected of having an intracranial
to 3,0 liters of liquid was introduced into the birth injury. Bloody cerebrospinal fluid was
right cubital vein. The following day, he felt a obtained. What hemorrhage occurred in this
dragging pain in the right shoulder. Objecti- case?
vely: on the inner surface of the shoulder
there is a longitudinal hyperemic zone, edema A. Subarachnoid
of skin, a tender cord. What complication B. Cephalohematoma
occurred in the patient? C. Epidural
D. Supratentorial
A. Acute thrombophlebitis E. Subtentorial
B. Venepuncture and edema of paravenous
tissue 41. A 36-year-old female patient complains
C. Paravenous tissue necrosis of intense pain in the knee joints and neck.
D. Acute lymphangitis In the morning she experiences pain in the
E. Paravenous tissue phlegmon interscapular region and leg joints; pain subsi-
des after warm-up gymnastics. The patient is
37. Some of the population of a city distri- overnourished, there is a clicking sound in the
ct have uneven teeth color. The individuals knees when squatting, the knees are somewhat
have white spots, transverse brown stripes on disfigured, painful on palpation. Blood test
the incisors. Occurrence of these symptoms is results: ESR- 18 mm/h, WBC- 8, 0 · 109 /l. Radi-
associated with the quality of drinking water ography reveals subchondral sclerosis in the
from a deep well. Which of the following left knee. What is the basis of this pathology?
components of water can be the cause of the
disease? A. Degenerative processes in cartilage
B. Autoimmune process in the synovium
A. F C. Deposition of urates (tophi) in the articular
B. Ca tissues
C. Mg D. Beta-haemolytic streptococcus
D. J E. Hemarthrosis
E. Fe
42. A 35-year-old female reports heart pain
38. A 22-year-old female patient has been (aching and drilling) occurring mainly in the
delivered by an ambulance team to a surgi- morning in autumn and spring and irradi-
cal clinic with symptoms of acute intestinal ating to the neck, back and abdomen; rapid
obstruction. It is known from the past hi- heartbeat; low vitality. Occurrence of this
story that 2 years ago she was operated for condition is not associated with physical acti-
acute destructive appendicitis. For two years, vity. In the evening, the patient’s condition
she has repeatedly complained of bloating improves. Study of somatic and neurological
and abdominal pain. Which of the followi- status, and ECG reveal no pathology. What
ng etiological factors has led to the intestinal pathology is most likely to have caused these
obstruction in the patient? clinical presentations?
A. Abdominal adhesions A. Somatization depression
B. Ileal diverticulum B. Resting stenocardia
C. Dolichosigma C. Pseudoneurotic schizophrenia
D. Diet violation D. Neurocirculatory asthenia
E. Helminthiasis E. Hypochondriacal depression
39. Mother of a 10-month-old baby reports 43. An hour before an elective surgery, a 56-
significant pallor, poor appetite, enlarged year-old patient of the surgical department
abdomen in the baby. As a neonate, the child got a dramatic increase in blood pressure,
underwent treatment in the in-patient hospi- tachycardia, hand tremor. The patient is
tal for jaundice and anemia. Objectively: the confused, anxious, depressed, fearful, is pessi-
skin is pale and jaundiced, teeth are absent, mistic about the operation outcome, refuses
abdomen is enlarged, spleen is palpable. the surgery. What tactics should be chosen by
Blood test results: Hb- 90 g/l, RBC- 3, 0·1012 /l, a surgeon?
color index - 0,9, microspherocytosis, reti-
culocytosis up to 20%, serum bilirubin - 37
mmol/l, unconjugated bilirubin - 28 mmol/l.
What type of anemia has occurred in the pati-
ent?
Krok 2 Medicine 2014 6

A. Start the surgery after correction of blood has been a smoker since childhood. Objecti-
pressure vely: to - 37, 4o C , respiratory rate is 26/min,
B. Isolate the patient Ps- 82/min, rhythmic. AP- 130/85 mm Hg.
C. Predict the psychological state of the patient There is limited breathing movement in the
D. Organize monitoring of the patient by right side of chest cavity, as well as percussive
medical personnel and mental health counselor dullness and diminished breath sounds. Radi-
E. Organize monitoring of the patient by his ograph shows a homogeneous opacity of the
family members pulmonary field on the right with the mediasti-
num displacement to the affected side. What
44. A 33-year-old female complains of escalati- is the most likely diagnosis?
ng spastic pain in the abdomen after the
psycho-emotional stress. The patient has A. Central lung cancer
intermittent bowel movements, that is 2-3 B. Pleural effusion
bowel movements after waking up alternate C. Pleuropneumonia
with constipation lasting for 1-2 days. Objecti- D. Pulmonary tuberculosis
vely: body weight is unchanged, there is E. Bronchiectasis
moderate pain on palpation of the sigmoid
colon. Hb- 130 g/l, WBC- 5, 2 · 109 /l, ESR- 9 48. A 33-year-old male patient developed a
mm/h. Proctosigmoidoscopy causes pain due condition that had a stormy clinical course:
to spastic bowel condition, intestinal mucosa chills, fever up to 39o C , vomiting, epigastric
is not changed. In the lumen there is a lot of pain, diarrhea with watery smelly feces. 6
mucus. What is the most likely diagnosis? hours before, he ate a raw egg, fried potatoes
with stewed meat, drank some juice. What
A. Irritable bowel syndrome pathogen is likely to have caused this conditi-
B. Crohn’s disease on?
C. Non-specific ulcerative colitis
D. Acute bowel ischemia A. Salmonella
E. Malabsorption syndrome B. Colibacillus
C. Campylobacter
45. An infant is 2 days old. He was born D. Shigella
full-term with signs of intrauterine infection, E. Vibrio cholerae
and therefore receives antibiotics. Neonates
should be given antibiotics at longer intervals 49. Chief physician of a polyclinic encharged
and lower doses compared to older children a district doctor with a task to determine the
and adults because: pathological prevalence of disease N in his
district. What document allows to estimate
A. Neonates have lower glomerular filtration the disease prevalence in the population of a
B. Neonates have lower concentration of medical district?
protein and albumin in blood
C. Neonates have a reduced activity of A. Prophylactic examinations register
glucuronyl transferase B. Statistic coupons (+)
D. Neonates have a decreased blood pH C. Statistic coupons (-)
E. Neonates have higher hematocrit D. Statistic coupons (+) and (-)
E. Vouchers for medical appointments
46. After a holiday in the Crimea, a 36-year-
old female patient presents with severe pain 50. A 76-year-old male consulted a therapi-
in the elbow joints, dyspnea and weakness. st about slow discharge of urine with a small
The body temperature is of 37, 6o C , the skin jet. The patient reported no cardiac problems.
is pale, there is erythema of cheeks and nose, Examination revealed atrial fibrillation with
lower lip ulceration. Visual inspection reveals a heart rate of 72/min and without pulse defi-
no changes in the joints, the right elbow cit. There are no signs of heart failure. ECG
movement is limited. There is murmur and confirms the presence of atrial fibrillation.
pleural friction in the lungs below the ri- From history we know that the arrhythmia
ght angle of the scapula. Cardiac sounds are was detected three years ago. What tactics for
muffled, there is tachycardia, gallop rhythm, the treatment of atrial fibrillation in the pati-
Ps- 114/min. AP- 100/60. What is the most li- ent should be chosen?
kely diagnosis?
A. Does not require treatment
A. SLE B. Digoxin
B. Rheumatic heart disease C. Verapamil
C. Rheumatoid arthritis D. Obzidan
D. Infectious allergic myocarditis E. Ajmaline
E. Dry pleurisy
51. A 53-year-old male has been admitted to
47. A 63-year-old male patient complains of a hospital for an attack of renal colic whi-
cough with expectoration of mucous blood- ch has repeatedly occurred throughout the
streaked sputum, asthma, low-grade fever, year. Objectively: in the region of auricles
general weakness. These presentations have and the right elbow some nodules can be
been observed for 3 months. The patient seen that are covered with thin shiny skin.
Krok 2 Medicine 2014 7

Ps- 88/min, AP- 170/100 mm Hg. There is bi- since he was 16, abuses alcohol, has a history
lateral costovertebral angle tenderness (posi- of CHD. The left lower extremity is colder
tive Pasternatsky’s symptom). The patient than the right one, the skin of extremities is
has been scheduled for examination. What dry, pedal pulse cannot be palpated, femoral
laboratory value would be most helpful for pulse is preserved. What is the most likely di-
making a diagnosis? agnosis?

A. Uric acid A. Obliterating endarteritis


B. Rheumatoid factor B. Diabetic angiopathy
C. ESR C. Leriche syndrome
D. Urine sediment D. Raynaud’s disease
E. Lactic acid E. Deep thrombophlebitis

52. A 58-year-old female patient complai- 56. A 38-year-old male complains of tonic
ns of spontaneous bruises, weakness, bleedi- tension of the masticatory muscles, so that
ng gums, dizziness. Objectively: the mucous he cannot open his mouth. 12 days before, he
membranes and skin are pale with numerous was bitten by an unknown dog. Objectively:
hemorrhages of various time of origin. Lymph there is pronounced tension and twitching of
nodes are not enlarged. Ps- 100/min, AP- the masticatory muscles. What is the most li-
110/70 mm Hg. There are no changes of kely diagnosis?
internal organs. Blood test results: RBC -
A. Tetanus
3,0·1012 /l, Нb - 92 g/l, colour index - 0,9, B. Rabies
anisocytosis, poikilocytosis, WBC - 10·109 /l, C. Hysteria
eosinophils - 2%, stab neutrophils - 12%, D. Trigeminal neuralgia
segmented neutrophils - 68%, lymphocytes E. Apyretic tetanus
- 11%, monocytes - 7%, ESR - 12 mm/h. What
laboratory test is to be determined next for 57. A 72-year-old male had had a moderate
making a diagnosis? headache. Two days later, he developed the
progressing speech disorders and weakness in
A. Platelets the right extremities. The patient has a history
B. Reticulocytes of myocardial infarction, arrhythmia. Study
C. Clotting time of the neurologic status revealed elements of
D. Osmotic resistance of erythrocytes motor aphasia, central paresis of the VII and
E. Fibrinogen XII cranial nerves on the right, central hemi-
paresis on the same side and hyperaesthesia.
53. A 48-year-old male in-patient undergoes What is the most likely diagnosis?
treatment for essential hypertension of II-
B stage. It is known from history that he A. Ischemic stroke
works in a design engineering office. His job B. Hemorrhagic stroke
involves neuro-emotional stress. Which of C. Transient ischemic attack
these foodstuffs do not stimulate the central D. Epidural hematoma
nervous system and can be recommended for E. Brain tumor
the patient?
58. A 28-year-old female patient has been
A. Whole milk admitted to the gynecology department for
B. Meat broths abdominal pain, spotting before and after
C. Mushroom broths menstruation for 5 days. The disease is associ-
D. Vegetable broths ated with the abortion which she had 2 years
E. Carbonated beverages ago. Anti-inflammatory treatment had no
effect. Bimanual examination findings: the
54. In an urban settlement situated on the ri- uterus is enlarged, tight, painful, smooth.
verbank, an outbreak of hepatitis A was regi- Hysteroscopy reveals dark red holes in the
stered. The disease might have water origin. fundus with dark blood coming out of them.
This assumption can be confirmed by growth What diagnosis can be made on the grounds
of the following values of water quality: of these clinical presentations?
A. Number of coli-phages A. Inner endometriosis
B. Escherichia coli index B. Polymenorrhea
C. Oxidability C. Hypermenorrhea
D. Presence of benign leptospirosis pathogen D. Submucous fibromatous node
E. Index of fecal coli-forms E. Dysfunctional uterine bleeding
55. A 48-year-old male patient complains of 59. 2 weeks after having quinsy, a 26-
pain in the lower extremities, especially when year-old male patient got facial edemata,
walking, intermittent claudication, numbness moderate pain in the sacrum. Objectively:
in the fingers, cold extremities, inability to body temperature is 37, 5o C , AP- 100/80 mm
walk more than 100 meters. Sleeps with his Hg. Urinalysis results: RBC- up to 100 fresh
leg lowered. The patient has been a smoker cells in per HPF, protein - 2,2 g/l, hyaline cyli-
Krok 2 Medicine 2014 8

nders - up to 10 per HPF, relative density - the heart’s need for oxygen without aggravati-
1002. What is the most likely diagnosis? ng the disease?
A. Acute glomerulonephritis A. Isosorbide dinitrate
B. Nephroma B. Corinfar
C. Acute pyelonephritis C. Atenolol
D. Urolithiasis D. Streptokinase
E. Chronic glomerulonephritis E. Aminophylline
60. Examination of a full-term 6-day-old infant 64. A 5-year-old girl has had thirst, polyuria,
revealed that different areas of skin had increased appetite for two months. At the
erythemas, flaccid bubbles, eroded surface, same time, there is a 3 kg decrease in body
cracks, peeling of the epidermis looking like weight. During the last week, these presentati-
being scalded with boiling water. There was ons got accompanied by nocturnal enuresis.
positive Nikolsky’s symptom. General condi- Examination revealed hyperglycemia at the
tion of the child was serious. The child was rate of 14 mmol/l. The child has been di-
restless, hypersensitive, febrile. What is the agnosed with type I diabetes. What is the most
most likely diagnosis in this case? likely genesis of this disease?
A. Ritter’s exfoliative dermatitis A. Autoimmune
B. Neonatal phlegmon B. Viral
C. Finger’s pseudofurunculosis C. Bacterial
D. Neonatal pemphigus D. Neurogenic
E. Epidermolysis E. Viral and bacterial
61. A 39-year-old female patient complains of 65. An 8-year-old child with a 3-year history
rapid fatigability, drowsiness, dry skin, hair of diabetes was hospitalized in hyperglycemic
loss, swelling of the face. A month ago, she coma. Specify the initial dose of insulin to be
underwent a surgery for thyrotoxicosis. The administered:
patient has the following gland dysfunction:
A. 0,1-0,2 U/kg of body weight per hour
A. Thyroid (hypothyroidism), due to B. 0,05 U/kg of body weight per hour
inadequate operative technique C. 0,2-0,3 U/kg of body weight per hour
B. Pituitary, due to a tumor D. 0,3-0,4 U/kg of body weight per hour
C. Adrenal E. 0,4-0,5 U/kg of body weight per hour
D. Parathyroid, due to the gland removal
during surgery 66. A 12-year-old girl undergoes regular
E. Ovarian, due to a tumor gastroenterological check-ups for duodenal
ulcer, biliary dyskinesia. What is the
62. A 27-year-old patient has a severe recommended frequency of anti-relapse
headache, nausea and vomiting. Objecti- treatment?
vely: body temperature is 38, 9o C , there is
a haemorrhagic stellate rash on the legs. A. Twice a year
The patient takes meningeal pose in bed. B. Every two months
Meningeal symptoms are strongly positive. C. Every 3 months
Deep reflexes are brisk, uniform. Pathologi- D. Once a year
cal reflexes are absent. It has been suspected E. Three times a year
that the patient has epidemic cerebrospinal
meningitis. Which of additional tests should 67. On the 2nd day of disease a 27-year-old
be performed in the first place to verify the patient complains of unbearable headache,
diagnosis? repeated vomiting. Objectively: the patient
is in a grave condition. He is conscious but
A. Lumbar puncture adynamic. Lies in a forced position with his
B. Echoencephalography head thrown back. There is no skin rash.
C. Rheoencephalography Nuchal muscles are evidently rigid, there are
D. Electroencephalography Kernig’s and Brudzinski’s signs. to - 39, 5o C ,
E. Survey craniogram Ps- 120/min, AP- 130/80 mm Hg. The leading
syndrome of this disease is caused by:
63. 3 hours before, a 68-year-old male pati-
ent got a searing chest pain radiating to the A. Liquor hypertension
neck and left forearm, escalating dyspnea. Ni- B. Liquor hypotension
troglycerin failed to relieve pain but somewhat C. Affection of the cranial nerve nuclei
reduced dyspnea. Objectively: there is cri- D. Haemorrhages in the adrenal glands
mson cyanosis of face. Respiratory rate is E. Hyperthermy
28/min. The patient has vesicular breathing
with isolated sibilant rales. Heart sounds are 68. Two years ago, a 46-year-old patient was
muffled, with a gallop rhythm. Ps- 100/min, diagnosed with stage I silicosis. Currently
AP- 100/65 mm Hg. ECG shows negative T- the patient complains of escalating dyspnea,
wave in V 2 − V 6 leads. What drug can reduce pain in the infrascapular regions. Radiograph
Krok 2 Medicine 2014 9

shows a diffuse enhancement and distorti-


on of lung markings, as well as multiple A. Rheumatoid arthritis
nodular shadows 2-4 mm in diameter. There is B. Osteoarthritis
interlobar pleural density on the right. Dense C. Gout
shadows are found in the hilar regions. Specify D. Pseudogout
the form of radiographic pulmonary fibrosis in E. Multiple myeloma
this case:
73. A 30-year-old female patient has been
A. Nodular delivered to a hospital for sudden dyspnea
B. Interstitial progessing to asthma, sensation of having a
C. Interstitial nodular "lump in the throat", hand tremor, fear of
D. Nodal death. The attack has developed for the first
E. Tumor-like time and is associated with a strong emotion.
There is no previous history. Objectvely: respi-
69. A 63-year-old female complains of general ratory rate - 28/min, Ps- 104/min, rhythmic,
weakness, a feeling of heaviness, compressi- AP- 150/85 mm Hg. The patient has rapid
on in the epigastrium, postprandial fullness, superficial vesicular breathing with extended
nausea, belching after meals. These symptoms expiration. Percussion findings: heart borders
have been observed for about 15 years. are not changed. Cardiac sounds are loud,
Objectively: body temperature is 36, 4o C , rhythmic. What is the most likely diagnosis?
respiratory rate - 20/min, Ps - 88/min, blood
pressure - 115/75 mm Hg. Skin and mucous A. Neurocirculatory asthenia
membranes are pale. Blood test results: RBC - B. Bronchial asthma
2,0·1012 /l, Hb - 100 g/l. Tests revealed parietal- C. Hypertensive crisis
cell antibodies. What is the most likely reason D. Cardiac asthma
for the development of anemia in this patient? E. Thyrotoxic crisis

A. Production of antibodies to intrinsic factor 74. A 42-year-old male patient wth essential
B. Disruption of hemoglobin synthesis hypertension presents with headache, palpi-
C. Disruption of erythropoietin synthesis tations, unexplained fear. Objectively: Ps-
D. Impaired iron absorption 100/min, AP- 200/100 mm Hg, the left border
E. Increased loss of iron of cardiac dullness is displaced by 1,5 cm to
the left, vesicular breathing is present. ECG
70. During dressing of a poorly-granulating shows sinus tachycardia, signs of left ventri-
wound Pseudomonas aeruginosa infection was cular hypertrophy. What drug should be admi-
revealed. What medication would be optimal nistered as an emergency?
for the wound d-bridement?
A. Obzidan
A. Boric acid solution B. Dibazol
B. Biogenic stimulators C. Reserpine
C. Sulfonamides D. Magnesium sulfate
D. Salicylic acid E. Furosemide
E. Antibiotics
75. A 37-year-old male patient has a histrory
71. A 39-year-old male patient complains of of diabetes of moderate severity. On the left
moderate pain and weakness in the shoulder, side of face the patient has a carbuncle. What
back and pelvic girdle muscles, that has been severe complication might have occurred in
progressing for the last 3 weeks; great di- the patient?
fficulty in getting out of bed, going up and
down the stairs, shaving. Dermatomyositis has A. Cavernous sinus thrombosis
been suspected. Blood test results: Hb- 114 g/l, B. Lymphangitis
C. Endarteritis
WBC- 10, 8 · 109 /l, eosinophils - 9%, ESR -22 D. Thrombophlebitis
mm/h, C-reactive protein - (++). The alterati- E. Thromboembolism
on in the following laboratory value wil be of
decisive diagnostic significance: 76. A 42-year-old female patient complains of
a dull pain in her left side, low-grade fever,
A. Creatine phosphokinase accelerated painful urination in small porti-
B. Ceruloplasmin ons. These presentations have been observed
C. Sialic acids for three years. For a long time, the pati-
D. dsDNA antibodies ent has had cystitis with frequent exacerbati-
E. Gamma-globulins ons, there is pulmonary tuberculosis in the
72. A 60-year-old female patient complains of past history. Urinalysis results: microscopic
recurrent pain in the proximal interphalangeal hematuria, leukocyturia. What is the most li-
and wrist joints, their periodic swelling and kely provisional diagnosis?
reddening that have been observed for 4 years.
X-ray picture represents changes in form of
osteoporosis, joint space narrowing and single
usuras. What is the most likely diagnosis?
Krok 2 Medicine 2014 10

A. Renal tuberculosis before the onset of the disease the patient had
B. Urolithiasis examined a dead calf. What is the most likely
C. Chronic pyelonephritis diagnosis?
D. Renal tumor
E. Chronic cystitis A. Cutaneous anthrax
B. Bubonic plague
77. A woman at 30 weeks pregnant has had C. Carbuncle
an attack of eclampsia at home. On admissi- D. Lymphocutaneous tularemia
on to the maternity ward AP- 150/100 mm E. Erysipelas
Hg. Predicted fetal weight is 1500 g. There
is face and shin pastosity. Urine potein is 81. An employee of a petrol station with 15
0, 66o /oo . Parturient canal is not ready for deli- years of service record having contact wi-
very. An intensive complex therapy has been th ethylated gasoline presents with memory
started. What is the correct tactics of this case impairment, bradycardia, sensation of havi-
management? ng a hair in the mouth, skin paresthesia. In
this case, one can assume intoxication with the
A. Delivery by cesarean section following substance:
B. Continue therapy and prolong pregnancy
for 1-2 weeks A. Tetraethyl lead
C. Continue therapy and prolong pregnancy B. Lead chloride
for 3-4 weeks C. Organophosphates
D. Labor induction by intravenous oxytocin or D. Benzene
prostaglandins E. Nitrobenzene
E. Treat preeclampsia and achieve the delivery
by way of conservative management 82. A 38-year-old female suddenly developed
acute inflammatory rash in form of roseolas,
78. Examination of a dead man who died papules, vesicles that are scattered on the skin
from hanging revealed that cadaver spots di- of trunk in irregular and predominantly focal
sappeared when pressed upon and restored manner. The rash appeared a few hours after
after 50 seconds, rigor mortis was moderately visiting a restaurant. The patient complains of
expressed only in the masticatory muscles itching skin. What is the most likely diagnosis?
and the muscles of neck and fingers. Body
temperature was 31o C . Specify the time of A. Toxicodermatosis
death: B. Atopic dermatitis
C. Contact dermatitis
A. 6-7 hours D. Eczema
B. 1-2 hours E. -
C. 16-24 hours
D. 8-10 hours 83. The institutions which take part in medi-
E. 10-18 hours cal examinations can be prevention and
treatment facilities, medical board of Mini-
79. A 65-year-old male patient complains of stry of Defense, medical board of Ministry of
dyspnea that is getting worse with exerti- Home Affairs, medico-social expert commi-
on, morning cough with expectoration of ssions, forensic medical boards etc. What insti-
mucous sputum. For about 15 years, he has tutions are responsible for temporary disabili-
been subject to regular medical check-up ty examination?
for chronic bronchitis. The patient takes
berodual (16 inhaled doses per day). Objecti- A. Prevention and treatment facilities
vely: body temperature is 36, 8o C , RR- 24/min, B. Sanitary-and-prophylactic institutions
Ps- 110/min, AP- 145/90 mm Hg. Auscultati- C. Medico-social expert commissions
on reveals a lot of dry rales above the lungs. D. Medical boards of the Ministry of Defense
FEV1- 65%. What is the optimal tactics of E. Medical boards of the Ministry of Home
further management of the patient? Affairs

A. To administer inhalation corticosteroids 84. After lifting a load, a 36-year-old male


B. To administer antibiotics patient has experienced a severe pain in the
C. To administer theophylline lumbar region, which spread to the right
D. To increase the daily dose of berodual leg and was getting worse when he moved
E. To include short-acting β2 -agonists in the his foot or coughed. Objectively: the long
therapy back muscles on the right are strained. Achi-
lles jerk is reduced on the right. There is a
80. A 49-year-old countryman got an itching pronounced tenderness of paravertebral poi-
papule on the dorsum of his right hand. In nts in the lumbar region. The straight leg raise
the centre there is a vesicle with serosangi- (Lasegue’s sign) is positive on the right. What
nous exudate. Within the next 2 days the additional tests should be performed in the fi-
patient developed a painless edema of hand rst place?
and forearm. On the 4th day the temperature
rose to 38, 5o C , in the right axillary region a
large painful lymph node was found. One day
Krok 2 Medicine 2014 11

A. Radiography of the spinal column A. Central-unit


B. Computed tomography B. Centralized
C. Magnetic resonance tomography C. Decentralized
D. Electromyography D. Free
E. Lumbar puncture E. Combined
85. A 17-year-old male patient consulted a 89. A 43-year-old female patient complai-
therapist about malaise, chills, runny nose, ns of dyspnea, swelling of legs, abdomen
aching muscles and joints, nausea and di- enlargement, pricking heart pain. She has a
arrhea. The patient asks to prescribe him history of tuberculous bronchadenitis, qui-
a lot of painkillers and sedatives (tramadol nsies. The patient’s condition deteriorated 6
or solpadein that help the best, and di- months ago. Objectively: cyanosis, bulging
azepam). Pharyngeal mucosa is pale pink, neck veins, vesicular breathing. Heart borders
clean. Auscultation reveals vesicular breathi- are not displaced. Heart sounds are muffled,
ng. Tachycardia is present. The pupils are di- Ps- 106/min, liver is +4 cm, ascites is present.
lated, there is sluggish response to light. There Low voltage on the ECG has been revealed.
are injection marks on the forearm skin. Duri- Radiograph shows a thin layer of calcium
ng examination, the patient’s manner is vulgar, deposits along the left contour of heart. What
irritable, rude and untruthful. Make a di- treatment should be recommended to the pati-
agnosis: ent?

A. Opioid addiction A. Treatment by a cardiac surgeon


B. Painkillers addiction B. Digitalis preparations
C. Sedative drug addiction C. Anti-TB drugs
D. Acute respiratory disease D. Diuretics
E. Food-born toxic infection E. Vasodilators, nitrates

86. A 4-year-old boy had untimely vacci- 90. A 26-year-old female patient has an 11-
nation. He complains of painful swallowing, year history of rheumatism. Four years ago
headache, inertness, fever. Objectively: the she suffered 2 rheumatic attacks. Over the last
child is pale, has enlarged anterior cervical 6 months there have been paroxysms of atri-
lymph nodes, swollen tonsils with cyanotic al fibrillation every 2-3 months. What option
hyperemia, tonsils are covered with gray-white of antiarrhythmic therapy or tactics should be
pellicles which cannot be easily removed. proposed?
When the pellicles are forcibly removed, the
tonsils bleed. What is the most likely di- A. Prophylactic administration of cordarone
agnosis? B. Immediate hospitalization
C. Defibrillation
A. Oropharyngeal diphtheria D. Lidocaine administration
B. Lacunar tonsillitis E. Heparin administration
C. Pseudomembranous tonsillitis
D. Infectious mononucleosis 91. Blood typing resulted in positive
E. Follicular tonsillitis isohemagglutination reaction with standard
sera of A(II) and B(III) groups and negative
87. Public nurseries are designed as a single or reaction with sera of 0(I) and AB(IV) groups.
several one- or two-storey buildings linked What is this result indicative of?
by covered walkways. Planning the structure
of preschool instiutions is based upon the A. Faulty standard sera
following principle: B. The first blood group
C. The second blood group
A. Principle of group isolation D. The third blood group
B. Principle of autonomy E. The fourth blood group
C. Principle of age distribution
D. Principle of sex distribution 92. A 9-year-old girl has been admitted to
E. Principle of age-sex distribution a hospital for an elevated body temperature
(39, 8o C ), painful dry cough, abdominal pain
88. A city somatic hospital with 300 beds has a on the right. Examination reveals dullness on
main building which houses the therapeutic percussion on the right, diminished breath
and surgical departments. Several separate sounds, crepitus. What study is required to
buildings house the maternity, pediatric and make a diagnosis?
radiologic departments that are connected to
the main building by underground walkways A. Radiography of the chest cavity
and above-ground covered skybridges. Speci- B. USI of the chest cavity
fy the building system of the hospital: C. Pleural puncture
D. Bronchoscopy
E. Bronhography
93. A newborn has purulent discharges from
the umbilical wound, the skin around the navel
Krok 2 Medicine 2014 12

is swollen. The baby’s skin is pale, with a is 0,586 mmol/l, plasma potassium - 7,2 mmol/l.
yellow-gray tint, generalized hemorrhagic rash What treatment is necessary for this patient?
is present. What is the most likely diagnosis?
A. Hemodialysis
A. Sepsis B. Large doses of verospiron
B. Hemorrhagic disease of the newborn C. Plasma volume expanders
C. Hemolytic disease of the newborn D. Glucocorticosteroids
D. Thrombocytopathy E. Heparin
E. Omphalitis
98. A 10-year-old child with a history of
94. A pregnant 26-year-old woman was admi- nonrheumatic carditis has periodic attacks
tted to a hospital for abdominal pain and manifested by heart pain, dyspnea, pallor, hi-
bleeding from the genital tract. Bimanual gh blood pressure, a dramatic increase in heart
examination revealed that uterus was the si- rate up to 180/min. What drug would be most
ze of 9 weeks of pregnancy, the cervical canal effective to treat this patient?
let a finger through. Fetal tissues could be
palpated in the orifice. There was moderate A. Obsidan
vaginal bleeding. What is the tactics of choice? B. Procainamide
C. Lidocaine
A. Instrumental extraction of fetal tissue D. Verapamil
B. Surveillance E. Ajmaline
C. Administration of hormones
D. Hemostatic and antianemic therapy 99. A 45-year-old male patient with acute
E. Therapy for the maintenance of pregnancy abscess of the left lung has suddenly
developed acute chest pain and dyspnea whi-
95. On the second day of the disease a 22- le coughing, tachycardia has increased. The
year-old male patient complains of high-grade control Ro-gram shows left lung collapse, the
fever, headache in the region of forehead and air in the left pleural cavity and a horizontal
superciliary arches, and during eye movement; fluid level. What is the mechanism of this
aching muscles and joints. Objectively: body complication?
temperature is 39o C . Face is hyperemic,
sclerae are injected. The mucous membrane A. Abscess burst into the pleural cavity
of the soft palate and posterior pharyngeal B. Bullae rupture of the left lung
wall is bright hyperemic and has petechial C. Inflammation spread to the visceral pleura
hemorrhages. What changes in the hemogram D. Atelectasis of the left lung
are typical for this disease? E. Acute cardiovascular insufficiency

A. Leukopenia 100. A 24-year-old male patient had been


B. Leukocytosis diagnosed with class III diffuse toxic goi-
C. Neutrocytosis ter. There is moderate hyperthyroidism. A
D. Anemia surgery was suggested, and the patient agreed
E. Accelerated ESR to it. What preoperative measures should be
taken for prevention of thyrotoxic crisis in the
96. A 44-year-old male patient complains of postoperative period?
severe non-localized abdominal pain, pain in
the right shoulder girdle, repeated vomiting, A. Administration of antithyroid drugs
red urine. The onset of the disease is associ- B. Minimally invasive surgical techniques
ated with alcohol consumption. The face is C. Bed rest
hyperemic. AP- 70/40 mm Hg. Abdominal D. Detoxification therapy
radiography reveals no pathological shadows. E. Administration of corticosteroids
Hemodiastase is 54 mg/h/l. Prothrombin is
46%. What is the provisional diagnosis? 101. A 26-year-old male patient complains of
pain in the right knee, which is getting worse in
A. Acute pancreatitis the morning. Two weeks before, he consulted
B. Acute myocardial infarction an urologist about prostatitis. Objectively:
C. Perforated gastric ulcer conjunctivitis is present. There is also peri-
D. Thrombosis of mesenteric vessels articular edema of the knee joint, redness of
E. Aneurysm of the abdominal aorta the overlying skin. Rheumatoid factor was not
detected. Until further diagnosis is specified,
97. A 41-year-old male patient was delivered it would be reasonable to start treatment with
to a hospital unconscious. During the previ- the following antibiotic:
ous 7 days he had been taking large doses
of biseptolum for a cold. The night before, A. Tetracyclines
he began complaining of dyspnea, especially B. Cephalosporins
when lying down, swollen legs, 2-day urinary C. Penicillins
retention. In the morning he had seizures and D. Aminoglycosides
lost consciousness. Objctively: noisy breathi- E. Lincosamides
ng at the rate of 30/min, edematous legs and
lumbar region, Ps- 50/min. Plasma creatinine 102. A 66-year-old female patient has been
Krok 2 Medicine 2014 13

admitted to a hospital for massive gross A. Furosemide


hematuria with release of shapeless blood B. Mannitol
clots, frequent painful urination. The pati- C. Hydrochlorthiazide
ent also reports a moderate weight loss wi- D. Spironolactone
thin 3-4 months. Gross hematuria that was E. Moduretic
not accompanied by pain and dysuria first
occurred three months ago for no apparent 106. For 3 days, a 28-year-old emale patient
reason, and after a few days the bleeding had had the body temperature increase up to
subsided independently. What is the most li- 38o C , weakness, poor appetite, nausea, a si-
kely diagnosis? ngle vomiting. On the 4th day the temperature
was normal, the condition improved, but the
A. Bladder tumor jaundice developed. Objectively: moderate
B. Urolithiasis ictericity of skin, +3 cm enlarged liver of
C. Renal tumor elastic consistency. Ortner’s, Kehr’s and
D. Chronic cystitis Voznesensky’s symptoms are negative. What
E. Acute cystitis test will verify the diagnosis?

103. A 36-year-old female pesented to a A. IgM Anti-HAV detection


gynecological hospital with a significant B. Complete blood count
bleeding from the genital tract and a 1-month C. Ultrasound of the abdomen
delay of menstruation. Bimanual examinati- D. Total bilirubin
on revealed soft barrel-shaped cervix. Uterus E. AST activity
was of normal size, somewhat softened.
Appendages were unremarkable on both si- 107. An 11-year-old girl has been immunized
des. Speculum examination revealed that the according to her age and in compliance with
cervix was cyanotic, enlarged, with the the the calendar dates. What vaccinations should
external orifice disclosed up to 0,5 cm. Uri- the children receive at this age?
ne hCG test was positive. What is the most A. Diphtheria and tetanus
likely diagnosis? B. TB
A. Cervical pregnancy C. Polio
B. Uterogestation D. Hepatitis B
C. Abortion in progress E. Pertussis
D. Threatened miscarriage 108. A 40-year-old male patient has had heavi-
E. Ectopic pregnancy ness in the epigastric region for the last 6
104. A 47-year-old female patient has an 8- months. He has not undergone any exami-
year history of ulcerative colitis, has been nations. The night before, he abused vodka. In
treated with glucocorticoids. She complai- the morning there was vomiting, and 30 mi-
ns of cramping pain in the umbilical regi- nutes after physical activity the patient experi-
on and left iliac region which has signifi- enced dizziness and profuse hematemesis.
cantly increased during the past 2 weeks, di- What pathology should be suspected in the
arrhea with mucus and blood 4-6 times a day, first place?
elevated body temperature up to 38 − 39o C , A. Mallory-Weis’s syndrome
headache and pain in the knee joints. Objecti- B. Menetrier’s disease
vely: the patient is in moderate condition, C. Gastric ulcer
Ps- 108/min, AP- 90/60 mm Hg; heart and D. Perforated ulcer
lungs are unremarkable; the tongue is moi- E. Zollinger-Ellison syndrome
st; abdominal muscle tone is significantly
decreased; peristaltic noises are absent. What 109. A 55-year-old patient complains of severe
complication developed in the patient? itching, burning and pain in the eyes, skin
redness in the outer corners of the palpebral
A. Toxic dilatation of the colon fissure. Objectively: skin around the outer
B. Perforation of the colon corners of the palpebral fissure is macerated,
C. Enterorrhagia eczematous, there are single moist cracks.
D. Stricture of the colon Palpebral conjunctiva is hyperemic, quaggy.
E. Colon carcinoma There are minor discharges in form of stringi-
105. A 10-year-old child has been admitted ng mucus. What is the most likely diagnosis?
to a hospital with a closed craniocerebral A. Chronic conjunctivitis
injury with a suspected cerebral edema. The B. Acute conjunctivitis
patient is in grave condition, unconscious. C. Sty
The dyspnea, tachycardia, hypertension are D. Blepharitis
present. Muscle tone is increased, there is E. Atopic eyelid dermatitis
nystagmus, pupillary and oculomotor reacti-
ons are impaired. The mandatory component 110. A 3-month-old infant has occipital
of intensive care is dehydration. What diuretic alopecia, restless sleep, excessive sweating.
is adequate in this case? What disease can you think of?
Krok 2 Medicine 2014 14

A. Rickets A. Apply a bandage, give an injection of


B. Spasmophilic diathesis vasodilators
C. Anemia B. Administer heart medications
D. Phosphate diabetes C. Put the feet into hot water
E. Chondrodystrophy D. Rub the feet with snow
E. Apply an alcohol compress
111. A 50-year-old male in a grave conditi-
on has been admitted to the intensive care 116. A 21-year-old male patient got a deep cut
unit. It is known from life history that the pati- wound in his right thigh. In the emergency
ent works in agriculture, and 3 hours ago was room a surgeon on duty performed pri-
engaged into insecticide treatment of crops mary debridement of the wound and pri-
for control of colorado potato beetle. Conditi- mary wound closure with a suture. After 4
on on admission: acrocyanosis, bronchorrhea, days, there appeared pain, redness, edema,
tachypnea, AP- 100/60 mm Hg, Ps- 44/min. purulent discharge from the wound gap, body
What method of efferent therapy would be temperature rose up to 39o C . What kind of
most appropriate at this stage? wound complication can you think of and what
actions should be taken?
A. Hemosorbtion
B. Hemodialysis A. Wound abscess, remove the sutures and
C. Plasmapheresis drain the wound
D. Lymphosorption B. Infiltration, apply a hot compress
E. Plasma dialysis C. Erysipelas, prescribe antibiotics
D. Tetanus, active-passive immunization agai-
112. An 80-year-old patient complains of nst tetanus
constantly urinating small amounts, a feeling E. Lymphangitis, apply a hot compress
of pressure in the lower abdomen. Objectively:
there is a suprapubic spherical bulging with 117. A general practitioner visited a 2-year-old
percussion dullness over it. What syndrome child and diagnosed him with measles. The
occurred in the patient? child attends a nursery, has a 5-year-old si-
ster. What document must be filled in for the
A. Paradoxical ischuria effective antiepidemic measures in the given
B. Urinary incontinence health locality?
C. Dysuria
D. Enuresis A. Emergency notification on infectious di-
E. Pollakisuria sease (form № 058/o)
B. Carer’s leave certificate
113. Carpathian region is characterized by C. Infant’s record (report form № 112/o)
permanently high (over 80%) air humidi- D. House call record (form № 031/o)
ty. In the cold season the population of this E. Sick leave
region feels very cold at moderately low
temperatures. This is due to an increase in 118. An employee has been diseased for 4
the heat transfer by: months, further treatment is necessary, the
patient is incapacitated. Who is authorized to
A. Convection provide further disability examination of this
B. Emission patient?
C. Evaporation
D. Conduction A. Medical and social expert board
E. Radiation B. Medical consultative board
C. Physician in charge and chief of department
114. An 18-year-old girl complains of breast D. Chief physician of a medical facility
pain and engorgement, headaches, irri- E. Deputy chief responsible for disability
tability, swelling of the lower extremities. examination
These symptoms have been observed since
menarche and occur 3-4 days before the 119. It has been suspected that a newborn has
regular menstruation. Gynecological exami- congenital diaphragmatic hernia (asphyctic
nation revealed no pathology. Make a di- incarceration). What study will allow to confi-
agnosis: rm the diagnosis?
A. Premenstrual syndrome A. Plan radiography of the chest cavity
B. Neurasthenia B. Plan abdominal radiography
C. Renal disease C. Irrigography
D. Mastopathy D. Pneumoirrigoscopy
E. Cardiovascular disorder E. Fibroesophagogastroduodenoscopy
115. A 56-year-old male patient has been 120. A 25-year-old female presented to a
delivered to the emergency department wi- women’s welfare clinic and reported the
th frostbite on both feet. What aid should be inability to get pregnant within 3 years of
rendered to the victim? regular sexual activity. Examination revealed
Krok 2 Medicine 2014 15

increased body weight, male pattern of she hears the voice of her brother who tells
pubic hair growth, excessive pilosis of thi- her to go home. The patient is anxious, suspi-
ghs, dense enlarged ovaries, monophasic basal cious, looks around all the time. Specify the
temperature. What is the most likely di- psychopathological syndrome:
agnosis?
A. Hallucinatory
A. Polycystic ovarian syndrome B. Generalized anxiety disorder
B. Adnexitis C. Paranoiac
C. Adrenogenital syndrome D. Paraphrenic
D. Premenstrual syndrome E. Depressive
E. Gonadal dysgenesis
125. During the first home visit to a full-term
121. A factory’s sectorial doctor selects a boy after his discharge from the maternity
group of persons who often fall ill for thorough hospital a pediatrician revealed a symmetri-
monitoring. At the same time he takes into cal swelling of mammae without skin changes
consideration the number of etiologically over them, swelling of the scrotum. The body
related cases with temporary disability in temperature was of 36, 5o C . The baby was
each of the employees over the last year. An calm, sucked the mother’s breast actively.
employee falls into this group if the number of What condition should you think of?
sickness cases is:
A. Hormonal crisis of the newborn
A. 4 or more B. Neonatal mastitis
B. 1 or more C. Sclerema
C. 2 or more D. Necrotic neonatal phlegmon
D. 3 or more E. Congenital adrenal dysfunction
E. 6 or more
126. A 30-year-old male patient complains
122. A 23-year-old female consulted a of inertness, low-grade fever, bleeding gums,
gynecologist on the 20th day postpartum peri- frequent quinsies, aching bones. Objecti-
od about pain in the left breast, purulent vely: the patient has pale skin and mucous
discharge from the nipple. Objectively: Ps- membranes, sternalgia, +2 cm liver, +5 cm
120/min, t - 39 C . The left breast is painful, painless spleen. Blood test results: RBC-
larger than the right one, hyperemic. In the 2, 7 · 1012 /l, Нb- 80 g/l, WBC- 3 · 109 /l,
upper quadrant there is an infiltrate sized eosinophils - 4%, basophils - 5%, blasts -
10x15 cm with a softening inside. Blood test 4segmented neutrophils - 17%, lymphocytes -
results: ESR- 50 mm/h, WBC- 15, 0 · 109 /l. 29%, myelocytes - 25%, promyelocytes - 12%,
What is the tactics of choice? monocytes - 2%, platelets - 80 · 109 /l, ESR -
57 mm/h. What test should be performed to
A. Refer to the surgical department for operati- verify the diagnosis?
ve treatment
B. Refer to the gynecology department A. Sternal puncture
C. Refer to the postpartum department B. Trephine biopsy
D. Refer to a polyclinic surgeon for conservati- C. Lymph node biopsy
ve treatment D. Lumbar puncture
E. Lance the breast abscess in the women’s E. Chest X-ray
health clinic
127. A 46-year-old male patient complains of
123. A family lives in the town situated within periodic epigastric pain that occurs at night.
the zone of radiation pollution. A 6-year-old Objectively: HR- 70/min, AP- 125/75 mm Hg,
child had been ill with ARVI for 19 days. The tenderness in the epigastric region is present.
child was undergoing outpatient treatment EGD confirms duodenal ulcer of 0,6 cm in
and was nursed by his mother, a cafe worker. diameter. Test for H. Pylori is positive. Whi-
Specify the order of disability examination: ch of the given antisecretory drugs will be
a compulsory element of the treatment regi-
A. Sick leave is granted for the entire period of men?
the child’s illness
B. Sick leave is granted for a total of 14 days, A. Omeprazole
and after that period is over, a carer’s leave B. Famotidine
certificate is issued C. Pirenzepine
C. Sick leave is not granted, only a carer’s leave D. Atropine
certificate is issued E. Maalox
D. Sick leave is granted for 14 days, after this
period is over, no document is issued 128. A 47-year-old male patient has been lately
E. Sick leave is granted for 7 days, after this complaining of compressing chest pain that
period is over, a carer’s leave certificate is issued occurs when he walks a distane of 700-800 m.
Once a week, he drinks 2 liters of beer. Rise in
124. A 49-year-old female patient with schi- arterial pressure has been observed for the last
zophrenia is all the time listening to somethi- 7 years. Objectively: Ps- 74/min, AP- 120/80
ng, insists that "there is a phone in her head"as mm Hg. The bicycle ergometry performed at
Krok 2 Medicine 2014 16

workload of 75 watts shows 2 mm ST -segment A. Botulism


depression in V 4 − V 6 leads. What is the most B. Shigellosis
likely diagnosis? C. Salmonellosis
D. Cholera
A. Exertional stenocardia, II functional class E. Yersiniosis
B. Exertional stenocardia, III functional class
C. Exertional stenocardia, IV functional class 133. A full-term neonate weighing 4500 g was
D. Vegetative-vascular dystonia of hypertensi- born asphyxiated with Apgar score of 4-6 poi-
ve type nts. During the delivery shoulder dystocia
E. Alcoholic cardiomyopathy occurred. Neurologic assessment revealed
non-focal neurologic symptoms, total flaccid
129. A puerperant is 28 years old. It’s the 3rd paresis of the upper extremities since the arm
day post-partum after a second, normal, term was atonic and pronated. Grasping, Babki-
delivery. The body temperature is of 36, 8o C , n’s and Moro’s reflexes were absent. What
Ps- 72/min, AP- 120/80 mm Hg. Mammary segments of spinal cord had been affected?
glands are moderately engorged, the nipples
are clean. Abdomen is soft, painless. The A. V - T hI
fundus is 3 fingers’ breadth below the navel. B. I - II
Moderate bloody lochia are present. What di- C. III - IV
agnosis can be made? D. T hI - T hV
E. T hV I - T hV 
A. Physiological course of the postpartum
period 134. A newborn (mother’s I pregnancy) wei-
B. Subinvolution of uterus ghing 3500 g has been found to have jaundice,
C. Postpartum metroendometritis lethargy, reduced reflexes. Objectively: second
D. Remains of placental tissue after childbirth grade jaundice of skin with saffron tint, li-
E. Lactostasis ver - +2 cm, spleen - +1 cm. Urine and feces
are yellow. Blood count: Hb- 100 g/l, RBC-
130. A 29-year-old unconscious patient has 3, 2 · 1012 /l, WBC- 18, 7 · 109 /l, mother’s blood
been delivered to a hospital. Objectively: skin type - 0(I) Rh(+), baby’s blood type - A(II)
and mucous membranes are pale, cyanotic, Rh(-), bilirubin - 170 mmol/l, indirect fraction.
breath sounds are dramatically diminished ALT, AST rates are normal. What is the most
on the right and cannot be auscultated in the likely disease in the child?
lower parts, at the level of the 6 rib along the
anterior axillary line there is a wound hole wi- A. Hemolytic disease of newborn, AB0-conflict
th moderate bleeding and passage of air duri- B. Perinatal hepatitis
ng inspiration. Radiography reveals a bullet in C. Hemolytic disease of newborn, Rh-conflict
the pleural cavity. What is the medical tactics D. Biliary atresia
of choice? E. Physiologic jaundice
A. Emergency thoracotomy 135. As a result of a road accident a 45-
B. Thoracoscopy with removal of bullet year-old male patient got multiple injuries,
C. Converting a tension pneumothorax into a namely closed fractures of the right humerus
simple (open) pneumothorax and the left antebrachial bones with a di-
D. Thoracostomy splacement of bone fragments, a closed blunt
E. Tight bandage on a wound abdominal injury. The patient was delivered
to the emergency department 30 minutes after
131. A patient had four generalized convulsive the injury. Objectively: the skin is pale. AP-
seizures within a day. Between the seizures the 90/20 mm Hg, there is pain and deformati-
patient did not come to waking consciousness on at the fracture sites. Abdomen is tense,
(was in a coma or stupor). Specify his state: palpation causes severe pain, there is rebound
tenderness (positive Blumberg’s sign). What is
A. Status epilepticus the treatment tactics of choice?
B. Frequent generalized seizures
C. Frequent jacksonian seizures A. Urgent diagostic laparotomy
D. Hysterical attacks B. Infusion therapy to stabilize blood pressure
E. Frequent complex partial seizures C. Fracture immobilization, analgesia
D. Local anesthetic blockade of fractures
132. A 12-year-old boy presents with nausea, E. Additional tests to specify the diagnosis
frequent repeated vomiting that first occurred
after eating canned vegetables. Objectively: 136. A 34-year-old male visited Tajikistan.
the patient has dry mucous membranes, After return, he complains of fever up
muscular hypotonia, anisocoria, mydriasis, to 40o C which occurs every second day
dysphagia and dysarthria. What is the most and is accompanied by chills, sweating.
likely diagnosis? Hepatosplenomegaly is present. Blood test
results: RBC- 3 · 1012 /l, Нb- 80 g/l, WBC- 4 ·
109 /l, eosinophils - 1%, stab neutrophils - 5%,
segmented neutrophils - 60%, lymphocytes -
Krok 2 Medicine 2014 17

24%, monocytes - 10%, ESR - 25 mm/h. What A. Pericardiocentesis and immediate


is the provisional diagnosis? thoracotomy
B. Oxygen inhalation
A. Malaria C. Puncture of the pleural cavity on the left
B. Infectious mononucleosis D. Conservative treatment, infusion of
C. Sepsis adrenomimetics
D. Typhoid fever E. Pleural cavity drainage
E. Leptospirosis
141. A puerperant is 32 years old, it’s her first
137. A 23-year-old male patient consulted childbirth, term precipitate labor, the III peri-
a doctor about pain occurring in the lower od is unremarkable, the uterus is contracted,
third of the thigh with weight bearing acti- tight. Examination of the birth canal revealed
vities and unloaded. The patient denies any a rupture in the left posterior vaginal wall that
injuries to the region. Objectively: the skin was closed with catgut. Two hours later, the
is of normal color, deep palpation reveals patient complained of a feeling of pressure on
pastosity and tenderness, movements of the the anus, pain in the perineum, minor vaginal
knee joint are limited. Radiograph of the di- discharges, edema of the vulva. These clinical
stal femoral metaepiphyseal region shows a presentations are indicative most likely of:
zone of degradation and spicules. In blood:
immature cells are present, there are no signs A. Vaginal hematoma
of inflammation. What is the most likely di- B. Hysterocervicorrhexis
agnosis? C. Hemorrhoids
D. Hysterorrhesis
A. Osteosarcoma E. Hypotonic bleeding
B. Hyperparathyroid dystrophy
C. Chronic osteomyelitis 142. A hospital nutrition unit received a batch
D. Multiple myeloma of beef. Sanitation physician examined the
E. Marble bone disease meat and revealed the presence of 5 bladder
worms per 40 cm2 of meat. Give the hygienic
138. It is planned to organize a rural outpati- assessment of meat:
ent clinic. The patients will be able to visit the
doctors of the following specialities: A. Liable to technical utilization
B. Conditionally admissible
A. Therapeutist, dentist, pediatrician, C. Adulterated
obstetrician-gynecologist D. Good-quality
B. Therapeutist, pediatrician, neurologist E. Poor-quality
C. Pediatrician, obstetrician-gynaecologist,
ophthalmologist 143. A 47-year-old female patient complains of
D. Pediatrician, therapeutist, ophthalmologist having pain on swallowing and difficult passi-
E. Obstetrician-gynaecologist, therapeutist ng of solid food for two months. The pati-
ent has taken to the liquid and semi-liquid
139. A 12-year-old girls has minor functi- food. During the last week the liquid food has
onal and morphological abnormalities: 1,0 D barely passed through. General condition is
myopia, reduced body resistance. The patient satisfactory, the patient is undernourished, the
has no history of chronic diseases. Over the appetite is preserved, there is a fear of eating.
last year, there were 4 cases of respiratory di- What is the provisional diagnosis?
seases. The girl belongs to the following health
group: A. Esophageal carcinoma
B. Esophageal stricture
A. II C. Esophageal foreign body
B. I D. Esophageal varices
C. III E. Esophageal achalasia
D. IV
E. V 144. A 31-year-old female patient complai-
ns of infertility, amenorrhea for 2 years after
140. 4 weeks after a myocardial infarction, the artificial abortion that was complicated by
a 56-year-old male patient developed acute endometritis. Objectively: examination of the
heart pain, pronounced dyspnea. Objectively: external genitalia revals no pathology, there is
the patient’s condition is extremely grave, female pattern of hair distribution. According
there is marked cyanosis of face, swelling and to the functional tests, the patient has biphasic
throbbing of neck veins, peripheral pulse is ovulatory cycle. What form of infertility is the
absent, the carotid artery pulse is rhythmic, case?
130 bpm, AP- 60/20 mm Hg. Auscultation
of heart reveals extremely muffled sounds, A. Uterine
percussion reveals heart border extension in B. Ovarian
both directions. What is the optimal treatment C. Pituitary
tactis for this patient? D. Hypothalamic
E. Immunological
Krok 2 Medicine 2014 18

145. A 49-year-old male patient consulted analyses of the daily urinary excretion of
a doctor about difficult swallowing, voice albumin revealed microalbuminuria (200-
hoarseness, weight loss. These symptoms have 300 mg/day). Glomerular filtration rate
been gradually progressing for the last 3 is 105 ml/min. Blood pressure is within
months. Objectively: the patient is exhausted, normal range. Normalization of the followi-
supraclavicular lymph nodes are enlarged. ng indicator should be the first-priority task
Esophagoscopy revealed no esophageal in the secondary prevention of diabetic
pathology. Which of the following studies is nephropathy:
most appropriate in this case?
A. Glycosylated hemoglobin
A. Computed tomography of chest and medi- B. C-peptide
astinum C. Blood insulin
B. X-ray of lungs D. Fasting glucose
C. Multiplanar imaging of esophagus E. Glycemia 2 hours after a meal
D. Radioisotope investigation of chest
E. Ultrasound investigation of mediastinum 150. 10 days after birth, a newborn developed
a sudden fever up to 38, 1o C . Objectively:
146. A 36-year-old male patient complains of the skin in the region of navel, abdomen
having headache, obstructed nasal breathing, and chest is erythematous; there are multi-
purulent nasal discharge for 2 weeks. A month ple pea-sized blisters with no infiltration at the
before, he had his right maxillary premolar fi- base; single bright red moist erosions with epi-
lled. Radiolography revealed an intense opaci- dermal fragments on the periphery. What is
ty of the right maxillary sinus. Diagnostic your provisional diagnosis?
puncture revealed a large amount of thick
malodorous crumbly pus. What is the most A. Epidemic pemphigus of newborn
likely diagnosis? B. Syphilitic pemphigus
C. Streptococcal impetigo
A. Chronic suppurative odontogenic sinusitis D. Vulgar impetigo
B. Acute purulent sinusitis E. Atopic dermatitis
C. Chronic purulent sinusitis
D. Chronic atrophic sinusitis 151. An emergency physician arrived to provi-
E. Tumor of the maxillary sinus de medical care for a hangman taken out
of the loop by his relatives. The doctor
147. A 5-year-old boy has a history of repeated revealed no pulse in the carotid arteries, lack
pneumonia, frequent acute respiratory viral of consciousness, spontaneous breathing and
diseases. Objectively: exertional dyspnea, mi- corneal reflexes; cadaver spots on the back
nor fatigabilty. There is a systolic murmur and posterior parts of extremities. A person
having its epicenter in the IV intercostal can be declared dead if the following sign is
space on the left. Left relative dullness is present:
found along the midclavicular line. Accordi-
ng to the findings of instrumental methods of A. Cadaver spots
examination (electrocardiography, echocardi- B. Lack of spontaneous breathing
ography), the patient has been diagnosed wi- C. Lack of corneal reflexes
th ventricular septal defect, subcompensation D. Pulselessness
stage. What is the main method of treatment? E. Unconsciousness

A. Operative therapy 152. A 69-year-old male patient has been


B. Phytotherapy hospitalized with hypothermia. Objectively:
C. Does not require treatment the patient is pale, has shallow breathing. AP-
D. Conservative treatment 100/60 mm Hg, Ps- 60/min. Palpation of the
E. Indomethacin abdomen and chest reveals no pathological
signs. The body temperature is of 34, 8o C .
148. A patient with autoimmune thyroi- The patient’s breath smells of alcohol. Give
ditis accompanied by multinodular goiter treatment recommendations:
underwent the right lobe ectomy and subtotal
resection of the left lobe. What drug should A. Warm bath + intravenous administration of
be administered to prevent postoperative warm solutions
hypothyroidism? B. Body warming with hot-water bags
C. Forced diuresis
A. L-thyroxine D. Rubbing with alcohol and snow
B. Merkazolil E. Antishock therapy
C. Iodomarin
D. Lithium drugs 153. A 26-year-old male patient consulted a
E. Insulin doctor abut sore throat, fever up to 38, 2o C .
A week before, the patient had quinsy, di-
149. A 49-year-old female patient has type dn’t follow medical recommendations. On
1 diabetes of moderate severity. The di- examination, the patient had forced position
sease is complicated by retinopathy and of his head, trismus of chewing muscles. Left
polyneuropathy. Besides that, repeated peritonsillar region is markedly hyperemic,
Krok 2 Medicine 2014 19

swollen. What is the provisional diagnosis? upper extremities and decreased pressure
in the lower ones, extension of the left
A. Left-sided peritonsillar abscess heart border, blowing systolic murmur in the
B. Meningitis interscapular region. ECG shows the hori-
C. Phlegmonous tonsillitis zontal axis of heart. Radiography reveals left
D. Pharyngeal diphtheria cardiomegaly, costal usuration. What is the
E. Tonsil tumour most likely diagnosis?
154. A 21-year-old female patient has been A. Aortarctia
hospitalized on an emergency basis because B. Aortic stenosis
of severe dyspnea, pain in the left side of C. Patent ductus arteriosus
chest. Body temperature is 38, 8o C . The condi- D. Ventricular septal defect
tion developed three days ago. Respiratory E. Atrial septal defect
rate is 42/min, auscultation reveals shallow
breathing. There is percussive dullness on 158. During the preventive examinati-
the right starting from the middle of the on a 17-year-old young man reports no
blade, breath sounds cannot be heard. The left health problems. Objectively: the patient is
border of heart is 3 cm displaced outwards. undernourished, asthenic; blood pressure is
Embryocardia is present, HR is 110/min. The 110/70 mm Hg, Ps- 80/min. Heart borders
right hypochondrium is painful on palpation. are within normal range. Auscultation reveals
What urgent therapeutic measures should be three apical heart sounds, murmurs are absent.
taken in this situation? ECG shows no pathological changes, PCG
registers the S3 occurring 0,15 seconds after
A. Emergency puncture of the pleural cavity the S2. How can you interpret these changes?
B. Administration of penicillin antibiotics
C. Injection of Lasix A. Physiologic S3
D. Injection of cardiac glycosides B. Fout-ta-ta-rou (three-component rhythm)
E. Transferring the patient to the thoracic C. Protodiastolic gallop rhythm
surgery department D. Presystolic gallop rhythm
E. Physiologic S4
155. A 77-year-old male patient complains of
inability to urinate, bursting pain above the 159. After having the flu, a 39-year-old
pubis. The patient developed acute condi- male patient with a history of Addison’s
tion 12 hours before. Objectively: full uri- disease developed a condition manifested
nary bladder is palpable above the pubis. by weakness, depression, nausea, vomi-
Rectal prostate is enlarged, dense and elastic, ting, diarrhea, hypoglycemia. AP- 75/50
well-defined, with no nodes. Interlobular mm Hg. Blood test results: low corti-
sulcus is distinct. Ultrasonography results: costerone and cortisol, 13-oxycorticosteroids,
prostate volume is 120 cm3 , it projects into the 17-oxycorticosteroids levels. What condition
bladder cavity, has homogeneous parenchyma. developed in the patient?
Prostate-specific antigen rate is of 5 ng/ml.
What is the most likely disease that caused A. Acute adrenal insufficiency
acute urinary retention? B. Acute gastritis
C. Acute enterocolitis
A. Prostatic hyperplasia D. Collapse
B. Prostate carcinoma E. Diabetes mellitus
C. Sclerosis of the prostate
D. Tuberculosis of the prostate 160. A female patient complains of being
E. Acute prostatitis unable to get pregnant for 5 years. A complete
clinical examination brought the following
156. At the first minute of life a full-term results: hormonal function is not impaired,
infant born with umbilical cord entangled urogenital infection hasn’t been found, on
around his neck has total cyanosis, apnea, hysterosalpingography both tubes were filled
HR- 80/min, hypotonia and areflexia. There with the contrast medium up to the isthmic
are no signs of meconium aspiration. After segment, abdominal contrast was not visuali-
the airway suctioning the newborn did not zed. The patient’s husband is healthy. What
start breathing. What is the next action of the tactics will be most effective?
doctor?
A. In-vitro fertilization
A. ALV with a 100% O2 mask B. Insemination with husband’s sperm
B. Intravenous administration of adrenaline C. ICSI within in-vitro fertilization program
C. Intravenous administration of etamsylate D. Hydrotubation
D. Tracheal intubation and ALV E. Laparoscopic tubal plasty
E. Stimulation of the skin along the spine
161. A 19-year-old primiparous woman wi-
157. Examination of an 11-year-old boy th a body weight of 54,5 kg gave birth at
revealed frequent nosebleeds, fatigue when 38 weeks gestation to a full-term live girl
walking, underdevelopment of the lower half after a normal vaginal delivery. The girl’s wei-
of the body, increased blood pressure in the ght was 2180,0 g, body length - 48 cm. It is
Krok 2 Medicine 2014 20

known from history that the woman has been suspected?


a smoker for 8 years, and kept smoking duri-
ng pregnancy. Pregnancy was complicated by A. Heavy-metal coniosis
moderate vomiting of pregnancy from 9 to 12 B. Silicosis
weeks pregnant, edemata of pregnancy from C. Silicatosis
32 to 38 weeks. What is the most likely cause D. Carbon pneumo coniosis
of low birth weight? E. Bronchitis

A. Fetoplacental insufficiency 166. A 12-year-old boy periodically has short


B. Low weight of the woman episodes (10-15 seconds) of a brief loss of
C. Woman’s age awareness with a dazed look and eyes stare in
D. First trimester preeclampsia an upright position, blank expression of face,
E. Third trimester preeclampsia absence of motions and subsequent amnesia.
Specify the described state:
162. In a cold weather, the emergency room
admitted a patient pulled out of the open A. Absence seizure
water. There was no respiratory contact wi- B. Obnubilation
th the water. The patient is excited, pale, C. Trance
complains of pain, numbness of hands and D. Fugue
feet, cold shiver. Respiratory rate is 22/min, E. Sperrung
AP- 120/90 mm Hg, Ps- 110/min, rectal
temperature is 34, 5o C . What kind of warming 167. Evaluation results of sanitary and hygiene
is indicated for this patient? conditions in a 4-bed ward were as follows:
ward area - 30 m2 , height - 3,2 m, temperature
A. Passive warming - 20o C , humidity - 55%, air velocity - 0,1 m/s,
B. Infusion of 37o C solutions window-to-floor area ratio - 1:5, daylight ratio
C. Hot compresses - 0,6%, concentration of carbon dioxide in the
D. Warm bath air - 0,1%. Which of the given indicators does
E. Hemodialysis with blood warming not meet hygienic requirements?
163. A week before, a 65-year-old male pati- A. Daylight ratio
ent suffered an acute myocardial infarction. B. Air velocity
His general condition has deteriorated: he C. Window-to-floor area ratio
complains of dyspnea at rest, pronounced D. Concentration of carbon dioxide in the air
weakness. Objectively: edema of the lower E. Ward area
extremities, ascites is present. Heart borders
are extended, paradoxical pulse is 2 cm di- 168. A 25-year-old patient works as a tractor
splaced from the apex beat to the left. What is driver. Four days ago, he got pain in the left
the most likely diagnosis? axillary region, general weakness, fever up to
38o C . He hadn’t sought medical helf until a
A. Acute cardiac aneurysm painful solid lump appeared in this region.
B. Recurrent myocardial infarction Objectively: in the left axilla there is a very
C. Acute pericarditis painful cone-shaped mass sized 3x2,5 cm, wi-
D. Cardiosclerotic aneurysm th a destruction in the center of the pointed
E. Pulmonary embolism vertex. The surrounding skin is hyperemic,
there are purulent discharges. What is the
164. A 41-year-old patient cosulted a most likely diagnosis?
dermatologist about discoloration, thickening,
brittleness of toenails. These symptoms have A. Hydradenitis
been present for about five years. Objecti- B. Carbuncle
vely: nail plates in all toes are thickened, of C. Furuncle
dirty yellow color, lustreless, crumble over the D. Lymphadenitis
edge. Microscopy of the nail plate material E. Abscess
treated with alkali revealed mycelial filaments.
Material inoculation onto Sabouraud medium 169. A 19-year-old male patient complains of
resulted in growth of the Trichophyton rubrum intense pain in the left knee joint. Objectively:
colony. What is the most likely diagnosis? the left knee joint is enlarged, the overlyi-
ng skin is hyperemic, the joint is painful on
A. Rubromycosis of toenails palpation. Blood test results: RBC- 3, 8 · 1012 /l,
B. Candidal onychia Hb- 122 g/l, lymphocytes - 7, 4 · 109 /l, platelets
C. Psoriasis of the nails - 183 · 109 /l. ESR- 10 mm/h. Duke bleeding
D. Nail dystrophy time is 4 minutes, Lee-White clotting time - 24
E. Epidermophytosis of nails minutes. A-PTT is 89 sec. Rheumatoid factor
165. Examination of an electric welder with is negative. What is the most likely diagnosis?
15 years of service record revealed dry rales
in the lower lung fields. Radiograph shows
diffuse nodules sized 3-4 mm in the middle
and lower lung fields. What disease can be
Krok 2 Medicine 2014 21

A. Hemophilia, hemarthrosis A. Sorting, medical assistance, evacuation


B. Werlhof’s disease B. Sorting, evacuation, treatment
C. Rheumatoid arthritis C. Medical assistance, evacuation, isolation
D. Thrombocytopathy D. Isolation, rescue activity, recovery
E. Hemorrhagic vasculitis, articular form E. Sorting, recovery, rescue activity
170. A 29-year-old female patient complai- 174. A 36-year-old injured has been taken
ns of dyspnea and palpitations on exertion. to the emergency station with open tibi-
According to her mother, as a child she had al fractures. Examination reveals bleeding:
heart murmur, did not undergo any exami- pulsating blood spurts out of the wound. What
nations. Objectively: the patient has pale skin, medical manipulations are required at this
Ps- 94/min, rhythmic. AP- 120/60 mm Hg. In stage of medical care?
the II intercostal space on the left auscultation
reveals a continuous rasping systolodiastolic A. Apply a tourniquet on the thigh proximal
murmur, diastolic shock above the pulmonary to the source of bleeding and transport the
artery. Blood and urine are unremarkable. patient to the operating room
What is the most likely diagnosis? B. Immobilize the fracture and transport the
patient to the operating room
A. Patent ductus arteriosus C. Apply a tourniquet on the thigh distal to the
B. Atrial septal defect source of bleeding and transport the patient to
C. Ventricular septal defect the operating room
D. Aortarctia D. Stop the bleeding by a compressive bandage
E. Tetralogy of Fallot and transport the patient to the operating room
E. Transport the patient to the operating room
171. A 6-year-old boy had had a quinsy. 9
days later, there appeared edemata of the 175. The outpatient department of a city
face, extremities and trunk, general health hospital works also as a 60-bed day hospital
condition deteriorated. Urine became turbid. for somatic patients. The department operates
Objectively: expressive edemata, ascites. AP- in a single-shift mode. What specialist selects
100/55 mm Hg, diuresis - 0,2 l of urine per patients for admission to the day hospital?
day. Results of the biochemical blood analysis:
total protein - 50 g/l, cholesterol - 11,28 A. District doctor
mmol/l, urea - 7,15 mmol/l, creatinine - 0,08 B. Resident doctor
mmol/l. Urinalysis results: leukocytes - 3-5 per C. Chief of the outpatient department
HPF, red blood cells are absent. What is the D. Deputy chief of the hospital
provisional diagnosis? E. Chief physician of the hospital

A. Acute glomerulonephritis 176. A 67-year-old female patient with


B. Acute pyelonephritis hypertensive crisis has asthma, cough with
C. Urolithiasis expectoration of frothy pink sputum, moist
D. Acute renal failure rales in the lungs. The patient stays in sitti-
E. Chronic glomerulonephritis ng position, respiratory rate is 40/min, AP-
214/136 mm Hg, heart rate - 102/min. What
172. A 13-year-old boy has had abdominal is the most rational tactics of this patient
pain, bloating, nausea, liquid fatty gray stool management?
with putrid smell for the last 3 years. Palpati-
on reveals epigastric tenderness, as well as A. Intravenous administration of furosemide
tenderness in the Desjardins’ pancreatic point, B. Urgent pneumography
Chauffard’s triangle; there is positive Mayo- C. Bed rest, lying position
Robson’s sign. Failure of exocrine pancreatic D. Intravenous administration of a β -blocker
function has been suspected. What is the most E. Tactics can be determined after ECG and
informative method for evaluating the state of chest radiography
exocrine pancreatic function?
177. A 23-year-old primigravida at 39 weeks
A. Fecal elastase-1 determination gestation has been admitted to the maternity
B. Blood serum trypsin determination ward with irregular contractions. The intensi-
C. Sonography of the pancreas ty of uterine contractions is not changing,
D. Blood and urine amylase determination the intervals between them stay long. Bi-
E. Scatological study manual examination reveals that the cervix
is centered, soft, up to 1,5 cm long. There is no
173. Explosion of a tank with benzene at a cervical dilatation. What diagnosis should be
chemical plant has killed and wounded a large made?
number of people. There are over 50 victims
with burns, mechanical injuries and intoxicati-
on. Specify the main elements of medical care
and evacuation of population in this situation:
Krok 2 Medicine 2014 22

A. Pregnancy I, 39 weeks, preliminary period A. Iron-deficiency anemia


B. Pregnancy I, 39 weeks, labor I, 1 period, the B. Acute leukemia
latent phase C. B12 -deficiency anemia
C. Pregnancy I, 39 weeks, labor I, period 1, the D. Vegetative-vascular dystonia
active phase E. Aplastic anemia
D. Pregnancy I, 39 weeks, birth I, 1 period, the
acceleration phase 182. During the ultrasound study of carotid
E. Pregnancy I, 39 weeks, pathological prelimi- and vertebral arteries a 74-year-old pati-
nary period ent developed a condition manifested by di-
zziness, weakness, nausea, transient loss of
178. A 49-year-old male patient complains of consciousness. Objectively: pale skin, AP-
retrosternal pain, heartburn, weight loss of 8 80/60 mm Hg, Ps- 96/min of poor volume.
kg over the last year, constipation, weakness. ECG shows sinus tachycardia, left ventricular
The patient has been a smoker for 20 years, hypertrophy. Focal neurological symptoms
and has a 10-year history of gastroesophageal were not found. What is the provisional di-
reflux disease. The patient is asthenic, has dry agnosis?
skin. EGD revealed an ulcer in the lower
third of the esophagus and esophageal stri- A. Carotid sinus syncope
cture accompanied by edema, hyperemia and B. Orthostatic syncope
multiple erosions of the mucosa. What study C. Morgagni-Adams-Stokes attack
is required for more accurate diagnosis? D. Complete atrioventricular block
E. Acute cerebrovascular accident
A. Biopsy of the esophageal mucosa
B. X-ray examination of the esophagus 183. On the 10th day postpartum a puerperant
C. Respiratory test for Helicobacter pylori woman complains of pain and heaviness in
D. pH-metry of the esophagus and the stomach the left breast. Body temperature is 38, 8o C ,
E. Fecal occult blood test Ps- 94 bpm. The left breast is edematic, the
supero-external quadrant of skin is hyperemic.
179. A 63-year-old male patient with persi- Fluctuation symptom is absent. The nipples di-
stent atrial fibrillation complains of moderate scharge drops of milk when pressed. What is a
dyspnea. Objectively: peripheral edemata are doctor’s further tactics?
absent, vesicular breathing is present, heart
rate - 72/min, AP- 140/90 mm Hg. What combi- A. Antibiotic therapy, immobilization and
nation of drugs will be most effective for the expression of breast milk
secondary prevention of heart failure? B. Compress to both breasts
C. Inhibition of lactation
A. Beta-blockers, ACE inhibitors D. Physiotherapy
B. Beta-blockers, cardiac glycosides E. Opening of the abscess and drainage of the
C. Cardiac glycosides, diuretics breast
D. Cardiac glycosides, ACE inhibitors
E. Diuretics, beta-blockers 184. During the breast self-exam a 37-year-old
female patient revealed a lump in the lower
180. A 57-year-old male patient had an attack inner quadrant of her left breast. Palpation
of retrosternal pain that lasted more than confirms presence of a mobile well-defined
1,5 hours. Objectively: the patient is inert, neoplasm up to 2 cm large. Peripheral lymph
adynamic, has pale skin, cold extremities, poor nodes are not changed. What is the way of
volume pulse, heart rate - 120/min, AP- 70/40 further management?
mm Hg. ECG shows ST elevation in II, III,
aVF leads. What condition are these changes A. Ultrasound examination of breasts,
typical for? mammography, fine-needle aspiration bi-
opsy
A. Cardiogenic shock B. Anti-inflammatory therapy, physiotherapy
B. Arrhythmogenic shock C. Radical mastectomy
C. Perforated gastric ulcer D. Ultrasound monitoring of genitals during
D. Acute pericarditis the entire course of antiestrogens therapy,
E. Acute pancreatitis systemic enzyme therapy, phytotherapy
E. Case follow-up
181. During an exam, a 22-year-old female
student fainted. She grew up in a family with 185. An 8-year-old child was hospitalized
many children, has a history of frequent acute for fever up to 39, 8o C , inertness, moderate
respiratory infections. Objectively: the patient headache, vomiting. Examination revealed
has pale skin and mucous membranes, split- meningeal symptoms. Lumbar puncture was
end hair, brittle nails. Blood test results: RBC- performed. The obtained fluid had raised
2, 7 · 1012 /l, Hb- 75 g/l, color index - 0,7, WBC- opening pressure, it was transparent, with
3, 2 · 109 /l, platelets - 210 · 109 /l, ESR- 30 mm/h. the cell count of 450 cells per 1 mcL (mai-
Blood serum iron is 6 mmol/l. What is the most nly lymphocytes - 90%), glucose level of 2,6
likely diagnosis? mmol/l. What causative agent might have
caused the disease in the child?
Krok 2 Medicine 2014 23

A. Enterovirus
B. Meningococcus A. Chronic adrenal insufficiency
C. Koch’s bacillus B. Diabetes mellitus
D. Staphylococcus C. Coronary artery disease
E. Pneumococcus D. Chronic pancreatitis
E. Pulmonary tuberculosis
186. A 25-year-old female has a self-detected
tumor in the upper outer quadrant of her ri- 190. A 42-year-old male patient has been
ght breast. On palpation there is a painless, delivered to a hospital in a grave condition
firm, mobile lump up to 2 cm in diameter, with dyspnea, cough with expectoration of
peripheral lymph nodes are not changed. In purulent sputum, fever up to 39, 5o C . The
the upper outer quadrant of the right breast first symptoms appeared 3 weeks ago. Two
ultrasound revealed a massive neoplasm with weeks ago, a local therapist diagnosed him wi-
increased echogenicity sized 21x18 mm. What th acute right-sided pneumonia. Over the last
is the most likely diagnosis? 3 days, the patient’s condition deteriorated:
there was a progress of dyspnea, weakness,
A. Fibroadenoma lack of appetite. Chest radiography confirms a
B. Lactocele rounded shadow in the lower lobe of the right
C. Diffuse mastopathy lung with a horizontal fluid level, the right si-
D. Mammary cancer nus is not clearly visualized. What is the most
E. Mastitis likely diagnosis?
187. A 49-year-old female patient complai- A. Abscess of the right lung
ns of itching, burning in the external geni- B. Acute pleuropneumonia
tals, frequent urination. The symptoms have C. Right pulmonary empyema
been present for the last 7 months. The pati- D. Atelectasis of the right lung
ent has irregular menstruation, once every E. Pleural effusion
3-4 months. Over the last two years she has
had hot flashes, sweating, sleep disturbance. 191. An 11-year-old boy complains of general
Examination revealed no pathological weakness, fever up to 38, 2o C , pain and swelli-
changes of the internal reproductive organs. ng of the knee joints, feeling of irregular
Complete blood count and urinalysis showed heartbeat. 3 weeks ago, the child had quinsy.
no pathological changes. Vaginal smear Knee joints are swollen, the overlying skin
contained 20-25 leukocytes per HPF, mixed and skin of the knee region is reddened, local
flora. What is the most likely diagnosis? temperature is increased, movements are li-
mited. Heart sounds are muffled, extrasystole
A. Menopausal syndrome is present, auscultation reveals apical systolic
B. Cystitis murmur that is not conducted to the left ingui-
C. Trichomonas colpitis nal region. ESR is 38 mm/h. CRP is 2+, anti-
D. Vulvitis streptolysin O titre - 400. What is the most
E. Bacterial vaginosis likely diagnosis?
188. A 45-year-old male patient complains of A. Acute rheumatic fever
acute pain in his right side irradiating to the B. Vegetative dysfunction
right thigh and crotch. The patient claims also C. Non-rheumatic carditis
to have frequent urination with urine which D. Juvenile rheumatoid arthritis
resembles a meat slops. The patient has no E. Reactive arthritis
previous history of this condition. There is
costovertebral angle tenderness on the right 192. A 28-year-old male patient complains
(positive Pasternatsky’s symptom). What is of sour regurgitation, cough and heartburn
the most likely diagnosis? that occurs every day after having meals,
when bending forward or lying down. These
A. Urolithiasis problems have been observed for 4 years.
B. Acute appendicitis Objective status and laboratory values are
C. Acute pyelonephritis normal. FEGDS revealed endoesophagitis.
D. Acute cholecystitis. Renal colic What is the leading factor in the development
E. Acute pancreatitis of this disease?
189. After a holiday in the Crimea, a 49- A. Failure of the lower esophageal sphincter
year-old male patient with a history of lung B. Hypersecretion of hydrochloric acid
tuberculosis felt increased weakness, peri- C. Duodeno-gastric reflux
odic dizziness, easing bowel movements with D. Hypergastrinemia
abdominal pain, the need for additional salti- E. Helicobacter pylori infection
ng his meals. The patient has noted that his
condition improves after some sweet tea and 193. On admission a 35-year-old female
validol taken sublingually. Objectively: there reports acute abdominal pain, fever up to
is an intense darkening of skin, AP- 70/50 mm 38, 8o C , mucopurulent discharges. The pati-
Hg, glycemia is 3,0 mmol/l. What is the possi- ent is nulliparous, has a history of 2 artificial
ble cause of health deterioration: abortions. The patient is unmarried, has sexual
Krok 2 Medicine 2014 24

contacts. Gynecological examination reveals scan showed a destruction area in the center
no uterus changes. Appendages are enlarged, of the shadow. Sputum analysis revealed
bilaterally painful. There is profuse purulent MTB. The patient was diagnosed with focal
vaginal discharge. What study is required to tuberculosis. What phases of tuberculosis are
confirm the diagnosis? the identified changes typical for?
A. Bacteriologic and bacteriascopic studies A. Infiltration and disintegration
B. Hysteroscopy B. Infiltration and dissemination
C. Curettage of uterine cavity C. Resorption and scarring
D. Vaginoscopy D. Disintegration and dissemination
E. Laparoscopy E. Calcification and resorption
194. A 20-year-old female consulted a 198. A 43-year-old female complains of si-
gynecologist about not having menstrual gnificant weakness, sore throat, occurrence
period for 7 months. History abstracts: of multiple unexplained bruises on her skin.
early childhood infections and frequent These symptoms have been present for a
tonsillitis, menarche since 13 years, regular week, the disease is associated with quinsy
monthly menstrual cycle of 28 days, painless which she had some time before. Objectively:
menstruation lasts 5-6 days. 7 months ago the body temperature - 38, 9o C , respiratory rate -
patient had an emotional stress. Gynecologi- 24/min, Ps- 110/min, AP- 100/65 mm Hg. The
cal examination revealed no alterations in the patient has pale skin, petechial rash on the
uterus. What is the most likely diagnosis? extremities, enlarged lymph nodes. Blood test
results: Hb- 80 g/l, RBC- 2, 2 · 1012 /l; WBC-
A. Secondary amenorrhea
B. Primary amenorrhea 3, 5 · 109 /l; blasts - 52%; eosinophils - 2%;
C. Algomenorrhea stab neutrophils - 3%; segmented neutrophils
D. Spanomenorrhea - 19%; lymphocytes - 13%; monocytes - 1%;
E. Cryptomenorrhea platelets - 35 · 109 /l. ESR - 47 mm/h. What test
is required to specify the diagnosis?
195. A 48-year-old female has been admi-
tted to the gynecology department for pain in A. Immunophenotyping
the lower right abdomen and low back pain, B. Protein electrophoresis
constipations. Bimanual examination findings: C. Lymph node biopsy
the uterus is immobile, the size of a 10-week D. Determination of anti-platelet antibody
pregnancy, has uneven surface. Aspirate from titer
the uterine cavity contains atypical cells. What E. Cytogenetic study
diagnosis can be made?
199. A 47-year-old male patient complains of
A. Hysterocarcinoma compressive chest pain that occurs both at rest
B. Cervical cancer and during light physical activity; irregular
C. Metrofibroma heartbeat. These problems arose 3 months
D. Colon cancer ago. The patient’s brother died suddenly at the
E. Chorionepithelioma age of 30. Objectively: Ps- 84/min, arrhythmic,
AP- 130/80 mm Hg. ECG confirms signs of
196. A 27-year-old male patient consulted left ventricular hypertrophy, abnormal Q-
a doctor about pain in the lower third waves in V 4 − V 6 leads. EchoCG reveals that
of the thigh with weight bearing activiti- interventricular septum is 1,7 cm, left ventri-
es and unloaded. Two years ago, the pati- cular wall thickness is 1,2 cm. What is the most
ent underwent treatment in the casualty likely diagnosis?
depatment for the open fracture of the lower
third of femur. The fracture healed slowly, the A. Hypertrophic cardiomyopathy
healing process was accompanied by prulent B. Neurocirculatory asthenia
inflammation. Objectively: edema of the lower C. Exertional angina
third of the thigh, elevated local temperature. D. Myocarditis
Radiograph shows signs of destruction and E. Pericarditis
sequestration. What is the most likely di-
agnosis? 200. Within the structure of the region’s
population the share of persons aged 0 to 14
A. Chronic post-traumatic osteomyelitis years is 25%, the share of persons aged 50
B. Osteosarcoma years and older is 30%. What concept most
C. Tuberculosis of femur accurately describes this demographic situati-
D. Hematogenous osteomyelitis on?
E. Multiple myeloma
A. Regressive type of population age structure
197. A 30-year-old male patient had been B. Progressive type of population age structure
admitted to the TB hospital because of the C. Cohort reproduction
following changes detected by fluorography: D. Stationary type of population age structure
an ill-defined shadow of low intensity up to 1 E. Immigration of population
cm in diameter in the S1 of the right lung. CT
Крок 2 Medicine (англомовний варiант, iноземнi студенти) 2015 рiк 1

1. 13 months after the first labor a 24- A. Droperidolum of 0,25% - 2,0 ml


year-old patient complained of amenorrhea. B. Dibazolum (Bendazole hydrochloride) of
Pregnancy ended in Caesarian section because 1% - 6,0 ml
of premature detachment of normally positi- C. Papaverine hydrochloride of 2% - 4,0 ml
oned placenta which resulted in blood loss at D. Hexenalum of 1% - 2,0 ml
the rate of 2000 ml due to disturbance of blood E. Pentaminum of 5% - 4,0 ml
clotting. Choose the most suitable investigati-
on: 5. A patient with high temperature came to
a first-aid post in the evening. The fact of
A. Estimation of gonadotropin rate temporary disability was established. Indicate
B. US of small pelvis the measure to be taken in this case:
C. Progesteron assay
D. Computer tomography of head A. The night duty doctor should issue a medical
E. Estimation of testosteron rate in blood serum certificate, which will be subsequently used for
issuing a sick list from the date of the previous
2. A 24-year-old primipara was hospitalised day
with complaints of discharge of the amniotic B. The sick list for 1 day should be issued
waters. The uterus is tonic on palpation. The C. The sick list for up to 3 days should be issued
position of the fetus is longitudinal, it is pressed D. The sick list for 3 days should be issued
with the head to pelvic outlet. Palpitation of E. No document should be issued
the fetus is rhythmical, 140 bpm, auscultated
on the left below the navel. Internal examinati- 6. A 10-year-old boy complains of pain in his
on: cervix of the uterus is 2,5 cm long, dense, left eye and strong photophobia after he has
the external opening is closed, light amniotic injured his left eye with a pencil at school. Left
waters are discharged. Point out the correct eye examination revealed: blepharospasm, ci-
component of the diagnosis: liary and conjunctival congestion, cornea is
transparent, other parts of eyeball have no
A. Antenatal discharge of the amniotic waters changes. Visus 0,9. Right eye is healthy, Visus
B. Early discharge of the amniotic waters 1,0. What additional method would you choose
C. The beginning of the 1st stage of labour first of all?
D. The end of the 1st stage of labour
E. Pathological preterm labour A. Staining test with 1% fluorescein
B. X-ray examination of orbit
3. A patient was delivered to a surgical C. Tonometria
department after a road accident with a closed D. Gonioscopia
trauma of chest and right-sided rib fracture. E. Cornea sensation-test
The patient was diagnosed with right-sided
pneumothorax; it is indicated to perform drai- 7. An infant was born with body mass 3 kg and
nage of pleural cavity. Pleural puncture should body length 50 cm. Now he is 3 years old. His
be made: brother is 7 years old, suffers from rheumatic
fever. Mother asked a doctor for a cardiac
A. In the 2nd intercostal space along the middle check up of the 3-year-old son. Where is the
clavicular line left relative heart border located?
B. In the 6th intercostal space along the posteri-
or axillary line A. 1 cm left from the left medioclavicular line
C. In the 7th intercostal space along the scapular B. 1 cm right from the left medioclavicular line
line C. Along the left medioclavicular line
D. In the projection of pleural sinus D. 1 cm left from the left parasternal line
E. In the point of the greatest dullness on E. 1 cm right from the left parasternal line
percussion 8. A baby was born at 36 weeks of gestation.
4. A primagravida with pregnancy of 37-38 Delivery was normal, by natural way. The baby
weeks complains of headache, nausea, pain in has a large cephalohematoma. The results of
epigastrium. Objectively: the skin is acyanotic. blood count are: Hb- 120g/l, Er- 3, 5 · 1012 /l,
Face is hydropic, there is short fibrillar twi- total serum bilirubin - 123 mmol/l, direct bili-
tching of blepharons, muscles of the face and rubin - 11 mmol/l, indirect - 112 mmol/l. What
the inferior extremities. The stare is fixed. BP are the causes of hyperbilirubinemia in this
- 200/110 mm Hg; sphygmus is of 92 bpm, case?
intense. Respiration rate is 32/min. Heart acti-
vity is rhythmical. Appreciable edemas of A. Erythrocyte hemolysis
the inferior extremities are present. Urine is B. Intravascular hemolysis
cloudy. What medication should be admini- C. Disturbance of the conjugative function of
stered? liver
D. Bile condensing
E. Mechanical obstruction of the bile outflow
9. A patient has been hospitalised. The onset of
the disease was gradual: nausea, vomiting, dark
urine, аcholic stool, yellowness of the skin and
scleras. The liver is enlarged by 3 cm. Jaundice
Крок 2 Medicine (англомовний варiант, iноземнi студенти) 2015 рiк 2

developed on the 14th day of the disease. The 14. A 13-year-old girl complains of periodic
liver diminished in size. What complication of prickly pain in the heart region. Percussi-
viral hepatitis caused deterioration of the pati- on revealed no changes of cardiac borders.
ent’s condition? Auscultation revealed arrhythmic enhanced
heart sounds, extrasystole at the 20-25 cardi-
A. Hepatic encephlopathy ac impulse. ECG showed the sinus rhythm,
B. Meningitis impaired repolarization, single supraventri-
C. Relapse of viral hepatitis cular extrasystoles at rest. What is the most
D. Cholangitis likely diagnosis?
E. Infectious-toxic shock
A. Somatoform autonomic dysfunction
10. An 18-year-old patient was admitted B. Rheumatism
to a hospital with complaints of headache, C. Nonrheumatic carditis
weakness, high fever, sore throat. Objectively: D. Myocardial degeneration
enlargement of all groups of lymph nodes was E. Intoxication syndrome
revealed. The liver is enlarged by 3 cm, spleen
- by 1 cm. In blood: leukocytosis, atypical 15. A 24-year-old patient visited a doctor
lymphocytes - 15%. What is the most probable complaining of enlargement of his submaxi-
diagnosis? llary lymph nodes. Objectively: submaxillary,
axillary and inguinal lymph nodes are enlarged.
A. Infectious mononucleosis Chest X-ray shows: enlarged lymph nodes
B. Acute lymphoid leukosis of mediastinum. Blood test: erythrocytes -
C. Diphtheria 3, 4 · 1012 /l, Hb- 100 g/l, blood colour index
D. Angina
E. Adenoviral infection - 0,88, platelets - 190 · 109 /l, leucocytes -
7, 5 · 109 /l, eosinophiles - 8%, band neutrophi-
11. A 60-year-old woman, mother of 6 children, les - 2%, segmented neutrophiles - 67%,
developed a sudden onset of upper abdomi- lymphocytes - 23%, ESR - 22 mm/h. What
nal pain radiating to the back, accompani- test must be prescribed to verify the cause of
ed by nausea, vomiting, fever and chills. lymphadenopathy?
Subsequently, she noticed yellow discolorati-
on of her sclera and skin. On physical exami- A. Open biopsy of lymph nodes
nation the patient was found to be febrile with B. Ultrasonography of abdominal cavity
temperature of 38, 9o C , along with right upper C. Mediastinum tomography
quadrant tenderness. The most likely diagnosis D. Puncture biopsy of lymph nodes
is: E. Sternal puncture

A. Choledocholithiasis 16. A 60-year-old woman has been sufferi-


B. Benign biliary stricture ng from arterial hypertension for 15 years.
C. Malignant biliary stricture After recurrent stroke she started complaini-
D. Carcinoma of the head of the pancreas ng about unmotivated bad mood, problems
E. Choledochal cyst with attention concentration; she forgets to
close the entrance door, cannot recall events
12. What juice is recommended to be included of the past day. Computer tomography shows
in a complex drug and diet therapy for patients areas of postinfarction changes in the cortical
suffering from gastric or duodenal ulcer and postfrontal areas. What is the most probable
high acidity of gastric juice to speed up ulcer diagnosis?
healing?
A. Vascular dementia
A. Potato, potato-carrot B. Alzheimer’s disease
B. Apple, apple-birch C. Huntington’s disease
C. Pumpkin D. Pick’s disease
D. Cabbage, cabbage-carrot E. Dissociative amnesia
E. Celery, parsley
17. Clinic of a research instutute for occupati-
13. A 40-year-old woman has been hospitalized onal diseases examined a worker who works at
for attacks of asphyxia, cough with phlegm. She a concentration plant and diagnosed him with
has a 4-year history of the disease. The first chronic dust bronchitis. The case is investigated
attack of asphyxia occurred during her stay in by a commission including the representatives
the countryside. Further attacks occurred whi- of: the plant, medical unit, territorial sanitation
le cleaning the room. After 3 days of inpatient center, department of Social Insurance Fund,
treatment the patient’s condition has signifi- trade union. According to the "regulation on
cantly improved. What is the most likely eti- investigation of. . . ", the commission should be
ological factor? headed by the representative of the following
authority:
A. Household allergens
B. Pollen
C. Infectious
D. Chemicals
E. Psychogenic
Крок 2 Medicine (англомовний варiант, iноземнi студенти) 2015 рiк 3

A. Territorial sanitation center mes joined by hemoptysis and pain in the ri-
B. Plant ght side of chest. Breathing is vesicular. X-ray
C. Social Insurance Fund shows darkening and sharp decrease in size of
D. Trade union the lower lobe distinctly visible on the X-ray
E. Medical unit image as a streak 2-3 cm wide situated at the
angle from lung root to the frontal costodi-
18. 4 days after a patient received a gunshot aphragmatic recess. The most likely diagnosis
wound of the middle third of the thigh soft ti- is:
ssues his condition suddenly began deteriorati-
ng. There are complaints of bursting pain in the A. Peripheral lung cancer
wound; pain increases during the last 12 hours. B. Bronchiectasis
Edema of skin and hypodermic tissue quickly C. Pneumonia
grows. Body temperature is 38, 2o C , heart rate D. Middle lobe syndrome
is 102/min. The wound edges gape, are dull in E. Interlobular pleurisy
color; the muscles, viable as of day before, now
protrude into the wound, look boiled, are dull 23. A 52-year-old patient, who has been sufferi-
in colour, have dirty-grey coating and fall apart ng from angina pectoris for 2 weeks, has
when being held with forceps. What infection more and more frequent pain attacks in the
has developed in the wound? area behind his sternum and his need for ni-
troglycerine increased. Objectively: the condi-
A. Anaerobic tion is of moderate severity. Skin is pale. Heart
B. Aerobic gram-negative sounds are weakened, rhythmic. Heart rate is
C. Putrid 84 per minute. ECG shows no signs of focal
D. Aerobic gram-positive myocardial damage. What is the most likely di-
E. Diphtheria of wound agnosis?
19. Bacterial analysis of air in a living space in A. Progressive angina pectoris
winter period by means of Krotov’s apparatus B. First-time angina pectoris
revealed that total number of microorganisms C. Stable FC II angina pectoris
in 1m3 of air was 7200. What is the allowed D. Variant angina pectoris
number of microorganisms for the air to be E. Acute cardiac infarction
characterized as "pure"?
24. An 8-year-old boy during preventive exami-
A. Up to 4500 nation was determined to have changes in
B. Up to 2500 his spine curvature in frontal plane: the ri-
C. Up to 3500 ght shoulder is lowered and flat, scapulae
D. Up to 5500 angles are of different height due to the ri-
E. Up to 7500 ght scapula being shifted down. Waist triangles
are pronounced on the both sides; longitudinal
20. Maximum permissible concentration of muscles of the back form muscle cushion on
carbon dioxide in the air is considered to be the left. What type of posture is detected in the
a sanitary index of air purity in a classroom. child?
What concentration of carbon dioxide in the
air is accepted as a permissible maximum? A. Scoliotic
B. Kyphotic
A. 0,1% C. Lordotic
B. 0,05% D. Stooping
C. 0,15% E. Corrected
D. 0,2%
E. 0,3% 25. A patient with cardiac infarction is on sick
leave with his medical certificate being conti-
21. When examining the parameters of desk nuously extended for 4 months according to a
natural lighting in a school class it was determi- standard procedure. The disease is progressi-
ned that: light angle is 25o , window angle is ng. The issue is put forward to refer the pati-
3o , window-to-floor area ratio is 1:4, dayli- ent to the industrial injury assessment board.
ght factor is 0,5%, and window-head-to-room- Who among the medical staff of medical and
depth ratio is 2. What parameter does not preventive treatment facility is authorized to
correspond to hygienic norms? do this?
A. Daylight factor A. Medical expert committee
B. Window angle B. Attending medical doctor
C. Window-to-floor area ratio C. Head doctor of hospital department
D. Window-head-to-room-depth ratio D. Deputy head doctor for assessment of
E. Light angle temporary disability
E. Head doctor
22. A 29-year-old patient works as a motor
mechanic. Anamnesis shows frequent exposure 26. A 9-year-old boy has been suffering from
to cold, exacerbation of chronic bronchitis bronchoectasis since he was 3. Exacerbations
attended by cough with relativly small amount occur quite often, 3-4 times a year. Conservati-
of mucopurulent sputum, subfebrility, someti- ve therapy results in short periods of remi-
Крок 2 Medicine (англомовний варiант, iноземнi студенти) 2015 рiк 4

ssion. The disease is progressing, the child 31. A 46-year-old patient once took part in eli-
has physical retardation. The child’s skin is mination of breakdown at an atomic power
pale, acrocyanotic, he has "watch glass"nail plant. Currently he is being treated at an in-
deformation. Bronchography revealed saccular patient hospital unit. He was diagnosed with
bronchiectases of the lower lobe of his right progressing somatoform autonomic dysfuncti-
lung. What is the further treatment tactics? on. This disease relates to the following group
of ionizing radiation effects:
A. Surgical treatment
B. Further conservative therapy A. Somato-stochastic
C. Physiotherapeutic treatment B. Somatic
D. Sanatorium-and-spa treatment C. Genetic
E. Tempering of the child’s organism D. Hormesis
E. Heterosis
27. A 29-year-old female patient complains of
dyspnea, heaviness and chest pain on the ri- 32. A patient complains of fatigue, lack of
ght, body temperature rise up to 37, 2o C . The appetite, pain and burning sensation in the
disease is associated with a chest trauma recei- tongue, numbness of the distal limbs, diarrhea.
ved 4 days ago. Objectively: skin is pale and Objectively: pale skin with lemon-yellow tint,
moist. Heart rate is 90 bpm, regular. Palpation face puffiness, brown pigmentation in the form
reveals a dull sound on the right, auscultati- of a "butterfly", bright red areas on the tongue.
on reveals significantly weakened vesicular The liver is 3 cm below the costal margin, soft.
breathing. In blood: RBCs - 2, 8 · 1012 /l, colour Blood count: RBCs - 1, 5 · 1012 /l, colour index -
index - 0,9, Hb- 100 g/l, WBCs - 8, 0 · 109 /l, ESR 1,2, WBCs - 3, 8·109 /l, thrombocytes - 180·109 /l,
- 17 mm/h. What results of diagnostic puncture eosinophiles - 0%, stab neutrophiles - 1%,
of the pleural cavity can be expected? segmented neutrophiles - 58%, lymphocytes -
38% monocytes - 3%, RBC macrocytosis. ESR
A. Haemorrhagic punctate - 28 mm/h. What diagnosis are these presentati-
B. Chylous liquid ons typical for?
C. Exudate
D. Transudate A. B12 -deficiency anemia
E. Purulent punctate B. Iron deficiency anemia
C. Aplastic anemia
28. A 54-year-old man had been drowning at D. Acute erythromyelosis
sea, when he was found and evacuated to the E. Chronic adrenal failure
shore. Objectively: unconscious, pale face, no
breathing can be auscultated, thready pulse. 33. A 25-year-old female patient complains of
Resuscitation measures allowed to save the marked weakness, sleepiness, blackouts, di-
man. What complication can develop in him zziness, taste disorder. The patient has a hi-
in the nearest future? story of menorrhagia. Objectively: the pati-
ent has marked weakness, pale skin, cracks
A. Pulmonary edema in the corners of her mouth, peeling nails,
B. Respiratory arrest systolic apical murmur. Blood test results: RBC
C. Encephalopathy - 3, 4 · 1012 /l, Hb- 70 g/l, colour index - 0,75,
D. Cardiac arrest
E. Bronchial spasm platelets - 140 · 109 /l, WBC- 6, 2 · 109 /l. What is
the most likely diagnosis?
29. A 7-year-old child complains of cramping
pain occuring after mental exertion, cold dri- A. Chronic posthemorrhagic anemia
nks and eating ice-cream. Instrumental exami- B. Acute leukemia
nation allowed to diagnose biliary dyskinesia of C. Acute posthemorrhagic anemia
hypertensive type. What group of drugs should D. B12 -deficiency anemia
be prescribed for treatment? E. Werlhof’s disease

A. Antispasmodics and choleretics 34. A 39-year-old patient complains of morning


B. Choleretics and cholekinetics headache, appetite loss, nausea, morning vomi-
C. Sedatives and cholekinetics ting, periodic nasal haemorrhages. The patient
D. Antioxydants had acute glomerulonephritis at the age of 15.
E. Antibiotics Examination revealed rise of arterial pressure
up to 220/130 mm Hg, skin haemorrhages on
30. Caries morbidity rate is 89% among resi- his arms and legs, pallor of skin and mucous
dents of a community. It is determined that membranes. What biochemical parameter is
fluorine content in water is 0,1 mg/l. What the most important for making diagnosis in
preventive measures should be taken? this case?

A. Water fluorination A. Blood creatinine


B. Tooth brushing B. Blood bilirubin
C. Fluorine inhalations C. Blood sodium
D. Sealant application D. Uric acid
E. Introduce more vegetables to the diet E. Fibrinogen
Крок 2 Medicine (англомовний варiант, iноземнi студенти) 2015 рiк 5

35. A child was taken to a hospital with focal treatment is going to last for another 1-2
changes in the skin folds. The child was anxi- months. Who is authorised to extend the
ous during examination, examination revealed duration of medical certificate for this patient?
dry skin with solitary papulous elements and
ill-defined lichenification zones. Skin erupti- A. Medical advisory commission after medico-
on was accompanied by strong itch. The child social expert commission examination
usually feels better in summer, his condition B. Medical superintendent
is getting worse in winter. The child has been C. Medical advisory commission after inpatient
on bottle feeding since he was 2 months old. treatment
He has a history of exudative diathesis. His D. District doctor by agreement with a
grandmother on his mother’s side has bronchi- department chief
al asthma. What is the most likely diagnosis? E. Medico-social expert commission

A. Atopic dermatitis 40. A 30-year-old woman complains of pain


B. Contact dermatitis in the heart area ("aching, piercing pain")
C. Seborrheal eczema that arises primarily in the morning hours in
D. Strophulus autumn and spring. Pain irradiates into the
E. Urticaria neck, back, abdomen and is attended by rapid
heart rate and low vital tonus. This conditi-
36. An electro-gas welding operator worki- on occurs independently from physical exerti-
ng at a machine workshop performs welding on. In the evening her condition improves.
and cutting of metal, which is accompanied by Somatic and neurologic state and ECG have no
intense UV-radiation. His welding station is pathologies. What pathology is likely to result
equipped with effective mechanical ventilati- in such clinical presentation?
on. What occupational disease is most likely to
develop in the electro-gas welding operator? A. Somatized depression
B. Rest angina pectoris
A. Photoelectric ophthalmia C. Pseudoneurotic schizophrenia
B. Heatstroke D. Somatoform autonomic dysfunction
C. Vegetative-vascular dystonia E. Hypochondriacal depression
D. Chronic overheating
E. Pneumoconiosis 41. A 59-year-old female patient attended
a maternity welfare clinic with complains
37. 2 weeks after recovering from tonsillitis an of bloody discharge from the genital tracts.
8-year-old boy developed edemas of face and Postmenopause is 12 years. Vaginal exami-
lower limbs. Objectively: the patient is in grave nation revealed that external genital organs
condition, BP - 120/80 mm Hg. Urine is of dark had signs of age involution, uterus cervix
brown colour. Oliguria is present. On urine was not erosive, small amount of bloody di-
analysis: specific gravity - 1,015, protein - 1,2 scharge came from the cervical canal. Uterus
g/l, RBCs are leached and cover the whole vi- is of normal size, uterine appendages are
sion field, granular casts - 1-2 in the vision field, unpalpable. Fornices were deep and painless.
salts are represented by urates (large quantity). What method should be applied for the di-
What is the most likely diagnosis? agnosis specification?
A. Acute glomerulonephritis with nephritic A. Separated diagnosic curretage
syndrome B. Laparoscopy
B. Acute glomerulonephritis with nephrotic C. Puncture of abdominal cavity through
syndrome posterior vaginal fornix
C. Acute glomerulonephritis with nephrotic D. Extensive colposcopy
syndrome, hematuria and hypertension E. Culdoscopy
D. Acute glomerulonephritis with isolated
urinary syndrome 42. A 41-year-old patient, a hunter, complai-
E. Nephrolithiasis ns of heaviness in the right subcostal area.
No other complaints recorded. Anamnesis
38. A 48-year-old patient has the followi- states causeless urticaria and skin itching that
ng symptoms: diffuse enlargement of thyroid occurred 1 year ago. Objectively: liver margin
gland, exophthalmus, weight loss up to 4 kg wi- is rounded, painless, and can be palpated 3
thin 2 months, excessive sweating. Objectively: cm below the costal arch. No other pathology
heart rate 105/min, BP - 180/70 mm Hg. Stool is was detected during physical examination of
normal. What therapy is advisable in the given the patient. Body temperature is normal. X-ray
case? reveals a hemispherical protrusion in the right
cupula of the diaphragm. What disease can be
A. Mercazolil (Thiamazole) suspected in the given case?
B. Potassium iodide
C. Propranolol A. Hydatid disease of liver
D. Iodomarin B. Liver abscess
E. Thyroxin C. Hepatocellular carcinoma
D. Metastatic tumor
39. A patient is on sick leave for 4 months E. Subdiaphragmatic abscess
continuously from the date of injury. The
Крок 2 Medicine (англомовний варiант, iноземнi студенти) 2015 рiк 6

43. A 14-year-old girl has been presenting wi- Objectively: focal neurological symptoms
th irritability and tearfulness for about a year. were not found. Pronounced meningeal
A year ago she was also found to have diffuse symptoms were revealed. BP - 120/80 mm
enlargement of the thyroid gland (II grade). Hg. According to clinical and liquorologi-
This condition was regarded as a pubertal cal findings the patient was diagnosed wi-
manifestation, the girl did not undergo any th subarachnoid haemorrhage. After admi-
treatment. The girl’s irritability gradually gave nistration of dehydrants the patient’s condi-
place to a complete apathy. The girl got tion somewhat improved. What is the main
puffy face, soft tissues pastosity, bradycardia, component of further emergency care?
constipations. Skin pallor and gland density
progressed, the skin got a waxen hue. What di- A. Coagulants
sease may be assumed? B. Anticoagulants
C. Antiaggregants
A. Autoimmune thyroiditis D. Fibrinolytics
B. Diffuse toxic goiter E. Corticosteroids
C. Thyroid carcinoma
D. Subacute thyroiditis 48. A 42-year-old man has undergone gastric
E. Juvenile basophilism analysis. Free hydrochloric acid is absent
at all phases. On endoscopy: pallor, gastric
44. A 26-year-old woman, who delivered a child mucosa loss, gastric folds are smoothed out.
7 months ago, has been suffering from nausea, On microscopy: glands atrophy with intestinal
morning vomiting, sleepiness for the last 2 metaplasia. What disease can be characterized
weeks. She breastfeeds the child, menstruation by such presentation?
is absent. She has not applied any contracepti-
ves. What method should be applied in order A. Chronic gastritis type A
to specify her diagnosis? B. Chronic gastritis type B
C. Chronic gastritis type C
A. Ultrasonic examination D. Menetrier’s disease
B. X-ray of small pelvis E. Stomach cancer
C. Palpation of mammary glands and pressing-
out of colostrum 49. A 24-year-old woman, teacher by professi-
D. Bimanual vaginal examination on, complains of dizziness and heart pain
E. Speculum examination irradiating to the left nipple. Pain is not associ-
ated with physical activity and cannot be reli-
45. Over a current year among workers of an eved by nitroglycerin, it abates after taking
institution 10% have not been ill a single time, Valocordin and lasts for an hour or more. The
30% have been ill once, 15% - twice, 5% - 4 patient has a nearly 2-year history of this di-
times, the rest - 5 and more times. What is the sease. Objectively: Heart rate - 76 bpm. BP
percentage of workers belonging to the I health - 110/70 mm Hg. Heart borders are normal,
group? heart sounds are clear. The ECG shows respi-
ratory arrhythmia. X-ray of the cervicothoracic
A. 55% spine shows no pathology. Lungs, abdomen are
B. 10% unremarkable. What changes in blood formula
C. 40% can be expected?
D. 60%
E. 22% A. No changes
B. Leukocytosis
46. A 34-year-old patient after vacation in C. Thrombocytopenia
Crimea has developed severe pain in her D. Leukemic hiatus
elbow joints, dyspnea and weakness. Body E. Increased ESR
temperature is 37, 6o C , skin pallor and
erythema on the cheeks and bridge of nose 50. Chief physician of a polyclinic tasked a di-
are observed, lip mucosa is ulcerated. The joi- strict doctor with determining the pathologi-
nts are not visibly deformed, movement of the cal prevalence of disease N in his district.
right elbow joint is restricted. Pleura friction What document allows to estimate the disease
sound is detected in the lungs on the right side prevalence in the population of a medical di-
below the angle of scapula. Heart sounds are strict?
dull, tachycardia, gallop rhythm, heart rate -
114/min, BP - 100/60 mm Hg. The most likely A. Prophylactic examinations register
diagnosis is: B. Statistic coupons (+)
C. Statistic coupons (-)
A. Systemic lupus erythematosus D. Statistic coupons (+) and (-)
B. Rheumatic carditis E. Vouchers for medical appointments
C. Rheumatoid arthritis
D. Infectious allergic myocarditis 51. A 32-year-old woman complains of di-
E. Dry pleurisy zziness, headache, palpitation, tremor. For
the last several months she has been under
47. Several hours before, a 28-year-old pati- outpatient monitoring for increased arterial
ent suddenly developed acute headache and pressure. Recently such attacks have become
repeated vomiting, then lost consciousness. more frequent and severe. Objectively: skin
Крок 2 Medicine (англомовний варiант, iноземнi студенти) 2015 рiк 7

is covered with clammy sweat, tremor of the A. Coliphage number


extremities is present. Heart rate - 110/min, BP B. Colibacillus index
- 220/140 mm Hg. Heart sounds are weakened. C. Oxidability
In blood: WBCs - 9, 8 · 109 /l, ESR - 22 mm/h. D. Non-icteric leptospirosis agent
Blood glucose - 9,8 millimole/l. What disease is E. Fecal coliform bacteria index
the most likely cause of this crisis? 56. A 25-year-old patient was delivered to
A. Pheochromocytoma an infectious diseases unit on the 3rd day
B. Essential hypertension of illness with complaints of headache, pain
C. Preeclampsia in lumbar spine and gastrocnemius muscles,
D. Primary hyperaldosteronism high fever, chill. Objectively: condition of
E. Diabetic glomerulosclerosis moderate severity. Scleras are icteric. Pharynx
is hyperemic. Tongue is dry with dry brown
52. A 45-year-old patient complains of pain coating. Abdomen is distended. Liver is
in the epigastric region, left subcostal area, enlarged by 2 cm. Spleen is not enlarged.
abdominal distension, diarrhea, loss of wei- Palpation of muscles, especially gastrocnemi-
ght. He has been suffering from this condition us muscles, is painful. Urine is dark in colour.
for 5 years. Objectively: tongue is moist with Stool is normal in colour. The most likely di-
white coating near the root; deep palpation of agnosis is:
abdomen reveals slight pain in the epigastric
region and Мауо-Robson’s point. Liver is pai- A. Leptospirosis
nless and protrudes 1 cm from the costal arch. B. Viral hepatitis type A
Spleen cannot be palpated. What disease can C. Malaria
be primarily suspected? D. Infectious mononucleosis
E. Yersiniosis
A. Chronic pancreatitis
B. Atrophic gastritis 57. A patient suffering from acute
C. Peptic stomach ulcer posttraumatic pain received an injection of
D. Chronic cholecystitis morphine that brought him a significant relief.
E. Chronic enteritis Which of the following mechanisms of action
provided for antishock effect of morphine in
53. A 58-year-old woman complains of this patient?
spontaneous bruises, weakness, bleeding gums,
dizziness. Objectively: the mucous membranes A. Stimulation of opiate receptors
and skin are pale with numerous hemorrhages B. Block of central cholinergic receptors
of various time of origin. Lymph nodes are not C. Stimulation of benzodiazepine receptors
enlarged. Heart rate - 100/min, BP- 110/70 D. Inhibition of dopamine mediation
mm Hg. There are no changes of internal E. Intensification of GABA-ergic reactions
organs. Blood test results: RBC - 3,0·1012 /l, 58. A 9-month-old child presents with fever,
Нb - 92 g/l, colour index - 0,9, anisocytosis, poi- cough, dyspnea. The symptoms appeared 5
kilocytosis, WBC - 10·109 /l, eosinophiles - 2%, days ago after a contact with a person havi-
stab neutrophiles - 12%, segmented neutrophi- ng URTI. Objectively: the child is in grave
les - 68%, lymphocytes - 11%, monocytes - 7%, condition. Temperature of 38o C , cyanosis of
ESR - 12 mm/h. What laboratory test should be nasolabial triangle is present. Respiration rate
determined additionally to make a diagnosis? - 54/min, nasal flaring while breathing. There
was percussion dullness on the right below the
A. Platelets scapula angle, and tympanic sound over the
B. Reticulocytes rest of lungs. Auscultation revealed bilateral fi-
C. Clotting time ne moist rales (crackles) predominating on the
D. Osmotic resistance of erythrocytes right. What is the most likely diagnosis?
E. Fibrinogen
A. Acute pneumonia
54. After examination a 46-year-old patient was B. URTI
diagnosed with left breast cancer T2N2M0, cli- C. Acute laryngotracheitis
nical group II-a. What will be the treatment D. Acute bronchitis
plan for this patient? E. Acute bronchiolitis
A. Radiation therapy + operation + 59. A 50-year-old patient complains about havi-
chemotherapy ng pain attacks in the right subcostal area for
B. Operation only about a year. The pain arises mainly after
C. Operation + radiation therapy taking fatty food. Over the last week the
D. Radiation therapy only attacks occurred daily and became more pai-
E. Chemotherapy only nful. On the 3rd day of hospitalization the pati-
55. In river-side urban community there was an ent presented with icteritiousness of skin and
outbreak of hepatitis type A possibly spread by scleras, light-coloured feces and dark urine. In
water. What indexes of river water quality can blood: neutrophilic leukocytosis - 13, 1 · 109 /l,
confirm this theory? ESR - 28 mm/h. What is the most likely di-
agnosis?
Крок 2 Medicine (англомовний варiант, iноземнi студенти) 2015 рiк 8

A. Chronic calculous cholecystitis other day. Two days ago the patient arrived
B. Chronic recurrent pancreatitis from Africa. Objectively: pale skin, subicteric
C. Fatty degeneration of liver sclera, significantly enlarged liver and spleen.
D. Chronic cholangitis, exacerbation stage What is the cause of fever attacks in this di-
E. Hypertensive dyskinesia of gallbladder sease?
60. A 52-year-old woman complains of face di- A. Erythrocytic schizogony
stortion. It appeared 2 days ago after exposure B. Tissue schizogony
to cold. Objectively: body temperature is C. Exotoxin of a causative agent
38, 2o C . Face asymmetry is present. Frontal D. Endotoxin of a causative agent
folds are flattened. Left eye is wider than the ri- E. Gametocytes
ght one and cannot close. Left nasolabial fold is
flattened, mouth corner is lowered. Examinati- 65. A 49-year-old patient 2 years ago was di-
on revealed no other pathology. Blood count: agnosed with stage 1 silicosis. He complains of
leukocytes - 10 · 109 /l, ESR - 20 mm/h. What is increased dyspnea and pain in the infrascapular
the most probable diagnosis? regions. X-ray shows diffuse intensification
and distortion of lung pattern with numerous
A. Facial neuritis nodular shadows 2-4 mm in diameter. Hardeni-
B. Trigeminus neuralgia ng of right interlobar pleura is detected. Lung
C. Hemicrania (migraine) roots are hardened and expanded. What X-ray
D. Ischemic stroke type of pneumosclerosis does the patient have?
E. Brain tumour
A. Nodular
61. A 28-year-old female patient with a six-year B. Interstitial
history of Raynaud’s syndrome has recently C. Interstitial-nodular
developed pain in the small joints of hands, di- D. Nodal
fficult movement of food down the esophagus. E. Tumor-like
What disease can be suspected in this case?
66. A 24-year-old patient complains of general
A. Systemic scleroderma weakness, dizziness, body temperature ri-
B. Periarteritis nodosa se up to 37, 5o C , sore throat, neck edema,
C. Rheumatoid arthritis enlargement of submaxillary lymph nodes.
D. Systemic lupus erythematosus Objectively: mucous membrane of oropharynx
E. Pseudotrichiniasis is edematic and cyanotic, tonsils are enlarged
and covered with films that spread beyond
62. An 8-year-old boy, who has been sufferi- the tonsils and cannot be easily removed.
ng from diabetes mellitus for 3 years, was What is the leading mechanism of this illness
delivered to a hospital in a condition of development?
hyperglycemic coma. Primary dose of insulin
should be prescribed basing on the following A. Action of bacterial exotoxin
calculation: B. Action of bacterial endotoxin
C. Allergic
A. 0,1-0,2 units/kg of body mass per hour D. Accumulation of suboxidated products
B. 0,05 units/kg of body mass per hour E. Bacteriemia
C. 0,2-0,3 units/kg of body mass per hour
D. 0,3-0,4 units/kg of body mass per hour 67. A 40-year-old woman with a history of
E. 0,4-0,5 units/kg of body mass per hour combined mitral valve disease with predomi-
nant stenosis complains of dyspnea, asthma
63. A 43-year-old female patient complains of attacks at night, heart problems. At present,
rash on the skin of her right leg, pain, weakness, she is unable to do even easy housework. What
body temperature rise up to 38o C . The disease is the optimal tactics of the patient treatment?
is acute. Objectively: there is an edema on the
skin of her right leg in the foot area, a well- A. Mitral commissurotomy
defined bright red spot in the irregular shape B. Implantation of an artificial valve
of flame tips, which is hot to touch. There are C. Antiarrhythmia therapy
isolated vesicles in the focus. What is your D. Treatment of heart failure
provisional diagnosis? E. Antirheumatic therapy
A. Erysipelas 68. A 30-year-old woman with a long history
B. Microbial eczema of chronic pyelonephritis complains of consi-
C. Contact dermatitis derable weakness, sleepiness, decrease in di-
D. Toxicoderma uresis down to 100 ml per day. BP is 200/120
E. Haemorrhagic vasculitis mm Hg. In blood: creatinine - 0,62 millimole/l,
hypoproteinemia, albumines - 32 g/l, potassi-
64. A 47-year-old patient came to see a doctor um - 6,8 millimole/l, hypochromic anemia,
on the 7th day of disease. The disease is increased ESR. What is the first step in the
acute: after the chill body temperature rose patient treatment tactics?
to 40o C and lasted up to 7 hours, then dropped
abruptly, which caused profuse sweat. There
were three such attacks occurring once every
Крок 2 Medicine (англомовний варiант, iноземнi студенти) 2015 рiк 9

A. Haemodialysis 73. A 12-year-old cild has been hit on the


B. Antibacterial therapy stomach. The patient is in moderately grave
C. Enterosorption condition, has a forced position in bed. The
D. Haemosorption skin is pale, heart rate - 122/min. Stress on
E. Blood transfusion the left costal arch causes a slight pain. Wei-
nert and Kulenkampff symptoms are positive.
69. A 43-year-old patient had right-sided deep Macroscopically the urine is unchanged. What
vein thrombosis of iliofemoral segment 3 years is the most likely diagnosis?
ago. Now he is suffering from the sense of
heaviness, edema of the lower right extremi- A. Spleen rupture, abdominal bleeding
ty. Objectively: moderate edema of shin, brown B. Left kidney rupture, retroperitoneal
induration of skin in the lower third of shin, hematoma
varix dilatation of superficial shin veins are C. Rupture of the pancreas
present. What is the most probable diagnosis? D. Liver rupture, abdominal bleeding
E. Rupture of a hollow organ, peritonitis
A. Postthrombophlebitic syndrome, varicose
form 74. Establishments participating in medical
B. Acute thrombosis of right veins examinations include: medical and preventi-
C. Lymphedema of lower right extremity ve treatment facility, hygiene and preventive
D. Parkes-Weber syndrome treatment facility, sociomedical expert commi-
E. Acute thrombophlebitis of superficial veins ttees, Ministry of Defence medical commi-
ttees, Ministry of Domestic Affairs medical
70. A 28-year-old woman has bursting pain committees, forensic medicine agency, etc.
in the lower abdomen during menstruati- Specify what service deals with sociomedical
on; chocolate-like discharges from vagina are assessment of temporary disability:
observed. It is known from the anamnesis that
the patient suffers from chronic adnexitis. Bi- A. Medical and preventive treatment facility
manual examination revealed a tumour-like B. Hygiene and preventive treatment facility
formation of heterogenous consistency 7х7 cm C. Sociomedical expert committees
large to the left from the uterus. The formation D. Ministry of Defence medical committees
is restrictedly movable, painful when moved. E. Ministry of Domestic Affairs medical commi-
What is the most probable diagnosis? ttees
A. Endometrioid cyst of the left ovary 75. An unconscious 35-year-old patient has
B. Follicular cyst of the left ovary been delivered by an ambulance to the intensi-
C. Fibromatous node ve care unit. Objectively: the patient is in
D. Exacerbation of chronic adnexitis semicoma. Moderate mydriasis is present. The
E. Tumour of sigmoid colon reaction of pupils to light is reduced. The pati-
ent is non-responsive to verbal instructions. BP
71. A 6-year-old child complains of frequent li- is150/100 mm Hg, tachycardia is present. Blood
quid stool and vomiting. On the 2nd day of contains methanol. What antidote should be
desease the child presented with inertness, administered?
temperature rise up to 38, 2o C , heart rate -
150 bpm, scaphoid abdomen, palpatory painful A. Ethanol
sigmoid colon, defecation 10 times a day with B. Unithiol
liquid, scarce stool with mucus and streaks of C. Thiamine chloride
green. What is a provisional diagnosis? D. Tavegil (Clemastine)
E. Naloxone
A. Shigellosis
B. Salmonellosis 76. After lifting a load, a 36-year-old male pati-
C. Escherichiosis ent has experienced a severe pain in the lumbar
D. Intestinal amebiasis region, which spread to the right leg and was
E. Yersiniosis getting worse when he moved his foot or
coughed. Objectively: the long back muscles on
72. A 65-year-old patient complains of pain in the right are strained. Achilles jerk is reduced
the lumbar spine, moderate disuria. He has on the right. There is a pronounced tenderness
been suffering from this condition for about of paravertebral points in the lumbar region.
half a year. Prostate volume is 45 cm3 (there The straight leg raise (Lasegue’s sign) is posi-
are hypoechogenic nodes in both lobes, capsule tive on the right. What additional tests should
invasion). Prostate-specific antigen is 60 ng/ml. be performed in the first place?
Prostate biopsy revealed an adenocarcinoma.
Which of the supplementary examination A. Radiography of the spinal column
methods will allow to determine the stage of B. Computed tomography
neoplastic process in this patient? C. Magnetic resonance tomography
D. Electromyography
A. Computer tomography of small pelvis E. Lumbar puncture
B. Lumbar spine X-ray
C. Excretory urography 77. A patient is active, lively, fussy. He resi-
D. Bone scintigraphy sts the examination. His speech is fast and
E. Chest X-ray loud, his statements are spontaneous and
Крок 2 Medicine (англомовний варiант, iноземнi студенти) 2015 рiк 10

inconsequent. Specify the psychopathological diagnosis is:


state of this patient:
A. Uterine rupture
A. Psychomotor agitation B. Initial uterine rupture
B. Catatonic agitation C. Threatened uterine rupture
C. Delirium D. Premature detachment of normally positi-
D. Behavioral disorder oned placenta
E. Paranoid syndrome E. Compression of inferior pudendal vein
78. A 4-year-old boy was vaccinated in vi- 82. A 26-year-old female patient has an 11-
olation of his vaccination schedule. There year history of rheumatism. Four years ago
are complaints of pain in the throat during she suffered 2 rheumatic attacks. Over the last
swallowing, headache, inertness, fever. Objecti- 6 months there have been paroxysms of atri-
vely: the child is pale; anterior cervical lymph al fibrillation every 2-3 months. What option
nodes are enlarged; tonsils are swollen, with of antiarrhythmic therapy or tactics should be
cyanotic hyperemy, and covered with grey- proposed?
white coating that cannot be removed; if it is
forcibly removed, tonsils bleed. The most li- A. Prophylactic administration of cordarone
kely diagnosis is: B. Immediate hospitalization
C. Defibrillation
A. Oropharyngeal diphtheria D. Lidocaine administration
B. Lacunar tonsillitis E. Heparin administration
C. Pseudomembranous (Vincent’s) tonsillitis
D. Infectious mononucleosis 83. A 47-year-old female patient complains of
E. Follicular tonsillitis cough with purulent sputum, pain in the lower
left chest, periodical body temperature rise.
79. A 26-year-old male patient complains of pi- She has been suffering from this condition for
ercing pain during breathing, cough, dyspnea. about 10 years. Objectively: "drumstick"distal
Objectively: to - 37, 3o C , respiration rate is phalanges. What examination would be the
19/min, heart rate is 92/min; BP is 120/80 mm most informative for making a diagnosis?
Hg. Vesicular respiration is observed. In the
inferolateral parts of chest auscultation in both A. Bronchography
inspiration and expiration phase revealed noi- B. Bronchoscopy
se that was getting stronger at phonendoscope C. Survey radiograph of lungs
pressing and could be still heard after cough. D. Pleural puncture
ECG showed no pathological changes. What is E. Bacteriological analysis of sputum
the most likely diagnosis?
84. A patient is 60-year-old, retired, worked
A. Acute pleuritis as deputy director of a research institute.
B. Intercostal neuralgia Behavioural changes appeared 2 years ago
C. Subcutaneous emphysema after the death of her husband: she stopped
D. Spontaneous pneumothorax looking after herself and leaving the house;
E. Pericarditis sicca then she stopped to clean the apartment and
cook. Mental status: temporal disorientation.
80. A child is 3-week-old. Since his birth peri- The patient does not understand many of the
odical vomiting is observed occuring several questions, is confused; does not know how
minutes after feeding. Vomit mass does not to cook soup or fasten a button. Her speech
exceed previous feeding volume. Body mass is is characterized by stumbling and logoclonia.
appropriate to the child’s age. What is the most She does not recognize doctors, fellow pati-
likely cause for the symptoms described? ents. She cries a lot but can not explain the
reason for tears. What is the mechanism of this
A. Pylorospasm pathology?
B. Esophageal chalasia
C. Adrenogenital syndrome A. Atrophy of the cerebral cortex
D. Pylorostenosis B. Atherosclerotic changes in cerebral vessels
E. Esophageal achalasia C. Serotonin deficiency
D. Impaired conversion of dopamine to
81. A 30-year-old parturient woman was deli- noradrenaline
vered to a maternity hospital with full-term E. Disorder of melatonin metabolism
pregnancy. She complains of severe lancinati-
ng pain in the uterus that started 1 hour ago, 85. An 8-year-old girl periodically has sudden
nausea, vomiting, cold sweat. Anamnesis states short-term heart pain, sensation of chest
cesarean section 2 years ago. Uterine contracti- compression, epigastric pain, dizziness, vomiti-
ons stopped. Skin and mucous membranes are ng. Objectively: the patient is pale, respiratory
pale. Heart rate is 100/min, BP is 90/60 mm Hg. rate - 40/min, jugular pulse is present. Heart
Uterus has no clear margins, is sharply painful. rate - 185 bpm, of poor volume. BP - 75/40
No heartbeat can be auscultated in the fetus. mm Hg. ECG taken during an attack shows
Moderate bloody discharge from the uterus ectopic P waves, QRS wave is not deformed.
can be observed. Uterus cervix is 4 cm open. At the end of an attack a compensatory pause
Presenting part is not visible. The most likely is observed. The most likely cause of the attack
Крок 2 Medicine (англомовний варiант, iноземнi студенти) 2015 рiк 11

is: A. Thermograph
B. Alcohol thermometer
A. Paroxysmal atrial tachycardia C. Mercury thermometer
B. Sinus tachycardia D. August’s psychrometer
C. Paroxysmal ventricular tachycardia E. Assmann psychrometer
D. Complete AV-block
E. Atrial fibrillation 91. Against the background of angina a patient
has developed pain in tubular bones. Exami-
86. During examination a patient is unconsci- nation revealed generalized enlargement
ous, his skin is dry and hot, face hyperemia of lymph nodes, hepatolienal syndrome,
is present. The patient has Kussmaul’s respi- sternalgia. In blood: RBCs - 3, 6 · 1012 /l, Hb-
ration, there is also smell of acetone in the 87 g/l, thrombocytes - 45 · 109 /l, WBCs -
air. Symptoms of peritoneum irritation are
positive. Blood sugar is 33 millimole/l. What 13 · 109 /l, blasts - 87%, stab neutrophiles - 1%,
emergency actions should be taken? segmented neutrophiles - 7%, lymphocytes -
5%, ESR - 55 mm/h. What is the most likely
A. Intravenous infusion of short-acting insulin diagnosis?
B. Intravenous infusion of glucose along with
insulin A. Acute leukemia
C. Introduction of long-acting insulin B. Erythremia
D. Intravenous infusion of neohaemodesum C. Chronic lymphocytic leukemia
along with glutamic acid D. Chronic myeloid leukemia
E. Intravenous infusion of sodium chloride E. Multiple myeloma
saline 92. For a long time to eradicate weeds on
87. A patient complains of evaginations in the agricultural lands herbicides retaining in envi-
region of anus that appear during defecation ronment have been used. Point out the most
and need resetting. Examination with anoscope probable way of herbicides reaching human
revealed 1x1 cm large evaginations of mucosa organism from soil:
above the pectineal line. What is the most A. Soil-plants-human
probable diagnosis? B. Soil-microorganisms-human
A. Internal hemorrhoids C. Soil-animals-human
B. Acute paraproctitis D. Soil-protozoa-human
C. External hemorrhoids E. Soil-insects-human
D. Anal fissure 93. A 52-year-old patient complains of pain in
E. - the right part of her chest, dyspnea, cough wi-
88. During the newborn infant’s examinati- th a lot of albuminoid sputum emitting foul
on redness of the skin was detected, which smell of "meat slops". Objectively: the pati-
occurred immediately after birth and became ent’s condition is grave, cyanosis is observed,
the most pronounced on the second day of the breathing rate is 31/min, percussion sound
infant’s life. What provisional diagnosis can be above the right lung is shortened, auscultation
made? revealed various moist rales (crackles). What is
the most probable diagnosis?
A. Simple erythema
B. Toxic erythema A. Lung gangrene
C. Transient erythema B. Lung abscess
D. Erythema nodosum C. Pleura empyema
E. Erythema anulare centrifugum D. Multiple bronchiectasis
E. Chronic pneumonia
89. A baby was born by a young smoker. The
labour was complicated by uterine inertia, diffi- 94. A 15-year-old patient suffers from
cult delivery of the baby’s head and shoulders. headache, nasal haemorrhages, sense of lower
The baby’s Apgar score was 4. Which of the extremity coldness. Objectively: muscles of
following is a risk factor for a spinal cord shoulder girdle are developed, lower extremi-
injury? ties are hypotrophied. Pulsation on the pedal
and femoral arteries is sharply dampened. BP
A. Difficult delivery of the head and shoulders is 150/90 mm Hg, 90/60 in the legs. Systolic
B. Young age of the mother murmur can be auscultated above carotid
C. Pernicious habits arteries. What is the most probable diagnosis?
D. Uterine inertia
E. Chronic hypoxia A. Aorta coarctation
B. Aorta aneurism
90. In order to study impact of microclimate on C. Aortal stenosis
the human organism it is necessary to make D. Aortal insufficiency
systematic observation of air temperature E. Coarctation of pulmonary artery
during the period of 3 days. Choose a devi-
ce that will allow to make the most precise 95. A patient with frostbite of both feet was
temperature records: delivered to an admission ward. What actions
should be taken?
Крок 2 Medicine (англомовний варiант, iноземнi студенти) 2015 рiк 12

A. Disturbed urine outflow from the kidney


A. To apply a bandage, to introduce vasodilating B. Increase in urine specific gravity
medications C. Ureteric twists
B. To administer cardiac medications D. Destruction of renal glomerules
C. To put feet into hot water E. Renal artery spasm
D. To rub feet with snow
E. To apply an alcohol compress 101. A 26-year-old woman has attended
maternity center complaining of her inability
96. A patient in a clinical death condition is bei- to become pregnant despite 3 years of regular
ng resuscitated through mouth-to-mouth artifi- sex life. Examination revealed the followi-
cial pulmonary ventilation and external cardi- ng: increased body weight; male-type pubic
ac massage. A doctor noticed that air does not hair; excessive pilosis of thighs; ovaries are
flow into the patient’s airways and his head and dense and enlarged; basal body temperature
torso are positioned at the same level. Why is is monophasic. The most likely diagnosis is:
artificial respiration not effective in the given
case? A. Ovaries sclerocystosis
B. Inflammation of uterine appendages
A. Tongue retraction C. Adrenogenital syndrome
B. Low breathing volume D. Premenstrual syndrome
C. External cardiac massage E. Gonadal dysgenesis
D. Probe is absent from stomach
E. The patient’s mouth is too small 102. A workshop resident doctor makes a li-
st of workers, who are often ill, for special
97. Hygienic assessment of a sample taken supervision. He takes into account the number
from the batch of grain revealed 2% of grains of etiologically connected cases resulting in
infected with microscopic Fusarium fungi. On temporary disability that occurred in the span
the grounds of laboratory analyses this batch of a year for each worker. How many such cases
of grain should be: should a worker have to be included in this
group?
A. Sold without restrictions
B. Tested for toxicity A. 4 and more
C. Used for forage production B. 1 and more
D. Used for ethanol production C. 2 and more
E. Destroyed D. 3 and more
E. 6 and more
98. Residents of an industrial community si-
tuated near a plant suffer from increased 103. A 28-year-old woman complains of
morbidity rate caused by nervous and endocri- increased intermenstrual periods up to 2
ne system conditions and kidney diseases. months, hirsutism. Gynaecological examinati-
Blood test: decrease of sulfhydric groups on revealed that the ovaries were enlarged,
content in blood. The pathologies developed painless, compact, uterus had no peculariti-
can be caused by environment being polluted es. Pelvic ultrasound revealed that the ovari-
by the following: es were 4-5 cm in diameter and had multiple
enlarged follicles on periphery. X-ray of skull
A. Mercury base showed that sellar region was dilated.
B. Cadmium What is the most probable diagnosis?
C. Boron
D. Chromium A. Stein-Leventhal syndrome (Polycystic ovary
E. Lead syndrome)
B. Algodismenorrhea
99. Various population groups mortality has C. Sheehan’s syndrome
been studied for a long time; territorial distri- D. Premenstrual syndrome
bution of population and resulting changes in E. Morgagni-Stewart syndrome
mortality trends are considered. What statstical
method can be applied? 104. A woman consulted a therapeutist
about fatigability, significant weight loss,
A. Time series analysis weakness, loss of appetite. She has been having
B. Standardized ratio analysis amenorrhea for 8 months. A year ago she born
C. Correlation and regression analysis a full-term child. Haemorrhage during labour
D. Assessment of parameter difference reliabi- made up 2 l. She got blood and blood substi-
lity tute transfusions. What is the most probable
E. Analysis of relative values diagnosis?
100. A 45-year-old patient with urolithiasis had A. Sheehan’s syndrome
an attack of renal colic. What is the mechanism B. Stein-Leventhal syndrome
of the attack development? C. Shereshevsky-Turner’s syndrome
D. Homological blood syndrome
E. Vegetovascular dystonia
105. The correlation between service record
Крок 2 Medicine (англомовний варiант, iноземнi студенти) 2015 рiк 13

and eosinophiles concentration in blood was A. Alimentary constitutive obesity, I stage,


studied in workers at dyeing shops of texti- android type
le factories. What index will be the most B. Itsenko-Cushing hypothalamic obesity, II
informative for the analysis of this data? stage, gynoid type
C. Alimentary constitutive obesity, III stage,
A. Correlation factor gynoid type
B. Student’s criterion D. Alimentary constitutive obesity, II stage,
C. Standardized index android type
D. Fitting criterion E. Itsenko-Cushing hypothalamic obesity, I
E. Sign index stage, android type
106. A 39-year-old woman suffering from schi- 110. A parturient woman is 27 year old, it was
zophrenia constantly strains to hear somethi- her second labour, delivery was at full-term,
ng insisting that "there is a phone connecti- normal course. On the 3rd day of postpartum
on in her brain and she hears her brother’s period body temperature is 36, 8o C , heart rate
voice demanding that she come back home". - 72/min, BP - 120/80 mm Hg. Mammary glands
The patient is anxious, suspicious, constantly are moderately swollen, nipples are clean.
looking around. Specify the psychopathologic Abdomen is soft and painless. Fundus of uterus
syndrome. is 3 fingers below the umbilicus. Lochia are
bloody, moderate. What is the most probable
A. Hallucinatory diagnosis?
B. Anxiety
C. Paranoid A. Physiological course of postpartum period
D. Paraphrenic B. Subinvolution of uterus
E. Depressive C. Postpartum metroendometritis
D. Remnants of placental tissue after labour
107. A 3-month-old girl has rhinitis, dyspnea, E. Lactostasis
dry cough. She has been sick for 2 days.
Objectively: pale skin, acrocyanosis, hypopnoe; 111. A 40-year-old patient suffers from
breathing rate is 80/min; over the whole influenza. On the 5th day of illness there are
pulmonary surface there is vesiculotympanitic pain behind sternum, cough with sputum,
(bandbox) resonance observed with numerous inertness. Temperature is 39, 5o C . Face is
bubbling rales (crackles). The most likely di- pale. Mucosa of conjunctivas and pharynx is
agnosis is: hyperemic. Heart rate is 120/min, breathing
rate is 38/min. In the lower lung segments
A. Acute bronchiolitis shortening of percussion sound and moist rales
B. Pneumonia (crackles) can be detected. What additional
C. Mucoviscidosis investigation should be performed first of all to
D. Foreign body in airways specify the diagnosis?
E. Acute bronchitis
A. Lung X-ray
108. A 30-year-old patient was hospitalised in B. ECG
an intensive care unit with a diagnosis of multi- C. Heart US
ple bee stings. Skin is pale and covered with D. Mantoux test
cold sweat. Pulse can be palpated only at the E. Spirography
carotid arteries and is 110/min; breating rate is
24/min, rhytmical, weakened. What drug must 112. A 45-year-old patient complains of body
be administered immediately? temperature rise up to 40o C , general weakness,
headache, painfulness and spastic muscle
A. Adrenaline hydrochloride intravenously contractions around the wound in the shin. He
B. Prednisolone intravenously received this wound 5 days ago when worki-
C. Adrenaline hydrochloride intramuscularly ng in his garden. He requested no medical
D. Dopamine intravenously care back then. What wound infection can be
E. Tavegyl (Clemastine) intravenously suspected?
109. A 24-year-old patient complains of gaining A. Tetanus
body mass and increased appetite. Objectively: B. Anthrax
built of hypersthenic type, body mass index C. Erysipelas
is 33,2 kg/m2 , waist circumference is 100 cm. D. Gram-positive
Waist to hips circumference ratio is 0,95. What E. Gram-negative
is the provisional diagnosis?
113. A 60-year-old woman started feeling
weakness, vertigo, rapid fatigability during the
last year. Recently there have been dyspnea
and paresthesia observed. Objectively: skin
and mucous membranes are pale and icteric.
Lingual papillae are smoothed out. Liver and
spleen are situated at the edge of costal arch.
Blood test: Hb- 70 g/l, erythrocytes - 1, 7·1012 /l,
blood color index - 1,2, macrocytes. What drug
Крок 2 Medicine (англомовний варiант, iноземнi студенти) 2015 рiк 14

can be prescribed on pathogenetic grounds? A. Acute bartholinitis


B. Furuncle of outer labia
A. Vitamin B12 C. Acute vulvovaginitis
B. Vitamin B6 D. Bartholin’s cyst
C. Ascorbic acid E. Carcinoma of vulva
D. Iron preparations
E. Vitamin B1 118. In a rural health care area there is a case of
child dying during the first month of his life. To
114. A 34-year-old patient complains of profuse analyse this situation, among other measures,
sweating at night, skin itching, weight loss an expert assessment of medical records is
(9 kg within the last 3 months). Examinati- performed. What medical document should be
on revealed malnutrition, skin pallor. Palpati- considered first?
on of neck and inguinal areas revealed dense
elastic lymph nodes of about 1 cm in diameter, A. Child development history
nonmobile, non-adhering to skin. What is the B. Infant development history
most probable diagnosis? C. Vaccination card
D. Outpatient medical record
A. Lymphogranulomatosis E. Child’s medical record
B. Chronic lymphadenitis
C. Lymphosarcoma 119. Examination of placenta revealed a defect.
D. Burkitt’s lymphoma An obstetrician performed manual investi-
E. Cancer metastases gation of uterine cavity, uterine massage.
Prophylaxis of endometritis in the postpartum
115. A 57-year-old woman complains of having period should involve the following actions:
a sensation of esophageal compresion, palpi-
tation, breathing difficulties when eating solid A. Antibacterial therapy
food, occasional vomiting with a full mouth, B. Instrumental revision of uterine cavity
"wet pillow"sign at night for the last 6 months. C. Haemostatic therapy
Objectively: body tempearture - 39o C , height - D. Contracting agents
168 cm, weight - 72 kg, heart rate - 76/min, BP- E. Intrauterine instillation of dioxine
120/80 mm Hg. X-ray revealed considerable di- 120. A 15-year-old boy has developed pain in
lation of esophagus and its constriction in the the area of his left knee joint. Objectively:
cardial part. What pathology is most likely to thigh soft tissues in the painful area are infi-
have caused dysphagia in this patient? ltrated, joint function is reduced. X-ray: there is
A. Achalasia cardiae a focus of destruction in the left distal femoral
B. Primary esophagism metaphysis, with periosteum detachment and
C. Hiatal hernia formation of Codman triangle in the bone
D. Esophageal carcinoma cortical layer at the margin of the defect. Chest
E. Reflux esophagitis X-ray reveals numerous microfocal metastases.
What pathology is the most likely to cause such
116. A 25-year-old man complains of pain presentation?
in the lower third of his left thigh, which
occurs both with and without physical exertion. A. Osteogenic sarcoma
Possibility of trauma is denied by the patient. B. Fibrosarcoma
Objectively: skin colour is normal; pastosity C. Chondrosarcoma
and pain can be detected with deep palpati- D. Ewing’s sarcoma
on; knee joint mobility is reduced. X-ray of E. Juxtacortical sarcoma
distal femoral metaphysis shows an area of 121. A 74-year-old man complains of pain in
destruction and spicule. Blood test: immature the left foot and dark color of the 1st toe. He
cells, no signs of inflammation. The most likely has been sick for 6 days. Objectively: heart
diagnosis is: rate is 84/min. Body temperature is 37, 8o C .
A. Osteogenic sarcoma The left foot resembles "chicken claw"due
B. Osteitis fibrosa cystica to soft tissues decreasing in volume, skin is
C. Chronic osteomyelitis dark red. The 1st toe is black, small amount of
D. Multiple myeloma dark substance is being discharged from under
E. Marble-bone disease the nail. Pulse over the peripheral arteries of
extremity is absent. The most likely diagnosis
117. A 37-year-old woman complains of sharp is:
pains in her external genitalia, edema of the
vulvar lips, pain when walking. Objectively: A. Gangrene
body temperature is 38, 7o C , heart rate is B. Phlegmon
98/min. Inside the right vulvar lip there is a C. Panaritium
dense, painful, tumor-like growth 5,0х4,5 cm in D. Erysipelas
size; skin and mucosa of the external genitalia E. Erysipeloid
are hyperemic, copious foul-smelling discharge 122. Head circumference of a 1-month-old boy
is observed. The most likely diagnosis is: with signs of excitement is 37 cm, prefontanel
is 2x2 cm large. After feeding the child regurgi-
tates small portions of milk; stool is normal
Крок 2 Medicine (англомовний варiант, iноземнi студенти) 2015 рiк 15

in its volume and composition. Muscle tone is


within norm. What is the most likely diagnosis? A. Diabetes mellitus type 2 with visceral
neuropathy
A. Pylorospasm B. Diabetes mellitus type 1 with ketoacidosis
B. Meningitis C. Diabetes mellitus type 1 with
C. Pylorostenosis encephalomyelopathy
D. Microcephaly D. Diabetes mellitus type 2 with nephropathy
E. Craniostenosis E. Diabetes mellitus type 2 with polyneuropathy
123. A 49-year-old patient complains of di- 127. A young woman suffering from seborrhea
fficulties when swallowing food, hoarse voi- oleosa has numerous light-brown and whi-
ce, weight loss. The patient has been suffering te spots on the skin of her torso and
from these symptoms for the last 3 months. The shoulders. The spots have clear margins,
symptoms are gradually progressing. Objecti- branny desquamation, no itching. What provi-
vely: the patient is emaciated, supraclavicular sional diagnosis can be made?
lymph nodes are enlarged. Esophagoscopy
revealed no pathologies of esophagus. What A. Pityriasis versicolor
investigation would be the most advisable in B. Torso dermatophytosis
the given case? C. Seborrheic dermatitis
D. Pityriasis rosea
A. Computed tomogram of chest and mediasti- E. Vitiligo
num
B. Lungs X-ray 128. 10 days after birth an infant developed
C. Multiplanar fluoroscopy of esophagus a sudden fever up to 38, 1o C . Objectively:
D. Radioisotopic examination of chest and the skin of navel, abdomen and chest is
mediastinum erythematous; there are multiple pea-sized
E. US of mediastinum blisters with no infiltration at the base; si-
ngle bright red moist erosions with epidermal
124. A 36-year-old male patient complains of fragments on the periphery. What is your provi-
having headache, obstructed nasal breathing, sional diagnosis?
purulent nasal discharge for 2 weeks. A month
before, he had his right maxillary premolar A. Epidemic pemphigus of newborn
filled. X-ray revealed an intense opacity of B. Syphilitic pemphigus
the right maxillary sinus. Diagnostic puncture C. Streptococcal impetigo
revealed a large amount of thick malodorous D. Vulgar impetigo
crumbly pus. What is the most likely diagnosis? E. Atopic dermatitis

A. Chronic suppurative odontogenic sinusitis 129. In a traffic accident two persons died. An
B. Acute purulent sinusitis appointed forensic medical expert was called
C. Chronic purulent sinusitis on duty to another accident location; therefore,
D. Chronic atrophic sinusitis according to the crininal law in force in Ukrai-
E. Tumor of the maxillary sinus ne, an investigator called in the following speci-
alist for examination of the location and bodi-
125. During a surgical operation necessitated es:
by the patient’s suffering from autoimmune
thyroiditis with concomitant multinodular goi- A. Nearest available medical doctor
ter the right lobe was removed and subtotal B. Medical assistant
resection of the left lobe was performed. C. Surgical nurse
What should be prescribed to the patient for D. Nearest available dispensing chemist
postoperative hypothyroidism prevention? E. Nearest available dental mechanic

A. L-thyroxin 130. A 35-year-old female patient has gained


B. Mercazolil (Thiamazole) 20 kg weight within a year with the normal di-
C. Iodomarin (Potassium iodide) et. She complains of chill, sleepiness, dyspnea.
D. Lithium preparations The patient’s mother and sister are corpulent.
E. Insulin Objectively: height - 160 cm, weight - 92 kg,
BMI - 35,9. Obesity is uniform, there are no
126. A 54-year-old patient has been suffering striae. The face is amimic. The skin is dry. The
from diabetes mellitus for 5 years, with diet tongue is thickened. Heart sounds are muffled.
being his only treatment. Within the last half Heart rate - 56/min, BP - 140/100 mm Hg. The
a year he lost 7 kg of body weight, complai- patient has constipations, amenorrhea for 5
ns of thirst, vertigo when raising from bed, months. TSH - 28 mkME/l (normal rate - 0,32-
decrease of erectile function, frequent stool, 5). Craniogram shows no pathology. What is
especially at night. Objectively: malnutrition, the etiology of obesity?
dry skin. BP in lying position is 160/90 mm
Hg; BP in standing position is 170/85 mm Hg. A. Hypothyroid
No edemas. Fasting plasma glucose level is B. Hypo-ovarian
12 mmol/l. Glycated hemoglobin accounts for C. Hypothalamic-pituitary
11%. Albumin excreted with urine is 20 mg per D. Alimentary and constitutive
day. The most likely diagnosis is: E. Hypercorticoid
Крок 2 Medicine (англомовний варiант, iноземнi студенти) 2015 рiк 16

131. A 45-year-old patient has been sufferi- A. Ileocecal invagination


ng from essential hypertension for 10 years. B. Gastrointestinal haemorrhage
After hot bath he suddenly developed sharp C. Wilm’s tumour
headache and recurrent vomiting. Objecti- D. Helminthic invasion
vely: pronounced meningeal symptoms; BP E. Pylorostenosis
is 180/110 mm Hg. He was hospitalised in a
neurology unit. What additional tests should 135. A 68-year-old patient complains of
be primarily prescribed? pain, slight swelling and creaking in the di-
stal joints of her fingers and knee joints.
A. Lumbar puncture, cerebrospinal fluid test She has been suffering from this condition
B. Ventriculopuncture for 4 years. Objectively: thickening of knee
C. Echoencephalography and interphalangeal joints, restricted mobi-
D. Rheoencephalography lity of these joints. Blood test: leukocytes -
E. Electroencephalography 5, 4 · 109 /l, ESR - 12 mm/h, fibrinogen - 3,5
g/l; Rheumatoid factor - 1:32; anti-streptolysin
132. A 77-years-old patient complains of inabi- O - 160 units; seromucoid - 0,20 units.; CRP+.
lity to urinate and bursting pain above his What mechanism of development is likely for
pubis. Acute onset of his condition occurred this condition?
12 hours ago. Objectively: overfilled urinary
bladder can be palpated above the pubis. On A. Glycosaminoglycanes synthesis disruption
rectal examination: prostate is enlarged, dense- B. Rheumatoid factor development
elastic, with clear margins and no nodules. C. Native DNA antibodies hyperproduction
Interlobar sulcus is pronounced. US examinati- D. Increased uric acid sunthesis
on: prostate volume is 120 cm3 , it protrudes E. Immune reaction to streptococcosis
into urinary bladder cavity, parenchyma is
homogeneous. Prostate specific antigen is 5 136. A 30-year-old male patient consulted a
ng/ml. What disease is the most likely to cause family doctor 2 months after he had been
acute urinary retention? operated for an open fracture of brachial bone.
Objectively: the patient’s condition is sati-
A. Prostatic hyperplasia sfactory; in the area of the operative wound
B. Prostate cancer there is a fistula with some purulent discharge,
C. Prostate sclerosis redness, fluctuation. X-ray image shows brachi-
D. Prostate tuberculosis al bone destruction with sequestra. What
E. Acute prostatitis complication arose in the postoperative peri-
od?
133. A patient with uterine fibromyoma si-
zed up to 8-9 weeks of pregnancy consulted A. Posttraumatic osteomyelitis
a gynaecologist about acute pain in the lower B. Hematogenic osteomyelitis
abdomen. Examination revealed pronounced C. Wound abscess
positive symptoms of peritoneal irritation, hi- D. Posttraumatic phlegmon
gh leukocytosis. Vaginal examination revealed E. Suture sinus
that the uterus was enlarged up to 9 weeks of
pregnancy due to the fibromatous nodes, one 137. A 40-year-old female patient complains of
of which was mobile and extremely painful. having a bulge on the anterior surface of neck
Appendages were not palpable. Discharges for 5 years. Objectively: Heart rate - 72 bpm,
were mucous, coming in moderate amounts. arterial pressure - 110/70 mm Hg, in the right
What is the treatment tactics? lobe of thyroid gland palpation reveals a mobi-
le 4x2 cm node, the left lobe is not palpable, the
A. Urgent surgery (laparotomy) basal metabolic rate is 6%. What is the most li-
B. Surveillance and spasmolytic therapy kely diagnosis?
C. Fractional diagnostic curettage of the uterine
cavity A. Nodular euthyroid goiter
D. Surgical laparoscopy B. Nodular hyperthyroid goiter
E. Surveillance and antibacterial therapy C. Riedel’s thyroiditis
D. Mixed euthyroid goiter
134. A 4-month-old child was admitted to E. The median cervical cyst
a surgical department 8 hours after the first
attack of anxiety. The attacks happen every 138. Estimation of community health level
10 minutes and last for 2-3 minutes, vomi- involved analysis of a report on diseases regi-
ting occurred once. Objectively: the child’s stered among the population of catchment area
condition is grave. Abdomen is soft, palpati- (reporting form 12). What index is calculated
on reveals a tumour-like formation in the right basing on this report?
iliac area. After rectal examination the doctor’s A. Common morbidity rate
finger was stained with blood. What is the most B. Index of pathological affection
probable diagnosis? C. Index of morbidity with temporary disability
D. Index of hospitalized morbidity
E. Index of basic non-epidemic morbidity
139. A tractor driver with the record of servi-
Крок 2 Medicine (англомовний варiант, iноземнi студенти) 2015 рiк 17

ce of 24 years has undergone palestesiometry vely: tuberous slightly movable lump can be
test (Vibration Sensitivity Measurement). palpated in the right subcostal area; veins
Test revealed increased vibration sensitivity of spermatic cord and scrotum are dilated.
threshold at the frequencies of 63-125-259 Hz Blood test: Hb - 86 g/l, ESR - 44 mm/h. Uri-
to 25 dB. Dynamometry is 20 kg on the right ne test: specific gravity - 1020, proteine - 0,99
and 16 kg on the left. Cold stimulus test is posi- g/l, erythrocytes - all field of vision, leukocytes
tive, time of hand temeperature restoration is - 4-6 in the field of vision. The provisional di-
52 minutes. Blanching at pressure symptom is agnosis is:
positive and equals 21. Hypesthesia of upper
and lower limbs is observed and can be classifi- A. Kidney tumor
ed as "gloves"and "socks"polyneuritic pattern. B. Urolithiasis
Make the provisional diagnosis. C. Acute pyelonephritis
D. Acute glomerulonephritis
A. Vibration disease, I stage, caused by combi- E. Nephroptosis
ned vibration
B. Vibration disease, I stage, caused by local 144. A 58-year-old patient complains of
vibration headache in the occipital region, nausea, choki-
C. Vibration disease, II stage, caused by combi- ng, opplotentes. The presentations appeared
ned vibration after a physical exertion. Objectively: the pati-
D. Vibration disease, II stage, caused by local ent is excited. Face is hyperemic. Skin is pale.
vibration Heart sounds are regular, the 2nd aortic sound
E. Vibration disease, I stage, caused by general is accentuated. BP - 240/120 mm Hg, heart
vibration rate - 92/min. Auscultation reveals some fine
moist rales (crackles) in the lower parts of the
140. A patient’s X-ray image (anteroposterior lungs. Liver is not enlarged. ECG shows signs
projection) shows deformation of lung pattern, of hypertrophy and left ventricular overload.
pneumofibrosis, reticular (honeycomb) lung What is the most likely diagnosis?
pattern of lower pulmonary segments, cylindric A. Complicated hypertensic crisis
and fusiform lumps. The most likely diagnosis B. Acute myocardial infarction, pulmonary
is: edema
A. Multiple bronchiectasis C. Bronchial asthma exacerbation
B. Right lung abscess D. Uncomplicated hypertensic crisis
C. Non-hospital-acquired pneumonia E. Community-acquired pneumonia
D. Lungs maldevelopment 145. A 37-year-old patient complains of pain
E. Diffuse pulmonary fibrosis in the lumbar spine, which increases during
141. A 41-year-old patient with Addi- walking, limited mobility, edema of the right si-
son’s disease had influenza. After that he de of abdomen. Focal tuberculosis is recorded
developed adynamia, depression, nausea, in the anamnesis. X-ray shows destruction
vomiting, diarrhea and hypoglycemia. BP of adjacent surfaces of the bodies of the 1-
is 75/50 mm Hg. Blood test: decreased 2 lumbar vertebrae, vertebral bodies height
content of corticosterone, hydrocortisone, is decreased, intervertebral fissure cannot be
13-oxycorticosteroids, 17-oxycorticosteroids. detected. Abdominal US reveals a growth
What condition has developed in the patient? 15x20 cm in size in the retroperitoneal space
and echo signs of liquid. What diagnosis can be
A. Acute adrenal gland insufficiency suspected?
B. Acute gastritis
C. Acute enterocolitis A. Lumbar tuberculous spondylitis
D. Collapse B. Fracture of the 1-2 lumbar vertebrae body
E. Diabetes mellitus C. Metastatic damage of vertebral column
D. Lumbar spondylolisthesis
142. After a contact with chemicals a plant E. Osteochondrosis
worker has suddenly developed stridor, voi-
ce hoarseness, barking cough, progressi- 146. A 45-year-old patient (14-year-long work
ng dyspnea. Objective examination reveals record as a house painter) upon the contact
acrocyanosis. What is your provisional di- with synthetic paint develops skin reddeni-
agnosis? ng, edema, severe itching and oozing lesions
on her face. Symptoms disappear after the
A. Laryngeal edema contact with this chemical substance stops
B. Laryngeal carcinoma but even the smell of paint alone is enough
C. PATE to make them reappear each time. Each
D. Pulmonary atelectasis recurrence is characterised by increased severi-
E. Pneumothorax ty of symptoms. What provisional diagnosis can
be made?
143. A 58-year-old patient complains of general
weakness, weight loss up to 10 kg within the
last 1,5 months, progressive pain in the small
of the back, raise of blood pressure to 220/160
mm Hg, subfebrile body temperature. Objecti-
Крок 2 Medicine (англомовний варiант, iноземнi студенти) 2015 рiк 18

A. Professional eczema A. Pyloric stenosis


B. Simple contact dermatitis B. Penetration of gastric ulcer
C. Allergic contact dermatitis C. Functional pyloric spasm
D. Urticaria D. Foreign body in the stomach (bezoar)
E. Toxicodermia E. Malignization of gastric ulcer
147. In cold season a patient was delivered to 151. A 5-year-old child has body temperature
a hospital. He was rescued from drowning in a risen up to febrile numbers, suffers from
body of water. There was no contact of his ai- inertness, weakness. Examination revealed
rways with water. The patient is anxious, pale, hemorrhage on the skin of limbs and torso.
complains of pain and numbness of limbs, has Enlargement of cervical and axillary lymph
cold shivering. Breathing rate is 22/min, BP nodes can be detected. The liver is 4 cm below
is 120/90 mm Hg, heart rate is 110/min, rectal the costal arch; the spleen is 6 cm below
temperature is 34, 5o C . What kind of warming the costal arch. Blood test: erythrocytes -
should be prescribed to the patient? 2, 3 · 1012 /l, Hb- 60 g/l, platelets - 40 · 109 /l,
A. Passive warming leukocytes - 32, 8 · 109 /l, eosinophiles - 1%,
band neutrophiles - 1%, segmented neutrophi-
B. Infusion of 37O c solutions les - 12%, lymphocytes - 46%, monocytes - 1%,
C. Hot compresses blasts - 40%, Duke’s bleeding time test result is
D. Heat bath 9 min. What examination is necessary to make
E. Hemodialysis with blood warming the diagnosis?
148. Survey radiograph of a 52-year-old worker A. Myelogram
of an agglomeration plant (28 years of experi- B. Lymph nodes biopsy
ence, the concentration of metal dust is 22-37 C. US of abdominal cavity
mg/m3 ) shows mildly pronounced interstitial fi- D. Detection of hepatitis markers
brosis with diffused contrast well-defined small E. Investigation of platelets dynamic functions
nodular shadows. The patient has no complai-
nts. Pulmonary function is not compromised. 152. A child from primipregnancy was born in
What is the provisional diagnosis? a term labor and has body weight of 4000 g
and body length of 57 cm. When born, he was
A. Siderosis nonresponsive to examination. Diagniosis is di-
B. Silicosis ffuse. Heart rate is 80/min. What resuscitation
C. Anthraco-silicatosis measures should be prescribed?
D. Silicatosis
E. Anthracosis A. Begin ALV with mask
B. Introduce 100% oxygen
149. A week before, a 65-year-old male patient C. Intubate and begin ALV
suffered an acute myocardial infarction. His D. Tactile stimulation
general condition has deteriorated: he complai- E. Administer naloxone
ns of dyspnea at rest, pronounced weakness.
Objectively: edema of the lower extremities, 153. A 20-year-old patient was delivered to a
ascites is present. Heart borders are extended, surgical unit complaining of an incised wound
paradoxical pulse is 2 cm displaced from the on his right forearm that has been bleeding
apex beat to the left. What is the most likely for 1,5 days. Suffers from general weakness,
diagnosis? vertigo, cold sweat, opplotentes. Skin and visi-
ble mucous membranes are pale. Heart rate is
A. Acute cardiac aneurysm 110/min, BP is 100/70 mm Hg. Blood test: Hb
B. Recurrent myocardial infarction is 100 g/l, erythrocytes 2, 5 · 1012 /l. What is the
C. Acute pericarditis cause for the paient’s general condition?
D. Cardiosclerotic aneurysm
E. Pulmonary embolism A. Posthemorrhagic anemia
B. Aplastic anemia
150. A 60-year-old patient complains of nearly C. Wound infection
permanent sensation of heaviness and fullness D. Concomitant disease
in the epigastrium, that increases after eating, E. Acute trombophlebitis
foul-smelling eructation, occasional vomiting
with food consumed 1-2 days ago, weight loss. 154. A 20-year-old parturient woman has the I
12 years ago he was first found to have an ulcer labor stage. The pregnancy is full-term. Labors
of pyloric channel. The patient has taken rani- occur every 3 minutes and last for 55 seconds.
tidine for periodic hunger pain. The patient’s Fetus presentation is polar, the head is pressed
condition has been deteriorating over the last to the small pelvis entrance. Heart rate of the
3 months. Objectively: splashing sound in the fetus is 150/min, distinct and rhythmic. Vagi-
epigastrium is present. What kind of compli- na examination: uterus cervix is smoothed out;
cation is it? mouth of the womb is 2 cm open; fetal bladder
is intact; the head is presented over the I plane
of small pelvis; moderate mucous-bloody di-
scharge is observed. What phase of the I labor
stage is it?
Крок 2 Medicine (англомовний варiант, iноземнi студенти) 2015 рiк 19

A. Latent A. Echocardiography
B. Active B. Electrocardiography
C. Slowing-down C. X-ray
D. Physiological preliminary period D. Phonocardiography
E. Primary uterine inertia E. Rheography
155. A 23-year-old woman after stress has 159. A 64-year-old patient has been hospitali-
developed thirst, polydipsia, polyuria, weight sed with complaints of progressive jaundice
loss, increasing fatigue. Later she developed that developed over 3 weeks ago without pain
nausea and somnolence, lost consciousness syndrome, general weakness, loss of appetite.
and was hospitalised. Glycemia is 27 mmol/l, Objectively: temperature is 36, 8o C , heart rate
acetone in urine is sharply positive. Treatment is 78/min, abdomen is soft and painless, peri-
for ketoacidotic coma was initiated. When toneum irritation symptoms are not detected,
would it be advisable to start preventive palpation reveals sharply enlarged tense
treatment of hypoglycemia by introduction of gallbladder. What disease can be characteri-
5% glucose solution? sed with these symptoms?
A. After glycemia rate drops to 13-14 mmol/l A. Cancer of pancreas head
B. 2 hours after beginning of insulinotherapy B. Duodenal ulcer
C. When patient becomes conscious C. Acute cholecystitis
D. After glycemia rate becomes normal D. Chronic cholecystitis
E. If glycemia decreases with the rate over 5 E. Cholecystitis caused by lambliasis
mmol/l per hour
160. A 22-year-old woman on a reduced diet,
156. A 53-year-old woman complains of weight vegetarian, attended a hospital with complai-
loss up to 10 kg within the last 2 years, liquid nts of distorted smell and taste perception and
foul-smelling stool two times a day that poorly lesions in the angles of her mouth. Objecti-
washes off the toilet, periodic bouts of nausea, vely: sclera is distinctly blue. Diagnosis: iron-
girdle pain in the upper abdomen. Objecti- deficiency anemia. What clinical syndrome is
vely: pain in Gubergrits zone (on the right from expressed primarily?
navel) and Mayo-Robson’s point. Biochemical
blood analysis: glucose - 3,2 mmol/l, bilirubin - A. Sideropenic
16,5 micromole/l, crude protein - 56,4 g/l. Urine B. Anemic
diastase - 426 g/h/l. D-xylose test (oral admi- C. Hemorrhagic
D. Hemolytic
nistration of 25 g of d-xylose) after 5 hours E. Myelodysplasic
reveals 3 g of xylose in urine. The most likely
diagnosis is: 161. A 15-year-old patient complains of
excessive body weight, headache, irritability,
A. Pancreatitis. Malabsorption syndrome rapid fatigability. Significant increase of body
B. Pseudomembranous colitis weight occurred at the age of 14. Objectively:
C. Nonspecific ulcerative colitis weight is 90 kg; height is 160 sm, proportional
D. Irritable bowel syndrome body built. Fatty tissue is distributed evenly.
E. Chronic gastritis
There are thin pink striae (stretch marks) on
157. A 23-year-old patient after intake of brake the thighs, abdomen and mammary glands. BP
fluid has developed anuria that has been lasting - 145/90 mm Hg. Provisional diagnosis is:
for 5 days already. Creatinine level increased up
to 0,769 mmol/l. What treatment tactics should A. Pubertate dyspituitarism
be chosen in the given case? B. Alimentary constitutive obesity
C. Somatoform autonomic dysfunction
A. Hemodialysis D. Itsenko-Cushing’s disease
B. Detoxification therapy E. Cushing’s syndrome
C. Antidotal therapy
D. Diuretics 162. A 6-year-old girl attended a general practi-
E. Plasmapheresis tioner with her mother. The child complains
of burning pain and itching in her external
158. A 15-year-old teenager has undergone genitalia. The girl was taking antibiotics the
medical examination in military recruitment day before due to her suffering from acute
center. The following was revealed: interval bronchitis. On examination: external genitalia
systolic murmur at the cardiac apex, accent are swollen, hyperemic, there is white deposit
of the II heart sound over the pulmonary accumulated in the folds. The most likely di-
artery, tachycardia. What additional examinati- agnosis is:
on method will be the most informative for
determining diagnosis? A. Candidal vulvovaginitis
B. Trichomoniasis
C. Nonspecific vulvitis
D. Helminthic invasion
E. Herpes vulvitis
163. A 44-year-old man complains of dyspnea
Крок 2 Medicine (англомовний варiант, iноземнi студенти) 2015 рiк 20

with sensation of lack of air on inhale, palpi- are detected, the skin covering them is bluish-
tations occurring during slight physical exerti- red in colour. Some nodules have fistulous
on, and shin edemas that appear in eveni- openings producing thick purulent mass. Body
ng and resolve in morning. His condition has temperature is 38, 5O c, general malaise. The
been lasting for 5 months already, deteriorati- most likely diagnosis is:
on is gradual. What method of instrumental
diagnostics allows to verify decrease of systolic A. Hydradenitis
function in this patient? B. Carbuncle
C. Cutaneous tuberculosis
A. Echocardiography D. Necrotizing ulcerative trichophytosis
B. Electrocardiography E. Pyoderma chancriformis
C. Computed tomogram
D. Phonocardiography 168. A patient complains of weight gain, chill,
E. Holter blood pressure monitoring edemas, xeroderma, somnolence, difficulties
with focusing. Objectively: height is 165 cm;
164. When playing in a kindergarten a 3- weight is 90 kg; body proportions are of female
year-old child sudenly developed dyspnea, type, to - 35, 8o C , heart rate - 58/min, BP -
paroxysmal compulsive dry cough. The face 105/60 mm Hg. Heart sounds are weakened,
is cyanotic, the eyes are tearful. Vomiting bradycardia is observed. Other internal organs
occurred several times. Breathing is weakened have no changes. Thyroid gland cannot be
over the whole right side of the chest. The palpated. Milk secretion from mammary
provisional diagnosis is: glands is observed. Hormone investigation
revealed increased levels of thyroid-stimulating
A. Foreign body
B. Obstructive bronchitis hormone (TSH) and prolactin, and decreased
C. Bronchial asthma level of thyroxine (T4). Which one is the cause
D. Hysteria fit for obesity?
E. Stenosing laryngotracheitis A. Primary hypothyroidism
165. A patient with otopyosis is in sharply B. Secondary hypothyroidism
deteriorating condition: he developed C. Prolactinoma
headache, vomiting, febrile temperature, D. Hypopituitarism
general hyperesthesia. Meningeal symptoms E. Adiposogenital dystrophy
and stagnant optic disks are observed. There 169. A 54-year-old patient complains of
is no focal symptoms. Liquor is turbid, blood weakness, jaundice, itching skin. Disease onset
pressure is high, albuminocytological dissoci- was 1,5 months ago: fever up to 39o C appeared
ation occurs with neutrophils. What disease at first, with progressive jaundice developed 2
can be suspected? weeks later. On hospitalisation jaundice was
A. Secondary purulent meningitis severely progressed. Liver cannot be palpated.
B. Meningoencephalitis Gallbladder is enlarged and painless. Blood
C. Serous meningitis bilirubin is 190 micromole/l (accounting for
D. Primary purulent meningitis direct bilirubin). Stool is acholic. What is the
E. Subarachnoid hemorrhage most likely jaundice genesis in this patient?

166. A 25-year-old patient is not married and A. Mechanical jaundice


has sexual relations with several partners. Duri- B. Hepatocellular jaundice
ng the last 3 months he noticed small amount of C. Hemolytic jaundice
mucoserous secretions produced from urethra. D. Caroli syndrome
Subjectively: periodical itching or burning pain E. Gilbert’s syndrome
in urethra. Two months ago pain in knee join
developed. Possibility of trauma or exposure 170. A 22-year-old patient complains
to cold is denied by the patient. During the of 8-months-long delay of menstruation.
last week eye discomfort is noted - lacrimation Anamnesis: menarche since the age of 12,5.
and itching. What provisional diagnosis can be Since the age of 18 menstruations are irregular.
suggested? No pregnancies. Mammary glands have normal
development; when the nipples are pressed,
A. Reactive arthritis milk drops are discharged. On gynecological
B. Rheumatoid arthritis examination: moderate uterus hypoplasia. On
C. Seasonal pollinosis hormonal examination: prolactin level exceeds
D. Bacterial nonspecific urethral conjunctivitis the norm two times. On computed tomogram
E. URTI with conjunctiva and joints affected of the sellar region: a space-occupying lesion
4 mm in diameter is detected. The most likely
167. A 19-year-old patient complains of severe diagnosis is:
pain in axillary crease. Condition onset was a
week ago after her swimming in cold river and
epilation. The next day painful "boil"appeared
that was becoming larger every day and
became a plum-sized tumor. Upon examinati-
on nodular conical growths joined together
Крок 2 Medicine (англомовний варiант, iноземнi студенти) 2015 рiк 21

A. Pituitary tumor 175. An emergency doctor has diagnosed a 32-


B. Lactation amenorrhea year-old woman with generalized convulsive
C. Stein–Leventhal syndrome (Polycystic ovary status epilepticus. The deterioration in the pati-
syndrome) ent’s condition is caused by a sudden gap in the
D. Sheehan’s syndrome epilepsy treatment. Specify the doctor’s further
E. Cushing’s disease tactics:
171. A 2-year-old child in a satisfactory conditi- A. Hospitalization in the intensive care unit
on periodically presents with moderate protei- B. Hospitalization in the department of
nuria, microhematuria. US results: the left ki- neurology
dney is undetectable, the right one is enlarged, C. Hospitalization in the department of
there are signs of double pyelocaliceal system. neurosurgery
What investigation is required to specify the D. Outpatient monitoring by a neuropathologist
diagnosis? E. Outpatient monitoring by a neurosurgeon
A. Excretory urography 176. A 19-year-old patient complains of
B. Micturating cystography dyspnea during physical exertion. He often has
C. Retrograde urography bronchitis and pneumonia. Cardiac murmur
D. Doppler study of renal vessels has been observed since his childhood. On
E. Radioisotope renal scan auscultation: there is splitting of II heart sound
over pulmonary arteria, systolic murmur in the
172. For the last 3 years a 12-year-old boy has third intercostal space near the left sternum
been suffering from stomachache, abdominal margin. ECG test shows right bundle-branch
distension, nausea, periodical liquid fatty stool, block. What is the provisional diagnosis?
grey in color, with rotten smell. On palpation:
pain in the epigastrium, Desjardins’ pancreatic A. Interatrial septum defect
point and Chauffard’s triangle; positive Mayo- B. Open arterial duct
Robson’s sign. Insufficiency of pancreas exocri- C. Coarctation of aorta
ne function is suspected. What method is the D. Aortic stenosis
most informative for pancreas exocrine functi- E. Mitral valve insufficiency
on assessment?
177. A 30-year-old patient was in a car accident.
A. Detection of elastase-1 in feces He is unconscious, pale, has thready pulse. In
B. Determining tripsin content in blood serum the middle third of the right thigh there is
C. Pancreas echography an extensive laceration with ongoing profuse
D. Determining amylase content in blood and external arterial bleeding. What urgent actions
urine must be taken to save the life of the patient?
E. Scatological test
A. Tourniquet above the wound of the right
173. A 34-year-old patient was delivered to thigh
a first-aid center with open fracture of the B. Tourniquet below the wound of the right
lower leg bones. Upon examination bleeding thigh
is stated: blood flows in pulsating bursts. What C. Artificial lung ventilation
actions should be taken at this stage of medical D. Precordial thump
aid? E. Plaster bar
A. Apply tourniquet to the thigh proximally to 178. A 75-year-old male patient complains of
the place of hemorrhage and deliver the patient slight pain in the right iliac region. The abdomi-
to operating room nal pain arose 6 days ago and was accompani-
B. Provide immobilization and deliver the ed by nausea. Surgical examination revealed
patient to a hospital moist tongue, heart rate - 76 bpm. BP - 130/80
C. Apply tourniquet to the thigh distally to the mm Hg. Abdomen is soft, slightly painful in
place of hemorrhage and deliver the patient to the right iliac region on deep palpation, the
operating room symptoms of the peritoneum irritation are
D. Apply pressure dressing to stop hemorrhage doubtful. In blood: RBCs - 4, 0 · 1012 /l, Hb-
and deliver the patient to operating room 135 g/l, WBCs - 9, 5 · 109 /l, stab neutrophiles
E. Deliver the patient to operating room - 5%, segmentonuclear - 52%, lymphocytes -
174. A 26-year-old patient with affective bi- 38%, monocytes - 5%, ESR - 20 mm/h. Specify
polar disorder has developed a condition mani- the doctor’s further tactics:
fested by mood improvement, behavioural A. Emergency operation for acute appendicitis
and sexual hyperactivity, verbosity, active body B. Hospitalization, dynamic surveillance
language, reduced need for sleep. Which of the C. Send the patient home
following drugs are most effective in this case? D. Refer the patient to a district therapist
A. Neuroleptics with a sedative effect E. Administration of additional examination:
B. Antidepressants with an activating effect abdominal ultrasound, x-ray contrast study of
C. Neuroleptics with an activating effect the gastrointestinal tract
D. Tranquilizers 179. On the 10th day postpartum a puerperant
E. Antidepressants with a sedative effect woman complains of pain and heaviness in
Крок 2 Medicine (англомовний варiант, iноземнi студенти) 2015 рiк 22

the left mammary gland. Body temperature is 183. A 70-year-old patient consulted a doctor
38, 8o C , Ps- 94 bpm. The left mammary gland is about arrhythmic cardiac activity, dyspnea.
edematic, the supero-external quadrant of skin Objectively: BP - 150/90 mm Hg, extrasystole
is hyperemic. Fluctuation symptom is absent. arrhythmia (10-12 beats per minute), left
The nipples discharge drops of milk when ventricular systolic dysfunction (ejection fracti-
pressed. What is a doctor’s further tactics? on at the rate of 42%). Which of anti-
arrhythmic drugs should be administered as
A. Antibiotic therapy, immobilization and initial therapy in this case?
expression of breast milk
B. Compress to both mammary glands A. Amiodarone
C. Inhibition of lactation B. Flecainide
D. Physiotherapy C. Encainide
E. Opening of the abscess and drainage of the D. Moracizine
mammary gland E. Digoxin
180. An 8-year-old child was hospitalized 184. A 45-year-old male patient complains of
for fever up to 39, 8o C , inertness, moderate acute pain in his right side irradiating to the
headache, vomiting. Examination revealed right thigh and crotch. The patient claims also
meningeal symptoms. Lumbar puncture was to have frequent urination with urine which
performed. The obtained fluid had raised resembles a "meat slops"in color. The patient
opening pressure, it was transparent, with has no previous history of this condition. There
the cell count of 450 cells per 1 mcL (mai- is costovertebral angle tenderness on the right
nly lymphocytes - 90%), glucose level of 2,6 (positive Pasternatsky’s symptom). What is the
mmol/l. What causative agent might have most likely diagnosis?
caused the disease in the child?
A. Urolithiasis
A. Enterovirus B. Acute appendicitis
B. Meningococcus C. Acute pyelonephritis
C. Koch’s bacillus D. Acute cholecystitis. Renal colic
D. Staphylococcus E. Acute pancreatitis
E. Pneumococcus
185. A 38-year-old man works within the
181. A 25-year-old patient during self- range of ionizing radiation. At a routine medi-
examination detected a tumor in the upper cal examination he presents no problems. In
external quadrant of the right mammary gland. blood: RBCs - 4, 5 · 1012 /l, Hb- 80 g/l, WBCs -
On palpation: painless, dense, mobile growth 2
cm in diameter is detected in the mammary 2, 8 · 109 /l, thrombocytes - 30 · 109 /l. Decide if
gland; no changes in the peripheral lymph this person can work with sources of ionizing
nodes are observed. On mammary glands US: radiation:
in the upper external quadrant of the right A. Working with radioactive substances and
mammary gland there is a space-occupying other sources of ionizing radiation is contraindi-
lesion of increased echogenicity 21х18 mm in cated
size. The most likely diagnosis is: B. The patient is allowed to work with radioacti-
A. Fibrous adenoma ve substances
B. Lacteal cyst C. The patient can only work with radioactive
C. Diffuse mastopathy substances of low activity
D. Breast cancer D. The patient can be allowed to work after an
E. Mastitis extended medical examination
E. The patient is allowed to work with radioacti-
182. A 20-year-old woman complains of feeling ve substances for the limited period of time
of air shortage, lingering dull pain in the heart
area, irritability. Objectively: general condition 186. A maternity patient breastfeeding for 1,5
is satisfactory, heart rate lability, BP is 130/60 weeks has attended a doctor. She considers
mm Hg. ECG reveals disruption of repolari- the onset of her disease to be when proporti-
zation proceses. The patient is diagnosed with onal breast engorgement occurred. Mammary
somatoform autonomic dysfunction of cardi- glands are painful. Body temperature is
al type. Specify the conditions of the patient 36, 6o C . Expression of breast milk is hindered.
treatment: The most likely diagnosis is:

A. Out-patient treatment A. Lactostasis


B. In-patient treatment in therapeutics B. Infiltrative mastitis
department C. Suppurative mastitis
C. In-patient treatment in cardiology D. Chronic cystic mastitis
department E. Gangrenous mastitis
D. In-patient treatment in cardiac surgery 187. A 12-year-old girl complains of general
department weakness, rise of body temperature up to
E. In-patient treatment in psychiatric 38, 2o C , pain and swelling of knee joints, feeli-
department ng of cardiac rhythm disruption. The chi-
Крок 2 Medicine (англомовний варiант, iноземнi студенти) 2015 рiк 23

ld had tonsillitis 3 weeks ago. The knee joi- ultrastructural change of ciliated epithelium.
nts are swollen, local raise of temperature is What is the basis of the given syndrome?
observed, mobility is reduced. Heart sounds
are weakened, extrasystole is present; at the A. Primary ciliary diskinesia
cardiac apex systolic noise can be auscultated, B. Proteoglycans insufficiency
which is not conducted to the left axillary regi- C. Surfactant deficit
on. ESR is 38 mm/h. CRP 2+. Antistreptolysin- D. Muscle cells atony
O titers are 400. The most likely disease is: E. Alpha-1-antitrypsin deficiency

A. Acute rheumatic fever 192. A newborn infant (the first labor, lasted
B. Somatoform autonomic dysfunction for 26 hours) is 1-day-old, postmature; body
C. Non-rheumatic carditis weight is 3850 g; body length is 52 cm. Deli-
D. Juvenile rheumatoid arthritis very was performed by applying obstetrical
E. Reactive arthritis forceps in sincipital presentation, Apgar score
is 1/3. The face is bluish-pale. The head is
188. A 60-year-old man has a diet consisting of thrown back; severe birth trauma is present; the
unvaried food staples: mostly cereals, potato, infant is excitable, shrill "cerebral scream"is
pasta; few vegetables and little fats (especially present; the eyes are half-open; facial expressi-
animal fats). During medical examination he on is attentive; hyperesthesia, hypersthenia and
complains of deterioration of his twilight visi- readiness for convulsions are present. Liquor
on. This condition can be caused by lack of: has high content of erythrocytes, lymphocytic
cytosis occurs. The most likely diagnosis is:
A. Retinol
B. Amino acids A. Subarachnoid hemorrhage
C. Fats B. Epidural hemorrhage
D. Calcium C. Subdural hemorrhage
E. Carbohydrates D. Intraventricular hemorrhage
E. Intracerebral hemorrhage
189. A 45-year-old patient was hospitalised
in surgical in-patient unit with intermuscular 193. A 74-year-old patient visited a urologist
phlegmon of the right thigh on the 6th day from with complaints of pain above the pubis and
the onset of disease. Abscess formed under the inability to urinate for 8 hours. At home he
fascia lata was diagnosed, lanced and widely had taken antispasmodics and had a warm bath
drained, necrotic tissues were removed. Anti- but no improvement occurred. Objectively:
biotic therapy with cephalosporines and the abdomen is soft and painful above the pubis;
III generation fluoroquinolones was prescri- dullness of percussion sound is observed above
bed, as well as immune corrective and detoxifi- the pubis. Murphy’s (Pasternatski’s) punch sign
cation infusion therapy (2,5 liters per day), di- is negative on the both sides. What condition
uresis stimulation, UV irradiation of blood and does the patient have?
plasmapheresis. What is the main component
of sepsis prevention? A. Acute urinary retention
B. Paradoxal ischuria
A. Surgical invasion C. Shronic urinary retention
B. General antibiotic therapy D. Anuria
C. Topical antibiotic therapy E. Oliguria
D. Immune correction therapy
E. Detoxification therapy 194. A 37-year-old patient has been undergoing
treatment of diskogenic lumbosacral radiculitis
190. An Rh-negative woman with 32-week- for a month. There is skin numbness observed
long term of pregnancy has been exami- at the lateral surface of the right lower extremi-
ned. It was observed that Rh-antibodies ti- ty, Achilles jerk is absent. MRI of lumbar spine
ter had increased four times within the last revealed intervertebral disk L5 -S1 prolapse up
2 weeks and was 1:64. First two pregnancies to 8 mm. Choose the further tactics for patient
ended in antenatal death of fetus caused by treatment:
hemolytic disease. What tactics of pregnancy
management should be chosen? A. Surgical extraction of intervertebral disk
B. Conservative treatment
A. Preterm delivery C. Stabilizing spinal surgery
B. Delivery at 37 weeks term D. Corset
C. Rh-antibody test in 2 weeks; if Rh-antibodies E. Manual therapy
increase in number conduct delivery
D. Introduction of anti-Rh immunoglobulin 195. A 40-year-old patient complains of
E. US examination to determine signs of fetal constant moderate pain in the lumbar spine
erythroblastosis and significantly reduced mobility. The pati-
ent has been suffering from this condition for
191. A 2-year-old child has been suffering the last 7 years since pain appeared first in the
since birth from recurring inflammatory di- sacrum area. X-ray: ankylosis of sacroiliac arti-
seases of lungs, purulent pansinusites, heari- culation, significant narrowing of intervertebral
ng deterioration, multiple cylindrical bronchi- joint fissures of lumbar vertebrae and calcifi-
ectases. Dextrocardia is observed. On biopsy: cation of spinal ligaments. What pathology is
Крок 2 Medicine (англомовний варiант, iноземнi студенти) 2015 рiк 24

most likely to cause such X-ray image? 198. A 30-year-old woman complains of
irregular copious painful menstruations, pain
A. Ankylosing spondylitis irradiates to the rectum. Anamnesis states 10-
B. Spinal tuberculosis year-long infertility. On bimanual examination:
C. Spinal osteochondrosis uterus is of normal size; uterine appendages
D. Vertebral osteochondropathy on the both sides are corded, with rectricted
E. Rheumatoid arthritis mobility, painful; there are dense nodular pai-
nful growths detected in the posterior fornix. A
196. A 7-year-old child became ill again 2 doctor suspects endometriosis. What method
weeks after he had tonsillitis. There are the allows to verify this diagnosis?
following complaints: temperature rise up to
38o C , hemorrhagic rash on the extremities, A. Laparoscopy
enlargement of the ankle joints. Blood test: B. Diagnostic curettage of uterine cavity
hemoglobin is 120 g/l, platelets are 170 · 109 /l, C. Paracentesis of posterior fornix
ESR is 30 mm/h. Urine test: proteinuria up D. Uterine probing
to 0,7 g/l, cylinders - 5-6 in the field of visi- E. Hysteroscopy
on, erythrocytes - 8-10 in the field of vision.
What mechanism of hemorrhagic syndrome is 199. A 14-year-old girl complains of tooth
present in the given case? caries; the tooth should be filled. Anamnesis
states that artificial mitral valve was installed 2
A. Vessel wall damage caused by immune years ago due to mitral insifficiency. What anti-
complexes bacterial drug should be prescribed to prevent
B. Platelet dysfunction infective endocarditis?
C. Suppression of hematopoietic stem cells
D. Decrease of adhesive-aggregative function A. Amoxicillin
of platelets B. Lincomycin
E. Vessel wall damage caused by bacteria C. Ceftriaxone
D. Erythromycin
197. A 48-year-old patient was delivered to a E. Midecamycin
hospital in-patient unit with uterine bleeding
that occurred after the 2-week-long delay of 200. A 30-year-old woman complains of inferti-
menstruation. Anamnesis states single birth. lity during her 10-year-long married life.
Examination of the uterine cervix with mirrors Menstruations occur since she was 14 and are
revealed no pathologies. On bimanual exami- irregular, with delays up to a month and longer.
nation: uterus is of normal size, painless, mobi- Body mass is excessive. Hirsutism is observed.
le; uterine appendages have no changes. Di- On bimanual examination: uterine body is
scharge is bloody and copious. What primary decreased in size; ovaries are increased in si-
hemostatic measure should be taken in the gi- ze, dense, painless, and mobile. The most likely
ven case? diagnosis is:
A. Fractional curettage of uterine cavity A. Stein–Leventhal syndrome (Polycystic ovary
B. Hormonal hemostasis syndrome)
C. Hemostatics B. Follicular cyst of ovaries
D. Uterine tamponade C. Genital endometriosis
E. Uterotonics D. Genital tuberculosis
E. Inflammatory tumor of ovaries
Крок 2 Medicine (англомовний варiант, iноземнi студенти) 2016 рiк 1

1. You are a doctor on duty. A patient after Common blood analysis: erythrocytes - 2,8
a successful resuscitation (drowning) was deli- Т/l, Hb- 80 g/l, color index - 0,78, anisocytosis,
vered to an admission room. BP is 90/60 mm poikilocythemia, serum iron - 10 mcmol/l.
Hg, heart rate is 120/min., respiration rate is What diagnosis is most likely?
26/min. The patient is unconscious, pupils are
moderately dilated, general clonic and tonic A. Iron-deficiency anemia
convulsions are observed. Make the diagnosis: B. B12 -deficient anemia
C. Autoimmune hemolytic anemia
A. Postresuscitation disease D. Aplastic anemia
B. Apparent death E. Hypoplastic anemia
C. Coma of unknown origin
D. Unconsciousness 6. A 24-year-old patient visited a doctor
E. Vegetative state complaining of enlargement of his submaxi-
llary lymph nodes. Objectively: submaxi-
2. A 32-year-old welder complains of llary, axillary and inguinal lymph nodes
weakness and fever. His illness initially are enlarged. Chest X-ray shows: enlarged
presented as tonsillitis one month earlier. On lymph nodes of mediastinum. Blood test:
examination: BT- 38, 9o C , RR- 24/min., HR- erythrocytes - 3, 4 · 1012 /l, Hb- 100 g/l, blood
100/min., BP- 100/70 mm Hg, hemorrhages colour index - 0,88, platelets - 190 · 109 /l,
on the legs, enlargement of the lymph nodes. leucocytes - 7, 5·109 /l, eosinophiles - 8%, band
CBC shows Hb- 70 g/l, RBC- 2, 2 · 1012 /l, neutrophiles - 2%, segmented neutrophiles -
WBC- 3, 0 · 109 /l with 32% of blasts, 1% of 67%, lymphocytes - 23%, ESR- 22 mm/hour.
eosinophiles, 3% of bands, 36% of segments, What test must be prescribed to verify the
20% of lymphocytes, and 8% of monocytes, cause of lymphadenopathy?
ESR- 47 mm/hour. What is the cause of
anemia? A. Open biopsy of lymph nodes
B. Ultrasonography of abdominal cavity
A. Acute leukemia C. Mediastinum tomography
B. Chronic lympholeukemia D. Puncture biopsy of lymph nodes
C. Aplastic anema E. Sternal puncture
D. B12 -deficient anemia
E. Chronic hemolytic anemia 7. A 32 year old patient complains of cardiac
irregularities, dizziness, dyspnea at physical
3. A regional cardiologist is tasked with the exertion. He has never suffered from such
development of a plan for medioprophylactic condition before. Objectively: Ps- 74/min.,
measures aimed at decrease of cardiovascular rhythmic. BP- 130/80 mm Hg. Auscultati-
mortality. What measures should be planned on revealed systolic murmur above aorta,
for secondary prevention? the first heart sound was normal. ECG
showed hypertrophy of the left ventricle, si-
A. Prevention of recurrences and complicati- gns of repolarization disturbance in the I , V5
ons and V6 leads. Echocardiogram revealed that
B. Referring patients for sanatorium-and-spa interventricular septum was 2 cm. What is the
treatment most probable diagnosis?
C. Prevention of diseases
D. Referring patients for in-patient treatment A. Hypertrophic cardiomyopathy
E. Optimization of life style and living conditi- B. Aortic stenosis
ons C. Essential hypertension
D. Myocardium infarction
4. An 8-year-old boy developed a temperature E. Coarctation of aorta
of 37, 5o C two days after his recovery from
the case of URTI. He complains of suffocati- 8. A 35-year-old patient’s wound with
on, heart pain. Objectively: the skin is pale, suppurative focus was surgically cleaned.
tachycardia, the I heart sound is weakened, On the 8th day after the surgery the wound
short systolyc murmur in the 4th intercostal cleared from its purulo-necrotic content and
area near the left edge of the breastbone. granulations appeared. However, against the
What heart disorder such clincal presentati- bacground of antibacterial therapy the body
on is characteristic of? temperature keeps at 38,5-39,5o C . There are
chills, excessive sweating, euphoria, heart rate
A. Nonrheumatic myocarditis is 120/min. What complication of local pyoi-
B. Primary rheumatic carditis nflammatory process can it be?
C. Myocardiodystrophy
D. Fallot’s tetrad A. Sepsis
E. Cardiomyopathy B. Purulent absorption fever
C. Trombophlebitis
5. A woman complains of muscle weakness D. Meningitis
and general fatigue, dyspnea, vertigo, bri- E. Pneumonia
ttleness of her hair and nails, an urge to
eat chalk. Anamnesis states uterine fibroid. 9. A 60-year-old woman has been sufferi-
Крок 2 Medicine (англомовний варiант, iноземнi студенти) 2016 рiк 2

ng from arterial hypertension for 15 years. A. Tuberculous spondylitis of the thoracic


After recurrent stroke she started complaini- spine
ng of unmotivated bad mood, problems with B. Spinal tumor
attention concentration; she forgets to close C. Multiple sclerosis
the entrance door, cannot recall events of D. Metastases into the spine
the past day. Computer tomography shows E. Osteochondrosis
areas of postinfarction changes in the cortical
postfrontal areas. What is the most probable 13. A pregnant woman is 28 years old.
diagnosis? Anamnesis: accelerated labor complicated by
the II degree cervical rupture. The followi-
A. Vascular dementia ng two pregnancies resulted in spontaneous
B. Alzheimer’s disease abortions at the terms of 12 and 14 weeks.
C. Huntington’s disease On mirror examination: the uterine cervix
D. Pick’s disease is scarred from previous ruptures at 9 and 3
E. Dissociative amnesia hours, the cervical canal is gaping. On vagi-
nal examination: the cervix is 2 cm long, the
10. Clinic of a research instutute for occupati- external orifice is open 1 cm wide, the internal
onal diseases examined a worker who works orifice is half-open; the uterus is enlarged to
at a concentration plant and diagnosed him the 12th week of pregnancy, soft, mobile, pai-
with chronic dust bronchitis. The case is nless, the appendages are without changes.
investigated by a commission including the What diagnosis would you make?
representatives of: the plant, clinic, local
SES, department of Social Insurance Fund, A. Isthmico-cervical insufficiency, habitual
trade union. According to the ”regulation on noncarrying of pregnancy
investigation of. . . ”, the commission should be B. Threatened spontaneous abortion
headed by the representative of the following C. Incipient abortion, habitual noncarrying of
authority: pregnancy
D. Cervical hysteromyoma, habitual noncarryi-
A. Local SES ng of pregnancy
B. Plant E. Cervical pregnancy, 12 weeks
C. Social Insurance Fund
D. Trade union 14. A 56-year-old patient complains of pain in
E. Clinic the epigastrium after eating, eructation, loss
of appetite, slight loss of weight, fatigabili-
11. A 37-year-old woman complains of ty. The patient smokes; no excessive alcohol
headaches, nausea, vomiting, spasms. The consumption. Objectively: pale mucosa, BP-
onset of the disease occurred the day before 110/70 mm Hg. The tongue is ”lacquered”.
due to her overexposure to cold. Objecti- The abdomen is soft, sensitive in the epi-
vely: fever up to 40o C ; somnolence; rigid gastric area. Blood test: erythrocytes - 3,0 T/l,
neck; Kernig’s symptom is positive on the Hb- 110 g/l, color index - 1,1; macrocytosis;
both sides; general hyperesthesia. Blood test: leukocytes - 5,5 g/l, ESR- 13 mm/hour. On
leucocytosis, increased ESR. Cerebrospinal fibrogastroduodenoscopy: atrophy of fundic
fluid is turbid, yellow-tinted. What changes of mucosa. What pathogenesis does this disorder
the cerebrospinal fluid are most likely? have?
A. Neutrophilic pleocytosis A. Producing antibodies to parietal cells
B. Lymphocytic pleocytosis B. Н.pylori persistence
C. Blood in the cerebrospinal fluid C. Alimentary factor
D. Xanthochromia in the cerebrospinal fluid D. Chemical factor
E. Albuminocytological dissociation E. Gastropathic effect
12. A 48-year-old woman complains of pain 15. A 26-year-old secundipara at 40 weeks of
in the thoracic spine, sensitivity disorder gestation arrived at a maternity ward after
in the lower body, disrupted motor functi- the beginning of labor activity. The bursti-
on of the lower limbs, body temperature ng of waters occurred 2 hours prior. The
rise up to 37, 5o C . She has been suffering fetus was in a longitudinal lie with cephalic
from this condition for 3 years. Treatment presentation. Abdominal circumference was
by various specialists was ineffective. X-ray 100 cm, fundal height - 42 cm. Contracti-
reveals destruction of adjacent surfaces of ons occurred every 4-5 minutes and lasted
the VIII and IX vertebral bodies. In the right 25 seconds each. Internal obstetric examinati-
paravertebral area at the level of lesion there on revealed cervical effacement, opening by 4
is an additional soft tissue shadow. What di- cm. Fetal bladder was absent. Fetal head was
agnosis is most likely? pressed against the pelvic inlet. What compli-
cation arose in the childbirth?
Крок 2 Medicine (англомовний варiант, iноземнi студенти) 2016 рiк 3

A. Early amniorrhea A. Peripheral lung cancer


B. Primary uterine inertia B. Bronchiectasis
C. Secondary uterine inertia C. Pneumonia
D. Discoordinated labor D. Middle lobe syndrome
E. Clinically narrow pelvis E. Interlobular pleurisy
16. Bacterial analysis of air in a living space in 20. A 62-year-old patient has been hospitali-
winter period by means of Krotov’s apparatus zed with complaints of pain in the thorax
revealed that total number of microorganisms on the right during breathing, dyspnea, dry
in 1 m3 of air was 7200. What is the allowed cough. Ten days ago he slipped and fell hitti-
number of microorganisms for the air to be ng his right side. On examintaion: the pati-
characterized as ”pure”? ent lies on the left side. The right side of the
thorax lags during breathing. On the right
A. Up to 4500 there are crepitation and pain in the III-IV ri-
B. Up to 2500 bs. Dullness of percussion sound and sharply
C. Up to 3500 diminished breath sounds can be observed.
D. Up to 5500 On X-ray: signs of exudate, fracture of the III-
E. Up to 7500 IV ribs. On pleurocentesis: blood is detected.
Choose the further tactics:
17. A 24-year-old patient received a
puncture injury below the Poupart’s ligament A. Transfer to a thoracic surgery department
accompanied by intense arterial bleeding. The B. Prescribe conservative therapy
best method to temporarily stop the bleeding C. Recurrent pleurocentesis
in the patient would be: D. Fixed bandage of the rib cage
E. Refer to a traumatologist
A. Compression band
B. Esmarch’s tourniquet 21. A 29-year-old female patient complains of
C. Maximum limb bending dyspnea, heaviness and chest pain on the ri-
D. Compressing a blood vessel with a clamp ght, body temperature rise up to 37, 2o C . The
E. Wound suturing disease is associated with a chest trauma recei-
ved 4 days ago. Objectively: skin is pale and
18. A 42-year-old woman has been hospitali- moist. Ps- 90/min., regular. Palpation reveals
zed with complaints of intense pain attacks a dull sound on the right, auscultation reveals
in the lumbar and right iliac areas, which significantly weakened vesicular breathing.
irradiate to the vulvar lips, frequent urination, In blood: RBCs- 2, 8 · 1012 /l, colour index -
nausea. The pain onset was acute. Objecti-
vely: the abdomen is soft, moderately pai- 0,9, Hb- 100 g/l, WBCs- 8, 0 · 109 /l, ESR- 17
nful in the right subcostal area, costovertebral mm/hour. What results of diagnostic puncture
angle tenderness on the right. Common uri- of the pleural cavity can be expected?
ne analysis: specific gravity - 1016, traces of A. Haemorrhagic punctate
protein, leukocytes - 6-8 in the vision field, B. Chylous liquid
erythrocytes - 12-16 in the vision field, fresh. C. Exudate
What diagnosis can be made? D. Transudate
A. Right-sided renal colic E. Purulent punctate
B. Acute right-sided pyelonephritis 22. Caries morbidity rate is 89% among resi-
C. Acute right-sided adnexitis dents of a community. It is determined that
D. Acute cholecystitis fluorine content in water is 0,1 mg/l. What
E. Acute appendicitis preventive measures should be taken?
19. A 29-year-old patient works as a A. Water fluorination
motor mechanic. Anamnesis shows frequent B. Tooth brushing
exposure to cold, exacerbation of chronic C. Fluorine inhalations
bronchitis attended by cough with relati- D. Sealant application
vly small amount of mucopurulent sputum, E. Introduce more vegetables to the diet
subfebrility, sometimes joined by hemoptysis
and pain in the right side of chest. Breathing 23. Examination of a group of persons living
is vesicular. X-ray shows shadows and sharp on the same territory revealed the following
decrease in size of the lower lobe distinctly vi- common symptoms: dark-yellow pigmentati-
sible on the X-ray image as a streak 2-3 cm wi- on of the tooth enamel, diffuse osteoporosis
de situated at the angle from lung root to the of bone apparatus, ossification of ligaments
frontal costodiaphragmatic recess. The most and joints, functional disorders of the central
likely diagnosis is: nervous system. This condition may be caused
by the excessive concentration of the followi-
ng microelement in food or drinking water:
Крок 2 Medicine (англомовний варiант, iноземнi студенти) 2016 рiк 4

A. Fluorine aside from tachycardia (102/min.) no other


B. Copper changes. On ECG: pathologic wave Q in I,
C. Nickel аVL, QS in V1, V2, V3 leads and ’domed’ ST
D. Iodine elevation with negative T. What diagnosis is
E. Cesium most likely?
24. In a pre-school educational establishment A. Acute left ventricular anterior myocardial
the menu consists of the following dishes: milk infarction
porridge from buckwheat, pasta with minced B. Variant angina pectoris
meat, cucumber salad, kissel (thin berry jelly), C. Aortic dissection
rye bread. What dish should be excluded from D. Acute left ventricular posterior myocardial
the menu? infarction
E. Exudative pericarditis
A. Pasta with minced meat
B. Milk porridge from buckwheat 29. A 48-year-old woman has thermal burns
C. Kissel (thin berry jelly) of both hands. The epidermis of the palms
D. Rye bread and backs of her hands is exfoliating, and bli-
E. Cucumber salad sters filled with serous liquid are forming. The
forearms are intact. What diagnosis is most
25. An infant has been born at the 41st week likely?
of gestation. The pregnancy was complicated
with severe gestosis of the second semester. A. 2-3A degree thermal burn
The weight of the baby is 2400 g, the height is B. 4 degree thermal burn
50 cm. Objectively: the skin is flabby, the layer C. 1 degree thermal burn
of subcutaneous fat is thin, hypomyotonia, D. 3B degree thermal burn
neonatal reflexes are weak. The internal E. 1-2 degree thermal burn
organs are without pathologic changes. This
newborn can be estimated as a: 30. A 39-year-old patient complains of morni-
ng headache, appetite loss, nausea, morning
A. Full-term infant with prenatal growth vomiting, periodic nasal haemorrhages. The
retardation patient had a case of acute glomerulonephri-
B. Premature infant tis at the age of 15. Examination revealed rise
C. Immature infant of arterial pressure up to 220/130 mm Hg, skin
D. Postmature infant haemorrhages on his arms and legs, pallor of
E. Full-term infant with normal body weight skin and mucous membranes. What biochemi-
cal parameter is most important for making
26. A patient suffering from infiltrati- diagnosis in this case?
ve pulmonary tuberculosis was prescribed
streptomycin, rifampicin, isoniazid, pyrazi- A. Blood creatinine
namide, vitamin C. One month after the begi- B. Blood bilirubin
nning of the treatment the patient started C. Blood sodium
complaining of reduced hearing and tinnitus. D. Uric acid
What drug has such a side effect? E. Fibrinogen
A. Streptomycin 31. A 30-year-old patient, who has
B. Isoniazid been suffering from headaches, suddenly
C. Rifampicin developed extreme headache after lifting a
D. Pyrazinamide heavy load, as if he had been hit over the
E. Vitamin C head. Nausea, vomiting, and slight dizzi-
ness are observed. In a day he developed
27. A woman has developed sudden thoracic pronounced meningeal syndrome and body
pain on the right with expectoration of pi- temperature up to 37, 6o C . A doctor suspects
nk sputum and body temperature rise up to subarachnoid hemorrhage. What additional
37, 7o C on the 4th day after the surgery for examination is necessary to confirm this di-
cystoma of the right ovary. On lung examinati- agnosis?
on: dullness of the lung sound on the lower
right is observed. Isolated moist crackles can A. Lumbar puncture with investigation of the
be auscultated in the same area. What compli- spinal fluid
cation is the most likely? B. Skull X-ray
C. Computed tomography of the brain
A. Pulmonary infarction D. Rheoencephalography
B. Pneumonia E. Angiography of the brain vessels
C. Pulmonary abscess
D. Exudative pleurisy 32. A worker of a blowing shop complains of
E. Pneumothorax headache, irritability, sight impairment - he
sees everything as if through a ”net”. Objecti-
28. A 58-year-old patient was delivered to an vely: hyperemic sclera, thickened cornea,
admission room with complaints of pain in the decreased opacity of pupils, visual acuity is
thorax on the left. On clinical examination:
Крок 2 Medicine (англомовний варiант, iноземнi студенти) 2016 рiк 5

0,8 in the left eye, 0,7 in the right eye. The operator?
worker uses no means of personal protection.
What diagnosis is most likely? A. Photoelectric ophthalmia
B. Heatstroke
A. Cataract C. Vegetative-vascular dystonia
B. Conjunctivitis D. Chronic overheating
C. Keratitis E. Pneumoconiosis
D. Blepharospasm
E. Progressive myopia 37. A 48-year-old patient was found to have
diffuse enlargement of the thyroid gland,
33. A 45-year-old woman is undergoing exophthalmia, weight loss of 4 kg in 2 months,
treatment for active rheumatism, combined sweating. Objectively: HR- 105/min., BP-
mitral valve failure. During her morning 140/70 mm Hg. Defecation act is normal.
procedures she suddenly sensed pain in the What kind of therapy is recommended in this
left hand, which was followed by numbness. case?
Pain and numbness continued to aggravate.
Objectively: the skin of the left hand is pale A. Mercazolil
and comparatively cold. Pulse in the hand B. Radioiodine
arteries is absent along the whole length. C. Propranolol
What treatment tactics is most efficient? D. Lugol’s solution
E. Thyroxine
A. Urgent embolectomy
B. Prescription of fibrinolytics and anti- 38. A woman addressed a doctor with
coagulants complaints of increased body temperature up
C. Prescription of antibiotics and antii- to 37, 8o C and moderately sore throat for the
nflammatory agents last 3 days. Objectively: mandibular lymph
D. Cardiac catheterization nodes are enlarged up to 3 cm. Palatine tonsils
E. Urgent thrombintimectomy are hypertrophied, covered with grey coating
that spreads to the uvula and anterior pillars
34. A 10-year-old boy had a case of viral of the fauces. What diagnosis is most likely?
hepatitis type B four years ago. Currently the
assumption was made about the formation of A. Oropharyngeal diphtheria
hepatic cirrhosis in the patient. What additi- B. Infectious mononucleosis
onal investigation can clarify the diagnosis? C. Pseudomembranous (Vincent’s) tonsillitis
D. Agranulocytosis
A. Renal needle biopsy E. Oropharyngeal candidiasis
B. Proteinogram
C. Echocholecystography 39. A 48-year-old man complains of constant
D. Markers of viral hepatitis type B pain in the upper abdomen, predominantly
E. Transaminase level measurement on the left, which aggravates after eating,
diarrhea, loss of weight. The patient has
35. A 40-year-old patient has acute onset alcohol use disorder. Two years ago he had
of disease caused by overexposure to cold. a case of acute pancreatitis. Blood amylase
Temperature has increased up to 39o C . is 4 g/hour·l. Feces analysis: steatorrhea,
Foul-smelling sputum is expectorated duri- creatorrhea. Blood sugar is 6,0 mmol/l. What
ng coughig. Various moist crackles can be treatment should be prescribed?
auscultated above the 3rd segment on the ri-
ght. Blood test: leukocytes - 15, 0 · 109 /l, stab A. Panzinorm forte (Pancreatin)
neutrophils - 12%, ESR- 52 mm/hour. On X- B. Insulin
ray: in the 3rd segment on the right there is a C. Gastrozepin (Pirenzepine)
focus of shadow 3 cm in diameter, low density, D. Contrykal (Aprotinin)
with fuzzy smooth margins and a clearing in E. No-Spa (Drotaverine)
its center. What disease is most likely in the
given case? 40. In 10 hours after eating canned
mushrooms a 27-year-old patient has
A. Pneumonia complicated by an abscess developed diplopia, bilateral ptosis, di-
B. Infiltrative tuberculosis srupted swallowing, shallow breathing with
C. Peripheral pulmonary cancer respiratory rate 40/min., muscle weakness,
D. Cystic echinococcosis enteroparesis. What measure should be taken
E. Pulmonary cyst first?
36. An electro-gas welding operator worki- A. Intubation of the trachea for artificial
ng at a machine workshop performs welding respiration
and cutting of metal, which is accompanied B. Gastrointestinal lavage
by intense UV-radiation. His welding station C. Introduction of antibotulinic serum
is equipped with efficient mechanical venti- D. Introduction of glucocorticosteroids
lation. What occupational disease is most E. Intravenous detoxication therapy
likely to develop in an electro-gas welding
41. A 32-year-old patient complains of
Крок 2 Medicine (англомовний варiант, iноземнi студенти) 2016 рiк 6

reddening, burning, and sensation of a foreign labor was diagnosed with cephalohematoma.
body in the right eye. The disease is acute. On On the 2nd day of life the child developed
examination: visual acuity of the both eyes jaundice; on the 3rd day of life there appeared
is 1,0. In the right eye there are hyperemy neurological changes: nystagmus, Graefe
and swelling of the conjunctiva, superficial syndrome. Urine is yellow, feces are golden-
injection. There is purulent discharge in the yellow. The mother’s blood group is А (II)
conjunctival sac. The cornea is clear. The color Rh− , the child’s - А (II) Rh+ . On the 3rd
and pattern of the iris are uncanged, the pupil day the results of the child’s blood test are as
is mobile. What diagnosis is most likely? follows: Hb- 200 g/l, erythrocytes - 6, 1 · 1012 /l,
A. Acute conjunctivitis blood bilirubin - 58 mcmol/l due to the
B. Acute iridocyclitis presence of its unconjugated fraction, Ht- 0,57.
C. Acute attack of glaucoma In this case the jaundice is caused by:
D. Foreign body of the cornea A. Craniocerebral birth injury
E. Acute dacryocystitis B. Physiologic jaundice
42. A patient is on the sick leave for 4 months C. Hemolytic disease of newborn
continuously from the date of injury. The D. Atresia of bile passages
treatment is going to last for another 1-2 E. Fetal hepatitis
months. Who has the right to extend the 46. On the 4th day after recovering from a
duration of medical certificate for this pati- cold a patient was hospitalized with complai-
ent? nts of solitary spittings of mucoid sputum.
A. Medical advisory commission after medico- On the 2nd day there was a single discharge
social expert commission examination of about 250 ml of purulent blood-streaked
B. Medical superintendent sputum. Objectively: the patient’s conditi-
C. Medical advisory commission after inpati- on is moderately severe. Respiratory rate -
ent treatment 28-30/min., Ps- 96/min., BP- 110/70 mm Hg.
D. District doctor upon agreement with a Respiration above the left lung is vesicular,
department chief weak above the right lung. There are vari-
E. Medico-social expert commission ous moist crackles above the lower lobe and
amphoric breath near the angle of scapula.
43. Monthly dysentery morbidity in the region What is the most likely diagnosis?
given in absolute figures is as follows: January
- 6; February - 9; March - 11; April - 10; May - A. Acute pulmonary abscess
16; June - 23; July - 19; August - 33; September B. Exudative pleuritis
- 58; October - 19; November - 11; December C. Acute focal pneumonia
- 5. Annual total is 220 cases. What graphic D. Pleural empyema
presentation would provide the best visual for E. Pyopneumothorax
monthly deviations of dysentery morbidity 47. A 46-year-old patient with temporari-
from the average? ly undetermined diagnosis was prescribed
A. Radar chart pleurocentesis based on the results of the X-
B. Map ray. The puncture yielded 1000 ml of a liquid
C. Cartogram with the following properties: clear, specific
D. Pie chart gravity - 1,010, protein content - 1%, Rivalta’s
E. Bar chart test is negative, erythrocytes - 2-3 in the field
of vision. What disorder are these pathologic
44. A 30-year-old woman complains of pain changes characteristic of?
in the heart area (”aching, piercing pain”) A. Cardiac failure
that arises primarily in the morning hours in B. Pleuropneumonia
autumn and spring. Pain irradiates into the C. Pleural mesothelioma
neck, back, abdomen and is attended by rapid D. Pulmonary tuberculosis
heart rate and low vital tonus. This condition E. Pulmonary cancer
occurs independently from physical exerti-
on. In the evening her condition improves. 48. A 14-year-old girl has been presenting wi-
Somatic and neurologic state and ECG have th irritability and tearfulness for about a year.
no pathologies. What pathology is likely to A year ago she was also found to have diffuse
result in such clinical presentation? enlargement of the thyroid gland (II grade).
A. Somatized depression This condition was regarded as a pubertal
B. Rest angina pectoris manifestation, the girl did not undergo any
C. Pseudoneurotic schizophrenia treatment. The girl’s irritability gradually gave
D. Somatoform autonomic dysfunction place to a complete apathy. The girl developed
E. Hypochondriacal depression puffy face, soft tissues pastosity, bradycardia,
constipations. Skin pallor and gland density
45. A full term baby born from the 1st progressed, the skin developed a waxen hue.
noncomplicated pregnancy with complicated What disease can be suspected?
Крок 2 Medicine (англомовний варiант, iноземнi студенти) 2016 рiк 7

A. Autoimmune thyroiditis 53. A 56-year-old patient with diffuse toxic


B. Diffuse toxic goiter goiter has ciliary arrhythmia, heart rate is 110-
C. Thyroid carcinoma 120/min., arterial hypertension, BP is 165/90
D. Subacute thyroiditis mm Hg. What drug besides Mercazolil (Thi-
E. Juvenile basophilism amazole) should be prescribed in this case?
49. In a maternity hospital a newborn had A. Propranolol
been presenting with cough attacks after eati- B. Radioactive iodine
ng. The child was discharged from the hospital C. Novocainamide (procainamide)
on the 18th day due to a case of pneumonia. D. Verapamil
During the further 1,5 months the child had E. Corinfar
2 cases of pneumonia. Periodically there are
cough attacks after eating, especially if the 54. A patient has been provisionally di-
child lies on the left side. Objectively: the II agnosed with pheochromocytoma at the stage
degree hypotrophy, isolated moist crackles, of intermission. BP is within norm, there is
dyspnea. Stool and diuresis are not disrupted. a tendency towards tachycardia. No urine
What diagnosis is most likely? pathologies. The decision has been made to
perform a provocative test with histamine.
A. Tracheoesophageal fistula What drug should be kept close at hand for
B. Mucoviscidosis emergency aid in case of positive test result?
C. Posthypoxic encephalopathy
D. Hernia of the esophageal opening A. Phentolamine
E. Tracheobronchomalacia B. Pipolphen
C. Nifedipine
50. A 26-year-old woman, who gave bi- D. Mesaton (Phenylephrine)
rth 7 months ago, has been suffering from E. Prednisolone
nausea, morning sickness, somnolence for
the last 2 weeks. The patient breasfeeds; no 55. A 54-year-old patient complains of
menstruation. She has been using no means of weakness, weight loss despite the unchanged
contraception. What method would be most appetite, frequent urination, skin itch for six
efficient in clarification of the diagnosis? months. Some time ago the patient underwent
treatment for furunculosis. She has not been
A. Ultrasound examined recently. Objectively: malnutriti-
B. Small pelvis radiography on, dry skin with scratch marks. Small lymph
C. Palpation of mammary glands and squeezi- nodes can be palpated in the axillary regions.
ng out colostrum Changes in the internal organs are absenr.
D. Bimanual abdominovaginal examination What test must be performed in the first
E. Mirror examination place?
51. A 2-year-old girl has a medical history A. Fasting blood sugar
of recurrent obstructive pneumonia. In the B. Complete blood count
lungs various moist and dry crackles can C. Endoscopy of stomach
be auscultated, breath sounds are dimini- D. Lymph node biopsy
shed. Sputum is thick, viscous and difficult E. Blood sterility testing
to expectorate. Drumstick fingers and physi-
cal developmental retardation are observed. 56. A child is 1 month 10 days old. ”Gushing”
What preliminary diagnosis can be made? vomiting has been observed since the age of 3
weeks. The vomit volume exceeds the volume
A. Pulmonary mucoviscidosis of the previous feeding. Objectively: the chi-
B. Recurrent bronchitis ld is inert. Skin elasticity and tissue turgor
C. Bronchial asthma are decreased. Hour-glass deformity sign is
D. Congenital polycystic lungs positive. The preliminary diagnosis is pyloric
E. Pulmonary tuberculosis stenosis. What treatment tactics should be
52. After a case of purulent otitis a 1-year-old chosen?
boy has developed pains in the upper third of A. Surgery
the left thigh, body temperature up to 39o C .
Objectively: swelling of the thigh in its upper B. Prescription of Cerucal (Metoclopramide)
third and smoothed out inguinal fold. The li- C. Atropinization
mb is in semiflexed position. Active and passi- D. Internal administration of Novocaine
ve movements are impossible due to severe E. Prescription of Pipolphen
pain. What diagnosis is most likely? 57. A 46-year-old woman complains of pain
A. Acute hematogenous osteomyelitis attacks in the right lumbar area, which irradi-
B. Acute coxitis ate to the lower abdomen, and nausea.
C. Intermuscular phlegmon This kind of pain attacks has never been
D. Osteosarcoma detected in the patient before. Survey X-
E. Brodie’s abscess ray of the abdominal cavity reveals no
pathologic shadows. Ultrasonic scan detects
Крок 2 Medicine (англомовний варiант, iноземнi студенти) 2016 рiк 8

a hyperechogenic growth 1,5 cm in diameter, 27 10 . What drug would be most efficient in
which reflects sound wave, in the enlarged ri- the treatment of this patient?
ght renal pelvis. What diagnosis is most likely?
A. Cryoprecipitate
A. Renal calculus B. Calcium chloride
B. Benign renal tumor C. Concentrated red cells
C. Renal cyst D. Aminocapronic acid
D. Renal tuberculosis E. Vicasol (Menadione)
E. Malignant renal tumor
62. A 67-year-old man complains of dyspnea
58. Several hours before, a 28-year-old pati- on exertion, attacks of retrosternal pain, di-
ent suddenly developed acute headache and zziness. He has no history of rheumatism.
repeated vomiting, then lost consciousness. Objectively: pale skin, acrocyanosis. There are
Objectively: focal neurological symptoms crackles in the lower lungs. There is systolic
were not found. Pronounced meningeal thrill in the II intercostal space on the ri-
symptoms were revealed. BP- 120/80 mm ght, coarse systolic murmur conducted to the
Hg. According to clinical and liquorologi- vessels of neck. BP- 130/90 mm Hg, heart rate
cal findings the patient was diagnosed wi- - 90/min., regular rhythm. The liver extends 5
th subarachnoid haemorrhage. After admi- cm from under the edge of costal arch, shin
nistration of dehydrants the patient’s condi- edemas are present. Specify the suspected
tion somewhat improved. What is the main valvular defect:
component of further emergency care?
A. Aortic stenosis
A. Coagulants B. Pulmonary artery stenosis
B. Anticoagulants C. Mitral insufficiency
C. Antiaggregants D. Ventricular septal defect
D. Fibrinolytics E. Tricuspid regurgitation
E. Corticosteroids
63. A 24-year-old female teacher complai-
59. A 24-year-old man has developed increasi- ns of dizziness and heart pain irradiating
ng headaches, vertigo, diplopia, paresis of the to the left nipple. Pain is not associated
facial muscles on the right, choking during with physical activity and cannot be reli-
swallowing. The signs appeared on the 5th eved by nitroglycerin, it abates after taki-
day of respiratory disorder. He was diagnosed ng Valocordin and lasts an hour or more.
with acute viral encephalitis. Determine the The patient has a nearly 2-year history of
main direction of emergency therapy: this disease. Objectively: Ps- 76/min., BP-
110/70 mm Hg. Heart borders are normal,
A. Zovirax (Aciclovir) heart sounds are clear. The ECG shows respi-
B. Glucocorticoids ratory arrhythmia. Radiograph of the cervi-
C. Ceftriaxone cothoracic spine reveals no pathology. Lungs,
D. Lasix (Furosemide) abdomen are unremarkable. What changes in
E. Neohaemodes blood formula can be expected?
60. A 23-year-old woman, who works as a A. No changes
milk and dairy inspector, after the miscarri- B. Leukocytosis
age suffers from high fever up to 38, 6o C , C. Thrombocytopenia
recurring chills, excessive sweating. Objecti- D. Leukemic hiatus
vely: polyadenitis, pain in the lumbosacral E. Increased ESR
spine, swollen left knee joint, enlarged liver
and spleen. What diagnosis is most likely? 64. A 51-year-old female patient complains of
frequent defecation and liquid blood-streaked
A. Brucellosis stools with mucus admixtures, diffuse pain in
B. Sepsis the inferolateral abdomen, 6 kg weight loss
C. Toxoplasmosis within the previous month. Objectively: body
D. Polyarticular rheumatoid arthritis temperature - 37, 4o C , malnutrition, skin is
E. Yersiniosis pale and dry. Abdomen is soft, sigmoid is pai-
nful and spasmodic, makes a rumbling sound.
61. A 16-year-old adolescent has been hospi- Liver is dense, painful, extends 3 cm below
talized with complaints of unceasing nasal the costal margin. What is the most likely di-
hemorrhage and unbearable pain in his ri- agnosis?
ght elbow joint. Objectively: the large joint
is enlarged and defigurated, the skin over A. Non-specific ulcerative colitis
the joint is hyperemic. Arthropathy signs can B. Bacillary dysentery
be observed in the other joints. Ps- 90/min. C. Sprue
Blood test: erythrocytes - 3, 9 · 1012 /l, Нb- 130 D. Intestinal enzymopathy
g/l, color index - 1,0, leukocytes - 5, 6 · 109 /l, E. Helminthic invasion
platelets - 220 · 109 /l, ESR- 6 mm/hour.
65. Chief physician of a polyclinic charged a
Lee-White coagulation time: start- 24 , end-
Крок 2 Medicine (англомовний варiант, iноземнi студенти) 2016 рiк 9

district doctor with a task to determine the A. Platelets


pathological prevalence of disease N in his B. Reticulocytes
district. What document allows to estimate C. Clotting time
the disease prevalence in the population of a D. Osmotic resistance of erythrocytes
medical district? E. Fibrinogen

A. Prophylactic examinations register 69. A 49-year-old patient complains of


B. Statistic coupons (+) swallowing disorder that intensifies duri-
C. Statistic coupons (-) ng eating solid food, hiccups, hoarse voice,
D. Statistic coupons (+) and (-) nausea, regurgitation, significant weight loss
E. Vouchers for medical appointments (15 kg within 2,5 months). Objectively: body
weight is reduced; the skin is pale and dry;
66. A 32-year-old woman complains of di- vesicular respiration; heart sounds are suffi-
zziness, headache, palpitation, tremor. For ciently sonorous; heart rate is rhythmic. The
the last several months she has been under abdomen is soft, no pain on palpation. The
outpatient observation for increased arteri- liver is not enlarged. What investigation is
al pressure. Since recently such attacks have most necessary for making the diagnosis in
become more frequent and severe. Objecti- this case?
vely: the skin is covered with clammy sweat,
tremor of the extremities is present. HR- A. Esophagoduodenoscopy with biopsy
110/min., BP- 220/140 mm Hg. Heart sounds B. Clinical blood analysis
are muffled. Blood test results: WBCs- 9, 8 · C. X-ray of the gastrointestinal tract
D. X-ray in the Trendelenburg position
109 /l, ESR- 22 mm/hour. Blood glucose - 9,8 E. Investigation of gastric secretion
millimole/l. What disease is the most likely
cause of this crisis? 70. A 9-month-old child presents with fever,
cough, dyspnea. The symptoms appeared
A. Pheochromocytoma
5 days ago after a contact with a person
B. Essential hypertension suffering from URTI. Objectively: the child
C. Preeclampsia is in grave condition. Temperature of 38o C ,
D. Primary hyperaldosteronism
cyanosis of nasolabial triangle is present.
E. Diabetic glomerulosclerosis Respiration rate - 54/min., nasal flaring duri-
67. A 45-year-old patient complains of pain ng breathing. There was percussion dullness
in the epigastric region, left subcostal area, on the right below the scapula angle, and
abdominal distension, diarrhea, loss of wei- tympanic sound over the rest of lungs.
ght. He has been suffering from this condition Auscultation revealed bilateral fine moist
for 5 years. Objectively: tongue is moist with crackles predominating on the right. What is
white coating near the root; deep palpation of the most likely diagnosis?
abdomen reveals slight pain in the epigastric
A. Acute pneumonia
region and Мауо-Robson’s point. Liver is pai- B. URTI
nless and protrudes 1 cm from the costal arch. C. Acute laryngotracheitis
Spleen cannot be palpated. What disease can
D. Acute bronchitis
be primarily suspected? E. Acute bronchiolitis
A. Chronic pancreatitis
71. A 50-year-old man, who works as a poli-
B. Atrophic gastritis sher at a combine-building factory, addressed
C. Peptic stomach ulcer the factory’s sectorial doctor with complai-
D. Chronic cholecystitis
nts of general fatigue, sensations of numbness
E. Chronic enteritis and pain in his fingers. Objectively: the skin
68. A 58-year-old woman complains of of his fingers is pale. Reaction to pain, tactile
spontaneous bruises, weakness, bleeding and thermal stimuli was revealed to be slightly
gums, dizziness. Objectively: the mucous disrupted. No disruptions can be observed wi-
membranes and skin are pale with numerous thin the other organs and systems. What di-
hemorrhages of various time of origin. Lymph sorder is most likely?
nodes are not enlarged. Heart rate - 100/min., A. Pneumatic hammer disease
BP- 110/70 mm Hg. There are no changes
B. Multiple neuritis
of internal organs. Blood test results: RBC- C. Raynaud’s disease
3, 0 · 1012 /l, Нb- 92 g/l, colour index - 0,9, D. Syringomyelia
anisocytosis, poikilocytosis, WBC - 10 · 109 /l, E. Deforming arthrosis
eosinophiles - 2%, stab neutrophiles - 12%,
segmented neutrophiles - 68%, lymphocytes 72. A 27-year-old woman complains of
- 11%, monocytes - 7%, ESR- 12 mm/hour. bleeding gums, nasal hemorrhages, multiple
What index should be determined additi- hematomas on the skin of her limbs and on
onally by a laboratory to make a diagnosis? the front of her torso, extreme general fati-
gue. Blood test: Hb- 64 g/l, erythrocytes -
2, 5 · 1012 /l, reticulocytes - 16%, platelets -
Крок 2 Medicine (англомовний варiант, iноземнi студенти) 2016 рiк 10

30 · 109 /l, ESR- 22 mm/hour. What approach adrenal glands, cranium X-ray. The tests
would be most efficient for treatment of this revealed no pathologies.The diagnosis of
pathology? exogenic constitutive obesity has been made.
What direction of therapy should be prioriti-
A. Splenectomy zed?
B. Dicynone (Etamsylate)
C. Platelet concentrate transfusion A. Reducing diet and exercise
D. Cytostatics B. Sanatorium-and-spa treatment
E. Group B vitamins C. Anorectic drugs
D. Dehydration therapy
73. A 23-year-old patient had taken 1 g of E. ”Fat-burning” methods
aspirin to treat acute respiratory infection.
After that he developed an asthmatic fit wi- 77. An 8-year-old child with a 3-year history
th labored expiration that was arrested by of diabetes was hospitalized in hyperglycemic
introduction of aminophylline. The patient’s coma. Specify the initial dose of insulin to be
medical history is not burdened with allergies. administered:
The patient has undergone two surgeries for
nasal polyposis in the past. What diagnosis is A. 0,1-0,2 U/kg of body weight per hour
most likely? B. 0,05 U/kg of body weight per hour
C. 0,2-0,3 U/kg of body weight per hour
A. Aspirin-induced asthma D. 0,3-0,4 U/kg of body weight per hour
B. Atopic bronchial asthma E. 0,4-0,5 U/kg of body weight per hour
C. Infectious allergic bronchial asthma
D. Exercise-induced asthma 78. A worker, who was involved in fire fi-
E. Symptomatic bronchospasm ghting inside the building that stored 2 kg
of mercury, has been delivered to a hospital
74. A 58-year-old woman undergoing with complaints of emotional expansiveness,
chemotherapy for her oncologic disorder has palpitations, excessive sweating, body tremor,
developed sore throat. Examination revealed heart pain. Within one day his condition
necrotic areas on the mucosa of the pharynx aggravated. Objectively: the skin is pale and
and tonsils. Many of her teeth are afflicted wi- moist. The patient is depressed. Permanent
th caries. In blood: neutrophilic granulocytes red dermographism, erethism, unstable BP
are practically absent against the background are observed. What drug is the serum in this
of leukopenia. Leukocytes are represented case?
mainly by lymphocytes and monocytes. What
disease can be suspected in the given case? A. Unithiol
B. Atropine sulfate
A. Agranulocitar tonsillitis C. Calcium tetacine
B. Lacunar tonsillitis D. Amyl nitrite
C. Pseudomembranous (Vincent’s) tonsillitis E. Dipyroxime
D. Syphilitic tonsillitis
E. Diphtheria 79. During meat testing Trichinella was
detected in diaphragm crura in one of the two
75. A patient complains of constant dull muscular tissue samples. What tactics should
pain in the perineum and suprapubic area, a doctor choose regarding this meat?
weak flow of urine, frequent difficult pai-
nful urination, nocturia. The patient has A. Technolgical disposal
been suffering from this condition for several B. Incineration
months, during which urination was becomi- C. Boiling under 1,5 atmosphere
ng increasingly difficult, and pain in the peri- D. Preservation in 10% salt solution
neum has developed. On rectal examinati- E. Freezing under -12o C
on: the prostate is enlarged (mainly its right 80. A 40-year-old patient was bitten by a stray
lobe), dense, asymmetrical, central fissure is dog an hour ago. On the left shin there is a
smoothed out, the right lobe is of stony densi- bite mark - the wound is 4x2x0,5 cm in size.
ty, painless, tuberous. What disease is it? What surgical aid would be most efficient in
A. Prostate cancer this case?
B. Prostate sclerosis A. Lavage with soapy water, retension sutures
C. Urolithiasis, prostatolith of the right lobe B. Aseptic dressing
D. Prostate tuberculosis C. Salve dressing
E. Chronic congestion prostatitis D. Blind suture
76. A child is 10 years old. The weight is E. Retension sutures
46 kg. Since birth the child has been gai- 81. A 30-year-old patient has been hospitali-
ning excessive weight. The parents are full- zed with diagnosis of intestinal obstruction.
bodied. The child has undergone the followi- The surgery revealed the obstruction of the
ng tests: carbohydrate tolerance, level of small intestine to be caused by a helminth
17-ketosteroids, blood electrolytes, US of ball. What kind of helminth is it?
Крок 2 Medicine (англомовний варiант, iноземнi студенти) 2016 рiк 11

A. Shigellosis
A. Ascaris B. Salmonellosis
B. Guinea worm C. Escherichiosis
C. Filariidae D. Intestinal amebiasis
D. Cysticercus E. Yersiniosis
E. Pinworm
86. The institutions that take part in medi-
82. Mother of a newborn suffers from chronic cal examinations include prevention and
pyelonephritis; she had a case of URTI before treatment facilities, medical board of Mini-
the delivery. The delivery is at term, for a stry of Defense, medical board of Ministry of
long period waters were not breaking. On Home Affairs, medico-social expert commi-
the 2nd day of life the child developed an ssions, forensic medical boards etc. What insti-
erythematic rash, later the rash developed tutions are responsible for temporary disabi-
into blisters approximately 1 cm in diameter lity examination?
filled with serous purulent substance. Ni-
kolsky’s symptom is positive. After the bli- A. Prevention and treatment facilities
sters had been lanced, erosions developed in B. Sanitary-and-prophylactic institutions
their place. The child is inert. The child’s body C. Medico-social expert commissions
temperature is subfebrile. What diagnosis is D. Medical boards of Ministry of Defense
most likely? E. Medical boards of Ministry of Home Affairs

A. Impetigo neonatorum 87. After a lengthy march an army regi-


B. Vesiculopustulosis ment has set camp for 3 days near a
C. Pseudofurunculosis settlement. Sanitary-hygienic investigation
D. Sepsis detected several water sources. Choose the
E. Ritter’s disease source that would satisfy the demands for
potable water the most under the given fi-
83. Three weeks after a case of acute tonsilli- eld conditions:
tis the patient is still weak, inert, subfebrile,
his retromaxillary lymph nodes are enlarged. A. Artesian well
Tonsils are flabby, stick together with arches, B. Brook
there are purulent plugs in lacunae. What is C. River
the most probable diagnosis? D. Rain water
E. Melt water
A. Chronic tonsillitis
B. Chronic pharyngitis 88. Clinical statistical investigation was
C. Acute lacunar tonsillitis performed to determine efficiency of a new
D. Paratonsillitis pharmacological preparation for patients wi-
E. Tonsillar tumour th ischemic heart disease. What parametric
test (coefficient) can be used to estimate reli-
84. The following indicators were calculated ability of the results?
to analyse population health and treatment
quality in a cardiological hospital: primary A. Student’s t-distribution
cardiovascular morbidity - 62%; total cardi- B. Sign test
ovascular morbidity - 483,55%; cardiovascular C. Matching factor
mortality - 10,9%; proportion of cardi- D. Wilcoxon signed-rank test
ovascular mortality within total mortality - E. Kolmogorov-Smirnov test
67,0%; primary disablement caused by cardi-
ovascular diseases - 16,2 per 10.000 populati- 89. A 30-year-old parturient woman was deli-
on. What indicator is an extensive value? vered to a maternity hospital with full-term
pregnancy. She complains of severe lanci-
A. Proportion of cardiovascular mortality nating pain in the uterus that started 1 hour
within total mortality ago, nausea, vomiting, cold sweat. Anamnesis
B. Primary cardiovascular morbidity states cesarean section 2 years ago. Uteri-
C. Total cardiovascular morbidity ne contractions stopped. Skin and mucous
D. Primary disablement caused by cardi- membranes are pale. Heart rate is 100/min.,
ovascular diseases BP is 90/60 mm Hg. Uterus has no clear margi-
E. Cardiovascular mortality ns, is sharply painful. No heartbeat can be
auscultated in the fetus. Moderate bloody
85. A 6-year-old child complains of frequent discharge from the uterus can be observed.
liquid stool and vomiting. On the 2nd day Uterus cervix is 4 cm open. Presenting part is
of disease the child presented with inertness, not visible. The most likely diagnosis is:
temperature rise up to 38, 2o C , Ps- 150/min.,
scaphoid abdomen, palpatory painful sigmoid
colon, defecation 10 times a day with liquid,
scarce stool with mucus and streaks of green.
What is the provisional diagnosis?
Крок 2 Medicine (англомовний варiант, iноземнi студенти) 2016 рiк 12

A. Uterine rupture
B. Initial uterine rupture A. Testicular feminization syndrome
C. Threatened uterine rupture B. Mayer-Rokitansky-Kuster-Hauser
D. Premature detachment of normally positi- syndrome
oned placenta C. Cushing’s syndrome
E. Compression of inferior pudendal vein D. Sheehan syndrome
E. Cushing’s disease
90. A parturient woman is 23 years old.
Internal obstetric examination shows the 94. A planner designs a heating system for a
uterine cervix to be completely open. Fetal pre-school educational establishment. The hi-
bladder is absent. Cephalic presentation is ghest air temperature should be in the followi-
observed in the plane of the small pelvic ng room:
outlet. Sagittal suture is at the longitudi-
nal section of the small pelvic outlet, small A. Game room of a nursery group
fontanel is situated closer to the uterus. What B. Common room of a preschool group
cephalic position will the newborn have duri- C. Bedroom of a nursery group
ng birth in this case? D. Bedroom of a preschool group
E. Gymnasium
A. Minor oblique lie
B. Longitudinal lie 95. A 27-year-old sexually active woman
C. Transverse lie complains of numerous vesicles on the right
D. Medium oblique lie sex lip, itch and burning. Eruptions regularly
E. Major oblique lie appear before menstruation and disappear
8-10 days later. What is the most likely di-
91. During the dynamic observation of a agnosis?
parturient woman in the second stage of labor
it was registered that the fetal heart rate A. Herpes simplex virus
decreased to 90-100/min. and did not normali- B. Bartholinitis
ze after contractions. Vaginal examination C. Primary syphilis
revealed the complete cervical dilatation, the D. Cytomegalovirus infection
fetal head filling the entire posterior surface E. Genital condylomata
of the pubic symphysis and sacral hollow;
the sagittal suture was in the anteroposteri- 96. A 35-year-old woman addressed a
or diameter of the pelvic outlet, the posterior gynecological in-patient department with
fontanelle was in front under the pubic arch. complaints of regular pains in her lower
What plan for further labour management abdomen, which increase during menstruati-
should be recommended? on, and dark-brown sticky discharge from
the genital tracts. On bimanual examinati-
A. Application of forceps minor on: the uterine body is slightly enlarged, the
B. Caesarean section appendages are not palpated. Mirror exami-
C. Episiotomy nation of the uterine cervix reveals bluish
D. Application of cavity forceps spots. What diagnosis is most likely?
E. Stimulation of labour activity through
intravenous injection of oxytocin A. Cervical endometriosis
B. Cervical erosion
92. An 8-year-old girl periodically has sudden C. Cervical polyp
short-term heart pains, sensation of chest D. Cervical cancer
compression, epigastric pain, dizziness, vomi- E. Cervical fibroid
ting. Objectively: the patient is pale, respi-
ratory rate - 40/min., jugular pulse is present. 97. A 10-year-old patient has a history of mild
Ps- 185/min., of poor volume. BP- 75/40 bronchial asthma. During a regular check-up
mm Hg. ECG taken during an attack shows the patient should be recommended:
ectopic P waves, QRS wave is not deformed. A. To avoid allergenic food
At the end of an attack a compensatory pause B. To avoid body tempering procedures
is observed. The most likely cause of the C. To avoid sports
attack is: D. To avoid spa treatment
A. Paroxysmal atrial tachycardia E. To avoid going to the seaside
B. Sinus tachycardia 98. A baby was born by a young smoker. The
C. Paroxysmal ventricular tachycardia labour was complicated by uterine inertia,
D. Complete AV-block difficult delivery of the baby’s head and
E. Atrial fibrillation shoulders. The baby’s Apgar score was 4.
93. A 16-year-old girl has primary Which of the following is a risk factor for a
amenorrhea, no pubic hair growth, normally spinal cord injury?
developed mammary glands; her genotype is
46 ХY; uterus and vagina are absent. What is
your diagnosis?
Крок 2 Medicine (англомовний варiант, iноземнi студенти) 2016 рiк 13

A. Difficult delivery of the head and shoulders te, depression. Objective examination reveals
B. Young age of the mother no pathologic changes of uterus and uterine
C. Pernicious habits appendages. What pathogenesis is characteri-
D. Uterine inertia stic of this disorder?
E. Chronic hypoxia
A. Decreased production of gonadotropin
99. An emergency situation at a chemical B. Hyperproduction of estrogen
plant caused acute occupational intoxicati- C. Hyperproduction of androgen
on. A doctor who revealed the case of ”acute D. Decreased production of progesterone
occupational disease (intoxication)” must E. Hyperproduction of prolactin
notify the following authority:
104. A patient in a clinical death condition is
A. Sanitary and epidemiological station being resuscitated through mouth-to-mouth
B. Plant administration artificial pulmonary ventilation and external
C. Trade union committee of the plant cardiac massage. A doctor noticed that air
D. Medical unit of the plant does not flow into the patient’s airways and
E. Ministry of Public Health of Ukraine his head and torso are positioned at the same
level. Why is artificial respiration not effective
100. A 27-year-old woman has been treated in in the given case?
a surgical department for pleural empyema
for 6 months. Multiple paracenteses of the A. Tongue retraction
pleural cavity were performed along with anti- B. Low breathing volume
bacterial treatment. The patient’s condition is C. External cardiac massage
slowly aggravating; attempts to fully stretch D. Probe is absent from stomach
the lung were unsuccessful. Choose the tacti- E. The patient’s mouth is too small
cs:
105. A 45-year-old patient with urolithiasis
A. Decortication of the lung had an attack of renal colic. What is the
B. Change antibiotics mechanism of the attack development?
C. Set constant active suction drain
D. Pulmonectomy A. Disturbed urine outflow from the kidney
E. Include hyperbaric oxygenation in the B. Increase in urine specific gravity
treatment C. Ureteric twists
D. Destruction of glomerules
101. A prematurely born girl is now 8 E. Renal artery spasm
months old. She has dyspnea, tachycardia,
hepatosplenomegaly, physical developmental 106. A 26-year-old woman has attended
lag, limb cyanosis. There is parasternal cardiac maternity center complaining of her inability
hump, auscultation revealed systolodiastolic to become pregnant despite 3 years of regular
murmur in the II intercostal space on the sex life. Examination revealed the followi-
left. BP is 90/0 mm Hg. What disease can be ng: increased body weight; male-type pubic
suspected? hair; excessive pilosis of thighs; ovaries are
dense and enlarged; basal body temperature
A. Patent ductus arteriosus is monophasic. The most likely diagnosis is:
B. Coarctation of aorta
C. Stenosis of aortal valve A. Ovaries sclerocystosis
D. Stenosis of pulmonary artery B. Inflammation of uterine appendages
E. Nonclosure of interventricular septum C. Adrenogenital syndrome
D. Premenstrual syndrome
102. Three days ago a boy underwent removal E. Gonadal dysgenesis
of a foreign body from under a nail plate. 2
days later he deeloped acute pulsating pain at 107. The process of open-cut mining requi-
the end of the nail bone which aggravated at res drilling and blasting operations, rock
pressing. Nail fold became hyperemic, body and ore excavation, transportation of ore
temperature rose up to 37, 5o C , there was a to fragmentation and sorting factories and
change in nail plate colour. What is the most transportation of barren rock to slag-heaps,
likely diagnosis? road building and maintenance, repair works.
What factor of production is most important
A. Subungual panaritium for miner’s health?
B. Erysipelas
C. Paronychia A. High content of dust in the air
D. Erysipeloid B. High content of explosion gas
E. Abscess C. Vibration
D. Noise
103. A 25-year-old woman complains of E. Adverse microclimate
menstruation retention lasting for 3 years.
The patient explains it by a difficult childbirth 108. A woman addressed a gynecologist
complicated with profuse hemorrhage, weight on the 20th day of puerperal period with
loss, brittleness and loss of hair, loss of appeti- complaints of pain in the left mammary gland,
Крок 2 Medicine (англомовний варiант, iноземнi студенти) 2016 рiк 14

puruent discharge from the nipple. Objecti- at the seaside. Objectively: face erythema,
vely: Ps- 120/min., body temperature is 39o C . edema of shin muscles. Heart sounds are
The left mammary gland is painful, larger than muffled, BP is 100/70 mm Hg. In blood:
the right one, the skin there is hyperemic; in ASAT activity is 0,95 millimole/h·l, ALAT-
the upper quadrant there is an infiltrate 10x15 1,3 micromole/h·l, aldolase - 9,2 IU/l, creati-
cm in size with soft center. Blood test: ESR- ne phosphokinase - 2,5 micromole Р/g·l. What
50 mm/hour, leukocytes - 15, 0 · 109 /l. What method of study would be most specific?
would be the treatment tactics?
A. Muscle biopsy
A. Transfer to a surgical department for surgi- B. ECG
cal treatment C. Echocardiogram
B. Refer to a gynecology department D. Electromyography
C. Refer to a postnatal department E. Determination of cortisol concentration in
D. Refer to a surgeon for conservative blood and urine
treatment
E. Lance the mammary gland abscess in a 113. A 14-year-old girl has fainted during a
maternity department meeting. The day before she complained of a
headache. The skin is pale, the limbs are cold,
109. The correlation between the service shallow breathing, heart sounds are muffled;
record and eosinophil concentration in blood heart rate is 51/min.; BP is 90/50 mm Hg. The
was studied in workers at dyeing shops of abdomen is soft. Meningeal symptoms are
textile factories. What index will be most negative. Make the provisional diagnosis:
informative for the analysis of this data?
A. Collapse
A. Correlation factor B. Unconsciousness
B. Student’s criterion C. Acute left ventricular failure
C. Standardized index D. Acute right ventricular failure
D. Fitting criterion E. Respiratory failure
E. Sign index
114. For the last 15 years a 48-year-old patient
110. A 3-month-old girl has rhinitis, dyspnea, has been working at the factory producing
dry cough. She has been sick for 2 synthetic resins. Lately he has been complai-
days. Objectively: pale skin, acrocyanosis, ning of significant general fatigue, headaches,
hypopnoe; breathing rate is 80/min.; over frequent urination (predominantly during the
the whole pulmonary surface there is day), red color of urine. What complication
vesiculotympanitic (bandbox) resonance of benzene nitrocompounds poisoning can be
observed with numerous bubbling crackles. suspected?
The most likely diagnosis is:
A. Malignant tumor of the urinary bladder
A. Acute bronchiolitis B. Chronic cystitis
B. Pneumonia C. Chronic prostatitis
C. Mucoviscidosis D. Acute glomerulonephritis
D. Foreign body in airways E. Chronic pyelonephritis
E. Acute bronchitis
115. A 60-year-old woman developed
111. A 24-year-old patient complains of gaini- weakness, vertigo, rapid fatigability during the
ng body mass and increased appetite. Objecti- last year. Recently there have been dyspnea
vely: built of hypersthenic type, body mass and paresthesia observed. Objectively: skin
index is 33,2 kg/m2 , waist circumference is and mucous membranes are pale and icteric.
100 cm. Waist to hips circumference ratio is Lingual papillae are smoothed out. Liver
0,95. What is the provisional diagnosis? and spleen are situated at the edge of costal
arch. Blood test: Hb- 70 g/l, erythrocytes -
A. Alimentary constitutive obesity, I stage, 1, 7·1012 /l, blood color index - 1,2, macrocytes.
android type What drug can be prescribed on pathogenetic
B. Itsenko-Cushing hypothalamic obesity, II grounds?
stage, gynoid type
C. Alimentary constitutive obesity, III stage, A. Vitamin B12
gynoid type B. Vitamin B6
D. Alimentary constitutive obesity, II stage, C. Ascorbic acid
android type D. Iron preparations
E. Itsenko-Cushing hypothalamic obesity, I E. Vitamin B1
stage, android type
116. In the morning upon waking a 65-year-
112. A 36 year old female patient complains old patient developed weakness in the right-
ofgeneral weakness, edemas of her face and side limbs, speech disorder, decreased sensi-
hands, rapid fatigability during walking, diffi- tivity of the left side of the body. On exami-
cult diglutition, cardiac irregularities. These nation: conscious, BP- 100/60 mm Hg, motor
symptoms developed 11 days after holiday aphasia, right-sided central hemiparesis and
Крок 2 Medicine (англомовний варiант, iноземнi студенти) 2016 рiк 15

hemihypalgesia. Make the preliminary di- mmol/l, potassium - 6,5 mmol/l, glomerular
agnosis: filtration rate - 10 ml/min. What tactics would
be leading in the patient’s treatment?
A. Ishemic stroke
B. Hemorrhagic stroke A. Hemodialysis
C. Encephalitis B. Antibacterial therapy
D. Brain tumor C. Sorbent agents
E. Subarachnoid hemorrhage D. Blood transfusion
E. Hypotensive therapy
117. The objective of a statistical study was to
find out the extent of seeking medical care by 121. In a rural health care area there is a case
the population. For this purpose 300 residents of child dying during the first month of life. To
of the area were interviewed. Information was analyse this situation, among other measures,
collected by means of a special questionnaire. an expert assessment of medical records is
What method of collecting information was performed. What medical document should
used by researchers? be considered first?
A. Anamnestic A. Child development history
B. Immediate registration B. Infant development history
C. Immediate examination C. Vaccination card
D. Doing extracts D. Outpatient medical record
E. - E. Child’s medical record
118. A 57-year-old woman complains of havi- 122. A region attended by a central regional
ng a sensation of esophageal compresion, hospital demonstrates increased hemorrhagic
palpitation, difficult breathing during eating stroke morbidity. Essential hypertension
solid food, occasional vomiting with a full morbidity, however, remains at the same level
mouth, ”wet pillow” sign at night for the last and is below the average level registered wi-
6 months. Objectively: body tempearture - thin the larger area. What managerial decision
39o C , height - 168 cm, weight - 72 kg, Ps- should be made in this case?
76/min., BP- 120/80 mm Hg. X-ray revealed
a considerable dilation of the esophagus and A. To design and implement measures for
its constriction in the cardial part. What early diagnostics of arterial hypertension
pathology is most likely to have caused B. To design and implement measures for
dysphagia in this patient? primary prevention of essential hypertension
C. To design and implement measures for
A. Achalasia cardiae secondary prevention of hypertension compli-
B. Primary esophagism cations
C. Hiatal hernia D. To design and implement measures for
D. Esophageal carcinoma professional development of medical workers
E. Reflux esophagitis E. To design and implement improved di-
spensary system for hypertensive patients
119. A 25-year-old man complains of pain
in the lower third of his left thigh, which 123. A 42-year-old man was delivered to a
occurs both with and without physical exerti- surgical in-patient department with complai-
on. Possibility of trauma is denied by the nts of icteric skin, pain in the right subcostal
patient. Objectively: skin colour is normal; area. Biochemical blood analysis: total bili-
pastosity and pain can be detected with deep rubin - 140 mcmol/l, direct bilirubin - 112
palpation; knee joint mobility is reduced. X- mcmol/l. On US: choledoch duct - 1,4 cm,
ray of distal femoral metaphysis shows an a concrement is detected in the distal area.
area of destruction and spicule. Blood test: Gallbladder is 40 cm, no concrements. What
immature cells, no signs of inflammation. The treatment tactics should be chosen?
most likely diagnosis is:
A. Endoscopic papillosphincterotomy
A. Osteogenic sarcoma B. Laparoscopic cholecystectomy
B. Osteitis fibrosa cystica C. Laparotomy with choledoch duct drain
C. Chronic osteomyelitis D. Laparotomy with cholecystectomy
D. Multiple myeloma E. Threatment in an infectious diseases hospi-
E. Marble-bone disease tal
120. A 30-year-old woman suffers from 124. 4 weeks after myocardial infarction a 56-
polycystic renal disease. She has been admi- year-old patient developed acute heart pain,
tted with signs of fatigue, thirst and nocturia. pronounced dyspnea. Objectively: the pati-
Diuresis is up to 1800 ml per day. BP is ent’s condition is extremely grave, there is
200/100 mm Hg. Blood test: erythrocytes - marked cyanosis of the face, swelling and
1, 8 · 109 /l, Hb- 68 g/l. Urine analysis: specific throbbing of the neck veins, peripheral pulse
gravity - 1005, leukocytes - 50-60, erythrocytes is absent, carotid artery pulse is rhythmic,
- 3-5 in the vision field, creatinine - 0,82 130/min., BP is 60/20 mm Hg. Auscultation of
Крок 2 Medicine (англомовний варiант, iноземнi студенти) 2016 рiк 16

the heart reveals extremely muffled sounds, ght shin and dorsal surface of foot. Objecti-
percussion reveals heart border extension in vely: weakness of the anterior tibial muscle,
both directions. What is the optimal treatment long extensor muscle of the right toes, short
tactis for this patient? extensor muscle of the right toes. Low Achi-
lles reflex on the right. Positive Lasegue’s si-
A. Pericardiocentesis and immediate gn. What examination method would be most
thoracotomy effective for specification of the diagnosis of
B. Oxygen inhalation L5 root discogenic compression?
C. Puncture of the pleural cavity on the left
D. Conservative treatment, infusion of A. Magnetic resonance scan
adrenomimetics B. Spinal column X-ray
E. Pleural cavity drainage C. Electromyography
D. Angiography
125. A 45-year-old man was delivered to E. Lumbar puncture
a hospital with complaints of vomiting wi-
th streaks of blood, loss of weight. On 129. A 23-year-old female patient has been
esophagofiberscopy a cauliflower-shaped suffering from mental disorder since the
mucosal growth was detected in the abdomi- age of 18, the course of disorder has no
nal esophagus. The mucosa there bleeds on remission periods. At hospital the patient
contact. What preliminary diagnosis can be mostly presents with non-purposeful fooli-
made? sh excitation: she makes stereotypic gri-
maces, exposes herself, publically masturbati-
A. Esophageal tumor ng, loudly laughs, repeating stereotypical
B. Barrett esophagus obscene shouts. The patient should be prescri-
C. Abdominal esophagitis bed:
D. Esophageal diverticulum
E. Esophageal achalasia A. Neuroleptics
B. Antidepressants
126. A 35-year-old man complains of intense C. Tranquilizers
lumbar pain irradiating to the inguinal area, D. Nootropics
external genitalia, thigh; frequent urination, E. Mood stabilizers
chill, nausea, vomiting. Objectively: positive
Pasternatsky’s symptom (costovertebral angle 130. During preventive ultrasound scan of
tenderness). Urine analysis revealed that abdomen performed during regular check-
RBCs and WBCs covered the total field of up in a school the following was revealed in
microscope; the urine exhibited high protein an 11-year-old student of the 5th grade: the
concentration. These clinical presentations left kidney is 3 cm below the normal posi-
were most likely caused by the following tion, its shape, size and structure are within
pathology: the norm, the contralateral kidney cannot be
observed at its proper place. The preliminary
A. Urolithiasis, renal colic diagnosis is as follows: congenital anomaly of
B. Cholelithiasis, biliary colic renal development, dystopic left kidney, right
C. Renal infarct kidney is absent or pelvic dystopic. What X-
D. Intestinal obstruction ray method would be required for making the
E. Osteochondrosis, acute radicular syndrome final diagnosis and determining the functional
capacity of both kidneys?
127. A 20-year-old patient complains of pain
in the left lumbar region, arterial pressure rise A. Renal dynamic scintigraphy
up to 160/110 mm Hg. US revealed that the B. Radioimmunoassay
structure and size of the right kidney were wi- C. Radionuclide renography
thin age norms, there were signs of 3 degree D. Thermography
hydronephrotic transformation of the left ki- E. Excretory urography
dney. Doppler examination revealed an addi-
tional artery running to the lower pole of the 131. A young woman suffering from
kidney. Excretory urogram shows a narrowing seborrhea oleosa has numerous light-brown
in the region of ureteropelvic junction. Speci- and white spots on the skin of her torso
fy the treatment tactics: and shoulders. The spots have clear margi-
ns, branny desquamation, no itching. What
A. Surgical intervention provisional diagnosis can be made?
B. Administration of spasmolytics
C. Administration of ACE inhibitors A. Pityriasis versicolor
D. Kidney catheterization B. Torso dermatophytosis
E. Administration of β -blockers C. Seborrheic dermatitis
D. Pityriasis rosea
128. As a result of load lifting a 68-year-old E. Vitiligo
woman developed acute pain in the lumbar
region, in a buttock, posterolateral surface 132. In a traffic accident two persons died.
of her right thigh, external surface of the ri- An appointed forensic medical expert was
Крок 2 Medicine (англомовний варiант, iноземнi студенти) 2016 рiк 17

called on duty to another accident locati- A. Hypothyroid


on; therefore, according to the crininal law B. Hypo-ovarian
in force in Ukraine, an investigator called in C. Hypothalamic-pituitary
the following specialist for examination of the D. Alimentary and constitutive
location and bodies: E. Hypercorticoid

A. Nearest available medical doctor 136. A 10 week pregnant woman was admi-
B. Medical assistant tted to a hospital for recurrent pain in the
C. Surgical nurse lower abdomen, bloody discharges from the
D. Nearest available dispensing chemist genital tracts. The problems developed after
E. Nearest available dental mechanic a case of URTI. The woman was registered
for antenatal care. Speculum examination
133. A patient is 28 years old. He has been revealed cyanosis of vaginal mucosa, clean
suffering from mental disorder since he was cervix, open cervical canal discharging blood
22. His current condition has changed acutely: and blood clots; the lower pole of the gestati-
for 3 days the patient has been refusing to onal sac was visible. What tactics should be
leave his home. He claims that there is a chosen?
”telepathy” occurring between him and other
people, through which he receives ”thoughts A. Curettage of the uterus
of strangers” and transmits his own thoughts B. Pregnancy maintenance therapy
for everyone to hear. He thinks his thoughts C. Expectant management, surveillance
and actions are manipulated through this D. Hysterectomy
”telepathy”. Make the preliminary diagnosis: E. Antiviral therapy

A. Paranoid schizophrenia 137. A patient with fibromyoma of uterus si-


B. Depressive episode zed up to 8-9 weeks of pregnancy consulted
C. Manic episode a gynaecologist about acute pain in the lower
D. Organic delirium abdomen. Examination revealed pronounced
E. Acute reaction to stress positive symptoms of peritoneal irritation, hi-
gh leukocytosis. Vaginal examination revealed
134. A 40-year-old patient is registered in a that the uterus was enlarged correspondi-
narcological dispensary. Somatically: skin is ng to 9 weeks of pregnancy due to the fi-
dramatically hyperemic, sclera are injected, bromatous nodes, one of which was mobile
hyperhidrosis is present. BP- 140/100 mm and extremely painful. Appendages were not
Hg, heart rate - 100/min. Mental state: palpable. There were moderate mucous di-
autopsychic orientation is intact, allopsychic scharges. What is the optimal treatment tacti-
orientation is distorted. The patient presents cs?
with motor anxiety. There is an expression
of fear on his face. He refuses to talk about A. Urgent surgery (laparotomy)
his problems and asks to release him immedi- B. Surveillance and spasmolytic therapy
ately, because he ”may be killed”. This state C. Fractional diagnostic curettage of the uteri-
developed in a day after one of his regular ne cavity
drinking bouts. What is your provisional di- D. Surgical laparoscopy
agnosis? E. Surveillance and antibacterial therapy
A. Delirium tremens 138. A 4-month-old child was admitted to
B. Organic delirium a surgical department 8 hours after the first
C. Paranoia attack of anxiety. The attacks happen every
D. Alcoholic hallucinosis 10 minutes and last for 2-3 minutes, vomi-
E. Alcoholic paranoid ting occurred once. Objectively: the child’s
condition is grave. Abdomen is soft, palpation
135. A 35-year-old female patient has gai- reveals a tumor-like formation in the right ili-
ned 20 kg weight within a year with the ac area. After rectal examination the doctor’s
normal diet. She complains of chill, sleepi- finger was stained with blood. What is the
ness, dyspnea. The patient’s mother and sister most probable diagnosis?
are corpulent. Objectively: height - 160 cm,
weight - 92 kg, BMI- 35,9. Obesity is uniform, A. Ileocecal invagination
there are no striae. The face is amimic. The B. Gastrointestinal haemorrhage
skin is dry. The tongue is thickened. Heart C. Wilm’s tumour
sounds are muffled. Heart rate - 56/min., D. Helminthic invasion
BP- 140/100 mm Hg. The patient has consti- E. Pylorostenosis
pations, amenorrhea for 5 months. TSH- 28
mkME/l (normal rate - 0,32-5). Craniogram 139. A woman has focal encephalitis in the
shows no pathology. What is the etiology of anamnesis. Her spatial orientation is not di-
obesity in this case? srupted. She has a sensation, as if ”everythi-
ng seems surreal: buildings are small, round
or distorted; trees are upside down; people
are very tall with thin limbs”. Determine the
Крок 2 Medicine (англомовний варiант, iноземнi студенти) 2016 рiк 18

psychopathologic syndrome: A. Streptoderma


B. Herpes
A. Derealization C. Atopic cheilitis
B. Depersonalization D. Dermatitis
C. Hallucinatory E. Eczema
D. Oneiric
E. Cenestopathic 144. After a contact with chemicals a plant
worker has suddenly developed stridor, voi-
140. A 56-year-old woman complains of itchi- ce hoarseness, barking cough, progressi-
ng skin of her torso, constant nausea, consti- ng dyspnea. Objective examination reveals
pation, sensation of heaviness and pain in acrocyanosis. What is your provisional di-
the right subcostal area, extreme general fati- agnosis?
gue. The patient suffers from biliary cirrhosis.
The skin is pale icteric. The abdomen is soft, A. Laryngeal edema
the liver protrudes 2,0 cm from under the B. Laryngeal carcinoma
margin of the right costal arch, sensitive on C. PATE
palpation. Biochemical investigation: total bi- D. Pulmonary atelectasis
lirubin - 142,0 mcmol/l, conjugated bilirubin E. Pneumothorax
- 139,0 mcmol/l, alanine aminotransferase -
0,98 mmol/hour·l, aspartate aminotransferase 145. An 18-year-old woman complains of
- 0,82 mmol/hour·l, alkaline phosphatase - 8,7 pains in her lower abdomen, purulent di-
mmol/hour·l. What drug should be prescribed scharge from the vagina, temperature rise
in the first place? up to 37, 8o C . Anamnesis states that she
had random sexual contact the day before
A. Ursodeoxycholic acid the signs appeared. She was diagnosed wi-
B. Sirepar th acute bilateral adnexitis. On additional
C. Allochol examination: leukocytes in the all field of visi-
D. Essentiale forte (Phospholipides) on, bacteria, diplococci with intracellular and
E. Livolin forte extracellular position. What agent is most li-
kely in the given case?
141. A 40-year-old female patient complai-
ns of having a bulge on the anterior surface A. Gonococcus
of neck for 5 years. Objectively: Ps- 72/min., B. Colibacillus
arterial pressure - 110/70 mm Hg, in the ri- C. Chlamydia
ght lobe of thyroid gland palpation reveals a D. Trichomonad
mobile node 4x2 cm in size, the left lobe is E. Staphylococcus
not palpable, the basal metabolic rate is 6%.
What is the most likely diagnosis? 146. A 45-year-old man has been suffering
from duodenal ulcer disease for 5 years. He
A. Nodular euthyroid goiter complains of weakness, dizziness, dryness
B. Nodular hyperthyroid goiter of the skin. Objectively: the skin and vi-
C. Riedel’s thyroiditis sible mucosa are pale, chapped lips; heart
D. Mixed euthyroid goiter rate is 100/min., BP- 100/70 mm Hg, systolic
E. Median cervical cyst murmur at all points on heart auscultati-
on. All other internal organs are unchanged.
142. Estimation of community health level Fecal occult blood test is positive. Blood test:
involved analysis of a report on diseases regi- erythrocytes - 3, 1 · 1012 /l, Hb- 88 g/l, color
stered among the population of district under index - 0,7, leukocytes - 4, 6 · 109 /l, platelets -
charge (reporting form 12). What index is 350·109 /l, ESR- 21 mm/hour, anisocytosis, poi-
calculated based on this report? kilocythemia, serum iron - 9,5 mcmol/l. What
A. Common morbidity rate treatment tactics would you choose?
B. Index of pathological affection A. Iron preparations, balanced diet
C. Index of morbidity with temporary disabili- B. Concentrated red cells transfusion
ty C. Intramuscular introduction of 500 mkg of
D. Index of hospitalized morbidity cyanocobalamin
E. Index of basic non-epidemic morbidity D. Corticosteroids, cytostatics
143. A 32-year-old woman addressed a E. Ascorbic acid, calcium chloride
dermatologist with complaints of slightly 147. A 58-year-old patient complains of a
itching rashes in the mouth angles. She has headache in the occipital region, nausea,
been suffering from this condition for 3 choking, opplotentes. The presentations
days. Objectively: there are isolated small appeared after a physical exertion. Objecti-
phlyctenas and superficial erosions covered vely: the patient is excited. Face is hyperemic.
in honey-yellow scabs against the background Skin is pale. Heart sounds are regular, the
of slight hyperemia. Make the diagnosis: 2nd aortic sound is accentuated. BP- 240/120
mm Hg, HR- 92/min. Auscultation reveals
some fine moist crackles in the lower lungs.
Крок 2 Medicine (англомовний варiант, iноземнi студенти) 2016 рiк 19

Liver is not enlarged. ECG shows signs of apex beat to the left. What is the most likely
hypertrophy and left ventricular overload. diagnosis?
What is the most likely diagnosis?
A. Acute cardiac aneurysm
A. Complicated hypertensic crisis B. Recurrent myocardial infarction
B. Acute myocardial infarction, pulmonary C. Acute pericarditis
edema D. Cardiosclerotic aneurysm
C. Bronchial asthma exacerbation E. Pulmonary embolism
D. Uncomplicated hypertensic crisis
E. Community-acquired pneumonia 152. A 60-year-old patient complains of nearly
permanent sensation of heaviness and fullness
148. A 45-year-old patient (14-year-long work in the epigastrium, which increases after eati-
record as a house painter) upon the contact ng, foul-smelling eructation, occasional vomi-
with synthetic paint develops skin reddeni- ting with food consumed 1-2 days ago, wei-
ng, edema, severe itching and oozing lesi- ght loss. 12 years ago he was first found to
ons on her face. Symptoms disappear after have an ulcer of pyloric channel. The patient
the contact with this chemical substance has taken ranitidine for periodic hunger pains.
stops but even the smell of paint alone is The patient’s condition has been deteriorating
enough to make them reappear each time. over the last 3 months. Objectively: splashing
Each recurrence is characterised by increased sound in the epigastrium is present. What ki-
severity of symptoms. What provisional di- nd of complication is it?
agnosis can be made?
A. Pyloric stenosis
A. Professional eczema B. Penetration of gastric ulcer
B. Simple contact dermatitis C. Functional pyloric spasm
C. Allergic contact dermatitis D. Foreign body in the stomach (bezoar)
D. Urticaria E. Malignization of gastric ulcer
E. Toxicodermia
153. A 5-year-old child has body temperature
149. A 32-year-old woman addressed a increased up to febrile numbers, suffers from
maternity clinic with complaints of inferti- inertness, weakness. Examination revealed
lity that has been lasting for 7 years. hemorrhage on the skin of limbs and torso.
Her menstrual cycle occurs in two phases. Enlargement of cervical and axillary lymph
Hysterosalpingography reveals obstruction of nodes can be detected. The liver is 4 cm
the uterine tubes in the ampullar areas, an below the costal arch; the spleen is 6 cm
adhesive process in the small pelvis can be below the costal arch. Blood test: erythrocytes
observed. What treatment is most advisable - 2, 3 · 1012 /l, Hb- 60 g/l, platelets - 40 ·
in this case? 109 /l, leukocytes - 32, 8 · 109 /l, eosinophiles
- 1%, band neutrophiles - 1%, segmented
A. Laparoscopy neutrophiles - 12%, lymphocytes - 46%,
B. Laparotomy monocytes - 1%, blasts - 40%, Duke’s bleedi-
C. Tubectomy ng time is 9 min. What examination is
D. Adnexectomy necessary to make the diagnosis?
E. Hydrotubation
A. Myelogram
150. Survey radiograph of a 52-year-old B. Lymph nodes biopsy
worker of an agglomeration plant (28 years C. US of abdominal cavity
of experience, the concentration of metal dust D. Detection of hepatitis markers
is 22-37 mg/m3 ) shows mildly pronounced E. Investigation of platelets dynamic functions
interstitial fibrosis with diffused contrast well-
defined small nodular shadows. The patient 154. A child from primipregnancy was born in
has no complaints. Pulmonary function is a term labor and has body weight of 4000 g
not compromised. What is the provisional di- and body length of 57 cm. When born, he was
agnosis? nonresponsive to examination. Diagniosis is
diffuse. Heart rate is 80/min. What resuscitati-
A. Siderosis on measures should be taken?
B. Silicosis
C. Anthraco-silicatosis A. Begin ALV with mask
D. Silicatosis B. Introduce 100% oxygen
E. Anthracosis C. Intubate and begin ALV
D. Tactile stimulation
151. A week ago a 65-year-old patient E. Administer naloxone
suffered an acute myocardial infarction, his
general condition deteriorated: he complai- 155. A 15-year-old teenager has undergone
ns of dyspnea at rest, pronounced weakness. medical examination in military recruitment
Objectively: edema of the lower extremities, center. The following was revealed: interval
ascites is present. Heart borders are extended, systolic murmur at the cardiac apex, accent
paradoxical pulse is observed 2 cm from the of the II heart sound over the pulmonary
Крок 2 Medicine (англомовний варiант, iноземнi студенти) 2016 рiк 20

artery, tachycardia. What additional exami- is noted - lacrimation and itching. What provi-
nation method will be most informative for sional diagnosis can be suggested?
determining a diagnosis?
A. Reactive arthritis
A. Echocardiography B. Rheumatoid arthritis
B. Electrocardiography C. Seasonal pollinosis
C. X-ray D. Bacterial nonspecific urethral conjunctivitis
D. Phonocardiography E. URTI with conjunctiva and joints affected
E. Rheography
160. A 19-year-old patient complains of severe
156. A 64-year-old patient has been hospi- pain in axillary crease. Condition onset was a
talised with complaints of progressive jaundi- week ago after her swimming in a cold ri-
ce that developed over 3 weeks ago without ver and epilation. The next day painful ”boi-
pain syndrome, along with general weakness, l” appeared that was becoming larger every
loss of appetite. Objectively: temperature is day and became a plum-sized tumor. Upon
36, 8o C , heart rate is 78/min., abdomen is soft examination nodular conical growths joined
and painless, peritoneum irritation symptoms together are detected, the skin covering them
are not detected, palpation reveals sharply is bluish-red in color. Some nodules have fi-
enlarged tense gallbladder. What disease can stulous openings producing thick purulent
be characterised with these symptoms? mass. Body temperature is 38, 5o C , general
malaise. The most likely diagnosis is:
A. Cancer of pancreas head
B. Duodenal ulcer A. Hydradenitis
C. Acute cholecystitis B. Carbuncle
D. Chronic cholecystitis C. Cutaneous tuberculosis
E. Cholecystitis caused by lambliasis D. Necrotizing ulcerative trichophytosis
E. Pyoderma chancriformis
157. A 6-year-old girl attended a general
practitioner with her mother. The child 161. Annual report of an in-patient ward
complains of burning pain and itching in her presents data about the number of patient
external genitalia. The girl was taking antibi- days and the number of patients, who have
otics the day before due to her suffering from undergone treatment within a year. What
acute bronchitis. On examination: external work indicator of the in-patient ward can be
genitalia are swollen, hyperemic, there is whi- calculated based on these data?
te deposit accumulated in the folds. The most
likely diagnosis is: A. Average duration of in-patient treatment of
a patient
A. Candidal vulvovaginitis B. Bed occupancy rate
B. Trichomoniasis C. Hospital bed turnover rate
C. Nonspecific vulvitis D. Mortality
D. Helminthic invasion E. Efficient use of the hospital bed capacity
E. Herpes vulvitis
162. ECG revealed the following in a 10-
158. A patient with otopyosis is in sharply year-old child: sharp acceleration of the heart
deteriorating condition: he developed rate - 240/min., P wave overlaps with T wave
headache, vomiting, febrile temperature, and deforms it, moderate lengthening of PQ
general hyperesthesia. Meningeal symptoms interval, QRS complex is without changes.
and stagnant optic disks are observed. There What pathology does this child have?
are no focal symptoms. Liquor is turbid, blood
pressure is high, albuminocytological dissoci- A. Paroxysmal atrial tachycardia
ation occurs with neutrophils. What disease B. Atrial hypertrophy
can be suspected? C. Ventricular hypertrophy
D. WPW syndrome
A. Secondary purulent meningitis E. Extrasystole
B. Meningoencephalitis
C. Serous meningitis 163. A 54-year-old patient complains of
D. Primary purulent meningitis weakness, jaundice, itching skin. Disease
E. Subarachnoid hemorrhage onset was 1,5 months ago: fever up to 39o C
appeared at first, with progressive jaundice
159. A 25-year-old patient is not married and developed 2 weeks later. On hospitalisati-
has sexual relations with several partners. on jaundice was severely progressed. Liver
During the last 3 months he noticed small cannot be palpated. Gallbladder is enlarged
amount of mucoserous secretions produced and painless. Blood bilirubin is 190 mi-
from urethra. Subjectively: periodical itching cromole/l (accounting for direct bilirubin).
or burning pain in urethra. Two months ago Stool is acholic. What is the most likely jaundi-
pain in the knee join developed. Possibility of ce genesis in this patient?
trauma or exposure to cold is denied by the
patient. During the last week eye discomfort
Крок 2 Medicine (англомовний варiант, iноземнi студенти) 2016 рiк 21

A. Mechanical jaundice A. Neuroleptics with a sedative effect


B. Hepatocellular jaundice B. Antidepressants with an activating effect
C. Hemolytic jaundice C. Neuroleptics with an activating effect
D. Caroli syndrome D. Tranquilizers
E. Gilbert’s syndrome E. Antidepressants with a sedative effect
164. A 22-year-old patient complains of 8- 168. A 32-year-old man complains of pain in
month delay of menstruation. Anamnesis: the chest on the left, dyspnea, temperature
menarche since the age of 12,5. Since the rise up to 38, 0o C , slight cough. The disease
age of 18 menstruations are irregular. No onset was 2 weeks ago after overexposure to
pregnancies. Mammary glands have normal cold. He had suffered from bronchoadenitis
development; when the nipples are pressed, in his childhood. The affected side lags during
milk drops are discharged. On gynecologi- breating; percussion reveals dull sound with
cal examination: moderate uterus hypoplasia. oblique margin in the lower left lung, where
On hormonal examination: prolactin level breathing is absent. The right heart border is
exceeds the norm two times. On computed displaced outwards. Mantoux test with 2 TU
tomogram of the sellar region: a space- resulted in a papule 16 mm in size. What di-
occupying lesion 4 mm in diameter is agnosis is most likely?
detected. The most likely diagnosis is:
A. Tuberculous pleurisy
A. Pituitary tumor B. Central carcinoma of the left lung
B. Lactation amenorrhea C. Congestion pneumonia
C. Stein-Leventhal syndrome D. Thromboembolism of the pulmonary artery
D. Sheehan’s syndrome branches
E. Cushing’s disease E. Community-acquired pneumonia
165. A 2-year-old child in a satisfactory condi- 169. An emergency doctor has diagnosed a
tion periodically presents with moderate 32-year-old woman with generalized convulsi-
proteinuria, microhematuria. US results: the ve status epilepticus. The deterioration in the
left kidney is undetectable, the right one is patient’s condition is caused by a sudden gap
enlarged, there are signs of double pyelocali- in the epilepsy treatment. Specify the doctor’s
ceal system. What study is required to specify further tactics:
the diagnosis?
A. Hospitalization in the intensive care unit
A. Excretory urography B. Hospitalization in the department of
B. Micturating cystography neurology
C. Retrograde urography C. Hospitalization in the department of
D. Doppler study of renal vessels neurosurgery
E. Radioisotope renal scan D. Outpatient monitoring by a neuropathologi-
st
166. A 13-year-old girl complains of fatigabili- E. Outpatient monitoring by a neurosurgeon
ty, frequent headaches, cardialgia. Eight years
ago she had a case of pyelonephritis. Urine 170. A 19-year-old patient complains of
analyses periodically revealed leukocyturia. dyspnea during physical exertion. He often
The child has undergone no further treatment. has bronchitis and pneumonia. Cardiac
On examination: increased BP up to 150/100 murmur has been observed since his chi-
mm Hg. Ultrasound investigation revealed si- ldhood. On auscultation: there is splitting of
gnificant reduction of the right kidney. What the II heart sound over pulmonary arteria,
process is leading in arterial hypertension systolic murmur in the third intercostal space
pathogenesis in this case? near the left sternum margin. ECG test shows
right bundle-branch block. What is the provi-
A. Hyperactivity of renin-angiotensin system sional diagnosis?
B. Disruption of water-electrolytic balance
C. Disruption of renal circulation A. Interatrial septum defect
D. Hypersympathicotonia B. Open arterial duct
E. Increased cortisol level C. Coarctation of aorta
D. Aortic stenosis
167. A 26-year-old patient with affective bi- E. Mitral valve insufficiency
polar disorder has developed a condition
manifested by mood improvement, behavi- 171. A 75-year-old male patient complains
oural and sexual hyperactivity, verbosity, acti- of slight pain in the right iliac region. The
ve body language, reduced need for sleep. abdominal pain arose 6 days ago and was
Which of the following drugs are most effi- accompanied by nausea. Surgical examination
cient in this case? revealed moist tongue, Ps- 76/min., BP- 130/80
mm Hg. Abdomen was soft, slightly painful in
the right iliac region on deep palpation, the
symptoms of the peritoneum irritation were
doubtful. In blood: RBCs- 4, 0 · 1012 /l, Hb- 135
Крок 2 Medicine (англомовний варiант, iноземнi студенти) 2016 рiк 22

g/l, WBCs- 9, 5 · 109 /l, stab neutrophils - 5%, lymph nodes are observed. On mammary
segmentonuclear - 52%, lymphocytes - 38%, glands US: in the upper external quadrant of
monocytes - 5%, ESR- 20 mm/hour. Specify the right mammary gland there is a space-
the doctor’s further tactics: occupying lesion of increased echogenicity
21х18 mm in size. The most likely diagnosis
A. Emergency operation for acute appendicitis is:
B. Hospitalization, dynamic surveillance
C. Send the patient home A. Fibrous adenoma
D. Refer the patient to a district therapist B. Lacteal cyst
E. Administration of additional examination: C. Diffuse mastopathy
abdominal ultrasound, x-ray contrast study of D. Breast cancer
the gastrointestinal tract E. Mastitis
172. An 8-year-old boy has a 2-year history of 176. 2 weeks after labour a parturient
blotchy itchy rash appearing after eating ci- woman developed breast pain being observed
trus fruit. The first eruption occurred at the for 3 days. Examination revealed body
age of 6 months after the introduction of jui- temperature at the rate of 39o C , chills,
ces to the baby’s diet. Father has a history of weakness, hyperaemia, enlargement, pain and
bronchial asthma, mother - that of allergic rhi- deformity of the mammary gland. On palpati-
nitis. What is the most likely diagnosis? on the infiltrate was found to have an area of
softening and fluctuation. What is the most
A. Atopic dermatitis likely diagnosis?
B. Psoriasis
C. Pityriasis Rosea A. Infiltrative-purulent mastitis
D. Urticaria B. Phlegmonous mastitis
E. Quincke’s edema C. Lactostasis
D. Serous mastitis
173. A 25-year-old patient has been admi- E. Mastopathy
tted to the hospital with the following
problems: weakness, sweating, itching, wei- 177. A 48-year-old woman has been suffering
ght loss, enlarged submandibular, cervical, from chronic pancreatitis for the last 7 years.
axillary, inguinal lymph nodes. Objectively: Lately she has been noticing an increase in
hepatomegaly. Lymph node biopsy revealed daily feces with foul smell, abdominal di-
giant Berezovsky-Reed-Sternberg cells, stention, gurgling. The patient complains of
polymorphocellular granuloma composed of diarrhea, weakness, fatigability, loss of appeti-
lymphocytes, reticular cells, neutrophils, eosi- te, loss of weight. What syndrome can be
nophils, fibrous tissue, plasma cells. What is suspected in this case?
the most likely diagnosis?
A. Malabsorption
A. Lymphogranulomatosis B. Irritable colon
B. Lymph node tuberculosis C. Maldigestion
C. Lymphoreticulosarcoma D. Exudative enteropathy
D. Cancer metastases to lymph nodes E. Endocrine gland failure
E. Macofollicular reticulosis
178. A 37-year-old man suffers from attacks
174. An 8-year-old child was hospitalized of unconsciousness, dyspnea during physi-
for fever up to 39, 8o C , inertness, moderate cal exertion, periodical sensations of heart
headache, vomiting. Examination revealed rate disorder. Father of the patient died
meningeal symptoms. Lumbar puncture was suddenly at the age of 45. Objectively: heart
performed. The obtained fluid had raised rate is 90/min., BP is 140/90 mm Hg. On
opening pressure, it was transparent, with heart US: ejection fraction - 49%, significant
the cell count of 450 cells per 1 mcl (mai- myocardium thickening of the left ventri-
nly lymphocytes - 90%), glucose level of 2,6 cle and interventricular septum. What drug
mmol/l. What causative agent might have should be prescribed for the treatment?
caused the disease in the child?
A. Bisoprolol
A. Enterovirus B. Enalapril
B. Meningococcus C. Phenyhydinum (Nifedipine)
C. Koch’s bacillus D. Hydrochlorothiazide
D. Staphylococcus E. Furosemide
E. Pneumococcus
179. During a regular check-up of a 50-year-
175. A 25-year-old patient during self- old woman a tumor was detected in her ri-
examination detected a tumor in the upper ght mammary gland. The tumor is 5 cm in
external quadrant of the right mammary diameter, dense, without clear margins. The
gland. On palpation: painless, dense, mobi- skin over the tumor resembles lemon rind,
le growth 2 cm in diameter is detected in the the nipple is inverted. The lymph node can
mammary gland; no changes in the peripheral be palpated in the axillary region. What di-
Крок 2 Medicine (англомовний варiант, iноземнi студенти) 2016 рiк 23

agnosis is most likely? muffled, extrasystole is present, auscultati-


on reveals apical systolic murmur that is not
A. Breast cancer conducted to the left inguinal region. ESR is
B. Lacteal cyst 38 mm/hour. CRP is 2+, antistreptolysin O ti-
C. Diffuse mastopathy tre - 400. What is the most likely diagnosis?
D. Mastitis
E. Breast lipoma A. Acute rheumatic fever
B. Vegetative dysfunction
180. A 20-year-old woman complains of C. Non-rheumatic carditis
sensation of air shortage, lingering dull pain D. Juvenile rheumatoid arthritis
in the heart area, irritability. Objectively: E. Reactive arthritis
general condition is satisfactory, heart rate
lability, BP is 130/60 mm Hg. ECG reveals di- 184. An 18-year-old woman complains
sruption of repolarization proceses. The pati- of mammary glands swelling, headaches,
ent is diagnosed with somatoform autonomic tearfulness, abdominal distension occurring
dysfunction of cardial type. Specify the condi- the day before menstruation. The symptoms
tions of the patient treatment: disappear with the beginning of menstruation.
Menstruations are regular, last for 5-6 days wi-
A. Out-patient treatment th interval of 28 days in between. Gynecologi-
B. In-patient treatment in therapeutics cal examination revealed no changes of
department internal genitals. What is your diagnosis?
C. In-patient treatment in cardiology
department A. Premenstrual syndrome
D. In-patient treatment in cardiac surgery B. Sheehan syndrome
department C. Stein-Leventhal syndrome
E. In-patient treatment in psychiatric D. Asherman’s syndrome
department E. Adrenogenital syndrome
181. A 38-year-old man’s workplace is within 185. A 24-year-old patient had been delivered
the area of effect of ionizing radiation. Duri- to a thoracic department with a chest injury,
ng regular medical check-up he expresses a fracture of the IV, V, VI ribs on the right.
no complaints. Blood test: erythrocytes - Plan radiography shows the fluid level in the
4, 5 · 1012 /l, Нb- 80 g/l, leukocytes - 2, 8 · 109 /l, pleural cavity reaching the III rib on the right.
platelets - 30 · 109 /l. Can this person continue Puncture blood contained clots. What is the
to work with sources of ionizing radiation? optimal treatment tactics?

A. Work with radioactive substances and other A. Emergency thoracotomy


sources of radiation is contraindicated B. Pleural puncture
B. Work with radioactive substances is allowed C. Thoracentesis and thoracostomy
C. Only work with low-level radioactive D. Hemostatic therapy
substances is allowed E. Medical thoracoscopy
D. Work with radioactive substances can be 186. A 30-year-old man was delivered to
allowed after detailed medical examination a neurosurgical department with complai-
E. Work with radioactive substances is allowed nts of constant headaches, nausea, vomi-
only for limited periods of time ting, fever, weakness of the right-side li-
182. A maternity patient breastfeeding for 1,5 mbs. Anamnesis states that one month ago
weeks has attended a doctor. She considers the patient had a surgery for left-sided
the onset of her disease to be when proporti- suppurative otitis and mastoiditis. He has
onal breast engorgement occurred. Mammary been undergoing treatment in an ENT
glands are painful. Body temperature is department. Approximately 2 weeks ago
36, 6o C . Expression of breast milk is hindered. the temperature increased, and the patient
The most likely diagnosis is: developed headaches. Objectively: heart rate
- 98/min., BP- 140/90 mm Hg, temperature -
A. Lactostasis 38, 3o C . Neurologically pronounced stiff neck:
B. Infiltrative mastitis bilateral Kernig’s symptom, unsteadiness
C. Suppurative mastitis during the Romberg’s maneuver. Computer
D. Chronic cystic mastitis tomography of the brain revealed a three-
E. Gangrenous mastitis dimensional growth with a capsule in the left
hemisphere. Make the diagnosis:
183. An 11-year-old boy complains of general
weakness, fever up to 38, 2o C , pain and swelli- A. Cerebral abscess
ng of the knee joints, sensation of irregular B. Echinococcus
heartbeat. 3 weeks ago, the child had a case C. Hemorrhage
of tonsillitis. Knee joints are swollen, the D. Hydrocephalus
overlying skin and skin of the knee region E. Arnold-Chiari malformation
is reddened, local temperature is increased,
movements are limited. Heart sounds are 187. Examination of a Rh-negative pregnant
Крок 2 Medicine (англомовний варiант, iноземнi студенти) 2016 рiк 24

woman at 32 weeks of gestation revealed A. Mitral valve stenosis


a four-time rise of Rh-antibody titer wi- B. Mitral valve failure
thin 2 weeks, the titer was 1:64. In the first C. Aortic outflow stenosis
two pregnancies the patient had experienced D. Aortic valve failure
antenatal fetal death due to hemolytic di- E. Tricuspid valve stenosis
sease. What is the optimal tactics of pregnancy
management? 191. A 64-year-old woman has been suffering
from diabetes mellitus for the last 14 years.
A. Early delivery Approximately 3 days ago the skin on the
B. Delivery at 37 weeks of gestation distal phalanx of the I toe on the left foot
C. Screening for Rh-antibodies 2 weeks later became cold and bluish-black in color. Mild
and early delivery in case of further titer rise pain is observed in the affected area. Pulse on
D. Introduction of anti-Rh (D) the pedal arteries cannot be detected, pulse
immunoglobulin on the popliteal artery is retained. Glycemia
E. Ultrasound for signs of hemolytic disease of is 12,4 mmol/l. US scan: stenosis of the shin
the fetus arteries, collateral compensated blood flow.
Ankle-brachial pressure index is 0,7. Foot X-
188. A 74-year-old patient visited a urologi- ray: destruction of the distal phalanx of the
st with complaints of pain above the pubis I toe. Determine the grade of diabetic foot
and inability to urinate for 8 hours. At home according to Wagner:
he had taken antispasmodics and had a
warm bath but no improvement occurred. A. IV
Objectively: the abdomen is soft and pai- B. I
nful above the pubis; dullness of percussion C. II
sound is observed above the pubis. Murphy’s D. III
(Pasternatski’s) punch sign is negative on the E. V
both sides. What condition does the patient 192. A 4-month-old boy has been undergoi-
have? ng in-patient treatment for pneumocystic
A. Acute urinary retention pneumonia for 4 weeks. The diagnosis has
B. Paradoxal ischuria been made based on clinical signs, typi-
C. Chronic urinary retention cal X-ray presentation, presence of severe
D. Anuria hypoxemia, positive dynamics caused by
E. Oliguria intravenous introduction of Biseptol (Co-
trimoxazole). Anamnesis states that enzyme-
189. A 7-year-old child became ill again 2 linked immuno sorbent assay (ELISA)
weeks after a case of tonsillitis. There are the detected antibodies to HIV in the umbilical
following complaints: temperature rise up to blood. Polymerase chain reaction (PCR) was
38o C , hemorrhagic rash on the extremities, performed on the child at the ages of 1 month
enlargement of the ankle joints. Blood test: and 3 months, and proviral DNA was detected
hemoglobin is 120 g/l, platelets are 170 · 109 /l, in the child’s blood. Viral load and number of
ESR is 30 mm/hour. Urine test: proteinuria CD4+-lymphocytes was not measured. Make
up to 0,7 g/l, cylinders - 5-6 in the field of vi- the diagnosis:
sion, erythrocytes - 8-10 in the field of vision.
What mechanism of hemorrhagic syndrome is A. HIV/AIDS
present in the given case? B. Infectious mononucleosis
C. Pneumonia
A. Vessel wall damage caused by immune D. Tuberculosis
complexes E. Adenovirus infection
B. Platelet dysfunction
C. Suppression of hematopoietic stem cells 193. A 35-year-old patient undergoing
D. Decrease of adhesive-aggregative function treatment for heart disorder in cardiologi-
of platelets cal department has developed complaints
E. Vessel wall damage caused by bacteria of acute sudden pain in the epigastrium,
temperature rise up to 38, 3o C . Blumberg’s
190. A 33-year-old patient has developed and Razdolsky’s (abduction of femur) signs
dyspnea during physical exertion, palpitati- are positive. What necessitates surgical aid in
ons, disruptions of heart rate, swollen legs. the given case?
In the childhood the patient had a case of
acute rheumatic fever that required in-patient A. Progress as a surgical disease
treatment. There were no further requests B. Extent of congenital and acquired
for medical care. Objectively: heart rate is development disorders
92/min., rhythmic; BP is 110/70 mm Hg. At C. Severity of the concomitant pathology
the apex the I heart sound is increased, triple D. Urgent aid is required to save the patient’s
rhythm, diastolic murmur. What heart disease life
is most likely? E. Activity of the heart disorder
194. A 5-year-old girl was hospitalized with
Крок 2 Medicine (англомовний варiант, iноземнi студенти) 2016 рiк 25

complaints of pain and swelling in the right A. Doxazosin


knee joint, temperature rise up to 38, 4o C and B. Propranolol
a rash diagnosed as erythema annulare centri- C. Indapamide
fugum. The signs developed 3 days after the D. Amlodipine
recovery from a case of acute respiratory di- E. Captopril
sease. Name the etiotropic drug to be prescri-
bed: 198. On the 9th day after childbirth the
obstetric patient developed high fever up
A. Augmentin to 38o C . She complains of pain in the right
B. Methotrexate mammary gland. The examination revealed
C. Metypred (Methylprednisolone) the following: a sharply painful infiltrate can
D. Diclofenac sodium be palpated in the right mammary gland, the
E. Captopril skin over the infiltrate is red, subareolar area
and nipple are swollen and painful. What is
195. During medical examination of a group your diagnosis?
of children under 4 years carried out by a
pediatric team in one of the African countri- A. Abscess of the right mammary gland
es a set of similar pathological signs was B. Mastopathy
detected in some of the children. The si- C. Cancer of the right mammary gland
gns are as follows: growth inhibition, mental D. Serous mastitis
changes, muscle atrophy, swellings, changes E. Fibrous cystic degeneration of the right
in hair and skin pigmentation. These children mammary gland
were diagnosed with kwashiorkor. What food
products should be added to the diet to treat 199. A 16-year-old girl addressed a doctor wi-
this disorder? th complaints of fatigability and dizziness. On
heart auscultation: systolic murmur in the II
A. Fish, vegetables, cereals intercostal area along the breastbone edge
B. Milk, meat, vegetables on the left. ECG revealed signs of the right
C. Vegetables, fruit ventricular hypertrophy. X-ray revealed di-
D. Cereals, fruit, berries latation of the the pulmonary artery trunk,
E. Poultry, fruit, berries enlargement of the right heart. What heart di-
sorder is it?
196. An 8-year-old girl was diagnosed wi-
th signs of Morgagni-Adams-Stokes disease A. Pulmonary artery outflow stenosis
that developed against the background of the B. Fallot’s tetrad
III degree atrioventricular heart block. What C. Defect of the interatrial septum
drug should be introduced intravenously for D. Coarctation of aorta
emergency aid? E. Pulmonary artery valve failure

A. Atropine 200. A 32-year-old patient complains of


B. Prednisolone excessive appetite, excess weight, dyspnea
C. Dobutamine during physical exertion. There are fat deposi-
D. Digoxin ts in the area of abdomen and shoulder gi-
E. Potassium chloride rdle. The skin is pale-pink, adult male pattern
of hair distribution is observed on the torso,
197. A 62-year-old man addresed a urologist no stretch marks. Heart rate is 90/min., BP
with complaints of frequent urination at night is 120/80 mm Hg, body build index equals
(5-6 times per night), sensation of incomplete 35. Blood sugar is 4,9 mmol/l, cholesterol is
voiding of the urinary bladder, pain in the 6,2 mmol/l. On ophthalmoscopy: fundus of
lower abdomen, slow urination. Anamnesis: the eye without changes. What provisional di-
the II degree essential hypertension (peak agnosis can be made?
BP is 160/100 mm Hg). Current case: the II
degree enlargement of the prostate gland, A. Primary alimentary constitutive obesity,
PSA is 2,2 ng/ml. Select the drug suitable for android type
long-term therapy of the patient’s combined B. Primary alimentary constitutive obesity,
pathology: gynoid type
C. Secondary hypothalamic obesity
D. Secondary neuroendocrine obesity
E. Secondary endocrine hypo-ovarian obesity
MINISTRY OF PUBLIC HEALTH OF UKRAINE

Department of human resources policy, education and science

Testing Board

Student ID Last name

Variant ___________________

Test items for licensing examination

Krok 2
MEDICINE
General Instruction
Every one of these numbered questions or unfinished statements in
this chapter corresponds to answers or statements endings. Choose the
answer (finished statements) that fits best and fill in the circle with the
corresponding Latin letter on the answer sheet.

Authors of items: Ahafonova O.O., Ambrozevych Z.M., Barannyk S.I., Berezniuk V.V., Berezov V.M.,
Bilenko O.A., Bilyk O.V., Bilyk V.D., Blikhar V.Ye., Bolbot Yu.K., Borzova O.Yu., Bukhtieieva E.R.,
Bukhtiyarova O.H., Buriak V.M., Butina L.I., Butvyn I.M., Chaika O.O., Chekanov S.L., Chelpan L.L.,
Chonka I.I., Chuiko Yu.M., Dankyna I.A., Dashchuk A.M., Demchenko T.V., Desiatska Yu.V., Drin T.M.,
Duplenko P.Yu., Dzis N.P., Filippova O.Yu., Franchuk O.A., Hahara V.F., Havrylova L.O., Henyk N.I.,
Herasymenko O.I., Hovalenkova O.L., Hrydasova V.D., Hubka O.V., Hyrla Ya.V., Kalinovska I.V.,
Kaliy V.V., Kalyberdenko V.B., Kandyba V.P., Karliychuk O.O., Kirieieva T.V., Klymenko A.V.,
Koliush O.I., Kompaniyets K.M., Kondratenko P.H., Kondratiev V.O., Konopkina L.I., Konovalova N.V.,
Kopchak O.V., Korovka S.Ya., Kovalchuk P.Ye., Kovalenko S.V., Kovtunenko R.V., Krut Yu.Ya.,
Kryvenko Z.F., Kryvonosov M.V., Kryzhanovsky Yu.M., Kudria V.I., Lavrinchuk I.O., Lakusta N.M.,
Lebediuk M.M., Leshchenko K.A., Liulka O.M., Makieieva N.I., Malovany V.V., Marushko Yu.V.,
Martsynik Ye.M., Melnychuk L.V., Mierkulova N.F., Mitiunina N.I., Miziuk V.V., Moroz I.M.,
Muravska O.M., Murtazin L.M., Mysak A.I., Nadraha O.B., Nechytailo Yu.M., Nerianov K.Yu.,
Nikolaichuk O.M., Nishkumai O.I., Nykoniuk T.R., Olifierovska R.P., Parashchuk Yu.S., Pashchenko I.V.,
Pertseva N.O., Petrynych V.V., Pisotska L.A., Pohorelov O.V., Proniv L.M., Pryshliak O.Ya.,
Radchenko O.M., Reitmaier M.Y., Riapolova T.L., Romanenko V.N., Rudai V.V., Rudenko M.M.,
Rudenko S.M., Ruzhytska O.O., Samardakova H.O., Semeniak A.V., Semenukha K.V., Sennikov I.A.,
Serheta I.V., Shapkin V.Ye., Shkrobanets I.D., Shorikov Ye.I., Shusterman T.Y., Shvyhar L.V.,
Sidykh N.M., Sierkova V.K., Sikorska M.V., Smachylo I.V., Smoliak L.L., Snizhko S.S., Soboleva N.P.,
Sokolov O.B., Soldak I.I., Sonnyk H.T., Sorochan V.V., Sotnik Yu.P., Suk V.H., Svyrydova V.V.,
Svystunov I.V., Sylenko H.Ya., Sychova V.V., Synoverska O.B., Tarasova V.I., Teliushchenko O.D.,
Tiuieva N.V., Todoriko L.D., Tokariev A.V., Tonkohlas O.A., Tretiakevych Z.M., Troian V.I.,
Tykhonova S.A., Ukhal M.I., Vankhanen N.V., Vatanska I.Yu., Volianska A.H., Voloshyna L.O.,
Vorokhta Yu.M., Vykhovanets T.A., Yakovenko I.K., Yermachenko T.P., Yevtushenko V.V.,
Yurchenko I.V., Yutanova A.V., Zakharov V.K., Zheliba M.D., Zoria A.V. and Committees of professional
expertise

Item reviewers. Agafonova O.O., Anisimov Ye.M., Bab’yak T.Ye., Chursina T.Ya., Dyndar O.A.,
Grygorov Yu.B., Grynzovs’ky A.M., Gubka V.O., Gutsalenko O.O., Kalinina S.Yu., Karapetyan K.G.,
Khrapach V.V., Kolesnyk O.M., Kolosovych I.V., Kondratyuk V.Ye., Kopcha V.S., Kravchenko O.V.,
Kutovy O.B., Kuz’mina I.Yu., Malanchuk L.M., Martynyuk L.P., Mishchenko V.P., Muravs’ka O.M.,
Petrushenko V.V., Prokhorova M.P., Pryshlyak O.Ya., Puzanova O.G., Pyptyuk O.V., Shestakova I.V.,
Shevtsova T.I., Stovban I.V., Tsvirenko S.M., Usenko S.G., Vakal’uk I.P., Volyans’ka A.G.

The book includes test items for use at licensing integrated examination “Krok 2. Medicine” and
further use in teaching.

The book has been developed for students of medical, pediatric and medical-and-prophylactic
faculties and academic staff of higher medical educational establishments.

Approved by Ministry of Public Health of Ukraine as examination and teaching


publication based on expert conclusions (Orders of MPH of Ukraine of
14.08.1998 №251, of 27.12.1999 №303, of 16.10.2002 №374, of 29.05.2003 №233).

© Copyright Testing Board.


Крок 2 Medicine (англомовний варiант, iноземнi студенти) 2017 рiк 1

1. A 32-year-old welder complains of 5. 5 weeks after hypothermia a 22-year-


weakness and fever. His illness initially old patient developed fever, weakness,
presented as tonsillitis one month earlier. On muscle pain, inability to move independently.
examination: body temperature - 38,9o C , RR- Objectively: tenderness, induration of
24/min., HR- 100/min., BP- 100/70 mm Hg, shoulder and shin muscles, restricted active
hemorrhages on the legs, enlargement of the movements, erythema on the anterior surface
lymph nodes. Complete blood count: Hb- 70 of the chest. There is a periorbital edema
g/l, RBC- 2, 2 · 1012 /l, WBC- 3, 0 · 109 /l wi- with heliotropic erythema. Gottron’s sign is
th 32% of blasts, 1% of eosinophils, 3% of present. What investigation is required for the
band neutrophils, 36% of segments, 20% of diagnosis verification?
lymphocytes, and 8% of monocytes, ESR- 47
mm/hour. What is the cause of anemia? A. Muscle biopsy
B. Aminotransferase activity
A. Acute leukemia C. Pneumoarthrography
B. Chronic lympholeukemia D. ASO titer
C. Aplastic anema E. Rheumatoid factor
D. B12 -deficient anemia
E. Chronic hemolytic anemia 6. A 37-year-old woman complains of
headaches, nausea, vomiting, spasms. The
2. After a 5-day-long celebration of his onset of the disease occurred the day before
daughter’s wedding a 65-year-old patient due to her overexposure to cold. Objecti-
”saw” in his yard many cats, chickens, and vely: fever up to 40o C ; somnolence; rigid
rats. He tried to chase them away, but was neck; Kernig’s symptom is positive on the
scared off when the animals started to scold both sides; general hyperesthesia. Blood test:
him and tried to harm him. Make the di- leucocytosis, increased ESR. Cerebrospinal
agnosis: fluid is turbid, yellow-tinted. What changes of
the cerebrospinal fluid are most likely?
A. Delirium tremens
B. Senile psychosis A. Neutrophilic pleocytosis
C. Schizophrenia B. Lymphocytic pleocytosis
D. Organic brain syndrome C. Blood in the cerebrospinal fluid
E. Reactive hallucinosis D. Xanthochromia in the cerebrospinal fluid
E. Albuminocytological dissociation
3. An 8-year-old boy developed a temperature
of 37, 5o C two days after his recovery from 7. A 44-year-old patient complains of diffi-
the case of URTI. He complains of suffocati- cult urination, sensation of incomplete uri-
on, heart pain. Objectively: the skin is pale, nary bladder emptying. Sonographic exami-
tachycardia, the I heart sound is weakened, nation of the urinary bladder near the
short systolyc murmur in the 4th intercostal urethra entrance revealed an oval well-
area near the left edge of the breastbone. defined hyperechogenic formation 2x3 cm
What heart disorder such clincal presentati- large that was changing its position during the
on is characteristic of? examination. What conclusion can be made?
A. Nonrheumatic myocarditis A. Stone
B. Primary rheumatic carditis B. Malignant tumour of the urinary bladder
C. Myocardiodystrophy C. Urinary bladder polyp
D. Fallot’s tetrad D. Prostate adenoma
E. Cardiomyopathy E. Primary ureter tumour
4. A 28-year-old woman has been delivered 8. 4 days after a patient received a gunshot
to a hospital with acute pain in the lower wound to the soft tissues of middle third
abdomen. There was a brief syncope. The of the thigh, his condition suddenly began
delay of menstruation is 2 months. Objecti- deteriorating. There are complaints of bursti-
vely: the patient has pale skin, BP- 90/50 mm ng pain in the wound; pain has been increasi-
Hg, Ps- 110/min. Lower abdomen is extremely ng during the last 12 hours. Edema of skin
painful. Vaginal examination reveals uterus and hypodermic tissue quickly grows. Body
enlargement. Promtov’s sign (pain during bi- temperature is 38,2o C , heart rate is 102/min.
manual gynecological examination) is posi- The wound edges gape, are dull in color;
tive. Right uterine appendages are enlarged the muscles, viable as of day before, now
and very painful. Posterior vault hangs over. protrude into the wound, look boiled, are dull
What is the most likely diagnosis? in color, have dirty-gray coating, and fall apart
when held with forceps. What infection has
A. Right-sided tubal pregnancy developed in the wound?
B. Right ovary apoplexy
C. Acute right-sided salpingoophoritis
D. Pelvioperitonitis
E. Incipient abortion
Крок 2 Medicine (англомовний варiант, iноземнi студенти) 2017 рiк 2

A. Anaerobic A. Surgical intervention


B. Aerobic gram-negative B. Continuation of conservative therapy
C. Putrid C. Physiotherapy
D. Aerobic gram-positive D. Sanatorium-and-spa treatment
E. Diphtheria of the wound E. Physical training
9. A 35-year-old patient has been sufferi- 13. Caries morbidity rate is 89% among resi-
ng from an illness for 3 days. 5 days ago he dents of a community. It is determined that
returned from a trip to Africa. The onset of fluorine content in water is 0,1 mg/l. What
disease was accompanied by fever up to 40o C , preventive measures should be taken?
chills, acute headache, myalgia. In the axi-
llary region the lymph node enlarged up to A. Water fluorination
3x6 cm can be palpated. The lymph node is B. Tooth brushing
dense, intensely painful, slightly mobile, wi- C. Fluorine inhalations
thout clear margins; the skin over the node D. Sealant application
is hyperenic and tight. Tachycardia is present. E. Introduction of more vegetables to the diet
Make the preliminary diagnosis:
14. A patient received flame burns of both
A. Plague hands. On the dorsal and palmar surface of
B. Sepsis the hands there are blisters filled with serous
C. Tularemia fluid. The wrist joint region is hyperemic. The
D. Lymphadenitis forearms were not injured. What is the provi-
E. Anthrax sional diagnosis?

10. A parturient woman complains of pain in A. II degree flame burn of the hands, 4% of
her mammary gland. In the painful area there body surface area
is an infiltration 3x4 cm in size with softened B. II degree flame burn of the hands, 2% of
center. Body temperature is 38,5o C . What is body surface area
the most likely diagnosis? C. IIIa degree flame burn of the hands, 4% of
body surface area
A. Acute suppurative mastitis D. III degree flame burn of the hands, 4% of
B. Pneumonia body surface area
C. Pleurisy E. IIb degree flame burn of the hands, 2% of
D. Milk retention body surface area
E. Birth trauma
15. During the periodic medical examination
11. A 52-year-old patient, who has been an assembly fitter (works on soldering detai-
suffering from angina pectoris, for 2 weeks ls) didn’t report any health problems. Closer
has increasingly frequent pain attacks in the examination revealed signs of asthenic-
area behind his sternum and his need for ni- vegetative syndrome. Blood included red
troglycerine has increased. Objectively: the blood cells with basophilic aggregations
condition is of moderate severity. The skin is and a somewhat higher number of reti-
pale. Heart sounds are weakened, rhythmic. culocytes, urine had a high concentration of
Heart rate is 84/min. ECG shows no signs of delta-aminolevulinic acid. The complex of
focal myocardial injury. What is the most li- symptoms indicates the initial stage of chronic
kely diagnosis? intoxication with:
A. Progressive angina pectoris A. Lead
B. First-time angina pectoris B. Manganese
C. Stable NYHA functional class II angina C. Mercury
pectoris D. Tin
D. Variant angina pectoris E. Ethanol
E. Acute cardiac infarction
16. A 29-year-old woman came to a
12. A 9-year-old boy has been suffering gynecologist with complaints of irritabili-
from multiple bronchiectasis since he was 3 ty, tearfulness, headache, nausea, occasional
years old. Exacerbations occur frequently (3- vomiting, pain in the heart area, tachycardia
4 times a year), after conservative therapy attacks, memory impairment, meteorism.
there are short remission periods. The di- These signs appear 6 days before menstruati-
sease progresses, the child is physically on and disappear the day before menstruation
underdeveloped, presents with pale skin, or during its first 2 days. On vaginal exami-
acrocyanosis, deformed nail plates in the nation: the uterus and uterine appendages
shape of ”clock-face”. Bronchography reveals are without alterations. What diagnosis is the
saccular bronchiectases in the lower lobe of most likely?
the right lung. What further treatment tactics
should be chosen?
Крок 2 Medicine (англомовний варiант, iноземнi студенти) 2017 рiк 3

A. Premenstrual syndrome A. Blood creatinine


B. Algodismenorrhea B. Blood bilirubin
C. Ovarian apoplexy C. Blood sodium
D. Genital endometriosis D. Uric acid
E. Neurosis E. Fibrinogen
17. Examination of a group of persons living 21. A worker of a glass-blowing workshop
on the same territory revealed the following complains of headache, irritability, visual
common symptoms: dark-yellow pigmentati- impairment - he sees everything as if through
on of the tooth enamel, diffuse osteoporosis a ”net”. Objectively: hyperemic sclera, thi-
of bone apparatus, ossification of ligaments ckened cornea, decreased opacity of pupi-
and joints, functional disorders of the central ls, visual acuity is 0,8 in the left eye, 0,7 in
nervous system. This condition may be caused the right eye. The worker uses no means of
by the excessive concentration of the followi- personal protection. What is the most likely
ng microelement in food or drinking water: diagnosis?
A. Fluorine A. Cataract
B. Copper B. Conjunctivitis
C. Nickel C. Keratitis
D. Iodine D. Blepharospasm
E. Cesium E. Progressive myopia
18. An infant has been born at the 41st week 22. For a week a 42-year-old patient has been
of gestation. The pregnancy was complicated suffering from fever attacks followed by high
with severe gestosis of the second semester. temperature, which occur each 48 hours. Body
The weight of the baby is 2400 g, the height is temperature raises up to 40o C and decreases
50 cm. Objectively: the skin is flabby, the layer in 3-4 hours with excessive sweating. The pati-
of subcutaneous fat is thin, hypomyotonia ent presents with loss of appetite and general
is observed, neonatal reflexes are weak. fatigue. The skin is pale and sallow. The li-
The internal organs are without pathologic ver and spleen are enlarged and dense on
alterations. This newborn can be assessed as palpation. What method of diagnosis verifi-
a: cation would be most efficient?
A. Full-term infant with prenatal growth A. Microscopy of blood smear and thick blood
retardation film
B. Premature infant B. Complete blood count
C. Immature infant C. Bacteriological analysis
D. Postmature infant D. Immune-enzyme assay
E. Full-term infant with normal body weight E. Microscopy of hanging blood drop
19. A patient suffering from infiltrati- 23. A 28-year-old patient is a drug addict. He
ve pulmonary tuberculosis was prescribed has been sick for a year, when noticed general
streptomycin, rifampicin, isoniazid, pyrazi- weakness, increased sweating, and weight
namide, vitamin C . One month after the begi- loss. He often had cases of respiratory di-
nning of the treatment the patient started seases. Within the last 2 days he demonstrates
complaining of reduced hearing and tinnitus. intermittent fever with profuse night sweati-
What drug has such a side effect? ng, increased general weakness, developed di-
arrhea with mucus and blood admixtures. On
A. Streptomycin examination: polylymphadenopathy, herpetic
B. Isoniazid rashes in the oral cavity; on abdominal
C. Rifampicin palpation: the liver and spleen are enlarged.
D. Pyrazinamide What is the most likely diagnosis?
E. Vitamin C
A. HIV-infection
20. A 39-year-old man complains of morni- B. Herpetic stomatitis
ng headaches, appetite loss, nausea, morni- C. Chronic lymphatic leukemia
ng vomiting, periodic nasal hemorrhages. The D. Colon cancer
patient had a case of acute glomerulonephritis E. Chronic sepsis
at the age of 15. Examination revealed rise of
arterial pressure up to 220/130 mm Hg, skin 24. A 40-year-old patient has acute onset
hemorrhages on his arms and legs, pallor of of disease caused by overexposure to cold.
skin and mucous membranes. What biochemi- Temperature has increased up to 39o C .
cal parameter is the most important for maki- Foul-smelling sputum is expectorated duri-
ng diagnosis in this case? ng coughig. Various moist crackles can be
auscultated above the 3rd segment on the ri-
ght. Blood test: leukocytes - 15, 0 · 109 /l, stab
neutrophils - 12%, ESR- 52 mm/hour. On X-
ray: in the 3rd segment on the right there is a
Крок 2 Medicine (англомовний варiант, iноземнi студенти) 2017 рiк 4

focus of shadow 3 cm in diameter, low density, A. Risk of late abortion with hemorrhage
with fuzzy smooth margins and a clearing in B. Risk of late abortion without hemorrhage
its center. What disease is most likely in the C. The process of late abortion
given case? D. Incomplete late abortion
E. Attempted late abortion
A. Pneumonia complicated by an abscess
B. Infiltrative tuberculosis 29. A 65-year-old man was diagnosed with
C. Peripheral pulmonary cancer B12 -deficient anemia and the treatment was
D. Cystic echinococcosis prescribed. A week later control blood test
E. Pulmonary cyst was performed. What would be the early indi-
cator of the therapy effectiveness?
25. A 48-year-old patient was found to have
diffuse enlargement of the thyroid gland, A. Increased number of reticulocytes
exophthalmia, weight loss of 4 kg in 2 months, B. Increased hemoglobin level
sweating. Objectively: HR- 105/min, BP- C. Megaloblastic hematopoiesis
140/70 mm Hg. Defecation act is normal. D. Normoblastic hematopoiesis
What kind of therapy is recommended in this E. Increased erythrocyte number
case?
30. A 35-year-old woman complains of heart
A. Mercazolil pain (”aching and drilling”) occurring mai-
B. Radioactive iodine nly in the morning in autumn and spring and
C. Propranolol irradiating to the neck, back and abdomen;
D. Lugol’s solution rapid heartbeat; low vitality. Occurrence of
E. Thyroxine this condition is not associated with physical
activity. In the evening, the patient’s condition
26. A 26-year-old woman is suspected to improves. Study of somatic and neurological
suffer from systemic lupus erythematosus due status and ECG reveal no pathology. What
to systemic lesions of skin, vessels, joints, pathology is most likely to have caused these
serous tunics, and heart that developed after clinical presentations?
photosensitization. The following is detected:
LE cells, antibodies to native DNA, isolated A. Somatization depression
anti-centromere antibodies, rheumatoid B. Resting stenocardia
factor is 1:100, Wassermann reaction is posi- C. Pseudoneurotic schizophrenia
tive, circulating immune complex is 120 uni- D. Neurocirculatory asthenia
ts. What immunological indicators are consi- E. Hypochondriacal depression
dered to be specific to this disease?
31. On the 4th day after recovery from a
A. DNA antibodies cold a patient was hospitalized with complai-
B. Rheumatoid factor nts of solitary spittings of mucoid sputum.
C. Anti-centromere antibodies On the 2nd day there was a single discharge
D. Immunoglobulin A of about 250 ml of purulent blood-streaked
E. Increased circulating immune complex sputum. Objectively: the patient’s conditi-
on is moderately severe. Respiratory rate -
27. A woman came to a doctor with complai- 28-30/min., Ps- 96/min., BP- 110/70 mm Hg.
nts of increased body temperature up to Respiration over the left lung is vesicular,
37,8o C and moderately sore throat for the last over the right lung - weakened. There are
3 days. Objectively: mandibular lymph nodes various moist crackles over the lower lobe
are enlarged up to 3 cm. Palatine tonsils are and amphoric breath sounds near the angle of
hypertrophied, with gray coating that spreads scapula. What is the most likely diagnosis?
to the uvula and anterior pillars of the fauces.
What is the most likely diagnosis? A. Acute pulmonary abscess
B. Exudative pleuritis
A. Oropharyngeal diphtheria C. Acute focal pneumonia
B. Infectious mononucleosis D. Pleural empyema
C. Pseudomembranous (Vincent’s) tonsillitis E. Pyopneumothorax
D. Agranulocytosis
E. Oropharyngeal candidiasis 32. A 65-year-old woman on abdominal
palpation presents with a tumor in the umbi-
28. A primigravida at the term of 20 weeks lical region and above it; the tumor is 13x8 cm
complains of pain in her lower abdomen, in size, moderately painful, non-mobile, pulsi-
smearing blood-streaked discharge from the ng. On auscultation systolic murmur can be
genital tracts. Uterine tone is increased, fetus observed. What is the most likely diagnosis?
is mobile. On vaginal examination: the uterus
is enlarged according to the term, uterine A. Abdominal aneurysm
cervix is shortened to 0,5 cm, external cervical B. Gastric tumor
orifice is open by 2 cm. What is the most likely C. Arteriovenous aneurysm
diagnosis? D. Tricuspid insufficiency
E. Bicuspid insufficiency
Крок 2 Medicine (англомовний варiант, iноземнi студенти) 2017 рiк 5

33. A 45-year-old man complains of cough fi- 37. A patient with suspected pheochromocytoma
ts and tickling in his nasopharynx. He had has normal blood pressure in the periods
been staying for 10 days in the polluted area between the atacks and a tendency towards
created by the Chornobyl nuclear power plant tachycardia. Urine test revealed no pathologi-
accident. Rhinoscopy shows signs of severe es. It was decided to use a provocative test
nasopharynx irritation. What radionuclide is with histamine. What medication should be
the cause of this irritation? prepared to provide emergency care in case
of positive test result?
A. Radioactive iodine
B. Radioactive cesium A. Phentolamine
C. Radioactive strontium B. Pipolphen
D. Radioactive plutonium C. Nifedipine
E. Radioactive cobalt D. Mesatonum
E. Prednisolone
34. A 20-year-old patient complains of severe
headache, double vision, weakness, fever, 38. A 42-year-old woman complains of severe
irritability. Objectively: body temperature is pulsing headache in the frontoparietal area,
38,1o C , the patient is reluctant to contact, vertigo, palpitations. She has been sufferi-
sensitive to stimuli. There are ptosis of ng from hypertension for 3 years. Significant
the left eyelid, exotropia, anisocoria S>D, increase of BP occurs 2-3 times per month
pronounced meningeal syndrome. On lumbar and lasts for 3-8 hours. The left ventricle is
puncture the cerebrospinal fluid flowed out enlarged, heart sounds are clear, heart rate
under a pressure of 300 mm Hg, the fluid - 105/min., BP- 225/115 mm Hg. ECG: signs
is clear, slightly opalescent. 24 hours later of left ventricular hypertrophy. What drug
there appeared fibrin film. Protein - 1,4 g/l, would be the most effective for termination
lymphocytes - 600/3 per mm3 , sugar - 0,3 of cerebral crisis attack?
mmol/l. What is the provisional diagnosis?
A. Labetalol
A. Tuberculous meningitis B. Hydrochlorothiazide (Hypothiazide)
B. Meningococcal meningitis C. Captopril
C. Lymphocytic (Armstrong’s) meningitis D. Losartan
D. Syphilitic meningitis E. Clonidine (Clophelin)
E. Mumps meningitis
39. A 45-year-old patient complains of pain
35. A 37-year-old worker during a fire ended in the epigastric region, left subcostal area,
up in the area of high CO concentration. abdominal distension, diarrhea, loss of wei-
He was delivered to a hospital in unconsci- ght. He has been suffering from this condition
ous state. Objectively: the skin of his face for 5 years. Objectively: the tongue is moist
and hands is crimson. Respiration rate is with white coating near the root; deep palpati-
20/min. ECG: alterations specific for hypoxic on of abdomen reveals slight pain in the epi-
myocardium. Hourly diuresis is 40 ml. Blood gastric region and Мауо-Robson’s point. Li-
test: erythrocytes - 4, 5 · 1012 /l, Нb- 136 ver is painless and protrudes by 1 cm from the
g/l, color index - 0,89, ESR- 3 mm/hour, costal arch. Spleen cannot be palpated. What
carboxyhemoglobin - 5%. What criterion disease can be primarily suspected?
allows determining the severity of the pati- A. Chronic pancreatitis
ent’s condition? B. Atrophic gastritis
A. Carboxyhemoglobin concentration C. Peptic stomach ulcer
B. Respiratory disorders D. Chronic cholecystitis
C. ECG results E. Chronic enteritis
D. Extent of trophic disorders 40. For three years a 31-year-old woman has
E. Development of chronic renal failure been complaining of pain and swelling of
36. After a case of purulent otitis a 1-year-old her radiocarpal and metacarpophalangeal
boy has developed pain in the upper third of articulations, their reduced mobility in the
the left thigh, body temperature up to 39o C . morning, which persisted up to 1,5 hours.
Objectively: swelling of the thigh in its upper Two weeks ago she developed pain, swelli-
third and smoothed out inguinal fold. The li- ng, and reddening of her knee joints, her
mb is in semiflexed position. Active and passi- body temperature increased up to 37,5oC .
ve movements are impossible due to severe The treatment was belated. Examination of
pain. What diagnosis is the most likely? the internal organs revealed no pathologic
alterations. Diagnosis of rheumatoid arthritis
A. Acute hematogenous osteomyelitis was made. What alterations are the most li-
B. Acute coxitis kely to be visible on the arthrogram?
C. Intermuscular phlegmon
D. Osteosarcoma
E. Brodie’s abscess
Крок 2 Medicine (англомовний варiант, iноземнi студенти) 2017 рiк 6

A. Joint space narrowing, usuration resins. Objectively: lesion foci are symmetri-
B. Joint space narrowing, subchondral cally localized on both hands. Against the
osteosclerosis background of erythema with blurred margi-
C. Cysts in the subchondral bone ns there are papulae, vesicles, erosions, crusts,
D. Numerous marginal osteophytes and scales. What is the most likely pathology?
E. Epiphyseal osteolysis
A. Occupational eczema
41. A 58-year-old woman complains of B. Idiopathic eczema
spontaneous bruises, weakness, bleeding C. Allergic dermatitis
gums, dizziness. Objectively: the mucous D. Simple contact dermatitis
membranes and skin are pale with numerous E. Erythema multiforme
hemorrhages of various time of origin. Lymph
nodes are not enlarged. Ps- 100/min, BP- 45. A 10-year-old boy with symptoms of
110/70 mm Hg. There are no alterations of arthritis and myocarditis was delivered into
internal organs. Blood test results: RBC- a hospital. Based on clinical examination the
3, 0 · 1012 /l, Нb- 92 g/l, color index - 0,9, preliminary diagnosis of juvenile rheumatoid
anisocytosis, poikilocytosis, WBC- 10 · 109 /l, arthritis was made. What symptom is the most
eosinophils - 2%, stab neutrophils - 12%, contributive for the diagnostics of this di-
segmented neutrophils - 68%, lymphocytes sease?
- 11%, monocytes - 7%, ESR- 12 mm/h. What A. Reduced mobility of the joints in the
laboratory test should be performed additi- morning
onally to make the diagnosis? B. Regional hyperemia of the joints
A. Platelets C. Affection of the large joints
B. Reticulocytes D. Enlarged heart
C. Clotting time E. Increased heart rate
D. Osmotic resistance of erythrocytes 46. A 25-year-old patient was delivered to
E. Fibrinogen an infectious diseases unit on the 3rd day of
42. After tonsillectomy a woman with illness with complaints of headache, pain in
systemic lupus erythematosus, who has been the lumbar spine and gastrocnemius muscles,
taking prednisolone for a year, developed high fever, chills. Objectively: condition is of
acute weakness, nausea, vomiting, pain in the moderate severity. Scleras are icteric. Pharynx
right iliac area, watery stool up to 5 times is hyperemic. Tongue is dry with dry brown
per day. Ps- 96/min., BP- 80/50 mm Hg. What coating. Abdomen is distended. Liver is
preventive therapy should have been admini- enlarged by 2 cm. Spleen is not enlarged.
stered prior to the surgery? Palpation of muscles, especially gastrocnemi-
us muscles, is painful. Urine is dark in color.
A. Increase of prednisolone dosage Feces are normal in color. What is the most
B. Prescription of desoxycorticosterone likely diagnosis?
acetate
C. Prescription of antibiotics A. Leptospirosis
D. Administration of Ringer’s solution B. Viral hepatitis type A
E. Administration of 10% N aCl solution C. Malaria
D. Infectious mononucleosis
43. A 45-year-old woman complains of E. Yersiniosis
paroxysmal intolerable facial pain on the left
with attacks that last for 1-2 minutes. Attacks 47. A 28-year-old woman complains of
are provoked by chewing. The disease onset skin hemorrhages after minor traumas and
was two month ago after overexposure to spontaneous appearance of hemorrhages on
cold. Objectively: pain at the exit points of the the front of her torso and extremities. On
trigeminal nerve on the left. Touching near examination: the skin is variegated (old and
the wing of nose on the left induces new pain new hemorrhages), bleeding gums. Blood
attack with tonic spasm of the facial muscles. platelets - 20·109 /l; in the bone marrow there is
What is the most likely diagnosis among those increased number of megakaryocytes and no
listed? platelet production. Treatment with steroid
hormones was effective. What disease is it?
A. Trigeminal neuralgia
B. Glossopharyngeal neuralgia A. Idiopathic thrombocytopenic purpura
C. Temporomandibular joint arthritis B. Hemophilia
D. Facial migraine C. Rendu-Osler-Weber disease (Hereditary
E. Maxillary sinusitis hemorrhagic telangiectasia)
D. Disseminated intravascular coagulation
44. A 28-year-old man complains of skin E. Acute vascular purpura
rash and itching on the both of his hands.
The condition persists for 1,5 years. The 48. A 74-year-old patient was delivered into
exacerbation of his condition he ascribes to admission room with clinical presentations of
the occupational contact with formaldehyde acute deep vein thrombosis of the shin. What
Крок 2 Medicine (англомовний варiант, iноземнi студенти) 2017 рiк 7

symptom is the most typical of this pathology? A. Renal amyloidosis


B. Chronic glomerulonephritis
A. Homans’ sign C. Acute glomerulonephritis
B. Rovsing’s sign D. Heart failure
C. Courvoisier’s sign E. Chronic pyelonephritis
D. Mayo-Robson’s sign
E. Grey Turner’s sign 52. A 23-year-old man had taken 1 g of aspirin
to treat acute respiratory infection. After that
49. A 9-month-old child presents with fever, he developed an asthmatic fit with labored
cough, dyspnea. The symptoms appeared 5 expiration that was arrested by introduction
days ago after a contact with a person wi- of aminophylline. The patient has no medical
th URTI. Objectively: the child is in grave history of allergies. The patient has undergone
condition. Temperature is 38o C , cyanosis of two surgeries for nasal polyposis in the past.
nasolabial triangle is present. RR- 54/min, What is the most likely diagnosis?
nasal flaring during breathing is observed.
There was percussion dullness on the right A. Aspirin-induced asthma
below the scapula angle and tympanic sound B. Atopic bronchial asthma
over the other areas of lungs. Auscultati- C. Infectious allergic bronchial asthma
on revealed bilateral fine moist crackles D. Exercise-induced asthma
predominating on the right. What is the most E. Symptomatic bronchospasm
likely diagnosis?
53. A 50-year-old patient was delivered to
A. Pneumonia a hospital with complaints of blood traces
B. URTI in urine. Urination is painless and undi-
C. Acute laryngotracheitis sturbed. Macrohematuria had been observed
D. Acute bronchitis for 3 days. Objectively: kidneys cannot be
E. Acute bronchiolitis palpated, suprapubic area is without alterati-
ons, external genitalia are non-pathologic. On
50. The mother of a 3-month-old child came rectal investigation: prostate is not enlarged,
to a family doctor with complaints of her painless, has normal structure. Cystoscopy
child being physically underdeveloped and revealed no alterations. What is the most li-
suffering from cough attacks and dyspnea. kely diagnosis?
Anamnesis: the child is the result of the
second full-term pregnancy with the risk of A. Renal carcinoma
miscarriage (the first child died of pulmonary B. Bladder tuberculosis
pathology at the age of 4 months, according C. Varicocele
to the mother). Body mass at birth is 2500 D. Dystopic kidney
g. Cough attacks were observed from the fi- E. Necrotic papillitis
rst days of life, twice the child was treated 54. A man complains of constant dull pain
for bronchitis. Considering the severity of the in the perineum and suprapubic area, weak
child’s condition the doctor made the referral flow of urine, frequent difficult painful
for hospitalization. What diagnosis was most urination, nocturia. The patient has been
likely stated in the referral? suffering from this condition for several
A. Mucoviscidosis (Cystic fibrosis) months, during which urination was becomi-
B. Acute obstructive bronchitis ng increasingly difficult, and pain in the peri-
C. Recurrent obstructive bronchitis neum has developed. On rectal examinati-
D. Pertussis on: the prostate is enlarged (mainly its right
E. Acute obstructive pneumonia lobe), dense, asymmetrical, central fissure is
smoothed out, the right lobe is of stony densi-
51. A 46-year-old man notes swollen legs, ty, painless, tuberous. What disease is it?
weakness, sensation of fullness and heavi-
ness in the right subcostal area; it is the A. Prostate cancer
first occurrence of these signs in the pati- B. Prostate sclerosis
ent. The patient has 20-year-long history of C. Urolithiasis, prostatolith of the right lobe
rheumatoid arthritis. The liver and spleen D. Prostate tuberculosis
are enlarged and dense. Blood creatinine - E. Chronic congestion prostatitis
0,23 mmol/l, proteinemia - 68 g/l, cholesterol
- 4,2 mmol/l, urine specific gravity - 1012, 55. A boy was born at 32 weeks of gestation. 2
proteinuria - 3,3 g/l, isolated wax-like cyli- hours after the birth he developed respiratory
nders, leached erythrocytes in the vision field, distress (RD). The RD severity assessed by
leukocytes - 5-6 in the vision field. What is the Silverman score was 5. The respiratory di-
most likely complication? sorders progressed, respiratory failure could
not be eliminated by Martin-Bouyer CPAP
(continuous positive airway pressure). X-ray
of lungs shows reticular and nodular pattern,
air bronchogram. What is the most likely
cause of respiratory distress syndrome?
Крок 2 Medicine (англомовний варiант, iноземнi студенти) 2017 рiк 8

60. An infant is 2,5 months old. The onset of


A. Hyaline membrane disease the disease was gradual, the child had normal
B. Segmental atelectasis body temperature but presented with slight
C. Bronchopulmonary dysplasia cough. Within a week the cough intensified,
D. Congenital pulmonary emphysema especially at night; on the 12th day the child
E. Edematous hemorrhagic syndrome developed cough fits occurring up to 20 times
per day and followed by vomiting. There was
56. An 8-year-old child with a 3-year-long one instance of respiratory arrest. Make the
history of diabetes was hospitalized in diagnosis:
hyperglycemic coma. Specify the initial dose
of insulin to be administered: A. Pertussis
B. Parainfluenza
A. 0,1-0,2 U/kg of body weight per hour C. Congenital stridor
B. 0,05 U/kg of body weight per hour D. Respiratory syncytial infection
C. 0,2-0,3 U/kg of body weight per hour E. Adenovirus infection
D. 0,3-0,4 U/kg of body weight per hour
E. 0,4-0,5 U/kg of body weight per hour 61. Vaginal examination reveals the head of
the fetus, which fills the posterior surface of
57. A 17-year-old young man complains of symphysis pubis and hollow of the sacrum.
general weakness, trismus, twitching of the The lower edge of symphysis pubis, ischi-
muscles in his left shin. 7 days ago he pierced adic spines, and sacrococcygeal joint can be
his foot with a nail. Objectively: at the sole palpated. Where in the lesser pelvis is the fetal
of the foot there is a wound, 0,3х0,2 mm in head situated?
size, with small amount of serous-purulent
discharge, the skin around the wound is A. In the narrow plane of lesser pelvis
hyperemic. What is the most likely diagnosis? B. In the wide plane of lesser pelvis
C. Above pelvic inlet
A. Tetanus D. In the area of brim
B. Phlegmon E. In the area of pelvic outlet
C. Osteomyelitis
D. Infected wound 62. A man works in casting of nonferrous
E. Erysipelas metals and alloys for 12 years. In the air
of working area there was registered high
58. A patient with trauma of the lower thi- content of heavy metals, carbon monoxi-
rd of the forearm volar surface caused by a de, and nitrogen. During periodic health
glass shard came to a first-aid center. Objecti- examination the patient presents with
vely: flexion of the IV and V fingers is asthenovegetative syndrome, sharp pains
impaired, sensitivity of the inner dorsal and in the stomach, constipations, pain in the
palmar surfaces of the hand and IV finger is hepatic area. In urine: aminolevulinic acid
decreased. What nerve is damaged? and coproporphyrin are detected. In blood:
A. Ulnar reticulocytosis, low hemoglobin level. Such
B. Radial intoxication is caused by:
C. Median A. Lead and lead salts
D. Musculocutaneous B. Tin
E. Axillary C. Carbon monoxide
59. A 58-year-old patient complains of D. Nitric oxide
pain in the lower left extremity, which E. Zinc
aggravates during walking, sensation of cold 63. A 30-year-old multigravida has been
and numbness in the both feet. The patient in labour for 18 hours. 2 hours ago the
has been suffering from this condition for 6 pushing stage began. Fetal heart rate is
years. Objectively: the skin is pale and dry, clear, rhythmic, 136/min. Vaginal examinati-
with hyperkeratosis. On the left shin hair is on reveals complete cervical dilatation, the
scarce, ”furrow” symptom of inflamed ingui- fetal head in the pelvic outlet plane. Sagittal
nal lymph nodes is positive. Pulse cannot be suture is in line with obstetric conjugate, the
detected over the pedal and popliteal arteri- occipital fontanel is near the pubis. The pati-
es and is weakened over the femoral artery. ent has been diagnosed with primary uterine
In the right limb popliteal artery pulsation is inertia. What is the further tactics of labour
retained. What is the most likely diagnosis? management?
A. Arteriosclerosis obliterans of the lower A. Outlet forceps
extremities B. Labour stimulation
B. Obliterating endarteritis C. Cesarean section
C. Femoral artery thrombosis D. Skin-head Ivanov’s forceps
D. Raynaud’s disease E. Vacuum extraction of the fetus
E. Buerger’s disease (thromboangiitis obli-
terans) 64. A patient, who had eaten canned
Крок 2 Medicine (англомовний варiант, iноземнi студенти) 2017 рiк 9

mushrooms (honey agaric) three days A. Lactational mastitis


ago, developed vision impairment (di- B. Abscess
plopia, mydriasis), speech disorder, disturbed C. Erysipelas
swallowing. What type of food poisoning D. Dermatitis
occurred in the patient? E. Tumor

A. Botulism 69. A 3-year-old girl is being treated at a


B. Food toxicoinfection resuscitation unit with diagnosis ”acute ki-
C. Fusariotoxicosis dney failure, oligoanuric stage”. ECG: high T
D. Honey agaric poisonong wave, extended QRS complex, displacement
E. Lead salts poisoning of S-T interval downwards below the isoline.
What electrolyte imbalance is it?
65. An infant is 3 weeks old. Since birth
there has been observed periodical vomiti- A. Hyperkalemia
ng within a few minutes after feeding. The B. Hypokalemia
amount of vomitive masses does not exceed C. Hypocalcemia
the volume of previous feeding. The infant D. Hypercalcemia
has age-appropriate body weight. What is the E. Hyperphosphatemia
most likely cause of this symptom? 70. Mother of an 8-year-old girl complains
A. Pylorospasm that the child is too short and has excessi-
B. Esophageal chalasia ve body weight. Objectively: obesity with fat
C. Adrenogenital syndrome deposits on the torso and face (round moon-
D. Pyloristenosis like face), acne, striae on the thighs and lower
E. Esophageal achalasia abdomen, hirsutism. What hormone can cause
such symptoms, when in excess?
66. A newborn with gestational age of 31
weeks presents with hypotonia and depressed A. Cortisol
consciousness. Hematocrit is 35%, general B. Thyroxin
cerebrospinal fluid analysis shows increased C. Testosterone
content of erythrocytes and protein, and low D. Insulin
glucose. These data correspond with the clini- E. Glucagon
cal presentation of:
71. A 9-year-old girl complains of fever up to
A. Intracranial hemorrhage 37,5oC , headache, inertness, weakness, loss
B. Meningitis of appetite, stomachache, and frequent pai-
C. Sepsis nful urination. Provisional diagnosis of acute
D. Anemia pyelonephritis is made. Clinical urine analysis:
E. Prenatal infection specific gravity - 1018, no protein, leukocytes
- 10-15 in the vision field. What investigati-
67. From urine of a 14-year-old boy with on method can verify the diagnosis of urinary
the exacerbation of secondary obstructive system infection?
pyelonephritis Pseudomonas aeruginosa was
isolated with a titer of 1000000 microbes per A. Bacteriological inoculation of urine
1 ml. What antibiotic is the most advisable in B. Rehberg test (creatinine clearance test)
this case? C. Zymnytsky test (measurement of daily
diuresis)
A. Ciprofloxacin D. Complete blood count
B. Ampicillin E. Clinical urine analyses, dynamic testing
C. Cefazolinum
D. Azithromycin 72. During assessment of work conditions
E. Chloramphenicol at the mercury thermometer manufacture,
content of mercury vapors in the air of worki-
68. A 20-year-old woman on the 10th day ng area is revealed to exceed maximum
after her discharge from the maternity ward concentration limit. Specify the main way of
developed fever up to 39o C and pain in her mercury penetration into the body:
left mammary gland. On examination the
mammary gland is enlarged, in its upper outer A. Respiratory organs
quadrant there is a hyperemic area. In this B. Intact skin
area a dense spot with blurred margins can be C. Damaged skin
palpated. The patient presents with lactostasis D. Gastrointestinal tract
and no fluctuation. Lymph nodes in the ri- E. Mucous tunics
ght axillary crease are enlarged and painful.
Specify the correct diagnosis: 73. During health assessment of car drivers
and police officers on point duty, the physici-
ans detected carboxyhemoglobin in the blood
of the patients, weakened reflex responses,
disturbed activity of a number of enzymes.
Крок 2 Medicine (англомовний варiант, iноземнi студенти) 2017 рiк 10

Revealed professional health disorders are A. Sheehan’s syndrome (postpartum hypopi-


most likely to be associated with the effect of: tuitarism)
B. Ovarian amenorrhea
A. Carbon monoxide C. Turner’s syndrome
B. Sulfurous anhydride D. Ovarian exhaustion syndrome
C. Mental stress E. Galactorrhea-amenorrhea syndrome
D. Aromatic hydrocarbons
E. Nitric oxide 78. On laboratory investigation of a pork
sample there is 1 dead trichinella detected
74. On the 3rd day of life a newborn, in 24 sections. This meat should be:
who had suffered birth asphyxia, developed
hemorrhage from the umbilical wound. A. Handed over for technical disposal
Laboratory analysis reveals hypocoagulati- B. Allowed for sale with no restrictions
on, thrombocytopenia, and hypothrombi- C. Processed and sold through public catering
nemia. What is the cause of such clinical network
developments? D. Processed for boiled sausage production
E. Frozen until the temperature of - 10o C is
A. Disseminated intravascular coagulation reached in the deep layers, with subsequent
B. Hemorrhagic disease of newborn exposure to cold for 15 days
C. Congenital angiopathy
D. Thrombocytopenic purpura 79. Examination of a 43-year-old man
E. Umbilical vessel trauma objectively revealed pallor of skin and
mucous membranes, loss of tongue papillae,
75. A 46-year-old woman came to a materni- transverse striation of fingernails, cracks in
ty clinic with complaints of moderate blood the mouth corners, tachycardia. Blood test
discharge from the vagina, which developed results: Hb- 90 g/l, anisocytosis, poikilocytosis.
after the menstruation delay of 1,5 months. The most likely causative factor of this condi-
On vaginal examination: the cervix is clean; tion is the inadequate intake of:
the uterus is not enlarged, mobile, painless;
appendages without changes. Make the di- A. Iron
agnosis: B. Copper
C. Zinc
A. Dysfunctional uterine bleeding D. Magnesium
B. Adenomyosis E. Selene
C. Ectopic pregnancy
D. Submucous uterine myoma 80. A 52-year-old woman presents with
E. Cancer of the uterine body affected mucosa in the mouth angles where
fissures, erosions, and ulcers develop; verti-
76. A patient has the second and third degree cal fissures appear on the lips during their
burns of the 15% of the body surface. On the closing (cheilosis); there are tongue alterati-
20th day after the trauma the patient presents ons (glossitis), angular stomatitis, seborrheic
with sharp increase of body temperature, dermatitis around the mouth and wings of
general weakness, rapid vesicular respiration; the nose, and pericorneal injection. The listed
facial features are sharpened, BP is 90/50 mm symptoms are characteristic of:
Hg, heart rate is 112/min. What complication
is it? A. B2 -hypovitaminosis
B. B1 -hypovitaminosis
A. Sepsis C. C -hypovitaminosis
B. Pneumonia D. P P -hypovitaminosis
C. Acute intoxication E. A-hypovitaminosis
D. Purulent bronchitis
E. Anaerobic infection 81. Two days ago a woman fell from the hei-
ght of 1,5 m. She complains of severe thoracic
77. A 30-year-old woman complains of pain on the left and dyspnea. Chest X-ray
amenorrhea that lasts for 2 years after she reveals hydropneumothorax on the left with
has given birth, loss of hair and body weight. fluid level at the 7th rib and the lung collapsed
The labor was complicated with hemorrhage by 1/3. The 6th-7th ribs are fractured along
caused by uterine hypotonia. Objectively the the scapular line. Serohemorrhagic fluid was
patient is of asthenic type, her external geni- obtained during thoracic puncture. What
talia are hypoplastic, the uterine body is small treatment tactics should be chosen?
in size and painless. No uterine appendages
can be detected. What is the most likely di-
agnosis?
Крок 2 Medicine (англомовний варiант, iноземнi студенти) 2017 рiк 11

A. Thoracocentesis on the left at the 7th shoulders, the head is bowed forward, the
intercostal space thorax is flattened, the stomach is bulging.
B. Thoracocentesis at the 2nd intercostal space In the vertebral column there are deepened
C. Repeated puncture of the pleural cavity cervical and lumbar flexures. What posture
D. Intubation of trachea with artificial does the child have?
pulmonary ventilation
E. Alcohol-novocaine block of the fracture A. Kyphosis
area B. Lordosis
C. Stooping
82. A patient complains of suppuration from D. Corrected
the ear and impaired hearing of the left ear, E. Normal
which have been observed for the past 6
years. The patient had periodical headaches, 86. A 46-year-old woman has been hospitali-
general indisposition, fever. Objectively: zed with open fracture of the left thigh in
otoscopy of the external auditory meatus its middle third. She underwent the surgery
revealed mucopurulent odorless substance. - fixation with extraosseous osteosynthesis
The eardrum is of normal color, with central plates. On the 4th day after the surgery
perforation. What is the most likely di- she developed pain in the wound, body
agnosis? temperature rose over 39o C . What measures
should be taken in this case?
A. Chronic mesotympanitis
B. Otosclerosis A. Undo the sutures, drain the wound, and
C. Acute otitis media prescribe antibiotics
D. Chronic epitympanitis B. Prescribe broad spectrum antibiotics and
E. Chronic sensorineural hearing loss hormonal agents
C. Administer antibiotics intraosseously and
83. In 2 hours after a traffic accident a 28-year- hypothermia locally
old man in grave condition was delivered to a D. Inject antibiotics into the area surroundi-
hospital. The patient complains of abdominal ng the wound, prescribe spasmolytics and
pain. He received a blow to the abdomen wi- analgesics
th the steering wheel. Objective examination E. Remove the fixation, prescribe sulfanilami-
revealed the following: the abdomen does not des
participate in respiration, is tense and acutely
painful on palpation; the abdominal muscles 87. A multigravida on the 38th week of her
are defensively tense, peritoneal irritation si- pregnancy complains of increased BP up to
gns are positive, hepatic dullness is absent. BP 140/90 mm Hg, edema of the shins for 2 weeks.
is 90/60 mm Hg, heart rate is 120/min. What In the last month she gained 3,5 kg of weight.
further treatment tactics should be chosen? Urine analysis: protein - 0,033 g/l. Make the
diagnosis:
A. Laparotomy
B. Laparoscopy A. Mild preeclampsia
C. Cold to the abdomen B. Moderate preeclampsia
D. Ultrasound investigation C. Pregnancy hypertension
E. Laparocentesis D. Severe preeclampsia
E. Pregnancy edema
84. A 28-year-old woman complains of
increased intervals between menstruations, 88. A 44-year-old patient with postinfarcti-
up to 2 months, and hirsutism. Gynecological on cardiosclerosis presents with frequent
examination revealed the following: ovaries heart rate disorders and lower extremity
are enlarged, painless, and dense; no alterati- edema. Objectively: Ps- 95/min., irregular,
ons of the uterus. US of the lesser pelvis: 10-12 extrasystoles per minute. BP- 135/90
ovaries are 4-5 cm in diameter, with numerous mm Hg. The 1st heart sound at the apex is
enlarged follicles on the periphery. X-ray of weakened. Pulmonary respiration is rough.
the skull base: sellar region is widened. What The liver is enlarged +2 cm. ECG: irregular
is the most likely diagnosis? sinus rhythm, heart rate - 95/min, frequent
polytopic ventricular extrasystoles. What anti-
A. Stein-Leventhal syndrome (polycystic arrhythmic drug is advisable in this case for
ovarian syndrome) treatment and prevention of extrasystole?
B. Algodismenorrhea
C. Sheehan syndrome (postpartum hypopitui- A. Amiodarone
tarism) B. Lidocaine
D. Premenstrual syndrome C. Mexiletine
E. Morgagni-Stewart-Morel syndrome D. Quinidine
(metabolic craniopathy) E. Novocainamide (Procainamide)

85. Posture of an 11-year-old boy was determi- 89. A 60 year-old woman has been suffering
ned during preventive examination. The chi- from weakness, dizziness, and fatigue over
ld presents with curled forward rounded the last year. Recently she has also developed
Крок 2 Medicine (англомовний варiант, iноземнi студенти) 2017 рiк 12

dyspnea, paresthesia. Objectively: the skin A. Hepatocellular carcinoma of the liver


and mucous membranes are pale and slightly B. Hepatocellular insufficiency
icteric. The tongue is smooth due to the loss of C. Hypersplenism
lingual papillae. Liver and spleen are located D. Cholestasis
at the costal margin. Blood count: Hb- 70 g/l, E. Congestive heart failure
RBC- 1, 7·1012 /l, color index - 1,2, macrocytes. 93. A 43-year-old man has undergone a
Administer the patient a pathogenetically surgery for osteomyelitis of the left thigh.
justified drug: On the 6th day the patient’s condition was
A. Vitamin B12 complicated with sepsis. Despite complex
B. Vitamin B6 therapy of sepsis on the 9th day the patient
C. Ascorbic acid persistently presents with temperature up to
D. Iron preparations 40o C , heart rate is 110/min., respiration rate
E. Vitamin B1 is 23/min., BP is 100/60 mm Hg. Blood test:
leukocytes - 16 · 109 /l, band neutrophils - 16%.
90. After excessive consumption of fatty food What phase of clinical course is it?
a 60-year-old woman suddenly developed
pain in her right subcostal area, nausea, bi- A. Catabolic
le vomiting, sharp bitter taste in her mouth. B. Anabolic
In 2 days she developed jaundice, her uri- C. Rehabilitation
ne darkened. Objectively: sclera and skin are D. Functional
icteric, the abdomen is distended, the liver is E. Tension
enlarged by 3 cm, soft and painful on palpati-
on, Ortner’s, Murphy’s, Kehr’s, Zakharyin’s, 94. A 23-year-old woman presents with di-
Mayo-Robson’s signs are positive. What di- abetes mellitus type 1. She complains of
agnostic technique should be used in the first weakness, headache, nausea, and vomiting.
place? Objectively: temperature is 37,6o C , heart rate
is 98/min., BP is 95/65 mm Hg, respiration rate
A. US of the gallbladder and bile duct is 32/min., loud. Smell of acetone is detected,
B. Fibrogastroduodenoscopy heart sounds are muffled, pulse is rhythmic.
C. X-ray of the abdomen The stomach is sensitive in the epigastrium.
D. Radionuclide scanning of the liver and Costovertebral angle tenderness (Murphy’s
gallbladder punch sign) is present. Blood glucose is 28,5
E. Laparoscopy mmol/l; blood leukocytes - 16, 5 · 109 /l. In uri-
ne: acetone ++, leukocytes - 25-40 in the vision
91. A 42-year-old woman working at a poultry field. Blood pH is 7,1. What correction method
farm complains of dyspnea, thoracic pain on would be the most advisable?
the left, increased body temperature up to 38-
39o C in the evening, and cough. The patient A. Infusion of normal saline + insulinotherapy
suffers from essential hypertension. Objecti- in small portions
vely: vesicular respiration in the lungs, vesi- B. Infusion of 5% glucose + insulinotherapy in
cular resonance without alterations. X-ray large portions
of both lungs reveals numerous small low- C. Infusion of 2,5% sodium bicarbonate +
intensity foci, 2-3 mm in size, located in a row insulinotherapy
along the blood vessels. ESR- 32 mm/hour. D. Intravenous administration of antibiotics +
What is the most likely diagnosis? insulinotherapy
E. Infusion of dextran solutions + insuli-
A. Acute disseminated tuberculosis notherapy
B. Infiltrative tuberculosis
C. Pulmonary carcinomatosis 95. A 57-year-old woman complains of a
D. Focal tuberculosis sensation of esophageal compression, palpi-
E. Community-acquired pneumonia tations, difficult breathing when eating solid
food, occasional vomiting with a full mouth,
92. A 65-year-old patient has been suffering ”wet pillow” sign at night for the last 6
from liver cirrhosis associated with hepati- months. Objectively: body tempearture -
tis C virus for 7 years. During the last 3 39o C , height - 168 cm, weight - 72 kg, Ps-
weeks the patient developed severe edema 76/min, BP- 120/80 mm Hg. X-ray study
of the lower extremities, the abdomen is si- revealed considerable dilation of esophagus
gnificantly distended with fluid. Ultrasound: and its constriction in the cardial part. What
signs of liver cirrhosis, portal hypertensi- pathology is the most likely to have caused
on, lymph nodes are in the area of hepatic dysphagia in this patient?
portal. Concentration of α-fetoprotein in
blood serum is 285 ng/ml. What complicati- A. Achalasia cardiae
on could have developed in this case? B. Primary esophageal spasm
C. Hiatal hernia
D. Esophageal carcinoma
E. Reflux esophagitis
Крок 2 Medicine (англомовний варiант, iноземнi студенти) 2017 рiк 13

96. A 47-year-old man is employed at the neck veins, peripheral pulse is absent, the
weaving workshop, has 15-year-long record carotid artery pulse is rhythmic, 130/min., BP
of service at this factory; his work conditi- is 60/20 mm Hg. Auscultation of the heart
ons are associated with high-frequency and reveals extremely muffled sounds, percussion
high-intensity noise. During periodical exami- reveals heart border extension in both directi-
nation he was diagnosed with occupational ons. What is the optimal treatment tactics for
deafness. What are the grounds for making this patient?
such a diagnosis?
A. Pericardiocentesis and immediate
A. Audiometry data and hygienic assessment thoracotomy
of working environment B. Oxygen inhalation
B. Record of service at this factory C. Puncture of the pleural cavity on the left
C. Noise characteristic at this factory D. Conservative treatment, infusion of
D. Central nervous system examination results adrenomimetics
E. Inner ear examination results E. Pleural cavity drainage
97. A 37-year-old woman complains of acute 101. A patient with chronic pancreatitis
pain in the genital area, swelling of the complains of diarrhea occurring up to 5 ti-
labia, pain when walking. Objectively: body mes per day (no blood traces), loss of body
temperature is 38,7o C , Ps- 98/min. In the weight, abdominal distention, dryness of skin,
interior of the right labia there is a dense, loss of hair, thirst, bleeding gums, convulsi-
painful tumor-like formation 5,0x4,5 cm in si- ons. Complete blood count: leukocytes -
ze, the skin and mucous membrane of genitals 5, 8 · 109 /l; Hb- 86 g/l; ESR- 15 mm/g; Blood
are hyperemic, there is profuse foul-smelling protein test: protein - 48 g/l; albumins - 28
discharge. What is the most likely diagnosis? g/l. What indicators of coprological analysis
would accompany this syndrom?
A. Acute bartholinitis
B. Labial furuncle A. Steatorrhea, creatorrhea
C. Acute vulvovaginitis B. Large amount of mucus, amylorrhea
D. Bartholin gland cyst C. Large amount of starch grains and cellulose
E. Carcinoma of vulva D. Gas bubbles, acid reaction
E. Large numbers of iodinophilous microbes
98. A woman undergoing in-patient treatment
for viral hepatitis type B developed headache, 102. A 35-year-old patient complains of
nausea, recurrent vomiting, memory lapses, heartburn, gasseous and sour eructation,
flapping tremor of her hands, rapid pulse. burning constricting pain behind the sternum
Sweet smell from the mouth is detected. Body and along the esophagus, developing when
temperature is 37,6o C , heart rate is 89/min. bowing the torso to the front. No previ-
What complication developed in the patient? ous examination; the patient takes almagel
at his own discretion, after which he notes
A. Acute liver failure improvement of his general state. Make the
B. Ischemic stroke provisional diagnosis:
C. Gastrointestinal hemorrhage
D. Hypoglycemic shock A. Gastroesophageal reflux disease
E. Meningoencephalitis B. Functional dyspepsia
C. Cardiospasm
99. A 42-year-old man was delivered to a D. Ulcer disease of the stomach
surgical in-patient department with complai- E. Ulcer disease of the duodenum
nts of icteric skin, pain in the right subcostal
area. Biochemical blood analysis: total bili- 103. The left hand of a newborn is extended
rubin - 140 mcmol/l, direct bilirubin - 112 in all its joints, stretched along the torso, and
mcmol/l. On US: choledoch duct - 1,4 cm, pronated in the forearm. Active movements
a concrement is detected in the distal area. of the shoulder joint are retained. The hand
Gallbladder is 40 cm, no concrements. What is flattened, atrophied, cold to touch, hangs
treatment tactics should be chosen? passively. Grasping and Babkin’s reflexes are
absent at the affected side. Hemogram indi-
A. Endoscopic papillosphincterotomy cators are normal. Make the most likely di-
B. Laparoscopic cholecystectomy agnosis:
C. Laparotomy with choledoch duct drain
D. Laparotomy with cholecystectomy A. Inferior distal obstetrical paralysis
E. Threatment in an infectious diseases hospi- B. Osteomyelitis
tal C. Proximal obstetrical paralysis
D. Complete obstetrical paralysis
100. 4 weeks after myocardial infarction a 56- E. Hypoxic-ischemic encephalopathy
year-old patient developed acute heart pain,
marked dyspnea. Objectively: the patient’s 104. A 34-year-old man is being treated for
condition is extremely grave, there is marked schizophrenia exacerbation in a psychiatric
cyanosis of face, swelling and throbbing of unit. Objectively: the patient remains in bed,
Крок 2 Medicine (англомовний варiант, iноземнi студенти) 2017 рiк 14

is sluggishly mobile, unresponsive, does not Objectively: the patient is exhausted, there
react to questions. His position is unvari- are enlarged supraclavicular lymph nodes.
ed, hypomimic, snout reflex and Dupre’s Esophagoscopy revealed no esophageal
symptom are present, muscles exhibit waxy pathology. Which of the following investigati-
flexibility. He has been remaining in this ons is the most appropriate in this case?
state for approximately a week. Feeding is
parenteral. Determine the neuromotor di- A. Computed tomography of chest and medi-
sturbance: astinum
B. X-ray of lungs
A. Catatonic stupor C. Multiplanar imaging of esophagus
B. Depressive stupor D. Radioisotope investigation of chest
C. Psychogenic stupor E. Ultrasound investigation of mediastinum
D. Anergic stupor
E. Exogenic stupor 109. A 15-year-old girl complains of dizzi-
ness and sensation of lack of air that she
105. A 14-year-old girl has been delivered to develops in emotionally straining situations.
a gynecological department with complaints Relief occurs after she takes corvalol. Objecti-
of profuse blood discharge from her genital vely: hyperhidrosis and marble-like pattern
tract for 2 weeks. Anamnesis: menstruation si- of the skin of her palms and feet. Clinical
nce 13, irregular, painful, profuse; the last one and instrumental examination revealed no
was 2 months ago. Objectively: pale skin and organic alterations of the central nervous,
mucosa, BP- 100/60 mm Hg, Hb- 108 g/l. The cardiovascular, and respiratory systems. What
abdomen is soft and painless on palpation. provisional diagnosis can be made?
Rectal examination revealed no pathologies
of reproductive organs. What condition is it? A. Somatoform autonomic dysfunction
B. Obstructive bronchitis
A. Juvenile uterine hemorrhage (Dysfuncti- C. Bronchial asthma
onal) D. Stenosing laryngotracheitis
B. Hypomenstrual syndrome E. Acute epiglottitis
C. Inflammation of uterine appendages (Pelvic
110. A 14-year-old patient complains of
inflammatory disease) alopecia foci on his scalp. The patient has
D. Pelviperitonitis been presenting with this condition for 2
E. Endometritis weeks. Objectively: on the scalp there are
106. An infant cries during urination, the several small oval foci with blurred margins.
foreskin swells and urine is excreted in The skin in the foci is pink-red, the hairs are
drops. What approach to treatment should broken off at 4-5 mm length or at skin level.
be chosen? Under Wood’s lamp there are no foci of green
luminescence detected. What disease is it?
A. Create an opening into the foreskin cavity
B. Prescription of α-adrenergic blocking A. Trichophytosis capitis
agents B. Syphilitic alopecia
C. Prescription of antispasmodic agents C. Alopecia areata
D. Urinary bladder catheterization D. Scleroderma
E. Epicystostomy E. Psoriasis

107. A 20-year-old patient complains of pain 111. A 38-year-old patient has been delivered
in the left lumbar region, arterial pressure rise by an ambulance to a surgical department
up to 160/110 mm Hg. US revealed that the with complaints of general weakness, indi-
structure and size of the right kidney were wi- sposition, black stool. On examination the
thin age norms, there were signs of the 3rd patient is pale, there are dotted hemorrhages
degree hydronephrotic transformation of the on the skin of his torso and extremiti-
left kidney. Doppler examination revealed es. On digital investigation there are black
an additional artery running to the lower feces on the glove. Blood test: Hb- 108 g/l,
pole of the kidney. Excretory urogram shows thrombocytopenia. Anamnesis states that si-
a narrowing in the region of ureteropelvic milar condition was observed 1 year ago.
junction. Specify the treatment tactics: Make the diagnosis:

A. Surgical intervention A. Thrombocytopenic purpura


B. Administration of spasmolytics B. Hemophilia
C. Administration of ACE inhibitors C. Ulcerative bleeding
D. Kidney catheterization D. Rectal tumor
E. Administration of β -blockers E. Nonspecific ulcerative colitis

108. A 49-year-old patient consulted a doctor 112. A 35-year-old patient developed an epi-
about difficult swallowing, voice hoarseness, leptic attack with tonoclonic spasms that
weight loss. These symptoms have been lasted for 3 minutes. After the attack the pati-
gradually progressing for the last 3 months. ent fell asleep but in 5 minutes the second
Крок 2 Medicine (англомовний варiант, iноземнi студенти) 2017 рiк 15

attack occurred. The first step of emergency 22. His current condition has changed acutely:
aid would be to: for 3 days the patient has been refusing
to leave his home. He claims that there is
A. Ensure patency of airways a ”telepathy” occurring between him and
B. Take blood from the vein for analysis the other people, through which he receives
C. Introduce diazepam intravenously ”thoughts of strangers” and transmits his own
D. Prescribe antiepileptic drugs thoughts for everyone to hear. He is convi-
E. Administer chloral hydrate via an enema nced that his thoughts and actions are being
manipulated through this ”telepathy”. Make
113. A 23-year-old woman has been suffering the preliminary diagnosis:
from a mental disease since the age of 18,
the course of disease has no remission peri- A. Paranoid schizophrenia
ods. At a hospital the patient mostly presents B. Depressive episode
with non-purposeful foolish excitation: she C. Manic episode
makes stereotypic grimaces, exposes herself, D. Organic delirium
publicly masturbates with a loud laughter, E. Acute reaction to stress
repeates stereotypical abusive shouts. The
patient should be prescribed: 118. A 35-year-old woman has gained 20 kg
weight within a year with the normal diet.
A. Neuroleptics She complains of chills, sleepiness, shortness
B. Antidepressants of breath. The patient’s mother and sister are
C. Tranquilizers corpulent. Objectively: height - 160 cm, wei-
D. Nootropics ght - 92 kg, BMI- 35,9. Obesity is uniform,
E. Mood stabilizers there are no striae. The face is amimic. The
skin is dry. The tongue is thickened. Heart
114. A young woman suffering from sounds are muffled. HR- 56/min, BP- 140/100
seborrhea oleosa has numerous light-brown mm Hg. The patient has been suffering from
and white spots on the skin of her torso amenorrhea for 5 months, has constipations.
and shoulders. The spots have clear margi- TSH- 28 mcIU/l (norm is 0,32-5). Craniogram
ns, branny desquamation, no itching. What shows no pathology. What is the etiology of
provisional diagnosis can be made? obesity?
A. Pityriasis versicolor A. Hypothyroid
B. Torso dermatophytosis B. Hypo-ovarian
C. Seborrheic dermatitis C. Hypothalamic-pituitary
D. Pityriasis rosea D. Alimentary and constitutive
E. Vitiligo E. Hypercorticoid
115. 10 days after birth a newborn developed 119. A patient complains of painless ”sores”
sudden fever up to 38,1o C . Objectively: the on his penis and inguinal lymph nodes
skin in the region of navel, abdomen and enlargement. Synthomycin emulsion that the
chest is erythematous; there are multiple pea- patient have been applying to the ”sores” was
sized blisters with no infiltration at the base; ineffective. Objectively: on the inner leaf of
isolated bright red moist erosions with epi- the foreskin there are three closely situated
dermal fragments are observed on the peri- rounded erosions, 0,5 cm in diameter, with
phery. What is the provisional diagnosis? dense infiltration that can be palpated at their
A. Epidemic pemphigus of newborn bases. Make the preliminary diagnosis:
B. Syphilitic pemphigus A. Primary syphilis
C. Streptococcal impetigo
D. Vulgar impetigo B. Herpes simplex (Herpes pro genitalis)
E. Atopic dermatitis C. Candidiasis of the inner leaf of the foreskin
D. Shingles
116. A man came to an urologist with E. Erythema multiforme
complains of painful urination, discharge
from urethra. The patient has been suffering 120. A 1,5-month-old child on breasfeedi-
from this condition for a week. Objectively: ng presents from birth with daily vomiting,
hyperemic urinary meatus, edema, purulent irregular liquid foamy feces, and meteori-
discharge. Microscopy of smears detected sm, which are resistant to antibacterial and
gram-negative bacteria. Specify the diagnosis: probiotic therapy; no increase of body mass
is observed. The child’s condition improved,
A. Acute gonorrheal urethritis when breastmilk was substituted with ”NAN
B. Trichomonas urethritis low lactose” formula. What pathology is it?
C. Candidal urethritis
D. Chlamydial urethritis
E. Chronic gonorrhea
117. A patient is 28 years old. He has been
suffering from mental disorder since he was
Крок 2 Medicine (англомовний варiант, iноземнi студенти) 2017 рiк 16

A. Lactase deficiency crackles in the lower segments of the lungs.


B. Intestinal lambliasis (Giardiasis) Heart sounds are weakened, the II heart
C. Infectious enteritis sound is accentuated over the pulmonary
D. Drug-induced enteritis artery. The liver is +3 cm. What complicated
E. Functional dyspepsia the clinical course of COPD in this patient?
121. A 12-year-old girl after a case of respi- A. Chronic pulmonary heart
ratory infection developed dyspnea at rest, B. Pulmonary embolism
paleness of skin. Heart rate is 110/min., BP C. Acute left ventricular failure
is 90/55 mm Hg. Heart sounds are muffled. D. Diffuse pneumosclerosis
Borders of relative heart dullness: right - the E. Community-acquired pneumonia
parasternal line, upper - the III rib, left -
1,0 cm outwards from the midclavicular line. 125. A multigravida at 39 weeks of gestation
Make the provisional diagnosis: presenting with regular labour activity for 8
hours has been delivered to a hospital; the
A. Infectious myocarditis waters broke an hour ago. She complains of
B. Functional cardiopathy headache, seeing spots. BP is 180/100 mm Hg.
C. Somatoform autonomic dysfunction Urine test results: protein - 3,3 g/l, hyaline
D. Hypertrophic cardiomyopathy cylinders. Fetal heart rate is 140/min, rhythmi-
E. Exudative pericarditis cal. Vaginal examination reveals complete
cervical dilatation, the fetal head is on the
122. A 48-year-old man complains of fati- pelvic floor, sagittal suture is in line with
gue, excessive sweating, severe skin itchi- obstetric conjugate, the occipital fontanel is
ng, undulant fever, enlarged cervical and under the pubis. What is the optimal tactics of
supraclavicular lymph nodes. Objectively: labour management?
paleness of skin and mucosa, cervical lymph
nodes are mobile, dense, elastic, walnut-sized, A. Outlet forceps
painless, not attached to the skin. Complete B. Cavity forceps
blood count: erythrocytes - 3, 0 · 1012 /l, Hb- C. Cesarean section
100 g/l, leukocytes - 14 · 109 /l, eosinophils - D. Vacuum extraction of the fetus
6%, basophils - 3%, band neutrophils - 11%, E. Conservative labour management
segmented neutrophils - 69%, lymphocytes -
126. After the contact with chemicals a plant
7, monocytes - 4%, platelets - 280 · 109 /l, ESR- worker has suddenly developed stridor, voi-
37 mm/hour. What method should be applied ce hoarseness, barking cough, progressi-
to verify the diagnosis? ng dyspnea. Objective examination reveals
A. Lymph node biopsy acrocyanosis. What is the provisional di-
B. Sternal puncture agnosis?
C. Muscle biopsy A. Laryngeal edema
D. Chest X-ray B. Laryngeal carcinoma
E. Lumbar puncture C. PATE
123. A patient after a blow to the head D. Pulmonary atelectasis
developed general symptoms of cerebral di- E. Pneumothorax
sturbance, nausea, vomiting, focal signs - 127. An 18-year-old woman complains of pain
hemi-hyperreflexia S>D, hemihyperesthesia in her lower abdomen, profuse purulent di-
on the left, marked meningeal syndrome. Nei- scharge from the vagina, temperature rise up
ther cranial X-ray nor computer tomography to 37,8o C . Anamnesis states that she had a
revealed any pathologies. What examination random sexual contact the day before the si-
method would allow making and clarification gns appeared. She was diagnosed with acute
of the diagnosis? bilateral adnexitis. On additional examinati-
A. Lumbar puncture on: leukocytes are present throughout all visi-
B. Echoencephalography on field, bacteria, diplococci with intracellular
C. Electroencephalography and extracellular position. What is the most
D. Angiography likely agent in the given case?
E. Pneumoencephalography A. Neisseria gonorrhoeae
124. A 72-year-old man complains of lower B. Escherichia coli
extremity edema, sensation of heaviness in C. Chlamydia trachomatis
the right subcostal area, dyspnea of rest. D. Trichomona vaginalis
For over 25 years he has been sufferi- E. Staphylococcus aureus
ng from COPD. Objectively: orthopnea, 128. A 58-year-old man complains of general
jugular venous distention, diffuse cyanosis, weakness, loss of 10 kg of weight within 1,5
acrocyanosis. Barrel chest is observed, months, progressive pain in the lumbar regi-
on percussion there is vesiculotympanitic on, increased blood pressure up to 220/160
(bandbox) resonance, sharply weakened vesi- mm Hg, low grade fever. Objectively: in the
cular respiration on both sides, moist crepitant
Крок 2 Medicine (англомовний варiант, iноземнi студенти) 2017 рiк 17

right hypochondrium deep palpation reveals a A. Cause of death


formation with uneven surface and low mobi- B. Manner of death
lity; veins of the spermatic cord and scrotum C. Time of death
are dilated. Blood test results: Hb- 86 g/l, D. Mode of death
ESR- 44 mm/h. Urine test results: specific E. Mechanism of death
gravity - 1020, protein - 0,99 g/l, RBC cover
the whole field of vision, WBC- 4-6 in the field 132. It is the 3rd day after the normal term
of vision. What is the provisional diagnosis? labor; the infant is rooming-in with the
mother and is on breastfeeding. Objectively:
A. Renal tumour the mother’s general condition is satisfactory.
B. Urolithiasis Temperature is 36,4o C , heart rate is 80/min.,
C. Acute pyelonephritis BP is 120/80 mm Hg. Mammary glands
D. Acute glomerulonephritis are soft and painless; lactation is moderate,
E. Nephroptosis unrestricted milk flow. The uterus is dense,
the uterine fundus is located by 3 fingers width
129. Anamnesis of a 30-year-old patient below the navel. Lochia are sanguino-serous,
includes closed thoracic injury. Lately the moderate in volume. Assess the dynamics of
patient has been suffering from increasing uterine involution:
dyspnea, sensation of heaviness in the right
subcostal area, and heart rate disturbances. A. Physiological involution
Objectively: acrocyanosis, bulging cervical B. Subinvolution
veins, ascites, edema of the lower extremiti- C. Lochiometra
es. Heart auscultation reveals muffled heart D. Pathologic involution
sounds, additional III heart sound is detected. E. Hematometra
Provisional diagnosis of constrictive peri-
carditis was made. What diagnostic technique 133. Survey radiograph of a 52-year-old
would NOT confirm the diagnosis? worker of an agglomeration plant (28-year-
long record of service, the concentration
A. US of abdomen of metal dust is 22-37 mg/m3 ) shows mi-
B. Computer tomography ldly pronounced interstitial fibrosis with di-
C. Echocardiography ffused contrasting well-defined small nodular
D. Magnetic resonance imaging shadows. The patient has no complaints.
E. Chest X-ray Pulmonary function is not compromised.
What is the provisional diagnosis?
130. A 72-year-old woman suffers from di-
abetes mellitus type 2, concomitant diseases A. Siderosis
are stage 2 hypertension and stage 2B heart B. Silicosis
failure. She takes metformin. Hypertensic cri- C. Anthraco-silicatosis
sis had occurred the day before, after whi- D. Silicatosis
ch the patient developed extreme weakness, E. Anthracosis
myalgias, thirst, dry mouth, polyuria. BP
is 140/95 mm Hg, heart rate is 98/min., 134. A 53-year-old woman complains of wei-
no edemas or smell of acetone detected. ght loss up to 10 kg within the last 2 years,
What measures should be taken to prevent liquid foul-smelling stool two times a day that
development of comatose state in the patient? poorly washes off the toilet, periodic bouts
of nausea, girdle pain in the upper abdomen.
A. Stop metformin, prescribe short-acting Objectively: pain in Gubergrits zone (on the
insulin right from navel) and at Mayo-Robson’s poi-
B. Double the dosage of metformin nt. Biochemical blood analysis: glucose - 3,2
C. Apply hypotonic solution of sodium chlori- mmol/l, bilirubin - 16,5 mcmol/l, crude protein
de - 56,4 g/l. Urine diastase/amylase - 426 g/h/l.
D. Additionally prescribe long-acting insulin D-xylose test (oral administration of 25 g of
E. Prescribe glibenclamide d-xylose) after 5 hours reveals 3 g of xylose in
131. The body of a 24-year-old woman with urine. The most likely diagnosis is:
probable signs of poisoning has been found on A. Pancreatitis. Malabsorption syndrome
the street. Forensic medical examination was B. Pseudomembranous colitis
requested by an investigator during exami- C. Nonspecific ulcerative colitis
nation of the site and the body. According D. Irritable bowel syndrome
to the Criminal Procedure Code currently in E. Chronic gastritis
force in Ukraine, forensic medical examinati-
on is required when it is necessary to determi- 135. A 15-year-old teenager has undergone
ne the: medical examination in military recruitment
center. The following was revealed: interval
systolic murmur at the cardiac apex, accent
of the II heart sound over the pulmonary
artery, tachycardia. What additional exami-
nation method will be the most informative
Крок 2 Medicine (англомовний варiант, iноземнi студенти) 2017 рiк 18

for determining diagnosis? A. Reactive arthritis


B. Rheumatoid arthritis
A. Echocardiography C. Seasonal pollinosis
B. Electrocardiography D. Bacterial nonspecific urethral conjunctivitis
C. X-ray E. Upper respiratory tract infection (URTI)
D. Phonocardiography that affects conjunctiva and joints
E. Rheography
140. A 19-year-old woman complains of
136. A 64-year-old patient has been hospitali- severe pain in the axillary crease. Conditi-
zed with complaints of progressive jaundice on onset occurred a week ago after her swi-
that developed over 3 weeks without pain mming in a cold river and epilation. The next
syndrome and is accompanied by general day a painful ”boil” appeared. The ”boil” was
weakness and loss of appetite. Objectively: increasing in size every day and became a
temperature is 36,8o C , heart rate is 78/min, plum-sized tumor. Upon examination there
abdomen is soft and painless, peritoneum irri- are nodular conical growths joined together
tation symptoms are not detected, palpati- detected, the skin covering them is bluish-red
on reveals sharply enlarged tense gallbladder. in color. Some nodules have fistulous openi-
What disease can be characterised by these ngs producing thick purulent mass. Body
symptoms? temperature is 38, 5o C , general malaise. What
is the most likely diagnosis?
A. Cancer of pancreas head
B. Duodenal ulcer A. Hydradenitis
C. Acute cholecystitis B. Carbuncle
D. Chronic cholecystitis C. Cutaneous tuberculosis
E. Cholecystitis caused by lambliasis D. Necrotizing ulcerative trichophytosis
E. Pyoderma chancriformis
137. A 6-year-old girl came to a general practi-
tioner with her mother. The child complains 141. A woman complains of weight gain,
of burning pain and itching in her external chills, edema, xeroderma, somnolence, diffi-
genitalia. The girl was taking antibiotics the culties with focusing. Objectively: height is
day before due to her suffering from acute 165 cm; weight is 90 kg; body proportions
bronchitis. On examination: external genitalia are of female type, to - 35,8o C , heart rate -
are swollen, hyperemic, there is white deposit 58/min, BP- 105/60 mm Hg. Heart sounds
accumulated in the folds. The most likely di- are weakened, bradycardia is observed. Other
agnosis is: internal organs have no alterations. Thyroid
gland cannot be palpated. Milk secretion
A. Candidal vulvovaginitis from mammary glands is observed. Hormone
B. Trichomoniasis test revealed increased levels of thyroid-
C. Nonspecific vulvitis stimulating hormone (TSH) and prolactin,
D. Helminthic invasion and decreased level of thyroxine (4 ). What
E. Herpes vulvitis is the cause of obesity?
138. A 37-year-old patient complains of pain A. Primary hypothyroidism
in the spinal column, reduced mobility. The B. Secondary hypothyroidism
condition persists for 7 years. ”Sway back” is C. Prolactinoma
observed, there is no movement in all spi- D. Hypopituitarism
nal regions. On X-ray: ”bamboo spine” is E. Adiposogenital dystrophy
detected. What is the most likely diagnosis?
142. ECG revealed the following in a 10-
A. Ankylosing spondylitis year-old child: sharp acceleration of the heart
B. Osteochondrosis rate - 240/min., P wave overlaps with T wave
C. Spondylitis deformans and deforms it, moderate lengthening of PQ
D. Tuberculous spondylitis interval, QRS complex is without alterations.
E. Spondylolisthesis What pathology does this child have?
139. A 25-year-old patient is not married and A. Paroxysmal atrial tachycardia
has sexual relations with several partners. B. Atrial hypertrophy
During the last 3 months he noticed a small C. Ventricular hypertrophy
amount of mucoserous discharge from the D. WPW syndrome
urethra. Subjectively: periodical itching or E. Extrasystole
burning pain in the urethra. Two months ago
pain in the knee joint developed. Possibility 143. A 54-year-old patient complains of
of trauma or exposure to cold is denied by the weakness, jaundice, itching skin. Disease
patient. During the last week eye discomfort onset was 1,5 months ago: fever up to 39o C
is noted - lacrimation and itching. What provi- appeared at first, with progressive jaundice
sional diagnosis can be made? developed 2 weeks later. On hospitalizati-
on jaundice was severely progressed. Liver
cannot be palpated. Gallbladder is enlarged
Крок 2 Medicine (англомовний варiант, iноземнi студенти) 2017 рiк 19

and painless. Blood bilirubin is 190 mcmol/l analyses periodically revealed leukocyturia.
(accounting for direct bilirubin). Stool is The child has undergone no further treatment.
acholic. What is the most likely jaundice On examination: increased BP up to 150/100
genesis in this patient? mm Hg. Ultrasound investigation revealed si-
gnificant reduction of the right kidney. What
A. Mechanical jaundice process is leading in arterial hypertension
B. Hepatocellular jaundice pathogenesis in this case?
C. Hemolytic jaundice
D. Caroli syndrome A. Hyperactivity of renin-angiotensin system
E. Gilbert’s syndrome B. Disturbance of water-electrolytic balance
C. Disturbance of renal circulation
144. During hemotransfusion the patient D. Hypersympathicotonia
developed nausea, tremor, lumbar and E. Increased cortisol level
retrosternal pain. On examination the skin
is hyperemic, later developed pallor; the pati- 148. A 3-month-old child presents with
ent presents with hyperhidrosis, labored respi- saffron-yellow coloring of the skin, sclera,
ration, pulse is 110/min., BP is 70/40 mm Hg. and mucous membranes. The abdomen is
Urine is black colored. What complication enlarged, hepatomegaly and splenomegaly
developed in the patient? are observed. In blood there is conjugated
bilirubin-induced hyperbilirubinemia. On
A. Posttransfusion shock intravenous cholangiocholecystography:
B. Acute renal failure opacified bile is discharged into the intesti-
C. Pulmonary embolism ne. Transaminase activity is normal. What is
D. Anaphylactic shock the most likely diagnosis?
E. Hypotonic crisis
A. Biliary atresia
145. A 22-year-old man suddenly developed B. Physiologic jaundice
extreme weakness, nausea, vomiting with C. Hemolytic disease of newborn
traces of blood. The patient is known to D. Crigler-Najjar syndrome
suffer from peptiv ulcer disease of duodenum E. Congenital hepatitis
and hemophilia A. Objectively: heart rate -
102/min., BP- 100/60 mm Hg. Complete blood 149. A 51-year-old woman complains of
count: erythrocytes - 3, 2 · 1012 /l, Hb- 98 g/l, headache, trembling, paresthesiae, palpi-
color index - 0,92, leukocytes - 7, 4 · 109 /l, tations, increased blood pressure up to
280/160 mm Hg. The day before she experi-
platelets - 240 · 109 /l, ESR- 11 mm/hour. What enced exhausting headache, vascular pulsati-
measure would most effectively decrease on, palpitations, asphyxia, stomachache,
hemorrhaging in this case? unbearable fear of coming death. The pati-
A. Cryoprecipitate ent paled and broke out in cold sweat. In
B. Aminocapronic acid urine there is increased content of vani-
C. Native plasma llylmandelic acid. What disease causes such
D. Direct transfusion of donor blood clinical presentation in the patient?
E. Platelet concentrate transfusion A. Pheochromocytoma
146. A 22-year-old woman complains of B. Conn’s syndrome (primary
amenorrhea for 8 months. Anamnesis states hyperaldosteronism)
that menarche occured at the age of 12,5. Si- C. Cushing’s syndrome
nce the age of 18 the patient has a history of D. Primary hypertension
irregular menstruation. The patient is nulli- E. Cushing’s disease
gravida. The mammary glands are developed
properly, nipples discharge drops of milk 150. A 52-year-old patient suffers from
when pressed. Hormone test: prolactin level marked dyspnea during physical exertion,
is 2 times higher than normal. CT reveals a non-productive cough. The patient’s condi-
bulky formation with diameter of 4 mm in tion has been persisting for 8 months. The
the region of sella. What is the most likely di- patient has been a smoker for 30 years. In
agnosis? the lungs there are cellophane-type crackles
auscultated on both sides. Respiration rate is
A. Pituitary tumour 26/min., oxygen saturation of blood is 92%.
B. Lactational amenorrhea On spirometry: moderate restrictive-type di-
C. Stein-Leventhal syndrome (polycystic ovary sturbance of external respiration. What is the
syndrome) most likely diagnosis?
D. Sheehan’s syndrome (postpartum hypopi-
tuitarism)
E. Cushing’s disease
147. A 13-year-old girl complains of fatigabili-
ty, frequent headaches, cardialgia. Eight years
ago she had a case of pyelonephritis. Urine
Крок 2 Medicine (англомовний варiант, iноземнi студенти) 2017 рiк 20

A. Idiopathic fibrosing alveolitis A. Lymphogranulomatosis (Hodgkin’s


B. Chronic obstructive pulmonary disease lymphoma)
(COPD) B. Lymph node tuberculosis
C. Chronic bronchitis C. Lymphoreticulosarcoma
D. Community-acquired pneumonia D. Cancer metastases to the lymph nodes
E. Sarcoidosis E. Macofollicular reticulosis
151. A 26-year-old patient with affective bi- 155. An 9-year-old child was hospitalized
polar disorder has developed a condition for fever up to 39,8o C , inertness, moderate
manifested by mood improvement, behavi- headache, vomiting. Examination revealed
oural and sexual hyperactivity, verbosity, acti- meningeal symptoms. Lumbar puncture was
ve body language, reduced need for sleep. performed. The obtained fluid was characteri-
Which of the following drugs would be most sed by increased opening pressure, was
effective in this case? transparent, with the cell count of 450 cells per
1 mcL (mainly lymphocytes - 90%), glucose
A. Neuroleptics with sedative effect level of 3,6 mmol/l. What agent could have
B. Antidepressants with activating effect caused the disease in the child?
C. Neuroleptics with activating effect
D. Tranquilizers A. Enterovirus
E. Antidepressants with sedative effect B. Neisseria meningitidis
C. Mycobacterium tuberculosis
152. A 19-year-old patient complains of D. Staphylococcus aureus
dyspnea on exertion. He often has bronchitis E. Streptococcus pneumoniae
and pneumonia. Since childhood the patient
presents with cardiac murmur. Auscultation 156. A 25-year-old woman has a self-detected
revealed splitting of the II sound above the tumor in the upper outer quadrant of her ri-
pulmonary artery, systolic murmur in the 3rd ght breast. On palpation there is a painless
intercostal space at the left sternal border. firm mobile lump up to 2 cm in diameter, peri-
ECG detected right bundle branch block. pheral lymph nodes are without alterations. In
What is the provisional diagnosis? the upper outer quadrant of the right breast
ultrasound revealed a massive neoplasm with
A. Atrial septal defect increased echogenicity sized 21x18 mm. What
B. Open ductus arteriosus is the most likely diagnosis?
C. Aortarctia
D. Aortic stenosis A. Fibroadenoma
E. Mitral insufficiency B. Lactocele
C. Diffuse mastopathy
153. A 30-year-old patient was in a car acci- D. Mammary cancer
dent. He is unconscious, pale, has thready E. Mastitis
pulse. In the middle third of the right thigh
there is an extensive laceration with ongoi- 157. A 68-year-old man complains of inabili-
ng profuse external arterial bleeding. What ty to urinate for a day. On attempt of urinary
urgent actions must be taken to save the life bladder catheterization there was detected a
of the patient? rough stricture in the membranous portion of
the urethra. What first aid tactics should be
A. Tourniquet above the wound of the right applied in this case?
thigh
B. Tourniquet below the wound of the right A. Troacar cystostomy
thigh B. Adenomectomy
C. Artificial lung ventilation C. Optical internal urethrotomy
D. Precordial thump D. α-adrenoblockers
E. Application of plaster bar E. Urinary bladder catheterization
154. A 25-year-old patient has been admi- 158. Heart X-ray of a 31-year-old man has
tted to the hospital with the following revealed the following: with tightly filled
problems: weakness, sweating, itching, wei- opacified esophagus there is a marginal filli-
ght loss, enlarged submandibular, cervical, ng defect in its middle third on the posterior
axillary, inguinal lymph nodes. Objectively: wall; the defect is 1,8x1,3 cm in size with clear
hepatomegaly. Lymph node biopsy revealed oval border. Mucosal folds are retained and
giant Berezovsky-Reed-Sternberg cells, envelop the defect; wall peristalsis and elasti-
polymorphocellular granuloma composed of city are not affected. There are no complai-
lymphocytes, reticular cells, neutrophils, eosi- nts regarding the condition of the patient’s
nophils, fibrous tissue, and plasma cells. What alimentary canal. Make the provisional di-
is the most likely diagnosis? agnosis:
Крок 2 Medicine (англомовний варiант, iноземнi студенти) 2017 рiк 21

A. Esophageal tumor general fatigue, low-grade fever persisting


B. Achalasia cardiae for 4 months, lumbar pain, and dysuria.
C. Esophageal burns Anamnesis includes frequent acute respi-
D. Diverticulum ratory diseases, overexposure to cold, low-
E. Barrett esophagus calorie diet, a case of pulmonary tuberculosis
in childhood. Clinical urine analysis: pH-
159. A patient with signs of general 4,8, leukocyturia, hematuria. Complete blood
overexposure to cold presenting with local count: leukocytosis, lymphocytosis, increased
frostbites of fingers has been delivered into ESR. Urography concludes: dilatation of
an admission room. Objectively: conscious, renal pelvis and calyceal system of both ki-
inert, speech is slow, the skin of the face is dneys, foci of calcification in the projection of
cold, body temperature is 34o C , heart rate is right kidney parenchyma. What is the most
68/min. What would be the actions of a doctor likely diagnosis?
on call?
A. Nephrotuberculosis
A. Hospitalize the patient to the surgical B. Right renal cyst
department C. Right renal carcinoma
B. Hospitalize the patient to the therapeutics D. Acute glomerulonephritis
department E. Chronic pyelonephritis
C. Hospitalize the patient to the traumatology
department 164. A full-term newborn (born with the body
D. Let the patient go home weight of 3900 g at gestational age of 39
E. Refer to a family doctor on the next day weeks) on the first day of his life developed
160. A 22-day-old infant developed respiratory disturbances: dyspnea, arrhythmic
subcutaneous red nodes from 1,0 to 1,5 cm in respiration, cyanosis attacks. On examination
size on the scalp; later the nodes suppurated. there is paradoxical respiration observed and
Temperature increased up to 37,7o C , intoxi- left side of the chest lags behind in the act of
cation symptoms appeared, regional lymph breathing. On auscultation the respiration is
nodes enlarged. Complete blood count: weakened in the lungs on the left. Neurologi-
anemia, leukocytosis, neutrocytosis, increased st diagnosed the patient with left-sided Erb-
ESR. What diagnosis will you make? Duchenne palsy. Complete blood count shows
no changes. What is the most likely diagnosis?
A. Pseudofurunculosis
B. Pemphigus A. Left-sided diaphragm paresis
C. Vesiculopustulosis B. Congenital pneumonia
D. Scalp phlegmon C. Left-sided pneumothorax
E. - D. Respiratory distress syndrome
E. Transient tachypnea of the newborn
161. During examination of a healthy infant,
the child takes a toy into his hands, turns 165. A 12-year-old child had three attacks of
from the back to the side; when lying on the acute rheumatic fever accompanied by cardi-
stomach he can firmly prop himself up on his tis. Examination revealed the symptoms of
forearms; the child laughs and makes joyful chronic tonsillitis, mitral insufficiency, and
exclamations. The age of the child is: carious teeth. What is the optimal method
of secondary prophylaxis?
A. 4 months
B. 2 months A. Year-round bicillin prophylaxis until the age
C. 5 months of 25
D. 1 month B. Course of cardiotrophic drugs twice a year
E. 6 months C. Year-round bicillin prophylaxis for 3 years
D. Tonsillectomy
162. After significant physical exertion a 66- E. Oral cavity sanitation
year-old man with deep vein thrombosis of
the extremities developed shortness of breath, 166. A 24-year-old patient had been delivered
intense pain in the chest on the left, marked to a thoracic department with a chest injury,
palpitations. The patient’s condition is grave, fracture of the IV, V, VI ribs on the right.
his face is cyanotic, the cervical veins are Plan radiography showed the fluid level in
swollen, BP is 60/40 mm Hg. What investi- the pleural cavity to be reaching the III rib
gation method would be the most advisable in on the right. Puncture contained blood clots.
this case? What is the optimal treatment tactics?

A. Selective angiopneumography A. Emergency thoracotomy


B. Chest X-ray B. Pleural puncture
C. Echocardiography C. Thoracentesis and thoracostomy
D. Magnetic resonance imaging of the chest D. Hemostatic therapy
E. Fiber-optic bronchoscopy E. Medical thoracoscopy

163. A 32-year-old woman complains of 167. A 3-year-old child has been delivered to
Крок 2 Medicine (англомовний варiант, iноземнi студенти) 2017 рiк 22

a hospital with complaints of pain in the legs, 171. Examination of a Rh-negative pregnant
fever, loss of appetite. Objectively: pale skin woman at 32 weeks of gestation revealed
and mucosa, hemorrhagic rash. Lymph nodes a four-time rise of Rh-antibody titer within
are enlarged, painless, dense and elastic, not 2 last weeks; the titer is 1:64. The first two
matted together. Bones, joints, and abdomen pregnancies resulted in antenatal fetal death
are painful. The liver and spleen are enlarged. due to hemolytic disease. What is the optimal
Hemogram: Hb- 88 g/l, color index - 1,3, tactics of pregnancy management?
platelets - 80 · 109 /l, leukocytes - 25, 8 · 109 /l,
lymphoblasts - 70%, ESR- 52 mm/hour. Make A. Early delivery
the provisional diagnosis: B. Delivery at 37 weeks of gestation
C. Screening for Rh-antibodies in 2 weeks and
A. Acute leukemia urgent delivery in case of further increase of
B. Thrombocytopenic purpura antibody titer
C. Acute rheumatic fever D. Introduction of anti-Rh (D)
D. Infectious mononucleosis immunoglobulin
E. Hemorrhagic vasculitis (Henoch-Schonlein E. Ultrasound for signs of hemolytic disease of
purpura) the fetus
168. A 60-year-old man has a diet consisting of 172. During last several weeks an 11-year-
unvaried food staples: mostly cereals, potato, old girl has been complaining of dyspnea and
pasta; few vegetables and little fats (especially edema of shins and feet after physical exerci-
animal fats). During medical examination he se. After a long rest or sleep through the night
complains of deterioration of his twilight visi- her edemas diminish significantly. On clini-
on. This condition can be caused by lack of: cal examination there are enlarged liver and
rasping systolic murmur over the cardiac area.
A. Retinol Blood and urine analyses are without changes.
B. Amino acids What is the most likely cause of the child’s
C. Fats edema?
D. Calcium
E. Carbohydrates A. Heart failure
B. Angioneurotic edema
169. A 14-year-old girl came to a general C. Acute pyelonephritis
practitioner with complaints of weakness, D. Hepatocirrhosis
loss of appetite, headache, rapid fatigabili- E. Nephrotic syndrome
ty. Her last menstruation was profuse and
lasted for 14 days after previous delay of 2 173. A 74-year-old man visited an urologist
months. Objectively: the skin is pale, heart with complaints of pain above the pubis and
rate is 90/min., BP is 110/70 mm Hg, Hb is inability to urinate for 8 hours. At home he
88 g/l. Rectal examination: the uterus and its had taken antispasmodics and had a warm
appendages are without changes, no discharge bath but no improvement occurred. Objecti-
from the genital tracts. What complication vely: the abdomen is soft and painful above
occurred in the patient? the pubis; dullness of percussion sound is
observed above the pubis. Murphy’s punch si-
A. Posthemorrhagic anemia gn (costovertebral angle tenderness) is negati-
B. Somatoform autonomic dysfunction of ve on both sides. What condition does the
hypotonic type patient have?
C. Migraine
D. Gastritis A. Acute urinary retention
E. Dysmenorrhea B. Paradoxal ischuria
C. Chronic urinary retention
170. A postpartum woman on the 12th day D. Anuria
after the normal delivery complains of pain E. Oliguria
localized in her left gastrocnemius muscle.
Body temperature is 37,2o C ; pulse is 85/min, 174. During preventive examination a 58-year-
rhythmic; blood pressure is 128/80 mm Hg. old man on chest X-ray presents with multi-
Mammary glands are soft and painless. The ple globular pale shadows 3 cm in diameter
uterus is behind the pubis. The left leg in the within parenchyma of the both lungs. Exami-
area of gastrocnemius muscle is by 3 cm larger nation in the oncologic hospital: the primary
than the right leg in the diameter. Internal focus is not found; transbronchial biopsy wi-
organs present no pathologies. What compli- th cytologic investigation detected cells of
cation can be suspected? glandular neoplasm. What tactics should the
physician choose?
A. Deep vein thrombosis of the shin
B. Iliofemoral thrombosis
C. Varicose veins of lower extremities
D. Endometritis
E. Myositis
Крок 2 Medicine (англомовний варiант, iноземнi студенти) 2017 рiк 23

A. Polychemotherapy courses on on the right, which developed late in


B. Exploratory laparotomy the night, and impaired walking. Objecti-
C. Exploratory thoracotomy vely: the metatarsophalangeal articulation
D. Laparoscopy is swollen, hyperemic, hot to touch, pai-
E. Symptomatic treatment at home nful on movement. In blood: erythrocytes -
175. A 36-year-old man developed a disease 5, 1·1012 /l, Нb- 155 g/l, leukocytes- 13, 0·109 /l,
with acute onset 6 hours ago. The patient ESR- 50 mm/hour, CRP- 46 mg/dl, uric
presents with pain in the epigastric, ileocecal, acid - 720 mcmol/l. X-ray of feet articulati-
and paraumbilical areas, vomiting, weakness, ons: osteoporosis, narrowing of interarti-
nausea, and body temperature of 38,5o C . cular spaces, numerous punched-out erosions.
Stool is liquid, profuse, frequent, retains fecal Make the preliminary diagnosis:
nature, foul-smelling, frothy, colored dark A. Gout
green. The stomach is moderately distended B. Osteoarthritis
and painful on palpation. The patient attri- C. Reactive arthritis
butes his disease to eating raw chicken eggs D. Rheumatoid arthritis
one day before the clinical signs of the disease E. Psoriatic arthritis
appeared. What is the most likely diagnosis?
179. A woman in her early- to mid-thirties
A. Salmonellosis has lost her consciousness 3-5 minutes ago.
B. Shigellosis On examination: the skin is pale, no pulse
C. Typhoid fever over the carotid arteries, no spontaneous
D. Cholera respiration, pupils are dilated; the patient is
E. Food toxicoinfection nonresponsive, presents with atony. The pati-
176. A 55-year-old woman came to a ent’s condition can be determined as:
gynecologist with complaints of leukorrhea A. Apparent death
and bloody discharge from the vagina after B. Natural death
5 years of menopause. Anamnesis states C. Syncope
no pregnancies. Bimanual examination: the D. Brain death
uterus and uterine appendages are without E. Coma
changes. During diagnostic curettage of the
uterine cavity the physician scraped off 180. A 32-year-old woman complains of body
enchephaloid matter. What is the most likely weight loss despite her increased appetite,
diagnosis in this case? nervousness, and tremor of the extremities.
Objectively: the skin is moist; the thyroid
A. Endometrial carcinoma gland is diffusely enlarged, painless, soft, and
B. Adenomyosis mobile. Blood test: increased level of T3,
C. Subserous uterine myoma T4, and thyroid-stimulating hormone (THS).
D. Cervical carcinoma What is the most likely diagnosis?
E. Ovarian carcinoma
A. Diffuse toxic goiter
177. A 10-year-old boy is delivered into a B. Thyroid carcinoma
polytrauma unit after he received a blunt C. Autoimmune (Hashimoto’s) thyroiditis
trauma of the thorax, having fallen from D. Thyroid adenoma
the bicycle. Upon hospitalization his blood E. Diffuse nontoxic goiter
pressure is 110/80 mm Hg, heart rate is 96/min.
Chest X-ray is noncontributive to the di- 181. A 57-year-old patient complains of
agnosis. Echocardiogram shows free liquid sensation of dryness and pain during
in the pericardial cavity, in the amount of up swallowing, frequent unbearable cough, the
to 100 ml. In an hour after the hospitalization voice is hoarse. Disease onset was abrupt. On
the patient started to develop increasing si- laryngoscopy: laryngeal mucosa is hyperemic,
gns of heart failure: jugular venous distention, vocal folds are swollen, laryngeal lumen
decreased blood pressure down to 90/70 mm contains viscous secretion. What diagnosis is
Hg, tachycardia up to 120/min. On auscultati- it?
on muffled heart sounds. What would be the
primary tactics of a physician? A. Acute laryngitis
B. Acute stenosing laryngotracheitis
A. Pericardiocentesis C. Bronchial asthma
B. Cardiac glycosides intravenously D. Flegmonous laryngitis
C. Constant oxygenotherapy E. Laryngeal diphtheria
D. Diuretics intravenously
E. Antibiotics intravenously 182. A 24-year-old pregnant woman on her
37th week of pregnancy has been delivered to
178. After the celebratory feast that took a maternity obstetric service with complaints
place the day before, a 35-year-old man was of weak fetal movements. Fetal heartbeats are
hospitalized with complaints of marked pain 95/min. On vaginal examination the uterine
within the I metatarsophalangeal articulati- cervix is tilted backwards, 2 cm long, external
Крок 2 Medicine (англомовний варiант, iноземнi студенти) 2017 рiк 24

orifice allows inserting a fingertip. Biophysi- A. Total protein content in the pleural fluid
cal profile of the fetus equals 4 points. What below 25 g/l
tactics of pregnancy management should be B. Presence of atypical cells
chosen? C. Total protein content in the pleural fluid
above 30 g/l
A. Urgent delivery via a cesarean section D. Specific gravity exceeding 1015
B. Treatment of placental dysfunction and E. Positive Rivalta’s test
repeated analysis of the fetal biophysical
profile on the next day 186. A 28-year-old woman complains of gi-
C. Doppler measurement of blood velocity in rdle pain in her epigastric and left subcostal
the umbilical artery areas with irradiation to the back, nausea,
D. Urgent preparation of the uterine cervix for and vomiting without relief. On examinati-
delivery on a surgeon observes stomach distension
E. Treatment of fetal distress, if ineffective, and meteorism. There are positive Mondor’s,
then elective cesarean section on the next day Mayo-Robson’s, and Cullen’s symptoms.
What is the most likely diagnosis?
183. During regular preventive gynecologi-
cal examination a 30-year-old woman was A. Acute pancreatitis
detected to have dark blue punctulated B. Acute cholecystitis
”perforations” on the vaginal portion of C. Acute intestinal obstruction
the uterine cervix. The doctor suspects D. Aortic dissecting aneurysm
endometriosis of the vaginal portion of the E. Splenic infarction
uterine cervix. What investigation method
would be most informative for diagnosis 187. A 36-year-old woman complains of
confirmation? pain in her lumbar area, which irradiates to
her lower right limb and increases during
A. Colposcopy, target biopsy of the cervix movements, and sensation of numbness in
B. US of small pelvis her limb. Objectively: palpation of the shin
C. Hysteroscopy and thigh muscles is painful, positive stretch
D. Curettage of the uterine cavity symptom on the right. MRI scan: herniati-
E. Hormone testing on of intervertebral disk L5-S1 4 mm in size.
What is the most likely diagnosis?
184. A 26-year-old woman came to a
gynecologist for a regular check-up. She has A. Vertebrogenic lumbago
no complaints. Per vaginum: the uterus lies in B. Vertebrogenic radicular syndrome of L5-S1
anteflexion, not enlarged, dense, mobile, pai- on the right
nless. On the left from the uterus in the area of C. Endarteritis of lower extremities
uterine appendages there is a mobile painless D. Spinal stroke
outgrowth that can be moved independently E. Acute myelitis
from the uterus. On the right the appendages
cannot be detected. What additional investi- 188. During routine medical examinati-
gation would be informative for diagnosis on a 35-year-old woman presents with
clarification? enlarged cervical and mediastinal lymph
nodes. Her overall health is satisfactory.
A. US of lesser pelvis ESR is 30 mm/hour. Cervical node biopsy
B. Metrosalpingography was performed. In the specimen there are
C. Examination for urogenital infection granulomas composed of epithelial and gi-
D. Colposcopy ant cells, no caseous necrosis detected. What
E. Colonoscopy is the most likely diagnosis?

185. A 57-year-old patient complains of A. Sarcoidosis


dyspnea at rest. The patient presents with B. Lymphogranulomatosis
orthopnea, acrocyanosis, bulging cervical vei- C. Infectious mononucleosis
ns. On percussion: dull sound over the lower D. Nonspecific lymphadenitis
lung segments; on auscultation: no respiratory E. Lymph node tuberculosis
murmurs. Heart rate is 92/min. Right-sided
cardiac dilatation is observed. The liver is 189. A 39-year-old woman complains of vi-
enlarged by 7 cm. Shins are swollen. Pleural olent pain in her left lumbar area with irradi-
effusion is suspected. What indicator would ation to the right iliac area. Several years
confirm the presence of transudate in this ago she was diagnosed with cholelithiasis
case? and urolithiasis. The patient’s condition is
moderately severe, the skin is dry. Ortner’s
symptom is negative; costovertebral angle
tenderness is observed on the right. The most
reasonable treatment tactics would be:
Крок 2 Medicine (англомовний варiант, iноземнi студенти) 2017 рiк 25

A. Spasmolytics and analgesics 194. A 32-year-old woman complains of epi-


B. Urgent hemodialysis sodes of intense fear that occur without visible
C. Peritoneal dialysis cause and last for 10-20 minutes; the episodes
D. Laparoscopic cholecystectomy are characterized by rapid pulse, sweating,
E. Antibiotics labored breathing, and vertigo. Specify the li-
kely diagnosis:
190. An 18-year-old patient complains of skin
rash. The patient has been suffering from this A. Panic disorder
condition for 5 years. The first instance of this B. Paranoid syndrome
disease occurred after a car accident. Objecti- C. Manic syndrome
vely: the patient presents with papular rash D. Simple schizophrenia
covered in silvery scales, ”thimble” symptom E. Claustrophobia
(small pits on the nails), affected joints. What
is the most likely diagnosis? 195. A resuscitation unit received a 46-year-
old woman, who has been suffering from
A. Psoriasis diabetes mellitus type 1 for approximately
B. Panaritium 30 years. Objectively: the skin is pale, heart
C. Onychomycosis sounds are weakened, BP is 170/100 mm
D. Lupus erythematosus Hg, lower limbs are markedly swollen. Blood
E. Rheumatism creatinine - 1125 mcmol/l, urea - 49,6 mmol/l,
potassium - 6.3 mmol/l, glucose - 7,6 mmol/l,
191. A woman complains of frequent, li- glomerular filtration rate - 5 ml/min. What
quid stool (up to 9-10 times per day) with treatment is indicated for the patient in the
mucus and blood admixtures, dull pain in the first place?
hypogastrium, weight loss of 4 kg within the
last year. Objectively: malnutrition, dry skin, A. Hemodialysis
low turgor, aphthous stomatitis. The stomach B. Kidney transplantation
is soft, the sigmoid colon is spastic and painful C. Hemofiltration
on palpation. Occult blood test is positive. Fi- D. Enterosorption
brocolonoscopy: edema, hyperemia, mucosal E. Conservative detoxification therapy
granulation, pseudopolyps, small ulcers with
irregular edges. Make the diagnosis: 196. A woman is on the 32nd week of her
second pregnancy. She complains of fever,
A. Nonspecific ulcerative colitis chills, nausea, vomiting, lumbar pain, and
B. Chronic enterocolitis dysuria. Costovertebral angle tenderness is
C. Colon cancer present on both sides. Urine analysis: pyuria,
D. Irritable bowel syndrome bacteriuria. Blood test: leukocytosis. What is
E. Crohn’s disease (regional enteritis) the most likely diagnosis?
192. A 48-year-old woman has been hospitali- A. Gestational pyelonephritis
zed due to development of tachysystolic atrial B. Cystitis
fibrillation. She has lost 5 kg of body weight C. Pyelitis
within 2 months. On palpation there is a node D. Glomerulonephritis
in the left lobe of the thyroid gland. What E. Latent bacteriuria
pathology resulted in the development of this
condition? 197. A 19-year-old young man complains of
cough with expectoration of purulent sputum
A. Toxic nodular goiter in amount of 100 ml per day, hemoptysis,
B. Aterosclerotic cardiosclerosis dyspnea, increased body temperature up
C. Chronic thyroiditis to 37,8oC , general weakness, weight loss.
D. Nontoxic nodular goiter The patient’s condition lasts for 4 years.
E. Autoimmune thyroiditis Exacerbations occur 2-3 times a year.
The patient presents with malnutrition,
193. On the 9th day after childbirth the pale skin, cyanosis of the lips, drumstick
obstetric patient developed high fever up (clubbed) fingers. Tympanic percussion sound
to 38o C . She complains of pain in the right in the lungs, weakened respiration, vari-
mammary gland. On examination a sharply ous numerous moist crackles in the lower
painful infiltrate can be palpated in the right pulmonary segments on the left can be
mammary gland, the skin over the infiltrate observed. In blood: erythrocytes - 3, 2 · 1012 /l,
is red, subareolar area and nipple are swollen
and painful. What is your diagnosis? leukocytes - 8, 4 · 109 /l, ESR- 56 mm/hour.
On X-ray: lung fields are emphysematous, the
A. Abscess of the right mammary gland left pulmonary root is deformed and dilated.
B. Mastopathy What is the most likely diagnosis?
C. Cancer of the right mammary gland
D. Serous mastitis
E. Fibrous cystic degeneration of the right
mammary gland
Крок 2 Medicine (англомовний варiант, iноземнi студенти) 2017 рiк 26

A. Multiple bronchiectasis of the left lung her son with violence. She had opened her
B. Chronic left-sided pneumonia window (on the 8th floor) and tried to jump
C. Chronic abscess of the left lung out, resisted the people, who were holding her
D. Left-sided pulmonary cystic dysplasia back. What drugs should be administered to
E. Suppuration of the cyst in the left lung terminate such condition of the patient?
198. A 37-year-old man working as a A. Antipsychotics
typesetter in a print shop complains of B. Antidepressants
rapid fatigability, paroxysmal attacks of C. Tranquilizers
stomachache, weak drooping hands. Exami- D. Nootropic agents
nation of neurological status revealed E. Antihypertensive drugs
hypotrophy of the forearm muscles.
Carporadial reflexes are sharply weakened. 200. A 43-year-old man, who has been abusi-
Sensitivity is not disturbed. Gums present ng alcohol and suffering from pulmonary
with dark blue border. What neurological tuberculosis, in the course of two weeks
pathology is it? gradually developed general weakness,
headache, diplopia, vomiting. Objectively:
A. Lead polyneuropathy ptosis on the left, anisocoria S>D, exotropia
B. Guillain-Barre syndrome (postinfectious of the left eye, neck stiffness; Kernig’s
polyneuritis) and Brudzinski’s signs are positive. In
C. Shingles cerebrospinal fluid: lymphocytic pleocytosis,
D. Ulnar neuropathy low glucose, precipitation of cerebrospinal
E. Brachial plexitis fluid resulted in production of fibrin film.
What is the most likely diagnosis?
199. A 54-year-old woman takes anti-
hypertensive drugs for hypertension. Havi- A. Tuberculous meningitis
ng discovered that her son was arrested and B. Subarachnoid hemorrhage
is under investigation, became agitated and C. Brainstem encephalitis
extremely anxious. She lost her orientation D. Acute myelitis
in place, stopped recognizing her relatives, E. Basal arachnoiditis
started hearing ”voices” threatening her and
INSTRUCTIONAL BOOK

Testing Board

TEST ITEMS FOR LICENSING EXAMINATION: KROK 2. MEDICINE.

Kyiv. Testing Board.


(English language).

Approved to print 24.04/№217. Paper size 60х84 1/8


Offset paper. Typeface. Times New Roman Cyr. Offset print.
Conditional print pages 24. Accounting publishing pages 28.
Issue. 2610 copies
List of abbreviations
List of abbreviations
ACTH Adrenocorticotrophic hormone
ADP Adenosine diphosphate
ALT, ALAT Alanin aminotranspherase
ALV Artificial lung ventilation
AP Arterial (blood) pressure
AST, AspAT Aspartat aminotranspherase
ATP Adenosine triphosphate
ВСG Bacillus Calmette-Guérin
BP Blood (arterial) pressure
CK Creatine kinase
CNS Central nervous system
СоА Coenzyme A
CRP C-reactive protein
CT Computed tomography
COX Cyclooxygenase
DIC Disseminated intravascular coagulation
DTP (DPT) Diphtheria-tetanus-pertussis vaccine
EDTA Ethylenediamine tetra-acetic acid
ELISA Enzyme-linked immunosorbent assay
ENT Ear, nose, and throat (as a department in a hospital)
EPR (ER) Endoplasmic reticulum
ESR Erythrocyte sedimentation rate
EV Enterovirus
FC Functional class
GABA Gamma aminobutyric acid
Hct, Ht Hematocrit
HDL High-density lipoproteins
HR Heart rate
IDL Intermediate-density lipoproteins
IU International unit
IUPAC International Union of Pure and Applied Chemistry
LDH Lactate dehydrogenase
LDL Low-density lipoproteins
LOX Lipoxygenase
MAO Monoamine oxidase
MRI Magnetic resonance imaging
NSAID Nonsteroidal anti-inflammatory drug
PE (PATE) Pulmonary embolism (Pulmonary artery thromboembolism)
PSA Prostate-specific antigen
RBC Red blood count
RR Respiratory rate
SES Sanitary-and-epidemiologic station
STD Sexually transmitted disease
STI Sexually transmitted infection
T/l Trillion/liter
TABT Typhoid-paratyphoid A and B plus tetanus toxoid vaccine
TMJ Temporomandibular joint
TSH Thyroid-stimulating hormone
TU Tuberculin unit
URTI Upper respiratory tract infection
V/f Vision field
VLDL Very-low-density lipoproteins
WBC White blood count
WPW Wolff-Parkinson-White syndrome
MINISTRY OF PUBLIC HEALTH OF UKRAINE

Department of human resources policy, education and science

Testing Board

Student ID Last name

Variant ___________________

Test items for licensing examination

Krok 2
MEDICINE
General Instruction
Every one of these numbered questions or unfinished statements in
this chapter corresponds to answers or statements endings. Choose the
answer (finished statements) that fits best and fill in the circle with the
corresponding Latin letter on the answer sheet.

Authors of items: Afanasievska M.S., Baburina O.A., Bahirian I.O., Bedeniuk A.D., Beriozov V.M.,
Besh L.V., Beziuk M.M., Bezruk T.O., Bielska O.A., Bilyk V.D., Bobrytska V.V., Borysova T.P.,
Budaieva I.V., Bukhtieieva E.R., Bukhtiyarova O.H., Buriak O.H., Butina L.I., Chaika O.O.,
Chebanenko R.O., Chemodanov E.B., Chichirelio-Konstantynovych K.D., Chuiko A.P., Dankyna I.A.,
Davydenko O.M., Desiatska Yu.V., Detsyk O.Z., Dobrovolska L.M., Dotsenko T.M., Dovhaliuk Yu.P.,
Duplenko P.Yu., Furman V.M., Halicheva N.O., Harahulia I.S., Herasymenko O.I., Hlumcher F.S.,
Hnatko O.P., Honcharova I.M., Hordieieva H.D., Hresko M.M., Hyrla Ya.V., Ivakhno O.P., Ivanchuk P.R.,
Kadochnykov V.S., Kalinovska I.V., Kalsada I.N., Kalutsky I.V., Kandyba V.P., Kanikovsky O.Ye.,
Kanovska L.V., Karliychuk O.O., Khomaziuk T.A., Kitura O.Ye., Koliush O.I., Koloskova O.K.,
Koltsova N.I., Kompaniyets K.M., Kondratenko P.H., Kondratiev V.O., Konopliova L.F., Konovalova N.V.,
Kopchak O.V., Korniyets N.H., Korobchansky V.O., Korobko O.A., Kovalchuk P.Ye., Kovtun A.I.,
Kriachkova L.V., Krylova V.Yu., Kucherenko O.S., Kudelia I.V., Kudintseva T.Z., Kudrevych O.M.,
Kudria V.I., Kuzmenko S.A., Lakusta N.M., Lavrinchuk I.O., Lebediuk M.M., Leshchenko K.A.,
Loskutova I.Ye., Lysenko D.A., Makieieva N.I., Malkovych N.M., Martsynik Ye.M., Martyniuk L.P.,
Mavrutenkov V.V., Mazorchuk S.H., Medvediev M.V., Miakinkova L.O., Mierkulova N.F., Mitiunina N.I.,
Mostovy Yu.M., Mysak A.I., Nadraha O.B., Nechytailo Yu.M., Nykoniuk T.R., Osetrova M.S., Osiadla E.S.,
Ospanova T.S., Ostropolets M.S., Ostrovsky I.M., Pavlovych L.V., Petriuk B.V., Pisotska L.A., Prus L.O.,
Pryshliak O.Ya., Radchenko O.M., Radutna O.A., Rak L.M., Reva V.B., Riabenko E.B., Riapolova T.L.,
Romanenko V.N., Rud V.O., Seleznova S.V., Semeniak A.V., Semenukha K.V., Serheta I.V.,
Shapkin V.Ye., Shcherbinin O.V., Shkondina O.F., Shkrobanets I.D., Shorikov Ye.I.,
Shostakovych-Koretska L.R., Shtyker S.Yu., Shumko B.I., Shvyhar L.V., Sikorska M.V., Sliepichko Yu.M.,
Slyva V.I., Smoliak L.L., Soboleva N.P., Sokolov O.B., Soldak I.I., Soloviov I.Ye., Sorochan V.V.,
Sotnik Yu.P., Suk V.H., Sushkov M.T., Sychova V.V., Sylenko H.Ia., Synoverska O.B., Syvozhelizov A.V.,
Talalaienko Yu.O., Todoriko L.D., Tokariev A.V., Tonkohlas O.A., Toropchyn V.I., Troian V.I.,
Tsariova N.M., Tsertiy V.P., Tsiapa N.A., Tsyvenko O.I., Tuchkina I.O., Vankhanen V.D., Volianska A.H.,
Vorokhta Yu.M., Yakovenko I.K., Yermolenko T.O., Yurieva L.M., Zakharchenko Yu.B., Zheliba M.D.,
Znakhurenko L.S., Zotov O.S. and Committees of professional expertise

Item reviewers. Ahafonova O.O., Anisimov Ye.M., Babiak T.Ye., Bobrytska V.V., Borysova T.M.,
Chursina T.Ya., Dyndar O.A., Holovanova I.A., Hryhorov Yu.B., Hubka V.O., Hutsalenko O.O.,
Kalinina S.Yu., Kanikovsky O.Ye., Karapetian K.H., Khomaziuk T.A., Kolesnyk O.M., Koloskova O.K.,
Kolosovych I.V., Kopcha V.S., Kravchenko O.V., Kutovy O.B., Kuzmina I.Yu., Malanchuk L.M.,
Martyniuk L.P., Mavrutenkov V.V., Medvediev M.V., Mishchenko V.P., Moroz L.V., Muravska O.M.,
Petrushenko V.V., Prokhorova M.P., Pryshliak O.Ya., Puzanova O.H., Pyptiuk O.V., Radchenko O.M.,
Shestakova I.V., Stovban I.V., Sydorchuk R.I., Tsvirenko S.M., Usenko S.H., Vakaliuk I.P., Volianska A.H.

The book includes test items for use at licensing integrated examination “Krok 2. Medicine” and
further use in teaching.

The book has been developed for students of medical, pediatric and medical-and-prophylactic
faculties and academic staff of higher medical educational establishments.

Approved by Ministry of Public Health of Ukraine as examination and teaching


publication based on expert conclusions (Orders of MPH of Ukraine of
14.08.1998 №251, of 27.12.1999 №303, of 16.10.2002 №374, of 29.05.2003 №233).

© Copyright Testing Board.


Krok 2 Medicine (англомовний варiант, iноземнi студенти) 2018 рiк 1

1. A patient with Morgagni-Adams-Stokes A. Spontaneous pneumothorax


syndrome has fainted when walking up the B. Hemothorax
stairs. The skin is pale, the pupils are dilated, C. Lobar pneumonia
tonoclonic spasms are observed, rib cage is D. Pulmonary embolism
immobile. Make the diagnosis: E. Acute pleurisy

A. Clinical death 6. A 34-year-old man on the 3rd day of ceftri-


B. Social death axone treatment for acute otitis (daily dosage
C. Preagony - 2 grams) developed diarrhea occurring 5-6 ti-
D. Agony mes per day. Feces are without mucus or blood
E. Biological death admixtures. Temperature is 36.6o C . Gregersen
reaction (occult blood in feces) is negative.
2. A 32-year-old welder complains of weakness Stool culture detected no pathogenic germs.
and fever. His illness initially presented as What is the most likely cause of diarrhea in this
tonsillitis one month earlier. On examination: case?
temperature - 38.9o C , respirations - 24/min.,
pulse - 100/min., blood pressure - 100/70 mm A. Antibiotic-associated diarrhea
Hg, hemorrhages on the legs, enlargement of B. Intestinal dysbiosis
the lymph nodes. Complete blood count shows C. Bacterial overgrowth syndrome
Hb- 70 g/L, RBC- 2.2 · 1012 /L, WBC- 3.0 · 109 /L D. Ulcerative colitis
with 32% of blasts, 1% of eosinophiles, 3% of E. Crohn’s disease (regional enteritis)
bands, 36% of segments, 20% of lymphocytes,
and 8% of monocytes, ESR- 47 mm/hour. What 7. A 24-year-old patient visited a doctor
is the cause of anemia in this case? complaining of enlargement of his submaxi-
llary lymph nodes. Objectively: submaxillary,
A. Acute leukemia axillary and inguinal lymph nodes are enlarged.
B. Chronic lympholeukemia Chest X-ray shows: enlarged lymph nodes
C. Aplastic anema of mediastinum. Blood test: erythrocytes -
D. B12 -deficient anemia 3.4 · 1012 /L, Hb- 100 g/L, blood colour index
E. Chronic hemolytic anemia - 0.88, platelets - 190 · 109 /L, leucocytes -
3. After a 5-day-long celebration of his 7.5 · 109 /L, eosinophiles - 8%, band neutrophi-
daughter’s wedding a 65-year-old patient ”saw” les - 2%, segmented neutrophiles - 67%,
in his yard many cats, chickens, and rats. He tri- lymphocytes - 23%, ESR - 22 mm/hour. What
ed to chase them away, but was scared off when test must be prescribed to verify the cause of
the animals started scolding him and tried to lymphadenopathy?
harm him. What is the likely diagnosis? A. Open biopsy of the lymph nodes
A. Delirium tremens B. Abdominal US
B. Senile psychosis C. Mediastinum tomography
C. Schizophrenia D. Puncture biopsy of the lymph nodes
D. Organic brain syndrome E. Sternal puncture
E. Reactive hallucinosis 8. During medical examination a cadet in the
4. A regional cardiologist is given a task to naval college was detected to have a painless
develop a plan for preventive measures ai- dense ulcer 1.5x0.5 in size in his perianal area at
med at decreasing cardiovascular mortality the 2 o’clock position. The ulcer floor resembles
rates. What measures should be planned for ”old fat”. What is the provisional diagnosis?
secondary prevention? A. Hard syphilitic chancre of the rectum
A. Prevention of recurrences and complications B. Rectal fissure
B. Referring patients for sanatorium-and-spa C. Rectal fistula
treatment D. Anal cancer
C. Prevention of diseases E. Anal crypt suppuration
D. Referring patients for inpatient treatment 9. A 7-year-old boy has severe pulmonary
E. Optimization of lifestyle and living conditions mucoviscidosis (cystic fibrosis). He complai-
5. During physical exertion a man suddenly ns of dyspnea and blood expectoration.
developed acute chest pain on the right and Objectively he presents with lagging physi-
dyspnea. Objectively he assumes forced half- cal development, acrocyanosis, hepatomegaly,
sitting position in the bed, presents with diffuse drumstick fingers, and nail plates resembling
cyanosis, resting tachypnea of 38/min., the ri- a ”clock face”. Provisional diagnosis of chronic
ght side of the thorax is enlarged and does not pulmonary heart disease is made. What exami-
participate in the respiratory process; percussi- nation would be the most informative for di-
on on the right reveals tympanic resonance and agnosis confirmation?
absence of respiration.What is the most likely A. Doppler echocardiography
diagnosis in this case? B. Electrocardiography
C. Chest X-ray
D. Rheography of the pulmonary artery
E. Ultrasound of the liver
Krok 2 Medicine (англомовний варiант, iноземнi студенти) 2018 рiк 2

10. A 10-year-old boy, who was outdoors in chills, acute headache, myalgia. In the axillary
windy and cold weather, developed moderate region the lymph node enlarged up to 3x6 cm
pain and tingling in his fingers and toes. When can be palpated. The lymph node is dense,
he had returned home, his parents noticed that intensely painful, slightly mobile, without clear
the tips of his fingers and toes were white and margins; the skin over the node is hyperemic
their sensitivity was lost. The affected areas and tight. Tachycardia is present. Make the
are warming up, the fingers are tingling and preliminary diagnosis:
in pain. Skin pallor changed into redness, ti-
ngling stopped, slight itching and swelling of A. Plague
the fingers appeared. Determine the frostbite B. Sepsis
degree in this child: C. Tularemia
D. Lymphadenitis
A. Frostbite of the I degree E. Anthrax
B. Perniosis
C. Frostbite of the II degree 15. Caries morbidity rate is 89% among resi-
D. Frostbite of the III degree dents of a community. It is determined that
E. Frostbite of the IV degree fluorine content in water is 0.1 mg/L. What
preventive measures should be taken?
11. A 25-year-old woman complains of fati-
gue, dizziness, hemorrhagic rashes on the skin. A. Water fluorination
She has been presenting with these signs for a B. Tooth brushing
month. Blood test: erythrocytes - 1.0 · 1012 /L, C. Fluorine inhalations
Hb- 37 g/L, colour index - 1.1, leukocytes - D. Sealant application
1.2 · 109 /L, platelets - 42 · 109 /L. What analysis E. Introduce more vegetables to the diet
would be the most advisable for diagnosis- 16. During appointment with the doctor
making in this case? a man complains of painful itching rashes
that appeared on his skin under the beard
A. Sternal puncture (bone marrow biopsy) and moustache one year ago, with frequent
B. Splenic biopsy exacerbations occurring throughout the year.
C. Liver biopsy Objectively the skin of the facial hair growth
D. Coagulation studies areas is bluish-purple, thickened, with pustules,
E. US of the gastrointestinal tract erosions, and scabs covering its moist surface.
12. A 12-year-old boy with hypertrophic cardi- The fistulous tracts are surrounded by the
omyopathy complains of dyspnea caused by area of loose pink-red granulation and di-
the slightest physical exertion. Echocardi- scharge pus. The skin resembles mulberry in
ography detected asymmetrical left ventricular appearance. Make the diagnosis:
hypertrophy, signs of pulmonary hypertension, A. Sycosis
and left ventricular dilatation, its ejection fracti- B. Acne rosacea
on is 59%. These developments are indicative C. Lupus erythematosus
of: D. Deep trichophytosis
A. Heart failure with preserved ejection fraction E. Tuberculous lupus
B. Heart failure with reduced ejection fraction 17. A 60-year-old man presents with ischemic
C. Primary pulmonary hypertension heart disease and heart failure of the IV class
D. Essential hypertension according to NYHA (New York Heart Associ-
E. Symptomatic arterial hypertension
ation) that manifests as dyspnea at rest. There
13. A 35-year-old man complains of rapidly are moist crackles in the patient’s lungs. Liver
incresing fatigue, palpitations, ”visual snow”, +4 cm, lower limbs are swollen. Ejection fracti-
dizziness. He has a history of peptic ulcer of the on is 25%. What sign is the most indicative
stomach. Objectively the skin is pale. Vesicular when determining functional class of heart fai-
respiration is observed in the lungs. Systolic lure according to NYHA?
murmur is detected over the cardiac apex, heart
rate is 100/min., BP is 100/70 mm Hg. The epi- A. Degree of dyspnea
gastrium is slightly tender on palpation. Blood B. Moist crackles in the lungs
C. Swollen lower limbs
test: erythrocytes - 3.2 · 1012 /L, Нb- 100 g/L, D. Decrease of ejection fraction
color index - 0.94. What type of anemia is the E. Extent of liver enlargement
most likely present in this case?
18. A 39-year-old man, a battery attendant,
A. Posthemorrhagic anemia suddenly developed weakness, loss of appeti-
B. Sideroblastic anemia te, nonlocalized colicky abdominal pains, and
C. Iron-deficiency anemia nausea. Objectively his skin is gray; there is
D. Hemolytic anemia pink-gray stripe on his gums; the stomach is
E. Hypoplastic anemia soft and sharply painful. Blood test detected
14. A 35-year-old patient has been sufferi- erythrocytes with basophilic stippling and
ng from an illness for 3 days. 5 days ago he anemia. The patient has a history of peptic
returned from a trip to Africa. The onset of di- ulcer disease of the stomach. There is tendency
sease was accompanied by fever up to 40o C , to constipation. What is the most likely provisi-
onal diagnosis?
Krok 2 Medicine (англомовний варiант, iноземнi студенти) 2018 рiк 3

5th intercostal nerves on the left the skin is


A. Saturnism (lead poisosning) hyperemic and there are tight clusters of small
B. Acute appendicitis vesicles filled with clear serous content. What is
C. Perforation of gastric ulcer the most likely diagnosis?
D. Acute cholecystitis
E. Chronic alcoholism A. Herpes zoster
B. Herpes simplex
19. A 9-month-old infant presents with delayed C. Streptococcal impetigo
tooth eruption and fontanel closure, weakness, D. Pemphigus
and excessive sweating. What type of hypovi- E. Dermatitis herpetiformis (Duhring’s disease)
taminosis is the most likely in this child?
24. A worker of a blowing shop complains of
A. Hypovitaminosis D headache, irritability, sight impairment - he sees
B. Hypovitaminosis C everything as if through a ”net”. Objectively:
C. Hypovitaminosis B1 hyperemic sclera, thickened cornea, decreased
D. Hypovitaminosis B6 opacity of pupils, visual acuity is 0.8 in the left
E. Hypovitaminosis A eye, 0.7 in the right eye. The worker uses no
means of personal protection. What diagnosis
20. A 10-year-old girl exhibits high level of

physical development ( + 3σ ), her body
is the most likely?
length increased by 10 cm within a year (whi- A. Cataract
ch is double the norm for her age group), the B. Conjunctivitis
number of permanent teeth corresponds wi- C. Keratitis
th the age norm (20), the development of her D. Blepharospasm
secondary sex characteristics is three years E. Progressive myopia
ahead of her age (Ма, Р, Ах, Menarche).
Development rate ahead of her biological age 25. For a week a 42-year-old patient has been
can occur due to: suffering from fever attacks followed by high
temperature, which occur every 48 hours. Body
A. Endocrine disorders temperature raises up to 40o C and decreases in
B. Acceleration 3-4 hours with excessive sweating. The patient
C. Certain components of her diet presents with loss of appetite and general fati-
D. Sports training gue. The skin is pale and sallow. The liver and
E. Deficient hygienic education spleen are enlarged and dense on palpation.
What method of diagnosis verification would
21. An infant has been born at the 41st week of be most efficient?
gestation. The pregnancy was complicated wi-
th severe gestosis of the second semester. The A. Microscopy of blood smear and thick blood
weight of the baby is 2400 g, the height is 50 film
cm. Objectively: the skin is flabby, the layer B. Complete blood count
of subcutaneous fat is thin, hypomyotonia, C. Bacteriological analysis
neonatal reflexes are weak. The internal organs D. Immune-enzyme assay
are without pathologic changes. This newborn E. Microscopy of hanging blood drop
can be assessed as a:
26. A 16-year-old adolescent living in a rural
A. Full-term infant with intrauterine growth area has been bitten in the shin by a stray
retardation dog. The wound is superficial. Regular vacci-
B. Premature infant nation against tetanus was received 3 months
C. Immature infant ago. What treatment tactics would be the most
D. Postmature infant advisable in this case?
E. Full-term infant with normal body weight
A. Antirabies vaccination
22. A patient suffering from infiltrati- B. Antirabies immunoglobulin
ve pulmonary tuberculosis was prescribed C. Tetanus toxoid adsorbed
streptomycin, rifampicin, isoniazid, pyrazi- D. Antitetanus serum
namide, vitamin C. One month after the begi- E. Antitetanus immunoglobulin
nning of the treatment the patient started
complaining of reduced hearing and tinnitus. 27. A 26-year-old woman is suspected to
What drug has such a side effect? suffer from systemic lupus erythematosus with
systemic lesions of skin, vessels, joints, serous
A. Streptomycin tunics, and heart tissue that developed after
B. Isoniazid photosensitization. The following is detected in
C. Rifampicin blood analysis: LE cells, antibodies to native
D. Pyrazinamide ds-DNA, isolated anti-centromere antibodies,
E. Vitamin C rheumatoid factor is 1:100, Wassermann reacti-
on is positive, circulating immune complex is
23. The doctor has an appointment with a pati- 120 units. What immunological indicators are
ent, who 2 days ago developed severe chest pain considered to be specific to this disease?
on the left, general weakness, high temperature,
and headache. Objectively along the 4th and
Krok 2 Medicine (англомовний варiант, iноземнi студенти) 2018 рiк 4

A. ds-DNA antibodies is 106/min., rhythmic. Blood pressure is 90/60


B. Rheumatoid factor mm Hg. Cardiac borders are unchanged, heart
C. Anti-centromere antibodies sounds are weakened, at the cardiac apex there
D. Immunoglobulin A is soft systolic murmur. What factor would be
E. Increased circulating immune complex the most indicative of the likely disease eti-
ology?
28. A group of 5 had been resting in a forest,
they were drinking alcohol and eating canned A. Anti-streptolysin O
mushrooms and cured fish. The next day two B. C-reactive protein
of them were hospitalized with disturbed visi- C. Creatine kinase
on, swallowing and respiration; the third one D. Rheumatoid factor
presented with acute general weakness and dry E. Seromucoid
mouth. The remaining two were healthy. A tick
was detected on the skin of one of the healthy 33. A 20-year-old patient complains of severe
group members. What is the most likely di- headache, double vision, weakness, fever,
agnosis? irritability. Objectively: body temperature is
38.1o C , the patient is reluctant to contact, sensi-
A. Botulism tive to stimuli. There are ptosis of the left
B. Tick-borne encephalitis eyelid, exotropia, anisocoria S>D, pronounced
C. Alcohol poisoning meningeal syndrome. On lumbar puncture
D. Mushroom poisoning the cerebrospinal fluid flowed out under a
E. Lyme borreliosis pressure of 300 mm Hg, the fluid is clear, sli-
ghtly opalescent. 24 hours later there appeared
29. A 23-year-old man complains of facial fibrin film. Protein - 1.4 g/L, lymphocytes - 600/3
edema, headache, dizziness, low urinary output,
per mm3 , sugar - 0.3 mmol/L. What is the provi-
urine discoloration (dark red). These complai- sional diagnosis?
nts arose after the patient had had a case of
acute tonsillitis. On examination there are faci- A. Tuberculous meningitis
al edema, the skin is pale, temperature is 37.4o C ; B. Meningococcal meningitis
heart rate is 86/min., blood pressure is 170/110 C. Lymphocytic (Armstrong’s) meningitis
mm Hg. Heart sounds are muffled, the II heart D. Syphilitic meningitis
sound is accentuated over the aorta. What eti- E. Mumps meningitis
ological factor is the most likely in this case?
34. A 32-year-old woman complains of increasi-
A. Beta-hemolytic streptococcus ng spastic pains in her lower abdomen that
B. Staphylococcus aureus occur after emotional stress. Bowel movements
C. Streptococcus viridans are intermittent: 2-3 defecations after waking
D. Streptococcus pyogenes in the morning alternate with constipations
E. Staphylococcus saprophyticus that last for 1-2 days. Objectively body mass
30. During examination a 4-month-old chi- is retained, palpation of the sigmoid colon is
ld with meningococcemia presents with moderately painful. Hb- 130 g/L, leukocytes -
acrocyanosis, cold extremities, tachypnea, and 5.2 g/L, ESR- 9 mm/hour. Rectoromanoscopy is
thready pulse, blood pressure of 30/0 mm Hg, painful due to spastic condition of the intestine,
anuria, and sopor. What clinical syndrome is it? intestinal mucosa is without changes. Intestinal
lumen contains large amounts of mucus. What
A. Septic shock is the most likely diagnosis in this case?
B. Neurotoxicosis
C. Exicosis A. Irritable bowel syndrome
D. Encephalic syndrome B. Crohn’s disease (regional enteritis)
E. Acute renal failure C. Nonspecific ulcerative colitis
D. Acute mesenteric ischemia
31. The pregnancy is full term. The body wei- E. Malabsorption syndrome
ght of the parturient woman is 62 kg. Fetus
is in a longitudinal lie, the head is engaged to 35. The inpatient surgery unit has introduced
the pelvic inlet. Belly circumference is 100 cm. the method of laparoscopic cholecystectomy
Uterine fundus height is 35 cm. What body wei- into its practice. As the result the average
ght of the fetus can be expected? duration of postoperative care provided to
the patients could be reduced to 3.4±0.8 days
A. 3.5 kg compared to 7.3±1.1 days that were required
B. 4 kg after non-laparoscopic cholecystectomy. What
C. 2.5 kg method of medical statistics can confirm the
D. 3 kg statistical significance of the difference between
E. 4.5 kg these two estimates?
32. An 18-year-old young man complains of
pain in his knee and elbow joints and body
temperature up to 39.5o C . One week and a half
earlier developed sore throat. On examinati-
on his body temperature is 38.5o C . Swelling of
the knee and elbow joints is observed. Pulse
Krok 2 Medicine (англомовний варiант, iноземнi студенти) 2018 рiк 5

A. Calculation of Student’s confidence interval This kind of pain attacks has never been
B. Calculation of correlation coefficient detected in the patient before. Plain abdomi-
C. Calculation of standardized ratio nal X-ray reveals no pathologic shadows.
D. Calculation of average values (measures of Ultrasound detects a hyperechogenic mass 1.5
central tendency) cm in diameter, which reflects sound wave, in
E. Calculation of relative values the enlarged right renal pelvis. What diagnosis
is the most likely?
36. A 37-year-old worker during a fire ended up
in the area of high CO concentration. He was A. Renal calculus
delivered to a hospital in unconscious state. B. Benign renal tumor
Objectively: the skin of his face and hands is cri- C. Renal cyst
mson. Respiration rate is 20/min. ECG: alterati- D. Renal tuberculosis
ons specific for hypoxic myocardium. Hourly E. Malignant renal tumor
diuresis is 40 ml. Blood test: erythrocytes -
4.5 · 1012 /L, Нb- 136 g/L, color index - 0.9, ESR- 41. A 60-year-old woman complains of
3 mm/hour, carboxyhemoglobin - 5%. What unbearable pain in her right subcostal area. She
criterion allows determining the severity of the has a history of acute pancreatitis. Temperature
patient’s condition? is 38.2o C . Objectively her sclera are icteric.
There are no signs of peritoneal irritation.
A. Carboxyhemoglobin concentration Ortner’s and Gubergrits’ symptoms are positi-
B. Respiratory disorders ve. Urine diastase is 320 U/L. What is the most
C. ECG results likely diagnosis?
D. Extent of trophic disorders
E. Development of chronic renal failure A. Exacerbation of chronic pancreatitis
B. Acute cholangitis
37. The right arm of a newborn is stretched C. Chronic cholecystitis
along the torso with all its joints extended; the D. Acute cholecystitis
shoulder is rotated inwards, while the forearm E. Pancreatic cancer
is pronated, the hand is in the position of
palmar flexion. Spontaneous movements are 42. A 64-year-old man complains of cough wi-
absent in the shoulder and elbow joints, passive th expectoration consisting of blood-streaked
movements are painless. What is the most likely mucus, dyspnea, low grade fever and general
diagnosis? fatigue. He has been presenting with these
symptoms for 3 months. He has been smoki-
A. Duchenne-Erb palsy, superior proximal type ng since early adolescence. Objectively to
B. Dejerine-Klumpke palsy, inferior distal type is 37.4o C , respirations are 26/min., pulse is
C. Total obstetric palsy 82/min., rhythmic, blood pressure is 130/85 mm
D. Osteomyelitis of the right humerus Hg. The right side of the thorax lags behi-
E. Poliomyelitis nd in the respiratory process, dull percussion
sound and acute decrease of breathing activity
38. In April during the medical examination are observed there. X-ray shows homogeneous
of various population groups, 27% of indivi- shadow of the lung field on the right with medi-
duals presented with low working ability and astinum displacement towards the affected side.
rapid fatigability. The following symptoms were What is the most likely diagnosis?
observed in the affected individuals: swollen
friable gingiva that bleeds when pressed, A. Central lung cancer
hyperkeratosis follicularis not accompanied by B. Exudative pleuritis
skin dryness. These symptoms most likely result C. Pleuropneumonia
from the following pathology: D. Pulmonary tuberculosis
E. Multiple bronchiectasis
A. C -hypovitaminosis
B. Parodontosis 43. For 4 days a 35-year-old man has been


C. A-hypovitaminosis treated in the resuscitation unit for acute renal
D. 1 -hypovitaminosis failure caused by compartment syndrome. The
E. Polyhypovitaminosis patient is disoriented. ECG shows high T
waves and right ventricular extrasystoles. His
39. A 1-year-old child with a case of URTI central venous pressure is 159 mmH2 O; for the
suddenly developed noisy respirations with di- last 3 hours auscultation has been detecting
fficult inspiration, intercostal retractions, and isolated moist crackles in the lungs. Respirati-
barking cough on the 2nd night after the di- ons are 32/min. Blood test: residual nitrogen -
sease onset. What is the most likely diagnosis? 62 mmol/L, K + - 7.1 mmol/L, Cl− - 78 mmol/L,
A. Stenosing laryngotracheobronchitis N a+ - 120 mmol/L, Ht- 0.32 L/L, Hb- 100 g/L,
blood creatinine - 0.9 mmol/L. In this case the
B. Acute pulmonary inflammation most advisable would be to perform:
C. Bronchial asthma
D. Acute bronchitis A. Hemodialysis
E. Acute bronchiolitis B. Plasmasorption
40. A 46-year-old woman complains of severe C. Hemosorption
D. Plasmafiltration
pain attacks in the right lumbar area, which E. Ultrafiltration
irradiate to the lower abdomen, and nausea.
Krok 2 Medicine (англомовний варiант, iноземнi студенти) 2018 рiк 6

44. A 55-year-old woman, a cook, complai- with tonic spasm of the facial muscles. What is
ns of pain in her right knee joint that has the most likely diagnosis among those listed?
been troubling her for a month and intensifi-
es in the evening. Objectively she is overwei- A. Trigeminal neuralgia
ght, the knee joint is swollen, creaks during B. Glossopharyngeal neuralgia
movement, palpation reveals localized pain. C. Temporomandibular joint arthritis
The 1st metatarsophalangeal articulation is D. Facial migraine
deformed on the both feet. No changes in blood E. Maxillary sinusitis
and urine are detected. What should be visible
on the patient’s X-ray? 48. A 10-year-old boy with symptoms of arthri-
tis and myocarditis was delivered into a hospi-
A. Joint space narrowing, marginal osteophytes tal. Based on clinical examination the prelimi-
B. Osteoporosis, joint space narrowing, singular nary diagnosis of juvenile rheumatoid arthritis
usurations was made. What symptom is the most contri-
C. Joint space narrowing, multiple usurations, butive for the diagnostics of this disease?
subluxations
D. Epiphyseal erosions, bony ankylosis A. Reduced mobility of the joints in the morning
E. Joint space narrowing, round bone defects B. Regional hyperemia of the joints
C. Affection of the large joints
45. A 52-year-old woman has been suffering D. Enlarged heart
for 2 years from dull, occasionally exacerbating E. Increased heart rate
pain in her right subcostal area, occurring after
eating high-fat foods, bitter taste in her mouth 49. A 25-year-old patient was delivered to
in the morning, constipations, and flatulence. an infectious diseases unit on the 3rd day
Objectively she has excess weight, her body of illness with complaints of headache, pain
temperature is 36.9o C ; there is a coating on the in lumbar spine and gastrocnemius muscles,
root of her tongue; the abdomen is moderately high fever, chill. Objectively: condition of
distended and painful in the area of gallbladder moderate severity. Scleras are icteric. Pharynx is
projection. What examination would be the hyperemic. Tongue is dry with dry brown coati-
most helpful for diagnosis-making? ng. Abdomen is distended. Liver is enlarged
by 2 cm. Spleen is not enlarged. Palpation of
A. Ultrasound muscles, especially gastrocnemius muscles, is
B. Duodenal intubation painful. Urine is dark in colour. Stool is normal
C. Cholecystography in colour. The most likely diagnosis is:
D. Duodenoscopy
E. Liver scanning A. Leptospirosis
B. Viral hepatitis type A
46. A 57-year-old woman complains of C. Malaria
weakness, dyspnea, loss of appetite, and li- D. Infectious mononucleosis
quid feces. She has been suffering from this E. Yersiniosis
condition for 2 years. Objectively she presents
with pale skin, subicteric sclera, and bright- 50. A 28-year-old woman complains of
red fissured tongue. Lymph nodes are not skin hemorrhages after minor traumas and
enlarged. Pulse - 100/min. BP- 105/70 mm Hg. spontaneous appearance of hemorrhages on
Liver +3 cm, the spleen cannot be palpated. the front of her torso and extremities. On
examination: the skin is variegated (old and
Blood test: erythrocytes - 1.2 · 1012 /L, Нb- 56
g/L, color index - 1.4, macrocytes, leukocytes new hemorrhages), bleeding gums. Blood
- 2, 5 · 109 /L, eosinophils - 1%, juvenile - 1%, platelets - 20 · 109 /L; in the bone marrow there
metamyelocytes - 1%, band neutrophils - 8%, is increased number of megakaryocytes and
segmented neutrophils - 47%, lymphocytes - no platelet production. Treatment with steroid
38%, monocytes - 4%, reticulocytes - 0.1%, hormones was effective. What is the likely di-
agnosis?
platelets - 100 · 109 /L, ESR- 30 mm/hour, indi-
rect bilirubin - 26 mmol/L. What changes can be A. Idiopathic thrombocytopenic purpura
expected in the bone marrow puncture materi- B. Hemophilia
al? C. Rendu-Osler-Weber disease (Hereditary
A. Prevalence of megaloblasts hemorrhagic telangiectasia)
B. Increased number of sideroblasts D. Disseminated intravascular coagulation
C. Erythroid hyperplasia E. Acute vascular purpura
D. Presence of blast cells 51. A 7-year-old boy has been an inpatient for
E. Prevalence of lymphoid tissue 1.5 months. He had been delivered to the hospi-
47. A 45-year-old woman complains of tal with complaints of edemas all over his body,
paroxysmal intolerable facial pain on the left low urine output, and headache. Clinical uri-
with attacks that last for 1-2 minutes. Attacks nalysis: proteins - 7.1 g/L, leukocytes - 1-2 in the
are provoked by chewing. The disease onset vision field, erythrocytes - 3-4 in the vision fi-
was two month ago after overexposure to cold. eld. During the course of treatment the edemas
Objectively: pain at the exit points of the tri- gradually dissipated, headache abated, diuresis
geminal nerve on the left. Touching near the normalized. Daily urine proteins - 3 g/L. Bi-
wing of nose on the left induces new pain attack ochemical blood test: total protein - 43.2 g/L,
Krok 2 Medicine (англомовний варiант, iноземнi студенти) 2018 рiк 7

urea - 5.2 mmol/L, cholesterol - 9.2 mmol/L. suprapubic area is without alterations, external
What glomerulonephritis syndrome is the most genitalia are non-pathologic. On rectal investi-
likely to be present in the patient? gation: prostate is not enlarged, painless, has
normal structure. Cystoscopy revealed no
A. Nephrotic alterations. What is the most likely diagnosis?
B. Nephritic
C. Isolated urinary A. Renal carcinoma
D. Hematuric B. Bladder tuberculosis
E. Mixed C. Varicocele
D. Dystopic kidney
52. The mother of a 3-month-old child came to E. Necrotic papillitis
a family doctor with complaints of her child
being physically underdeveloped and suffering 56. A 36-year-old man has been complaini-
from cough attacks and dyspnea. Anamnesis: ng of marked weakness, low appetite, and
the child is the result of the second full-term nausea for the last year, year and a half. Wi-
pregnancy with the risk of miscarriage (the first thin the last year he has lost 10 kg of body
child died of pulmonary pathology at the age of mass. The skin is darkened, especially on his
4 months, according to the mother). Body mass face, neck, and arms. Skin folds and nipples
at birth is 2500 g. Cough attacks were observed are hyperpigmented, there are pigment spots
from the first days of life, twice the child was on the patient’s inner thighs. Pulse is 60/min.,
treated for bronchitis. Considering the severi- blood pressure is 80/50 mm Hg. What is the
ty of the child’s condition the doctor made the provisional diagnosis?
referral for hospitalization. What diagnosis was
most likely stated in the referral? A. Adrenal insufficiency
B. Diabetes mellitus
A. Mucoviscidosis (Cystic fibrosis) C. Chronic gastritis
B. Acute obstructive bronchitis D. Hemochromatosis
C. Recurrent obstructive bronchitis E. Cholestatic hepatitis
D. Pertussis
E. Acute obstructive pneumonia 57. A 5-year-old child that contacts with vi-
ral hepatitis in the kindergarten presents wi-
53. A 10-year-old girl complains of th increased body temperature up to 38o C ,
stomachache that appears and intensifies after weakness, low appetite, single case of vomiti-
she eats rough or spicy food, sour eructation, ng, dull pain in the subcostal area on the right.
heartburn, frequent constipations, headaches, The child is provisionally diagnosed with viral
irritability. She has been presenting with these hepatitis. What examination would be the most
signs for 12 months. Her meals are irregular and informative for diagnosis confirmation?
consist of dry food. Objectively her diet is suffi-
cient in calories. The tongue is moist with white A. ALT activity in blood
coating near the root. The abdomen is soft and B. Urine analysis for bile pigments
painful in the epigastrium. What method would C. Feces analysis for stercobilin
be optimal for diagnosis-making in this case? D. Blood test for bilirubin
E. Thymol turbidity test
A. Esophagogastroduodenoscopy
B. Intragastric pH-metry 58. A patient with trauma of the lower thi-
C. Fractional gastric analysis (Fractional test rd of the forearm volar surface caused by a
glass shard came to a first-aid center. Objecti-
meals) vely: flexion of the IV and V fingers is impai-
D. Phase-contrast X-ray imaging red, sensitivity of the inner dorsal and palmar
E. Biochemical blood test surfaces of the hand and IV finger is decreased.
54. A 23-year-old patient had taken 1 g of aspi- What nerve is damaged?
rin to treat acute respiratory infection. After A. Ulnar
that he developed an asthmatic fit with labored B. Radial
expiration that was arrested by introduction of C. Median
aminophylline. The patient’s medical history is D. Musculocutaneous
not burdened with allergies. The patient has E. Axillary
undergone two surgeries for nasal polyposis in
the past. What diagnosis is most likely? 59. A man diagnosed with closed-angle
glaucoma, grade IIa, of the right eye is regi-
A. Aspirin-induced asthma stered for regular medical check-ups. In the
B. Atopic bronchial asthma evening an acute glaucoma attack occurred in
C. Infectious allergic bronchial asthma his right eye; an ambulance was called. What
D. Exercise-induced asthma emergency aid would be optimal in this case?
E. Symptomatic bronchospasm
55. A 50-year-old patient was delivered to a
hospital with complaints of blood traces in
urine. Urination is painless and undisturbed.
Macrohematuria had been observed for 3
days. Objectively: kidneys cannot be palpated,
Krok 2 Medicine (англомовний варiант, iноземнi студенти) 2018 рiк 8

A. Pilocarpine, Diacarb (Acetazolamide), lytic A. Pertussis


mixture B. Parainfluenza
B. Atropine eye drops C. Congenital stridor
C. Antibiotic eye drops, broad-spectrum D. Respiratory syncytial infection
D. Sulfacetamide sodium eye drops E. Adenovirus infection
E. Dexamethasone eye drops
64. A man works in casting of nonferrous
60. A 58-year-old patient complains of pain in metals and alloys for 12 years. In the air of
the lower left extremity, which aggravates duri- working area there was registered high content
ng walking, and sensation of cold and numbness of heavy metals, carbon monoxide, and ni-
in the both feet. The patient has been sufferi- trogen. During periodic health examination
ng from this condition for 6 years. Objectively: the patient presents with asthenovegetati-
the skin is pale and dry, with hyperkeratosis. ve syndrome, sharp pains in the stomach,
On the left shin hair is scarce. Pulse cannot be constipations, pain in the hepatic area. In uri-
detected over the pedal and popliteal arteries ne: aminolevulinic acid and coproporphyrin
and is weakened over the femoral artery. On are detected. In blood: reticulocytosis, low
the right limb pulsation of the popliteal artery hemoglobin level. Such intoxication is caused
is retained. What is the most likely diagnosis? by:
A. Atherosclerosis obliterans of the lower A. Lead and lead salts
extremities B. Tin
B. Obliterating endarteritis C. Carbon monoxide
C. Femoral artery thrombosis D. Nitric oxide
D. Raynaud’s disease E. Zinc
E. Buerger’s disease (thromboangiitis obli-
terans) 65. Due to introduction of a new treatment
method, average duration of therapy in
61. A 22-year-old woman, gravida 1, para 0 the experimental group was 12.3±0.2 days
arrived with complaints of sharply painful compared to 15.4±0.4 days in the control group
contractions that occur every 4-5 minutes and that was treated by the old method. What
last for 25-30 seconds. Amniotic fluid did calculations should be made to estimate the
not burst. The fetus is in transverse lie, fetal statistical significance of the difference in the
heartbeats are not affected. Contraction ring is results?
acutely painful, located obliquely at the umbili-
cus. What is the most likely diagnosis? A. T-test (Student’s t-distribution)
B. Sign test (Z-test)
A. Impending uterine rupture C. Matching factor (chi-squared test)
B. Uterine tetany D. Wilcoxon T-test
C. Excessive uterine activity during labor E. Kolmogorov-Smirnov test
D. Discoordinated labor
E. Uterine rupture 66. Establishments participating in medical
examinations include: medical and preventi-
62. During regular check-up the doctor exami- ve treatment facilities, hygiene and preventi-
nes a young woman, a student, with height of ve treatment facilities, sociomedical expert
162 cm and weight of 59 kg. She complains committees, Ministry of Defence medical
that in the evening she becomes unable to see committees, Ministry of Domestic Affairs
clearly the objects around her. Objectively her medical committees, forensic medicine agency,
skin is dry, presents with hyperkeratosis. Her etc. Specify what service deals with sociomedi-
daily ration has the following vitamin content: cal assessment of temporary disability:
vitamin A - 0.5 mg, vitamin B1 - 2.0 mg, vitamin
B2 - 2.5 mg, vitamin B6 - 2 mg, vitamin C - 70 A. Medical and preventive treatment facilities
mg. Make the diagnosis: B. Hygiene and preventive treatment facilities
C. Sociomedical expert committees
A. A-hypovitaminosis D. Ministry of Defence medical committees
B. B1 -hypovitaminosis E. Ministry of Domestic Affairs medical commi-
C. B2 -hypovitaminosis ttees
D. B6 -hypovitaminosis
E. C -hypovitaminosis 67. A 14-year-old boy presents with moderate
bronchial asthma in its exacerbation period.
63. An infant is 2.5 months old. The onset of the What drug should be prescribed to stop an
disease was gradual, the child had normal body acute attack of expiratory dyspnea?
temperature but presented with slight cough.
Within a week the cough intensified, especially A. Salbutamol
at night; on the 12th day the child developed B. Cromolyn sodium (Cromoglicic acid)
cough fits occurring up to 20 times per day and C. Dexamethasone
followed by vomiting. There was one instance D. Lasolvan (Ambroxol)
of respiratory arrest. Make the diagnosis: E. Strophanthine (cardiac glycosides)
68. A newborn with gestational age of 31
weeks presents with hypotonia and depressed
consciousness. Hematocrit is 35%, general
Krok 2 Medicine (англомовний варiант, iноземнi студенти) 2018 рiк 9

cerebrospinal fluid analysis shows increased should the infant be put to the breast?
content of erythrocytes and protein, and low
glucose. These data correspond with the clini- A. In the delivery room
cal presentation of: B. After 12 hours
C. After 2 hours
A. Intracranial hemorrhage D. On the 2nd day
B. Meningitis E. On the 3rd day
C. Sepsis
D. Anemia 74. During assessment of work conditions at the
E. Prenatal infection factory manufacturing mercury thermometers,
the content of mercury vapors in the air of the
69. A 16-year-old girl has primary amenorrhea, working area is revealed to exceed the maxi-
no pubic hair growth, normally developed mum concentration limit. Specify the main
mammary glands; her genotype is 46 ХY; uterus pathway of human body exposure to mercury:
and vagina are absent. What is your diagnosis?
A. Respiratory organs
A. Testicular feminization syndrome B. Intact skin
B. Mayer-Rokitansky-Kuster-Hauser syndrome C. Damaged skin
C. Cushing syndrome D. Gastrointestinal tract
D. Sheehan syndrome E. Mucous tunics
E. Cushing disease
75. In the process of hiring, a prospective
70. 6 hours ago the waters of a 30-year-old employee has undergone preventive medical
gravida 1, para 0, burst; her preliminary peri- examination and was declared fit to work in
od was pathologic and lasted for over 2 days; this manufacturing environment. What type of
the term of pregnancy is 39 weeks. No labor preventive medical examination was it?
activity is observed. Fetal head presents above
the pelvic inlet. Fetal heartbeats are 142/min., A. Preliminary
clear and rhytmic. On vaginal examination the B. Scheduled
uterine cervix is not dilated. What further tacti- C. Periodical
cs should the doctor choose? D. Specific
E. Comprehensive
A. Perform cesarean section
B. Induce cervical dilation with prostaglandins 76. A 47-year-old woman came to the admissi-
C. Stimulate the labor with oxytocin on room with complaints of general weakness,
D. Wait for the onset of spontaneous labor dizziness, vomiting with blood clots. Conditi-
E. Prolong the pregnancy, while providing on onset was 3 hours ago. The patient has no
antibacterial treatment preceding illnesses. Blood pressure is 90/60
mm Hg, pulse is 106/min., of poor volume.
71. Mother of an 8-year-old girl complains that The abdomen is soft, with mild tenderness
the child is too short and has excessive body in the epigastrium. Blood test: erythrocytes -
weight. Objectively: obesity with fat deposits on 2.1·1012 /L, Нb- 70 g/L, hematocrit - 28%. What
the torso and face (round moon-like face), acne, tactics should the doctor on duty choose?
striae on the thighs and lower abdomen, hirsuti-
sm. What hormone can cause such symptoms, A. Consult the surgeon
when in excess? B. Refer the patient to the family doctor
C. Give spasmolytics
A. Cortisol D. Perform gastric lavage
B. Thyroxin E. Make an appointment for colonoscopy
C. Testosterone
D. Insulin 77. A 23-year-old woman came the the
E. Glucagon gynecologist with complaints of blood smears
from her genital tracts that have been observed
72. A 9-year-old girl complains of fever up for a long time. Her menstruation has been
to 38.5o C , headache, inertness, weakness, loss delayed for 8 weeks. Examination shows the
of appetite, stomachache, and frequent pai- uterine body to be enlarged up to 14 weeks of
nful urination. Provisional diagnosis of acute pregnancy. US detected a vesicular mole. What
pyelonephritis is made. Clinical urine analysis: tactics should the doctor choose?
specific gravity - 1016, no protein, leukocytes
- 10-15 in the vision field. What investigation A. Curettage of the uterine cavity
method can verify the diagnosis of urinary tract B. Hormonal treatment
infection? C. Hemostatic treatment
D. Supravaginal uterine amputation
A. Bacteriological inoculation of urine E. Uterectomy
B. Rehberg test (creatinine clearance test)
C. Zymnytsky test (density measurement of 78. A 52-year-old patient complains of pain
daily diuresis) in the right part of her chest, dyspnea, cough
D. Complete blood count with large amounts of foamy sputum emitti-
E. Clinical urine analyses, dynamic testing ng foul smell and resembling ”meat slops”.
Objectively: the patient’s condition is grave,
73. A newborn has Apgar score of 9. When cyanosis is observed, breathing rate is 31/min.,
Krok 2 Medicine (англомовний варiант, iноземнi студенти) 2018 рiк 10

percussion sound above the right lung is 83. A population of a small town often
shortened, auscultation revealed various moi- presents with registered cases of juvenile
st rales (crackles). What is the most likely di- cardiomyopathy, atherosclerosis, hypertensi-
agnosis? on, endocrinopathy, chronic dermatitis, and
arthralgia - signs of Keshan disease. What is
A. Lung gangrene the most likely cause of this pathology?
B. Lung abscess
C. Empyema of pleura A. Selenium deficiency in the environment
D. Multiple bronchiectasis B. Excessive iron in the environment
E. Chronic pneumonia C. Strontium deficiency in the environment
D. Excessive zinc in the environment
79. A man complains of sore throat on the left, E. Excessive manganese in the environment
pain in his left ear, to up to 39o C , and nasal
sound of his voice. Disease onset was 5 days 84. In 2 hours after a traffic accident a 28-year-
ago. Marked trismus and increased salivation old man in grave condition was delivered to a
are observed. The head tilts to the left shoulder. hospital. The patient complains of abdominal
Left side of the soft palate presents with swelli- pain. He received a blow to the abdomen wi-
ng, hyperemia, and infiltration. Retromandi- th the steering wheel. Objective examination
bular lymph nodes on the left are acutely pai- revealed the following: the abdomen does not
nful on palpation. Otoscopy results are normal. participate in respiration, is tense and acutely
Make the diagnosis: painful on palpation; abdominal guarding is
present, peritoneal irritation signs are positive,
A. Left-sided peritonsillar abscess hepatic dullness is absent. BP is 90/60 mm Hg,
B. Retropharyngeal abscess heart rate is 120/min. What further treatment
C. Parapharyngeal phlegmon tactics should be chosen?
D. Peritonsillitis on the left
E. Cervical phlegmon on the left A. Laparotomy
B. Laparoscopy
80. A patient has the second and third degree C. Cold to the abdomen
burns of the 15% of the body surface. On the D. Abdominal X-ray
20th day after the trauma the patient presents E. Laparocentesis
with sharp increase of body temperature,
general weakness, rapid vesicular respiration; 85. A 48-year-old woman was arrived to the
facial features are sharpened, BP is 90/50 mm surgical unit with wounds in her thigh. On
Hg, heart rate is 112/min. What complication is examination the wound surface has dirty-gray
it? coating with unpleasant sweet smell. Wound
content resembles raspberry jelly. Skin ti-
A. Sepsis ssues around the wound are glossy and turgid.
B. Pneumonia Palpation reveals moderate crepitation in the
C. Acute intoxication tissues. What microflora is the most likely to
D. Purulent bronchitis cause such inflammation?
E. Anaerobic infection
A. Anaerobic clostridial
81. A patient in the state of clinical death is bei- B. Anaerobic non-clostridial
ng resuscitated through mouth-to-mouth artifi- C. Streptococci
cial pulmonary ventilation and external cardiac D. Staphylococci
massage. A doctor noticed that air does not E. Blue pus bacillus
flow into the patient’s airways and his head and
torso are positioned at the same level. Why 86. The gynecology unit received a patient
is artificial respiration ineffective in the given with uterine bleeding that started 6 hours
case? after induced abortion at the term of 11-12
weeks. Objectively the skin is pale, pulse is
A. Tongue retraction 100/min., blood pressure is 100/70 mm Hg. On
B. Low breathing volume vaginal examination the uterus is painless, its
C. External cardiac massage enlargement corresponds to the 10th week of
D. Probe is absent from the stomach pregnancy; uterine cervix is dilated enough to
E. The patient’s mouth is too small let in one finger, there are fragments of the ferti-
lized ovum. What actions should be taken next:
82. An employee has been sick for 4 months,
further treatment is necessary, the patient is A. Urgent repeated curettage of the uterine
unable to work. Who is authorized to provide cavity
further disability examination of this patient? B. Uterotonic drugs
C. Treatment for acute anemia
A. Sociomedical expert committee D. Antibacterial agents
B. Medical consultative board E. Prescribe rest and continue to monitor the
C. Physician in charge and the head of the patient’s condition
department
D. Chief physician of a medical facility 87. A woman came to the general practitioner
E. Deputy chief responsible for disability exami- with complaints of fatigability, significant wei-
nation ght loss, weakness, and loss of appetite. She has
been presenting with amenorrhea for the last
Krok 2 Medicine (англомовний варiант, iноземнi студенти) 2018 рiк 11

8 month. One year ago she gave birth to a live A. Undo the sutures, drain the wound, and
full-term child. Blood loss during delivery was prescribe antibiotics
2 liters. The woman received blood transfusion B. Prescribe broad spectrum antibiotics and
and blood components. What is the most likely hormonal agents
diagnosis? C. Administer antibiotics intraosseously and
hypothermia locally
A. Sheehan’s syndrome (postpartum hypopitui- D. Inject antibiotics into the area surrounding
tarism) the wound, prescribe spasmolytics and analgesi-
B. Stein-Leventhal syndrome (polycystic ovary) cs
C. Turner’s syndrome E. Remove the fixation, prescribe sulfanilamides
D. Homologous blood syndrome
E. Somatoform autonomic dysfunction 92. A woman with blood group B(III) Rh(+)
gave birth to a full-term healthy boy. Exami-
88. To assess the effectiveness of medical nation on the 3rd day of the infant’s life shows
technologies and determine the power and di- him to have icteric tint to his skin. The chi-
rection of their effect on the public health indi- ld has no problems with suckling, sleep is
cators, the research was conducted to study the nondisturbed. The abdomen is soft, the li-
immunization rate of children and measles inci- ver protrudes by 2 cm from under the costal
dence rate by district. What method of statisti- margin. Complete blood count: hemoglobin -
cal analysis should be applied in this case? 200 g/L, erythrocytes - 5.5 · 1012 /L, total bi-
lirubin - 62 mcmol/L, indirect bilirubin - 52
A. Calculation of correlation coefficient mcmol/L. What condition can be suspected?
B. Calculation of morbidity index among the
nonvaccinated A. Physiologic jaundice
C. Calculation of coefficient of agreement B. Congenital hepatitis
D. Calculation of standardized ratio C. Hemolytic disease of the newborn due to Rh
E. Calculation of statistical significance of the incompatibility
difference between two estimates D. Biliary atresia
E. Hemolytic disease of the newborn due to
89. A 22-year-old woman complains of itchi- ABO incompatibility
ng and profuse discharge from her genital
tracts. The condition developed 10 days ago 93. A 45-year-old man developed constricting
after a sexual contact. Bacterioscopy of a di- retrosternal pain that occurs during walks at
scharge sample detected trichomonads. What the distance of 200 m. Objectively heart rate
drug should be prescribed for treatment in this is 80/min., BP is 160/90 mm Hg. During cardi-
case? opulmonary exercise test at 50 W there is a
depression of S-T segment by 3 mm below the
A. Metronidazole isoline in V3-V4. What is the provisional di-
B. Ampicillin agnosis?
C. Erythromycin
D. Zovirax (Acyclovir) A. Exertional angina pectoris, functional class
E. Valcyclovir III
B. Exertional angina pectoris, functional class
90. A 30-year-old patient was hospitalized in an IV
intensive care unit with a diagnosis of multi- C. Exertional angina pectoris, functional class II
ple bee stings. Skin is pale and covered wi- D. Somatoform autonomic dysfunction,
th cold sweat. Pulse can be palpated only at hypertension type
the carotid arteries and is 110/min.; breathing E. Alcoholic myocardiodystrophy
rate is 24/min., rhytmical, weakened. What drug
must be administered immediately? 94. A multigravida on the 38th week of her
pregnancy complains of increased BP up to
A. Epinephrine hydrochloride 140/90 mm Hg, edema of the shins for 2 weeks.
B. Prednisolone In the last month she gained 3.5 kg of weight.
C. Norepinephrine hydrochloride Urine analysis: protein - 0.033 g/L. Make the
D. Dopamine diagnosis:
E. Tavegyl (Clemastine)
A. Mild preeclampsia
91. A 46-year-old woman has been hospitalized B. Moderate preeclampsia
with open fracture of the left thigh in its middle C. Pregnancy hypertension
third. She underwent the surgery - fixation wi- D. Severe preeclampsia
th extraosseous osteosynthesis plates. On the E. Pregnancy edema
4th day after the surgery she developed pain in
the wound, body temperature rose over 39o C . 95. For the last 15 years a 48-year-old pati-
What measures should be taken in this case? ent has been working at the factory producing
synthetic resins. Lately he has been complai-
ning of significant general fatigue, headaches,
frequent urination (predominantly during the
day), red color of urine. What complication
of benzene nitrocompounds poisoning can be
suspected?
Krok 2 Medicine (англомовний варiант, iноземнi студенти) 2018 рiк 12

A. Anamnestic (history-taking)
A. Malignant tumor of the urinary bladder B. Immediate registration
B. Chronic cystitis C. Immediate examination
C. Chronic prostatitis D. Doing extracts
D. Acute glomerulonephritis E. -
E. Chronic pyelonephritis
100. A 30-year-old woman complains of
96. A 44-year-old patient with postinfarcti- increased body weight and problems with
on cardiosclerosis presents with frequent physical exertion. Her parents are of increased
heart rate disorders and lower extremity body weight as well; typical meals in their fami-
edema. Objectively: Ps- 95/min., irregular, 10- ly are high in fats and carbohydrates. Objecti-
12 extrasystoles per minute. BP- 135/90 mm Hg. vely her blood pressure is 135/80 mm Hg, pulse
The 1st heart sound at the apex is weakened. is 89/min., weight is 87 kg, height is 165 cm.
Pulmonary respiration is rough. The liver is The patient’s skin is clear, with even distributi-
enlarged +2 cm. ECG: irregular sinus rhythm, on of subcutaneous fat; the thyroid gland is
heart rate - 95/min., frequent polytopic ventri- not enlarged; there are no menstrual cycle di-
cular extrasystoles. What antiarrhythmic drug sturbances. What obesity prevention methods
is advisable in this case for treatment and would be the most advisable in this case?
prevention of extrasystole?
A. Dietary treatment, graduated exercise
A. Amiodarone B. Intensive training regimen
B. Lidocaine C. Gastroplasty or gastrojejunal shunt
C. Mexiletine D. Inhibitors of gastrointestinal lipases
D. Quinidine E. Anorectic drugs
E. Novocainamide (Procainamide)
101. A 43-year-old woman complains of pain in
97. A 60-year-old woman started feeling the lumbar area, which irradiates to her left leg
weakness, vertigo, rapid fatigability during the and aggravates on movement, and sensation
last year. Recently she has developed dyspnea of numbness in this leg. Objectively palpati-
and paresthesia observed. Objectively: skin and on of her shin and thigh is painful, there are
mucous membranes are pale and icteric. Li- painful stretch symptoms of on the left and
ngual papillae are smoothed out. Liver and gastrocnemius cramps. There is no sensory loss
spleen are at the edge of costal arch. Blood test: or weakening of reflex responses. Make the di-
Hb- 70 g/L, erythrocytes - 1.7 · 1012 /L, blood agnosis:
color index - 1.2, macrocytes. What drug can be
prescribed on pathogenetic grounds? A. Vertebrogenous lumbar ischialgia on the left
B. Vertebrogenous radicular syndrome of L5-S1
A. Vitamin B12 on the left
B. Vitamin B6 C. Left-sided coxitis
C. Ascorbic acid D. Endarteritis of the lower extremities
D. Iron preparations E. Spinal stroke
E. Vitamin B1
102. A 3-day-old infant with hyperbili-
98. After excessive consumption of fatty food rubinemia (428 mcmol/L) developed di-
a 60-year-old woman suddenly developed pain sturbances manifesting as periodical excitati-
in her right subcostal area, nausea, bile vomi- on and convulsions against the background of
ting, sharp bitter taste in her mouth. In 2 days inertness, hypotension, hypodynamia, and inhi-
she developed jaundice, her urine darkened. bition of unconditioned reflexes, convergent
Objectively: sclera and skin are icteric, the strabismus, rotational nystagmus, and setting-
abdomen is distended, the liver is enlarged by sun eye phenomenon. What is the most likely
3 cm, soft and painful on palpation, Ortner’s, cause of such symptoms?
Murphy’s, Kehr’s, Zakharyin’s, Mayo-Robson’s
signs are positive. What diagnostic technique A. Bilirubin encephalopathy
should be used in the first place to confirm the B. Craniocerebral injury
diagnosis? C. Brain tumor
D. Hydrocephalus
A. Ultrasound of the gallbladder and bile duct E. Infantile cerebral paralysis
B. Fibrogastroduodenoscopy
C. Abdominal X-ray 103. A woman undergoing in-patient treatment
D. Radionuclide scanning of the liver and for viral hepatitis type B developed headache,
gallbladder nausea, recurrent vomiting, memory lapses,
E. Laparoscopy flapping tremor of her hands, rapid pulse.
Sweet smell from the mouth is detected. Body
99. The objective of a statistical research was to temperature is 37.6o C , heart rate is 89/min.
find out to what extent the population peruses What complication developed in the patient?
the available medical services. For this purpose
300 residents of the area were interviewed.
Information was collected by means of a speci-
al questionnaire. What method of collecting
information was used by the researchers?
Krok 2 Medicine (англомовний варiант, iноземнi студенти) 2018 рiк 13

A. Acute liver failure 108. A 38-year-old patient has been delivered


B. Ischemic stroke by an ambulance to a surgical department with
C. Gastrointestinal hemorrhage complaints of general weakness, indisposition,
D. Hypoglycemic shock black stool. On examination the patient is pale,
E. Meningoencephalitis there are dotted hemorrhages on the skin of his
torso and extremities. On digital investigation
104. A patient with chronic pancreatitis there are black feces on the glove. Blood test:
complains of diarrhea occurring up to 5 ti- Hb- 108 g/L, thrombocytopenia. Anamnesis
mes per day (no blood traces), loss of body states that similar condition was observed 1
weight, abdominal distention, dryness of skin, year ago. Make the diagnosis:
loss of hair, thirst, bleeding gums, convulsions.
Complete blood count: leukocytes - 5.8 · 109 /L; A. Thrombocytopenic purpura
Hb- 86 g/L; ESR- 15 mm/g; Blood protein B. Hemophilia
test: total protein - 48 g/L; albumins - 28 g/L. C. Bleeding from an ulcer
What indicators of coprological analysis would D. Rectal tumor
accompany this syndrom? E. Nonspecific ulcerative colitis
A. Steatorrhea, creatorrhea 109. A 35-year-old patient developed an epi-
B. Large amount of mucus, amylorrhea leptic attack with tonoclonic spasms that lasted
C. Large amount of starch grains and cellulose for 3 minutes. After the attack the patient
D. Gas bubbles, acid reaction fell asleep but in 5 minutes the second attack
E. Large numbers of iodinophilous microbes occurred. The first step of emergency aid would
be to:
105. A 43-year-old woman complains of persi-
stent stomachache with recurrent pain attacks, A. Ensure patency of airways
nausea, repeated vomiting with stagnant bowel B. Take blood from the vein for analysis
content, abdominal distension, and flatulence. C. Introduce diazepam intravenously
She has been presenting with these signs for D. Prescribe antiepileptic drugs
7 hours. Pulse is 116/min. The tongue is dry E. Administer chloral hydrate via enema
and brown. The abdomen is symmetrically
distemded, soft, painful. Percussion reveals 110. A young woman suffering from seborrhea
tympanitis. On auscultation there are bowel oleosa has numerous light-brown and whi-
sounds with metallic overtone, splashing, and te spots on the skin of her torso and
dripping. Make the diagnosis: shoulders. The spots have clear margins, branny
desquamation, no itching. What provisional di-
A. Acute intestinal obstruction agnosis can be made?
B. Acute necrotizing pancreatitis
C. Acute destructive cholecystitis A. Pityriasis versicolor
D. Acute erosive gastritis B. Torso dermatophytosis
E. Acute nonspecific colitis C. Seborrheic dermatitis
D. Pityriasis rosea
106. After overexposure to cold a 45-year-old E. Vitiligo
woman developed acute pain in her suprapubic
and lumbar areas during urination, sharp pains 111. A patient is 28 years old. He has been
at the end of urination, false urges to urinate. suffering from mental disorder since he was
Urine is turbid with blood streaks. The doctor 22. His current condition has changed acutely:
suspects urinary tract infection. What results of for 3 days the patient has been refusing to
laboratory analysis would be the most indicati- leave his home. He claims that there is a
ve of such infection? ”telepathy” occurring between him and other
people, through which he receives ”thoughts
A. Leukocyturia, gross hematuria of strangers” and transmits his own thoughts
B. Gross hematuria for everyone to hear. He thinks his thoughts
C. Increased blood creatinine and blood urea and actions are manipulated through this
D. Daily proteinuria under 3.0 ”telepathy”. Make the preliminary diagnosis:
E. Daily proteinuria over 3.0
A. Paranoid schizophrenia
107. A 15-year-old adolescent girl came the B. Depressive episode
the gynecologist with complaints of painful C. Catatonic episode
menstruations that are accompanied by nausea, D. Organic delirium
vomiting, and dizziness. Her menarche was at E. Acute reaction to stress
12. Menstruations became painful since she was
14, remain regular. What treatment should be 112. A 3-year-old child presents with sharp
prescribed in this case? deterioration of his general condition. He has
a history of purulent otitis. His temperature is
A. Analgesics, antispasmodics, antiprostaglandi- now 38.5o C . The left leg is pressed to the torso,
ne therapy active movements are absent, the lower thi-
B. Antiinflammatory treatment only rd of the thigh and knee joint are thickened,
C. Antihemorrhagic agents hyperemic, with localized fever. Axial load
D. Antiandrogen therapy leads to acute discomfort of the patient. What
E. Vitamin supplements is the most likely diagnosis?
Krok 2 Medicine (англомовний варiант, iноземнi студенти) 2018 рiк 14

A. Epiphyseal osteomyelitis on the left of the lungs. Heart sounds are weakened, the II
B. Left hip fracture heart sound is accentuated over the pulmonary
C. Rheumatoid arthritis artery. The liver is +3 cm. What complicated the
D. Osteogenic sarcoma clinical course of COPD in this patient?
E. Hygroma of the knee
A. Chronic pulmonary heart
113. A 19-year-old student was urgently hospi- B. Pulmonary embolism
talized due to marked dyspnea and chest pain C. Acute left ventricular failure
on the left. Her body temperature is 38.8o C . D. Diffuse pneumosclerosis
She has been presenting with these signs for E. Community-acquired pneumonia
3 days. Respiratory rate is 42/min., shallow.
Percussion sound is dull to the left from the 117. A burn victim with flame burns of the
center of the scapula, no respiration can be IIIA-B and IV degrees on his face, neck, and
auscultated. The left heart border is displaced anterior surface of the thorax was brought into
outwards by 3 cm. Embryocardia and heart rate the admission room. The hairs in his nostri-
of 110/min are observed. Palpation of the right ls are burnt, his labial and glossal mucosa are
subcostal area is painful. What urgent measures gray-white. The voice is hoarse; respirations are
should be taken in this case? frequent and shallow; the patient has trumpet-
like cough that produces soot-streaked sputum.
A. Urgent thoracocentesis The signs of respiratory failure were progressi-
B. Prescription of penicillin antibiotics ng, while the patient was being transported into
C. Administration of furosemide the intensive care unit. What emergency care
D. Administration of cardiac glycosides must be provided to this patient?
E. Referral into thoracic surgery unit
A. Intubation of the trachea and mechanical
114. A 20-year-old woman, gravida 2, para 1 has ventilation
been in labor for 4 hours. Her condition is sati- B. Tracheostomy
sfactory. Moderately painful contractions occur C. Administration of bronchial spasmolytics
every 3 minutes and last for 35-40 seconds. D. Administration of respiratory analeptics
The waters have not burst yet. The fetus is E. Inhalation of moisturized oxygen
in longitudinal position. Fetal heartbeats are
136/min., clear and rhytmic. Major segment of 118. A 72-year-old man with pnaumonia
the fetal head is engaged to the pelvic inlet. complains of marked dyspnea, chest pain,
Vaginal examination shows smooth cervix of severe cough with expectoration, to is 39.5-
6cm, amniotic sac is intact, sagittal suture is in 40o C , no urination for a whole day. Objecti-
the left oblique diameter, occipital fontanel is vely the patient is conscious. Respiratory rate
on the right near the symphysis pubis. What is 36/min. Over the right lower pulmonary lobe
stage of the labor is it? percussion sound is dull; on auscultation there
is bronchial respiration and numerous moist
A. Active phase of the first stage of normal labor crackles. Blood pressure is 80/60 mm Hg. Heart
B. Latent phase of the first stage of normal labor rate is 120/min. Heart sounds are muffled, there
C. The second stage of normal labor is tachycardia. What tactics should the family
D. Precursors of childbirth doctor choose in the management of this pati-
E. Preliminary stage ent?
115. A 1.5-month-old child on breastfeedi- A. Hospitalization into intensive care unit
ng presents from birth with daily vomiting, B. Outpatient treatment
irregular liquid foamy feces, and flatulence, C. Treatment in the day patient facility
which are resistant to antibacterial and probi- D. Hospitalization into pulmonology unit
otic therapy; no increase of body mass is E. Hospitalization into neurology unit
observed. The child’s condition improved,
when breastmilk was substituted. What 119. Estimation of community health level
pathology is it? involved analysis of a report on diseases regi-
stered among the population of district under
A. Lactase deficiency charge (reporting form 12). What index is
B. Intestinal lambliasis (Giardiasis) calculated based on this report?
C. Infectious enteritis
D. Drug-induced enteritis A. Prevalence
E. Functional dyspepsia B. Index of pathological affection
C. Index of morbidity with temporary disability
116. A 72-year-old man complains of lower D. Index of hospitalized morbidity
extremity edema, sensation of heaviness in the E. Index of basic non-epidemic morbidity
right subcostal area, dyspnea in rest. For over
25 years he has been suffering from COPD. 120. A 72-year-old woman suffers from diabetes
Objectively: orthopnea, jugular venous di- mellitus type II, concomitant diseases are stage
stention, diffuse cyanosis, acrocyanosis. Barrel II hypertension and stage IIB heart failure.
chest is observed, on percussion there is vesi- She takes metformin. Hypertensic crisis had
culotympanitic (bandbox) resonance, sharply occurred the day before, after which the patient
weakened vesicular respiration on both sides, developed extreme weakness, myalgias, thirst,
moist crepitant crackles in the lower segments dry mouth, polyuria. BP is 140/95 mm Hg, heart
rate is 98/min., no edemas or smell of acetone
Krok 2 Medicine (англомовний варiант, iноземнi студенти) 2018 рiк 15

detected. What measures should be taken to complex. BP is 185/105 mm Hg. What additi-
prevent development of comatose state in the onal examination would you recommend to the
patient? patient in the first place?
A. Stop metformin, prescribe short-acting A. Lumbar puncture
insulin B. Ventriculopuncture
B. Double the dosage of metformin C. Echoencephalography
C. Apply hypotonic solution of sodium chloride D. Rheoencephalography
D. Additionally prescribe long-acting insulin E. Electroencephalography
E. Prescribe glibenclamide
125. During regular examination of a 2-year-
121. The body of a 24-year-old woman with old boy, he presents with enlarged left ki-
probable signs of poisoning has been found on dney, painless on palpation. The right ki-
the street. Forensic medical examination was dney was undetectable on palpation. Excretory
requested by an investigator during examinati- urography shows no contrast on the right.
on of the site and the body. According to the Cytoscopy detected hemiatrophy of the urinary
Criminal Procedure Code currently in force in bladder trigone, the right ureteral orifice is not
Ukraine, forensic medical examination is requi- detected. What pathology is it?
red when it is necessary to determine the:
A. Agenesis of the right kidney
A. Cause of death B. Dystopia of the right kidney
B. Manner of death C. Hypoplasia of the right kidney
C. Time of death D. Agenesis of the right ureter
D. Mode of death E. Ectopic right ureteral orifice
E. Mechanism of death
126. A 5-year-old child has body temperature
122. It is the 3rd day after the normal term risen up to febrile numbers, suffers from
labor; the infant is rooming-in with the mother inertness, weakness. Examination revealed
and is on breastfeeding. Objectively: the hemorrhage on the skin of limbs and torso.
mother’s general condition is satisfactory. Enlargement of cervical and axillary lymph
Temperature is 36.4o C , heart rate is 80/min., nodes can be detected. The liver is 4 cm
BP is 120/80 mm Hg. Mammary glands are soft below the costal arch; the spleen is 6 cm
and painless; lactation is moderate, unrestri- below the costal arch. Blood test: erythrocytes
cted milk flow. The uterus is dense, the uterine - 2.3 · 1012 /L, Hb- 60 g/L, platelets - 40 · 109 /L,
fundus is located by 3 fingers width below the leukocytes - 32.8 · 109 /L, eosinophiles - 1%,
navel. Lochia are sanguino-serous, moderate band neutrophiles - 1%, segmented neutrophi-
in volume. Assess the dynamics of uterine les - 12%, lymphocytes - 46%, monocytes - 1%,
involution: blasts - 40%, Duke’s bleeding time test result is
9 min. What examination is necessary to make
A. Physiological involution the diagnosis?
B. Subinvolution
C. Lochiometra A. Myelogram (bone marrow biopsy)
D. Pathologic involution B. Lymph nodes biopsy
E. Hematometra C. Abdominal US
123. A 27-year-old man was hospitalized in D. Detection of hepatitis markers
severe condition 50 minutes after receiving a E. Analysis of dynamic platelet function
penetrating wound to the left side of the chest. 127. A 36-year-old man complains of marked
Objectively the patient is in a stupor, his skin is dyspnea and cardiac pain. He ascribes his di-
pale and acrocyanotic. Pulse is 120/min., of poor sease to the case of influenza that he had 2
volume, weak. Blood pressure is 80/40 mm Hg. weeks ago. Objectively he leans forward when
Heart sounds are muffled, cardiac borders are sitting. The face is swollen, cyanotic, cervical
markedly expanded. In the III intercostal area veins are swollen. Heart borders are extended
along the parasternal line on the left there is on the both sides, heart sounds are muffled,
a stab-incised wound. Plain chest X-ray shows heart rate = Ps = 118/min., BP is 90/60 mm Hg.
enlarged heart shadow with smoothed out wai- Blood test: ESR is 16 mm/hour. ECG shows
st of the heart, there is hemothorax on the left low voltage. X-ray shows trapezoidal cardiac
to the 5th rib. What contributes the most to the silhouette and signs of pulmonary congestion.
severity of the patient’s condition? Choose the treatment tactics:
A. Cardiac tamponade A. Pericardial puncture (pericardiocenthesis)
B. Acute heart failure B. Diuretics
C. Cardiac rhythm disturbance C. Antibiotics
D. Blood loss D. Pericardectomy
E. Hemothorax and acute respiratory failure E. Glucocorticosteroids
124. A 59-year-old patient suffering from 128. A 25-year-old patient is not married and
hypertension was delivered to the hospital has sexual relations with several partners. Duri-
with complaints of acute headache, nausea, ng the last 3 months he noticed small amount of
recurrent vomiting. On examination she mucoserous secretions produced from urethra.
presents with acute meningeal symptom
Krok 2 Medicine (англомовний варiант, iноземнi студенти) 2018 рiк 16

Subjectively: periodical itching or burning pain 132. A 28-year-old woman complains of nausea,
in urethra. Two months ago pain in knee join stomachache, pain in her tongue, and liquid
developed. Possibility of trauma or exposure feces. Three days ago she ate poorly salted
to cold is denied by the patient. During the pike caviar. Objectively her skin is pale, the
last week eye discomfort is noted - lacrimation tongue looks ”lacquered” (bald tongue). Pulse
and itching. What provisional diagnosis can be is 100/min., with muffled heart sounds and
suggested? systolic murmur over the cardiac apex. Blood
pressure is 95/50 mm Hg. The liver is enlarged
A. Reactive arthritis by 3 cm. Hemogram shows anemia, eosinophi-
B. Rheumatoid arthritis ls - 18%. Oval helminth eggs were detected in
C. Seasonal pollinosis feces. Make the provisional diagnosis:
D. Bacterial nonspecific urethral conjunctivitis
E. URTI with conjunctiva and joints affected A. Diphyllobothriasis
B. Trichinosis
129. A woman complains of weight gain, chi- C. Teniasis
lls, edema, xeroderma, somnolence, difficulti- D. Taeniarhynchosis
es with focusing. Objectively: height is 165 E. Ascaridiasis
cm; weight is 90 kg; body proportions are of
female type, to - 35,8o C , heart rate - 58/min., BP- 133. A 30-year-old woman complains of milk
105/60 mm Hg. Heart sounds are weakened, discharge from her breasts and no menstruati-
bradycardia is observed. Other internal organs on for the last 5 months. One physiologic
have no alterations. Thyroid gland cannot be childbirth was 4 years ago. There are no
palpated. Milk secretion from mammary glands maldevelopments of mammary glands. Bi-
is observed. Hormone test revealed increased manual examination revealed diminished
levels of thyroid-stimulating hormone (TSH) uterus and normal sized ovaries. MRI-
and prolactin, and decreased level of thyroxine

scan shows no brain pathologies. Thyroid-
( 4 ). What is the cause of obesity? stimulating hormone is within normal limits.
Serum prolactin is high. What is the most likely
A. Primary hypothyroidism diagnosis?
B. Secondary hypothyroidism
C. Prolactinoma A. Hyperprolactinemia
D. Hypopituitarism B. Hypothyroidism
E. Adiposogenital dystrophy C. Polycystic ovaries
D. Pituitary adenoma
130. A 54-year-old patient complains of E. Sheehan’s syndrome (postpartum hypopitui-
weakness, jaundice, itching skin. Disease onset tarism)
was 1.5 months ago: fever up to 39o C appeared
at first, with progressive jaundice developed 2 134. A 25-year-old woman during self-
weeks later. On hospitalisation jaundice was examination detected a tumor in the upper
severely progressed. Liver cannot be palpated. external quadrant of her right mammary gland.
Gallbladder is enlarged and painless. Blood bi- On palpation: painless, dense, mobile growth 2
lirubin is 190 mcmol/L (accounting mainly for cm in diameter is detected in the mammary
direct bilirubin). Stool is acholic. What is the gland; no changes in the peripheral lymph
most likely reason for jaundice in this patient? nodes are observed. On US of the mammary
glands: in the upper external quadrant of the ri-
A. Mechanical jaundice ght mammary gland there is a space-occupying
B. Hepatocellular jaundice lesion of increased echogenicity 21х18 mm in
C. Hemolytic jaundice size. The most likely diagnosis is:
D. Caroli syndrome
E. Gilbert’s syndrome A. Fibrous adenoma
B. Breast cyst
131. A 23-year-old man came to the surgeon C. Diffuse mastopathy
with complaints of pain, redness of the skin, D. Breast cancer
and swelling in the area of his proximal E. Mastitis
interphalangeal joint of the III finger on the
right hand. Six days ago he pricked his finger 135. A 45-year-old woman underwent one year
with a wire. Objectively the III finger on the ago mastectomy followed by chemo- and radi-
right hand is swollen, hyperemic, prominent in ation therapy. She now complains of dyspnea at
the projection of interphalangeal joint, sharply rest and temperature up to 37.2o C . Her general
painful on touch and during movements. Finger condition is severe, acrocyanosis is observed.
mobility is reduced. Fluctuation sign is present. The right side of her chest practically does not
What diagnosis corresponds to the given clini- participate in respiration. Percussion reveals
cal presentation? a dull sound below the 3rd rib; auscultation
detects acute weakening of the respiratory
A. Articular panaritium sounds. Pleural puncture on the right has yi-
B. Bone panaritium elded a large amount of hemorrhagic exudate.
C. Subcutaneous panaritium What complication has developed in the pati-
D. Pandactylitis ent?
E. Finger furuncle
Krok 2 Medicine (англомовний варiант, iноземнi студенти) 2018 рiк 17

A. Carcinomatous pleuritis A. Nephrotuberculosis


B. Acute pleural empyema B. Right renal cyst
C. Acute right-sided pleuropneumonia C. Right renal carcinoma
D. Right lung abscess D. Acute glomerulonephritis
E. Pulmonary embolism E. Chronic pyelonephritis
136. A 37-year-old man suffers from attacks 140. A 45-year-old woman came to the materni-
of unconsciousness, dyspnea during physical ty clinic with complaints of periodical pains in
exertion, periodical sensations of heart rate di- her mammary glands that start 1 day before
sorder. Father of the patient died suddenly at menstruation and stop after the menstruati-
the age of 45. Objectively: heart rate is 90/min., on begins. Palpation of the mammary glands
BP is 140/90 mm Hg. On heart US: ejection detects diffuse nodes predominantly in the
fraction - 55%, significant myocardium thi- upper outer quadrants. What is the most likely
ckening of the left ventricle and interventri- diagnosis?
cular septum. What drug should be prescribed
for the treatment? A. Fibrocystic mastopathy
B. Breast cancer
A. Bisoprolol C. Mastitis
B. Enalapril D. Hyperprolactinemia
C. Phenyhydinum (Nifedipine) E. Breast cyst
D. Hydrochlorothiazide
E. Furosemide 141. A woman complains of temperature
increase up to 39o C , sharp pains in her lower
137. A 60-year-old man complains of discomfort abdomen, and sanguinopurulent discharge
when swallowing solid food, which he has been from her genital tracts. From her case history it
observing for a month. He changed his diet is known that 6 days ago she underwent illegal
to semiliquid food products. At first the di- abortion. Objectively her blood pressure is
scomfort had abated but later it renewed despi- 100/60 mm Hg, pulse is 110/min. Abdominal ri-
te the change in the diet. The patient developed gidity, rebound tenderness (Bloomberg’s sign),
gaseous eructation and hoarse voice. What and painful palpation of the lower abdomen are
examination should be performed to clarify the observed. On bimanual examination the uterus
diagnosis? is enlarged up to 7 weeks of pregnancy, painful,
and soft; posterior vaginal fornix overhangs.
A. Esophagoscopy with biopsy Make the diagnosis:
B. Urea breath test for H. pylori
C. Diurnal variations of ECG parameters A. Pelviperitonitis
D. Esophageal pH monitoring B. Endometritis
E. Abdominal US C. Acute adnexitis
D. Pyosalpinx
138. A 22-day-old infant developed E. Metroendometritis
subcutaneous red nodes from 1.0 to 1.5 cm in
size on the scalp; later the nodes suppurated. 142. An 8-year-old girl with complaints of pai-
Temperature increased up to 37.7o C , intoxicati- nful urination, frequent low-volume urinati-
on symptoms appeared, regional lymph nodes on, and leukocyturia was diagnosed with acute
enlarged. Complete blood count: anemia, cystitis. 10 days before the disease onset she
leukocytosis, neutrocytosis, increased ESR. was treated by the gynecologist for acute vulvi-
What diagnosis will you make? tis. 5 days ago she presented with mild catarrhal
symptoms. Her mother ascribes the child’s di-
A. Pseudofurunculosis sease to her overexposure to cold. Specify the
B. Pemphigus most likely infection route:
C. Vesiculopustulosis
D. Scalp phlegmon A. Ascending
E. - B. Descending
C. Hematogenic
139. A 32-year-old woman complains of D. Contact
general fatigue, low-grade fever persisting E. Lymphogenic
for 4 months, lumbar pain, and dysuria.
Anamnesis includes frequent acute respi- 143. A 74-year-old patient visited a urologist
ratory diseases, overexposure to cold, low- with complaints of pain above the pubis and
calorie diet, a case of pulmonary tuberculosis inability to urinate for 8 hours. At home he
in childhood. Clinical urine analysis: pH- had taken antispasmodics and had a warm bath
4.8, leukocyturia, hematuria. Complete blood but no improvement occurred. Objectively:
count: leukocytosis, lymphocytosis, increased abdomen is soft and painful above the pubis;
ESR. Urography concludes: dilatation of renal dullness of percussion sound is observed above
pelvis and calyceal system of both kidneys, foci the pubis. Murphy’s (Pasternatski’s) punch sign
of calcification in the projection of right kidney is negative on the both sides. What condition
parenchyma. What is the most likely diagnosis? does the patient have?
Krok 2 Medicine (англомовний варiант, iноземнi студенти) 2018 рiк 18

A. Acute urinary retention A. Pericardiocentesis (pericardial puncture)


B. Paradoxal ischuria B. Cardiac glycosides intravenously
C. Chronic urinary retention C. Constant oxygenotherapy
D. Anuria D. Diuretics intravenously
E. Oliguria E. Antibiotics intravenously
144. An excavator operator with 20 years of 148. A patient has gradually lost consciousness.
work experience at the opencast ore mine The skin is pale and dry. There is smell of
undergoes regular medical examination. He ammonia from the mouth. Respirations are
presents with signs of pneumoconiosis. What deep and noisy. Heart sounds are muffled, peri-
type of pneumoconiosis is the most likely in cardial friction rub is present. Blood pressure
this case? is 180/130 mm Hg. Blood test: Нb- 80 g/L,
leukocytes - 12 · 109 /L, blood glucose - 6.4
A. Siderosis mmol/L, urea - 50 mmol/L, creatinine - 1200
B. Silicosis mcmol/L, blood osmolality - 350 mOsmol/kg
C. Anthracosis H2 O. No urinary excretion. Make the diagnosis:
D. Asbestosis
E. Silicatosis A. Uremic coma
B. Hyperglycemic coma
145. A 55-year-old woman came to a C. Acute renal failure
gynecologist with complaints of leukorrhea and D. Acute disturbance of cerebral circulation
bloody discharge from the vagina after 5 years E. Hyperosmolar coma
of menopause. Anamnesis states no pregnanci-
es. Bimanual examination: the uterus and uteri- 149. A 26-year-old woman has been undergoing
ne appendages are without changes. During treatment for community-acquired pneumonia
diagnostic curettage of the uterine cavity the for 10 days. It is known that her husband had
physician scraped off enchephaloid matter. been treated for drug addiction. Sequenti-
What is the most likely diagnosis in this case? al intravenous administration of Amoksi-
klav (Amoxicillin+Clavunate) + Levofloxacin
A. Endometrial carcinoma combination and vancomycin in the prescribed
B. Adenomyosis dosage was ineffective. Within the last two days
C. Subserous uterine myoma the patient’s dyspnea and intoxication acutely
D. Cervical carcinoma exacerbated, bilateral pulmonary infiltrates are
E. Ovarian carcinoma observed. What is the most likely cause of the
146. A 5-year-old boy complains of severe medication ineffectiveness?
dyspnea and sensation of lack of air. Objecti- A. HIV infection and pneumocystic pneumonia
vely the child assumes orthopneic position, B. Tuberculosis mycobacterium infection with
presents with marked peripheral cyanosis, development of tuberculosis
drumstick fingers, nail plates resembling a C. Idiopathic fibrosing alveolitis
”clock face”, the borders of cardiac dullness are D. Infection with polyresistant bacterial strains
bilaterally extended; coarse systolic murmur E. Cancer metastases in the pulmonary tissues
can be detected over the whole surface of the
heart and is especially pronounced in the II 150. A 35-year-old man complains of persisti-
intercostal area on the left near the sternum. ng enlargement of his peripheral lymph nodes
What disease can be characterized by such that cause him no discomfort. The case history
presentations? states that the first lymph nodes to enlarge
were cervical, supraclavicular, and axillary;
A. Fallot’s tetrad new groups of lymph nodes emerged. Objecti-
B. Dilated cardiomyopathy vely the lymph nodes are soft and elastic on
C. Defect of the interventricular septum palpation, enlarged, painless, not fixed to the
D. Primary bacterial endocarditis surrounding tissue. What examination method
E. Defect of the interatrial septum would be the most informative for early di-
147. A 10-year-old boy is delivered into a agnostics of this disease?
polytrauma unit after he received a blunt A. Needle biopsy
trauma of the thorax, having fallen from the bi- B. Magnetic resonance tomography
cycle. Upon hospitalization his blood pressure C. Radioisotope scanning of the skeleton
is 110/80 mm Hg, heart rate is 96/min. Chest D. Ultrasound
X-ray is noncontributive to the diagnosis. E. X-ray
Echocardiogram shows free liquid in the peri-
cardial cavity, in the amount of up to 100 151. A 32-year-old woman complains of body
ml. In an hour after the hospitalization the weight loss despite her increased appetite,
patient started to develop increasing signs nervousness, and tremor of the extremities.
of heart failure: jugular venous distention, Objectively: the skin is moist; the thyroid gland
decreased blood pressure down to 90/70 mm is diffusely enlarged, painless, soft, and mobi-
Hg, tachycardia up to 120/min. On auscultation le. Blood test: increased level of T3, T4, and
muffled heart sounds. What would be the pri- decreased thyroid-stimulating hormone (TSH).
mary tactics of a physician? What is the most likely diagnosis?
Krok 2 Medicine (англомовний варiант, iноземнi студенти) 2018 рiк 19

A. Diffuse toxic goiter A. Colposcopy, target biopsy of the cervix


B. Thyroid carcinoma B. US of the lesser pelvis
C. Autoimmune (Hashimoto’s) thyroiditis C. Hysteroscopy
D. Thyroid adenoma D. Curettage of the uterine cavity
E. Diffuse nontoxic goiter E. Hormone testing
152. A 64-year-old man suddenly sensed pain in 156. A 26-year-old woman came to a
his occipital area, dizziness, general weakness. gynecologist for a regular check-up. She has
He has a 15-year-long history of hypertension. no complaints. Per vaginum: the uterus lies in
Objectively the skin and mucosa are of normal anteflexion, not enlarged, dense, mobile, pai-
color. Auscultation reveals vesicular respiration nless. On the left from the uterus in the area of
across the lung surface. At the cardiac apex the uterine appendages there is a mobile painless
I heart sound is weakened, the II heart sound outgrowth that can be moved independently
is accentuated over the aorta. Pulse is 84/min., from the uterus. On the right the appendages
blood pressure is 180/100 mm Hg. Other body cannot be detected. What additional investi-
organs and systems are unaffected. What drug gation would be informative for diagnosis clari-
should be prescribed in the first place? fication?
A. Captopril A. Ultrasound of the lesser pelvis
B. Ramipril B. Metrosalpingography
C. Urapidil C. Examination for urogenital infection
D. Perindopril D. Colposcopy
E. Amlodipine E. Colonoscopy
153. During winter epidemics of influenza 157. A 38-year-old man underwent surgical
caused predominantly by virus А/California/04/2009 treatment of a wound with a suppuration
(H1N1), on the 2nd day after the disease onset focus. On the 8th day after the procedure the
a 30-year-old hospitalized man presented with wound cleared of purulo-necrotic discharge
high fever, dry cough, myalgia, headache, and and granulations appeared. However, against
general weakness. What should be prescribed the background of antibacterial therapy, the
as etiotropic treatment in this case? patient’s body temperature persists as high
as 38.5-39.5o C ; chills, excessive sweating, and
A. Neuraminidase inhibitors (Oseltamivir) euphoria are observed in the patient; heart
B. Antibiotics rate is 120/min. What complication of the
C. Immunoglobulin local suppurative inflammatory process can be
D. Interferon inducers suspected?
E. Acyclovir
A. Sepsis
154. A 24-year-old pregnant woman on her B. Purulent-resorptive fever
37th week of pregnancy has been delivered C. Trombophlebitis
to a maternity obstetric service with complai- D. Meningitis
nts of weak fetal movements. Fetal heartbeats E. Pneumonia
are 95/min. On vaginal examination the uterine
cervix is tilted backwards, 2 cm long, external 158. A man complains of high fever, pain in the
orifice allows inserting a fingertip. Biophysical area of his right mastoid bone, and purulent
profile of the fetus equals 4 points. What tactics discharge from the right ear. One week ago
of pregnancy management should be chosen? he had a case of URTI. Objectively the ri-
ght auricle protrudes, the skin behind the ear
A. Urgent delivery via cesarean section is hyperemic and pastose; on palpation of the
B. Treatment of placental dysfunction and mastoid bone the pain intensifies; the auditory
repeated analysis of the fetal biophysical profile meatus is filled with thick pus, posterosuperior
on the next day meatal wall sags; the tympanic membrane is red
C. Doppler measurement of blood velocity in and perforated. Make the diagnosis:
the umbilical artery
D. Urgent preparation of the uterine cervix for A. Acute mastoiditis
delivery B. Furuncle of the external auditory meatus
E. Treatment of fetal distress, if ineffective, then C. Acute otitis media
elective cesarean section on the next day D. Acute otitis externa diffusa
E. Exacerbation of chronic mesotympanitis
155. During regular preventive gynecological
examination a 30-year-old woman was detected 159. A 52 year old man came to see his fami-
to have dark blue punctulated ”perforations” ly physician complaining of pain in the chest.
on the vaginal portion of the uterine cervix. After taking history and performing physical
The doctor suspects endometriosis of the vagi- exam the doctor decided to direct the patient
nal portion of the uterine cervix. What investi- to cardiologist for a consultation. What level of
gation method would be most informative for medical care is being proposed to the patient?
diagnosis confirmation?
Krok 2 Medicine (англомовний варiант, iноземнi студенти) 2018 рiк 20

A. Secondary healthcare Examination revealed her mammary gland


B. Emergency healthcare to be significantly enlarged and deformed;
C. Primary healthcare breast tissue fluctuations and lymphadenitis are
D. Tertiare healtcare observed. What type of mastitis is the most li-
E. Palliative care kely?
160. A 30-year-old woman made an appoi- A. Phlegmonous mastitis
ntment with the family doctor for scheduled B. Serous mastitis
vaccination of her 2-year-old child. What type C. Infiltrative mastitis
of healthcare provides such medical services? D. Lactostasis
E. Mammary edema
A. Primary healthcare
B. Emergency aid 165. When her car collided with a tree, a 37-
C. Secondary healthcare year-old woman felt sharp pain in her left hip
D. Tertiary healthcare joint. She was unable to get out of the car.
E. Palliative care Her position is forced, the hip is pressed to
the abdomen, fixed, and rotated inwards; the li-
161. A 28-year-old man after car accident recei- mb is flexed in the knee, any attempt to change
ved a wound to the right side of his chest infli- the position results in sharp pain. Make the di-
cted by a sharp metal object. A foamy liquid agnosis:
flows out from the wound, there are tympani-
tis and acutely weakened respirations in the A. Closed dislocation of the left hip
right. Blood pressure is 70/30 mm Hg, pulse is B. Contusion of the left hip joint
120/min., Hb is 28 g/L. X-ray shows collapsed C. Hemarthrosis of the left hip joint
right lung, horizontal fluid level is at the 3rd rib. D. Arthritis of the left hip joint
What treatment tactics should be chosen? E. Closed cervical fracture of the left hip
A. Urgent thoracotomy 166. On the 9th day after childbirth the
B. Delayed thoracotomy obstetric patient developed high fever up to
C. Drain the right pleural cavity 38o C . She complains of pain in the right
D. Apply occlusive dressing to the wound mammary gland. The examination revealed the
E. Conservative therapy following: a sharply painful infiltrate can be
palpated in the right mammary gland, the skin
162. A 38-year-old woman has been working over the infiltrate is red, subareolar area and
as a milker for 15 years. She made an appoi- nipple are swollen and painful. What is your
ntment with the doctor due to development of diagnosis?
red rashes on her hands, predominantly in the
interdigital space. The rashes are weeping, itchi- A. Abscess of the right mammary gland
ng, and expanding on her skin. Examination of B. Mastopathy
her hands shows her nail plates to be yellow C. Cancer of the right mammary gland
and brittle. These presentations aggravate duri- D. Serous mastitis
ng work. Make the provisional diagnosis: E. Fibrous cystic degeneration of the right
mammary gland
A. Occupational eczema
B. Scabies 167. During reanimation procedures an
C. Pemphigus attempt to intubate trachea was made. Nei-
D. Pyoderma ther glottis nor epiglottis can be visualized via
E. Dermatophytosis laryngoscopy. What tactics should be chosen in
this case?
163. An 18-year-old patient complains of skin
rash. The patient has been suffering from this A. Insert a laryngeal mask
condition for 5 years. The first instance of this B. Perform a bag-valve-mask ventilation with
disease occurred after a car accident. Objecti- Ambu bag
vely: the patient presents with papular rash C. Continue the attempts until the intubation is
covered in silvery scales, ”thimble” symptom successful
(small pits on the nails), affected joints. What is D. Temporarily cease the chest compressions to
the most likely diagnosis? make intubation easier
E. Discontinue the attempts for mechanical
A. Psoriasis ventilation
B. Panaritium
C. Onychomycosis 168. A 48-year-old woman developed insomnia,
D. Lupus erythematosus depressive mood, anxiety, fears and suicidal
E. Rheumatism thoughts after the death of her husband that
occurred one month ago. During her stay in the
164. A postparturient woman, who has been hospital she speaks in a low voice, is depressed,
breastfeeding for 3 weeks, made an appoi- anxious, avoids sleeping, refuses to eat. What
ntment with the doctor. For the last 6 days medications should be prescribed in this case?
she has been feeling unwell, complains of body
temperature of 38-39o C , general weakness; wi-
thin the last 2 days she developed pain and
redness in the area of her right mammary gland.
Krok 2 Medicine (англомовний варiант, iноземнi студенти) 2018 рiк 21

A. Antidepressants 173. On ultrasound of the thyroid gland, a 47-


B. Antipsychotics year-old woman presents with a hypoechoic
C. Group B vitamins node 1.6 cm in diameter with blurred margi-
D. Nootropics ns and intranodular hypervascularization.
E. Anticonvulsants The doctor suspects thyroid carcinoma. What
method should be used to verify the diagnosis?
169. A 39-year-old man suffers from chronic
adrenal insufficiency and receives replacement A. Fine-needle aspiration biopsy
glucocorticoid therapy (hydrocortisone - 15 B. Thyroid scintigraphy
mg/day). He is to undergo elective surgery C. Case monitoring
for calculous cholecystitis. What medication D. Determine TSH level in the blood
adjustment should be made on the day of the E. Positron emission tomography (PET)
surgery to prevent the development of acute
adrenal insufficiency? 174. A 19-year-old young man complains of
cough with expectoration of purulent sputum
A. Increase the dosage by 2-3 times in amount of 100 ml per day, hemoptysis,
B. Cancel the drug for the day of the surgery dyspnea, increased body temperature up to
C. Add mineralocorticoid 37.8o C , general weakness, weight loss. The
D. Add antibiotic patient’s condition has been persisting for
E. Prescribe large volume intravenous fluid 4 years. Exacerbations occur 2-3 times per
infusion year. The patient presents with malnutriti-
on, pale skin, cyanosis of the lips, drumsti-
170. A 22-year-old man at 18:00 developed ck (clubbed) fingers. Tympanic percussi-
persisting dull pain in the epigastrium. Three on sound in the lungs, weakened respirati-
hours later nausea appeared, he vomited on, various numerous moist crackles in the
once. By the morning the pain shifted to lower pulmonary segments on the left can be
the right iliac area. Body temperature rose observed. In blood: erythrocytes - 3.2 · 1012 /L,
to 38.6o C , developed tachycardia of 110/min.
On examination there are muscle rigidity and leukocytes - 8.4 · 109 /L, ESR- 56 mm/hour. On
X-ray: lung fields are emphysematous, the left
Bloomberg’s sign (rebound tenderness) in the pulmonary root is deformed and dilated. What
right iliac area of the anterior abdominal wall. is the most likely diagnosis?
Plain x-ray of the abdomen shows no fluid
levels, free air under the diapragm on the ri- A. Multiple bronchiectasis of the left lung
ght. Make the diagnosis: B. Chronic left-sided pneumonia
C. Chronic abscess of the left lung
A. Perforation of a gastric ulcer D. Left-sided pulmonary cystic dysplasia
B. Renal colic E. Suppuration of the cyst in the left lung
C. Acute appendicitis
D. Acute pancreatitis 175. An 8-year-old child presents with blood
E. Acute cholecystitis pressure up to 180/100 mm Hg in the upper
limbs accompanied by headaches, tinnitus,
171. 40-50 minutes after the completion of occasional nosebleeds, and high fatigabili-
repair works conducted in a closed garage, ty. On examination there is no pulse over
with car engine running, the repair workers the leg arteries. ECG shows left ventricular
developed severe headache in the temporal hypertrophy. MRI-scan shows aortic narrowi-
area, nausea, tinnitus, vertigo, etc. These ng to 5 mm in the typical place. Coarctation
symptoms are characteristic of acute poisoni- of aorta is diagnosed. What kind of treatment
ng with: should be prescribed in this case?
A. Carbon monoxide A. Surgical
B. Aldehydes B. Conservative
C. Organochlorides C. Physical therapy
D. Hydrogen sulfide D. Case monitoring
E. Fluoride E. Abstain from surgery in favor of complex
172. After a long drive with the window open conservative therapy
a man developed facial asymmetry; he cannot 176. A 6-year-old girl arrived to the in-patient
close his right eye, his right nasolabial fold is unit with complaints of enlargement of the
smoothed out, movements of expression are lower third of her right thigh. According to the
absent on the right, there is a disturbance of case history, she has been stepping carefully
gustatory sensation in the tongue on the ri- on her right leg and limping for 6 months.
ght. No other neurological pathologies were Blood test detected anemia. X-ray of the ri-
detected. What disease can be provisionally di- ght thigh shows a round bone defect with clear
agnosed in this patient? margins resembling melting sugar in the distal
A. Neuropathy of the facial nerve metaphysis. What provisional diagnosis can be
B. Neuropathy of the trigeminal nerve made in this case?
C. Trigeminal ganglionitis
D. Neuropathy of the oculomotor nerve
E. Ischemic stroke
Krok 2 Medicine (англомовний варiант, iноземнi студенти) 2018 рiк 22

A. Osteogenic sarcoma of the right femur


B. Rheumatoid arthritis of the right knee joint A. Scabies
C. Acute hematogenous osteomyelitis of the B. Pyoderma
distal femoral metaphysis on the right C. Dermatitis
D. Tuberculous osteitis of the distal femoral D. Eczema
metaphysis on the right E. Shingles
E. Giant cell tumor of the right femur
181. A 28-year-old man, a teacher, after emoti-
177. A 37-year-old man working as a typesetter onal stress developed painful muscle spasms in
in a print shop complains of rapid fatigabili- his right hand that occur during writing; now
ty, paroxysmal attacks of stomachache, weak he has to hold the pen between the second and
drooping hands. Examination of neurologi- third fingers. He has no problems with typing
cal status revealed hypotrophy of the forearm or writing on the blackboard; no other motor
muscles. Carporadial reflexes are sharply disturbances or neurological pathologies are
weakened. Sensitivity is not disturbed. Gums detected. What is the most likely diagnosis?
present with dark blue border. What neurologi-
cal pathology is it? A. Writer’s cramp
B. Cortical agraphia
A. Lead polyneuropathy C. Parkinsonism
B. Guillain-Barre syndrome (postinfectious D. Neuropathy of the right ulnar nerve
polyneuritis) E. Neuropathy of the right radial nerve
C. Shingles
D. Ulnar neuropathy 182. A 32-year-old pregnant woman at the term
E. Brachial plexitis of 5-6 weeks was vaccinated against influenza
along with her whole family. At that time she
178. The mother of an epileptic son complai- was not aware of her pregnancy. The pregnancy
ns of her son periodically presenting with irri- is wanted. The woman needs an advice from
table depression, when he becomes excited and the family doctor regarding the maintenance of
prone to physical agression and violence. These her pregnancy, namely whether there is a risk of
moods last for 5-10 minutes, after which the fetal malformations because of received vacci-
patient exhausts himself and falls asleep. Upon nation. What advice should the doctor give in
waking he is depressed, sad, cannot recall his this case?
actions or recalls them only partially. What
psychopathologic condition is it? A. Vaccination against influenza is safe during
pregnancy
A. Dysphoria B. Therapeutic abortion is recommended
B. Dysthymia C. Immediate ultrasound of the lesser pelvis is
C. Epileptic delirium necessary
D. Pathologic affect D. Test for antibodies against influenza virus is
E. Ambulatory automatism necessary
E. An infectious diseases specialist must be
179. A 44-year-old woman has undergone consulted
subtotal thyroid resection due to diffuse toxic
goiter. On the second day after the surgery the 183. A 15-year-old girl suffers from systemic
patient’s condition deteriorated; she developed lupus erythematosus and has been receiving
palpitations, dyspnea, sweating, and diarrhea, prednisolone therapy in the daily dosage of 2
and became fearful The patient is anxious, mg/kg for the last 6 weeks. The plans are made
her skin is moist and hot to the touch. Her to gradually lower the dosage of the medicine.
temperature is 39.2o C . Heart sounds are No clinical signs of her disease are observed.
muffled, tachycardia is observed, pulse is Previously she has received no immunization
160/min., blood pressure is 85/40 mm Hg. The against measles. Due to measles outbreak it is
stomach is soft and painless. What should be necessary to develop the immunity against this
measured to clarify the patient’s condition? infection in the patient. When can she be vacci-
nated?
A. Thyroid hormones
B. Blood sugar A. At the present time
C. 17-KS and 17-OCS in urine B. After 2 weeks of prednisolone therapy in the
D. Blood epinephrine and norepineprine dosage lower than 2 mg/kg/24 hours
E. Serum transaminases C. 1 month after the prednisolone therapy is
complete
180. The dermatologist has an appointment wi- D. Immediately after the prednisolone therapy
th a 30-year-old man that complains of severely is complete
itching rashes that especially disturb him at ni- E. Never due to medical contraindications
ght. The rashes developed 2 weeks ago, after
he had returned from a travel. Objectively 184. A 45-year-old veterinary worker has made
on the lateral surfaces of his fingers, hands, an appointment with the doctor for regular
wrists, elbows, lower abdomen, genitals, and examination. In his duties he frequently deals
thighs there are paired papulovesicles, single with animals, however he denies working wi-
pustules, and scratch marks. What disease can th rabies-affected animals. Previously he has
be suspected? received no antirabic vaccination. What should
the doctor recommend in this case?
Krok 2 Medicine (англомовний варiант, iноземнi студенти) 2018 рiк 23

tral heart disease with prevalence of III degree


A. Preventive immunization with antirabic stenosis; heart failure IIA with retained left
vaccine ventricular ejection fraction, functional class
B. Vaccination in case of contact with sick III (NYHA). What tactics of vaccination agai-
animal nst respiratory infections should be chosen to
C. Preventive immunization with rabies provide secondary prevention of exacerbations
immunoglobulin and to avoid heart failure decompensation in
D. Administration of antirabic vaccine and this patient?
rabies immunoglobulin
E. Preventive immunization with anti-rabies A. Scheduled yearly vaccination against
serum influenza and pneumococci
B. Vaccination should be combined with antibi-
185. A healthy child 1 year and 5 months of otic administration
age is being vaccinated against hepatitis B. The C. Vaccination is contraindicated due to severe
child did not receive the first dose of the vacci- heart failure
ne previously, while in the maternity hospital. D. Any vaccination is contraindicated due to
The doctor makes an individual vaccination elderly age of the patient
schedule for this child and plans the admini- E. Any vaccination is contraindicated due to
stration of the next dose of the vaccine. What is mitral valve disease
the minimum interval between doses of vaccine
in this case? 189. A 69-year-old woman was diagnosed wi-
th the following: ischemic heart disease; stable
A. 1 month exertional angina pectoris, FC III; heart fai-
B. 6 months lure IIA with retained left ventricular ejecti-
C. 2 months on fraction, functional class III (NYHA).
D. 3 months What vaccine should be chosen for influenza
E. 12 months prevention and to avoid destabilization of the
186. A 46-year-old man came to the surgeon’s patient’s condition?
office. He complains of twitching sensation A. Type of influenza vaccine is not important
in the wound on his left foot, insomnia, and B. Inactivated influenza vaccine (IIV)
anxiety. According to the patient, he received
this wound 5 days ago, when he accidentally C. Recombinant influenza vaccine (RIV)
stepped on a glass shard, while on the beach. He D. Vaccination is contraindicated due to severe
requested no medical assistance. Objectively heart failure
the patient’s general condition is satisfactory, E. Vaccination is contraindicated due to elderly
pulse is 75/min., blood pressure is 130/80 mm age of the patient
Hg, temperature is 36.9o C . On the plantar 190. A 60-year-old man presents with
surface of his foot there is a wound 1.5 cm subcompensated viral liver cirrhosis (HCV),
long and up to 3 cm deep. The wound edges Child-Pugh class B. What tactics should be
are moderately hyperemic, no discharge from chosen regarding the vaccination against
the wound is observed. What disease can be influenza in this case?
suspected in this patient?
A. Tetanus A. Scheduled yearly vaccination
B. In case of influenza outbreak
B. Diphtheria C. Combined with antiviral drugs
C. Anthrax D. Contraindicated due to disease progression
D. Fasciitis stage, as shown by Child-Pugh class
E. Phlegmon E. Contraindicated due to elderly age of the
187. A 45-year-old woman has been suffering patient
from rheumatoid arthritis for 10 years and takes 191. A 20-year-old student was brought to
methotrexate twice a week. What statement the first-aid center. He has a closed fracture
regarding vaccination against pneumococci of the left forearm and a contused lacerated
(23-valent vaccine) would conform to the wound on his left shin. After the patient recei-
recommendations for the management of ved initial wound management, he presented
rheumatoid arthritis issued by the European the documents confirming that he has recei-
League Against Rheumatism in 2010? ved all the necessary preventive vaccination
A. Vaccination is recommended as scheduled. What should the doctor do to
B. Vaccination is not recommended prevent tetanus in this patient?
C. Vaccination is contraindicated to the patients A. Dynamic case monitoring
who take methotrexate B. Administration of tetanus immunoglobulin
D. Vaccination is contraindicated in cases when C. Administration of anti-tetanus serum
inflammatory process is active D. Antibiotic therapy
E. Vaccination necessitates increase in the E. Administration of tetanus toxoid
dosage of the long-term medicines
192. A 17-year-old girl has made an appoi-
188. A 65-year-old woman was diagnosed with ntment with the doctor. She plans to begin her
the following: chronic rheumatic heart disease, sex life. No signs of gynecological pathology
I degree of rheumatic activity; combined mi-
Krok 2 Medicine (англомовний варiант, iноземнi студенти) 2018 рiк 24

were detected. In the family history there was a sease?


case of cervical cancer that occurred to the pati-
ent’s grandmother. The patient was consulted A. Rubella virus
about the maintenance of her reproductive B. Epstein-Barr virus
health. What recommendation will be the C. Streptococcus
most helpful for prevention of invasive cervi- D. Mumps virus
cal cancer? E. Neisseria meningitis

A. Vaccination against human papillomavirus 197. During administration of planned DPT


(HPV) vaccination the child suddenly developed acute
B. Vitamins, calcium, omega-3 anxiety, signs of pain response, dyspnea, grunti-
C. Immunomodulators ng respirations, cutis marmorata, cold sweat.
D. Antiviral and antibacterial drugs Objectively the child’s consciousness is di-
E. Timely treatment of sexually transmitted sturbed, heart rate is 150/min., blood pressure
diseases is 60/40 mm Hg, heart sounds are muffled. The
child was diagnosed with anaphylactic shock.
193. A 6-month-old infant is not vaccinated. What drug should be administered first?
The physician recommends a DPT (diphtheria,
pertussis, tetanus) vaccination but the mother A. Epinephrine
is absolutely against this procedure. Choose the B. Lasix (Furosemide)
most substantial argument in favor of vaccinati- C. Suprastin (Chloropyramine)
on: D. Euphylline (Aminophylline)
E. Analgin (Metamizole)
A. Risk of lethal consequences
B. Epidemic risk for the others 198. Among first-year schoolchildren there was
C. Personal professional experience a case of measles registered. A 7-year-old boy
D. High quality of vaccines from the same group was not vaccinated against
E. - measles due to refusal of his parents. His clini-
cal history has no cases of measles in the past
194. In autumn a 45-year-old man was and is not contraindicatory to immunobiologi-
recommended an elective surgery for coronary cal agents. Choose the most rational tactics of
artery bypass grafting due to multivessel measles prevention in this schoolboy:
coronary artery disease. The patient has never
received anti-influenza vaccination. Why would A. Measles-Mumps-Rubella vaccine
the family doctor offer a scheduled yearly vacci- B. Isolation for 20 days
nation against influenza to this patient? C. Antiviral agents
D. Antibiotics
A. Decrease the risk of mortality due to E. Immunomodulators
pneumonia and heart failure
B. Secondary prevention of exacerbations of 199. A 40-year-old man developed fever up to
chronic ischemic heart disease 37.5o C and macular rash 10 days after the fi-
C. Immunoprophylaxis of postoperative rst dose of MMR (Measles-Mumps-Rubella)
pulmonary complications vaccine was administered. The vaccination was
D. Primary prevention of influenza during considered necessary as there was a measles
postoperative care outbreak in the city and the patient had not
E. Prevention of seasonal influenza received MMR vaccination in his childhood. Is
revaccination with MMR vaccine possible?
195. A 28-year-old woman has made an
appointment with the family doctor to recei- A. Possible
ve vaccination against influenza. However, B. Forbidden
having collected the patient’s medical hi- C. After a course of glucocorticoids treatment
story, the doctor claimed this procedure to D. Simultaneously with antihistamines
be absolutely contraindicated for this woman. E. Under supervision in the infectious diseases
What anamnestic data is the absolute contrai- inpatient ward
ndication to vaccination?
200. A 26-year-old man is undergoing a regular
A. Egg white intolerance check-up. One year ago he had a case of tonsi-
B. Pregnancy at 30 weeks llar diphtheria complicated with myocarditis.
C. Blood hemoglobin - 109 g/L Presently his condition is satisfactory, no si-
D. Body temperature - 37.2o C gns of cardiovascular failure; ECG shows first-
E. Psoriasis in the remission phase degree atrioventricular block. What vaccine
was administered to this man according to his
196. A 26-year-old man complains of chills, age?
rhinitis, dry cough, and fever up to 38o C .
Examination shows him to be in a moderately A. Adsorbed diphtheria tetanus vaccine (modi-
severe condition; there are small pale pink fied)
non-merging spots on the skin of his back, B. Acellular DPT vaccine
abdomen, and extremities. Palpation reveals C. Tetanus anatoxin
enlarged occipital and axillary lymph nodes. No D. Oral polio vaccine (OPV)
information about vaccination history could be E. BCG vaccine
obtained. What is the likely etiology of this di-
Krok 2 Medicine (англомовний варiант, iноземнi студенти) 2018 рiк 25

Laboratory values
Complete blood count
Laboratory value Normal reference range
Male: 4.3 − 5.9 · 1012 /L
Erythrocyte count
Female: 3.5 − 5.5 · 1012 /L
Male: 135-175 g/L
Hemoglobin, blood
Female: 120-160 g/L
Color index 0.85- 1.05
Reticulocyte count 0.5 - 1.5%
Platelet count 150 − 400 · 109 /L
Leukocyte count 4.0 − 9.0 · 109 /L
Basophils 0 − 0.065 · 109 /L (0-1%)
Eosinophils 0.02 − 0.30 · 109 /L (0.5 - 5.0%)
Band neutrophils 0.04 − 0.30 · 109 /L (1 - 6%)
Segmented neutrophils 2.0 − 5.50 · 109 /L (47 - 72%)
Monocytes 0.09 − 0.60 · 109 /L (3 - 11%)
Lymphocytes 1.2 − 3.0 · 109 /L (19 - 37%)
Erythrocyte sedimentation rate Male: 0 - 15 mm/h
(ESR) Female: 0 - 20 mm/h
Male: 41 - 53%
Hematocrit
Female: 36 - 46%
Biochemical blood analysis (Metabolic panel)
Total proteins 60-78 g/L
Albumin 35-50 g/L (52-65%)
Globulin: 23-35 g/L (35-48%)
α1 -Globulin 2-4 g/L (4.2-7.2%)
α2 -Globulin 5-9 g/L (6.8-12%)
β -Globulin 6-11 g/L (9.3-15%)
γ -Globulin 11-15 g/L (15-19%)
Immunoglobulins:
IgD 0 - 0.15 g/L
IgG 6.5-15 g/L
IgM 0.4-3.45 g/L
IgA 0.76-3.90 g/L
IgE 0-380 kU/L
Bilirubin:
Total 2-17 mcmol/L
Indirect (unconjugated) 2-17 mcmol/L
Direct (conjugated) 0-5 mcmol/L
Triglycerides 0.59-1.77 mmol/L
Total cholesterol 3.9-6.2 mmol/L
Lipoproteins:
high-density lipoproteins (β -Lipoproteins) <4.2 mmol/L
low-density lipoproteins (α-Lipoproteins) 0.8-1.8 mmol/L
Glucose, blood 3.3-6.1 mmol/L (fasting)
Glycated hemoglobin 6%
Iron, blood 9-30 mcmol/L
Potassium, plasma 3.5-5.0 mmol/L
Sodium, plasma 136-145 mmol/L
Calcium, plasma 0.75-2.5 mmol/L
Krok 2 Medicine (англомовний варiант, iноземнi студенти) 2018 рiк 26

Magnesium, plasma 0.75 - 1 mmol/L


Phosphorus (inorganic), serum 0.646-1.292 mmol/L
Chloride, blood 95-105 mmol/L
Urea, serum 3.33 - 8.32 mmol/L
Creatinine 53 - 106 mcmol/L
Male: 0.12-0.38 mcmol/L
Uric acid
Female: 0.12-0.46 mcmol/L
Lactate dehydrogenase (LDH) 45-90 U/L
α-Amylase (diastase), blood 25-125 U/L
Aspartate aminotransferase (AST) 15-40 U/L
Alanine aminotransferase (ALT) 10-40 U/L
Phosphatase (alkaline) 30-115 U/L
Coagulogram
Prothrombin time 80-100% (< 12 seconds)
Fibrinogen 2-4 g/L
Partial thromboplastin time (activated) < 28 seconds
Lee-White coagulation time 5-10 minutes
Bleeding time (Duke method) < 4 minutes
Acid-base balance
pH arterial blood 7.35-7.45
Pco2 arterial blood 33-45 mm Hg
Po2 arterial blood 75 - 105 mm Hg
Bicarbonate, blood (HCO3 ) 22-28 mmol/L
Other blood values
Cortisol, serum 230 -750 nmol/L
Osmolality, serum 275 - 295 mOsmol/kg H2 O
Thyroid-stimulating hormone, serum or plasma 0.5-5 mIU/L
Thyroxine (T4), serum 65-155 nmol/L
Triiodothyronine (T3), serum 1.77 - 2.43 nmol/L
Male: 15-200 mcg/L
Ferritin, serum
Female: 12-150 mcg/L
Thymol turbidity test <5U
C-reactive protein (CRP) <6 mg/dL
Antistreptolysin 0 (ASL0) 250 U
Antistreptohyaluronidase (ASH) 250 U
Urine
Urine specific gravity 1.016-1.022
Nechiporenko’s test:
leukocytes < 2 · 106 /л
erythrocytes < 1 · 106 /л
Proteins, total <45.0 - 75.0 mg/24 h (<0.033 g/L)
Calcium 2.5 - 7.5 mmol/24 h
Male: 97 - 137 mL/min
Creatinine clearance
Female: 88 - 128 mL/min
Oxalate 90 - 445 mcmol/L
Male: 27.7 - 79.7 mcmol/24 h
17-Ketosteroids (17-KS)
Female: 17.4 - 55.4 mcmol/24 h
17-Hydroxycorticosteroids (17-OCS) 5.5 - 27.6 mcmol/24 h
α-Amylase (diastase), urine 28-160 U/L
INSTRUCTIONAL BOOK
Testing Board

TEST ITEMS FOR LICENSING EXAMINATION: KROK 2. MEDICINE.

Kyiv. Testing Board.


(English language).

Approved to print 12.04/№214. Paper size 60х84 1/8


Offset paper. Typeface. Times New Roman Cyr. Offset print.
Conditional print pages 28. Accounting publishing pages 32.
Issue. 2493 copies
List of abbreviations
ACTH Adrenocorticotrophic hormone
ADP Adenosine diphosphate
ALT, ALAT Alanin aminotranspherase
ALV Artificial lung ventilation
AP Arterial (blood) pressure
AST, AspAT Aspartat aminotranspherase
ATP Adenosine triphosphate
ВСG Bacillus Calmette-Guérin
BP Blood (arterial) pressure
CK Creatine kinase
CNS Central nervous system
СоА Coenzyme A
CRP C-reactive protein
CT Computed tomography
COX Cyclooxygenase
DIC Disseminated intravascular coagulation
DTP (DPT) Diphtheria-tetanus-pertussis vaccine
EDTA Ethylenediamine tetra-acetic acid
ELISA Enzyme-linked immunosorbent assay
ENT Ear, nose, and throat (as a department in a hospital)
EPR (ER) Endoplasmic reticulum
ESR Erythrocyte sedimentation rate
EV Enterovirus
FC Functional class
GABA Gamma aminobutyric acid
Hct, Ht Hematocrit
HDL High-density lipoproteins
HR Heart rate
IDL Intermediate-density lipoproteins
IU International unit
IUPAC International Union of Pure and Applied Chemistry
LDH Lactate dehydrogenase
LDL Low-density lipoproteins
LOX Lipoxygenase
MAO Monoamine oxidase
MRI Magnetic resonance imaging
NSAID Nonsteroidal anti-inflammatory drug
PE (PATE) Pulmonary embolism (Pulmonary artery thromboembolism)
PSA Prostate-specific antigen
RBC Red blood count
RR Respiratory rate
SES Sanitary-and-epidemiologic station
STD Sexually transmitted disease
STI Sexually transmitted infection
T/l Trillion/liter
TABT Typhoid-paratyphoid A and B plus tetanus toxoid vaccine
TMJ Temporomandibular joint
TSH Thyroid-stimulating hormone
TU Tuberculin unit
URTI Upper respiratory tract infection
V/f Vision field
VLDL Very-low-density lipoproteins
WBC White blood count
WPW Wolff-Parkinson-White syndrome
Krok 2 – 2014 Therapy Base
1. A 47 y.o. woman complains of having paroxysmal headaches for the last 5
years. The pain is one-sided, intense, localised in frontal region of head,
accompanied by nausea and stomach discomfort, begins one of a sudden.
Onset is usually preceded by vision reduction. Anamnesis gives evidence of
periodical AP rise, but at the moment the woman doesnt take any medicines.
Inbetween the onsets of headache her state is satisfactory. Objectively: high-
calorie diet (body weight index - 29), AP- 170/95 mm Hg. Neurologic state has
no pecularities. What is the most probable diagnosis?
A. Migraine
B. Epilepsy
C. Hypertensive encephalopathy
D. Benign intracranial hypertension
E. Chronic subdural hematoma

2. A 7 y.o. boy suddenly felt pain in his right knee, it became edematic. The day
before he took part in a cross-country race. Family anamnesis has no data
about hemophilia and bleeding sickness. Objectively: body temperature is
37,5°C. The knee is painful, hot to the touch, edematic with local tissue
tension over it. Blood count: Hb- 123 g/L, leukocytes - 5,6×109/L,
thrombocytes - 354×109/L, prothrombin time - 12 seconds (normally 10-15
seconds), partly activated thromboplastin time - 72 seconds (normally 35-45
seconds). Hemorrhage time is normal, VIII:C factor is 5% of norm. What is
the most probable diagnosis?
A. Hemophilia A
B. Hemophilia B
C. Vitamin K deficiency
D. Thrombocytopenia
E. Schoenlein-Henoch disease

3. On the 3rd day after the acute anterior myocardial infarction a 55 y.o. patient
complains of dull ache behind his breast bone, that can be reduced by bending
forward, and of dyspnea. Objectively: AP- 140/180 mm Hg, heart sounds are
dull. ECG results: atrial fibrillation with frequence of ventricular contractions
at the rate of 110/min, pathological Q wave and S-T segment raising in the
right chest leads. The patient refused from thrombolisis. What is the most
probable diagnosis?
A. Acute pericarditis
B. Pulmonary embolism
C. Dissecting aortic aneurysm
D. Dresslers syndrome
E. Tietzes syndrome
4. A 54 y.o. man was admitted to the hospital with complaints of sudden intense
headache in occipital region and vomiting. In the medical hystory: moderate
arterial hypertension, the patient was taking hydrochlorothiazide. Three days
ago he consulted a therapeutist about intense headache that was suppressed
by an analgetic. Objectively: consciousness is confused, left pupil is mydriatic.
Evident photophobia and tension of neck muscles. Left-side hemiparesis with
increased muscle tonus and reflexes. Body temperature is low, rash is absent.
AP- 230/130 mm Hg, Ps- 50 bpm, BR- 12/min. What is your preliminary
diagnosis?
A. Acute subdural hematoma
B. Disseminated sclerosis
C. Myasthenia
D. Migraine
E. Acute bacterial meningitis

5. A 51 y.o. women was taken to the emergency department in convulsive status


epilepticus. The first means of medical management should be:
A. Ensuring that the airway is open and the patient is oxygenating
B. Administering an instravenous bolus of 50% dextrose
C. Inducing pentobarbital coma
D. Injecting 5 mg of diazepam followed by a loading dose of phenytoin
E. Inserting a tongue blade

6. A 38 y.o. man complains of having occasional problems with swallowing of


both hard and fluid food for many months. Sometimes he feels intense pain
behind his breast bone, epecially after hot drinks. There are asphyxia onsets at
night. He has not put off weight. Objectively: his general condition is
satisfactory, skin is of usual colour. Examination revealed no changes of
gastrointestinal tract. X-ray picture of thorax organs presents esophagus
dilatation with level of fluid in it. What is the preliminary diagnosis?
A. Esophagus achalasia
B. Gastroesophageal reflux
C. Cancer of esophagus
D. Myastenia
E. Esophagus candidosis

7. A 35 y.o. woman consulted a doctor about occasional pains in paraumbilical


and iliac region that reduce after defecation or passage of gases. Defecation
takes place up to 6 times a day, stool is not solid, with some mucus in it.
Appetite is normal, she has not put off weight. First such symptoms appeared
1,5 year ago, but colonoscopy data reveals no organic changes. Objectively:
abdomen is soft, a little bit painful in the left iliac region. Blood and urine are
normal. What is the preliminary diagnosis?

A. Irritable bowels syndrome


B. Dispancreatism
C. Crohns disease
D. Celiac disease
E. Pseudomembranous colitis

8. The physician must undertake measures for primary prophylaxis of iron


deficiency anemia. Which of the following categories of patient are subject to
such primary prophylactic measures?
A. Pregnant women
B. All children
C. Patients after 60
D. Patients after operation
E. Workers of industrial enterprises

9. A patient with unstable angina pectoris was given the following complex
treatment: anticoagulants, nitrates, a-adrenoblockers. However on the third
day of treatment the pain still romains. Which in vestigation shoud be carried
out to establish diagnosis?
A. Coronarography
B. Test with dosed physical exercises
C. Stress-echocardiogram
D. Esophageal electrocardiac stimulator
E. Myocardial scintigraphy

10. The 28 y.o. woman applied to doctor because of limited loss of the hair. In the
anamnesis - she had frequent headache indisposition, arthromyalgia, fever,
irregular casual sexual life, drug user. RW is negative. What examination must
be done first?
A. Examination for HIV
B. Examination for trichomoniasis
C. Examination for gonorrhea
D. Examination for neuropathology
E. Examination for fungi

11. A 35 y.o. woman was admitted to thoracic surgery department with fever up to
40°C, onset of pain in the side caused by deep breathing, cough with
considerable quantity of purulent sputum and blood with bad smell. What is
the most likely diagnosis?
A. Abscess of the lung
B. Complication of liver echinococcosis
C. Actinomycosis of lungs
D. Pulmonary tuberculosis
E. Bronchiectatic disease

12. A 67 y.o. patient complains of palpitation, dizziness, noise in ears, feeling of


shortage of air. Objectively: pale, damp skin. Vesicular respiration, respiratory
rate- 22 per min, pulse- 200 bpm, AP- 100/70 mm Hg. On ECG: heart rate-
200 bmp, ventricular complexes are widened, deformed, location of segments
ST and of wave T is discordant. The wave is not changed, superimposes
QRST. What kind of arrhythmia is present?
A. Paroxismal ventricular tachycardia
B. Ventricular extrasystole
C. Atrial tachycardia
D. Sinus tachycardia
E. Atrial flutter

13. Male 30 y.o., noted growing fingers and facial scull, changed face. Complains
of poor eyesight, weakness, skin darkening, loss of body weight. X-ray shows
broadening of sella turcica, thinning of tuberculin sphenoidale, signs of
increased intracranial pressure. What diagnosis can you make?
A. Adenoma of hypophysis
B. Tumor of pondo-cerebellar corner
C. Optico - hiasmatic arachnoiditis
D. Encephalitis of truncus
E. Adrenal gland tumor

14. A patient complains of a tormental (agonizing) cough with expectoration of up


to 600 ml/daily purulent chocolatecolor sputum with a decay smell. Onset of
illness was abrupt, to- 39°C, fever of irregular type. There is the area of
darkening with a cavity in a center on X-ray film, with irregular contours and
level of liquid. What disease is the question?
A. Gangrene of lung
B. Bronchiectatic illness
C. Lobar pneumonia
D. Pneumonia complicated by an abscess
E. Tuberculosis

15. A 24 y.o. patient complains of nausea, vomiting, headache, shortness of


breath. He had an acute nephritis being 10 y.o. Proteinuria was found out in
urine. Objectively: a skin is grey-pale, the edema is not present. Accent of II
tone above aorta. BP 140/100-180/100 mm Hg. Blood level of residual N2-
6,6 mmol/L, creatinine- 406 mmol/L. Days diuresis- 2300 ml, nocturia.
Specific density of urine is 1009, albumin- 0,9 g/L, WBC- 0-2 in f/vis. RBC.-
single in f/vis., hyaline casts single in specimen. Your diagnosis?
A. Chronic nephritis with violation of kidney function
B. Hypertensive illness of the II degree
C. Stenosis of kidney artery
D. Nephrotic syndrome
E. Feochromocitoma

16. A 33 y.o. male patient was admitted to a hospital. A patient is pale, at an


attempt to stand up he complains of strong dizziness. There was vomiting like
coffee-grounds approximately hour ago. BP- 90/60 mm Hg., pulse- 120
b/min. In anamnesis, a patient has suffered from ulcer of the stomach,
painless form during 4 years. An ulcer was exposed at gastrofiberoscopy. Your
diagnosis:
A. Ulcer of stomach, complicated with bleeding
B. Erosive gastritis
C. Ulcer of duodenum, complicated with bleeding
D. Acute pleurisy
E. Acute myocardial infarction, abdominal form

17. A 48-year-old patient complains of heaviness in the right hypochondrium,


itching of the skin.He had been treated in infectious diseases hospital
repeatedly due to icterus and itch. On physical exam: meteorism, ascitis,
dilation of abdominal wall veins, protruded umbilicus, spleen enlargement.
What can be diagnosed in this case?
A. Liver cirrhosis
B. Cancer of the head of pancreas
C. Viral hepatitis B
D. Gallstones
E. Cancer of the liver

18. A 25-year-old man has facial edema, moderate back pains. His temperature is
37,5°C, BP 180/100 mm Hg, hematuria [up to 100 in v/f], proteinuria [2,0
g/L], hyaline casts - 10 in v/f., specific gravity -1020. The onset of the disease
is probably connected with acute tonsillitis that started 2 weeks ago. What is
the most probable diagnosis?
A. Acute glomerulonephritis
B. Urolithiasis
C. Chronic glomerulonephritis
D. Acute pyelonephritis
E. Cancer of the kidney
19. In the development of the inflammation processes glucocorticoids reduce the
level of certain most important active enzyme. It results also in the reducing of
the synthesis of prostaglandins and leucotrienes which have a key role in the
development of inflammation processes. What is the exact name of this
enzyme?
A. Phospholipase A2
B. Arachidonic acid
C. Cyclooxygenase – 1
D. Cyclooxygenase – 2
E. Lipoxygenase

20. A 30 y.o. female with rheumatoid arthritis of five years duration complains of
pain in the first three fingers of her right hand over past 6 weeks. The pain
seems especially severe at night often awakening her from sleep.The most
likelly cause is?
A. Carpal tunnel syndrome
B. Sensory peripheral neuropathy
C. Rheumatoid arthritis without complication
D. Rheumatoid vasculitis
E. Atlanto-axial sublaxation of cervical spine

21. A 19-year-old man has suffered from moderate mental retardation since
childhood. The patient is illiterate, can take care of himself, do simple
household work and other kinds of easy work under supervision. What does
his rehabilitation [tertiary prevention] require?
A. All the options
B. None of the above mentioned
C. Physical work under supervision
D. Supervision of a social worker
E. Supervision of relatives (if any)

22. A 38 y.o. patient was urgently admitted to the hospital with complaints of
sudden weakness, dizziness, loss of consciousness, body weight loss, nausea,
vomiting, severe pain in epigastric area, diarrhea, skin hyperpigmentation.
What is the most probable diagnosis?
A. Addisonic crisis
B. Meningoencephalitis
C. Pellagra
D. Scleroderma
E. Acute gastroenteritis

23. An unconscious patient presents with moist skin, shallow breathing. There are
signs of previous injection on the shoulders and hips. BP- 110/70 mm Hg.
Tonus of skeletal muscles and reflexes are increased. Cramps of muscles of the
extremities are seen. What is the most likely disorder?
A. Hypoglycemic coma
B. Hyperlactacidotic coma
C. Stroke
D. Hyperglycemic coma
E. Hyperosmolar coma

24. A patient was admitted to the hospital on the 7th day of the disease with
complaints of high temperature, headache, pain in the muscles, especially in
calf muscles. Dermal integuments and scleras are icteric. There is
hemorrhagic rash on the skin. Urine is bloody. The patient was fishing two
weeks ago. What is the most likely diagnosis?
A. Leptospirosis
B. Salmonellosis
C. Yersiniosis
D. Brucellosis
E. Trichinellosis

25. A 60-year-old woman, mother of 6 children, developed a sudden onset of


upper abdominal pain radiating to the back, accompanied by nausea,
vomiting, fever and chills. Subsequently, she noticed yellow discoloration of
her sclera and skin. On physical examination the patient was found to be
febrile with temp of 38,9°C, along with right upper quadrant tenderness. The
most likely diagnosis is:
A. Choledocholithiasis
B. Benign biliary stricture
C. Carcinoma of the head of the pancreas
D. Choledochal cyst
E. Malignant biliary stricture

26. A 42-year-old woman suffers from bronchial asthma, has an acute attack of
bronchial asthma. What medication from the listed below is contraindicated
to render a first aid?
A. Euphylinum
B. Corazolum
C. Izardin
D. Morphinum hydrochloride
E. Strophanthin hydrochloride

27. 4 days ago a 32-year-old patient caught a cold: he presented with sore throat,
fatigue. The next morning he felt worse, developed dry cough, body
temperature rose up to 38,2°C, there appeared muco-purulent expectoration.
Percussion revealed vesicular resonance over lungs, vesicular breathing
weakened below the angle of the right scapula, fine sonorous and sibilant
wheezes. What is the most likely diagnosis?
A. Focal right-sided pneumonia
B. Pulmonary carcinoma
C. Pulmonary gangrene
D. Bronchial asthma
E. Acute bronchitis

28. A 62-year-old patient complaining of enlargement of cervical, supraclavicular


and axillary lymph nodes, subfebrile temperature for the last 3 months has
been admitted to a hospital. In blood: WBCs – 64×109/l, lymphocytes - 72%.
What method of study should be used to specify the diagnosis?
A. Myelogram
B. X-rays
C. Thermography
D. Lymphography
E. Lymphoscintigraphy

29. A 38 y.o. woman complains of a purulent discharge from the left nostril. The
body temperature is 37,5°C. The patient has been ill for a week and associates
her illness with common cold. There are a pain and tenderness on palpation of
her left cheek. The mucous membrane in the left nasal cavity is red and
turgescent. The purulent exudate is seen in the middle meatus in maxillary.
What is the most probable diagnosis?
A. Acute purulent maxillary sinusitis
B. Acute purulent sphenoiditis
C. Acute purulent frontitis
D. Acute purulent ethmoiditis

30. A 40-year-old female patient has been hospitalized for attacks of asphyxia,
cough with phlegm. She has a 4-year history of the disease. The first attack of
asphyxia occurred during her stay in the countryside. Further attacks occurred
while cleaning the room. After 3 days of inpatient treatment the patients
condition has significantly improved. What is the most likely etiological
factor?
A. Household allergens
B. Pollen
C. Chemicals
D. Psychogenic
E. Infectious
31. The complications of acute cholecystitis which require surgical intervention
are as follows EXCEPT:
A. Jaundice
B. Gall-bladder perforation
C. Cholangitis conditioned by the presence of stones in the bile tract
D. Empyema of the gall-bladder
E. Emphysematous gall-bladder

32. A 22-year-old girl has been complaining of having itching rash on her face for
2 days. She associates this disease with application of cosmetic face cream.
Objectively: apparent reddening and edema of skin in the region of cheeks,
chin and forehead; fine papulovesicular rash. What is the most likely
diagnosis?
A. Allergic dermatitis
B. Erysipelas
C. Neurodermatitis
D. Dermatitis simplex
E. Eczema

33. A schizophrenic patient considers himself to be "an outstanding scientist, a


brilliant composer and an unrivalled artist". He complains that "family and
friends are always jealous of him and want to poison him". Determine the
psychopathological syndrome:
A. Paranoiac
B. Paratrophic
C. Hebephrenic
D. Paranoid
E. Manic

34. A 43-year-old alcohol abuser had not consumed alcohol for the last two days.
In the evening he claimed to see rats and feel like they bite his feet. The
patient is disoriented, agitated, all the time attempts to run somewhere.
Specify the psychopathological syndrome:
A. Delirious
B. Gansers syndrome
C. Oneiroid
D. Amential
E. Choreatic

35. A 42-year-old patient complains of back pain, darkened urine, general


weakness, dizziness that occurred after treating a cold with aspirin and
ampicillin. Objectively: the patient is pale, with subicteric sclerae. HR - 98
bpm. Liver - +2 cm, spleen - +3 cm. In blood: RBCs - 2,6×1012/l, Hb - 60 g/l,
CI - 0,9, WBCs - 9,4×109/l, basophils - 0,5%, eosinophils - 3%, stab
neutrophils - 6% segmented neutrophils - 58%, lymphocytes - 25%,
monocytes - 7%, ESR - 38 mm/hour, reticulocytes - 24%. Total bilirubin - 38
millimole/l. What complication occurred in the patient?
A. Acquired hemolytic anemia
B. Cholelithiasis
C. Toxic hepatitis
D. Agranulocytosis
E. Paroxysmal nocturnal hemoglobinuria

36. A hospital has admitted a 52-year-old patient with disseminated pulmonary


tuberculosis, complaints of acute pain in the right half of chest, that developed
after going upstairs to the 3rd floor; cough, dyspnea at rest, marked cyanosis.
What kind of complication should suspected first of all?
A. Spontaneous pneumothorax
B. Cardiac failure
C. Pleuritis
D. Acute myocardial infarction
E. Pulmonary failure

37. A 38-year-old male patient has been taking alcohol for 3 years. 3 days after a
regular drinking period he felt anxiety and fear. It appeared to him that he was
surrounded by spiders and worms, pursued by some "condemnatory voices".
His behaviour became aggressive. The patient demonstrated correct self-
awareness but impairment of temporal and spatial orientation. What is the
most likely diagnosis?
A. Delirium alcoholicum
B. Alcoholic hallucinosis
C. Pathologic intoxication
D. Alcoholic encephalopathy
E. Alcoholic paranoia

38. A painter working at a motorcar plant has been diagnosed with moderately
severe intoxication with amide compounds of benzene. The in-patient
treatment resulted in a considerable health improvement. What expert
decision should be made in this case?
A. The patient should be issued a sick list for out-patient treatment
B. The patient should be referred to the medio-social expert commission for
evaluation of percentage of work capicty loss
C. The patient may get back to work providing he will keep to hygiene and
sanitary regulations
D. The patient should be referred to the medio-social expert commission for
attributing the disability group because of an occupational disease
39. A 16-year-old patient who has a history of intense bleedings from minor cuts
and sores needs to have the roots of teeth extracted. Examination reveals an
increase in volume of the right knee joint, limitation of its mobility. There are
no other changes. Blood analysis shows an inclination to anaemia (Hb- 120
g/l). Before the dental intervention it is required to prevent the bleeding by
means of:
A. Cryoprecipitate
B. Fibrinogen
C. Calcium chloride
D. Dried blood plasma
E. Epsilon-aminocapronic acid

40. A 44-year-old patient complains about difficult urination, sensation of


incomplete urinary bladder emptying. Sonographic examination of the urinary
bladder near the urethra entrance revealed an oval well-defined
hyperechogenic formation 2x3 cm large that was changing its position during
the examination. What conclusion can be made?
A. Concrement
B. Urinary bladder polyp
C. Malignant tumour of the urinary bladder
D. Prostate adenoma
E. Primary ureter tumour

41. A 7-year-old boy had complained of headache, nausea, fatigue for 3 weeks. His
condition gradually deteriorated, headache and general weakness progressed.
The boy had bronchitis at the age of 3. His father has a history of pulmonary
tuberculosis. Objectively: body temperature 37,5°C, conscious, lies supine,
with the hip and knee flexed to 90 degrees, nuchal rigidity +6 cm, partial
ptosis of the right eyelid, the dilated right pupil. General hyperalgesia is
present. Liquor: transparent, pressure - 400 mm of water column, protein -
1,5%, cytosis - 610/3 with predominant lymphocytes, sugar - 1,22 mmol/l,
chlorides - 500 mmol/l. What is the most likely diagnosis?
A. Tuberculous meningitis
B. Epidemic cerebrospinal meningitis
C. Pneumococcal meningitis
D. Serous meningitis
E. Secondary purulent meningitis

42. A 35-year-old patient complains about pain and morning stiffness of hand
joints and temporomandibular joints that lasts over 30 minutes. She has had
these symptoms for 2 years. Objectively: edema of proximal interphalangeal
digital joints and limited motions of joints. What examination should be
administered?
A. Roentgenography of hands
B. Rose-Waaler reaction
C. Complete blood count
D. Immunogram
E. Proteinogram

43. A 69-year-old female patient complains of temperature rise up to 38,3°C,


haematuria. ESR - 55 mm/h. Antibacterial therapy turned out to be
ineffective. What diagnosis might be suspected?
A. Renal cancer
B. Chronic glomerulonephritis
C. Renal amyloidosis
D. Polycystic renal disease
E. Urolithiasis

44. Examination of a 35-year-old patient with rheumatism revealed that the right
heart border was 1 cm displaced outwards from the right parasternal line, the
upper border was on the level with inferior margin of the 1st rib, the left
border was 1 cm in from the left midclavicular line. Auscultation revealed
atrial fibrillation, loud apical first sound, diastolic shock above the pulmonary
artery. Echocardiocopy revealed abnormal pattern of the mitral valve motion.
What heart disease is characterized by these symptoms?
A. Mitral stenosis
B. Mitral valve insufficiency
C. Mitral valve prolapse
D. Aortic stenosis
E. Tricuspid valve insufficiency

45. A 54-year-old male patient works as an engineer. At the age of 35, he got
infected with syphilis and treated it with "traditional remedies". About 5 years
ago, he became forgetful, unable to cope with work, told cynical jokes, bought
useless things, collected cigarette butts in the street. Objectively: the patient is
indifferent, has slow speech, dysarthria, can make only primitive judgments, is
unabe to perform simple arithmetic operations or explain simple metaphors.
The patient is untidy, takes no interest in anything, passive. Considers himself
to be completely healthy. Qualify mental condition of the patient:
A. Total dementia
B. Somnolentia
C. Hysterical pseudodementia
D. Korsakoffs (amnesic) syndrome
E. Lacunar (dysmnestic) dementia
46. A 47-year-old patient complains of insomnia, heaviness over his entire body,
constantly depressed mood. He considers himself good-for-nothing,
inadequate. Believes that he is a burden to his family, wants to die. The patient
is depressed, inactive, has a hypomimic face with sorrowful expression. He
speaks quietly and monotonely,gives short answers. What is the most likely
diagnosis?
A. Major depressive disorder
B. Initial stage of Alzheimers disease
C. Neurotic depression
D. Late-onset schizophrenia
E. Atherosclerotic depression

47. A patient, aged 16, complains of headache, mainly in the frontal and temporal
areas, superciliary arch, appearing of vomiting at the peak of headache, pain
during the eyeballs movement, joints pain. On examination: excited, to- 39°C,
Ps- 110/min. Tonic and clonus cramps. Uncertain meningeal signs. What is
the most likely diagnosis?
A. Influenza with cerebral edema manifestations
B. Parainfluenza
C. Adenovirus infection
D. Influenza, typical disease duration
E. Respiratory syncytial virus

48. A 64 y.o. patient has developed of squeering substernal pain which had
appeared 2 hours ago and irradiated to the left shoulder, marked weakness.
On examination: pale skin, cold sweat. Pulse- 108 bpm, AP- 70/50 mm Hg,
heart sound are deaf, vesicular breathing, soft abdomen, painless, varicouse
vein on the left shin, ECG: synus rhythm, heart rate is 100 bmp, ST-segment is
sharply elevated in II, III aVF leads. What is the most likely disorder?
A. Cardiogenic shock
B. Disquamative aortic aneurizm
C. Cardiac tamponade
D. Cardiac asthma
E. Pulmonary artery thromboembolia

49. A 64-year-old patient complains of severe pain in the right side of chest,
dyspnea, dry cough which appeared suddenly on exertion. Objectively: the
right side of the chest lags behind in the act of breathing. Percussion reveals
tympanic sound. Auscultation reveals pronouncedly diminished breath
sounds on the right. Ps- 100/min, weak, arrhythmic. AP- 100/50 mm Hg.
Cardiac sounds are decreased. What disease can be suspected in this patient?
A. Right-sided pneumothorax
B. Right-sided dry pleurisy
C. Right-sided hydrothorax
D. Right-sided pleuropneumonia
E. PATE

50. A 29-year-old female patient complains of dyspnea, heaviness and chest pain
on the right, body temperature rise up to 37,2°C. The disease is associated
with a chest trauma received 4 days ago. Objectively: skin is pale and moist.
Ps- 90 bpm, regular. Palpation reveals a dull sound on the right, auscultation
reveals significantly weakened vesicular breathing. In blood: RBCs -
2,8×1012/l, colour index - 0,9, Hb- 100 g/l, WBCs - 8,0×109/l, ESR - 17
mm/h. What results of diagnostic puncture of the pleural cavity can be
expected?
A. Haemorrhagic punctate
B. Transudate
C. Purulent punctate
D. Chylous liquid
E. Exudate

51. A 54-year-old drowned man was rescued from the water and delivered to the
shore. Objectively: the man is unconscious, pale, breathing cannot be
auscultated, pulse is thready. Resuscitation measures allowed to save the
patient. What complications may develop in the near future?
A. Pulmonary edema
B. Encephalopathy
C. Respiratory arrest
D. Cardiac arrest
E. Bronchospasm

52. An 18-year-old patient since childhood suffers from bleeding disorder after
minor injuries. His younger brother also has bleeding disorders with
occasional haemarthrosis. Which laboratory test will be informative for
diagnosis verification?
A. Clotting time
B. Thrombocyte count
C. Determination of prothrombin time
D. Fibrinogen rate
E. Blood clot retraction

53. A 28-year-old patient complains of periodic compressing heart pain. His


brother died at the age of 34 from a cardiac disease with similar symptoms.
Objectively: the patients skin is pale. Heart borders display no significant
deviations. Heart sounds are loud, there is a systolic murmur above all the
points with a peak above the aorta. Echocardioscopy reveals thickening of the
interventricular septum in the basal parts, reduction of left ventricular cavity.
What drug should be administered in order to prevent the disease
progression?
A. Metoprolol
B. Nitroglycerin
C. Furosemide
D. Captopril
E. Digoxin

54. A patient, aged 40, has been ill during approximately 8 years, complains of
pain in the lumbar part of the spine on physical excertion, in cervical and
thoracal part (especially when coughing), pain in the hip and knee joints on
the right. On examination: the body is fixed in the forward inclination with
head down, gluteal muscles atrophy. Spine roentgenography: ribs
osteoporosis, longitudinal ligament ossification. What is the most likely
diagnosis?
A. Ancylosing spondyloarthritis
B. Tuberculous spondylitis
C. Spondyloarthropatia on the background of Reiters disease
D. Spread osteochondrosis of the vertebral column
E. Psoriatic spondyloarthropatia

55. A worker, aged 38, working in the slate production during 15 years, complains
of expiratory exertional dyspnea, dry cough. On examination: deafening of the
percutory sounds in interscapular region, rough breath sounds, dry
disseminated rales. On fingers skin - greyish warts. Factorys sectorial doctor
suspects asbestosis. Which method is the most informative for diagnosis
verification?
A. Thorax roentgenography
B. Bronchoscopy
C. Bronchoalveolar lavage
D. Blood gases examination
E. Spirography

56. A 37 y.o. woman is suffering from squeezing substernal pain on physical


exertion. On examination: AP- 130/80 mm Hg, heart rate=pulse rate 72 bpm,
heart boarders are dilated to the left side, aortic systolic murmur. ECG- signs
of the left venticle hypertrophy. What method of examination is the most
informative in this case?
A. Echocardiography
B. Sphygmography
C. X-ray
D. Phonocardiography
E. Coronarography

57. A 58-year-old woman complains of osteoarthrosis of knee-joint. For 2 weeks


she had been receiving an in-patient medical treatment. She was discharged
from the hospital in satisfactory condition with complaints of minor pain after
prolonged static work. Local hyperemia and exudative effects in the area of
joints are absent. What further tactics is the most expedient?
A. Outpatient treatment
B. Refferral to MSEC
C. Orthopedist consultation
D. Repeated in-patient treatment
E. Conducting arthroscopy

58. A 36-year-old female has a 7-year history of pollen allergy. Over the last 2
years in August and September (during ragweed flowering), the patient has
had 2-3 asthma attacks that could be treated with one dose of salbutamol.
Objectively: body temperature - 36,5°C, respiratory rate - 18/min, Ps -
78/min, AP - 115/70 mm Hg. There is vesicular breathing above the lungs.
Cardiac sounds are sonorous, of regular rhythm. What drug would be most
effective to prevent asthma attacks during the critical season for the patient?
A. Intalum inhalation
B. Atrovent inhalation
C. Berotec inhalation
D. Suprastin administration
E. Theopecum administration

59. A 42-year-old patient applied to hospital with complaints of pain behind the
sternum with irradiation to the left scapula. The pain appears during
significant physical work, this lasts for 5-10 minutes and is over on rest. The
patient is sick for 3 weeks. What is the preliminary diagnosis?
A. IHD:First established angina pectoris
B. IHD:Stable angina pectoris of effort IV FC
C. IHD:Progressive angina pectoris
D. IHD:Variant angina pectoris (Prinzmetals)
E. IHD:Stable angina pectoris of effort I FC

60. Medical examination of a 19-year-old worker revealed generalized


lymphadenopathy mainly affecting the posterior cervical, axillary and ulnar
lymph nodes. There are multiple injection marks on the elbow bend skin. The
man denies taking drugs, the presence of injection marks ascribes to influenza
treatment. Blood count: RBCs- 3,2×1012/l, Hb- 100 g/l, WBCs- 3,1×109/l,
moderate lymphopenia. What study is required in the first place?
A. ELISA for HIV
B. Lymph node biopsy
C. Sternal puncture
D. Immunogram
E. X-ray of lungs

61. Medical examination of a 19-year-old worker revealed generalized


lymphadenopathy mainly affecting the posterior cervical, axillary and ulnar
lymph nodes. There are multiple injection marks on the elbow bend skin. The
man denies taking drugs, the presence of injection marks ascribes to influenza
treatment. Blood count: RBCs- 3,2×1012/l, Hb- 100 g/l, WBCs- 3,1×109/l,
moderate lymphopenia. What study is required in the first place?
A. Lymph node biopsy
B. ELISA for HIV
C. Sternal puncture
D. Immunogram
E. X-ray of lungs

62. A 24-year-old patient felt sick in 16 hours after dried fish intake. There was
nausea, vomiting, weakness, flabbiness, double vision. On physical exam,
there was decrease of a muscle tone, anisocoria, flaccid swallowing and tendon
reflex. What is the most probable diagnosis?
A. Botulism
B. Acute gastritis
C. Salmonellosis
D. Acute encephalitis
E. Food toxicoinfection

63. A 72-year-old patient after operation due to holecystectomia was prescribed


gentamicin (80 mg every 8 hours) and cephalothin (2 g every 6 hours) due to
fever. In 10 days there was an increase of creatinine up to 310 mumol/L. BP -
130/80 mm Hg, daily quantity of the urine is 1200 mL. Urine tests are without
pathology. Ultrasound: the size of kidneys is normal. What is the most
probable reason for renal failure?
A. Nephrotoxity of gentamicin
B. Acute glomerulonephritis
C. Unequal infusion of the liqiud
D. Hepatorenal syndrome
E. Cortical necrosis of kidneys

64. A 40 y.o. patient was admitted to the gasteroenterology with skin itching,
jaundice, discomfort in the right subcostal area, generalized weakness. On
examination: skin is jaundice, traces of scratches, liver is +5 cm, splin is 6x8
cm. In blood: alkaline phosphatase - 2,0 mmol/(hourL), general bilirubin - 60
mkmol/L, cholesterol - 8,0 mmol/L. What is the leading syndrome in the
patient?
A. Cholestatic
B. Cytolytic
C. Asthenic
D. Liver-cells insufficiency
E. Mesenchymal inflammatory

65. A 55-year-old male had been treated at the surgical department for acute
lower-extremity thrombophlebitis. On the 7th day of treatment he suddenly
developed pain in the left part of chest, dyspnea and cough. Body temperature
was 36,1°C, respiratory rate - 36/min. The patient was also found to have
diminished breath sounds without wheezing. Ps- 140/min, thready. AP- 70/50
mm Hg. The ECG shows QIII-S1 syndrome. What is the most likely diagnosis?
A. Pulmonary embolism
B. Bronchial asthma
C. Pneumothorax
D. Myocardial infarction
E. Cardiac asthma

66. A 51-year-old female is a weaving factory worker with 15 years of service


record. During a regular preventive examination she complained of frequent
headaches, poor sleep, tingling in the heart, irritability, rapid fatigability,
hearing impairment. For years, the noise level has exceeded the maximum
allowable concentration by 10-15 dB. A year ago, the patient underwent a
course of treatment for essential hypertension. Specify the most likely
diagnosis:
A. Noise disease
B. Neurasthenia
C. Arteriosclerotic encephalopathy
D. Asthenic-vegetative syndrome
E. Essential hypertension

67. A 26-year-old patient undergoes a course of treatment due to chronic


glomerulonephritis. The treatment was successful, normalization of all the
characteristics was recorded. What sanitorium and health resort treatment
could be recommended?
A. The south coast of the Crimea
B. Not recommended
C. Myrhorod
D. Truskavets
E. Morshyn
68. After a wasp-bite there was an itching of skin, hoarse voice, barking cough,
anxiety. On physical exam: there is edema of lips, eyelids, cyanosis. What
medicine is to be taken first?
A. Prednisolone
B. Adrenalin
C. Lasix
D. Seduxen
E. Euphylin

69. A 16-year-old adolescent was vaccinated with DTP. In eight days there was
stiffness and pain in the joints, subfebrile temperature, urticarial skin
eruption, enlargement of inguinal, cervical lymph nodes and spleen. What
kind of allergic reaction is observed?
A. Immunocomplex
B. Hypersensitivity of delayed type
C. Hypersensitivity of immediate type
D. Cytoxic

70. 2 weeks after recovering from angina a 29-year-old patient noticed face
edemata, weakness, decreased work performance. There was gradual progress
of dyspnea, edemata of the lower extremities, lumbar spine. Objectively: pale
skin, weakening of the heart sounds, anasarca. AP- 160/100 mm Hg. In urine:
the relative density - 1021, protein - 5 g/l, erythrocytes - 20-30 in the field of
vision, hyaline cylinders - 4-6 in the field of vision. What is the most likely
diagnosis?
A. Acute glomerulonephritis
B. Infectious allergic myocarditis
C. Myxedema
D. Essential hypertension
E. Acute pyelonephritis

71. A 56-year-old scientist experiences constricting retrosternal pain several times


a day while walking for 100-150 m. The pain lasts for up to 10 minutes and can
be relieved by nitroglycerine. Objectively: the patient is overweight, heart
borders exhibit no abnormalities, heart sounds are rhythmic, Ps- 78 bpm, AP-
130/80 mm Hg. ECG contains low amplitude of T wave in V4-5. What disease
might be suspected?
A. Stable FC III stenocardia
B. Stable FC I stenocardia
C. Stable FC IV stenocardia
D. Stable FC II stenocardia
E. Instable stenocardia
72. In autumn a 25-year-old patient developed stomach ache arising 1,5-2 hours
after having meals and at night. He complains of pyrosis and constipation.
The pain is getting worse after consuming spicy, salty and sour food, it can be
relieved by means of soda and hot-water bag. The patient has been suffering
from this disease for a year. Objectively: furred moist tongue. Abdomen
palpation reveals epigastrial pain on the right, resistance of abdominal
muscles in the same region. What is the most likely diagnosis?
A. Duodenal ulcer
B. Diaphragmatic hernia
C. Chronic pancreatitis
D. Stomach ulcer
E. Chronic cholecystitis

73. A 51 y.o. woman complains of dull pain in the right subcostal area and
epigastric area, nausea, appetite decline during 6 months. There is a history of
gastric peptic ulcer. On examination: weight loss, pulse is 70 bpm, AP is
120/70 mm Hg. Diffuse tenderness and resistance of muscles on
palpation.There is a hard lymphatic node 1x1cm in size over the left clavicle.
What method of investigation will be the most useful?
A. Esophagogastroduodenoscopy with biopsy
B. Stomach X-ray
C. pH-metry
D. Ultrasound examination of abdomen
E. Ureatic test

74. On the 20th of June a townsman was brought to clinic. The disease broke out
acutely, starting with fever, rise in temperature to 38-39°C. There also was
weakness, acute headache, nausea, vomiting, pain all over the body, sleep
disorder. On physical examination: hyperemia of skin of face, neck, thorax.
Meningeal signs are positive. 12 days ago the patient returned from the
Siberia, from the forest. What is the most probable diagnosis?
A. Tick-borne encephalitis
B. Pseudotuberculosis
C. Epidemic typhus
D. Influenza
E. Omsk hemorrhagic fever

75. A 37-year-old patient has sudden acute pain in the right epigastric area after
having fatty food. What method of radiological investigation is to be used on
the first stage of examining the patient?
A. Ultrasonic
B. Thermographic
C. Radionuclid
D. Roentgenological
E. Magnetic-resonance

76. A man, aged 68, complains of tiredness, sweating, enlargement of cervical,


submaxillary and axillary lymph nodes. Blood test: WBC- 35x109/L,
lymphocytes - 60%, Botkin and Gumprecht bodies, level of haemoglobin and
quantity of thrombocytes is normal. Myelogram showed 40% of lymphocytes.
What is the most probable diagnosis?
A. Chronic lympholeucosis
B. Chronic myeloleucosis
C. Acute leucosis
D. Tuberculous lymphadenitis
E. Lymphogranulomatosis

77. A 38-year-old patient is under observation having polyneuritic syndrome with


considerable loss of weight, fever, rise in BP. Blood test:: considerable
inflammatory changes. What examination is the most expedient to make the
diagnosis?
A. Muscular biopsy with histological investigation of the material
B. Blood culture
C. Determination of HLA antigens
D. Determination of antinuclear antibodies
E. Electromyography

78. A 32-year-old male patient has been suffering from pain in the sacrum and
coxofemoral joints, painfulness and stiffness in the lumbar spine for a year.
ESR- 56 mm/h. Roentgenography revealed symptoms of bilateral sacroileitis.
The patient is the carrier of HLA B27 antigen. What is the most likely
diagnosis?
A. Ankylosing spondylitis
B. Rheumatoid arthritis
C. Spondylosis
D. Reiters disease
E. Coxarthrosis

79. A 58-year-old female patient complains about periodical headache, dizziness


and ear noise. She has been suffering from diabetes mellitus for 15 years.
Objectively: heart sounds are rhythmic, heart rate is 76/min, there is diastolic
shock above aorta, AP is 180/110 mm Hg. In urine: OD- 1,014. Daily loss of
protein with urine is 1,5 g. What drug should be chosen for treatment of
arterial hypertension?
A. Inhibitor of angiotensin converting enzyme
B. alpha-blocker
C. Calcium channel antagonist
D. beta-blocker
E. Thiazide diuretic

80.A 26 y.o. male patient with postoperative hypothyroidism take thyroxine 100
mg 2 times a day. He has developed tachycardia, sweating, irritability, sleep
disorder. Determine further treatment tactics.
A. To decrease thyroxine dosage
B. To administer betablockers
C. To administer sedatives
D. To add mercasolil to the treatment
E. To increase thyroxine dosage

81. A 28-year-old man was discharged from the hospital after having an out-of -
hospital pneumonia. He has no complaints. On physical exam: his
temperature is 36,6°C, RR-18/min, Ps - 78 bpm, BP- 120/80 mm Hg. During
ausculation there is harsh respiration to the right of the lower part of the lung.
Roentgenologically: infiltrative changes are absent, intensification of the
pulmonary picture to the right in the lower lobe. How long should the doctor
keep the patient under observation?
A. 12 months
B. 3 months
C. Permanently
D. 6 months
E. 1 month

82. A 20-year-old adolescent lives in the nidus of tuberculous infection. The


tuberculine Mantoux test with 2 TU was determined as hyperergic. What signs
determine the hyperergic test of this adolescent?
A. 6 mm papula, necrosis
B. 24 mm hyperemia
C. 12 mm hyperemia
D. 4 mm papula
E. 20 mm papula

83. A survey radiograph of a miner (24 years of service record, the dust
concentration in the workplace is at the rate of 260-280 mg/m3 with 15% of
free silica) shows lung changes that are typical for pneumoconiosis. What type
of pneumoconiosis is it?
A. Anthracosilicosis
B. Anthracosilicatosis
C. Silicosis
D. Carboconiosis
E. Silicatosis
84. A patient complains of pathological lump, appearing in the right inguinal
region on exercise. The lump is round-shaped, 4 cm in diameter, on palpation:
soft elastic consistency, is positioned near the medial part of Pouparts
ligament. The lump is situated inwards from the spermatic cord. What is the
most probable preliminary diagnosis?
A. Right-sided direct inguinal hernia
B. Lipoma of the right inguinal area
C. Right-sided femoral hernia
D. Right-sided oblique inguinal hernia
E. Varicose veins of the right hip

85. A 35-year-old man was operated on peptic ulcer of the stomach. Mass deficit
of the body is 10 kg. The level of glucose after operation in the undiluted
cellular blood on an empty stomach is 6,7 mmol. During repeated examination
- 11,1 mmol (after meal), level of HbA1c - 10%. Could you please make an
interpretation of the given data?
A. Diabetes mellitus
B. Diabetes mellitus risk group
C. Postoperative hyperinsulinemia
D. Norm
E. Disordered tolerance to glucose

86. A 52 y.o. woman complains of weakness, painful itching after washing and
bathing, sensation of heaviness in the head. On examination: hyperemia of
skin of face, neck, extremities.BP- 180/100 mm Hg. Spleen is 4 cm below the
rib arch edge. What is the most probable diagnosis?
A. Erythremia
B. Dermatomyositis
C. Essential hypertension
D. Allergic dermatitis
E. Systemic sclerodermia

87. A 37-year-old patient was brought to resuscitation unit. General condition of


the patient is very serious. Sopor. The skin is grey, moist. Turgor is decreased.
Pulse is rapid, intense. BP - 160/110 mm Hg, muscle tonus is increased.
Hyperreflexia. There is an ammonia odor in the air. What is the presumptive
diagnosis?
A. Uraemic coma
B. Cerebral coma
C. Hyperglycemic coma
D. Alcoholic coma
E. Hypoglycemic coma
88. A 57-year-old man complains of shortness of breath, swelling on shanks,
irregularity in cardiac work, pain in the left chest half with irradiation to the
left scapula.Treatment is uineffective. On physical exam: hearts sounds are
diminished, soft systolic murmur on the apex. Ps - 100/min, arrhythmical, BP
- 115/75 mm Hg. The liver is +2 cm, painful. Roentgenoscopy: enlargement of
heart shadow to all sides, pulsation is weak. Electrocardiogram (ECG):
leftventricled extrasystolia, decreased voltage. What method of investigation is
necessary to do to determine the diagnosis?
A. Echocardiography
B. X-ray kymography
C. Coronarography
D. ECG in the dynamics
E. Veloergometria

89. A 55 y.o. patient complains of distended abdomen and rumbling, increased


winds evacuation, liquid foamy feces with sour smell following the diary
products consumption. What is the correct name of this syndrome?
A. Syndrome of fermentative dyspepsia
B. Dyskinesia syndrome
C. Malabsorption syndrome
D. Syndrome of decayed dyspepsia
E. Syndrome of fatty dyspepsia

90. A 54-year-old patient has an over 20-year history of femoral osteomyelitis.


Over the last month she has developed progressing edemata of the lower
extremities. Urine test reveals: proteinuria at the rate of 6,6 g/l; in blood:
dysproteinemia in form of hypoalbuminemia, increase in a2- and γ-globulin
rate, ESR - 50 mm/h. What is the most likely diagnosis?
A. Secondary renal amyloidosis
B. Chronic glomerulonephritis
C. Systemic lupus erythematosus
D. Acute glomerulonephritis
E. Myelomatosis

91. In an inhabited locality there is an increase of diphtheria during the last 3


years with separate outbursts in families. What measure can effectively
influence the epidemic process of diphtheria and reduce the morbidity rate to
single cases?
A. Immunization of the population
B. Disinfection in disease focus
C. Detection of carriers
D. Hospitalization of patients
E. Early diagnostics

92. A 14-year-old victim was drawn out of the water in winter after 15 minutes of
being in the water. The victim shows no vital signs. What measures are to be
taken?
A. To release respiratory tract from water, to create drain position
and to take on measures to restore respiration and blood
circulation
B. To transport the victim to the nearest warm room to carry out reanimation
measures
C. Not to carry out reanimation measures
D. Not to waste time on the release of respiratory tract from water, to take on
cardiopulmonary reanimation
E. To transport the victim to the nearest hospital to carry out reanimation
measures

93. An electro-gas welding operator working at a machine workshop performs


welding and cutting of metal, which is accompanied by intense UV-radiation.
His welding station is equipped with effective mechanical ventilation. What
occupational disease is most likely to develop in an electro-gas welding
operator?
A. Photoelectric ophthalmia
B. Vegetative-vascular dystonia
C. Heatstroke
D. Chronic overheating
E. Pneumoconiosis

94. A woman complains of high temperature to 38°C, mild pain in the throat
during 3 days. On examination: angle lymphatic nodes of the jaw are 3 cm
enlarged, palatinel tonsils are enlarged and coated with grey plaque which
spreads to the uvula and frontal palatinel arches. What is the most probable
diagnosis?
A. Larynx dyphtheria
B. Agranulocytosis
C. Oropharyngeal candidosis
D. Infectious mononucleosis
E. Vincents angina

95. A 48-year-old male patient complains of constant pain in the upper abdomen,
mostly on the left, that is getting worse after taking meals; diarrhea, weight
loss. The patient is an alcohol abuser. 2 years ago he had acute pancreatitis.
Blood amylase is 4 g/hl. Coprogram shows steatorrhea, creatorrhea. Blood
glucose is 6,0 mmol/l. What treatment is indicated for this patient?
A. Panzinorm forte
B. Contrycal
C. No-spa
D. Insulin
E. Gastrozepin

96. A 60-year-old female patient had been admitted to a hospital for acute
transmural infarction. An hour later the patients contition got worse. She
developed progressing dyspnea, dry cough. Respiratory rate - 30/min, heart
rate - 130/min, AP- 90/60 mm Hg. Heart sounds were muffled, there was also
diastolic shock on the pulmonary artery. The patient presented with medium
moist rales in the lower parts of lungs on the right and on the left. Body
temperature - 36,4°C. What drug should be given in the first place?
A. Promedol
B. Dopamine
C. Aminophylline
D. Heparin
E. Digoxin

97. A 62-year-old male has been hospitalized in the intensive care unit with a
continuous attack of retrosternal pain that cannot be relieved by nitroglycerin.
Objectively: AP- 80/60 mm Hg, heart rate - 106/min, breathing rate - 22/min.
Heart sounds are muffled, a gallop rhythm is present. How would you explain
the AP drop?
A. Reduction in cardiac output
B. Reduction in peripheral resistance
C. Adrenergic receptor block
D. Internal haemorrhage
E. Blood depositing in the abdominal cavity

98. A 35-year-old female reports heart pain (aching and drilling) occurring mainly
in the morning in autumn and spring and irradiating to the neck, back and
abdomen; rapid heartbeat; low vitality. Occurrence of this condition is not
associated with physical activity. In the evening, the patients condition
improves. Study of somatic and neurological status, and ECG reveal no
pathology. What pathology is most likely to have caused these clinical
presentations?
A. Somatization depression
B. Pseudoneurotic schizophrenia
C. Resting stenocardia
D. Neurocirculatory asthenia
E. Hypochondriacal depression
99. A 18 y.o. male patient complains of pain in knee and ankle joints, temperature
elevation to 39,5°C. He had a respiratory disease 1,5 week ago. On
examination: temperature- 38,5°C, swollen knee and ankle joints, pulse- 106
bpm, rhythmic, AP- 90/60 mm Hg, heart borders without changes, sounds are
weakened, soft systolic apical murmur. What indicator is connected with
possible etiology of the process?
A. Antistreptolysine-0
B. Rheumatic factor
C. Seromucoid
D. 1-antitrypsine
E. Creatinkinase

100. A 30 y.o. male patient complains of itching of the skin which intensifies
in the evening. He has been ill for 1,5 months. On examination: there is rash
with paired papules covered with bloody crusts on the abdomen, hips,
buttocks, folds between the fingers, flexor surfaces of the hand. There are
traces of line scratches. What additional investigations are necessary to make
diagnosis?
A. Examination of rash elements scrape
B. Blood glucose
C. Examination for helmints
D. Determination of dermographism
E. Serologic blood examination

101. A 50-year-old locksmith has a long-term record of work under the


effect of mercury vapors with concentration exceeding MPC by 5-10 times.
Clinical examination revealed the lability of vasomotors of skin, pulse and
arterial pressure; total hyperhydrosis; asymmetric innervation of facial and
lingual muscles, positive subcortical reflexes, intention tremor. Against the
background of increased emotional excitability the patient presents with lack
of self-confidence, shyness. A dentist found him to have parodontosis, chronic
stomatitis. What disease can be suspected?
A. Chronic mercury intoxication
B. Parkinsons syndrome
C. Vascular encephalopathy
D. Acute mercury intoxication
E. Residual effects of neuroinfection

102. 4 hours after having meals a patient with signs of malnutrition and
steatorrhea experiences stomach pain, especially above navel and to the left of
it. Diarrheas take turns with constipation lasting up to 3-5 days. Palpation
reveals moderate painfulness in the choledochopancreatic region. The
amylase rate in blood is stable. X-ray reveals some calcifications located above
navel. What is the most likely diagnosis?
A. Chronic pancreatitis
B. Duodenal ulcer
C. Chronic gastroduodenitis
D. Zollinger-Ellison syndrome
E. Chronic calculous cholecystitis

103. A 58 y.o. male patient is examined by a physician and suffers from


general weakness, fatigue, mild pain in the left subcostal area, sometimes
frequent painful urination. Moderate splenomegaly has been revealed. Blood
test: neutrophilic leukocytosis with the progress to myelocyte; basophil- 2%;
eosinophil- 5%. There is a urate crystales in urine, erythrocyte- 2-3 in the field
of vision. What is the preliminary diagnosis?
A. Chronic myeloleucosis
B. Lymphogranulomatosis
C. Urolithiasis
D. Hepar cirrhosis
E. Leukemoid reaction

104. A 56-year-old patient with diffuse toxic goiter has ciliary arrhythmia
with pulse rate of 110 bpm, arterial hypertension, AP- 165/90 mm Hg. What
preparation should be administered along with mercazolil?
A. Propranolol
B. Radioactive iodine
C. Verapamil
D. Corinfar
E. Procaine hydrochloride

105. A 54-year-old patient complains of weakness, weight loss despite the


unchanged appetite, frequent urination, skin itch for six months. Some time
ago the patient underwent treatment for furunculosis. She hasnt been
examined recently. Objectively: malnutrition, dry skin with signs of
scratching. Small lymph nodes can be palpated in the axillary regions.
Changes in the internal organs are absent. What testing must be administered
in the first place?
A. Blood sugar test on an empty stomach
B. Complete blood count
C. Lymph node biopsy
D. Blood sterility testing
E. Endoscopy of stomach

106. A 43 y.o. woman complains of severe pain in the right abdominal side
irradiating in the right supraclavicular area, fever, dryness and bitterness in
the mouth. There were multiple vomitings without relief. Patient relates the
onset of pain to the taking of fat and fried food. Physical examination: the
patient lies on the right side, pale, dry tongue, tachycardia. Right side of
abdomen is painful during palpation and somewhat tense in right
hypochondrium. What is the most likely diagnosis?
A. Perforative ulcer
B. Acute bowel obstruction
C. Acute cholecystitis
D. Acute appendicitis
E. Right-sided renal colic

107. Several hours before, a 28-year-old patient suddenly developed acute


headache and repeated vomiting, then lost consciousness. Objectively: focal
neurological symptoms were not found. Pronounced meningeal symptoms
were revealed. AP - 120/80 mm Hg. According to clinical and liquorological
findings the patient was diagnosed with subarachnoid haemorrhage. After
administration of dehydrants the patients condition somewhat improved.
What is the main component of further emergency care?
A. Coagulants
B. Antiaggregants
C. Corticosteroids
D. Fibrinolytics
E. Anticoagulants

108. On the 5th day of a respiratory disease accompanied by fever a 24-year-


old man developed progressing headaches, systemic dizziness, double vision,
facial muscles paresis on the right, choking from swallowing. He was
diagnosed with acute viral encephalitis. Identify the main tendency of the
emergency treatment:
A. Zovirax
B. Haemodezum
C. Ceftriaxone
D. Glucocorticoids
E. Lasix

109. A 24-year-old man on the 5th day of acute respiratory disease with high
grade temperature started having strong headaches, systemic dizziness,
sensation of double vision, paresis of mimic muscles to the right, tickling by
swallowing. Diagnosis: Acute viral encephalitis. Determine the basic direction
of the emergent therapy.
A. Zovirax
B. Lasix
C. Hemodesis
D. Glucocorticoids
E. Cephtriaxon
110. A 30-year-old patient was delivered to the admission ward of the
infectious disease department. The disease had started acutely on the
background of normal temperature with the appearance of frequent, liquid,
profuse stool without pathological impurities. Diarrhea was not accompanied
by abdominal pain. 12 hours later there appeared recurrent profuse vomiting.
The patient rapidly developed dehydration. What is the most likely diagnosis?
A. Cholera
B. Shigellosis
C. Salmonellosis
D. Campylobacteriosis
E. Staphylococcal food toxicoinfection

111. A 65 y.o. woman complains of complicated mouth opening following


foot trauma 10 days ago. Next day she ate with difficulties, there were muscles
tension of back, the back of the head and abdomen. On the third day there was
tension of all muscle groups, generalized convulsions every 10-15 min. What is
the most probable diagnosis?
A. Tetanus
B. Meningoencephalitis
C. Tetania
D. Hemorrhagic stroke
E. Epilepsy

112. Gastric juice analysis of a 42-year-old male patient revealed absence of


free hydrochloric acid at all stages. Endoscopy revealed pallor, thinning of
gastric mucosa, smoothed folds. Microscopically the atrophy of glands with
intestinal metaplasia was found. What disease is this situation typical for?
A. Chronic type A gastritis
B. Menetrier disease
C. Stomach cancer
D. Chronic type B gastritis
E. Chronic type C gastritis

113. A 45-year-old female patient complains of frequent liquid stools with a


lot of mucus, pus and blood; pain across the abdomen, loss of 7 kg within 6
months. She has a 1-year history of non-specific ulcerative colitis. What group
of drugs should be preferred for this patient?
A. Corticosteroids
B. Sulfonamides
C. Polyenzymes
D. Nitrofurans
E. Antibacterial
114. A 54-year-old male patient complains of aching pain in the lumbar
region, that is getting worse after standing in an upright position, physical
exercise, supercooling. The patient also reports of experiencing weakness in
the afternoon. Pain in the lumbar region, said about 10 years old. Objectively:
pale skin, to- 37,2°C, AP- 180/100 mm Hg, minor costovertebral angle
tenderness (Pasternatsky symptom). In blood: RBCs - 3,5×1012/l, WBCs -
6,5×109/l, ESR - 22 mm/h. In urine: the relative density - 1010, leukocytes -
12-15 in the field of vision, erythrocytes - 2-3 in the field of vision. Urine
bacterial count - 100000 in 1 ml. What is the most likely diagnosis?
A. Chronic pyelonephritis
B. Amyloidosis
C. Polycystic renal disease
D. Nephrolithiasis
E. Chronic glomerulonephritis

115. A 67-year-old male complains of dyspnea on exertion, attacks of


retrosternal pain, dizziness. He has no history of rheumatism. Objectively:
pale skin, acrocyanosis. There are rales in the lower parts of lungs. There is
systolic thrill in the II intercostal space on the right, coarse systolic murmur
conducted to the vessels of neck. AP- 130/90 mm Hg, heart rate - 90/min,
regular rhythm. The liver extends 5 cm under the edge of costal arch, shin
edemata are present. Specify the assumed valvular defect:
A. Aortic stenosis
B. Tricuspid regurgitation
C. Mitral insufficiency
D. Pulmonary artery stenosis
E. Ventricular septal defect

116. A 24-year-old female teacher complains of dizziness and heart pain


irradiating to the left nipple. Pain is not associated with physical activity and
cannot be relieved by nitroglycerin, it abates after taking Valocordin and lasts
an hour or more. The patient has a nearly 2-year history of this disease.
Objectively: Ps- 76 bpm. AP- 110/70 mm Hg. Heart borders are normal, heart
sounds are clear. The ECG shows respiratory arrhythmia. Radiograph of the
cervicothoracic spine shows no pathology. Lungs, abdomen are unremarkable.
What changes in blood formula can be expected?
A. No changes
B. Leukocytosis
C. Leukemic hiatus
D. Increased ESR
E. Thrombocytopenia
117. A 51-year-old female patient complains of frequent defecation and
liquid blood-streaked stools with mucus admixtures, diffuse pain in the
inferolateral abdomen, 6 kg weight loss over the previous month. Objectively:
body temperature - 37,4°C, malnutrition, skin is pale and dry. Abdomen is
soft, sigmoid is painful and spasmodic, makes a rumbling sound. Liver is
dense, painful, extends 3 cm below the costal margin. What is the most likely
diagnosis?
A. Non-specific ulcerative colitis
B. Sprue
C. Bacillary dysentery
D. Intestinal enzymopathy
E. Helminthic invasion

118. A 18 y.o. female student complains of dyspnea during the intensive


exertion. The condition became worse half a year ago. On examination: pulse
rate is 88 bpm, accelerated, AP- 180/20 mm Hg, pale skin, heart borders are
dilated to the left and up. There is systolic-diastolic murmur in the 2hd
intercostal space, S2 at pulmonary artery is accentuated. ECG has revealed
both ventricles hypertrophy. Thoracic X-ray has revealed pulsation and
protrusion of the left ventricle, lung trunk. What doctors tactics should be?
A. Cardiosurgeon consultation
B. Administration of therapeutic treatment
C. Exemption from physical exercises
D. Continuation of investigation
E. Dispensary observation

119. A 49-year-old male patient complains of dyspnea of combined nature,


cough, shin edemata, abdomen enlargement due to ascites. He has a 20-year
history of chronic bronchitis. For the last 3 years he has been disabled (group
II) because of cardiac changes. Objectively: mixed cyanosis, edemata. Ps -
92/min, rhythmic, AP - 120/70 mm Hg, respiration rate - 24/min. There is
accentuation of the second sound above the pulmonary artery. Auscultation
reveals the box resonance above the lungs. There are also dry rales over the
entire surface of lungs. What is the mechanism of heart changes development
in this patient?
A. Euler-Liljestrand reflex
B. Cardiovascular reflex
C. Respiratory reflex
D. Kitaevs reflex
E. Bainbridge reflex

120. A 24-year-old man on the second day of the disease with a sudden
onset complains of a strong headache in temples and in the area of orbits, dull
pain in the body, dry painful cough. His temperature is 39°C. Adynamic.
Mucous membrane of oropharynx is "flaming", rales are not ausculated. What
is the most probable diagnosis?
A. Influenza
B. Respiratory mycoplasmosis
C. Meningococcus infection
D. Pneumonia
E. Parainluenza

121. A 32-year-old female complains of dizziness, headache, palpitation,


tremor. For the last several months she has been under outpatient observation
for the increased arterial pressure. Since recently such attacks have become
more frequent and severe. Objectively: skin is covered with clammy sweat,
tremor of the extremities is present. HR- 110/min, AP- 220/140 mm Hg.
Heart sounds are muffled. Blood test results: WBCs - 9,8×109/l, ESR - 22
mm/h. Blood glucose - 9,8 millimole/l. What disease is the most likely cause
of this crisis?
A. Pheochromocytoma
B. Preeclampsia
C. Essential hypertension
D. Primary hyperaldosteronism
E. Diabetic glomerulosclerosis

122. A 76-year-old male consulted a therapist about slow discharge of urine


with a small jet. The patient reported no cardiac problems. Examination
revealed atrial fibrillation with a heart rate of 72/min and without pulse
deficit. There are no signs of heart failure. ECG confirms the presence of atrial
fibrillation. From history we know that the arrhythmia was detected three
years ago. What tactics for the treatment of atrial fibrillation in the patient
should be chosen?
A. Does not require treatment
B. Obzidan
C. Ajmaline
D. Digoxin
E. Verapamil

123. A 43-year-old female patient complains of unstable defecation with


frequent constipations, abdominal swelling, headache, sleep disturbance.
Body weight is unchanged. What disease are these clinical presentations
typical for?
A. Irritable colon syndrome
B. Chronic pancreatitis
C. Colorectal cancer
D. Chronic atrophic gastritis
E. Chronic enteritis
124. A 43-year-old man who often contacts with ethyl gasoline was admitted
to a hospital with complaints of general weakness, dizziness, memory
impairment, sleepiness at daytime and insomnia at night, sensation of a hair
in the mouth, colicky pains in the right subcostal region. What is the most
likely diagnosis?
A. Chronic tetraethyl lead intoxication
B. Chronic lead intoxication
C. Chronic mercury intoxication
D. Alcoholic delirium
E. Chronic manganese intoxication

125. A 35-year-old patient has been in the intensive care unit for acute renal
failure due to crush for 4 days. Objectively: the patient is inadequate.
Breathing rate - 32/min. Over the last 3 hours individual moist rales can be
auscultated in lungs. ECG shows high T waves, right ventricular extrasystoles.
CVP - 159 mm Hg. In blood: the residual nitrogen - 62 millimole/l, K+- 7,1
millimole/l, Cl- - 78 millimole/l, Na+- 120 millimole/l, Ht - 0,32, Hb - 100 g/l,
blood creatinine - 0,9 millimole/l. The most appropriate method of treatment
would be:
A. Hemodialysis
B. Hemosorption
C. Ultrafiltration
D. Plasma filtration
E. Plasma sorption

126. A 45-year-old man was brought to clinic with complaints of the pain
that started suddenly in the left chest part and epigastric area, shortness of
breath, nausea, one-time vomiting. The acute pain started after weight-lifting.
On physical exam: shallow breathing, RR - 38/min, left chest part is behind
during respiration, by percussion - tympanitic sound, respiration is not
ausculated. Ps - 110 bpm, of weak filling. BP- 100/60 mm Hg, insignificant
displacement of heart to the right, sounds are dull. What examination is the
most expedient to do first?
A. Roentgenography
B. Esophagogastroscopy
C. Ultrasound of the abdominal cavity
D. Electrocardiography
E. Bronchoscopy

127. A 35 y.o. woman is suspected of aplastic anemia. The bone marrow


punction has been administered with the diagnostic purpose. What changes in
the marrow punctatum are suggested?
A. Replacement of marrow elements with adipose tissue
B. Absolute lymphocytosis
C. Prevalence of megaloblasts
D. Replacement of marrow elements with fibrous tissue
E. Presence of blast cells

128. A 58-year-old female patient complains of spontaneous bruises,


weakness, bleeding gums, dizziness. Objectively: the mucous membranes and
skin are pale with numerous hemorrhages of various time of origin. Lymph
nodes are not enlarged. Ps is 100/min, AP - 110/70 mm Hg. There are no
changes of internal organs. Blood test results: RBC - 3,0*10^12/l, ?b - 92 g/l,
colour index - 0,9, anisocytosis, poikilocytosis, WBC – 10*10^9/l, eosinophils
- 2%, stab neutrophils - 12%, segmented neutrophils - 68%, lymphocytes -
11%, monocytes - 7%, ESR - 12 mm/h. What laboratory test is to be
determined next for making a diagnosis?
A. Platelets
B. Fibrinogen
C. Clotting time
D. Reticulocytes
E. Osmotic resistance of erythrocytes

129. A 47-year-old woman underwent a thyroid gland resection on account


of nodular euthyroid goiter. What preparations are most likely to prevent the
disease recurrence?
A. Thyroid hormones
B. Thyrotropin
C. Mercazolil
D. Antistruminum (potassium iodide)
E. Radioactive iodine

130. A 55 y.o. male patient complains of weakness during 2 months, pain in


the right side of the thorax, cough, blood-streaked sputum. On X-ray:
intensive triangle shadow in the area of lower lobe that is connected to
mediastinum. What is the most likely disorder in the lungs?
A. Central cancer of lungs
B. Pleuropneumonia
C. Bronchiectasia
D. Tuberculosis of lungs
E. Pulmonary infarction

131. A 60 y.o. patient experiences acute air insufficiency following of the


venoectomy due to subcutaneous vein thrombophlebitis 3 days ago. Skin
became cianotic, with grey shade. Marked psychomotor excitement,
tachypnea, substernal pain. What postoperative complication has occured?
A. Thromboembolia of pulmonary artery
B. Hypostatic pneumonia
C. Hemorrhagia
D. Myocardial infarction
E. Valvular pneumothorax

132. A 19-year-old woman complains of pain in the abdomen and joints,


asks for more analgetics and somnifacient injections. The patient was
examined. Gynecological and urological pathologies are absent. There are
signs of previous punctures along superficial veins of the extremities. The
patient does not explain the origin of punctures. Tendon reflexes of upper and
lower extremities are the same, quick. Photoreaction of the pupil of the eye is
weak. The tongue is grey coated. During communication the patient in
affectively not even-tempered. There is diarrhea without pathologic
inclusions. What tactics is necessary to improve the condition of this patient?
A. Consultation of an expert in narcology
B. Consultation of infectious diseases doctor
C. Additional consultation of surgeon
D. Prescription of medications the patient asks for
E. Treatment with antibiotics

133. A patient has an over a year-old history of fast progressive rheumatoid


arthritis. X-raying confirms presence of marginal erosions. What basic drug
would be the most appropriate in this case?
A. Methotrexate
B. Chloroquine
C. Diclofenac sodium
D. Aspirin
E. Prednisolone

134. A female rheumatic patient experiences diastolic thoracic wall tremor


(diastolic thrill), accentuated S1 at apex, there is diastolic murmur with
presystolic intensification, opening snap, S2 accent at pulmonary artery. What
rind of heart disorder is observed?
A. Mitral stenosis
B. Mitral valve insufficiency
C. Opened arterial duct
D. Aortic valve insufficiency
E. Pulmonary artery stenosis
135. A 23-year-old patient complains of a dull ache, sensation of heaviness
and distention in the epigastrium immediately after meals, foul-smelling
eructation; dry mouth, empty stomach nausea, diarrhea. Objectively: the skin
is pale, the patient is of thin build. Abdomen is soft on palpation, there is
epigastric pain. The liver does not extend beyond the costal arch. In blood: Hb
- 110 g/l, RBCs - 3,4×1012/l, WBC count is normal. ESR - 16 mm/h. What is
the most informative study that will allow make a diagnosis?
A. Esophageal gastroduodenoscopy
B. Study of gastric juice
C. Duodenal probing
D. pH-metry
E. X-ray of digestion organs

136. A 49-year-old patient complains of deglutition problems, especially


with solid food, hiccups, voice hoarseness, nausea, regurgitation, significant
weight loss (15 kg within 2,5 months). Objectively: body weight is reduced.
Skin is pale and dry. In lungs: vesicular breathing, heart sounds are loud
enough, heart activity is rhythmic. The abdomen is soft, painless on palpation.
Liver is not enlarged. What study is required to make a diagnosis?
A. Esophageal duodenoscopy along with biopsy
B. X-ray of digestive tract organs
C. Study of gastric secretion
D. X-ray in Trendelenburgs position
E. Clinical blood test

137. A 60-year-old patient has been admitted to a hospital with complaints


of dyspnea, tightness in the right subcostal area, abdomen enlargement. These
presentations have been progressing for a year. Heart auscultation reveals
presystolic gallop rhythm. Objectively: swelling of the neck veins, ascites,
palpable liver and spleen. What disease requires differential diagnostics?
A. Constrictive pericarditis
B. Pulmonary embolism
C. Lung cancer with invasion to the pleura
D. Hepatocirrhosis
E. Chronic pulmonary heart

138. A 40-year-old patient, the forester, complains of severe headache, body


temperature rise up to 39,5°C, trembling limbs. From the patients history we
know that he had seriously cut his hand during the dissection of a killed fox.
Objectively: depressed mood. The patient asks not to turn on the light or open
the door. Any noise causes apparent motor excitation. When he saw a carafe of
water, he developed convulsive throat spasms. What tactics should an
emergency doctor choose?
A. Deliver the patient to the infectious disease hospital
B. Deliver the patient to the neurological department
C. Let him stay at home and consult a psychiatrist
D. Deliver the patient to the psychiatric hospital
E. Deliver the patient to the resuscitation department

139. A 28-year-old woman has a 12-year history of chronic


glomerulonephritis with latent course. Over the past six months she has
developed general weakness, loss of appetite, low work performance, nausea.
The patient complains of headache, pain in the joints. On examination:
anemia, blood urea - 34,5 millimole/l, blood creatinine - 0,766 millimole/l,
hyperkalemia. What complication has developed?
A. Chronic renal insufficiency
B. Renal amyloidosis
C. Pyelonephritis
D. Acute renal insufficiency
E. Nephrotic syndrome

140. A 72-year-old male had had a moderate headache. Two days later, he
developed the progressing speech disorders and weakness in the right
extremities. The patient has a history of myocardial infarction, arrhythmia.
Study of the neurologic status revealed elements of motor aphasia, central
paresis of the VII I XII cranial nerves on the right, central hemiparesis on the
same side and hyperaesthesia. What is the most likely diagnosis?
A. Ischemic stroke
B. Transient ischemic attack
C. Hemorrhagic stroke
D. Epidural hematoma
E. Brain tumor

141. A 70 y.o. male patient with mild headaches complains of speech


disorder, weakness in right limbs. There was a history of miocardial infarction
and arrhythmia. On nu eroligical examination there are elements of motor
aphasia, central paresis of VII and XII cranial nerves pairs on the right side,
cental type of hemiparesis and hemihyperesthisia on the same side. What is
the most probable diagnosis?
A. Ischemic stroke
B. Cerebral tumor
C. Transitory ischemic attack
D. Hemorrhagic stroke
E. Epidural hematoma

142. After treating a field with pesticides a machine operator presents with
great weakness, headache, nausea, vomiting, diarrhea, visual impairment,
watery eyes. Objectively: the patient is excited, hypersalivation, hyperhidrosis,
muscle fibrillation of tongue and eyelids are oberved. Pupils are narrowed,
there is tachycardia, lung auscultation reveals moist small and medium
bubbling rales. In blood: changed level of cholinesterase activity. What is the
most likely diagnosis?
A. Intoxication with organophosphorous pesticides
B. Intoxication with carbamic acid derivatives
C. Intoxication with organomercurial pesticides
D. Intoxication with organochlorine pesticides
E. Intoxication with arsenic-containing pesticides

143. A 40-year-old man is ill with autoimmune hepatitis. Blood test: A/G
ratio 0,8, bilirubin – 42 mumol/L, transaminase: ALT- 2,3 mmol g/L, AST -
1,8 mmol g/L. What is the most effective means in treatment from the given
below?
A. Glucocorticoids, cytostatics
B. Antibacterial medication
C. Antiviral medications
D. Hemosorbtion, vitamin therapy
E. Hepatoprotectors

144. A farmer hurt his right foot during working in a field and came to the
emergency station. He doesnt remember when he got last vaccination and he
has never served in the army. Examination of his right foot revealed a
contaminated wound up to 5-6 cm long with uneven edges. The further
treatment tactics will be:
A. To make an injection of tetanus anatoxin and antitetanus serum
B. To make an injection of antitetanus serum
C. To administer an antibiotic
D. Surgical d-bridement only
E. To make an injection of tetanus anatoxin

145. A 35-year-old patient has been admitted to a hospital for pain in the left
sternoclavicular and knee joints, lumbar area. The disease has an acute
character and is accompanied by fever up to 38°C. Objectively: the left
sternoclavicular and knee joints are swollen and painful. In blood: WBCs -
9,5×109/l, ESR - 40 mm/h, CRP - 1,5 millimole/l, fibrinogen - 4,8 g/l, uric
acid - 0,28 millimole/l. Examination of the urethra scrapings reveals
chlamydia. What is the most likely diagnosis?
A. Reiters syndrome
B. Rheumatoid arthritis
C. Gout
D. Rheumatic arthritis
E. Bechterews disease
146. A 20 daily y.o. female patient is suffering from chronic bronchitis.
Recently there has been production about 0,5 L of purulent sputum with
maximum discharge in the morning. Fingers are like "drum sticks", there are
"watching glass" nails. What is the most probable diagnosis?
A. Bronchiectasia
B. Pneumonia
C. Gangrene of lungs
D. Tuberculosis
E. Chronic bronchitis

147. Topographic percussion of lungs in a patient who got a serious job-


related barotrauma revealed that the lower lungs borders were located one rib
below normal, there was a significant increase in both lungs height and
Kronigs isthmus. What disease should be suspected in the first place?
A. Pulmonary emphysema
B. Exudative pleuritis
C. Bronchial asthma
D. Pneumothorax
E. Chronic bronchitis

148. An 18 y.o. girl complains of weakness, dizziness, loss of appetite,


menorrhagia. There are many-coloured petechiae on the skin of the upper
extremities. Blood test: Hb- 105 g/l; RBC- 3,2×1012/L; C.I.- 0,95; thromb.-
20×109/L. The sedimentation time according to Lee White is 5; hemorrhagia
duration according to Duke is 8, "pinch and tourniquet" test is positive. What
is the most probable diagnosis?
A. Idiopathic thrombocytopenic purpura
B. Hemorrhagic diathesis
C. Hemophilia
D. Iron deficiency anemia
E. Marchiafava-Michelis disease

149. A 28 y.o. male patient was admitted to the hospital because of high
temperature 39°C, headache, generalized fatigue, constipation, sleep disorder
for 9 days. There are sporadic roseolas on the abdomen, pulse- 78 bpm, liver is
enlarged for 2 cm. What is the most probable diagnosis?
A. Abdominal typhoid
B. Leptospirosis
C. Sepsis
D. Typhus
E. Brucellosis
150. A 50-year-old patient complains about having pain attacks in the right
subcostal area for about a year. He pain arises mainly after taking fattening
food. Over the last week the attacks occurred daily and became more painful.
On the 3rd day of hospitalization the patient presented with icteritiousness of
skin and scleras, light-colored feces and dark urine. In blood: neutrophilic
leukocytosis - 13,1×109/l, ESR- 28 mm/h. What is the most likely diagnosis?
A. Chronic calculous cholecystitis
B. Fatty degeneration of liver
C. Hypertensive dyskinesia of gallbladder
D. Chronic cholangitis, exacerbation stage
E. Chronic recurrent pancreatitis

151. A 20 y.o. patient with bronchial asthma experiences dyspnea attacks 3-


4 times a week. Nocturnal attacks are 1 time a week. FEV1- 50% of necessary
figures, during the day its variations is 25%. What is the severity of bronchial
asthma condition?
A. Moderate severity condition
B. Intermittent flow
C. Serious condition
D. Mild condition
E. Asthmatic status

152. A 40 y.o. man complains of headache in occipital area. On physical


examination: the skin is pale; face and hand edema, BP- 170/130 mm Hg. On
Ech°CG: concentric hypertrophy of the left ventricle. Ultrasound examination
of the kidneys reveals thinned cortical layer. Urine analysis shows proteinuria
of 3,5 g/day. What is the probable diagnosis?
A. Essential arterial hypertension
B. Chronic glomerulonephritis
C. Cushings disease
D. Polycystic disease of the kidneys
E. Chronic pyelonephritis

153. A 27-year-old patient has a severe headache, nausea and vomiting.


Objectively: body temperature is 38,9°C, there is a haemorrhagic stellate rash
on the legs. The patient takes meningeal pose in bed. Meningeal symptoms are
strongly positive. Deep reflexes are brisk, uniform. Pathological reflexes are
absent. It has been suspected that the patient has epidemic cerebrospinal
meningitis. Which of additional tests should be performed in the first place to
verify the diagnosis?
A. Lumbar puncture
B. Rheoencephalography
C. Survey craniogram
D. Electroencephalography
E. Echoencephalography

154. After a serious nervous stress a 35-year-old patient has developed on


the dorsal surface of hands redness and swelling that were later replaced by
small inflammatory nodules, vesicles and following erosion with a significant
serous discharge. The process is accompanied by severe itching. What is the
most likely diagnosis?
A. True eczema
B. Toxicoderma
C. Microbal eczema
D. Allergic dermatitis
E. Simple contact dermatitis

155. A 36-year-old patient complains of skin rash that appeared a week ago
and doesnt cause any subjective problems. Objectively: palm and sole skin is
covered with multiple lenticular disseminated papules not raised above the
skin level. The papules are reddish, dense on palpation and covered with
keratinous squamae. What is the provisional diagnosis?
A. Secondary syphilis
B. Palmoplanar psoriasis
C. Palm and sole callosity
D. Palmoplanar rubrophytosis
E. Verrucosis

156. In the morning a patient had nausea, abdominal discomfort, single


vomiting, dry mouth. In the evening, the patient presented with the increasing
general weakness, double vision, difficult swallowing of solid food.
Objectively: ptosis, mydriasis, anisocoria, absence of gag and pharyngeal
reflex, dry mucous membranes. The previous evening the patient had dinner
with canned food and alcohol. What is the presumptive diagnosis?
A. Botulism
B. Acute ischemic stroke
C. Poliomyelitis
D. Food toxicoinfection
E. Intoxication with unknown poison

157. A 30-year-old patient complains of paroxysmal abdominal pain,


frequent liquid stools up to 10 times a day. Throughout the first 3 days he had
a fever, since the 2nd day of disease there were scant liquid stools mixed with
mucus. On palpation: tenderness of all colon segments. Sigmoid colon was
found spastic. What is your provisional diagnosis?
A. Acute dysentery
B. Salmonellosis
C. Intestinal amebiasis
D. Cholera
E. Balantidiasis

158. A 38-year-old woman experiences episodic increases in arterial


pressure up to 240/120 mm Hg, which is accompanied by nausea, vomiting,
tachycardia, increased sweating, hyperglycemia. The attack is usually followed
by the excessive urination. Renal sonography reveals an additional formation
adjacent to the upper pole of the right kidney and possibly belonging to the
adrenal gland. What laboratory test will allow to clarify the diagnosis?
A. Determination of urinary excretion of catecholamines and
vanillylmandelic acid
B. Estimation of glomerular filtration rate by measuring endogenous creatinine
clearance
C. Blood test for renin level
D. Blood test for thyroxine and thyrotrophic hormone
E. Blood test for insulin and C-peptide

159. A 32-year-old patient has a 3-year history of asthma attacks, that can
be hardly stopped with berotec. Over a few last months he has experienced
pain in the joints and sensitivity disorder of legs and feet skin. Ps - 80/min,
AP - 210/100 mm Hg. In blood: eosinophilia at the rate of 15%. What disease
can be suspected in this case?
A. Periarteritis nodosa
B. Systemic scleroderma
C. Systemic lupus erythematosus
D. Dermatomyositis
E. Wegeners disease

160. 3 hours before, a 68-year-old male patient got a searing chest pain
radiating to the neck and left forearm, escalating dyspnea. Nitroglycerin failed
to relieve pain but somewhat reduced dyspnea. Objectively: there is crimson
cyanosis of face. Respiratory rate is 28/min. The patient has vesicular
breathing with isolated sibilant rales. Heart sounds are muffled, with a gallop
rhythm. Ps - 100/min, AP - 100/65 mmHg. ECG shows negative T-wave in
V2-V6 leads. What drug can reduce the hearts need for oxygen without
aggravating the disease?
A. Isosorbide dinitrate
B. Atenolol
C. Corinfar
D. Streptokinase
E. Aminophylline
161. A 46-year-old patient complains of sudden palpitation, which is
accompanied by pulsation in the neck and head, fear, nausea. The palpitation
lasts for 15-20 minutes and is over after straining when holding her breath.
What kind of cardiac disorder may be suspected?
A. An attack of supraventricular paroxysmal tachycardia
B. An attack of atrial flutter
C. An attack of extrasystolic arrhythmia
D. An attack of ciliary arrhythmia
E. An attack of ventricular paroxysmal tachycardia

162. A 5-grade pupil complains about extensive skin rash accompanied by


intensive itch, especially at night. Objectively: there are small red papules set
mostly in pairs in the region of interdigital folds on both hands, on the flexor
surface of radicarpal articulations, abdomen and buttock skin as well as
internal surface of thighs. In the centre of some papules vesicles or
serohaemorrhagic crusts can be seen. There are multiple excoriations. What is
the most likely diagnosis?
A. Scabies
B. Dermatitis
C. Toxicoderma
D. Eczema
E. Ringworm of body

163. A welder at work got the first-degree burns of the middle third of his
right shin. 5 days later the skin around the burn became edematic and itchy.
Objectively: on a background of a well-defined erythema there is polymorphic
rash in form of papules, vesicles, pustules, erosions with serous discharge.
What is the most likely diagnosis?
A. Microbal eczema
B. True eczema
C. Occupational eczema
D. Streptococcal impetigo
E. Toxicoderma

164. A 58-year-old patient has a 3-year history diabetes mellitus type II. He
has been keeping to a diet and regularly taking glyburide. He has been
delivered to a hospital on an emergency basis for acute abdomen. Objectively:
the patient is of supernutrition type. The skin is dry. In the lungs vesicular
breathing can be auscultated. Heart sounds are regular, 90/min. AP- 130/70
mm Hg. The symptom of "wooden belly" is visible. Blood sugar - 9,8
millimole/l. The patients has indication for laparotomy. What is the most
appropriate way of further treatment of diabetes?
A. To administer short insulin
B. To administer 1 tablet of Maninil three times a day
C. To administer Semilong to be taken in the morning and insulin - in the
evening
D. To continue taking glyburide
E. To administer 1 tablet of Glurenorm three times a day

165. A 56 y.o. man, who has taken alcoholic drinks regularly for 20 years,
complains of intensive girdle pain in the abdomen. Profuse nonformed stool
2-3- times a day has appeared for the last 2 years, loss of weight for 8 kg for 2
years. On examination: abdomen is soft, painless. Blood amylase - 12g/L.
Feces examination-neutral fat 15 g per day, starch grains. What is the most
reasonable treatment at this stage?
A. Pancreatine
B. Aminocapron acid
C. Imodium
D. Levomicytine
E. Contrykal

166. A 43-year-old female patiet complains of eruption on her right leg skin,
pain, weakness, body temperature rise up to 38°C. The disease is acute.
Objectively: there is an edema on the right leg skin in the region of foot, a
well-defined bright red spot in form of flame tips which feels hot. There are
isolated vesicles in focus. What is your provisional diagnosis?
A. Erysipelas
B. Haemorrhagic vasculitis
C. Contact dermatitis
D. Microbial eczema
E. Toxicoderma

167. A 45-year-old patient complains of some painless nodular elements


tending to peripheral growth and fusion. He has a 2-year history of this
disease. Aggravation takes place mainly in spring. In anamnesis: the patients
father had similar skin lesions. Objectively: pathological elements looke like
guttate and nummular nodules, plaques covered with white scales. What is
your provisional diagnosis?
A. Psoriasis
B. Lichen ruber planus
C. Pityriasis rosea
D. Seborrheic eczema
E. Neurodermitis

168. A 47-year-old patient came to see a doctor on the 7th day of disease.
The disease developed very fast: after the chill body temperature rose up to
40°C and lasted up to 7 hours, then it dropped abruptly, which caused profuse
sweat. There were three such attacks occuring once in two days. Two days ago
the patient arrived from Africa. Objectively: pale skin, subicteric sclera,
significantly enlarged liver and spleen. What is the cause of fever attacks in
this disease?
A. Erythrocytic schizogony
B. Exotoxin of a causative agent
C. Gametocytes
D. Endotoxin of a causative agent
E. Tissue schizogony

169. On the 2nd day of disease a 27-year-old patient complains of


unbearable headache, repeated vomiting. Objectively: the patient is in a grave
condition. He is conscious but adynamic. Lies in a forced position with his
head thrown back. There is no skin rash. Nuchal muscles are evidently rigid,
there are Kernigs and Brudzinskis signs. to - 39,5°C, Ps -120/min, AP -
130/80 mm Hg. The leading syndrome of this disease is caused by:
A. Liquor hypertension
B. Affection of the cranial nerve nuclei
C. Liquor hypotension
D. Haemorrhages in the adrenal glands
E. Hyperthermy

170. On the 2nd day of illness a 27-year-old patient complains of unbearable


headache, repeated vomiting. Objectively: the patient is in a grave condition.
He is conscious but adynamic. Lies in a forced position with his head thrown
back. There is no skin rash. Nuchal muscles are evidently rigid, there are
Kernigs and Brudzinskis signs. to- 39,5°C, Ps- 120/min, AP- 130/80 mm Hg.
The leading syndrome of this disease is caused by:
A. Liquor hypertension
B. Liquor hypotension
C. Haemorrhages into the adrenal glands
D. Hyperthermy
E. Affection of the cranial nerve nuclei

171. A 43 y.o. woman complains of shooting heart pain, dyspnea,


irregularities in the heart activity, progressive fatigue during 3 weeks. She had
acute respiratory disease a month ago. On examination: AP- 120/80 mm Hg,
heart rate 98 bpm, heart boarders +1,5 cm left side, sounds are muffled, soft
systolic murmur at apex and Botkins area; sporadic extrasystoles. Liver isnt
palpated, there are no edema. Blood test: WBC- 6,7×109/L, sedimentation
rate- 21 mm/hour. What is the most probable diagnosis?
A. Acute myocarditis
B. Climacteric myocardiodystrophia
C. Rheumatism, mitral insufficiency
D. Hypertrophic cardiomyopathy
E. Ichemic heart disease, angina pectoris

172. A 63-year-old female complains of general weakness, a feeling of


heaviness, compression in the epigastrium, postprandial fullness, nausea,
belching after meals. These symptoms have been observed for about 15 years.
Objectively: body temperature is 36,4°C, respiratory rate - 20/min, Ps -
88/min, blood pressure - 115/75 mm Hg. Skin and mucous membranes are
pale. Blood test results: RBC - 2,0x1012/l, Hb - 100 g/l. Tests revealed
parietal-cell antibodies. What is the most likely reason for the development of
anemia in this patient?
A. Production of antibodies to intrinsic factor
B. Disruption of erythropoietin synthesis
C. Disruption of hemoglobin synthesis
D. Impaired iron absorption
E. Increased loss of iron

173. A 37-year-old woman is sick with bronchial asthma for 15 years.


Recenlty asthmatic attacks occur 4-5 times per week, night attacks -2-3 times
per month. To stop attacks, the patient takes salbutamol. On physical exam:
condition is relatively satisfactory. RR - 20/min, Ps is 76 bpm, BP - 120/80
mm Hg. Respiration in lungs is vesicular. Cardiac sounds are muted, rhythm
is normal. What medication should be prescribed to prevent attacks of
bronchial asthma on the first stage?
A. Cromoglycat sodium
B. Injection of corticosteroids
C. Inhalation corticosteroids
D. Regular dose of salbutamol
E. Tabletted corticosteroids

174. A 52 y.o. male patient has become ill gradually. There is pain in the left
side of the thorax during 2 weeks, elevation of temperature till 38-39°C. On
examination: left chest side falls behind in breathing movement no voice
tremor over the left lung. Dullness that is more intensive in lower parts of this
lung. Right heart border is deviated outside. Sharply weakened breathing over
the left lung, no rales. Heart sounds are mufflet, tachycardia. What is the most
probable diagnosis?
A. Exudative pleuritis
B. Spotaneous pneumothorax
C. Cirrhotic tuberculosis
D. Infarction-pneumonia
E. Atelectasis of lung
175. A 50-year-old patient was hospitalized in severe condition with
complaints of chills, high grade temperature, dryness in the mouth, multiple
vomiting, pain in the epigastrium, frequent watery, foamy, dirty green color
stool of unpleasant odor. The tongue and the skin are dry. BP - 80/40 mm Hg.
What first aid is necessary for the patient?
A. Intravenous injection of sodium solutions
B. Hemosorbtion
C. To prescribe polyglucin
D. Fresh-frozen plasma transfusion
E. Sympathomimetics

176. A 42-year-old female patient suffers from micronodular cryptogenic


cirrhosis. Over the last week her condition has deteriorated: she developed
convulsions, mental confusion, progressing jaundice. What study may give
reasons for such aggravation?
A. Determination of serum ammonia
B. Determination of ALAT and ASAT
C. Determination of alkaline phosphatase
D. Determination of cholesterol ethers
E. Determination of alpha-phetoprotein

177. A patient has chronic heart failure of the II stage. He takes furosemide
regularly three times a week. He had developed bronchopneumonia and had
been administered combined pharmacotherapy. On the fifth day of therapy
the patient complained of hearing impairment. What drug coadministered
with furosemide might have caused the hearing loss?
A. Gentamicin
B. Mucaltin
C. Nystatin
D. Linex
E. Tavegil

178. A 60-year-old female patient complains of recurrent pain in the


proximal interphalangeal and wrist joints, their periodic swelling and
reddening that have been observed for 4 years. X-ray picture represents
changes in form of osteoporosis, joint space narrowing and single usuras.
What is the most likely diagnosis?
A. Rheumatoid arthritis
B. Osteoarthritis
C. Pseudogout
D. Multiple myeloma
E. Gout
179. A 37-year-old woman complains of generalized fatigue, irritability,
dysphagia, chalk hunger. On physical exam: t- 36,5°C, respirations - 20/min,
Ps - 96 bpm, BP - 110/70 mm Hg. Satisfactory nourishment. The skin and
visible mucous membranes are pale. Blood test: Hb -70g/L, erythrocytes -
3,4×1012/L, CI - 0,7, reticulocytes - 2%, leucocytes - 4,7×109/L, eosinophilis. -
2%, band neutrophils - 3%, segmented neutrophils - 64%, lymphocytes – 26%,
monocytes - 5%, ESR - 15 mm/min. Serum ferrum - 7,3 mumol/L, total
protein - 70g/L. Deficit of what factor caused the development of the disease?
A. Ferrum
B. Vitamin B12
C. Vitamin B6
D. Protein
E. Folic acid

180. A 28-year-old patient has been hospitalized for the pain in the
epigastric region. He has a 10-year history of duodenal ulcer (DU). Recently,
the pain character has changed: it became permanent, persistent, irradiating
to the back. There are general weakness, dizziness, fatigue. The patient has
put off weight. Objectively: HR- 68/min, AP- 120/80 mm Hg. What is most
likely cause of deterioration?
A. Penetration
B. Haemorrhage
C. Exacerbation of duodenal ulcer
D. Stenosis development
E. Perforation of duodenal wall

181. A 57-year-old male patient complains of dyspnea on exertion, heaviness


in the right hypochondrium and shin edemata towards evening. Objectively:
temperature - 38,1°C, HR- 20/min, Ps=92/min, AP- 140/90 mm Hg. There is
apparent kyphoscoliosis. In the lungs single dry rales can be auscultated.
Heart sounds are muffled, rhythmic. ECG: Rv1+Sv5=15 mm. X-ray picture
shows the bulging of pulmonary artery cone, right ventricle enlargement.
What is the most likely cause of this condition?
A. Pulmonary heart
B. Dilatation cardiomyopathy
C. Atherosclerotic cardiosclerosis
D. Mitral stenosis
E. Primary pulmonary hypertension

182. A 30-year-old female patient has been delivered to a hospital for


sudden dyspnea progessing to asthma, sensation of having a "lump in the
throat", hand tremor, fear of death. The attack has developed for the first time
and is associated with a strong emotion. There is no previous history.
Objectvely: respiratory rate - 28/min, Ps - 104/min, rhythmic, AP - 150/85
mm Hg. The patient has rapid superficial vesicular breathing with extended
expiration. Percussion findings: heart borders are not changed. Cardiac
sounds are loud, rhythmic. What is the most likely diagnosis?
A. Neurocirculatory asthenia
B. Bronchial asthma
C. Cardiac asthma
D. Thyrotoxic crisis
E. Hypertensive crisis

183. A 24-year-old emotionally-labile woman presents with irritation,


depressed mood, palpitation, shooting pain in the heart area, generalized
fatigue following the divorce. On examination: palm hyperhydrosis, pulse rate
- 72-78 bpm, labile, heart without changes. ECG is normal. What is the most
probable pathology in this case?
A. Neurasthenia
B. Ipochondric neurosis
C. Schizophrenia
D. Depressive neurosis
E. Compulsive neurosis

184. A 30-year-old woman with a long history of chronic pyelonephritis


complains about considerable weakness, sleepiness, decrease in diuresis down
to 100 ml per day. AP- 200/120 mm Hg. In blood: creatinine - 0,62
millimole/l, hypoproteinemia, albumines - 32 g/l, potassium - 6,8 millimole/l,
hypochromic anemia, increased ESR. What is the first step in the patient
treatment tactics?
A. Haemodialysis
B. Enterosorption
C. Antibacterial therapy
D. Haemosorption
E. Blood transfusion

185. A patient had macrofocal myocardial infarction. He is overweight for


36%, AP is 150/90 mm Hg, blood sugar- 5,9 mmol/L, general cholesterol- 4,9
mmol/L, uric acid- 0,211 mmol/L. Which risk factor should be urgently
eradicated during the secondary prevention?
A. Obesity
B. Arterial hypertension
C. Hypercholesterolemia
D. Hyperuricemia
E. Hyperglycemia
186. A 36-year-old female patient complains of bruises on the body, gingival
haemorrhage, general weakness. A month ago she had a severe domestic
poisoning with some pesticide (the patient can not remember the name). She
has a 7-year record of working in contact with petroleum products,
particularly benzene. In blood: RBCs - 3,2×1012/l, WBCs - 2,7×109/l,
thrombocytes – 70×109/l. What is the most likely pathology?
A. Benzene intoxication
B. Chronic fatigue Syndrome
C. Organochlorine pesticide Intoxication
D. Organophosphorus pesticide intoxication
E. Mercury-containing pesticide intoxication

187. While staying in a stuffy room a 19-year-old emotionally labile girl


developed severe weakness, dizziness, blackout, nausea and loss of
consciousness without convulsions. Objectively: the patient is unconscious,
the skin is pale, extremities are cold. AP- 90/60 mm Hg, Ps- 96/min,
deficient, breathing is shallow. Pupillary and tendon reflexes are present.
There are no pathological signs. What is the most likely diagnosis?
A. Syncope
B. Transient ischemic attack
C. Epileptic attack
D. Vegetovascular paroxysm
E. Hysterical neurosis

188. A patient complains of frequent, bulky, frothy stools with greenish


mucus, cramping pain in the umbilical region, abdominal murmur, body
temperature at the rate of 39°C. The patient associates the disease with
consumption of soft-boiled eggs. What is the most likely pathogen?
A. Salmonella
B. Shigella
C. Yersinia
D. Enteropathogenic E.Coli
E. Vibrio cholerae El Tor

189. A 40-year-old female patient complains of headache, dizziness, muscle


weakness, occasional cramps in the extremities. She has been taking
antihypertensive medications for 10 years. AP- 180/100 mm Hg. Blood
potassium - 1,8 millimole/l, sodium - 4,8 millimole/l. In urine: alkaline
reaction, the relative density - 1012, protein and sugar are not found, WBCs -
3-4 in the field of vision, RBCs - 1-2 in the field of vision. Conns syndrome is
suspected. Which drug should be chosen for the treatment of arterial
hypertension?
A. Spironolactone
B. Propanolol
C. Hydrochlorothiazide
D. Clonidine
E. Enalapril

190. An 18-year-old patient presents no problems. Percussion reveals that


heart borders are displaced to the right and left by 1 cm, there is a coarse
systolic murmur with its epicenter within the 4th intercostal space on the left.
What is the most informative examination to confirm the clinical diagnosis?
A. Ventriculography
B. PCG
C. Polycardiography
D. Echocardiography
E. ECG

191. A 56-year-old patient complains of having persistent chest pain on the


right for the last 2 months. The pain is not associated with respiration. He also
complains of cough with blood-streaked sputum, weakness, decreased
performance, fatigue. Chest radiograph shows a globular shade of 4x6 cm
connected to the root of the lung in the lower part of the right lung. What is
the most likely diagnosis?
A. Peripheral lung cancer
B. Lung abscess
C. Tuberculoma
D. Pneumonia
E. Metastasis

192. A 18-year-old patient had subtotal strumectomy due to malignant


capillary cystadenoma of the thyroid gland. In 2 months there was a suspicion
of metastasis presence in the lungs. What rontgenological method is to be
used first?
A. Roentgenography of lungs
B. Angiopneumonography
C. Roentgenoscopy of lungs
D. Bronchography
E. Bronchoscopy

193. A 58-year-old patient was diagnosed basal-cell skin cancer, 1st stage.
Tumor is up to 1 cm in size and with up to 0,5 cm deep infiltration in tissues.
Tumor is localized in the right nasolabial area. Choose the most optimal
method of treatment.
A. Short-distance roentgenotherapy
B. Surgical treatment
C. Long-distance gamma therapy
D. Long-distance roentgenotherapy
E. Chemotherapy

194. A 27-year-old patient complains of nasal haemorrhages, multiple


bruises on the anterior surface of the trunk and extremities, sudden weakness.
In blood: Hb- 74 g/l, reticulocytes - 16%, RBCs - 2,5×1012/l, platelets –
30×109/l, ESR- 25 mm/h. What is the most effective measure for the
treatment of thrombocytopenia?
A. Splenectomy
B. Vitamin B12
C. Hemotransfusion
D. Iron preparations
E. Cytostatics

195. 2 days ago a patient presented with acute pain in the left half of chest,
general weakness, fever and headache. Objectively: between the 4 and 5 rib on
the left the skin is erythematous, there are multiple groups of vesicles 2-4 mm
in diameter filled with transparent liquid. What diease are these symptoms
typical for?
A. Herpes zoster
B. Herpes simplex
C. Pemphigus
D. Streptococcal impetigo
E. Herpetiform Duhrings dermatosis

196. A woman while working in vegetable garden developed severe pain in


the loin. Lasagues and Nery tension signs are obviously marked on the right.
Lumbar lordosis is smoothed, movements are harshly restrained in lumbar
part of the spine. Right ankle (Achilles) reflex is absent. What kind of disease
can it be?
A. Lumbar-sacral radiculitis
B. Hepatic colic
C. Lumbalgia
D. Renal colic
E. Neuritis of femoral nerve

197. After lifting a load a patient felt undurable pain in the loin. He was
diagnosed with acute lumbosacral radiculitis. Which of the following is
contraindicated for this patient?
A. Warming procedures
B. Vitamins of B group
C. Intravenous injection of aminophylline
D. Dehydrating drugs
E. Analgetics

198. An unconscious 35-year-old patient has been delivered by an


ambulance to the intensive care unit. Objectively: the patient is in semicoma.
Moderate mydriasis is present. The reaction of pupils to light is reduced. The
reaction to verbal instructions is missing. AP is150/100 mm Hg, there is
tachycardia. Blood contains methanol. What antidote should be administered?
A. Ethanol
B. Thiamine chloride
C. Naloxone
D. Tavegil
E. Unithiol

199. After lifting a load, a 36-year-old male patient has experienced a severe
pain in the lumbar region, which spread to the right leg and was getting worse
when he moved his foot or coughed. Objectively: the long back muscles on the
right are strained. Achilles jerk is reduced on the right. There is a pronounced
tenderness of paravertebral points in the lumbar region. The straight leg raise
(Lasegues sign ) is positive on the right. What additional tests should be
performed in the first place?
A. Radiography of the spinal column
B. Lumbar puncture
C. Magnetic resonance tomography
D. Computed tomography
E. Electromyography

200. A female, aged 20, after smoking notices a peculiar inebriation with the
feeling of burst of energy, elation, irreality and changing of surroundings: the
world gets full of bright colours, the objects change their dimensions, peoples
faces get cartoon features, loss of time and space judgement. What is the most
likely diagnosis?
A. Cocainism
B. Morphinism
C. Nicotinism
D. Cannabism
E. Barbiturism

201. A 75 y.o patient can not tell the month, date and season of the year.
After long deliberations she manages to tellher name. She is in irritable and
dissatisfied mood. She always carries a bundle with belongings with her, hides
a parcel with bread, shoes in her underwear in her bosom as well as
"invaluable books". What is the most probable diagnosis?
A. Senile dementia
B. Behaviour disorder
C. Dissociated personality (psychopathy)
D. Atherosclerotic (lacunar) dementia
E. Presenile melancholia

202. A 17-year-old male patient consulted a therapist about malaise, chills,


runny nose, aching muscles and joints, nausea and diarrhea. The patient asks
to prescribe him a lot of painkillers and sedatives (tramadol or solpadein that
help the best, and diazepam). Pharyngeal mucosa is pale pink, clean.
Auscultation reveals vesicular breathing. Tachycardia is present. The pupils
are dilated, there is sluggish response to light. There are injection marks on
the forearm skin. During examination, the patients manner is vulgar, irritable,
rude and untruthful. Make a diagnosis:
A. Opioid addiction
B. Sedative drug addiction
C. Painkillers addiction
D. Acute respiratory disease
E. Food-born toxic infection

203. While lifting a heavy load a 39-year-old patient suddenly felt a severe
headache, pain in the interscapular region, and started vomiting. Objectively:
the pulse is rhythmic, 60/min, AP- 180/100 mm Hg. The patient is agitated.
He presents with photophobia, hyperacusis. There are positive Kernigs and
Brudzinskis signs on both sides. In blood: WBCs – 10×109/l. CSF is bloody,
cytosis is 240/3. What is the most likely diagnosis?
A. Subarachnoid haemorrhage
B. Meningococcal meningitis
C. Ischemic stroke
D. Sympathoadrenal crisis
E. Acute hypertonic encephalopathy

204. A 26-year-old male patient complains of piercing pain during


breathing, cough, dyspnea. Objectively: to- 37,3°C, respiration rate - 19/min,
heart rate = Ps- 92/min; AP- 120/80 mm Hg. Vesicular respiration. In the
inferolateral parts of chest auscultation in both inspiration and expiration
phase revealed noise that was getting stronger at phonendoscope pressing and
can be still heard after cough. ECG showed no pathological changes. What is
the most likely giagnosis?
A. Acute pleuritis
B. Subcutaneous emphysema
C. Intercostal neuralgia
D. Spontaneous pneumothorax
E. Pericarditis sicca
205. A 45-year-old patient, a sailor, was hospitalized on the 2nd day of the
disease. A week ago he returned from India. Complains of body temperature
of 41°C, severe headache, dyspnea, cough with frothy rusty sputum.
Objectively: the patient is pale, mucous membranes are cyanotic, breathing
rate is 24/min, tachycardia is present. In lungs: diminished breath sounds,
moist rales over both lungs, crepitation. What is the most likely diagnosis?
A. Pneumonic plaque
B. Influenza
C. Sepsis
D. Ornithosis
E. Miliary tuberculosis

206. HIV displays the highest tropism towards the following blood cells:
A. T-helpers
B. Thrombocytes
C. Erythrocytes
D. T-suppressors
E. T-killers

207. A 25-year-old patient complains of general weakness, dry cough,


sweating, subfebrile temperature. Objectively: lung auscultation reveals
vesicular resiration with no wheezing. Fluorogram shows focal shadows of
high intensity in the 1-2 segments of the right lung. Mantoux test gave a
reaction of 16 mm of induration. What clinical form of tuberculosis is most
likely?
A. Focal
B. Infiltrative
C. Tuberculoma
D. Miliary
E. Disseminated

208. A 22-year-old patient is a clerk. His working day runs in a conditioned


room. In summer he was taken by an acute disease with the following
symptoms: fever, dyspnea, dry cough, pleural pain, myalgia, arthralgia.
Objectively: moist rales on the right, pleural friction rub. X-ray picture showed
infiltration of the inferior lobe. In blood: WBC – 11×109/l, stab neutrophils -
6%, segmented neutrophils - 70%, lymphocytes - 8%, ESR - 42 mm/h. What is
the ethiological factor of pneumonia?
A. Legionella
B. Pneumococcus
C. Streptococcus
D. Mycoplasm
E. Staphylococcus

209. A 43-year-old female patient complains of dyspnea, swelling of legs,


abdomen enlargement, pricking heart pain. She has a history of tuberculous
bronchadenitis, quinsies. The patients condition deteriorated 6 months ago.
Objectively: cyanosis, bulging neck veins, vesicular breathing. Heart borders
are not displaced. Heart sounds are muffled, Ps is 106/min, liver is +4 cm,
ascites is present. Low voltage on the ECG has been revealed. Radiograph
shows a thin layer of calcium deposits along the left contour of heart. What
treatment should be recommended to the patient?
A. Treatment by a cardiac surgeon
B. Digitalis preparations
C. Diuretics
D. Vasodilators, nitrates
E. Anti-TB drugs

210. A 53-year-old female patient complains of cardiac pain and rhythm


intermissions. She has experienced these presentations since childhood. The
patients father had a history of cardiac arrhythmias. Objectively: the patient is
in grave condition, Ps- 220 bpm, AP- 80/60 mm Hg. ECG results: heart rate -
215/min, extension and deformation of QRS complex accompanied by
atrioventricular dissociation; positive P wave. Some time later heart rate
reduced down to 45/min, there was a complete dissociation of P wave and
QRST complex. Which of the following will be the most effective treatment?
A. Implantation of the artificial pacemaker
B. Calcium antagonists
C. Cardiac glycosides
D. beta-adrenoreceptor blocking agents
E. Cholinolytics

211. A 26-year-old female patient has an 11-year history of rheumatism.


Four years ago she suffered 2 rheumatic attacks. Over the last 6 months there
have been paroxysms of atrial fibrillation every 2-3 months. What option of
antiarrhythmic therapy or tactics should be proposed?
A. Prophylactic administration of cordarone
B. Heparin administration
C. Defibrillation
D. Immediate hospitalization
E. Lidocaine administration

212. A 49-year-old patient complains of dyspnea, cough. There are no


sputum discharges. He has repeatedly used salbutamol and intal but with no
effect. Objectively: he is only able to sit while leaning on the table. Cyanosis of
face, acrocyanosis are present. Breathing is shallow, laboured, in some parts it
cannot be auscultated; there are diffuse rales, expiration is significantly
prolonged. Heart sounds are muffled, tachycardia is present. Ps - 112/min,
AP- 110/70 mm Hg. Liver is located near the costal arch. There are no
peripheral edemata. What is your provisional diagnosis?
A. Status asthmaticus
B. Chronic obstructive bronchitis
C. Foreign object aspiration
D. Cardiac asthma
E. Bronchiale asthma, moderate gravity

213. Thrombosis of the coronary artery caused myocardial infarction. What


mechanisms of injury will be the dominating ones in this disease?
A. Calcium mechanisms
B. Acidotic mechanisms
C. Lipid mechanisms
D. Protein mechanisms
E. Electrolytoosmotic mechanisms

214. On the second day of the disease a 22-year-old male patient complains
of high-grade fever, headache in the region of forehead and superciliary
arches, and during eye movement; aching muscles and joints. Objectively:
body temperature is 39°C. Face is hyperemic, sclerae are injected. The mucous
membrane of the soft palate and posterior pharyngeal wall is bright hyperemic
and has petechial hemorrhages. What changes in the hemogram are typical for
this disease?
A. Leukopenia
B. Neutrocytosis
C. Accelerated ESR
D. Anemia
E. Leukocytosis

215. A female patient consulted a dermatologist about the rash on the trunk
and extremities. Objectively: interdigital folds, flexor surfaces of wrists and
navel region are affected with pairs of nodulo-cystic eruptions and crusts. The
rash is accompanied by skin itch that is getting stronger at night. What
external treatment should be administered?
A. 20% benzyl benzoate emulsion
B. 5% tetracycline ointment
C. 2% sulfuric paste
D. 5% sulfuric ointment
E. 5% naphthalan ointment
216. A 54 y.o. male patient suffers from dyspnea during mild physical
exertion, cough with sputum which is excreted with diffculty. On examination:
diffuse cyanosis. Is Barrel-chest. Weakened vesicular breathing with
prolonged expiration and dry whistling rales. AP is 140/80 mm Hg, pulse is
92 bpm, rhythmic. Spirography: vital capacity (VC)/predicted vital capacity-
65%, FEV1/FVC– 50%. Determine the type of respiratory insufficiency (RI).
A. RI of mixed type with prevailing obstruction
B. RI of mixed type with prevailing resriction
C. There is no RI
D. RI of restrictive type
E. RI of obstructive type

217. An ambulance had been called to a 48-year-old man. According to his


relatives, the patient had had three attacks of unconsciousness accompanied
by convulsions within 24 hours. On examination the doctor witnessed the
following attack: the patient lost consciousness and fell to the floor, developed
tonic, then clonic convulsions of trunk and extremities. The attack lasted 1
minute and ended with involuntary urination. Specify the kind of attack:
A. Epileptic seizure
B. Attack of hysteria
C. Vegetative crisis
D. Syncope
E. Coma

218. A patient is 60 years old, retired, worked as deputy director of a


research institute. Behavioural changes appeared 2 years ago after the death
of her husband: she stopped looking after herself and leaving the house; then
she refused to clean the apartment and cook. Mental status: temporal
disorientation. The patient does not understand many of the questions, is
confused; does not know how to cook soup or fasten a button. Her speech is
characterized by stumbling and logoclonia. She does not recognize doctors,
fellow patients. She cries a lot but can not explain the reason for tears. What is
the mechanism of this pathology?
A. Atrophy of the cerebral cortex
B. Atherosclerotic changes in cerebral vessels
C. Impaired conversion of dopamine to noradrenaline
D. Disorder of melatonin metabolism
E. Serotonin deficiency

219. A 26-year-old patient has abused alcohol since the age of 16, needs a
morning-after drink to cure hangover. He takes alcohol nearly every day, "a
little at a time". Twice a week he gets severely drunk. The patient works as a
motor mechanic, over the last 2 years work conflicts have become more
frequent. What medical and tactical actions should be taken in this case?
A. Voluntary consultation and treatment at an addiction clinic
B. Compulsory treatment
C. Referral to medical-social expert commission for assessment of his working
ability
D. Consultation with a psychologist
E. Referral to treatment at an activity therapy centre

220. A 47-year-old female patient has an 8-year history of ulcerative colitis,


has been treated with glucocorticoids. She complains of cramping pain in the
umbilical region and left iliac region which has significantly increased during
the past 2 weeks, diarrhea with mucus and blood 4-6 times a day, elevated
body temperature up to 38-39°C, headache and pain in the knee joints.
Objectively: the patient is in moderate condition, Ps - 108/min, AP - 90/60
mm Hg; heart and lungs are unremarkable; the tongue is moist; abdominal
muscle tone is significantly decreased; peristaltic noises are absent. What
complication developed in the patient?
A. Toxic dilatation of the colon
B. Enterorrhagia
C. Colon carcinoma
D. Stricture of the colon
E. Perforation of the colon

221. A 26-year-old patient with left lower lobe pneumonia experiences an


acute chest pain on the left during coughing. Objectively: diffuse cyanosis,
extension of the left side of chest. Percussion reveals high tympanitis.
Auscultation reveals no respiratory murmurs above the left side of chest.
There is a deviation of the right cardiac border towards the midclavicular line.
What examination will be the most informative?
A. X-Ray
B. Spirography
C. Bronchography
D. Bronchoscopy
E. Pneumotachometry

222. A male patient presents with swollen ankles, face, eyelids, elevated AP-
160/100 mm Hg, pulse- 54 bpm, daily loss of albumine with urine- 4g. What
therapy is pathogenetic in this case?
A. Corticosteroids
B. Diuretics
C. Calcium antagonists
D. Antibiotics
E. NSAID
223. After myocardial infarction, a 50-year-old patient had an attack of
asthma. Objectively: bubbling breathing with frequency of 32/min, cough with
a lot of pink frothy sputum, acrocyanosis, swelling of the neck veins. Ps-
108/min, AP- 150/100 mm Hg. Heart sounds are muffled. Mixed moist rales
can be auscultated above the entire lung surface. What drug would be most
effective in this situation?
A. Nitroglycerin intravenously
B. Dopamine intravenously
C. Aminophylline intravenously
D. Pentamin intravenously
E. Strophanthin intravenously

224. During dynamic investigation of a patient the increase of central


venous pressure is combined with the decrease of arterial pressure. What
process is proved by such combination?
A. Increase of bleeding speed
B. Presence of hypervolemia
C. Shunting
D. Developing of cardiac insufficiency
E. Depositing of blood in venous channel

225. A male patient complains of heartburn which gest stronger while


bending the body, substernal pain during swallowing. There is a hiatus hernia
on X-ray. What disoeder should be expected at gastroscopy?
A. Gastroesophageal reflux
B. Gastric peptic ulcer
C. Chronic gastritis
D. Acute erosive gastritis
E. Duodenal peptic ulcer

226. A 43 y.o. male complains of stomach pain, which relieves with


defecation, and is accompanied by abdominal winds, rumbling, the feeling of
incomplete evacuation or urgent need for bowel movement, constipation or
diarrhea in alternation. These symptoms have lasted for over 3 months. No
changes in laboratory tests. What is the most likely diagnosis?
A. Irritable bowel syndrome
B. Colitis with hypertonic type dyskinesia
C. Atonic colitis
D. Chronic enterocolitis, exacerbation phase
E. Spastic colitis

227. Against the background of angina a patient has developed pain in


tubular bones. Examination revealed generalized enlargement of lymph
nodes, hepatolienal syndrome, sternalgia. In blood: RBCs - 3,6×1012/l, Hb- 87
g/l, thrombocytes – 45×109/l, WBCs – 13×109/l, blasts - 87%, stab
neutrophils - 1%, segmented neutrophils - 7%, lymphocytes - 5%, ESR - 55
mm/h. What is the most likely diagnosis?
A. Acute leukemia
B. Multiple myeloma
C. Chronic lymphocytic leukemia
D. Erythremia
E. Chronic myeloid leukemia

228. A 49-year-old female patient with schizophrenia is all the time listening
to something, insists that "there is a phone in her head" as she hears the voice
of her brother who tells her to go home. The patient is anxious, suspicious,
looks around all the time. Specify the psychopathological syndrome:
A. Hallucinatory
B. Depressive
C. Paranoiac
D. Generalized anxiety disorder
E. Paraphrenic

229. A 43-year-old male patient undergoing treatment for peptic ulcer


complains of weakness, dizziness, coffee-ground vomiting, melena. After
administration of haemostatics the patients condition has not improved, fresh
blood has shown up in the vomit, skin bruises of different sizes have appeared.
In blood: thrombocytes – 50×109/l, Lee-White clotting time - 35 minutes,
APTT - 80 seconds. In this case it is most rational to administer the following
preparation:
A. Fresh frozen plasma
B. Rheopolyglucinum
C. Vikasol
D. Heparin
E. Fibrinogen

230. A 38-year-old patient complains of inertness, subfebrile temperature,


enlargement of lymph nodes, nasal haemorrhages, bone pain. Objectively: the
patients skin and mucous membranes are pale, palpation revealed enlarged
painless lymph nodes; sternalgia; liver was enlarged by 2 cm, spleen - by 5 cm,
painless. In blood: erythrocytes - 2,7×1012/l, Hb- 84 g/l, leukocytes –
58×109/l, eosinophils - 1%, stab neutrophils - 2%, segmented neutrophils -
12%, lymphocytes - 83%, lymphoblasts - 2%, smudge cells; ESR- 57 mm/h.
What is the most likely diagnosis?
A. Chronic lymphatic leukemia
B. Acute lymphatic leukemia
C. Chronic myeloleukemia
D. Acute myeloleukemia
E. Lymphogranulomatosis

231. A 30-year-old male patient complains of inertness, low-grade fever,


bleeding gums, frequent quinsies, aching bones. Objectively: the patient has
pale skin and mucous membranes, sternalgia, +2 cm liver, +5 cm painless
spleen. Blood test results: RBC - 2,7×1012/l, ?b - 80 g/l, WBC – 3×109/l,
eosinophils - 4%, basophils - 5%, blasts - 4%, stab neutrophils - 2%,
segmented neutrophils - 17%, lymphocytes - 29%, myelocytes - 25%,
promyelocytes - 12%, monocytes - 2%, platelets – 80×109/l, ESR - 57 mm/h.
What test should be performed to verify the diagnosis?
A. Sternal puncture
B. Lumbar puncture
C. Chest X-ray
D. Trephine biopsy
E. Lymph node biopsy

232. A 24-year-old patient complains about putting on weight, limosis.


Objectively: the patients constitution is of hypersthenic type, body weight
index is 33,2 kg/m2, waist circumference is 100 cm. Correlation of waist
circumference to the thigh circumference is 0,95. What is the most likely
diagnosis?
A. Alimentary constitutional obesity of the I stage, abdominal type
B. Alimentary constitutional obesity of the III stage, gynoid type
C. Hypothalamic Itsenko-Cushing obesity of the II stage, gynoid type
D. Alimentary constitutional obesity of the II stage, abdominal type
E. Hypothalamic Itsenko-Cushing obesity of the I stage, abdominal type

233. A 47-year-old male patient has been lately complaining of compressing


chest pain that occurs when he walks a distance of 700-800 m. Once a week,
he drinks 2 liters of beer. Rise in arterial pressure has been observed for the
last 7 years. Objectively: Ps - 74/min, AP - 120/80 mm Hg. The bicycle
ergometry performed at workload of 75 watts shows 2 mm ST-segment
depression in V4-V6 leads. What is the most likely diagnosis ?
A. Exertional stenocardia, II functional class
B. Exertional stenocardia, IV functional class
C. Exertional stenocardia, III functional class
D. Vegetative-vascular dystonia of hypertensive type
E. Alcoholic cardiomyopathy

234. A 58-year-old patient complains about sensation of numbness, sudden


paleness of II-IV fingers, muscle rigidness, intermittent pulse. The patient
presents also with polyarthralgia, dysphagia, constipations. The patients face
is masklike, solid edema of hands is present. The heart is enlarged;
auscultation revealed dry rales in lungs. In blood: ESR- 20 mm/h, crude
protein - 85/l, ?-globulines - 25%. What is the most likely diagnosis?
A. Systemic scleroderma
B. Systemic lupus erythematosus
C. Raynauds disease
D. Dermatomyositis
E. Rheumatoid arthritis

235. A 45-year-old man has been exhibiting high activity for the last 2
weeks, he became talkative, euphoric, had little sleep, claimed being able "to
save the humanity" and solve the problem of cancer and AIDS, gave money
the starangers. What is the most likely diagnosis?
A. Maniacal onset
B. Schizo-affective disorder
C. Catatonic excitation
D. Panic disorder
E. Agitated depression

236. A patient had four generalized convulsive seizures within a day.


Between the seizures the patient did not come to waking consciousness (was
in a coma or stupor). Specify his state:
A. Status epilepticus
B. Frequent complex partial seizures
C. Frequent jacksonian seizures
D. Frequent generalized seizures
E. Hysterical attacks

237. A 35-year-old patient complains of heartburn, sour eructation, burning,


compressing retrosternal pain and pain along the esophagus rising during
forward bending of body. The patient hasnt been examined, takes Almagel on
his own initiative, claims to feel better after its taking. Make a provisional
diagnosis:
A. Gastroesophageal reflux disease
B. Cardiospasm
C. Duodenal ulcer
D. Gastric ulcer
E. Functional dyspepsia

238. As a result of lifting a load a 62-year-old female felt acute pain in the
lumbar region, in a buttock, posterolateral surface of her right thigh, external
surface of the right shin and dorsal surface of foot. Objectively: weakness of
the anterior tibial muscle, long extensor muscle of the right toes, short
extensor muscle of the right toes. Low Achilles reflex on the right. Positive
Lasegues sign. What examination method would be the most effective for
specification of the diagnosis of discogenic compression of L5 root?
A. Magnetic resonance scan
B. Lumbar puncture
C. Electromyography
D. Spinal column X-ray
E. Angiography

239. A 45-year-old female patient complaining of general weakness, nausea


and vomiting hass been delivered to a hospital by the ambulance. Recently
there has been a lack of appetite, weight loss. Objectively: hyperpigmentation
of skin, blood pressure at the rate of 70/45 mm Hg, bradycardia. Additional
studies revealed the reduced concentration of aldosterone and cortisol in
blood, decreased excretion of 17-ketosteroids and 17-oxyketosteroids in the
urine, hyponatremia, chloropenia, hypokalemia. What therapeutic measures
are required?
A. To administer glucocorticoids, mineralocorticoids, and a diet with
a high content of cooking salt
B. To administer aldosterone
C. To administer insulin
D. To prescribe a diet with a high content of cooking salt
E. To administer prednisolone

240. A 23-year-old female patient has a mental disease since the age of 18,
the course of disease has no remission periods. At a hospital the patient
mostly presents with non-purposeful foolish excitation: she makes stereotypic
grimaces, exposed, masturbating in front of a loud laugh, repeating the
stereotypical abusive shouts. The patient should be assigned:
A. Neuroleptics
B. Nootropics
C. Mood stabilizers
D. Antidepressants
E. Tranquilizers

241. A 40-year-old patient is registered in a narcological dispensary.


Somatically: skin is dramatically hyperemic, sclera are injected, hyperhidrosis
is present. AP- 140/100 mm Hg, heart rate - 100/min. Mental state:
autopsychic orientation is intact, allopsychic orientation is distorted. The
patient presents with motor anxiety. There is a look of fear on his face. He
refuses to talk about his problems and asks to release him immediately,
because he "may be killed." This state developed a day after a regular drinking
bout. What is your provisional diagnosis?
A. Delirium tremens
B. Paranoia
C. Organic delirium
D. Alcoholic hallucinosis
E. Alcoholic paranoid

242. During the preventive examination a 17-year-old young man reports no


health problems. Objectively: the patient is undernourished, asthenic; blood
pressure is 110/70 mm Hg, Ps - 80/min. Heart borders are within normal
range. Auscultation reveals three apical heart sounds, murmurs are absent.
ECG shows no pathological changes, PCG registers the S3 occurring 0,15
seconds after the S2. How can you interpret these changes?
A. Physiologic S3
B. Physiologic S4
C. Protodiastolic gallop rhythm
D. Fout-ta-ta-rou (three-component rhythm)
E. Presystolic gallop rhythm

243. A patient is being prepared for the operation on account of varix


dilatation of lower extremities veins. Examination of the patients soles
revealed flour-like desquamation along the skin folds. All the toenails are
greyish-yellow, thickened and partially decayed. What dermatosis should be
suspected?
A. Rubromycosis
B. Microsporia
C. Microbial eczema
D. Pityriasis versicolor
E. Candidosis

244. A 14-year-old patient with signs of internal haemorrhage has been


taken to a hospital after a fight. He has had haemophilia A since childhood. He
has been diagnosed with retroperitoneal hematoma. What should be
administered in the first place?
A. Cryoprecipitate
B. Dried plasma
C. Aminocapronic acid
D. Platelet concentrate
E. Fresh blood

245. A 58-year-old patient complains of a headache in the occipital region,


nausea, choking, opplotentes. The presentations appeared after a physical
exertion. Objectively: the patient is excited. Face is hyperemic. Skin is pale.
Heart sounds are regular, the 2nd aortic sound is accentuated. AP- 240/120
mm Hg, HR- 92/min. Auscultation reveals some fine moist rales in the lower
parts of the lungs. Liver is not enlarged. ECG shows signs of hypertrophy and
left ventricular overload. What is the most likely diagnosis?
A. Complicated hypertensic crisis, pulmonary edema
B. Bronchial asthma exacerbation
C. Acute myocardial infarction, pulmonary edema
D. Uncomplicated hypertensic crisis
E. Community-acquired pneumonia

246. A 37-year-old patient complains of pain in the lumbar spine that is


getting stronger during walking; restricted mobility, edema of the right side of
abdomen. He has a history of focal tuberculosis. X-ray picture shows the
destruction of the adjacent surfaces of the 1-2 vertebral bodies of the lumbar
spine, vertebral body height is decreased, intervertebral foramen is
undetectable. Abdominal ultrasound reveals a 15x20 cm formation in the
retroperitoneal space, there are echo signals of fluid presence. What is the
most likely diagnosis?
A. Tuberculous spondylitis of the lumbar spine
B. Spondylolisthesis of the lumbar spine
C. Osteochondrosis
D. Fracture of the 1-2 vertebral bodies of the lumbar spine
E. Spinal metastases

247. A 43-year-old female patient was delivered to the hospital in grave


condition. She has a history of Addisons disease. The patient had been
regularly taking prednisolone but a week before she stopped taking this drug.
Objectively: sopor, skin and visible mucous membranes are pigmented, skin
and muscle turgor is decreased. Heart sounds are muffled, rapid. AP- 60/40
mm Hg, heart rate - 96/min. In blood: Na - 120 millimole/l, K - 5,8
millimole/l. Development of this complication is primarily caused by the
deficit of the following hormone:
A. Cortisol
B. Corticotropin (ACTH)
C. Noradrenaline
D. Adrostendion
E. Adrenaline

248. In a cold weather, the emergency room admitted a patient pulled out of
the open water. There was no respiratory contact with the water. The patient
is excited, pale, complains of pain, numbness of hands and feet, cold shiver.
Respiratory rate is 22/min, AP - 120/90 mm Hg, Ps - 110/min, rectal
temperature is 34,5°C. What kind of warming is indicated for this patient?
A. Passive warming
B. Hot compresses
C. Hemodialysis with blood warming
D. Warm bath
E. Infusion of 37°C solutions

249. Survey radiograph of a 52-year-old worker of an agglomeration plant


(28 years of experience, the concentration of metal dust is 22-37 mg/m3)
shows mildly pronounced interstitial fibrosis with diffused contrast well-
defined small nodular shadows. The patient has no complaints. Pulmonary
function is not compromised. What is the provisional diagnosis?
A. Siderosis
B. Anthraco-silicatosis
C. Silicosis
D. Silicatosis
E. Anthracosis

250. A 60-year-old patient complains of nearly permanent sensation of


heaviness and fullness in the epigastrium, that increases after eating, foul-
smelling eructation, occasional vomiting with food consumed 1-2 days ago,
weight loss. 12 years ago he was found to have an ulcer of pyloric channel. The
patient has taken ranitidine for periodic hunger pain. The patients condition
has been deteriorating over the last 3 months. Objectively: splashing sound in
the epigastrium is present. What kind of complication is it?
A. Pyloric stenosis
B. Functional pyloric spasm
C. Malignization of gastric ulcer
D. Foreign body in the stomach (bezoar)
E. Penetration of gastric ulcer

251. A 52-year-old patient works as a secretary and has 30 year record of


service. She complains of spasms in her right hand during working and
inability to type and write. Up to 80% of her work involves hand load. The
patient has been presenting with these symptoms for 2 years. Objectively: the
right hand is tense, there is an increase in muscle tone, attempts to write cause
spasms. Examination revealed no pathological changes of CNS. What is the
most likely diagnosis?
A. Spastic form of coordination neurosis
B. Chronic manganese intoxication
C. Paretic form of coordination neurosis
D. Neuralgic form of coordination neurosis
E. Hysteric neurosis

252. Examination of an electric welder with 15 years of service record


revealed dry rales in the lower lung fields. Radiograph shows diffuse nodules
sized 3-4 mm in the middle and lower lung fields. What disease can be
suspected?
A. Heavy-metal coniosis
B. Carbon pneumo coniosis
C. Bronchitis
D. Silicosis
E. Silicatosis

253. A 22-year-old vegetarian patient with signs of malnutrition consulted a


doctor about smell and taste distortion, angular stomatitis. Objectively:
expressively blue sclerae. The patient was diagnosed with iron deficiency
anemia. What is the dominating clinical syndrome?
A. Sideropenic
B. Myelodysplastic
C. Haemologic
D. Anaemic
E. Haemolytic

254. A patient complains of retrosternal pain, difficult swallowing, over 10


kg weight loss within three months, general weakness. In blood: hypochromic
anaemia, neutrophilic leukocytosis. In feces: weakly positive Gregersens
reaction. On esophagram a filling defect with ill-defined serrated edges shows
up along a large portion of the esophagus. What is the most likely diagnosis?
A. Esophageal carcinoma
B. Esophageal achalasia
C. Benign tumour
D. Peptic ulcer
E. Sideropenic dysphagia

255. A 12-year-old boy periodically has short episodes (10-15 seconds) of a


brief loss of awareness with a dazed look and eyes stare in an upright position,
blank expression of face, absence of motions and subsequent amnesia. Specify
the described state:
A. Absence seizure
B. Fugue
C. Sperrung
D. Obnubilation
E. Trance

256. A 19-year-old male patient complains of intense pain in the left knee
joint. Objectively: the left knee joint is enlarged, the overlying skin is
hyperemic, the joint is painful on palpation. Blood test results: RBC -
3,8×1012/l, Hb - 122 g/l, lymphocytes - 7,4×109/l, platelets – 183×109/l. ESR
- 10 mm/h. Duke bleeding time is 4 minutes, Lee-White clotting time - 24
minutes. A-PTT is 89 s. Rheumatoid factor is negative. What is the most likely
diagnosis?
A. Hemophilia, hemarthrosis
B. Hemorrhagic vasculitis, articular form
C. Rheumatoid arthritis
D. Werlhofs disease
E. Thrombocytopathy

257. Explosion of a tank with benzene at a chemical plant has killed and
wounded a large number of people. There are over 50 victims with burns,
mechanical injuries and intoxication. Specify the main elements of medical
care and evacuation of population in this situation:
A. Sorting, medical assistance, evacuation
B. Isolation, rescue activity, recovery
C. Sorting, recovery, rescue activity
D. Sorting, evacuation, treatment
E. Medical assistance, evacuation, isolation

258. An emergency doctor has diagnosed a 32-year-old woman with


generalized convulsive status epilepticus. The deterioration in the patients
condition is caused by a sudden gap in the epilepsy treatment. Specify the
doctors further tactics:
A. Hospitalization in the intensive care unit
B. Outpatient monitoring by a neuropathologist
C. Outpatient monitoring by a neurosurgeon
D. Hospitalization in the department of neurology
E. Hospitalization in the department of neurosurgery

259. A 63-year-old male patient with persistent atrial fibrillation complains


of moderate dyspnea. Objectively: peripheral edemata are absent, vesicular
breathing is present, heart rate - 72/min, AP - 140/90 mm Hg. What
combination of drugs will be most effective for the secondary prevention of
heart failure?
A. Beta-blockers, ACE inhibitors
B. Cardiac glycosides, diuretics
C. Diuretics, beta-blockers
D. Cardiac glycosides, ACE inhibitors
E. Beta-blockers, cardiac glycosides

260. A 57-year-old male patient had an attack of retrosternal pain that lasted
more than 1,5 hours. Objectively: the patient is inert, adynamic, has pale skin,
cold extremities, poor volume pulse, heart rate - 120/min, AP - 70/40 mm Hg.
ECG shows ST elevation in II, III, aVF leads. What condition are these
changes typical for?
A. Cardiogenic shock
B. Acute pericarditis
C. Acute pancreatitis
D. Arrhythmogenic shock
E. Perforated gastric ulcer

261. A 42-year-old female lives in the basement, is unemployed,


undernourished. She complains of having general weakness, hair loss, brittle
nails for six months, likes to eat chalk. Objectively: the patient is emaciated,
pale, has dry skin. Peripheral lymph nodes are not enlarged. Liver is +1,5 cm.
In blood: RBCs - 1,8×1012/l, Hb- 62 g/l, colour index - 0,78, reticulocytes -
0,5 o/oo, ESR- 18 mm/h. Leukogram exhibits no pathology. What is a
provisional diagnosis?
A. Nutritional iron deficiency anaemia
B. B12-deficiency anaemia
C. Chronic hepatitis
D. Acquired haemolytic anaemia
E. Congenital haemolytic anaemia

262. A 20-year-old patient complains of breath shortness, continuous dull


heart pain, irritability. Objectively: general condition is satisfactory, the pulse
is labile, AP- 130/60 mm Hg. ECG shows repolarization disorder. The patient
has been diagnosed with cardiac-type neurocirculatory dystonia. The patient
should receive treatment under the following conditions:
A. Outpatient treatment
B. Inpatient treatment at the psychiatric department
C. Inpatient treatment at the cardiology department
D. Inpatient treatment at the therapeutic department
E. Inpatient treatment at the cardiac surgery department

263. A 45-year-old male patient complains of acute pain in his right side
irradiating to the right thigh and crotch. The patient claims also to have
frequent urination with urine which resembles a meat slops. The patient has
no previous history of this condition. There is costovertebral angle tenderness
on the right (positive Pasternatskys symptom). What is the most likely
diagnosis?
A. Urolithiasis
B. Acute appendicitis
C. Acute cholecystitis.
D. Acute pancreatitis
E. Acute pyelonephritis
264. A 38-year-old male works within the range of ionizing radiation. At a
routine medical examination he presents no problems. In blood: RBCs -
4,5×1012/l, Hb- 80 g/l, WBCs - 2,8×109/l, thrombocytes – 30×109/l. Decide
if this person can work with sources of ionizing radiation:
A. Working with radioactive substances and other sources of ionizing
radiation is contraindicated
B. The patient can only work with radioactive substances of low activity
C. The patient is allowed to work with radioactive substances for the limited
period of time
D. The patient can be allowed to work after an extended medical examination
E. The patient is allowed to work with radioactive substances

265. A patient who undergoes treatment at a tuberculosis clinic has


complained of having progressing headache for the last 3 weeks. Neurological
examination reveals rigidity of occipital muscles, no focal symptoms. What is
your provisional diagnosis?
A. Tuberculous meningitis
B. Myelitis
C. Convexital arachnoiditis
D. Chorea minor
E. Brain tumour

266. A patient with chronic suppurative otitis has developed severe


headache, vomiting, body temperature rise. The meningeal symptoms are
present. There are no focal neurological symptoms. The further tactics of a
doctor should be:
A. Urgent hospitalization and diagnostic lumbar puncture
B. Regular medical check-up
C. Skull radiography
D. Administration of anti-inflammatory drugs
E. Referral for a consultation with otolaryngologist

267. A 28-year-old male patient complains of sour regurgitation, cough and


heartburn that occurs every day after having meals, when bending forward or
lying down. These problems have been observed for 4 years. Objective status
and laboratory values are normal. FEGDS revealed endoesophagitis. What is
the leading factor in the development of this disease?
A. Failure of the lower esophageal sphincter
B. Duodeno-gastric reflux
C. Helicobacter pylori infection
D. Hypergastrinemia
E. Hypersecretion of hydrochloric acid
268. Routine examination of a 16-year-old boy revealed the presence of
three heart sounds on auscultation. The third sound is low and occurs in early
diastole, there is no additional murmur. In history: pneumonia six months
ago. The patient presents no problems. Examination revealed hyposthenia,
underdevelopment of muscles. Laboratory and instrumental studies reveald
no peculiarities. What is the origin of the additional heart sound?
A. Physiological III sound
B. Protodiastolic gallop rhythm
C. The sound of the tricuspid valve opening
D. Pericardial diastolic sound
E. The sound of the mitral valve opening

269. A patients condition is getting worse towards evening: she becomes


excited, complains of "internal anxiety", "a weight on her heart", foreboding of
evil - "something evil will happen to me or my family". The patient is sad,
melancholic, has poor appetite and sleep disorders. Specify the kind of mental
disorder:
A. Anxious depression
B. Somatized depression
C. Hypochondriac depression
D. Agitated depression
E. Endogenous depression

270. A 30-year-old male patient had been admitted to the TB hospital


because of the following changes detected by fluorography: an ill-defined
shadow of low intensity up to 1 cm in diameter in the S1 of the right lung. CT
scan showed a destruction area in the center of the shadow. Sputum analysis
revealed MTB. The patient was diagnosed with focal tuberculosis. What
phases of tuberculosis are the identified changes typical for?
A. Infiltration and disintegration
B. Infiltration and dissemination
C. Disintegration and dissemination
D. Calcification and resorption
E. Resorption and scarring

271. A 43-year-old female complains of significant weakness, sore throat,


occurrence of multiple unexplained bruises on her skin. These symptoms have
been present for a week, the disease is associated with quinsy which she had
some time before. Objectively: body temperature - 38,9°C, respiratory rate -
24/min, Ps - 110/min, AP - 100/65 mm Hg. The patient has pale skin,
petechial rash on the extremities, enlarged lymph nodes. Blood test results:
Hb - 80 g/l, RBC - 2,2*10^12/l; WBC - 3,5*10^9/l; blasts - 52%; eosinophils -
2%; stab neutrophils - 3%; segmented neutrophils - 19%; lymphocytes - 13%;
monocytes - 1%; platelets – 35*10^9/l. ESR - 47 mm/h. What test is required
to specify the diagnosis?
A. Immunophenotyping
B. Lymph node biopsy
C. Protein electrophoresis
D. Determination of anti-platelet antibody titer
E. Cytogenetic study

272. A 47-year-old male patient complains of compressive chest pain that


occurs both at rest and during light physical activity; irregular heartbeat.
These problems arose 3 months ago. The patients brother died suddenly at the
age of 30. Objectively: Ps - 84/min, arrhythmic, AP - 130/80 mm Hg. ECG
confirms signs of left ventricular hypertrophy, abnormal Q-waves in V4-V6
leads. Ech°CG reveals that interventricular septum is 1,7 cm, left ventricular
wall thickness is 1,2 cm. What is the most likely diagnosis?
A. Hypertrophic cardiomyopathy
B. Neurocirculatory asthenia
C. Myocarditis
D. Pericarditis
E. Exertional angina

Krok 2 – 2013 Gynecology Base


1. A 52-year-old woman suffering from obesity, complains of bloody discharges
from sexual paths during 4 days. Last normal menses were 2 years ago.
Histological investigation of biopsy of the endometrium has revealed
adenomatous hyperplasia. What reason from the mentioned below caused the
development of disease?
A. Excessive transformation of preandrogens from adipose tissues
B. Supersecretion of androgens by the cortex of paranephroses.
C. Poor aromatization of preandrogens due to hypothyroidism
D. Hypersecretion of estrogens by tissues of the organism
E. The increased contents of follicle-stimulating hormone

2. A 40-year-old woman complains of colic pains in the lower part of abdomen


and abundant bloody discharges from genital tract. Last 2 years she had
menses for 15-16 days, abundant, with clots, painful. Had 2 medical abortions.
In bimanual investigation: from the canal of the cervix uteri - a fibromatous
node, 3 cm in diameter, on the thin stem. Discharges are bloody,
moderate.Choose the correct tactics
A. Operation: untwisting of born node
B. Hormonal hemostasis
C. Supravaginal ablation of the uterus without ovaries
D. Hysterectomy without ovaries
E. Phase by phase vitamin therapy

3. A 40-year-old woman complains of yellow color discharges from the vagina.


Bimanual examination: no pathological changes. Smear test: Trichomonas
vaginalis and mixed flora. Colposcopy: two hazy fields on the front labium,
with a negative Iodum probing. What is your tactics?
A. Treatment of specific colpitis with the subsequent biopsy
B. Diathermocoagulation of the cervix uteri
C. Cervix ectomy
D. Cryolysis of cervix uteri
E. Specific treatment of Trichomonas colpitis

4. A 32 y.o. woman consulted a gynecologist about having abundant long menses


within 3 months. Bimanual investigation: the body of the uterus is enlarged
according to about 12 weeks of pregnancy, distorted, tuberous, of dense
consistence. Appendages are not palpated. Histological test of the uterus body
mucosa: adenocystous hyperplasia of endometrium. Optimal medical tactics:
A. Surgical treatment
B. Radial therapy
C. Phase by phase vitamin therapy
D. Hormonetherapy
E. Phytotherapy

5. A woman complains of having slight dark bloody discharges and mild pains in
the lower part of abdomen for several days. Last menses were 7 weeks ago.
The pregnancy test is positive. Bimanual investigation: the body of the uterus
indicates for about 5-6 weeks of pregnancy, it is soft, painless. In the left
appendage there is a retort-like formation, 7x5 cm large, mobile, painless.
What examination is necessary for detection of fetus localization?
A. Ultrasound
B. Hromohydrotubation
C. Cystoscopy
D. Colposcopy
E. Hysteroscopy

6. A woman was hospitalised with full-term pregnancy. Examination: the uterus


is tender, the abdomen is tense, cardiac tones of the fetus are not auscultated.
What is the most probable complication of pregnancy?
A. Premature detachment of normally posed placenta
B. Hydramnion
C. Back occipital presentation
D. Premature labor
E. Acute hypoxia of a fetus

7. By the end of the 1st period of physiological labor clear amniotic fluid came
off. Contractions lasted 35-40 sec every 4-5min. Heartbeat of the fetus was
100 bpm. The BP was 140/90 mm Hg. What is the most probable diagnosis?
A. Acute hypoxia of the fetus
B. Premature detachment of normally posed placenta
C. Premature labor
D. Back occipital presentation
E. Hydramnion

8. A pregnant woman in her 40th week of pregnancy undergoes obstetric


examination: the cervix of uterus is undeveloped. The oxytocin test is
negative. Examination at 32 weeks revealed: AP 140/90 mm Hg, proteinuria 1
g/l, peripheral edemata. Reflexes are normal. Choose the most correct tactics:
A. Labour stimulation after preparation
B. Complex therapy of gestosis for 7 days
C. Complex therapy of gestosis for 2 days
D. Absolute bed rest for 1 month
E. Caesarian section immediately

9. A 26 year old woman had the second labour within the last 2 years with
oxytocin application. The childs weight is 4080 g. After the placent birth there
were massive bleeding, signs of hemorrhagic shock. Despite the injection of
contractive agents, good contraction of the uterus and absence of any cervical
and vaginal disorders, the bleeding proceeds. Choose the most probable cause
of bleeding:
A. Atony of the uterus
B. Hypotonia of the uterus
C. Hysterorrhexis
D. Injury of cervix of the uterus
E. Delay of the part of placenta

10. Which gestational age gives the most accurate estimation of weeks of
pregnancy by uterine size?
A. Less than 12 weeks
B. Between 12 and 20 weeks
C. Between 31 and 40 weeks
D. Over 40 weeks
E. Between 21 and 30 weeks
11. A woman is admitted to maternity home with discontinued labor activity and
slight bloody discharges from vagina. The condition is severe, the skin is pale,
consciousness is confused. BP is 80/40 mm Hg. Heartbeat of the fetus is not
heard. There was a Cesarian section a year ago. Could you please determine
the diagnosis?
A. Hysterorrhesis
B. Placental presentation
C. Cord presentation
D. Expulsion of the mucous plug from cervix uteri
E. Premature expulsion of amniotic fluid

12. Rise in temperature up to 39°C was registered the next day after a woman had
labor. Fetal membranes rupture took place 36 hours prior to labors. The
examination of the bacterial flora of cervix uteri revealed the following:
haemolytic streptococcus of group A. The uterus tissue is soft, tender.
Discharges are bloody, with mixing of pus. Establish the most probable
postnatal complication
A. Metroendometritis
B. Infected hematoma
C. Thrombophlebitis of veins of the pelvis
D. Infective contamination of the urinary system
E. Apostatis of stitches after the episiotomy

13. On the first day after labour a woman had the rise of temperature up to 39°C.
Rupture of fetal membranes took place 36 hours before labour. Examination
of the bacterial flora of cervix of the uterus revealed hemocatheretic
streptococcus of A group. The uterus body is soft, tender. Discharges are
bloody, with admixtures of pus. Specify the most probable postnatal
complication:
A. Metroendometritis
B. Infectious hematoma
C. Apostasis of sutures after the episiotomy
D. Infective contamination of the urinary system
E. Thrombophlebitis of veins of the pelvis

14. A woman of a high-risk group (chronic pyelonephritis in anamnesis) had


vaginal delivery. The day after labour she complained of fever and loin pains,
frequent urodynia. Specify the most probable complication:
A. Infectious contamination of the urinary system
B. Infectious hematoma
C. Thrombophlebitis of veins of the pelvis
D. Endometritis
E. Apostasis of sutures after episiotomy
15. 13 months after the first labor a 24-year-old patient complained of
amenorrhea. Pregnancy ended in Caesarian section because of premature
detachment of normally positioned placenta which resulted in blood loss at
the rate of 2000 ml owing to disturbance of blood clotting. Choose the most
suitable investigation:
A. Estimation of gonadotropin rate
B. USI of small pelvis organs
C. Computer tomography of head
D. Estimation of testosteron rate in blood serum
E. Progesteron assay

16. In 13 months after the first labor a 24-year-old woman complains of


amenorrhea. Cesarian section was conducted as a result of premature
detachment of normally posed placenta. Hemorrhage has made low fidelity of
2000 ml due to breakdown of coagulation of blood. Choose the most suitable
investigation
A. Determination of the level of gonadotropin
B. Determination of the contents of testosteron-depotum in blood serum
C. Progesteron test
D. Ultrasound of organs of a small pelvis
E. Computer tomography of the head

17. In the woman of 24 years about earlier normal menstrual function, cycles
became irregular, according to tests of function diagnostics - anovulatory. The
contents of prolactin in blood is boosted. Choose the most suitable
investigation:
A. Computer tomography of the head
B. Determination of the level of gonadotropins
C. Progesterone assay
D. Determination of the contents of testosteron-depotum in blood serum
E. USI of organs of small pelvis

18. A 20-year-old woman is having timed labor continued for 4 hours. Light
amniotic fluid came off. The fetus head is pressed to the orifice in the small
pelvis. The anticipated fetus mass is 4000,0 gpm 200,0 g. Heartbeat of the
fetus is normal. Intrinsic examination: cervix is absent, disclosure – 2 cm, the
fetal membranes are not present. The head is in 1-st plane of the pelvis, a
sagittal suture is in the left slanting dimension. What is the purpose of
glucose-calcium-hormone - vitaminized background conduction?
A. Prophylaxes of weakness of labor activity
B. Labor stimulation
C. Antenatal preparation
D. Treatment of weakness of labor activity.
E. Fetus hypoxia prophylaxes

19. A woman in her 39th week of pregnancy, the second labour, has regular birth
activity. Uterine contractions take place every 3 minutes. What criteria
describe the beginning of the II labor stage the most precisely?
A. Cervical dilatation by no less than 4 cm
B. Rupture of fetal bladder
C. Duration of uterine contractions over 30 seconds
D. Cervical smoothing over 90%
E. Presenting part is in the lower region of small pelvis

20. A 24 years old primipara was hospitalised with complaints about discharge of
the amniotic waters. The uterus is tonic on palpation. The position of the fetus
is longitudinal, it is pressed with the head to pelvic outlet. Palpitation of the
fetus is rhythmical, 140 bpm, auscultated on the left below the navel. Internal
examination: cervix of the uterus is 2,5 cm long, dense, the external os is
closed, light amniotic waters out of it. Point a correct component of the
diagnosis:
A. Antenatal discharge of the amniotic waters
B. The beginning of the 1st stage of labour
C. Pathological preterm labour
D. The end of the 1st stage of labour
E. Early discharge of the amniotic waters

21. A 29 year old patient underwent surgical treatment because of the benign
serous epithelial tumour of an ovary. The postoperative period has elapsed
without complications. What is it necessary to prescribe for the
rehabilitational period:
A. Hormonotherapy and proteolytic enzymes
B. Lasertherapy and enzymotherapy
C. Antibacterial therapy and adaptogens
D. Magnitotherapy and vitamin therapy
E. The patient does not require further care

22. A 34 y.o. woman in her 29-th week of pregnancy, that is her 4-th labor to
come, was admitted to the obstetric department with complaints of sudden
and painful bloody discharges from vagina that appeared 2 hours ago. The
discharges are profuse and contain grumes. Cardiac funnction of the fetus is
rhytmic, 150 strokes in the minute, uterus tone is normal. The most probable
provisional diagnosis will be:
A. Placental presentation
B. Bloody discharges
C. Disseminated intravascular coagulation syndrome
D. Detachment of normally located placenta
E. Vasa previa

23. A 34-year-old woman with 10-week pregnancy (the second pregnancy) has
consulted gynaecologist to make a record in patient chart. There was a
hydramnion previous pregnancy, the birth weight of a child was 4086 g. What
tests are necessary first of all?
A. The test for tolerance to glucose
B. Ultrasound of the fetus
C. Bacteriological test of discharge from the vagina
D. Determination of the contents of alpha fetoprotein
E. Fetus cardiophonography

24. A 26 y.o. woman complains of sudden pains in the bottom of abdomen


irradiating to the anus, nausea, giddiness, bloody dark discharges from sexual
tracts for one week, the delay of menses for 4 weeks. Signs of the peritoneum
irritation are positive. Bimanual examination: borders of the uterus body and
its appendages are not determined because of sharp painfullness. The
diverticulum and painfullness of the back and dextral fornixes of the vagina
are evident. What is the most probable diagnosis?
A. Broken tubal pregnancy
B. Acute appendicitis
C. Acute right-side adnexitis
D. Apoplexy of the ovary
E. Torsion of the crus of the ovary tumour

25. At the gynaecological department there is a patient of 32 years with the


diagnosis: "acute bartholinitis".Body temperature is 38,20C, leucocytes count
10,4109/L, the ESR is 24 mm/hour. In the area of big gland of the vestibulum
- the dermahemia, the sign of the fluctuation, sharp tenderness (pain). What is
the most correct tactics of the doctor?
A. Surgical dissecting, a drainage of an abscess of the gland,
antibiotics
B. Surgical dissection, drainage of the abscess of the gland
C. Antibiotics, detoxication and biostimulants.
D. Antibiotic therapy
E. Antibiotics, Sulfanilamidums

26. A primagravida with pregnancy of 37-38 weeks complains of headache,


nausea, pain in epigastrium. Objective: the skin is acyanotic. Face is hydropic,
there is short fibrillar twitching of blepharons, muscles of the face and the
inferior extremities. The look is fixed. AP- 200/110 mm Hg; sphygmus of 92
bpm, intense. Respiration rate is 32/min. Heart activity is rhythmical.
Appreciable edemata of the inferior extremities are present. Urine is cloudy.
What medication should be administered?
A. Droperidolum of 0,25% - 2,0 ml
B. Papaverine hydrochloride of 2% - 4,0 ml
C. Dibazolum of 1% - 6,0 ml
D. Hexenalum of 1% - 2,0 ml
E. Pentaminum of 5% - 4,0 ml

27. An onset of severe preeclampsia at 16 weeks gestation might be caused by:


A. Hydatidiform mole
B. Twin gestation
C. Anencephaly
D. Maternal renal disease
E. Interventricular defect of the fetus

28. A woman had the rise of temperature up to 39°C on the first day after labour.
The rupture of fetal membranes took place 36 hours before labour. The
investigation of the bacterial flora of cervix of the uterus revealed
hemocatheretic streptococcus of group A. The uterus body is soft, tender.
Discharges are bloody, mixed with pus. Specify the most probable postnatal
complication:
A. Metroendometritis
B. Infected hematoma
C. Thrombophlebitis of pelvic veins
D. Infection of the urinary system
E. Apostatis of junctures after the episiotomy

29. A 24 y.o. patient 13 months after the first labour consulted a doctor about
amenorrhea. Pregnancy has concluded by a Cesarean section concerning to a
premature detachment of normally posed placenta hemorrhage has made low
fidelity 2000 ml owing to breakdown of coagulability of blood. Choose the
most suitable investigation:
A. Determination of the level of Gonadotropins
B. Progesteron assay
C. USI of organs of a small pelvis
D. Computer tomography of the head
E. Determination of the contents of Testosteron-Depotum in Serum of blood

30. A 34 year old woman in the 10th week of gestation (the second pregnancy)
consulted a doctor of antenatal clinic in order to be registered there. In the
previous pregnancy hydramnion was observed, the childs birth weight was
4086 g. What examination method should be applied in the first place?
A. The test for tolerance to glucose
B. A cardiophonography of fetus
C. US of fetus
D. Determination of the contents of fetoproteinum
E. Bacteriological examination of discharges from vagina

31. A 10 y.o. boy was ill with angina 2 weeks ago, has complaints of joint pain and
stiffness of his left knee and right elbow. There was fever (38,50) and ankle
disfunction, enlargement of cardiac dullness by 2 cm, tachycardia, weakness
of the 1st sound, gallop rhythm, weak systolic murmur near apex. What
diagnosis corresponds with such symptoms?
A. Acute rheumatic fever
B. Systemic lupus erythematosis
C. Reiters disease
D. Reactive arthritis
E. Juvenile rheumatoid arthritis

32. The disease began acutely. The frequent watery stool developed 6 hours ago.
The bodys temperature is normal. Then the vomiting was joined. On
examination: his voice is hoarse, eyes are deeply sunken in the orbits. The
pulse is frequent. Blood pressure is low. There is no urine. What is the
preliminary diagnosis?
A. Cholera
B. Salmonellosis
C. Toxic food-borne infection
D. Dysentery
E. Typhoid fever

33. At term of a gestation of 40 weeks height of standing of a uterine fundus is less


then assumed for the given term. The woman has given birth to the child in
weight of 2500 g, a length of a body 53 cm, with an assessment on a scale of
Apgar of 4-6 points. Labor were fast. The cause of such state of the child were:
A. Chronic fetoplacental insufficiency
B. Placental detachment
C. Prematurity
D. Infection of a fetus
E. Delay of an intra-uterine fetation

34. A pregnant woman may be diagnosed with hepatitis if it is confirmed by the


presence of elevated:
A. SGOT (ALT)
B. WBCs
C. BUN
D. Alkaline phosphatase
E. Sedimentation rates

35. A woman, aged 40, primigravida, with infertility in the medical history, on the
42-43 week of pregnancy. Labour activity is weak. Longitudinal presentation
of the fetus, I position, anterior position. The head of the fetus is engaged to
pelvic inlet. Fetus heart rate is 140 bmp, rhythmic, muffled. Cervix dilation is
4 cm. On amnioscopy: greenish colour of amniotic fluid and fetal membranes.
Cranial bones are dense, cranial sutures and small fontanel are diminished.
What should be tactics of delivery?
A. Caesarean section
B. Amniotomy, labour stimulation, fetal hypoxia treatment
C. Fetal hypoxia treatment, conservative delivery
D. Medication sleep, amniotomy, labour stimulation
E. Fetal hypoxia treatment, in the ?? period - forceps delivery

36. An endometrial adenocarcinoma that has extended to the uterine serosa


would be classified as stage:
A. IIIA
B. IIA
C. IVAB
D. IIB
E. IC

37. Which of the methods of examination is the most informative in the


diagnostics of a tube infertility?
A. Laparoscopy with chromosalpingoscopy
B. Transvaginal echography
C. Bicontrast pelviography
D. Pertubation
E. Hysterosalpingography

38. A pregnant woman (35 weeks), aged 25, was admitted to the hospital because
of bloody discharges. In her medical history there were two artificial
abortions. In a period of 28-32 weeks there was noted the onset of
hemorrhage and USD showed a placental presentation. The uterus is in
normotonus, the fetus position is transversal (Ist position). The heartbeats is
clear, rhythmical, 140 bpm. What is the further tactics of the pregnant woman
care?
A. To perform a delivery by means of Cesarean section
B. To introduct the drugs to increase the blood coagulation and continue
observation
C. To keep the intensity of hemorrhage under observation and after the bleeding
is controlled to prolong the pregnancy
D. Stimulate the delivery by intravenous introduction of oxytocin
E. To perform the hemotransfusion and to prolong the pregnancy

39. A woman, primagravida, consults a gynecologist on 05.03.2012. A week ago


she felt the fetus movements for the first time. Last menstruation was on
10.01.2012. When should she be given maternity leave?
A. 8 August
B. 11 July
C. 5 September
D. 25 July
E. 22 August

40. Condition of a parturient woman has been good for 2 hours after live birth:
uterus is thick, globe-shaped, its bottom is at the level of umbilicus, bleeding
is absent. The clamp put on the umbilical cord remains at the same level,
when the woman takes a deep breath or she is being pressed over the
symphysis with the verge of hand, the umbilical cord drows into the vagina.
Bloody discharges from the sexual tracts are absent. What is the doctors
further tactics?
A. To do manual removal of afterbirth
B. To apply Abduladze method
C. To do curettage of uterine cavity
D. To introduct oxitocine intravenously
E. To apply Credes method

41. The woman who has delivered twins has early postnatal hypotonic uterine
bleeding reached 1,5% of her bodyweight. The bleeding is going on.
Conservative methods to arrest the bleeding have been found ineffective. The
conditions of patient are pale skin, acrocyanosis, oliguria. The woman is
confused. The pulse is 130 bpm, BP– 75/50 mm Hg. What is the further
treatment?
A. Uterine extirpation
B. Uterine vessels ligation
C. Supravaginal uterine amputation
D. Inner glomal artery ligation
E. Putting clamps on the uterine cervix

42. A 26 y.o. woman complains of a mild bloody discharge from the vagina and
pain in the lower abdomen. She has had the last menstruation 3,5 months ago.
The pulse is 80 bpm. The blood pressure (BP) is 110/60 mm Hg and body
temperature is 36,60C. The abdomen is tender in the lower parts. The uterus
is enlarged up to 12 weeks of gestation. What is your diagnosis?
A. Inevitable abortion
B. Disfunctional bleeding
C. Incomplete abortion
D. Incipient abortion
E. Complete abortion

43. 18 y.o. woman complains of pain in the lower abdomen. Some minutes before
she has suddenly appeared unconscious at home. The patient had no menses
within last 3 months. On examination: pale skin, the pulse- 110 bpm, BP-
80/60 mm Hg. The Schyotkins sign is positive. Hb- 76 g/L. The vaginal
examination: the uterus is a little bit enlarged, its displacement is painful.
There is also any lateral swelling of indistinct size. The posterior fornix of the
vagina is tendern and overhangs inside. What is the most probable diagnosis?
A. Impaired extrauterine pregnancy
B. Acute salpingoophoritis
C. Acute appendicitis
D. Ovarian apoplexy
E. Twist of cystoma of right uterine adnexa

44. A 20 y.o. pregnant woman with 36 weeks of gestation was admitted to the
obstetrical hospital with complains of pain in the lower abdomen and bloody
vaginal discharge. The general condition of the patient is good. Her blood
pressure is 120/80 mm Hg. The heart rate of the fetus is 140 bpm, rhythmic.
Vaginal examination: the cervix of the uterus is formed and closed. The
discharge from vagina is bloody up to 200 ml per day. The head of the fetus is
located high above the minor pelvis entry. A soft formation was defined
through the anterior fornix of the vagina. What is the probable diagnosis?
A. Placental presentation
B. Incipient abortion
C. Uterine rupture
D. Premature placental separation
E. Threatened premature labor

45. In the gynecologic office a 28 y.o. woman complains of sterility within three
years. The menstrual function is not impaired. There were one artificial
abortion and chronic salpingo-oophoritis in her case history. Oral
contraceptives were not used. Her husbands analysis of semen is without
pathology. What diagnostic method will you start from the workup in this case
of sterility?
A. Hysterosalpingography
B. Diagnostic scraping out of the uterine cavity
C. Hysteroscopia
D. Hormone investigation
E. Ultra sound investigation

46. A 28-year-old patient underwent endometrectomy as a result of incomplete


abortion. Blood loss was at the rate of 900 ml. It was necessary to start
hemotransfusion. After transfusion of 60 ml of erythrocytic mass the patient
presented with lumbar pain and fever which resulted in hemotransfusion
stoppage. 20 minutes later the patients condition got worse: she developed
adynamia, apparent skin pallor, acrocyanosis, profuse perspiration. to-
38,5°C, Ps- 110/min, AP- 70/40 mm Hg. What is the most likely diagnosis?
A. Hemotransfusion shock
B. Septic shock
C. Hemorrhagic shock
D. Anaphylactic shock
E. DIC syndrome

47. A 58-year-old female patient came to the antenatal clinic complaining of


bloody light-red discharges from the genital tracts. Menopause is 12 years.
Gynaecological examination revealed age involution of externalia and vagina;
uterine cervix was unchanged, there were scant bloody discharges from
uterine cervix, uterus was of normal size; uterine appendages were not
palpable; parametria were free. What is the most likely diagnosis?
A. Uterine carcinoma
B. Abnormalities of menstrual cycle of climacteric nature
C. Atrophic colpitis
D. Cervical carcinoma
E. Granulosa cell tumor of ovary

48. The results of a separate diagnostic curettage of the mucous of the uterus
cervix and body made up in connection with bleeding in a postmenopausal
period: the scrape of the mucous of the cervical canal revealed no pathology,
in endometrium - the highly differentiated adenocarcinoma was found.
Metastases are not found. What method of treatment is the most correct?
A. Surgical treatment and hormonotherapy
B. Surgical treatment + chemotherapy
C. Radial therapy
D. Surgical treatment and radial therapy

49. A 27 y.o. woman complains of having the disoders of menstrual function for 3
months, irregular pains in abdomen. On bimanual examination: in the dextral
appendage range of uterus there is an elastic spherical formation, painless, 7
cm in diameter. USI: in the right ovary - a fluid formation, 4 cm in diameter,
unicameral, smooth. What method of treatment is the most preferable?
A. Prescription of an estrogen-gestogen complex for 3 months with
repeated examination
B. Dispensary observation of the patient
C. Operative treatment
D. Anti-inflammatory therapy
E. Chemotherapeutic treatment

50. A 40 year old patient complains of yellowish discharges from the vagina.
Bimanual examination revealed no pathological changes. The smear contains
Trichomonas vaginalis and blended flora. Colposcopy revealed two hazy fields
on the frontal labium, with a negative Iodine test. Your tactics:
A. Treatment of specific colpitis and subsequent biopsy
B. Cervix ectomy
C. Cryolysis of cervix of the uterus
D. Diathermocoagulation of the cervix of the uterus
E. Specific treatment of Trichomonas colpitis

51. A 28-year-old parturient complains about headache, vision impairment,


psychic inhibition. Objectively: AP- 200/110 mm Hg, evident edemata of legs
and anterior abdominal wall. Fetus head is in the area of small pelvis. Fetal
heartbeats is clear, rhythmic, 190/min. Internal examination revealed
complete cervical dilatation, fetus head was in the area of small pelvis. What
tactics of labor management should be chosen?
A. Forceps operation
B. Embryotomy
C. Stimulation of labor activity
D. Conservative labor management with episiotomy
E. Cesarean

52. A 28 year old woman had the second labour and born a girl with
manifestations of anemia and progressing jaundice. The child’s weight was
3400 g, the length was 52 cm. The womans blood group is B (III) Rh-, the
fathers blood group is A (III) Rh+, the child’s blood group is B (III) Rh+. What
is the cause of anemia?
A. Rhesus incompatibility
B. Intrauterine infection
C. Antigen B incompatibility
D. Antigen A incompatibility
E. Antigen AB incompatibility

53. A 48 year old female patient complains about contact haemorrhage. Speculum
examination revealed hypertrophy of uterus cervix. It resembles of
cauliflower, it is dense and can be easily injured. Bimanual examination
revealed that fornices were shortened, uterine body was nonmobile. What is
the most probable diagnosis?
A. Cervical carcinoma
B. Cervical pregnancy
C. Cervical papillomatosis
D. Metrofibroma
E. Endometriosis

54. A 37 y.o. primigravida woman has been having labor activity for 10 hours.
Labor pains last for 20-25 seconds every 6-7 minutes. The fetus lies in
longitude, presentation is cephalic, head is pressed upon the entrance to the
small pelvis. Vaginal examination results: cervix of uterus is up to 1 cm long,
lets 2 transverse fingers in. Fetal bladder is absent. What is the most probable
diagnosis?
A. Primary uterine inertia
B. Secondary uterine inertia
C. Discoordinated labor activity
D. Pathological preliminary period
E. Normal labor activity

55. Laparotomy was performed to a 54 y.o. woman on account of big formation in


pelvis that turned out to be one-sided ovarian tumor along with considerable
omental metastases. The most appropriate intraoperative tactics involves:
A. Ablation of omentum, uterus and both ovaries with tubes
B. Ablation of omentum and both ovaries with tubes
C. Biopsy of an ovary
D. Biopsy of omentum
E. Ablation of an ovary and omental metastases

56. A parturient complains about pain in the mammary gland. Palpation revealed
a 3?4 cm large infiltration, soft in the centre. Body temperature is 38,5°C.
What is the most probable diagnosis?
A. Acute purulent mastitis
B. Pneumonia
C. Retention of milk
D. Birth trauma
E. Pleuritis

57. A 43 y.o. patient complains of formation and pain in the right mammary
gland, rise of temperature up to 37,20C during the last 3 months. Condition
worsens before the menstruation. On examination: edema of the right breast,
hyperemia, retracted nipple. Unclear painful infiltration is palpated in the
lower quadrants. What is the most probable diagnosis?
A. Cancer of the right mammary gland
B. Right-side chronic mastitis
C. Right-side acute mastitis
D. Premenstrual syndrome
E. Tuberculosis of the right mammary gland

58. A 14 year old girl complains of profuse bloody discharges from genital tracts
during 10 days after suppresion of menses for 1,5 month. Similiar bleedings
recur since 12 years on the background of disordered menstrual cycle. On
rectal examination: no pathology of the internal genitalia. In blood: ?b - 70
g/l, RBC- 2,3×1012/l, Ht - 20. What is the most probable diagnosis?
A. Juvenile bleeding, posthemorrhagic anemia
B. Polycyst ovarian syndrome
C. Incomplete spontaneous abortion
D. Hormonoproductive ovary tumor
E. Werlholfs disease

59. A 33-year-old woman was urgently brought to clinic with complaints of the
pain in the lower part of the abdomen, mostly on the right, irradiating to
rectum, she also felt dizzy. The above mentioned complaints developed
acutely at night. Last menses were 2 weeks ago. On physical exam: the skin is
pale, Ps - 92 bpm, t- 36,6OC, BP- 100/60 mm Hg. The abdomen is tense,
slightly tender in lower parts, peritoneal symptoms are slightly positive. Hb-
98 g/L. What is the most probable diagnosis?
A. Apoplexy of the ovary
B. Abdominal pregnancy
C. Renal colic
D. Acute appendicitis
E. Intestinal obstruction

60. A secundipara has regular birth activity. Three years ago she had cesarean
section for the reason of acute intrauterine hypoxia. During parodynia she
complains of extended pain in the area of postsurgical scar. Objectively: fetus
pulse is rhythmic - 140 bpm. Vaginal examination shows 5 cm cervical
dilatation. Fetal bladder is intact. What is the tactics of choice?
A. Cesarean section
B. Waiting tactics of labor management
C. Vaginal delivery
D. Augmentation of labour
E. Obstetrical forceps
61. A 54-year-old female patient consulted a doctor about bloody discharges from
the genital tracts after 2 years of amenorrhea. USI and bimanual examination
revealed no genital pathology. What is the tactics of choice?
A. Fractional biopsy of lining of uterus and uterine mucous
membranes
B. Styptic drugs
C. Estrogenic haemostasia
D. Hysterectomy
E. Contracting drugs

62. Examination of a just born placenta reveals defect 2x3 cm large. Hemorrhage
is absent. What tactic is the most reasonable?
A. Manual uretus cavity revision
B. Instrumental uterus cavity revision
C. External uterus massage
D. Prescription of uterotonic medicines
E. Parturient supervision

63. A 27 y.o. gravida with 17 weeks of gestation was admitted to the hospital.
There was a history of 2 spontaneous miscarriages. On bimanual examination:
uterus is enlarged to 17 weeks of gestation, uterus cervix is shortened, isthmus
allows to pass the finger tip. The diagnosis is isthmico-cervical insufficiency.
What is the doctors tactics?
A. To place suture on the uterus cervix
B. To administer hormonal treatment
C. To perform amniocentesis
D. To administer tocolytic therapy
E. To interrupt pregnancy

64. A 27-year-old woman presents at the maternity welfare centre because of


infertility. She has had sexual life in marriage for 4 years, doesnt use
contraceptives. She hasnt get pregnant. On examination: genital development
is without pathology, uterine tubes are passable, basal (rectal) temperature is
one-phase during last 3 menstrual cycles. What is the infertility cause?
A. Anovular menstrual cycle
B. Abnormalities in genital development
C. Chronic adnexitis
D. Immunologic infertility
E. Genital endometriosis

65. A 43 y.o. woman complains of contact hemorrhages during the last 6 months.
Bimanual examination: cervix of the uterus is enlarged, its mobility is
reduced. Mirrors showed the following: cervix of the uterus is in the form of
cauliflower. Chrobak and Schiller tests are positive. What is the most probable
diagnosis?
A. Cancer of cervix of the uterus
B. Polypus of the cervis of the uterus
C. Nascent fibroid
D. Leukoplakia
E. Cervical pregnancy

66. A 26-year-old woman gave birth to a child 6 months ago. She applied to
gynecologist complaining of menstruation absence. The child is breast-fed.
Vagina exam: uterus is of normal form, dense consistence. What is the most
probable diagnosis?
A. Physiological amenorrhea
B. Pseudoamenorrhea
C. Ashermans syndrome
D. Sheehans syndrome
E. Gestation

67. A primagravida in her 20th week of gestation complains about pain in her
lower abdomen, blood smears from the genital tracts. The uterus has an
increased tonus, the patient feels the fetus movements. Bimanual examination
revealed that the uterus size corresponded the term of gestation, the uterine
cervix was contracted down to 0,5 cm, the external orifice was open by 2 cm.
The discharges were bloody and smeary. What is the most likely diagnosis?
A. Incipient abortion
B. Abortion in progress
C. Risk of abortion
D. Incomplete abortion
E. Missed miscarriage

68. Full-term pregnancy. Body weight of the pregnant woman is 62 kg. The fetus
has the longitudinal position, the fetal head is pressed against the pelvic inlet.
Abdominal circumference is 100 cm. Fundal height is 35 cm. What is the
approximate weight of the fetus?
A. 3 kg 500 g
B. 2 kg 500 g
C. 4 kg
D. 3 kg
E. 4 kg 500 g

69. A patient was admitted to the hospital with complaints of periodical pain in
the lower part of abdomen that gets worse during menses, weakness, malaise,
nervousness, dark bloody smears from vagina directly before and after
menses. Bimanual examination revealed that uterus body is enlarged,
appendages cannot be palpated, posterior fornix has tuberous surface.
Laparoscopy revealed: ovaries, peritoneum of rectouterine pouch and
pararectal fat have "cyanotic eyes". What is the most probable diagnosis?
A. Disseminated form of endometriosis
B. Tuberculosis of genital organs
C. Ovarian cystoma
D. Polycystic ovaries
E. Chronic salpingitis

70. A gravida with 7 weeks of gestation is referred for the artificial abortion. On
operation while dilating cervical canal with Hegar dilator No.8 a doctor
suspected uterus perforation. What is immediate doctors tactics to confirm
the diagnosis?
A. Probing of uterus cavity
B. Ultrasound examination
C. Bimanual examination
D. Laparoscopy
E. Metrosalpingography

71. A pregnant woman in her 8th week was admitted to the hospital for artificial
abortion. In course of operation during dilatation of cervical canal of uterus by
means of Hegars dilator No.8 the doctor suspected uterus perforation. What is
the immediate tactics for confirmation of this diagnosis?
A. Uterine probing
B. Bimanual examination
C. Laparoscopy
D. Metrosalpingography
E. US examination

72. A 59 year old female patient applied to a maternity welfare clinic and
complained about bloody discharges from the genital tracts. Postmenopause is
12 years. Vaginal examination revealed that external genital organs had signs
of age involution, uterus cervix was not erosive, small amount of bloody
discharges came from the cervical canal. Uterus was of normal size, uterine
appendages were unpalpable. Fornices were deep and painless. What method
should be applied for the diagnosis specification?
A. Separated diagnosic curretage
B. Puncture of abdominal cavity through posterior vaginal fornix
C. Laparoscopy
D. Extensive colposcopy
E. Culdoscopy
73. A 25-year-old woman complains of profuse foamy vaginal discharges, foul,
burning and itching in genitalia region. She has been ill for a week.
Extramarital sexual life. On examination: hyperemia of vaginal mucous,
bleeding on touching, foamy leucorrhea in the urethral area. What is the most
probable diagnosis?
A. Trichomonas colpitic
B. Gonorrhea
C. Vagina candidomicosis
D. Bacterial vaginosis
E. Chlamydiosis

74. A 26 year old woman who delivered a child 7 months ago has been suffering
from nausea, morning vomiting, sleepiness for the last 2 weeks. She suckles
the child, menstruation is absent. She hasn’t applied any contraceptives. What
method should be applied in order to specify her diagnosis?
A. Ultrasonic examination
B. Speculum examination
C. Palpation of mammary glands and pressing-out of colostrum
D. Roentgenography of small pelvis organs
E. Bimanual vaginal examination

75. A newborns head is of dolichocephalic shape, that is front-to-back elongated.


Examination of the occipital region revealed a labour tumour located in the
middle between the prefontanel and posterior fontanel. Specify the type of
fetal presentation:
A. Posterior vertex presentation
B. Presentation of the bregma
C. Face presentation
D. Brow presentation
E. Anterior vertex presentation

76. A woman consulted a doctor on the 14th day after labour about sudden pain,
hyperemy and induration of the left mammary gland, body temperature rise
up to 39°C, headache, indisposition. Objectively: fissure of nipple,
enlargement of the left mammary gland, pain on palpation. What pathology
would you think about in this case?
A. Lactational mastitis
B. Phlegmon of mammary gland
C. Fibrous adenoma of the left mammary gland
D. Lacteal cyst with suppuration
E. Breast cancer
77. A young woman applied to gynecologist due to her pregnancy of 4-5 weeks.
The pregnancy is desirable. Anamnesis stated that she had rheumatism in the
childhood. Now she has combined mitral heart disease with the priority of
mitral valve deficiency. When will she need the inpatient treatment (what
periods of pregnancy)?
A. 8-12 weeks, 28–32 weeks, 37 weeks
B. 16 weeks, 34 weeks, 39-40 weeks
C. 6-7weeks, 16 weeks, 38 weeks
D. 10-12 weeks, 24 weeks, 37-38 weeks
E. 12-16 weeks, 27-28 weeks, 37-38 weeks

78. A woman in the first half of pregnancy was brought to clinic by an ambulance.
Term of pregnancy is 36 weeks. She complains of intensive pain in the
epigastrium, had vomiting for 2 times. Pain started after the patient had eaten
vinaigrette. Swelling of lower extremities. BP - 140/100 mm Hg. Urine became
curd after boiling. What is the most probable diagnosis?
A. Preeclampsia
B. Dropsy of pregnant women
C. Exacerbation of pyelonephritis
D. Nephropathy of the 3rd degree
E. Food toxicoinfection

79. A 13 year old girl consulted the school doctor on account of moderate bloody
discharge from the genital tracts, which appeared 2 days ago. Secondary
sexual characters are developed. What is the most probable cause of bloody
discharge?
A. Menarche
B. Haemophilia
C. Juvenile hemorrhage
D. Endometrium cancer
E. Werlhofs disease

80.In 10 min after childbirth by a 22-year-old woman, the placenta was


spontaneousely delivered and 100 ml of blood came out. Woman weight - 80
kg, infant weight - 4100 g, length - 53 cm. The uterus contracted. In 10
minutes the hemorrhage renewed and the amount of blood constitued 300 ml.
What amount of blood loss is permissible for this woman?
A. 400 ml
B. 500 ml
C. 300 ml
D. 650 ml
E. 1000 ml
81. A pregnant woman was registered in a maternity welfare clinic in her 11th
week of pregnancy. She was being under observation during the whole term,
the pregnancy course was normal. What document must the doctor give the
pregnant woman to authorize her hospitalization in maternity hospital?
A. Exchange card
B. Sanitary certificate
C. Individual prenatal record
D. Appointment card for hospitalization
E. Medical certificate

82. After examination a 46-year-old patient was diagnosed with left breast cancer
T2N2M0, cl. gr. II-a. What will be the treatment plan for this patient?
A. Radiation therapy + operation + chemotherapy
B. Radiation therapy only
C. Chemotherapy only
D. Operation only
E. Operation + radiation therapy

83. Immediately after delivery a woman had haemorrhage, blood loss exceeded
postpartum haemorrhage rate and was progressing. There were no symptoms
of placenta detachment. What tactics should be chosen?
A. Manual removal of placenta and afterbirth
B. Instrumental revision of uterine cavity walls
C. Uterus tamponade
D. Removal of afterbirth by Credes method
E. Intravenous injection of methylergometrine with glucose

84. A 30 y.o. primigravida woman has got intensive labor pain every 1-2 minutes
that lasts 50 seconds. The disengagement has started. The perineum with the
height of 4 cm has grown pale. What actions are necessary in this situation?
A. Episiotomy
B. Perineum protection
C. Vacuum extraction of fetus
D. Expectant management
E. Perineotomy

85. A 30-year-old gravida consulted a gynecologist about bright red bloody


discharges from the vagina in the 32 week of gestation. She was hospitalized
with a suspicion of placental presentation. Under what conditions is it rational
to conduct the internal examination in order to make a diagnosis?
A. In the operating room prepared for the operation
B. In the examination room of antenatal clinic
C. In the delivery room keeping to all the aseptics regulations
D. The examination is not to be conducted because of risk of profuse
haemorrhage
E. In the admission ward of maternity hospital

86. A 28 y.o. primagravida, pregnancy is 15-16 weaks of gestation, presents to the


maternity clinics with dull pain in the lower part of the abdomen and in
lumbar area. On vaginal examination: uterus cervix is 2,5 cm, external
isthmus allows to pass the finger tip. Uterus body is enlarged according to the
pregnancy term. Genital discharges are mucous, mild. What is the diagnosis?
A. Threatened spontaneous abortion
B. Stopped pregnancy
C. Placenta presentation
D. Hydatid molar pregnancy
E. Spontaneous abortion which has begun

87. A primapara with pelvis size 25-28-31-20 cm has active labor activity. Waters
poured out, clear. Fetus weight is 4500 g, the head is engaged to the small
pelvis inlet. Vastens sign as positive. Cervix of uterus is fully dilated. Amniotic
sac is absent. The fetus heartbeat is clear, rhythmic, 136 bpm. What is the
labor tactics?
A. Caesarean section
B. Stimulation of the labor activity
C. Obstetrical forseps
D. Vacuum extraction of the fetus
E. Conservative tactics of labor

88. Internal obstetric examination of a parturient woman revealed that the


sacrum hollow was totally occupied with fetus head, ischiadic spines couldnt
be detected. Sagittal suture is in the straight diameter, occipital fontanel is
directed towards symphysis. In what plane of small pelvis is the presenting
part of the fetus?
A. Plane of pelvic outlet
B. Plane of pelvic inlet
C. Over the pelvic inlet
D. Wide pelvic plane
E. Narrow pelvic plane

89. A 30 y.o. woman has the 2-nd labour that has been lasting for 14 hours.
Hearbeat of fetus is muffled, arrhythmic, 100/min. Vaginal examination:
cervix of uterus is completely opened, fetus head is level with outlet from
small pelvis. Saggital suture is in the straight diameter, small crown is near
symphysis. What is the further tactics of handling the delivery?
A. Use of obstetrical forceps
B. Cesarean section
C. Stimulation of labour activity by oxytocin
D. Cranio-cutaneous (Ivanovs) forceps
E. Use of cavity forceps

90. During examination of a patient, masses in the form of condyloma on a broad


basis are found in the area of the perineum. What is the tactics of the doctor?
A. To send a woman into dermatological and venerological centre
B. Antiviral treatment
C. Surgical ablation of condyloms
D. Cryodestruction of condyloms
E. Chemical coagulator treatment

91. A 28 year old woman has bursting pain in the lower abdomen during
menstruation; chocolate-like discharges from vagina. It is known from the
anamnesis that the patient suffers from chronic adnexitis. Bimanual
examination revealed a tumour-like formation of heterogenous consistency
7х7 cm large to the left from the uterus. The formation is restrictedly movable,
painful when moved. What is the most probable diagnosis?
A. Endometrioid cyst of the left ovary
B. Follicular cyst of the left ovary
C. Exacerbation of chronic adnexitis
D. Tumour of sigmoid colon
E. Fibromatous node

92. Vaginal inspection of a parturient woman revealed: cervix dilation is up to 2


cm, fetal bladder is intact. Sacral cavity is free, sacral promontory is reachable
only with a bent finger, the inner surface of the sacrococcygeal joint is
accessible for examination. The fetus has cephalic presentation. Sagittal
suture occupies the transverse diameter of pelvic inlet, the small fontanel to
the left, on the side. What labor stage is this?
A. Cervix dilatation stage
B. Prodromal stage
C. Placental stage
D. Stage of fetus expulsion
E. Preliminary stage

93. A 68-year-old patient consulted a doctor about a tumour in her left mammary
gland. Objectively: in the upper internal quadrant of the left mammary gland
there is a neoplasm up to 2,5 cm in diameter, dense, uneven, painless on
palpation. Regional lymph nodes are not enlarged. What is the most likely
diagnosis?
A. Cancer
B. Fibroadenoma
C. Cyst
D. Mastopathy
E. Lipoma

94. A 40-year-old female patient has been observing profuse menses accompanied
by spasmodic pain in the lower abdomen for a year. Bimanual examination
performed during menstruation revealed a dense formation up to 5 cm in
diameter in the cervical canal. Uterus is enlarged up to 5-6 weeks of
pregnancy, movable, painful, of normal consistency. Appendages are not
palpable. Bloody discharges are profuse. What is the most likely diagnosis?
A. Nascent submucous fibromatous node
B. Cervical carcinoma
C. Algodismenorrhea
D. Cervical myoma
E. Abortion in progress

95. A 29-year-old patient complains of sterility. Sexual life is for 4 years being
married, does not use contraception. There was no pregnancy before. On
physical examination, genitals are developed normally. Uterine tubes are
passable. Rectal temperature during three menstrual cycles is monophase.
What is the most probable reason for sterility?
A. Anovulatory menstrual cycle
B. Genital endometriosis
C. Anomalies of genitals development
D. Chronic adnexitis
E. Immunologic sterility

96. A 45 y.o. woman complains of contact bleedings during 5 months. On


speculum examination: hyperemia of uterus cervix, looks like cauliflower,
bleeds on probing. On bimanual examination: cervix is of densed consistensy,
uterus body isnt enlarged, mobile, nonpalpable adnexa, parametrium is free,
deep fornixes. What is the most likely diagnosis?
A. Cancer of cervix of uterus
B. Fibromatous node which is being born
C. Cancer of body of uterus
D. Cervical pregnancy
E. Polypose of cervix of uterus

97. 10 minutes after delivery a woman discharged placenta with a tissue defect
5х6 cm large. Discharges from the genital tracts were profuse and bloody.
Uterus tonus was low, fundus of uterus was located below the navel.
Examination of genital tracts revealed that the uterine cervix, vaginal walls,
perineum were intact. There was uterine bleeding with following blood
coagulation. Your actions to stop the bleeding:
A. To make manual examination of uterine cavity
B. To introduce an ether-soaked tampon into the posterior fornix
C. To administer uterotonics
D. To put an ice pack on the lower abdomen
E. To apply hemostatic forceps upon the uterine cervix

98. On the 5th day after labor body temperature of a 24-year-old parturient
suddenly rose up to 38,7°C. She complains about weakness, headache,
abdominal pain, irritability. Objectively: AP- 120/70 mm Hg, Ps- 92 bpm, to-
38,7°C. Bimanual examination revealed that the uterus was enlarged up to 12
weeks of pregnancy, it was dense, slightly painful on palpation. Cervical canal
lets in 2 transverse fingers, discharges are moderate, turbid, with foul smell.
In blood: skeocytosis, lymphopenia, ESR - 30 mm/h. What is the most likely
diagnosis?
A. Endometritis
B. Pelviperitonitis
C. Lochiometra
D. Metrophlebitis
E. Parametritis

99. A 20 y.o. patient complains of amenorrhea. Objectively: hirsutism, obesity


with fat tissue prevailing on the face, neck, upper part of body. On the face
there are acne vulgaris, on the skin - striae cutis distense. Psychological and
intellectual development is normal. Gynecological condition: external genitals
are moderately hairy, acute vaginal and uterine hypoplasia. What diagnosis is
the most probable?
A. Itsenko-Cushing syndrome
B. Shichans syndrome
C. Babinski-Froehlich syndrome
D. Turners syndrome
E. Stein-Leventals syndrome

100. A 27 y.o. woman suffers from pyelonephritits of the only kidney. She
presents to the maternity welfare centre because of suppresion of menses for
2,5 months. On examination pregnancy 11 weeks of gestation was revealed. In
urine: albumine 3,3 g/L, leucocytes cover the field of vision. What is doctors
tactics in this case?
A. Immediate pregnancy interruption
B. Pregnancy interruption after urine normalization
C. Pregnancy interruption at 24-25 weeks
D. Maintenance of pregnancy till delivery term
E. Maintenance of pregnancy till 36 weeks
101. An 18-year-old primigravida in her 27-28 week of gestation underwent
an operation on account of acute phlegmonous appendicitis. In the
postoperative period it is necessary to take measures for prevention of the
following pregnancy complication:
A. Noncarrying of pregnancy
B. Late gestosis
C. Fetus hypotrophy
D. Intestinal obstruction
E. Premature placenta detachment

102. A 24-year-old female patient complains of acute pain in the lower


abdomen that turned up after a physical stress. She presents with nausea,
vomiting, dry mouth and body temperature 36,6°C. She has a right ovarian
cyst in history. Bimanual examination reveals that uterus is dense, painless, of
normal size. The left fornix is deep, uterine appendages arent palpable, the
right fornix is contracted. There is a painful formation on the right of uterus.
Its round, elastic and mobile. It is 7х8 cm large. In blood: leukocytosis with
the left shit. What is the most likely diagnosis?
A. Ovarian cyst with pedicle torsion
B. Subserous fibromyoma of uterus
C. Extrauterine pregnancy
D. Acute metritis
E. Right-sided pyosalpinx

103. A parturient woman is 23 years old. Vaginal obstetric examination


reveals full cervical dilatation. There is no fetal bladder. Fetal head is in the
plane of pelvic outlet. Sagittal suture is in mesatipellic pelvis, anterior fontanel
is closer to pubes. The fetal head diameter in such presentation will be:
A. Suboccipito-bregmaticus
B. Fronto-occipitalis recta
C. Suboccipitio-frontalis
D. Mento-occipitalis
E. Biparietal

104. A 42-year-old woman has had hyperpolymenorrhea and progressing


algodismenorrhea for the last 10 years. Gynaecological examination revealed
no changes of uterine cervix; discharges are moderate, of chocolate colour,
uterus is slightly enlarged and painful, appendages are not palpable, the
fornices are deep and painless. What is the most likely diagnosis?
A. Uterine endometriosis
B. Uterine carcinoma
C. Endomyometritis
D. Adnexal endmetriosis
E. Subserous uterine fibromyoma

105. On the tenth day after discharge from the maternity house a 2-year-old
patient consulted a doctor about body temperature rise up to 39°C, pain in the
right breast. Objectively: the mammary gland is enlarged, there is a
hyperemized area in the upper external quadrant, in the same place there is an
ill-defined induration, lactostasis, fluctuation is absent. Lymph nodes of the
right axillary region are enlarged and painful. What is the most likely
diagnosis?
A. Lactational mastitis
B. Tumour
C. Erysipelas
D. Abscess
E. Dermatitis

106. During the dynamic observation over a parturient woman in the second
stage of labor it was registered that the fetal heart rate fell down to 90-
100/min and didn't come to normal after contractions. Vaginal examination
revealed the complete cervical dilatation, the fetal head filling the entire
posterior surface of the pubic symphysis and sacral hollow; the sagittal suture
lied in the anteroposterior diameter of the pelvic outlet, the posterior
fontanelle was in front under the pubic arch. What plan for further labour
management should be recommended?
A. Application of forceps minor
B. Stimulation of labour activity by intravenous injection of oxytocin
C. Episiotomy
D. Caesarean section
E. Application of cavity forceps

107. A 27-year-old sexually active female complains of numerous vesicles on


the right sex lip, itch and burning. Eruptions regularly turn up before
menstruation and disappear 8-10 days later. What is the most likely
diagnosis?
A. Herpes simplex virus
B. Primary syphilis
C. Genital condylomata
D. Cytomegalovirus infection
E. Bartholinitis

108. A 26-year-old woman complains of having bloody discharges from the


genitals for the last 14 days, abdominal pain, general fatiguability, weakness,
weight loss, body temperature rise, chest pain, obstructed respiration. 5 weeks
ago she underwent induced abortion in the 6-7 week of gestation. Objectively:
the patient is pale and inert. Bimanual examination revealed that the uterus
was enlarges up to 8-9 weeks of gestation. In blood: Hb- 72 g/l. Urine test for
chorionic gonadotropin gave the positive result. What is the most likely
diagnosis?
A. Chorioepithelioma
B. Metroendometritis
C. Uterine fibromyoma
D. Uterine carcinoma
E. Uterus perforation

109. A 28-year-old patient complains of discomfort, acute pain in the lower


third of the left labia majora. The disease began suddenly after menstruation.
Objectively: body temperature is 38°C. The left labia majora has a formation
to 3 cm diameter, with hyperemic surface, extremely painful to the touch, with
symptoms of fluctuation. What is the most likely diagnosis?
A. Acute bartholinitis
B. Hypertrophy of the labia
C. Vulvar fibroid
D. Vulvar cancer
E. Bartholin gland cyst

110. A 28-years-old woman complains of nausea and vomiting about 10


times per day. She has been found to have body weight loss and xerodermia.
The pulse is 100 bpm. Body temperature is 37,2°C. Diuresis is low. USI shows
5-6 weeks of pregnancy. What is the most likely diagnosis?
A. Moderate vomiting of pregnancy
B. I degree preeclampsia
C. Food poisoning
D. Premature abortion
E. Mild vomiting of pregnancy

111. A 40 week pregnant secundipara is 28 years old. Contractions are very


active. Retraction ring is at the level of navel, the uterus is hypertonic, in form
of hourglass. On auscultation the fetal heart sounds are dull, heart rate is
100/min. AP of the parturient woman is 130/80 mm Hg. What is the most
likely diagnosis?
A. RIisk of hysterorrhexis
B. Attack of eclampsia
C. Disturbed labour
D. Mazolysis
E. Complete hysterorrhexis
112. After delivery and revision of placenta there was found the defect of
placental lobule. General condition of woman is normal, uterus is firm, there
is moderate bloody discharge. Speculum inspection of birth canal shows
absence of lacerations and raptures. What action is necessary?
A. Manual exploration of the uterine cavity
B. Urine drainage, cold on the lower abdomen
C. Introduction of hemostatic medications
D. External massage of uterus
E. Introduction of uterine contracting agents

113. A 25 y.o. patient complains of body temperature rise up to 37°C, pain at


the bottom of her abdomen and vaginal discharges. Three days ago, when she
was in her 11th week of pregnancy, she had an artificial abortion. Objectibely:
cervix of uterus is clean, uterus is a little bit enlarged in size, painful.
Appendages cannot be determined. Fornixes are deep, painless. Vaginal
discharges are sanguinopurulent. What is the most probable diagnosis?
A. Postabortion endometritis
B. Hematometra
C. Postabortion uterus perforation
D. Parametritis
E. Pelvic peritonitis

114. A 25 y.o. pregnant woman in her 34th week was taken to the maternity
house in grave condition. She complains of headache, visual impairment,
nausea. Objectively: solid edemata, AP- 170/130 mm Hg. Suddenly there
appeared fibrillary tremor of face muscles, tonic and clonic convulsions,
breathing came to a stop. After 1,5 minute the breathing recovered, there
appeared some bloody spume from her mouth. In urine: protein - 3,5 g/L.
What is the most probable diagnosis?
A. Eclampsia
B. Epilepsy
C. Cerebral edema
D. Stomach ulcer
E. Cerebral hemorrhage

115. A 51-year-old patient complains of having intensive bloody discharges


from vagina for 15 days after delay of menstruation for 2,5 months. In
anamnesis: disorders of menstrual function during a year, at the same time
she felt extreme irritability and had sleep disorders. US examination results:
uterus corresponds with age norms, appendages have no pecularities,
endometrium is 14 mm thick. What is the doctors tactics?
A. Diagnostic curettage of uterine cavity
B. Hysterectomy
C. TORCH-infection test
D. Supravaginal amputation of uterus without appendages
E. Conservative treatment of bleeding

116. An 18 y.o. patient complains of painfulness and swelling of mammary


glands, headaches, irritability, edemata of lower extremities. These symptoms
have been present since the begin of menarche, appear 3-4 days before regular
menstruation. Gynecological examination revealed no pathology. What is the
most probable diagnosis?
A. Premenstrual syndrome
B. Renal disease
C. Neurasthenia
D. Mastopathy
E. Disease of cardiovascular system

117. A 22-year-old female patient complains of dull pain in her right iliac
area that she has been experiencing for a week, morning sickness and
gustatory change. She has a histrory of menstruation delay for 3 weeks.
Objectively: AP- 80/50 mm Hg, pulse is 78 bpm, body temperature is 37°C.
Bimanual examination reveals that uterus is enlarged, soft, mobile and
painless. Uterine appendages are palpable on the right, there is a dense,
elastic and moderately painful formation 3x4 cm large. What is the most likely
diagnosis?
A. Progressing fallopian pregnancy
B. Right ovarian cyst
C. Interrupted fallopian pregnancy
D. Uterogestation
E. Acute appendicitis

118. A 30 y.o. parturient woman was taken to the maternity house with
complaints of having acute, regular labour pains that last 25-30 seconds every
1,5-2 minutes. Labour activity began 6 hours ago. Uterus is in higher tonus,
head of the fetus is above the opening into the small pelvis. Fetal heartbeat is
136/min. P.V: cervical dilatation is 4 cm, uterine fauces is spasming at a
height of parodynia. Head is level with opening into the small pelvis, it is
being pushed off. What is the most probable diagnosis?
A. Discoordinated labour activity
B. Pathological preliminary period
C. Normal labour activity
D. Primary powerless labour activity
E. Secondary powerless labour activity

119. A primigravida woman appealed to the antenatal clinic on the 22.03.03


with complaints of boring pain in the lower part of abdomen. Anamnesis
registered that her last menstruation was on the 4.01.03. Bimanual
examination revealed that uterine servix is intact, external fauces is closed,
uterus is enlarged up to the 9-th week of pregnancy, movable, painless. What
complication can be suspected?
A. Risk of abortion in the 9-th week of pregnancy
B. Vesicular mole
C. Abortion that started in the 9-th week of pregnancy
D. Hysteromyoma

120. A 25-year-old female patient complains about having amenorrhea for 3


years. She associates it with difficult labour complicated by massive
hemorrhage. She also complains of loss of weight, hair fragility and loss, lack
of appetite and depression. Objective examination reveals no pathological
changes of uterus and its appendages. What is the desease pathogenesis?
A. Hypoproduction of gonadotropin
B. Hypoproduction of progesterone
C. Hyperproduction of prolactin
D. Hyperproduction of estrogens
E. Hyperproduction of androgens

121. A 30-year-old patient consulted a doctor about menstruation absence


for 2 years after labour, loss of hair, body weight loss. The labour was
complicated by a haemorrhage caused by uterus hypotonia. Objectively: the
patient is asthenic, external genitals are hypoplastic, the uterus body is small
and painless. The appendages are not palpaple. What is the most likely
diagnosis?
A. Sheehans syndrome
B. Turners syndrome
C. Ovarian amenorrhea
D. Exhausted overy syndrome
E. Galactorrhea-amenorrhea syndrome

122. A 28-year-old patient has been admitted to the gynecological


department three days after a casual coitus. She complains about pain in her
lower abdomen and during urination, profuse purulent discharges from the
vagina, body temperature rise up to 37,8°C. The patient was diagnosed with
acute bilateral adnexitis. Supplemental examination revealed: the 4th degree
of purity of the vaginal secretion, leukocytes within the whole visual field,
diplococcal bacteria located both intra- and extracellularly. What is the
etiology of acute adnexitis in this patient?
A. Gonorrheal
B. Trichomonadal
C. Staphylococcal
D. Colibacterial
E. Chlamydial

123. A 25-year-old woman came to a maternity welfare clinic and


complained about being unable to conceive within 3 years of regular sexual
life. Examination revealed weight gain, male pattern of hair distribution on
the pubis, excessive pilosis of thighs. Ovaries were dense and enlarged, basal
temperature was monophase. What is the most likely diagnosis?
A. Sclerocystosis of ovaries
B. Gonadal dysgenesis
C. Adrenogenital syndrome
D. Tubo-ovaritis
E. Premenstrual syndrome

124. A 28 year old patient complained about prolongation of intermenstrual


periods up to 2 months, hirsutism. Gynaecological examination revealed that
the ovaries were enlarged, painless, compact, uterus had no pecularities.
Pelvic ultrasound revealed that the ovaries were 4-5 cm in diameter and had
multiple enlarged follicles on periphery. Roentgenography of skull base
showed that sellar region was dilated. What is the most probable diagnosis?
A. Stein-Leventhal syndrome
B. Premenstrual syndrome
C. Morgagni-Stewart syndrome
D. Algodismenorrhea
E. Sheehans syndrome

125. A 32-year-old gravida complains of episodes of unconsciousness,


spontaneous syncopes that are quickly over after a change of body position. A
syncope can be accompanied by quickly elapsing bradycardia. There are no
other complications of gestation. What is the most likely reason for such
condition?
A. Postcava compresseion by the gravid uterus
B. Psychosomatic disorders
C. Pressure fall in the veins of extremities
D. Pressure rise in the veins of extremities
E. Vegetative-vascular dystonia (cardiac type)

126. A woman consulted a therapeutist about fatigability, significant weight


loss, weakness, loss of appetite. She has had amenorrhea for 8 months. A year
ago she born a full-term child. Haemorrhage during labour made up 2 L. She
got blood and blood substitute transfusions. What is the most probable
diagnosis?
A. Sheehans syndrome
B. Shereshevsky-Turners syndrome
C. Stein-Leventhal syndrome
D. Homological blood syndrome
E. Vegetovascular dystonia

127. A 54-year-old female patient consulted a gynaecologist about bloody


discharges from the vagina for 1 month. Last menstruation was 5 years ago.
Gynaecological examination revealed no pathological changes. What is the
tactics of choice?
A. Diagnostic fractional curettage of uterine cavity
B. Colposcopy
C. Cytosmear
D. Symptomatic therapy
E. USI

128. A 28-year-old female patient complains of having haemorrhage from


the genital tracts for 1 month. 6 months ago she had natural delivery and gave
birth to a girl weighing 3100 g. Objectively: the uterus is enlarged to 9-10
weeks, mobile, painless, of heterogenous consistency. Examination reveals
vaginal cyanosis, anaemia and body temperature rise up to 37,8°C. There is a
significant increase in hCG concentration in the urine. What is your
provisional diagnosis?
A. Uterine chorionepithelioma
B. Uterine fibromyoma
C. Hydatidiform mole
D. Pregnancy
E. Endometritis

129. An ambulance delivered a 21-year-old woman to the gynaecological


department with complaints of colicky abdominal pain and bloody discharges
from the genital tracts. Bimanual examination revealed that uterus was soft,
enlarged to the size of 6 weeks of gestation, a gestational sac was palpated in
the cervical canal. Uterine appendages werent palpable. Fornices are free,
deep and painless. Discharges from the genital tracts are bloody and profuse.
What is the most likely diagnosis?
A. Abortion in progress
B. Threat of abortion
C. Interrupted fallopian pregnancy
D. Incipient abortion
E. Cervical pregnancy

130. A 33 y.o. woman survived two operations on account of extrauterine


pregnancy, both uterine tubes were removed. She consulted a doctor with a
question about possibility of having a child. What can be advised in this case?
A. Extracorporal fertilization
B. Insemination with her husband’s semen
C. Artificial fertilization with donor’s semen
D. Induction of ovulation
E. Substitutional maternity

131. On the fifth day after a casual sexual contact a 25-year-old female
patient consulted a doctor about purulent discharges from the genital tracts
and itch. Vaginal examination showed that vaginal part of uterine cervix was
hyperemic and edematic. There was an erosive area around the external
orifice of uterus. There were mucopurulent profuse discharges from the
cervical canal, uterine body and appendages exhibited no changes.
Bacterioscopic examination revealed bean-shaped diplococci that became red
after Grams staining. What is the most likely diagnosis?
A. Acute gonorrheal endocervicitis
B. Candidal vulvovaginitis
C. Bacterial vaginism
D. Clamydial endocervicitis
E. Trichomonal colpitis

132. A 26 year old woman complains about edemata, swelling and


painfulness of mammary glands, headache, tearfulness, irritability. These
signs turn up 5 days before menstruation and disappear after its start. What
clinical syndrome is it?
A. Premenstrual syndrome
B. Adrenogenital syndrome
C. Postcastration syndrome
D. Climacteric syndrome
E. Stein-Leventhal syndrome

133. A 49-year-old woman complains about headache, head and neck going
hot, increased perspiration, palpitation, arterial pressure rise up to 170/100
mm Hg, irritability, insomnia, tearfulness, memory impairment, rare and
scarce menses, body weight increase by 5 kg over the last half a year. What is
the most likely diagnosis?
A. Climacteric syndrome
B. Arterial hypertension
C. Postcastration syndrome
D. Premenstrual syndrome
E. Vegetative-vascular dystonia

134. A 30-year-old female patient has been delivered to the gynaecological


department with complaints of acute pain in the lower abdomen and body
temperature 38,8°C. In history: sexual life out of wedlock and two artificial
abortions. Gynaecological examination reveals no changes of uterine. The
appendages are enlarged and painful on both sides. Vaginal discharges are
purulent and profuse. What study is required to confirm a diagnosis?
A. Bacteriological and bacterioscopic analysis
B. Curettage of uterine cavity
C. Laparoscopy
D. Colposcopy
E. Hysteroscopy

135. A parturient woman is 27 year old, it was her second labour, delivery
was at term, normal course. On the 3rd day of postpartum period body
temperature is 36,8°C, Ps - 72/min, AP - 120/80 mm Hg. Mammary glands
are moderately swollen, nipples are clean. Abdomen is soft and painless.
Fundus of uterus is 3 fingers below the umbilicus. Lochia are bloody,
moderate. What is the most probable diagnosis?
A. Physiological course of postpartum period
B. Lactostasis
C. Postpartum metroendometritis
D. Subinvolution of uterus
E. Remnants of placental tissue after labour

136. A parturient woman is 25 years old, it is her second day of postpartum


period. It was her first full-term uncomplicated labour. The lochia should be:
A. Bloody
B. Mucous
C. Sanguino-serous
D. Purulent
E. Serous

137. A 32-year-old patient consulted a doctor about being inable to get


pregnant for 5-6 years. 5 ago the primipregnancy ended in artificial abortion.
After the vaginal examination and USI the patient was diagnosed with
endometrioid cyst of the right ovary. What is the optimal treatment method?
A. Surgical laparoscopy
B. Conservative therapy with estrogen-gestagenic drugs
C. Anti-inflammatory therapy
D. Hormonal therapy with androgenic hormones
E. Sanatorium-and-spa treatment

138. A woman is 34 years old, it is her tenth labor at full term. It is known
from the anamnesis that the labor started 11 hours ago, labor was active,
painful contractions started after discharge of waters and became continuous.
Suddenly the parturient got knife-like pain in the lower abdomen and labor
activity stopped. Examination revealed positive symptoms of peritoneum
irritation, ill-defined uterus outlines. Fetus was easily palpable, movable. Fetal
heartbeats wasnt auscultable. What is the most probable diagnosis?
A. Rupture of uterus
B. Risk of uterus rupture
C. II labor period
D. Uterine inertia
E. Discoordinated labor activity

139. Examination of placenta revealed a defect. An obstetrician performed


manual investigation of uterine cavity, uterine massage. Prophylaxis of
endometritis in the postpartum period should involve following actions:
A. Antibacterial therapy
B. Haemostatic therapy
C. Intrauterine instillation of dioxine
D. Contracting agents
E. Instrumental revision of uterine cavity

140. A 10-week pregnant woman was admitted to a hospital for recurrent


pain in the lower abdomen, bloody discharges from the genital tracts. The
problems turned up after ARVI. The woman was registered for antenatal care.
Speculum examination revealed cyanosis of vaginal mucosa, clean cervix,
open cervical canal discharging blood and blood clots; the lower pole of the
gestational sac was visible. What tactics should be chosen?
A. Curettage of the uterus
B. Pregnancy maintenance therapy
C. Hysterectomy
D. Antiviral therapy
E. Expectant management, surveillance

141. A 30 year old patient complains about inability to become pregnant


over 3 years of married life. The patient is of supernutrition type, she has hair
along the median abdominal line, on the internal thigh surface and in the
peripapillary area. Menses started at the age of 16, they are infrequent and
non-profuse. US revealed that the uterus was of normal size, ovaries were
4х5х5 cm large and had a lot of cystic inclusions. What is the most probable
diagnosis?
A. Polycystic ovaries
B. Menstrual irregularity
C. Bilateral ovarian tumours
D. Ovarian cystoma
E. Chronic oophoritis
142. A 29-year-old patient complains of absent menstruation for a year,
milk discharge from the nipples when pressed, loss of lateral visual fields. X-
ray shows an expansion of the sella turcica. What is the most likely cause of
this condition?
A. Pituitary tumour
B. Mammary tumour
C. Ovarian tumor
D. Pregnancy
E. Functional disorder of the hypothalamic-pituitary-ovarian system

143. A patient with fibromyoma of uterus sized up to 8-9 weeks of


pregnancy consulted a gynaecologist about acute pain in the lower abdomen.
Examination revealed pronounced positive symptoms of peritoneal irritation,
high leukocytosis. Vaginal examination revealed that the uterus was enlarged
corresponding to 9 weeks of pregnancy due to the fibromatous nodes, one of
which was mobile and extremely painful. Appendages were not palpable.
There were moderate mucous discharges. What is the optimal treatment
tactics?
A. Urgent surgery (laparotomy)
B. Surveillance and spasmolytic therapy
C. Surgical laparoscopy
D. Surveillance and antibacterial therapy
E. Fractional diagnostic curettage of the uterine cavity

144. A pregnant woman was delivered to the gynecological unit with


complaints of pain in the lower abdomen and insignificant bloody discharges
from the genital tracts for 3 hours. Last menstruation was 3 months ago.
Vaginal examination showed that body of womb was in the 10th week of
gestation, a fingertip could be inserted into the external orifice of uterus,
bloody discharges were insignificant. USI showed small vesicles in the uterine
cavity. What is the most likely diagnosis?
A. Grape mole
B. Incipient abortion
C. Abortion in progress
D. Threat of spontaneous abortion
E. Incomplete abortion

145. A 49-year-old patient undergoes regular medical check-up for uterine


fibromyoma. Within the last year the uterus has enlarged up to 20 weeks of
gestation. What is the rational way of treatment?
A. Surgical treatment
B. Hormonal therapy
C. Embolization of uterine arteries
D. Treatment with prostaglandin inhibitors
E. Further surveillance

146. A primigravida is 22 years old. She has Rh(-), her husband has Rh(+).
Antibodies to Rh weren’t found at 32 weeks of pregnancy. Redetermination of
antibodies to Rh didnt reveal them at 35 weeks of pregnancy as well. How
often should the antibodies be determined hereafter?
A. Once a week
B. Once in three weeks
C. Once in two weeks
D. Monthly
E. There is no need in further checks

147. A maternity house has admitted a primagravida complaining of


irregular, intense labour pains that have been lasting for 36 hours. The woman
is tired, failed to fall asleep at night. The fetus is in longitudinal lie, with
cephalic presentation. The fetus heartbeat is clear and rhythmic, 145/min.
Vaginal examination revealed that the uterine cervix was up to 3 cm long,
dense, with retroflexion; the external orifice was closed; the discharges were of
mucous nature. What is the most likely diagnosis?
A. Pathological preliminary period
B. Secondary uterine inertia
C. Primary uterine inertia
D. Uterine cervix dystocia
E. Physiological preliminary period

148. A 14-year-old girl complains of pain in vaginal area and lower abdomen
that last for 3-4 days and have been observed for 3 months about the same
time. Each time pain is getting worse. Objectively: mammary glands are
developed, hairiness corresponds to the age. The virginal membrane is intact,
cyanotic and protruded. She has never had menstruation. She has been
diagnosed with primary amenorrhea. What is the reason of amenorrhea?
A. Hymen atresia
B. Sexual development delay
C. Babinski-Frohlich syndrome
D. Turners syndrome
E. Pregnancy

149. A multigravida with Rh-isosensitization was found to have a decrease


in anti-Rh titer from 1:32 to 1:8 at 33-34 weeks of gestation. Ultrasound
revealed double contour of head, ebnlargement of fetal liver, placental
thickness of 50 mm. The patient has indication for:
A. Premature delivery
B. Repeated (after 2 weeks) USI
C. Administration of anti-Rh gamma globulin
D. Course of desensitizing therapy
E. Plasmapheresis

150. A 13-year-old girl was admitted to the gynecological department with


heavy bleeding, which appeared after a long delay of menstruation. Shortly
before, the girl suffered a serious psychotrauma. Her menarche occurred at
the age of 11, she has a 30-day cycle with 5 to 6 days of moderate, painless
bleeding. The patient is somatically healthy, of normosthenic constitution
with height of 160 cm, weight of 42 kg. The patient is pale. Rectoabdominal
examination revealed that the uterus was of normal size and consistency,
anteflexio-versio, the appendages were not changed. What is the most likely
diagnosis?
A. Juvenile bleeding
B. Amenorrhea
C. Hysteromyoma
D. Ovarian cyst
E. Girl is healthy

151. A 38-year-old female patient complains about hot flashes and feeling of
intense heat arising up to 5 times a day, headaches in the occipital region
along with high blood pressure, palpitations, dizziness, fatigue, irritability,
memory impairment. 6 months ago the patient underwent extirpation of the
uterus with its appendages. What is the most likely diagnosis?
A. Post-castration syndrome
B. Secondary psychogenic amenorrhea
C. Physiological premenopause
D. Premenstrual syndrome
E. Early pathological menopause

152. A 27-year-old patient complains of irritability, tearfulness, depression,


and sometimes aggressiveness, headache, nausea, vomiting, swelling of the
mammary glands. The mentioned problems arise 5-6 days before
menstruation and gradually progress until menstruation, 3 days after it the
problems disappear. What is the most likely diagnosis?
A. Premenstrual syndrome
B. Secondary psychogenic amenorrhea
C. Premature pathological climacterium
D. Preclimacterium syndrome
E. Algomenorrhea
153. 20 minutes after a normal delivery at 39 weeks a puerpera had a single
temperature rise up to 38°C. Objectively: the uterus is dense, located between
the navel and the pubis, painless. Lochia are bloody, of small amount. Breasts
are moderately soft and painless. What is the optimal tactics?
A. Further follow-up
B. Manual examination of the uterine cavity
C. Expression of breast
D. Antibiotic therapy
E. Appointment antipyretic

154. On the 10th day postpartum a puerperant woman complains of pain


and heaviness in the left mammary gland. Body temperature is 38,8°C, Ps- 94
bpm. The left mammary gland is edematic, the supero-external quadrant of
skin is hyperemic. Fluctuation symptom is absent. The nipples discharge
drops of milk when pressed. What is a doctors further tactics?
A. Antibiotic therapy, immobilization and expression of breast milk
B. Inhibition of lactation
C. Opening of the abscess and drainage of the mammary gland
D. Physiotherapy
E. Compress to both mammary glands

155. A 30-year-old female patient complains of milk discharge from the


mammary glands, 5-month absence of menstruation. She had one
physiological labour four years ago. Objectively: mammary glands are
normally developed. Bimanual examination reveals that the uterus is
decreased in size, the ovaries are of normal size. MRI-scan shows no cerebral
pathologies. Concentration of thyroid-stimulating hormone is normal. The
serum prolactin level is increased. What is the most likely diagnosis?
A. Hyperprolactinemia
B. Polycystic ovary syndrome
C. Sheehan syndrome
D. Pituitary adenoma
E. Hypothyroidism

156. During self-examination a 22-year-old patient revealed a mammary


tumour. Palpation revealed a firm, painless, mobile formation up to 2 cm,
peripheral lymph nodes were not changed. USI results: in the superior
external quadrant of the right mammary gland there was a big formation of
increased echogenicity, sized 18x17 mm. The patient was provisionally
diagnosed with fibroadenoma. What is a doctor’s further tactics?
A. Surgical removal of the tumour prior to pregnancy
B. Radical mastectomy
C. Nonsteroid anti-inflammatory drugs, oral contraceptives
D. Dynamic follow-up
E. Surgical treatment after pregnancy

157. 2 weeks after labour a parturient woman developed breast pain being
observed for 3 days. Examination revealed body temperature at the rate of
39°C, chills, weakness, hyperaemia, enlargement, pain and deformity of the
mammary gland. On palpation the infiltrate was found to have an area of
softening and fluctuation. What is the most likely diagnosis?
A. Infiltrative-purulent mastitis
B. Lactostasis
C. Mastopathy
D. Serous mastitis
E. Phlegmonous mastitis

158. Preventive examination of a 50-year-old woman revealed a dense


tumour of the right mammary gland up to 5 cm in diameter without distinct
outlines. The skin over the tumour looked like lemon peel. Palpation revealed
a lymph node in the axillary region. What is the most likely diagnosis?
A. Breast cancer
B. Breast lipoma
C. Diffuse mastopathy
D. Lactocele
E. Mastitis

Krok 2 – 2013 Hygiene Base


1. A number of viable fetuses per 1000 women at the age between 15 and 44 is
determined by:
A. Genital index
B. Birth rate
C. Obstetric rate
D. Perinatal rate
E. Reproductive level

2. Point out the unit for statistical observation for the determination of blood
sugar level influence on a wound surface healing during postoperative period
A. The patient in a postoperative period
B. The patient who has a wound surface
C. The patient who was discharged to outpatient treatment
D. Blood sugar level
E. Blood test
3. What information gathering method is preferable to study housing conditions
of medical students during training period?
A. Questionaire
B. Statistical
C. Materials selection
D. Interviewing
E. Directed selection method

4. What methods of the collecting of the information is preferable for study of


housing conditions of students of medical HIGH SCHOOL for a training
period?
A. Questioning
B. A method of the directed selection
C. Statistical
D. Interviewing
E. Selecting of materials

5. Choose a method of a graphic representation of monthly information about


number of the registered cases of acute intestinal infection and their
comparisons to the average monthly values, obtained for 5 previous years:
A. The linear diagram
B. The curvilinear diagram
C. The sector diagram
D. The radial diagram
E. The figured diagram

6. The parameter of infantile mortality for the last year was - 16,3, in present
year - 15,7. Name a kind of the diagram that can be used for a graphic
representation of it:
A. Stylar
B. Radial
C. Intrastylar
D. Linear
E. Sector

7. The average body lenth of newborn boys is 50,9 cm at a sigma 1,66; and
average mass - 3432 at a sigma 5,00. What criterion is necessary in order to
compare degree of variability of these signs?
A. Coefficient of variation
B. Limit
C. Sigma
D. Amplitude
E. Coefficient of association
8. What method is applied to establish rate of correlation between age of men
and their mortality due to myocardial infarction?
A. Method of grade correlation (Spirman)
B. The Indirect method (Student)
C. Method of graduated correlation (Armler)
D. Correlation ratio
E. The quadrate method (Pirson)

9. Indicate the registration medical document for the patient, who 21.02. was
addressed to the doctor with diagnosis ARVD for the first time in this year:
A. The statistical coupon is to be filled in and it is necessary to deliver
on a sign (+)
B. The statistical coupon is to be filled in, but a sign (+) is not necessary to be put
in
C. The necessary registration form is not indicated
D. It is necessary to fill in the emergency notice on a case of a contagion
E. The statistical coupon for registration of final diagnosis is not necessary

10. Define the basic registration document at the profound study of a case rate
with temporary lost labor ability at the industrial enterprise:
A. A card of the personal account of a case rate
B. The sick-leave certificate
C. The Report on reasons of a temporary lost labor ability
D. A ambulatory medical card
E. The inpatient medical record

11. Head of a department and a trade-union group have appealed to the head of a
hospital about dismissal of the senior nurse who has 17 year record of service.
The facts of charge were confirmed and recognized by the nurse herself. This
nurse lives with a daughter (who is divorced and unemployed) and a 9-month-
old grandson. Make an administrative decision:
A. To continue the worker in office with a warning of dismissal in case
of repeated violation of labor discipline
B. To discharge the worker, i.e. to satisfy demands of the collective
C. To shift the solution of this problem on other officials or public organizations
D. To issue the sick list

12. 25 unorganized children in the age 2-3 year will be observed on a pediatric
district it in the current year. What scheduled number of initial visitations will
make to this group of children?
A. 50
B. 100
C. 200
D. 20
E. 40

13. A child is 6 years old. Within one year of observation he had URI that lasted 8
days. Physical state is satisfactory. Specify hi health group:
A. I
B. III (b)
C. III (c)
D. II
E. III (a)

14. A 38 year old man was admitted to a hospital from his working place on July
19 because of hip fracture. He was invalid till November 19. Requires
prolongation of treatment. Who decides on the issue of further temporary
invalidity?
A. Specialized (traumatologic) MSEC
B. DCC
C. Interregional general MSEC
D. Regional MSEC
E. The head physician of a polyclinic

15. What is the maximum duration of medical certificate in case of tuberculosis?


A. 2 months
B. 2 weeks
C. 10 months
D. Month
E. Week

16. An engineer-chemist at the age of 47 often fells ill with an occupational skin
disease. Who makes a decision to transfer him to other job accepts?
A. DCC
B. A head physician
C. The chief of shop
D. MSEC
E. The attending physician

17. A patient with high temperature came to a first-aid post in the evening. The
fact of temporary disability was established. Indicate the order of examination
in this case:
A. The night duty doctor should issue a medical certificate, which will
be subsequently used for issuing a sick list from the date of the
previous day
B. The sick list for up to 3 days should be issued
C. The sick list for 1 day should be issued
D. The sick list for 3 days should be issued
E. Any document shouldnt be issued

18. A sample of milk was taken for testing from a 5 ton milk batch. Lab analysis
showed the following: fat content 2%, specific density- 1,04 g/cm3, acidity
210C, reductase probe – weak positive. What way the product is to be used in?
What would you advise?
A. Sell but inform customers about milk quality
B. Annihilate the product
C. Utilize technically
D. Write the product off for animal feeding
E. Sell without limitations

19. A 5 tons milk batch was sampled. The lab analysis revealed: fat content 2%,
specific density - 1,04 g/cm3, acidity - 210?, reductase probe - weak-positive.
What way is the product to be used in?
A. Sell but inform customers about milk quality
B. Technical utilization
C. Do the product away
D. Sell without limitations
E. Discard for animal feeding

20. The student has the following devices: Geiger counter, Ebert counter, Krotovs
apparatus, Mischuk device, Ebert device. What device can he use to assess air
germ pollution?
A. Krotovs apparatus
B. Geigers counter
C. Eberts counter
D. Mischuks device
E. Eberts device

21. Thyreotoxicosis patient is in the two-place hospital ward of therapeutic


department. The area of the ward is 18 m2, height is 3 m, ventilation rate is
2,5/h. Air temperature is 20°C, relative humidity is 45%, air movement
velocity is 0,3 m/sec, light coefficient is 1/5, noise level constitutes 30 dB.
Make a hygienic assessment of these conditions
A. Discomfortable microclimate
B. All conditions are OK
C. Poor lighting
D. Non-effective ventilation
E. High level of noise

22. A 9 y.o. girl has an average height and harmonic growth development. She was
ill with acute respiratory infection for five times. Define the group of her
health
A. 2nd group
B. 3rd group
C. 5th group
D. 4th group
E. 1st group

23. A student lives in the modern house in the flat with a complete set of sanitary
equipment (WC, bath, shower, local water heater). How much water
consumption has he got?
A. 160-200 L/day
B. 10-15 L/day
C. 300-400 L/day
D. 500 -600 L/day
E. 50-100 L/day

24. A 30-year-old patient with complaints of occipital headache, disturbed sleep


with nightmares came to a policlinic. BP was 150/95 mm Hg. He was
diagnosed with hypertensic crisis. The patient should be registered in the
following dispensary group for arterial hypertension surveillance:
A. In the second
B. In the first
C. In the third
D. In the fifth
E. In the fourth

25. A young patient who came to a policlinic was diagnosed with the 1 stage of
hypertension. How often should he undergo the medical check-up?
A. Twice a year
B. 5 times a year
C. 3 times a year
D. Once a year
E. 4 times a year
26. 25 children at the age of 2-3 years who dont attend any child welfare
institutions should be observed by a district pediatrician within the current
year. How many initial visits of this group of children should be planned?
A. 50
B. 100
C. 200
D. 20
E. 40

27. Studying of pulmonary tuberculosis incidence provided data about patients


socioeconomic living conditions and bad habits. What method allows to
estimate the impact of these factors on tuberculosis incidence?
A. Calculation of correlation coefficient
B. Standardized index calculation
C. Calculation of reliability coefficient
D. Calculation of correspondence index
E. Calculation of regression coefficient

28. A teacher of a secondary school was diagnosed with pulmonary tuberculosis.


What is the maximum duration of his medical certificate?
A. Ten months
B. Four months
C. A month
D. Two months
E. Five months

29. An employee of a private company was ill with acute respiratory viral
infection. Consulted a district doctor, who determined the fact of temporary
loss of working ability, but refused to issue a sick-list, arguing that the patient
worked in the private and not state-owned company. Should the sick-list be
issued to the employees of private companies?
A. Issued regardless of companys ownership
B. Issued only on condition of payment guarantee by the companys proprietor
C. Issued only to empties of state-owned companies
D. Issued a medical certificate of a set form
E. Issued a medical certificate of a free form

30. Clinic of a research instutute for occupational diseases examined a worker


who works at a concentration plant and diagnosed him with chronic dust
bronchitis. The case is investigated by a commission including the
representatives of: the plant, clinic, territorial SES, department of Social
Insurance Fund, trade union. According to the "regulation on investigation
of…", the commission should be headed by the representative of the following
authority:
A. Territorial SES
B. Clinic
C. Social Insurance Fund
D. Plant
E. Trade union

31. Basing upon the data of laboratory assessment of sanitary state of soil in a
certain territory, the soil was found to be low-contaminated according to the
sanitary indicative value; contaminated according to the coli titer; low-
contaminated according to the anaerobe titer (Cl. Perfringens). This is
indicative of:
A. Fresh fecal contamination
B. Old fecal contamination
C. Insufficient insolation and aeration of soil
D. Constant entry of organic protein contaminations
E. Insufficient intensity of soil humification

32. A maternity hospital registered 616 live births, 1 stillbirth, 1 death on the 5th
day of life over a 1 year period. What index allows the most precise estimation
of this situation?
A. Perinatal mortality
B. Natality
C. Crude mortality rate
D. Neonatal mortality
E. Natural increase

33. A patient complained about problems with pain and tactile sensitivity, pain in
the nail bones at the end of the working day. He works at a plant with
mechanical devices. What pathology can be suspected?
A. Vibration disease
B. Hypovitaminosis of D1
C. Noise disease
D. Caisson disease
E. Overwork symptoms

34. Bacterial analysis of air in a living space in winter period by means of Krotovs
apparatus revealed that total number of microorganisms in 1m3 of air was
7200. What is the permissible number of microorganisms for the air to be
characterized as "pure"?
A. Up to 4500
B. Up to 5500
C. Up to 7500
D. Up to 2500
E. Up to 3500

35. Maximum permissible concentration of carbon dioxide in the air is considered


to be a sanitary index of air purity in a classroom. What concentration of
carbon dioxide in the air is accepted as maximum permissible?
A. 0,1%
B. 0,3%
C. 0,15%
D. 0,05%
E. 0,2%

36. Study of actual diet of an adult revealed the following: proteins make up 16%
of energy value of daily ration, fats - 25%, carbohydrates - 59%. Evaluate
compliance of protein, fat and carbohydrate share in the energy value of daily
ration with the recommended shares of these nutrients?
A. Carbohydrate share is insufficient, there is excess of proteins
B. Carbohydrate share is insufficient
C. Nutrient content complies with the recommended shares of energy value
D. Carbohydrate share is excessive
E. Fat share is insufficient

37. A patient who has been consuming refined foodstuffs for a long time
complains about headache, fatiguability, depression, insomnia, irritability.
Objectively: muscle asthenia, pain and cramps in the gastrocnemius muscles,
during walking the patient lands onto his heel first, then on the external edge
of foot. Cardiovascular system exhibits tachycardia, hypoxia, dystrophic
changes of myocardium. There are also gastrointestinal disorders. What is the
most likely diagnosis?
A. Hypovitaminosis B1
B. Hypovitaminosis B6
C. Hypovitaminosis B15
D. Hypovitaminosis B2
E. Hypovitaminosis B12

38. A patient who had eaten mushrooms in the morning was delivered to the
infectious diseases hospital at night. The disease development was rapid. The
patient presented with stomach pain, frequent diarrhea, intractable vomiting,
burning thirst, headache and dizziness. He died on the third day. What
mushrooms are most likely to have caused mycetismus?
A. Deadly amanita
B. Fly agarics
C. Russules
D. Sulfur-tufts
E. Morels

39. Periodical survey of a worker of a chemicals plant revealed a malignant


neoplasm on the urinary bladder. This occupational disease was the most
probably caused by contact with the following industrial poison:
A. Benzidine
B. Vinyl chloride
C. Asbestos
D. Arsenic
E. Nickel carbonyl

40. A 37 year old patient applied to a local therapeutist. As a result of exacerbation


of chronic obstructive bronchitis the patient had been temporarily disabled for
117 days within 1 year. What tactics will be legally correct?
A. The patient should be referred to the medicosocial expertise
B. The patient should be referred to the sanatorium-and-spa treatment
C. The patient should be referred to the medical consultation comission for
extension of medical certificate
D. The therapeutist should extend a medical certificate
E. The therapeutist should issue a new medical certificate

41. A 46-year-old patient once took part in elimination of breakdown at an atomic


power plant. Currently he is being treated at an in-patient hospital. He was
diagnosed with progressing vegetative insufficiency. This disease relates to the
following group of ionizing radiation effects:
A. Somato-stochastic
B. Genetic
C. Somatic
D. Hormesis
E. Heterosis

42. In treatment and prevention establishments, regardless of their organisational


and proprietary form, the rights of patients should be observed. Which of
these rights is the most significant?
A. The right to the protection of the patients interests
B. The right to be heard
C. The right to the protection from incompetence
D. The right to the free choice
E. The right to the information

43. A military unit stopped for 3-days rest in an inhabited locality after a long
march. The sanitary-epidemiological reconnaissance found several water
sources. It is necessary to choose the source complying with the hygienic
standards for drinking water in the field conditions:
A. Artesian well water
B. River water
C. Melt snow water
D. Rain water
E. Spring water

44. A district doctor was commissioned with a task to work out a plan of
treatment-and-prophylaxis actions for the population of his district. What
actions of secendary prophylaxis must he include into this plan?
A. Prevention of disease complications
B. Elimination of disease causes
C. Disease prevention
D. Improvement of populations living conditions
E. Rehabilitation actions

45. The district pediatrician is charged with the analysis of infant mortality. What
is taken for the unit of observation in infant mortality investigation?
A. A baby dead at the age up to 12 months
B. A baby dead at birth
C. A baby dead at the age over 28 days
D. A baby dead at the age up to 1 months
E. A baby dead at the age up to 6 days

46. Chief district pediatrician has to carry out analysis of infant mortality rate.
What should he take as a unit of the observation?
A. Child death case at the age up to 1 year
B. Child death case on labor
C. Child death case after 28 days of life
D. Child death case at the age up to the first month
E. Child death case during first 7 days of life
47. Deputy of chief medical officer carried out a study of morbidity rate for
population which had been served at the polyclinics within the last 5 years.
What statistical values can help in calculation of morbidity rates?
A. Relative values
B. Standard values
C. Absolute values
D. Dynamic series
E. Average values

48. There were registered 500 cases of urolithiasis per 10000 inhabitants. What
kind of statictical indices is presented?
A. Prevalence rate
B. Incidence rate
C. Index of compliance
D. Correlation coefficient
E. Index of visualization

49. At year-end hospital administration has obtained the following data: annual
number of treated patients and average annual number of beds used for
patients treatment. What index of hospital work can be calculated on the base
of this data?
A. Bed turnover
B. Average bed idle time
C. Average annual bed occupancy
D. Bed resources of the hospital
E. Average duration of patients presence in the hospital

50. Examination of a 13-year-old boy reveals that his body length is 147 ?m (+2),
body weight - 38 kg (+1,5), circumference of chest - 72 cm (+0,2). Estimate
the harmonicity of the childs physical development:
A. Disharmonious
B. Supernormal
C. Above the average
D. Harmonious
E. Sharply disharmonious

51. A childrens health camp received a party of tinned food. External examination
of the tins revealed that they had deep dents, could be easily concaved when
pressed and wouldnt immediately return to the initial state; rust was absent;
the tins were greased with inedible fat. Specify the bloat type:
A. Physical
B. Physicochemical
C. Biological
D. Chemical
E. Combined

52. On physiologic-sanitary examination of railway department work it was


revealed that loaders work is of III degree of difficulty. They unload vagons
with sand, manually break coagulated mass by shovel and shift it. What
criteria was used to evaluate work of loaders?
A. Maximun load weigh which is shifted
B. Time of active activities,% to the shift duration
C. Intellectual efforts
D. Time of passive observation,% to the shift duration
E. Value of static loading for the shift

53. Workers of fishery are subjected to low temperatures of the air (from 5 till
150C). Diseases of what organs and systems are the most frequent among
workers of such enterprises?
A. Respiratory system
B. Cardiovascular system
C. Liver
D. Gastrointestinal tract
E. Blood

54. During the periodic medical examination an assembly fitter (works on


soldering details) didnt report any health problems. Closer examination
revealed signs of asthenic-vegetative syndrome. Blood included red blood cells
with basophilic aggregations and a somewhat higher number of reticulocytes,
urine had a high concentration of delta-aminolevulinic acid. The complex of
symptoms indicates the initial stage of chronic intoxication with:
A. Lead
B. Mercury
C. Ethanol
D. Tin
E. Manganese

55. An outpatient hospital made record of 11600 diseases within one year. Among
them influenza and ARD make up 5800, circulatory system diseases - 3480,
digestion diseases - 1300, other diseases - 1020. What relative index can be
calculated according this data?
A. Extensive
B. Correlation
C. Intensive
D. Visualization
56. Prevalence of a disease in region N amounted 1156 occurences per 1000 of
inhabitants. What of the mentioned indices characterizes the disease
prevalence?
A. Intensive
B. Visual index
C. Standardized
D. Extensive
E. Ratio

57. A pupil of the 8th form after trauma has acute atrophy of the left arm muscles,
tonus of which is distinctly decreased, active movements are only in the left
joint, pupils foot is deformed. Function of support of the left leg is absent,
support function of the right leg is preserved. The boy wears an orthopedic
footwear. What group of physical training does the boy belong to?
A. Special
B. Basic
C. Other
D. Preparatory
E. Additional

58. District physician was charged with plan drafting concerning medical and
preventive measures among the population in the area he is assigned to. What
measures must he include in this plan as regards primary prevention of
illness?
A. Prevention of disease onset
B. Referral of patients to sanatorium
C. Measures to improve patients life conditions
D. Prevention of disease complications
E. Measures to increase patients life quality

59. A therapeutist needs to analyze adult health in the area of service. Which
groups of indicators will be included into this analysis?
A. Demographic, sickness rates, disability
B. Sickness rates, disability, death rates
C. Demographic, sickness rates, physical development
D. Birth rates, sickness rates, disability
E. Sickness rates, death rates, physical development

60. A district pediarician has carried out infant mortality rate analysis in his area.
What data has been used?
A. Mortality of children under 1 y.o. structured by age, sex, causes
B. Mortality of district adolescents
C. Mortality of district newborn
D. Mortality of children under 1 y.o., natimortality
E. Hospital mortality of children, structured by age

61. At the radiological unit of a hospital gamma-devices of radiotherapy of "Agat"


type and other closed sources of ionizing radiation are used for treatment of
malignant neoplasms. What measures are to be taken to protect personnel
during working with radioactive sources of such type?
A. Reduction of working time and screening of the source
B. Capsulation of devices and organization of room ventilation
C. The increase of distance to the source and individual hygiene compliance
D. Systematical cleansing of surfaces from the radioactive contamination and
shortening of working time
E. Screening of the source and the use of means of individual protection of
respiration organs

62. A patient undergoes inpatient treatment with the diagnosis of acute


pancreatitis. To spare pancreas as much as possible the doctor prescribed for
him starvation for 1-3 days. What products is the patient allowed to eat during
recovery period after cancelling of starvation?
A. Potato and carrot mash
B. Grape juice
C. Boiled meat
D. Broth
E. Milk

63. A local doctor has to prepare a report about the health condition of the
population of his region. What medical indexes of population health condition
should he use?
A. Morbidity, disabilities, demographic, physical development
B. Way of life, genetic, pollution
C. Social welfare, satisfaction of life quality
D. Average treatment duration, complications
E. Average longevity

64. To study physical development of children and adolescents, anthropometric


studies are widely used. Choose a physiometric method of study from the
below given
A. Determination of vital capacity of lungs
B. Measurement of growth
C. Determination of vertebra form
D. Determination of body weight
E. Determination of thorax form

65. Study of morbidity with temporary disability among workers of a machine


building plant revealed that average duration of a case was 20 days. What
diseases influenced upon the index value?
A. Chronic
B. Acute
C. Preexisting diseases
D. Hard to determine
E. Subacute

66. A patient consulted a doctor about acute respiratory viral infection. The
patient was acknowledged to be off work. The doctor issued him a medical
certificate for 5 days. The patient is not recovering. What measures should the
doctor take in order to legalize the further disability of patient?
A. To prolong the medical certificate at his own discretion but no
more than for 10 days in total
B. To send the patient to the medical social expert commission
C. To prolong the medical certificate together with department superintendent
D. To prolong the medical certificate at his own discretion but no more than for 6
days in total
E. To send the patient to the medical consultative commission

67. A patient is on the sick leave for 4 months continuously from the date of
injury. The treatment is going to last for 1-2 months more. Who has the right
to extend the duration of medical certificate for this patient?
A. Medical advisory commission after medico-social expert
commission examination
B. Medical advisory commission after inpatient treatment
C. Medical superintendent
D. District doctor by agreement with a department chief
E. Medico-social expert commission

68. Working conditions of a building company worker are characterized by


cooling microclimate effect, silica-containing dust, caustic alkali (quicklime)
and noise. What medical expert should be the chief of the commission that
periodically examines the workers of the mentioned category?
A. Therapeutist
B. Ophthalmologist
C. Dermatologist
D. Otolaryngologist
E. Neurologist
69. It is determined that 30 of a 100 women with risk factor had preterm labor,
and of a 100 women without risk factor 5 women had preterm labor. What
method of statistic data processing should the doctor use in order to estimate
reliability of differences between the compared groups?
A. Students criterion calculation
B. Relative numbers calculation
C. Correlation analysis
D. Standardization method
E. Average computing

70. 350 workers of a metalurgical plant had to undergo a yearly preventive


examination. A territorial polyclinic carried out preventive examination of 325
workers. As a result of it, 1 worker was recognized as temporarily disabled, 15
workers underwent further rehabilitation at an after-work sanatorium, 10
workers were provided with diet meal. What index characterizing the
preventive work of the polyclinic should be applied in this case?
A. Coverage of preventive medical examinations
B. Percentage of people who underwent rehabilitation at an after-work
sanatorium
C. Frequency of case detection during examinations
D. Percentage of people who were provided with diet meal
E. Percentage of temporarily disabled people

71. District doctor of rural medical department was called to a 42-year-old


patient. While examining the patient, the doctor suspected the dysentery.
What document must the doctor issue?
A. Urgent notification of infectious disease
B. Statistical coupon of final (precise) diagnosis
C. Abstract of outpatient medical card
D. Report addressed to Head of the village
E. Infectious disease report

72. During coal extraction in a mine the concentration of coal dust in the working
area is 450 mg/m3 (MPC is 10 mg/m3). What occupational respiratory
disease may develop in miners?
A. Anthracosis
B. Siderosis
C. Talcosis
D. Allergic nasopharyngitis
E. Byssinosis
73. A district doctor keeps the record of reconvalescents after infectious diseases,
people who are disposed to frequent and long-lasting diseases, patients with
chronic pathologies. What category of patients should belong to the III health
group?
A. People with chronic diseases
B. People with chronic pathologies and disposed to frequent and long-lasting
diseases
C. People disposed to frequent and long-lasting diseases
D. Reconvalescents after infectious diseases and patients with chronic
pathologies
E. All above mentioned categories

74. Over a current year among workers of an institution 10% havent been ill a
single time, 30% have been ill once, 15% - twice, 5% - 4 times, the rest - 5 and
more times. What is the percentage of workers relating to the I health group?
A. 55%
B. 22%
C. 40%
D. 10%
E. 60%

75. During a regular medical examination at a metallurgical plant 20% of workers


were found overweight (body weight was 5-14% higher than normal), and had
early signs of obesity (grade I-II) with Quetelet index from 26 to 30. What
products share must be reduced in the diet of this group of people in the first
place in order to normalize their body weight?
A. Bakery products
B. Fruit
C. Vegetables
D. Milk and dairy products
E. Meat and fish products

76. Chief physician of a polyclinic incharged a district doctor with a task to


determine the pathological prevalence of disease N in his district. What
document allows to estimate the disease prevalence in the population of a
medical district?
A. Prophylactic examinations register
B. Statistic coupons (-)
C. Vouchers for medical appointments
D. Statistic coupons (+) and (-)
E. Statistic coupons (+)
77. In a forest summer camp children have variable procedures to harden their
organisms. What procedure has the most hardening power?
A. Contrast shower
B. Morning exercises on the fresh air
C. Walking on the fresh air
D. Bath with hydromassage
E. Hygienic shower

78. There is a dynamic growth of number of congenital abnormalities such as


central paralysis, newborns blindness, idiocy among the population that lives
near to pesticides production enterprise. Compounds of which pollutant can
cause the development of this pathology?
A. Mercury
B. Cadmium
C. Chrome
D. Iron
E. Strontium

79. Ats planned to construct multifield a new hospital in one of the cental city
districts. What building type is the most appropriate in this case?
A. Centralized and blocked
B. Centralized
C. Mixed
D. Blocked
E. Decentralized

80.During inspection of sanitary conditions of studying at a technical university it


was necessary to evaluate the visual regimen of students, who study from 9
a.m to 3 p.m. What index of natural light will be the most informative?
A. Natural light coefficient
B. Time of the room insolation
C. Presence of mixed (superolateral) light
D. Light coefficient
E. Depth of study room

81. 200 patients suffering from essential hypertension were examined in order to
obtain data about patients arterial pressure and age. What statistic value
should be applied in order to measure relation between these characteristics?
A. Correlation coefficient
B. Representation error
C. Sygmal deviation
D. Students coefficient
E. Coefficient of variation
82. A doctor of the general practice has registered the following death causes for
the previous year: the first place was taken by cardiovascular diseases (60%),
the second - by tumors (18%), then - traumas (8,3%) etc. What diagrams will
provide the most substantial information about the registered ocurrences?
A. Pie diagram
B. Line diagram
C. Column diagram
D. Circle diagram
E. Cartogram

83. On medical observation a doctor identified girl (162 cm tall and 59 kg weight)
who complained loss of ability to see surrounding objects clearly in the
evening. On examination: dry skin, hyperkeratosis. Her daily ration includes
the following vitamines: vitamine A– 0,5 mg, vit.B1– 2,0 mg, vit.B2– 2,5 mg,
vit.B6– 2 mg, vit.C– 70 mg. What is the hypovitaminosis type?
A. A-hypovitaminosis
B. B1-hypovitaminosis
C. B6-hypovitaminosis
D. C-hypovitaminosis
E. B2-hypovitaminosis

84. As a result of prophylactic medical examination a 35 year old woman was


diagnosed with alimentary and constitutive obesity of the III degree. It is
known from her anamnesis that the patient doesnt observe rules of rational
nutrition: she often overeats, the last food intake is usually 10-15 minutes
before going to bed, prefers fattening and rich in carbohydrates food. What is
the main alimentary risk factor of obesity development?
A. Energetic unprofitableness of nutrition
B. Violation of dietary pattern
C. Excess of fats
D. Excess of carbohydrates
E. Lack of cellulose

85. The air of a foundry workers working zone contains condensation aerosol with
dust particles sized 2 nm (90%), 2-5 nm (2%), over 5 nm(6%), below 2 nm
(about 2%). Characterize the dust dispersivity:
A. Fine-dispersed
B. Coarsely dispersed
C. Mist
D. Ultrafine-dispersed
E. Median-dispersed
86. The major repair of a hospital included renewal of colour design of hospital
premises because it is of great psychological and aesthetical importance; and
so the walls of patient wards will be painted under consideration of:
A. Windows orientation
B. Wall reflection coefficient
C. Creation of cozy atmosphere
D. Hospital profile
E. Diseases of patients who will be staying in these wards

87. Atmospheric air of an industrial centre is polluted with the following wastes of
metallurgical plants: sulphuric, nitric, metal, carbon oxides that have negative
influence upon the inhabitant’s health. The effect of these hazards can be
characterized as:
A. Combined
B. Mixed
C. Associated
D. Complex
E. Adjacent

88. Study of morbidity rate in a city N revealed that population of different


administrative districts differed in age structure. What statistic method allows
to eliminate influence of this factor upon morbidity indices?
A. Standardization
B. Correlative regressive analysis
C. Calculation of average values
D. Analysis of dynamic series
E. Wilcoxons t-criterion

89. A driver had been fixing a car in a closed garage and afterwards complained
about headache, dizziness, nausea, muscle asthenia, sleepiness. Objectively:
pulse and respiratory rate elevation, excitement, hypertension, delirium of
persecution. What is the most likely diagnosis?
A. Intoxication with carbon oxide
B. Posttraumatic encephalopathy
C. Asthenovegetative syndrome
D. Hypertensive crisis
E. Intoxication with ethyl gasoline

90. An outbreak of food poisoning was recorded in an urban settlement. The


illness was diagnosed as botulism on the grounds of clinical presentations.
What foodstuffs should be chosen for analysis in the first place in order to
confirm the diagnosis?
A. Tinned food
B. Cabbage
C. Pasteurized milk
D. Potatoes
E. Boiled meat

91. A worker who undergoes regular medical check-up for duodenal ulcer
received a subsidized 24-day sanatorium voucher from his plant. The term of
annual leave of a worker is 24 calender days, it will take 4 days more to get to
the sanatorium and back home. What is the procedure of obtaining a 4-day
sick-leave?
A. Medical Expert Commission issues a 4-day sick list
B. The doctor in charge issues a 4-day sick list
C. Medical Expert Commission issues a 28-day sick list
D. The doctor in charge issues a health certificate and sanatorium patients file for
28 days
E. Medical Expert Commission issues a 4-day health certificate

92. A 42 year old metalworker has been working at the turning machine for
production of heavy large-size parts for 5 years. His work requires using of
hand and pedal levers that involves considerable physical force. What means
for osteoarthrosis prevention should be recommended?
A. To limit physical work
B. To go in for weightlifting
C. To administer protein-and-vitamin diet
D. To administer protein-and-carbohydrate diet
E. To improve health at the Black sea coast

93. Administration of a plant producing red lead paint intends to form a group of
medical specialists for periodical medical examinations. What specialist must
be obligatory included into this group?
A. Neuropathologist
B. Psychiatrist
C. Otolaryngologyst
D. Dermatologist
E. Gynaecologist

94. The total area of a ward at the therapeutical department is 28 m2. What is the
maximum number of beds that can be exploited in this ward?
A. 4
B. 2
C. 1
D. 3
E. 5
95. The amount of ultraviolet radiation dose was measured in minutes. What
device was applied for measurement of the biodose?
A. Gorbachevs biodosimeter
B. Actinometer
C. UV-meter
D. Radiometer
E. Catathermometer

96. The institutions which take part in medical examinations can be prevention
and treatment facilities, medical board of Ministry of Defence, medical board
of Ministry of Home Affairs, medico-social expert commissions, forensic
medical boards etc. What institutions are responsible for temporary disability
examination?
A. Prevention and treatment facilities
B. Medical boards of Ministry of Home Affairs
C. Medico-social expert commissions
D. Sanitary-and-prophylactic institutions
E. Medical boards of Ministry of Defense

97. A mother who is on partially paid maternity leave got sick and was
hospitalized. What document is to be issued to a working father who will be
taking care of a child during his mother’s illness?
A. Sick-leave
B. Certificate of mother’s illness
C. Extract from the medical card of out- or in-patient
D. Free-form certificate
E. Certificate of child’s care necessity

98. Clinical and statistical study was devoted to the effect of a new
pharmacological medication upon the patients with coronary heart disease.
What parametric criterion (coefficient) can be used for estimation of results
validity?
A. Students coefficient (t)
B. Kolmogorov-Smirnovs criterion
C. Conformity coefficient
D. Sign criterion
E. Wilcoxons t-criterion

99. It is planned to make complete isolation boxes in the infectious department in


order to prevent nosocomial airborne infections. The boxes consist of a
tambour, a ward and a lock chamber. What structure should be also included
in a complete isolation box?
A. Bathroom unit
B. Doctors consulting room
C. Manipulation room
D. Patients examination room
E. Nursing room

100. Educational rooms are illuminated with various lighting fittings. What
type of lighting fittings is the most appropriate in respect of hygienic norms?
A. Indirect light fittings
B. Semi-reflected light fittings
C. Combined light fittings
D. Ambient light fittings
E. Direct light fittings

101. In terms of megacalorie (1000 kcal = 4184 kJ) the ration of an adult
includes 30 g of proteins, 37 g of fats, 137 g of carbohydrates, 15 mg of vitamin
C, 0,6 mg of thiamine (vitamin D1). The ration is textbfUNBALANCED as to
the contents of:
A. Vitamin C
B. Fats
C. Proteins
D. Carbohydrates
E. Thiamine

102. A city somatic hospital with 300 beds consists of the main building
which houses the therapeutic and surgical departments. Several separate
buildings house the maternity, pediatric and radiologic departments that are
connected to the main building by underground walkways and above-ground
covered skybridges. Specify the building system of the hospital:
A. Central-unit
B. Decentralized
C. Centralized
D. Free
E. Combined

103. A heat station working on solid fuel is located in a residential district.


On cloudy foggy days in december there was an increase in diseases with
upper airway affection and signs of general intoxication. There were also
mortal cases among the elderly people. What is the most likely factor that
provoked toxic effect?
A. Suspended materials
B. Calm
C. Temperature gradient
D. Low air temperature
E. High air humidity

104. Indices that characterize population health include demographic


indices. What environment is used for calculation of these indices?
A. Population number
B. Number of hospitalized people
C. Employment number
D. Number of patients
E. Number of population being liable to preventive examination

105. While making sanitary examination of burn unit for adults it was stated
that wards for 4 persons are of 28 m2 square. What should be the minimum
ward area in this unit?
A. 40 m2
B. 30 m2
C. 52 m2
D. 24 m2
E. 28 m2

106. The results of 5 year monitoring allowed to estimate the level of


environmental influence upon health indices of population. What statistic
method should be chosen?
A. Calculation of correlation coefficient
B. Calculation of dynamic indices
C. Calculation of coefficient of difference validity
D. Calculation of conformity coefficient
E. Calculation of regression coefficient

107. In order to improve organism tolerance of boarding-school pupils a


doctor developed a program. The program is based upon the following
principles: graduality, consistency, individuality, coomplexity. What of the
main principles of organism tempering wasnt taken into account?
A. Increase of resistance
B. Increase of influence intensity
C. Autodefense increase
D. Increase of influence force
E. Systematicness
108. A student analyzes noise level of cold-pressing process. What device
should be applied for this hygienic study?
A. Noise and vibration analyzer
B. Noise analyzer
C. Actinometer
D. Pyranometer
E. Sound tester

109. Environmental pollution is prevented by mechanical separation of


nontoxic solid domestic waste. Specify the method which can be used for
mechanical utilization of these wastes:
A. Compressing of wastes into building blocks
B. Waste neutralization in biothermal boxes
C. Burning as power-plant fuel
D. Hydrolysis
E. Burial of wastes

110. An emergency situation at a chemical plant caused acute occupational


intoxication. A doctor who revealed the case of "acute occupational disease
(intoxication)" must notify the following authority:
A. Sanitary and epidemiological station
B. Ministry of Health of Ukraine
C. Trade union committee of the plant
D. Plant administration
E. Medical unit of the plant

111. At a machine-building plant the casts are cleaned by means of abrasion


machines that are a source of local vibration. What are the most efficient
preventive measures for preventing harmful effect of vibration on workers
organisms?
A. Use of gloves that reduce vibration
B. Hand massaging
C. Preliminary and periodical medical examinations
D. Warm hand baths
E. Giving sanitary instructions to the workers

112. In order to study impact of microclimate upon the human organism it


is necessary to make systematic observation of air temperature over 3 days.
Choose a device that will allow to make the most precise temperature records:
A. Thermograph
B. Augusts psychrometer
C. Assmann psychrometer
D. Alcohol thermometer
E. Mercury thermometer

113. The Carpathian region is characterized by constant high humidity of


atmospheric air (over 80%). Inhabitants of this region feel severe cold in
corresponding season at a medium low temperature. Its caused by heat
emission by:
A. Convection
B. Vaporization
C. Radiation
D. Conduction

114. Poorly refined wastes of an industrial plant are usually thrown into the
river that supplies drinking water. It causes perishing of some
microorganisms, disturbs processes of water self-purification and worsens its
quality that can have negative influence upon people’s health. How is this
effect of environmental factors called?
A. Indirect
B. Complex
C. Combined
D. Direct
E. Associated

115. In order to reduce weed growth on agricultural land, some herbicides


have been used for a long time. In terms of environmental stability these
herbicides are rated as stable. Specify the most likely route of their entry into
the human body:
A. Soil-plants-humans
B. Soil-insects-humans
C. Soil-animals-humans
D. Soil-microorganisms-humans
E. Soil-protozoa-humans

116. In a city with population 400000 people 5600 fatal cases were
recorded, including 3300 cases because of blood circulation diseases, 730 -
because of tumors. What index will allow to characterize mortality from blood
circulation diseases in this city?
A. Intensive index
B. Visuality index
C. Correlation index
D. Extensive index
E. Relative intensity index
117. A selective population research study was aimed at exploring the effect
of air emissions from a metallurgical plant on the obstructive bronchitis
morbidity in a city. The calculated correlation coefficient was +0,79. Evaluate
the strength and direction of the relationship:
A. Direct, strong
B. Inverse, average
C. Inverse, strong
D. Direct, average

118. Statistic of patients of common medical practice displays constant


increase in elderly and old people number. What kind of pathology is expected
to prevail in the morbidity structure of population in question?
A. Chronic pathology
B. Non-epidemic pathology
C. Infectious pathology
D. Acute pathology
E. Occupational pathology

119. In the current year general practitioners of the municipal polyclinic


have referred 11 patients with coronary artery disease to the in-patient
hospital. In 3 cases the diagnosis wasnt confirmed. What managerial decision
should be made in such case?
A. Analysis of each case of diagnostic divergence
B. Analysis of material and technical basisof the polyclinic
C. Analysis of diagnostic examination quality
D. Analysis of medical check-up quality
E. Analysis of doctor’s skill level

120. Hygienic expertise of a sample taken from the batch of grain revealed
that 2% of grains were infected with microscopic Fusarium fungi. On the
ground of laboratory analyses this batch of grain should be:
A. Sold without restrictions
B. Used for ethanol production
C. Destroyed
D. Tested for toxicity
E. Used for forage production

121. Among the inhabitants of a workmens settlement located near an


industrial plant the cases of nervous and endocrine system diseases as well as
renal diseases became more frequent. Blood analyses revealed a decrease in
sulfhydryl groups. These pathologies might have been caused by the following
substance released into the environment:
A. Mercury
B. Lead
C. Boron
D. Cadmium
E. Chromium

122. Examination of a 43-year-old man objectively revealed pallor of skin


and mucous membranes, loss of tongue papillae, transverse striation of
fingernails, cracks in the mouth corners, tachycardia. Blood test results: Hb-
90 g/l, anisocytosis, poikilocytosis. The most likely causative agent of this
state is inadequate intake of:
A. Iron
B. Copper
C. Magnesium
D. Selene
E. Zinc

123. During the medical examination a port crane operator complained of


dizziness, nausea, sense of pressure against tympanic membranes, tremor,
dyspnoea, cough. He works aloft, the work is connected with emotional stress.
Workers are affected by vibration (general and local), noise, ultrasound,
microclimate that warms in summer and cools in winter. What factor are the
worker’s complaints connected with?
A. Infrasound
B. Intensity of work
C. Altitude work
D. Noise
E. Vibration

124. It is required to analyze the level of daylight illumination in a ward of


therapeutics unit. What device should be applied to estimate the level of
daylight illumination?
A. Illuminometer
B. Katathermometer
C. Psychrometer
D. Actinometer
E. Anemometer

125. During the medical examination at school the schoolchildren had to


undergo plantography. After the analysis of footprints platypodia was found in
30% of pupils. What is the percentage of the flatfoot isthmus?
A. 65%
B. 50%
C. 45%
D. 55%
E. 30%

126. A factory’s sectorial doctor selects a group of persons who often fall ill
for thorough monitoring. At the same time he takes into consideration the
number of etiologically related cases with temporary disability in each of the
employees over the last year. An employee falls into this group if the number
of sickness cases is:
A. 4 or more
B. 1 or more
C. 3 or more
D. 6 or more
E. 2 or more

127. Production areas of a greenhouse complex have the following


microclimate parameters: air temperature - 42°C, humidity - 98%, air velocity
- 0,05 mps, temperature of enclosing surfaces - 15°C. Characterize the
microclimate of production areas:
A. Overheated
B. Uncomfortable
C. Satisfactory
D. Comfortable
E. Cooling

128. Sanitary-veterinary examination of a cow carcass revealed measle


contamination (2-3 measles per 10 cm2). What tactics should be chosen in
respect of this meat consumption?
A. The whole carcass should be technically disposed
B. The carcass should be used for the production of canned meat
C. Meat should be disinfected by freezing
D. Meat should be disinfected by boiling
E. Meat can be sold without any restrictions

129. A factory worker has ARD complicated by acute bronchitis. He receives


treatment in the outpatient setting. The attending doctor has issued him a
medical certificate for 5 days and then extended its duration by 5 more days.
Patient cant get down to work because of his health status. Who should extend
the duration of medical certificate for this patient?
A. A department chief
B. Deputy medical superintendent in charge of medical treatment
C. Medical advisory commission
D. Medical superintendent
E. Deputy medical superintendent in charge of temporary disability examination
130. Preventive examination of an 11 year old boy helped to determine his
habitus type. It was established that the child’s shoulders were deviated and
brought forward, with forward flexion of head, the thorax was flattened,
abdomen was convex. The child’s backbone had signs of deepened cervical
and lumbar curvatures. What habitus is it?
A. Kyphosis
B. Normal
C. Round-shouldered
D. Lordosis
E. Corrected

131. A plot of land with total area of 2,0 hectare was intended for building of
a hospital. The maximal capacity of the hospital will be:
A. 100 beds
B. 400 beds
C. 200 beds
D. 800 beds
E. Over 1000 beds

132. During hygienic examination of a hospital it was established that the


area for each bed in a double ward was: in the infectious department for
children - 7x2, in the infectious department for adults – 8x2, in the burns
department – 9x2, in the radiological department – 10x2, in the critical care
department – 13x2. In which wards the area for each bed doesnt correspond
with hygienic requirements?
A. In burns wards
B. In critical care wards
C. In infectious wards for adults
D. In infectious wards for children
E. In radiological wards

133. According to the report of water quality control, drinking city water has
the following characteristics: turbidity - 1,5 mg/m3, odour - 3 points, metallic
taste - 2 points, pale yellow colour, colour index - 20o, temperature - 12o.
Which of these factors doesnt comply with hygienic requirements?
A. Odour
B. Turbidity
C. Temperature
D. Taste
E. Colour index
134. A 2 year old child has been ill with acute respiratory viral infection of
upper thrice a year - in February, in April and in December. How should these
occurrences be recorded?
A. It is necessary to fill in 3 statistic talons signed (+)
B. It is necessary to fill in 1 statistic talon signed (-)
C. It is necessary to fill in 1 statistic talon signed (+)
D. It is necessary to fill in 3 statistic talons signed (-)
E. It is necessary to fill in 1 statistic talon signed (+) and 2 statistic talons signed
(-)

135. A worker diagnosed with "acute dysentery" was sent to the infectious
department by a doctor of aid post. What document should be used for
registration of this disease?
A. Urgent report on infectious disease
B. Inpatients card
C. Statistic card of the patient who left in-patient hospital
D. Statistic coupon for registration of final diagnoses
E. Outpatients card

136. Researchers studied disease incidence of influenza and acute


respiratory viral infection within the last 5 years. What kind of graphic
presentation should be used for the best visualization of this data?
A. Linear diagram
B. Pie diagram
C. Histogram
D. Radial diagram
E. Bar diagram

137. The correlation between the service record and eosinophil


concentration in blood was studied in workers at dyeing shops of textile
factories. What index will be the most informative for the analysis of this data?
A. Correlation factor
B. Fitting criterion
C. Sign index
D. Students criterion
E. Standardized index

138. The objective of a statistical study was to find out the extent of seeking
medical care by the population. For this purpose, 300 residents of the area
were interviewed. Information was collected by means of a special
questionnaire. What method of collecting information was used by
researchers?
A. Anamnestic
B. Immediate examination
C. Doing extracts
D. Immediate registration

139. Analysis of organization of medical care in a regional centre has shown


that every year about 12% of patients receive inpatient care for diseases that
dont require round-the-clock monitoring and intensive care. What are the
most appropriate organizational changes required to address this problem?
A. Development of medical care forms replacing the in-patient care
B. Restructuring of specialized care
C. Upgrading of hospital facilities
D. Changes to the statute of outpatient clinics
E. Development of primary care

140. A city’s population is 400000 inhabitants in 2005 there were registered


5600 deaths, including 3300 cases caused by cardiovascular diseases, 730 - by
tumours. Which of the following indicators allows to characterize the share of
the circulatory system diseases as the cause of death in the city?
A. Extensive index
B. Index of relative intensity
C. Intensive index
D. Demonstrativeness index
E. Ratio

141. It is planned to organize a rural outpatient clinic. The patients will be


able to visit the doctors of the following specialities:
A. Therapeutist, dentist, pediatrician, obstetrician-gynecologist
B. Therapeutist, pediatrician, neurologist
C. Pediatrician, therapeutist, ophthalmologist
D. Obstetrician-gynaecologist, therapeutist
E. Pediatrician, obstetrician-gynaecologist, ophthalmologist

142. In a rural health district, a child died in the first month of life. In order
to analyze this situation it was necessary to complete expert evaluation of
medical records. Which medical document was analysed in the first place?
A. Child developmental history record
B. Childs medical record
C. Record of vaccinations
D. Neonatal record
E. Medical record of an outpatient
143. A department chief of an in-patient hospital is going to inspect resident
doctors as to observation of medical-technological standards of patient
service. What documentation should be checked for this purpose?
A. Health cards of in-patients
B. Treatment sheets
C. Statistic cards of discharged patients
D. Registry of operative interventions
E. Annual report of a patient care institution

144. The head of prenatal care department intends to complete expert


evaluation of compliance with medical and technological standards of
pregnancy follow-up. What documents must be verified for this purpose?
A. Individual records of pregnant and postpartum women
B. Neonatal record
C. Medical records of outpatients
D. Prenatal records
E. Labour and delivery record

145. Workers of a laboratory producing measuring devices (manometers,


thermometers etc.) complain about a metallic taste in mouth, stomatitis,
dyspepsia, sleep disturbance, unsteady walk, abrupt decrease in cardiac
activity. These presentations must have been caused by the intoxication with
the following substance:
A. Mercury
B. Manganese
C. Tetraethyl lead
D. Toluol
E. Lead

146. While assessing the health status of graduates of a secondary school,


the doctor found one of them to have grade 3 tonsillar hypertrophy, chronic
rhinitis and vegetative-vascular dystonia. The organism functionality is
reduced. This student belongs to the following health group:
A. III
B. IV
C. V
D. II
E. I

147. At first appointment with an obstetrician-gynaecologist a pregnant


woman is referred to other medical specialists. She must be obligatory
examined by the following specialists:
A. Therapeutist and dentist
B. Dentist and phthisiatrician
C. Dentist and cardiologist
D. ENT and ophthalmologist
E. Therapeutist and endocrinologist

148. Estimation of physical development of a child involved dynamometry


and estimation of body weight and length, annual gain in body length, chest
circumference, number of permanent teeth, secondary sexual characters, lung
vital capacity. Which of the mentioned indices relates to the physio metric
ones?
A. Lung vital capacity, dynamometry
B. Number of permanent teeth
C. Annual gain in body length
D. Body length and weight, chest circumference
E. Secondary sexual characters

149. Estimation of community health level involved analysis of a report on


diseases registered among the population of district under charge (reporting
form 12). What index is calculated on the grounds of this report?
A. Common morbidity rate
B. Index of basic non-epidemic morbidity
C. Index of morbidity with temporary disability
D. Index of pathological affection
E. Index of hospitalized morbidity

150. A 50-year-old male suburbanite underwent treatment in rural


outpatient clinic for pneumonia. The treatment didn’t have effect and the
disease got complicated by exudative pleuritis. What prevention and
treatment facility should the patient be referred to for further aid?
A. Central district hospital
B. Phthisio-pulmonological dispensary
C. Regional hospital
D. Municipal hospital
E. Tuberculosis dispensary

151. A 46-year-old patient was issued a 10-day sick list because of


exacerbation of chronic cholecystitis. The patients general condition got
better, but the clinical manifestations of the disease are still present. What
authority is entitled to extend the sick list?
A. Medical Expert Commission
B. Deputy head doctor for medical-labour expertise
C. Head doctor
D. Family doctor
E. Deputy head doctor for terapeutic management

152. A 59-year-old male patient with essential hypertension of stage II is


registered with the dispensary department of a polyclinic. The patient
regularly takes ACE inhibitors and calcium antagonists. How often should a
therapeutist examine this patient (except for exacerbation periods)?
A. Every 3 months
B. Every 6 months
C. Once a year
D. Every 9 months
E. Every 4 months

153. A municipal hospital reported on the number of operated patients


including fatal outcomes following the operations. Which index of hospital
work can be calculated on the ground of this data?
A. Postoperative lethality
B. Index of late hospitalization since a disease incursion
C. Total lethality
D. Standardized lethality

154. It is planned to build a multi-disciplinary hospital with 500 beds in a


town. Specify the location of a polyclinic within the medical centre:
A. At the main entrance
B. In the centre of the territory near medical buildings
C. In the garden and park area
D. It is not allowed to place the polyclinic within the centre territory
E. In the service zone

Krok 2 – 2013 Pediatrics Base


1. An 8 year old child has low-grade fever, arthritis, colicky abdominal pain and
a purpuric rash llocalized on the lower extremities. laboratory studies reveal a
guaiac-positive stool, urinalysis with red blood cell (RBC) casts and mild
proteinuria, and a normal platelet count. The most likely diagnosis is:
A. Henoch-Schonleins vasculitis
B. Idiopathic thrombocytopenic purpura
C. Poststreptococcal glomerulonephritis
D. Systemic lupus erythematosus (SLE)
E. Rocky Mountain spotted fever

2. A young man has painful indurations in the peripapillary regions of both


mammary glands. The most reasonable action will be:
A. To leave these indurations untouched
B. To cut and drain them
C. To remove them
D. To take an aspirate for bacterial inoculation and cytology
E. To administer steroids locally

3. A 9 year old girl with a history of intermittent wheezing for several years is
brought to the pediatrician. The child has been taking no medications for
some time. Physical examination reveals agitation and perioral cyanosis.
Intercostal and suprasternal retractions are present. The breath sounds are
quiet, and wheezing is audible bilaterally. The child is admitted to the
hospital. Appropriate interventions might include all of the following text of
EXCEPT:
A. Prescribe nebulized cromolyn sodium
B. Prescribe nebulized metaproterenol
C. Administer supplemental oxygen
D. Prescribe intravenous aminophylline
E. Prescribe intravenous corticosteroids

4. Routine examination of a child with a history of bronchial asthma reveals AP


of 140/90 mm Hg. The most likely cause of the hypertension is:
A. Renal disease
B. Chronic lung disease
C. Theophylline overdose
D. Coarctation of the aorta
E. Obesity

5. Patient with thyreotoxicosis is in the 2 beds hospital ward of therapeutic


department. The area of the ward is 18 m2, height 3 m, ventilation rate 2,5/hr.
Air temperature - 20°C, relative humidity - 45%, air movement velocity - 0,3
m/s, light coefficient - 1/5, noise level - 30 dB. Do hygienic evaluation of the
conditions meet the standards?
A. Discomfortable microclimate
B. High level of noise
C. All conditions meet the requirements
D. Non-effective ventilation
E. Poor lighting

6. The child is 11 m.o. He suffers from nervous-arthritic diathesis. The increased


synthesis of what acid is pathogenic at nervous-arthritic diathesis?
A. Uric acid
B. Phosphoric acid
C. Acetic acid
D. Hydrochloric acid
E. Sulfuric acid

7. A 10-year-old child complains of fever (temperature is 39OC), frequent


painful urination [pollakiuria]. Urine test: proteinuria [0,066 g/L],
leukocytouria [entirely within eyeshot], bacteriuria [105 colony forming
units/mL]. What is the most probable diagnosis?
A. Acute pyelonephritis
B. Dysmetabolic nephropathy
C. Acute glomerulonephritis
D. Acute cystitis
E. Urolithiasis

8. A 8-year-old boy has suffered from tonsilitis. In 2 weeks he started


complaining of migratory joint pain, edema of joints, restriction of
movements, fever. On examination, an acute rheumatic heart disease, activity
of the III-rd degree, primary rheumocarditis, polyarthritis; acute course of
disease, cardiovascular failure IIA. What medication is to be prescribed?
A. Prednisone
B. Diprazinum
C. Erythromycin
D. Cefazolin
E. Delagil

9. The 10 y.o. boy has complains on headache, weakness, fever 40°C, vomiting,
expressed dyspnea, pale skin with flush on right cheek, lag of right hemithorax
respiratory movement, dullness on percussion over low lobe of right lung,
weakness of vesicular respiration in this zone. The abdomen is painless and
soft at palpation. Which disease lead to these symptoms and signs?
A. Pneumonia croupousa
B. Flu
C. Acute appendicitis
D. Intestinal infection
E. Acute cholecystitis

10. A patient with acute respiratory viral infection (3rd day of disease) complains
of pain in lumbar region, nausea, dysuria, oliguria. Urinalysis - hematuria
(100-200 RBC in eyeshot spot), specific gravity - 1002. The blood creatinin
level is 0,18 millimole/l, potassium level - 6,4 millimole/l. Make the diagnosis:
A. Acute interstitial nephritis
B. Acute cystitis
C. Acute renal colic
D. Acute renal failure
E. Acute glomerylonephritis

11. A baby boy was born in time, it was his mothers 1st pregnancy. The jaundice
was revealed on the 2nd day of life, then it progressed. The adynamia,
vomiting and hepatomegaly were presented. The indirect bilirubin level was
275 mcmol/L, the direct bilirubin level - 5 mcmol/L, Hb- 150 g/L. Mothers
blood group - 0(I), Rh+, childs blood group - A(II), Rh+. Make a diagnosis
A. Hemolytic disease of newborn (ABO incompatibility), icteric type
B. Hepatitis
C. Hemolytic disease of newborn (Rh - incompatibility)
D. Physiological jaundice
E. Jaundice due to conjugation disorder

12. A neonate was born from the 1st gestation on term. The jaundice was revealed
on the 2nd day of life, then it became more acute. The adynamia, vomiting and
hepatomegaly were observed. Indirect bilirubin level was 275mumol/L, direct
bilirubin level - 5mumol/L, Hb- 150 g/l. Mothers blood group - 0(I), Rh+,
child’s blood group - A(II), Rh+. What is the most probable diagnosis?
A. Hemolytic disease of the neonate (AB0 incompatibility), icteric
type
B. Jaundice due to conjugation disorder
C. Physiological jaundice
D. Hemolytic disease of the neonate (Rh - incompatibility)
E. Hepatitis

13. A 3 month old infant suffering from acute segmental pneumonia has dyspnea
(respiration rate - 80 per minute), paradoxical breathing, tachycardia, total
cyanosis. Respiration and pulse - ratio is 1:2. The heart dullness under normal
size. Such signs characterise:
A. Respiratory failure of III degree
B. Respiratory failure of II degree
C. Respiratory failure of I degree
D. Myocarditis
E. Congenital heart malformation

14. The 7 m.o. infant is suffering from acute pneumonia which was complicated
by cardiovascular insufficiency and respiratory failure of II degree. The
accompanied diagnosis is malnutrition of II degree. Choose the best variant of
therapy:
A. Ampiox and Amicacin
B. Penicillin and Ampiox
C. Ampiox and Polymixin
D. Gentamycin and Macropen
E. Macropen and Penicillin

15. A 3 year old child has been suffering from fever, cough, coryza, conjunctivitis
for 4 days. He has been taking sulfadimethoxine. Today it has fever up to 39°C
and maculopapular rash on its face. Except of rash the child’s skin has no
changes. What is your diagnosis?
A. Measles
B. Pseudotuberculosis
C. Rubella
D. Allergic rash
E. Scarlet fever

16. A 2 year old girl has been ill for 3 days. Today she has low grade fever, severe
catarrhal presentations, slight maculopapular rash on her buttocks and
enlarged occipital lymph nodes. What is your diagnosis?
A. Rubella
B. Measles
C. Pseudotuberculosis
D. Adenoviral infection
E. Scarlet fever

17. A 3 year old boy fell ill abruptly: fever up to 39°C, weakness, vomiting.
Haemorrhagic rash of various size appeared on his lower limbs within 5 hours.
Meningococcemia with infective - toxic shock of the 1 degree was diagnosed.
What medications should be administered?
A. Chloramphenicol succinate and prednisone
B. Ampicillin and immunoglobulin
C. Penicillin and immunoglobulin
D. Penicillin and prednisone
E. Chloramphenicol succinate and interferon

18. A 7 year old girl has mild form of varicella. Headache, weakness, vertigo,
tremor of her limbs, ataxia, then mental confusion appeared on the 5th day of
illness. Meningeal signs are negative. Cerebrospinal fluid examination is
normal. How can you explain these signs?
A. Encephalitis
B. Myelitis
C. Neurotoxic syndrome
D. Meningitis
E. Meningoencephalitis
19. A 7 y.o. girl fell ill abruptly: fever, headache, severe sore throat, vomiting.
Minute bright red rash appear in her reddened skin in 3 hours. It is more
intensive in axillae and groin. Mucous membrane of oropharynx is hyperemic.
Greyish patches is on the tonsills. Submaxillary lymph nodes are enlarged and
painful. What is your diagnosis?
A. Scarlet fever
B. Enteroviral infection
C. Rubella
D. Measles
E. Pseudotuberculosis

20. An 8-year-old boy fell ill acutely: he presents with fever, weakness, headache,
abdominal pain, recurrent vomiting, then diarrhea and tenesmus. Stools occur
12 times daily, are scanty, contain a lot of mucus, pus, streaks of blood. His
sigmoid gut is tender and hardened. What is your diagnosis?
A. Dysentery
B. Staphylococcal gastroenteritis
C. Escherichiosis
D. Salmonellosis
E. Cholera

21. The child has complains of the "night" and "hungry" abdominal pains. At
fibroscopy in area a bulbus ofa duodenum the ulcerrative defect of 4 mms
diameter is found, the floor is obtected with a fibrin, (H.p +). Administer the
optimum schemes of treatment:
A. Omeprasole - Trichopolum - Claritromicin
B. Trichopolum
C. Maalox - Ranitidin
D. De-nol
E. Vicalinum - Ranitidin

22. A woman delivered a child. It was her fifth pregnancy but the first delivery.
Mothers blood group is A(II)Rh-, newborns - A(II)Rh+. The level of indirect
bilirubin in umbilical blood was 58 micromole/l, haemoglobin - 140 g/l, RBC-
3,8×1012/l. In 2 hours the level of indirect bilirubin turned 82 micromole/l.
The hemolytic disease of newborn (icteric-anemic type, Rh-incompatibility)
was diagnosed. Choose the therapeutic tactics:
A. Replacement blood transfusion (conservative therapy)
B. Symptomatic therapy
C. Antibiotics
D. Conservative therapy
E. Blood transfusion (conservative therapy)
23. A mother with an infant visited the paediatrician for expertise advice. Her
baby was born with body weight 3,2 kg and body length 50 cm. He is 1-year-
old now. How many teeth the baby should have?
A. 8
B. 6
C. 12
D. 10
E. 20

24. A mother consulted a pediatrician about her son. Her son was born with body
mass of 3 kg and length of 48 cm. He is 1 year old now. What is the required
normal mass?
A. 10,5 kg
B. 12,0 kg
C. 15,0 kg
D. 9,0 kg
E. 11,0 kg

25. 6 m.o. infant was born with body’s mass 3 kg and length 50 cm. He is given
natural feeding. How many times per day the infant should be fed?
A. 5
B. 6
C. 7
D. 8
E. 4

26. Infant is 6,5 months now and is given natural feeding since birth. Body mass
was 3,5 kg, with length 52 cm at birth. How many times per day the
supplement (up feeding) should be given?
A. 2
B. 3
C. 0
D. 4
E. 1

27. A 2-month old healthy infant with good appetite is given artificial feeding
since he turned 1 month old. When is it recommended to start the corrective
feeding (fruit juice)?
A. 4,0 months
B. 1,5 months
C. 3,0 months
D. 1,0 months
E. 2,0 months
28. An infant was born with body mass 3 kg and body length 50 cm. Now he is 3
years old. His brother is 7 years old, suffers from rheumatic fever. Mother
asked the doctor for a cardiac check up of the 3-year-old son. Where is the left
relative heart border located?
A. 1 cm left from the left medioclavicular line
B. Along the left medioclavicular line
C. 1 cm right from the left parasternal line
D. 1 cm left from he left parasternal line
E. 1 cm right from the left medioclavicular line

29. A boy of 7 y.o. had an attack of asthma and distant whistling rales after
playing with a dog. In the medical history: atopic dermatitis caused by eating
eggs, chicken, beef. What group of allergens is the reason of the development
of bronchial asthma attacks?
A. Epidermal
B. Pollen
C. Chemical
D. Itch mite
E. Dust

30. A 14-year-old boy has rheumatism. Over the last 2 years he has had 3
rheumatic attacks. What course of rheumatism does the patient have?
A. Prolonged
B. Latent
C. Persistent-reccurent
D. Acute
E. Subacute

31. The patient with acquired heart failure has diastolic pressure of 0 mm Hg.
What heart failure does the child have?
A. Aortal insufficiency
B. Aortal stenosis
C. Mitral stenosis
D. Mitral insufficiency
E. Rheumatism

32. A 12 year old child has the ulcer disease of stomach. What is the etiology of
this disease?
A. Intestinal bacillus
B. Influenza
C. Salmonella
D. Helicobacter pylory
E. Lambliosis

33. A nine-year-old child is at a hospital with acute glomerulonephritis. Clinical


and laboratory examinations show acute condition. What nutrients must not
be limited during the acute period of glomerulonephritis?
A. Carbohydrates
B. Liquid
C. Fats
D. Proteins
E. Salt

34. An 18-month-old child was taken to a hospital on the 4-th day of the disease.
The disease began acutely with temperature 39, weakness, cough,
breathlessness. He is pale, cyanotic, has had febrile temperature for over 3
days. There are crepitative fine bubbling rales on auscultation. Percussion
sound is shortened in the right infrascapular region. X-ray picture shows non-
homogeneous segment infiltration 8-10 mm on the right, the intensification of
lung pattern. Your diagnosis:
A. Segmentary pneumonia
B. Bronchiolitis
C. Interstitial pneumonia
D. Grippe
E. Bronchitis

35. A 9-year-old girl has attacks of abdominal pain after fried food. No fever. She
has pain in Cera point. The liver is not enlarged. Portion B [duodenal probe] -
50 ml. What is your diagnosis?
A. Biliary tracts dyskinesia, hypotonic type
B. Acute colitis
C. Peptic ulcer
D. Chronic duodenum
E. Hepatocirrhosis

36. A baby was born at 36 weeks of gestation. Delivery was normal, by natural
way. The baby has a large cephalohematoma. The results of blood count are:
Hb- 120g/l, Er- 3,5×1012/l, total serum bilirubin - 123 mmol/l, direct
bilirubin - 11 mmol/l, indirect - 112 mmol/l. What are causes of
hyperbilirubinemia in this case?
A. Erythrocyte hemolysis
B. Bile condensing
C. Mechanical obstruction of the bile outflow
D. Intravascular hemolysis
E. Disturbance of the conjugative function of liver

37. A 4-month-old girl with blond hair and blue eyes has "mousy" odor of sweat
and urine, delayed psychomotoric development. The most typical laboratory
data for this disorder is:
A. Positive urine ferric chloride test
B. High concentration of chlorides in sweat
C. Low level of thyroid gland hormones in blood
D. High level of oxyproline in urine
E. High level of glycosaminoglycanes in urine

38. A neonate is 5 days old. What vaccination dose of BCG vaccine (in mg) is
necessary for vaccination of this child?
A. 0,05 mg
B. 0,2 mg
C. 0,075 mg
D. 0,025 mg
E. 0,1 mg

39. 7 y.o. boy with chronic sinusitis and rercurent pulmonary infections has chest
X-ray demonstrating a right-sided cardiac silhouette. What is the most likely
diagnosis?
A. Kartagener syndrome
B. Laryngotracheomalacia
C. alpha-antitrypsin deficiency
D. Cystic fibrosis (mucoviscidosis)
E. Bronchiolitis obliterans

40. A 2,9-kg term male infant is born to a mother who developed polyhydramnios
at 34 weeks gestation. At birth, the Apgar scores were 9 and 9. The infant
develops choking and cyanosis with the first feed. In addition, is unable to
place a nasogastric tube. What is the most likely diagnosis?
A. Esophageal atresia
B. Laryngomalacia
C. Choanal atresia
D. Tracheal atresia
E. Respiratory distress syndrome

41. Full term newborn has developed jaundice at 10 hours of age. Hemolytic
disease of newborn due to Rh-incompatibility was diagnosed. 2 hours later the
infant has indirect serum bilirubin level increasing up to 14 mmol/L. What is
most appropriate for treatment of hyperbilirubinemia in this infant?
A. Exchange blood transfusion
B. Phototherapy
C. Intestinal sorbents
D. Infusion therapy
E. Phenobarbital

42. A 4-year-old girl was playing with her toys and suddenly she got an attack of
cough, dyspnea. Objectively: respiration rate - 45/min, heart rate - 130/min.
Percussion revealed dullness of precatory sound on the right in the lower
parts. Auscultation revealed diminished breath sounds with bronchial
resonance on the right. X-ray picture showed shadowing of the lower part of
lungs on the right. Blood analysis revealed no signs of inflammation. The child
was diagnosed with foreign body in the right bronchus. What complication
caused such clinical presentations?
A. Atelectasis
B. Pneumothorax
C. Emphysema
D. Bronchitis
E. Pneumonia

43. A man, 42 years old, died in a road accident after the hemorrhage on the spot,
because of acute hemorrhagic anemia. What minimum percent of the whole
blood volume could result in death by acute hemorrhage?
A. 25-30%
B. 15-20%
C. 35-50%
D. 6-9%
E. 10-14%

44. A 6-week old child is admitted because of tachypnea. Birth had been
uneventful, although conjunctivitis developed on the third day of life and
lasted for about 2 weeks. Physical examination reveals tachypnea, bilateral
inspiratory crackles and single expiratory wheezing. Bilateral pneumonia is
evident on chest X-ray. The child is afebrile and has no history of fever. White
blood cell count is 15cdot109/l, with 28% of eosinophils. The most likely cause
of this child’s symptoms is:
A. Clamydia trachomanis
B. Visceral larva migrans
C. Varicella
D. Pneumocystis carinii
E. Mycoplasma pneumoniae
45. A 6 y.o. asthmatic child was taken to the emergency hospital because of severe
coughing and wheezing for the last 24 hours. Physical examination reveals
that the child is excitable, has intercostal and suprasternal retractions,
expiratory wheezing throughout all lung fields, RR- 60/min. Initial treatment
may include the prescription of:
A. Subcutaneous epinephrine
B. Parenteral gentamicyn
C. Intravenous fluids in the first 2 h to compensate water deficiency
D. Parenteral phenobarbital
E. N-acetyl cysteine and cromolyn by inhalation

46. A full term infant was born after a normal pregnancy, delivery, however, was
complicated by marginal placental detachment. At 12 hours of age the child,
although appearing to be in good health, passes a bloody meconium stool. For
determining the cause of the bleeding, which of the following diagnostic
procedures should be performed first?
A. Barium enema
B. Platelet count, prothrombin time, and partial thromboplastin time
C. Gastric lavage with normal saline
D. An Apt test
E. An upper gastrointestinal series

47. In the 43rd week of gestation a long, thin infant was delivered. He is apneic,
limp, pale, and covered with "pea soup" amniotic fluid. The first step in the
resuscitation of this infant at delivery should be:
A. Suction of the trachea under direct vision
B. Artificial ventilation with endotracheal tube
C. Catheterization of the umbilical vein
D. Administration of 100% oxygen by mask
E. Artificial ventilation with bag and mask

48. A newborn infant has mild cyanosis, diaphoresis, poor peripheral pule,
hepatomegaly and cardiomegaly. Respiratory rate is 60 breaths per minute,
and heart rate is 230 beats per minute. The child most likely has congestive
heart failure caused by:
A. Paroxysmal atrial tachycardia
B. Hypoplastic left heart syndrome
C. A large atrial septal defect and valvular pulmonary stenosis
D. A ventricular septal defect and transposition of the great vessels
E. Atrial flutter and partial atrioventricular block
49. A 6-year-old boy was brought to the emergency room with a 3-hour history of
fever up to 39,5°C and sore throat. The child looks alert, anxious and has a
mild inspiratory stridor. You should immediately:
A. Prepare to establish an airway
B. Order a chest x-ray and lateral view of the neck
C. Admit the child and place him in a mist tent
D. Examine the throat and obtain a culture
E. Obtain an arterial gas and start an IV line

50. A 7 d.o. boy is admitted to the hospital for evaluation of vomiting and
dehydration. Physical examination is otherwise normal except for minimal
hyperpigmentation of the nipples. Serum sodium and potassium
concentrations are 120 meq/L and 9 meq/L respectively. The most likely
diagnosis is:
A. Congenital adrenal hyperplasia
B. Hyperaldosteronism
C. Secondary hypothyroidism
D. Pyloric stenosis
E. Panhypopituitarism

51. A 7 y.o. boy has crampy abdominal pain and a rash on the back of his legs and
buttocks as well as on the extensor surfaces of his forearms. Laboratory
analysis reveals proteinuria and microhematuria. He is most likely to be
affected by:
A. Anaphylactoid purpura
B. Polyarteritis nodosa
C. Dermatomyositis
D. Systemic lupus erythematosus
E. Poststreptococcal glomerulonephritis

52. A 5-year-old boy was progressively getting worse compared to the previous 2
months. A chest x-ray has shown right middle lobe collapse. A tuberculin skin
test was strongly positive. What is the most characteristic finding in primary
tuberculosis?
A. Hilar or paratracheal lymph node enlargement
B. Hematogenous dissemination leading to extrapulmonary tuberculosis
C. Cavity formation
D. Atelectasis with obstructive pneumonia
E. Miliary tuberculosis

53. A girl is 12-year-old. Yesterday she was overcooled. Now she is complaining on
pain in suprapubic area, frequent painful urination by small portions,
temperature is 37,8°C. Pasternatsky symptom is negative. Urine analysis:
protein - 0,033 g/L, WBC- 20-25 in f/vis, RBC- 1-2 in f/vis. What diagnosis is
the most probable?
A. Acute cystitis
B. Urolithiasis
C. Acute glomerulonephritis
D. Dysmetabolic nephropathy
E. Acute pyelonephritis

54. The girl of 11 y.o. She is ill for 1 month. She has "butterfly"-type rash on face
(spots and papules), pain and swelling of small joints on arms and legs, signs
of stomatitis (small-sized ulcers in mouth). CBC: Hb– 80 g/L, RBC–
2,91012/L, WBC– 15109/L, ESR- 40 mm/hour. Urinalysis: protein– 0,33 g/L.
What is the most probable diagnosis?
A. Systemic lupus erythematosus
B. Acute rheumatic fever
C. Dermatomyositis
D. Juvenile rheumatoid arthritis, systemic type
E. Periarteriitis nodosa

55. An infant aged 1 year on the third day of common cold at night developed
inspiratory stridor, hoarse voice and barking cough. Physical examination
revealed suprasternal and intercostal chest retractions. There is a bluish skin
discoloration moistly seen over the upper lip. The respiratory rate is 52 per
min and pulse- 122 bpm. The body temperature is 37,50C. What disease does
the infant have?
A. Acute infectious croup due to viral laryngotracheitis
B. Acute laryngitis
C. Acute bronchiolitis with respiratory distress
D. Acute epiglottitis
E. Bronchopneumonia without complications

56. A newborn aged 3 days with hyperbilirubinemia (428 mkmol/L) developed


following disorders. From beginning there were severe jaundice with poor
suckling, hypotomia and hypodynamia. Little bit later periodical excitation,
neonatal convulsions and neonatal primitive reflexes loss are noted. Now
physical examination reveals convergent squint, rotatory nystagmus and
setting sun eye sign. How to explain this condition?
A. Encephalopathy due to hyperbilirubinemia
B. Brain tumour
C. Spastic cerebral palsy
D. Hydrocephalus
E. Skull injury
57. A child is 2 years old. The child complains of hoarse voice, dyspnea with
obstructed inspiration. The disease started 3 days ago from dry cough and
nose stuffiness. Objectively: general condition is unbalanced, stridor is
present. The childs skin is pale. Body temperature is 37,7°C. The palatine
arches are hyperemic. There is no deposit. Heart sounds are rhythmic.
Auscultation of lungs reveals rough breathing sounds, crepitation is absent.
Parainfluenza virus has been detected in nasopharynx lavage. What is the
most likely diagnosis?
A. Acute laryngotracheitis
B. Diphtheria
C. Laryngospasm
D. Epiglottitis
E. Foreign body

58. A 3-year-old child has been admitted to a hospital because of ostealgia and
body temperature rise up to 39°C. Objectively: the patient is in grave
condition, unable to stand for ostealgia, there is apparent intoxication, lymph
nodesare enlarged up to 1,5 cm. Liver can be palpated 3 cm below the costal
margin, spleen - 2 cm below the costal margin. In blood: RBCs - 3,0×1012/l,
Hb- 87 g/l, colour index - 0,9, thrombocytes - 190cdot109/l, WBCs -
3,2cdot109/l, eosinophils - 1, stab neutrophils - 1, segmented neutrophils - 0,
lymphocytes - 87, monocytes - 2, ESR - 36 mm/h. What examination should
be conducted in order to specify the diagnosis?
A. Sternal puncture
B. Computer tomography
C. Lymph node puncture
D. Ultrasound
E. Lymph node biopsy

59. Apgar test done on a newborn girl at 1st and 5th minute after birth gave the
result of 7-8 scores. During the delivery there was a short-term difficulty with
extraction of shoulder girdle. After birth the child had the proximal extremity
dysfunction and the arm couldnt be raised from the side. The shoulder was
turned inwards, the elbow was flexed, there was also forearm pronation,
obstetric palsy of brachial plexus. What is the clinical diagnosis?
A. Duchenne-Erb palsy
B. Trauma of thoracic spine
C. Intracranial haemorrhage
D. Trauma of right hand soft tissues
E. Right hand osteomyelitis

60. Examination of a 9-month-old girl revealed skin pallor, cyanosis during


excitement. Percussion revealed transverse dilatation of cardiac borders.
Auscultation revealed continuous systolic murmur to the left of the breastbone
in the 3-4 intercostal space. This murmur is conducted above the whole
cardiac region to the back. What congenital cardiac pathology can be
suspected?
A. Defect of interventricular septum
B. Defect of interatrial septum
C. Fallots tetrad
D. Pulmonary artery stenosis
E. Coarctation of aorta

61. A worker was temporarily off work because of illness for 16 days, was under
out-patient treatment. The doctor in charge issued a sick-list first for 5 days,
then prolonged it for 10 days. Who can further prolong the sick-list of this
patient?
A. The doctor in charge of the case together with the head of
department
B. Deputy head physician on the working ability expertise
C. The head of department
D. Working ability expertise committee
E. The doctor in charge of the case with the permission of the head of
department

62. A 13 y.o. patient was treated in dermatological hospital for atopic dermatitis
exacerbation. He was discharged in the condition of clinical remission. What
recommendations should the doctor give to prevent exacerbations?
A. Use of neutral creams to protect skin
B. Systematic skin disinfection
C. Avoidance of skin insolation
D. Frequent skin washing with detergents
E. Systematic use of local corticosteroids

63. On the 21 day after appearance of vesiculous chickenpox rash a 7-year-old


child developed ataxia, nystagmus, intention tremor, muscle hypotonia.
Liquor analysis shows a low-grade lymphocytic pleocytosis, slightly increased
protein rate. What complication is it?
A. Encephalitis
B. Postherpetic neuralgia
C. Pneumonitis
D. Purulent meningitis
E. Acute nephritis

64. An 8-year-old boy suffering from haemophilia was undergoing transfusion of


packed red cells. Suddenly he felt pain behind the breastbone and in the
lumbar area, dyspnea, cold sweat. Objectively: pale skin, heart rate - 100/min,
AP - 60/40 mm Hg; oliguria, brown urine. For the treatment of this
complication the following drug should be administered:
A. Prednisolone
B. Adrenaline
C. Lasix
D. Aminophylline
E. Analgine

65. A 3-year-old child has been diagnosed with type I diabetes mellitus,
hyperosmolar coma. The laboratory confirmed the diagnosis. Which
laboratory findings are characteristic for such condition?
A. High hyperglycemia without ketonemia
B. Hyperglycemia and glucosuria
C. Hyperglycemia and high indicators of acid-base balance
D. Hyperglycemia and ketonuria
E. Hyperglycemia and ketonemia

66. A 3-year-old child was playing in a playpen when he suddenly developed


paroxysmal cough and shortness of breath. Objectively: dry cough, mixed
dyspnea. Lung auscultation revealed some wheezes. Breathing sounds on the
right are diminished. The child doesnt mix with other children. Immunization
is age-appropriate. What pathological condition can be suspected?
A. Foreign body in the respiratory tracts
B. Pertussis
C. Bronchial asthma
D. Pneumonia
E. Acute respiratory viral infection

67. A 10-year-old child has been folowed-up for the dilated cardiomyopathy. The
child presents with dyspnea, cardialgia. There are dense, nonmobile edemata
on the lower extremities and sacrum. Ps- 120/min. The cardiac borders are
extended transversely. Heart sounds are muffled, there is blowing systolic
murmur at the apex and over the xiphoid process. Liver is 3 cm enlarged,
urine output is reduced. The blood total protein - 58.6 g/l. In urine: protein -
0,025 g/l, WBCs - 2-4 in the field of vision, RBCs - 2-3 in the field of vision.
What is the main mechanism of edema syndrome development:
A. Venous congestion of greater circulation
B. Peripheral circulation disorder
C. Venous congestion of lesser circulation
D. Secondary nephropathy development
E. Hypoproteinemia
68. After objective clinical examination a 12-year-old child was diagnosed with
mitral valve prolapse. What complementary instrumental method of
examination should be applied for the diagnosis confirmation?
A. Echocardiography
B. Veloergometry
C. Phonocardiography
D. Roentgenography of chest
E. ECG

69. A full-term child survived antenatal and intranatal hypoxia, it was born in
asphyxia (2-5 points on Apgar score). After birth the child has progressing
excitability, there are also vomiting, nystagmus, spasms, strabismus,
spontaneous Moros and Babinskys reflexes. What localization of intracranial
hemorrhage is the most probable?
A. Subarachnoid hemorrhage
B. Hemorrhages into the brain ventricles
C. Subdural hemorrhage
D. Small cerebral tissue hemorrhages
E. Periventricular hemorrhages

70. A 15 y.o. boy was twice attacked by bees, as a result he had severe anaphylactic
shock. What is the most effective prophylaxis method?
A. Desensibilisation by means of bee venom extract
B. Long-term prophylactic treatment with antihistamines
C. Protective clothing
D. Limitation of outside staying during summer months
E. Prescription of corticosteroids for summer

71. A 9-year-old boy has been suffering from bronchoectasis since he was 3.
Exacerbations occur quite often, 3-4 times a year. Conservative therapy results
in short periods of remission. The disease is progressing, the child has
physical retardation. The childs skin is pale, acrocyanotic, he has "watch glass"
nail deformation. Bronchography revealed saccular bronchiectases of the
lower lobe of his right lung. What is the further treatment tactics?
A. Surgical treatment
B. Physiotherapeutic treatment
C. Further conservative therapy
D. Sanatorium-and-spa treatment
E. Tempering of the childs organism

72. A child with tetralogy of Fallot is most likely to exhibit:


A. Increased pressure in the right ventricle
B. Normal oxygen tension (PaO2) in the left ventricle
C. Increased pulse pressure
D. Increased pulmonary blood flow
E. Normal pressure gradient across the pulmonary valve

73. A 2-months-old child after preventive vaccination had a prolonged


hemorrhage from the vaccination place and due to those an intramuscular
hematoma. During examination of the child a considerable rise of
prothrombin consumption and a significant prolongation of the activated
partial thromboplastic time were found. What is the most probable diagnosis?
A. Hemophilia
B. Inborn afibrinogenemia
C. Henoch-Schoenlein disease
D. Werlhofs disease
E. Hemorrhagic disease of the neonate

74. A 10 y.o. boy with hemophilia has signs of acute respiratory viral infection
with fever. What of the mentioned antifebrile medications are contraindicated
to this patient?
A. Acetylsalicylic acid
B. Pipolphen
C. Panadol extra
D. Paracetamol
E. Analgin

75. A 7-year-old child is sick for 2 weeks with running nose, was taking nasal
drops. The boy suffers with alimentary allergy. He applied to doctor due to
suppurative and bloody discharges from nose, maceration of ala nasi and
upper lip. Rhinoscopy results: there are whitish-greyish areas at nasal septum.
Mucous membrane of oropharynx is not changed. What is the most probable
disease?
A. Diphtheria of the nose
B. Sinusitis (maxillar sinus)
C. Rhinovirus
D. Adenovirus
E. Allergic rhinitis

76. A 10-year-old boy underwent treatment in cardiological department for


rheumatism, I acute attack of rheumatic fever, active phase, II degree. The
patient was discharged in satisfactory condition. Which drug should be chosen
for prevention of rheumatism recurrence?
A. Bicillinum-5
B. Ampicillin
C. Oxacillin
D. Bicillinum-1
E. Erythromycin

77. A child is 4 years old, has been ill for 5 days. There are complaints of cough,
skin rash, to- 38,2°C, face puffiness, photophobia, conjunctivitis. Objectively:
there is bright, maculo-papulous, in some areas confluent rash on the face,
neck, upper chest. The pharynx is hyperemic. There are seropurulent
discharges from the nose. Auscultation revealed dry rales in lungs. What is the
most likely diagnosis?
A. Measles
B. Enterovirus exanthema
C. Scarlet fever
D. Adenoviral infection
E. Rubella

78. A 10 month old boy has been ill for 5 days after consumption of unboiled milk.
Body temperature is 38-39°C, there is vomiting, liquid stool. The child is pale
and inert. His tongue is covered with white deposition. Heart sounds are
muffled. Abdomen is swollen, there is borborygmus in the region of ubbilicus,
liver is enlarged by 3 cm. Stool is liquid, dark-green, with admixtures of
mucus, 5 times a day. What is the most probable diagnosis?
A. Salmonellosis
B. Staphylococcal enteric infection
C. Acute shigellosis
D. Rotaviral infection
E. Escherichiosis

79. A 3 year old child with weight deficiency suffers from permanent moist cough.
In history there are some pneumonias with obstruction. On examination:
distended chest, dullness on percussion over the lower parts of lungs. On
auscultation: a great number of different rales. Level of sweat chloride is 80
millimol/l. What is the most probable diagnosis?
A. Mucoviscidosis (cystic fibrosis)
B. Bronchial asthma
C. Bronchiectasis
D. Pulmonary hypoplasia
E. Recurrent bronchitis

80.A 12 y.o. child with acute glomerulonephritis presented with hypertensive


syndrom during first days of the disease. What is the role of angiotesin II in
the pathogenesis?
A. Intensifies production and secretion of aldosterone
B. Infibits deppresive action of prostaglandins
C. Increases heart output
D. Increases erythropoetin production
E. Increases renine level

81. A full-term infant is 3 days old. On the different parts of skin there are
erythemas, erosive spots, cracks, areas of epidermis peeling. The infant has
scalded skin syndrome. Nikolskys symptom is positive. General condition of
the infant is grave. Anxiety, hyperesthesia, febrile temperature are evident.
What is the most probable diagnosis?
A. Exfoliative dermatitis
B. Fingers pseudofurunculosis
C. Mycotic erythema
D. Impetigo neonatorum
E. Phlegmon of newborn

82. District pediatrician examines a healthy carried 1-month-old child. The child
is breast-fed. Prophylaxis of what disease will the doctor recommend to do
first?
A. Rachitis
B. Anemia
C. Spasmophilia
D. Parathropy
E. Hypotrophia

83. A 7-year-old boy has been managed for a month. Immediately after
hospitalization there were apparent edemata, proteinuria - 7,1 g/l, daily urine
protein - 4,2 g. Biochemical blood test shows persistent hypoproteinemia
(43,2 g/l), hypercholesterolemia (9,2 millimole/l). The patient is most likely
have the following type of glomerulonephritis:
A. Nephrotic
B. Combined
C. Isolated urinary
D. Nephritic
E. Hematuric

84. A 3 y.o. girl has had a temperature rise up to 38°C, rhinitis, dry superficial
cough, flabbiness, appetite loss. Palpation didnt reveal any changes over her
lungs. Percussion sound has a wooden resonance, auscultation revealed
puerile breathing, no rales. In blood: leukopenia, lymphocytosis, increased
ESR. What is the most probable diagnosis?
A. Acute simple tracheitis
B. Recurrent bronchitis, acute condition
C. Bilateral microfocal pneumonia
D. Acute simple bronchitis
E. Acute obstructive bronchitis

85. A 5-year-old girl with the transitory immunodeficiency according to T-system


has a clinical picture of a right-sided pneumonia during 2 months. How
pneumonia progress can be described?
A. Delaying
B. Acute
C. Chronic
D. Recidivating
E. Wavelike

86. A 12 y.o. girl took 2 pills of aspirine and 4 hours later her body temperature
raised up to 39-40°C. She complains of general indisposition, dizziness,
sudden rash in form of red spots and blisters. Objectively: skin lesions
resemble of second-degree burns, here and there with erosive surface or
epidermis peeling. Nikolskys symptom is positive. What is the most probable
diagnosis?
A. Acute epidermal necrolisis
B. Polymorphous exudative erythema
C. Duhrings disease
D. Bullous dermatitis
E. Pemphigus vulgaris

87. A 5-year-old child had an attack of palpitation with nausea, dizziness,


generalized fatigue. On ECG: tachycardia with heartbeat rate of 220/min.
Ventricle complexes are deformed and widened. P wave is absent. What
medication is to be prescribed to provide first aid?
A. Lydocain
B. Novocainamides
C. Strophantin
D. Isoptin
E. Seduxen

88. Examination of a 4 month old child revealed some lemon-yellow squamae


with fatty crusts on the scalp. What is the most probable diagnosis?
A. Gneiss
B. Pseudofurunculosis
C. Infantile eczema
D. Milk crust
E. Strophulus
89. A neonate from gestation with severe gestosis of the second half was born on
the 41st week with 2400 g birth weight and 50cm long. On physical
examination: skin is flaccid, subcutaneous fatty cellular tissue is thin, muscle
hypotonia, new-born period reflexes are decreased. Internal organs are
without pathological changes. How would you estimate this child?
A. Term infant with pre-natal growth retardation
B. Immature infant
C. Premature infant
D. Postmature infant
E. Term infant with normal body weight

90. A child was taken to a hospital with focal changes in the skin folds. The child
was anxious during examination, examination revealed dry skin with solitary
papulous elements and ill-defined lichenification zones. Skin eruption was
accompanied by strong itch. The child usually feels better in summer, his
condition is getting worse in winter. The child has been artificially fed since he
was 2 months old. He has a history of exudative diathesis. Grandmother by his
mothers side has bronchial asthma. What is the most likely diagnosis?
A. Atopic dermatitis
B. Contact dermatitis
C. Strophulus
D. Urticaria
E. Seborrheal eczema

91. A boy, aged 9, was examined: height - 127 cm (-0,36), weight - 28,2 kg
(+0,96), chest circumference - 64,9 cm (+0,66), lung vital capacity - 1520 ml
(-0,16). What is the complex assessment of the childs physical development?
A. Harmonious
B. Apparently disharmonious
C. Below the average
D. Excessive
E. Disharmonious

92. A child is 7 months old. Birth weight was 3450, the child is breastfed.
Supplemental feeding was introduced on time. Determine the daily protein
requirements for the child:
A. 3,0 g/kg
B. 2,0 g/kg
C. 3,5 g/kg
D. 4,0 g/kg
E. 2,5 g/kg
93. 2 weeks after recovering from angina an 8-year-old boy developed edemata of
face and lower limbs. Objectively: the patient is in grave condition, AP-
120/80 mm Hg. Urine is of dark brown colour. Oliguria is present. On urine
analysis: relative density - 1,015, protein - 1,2 g/l, RBCs are leached and cover
the whole vision field, granular casts - 1-2 in the vision field, salts are
represented by urates (big number). What is the most likely diagnosis?
A. Acute glomerulonephritis with nephritic syndrome
B. Acute glomerulonephritis with nephrotic syndrome, hematuria and
hypertension
C. Acute glomerulonephritis with nephrotic syndrome
D. Acute glomerulonephritis with isolated urinary syndrome
E. Nephrolithiasis

94. A 14 year old child suffers from vegetovascular dystonia of pubertal period. He
has got sympathoadrenal atack. What medicine should be used for attack
reduction?
A. Obsidan
B. No-shpa
C. Aminophylline
D. Corglicone
E. Amysyl

95. A child is 9 months old. The patients body temperature is 36,7°C, the skin is
pale, humid, there is pain in leg muscles. There is no extremities mobility,
sensitivity is present. The child has been diagnosed with poliomyelitis. The
causative agent of this disease relates to the following family:
A. Picornavirus
B. Paramyxovirus
C. Adenovirus
D. Rotavirus
E. Tohovirus

96. A 4 month old child fell seriously ill: body temperature rose up to 38,5°C, the
child became inert and had a single vomiting. 10 hours later there appeared
rash over the buttocks and lower limbs in form of petechiae, spots and
papules. Some haemorrhagic elements have necrosis in the centre. What is the
most probable disease?
A. Meningococcemia
B. Influenza
C. Scarlet fever
D. Haemorrhagic vasculitis
E. Rubella
97. A 5-year-old child had strong headache, vomiting, ataxy, dormancy,
discoordination of movements, tremor of the extremities on the 8th day of the
disease. It was followed by rise in body temperature, vesiculosis rash mainly
on the skin of the body and the hairy part of the head. At the second wave of
the fever a diagnosis of encephalitis was given. What disease complicated
encephalitis in this case?
A. Chicken pox
B. Herpetic infection
C. German measles
D. Measles
E. Enterovirus infection

98. A 13 year old girl was admitted to the cardiological department because of
pain in the muscles and joints. Examination of her face revealed an edematic
erythema in form of butterfly in the region of nose bridge and cheeks. What is
the most probable diagnosis?
A. Systemic lupus erythematosus
B. Dermatomyositis
C. Periarteritis nodosa
D. Rheumatoid arthritis
E. Rheumatism

99. A 4 y.o. boy was admitted to the hospital with complaints of dyspnea, rapid
fatigability. His anamnesis registers frequent respiratory diseases. On
percussion: heart borders are dilatated to the left and upwards. On
auscultation: amplification of the SII above pulmonary artery, a harsh
systolodyastolic "machine" murmur is auscultated between the II and the III
rib to the left of breast bone, this murmur is conducted to all other points
including back. AP is 100/20 mm Hg. What is the most probable diagnosis?
A. Opened arterial duct
B. Interatrial septal defect
C. Valvar aortic stenosis
D. Interventricular septal defect
E. Isolated stenosis of pulmonary arterial orifice

100. A 12 year old girl complains about abrupt weakness, nausea, dizziness,
vision impairment. The day before she ate home-made stockfish, beef.
Examination revealed skin pallor, a scratch on the left knee, dryness of
mucous membranes of oral pharynx, bilateral ptosis, mydriatic pupils. The girl
is unable to read a simple text (mist over the eyes). What therapy would be the
most adequate in this case?
A. Parenteral introduction of polyvalent antibotulinic serum
B. Gastric lavage
C. Parenteral introduction of antitetanus serum
D. Parenteral disintoxication
E. Parenteral introduction of antibiotics

101. A child from the first non-complicated pregnancy but complicated


labor had cephalhematoma. On the second day there developed jaundice. On
the 3th day appeared changes of neurologic status: nystagmus, Graefes sign.
Urea is yellow, feces- golden-yellow. Mothers blood group is A(II)Rh-, child-
A(II)Rh+. On the third day childs Hb- 200 g/L, RBC- 6,1*10^12/L, bilirubin
in blood - 58 mk mol/L due to unconjugated bilirubin, Ht- 0,57. What is the
childs jaundice explanation?
A. Brain delivery trauma
B. Hemolytic disease of newborn
C. Fetal hepatitis
D. Bile ducts atresia
E. Physiologic jaundice

102. A full-term baby (the 1st uncomplicated pregnancy, difficult labour)


had a cephalogematoma. On the 2nd day there was jaundice, on the third the
following changes in neurological status appeared: nystagmus, Graefe
syndrome. Urine was yellow, feces were of golden-yellow colour. Mothers
blood group is A (II) Rh-, the babys one - A (II) Rh+. On the third day the
childs Hb was 200g/l, RBCs - 6,1×1012/l, blood bilirubin - 58 micromole/l at
the expense of unbound fraction. What caused the jaundice in the child?
A. Craniocerebral birth trauma
B. Biliary atresia
C. Fetal hepatitis
D. Physiological jaundice
E. Neonatal anaemia

103. After birth a child was pale and had arrhythmical breathing. Oxygen
therapy didnt have any effect. Pulse was weak and rapid. It was difficult to
measure arterial pressure accurately. There were no edemata. What is the
most likely reason for these symptoms?
A. Asphyxia
B. Intracranial haematoma
C. Congenital pneumonia
D. Intrauterine sepsis
E. Congestive heart failure

104. A child was delivered severely premature. After the birth the child has
RI symptoms, anasarca, fine bubbling moist rales over the lower lobe of the
right lung. Multiple skin extravasations, bloody foam from the mouth have
occured after the 2 day. On chest X-ray: atelectasis of the lower lobe of the
right lung. In blood: Hb-100 g/L, Ht- 0,45. What is the most probable
diagnosis?
A. Edematous-hemorrhagic syndrome
B. Disseminated intravascular clotting syndrome
C. Hyaline membrane disease
D. Congenital pneumonia
E. Pulmonary edema

105. An infant is 2 d.o. It was full-term born with signs of intrauterine


infection, thats why it was prescribed antibiotics. Specify, why the gap
between antibiotic introductions to the new-born children is longer and
dosage is smaller compared to the older children and adults?
A. The newborns have a lower level of glomerular filtration
B. The newborns have bigger hematocrit
C. The newborns have reduced activity of glucuronil transferase
D. The newborns have lower concentration of protein and albumins in blood
E. The newborns have diminished blood pH

106. A 10-year-old child is sick with chronic viral hepatitis B with marked
activity of the process. Total bilirubin - 70mumol/L, direct - 26mumol/L,
indirect - 44mumol/L. AST - 6,2 mmol/L,ALT- 4,8 mmol/L. What mechanism
underlies the transaminase level increase of this patient?
A. Cytolysis of hepatocytes
B. Failure of the synthetical function of the liver
C. Intrahepatic cholestasis
D. Failure of bilirubin conjugation
E. Hypersplenism

107. A 12-year-old girl applied to doctor with complaints of swelling on the


front part of the neck. The doctor diagnosed hyperplasia of the thyroid gland
of the second degree, euthyroidism. Ultrasound suspected autoimmune
thyroiditis. Blood was taken for titre of antibodies to thyroglobulin. What titre
of antibodies will be diagnostically important?
A. 1:100
B. 1:200
C. 1:250
D. 1:50
E. 1:150

108. A 14-year-old girl has been presenting with irritability and tearfulness
for about a year. A year ago she was also found to have diffuse enlargement of
the thyroid gland (II grade). This condition was regarded as a pubertal
manifestation, the girl didnt undergo any treatment. The girls irritability
gradually gave place to a complete apathy. The girl got puffy face, soft tissues
pastosity, bradycardia, constipations. Skin pallor and gland density
progressed, the skin became of a waxen hue. What disease may be suspected?
A. Autoimmune thyroiditis
B. Thyroid carcinoma
C. Diffuse toxic goiter
D. Subacute thyroiditis
E. Juvenile basophilism

109. In the anamnesis of a 2-year-old girl there are recurrent pneumonias


with signs of obstruction. There are heterogeneous moist and dry rales,
respiration is weakened. Dense, viscous secretion is difficult to hawk. There
are "drumsticks", physical retardation. What is the most probable diagnosis?
A. Mucoviscidosis, pulmonary form
B. Bronchial asthma
C. Recidivating bronchitis
D. Congenital pulmonary polycystosis
E. Pulmonary tuberculosis

110. On the 3rd day of life a baby presented with haemorrhagic rash, bloody
vomit, black stool. Examination revealed anaemia, extended coagulation time,
hypoprothrombinemia, normal thrombocyte rate. What is the optimal
therapeutic tactics?
A. Vitamin K
B. Epsilon-aminocapronic acid
C. Sodium ethamsylate
D. Fibrinogen
E. Calcium gluconate

111. A 2 month old full-term child was born with weight 3500 g and was on
the mixed feeding. Current weight is 4900 g. Evaluate the current weight of
the child:
A. Corresponding to the age
B. Paratrophy of the I grade
C. Hypotrophy of the I grade
D. 150 g less than necessary
E. Hypotrophy of the II grade

112. A 2 m.o. breast-fed child suffers from cheek skin hyperemia, sporadic
papulous elements on the skin of the chest and back following the apple juice
introduction. The child is restless. What is the initial pediatritians tactics?
A. Clarify mothers diet and exlude obligate allergens
B. Apply ointment with corticosteroids to affected skin areas
C. Administer general ultraviolet irradiation
D. Refer to prescribe dermathologist
E. Treat with claritine

113. A 5 month old boy was born prematurely, he didnt suffer from any
disease at the infant age and later on. Examination at an outpatients hospital
revealed paleness of skin, sleepiness. Blood count: Hb - 95 g/l, erythrocytes -
3,5×1012/l, reticulocytes - 90/_00, colour index - 0,7, osmotic stability of
erythrocytes - 0,44-0,33%, serum iron - 4,9 micromole/l. What is the most
probable cause of anemia?
A. Iron deficit
B. B12 deficit
C. Infectious process
D. Hemogenesis immaturity
E. Erythrocyte hemolysis

114. A 7 y.o. child had elevation of temperature tol 400C in anamnesis. For
the last 3 months he presents fusiform swelling of fingers, ankle joints and
knee joint, pain in the upper part of the sternum and cervical part of the spinal
column. What is the most probable diagnosis?
A. Juvenile rheumatic arthritis
B. Septic arthritis
C. Osteoarthrits
D. Rheumatism
E. Toxic synovitis

115. An 8 year old girl complains about joint pain, temperature rise up to
38°C, dyspnea. Objectively: the left cardiac border is deviated by 2,5 cm to the
left, tachycardia, systolic murmur on the apex and in the V point are present.
Blood count: leukocytes - 20,0cdot109/l, ESR - 18 mm/h. What sign gives the
most substantial proof for rheumatism diagnosis?
A. Carditis
B. Leukocytosis
C. Accelerated ESR
D. Fever
E. Arthralgia

116. A 5 y.o. child with stigmas of dysembryogenesis (small chin, thick lips,
opened mouth, hyperthelorismus) has systolic murmur in the second
intercostal to the right of the sternum. The murmur passes to the neck and
along the sternum left edge. The pulse on the left brachial artery is weakened.
BP on the right arm is 110/60 mm Hg, on the left - 100/60 mm Hg. ECG
results: hypertrophy of the right ventricle. What defect is the most probable?
A. Aortic stenosis
B. Coarctation of the aorta
C. Open aortic duct
D. Defect of interventricular septum
E. Defect of interatrial septum

117. A 1,5-year-old child fell ill acutely with high temperature 38°C,
headache, fatigue. The temperature declined on the fifth day, muscular pain in
the right leg occured in the morning, there were no movements and tendon
reflexes, sensitivity was reserved. What is the initial diagnosis?
A. Polyomyelitis
B. Polyartropathy
C. Viral encephilitis
D. Osteomyelitis
E. Hip joint arthritis

118. A 3-year-old child has been delivered to a hospital in soporose state


with considerable amyotonia, inhibition of tendon and periosteal reflexes.
Miosis and asthenocoria are also present. Corneal reflexes are preserved.
Pulse is rapid and weak. AP- 80/50 mm Hg. The parents suspect the child of
accidental taking some tablets. Such clinical presentations are typical for
intoxication with the following tableted drugs:
A. Tranquilizers
B. Antihypertensive drugs
C. Antropine drugs
D. Barbiturates
E. Beta-2-adrenoceptor agonists

119. A 2 m.o. child with birth weight 5100 g has jaundice, hoarse cry,
umbilical hernia, physical development lag. Liver is +2 cm enlarged, spleen is
not enlarged. In anamnesis: delayed falling-away of umbilical cord rest. In
blood: Hb- 120 g/L, erythrocytes - 4,5x1012/L, ESR- 3 mm/h. Whole serum
bilirubin is 28 mcmole/L, indirect - 20 mcmole/L, direct - 8 mcmole/L. What
is the most probable diagnosis?
A. Congenital hypothyreosis
B. Congenital hepatitis
C. Conjugated jaundice
D. Cytomegalovirus infection
E. Hemolitic anemia

120. A 5-year-old child developed an acute disease starting from body


temperature rise up to 38,5°C, running nose, cough and conjunctivitis. On the
4th day the child presented with maculo-papular rash on face. Body
temparature rose again up to 39,2°C. Over the next few days the rash spread
over the whole body and extremities. Mucous membrane of palate was
hyperemic, there was whitish deposition on cheek mucous membrane next to
molars. What is your provisional diagnosis?
A. Measles
B. Enterovirus diseases
C. Rubella
D. Acute viral respiratory infection
E. Yersinia

121. A 3 year old child fell acutely ill, body temperature rose up to 39,5°C,
the child became inert, there appeared recurrent vomiting, headache.
Examination revealed positive meningeal symptoms, after this lumbal
puncture was performed. Spinal fluid is turbid, runs out under pressure,
protein concentration is 1,8 g/l; Pandy reaction is +++, sugar concentration is
2,2 millimole/l, chloride concentration - 123 millimole/l, cytosis is
2,35cdot109 (80% of neutrophils, 20% of lymphocytes). What is the most
probable diagnosis?
A. Purulent meningitis
B. Subarachnoid haemorrhage
C. Brain tumour
D. Serous viral meningitis
E. Serous tuberculous meningitis

122. A 13 y.o. girl complains of having temperature rises up to febrile figures


for a month, joint ache, periodical skin rash. Examination revealed steady
enhancing of ESR, LE-cells. What is the most probable diagnosis?
A. Systematic lupus erythematosus
B. Acute lymphoblast leukosis
C. Rheumatics
D. Juvenile rheumatoid arthritis
E. Systematic scleroderma

123. A child is 1 year old. Ater the recent introduction of complementary


feeding the child has presented with loss of appetite, diarrhea with large
amounts of feces and occasional vomiting, body temperature is normal.
Objectively: body weight is 7 kg, the child is very pale, there are edemata of
both legs, abdomen is significantly enlarged. Coprogram shows many fatty
acids and soaps. The child has been diagnosed with celiac disease and
administered the gluten-free diet. What is to be excluded from the ration?
A. Cereals - wheat and oats
B. Milk and dairy products
C. Animal protein
D. High digestible carbohydrates
E. Fruit

124. A 7-year-old child was brought to a doctor for a check. The child has a
4-year history of bronchial asthma, asthma attacks occur mainly in spring and
summer. Allergy tests revealed hypersensitivity to poplar seed tufts, field
herbs. What recommendation should be given?
A. Specific hyposensitization
B. Treatment at a health resort
C. Needle reflexotherapy
D. Phytotherapy
E. Physiotherapy

125. A 9-month-old child presents with fever, cough, dyspnea. The


symptoms appeared 5 days ago after a contact with a person having ARVI.
Objectively: the child is in grave condition. Temperature of 38°C, cyanosis of
nasolabial triangle is present. RR- 54/min, nasal flaring while breathing.
There was percussion dullness on the right below the scapula angle, and
tympanic sound over the rest of lungs. Auscultation revealed bilateral fine
moist rales predominating on the right. What is the most likely diagnosis?
A. Acute pneumonia
B. Acute laryngotracheitis
C. Acute bronchiolitis
D. Acute bronchitis
E. ARVI

126. An 8 y.o. boy complains of constant cough along with discharge of


greenish sputum, dyspnea during physical activities. At the age of 1 year and 8
months he fell ill for the first time with bilateral pneumonia that had
protracted course. Later on there were recurrences of the disease 5-6 times a
year, during the remission periods there was constant productive cough. What
examination results will be the most important for making a final diagnosis?
A. Bronchography
B. Bronchoscopy
C. Spirography
D. Roentgenography of thorax organs
E. Bacterial inoculation of sputum

127. A mother of a 5 y.o. girl consulted a doctor about doughters involuntary


urination at night, nightmares, sleep disorders, slow gaining of body weight.
Objectively: malnutrition, intellectual development is good, the girl can read
and explains common situations quite adultly. Her skin is very pale, liver is
enlarged in size. Her mother suffers from holetithiasis. What type of diathesis
is the most probable in the childs case?
A. Gouty diathesis
B. Exudative diathesis
C. Urine acid diathesis
D. Allergic diathesis
E. Lymphohypoplastic diathesis

128. A 10 year old girl complains about abdominal pain that is arising and
getting worse after eating rough or spicy food. She complains also about sour
eructation, heartburn, frequent constipations, headache, irritability. She has
been suffering from this for 12 months. Objectively: the girls diet is adequate.
Tongue is moist with white deposit at the root. Abdomen is soft, painful in its
epigastric part. What study method will help to make a diagnosis?
A. Esophagogastroduodenoscopy
B. Fractional examination of gastric juice
C. Intragastral pH-metry
D. Contrast roentgenoscopy
E. Biochemical blood analysis

129. A 40 h.o. child age has hyperosthesia, CNS depression, dyspepsia.


Sepsis is suspected. What should the differential diagnosis be made with?
A. Hypoglycemia
B. Hyperbilirubinemia
C. Hypomagnesemia
D. Hyperkaliemia
E. Hypocalcemia

130. A 1,5 y.o. child fell seriously ill: chill, body temperature rise up to
40,1°C, then rapid dropping to 36,2°C, skin is covered with voluminous
hemorrhagic rash and purple cyanotic spots. Extremities are cold, face
features are sharpened. Diagnosis: meningococcosis, fulminant form,
infection-toxic shock. What antibiotic must be used at the pre-admission
stage?
A. Soluble Levomycetine succinate
B. Sulfamonometoxin
C. Lincomycin
D. Penicillin
E. Gentamycin

131. A 10 year old boy suffers from chronic viral hepatitis type B with
maximal activity. What laboratory test can give the most precise characteristic
of cytolysis degree?
A. Transaminase test
B. Takata-Ara test
C. Weltmans coagulation test
D. Prothrombin test
E. Test for whole protein

132. A 6 y.o child complains of thirst, polyuria, increased appetite for 2


months with weight loss for 3 kg. There has been nocturnal enuresis during
last week. On examination: hyperglycemia 14 mol/L. The diagnosis is diabetis
mellitus I type. What is the genesis of this disease?
A. Autoimmune
B. Bacterial
C. Viral
D. Neurogenic
E. Virus-bacterial

133. A 10 y.o. child who is at oligoanuretic stage of acute renal insufficiency


has got sensations of pricking in the mucous membrane of oral cavity and
tongue, extremities numbness, reduced reflexes, respiratory disturbance,
arrhythmia. What are these symptoms caused by?
A. Hyperkaliemia
B. Alkalosis
C. Hyperazotemia
D. Hyponatremia
E. Acidosis

134. Examination of a 12 year old child revealed diffuse thyroid enlargement


of the II degree. Heart auscultation revealed dullness of heart sounds, heart
rate was 64/min. The child has frequent constipations, anemia. Concentration
of thyreoglobulin antibodies is increased. What disease might have caused
such symptoms?
A. Autoimmune thyroiditis
B. Diffuse toxic goiter
C. Thyroid hyperplasia
D. Endemic goiter
E. Thyroid carcinoma

135. An 8-year-old girl has been admitted to the cardiology department.


Objectively: there is a skin lesion over the extensor surfaces of joints with
atrophic cicatrices, depigmentation, symmetrical affection of skeletal muscles
(weakness, edema, hypotrophy). What disease are these changes most typical
for?
A. Dermatomyositis
B. Systemic lupus erythematosus
C. Reiters disease
D. Systemic scleroderma
E. Nodular periarteritis

136. A 13 y.o. teenager who suffers from hemophilia A was taken to the
hospital after a fight at school. His diagnosis is right-sided hemarthros of knee
joint, retroperitoneal hematoma. What should be primarily prescribed?
A. Fresh frozen plasma
B. Washed thrombocytes
C. Dry plasma
D. Placental albumin
E. Aminocapronic acid

137. A 3 m.o. child fell seriously ill, body temperature rised up to 37,80C,
there is semicough. On the 3-rd day the cough grew worse, dyspnea appeared.
On percussion: tympanic sound above lungs, on auscultation: a lot of fine
moist and wheezing rales during expiration. What is the most probable
diagnosis?
A. Acute respiratory viral infection, bronchiolitis
B. Acute respiratory viral infection, bronchitis
C. Acute respiratory viral infection, focal pneumonia
D. Acute respiratory viral infection, bronchitis with asthmatic component
E. Acute respiratory viral infection, bronchopneumonia

138. On the 1st day of life a full-term girl (2nd labour) weighing 3500g, with
Apgar score of 8 points, presented with jaundice. Indirect bilirubin of blood -
was 80 micromole/l, 6 hours later - 160 micromole/l. What is the optimal
method of treatment?
A. Exchange blood transfusion
B. Enterosorbents
C. Infusion therapy
D. Phototherapy
E. Phenobarbital treatment

139. A child was born at a gestational age of 34 weeks in grave condition.


The leading symptoms were respiratory distress symptoms, namely sonorous
and prolonged expiration, involving additional muscles into respiratory
process. The Silverman score at birth was 0 points, in 3 hours it was 3 points
with clinical findings. Which diagnostic study will allow to diagnose the form
of pneumopathy?
A. X-ray of chest
B. Clinical blood test
C. Proteinogram
D. Immunoassay
E. Determination of blood gas composition

140. A 10-year-old girl consulted a doctor about thirst, frequent urination,


weight loss. She has been observing these symptoms for about a month.
Objectively: no pathology of internal organs was revealed. What laboratory
analysis should be carried out in the first place?
A. Blood glucose analysis on an empty stomach
B. Glucosuric profile
C. Acetone in urine test
D. Glucose in urine test on the base of daily diuresis
E. Glucose tolerance test

141. A 6-year-old child complains of frequent liquid stool and vomiting. On


the 2nd day of desease the child presented with inertness, temperature rise up
to 38,2°C, Ps- 150 bpm, scaphoid abdomen, palpatory painful sigmoid colon,
defecation 10 times a day with liquid, scarce stool with mucus and streaks of
green. What is a provisional diagnosis?
A. Shigellosis
B. Intestinal amebiasis
C. Yersiniosis
D. Salmonellosis
E. Escherichiosis

142. After a 10-year-old child had been bitten by a bee, he was delivered to a
hospital. There were lip, face and neck edemata. The patient felt hot and short
of breath. Objectively: breathing was laboured and noisy. There were foamy
discharges from the mouth, cough. The skin was pale and cold. There was
bradypnoea. Heart sounds were muffled and arrhythmic. Thready pulse was
present. What diagnosis was made by the expert in resuscitation?
A. Anaphylactic shock
B. Bronchial asthma
C. Quinckes edema
D. Acute cardiovascular collapse
E. Cerebral coma

143. A 13-year-old girl complains of fever up to 37,4°C during the last 2


months after recovering from ARVI. Objectively: malnutrition, diffuse grade II
enlargement of the thyroid gland feeling dense on palpation, exophthalmos,
tachycardia. What kind of pathological syndrome is it?
A. Thyrotoxicosis
B. Thymomegaly
C. Hypoparathyroidism
D. Hypothyroidism
E. Hyperparathyroidism

144. A 3-year-old girl presents with pertussis-like cough with thick sputum.
There have been persistent changes in lungs since the age of 6 months when
she was first diagnosed with acute pneumonia. Chloride concentration in the
perspiration is 112 mEq/l. The child has been diagnosed with mucoviscidosis.
What is the basis for autosomal recessive disease - mucoviscidosis?
A. Inadequate transport of sodium and chloride ions
B. Pulmonary artery hypoplasia
C. Deposition of calcium triphosphates and carbotates in the alveoles
D. alpha_1-antitrypsin deficiency
E. Pulmonary cysts

145. From urine of a 14-year-old boy with the exacerbation of secondary


obstructive pyelonephritis Pseudomonas aeruginosa was isolated with a titer
of 1000000 microbes per 1 ml. Which antibiotic is most advisable to be
administered in this case?
A. Ciprofloxacin
B. Cefazolinum
C. Chloramphenicol
D. Azithromycin
E. Ampicillin

146. A 14-year-old boy with a history of chronic tonsillitis and sinusitis has
developed a feeling of heart irregularities and additional pulse. HR- 83/min.
ECG results: regular impulses with no visible P wave that occur every two
sinus contractions, QRS complex is dramatically deformed and prolonged to
over 0,11 s, T wave is discordant followed by a complete compensatory pause.
Specify the arrhythmia type:
A. Trigeminal extrasystole
B. Partial AV-blockade
C. Left bundle branch block
D. Complete AV-block
E. Bigeminal extrasystole

147. An 8-year-old girl periodically has sudden short-term heart pain,


sensation of chest compression, epigastric pain, dizziness, vomiting.
Objectively: the patient is pale, respiratory rate - 40/min, jugular pulse is
present. Ps- 185 bpm, of poor volume. AP- 75/40 mm Hg. ECG taken during
an attack shows ectopic P waves, QRS wave is not deformed. At the end of an
attack a compensatory pause is observed. The most likely cause of the attack
is:
A. Paroxysmal atrial tachycardia
B. Sinus tachycardia
C. Complete AV-block
D. Atrial fibrillation
E. Paroxysmal ventricular tachycardia

148. A 1-month-old child became restless and presented with an increase in


head sweating. Its known from the history that the child has been fed with
cows milk since birth (September 5). Examination revealed craniotabes. A
doctor administered a course of UV radiation. Decide, if the child needs
ergocalciferol:
A. 2-2,5 months after the UVR withdrawal
B. In combination with UVR
C. A month after the UVR withdrawal
D. Immediately after the UVR withdrawal
E. Does not need

149. 15 minutes after the second vaccination with DTP vaccine a 4-month-
old boy exhibited the symptoms of Quinckes edema. What medication should
be given for emergency aid?
A. Prednisolone
B. Adrenalin
C. Heparin
D. Furosemide
E. Seduxen

150. A baby is 3 months old. The mother consulted a pediatrician about lack
of breast milk. After several test weighings it was found that the child had to
receive supplementary feeding. What is the optimal milk formula for this
child?
A. Malysh
B. Malutka
C. Milk formula No. 3
D. Milk formula No. 2
E. Whole cow’s milk

151. Examination of a newborn revealed skin redness that appeared


immediately after birth and reached the maximum intensity on the second day
of life. What is your provisional diagnosis?
A. Simple erythema
B. Annular erythema
C. Transient erythema
D. Toxic erythema
E. Erythema nodosum
152. A child is 2 days old. He was born with a weight of 2900 kg, body
length of 50 cm. On examination the skin is intensely red, elastic, with
preserved turgor. Puerile respiration is present. Respiration rate - 40/min,
cardiac sounds are rhythmic, sonorous. HR- 138/min. The abdomen is soft.
The liver extends 2 cm below the costal margin. Diuresis is sufficient. Stool is
in form of meconium. What is the most likely diagnosis?
A. Physiological erythema of the newborn
B. Neonatal phlegmon
C. Exfoliative Ritters dermatitis
D. Erysipelas
E. Toxic erythema of the newborn

153. A full-term baby was born with body weight of 3200 g, body length of
50 cm, Apgar score - 8-10 points. What is the optimum time for the first
breast-feeding?
A. First 30 minutes
B. First 24 hours
C. First 6 hours
D. First 48 hours
E. After 48 hours

154. A 3-year-old child has been taken to a pediatrician. He has no recent


history of any diseases. Objective examination revealed no pathology of the
internal organs. The child needs the routine immunization against the
following disease:
A. Poliomyelitis
B. Type B hepatitis
C. Measles, rubella, parotitis
D. Diphtheria and tetanus
E. Pertussis

155. A 6-year-old child has duodenal ulcer. What antibacterial drug should
be co-administered together with metronidazole and De-Nol in order to
eradicate Helicobacter pylori infection?
A. Amoxicillin
B. Sulfadimethoxinum
C. Oleandomycin
D. Tetracycline
E. Biseptol
156. A baby born after fast labour has palsy of hand muscles. Grasp reflex is
absent, as well as hand-to-mouth reflex. Hand sensitivity is absent. What is
the most likely diagnosis?
A. Dejerine-Klumpke palsy
B. Total lesion of the brachial plexus
C. Bernard-Horner syndrome
D. Muscle paresis
E. Duchenne-Erbs palsy

157. A child is 12 years old. He complains of a dull aching pain in the


epigastrium and right hypochondrium, that is getting worse after taking fatty
or fried food, headache, weakness, nausea, low-grade fever. Abdominal
palpation reveals a marked resistance of muscles in the right hypochondrium,
positive Kerrs, Ortners, Murphys symptoms. What is the most likely
diagnosis?
A. Chronic cholecystitis
B. Acute appendicitis
C. Acute gastritis
D. Acute pancreatitis
E. Viral hepatitis

158. A 3-month-old girl presents with rhinitis, dyspnea, dry cough. These
manifestations has been observed for two days. Objectively: the child has pale
skin, acrocyanosis, shallow respiration at the rate of 80/min. Percussion
reveals handbox resonance over the whole surface of lungs, massive fine rales.
What is the most likely diagnosis?
A. Acute bronchiolitis
B. Acute bronchitis
C. Mucoviscidosis
D. Pneumonia
E. Foreign body of the airway

159. A newborn (mothers I pregnancy) weighing 3500 g presents with


jaundice, lethargy, reduced reflexes. Objectively: second grade jaundice of skin
with saffron tint, liver - +2cm, spleen - +1 cm. Urine and feces are yellow.
Blood count: Hb- 100 g/l, RBCs - 3,2*10^12/l, WBCs - 18,7*10^9/l, mothers
blood type - 0(I) Rh(+), babys blood type - ?(II) Rh(-), bilirubin - 170 mmol/l,
indirect fraction. ALT, AST rates are normal. What disease is the child most
likely to have?
A. Hemolytic disease of newborn, AB0-conflict
B. Hemolytic disease of newborn, Rh-conflict
C. Perinatal hepatitis
D. Biliary atresia
E. Physiologic jaundice
160. A 10-year-old girl was admitted to a hospital with carditis
presentations. It is known from the anamnesis that two weeks ago she had
exacerbation of chronic tonsillitis. What is the most likely etiological factor in
this case?
A. Streptococcus
B. Proteus
C. Pneumococcus
D. Staphylococcus
E. Klebsiella

161. All the joints on the left elbow of a newborn are extended, the whole
arm hangs vertically along the trunk with the forearm pronated. Active
movements in the elbow joint are absent but present in the shoulder joint. The
hand is flattened, atrophied, cold to the touch, hangs passively. Grasp reflex
and hand-mouth reflex on the affected side are missing. Haemogram values
are normal. What is the most likely diagnosis?
A. Inferior distal obstetrical paralysis
B. Proximal obstetrical paralysis
C. Hypoxic-ischemic encephalopathy
D. Complete obstetrical paralysis
E. Osteomyelitis

162. Head circumference of a 1-month-old boy with signs of excitement is 37


cm, prefontanel is 2x2 cm large. After feeding the child regurgitates small
portions of milk; stool is normal in respect of its volume and composition.
Muscle tonus is within norm. What is the most likely diagnosis?
A. Pylorospasm
B. Microcephaly
C. Craniostenosis
D. Meningitis
E. Pylorostenosis

163. On the second day after preventive vaccination a 2-year-old boy


presented with abdominal pain without clear localization, body temperature
rose up to 38°C. On the third day the child got red papular haemorrhagic
eruption on the extensor surfaces of limbs and around the joints. Knee joints
were edematic and slightly painful. Examination of other organs and systems
revealed no pathological changes. What is the most likely diagnosis?
A. Haemorrhagic vasculitis
B. Thrombocytopenic purpura
C. Urticaria
D. DIC syndrome
E. Meningococcemia

164. On the 6th day of life a child got multiple vesicles filled with
seropurulent fluid in the region of occiput, neck and buttocks. General
condition of the child is normal. What disease should be suspected?
A. Vesiculopustulosis
B. Impetigo neonatorum
C. Impetigo
D. Epidermolysis bullosa
E. Miliaria

165. A patient is 14 years old. Cytochemical study of punctate revealed 40%


of blasts, there was negative reaction to peroxidase and with Sudan black,
positive reaction to glycogen. Specify the form of acute leukemia:
A. Lymphoblastic
B. Promyelocytic
C. Undifferentiated
D. Myeloblastic
E. Monoblastic

166. Six months ago, a 5-year-old child was operated for CHD. For the last 3
weeks he has complained of fever, heart pain, aching muscles and bones.
Examination results: "white-coffee" skin colour, auscultation revealed systolic
murmur in the region of heart along with a noise in the III-IV intercostal
space. Examination of fingertips revealed Janeway lesions. What is your
provisional diagnosis?
A. Infectious endocarditis
B. Nonrheumatic carditis
C. Sepsis
D. Acute rheumatic fever
E. Typhoid fever

167. The condition of a 3-year-old child with acute non-rheumatic


myocarditis has suddenly deteriorated: he presents with anxiety, acrocyanosis,
peripheral edemata, dyspnea. Auscultation of lungs reveals fine moist rales on
both sides mainly in the lower parts. AP- 65/40 mm Hg. HR- 150/min, heart
sounds are muffled, arrhythmic (extrasystole). Liver is +4 cm. Oliguria is
present. The child has been diagnosed with acute heart failure. Which method
of examination is most informative for assessing the childs status dynamics?
A. Echocardiography
B. Diuresis monitoring
C. 24-hour monitoring of heart rhythm
D. Monitoring of K+, Na+ concentration in blood
E. ECG

168. A hospital admitted an 11-year-old boy diagnosed with medium-severe


asthma, exacerbation period. In order to arrest the attacks the boy was
administered broncholytic nebulizer therapy. During the day the childs
condition stabilized. What is the most appropriate method for further
monitoring of respiratory function in this patient?
A. Peak flowmetry
B. Veloergometry
C. Pneumotachometry
D. Spirometry
E. Bronchodilatation tests

169. A full-term newborn was born with body weight of 4000 g, body length
of 57 cm. Reaction to the postnatal check was absent. There was diffuse
cyanosis, heart rate of 80/min. What resuscitation measures should be taken?
A. Start ALV with a mask
B. Intubate the child and start ALV
C. Give 100% oxygen
D. Start tactile stimulation
E. Give an injection of naloxone

170. A 2-year-old child in a satisfactory condition periodically presents with


moderate proteinuria, microhematuria. USI results: the left kidney is
undetectable, the right one is enlarged, there are signs of double pyelocaliceal
system. What study is required to specify the diagnosis?
A. Excretory urography
B. Retrograde urography
C. Radioisotope renal scan
D. Doppler study of renal vessels
E. Micturating cystography

171. An 8-year-old boy has a 2-year history of blotchy itchy rash appearing
after eating citrus fruit. The first eruption occurred at the age of 6 months
after the introduction of juices to the babys diet. Father has a history of
bronchial asthma, mother - that of allergic rhinitis. What is the most likely
diagnosis?
A. Atopic dermatitis
B. Urticaria
C. Quinckes edema
D. Psoriasis
E. Pityriasis Rosea
172. A 3-year-old child with ARVI had been administered biseptol,
paracetamol, nazoferon. On the third day of treatment the babys condition
deteriorated: he developed sore throat, stomatitis, conjunctivitis,
hypersalivation, painful dark red spots on the neck, face, chest and legs, then
the spots were replaced with vesicles. Examination revealed lesions of mucous
membranes around the mouth and anus. What is your provisional diagnosis?
A. Stevens-Johnson syndrome
B. Bullous dermatitis
C. Chickenpox
D. Atopic dermatitis
E. Serum sickness

173. A 12-year-old child had three attacks of acute rheumatic fever


accompanied by carditis. Examination revealed the symptoms of chronic
tonsillitis, mitral insufficiency, carious teeth. What is the optimal method of
secondary prophylaxis?
A. Year-round bicillin prophylaxis till the age of 25
B. Course of cardiotrophic drugs twice a year
C. Tonsillectomy
D. Oral cavity sanitation
E. Year-round bicillin prophylaxis for 3 years

174. A 7-year-old female child has developed an acute condition. She


complains of a headache, two onsets of vomiting. Objectively: deferred
reactions, body temperature - 39,3°C, pronounced hyperesthesia, nuchal
rigidity, positive superior and inferior Brudzinskis signs, symmetric Kernigs
sign. What is the provisional diagnosis?
A. Meningitis
B. Craniocerebral trauma
C. Encephalitis
D. Toxic encephalopathy
E. Food toxicoinfection

Krok 2 – 2013 Surgery Base


1. A 76 y.o. woman complains of progressing swallowing disorder, mostly she
has had problems with solid food for the last 6 weeks. Sometimes she has
regurgitation of solid masses. Swallowing is not painful. She lost 6 kg. 10 years
ago she had myocardiac infarction, she takes constantly aspirine and
prolonged nitrates. She consumes alcochol in moderate proportions, smokes.
Objectively: icteric skin, neck has no pecularities, lymph nodes are not
enlarged. Thorax has no changes, cardiovascular system has no evident
changes. Liver is +3 cm. What is the preliminary diagnosis?
A. Cancer of esophagus
B. Esophageal achalasia
C. Diffuse constriction of esophagus
D. Diaphragmatic hernia
E. Myasthenia

2. A 65 y.o. man who has problems with urination as a result of benign prostate
gland adenoma dveloped fever and chill, hypotension, sinus tachycardia. Skin
is warm and dry. Clinical blood analysis revealed absolute granulocytopenia.
These hemodynamic changes are most likely to be caused by:
A. Endotoxemia with activation of complement system
B. Secondary circulation insufficiency with retained systolic function as a result
of peripheral vasoconstriction
C. Secondary endothelial changes as a result of bacterial lesion
D. Reflex vagus stimulation with lowered cardiac output
E. Secondary reflex vasodilatation as a result of lowered cardiac output

3. A 60 y.o. man complains of sense of heaviness in the region of scrotum.


Objectively: scrotum edema in the left part. Testicle is of normal size, but
there is a soft, scrotum limited edema over it that can be pressed and
disappears when the patient lies down. What is the preliminary diagnosis?
A. Varicocele
B. Varicosity of subcutaneous veins
C. Ectopic testicle
D. Inguinal lymphadenopathy
E. Inguinal hernia

4. A patient was delivered to a surgical department after a road accident with a


closed trauma of chest and right-sided rib fracture. The patient was diagnosed
with right-sided pneumothorax, it is indicated to perform drainage of pleural
cavity. Pleural puncture should be made in:
A. In the 2nd intercostal space along the middle clavicular line
B. In the projection of pleural sinus
C. In the point of the greatest dullness on percussion
D. In the 6th intercostal space along the posterior axillary line
E. In the 7th intercostal space along the scapular line

5. A 40 year old woman has changes of mammary gland. What are the most
often symtomps that precede the malignization?
A. Skin induration with inverted nipple
B. Painful movable induration
C. Bloody discharges from the nipple
D. Pure discharges from the nipple
E. Painless movable induration

6. Name a statistical observation unit for determination of influence amount of


bloodsugar on the healing of wounds surface in a postoperative period:
A. The patient in a postoperative period
B. The patient who was discharged on an after-care
C. Blood analysis
D. An amount of bloodsugar
E. The patient who has a wound surface

7. A 20-year-old patient was delivered to the hospital in summer from the street
with haemorrage from the brachial artery. First medical aid involved
application of a tourniquet for provisional arrest of bleeding. What is the
maximal exposure of the tourniquet?
A. 120 minutes
B. 15 minutes
C. 60 minutes
D. 180 minutes
E. 30 minutes

8. A 9 y.o. child with diagnosis "chronic tonsillitis" stands dispanserization


control. Within 1 year of observation there was one exacerbation of disease.
Physical condition is satisfactory. The general state is not infringed. Define
group of health:
A. III (a)
B. III (b)
C. III (c)
D. II-d
E. I-st

9. A 38 year old man, previously in good health, suddenly develops severe


abdominal pain radiating from the left loin to groin and accompanied by
nausea, perspiration and the need for frequent urination. He is restless,
tossing in bed but has no abnormal findings. The most likely diagnosis is:
A. Leftsided renal colic
B. Sigmoid diverticulitis
C. Retroperitoneal haemorrhage
D. Torsion of the left testicle
E. Herpes zoster

10. A 40 year old woman has a self-detected hard breast mass. The procedure of
choice for confirming the diagnosis is:
A. Excision biopsy
B. Mammography
C. Ultrasonography
D. Aspiration biopsy with cytology
E. Thermography

11. During investigation for chronic, severe, epigastric pain, a 40 year old
alcoholic man is found to have multiple areas of narrowing alternating with
dilatation ("chain of lakes" appearance) of the main pancreatic duct. The
operation of choice is:
A. Lateral pancreaticojejunostomy
B. Sphincterotomy
C. Total pancreatectomy
D. Distal pancreatectomy
E. Distal pancreaticojejunostomy

12. The treatment of choice for duodenal obstruction caused by secondary


duodenal hematoma that developed a few days after blunt abdominal injury
is:
A. Nasogastric decompression and parenteral alimentation
B. Tube duodenostomy
C. Duodenojejunostomy
D. Retrocolic gastrojejunostomy
E. Immediate exploration

13. An anestesiologist gives narcosis to the patient, he uses a non-reversive


contour. Anesthetic is halothane. Air temperature in the operation room is
21°C, humidity 50%, level of noise 30 dB. What occupational hazard is the
principal one under these conditions?
A. Air pollution with anesthetic
B. High level of noise
C. Improper occupational microclimate
D. Mental overfatigue
E. Compelled working pose

14. A patient suddenly felt an acute chest pain irradiating to the left arm.
Objectively: the patient is excited, with pale skin. Breathing rate - 38/min, AP
- 180/110 mm Hg. Later the patient lost consciousness and fell down. Pulse on
the great vessels was absent, the pupils were equally dilated. What is the most
likely diagnosis?
A. Clinical death
B. Heart attack
C. Disorder of the cerebral circulation
D. Agonal state
E. Coma

15. A 37-year-old patient has come to clinic being wounded in the area of
umbilicus an hour ago.On physical exam, there is a prick and cut painful
wound of about 0,5x1 cm around umbilicus on the abdominal skin with slight
bleeding.How would you help this patient?
A. Laparotomy, abdominal cavity organs inspection. Primary surgical
processing of the wound
B. Aseptic bandage
C. Suture on the wound
D. Drainage of the wound with rubber strip
E. Inspection of the wound with canal probe

16. If a child has adherent fingers on his right hand, then what will be your
diagnosis?
A. Syndactyly
B. Ectromelia
C. Macrodactyly
D. Polydactyly
E. Ectrodactyly

17. A 34-year-old patient was bitten by a dog 3 hours ago.There is a wound by dog
teeth without bleeding in the left arm. What surgical help would you provide
to this patient?
A. Clean wound with detergent water and apply anti-septic
B. Complete suture of the wound
C. Incomplete suture of the wound
D. Aseptic bandage
E. Cream bandage

18. A 63 year old patient was diagnosed with purulent mediastinitis. What of the
below listed diseases are NOT the cause of purulent mediastinitis?
A. Cervical lymphadenitis
B. Perforation of the thoracic part of the oesophagus
C. Iatrogenic injury of the trachea
D. Deep neck phlegmon
E. Perforation of the cervical part of the oesophagus

19. Purulent mediastinitis is diagnosed at a 63-year-old patient. What diseases


from the stated below textbfCANNOT cause the purulent mediastinitis?
A. Cervical lymphadenitis
B. Perforation of the cervical part of the easophagus
C. Deep neck phlegmon
D. Perforation of the thoracic part of the easophagus
E. Iatrogenic injury of the trachea

20. The diagnosis of a right-sided pneumothorax is made to a 36- year-old


patient. What method of treatment is indicated to the patient?
A. Drainage of the pleural cavity
B. Thoracotomy
C. Symptomatic therapy
D. Antiinflammation therapy
E. Pleural puncture

21. A 36 year old patient was diagnosed with right-sided pneumothorax. What
method of treatment is indicated to the patient?
A. Surgical treatment: drainage of the pleural cavity
B. Thoracotomy
C. Symptomatic therapy
D. Antiinflammation therapy
E. Pleural puncture

22. A 16 year old patient with complaints of frequent pain in the abdomen was
diagnosed with melanoma, examination revealed also pigmentation of the
mucosa and skin, polyp in the stomach and large intestine. It is know that the
patients mother has an analogous pigmentation and has been often treated for
anemia. What disease is suspected?
A. Peytz-Egerss polyposis
B. Tuberculosis of the intestine
C. Chrons disease
D. Adolescent polyposis
E. Hirschprungs disease

23. A 41 year old patient was admitted to the intensive care unit with
haemorrhagic shock due to gastric bleeding. He has a history of hepatitis B
during the last 5 years. The source of bleeding are esophageal veins. What is
the most effective method for control of the bleeding?
A. Introduction of obturator nasogastric tube
B. Operation
C. Administration of plasma
D. Intravenous administration of pituitrin
E. Hemostatic therapy
24. It is suspected that a 34 year old patient has an abscess of Douglas pouches.
What diagnostic method is to be chosen?
A. Digital examination of rectum
B. Percussion and auscultation of stomach
C. R-scopy of abdominal cavity
D. Rectoromanoscopy
E. Laparoscopy

25. A patient has restrained umbilateral hernia complicated by phlegmon hernia,


it is necessary to take following actions:
A. Herniotomy by Mayo-Sapezhko
B. Herniotomy by Mayo
C. Herniotomy by Lekser
D. Herniotomy by Grenov
E. Herniotomy by Sapezhko

26. A 52 year old man has recurrent transient ischemic attacks. Auscultation of
the carotid arteries detected murmur. What diagnostic method is to be applied
in the first place?
A. Ultrasound dopplerography
B. CT of the brain
C. Cerebral angiography
D. Electroencephalography
E. MRI of the brain

27. For the persons who live in a hot area after an accident at a nuclear object, the
greatest risk within the first decade is represented by cancer of:
A. Thyroid gland
B. Reproduction system organs
C. Lungs
D. Breast
E. Skin

28. A 10 year old boy complains about pain in his left eye and strong photophobia
after he had injured his left eye with a pencil at school. Left eye examination:
blepharospasm, ciliary and conjunctival congestion, cornea is transparent,
other parts of eyeball have no changes. Visus 0,9. Right eye is healthy, Visus
1,0. What additional method would you choose first of all?
A. Staining test with 1% fluorescein
B. X-ray examination of orbit
C. Gonioscopia
D. Cornea sensation-test
E. Tonometria
29. A patient had disorder of nasal respiration, mucopurulent discharges from
nose, headache for 3 weeks. At anterior rhinoscopy in middle nasal meathus
the stria of pus, edema, hyperemia of the mucosa of the nose have been
determined. What diagnostic method is necessary to administer first of all?
A. An X-ray of paranasal sinuses
B. General blood test
C. Bacteriology analysis of the nasal mucous
D. CT of a skull
E. Punction of the maxillar sinus

30. A patient with acute purulent otitis media complicated by mastoiditis was
admitted to a hospital. Roentgenogram of mastoid processes showed the
shadiowing of the cellular system on the lesion, absence of bone septa was
present. What are the necessary therapeutic actions at the second stage of
mastoiditis?
A. Mastoidotomy
B. Radical operation on the middle ear
C. Cateterization of the Eustachian tube
D. Tympanoplasty
E. Paracentesis of the drum

31. A 35-year-old woman was admitted to resuscitation department in asthmatic


status. What is the most trustworthy criterion of breath effectiveness?
A. PaCO2 and PaO2
B. Minute respiratory volume
C. Determination of “dead” space
D. Respiratory volume
E. Respiration rate

32. During dynamic investigation of a patient the increase of central venous


pressure is combined with the decrease of arterial pressure. What process is
proved by such combination?
A. Developing of cardiac insufficiency
B. Increase of bleeding speed
C. Shunting
D. Depositing of blood in venous channel
E. Presence of hypervolemia

33. A 22 year old patient was admitted to trauma center with complaints of pain
in the left ankle joint that was getting worse during moving and weight
bearing. On the clinical examination it was found that the patient had the
closed fracture of medial malleolus without displacement. In which position
should the foot be fixed in plaster cast?
A. At right angle with varus positioning of the foot
B. In position of supination
C. In position of dorsal flexion of foot
D. In position of planter flexion of foot
E. In position of pronation

34. A 74 y.o. patient has been ill with benign prostate hyperplasy for the last 5
years. 4 days ago, after alcochol consumption, there was an acute retention of
urination. At the pre-admission stage his urinary bladder was catheterized
with metallic catheter. Examination revealed: right epididymis is enlarged,
thick and painful, there are purulent discharges from urethra. What way of
emergency care must be chosen?
A. Trocar or open epicystostomy
B. Microwave thermotherapy of prostate
C. Placing of intraprostatic stent
D. Transuretral resection or prostatectomy
E. Introduction of permanent urethral catheter

35. A rounded well-defined shadow was found in the costo-vertebral angle on the
chest roentgenogram of an otherwise healthy 9 year old girl. Make a
preliminary diagnosis:
A. Ganglioneuroma
B. Sympatoblastoma
C. Sympatogonioma
D. Sarcoma of the vertebra
E. Ganglioneuroblastoma

36. A 4 y.o. child attends the kindergarten. Complains of poor appetite, fatigue.
Objective examination: skin and mucous membrane are pale, child is asthenic.
In the hemogram: hypochromatic anemia 1st, leucomoide reaction of the
eosinophile type. What pathology must be excluded first of all?
A. Helminthic invasion
B. Atrophic gastritis
C. Hypoplastic anemia
D. Lymphoprolipherative process
E. Duodenal ulcer

37. A 33 year old male patient was brought to Emergency Department with the
signs of cardiovascular collapse: BP - 60/30 mm Hg, Ps - 140 bpm, the skin is
pale and moist, diuresis 20 ml/h, Hb - 80 g/l, red blood cell count -
2,5×1012/l. The reduction of blood volume averages:
A. 30-40%
B. 15-20%
C. 25-30%
D. 20-25%
E. 10-15%

38. A 19 year old girl was admitted to emergency department: unconsciousness,


cyanosis, myotic pupils are present, superficial breathing is 12/min. BP is
90/60 mm Hg, Ps- 78/min. Choose the action necessary in this clinical
situation:
A. Controlled respiration
B. Caffeine injection
C. Cordiamine injection
D. Gastric lavage
E. Oxygen inhalation

39. What preparations are used for prevention of fungal infection?


A. Fluconozol, Orungol, Nisoral
B. Rubomycin, Bleomycin, Mytomycin C
C. Captopril, Enalapril
D. Isoniazid, Ftibazid, Pyrazinamid
E. Cytosar, Cormyctin, Lomycitin

40. What developes most often after accidental intake of Hydrochloric acid?
A. Cardiac insufficiency
B. Acute pancreatitis
C. Kutlings syndrome
D. Cushings syndrome
E. Deyladss syndrome

41. 30 y.o. woman, had mild trauma of 5th finger of the left hand 15 days ago. She
has treated her self at home. She presents to the hospital due to deterioration
of the condition and temperature rise. Objectively: hyperemia and swelling on
the ventral surface of finger. Restricted movements of the finger. X-ray of the
left hand: an early stage of ?steomyolitis of the fifth finger could not be
excluded. The diagnosis: panaris of 5th finger of the left hand. What form of
panaris has occurred in the patient?
A. Bony
B. Paronychia
C. Hypodermic
D. Tendon type
E. Joints type
42. A 36 y.o. patient is diagnosed with right sided pneumothorax. What method of
treatment is indicated to the patient?
A. Surgical drainage of the pleural cavity
B. Symptomatic therapy
C. Antiinflammation therapy
D. Pleural puncture
E. Thoracotomy

43. A 28 year old woman was admitted to the emergency room with a slightly
reddened, painful "knot" 8 cm above the medial malleolus. Examination in the
standing position demonstrates a distended vein above and below the mass.
There are no other abnormalities on physical examination. The most likely
diagnosis is:
A. Superficial venous thrombosis
B. Insect bite
C. Subcutaneous hematoma
D. Cellulitis
E. Early deep vein thrombosis

44. A 30 year old man complains of acute pain in his right ear, hearing loss, high
temperature for three days. Objectively: right ear whispering language - 0,5
mm, external ear is intact, otoscopically - eardrum protrusion, hyperemia and
swelling, loss of landmarks. What disease is it?
A. Acute purulent otitis media
B. Acute mastoiditis
C. Chronic purulent otitis media
D. Eustachian tube disfunction
E. Chronic secretory otitis media

45. A 22 y.o. man complains of acute throat pain, increasing upon swallowing
during 3 days. Body temperature 38,3°C, neck lymph nodules are slightly
enlarged and painful. Pharyngoscopically - tonsilar hyperemia, enlargement
and edema, tonsils are covered by round yellow fibrinous patches around
crypts openings. Beta-haemolytic streptococcus in swab analysis. What is the
diagnosis?
A. Acute membranous tonsilitis
B. Acute follicular tonsilitis
C. Infectious mononucleosis
D. Pharyngeal candidosis
E. Pharyngeal diphtheria
46. A patient is staying in the hospital with the diagnosis of abdominal typhus.
During the 3-d week from the beginning of the disease the patient stopped
keeping diet and confinement to bed. As a result the body temperature and
rapid pulse decreased and melena appeared. What kind of complications
should we think about first of all?
A. Intestinal haemorrhage
B. Thrombophlebitis
C. Nephroso-nephritis
D. Hepatite
E. Meningitis

47. A 45-year-old woman, mother of four children, comes to the emergency room
complaining of a sudden onset of the epigastric and right upper quadrant
pain, radiating to the back, accompanied by vomiting. On examination,
tenderness is elicited in the right upper quadrant, bowel sounds are decreased,
and laboratory data shows leukocytosis, normal serum levels of amylase,
lipase, and bilirubin. The most likely diagnosis is:
A. Acute cholecystitis
B. Perforated peptic ulcer disease
C. Sigmoid diverticulitis
D. Acute pancreatitis
E. Myocardial infarction

48. During an operation for presumed appendicitis the appendix was found to be
normal; however, the terminal ileum is evidently thickened and feels rubbery,
its serosa is covered with grayish-white exudate, and several loops of
apparently normal small intestine are adherent to it. The most likely diagnosis
is:
A. Crohns disease of the terminal ileum
B. Ulcerative colitis
C. Acute ileitis
D. Ileocecal tuberculosis
E. Perforated Meckels diverticulum

49. A 50 year old woman with a 2-year history of mild, diffuse, tender thyroid
enlargement complains of 10 pound weight gain and fatigue. What is the most
probable diagnosis?
A. Hashimotos thyroiditis
B. Riedels thyroiditis
C. Suppurative thyroiditis
D. Papillary thyroid carcinoma
E. Subacute thyroiditis
50. A severely traumatized patient who has been receiving prolonged parenteral
alimentation develops diarrhea, mental depression, alopecia and perioral and
periorbital dermatitis. Administration of which of the following trace elements
is most likely to reverse these complications?
A. Zinc
B. Silicon
C. Copper
D. Iodine
E. Selenium

51. A 38 y.o. woman was hospitalized to the surgical unit with vomiting and acute
abdominal pain irradiating to the spine. On laparocentesis hemmorhagic fluid
is obtained. What disease should be suspected?
A. Acute pancreatitis
B. Acute appendicitis
C. Acute enterocolitis
D. Renal colic
E. Perforated gastric ulcer

52. The 67 y.o. patient had 5 recurrent fractures of the lower extremities without
considerable cause within 5 years. O-shaped deformity of the legs in the knee
joints has appeared. The skull, pelvis and lower extremities X-Ray shows the
thickening of flat bones. In the long bones there is a hyperostosis along the
bone axis.The blood test has not revealed any inflammation activity. Serum
calcium is normal. What disease do you consider in this case?
A. Pagets disease
B. Hyperparathyoid dystrophy
C. Myeloma
D. Mottled disease (marble disease)
E. Chronic osteomyelitis

53. A 33 y.o. patient was admitted to the reception room of the Central District
Hospital. He complains of a severely painful swelling localized on posterior
neck, fever up to 38,40C and general weakness. In anamnesis: diabetes
mellitus within 5 years. On physical examination on the posterior neck surface
there is an infiltrate elevated above surrounding skin. The tissues affected by
swelling are tense and blue reddish discoloration in central area. There are
also several purulent necrotic pustules which are connected with each other
and form a large skin necrosis. A thinned necrotic skin of this swelling has
holes looking like sieve, pus discharges through out. What disease should a
doctor consider first of all?
A. Carbuncle
B. Acute skin cellulitis
C. Skin abscess
D. Carbuncle associated with anthrax
E. Furuncle

54. A 19 y.o. man was admitted to the reception department in 20 minutes after
being wounded with the knife to the left chest. The patient is confused. The
heart rate is 96 bpm and BP- 80/60 mm Hg. There are the dilated neck veins,
sharply diminished apical beat and evident heart enlargement What kind of
penetrative chest wound complications has developed in patient?
A. Pericardium tamponade
B. Closed pneumothorax
C. Valve-likes pneumothorax
D. Massive hemothorax
E. Open pneumothorax

55. A 35 y.o. patient complains of a difficult swallowing, pain behind the


breastbone. He can eat only liquid food. While swallowing sometimes he has
attacks of cough and dyspnea. Above mentioned complaints are progressing.
It is known that the patient has had a chemical burn of esophagus one month
ago. What complication does the patient have?
A. Corrosive esophagitis and stricture
B. Esophageal diverticula
C. Esophagitis
D. Cardiac achalasia
E. Cardiac insufficiency

56. Survey radiograph of chest of a 62 year old smoker who often suffers from
"pneumonias" showed a triangle shadow in the right lung, its vertex is
pointing towards the lung root. It also showed deviation of heart and
mediastinum shadows towards the lesion. What is the most probable
diagnosis?
A. Cenral cancer of lung
B. Lung cyst
C. Lung abscess
D. Peripheral cancer of lung
E. Atelectasis

57. A patient operated for acute paraproctitis undergoes antibacterial and


detoxification therapy, the local course of the disease has the positive
dynamics. Since the operation the patient has had chills, pyrexia, tachycardia,
euphoria for five days. The doctor suspected sepsis. What study will confirm
the diagnosis?
A. Blood culture for a pathogen
B. Liver ultrasound
C. Determining the rate of average-weight molecules
D. Determining the rate of microbial contamination of wound
E. X-ray of lungs

58. A 44-year-old patient has been admitted to a hospital with complaints of dull,
aching pain in the left lumbar region, the admixture of pus in the urine.
Examination revealed a grade II staghorn calculus on the left. What method of
treatment is indicated for this patient?
A. Surgery
B. Distance lithotripsy
C. Ascending litholysis
D. Conservative therapy
E. Contact lithotripsy

59. An emergency team has delivered to a hospital an unconscious patient found


lying in the street in winter. Objectively: the patient is pale, with superficial
respiration; bradycardia with heartrate 54/min, to- 35,0°C. AP- 100/60 mm
Hg. Palpation of chest and abdomen revealed no peritoneal symptoms. There
is a smell of alcohol from the patients mouth. What is the most likely
diagnosis?
A. Hypothermia
B. Apparent death
C. Frostbite of trunk and extremities
D. Acute cardiovascular insufficiency

60. A 48-year-old patient got a job-related injury of a hypodermic varicose vein on


his shin that was accompanied by the intensive phleborrhagia. Choose the
optimal variant of first aid:
A. Pressure bandage and limb strapping
B. Application of Esmarchs tourniquet beneath the injury
C. Maximal limb flexion in knee joint
D. Occlusion of femoral artery in a typical place
E. Application of Esmarchs tourniquet above the injury

61. A 24-year-old patient got a puncture injury below the Pouparts ligament
accompanied by intense arterial bleeding. The best method to temporarily
stop the bleeding in the patient would be:
A. Compression band
B. Compressing a blood vessel with a clamp
C. Wound suturing
D. Esmarchs tourniquet
E. Maximum limb bending
62. While making a round, a doctor had noticed the edema of the right shoulder of
a 26-day-old child with diagnosis of "umbilical sepsis". Active movements in
the shoulder joint were absent, right hand was hanging down. Mother stated
that her childs condition had worsened for the last 24 hours what resulted in
childs refusal to be breast-fed, restlessness, weeping while swaddling, rise in
body temperature up to 38,8°C. What is the most probable preliminary
diagnosis?
A. Epiphysial osteomyelitis of the humerus
B. Fracture of the clavicle
C. Traumatic brachioplexitis
D. Fracture of the humerus
E. Phlegmon of the shoulder

63. A 65-year-old patient complains of dull pain in the rectum during and after
defecation, discharge of mucus and small amount of blood mixed up with
mucus and faeces. The discharged blood is of dark red color, sometimes with
small amount of clots. The patient is sick for 8 months, has lost some weight.
On digital examination, there is a round constriction of the rectum with
infiltrate at a height of 4-5 cm from the anus. What is the most probable
diagnosis?
A. Cancer of the medium-ampullar section of the rectum
B. Cicatricial stenosis of the rectum
C. Crohns disease
D. Non-specific ulcer colitis
E. Chronic paraproctitis

64. A 52 y.o. patient fell from 3 m height on the flat ground with the right lumbar
area. He complains of pain in this area. There is microhematuria in the urea.
Excretory urography revealed that kidneys functioning is satisfactory. What is
the most probable diagnosis?
A. Kidneys contusion
B. Multiple kidneys ruptures
C. Subcapsular kidneys rupture
D. Paranephral hematoma
E. Kidneys abruption

65. A female patient has been suffering from pain in the right subcostal area,
bitter taste in the mouth, periodical bile vomiting for a month. The patient put
off 12 kg. Body temperature in the evening is 37,6°C. Sonography revealed
that bile bladder was 5,5?2,7 cm large, its wall - 0,4 cm, choledochus - 0,8 cm
in diameter. Anterior liver segment contains a roundish hypoechoic formation
up to 5 cm in diameter and another two up to 1,5 cm each, walls of these
formations are up to 0,3 cm thick. What is the most likely diagnosis?
A. Alveolar echinococcus of liver
B. Liver abscess
C. Liver cancer
D. Cystous liver cancer
E. Paravesical liver abscesses

66. A 36-year-old woman applied to doctor with complaints of the enlargement of


the right mammary gland. On physical exam: the right mammary gland is
enlarged, hardened; nipple with areola is pasty and hydropic, looks like
"lemon peel". There palpates a lymph node in the right axilla, 1,5 cm in
diameter, dense consistence, not mobile. What is the most probable
diagnosis?
A. Hydropic and infiltrative cancer of the mammary gland
B. Diffusive mastopathy
C. Erysipelatouslike cancer of the mammary gland
D. Pagets cancer
E. Acute mastitis

67. A 43 year old patient had cholecystectomy 6 years ago because of chronic
calculous cholecystitis. Lately he has been suffering from pain in the right
subcostal area and recurrent jaundice. Jaundice hasnt gone for the last 2
weeks. Stenosing papillitis 0,5 cm long has been revealed. What is the best
way of treatment?
A. To perform endocsopic papillosphincterotomy
B. To perform external choledoch drainage
C. To perform choledochoduodenostomy
D. To perform transduodenal papillosphincterotomy
E. To treat conservatively: antibiotics, spasmolytics, antiinflammatory drugs

68. A 54-year-old patient applied with complaints of acute pain in the shoulder-
joint. 10 minutes earlier he fell in the street with his arms stretched. Doctor of
the traumatological unit noticed an acute deformation of the right shoulder-
joint, the impossibilty of active movement in the right extremity.
Roentgenologically: uncoincidence of articulating surfaces. What is the most
probable diagnosis?
A. Dislocation of the clavicle
B. Dislocation of the scapula
C. Fracture of the humerus
D. Dislocation of the humerus
E. Contusion of the humerus
69. A 62-year-old patient has been delivered to the surgical department with
complaints of sudden pain in the umbilical region irradiating to the back and
groin, which was accompanied by a syncope. Objectively: the patient is in
grave condition, umbilical region is tender on palpation, bowel sounds are
diminished. AP drop is present. USI reveals: free fluid in the abdomen,
thickening of the wall of the abdominal aorta. The most likely diagnosis is:
A. Rupture of abdominal aortic aneurism
B. Peritonitis
C. Acute appendicitis
D. Stomach ulcer perforation
E. Acute pancreatitis

70. Parents of a 2-year-old boy applied to clinic complaining of right testicle


absence in the scrotum of a boy. While examining the boy, hypoplasia of the
right half of the scrotum was revealed, absence of the testicle. Testicle is
miniaturized, it palpitates along the inguinal canal but it could not be moved
down to scrotum. What is the most probable diagnosis?
A. Right-sided cryptorchism, inguinal form
B. Ectopia of the right testicle, pubic form
C. Left-sided monorchism
D. Retraction of the right testicle (pseudocryptorchism)
E. Right-sided cryptorchism, abdominal form

71. A 62-year-old patient complains of the pain behind the sternum, bad passing
of solid and liquid food, bad breath, increased salivation weight, loss of 15 kg
during the period of 2 months. Appetite is retained. On physical exam: face
features are sharpened. The skin is pale, with sallow tint, its turgor is
decreased. The liver is not enlarged. Blood Hb - 86g/L. Gregersen reaction is
positive. What kind of pathology caused the given clinical situation?
A. Esophagus cancer
B. Chronic non-specific esophagitis
C. Cicatricial constriction of esophagus
D. Benign growth of esophagus
E. Achalasia of esophagus

72. A patient suffers from suddenly arising crampy pain in the right loin area. 2
hours after the pain had started, hematuria took place. Loin X-ray: no
pathological shadows. Ultrasound: pyelocaliectasis on the right, the left part is
normal. What is the most probable diagnosis?
A. Stone of the right kidney, renal colic
B. Acute appendicitis
C. Twist of the right ovary cyst
D. Tumour of the right kidney pelvis
E. Intestine invagination
73. A patient complains of an extremely intense pain in epigastrium. He has
peptic ulcer disease of duodenum for 10 years. The patient is in the forced
position being on the right side with legs abducted to stomach. Abdomen has
acute tenderness in the epigastrium. Guarding contraction of the abdominal
wall muscles is observed. What is the preliminary diagnosis?
A. Perforation of ulcer
B. Thrombosis of mesenteric vessels
C. Acute condition of peptic ulcer disease
D. Acute pancreatitis
E. Penetration of ulcer into pancreas

74. A 30-year-old patient complains of pain, hyperemia along subcutaneous veins,


rise in body temperature. While examining the large shin subcutaneous vein,
there is hyperemia, pain by pressing. Homanss and Lusess symptoms are
negative. What is the preliminary diagnosis?
A. Acute thrombophlebitis of subcutaneous veins
B. Acute ileophemoral phlebothrombosis
C. Embolism of aorta
D. Thrombosis of aorta
E. Lymphostasis

75. A 30-year-old man has suffered from traffic accident. Consciousness is absent.
Pulse on the carotid artery is undeterminable, no respiration. There is a wide
leather belt on mans waist. What measures are to be taken?
A. Conduct an artificial ventilation of lungs and closed-chest cardiac
massage after having released from the belt
B. Do not touch the victim until the arrival of the police
C. Put the victim with his back on the shield
D. Start immediate artificial ventilation of lungs and closed-chest cardiac
massage
E. Turn the victim to the right side

76. An emergency team deliverd a 83-year-old patient complaining of inability of


her right leg to support the body after falling on her right side. Objectively: the
patient lies on a gurney, her right leg is rotated outwards, the outside edge of
foot touches the bed. There is positive straight leg raising sign. What is your
provisional diagnosis?
A. Femoral neck fracture
B. Hip dislocation
C. Femoral diaphysis fracture
D. Hip joint contusion
E. Cotyloid cavity fracture
77. A 50-year-old patient complains of bursting pain in the left lower limb that is
getting worse on exertion, swelling in the region of shin and foot. Objectively:
left shin and foot are doughy, skin of the lower shin is indurated and has a
bronze tint, subcutaneous veins are dilated, there is an ulcer with necrotic
masses. What is the most likely diagnosis?
A. Postthrombophlebitic syndrome
B. Deep vein thrombosis of the lower limbs
C. Gangrene of the lower extremity
D. Chronic arterial insufficiency
E. Acute arterial thrombosis

78. 3 days ago a 29-year-old patient presented with pulsating pain in the region of
rectum, fever, general weakness. Objectively: local tenderness in the anal
region in the 6 oclock position. Digital investigation of rectum revealed a
painful infiltration reaching the pectinate line. What is the most likely
diagnosis?
A. Acute periproctitis
B. Acute prostatitis
C. Acute haemorrhoids
D. Acute anal fissure
E. Rectum tumour

79. A 65-year-old patient complains of gradual worsening of the left eye vision
during 10 months. On physical examination: acuty of vision of the left eye is
0,01, not correctable. The eye is quiet, pupil of the eye is grayish, reflex from
the eye-ground is absent. Intraocular pressure is 18 mm/Hg. What is the most
probable preliminary diagnosis?
A. Senile cataract
B. Open-angle glaucoma
C. Leukoma of the cornea
D. Exfoliation of the retina
E. Disorder of blood circulation in retina vessels

80.A 36 y.o. patient was admitted to the hospital with sharp pain in substernal
area following occasional swallowing of a fish bone. On esophagoscopy the
foreign body wasnt revealed. The pain increased and localized between
scapulas. In a day temperature elevated, condition became worse, dysphagia
intensified. What complication has developed?
A. Perforation of esophagus with mediastinitis
B. Aspirative pneumonia
C. Obstruction of esophagus
D. Esophageal hemorrhage
E. Pulmonary atelectasis

81. A 19 year old patient was admitted to a hospital with acute destructive
appendicitis. He suffers from hemophilia of B type. What antihemophilic
medications should be included in pre- and post-operative treatment plan?
A. Fresh frozen plasma
B. Cryoprecipitate
C. Native plasma
D. Dried plasma
E. Fresh frozen blood

82. On the 4th day after recovering from a cold a patient was hospitalized with
complaints of solitary spittings of mucoid sputum. On the 2nd day there was a
single discharge of about 250 ml of purulent blood-streaked sputum.
Objectively: the patients condition is moderately severe. Respiratory rate - 28-
30/min, Ps- 96 bpm, AP- 110/70 mm Hg. Respiration above the left lung is
vesicular, weak above the right lung. There are moist rales of different types
above the lower lobe and amphoric breath near the angle of scapula. What is
the most likely diagnosis?
A. Acute pulmonary abscess
B. Acute focal pneumonia
C. Pyopneumothorax
D. Pleural empyema
E. Exudative pleuritis

83. A 24 y.o. male patient was transferred to the chest surgery department from
general surgical department with acute post-traumatic empyema of pleura. On
the X-ray: wide level horizontal of fluid on the right. What method of
treatment should be prescribed?
A. Punction and drainage of pleural cavity
B. Thoracoplasty
C. Lobectomy
D. Decortication of pleura
E. Pneumoectomy

84. A patient complains of intense pressing pain in the pharynx, mainly to the
right, impossibility to swallow even liquid food. The illness started 5 days ago.
The patients condition is grave. Body temperature - 38,90C, speech is
difficult, voice is constrained, difficulties in opening the mouth. Submaxillary
glands to the right are painful, enlarged. What is the most probable diagnosis?
A. Peritonsillar abscess
B. Pharyngeal tumour
C. Diphtheria
D. Vincents disease
E. Phlegmonous tonsillitis

85. In a 65 y.o. female patient a tumor 13x8 cm in size in the umbilical area and
above is palpated, mild tenderness on palpation, unmovable, pulsates. On
ausculation: systolic murmur. What is the most probable diagnosis?
A. Abdominal aortic aneurism
B. Tricuspid valve insufficiency
C. Mitral insufficiency
D. Stomach tumor
E. Arterio-venous aneurism

86. A patient with suspicion of pelvic bones fraction is under examination


conducted by the doctor who presses alae ilii in medial direction with his both
hands. What causes painful syndrome of the patient?
A. Disorder of continuity of the pelvic ring
B. Fracture of sciatic bones
C. Fracture of the sacrum
D. Retroperitoneal hematoma
E. Traumatic injury of the intrapelvic organs

87. A 30-year-old man was injured in a fire and got thermic burns of III-A and
III-B degree that affected 20% of the total skin. AP - 110/70 mm Hg, heart rate
-120/min. What transfusion means should be used for blind infusion before
transportation?
A. Saline solutions
B. Fresh frozen plasma
C. Albumin
D. Polyglycine
E. 10% glucose solution

88. A 24 y.o. woman consulted a doctor about continued fever, night sweating.
She lost 7 kg within the last 3 months. She had casual sexual contacts.
Objectively: enlargement of all lymph nodes, hepatolienal syndrome. Blood
count: leukocytes - 2,2109/L. What disease can be suspected?
A. HIV-infection
B. Tuberculosis
C. Chroniosepsis
D. Infectionous mononucleosis
E. Lymphogranulomatosis
89. Development of chronic venous insufficiency of lower extremities depends on
the functional condition of so-called musculovenous pump. This term refers to
the following group of muscles:
A. Shin muscles
B. Buttock region muscles
C. Foot muscles
D. Thigh muscles
E. Abdominal wall muscles

90. A 27-year-old patient was brought to clinic with a crashed wound of the
posterior surface of the right shin in 2 hours after the accident happened.
During surgical treatment pieces of dirty clothes and gravel were removed.
What actions from the given below are the decisive ones to prevent an
anaerobic infection?
A. Radical surgical treatment
B. Rational antibiotic therapy
C. Hyperbaric oxygenation
D. Introduction of the preventive dose of an anti-gangrene serum
E. Introduction of the medical dose of the anti-gangrene serum

91. A 32-year-old patient lives in an area endemic for echinococcosis. In the last 6
months he reports of pain in the right subcostal area, fever. He is suspected to
have liver echinococcosis. What study would be the most informative in this
case?
A. USI
B. Biochemical laboratory examination
C. Liver scanning
D. Angiography
E. Survey radiography of abdominal cavity

92. In morgue there are dead bodies with the following causes of death:
electrotrauma; rupture of the spleen with acute anemia. There is one unknown
person; one ethyl alcohol poisoned person and one drowned man. What dead
body should the blood group be determined for?
A. All dead bodies of the unknown persons
B. Body of person with internal hemorrhage
C. Body of person with a sudden death
D. Body of poisoned person
E. Body of drowned man

93. A 50 y.o. patient was admitted to the hospital with complaints of blood in
urine. There was no pain or urination disorders, hematuria has lasted for 3
days. Objectively: kidneys are not palpable, suprapubic region has no
pecularities, external genitals have no pathology. Rectal examination revealed:
prevesical gland is not enlarged. Cytoscopy revealed no changes. What disease
would you think about first of all?
A. Cancer of kidney
B. Varicocele
C. Tuberculosis of urinary bladder
D. Kidney dystopy
E. Necrotic papillitis

94. A patient complains of nycturia, constant boring pain in perineum and


suprapubic region, weak urine jet, frequent, obstructed and painful urination.
He has been ill for several months, pain in perineum appeared after getting
problems with urination. Rectal examination revealed that prostate is
enlarged (mostly owing to the right lobe), dense, asymmetric, its central
sulcus is flattened, the right lobe is dense, painless, tuberous. What disease
can it be?
A. Cancer of prostate
B. Urolithiasis, stone of the right lobe of prostate
C. Chronic congestive prostatitis
D. Prostate tuberculosis
E. Prostate sclerosis

95. The patient complains of a painful swelling in the chin region, malaise,
headache. Examination reveals an acutely inflamed cone-shaped dense node.
The skin over it is tense, red. In the center of the node there is an ulcer with
overhanging edges and a necrotic core of a dirty-green colour. Submandibular
lymph nodes on the right are enlarged and painful. What is the most likely
diagnosis?
A. Furuncle
B. Carbuncle
C. Tuberculosis
D. Tertiary syphilis (gummatous form)
E. Parasitic sycosis

96. On the 5th day after a surgery for colon injury a patient complains of bursting
pain in the postoperative wound, weakness, drowsiness, headache, fever up to
40°C. Objectively: the skin around the wound is swollen, there is gas
crepitation. The wound discharges are scarce foul-smelling, of dark-gray
colorl. What is the most likely diagnosis?
A. Anaerobic clostridial wound infection
B. Postoperative wound infection
C. Abscess
D. Erysipelas
E. Phlegmon
97. A childrens surgical unit admitted a 1-month-old boy who had been prenatally
diagnosed with the left-sided pyelectasis. Such studies as drip infusion
urography, cystography and USI allowed to reveal initial hydronephrosis.
There is no information confirming the secondary pyelonephritis. What tactics
of this patient management is most advisable?
A. 6-month surveillance
B. Anderson-Hynes operation
C. Urgent nephrostomy
D. There is no need in further surveillance and treatment
E. Antibacterial therapy

98. A 60 y.o. man complains of significant pain in the right eye, photophobia,
lacrimation, reduced vision of this eye, headache of the right part of the head.
Pain occured 2 days ago. On examination: Vis OD- 0,03, congested injection
of the eye ball, significant cornea edema, front chamber is deep, pupil is
narrow, athrophic iris, there is optic nerve excavation on the eye fundus,
intraocular pressure- 38 mm Hg. Vis OS- 0,8 unadjustable. The eye is calm,
healthy. Intraoccular pressure- 22 mm Hg. What is the most probable
diagnosis?
A. Acute glaucoma attack
B. Eye nerves neuritis
C. Maculodystrophy
D. Right eyes uveitis
E. Right eyes keratitis

99. A 54-year-old female patient has been admitted to a hospital 12 days after the
beginning of acute pancreatitis. Objectively: the patient is in grave condition.
The body temperature is hectic. Ps - 112 bpm. The abdomen is swollen.
Epigastrium palpation reveals a very painful infiltration falling in the
localization of pancreas. Abdominal cavity contains fluid. There is an edema of
the left lumbar region. In blood: WBCs - 18cdot109/l. What is the required
tactics of the patients treatment?
A. Surgical treatment
B. Massive antibacterial therapy
C. Peritoneal dialysis
D. Further conservative treatment
E. Increase in antienzymes

100. To replace the blood loss replacement 1000 ml of the same group of
Rhesus-compatible donated blood was transfused to the patient. The blood
was conserved by sodium citrate. At the end of hemotransfusion there
appeared excitement, pale skin, tachycardia, muscles cramps in the patient.
What complication should be suspected?
A. Citrate intoxication
B. Anaphylactic shock
C. Pyrogenous reaction
D. Citrate shock
E. Allergic reaction

101. A 20 y.o. patient suddely felt ill 12 hours ago. There was pain in
epigactric area, nausea, sporadic vomiting. He had taken alcohol before. In
few hours the pain localized in the right iliac area. On examination: positive
rebound tenderness symptoms. WBC- 12,2*10^9/L. What is the most
probable diagnosis?
A. Acute appendicitis
B. Perforated ulcer
C. Acute pancreatitis
D. Rightside kidney colic
E. Acute cholecystitis

102. A patient, aged 58, was fishing in the winter. On return home after
some time felt some pain in the feet. Consulted a doctor. On examination: feet
skin was pale, then after rewarming became red, warm to the touch. Edema is
not significant, limited to the toes. All types of sensitivity are preserved. No
blisters. What degree of frostbite is observed?
A. I degree
B. II degree
C. IV degree
D. V degree
E. III degree

103. After an accident a patient complains of pain in the hip joint.


Objectively: the leg is in the position of flexion, adduction and internal
rotation, significantly contracted. There is elastic resistance to passive
adduction or abduction of the extremity. Major trochanter is located high
above the Roser-Nelaton line. A significant lordosis is present. What is your
provisional diagnosis?
A. Iliac dislocation of hip
B. Cotyloid cavity fracture with a central dislocation of hip
C. Femoral neck fracture with a displacement
D. Inferoposterior dislocation of hip
E. Pertrochanteric fracture of hip
104. Three weeks after acute angina the patient is still weak, inert,
subfebrile, his retromaxillary lymph nodes are enlarged. Tonsils are flabby,
stick together with arches, there are purulent plugs in lacunae. What is the
most probable diagnosis?
A. Chronic tonsillitis
B. Tonsillar tumour
C. Acute lacunar tonsillitis
D. Chronic pharyngitis
E. Paratonsillitis

105. A 43 year old patient had right-sided deep vein thrombosis of


iliofemoral segment 3 years ago. Now he is suffering from the sense of
heaviness, edema of the lower right extremity. Objectively: moderate edema of
shin, brown induration of skin in the lower third of shin, varix dilatation of
superficial shin veins are present. What is the most probable diagnosis?
A. Postthrombophlebitic syndrome, varicose form
B. Acute thrombosis of right veins
C. Parkes-Weber syndrome
D. Acute thrombophlebitis of superficial veins
E. Lymphedema of lower right extremity

106. A child undergoes in-patient treatment for acute staphylococcal


destruction of the right lung. Unexpectedly he developed acute chest pain on
the right, dyspnea, cyanosis. The right side of chest lags behind in the
respiratory act. Percussion reveals dullness in the lower parts on the right,
bandbox resonance in the upper parts. Borders of the relative cardiac dullness
are shifted to the left. What complication has most likely developed?
A. Right-sided pyopneumothorax
B. Spontaneous pneumothorax
C. Pleural empyema
D. Exudative pleuritis
E. Right lung abscess

107. A 98 y.o. male patient complains of pain in the left lower limb which
intensifies on walking, feeling of cold and numbness in both feet. He has been
ill for 6 years. On examination: pale dry skin, hyperkeratosis. Hairy covering is
poorly developed on the left shin. "Furrow symptom " is positive on the left.
Pulse on foot arteries and popliteal artery isnt palpated, on the femoral artery
its weak. On the right limb the artery pulsation is reserved. What is the most
probable diagnosis?
A. Arteriosclerosis obliterans
B. Buergers disease (thromboangiitis obliterans)
C. Hemoral arthery thombosis
D. Obliterating endarteritis
E. Raynaulds disease

108. Examination of the corpse of a man who died from hanging reveals:
death spots disappear when pressed upon and restore after 50 seconds, rigor
mortis is moderately expressed only in the masticatory muscles as well as neck
and finger muscles, body temperature is of 31°C. The time of death:
A. 6-7 hours ago
B. 1-2 hours ago
C. 8-10 hours ago
D. 10-18 hours ago
E. 16-24 hours ago

109. After contusion of the right eye a patient complains of sudden loss of
vision with remaining light perception. Objectively: the eye is not irritated.
The cornea is transparent. Pupil reacts to light. The pupil area is black. The
fundus reflex is absent. What is the most likely cause of vision loss?
A. Hemophthalmia
B. Traumatic cataract
C. Optic nerve avulsion
D. Acute occlusion of retinal vessels
E. Retinal detachment

110. A 65-year-old patient complains of pain in the lumbar spine, moderate


disuria. He has been suffering from these presentations for about half a year.
Prostate volume is 45 cm3 (there are hypoechogenic nodes in both lobes,
capsule invasion). The rate of prostate-specific antigen is 60 ng/l. Prostate
biopsy revealed an adenocarcinoma. Which of the supplemental examination
methods will allow to determine the stage of neoplastic process in this
patient?
A. Computer tomography of pelvis
B. Excretory urography
C. Roentgenography of chest
D. Bone scintigraphy
E. Roentgenography of lumbar spine

111. X-ray picture of chest shows a density and an abrupt decrease in the
upper lobe of the right lung. The middle and lower lobe of the right lung
exhibit significant pneumatization. The right pulmonary hilum comes up to
the dense lobe. In the upper and middle parts of the left pulmonary field there
are multiple focal shadows. In the basal region of the left pulmonary field
there are clear outlines of two annular shadows with quite thick and irregular
walls. What disease is this X-ray pattern typical for?
A. Fibro-cavernous pulmonary tuberculosis
B. Abscessing pneumonia
C. Atelectasis of the right upper lobe
D. Peripheral cancer
E. Pancoast tumour

112. A patient complains of impaired far vision. Previously his eyes often
turned red and hurt. Objectively: the eyes are not irritated, the cornea is
transparent, anterior chambers are median deep, their liquid is transparent.
The iris of the right eye has not changed in colour, its pattern is unchanged.
The pupil is of irregular shape, scalloped. Biomicroscopy of the crystalline lens
reveals the areas of opacity and vacuoles. Make a diagnosis:
A. Complicated cataract of the right eye
B. Tetanic cataract of the right eye
C. Radiation cataract of the right eye
D. Senile cataract of the right eye
E. Diabetic cataract of the right eye

113. A 26-year-old patient complains of experiencing pain in the right hand


for 4 days. The pain arose at the site of corns on the palmar surface at the base
of the II and III fingers. Some time later the dorsum of hand became
edematic. I and III fingers are half-bent in the interphalangeal joints, the hand
looks like "rake". What is the most likely diagnosis?
A. Phlegmon of the second interdigital space of the right hand
B. Corn abscess of the right hand
C. Tendovaginitis
D. U-shaped phlegmon of the right hand
E. Adenophlegmon of the right hand

114. A 30-year-old patient had deep burn covering 30% of body 30 days ago.
Now he presents with continued fever, loss of appetite, night sweats. Burned
surface weakly granulates. What is the stage of burn disease?
A. Septicotoxemia
B. Convalescence
C. Secondary burn shock
D. Primary burn shock
E. Acute burn toxemia

115. A 29 y.o. patient was admitted to the hospital with acute girdle pain in
epigastric area, vomiting in 1 hour after the meal. On examination: pale,
acrocyanosis. Breathing is frequent, shallow. Abdominal muscles are tensed,
positive Schotkin-Blumbergs symptom. What is the maximal term to make a
diagnosis?
A. In 2 hours
B. In 1 hour
C. In 0,5 hours
D. In 3 hours
E. In 6 hours

116. A surgeon examined a 42-year-old patient and diagnosed him with


right forearm furuncle at the purulo-necrotic stage. The furuncle was lanced.
At the hydration stage the wound dressing should enclose the following
medication:
A. Hypertonic solution
B. Ichthyol ointment
C. Dimexide
D. Chloramine
E. Vishnevsky ointment

117. A 30 year old patient undergoes treatment because of Werlhofs disease.


Objectively: the patient is pale, there are petechial haemorrhages on the
extension surfaces of forearms. Ps is 92 bpm, AP is 100/60 mm Hg. The lower
edge of spleen is at a level with umbilicus. Blood count: erythrocytes:
2,8×1012/l, Hb - 90 g/l, Ht - 0,38, thrombocytes - 30cdot109/l. The patient is
being prepared for splenectomy. What transfusion medium should be chosen
in the first place for the preoperational preparation?
A. Thrombocytic mass
B. Washed erythrocytes
C. Native erythrocytic mass
D. Stored blood
E. Erythrocytic suspension

118. A 19 year old boy was admitted to a hospital with closed abdominal
trauma. In course of operation multiple ruptures of spleen and small intestine
were revealed. AP is falling rapidly, it is necessary to perform
hemotransfusion. Who can specify the patients blood group and rhesus
compatibility?
A. A doctor of any speciality
B. A laboratory physician
C. A traumatologist
D. An anaesthesilogist
E. A surgeon

119. A 36-year-old man was delivered to the surgical department an hour


after a road accident. His condition is getting worse: respiratory insufficiency
is progressing, there are cardiac abnormalities. Clinical and roentgenological
investigations revealed mediastinal displacement. What process has caused
this complication?
A. Valvular pneumothorax
B. Mediastinitis
C. Closed pneumothorax
D. Open pneumothorax
E. Subcutaneous emphysema

120. A victim of the car crash was brought to neurosurgery due to


craniocerebral trauma. During examination symptoms of focused injury of the
cerebrum are found, cerebrum contusion is suspected. What method of
examination is to be used?
A. Computed tomography of the cerebrum
B. X- ray of cranium bones
C. Ultrasound examination of vessels of the cerebrum
D. Eye-ground examination
E. Antiography of vessels of the cerebrum

121. A 35 y.o. female patient was admitted to the surgical department with
symptoms of ulcerative gastric hemorrhage. Its been the third hemorrhage for
the last 2 years. After conservative treatment vomiting with blood stopped,
hemoglobin elevated from 60 till 108 g/L. General condition became better.
But profuse vomiting with blood reoccured in 2-3- hours. Hemoglobin
decreased to 93,1 g/L then to 58,1 g/L. What is the tactics of treatment?
A. Urgent surgery
B. Taylors treatment
C. Conservative treatment
D. Deferred surgery
E. Conservative treatment with following surgery

122. A 39-year-old patient complains of a tumour on the anterior surface of


her neck. The tumour has been observed for 2 years. It is nonmobile and has
enlarged recently. The patient has a changed tone of voice, a sense of pressure.
Objectively: in the left lobe of the thyroid gland a 3 cm node is palpable; it is
very dense, tuberous, painless. Cervical lymph nodes are enlarged. Functional
status of the thyroid gland is unchanged. What is the most likely diagnosis?
A. Thyroid gland cancer
B. Chronic lymphomatous Hashimotos thyroiditis
C. Chronic fibrous Riedels thyroiditis
D. Nodular euthyroid goiter
E. Nodular hyperthyroid goiter
123. An unconscious victim in severe condition is brought to clinic. It is
known that the patient touched the bare wire with his hand and during 5
minutes was under the influence of an alternating current with voltage of 220
V. Physical exam: skin is pale, cold by touch. Breath is weakened, BP - 90/50
mm Hg, Ps - 60 bpm, arrhythmical. There are fields of necrosis of the skin on
the right hand and on the right foot. What is the preliminary diagnosis?
A. Electroburn of the right foot and right hand
B. Electrotrauma, acute cardiovascular failure
C. High voltaged electroburn of the right foot and shank
D. Vascular collapse due to electric current lesion

124. A 47-year-old patient complains about cough with purulent sputum,


pain in the lower part of the left chest, periodical body temperature rise. She
has been suffering from these presentations for about 10 years. Objectively:
"drumstick" distal phalanges. What examination would be the most
informative for making a diagnosis?
A. Bronchography
B. Bacteriological analysis of sputum
C. Survey radiograph of lungs
D. Bronchoscopy
E. Pleural puncture

125. A victim of a road accident, aged 44, is operated on account of


intraperitoneal haemorrhage. In which case can the patients blood from the
abdominal cavity be used for autotransfusion?
A. Stomach rupture
B. Splenic rupture
C. Small intestines rupture
D. Bladder rupture
E. Liver rupture

126. A man with a stab-wound of the right foot applied to doctor. He


stepped on the board with nail two hours ago. In the patient medical chart it is
stated that 3 years ago he passed the whole course of vaccination against
tetanus. What is the tactics of doctor to prevent tetanus in this case?
A. Do not conduct specific prophylaxis
B. Administer 3000 U of anti-tetanic serum
C. Administer 1 ml tetanus toxoid and 3000 U of anti-tetanic serum
D. Administer 0,5 ml of tetanus toxoid
E. Administer 1,0 ml of tetanus toxoid

127. A 42-year-old patient with acute haemorrhage and class III blood loss
underwent blood transfusion and got 1,8 l of preserved blood and erythromass
of the same group and Rh. After the transfusion the patient complained of
unpleasant retrosternal sensations, his arterial pressure dropped to 100/60
mm Hg, there appeared convulsions. Blood serum calcium was at the rate of
1,7 millimole/liter. What is the mechanism of this complication development?
A. Citrate binds calcium ions, hypocalcemia impairs myocardial
function
B. Citrate causes the development of metabolic acidosis
C. Citrate is cardiotoxic and nephrotoxic
D. The increased citrate rate causes convulsions
E. Citrate binds potassium causing severe hypokalemia

128. A 40-year-old woman with acute gastrointestinal hemorrhage was


transfused 400 ml of the preserved blood after having conducted all
compatibility tests. After hemotransfusion the condition of the patient became
worse. She started having headache, muscles pain, her temperature has risen
up to 38,8°C. What is the reason for such condition of the patient?
A. Pyrogenic reaction of medium severity
B. Allergic reaction
C. Air embolism
D. Bacterial and toxic shock development
E. Hemotransfusional shock development

129. A 27-year-old patient with a knife stomach wound has been delivered to
a hospital 4 hours after injury. Objectively: the patient is in grave condition.
Ps- 120 bpm, weak. AP- 70/40 mm Hg. Laparotomy revealed a lot of liquid
blood in the abdominal cavity. The patient has been found to have bleeding
from the mesenteric vessels of the small intestine. Damage to hollow organs
has not been revealed. What is the best way to restore the blood loss?
A. Autoblood reinfusiont
B. Transfusion of washed erythrocytes
C. Erythromass transfusion
D. Transfusion of fresh frozen plasma
E. Rheopolyglucinum transfusion

130. A patient presented to a hospital with a carbuncle of the upper lip. The
body temperature is 39°C. There is a pronounced edema of the upper lip and
eyelids. What is the surgeons tactics of choice?
A. Hospitalize in the surgical unit
B. Disclose the carbuncle and administer out-patient treatment
C. Administer out-patient course of antibiotics
D. Administer physiotherapy
E. Disclose the carbuncle and administer antibiotics
131. A patient, aged 25, suffering from stomach ulcer. Had a course of
treatment in the gastroenterological unit. 2 weeks later developed constant
pain, increasing and resistant to medication. The abdomen is painful in
epigastric area, moderate defence in pyloroduodenal area. Which
complication development aggravated the patients state?
A. Malignisation
B. Penetration
C. Haemorrhage
D. Stenosis
E. Perforation

132. A 62-year-old male patient complains of intense pain in the left leg that
suddenly arose three hours before, leg numbness and coldness. During the
year there has been pain in the leg while walking, hypersensitivity to cooling.
Objectively: the left foot and shin have marbled skin, subcutaneous veins are
collapsed. The foot is cold, active movements of the foot and toes are
preserved. Pulse is present only on the femoral artery. There is rough systolic
murmur above the artery. Make a provisional diagnosis:
A. Acute occlusion of the left femoral artery
B. Stenosis of the left popliteal artery
C. Acute arterial thrombosis ileofemoral
D. Acute thrombophlebitis
E. Occlusive disease

133. A patient complains about pyrosis and permanent pain behind his
breastbone. When he bends forward after eating there appears regurgitation.
Roentgenological examination revealed extrasaccular cardiofunctional hernia
of esophageal opening of diaphragm. Esophagoscopy revealed signs of reflux-
esophagitis. What is the necessary tretment tactics?
A. Operation in a surgical department
B. Conservative treatment in a surgical department
C. Treatment at a health resort
D. Conservative treatment in an outpatients clinic
E. Conservative treatment in the therapeutic hospital

134. A patient aged 18 with a cranial injury was in comatose state during
several hours. In post-comatose period gets tired quickly, non-productive in
dialog - in the beginning answers 2-3 questions, then gets tired and can not
understand the point of the question. Which psychotropic should be given to
the patient to prevent psychoorganic syndrome?
A. Nootropics
B. Tranquillisers
C. Antidepressants
D. Neuroleptics
E. Stimulators

135. Examination of a 38-year-old patient who had been hit with a blunt
object on the left side of chest revealed a fracture of the X rib with fragments
displacement, parietal pneumothorax. The patient complains of pain in the
left subcostal area. Objectively: the patient is pale, AP- 80/40 mm Hg, Ps-
138/min, of poor volume. USI reveals fluid in the left abdomen. Splenic
rupture is present. What treatment tactics should be chosen?
A. Drainage of the left pleural cavity followed by laparotomy
B. Immediate laparotomy and alcohol-novocaine block of the X rib
C. Left-sided thoracotomy immediately followed by laparotomy
D. Anti-schock actions followed by laparotomy after the arterial pressure rise
E. Immediate upper median laparotomy followed by drainage of the left pleural
cavity

136. On the 2nd day after a surgery for toxic mixed goiter IV a 35-year-old
patient complains of heart pain. ECG shows prolonged QT intervals.
Chvosteks and Trousseau symptoms cannot be clearly defined. The patient is
provisionally diagnosed with latent tetany. What study will allow to confirm
the diagnosis?
A. Determination of blood calcium and phosphor
B. Determination of potassim
C. Determination of thyrotropic hormone
D. Determination of sodium
E. Determination of thyroid hormones

137. A patient with bilateral hydrothorax has repeatedly undergone pleural


puncture on both sides. After a regular puncture the patients condition has
become worse: he presents with fever, chest pain. The next day, the attending
physician performing pleural puncture revealed some pus on the right. What
is the mechanism of acute right-sided empyema development?
A. Contact-and-aspiration
B. Hematogenous
C. Lymphogenous
D. Implantation
E. Aerial
138. A 25 y.o. patient was admitted with chest trauma. Clinical and X-ray
examination have revealed tense pneumothorax on the left. What emergency
treatment should be undertaken?
A. Pleural cavity drainage
B. Oxigenotherapy
C. Analgetics
D. Intubation
E. Intravenous infusions

139. A 38 y.o. patient lifted a heavy object that resulted in pain in the
lumbar part of spine irradiating to the posterior surface of his left leg. The
pain increases during changing body position and also in the upright position.
Examination revealed positive symptoms of tension. What is the preliminary
diagnosis?
A. Pathology of intercostal disks
B. Spinal cord tumor
C. Polyneuritis
D. Myelopathy
E. Arachnomyelitis

140. Two hours ago a 38-year-old patient got pain in his right shin. He was
diagnosed with popliteal artery embolism, acute arterial insufficiency of grade
I. What is the most appropriate therapeutic tactics?
A. Embolectomy
B. Destruction of the embolus by the catheter
C. Bypass grafting
D. Amputation at the middle of shin
E. Resection of the popliteal artery

141. A 38-year-old patient has suddenly developed pain in the left side of his
chest, suffocation. Objectively: moderately grave condition, Ps - 100/min, AP-
90/60 mm Hg, breath sounds on the left cannot be auscultated. Chest
radiography shows the collapse of the left lung up to 1/2. What kind of
treatment should be administered?
A. Passive thoracostomy
B. Operative therapy
C. Active thoracostomy
D. Rest, resolution therapy
E. Pleural puncture

142. A 47-year-old female patient complains of leg heaviness, fatigue when


standing and walking. This feeling disappears when she takes a horizontal
position. Objectively: dilatation of the superficial veins of the left shin and
thigh with pigmentation and trophic skin disorders. What functional test
should the examination be started with?
A. Trendelenburgs test
B. Pratt test 1
C. Pratt test 2
D. Sheinis test
E. Perthes test

143. A neonatologist examining a full-term mature baby revealed the


shortening and external rotation of the newborns lower extremity. Clinical
examination revealed positive Ortolani sign, symptom of non-vanishing pulse,
additional skin folds on the internal surface of thigh. What is the most likely
diagnosis?
A. Congenital hip dislocation
B. Femoral neck fracture
C. Fracture of the femoral shaft
D. Dysplasia of the hip joint
E. Varus deformity of the femoral neck

144. A child is being discharged from the surgical department after


conservative treatment of invagination. What recommendations should doctor
give to mother to prevent this disease recurrence?
A. Strict following of feeding regimen
B. Common cold prophilaxis
C. Gastro-intestinal disease prevention
D. Hardening of the child
E. Feces observation

145. A 10-year-old child has been admitted to a hospital with a closed


craniocerebral injury with suspected cerebral edema. The patient is in grave
condition, unconscious. The dyspnea, tachycardia, hypertension are present.
Muscle tone is increased, there is nystagmus, pupillary and oculomotor
reactions are disturbed. The mandatory component of intensive care is
dehydration. What diuretic is adequate in this case?
A. Furosemide
B. Spironolactone
C. Moduretic
D. Mannitol
E. Hydrochlorthiazide

146. A 40-year-old patient underwent an operation for a lumbar phlegmon.


Body temparature rose again up to 38°C, he got intoxication symptoms, there
was an increase of leukocyte number in blood. The wound that was nearly free
from necrotic tissues and full of granulations started to discharge pus, the
granulations turned pale. What complication developed in this patient?
A. Sepsis
B. Erysipeloid
C. Erysipelas
D. Putrid phlegmon
E. Allergic reaction

147. After a car accident a 37-year-old patient has an acute pain and
bleeding in the middle third of his right thigh. Objectively: there is a wound on
the anterior surface of the right thigh with massive bleeding, abnormal
mobility at the level of the middle third of the thigh. The first aid is to be
started with:
A. Digital occlusion of the femoral artery
B. Tourniquet application
C. Venipuncture and intravenous infusion of polyglycine
D. Immobilization of the extremity with a transportation splint
E. Injection of pain-killers

148. A 42-year-old builder consulted a doctor about a foot injury with a nail
that he got in the morning of the same day. The wound was washed with
water. Three years ago he was vaccinated against tetanus. Examination
established satisfactory condition of the patient. The left foot was slightly
edematic, there was a stab wound on the sole. In order to prevent tetanus it is
primarily required to:
A. Give an intravenous injection of 0,5 ml of tetanus anatoxin
B. Treat the wound with suds
C. Administer a course of antibiotic therapy
D. Give an intravenous injection of 1 ml of tetanus anatoxin, 3000 IU of
antitetanus serum
E. Give an intravenous injection of 3000 IU of antitetanus serum

149. A 9-year-old boy fell from a tree and hit the occipital region, there was
a momentary loss of consciousness. Objectively: the childs condition is
satisfactory, he complains of the headache and dizziness. The X-ray of skull
reveals a comminuted depressed fracture of occipital bone in the region of
inion. What treatment is indicated for the patient?
A. Surgical intervention
B. Hemostatic therapy
C. Anti-inflammatory therapy
D. Therapeutic lumbar punctures
E. Complex conservative treatment
150. A 40-year-old female patient has a history of rheumatism. She
complains about acute pain in her left eye, especially at night, vision
impairment, photophobia, lacrimation. The patient cannot suggest any
reasons for the disease. Objectively: weak pericorneal injection, flattening of
iris relief, iris discoloration. What is the most likely diagnosis?
A. Iridocyclitis
B. Choroiditis
C. Acute attack of glaucoma
D. Iritis
E. Keratitis

151. 14 days ago a 49-year-old patient was operated on for perforated


appendicitis and disseminated fibrinopurulent peritonitis. The postoperative
period was uneventful. On the 9th day the patient presented with low-grade
fever, abdominal pain, frequent liquid stools. USI of the left mesogastrium
reveals a fluid formation 9x10 cm large. In blood: leukocytosis with the left
shift. What is your provisional diagnosis?
A. Interloop abscess
B. Left kidney cyst
C. Spleen abscess
D. Abdominal cavity tumour
E. Liver abscess

152. A patient, aged 81, complains of constant urinary excretion in drops,


feeling of fullness in the lower abdomen. On examination: above pubis
there is a spherical protrusion, over which there is a dullness of percussion
sound, positive suprapubic punch. What symptom is observed in this patient?
A. Paradoxal ischuria
B. Urinary incontinence
C. Enuresis
D. Pollakiuria
E. Dysuria

153. A 67 y.o. patient complains of dyspnea, breast pain, common weakness.


He has been ill for 5 months. Objectively: t0- 37,3°C, Ps- 96/min. Vocal
tremor over the right lung cannot be determined, percussion sound is dull,
breathing cannot be auscultated. In sputum: blood diffusively mixed with
mucus. What is the most probable diagnosis?
A. Lung cancer
B. Focal pulmonary tuberculosis
C. Exudative pleuritis
D. Macrofocal pneumonia
E. Bronchoectatic disease
154. A 52 year old patient complains about pain in the right part of her
chest, dyspnea, cough with a lot of foul-smelling albuminoid sputum in form
of "meat slops". Objectively: the patients condition is grave, cyanosis is
present, breathing rate is 31/min, percussion sound above the right lung is
shortened, auscultation revealed different rales. What is the most probable
diagnosis?
A. Lung gangrene
B. Chronic pneumonia
C. Pleura empyema
D. Lung abscess
E. Multiple bronchiectasis

155. 3 hours ago a 65-year-old female patient felt sharp abdominal pain
irradiating to the right scapula, there was a single vomiting. She has a history
of rheumatoid arthritis. Objectively: pale skin, AP- 100/60 mm Hg, Ps-
60/min. Abdomen is significantly painful and tense in the epigastrium and
right subcostal areat, there are positive symptoms of parietal peritoneum
irritation over the right costal arch, that is tympanitis. What is the tactics of an
emergency doctor?
A. To take the patient to the surgical hospital
B. To take the patient to the rheumatological department
C. To perform gastric lavage
D. To inject pain-killers and watch the dynamics
E. To inject spasmolytics

156. A 15 year old patient suffers from headache, nasal haemorrhages, sense
of lower extremity coldness. Objectively: muscles of shoulder girdle are
developed, lower extremities are hypotrophied. Pulsation on the pedal and
femoral arteries is sharply dampened. AP is 150/90 mm Hg, 90/60 on the
legs. Systolic murmur can be auscultated above carotid arteries. What is the
most probable diagnosis?
A. Aorta coarctation
B. Aortal stenosis
C. Aorta aneurism
D. Aortal insufficiency
E. Coarctation of pulmonary artery

157. A 15 y.o. patient has developmental lag, periodical skin yellowing.


Objectively: spleen is 16?12?10 cm large, holecistolithiasis, skin ulcer on the
lower third of his left crus. Blood count: erythrocytes - 3,0×1012/L, Hb- 90
g/L, C.I.- 1,0, microspherocytosis, reticulocytosis. Blood bilirubin - 56
mmole/L, indirect bilirubin - 38 mmole/L. Choose the way of treatment:
A. Splenectomy
B. Omentohepatopexy
C. Portocaval anastomosis
D. Spleen transplantation
E. Omentosplenopexy

158. A 72-year-old patient complains of pain and bleeding during


defecation. Digital rectal investigation revealed a tumour of the anal canal.
After verification of the diagnosis the patient was diagnosed with squamous
cell carcinoma. The secondary (metastatic) tumour will be most probably
found in:
A. Lungs
B. Mediastinum
C. Brain
D. Liver
E. Pelvic bones

159. A patient with frostbite of both feet was delivered to the admission
ward. What actions should be taken?
A. To apply a bandage, to introduce vasodilating medications
B. To administer cardiac medications
C. To rub feet with snow
D. To apply an alcohol compress
E. To put feet into hot water

160. 10 years ago a patient had a fracture in the middle one-third of his left
femoral bone, and during the last 7 years he has been having acute
inflammation in the area of old fracture accompanied by formation of a fistula
through which some pus with small fragments of bone tissue is discharged.
After a time the fistula closes. What complication of the fracture is it?
A. Chronic osteomyelitis
B. Soft tissue phlegmon
C. Trophic ulcer
D. False joint
E. Bone tuberculosis

161. A patient has a stab wound on his right foot. On the fourth day after
injury the patients body temperature rose up to 38°C, inguinal lymph nodes
became enlarged and painful, skin over them reddened. What complication
might be suspected?
A. Lymphadenitis
B. Phlegmon
C. Lymphangitis
D. Tetanus
E. Erysipelas

162. During a surgery on a 30-year-old patient a dark ileo-ileal


conglomerate was discovered, the intussusceptum intestine was considered to
be unviable. The intussuscipiens intestine was dilated to 7-8 cm, swollen, full
of intestinal contents and gases. What pathology led to the surgery?
A. Invagination (combined) obstruction
B. Strangulation obstruction
C. Paralytic obstruction
D. Spastic obstruction
E. Obturation obstruction

163. A 52 year old patient complains about headache, weakness of his upper
left extremity. Neurological symptoms become more intense during physical
stress of the left extremity. Pulsation on the arteries of the left extremity is
sharply dampened but it remains unchanged on the carotid arteries. What is
the most probable diagnosis?
A. Occlusion of the left subclavicular artery, steal syndrome
B. Raynauds syndrome
C. Occlusion of brachiocephalic trunk
D. Takayasus disease
E. Thoracal outlet syndrome

164. A 43-year-old patient had been admitted to a hospital with clinical


presentations of ischiorectal periproctitis. On the 12th day of treatment the
patients condition deteriorated: there was an increase in the rate of
intoxication and hepatic failure, the body temperature became hectic, AP was
100/60 mm Hg. USI of liver revealed a hydrophilic formation. In blood: WBCs
- 19,6cdot109/l, RBCs.- 3,0×1012/l, Hb- 98 g/l. What complication was
developed?
A. Liver abscess
B. Pylephlebitis
C. Liver necrosis
D. Budd-Chiari syndrome
E. Hepatic cyst

165. A 57 year old patient abruptly lost the sight of one eye. Examination
revealed that his sight made up 0,02 excentrically, eye fundus has
haemorrhages of different forms and sizes ("squashed tomato" symptom).
Disc of optic nerve is hyperemic. In anamnesis general vascular pathology is
recorded. Direct-acting anticoagulants were administered. What is the most
probable diagnosis?
A. Thrombosis of central vein of retina
B. Hypertensive angioneuropathy
C. Diabetic retinopathy
D. Embolism of central artery of retina
E. Hypertensive angiopathy

166. Half an hour ago a 67-year-old patient with a hernia picked up a heavy
object, which caused acute pain in the region of hernia protrusion, the hernia
couldnt be reduced. Objectively: the hernia in the right inguinal region is
roundish, tight, moderately painful; during palpation it was reduced back to
the cavity, the pain was gone. Specify the further medical tactics:
A. Inpatient surveillance
B. Planned hernioplasty a month later
C. Planned hernioplasty a year later
D. Immediate hernioplasty
E. Immediate laparotomy

167. A 55 year old patient felt suddenly sick in a hospital corridor, he was
immediately examined by a doctor. Examination revealed that the patients
skin was pale, autonomous respiration was absent, pulse on carotid arteries
couldnt be felt, pupils were mydriatic. What action should be taken at the
beginning of cardiac resuscitation?
A. Precordial thump
B. Defibrillation
C. Closed-chest cardiac massage
D. Mouth-to-mouth ventilation
E. Restoration of airway patency

168. A 30-year-old patient has been admitted to the intensive care unit for
multiple bee stings. The skin is covered with cold sweat. The pulse is felt just
on the carotid arteries, 110 bpm, respiration is 24/min, rhythmic, weakened.
Which drug should be given in the first place?
A. Adrenalin hydrochloride intravenously
B. Adrenalin hydrochloride intramuscularly
C. Prednisolone intravenously
D. Dopamine intravenously
E. Tavegil intravenously

169. A 25-year-old victim of a road accient complains of chest pain, dyspnea.


Objectively: the patient is in a grave condition, Ps- 120/min, AP- 90/70 mm
Hg.There is pathological mobility of fragments of III-V ribs on the right.
Percussion reveals a box sound over the right lung, breathing sounds cannot
be auscultated on the right. What examination should be administered in the
first place?
A. X-ray of chest organs
B. Pleural puncture
C. Bronchoscopy
D. USI of chest organs
E. Thoracoscopy

170. After the pneumatic dilatation of oesophageal stricture, a patient


developed acute retrosternal pain getting worse when throwing the head back
and swallowing. Objectively: dilatation of the neck veins, dropped beat pulse,
signs of purulent intoxication, oliguria, emphysema of the upper portion of
chest. What disease can be suspected?
A. Suppurative mediastinitis
B. Acute myocardial infarction
C. Spontaneous pneumothorax
D. Thrombosis of the superior vena cava
E. Pleural empyema

171. A 6-year-old girl drank some coloured fizzy drink which gave her a
feeling of pressure in the throat. 30 minutes later the childs lips got swollen,
then edema gradually spread over the whole face, laryngeal breathing became
difficult. The child is excited. Ps- 120/min, breathing rate - 28/min, breathing
is noisy, indrawing of intercostal spaces is observed. What basic aid is most
appropriate for the restoration of laryngeal breathing?
A. Corticosteroids
B. Sedative drugs
C. Antibacterial drugs
D. Conicotomy
E. Tracheostomy

172. An 8-month-old baby has had problems with nasal breathing and
muco-purulent discharge from the nose for a week. Examination reveals a
rhinedema, muco-purulent discharges from the middle nasal meatus as well
as on the back of pharynx. What disease are these symptoms most typical for?
A. Ethmoiditis
B. Maxillary sinusitis
C. Hemisinusitis
D. Frontitis
E. Sphenoiditis

173. A 28-year-old female patient has been admitted to a hospital. She


states to be ill for 12 years. On examination she has been diagnosed with
bronchiectasis with affection of the left lower lobe of lung. What is the optimal
treatment tactics for this patient?
A. Left lower lobectomy
B. Antibiotic therapy
C. Bronchopulmonary lavage
D. Active drainage of the left pleural cavity
E. Left-sided pneumoectomy

174. 4 weeks after myocardial infarction a 56-year-old patient developed


acute heart pain, pronounced dyspnea. Objectively: the patients condition is
extremely grave, there is marked cyanosis of face, swelling and throbbing of
neck veins, peripheral pulse is absent, the carotid artery pulse is rhythmic, 130
bpm, AP is 60/20 mm Hg. Auscultation of heart reveals extremely muffled
sounds, percussion reveals heart border extension in both directions. What is
the optimal treatment tactis for this patient?
A. Pericardiocentesis and immediate thoracotomy
B. Puncture of the pleural cavity on the left
C. Pleural cavity drainage
D. Conservative treatment, infusion of adrenomimetics
E. Oxygen inhalation

175. Esophagus wall of a 72 year old patient with severe concomitant


pathology was injured during urgent fibroesophagogastroscopy. This resulted
in progressing of acute respiratory failure and collapse of the left lung. What
aid should be rendered?
A. Drainage of pleural cavity by Bullaux method, mediastinum
drainage, antibacterial therapy
B. Endoscopic closure of esophagus wound, drainage
C. Left-sided thoracotomy, closure of esophagus and mediastinum wound
D. Buelaus drainage of pleural cavity, antibacterial therapy
E. Left-sided thoracotomy, closure of esophagus wound

176. A 17-year-old patient complains of pain in the area of the left knee
joint. Soft tissues of thigh in the affected region are infiltrated, joint function
is limited. X-ray picture of the distal metaepiphysis of the left femur shows a
destruction focus with periosteum detachment and Codmans triangle found at
the defect border in the bone cortex. X-ray of chest reveals multiple small
focal metastases. What treatment is indicated?
A. Palliative chemotherapy
B. Disarticulation of the lower extremity
C. Amputation of the lower extremity
D. Radioiodine therapy
E. Distance gamma-ray therapy
177. A 35-year-old victim of a road accident has got an injury of the right
side of his chest. Objectively: respiration rate - 28-30/min, respiration is
shallow, restricted respiratory excursion and acrocyanosis are present. Ps- 110
bpm, AP- 90/60 mm Hg. Respiratory sounds over the right lung cannot be
auscultated. Chest radiograph shows fractures of the VI-VII ribs on the right,
the right pleural cavity contains both air and fluid, with the fluid at about the
level of the V rib, the shadow of the mediastinum is displaced to the left. What
first aid should be provided to the victim?
A. Puncture of the pleural cavity
B. Antibiotic administration
C. Artificial ventilation of lungs
D. Urgent thoracotomy
E. Vagosympathetic blockade

178. On the first day after a surgery for diffuse toxic goiter a patient
developed difficulty breathing, cold sweats, weakness. Objectively: pale skin,
body temperature - 38,5°C, RR - 25/min, Ps- 110/min, AP- 90/60 mm Hg.
What early postoperative complication occurred in the patient?
A. Thyrotoxic crisis
B. Acute thyroiditis
C. Compression of the trachea by the hematoma
D. Hypothyroid crisis
E. Postoperative tetany

179. A 20-year-old patient complains of pain in the left lumbar region,


arterial pressure rise up to 160/110 mm Hg. USI revealed that the structure
and size of the right kidney were within age norms, there were signs of 3
degree hydronephrotic transformation of the left kidney. Doppler examination
revealed an additional artery running to the lower pole of the kidney.
Excretory urogram shows a narrowing in the region of ureteropelvic junction.
Specify the therapeutic tactics:
A. Surgical intervention
B. Kidney catheterization
C. Administration of beta-blockers
D. Administration of spasmolytics
E. Administration of ACE inhibitors

180. A hospital admitted a patient with coarse breathing (obstructed


inspiration), skin cyanosis, tachycardia and arterial hypertension. He has a
history of bronchial asthma. An hour ago he was having salbutamol inhalation
and forgot to remove a cap that was aspired while taking a deep breath. What
measures should the doctor take?
A. To perform the Heimlich manoever
B. To send for an anesthesiologist and wait for him
C. To perform conicotomy immediately
D. To use an inhalation of beta_2-adrenoceptor agonist
E. To make a subcutaneous injection of dexamethasone

181. An emergency physician arrived to provide medical care for a hangman


taken out of the loop by his relatives. The doctor revealed no pulse in the
carotid arteries, absence of consciousness, spontaneous breathing and corneal
reflexes; cadaver spots on the back and posterior parts of extremities. A
person can be declared dead if the following sign is present:
A. Cadaver spots
B. Absence of spontaneous breathing
C. Pulselessness
D. Unconsciousness
E. Absence of corneal reflexes

182. A patient undergoing treatment for the left-sided destructive


pneumonia presents with deterioration of his general condition, progressing
dyspnea, cyanosis. Objectively: the left side of chest is not involved in
respiration, breathing sounds cannot be auscultated. Radiograph shows a
shadow reaching the 5 rib with a horizontal fluid level and a radiolucency
above it, the mediastinum is displaced to the right. What is the medical
tactics?
A. Thoracostomy
B. Endotracheal intubation
C. Emergency bronchoscopy
D. Infusion and antibacterial therapy
E. Open thoracotomy

183. A 26-year-old patient consulted a doctor abut sore throat, fever up to


38,2°C. A week ago, the patient had angina, didnt follow medical
recommendations. On examination, the patient had forced position of his
head, trismus of chewing muscles. Left peritonsillar region is markedly
hyperemic, swollen. What is the provisional diagnosis?
A. Left-sided peritonsillar abscess
B. Phlegmonous angina
C. Tonsil tumour
D. Diphtheria of the pharynx
E. Meningitis

184. A 4 month old child was admitted to a surgical department 8 hours


after the first attack of anxiety. The attacks happen every 10 minutes and last
for 2-3 minutes, there was also one-time vomiting. Objectively: the childs
condition is grave. Abdomen is soft, palpation reveals a tumour-like formation
in the right iliac area. After rectal examination the doctors finger was stained
with blood. What is the most probable diagnosis?
A. Ileocecal invagination
B. Pylorostenosis
C. Wilms tumour
D. Gastrointestinal haemorrhage
E. Helminthic invasion

185. A 3-year-old male patient consulted a family doctor 2 months after he


had been operated for an open fracture of brachial bone. Objectively: the
patients condition is satisfactory, in the region of the operative wound there is
a fistula with some purulent discharge, redness, fluctuation. X-ray picture
shows brachial bone destruction with sequestra. What complication arose in
the postoperative period?
A. Posttraumatic osteomyelitis
B. Wound abscess
C. Suture sinus
D. Posttraumatic phlegmon
E. Hematogenic osteomyelitis

186. A 40-year-old female patient complains of having a bulge on the


anterior surface of neck for 5 years. Objectively: Ps- 72 bpm, arterial pressure
- 110/70 mm Hg, in the right lobe of thyroid gland palpation reveals a mobile
4x2 cm node, the left lobe is not palpable, the basal metabolic rate is 6%. What
is the most likely diagnosis?
A. Nodular euthyroid goiter
B. Nodular hyperthyroid goiter
C. Mixed euthyroid goiter
D. The median cervical cyst
E. Riedels thyroiditis

187. Forensic medical expertise of corpse of a newborn revealed: body


weight 3500 g, body length 50 cm, the umbilical cord was smooth, moist,
glossy, without any signs of drying. Hydrostatic tests were positive. The test
results are the evidence of:
A. Live birth
B. Primary atelectasis
C. Stillbirth
D. Secondary atelectasis
E. Hyaline membrane disease
188. A week ago a 65-year-old patient suffered an acute myocardial
infarction, his general condition deteriorated: he complains of dyspnea at rest,
pronounced weakness. Objectively: edema of the lower extremities, ascites is
present. Heart borders are extended, paradoxical pulse is 2 cm displaced from
the apex beat to the left. What is the most likely diagnosis?
A. Acute cardiac aneurysm
B. Acute pericarditis
C. Recurrent myocardial infarction
D. Cardiosclerotic aneurysm
E. Pulmonary embolism

189. A 75-year-old male patient complains of slight pain in the right iliac
region. The abdominal pain arose 6 days ago and was accompanied by nausea.
Surgical examination revealed moist tongue, Ps- 76 bpm. AP- 130/80 mm Hg.
Abdomen was soft, slightly painful in the right iliac region on deep palpation,
the symptoms of the peritoneum irritation were doubtful. In blood: RBCs -
4,0*10^12/l, Hb- 135 g/l, WBCs - 9,5*10^9/l, stab neutrophils - 5%,
segmentonuclear - 52%, lymphocytes - 38%, monocytes - 5%, ESR - 20 mm/h.
Specify the doctors further tactics:
A. Emergency operation for acute appendicitis
B. Send the patient home
C. Hospitalization, dynamic surveillance
D. Refer the patient to a district therapist
E. Administration of additional examination: abdominal ultrasound, x-ray
contrast study of the gastrointestinal tract

190. A 56-year-old patient was undergoing a surgery for suture repair of


perforated ulcer. During the operation the cardiomonitor registered
ventricular fibrillation. The first-priority measure should be:
A. Electrical defibrillation
B. Injection of calcium chloride
C. Injection of lidocaine
D. Injection of adrenalin
E. Injection of atropine

191. A 78-year-old patient complains of severe pain in the lower abdomen,


perineum and rectum; intense urge to urinate and inability to urinate without
assistance. Abdomen palpation reveals a moderately painful globular
formation above the pubis, there is percussion dullness over the formation.
What is the most likely diagnosis?
A. Acute urinary retention
B. Chronic urinary retention
C. Paradoxical ischuria
D. Cystitis
E. Chronic incomplete urinary retention

192. A 24-year-old patient had been delivered to the thoracic department


with a chest injury, a fracture of the IV, V, VI ribs on the right. Plan
radiography shows the fluid level in the pleural cavity reaching the III rib on
the right. Puncture blood contained clots. What is the optimal treatment
tactics?
A. Emergency thoracotomy
B. Pleural puncture
C. Hemostatic therapy
D. Medical thoracoscopy
E. Thoracentesis and thoracostomy

Krok 2 – 2013 Therapy Base


1. A 47 y.o. woman complains of having paroxysmal headaches for the last 5
years. The pain is one-sided, intense, localised in frontal region of head,
accompanied by nausea and stomach discomfort, begins one of a sudden.
Onset is usually preceded by vision reduction. Anamnesis gives evidence of
periodical AP rise, but at the moment the woman doesnt take any medicines.
Inbetween the onsets of headache her state is satisfactory. Objectively: high-
calorie diet (body weight index - 29), AP- 170/95 mm Hg. Neurologic state has
no pecularities. What is the most probable diagnosis?
A. Migraine
B. Hypertensive encephalopathy
C. Epilepsy
D. Chronic subdural hematoma
E. Benign intracranial hypertension

2. A 7 y.o. boy suddenly felt pain in his right knee, it became edematic. The day
before he took part in a cross-country race. Family anamnesis has no data
about hemophilia and bleeding sickness. Objectively: body temperature is
37,5°C. The knee is painful, hot to the touch, edematic with local tissue
tension over it. Blood count: Hb- 123 g/L, leukocytes - 5,6109/L,
thrombocytes - 354109/L, prothrombin time - 12 seconds (normally 10-15
seconds), partly activated thromboplastin time - 72 seconds (normally 35-45
seconds). Hemorrhage time is normal, VIII:C factor is 5% of norm. What is
the most probable diagnosis?
A. Hemophilia A
B. Schoenlein-Henoch disease
C. Hemophilia B
D. Vitamin K deficiency
E. Thrombocytopenia
3. On the 3rd day after the acute anterior myocardial infarction a 55 y.o. patient
complains of dull ache behind his breast bone, that can be reduced by bending
forward, and of dyspnea. Objectively: AP- 140/180 mm Hg, heart sounds are
dull. ECG results: atrial fibrillation with frequence of ventricular contractions
at the rate of 110/min, pathological Q wave and S-T segment raising in the
right chest leads. The patient refused from thrombolisis. What is the most
probable diagnosis?
A. Acute pericarditis
B. Dresslers syndrome
C. Tietzes syndrome
D. Pulmonary embolism
E. Dissecting aortic aneurysm

4. A 54 y.o. man was admitted to the hospital with complaints of sudden intense
headache in occipital region and vomiting. In the medical hystory: moderate
arterial hypertension, the patient was taking hydrochlorothiazide. Three days
ago he consulted a therapeutist about intense headache that was suppressed
by an analgetic. Objectively: consciousness is confused, left pupil is mydriatic.
Evident photophobia and tension of neck muscles. Left-side hemiparesis with
increased muscle tonus and reflexes. Body temperature is low, rash is absent.
AP- 230/130 mm Hg, Ps- 50 bpm, BR- 12/min. What is your preliminary
diagnosis?
A. Acute subdural hematoma
B. Myasthenia
C. Migraine
D. Acute bacterial meningitis
E. Disseminated sclerosis

5. A 51 y.o. women was taken to the emergency department in convulsive status


epilepticus. The first means of medical management should be:
A. Ensuring that the airway is open and the patient is oxygenating
B. Inserting a tongue blade
C. Injecting 5 mg of diazepam followed by a loading dose of phenytoin
D. Inducing pentobarbital coma
E. Administering an instravenous bolus of 50% dextrose

6. A 38 y.o. man complains of having occasional problems with swallowing of


both hard and fluid food for many months. Sometimes he feels intense pain
behind his breast bone, epecially after hot drinks. There are asphyxia onsets at
night. He has not put off weight. Objectively: his general condition is
satisfactory, skin is of usual colour. Examination revealed no changes of
gastrointestinal tract. X-ray picture of thorax organs presents esophagus
dilatation with level of fluid in it. What is the preliminary diagnosis?
A. Esophagus achalasia
B. Esophagus candidosis
C. Gastroesophageal reflux
D. Myastenia
E. Cancer of esophagus

7. A 35 y.o. woman consulted a doctor about occasional pains in paraumbilical


and iliac region that reduce after defecation or passage of gases. Defecation
takes place up to 6 times a day, stool is not solid, with some mucus in it.
Appetite is normal, she has not put off weight. First such symptoms appeared
1,5 year ago, but colonoscopy data reveals no organic changes. Objectively:
abdomen is soft, a little bit painful in the left iliac region. Blood and urine are
normal. What is the preliminary diagnosis?
A. Irritable bowels syndrome
B. Celiac disease
C. Pseudomembranous colitis
D. Dispancreatism
E. Crohns disease

8. The physician must undertake measures for primary prophylaxis of iron


deficiency anemia. Which of the following categories of patient are subject to
such primary prophylactic measures?
A. Pregnant women
B. Patients after 60
C. Patients after operation
D. Workers of industrial enterprises
E. All children

9. A patient with unstable angina pectoris was given the following complex
treatment: anticoagulants, nitrates, /alpha-adrenoblockers. However on the
third day of treatment the pain still romains. Which in vestigation shoud be
carried out to establish diagnosis?
A. Coronarography
B. Esophageal electrocardiac stimulator
C. Myocardial scintigraphy
D. Stress-echocardiogram
E. Test with dosed physical exercises

10. The 28 y.o. woman applied to doctor because of limited loss of the hair. In the
anamnesis - she had frequent headache indisposition, arthromyalgia, fever,
irregular casual sexual life, drug user. RW is negative. What examination must
be done first?
A. Examination for HIV
B. Examination for neuropathology
C. Examination for fungi
D. Examination for trichomoniasis
E. Examination for gonorrhea

11. A 35 y.o. woman was admitted to thoracic surgery department with fever up to
400C, onset of pain in the side caused by deep breathing, cough with
considerable quantity of purulent sputum and blood with bad smell. What is
the most likely diagnosis?
A. Abscess of the lung
B. Complication of liver echinococcosis
C. Actinomycosis of lungs
D. Pulmonary tuberculosis
E. Bronchiectatic disease

12. A 67 y.o. patient complains of palpitation, dizziness, noise in ears, feeling of


shortage of air. Objectively: pale, damp skin. Vesicular respiration, respiratory
rate- 22 per min, pulse- 200 bpm, AP- 100/70 mm Hg. On ECG: heart rate-
200 bmp, ventricular complexes are widened, deformed, location of segments
ST and of wave T is discordant. The wave P is not changed, superimposes
QRST, natural conformity between P and QRS is not present. What kind of
arrhythmia is present?
A. Paroxismal ventricular tachycardia
B. Atrial tachycardia
C. Atrial flutter
D. Sinus tachycardia
E. Ventricular extrasystole

13. Male 30 y.o., noted growing fingers and facial scull, changed face. Complains
of poor eyesight, weakness, skin darkening, loss of body weight. X-ray shows
broadening of sella turcica, thinning of tuberculin sphenoidale, signs of
increased intracranial pressure. What diagnosis can you make?
A. Adenoma of hypophysis
B. Optico - hiasmatic arachnoiditis
C. Encephalitis of truncus
D. Adrenal gland tumor
E. Tumor of pondo-cerebellar corner

14. A patient complains of a tormental (agonizing) cough with expectoration of up


to 600 ml/daily purulent chocolatecolor sputum with a decay smell. Onset of
illness was abrupt, t0- 390C, fever of irregular type. There is the area of
darkening with a cavity in a center on X-ray film, with irregular contours and
level of liquid. What disease is the question?
A. Gangrene of lung
B. Lobar pneumonia
C. Bronchiectatic illness
D. Tuberculosis
E. Pneumonia complicated by an abscess

15. A 24 y.o. patient complains of nausea, vomiting, headache, shortness of


breath. He had an acute nephritis being 10 y.o. Proteinuria was found out in
urine. Objectively: a skin is grey-pale, the edema is not present. Accent of II
tone above aorta. BP 140/100-180/100 mm Hg. Blood level of residual N_2-
6,6 mmol/L, creatinine- 406 mmol/L. Days diuresis- 2300 ml, nocturia.
Specific density of urine is 1009, albumin- 0,9 g/L, WBC- 0-2 in f/vis. RBC.-
single in f/vis., hyaline casts single in specimen. Your diagnosis?
A. Chronic nephritis with violation of kidney function
B. Nephrotic syndrome
C. Stenosis of kidney artery
D. Feochromocitoma
E. Hypertensive illness of the II degree

16. A 33 y.o. male patient was admitted to a hospital. A patient is pale, at an


attempt to stand up he complains of strong dizziness. There was vomiting like
coffee-grounds approximately hour ago. BP- 90/60 mm Hg., pulse- 120
b/min. In anamnesis, a patient has suffered from ulcer of the stomach,
painless form during 4 years. An ulcer was exposed at gastrofiberoscopy. Your
diagnosis:
A. Ulcer of stomach, complicated with bleeding
B. Ulcer of duodenum, complicated with bleeding
C. Acute pleurisy
D. Acute myocardial infarction, abdominal form
E. Erosive gastritis

17. A 48-year-old patient complains of heaviness in the right hypochondrium,


itching of the skin.He had been treated in infectious diseases hospital
repeatedly due to icterus and itch. On physical exam: meteorism, ascitis,
dilation of abdominal wall veins, protruded umbilicus, spleen enlargement.
What can be diagnosed in this case?
A. Liver cirrhosis
B. Cancer of the head of pancreas
C. Viral hepatitis B
D. Gallstones
E. Cancer of the liver

18. A 25-year-old man has facial edema, moderate back pains. His temperature is
37,5OC, BP 180/100 mm Hg, hematuria [up to 100 in v/f], proteinuria [2,0
g/L], hyaline casts - 10 in v/f., specific gravity -1020. The onset of the disease
is probably connected with acute tonsillitis that started 2 weeks ago. What is
the most probable diagnosis?
A. Acute glomerulonephritis
B. Chronic glomerulonephritis
C. Cancer of the kidney
D. Acute pyelonephritis
E. Urolithiasis

19. In the development of the inflammation processes glucocorticoids reduce the


level of certain most important active enzyme. It results also in the reducing of
the synthesis of prostaglandins and leucotrienes which have a key role in the
development of inflammation processes. What is the exact name of this
enzyme?
A. Phospholipase A2
B. Lipoxygenase
C. Arachidonic acid
D. Cyclooxygenase – 1
E. Cyclooxygenase – 2

20. A 30 y.o. female with rheumatoid arthritis of five years duration complains of
pain in the frist three fingers of her right hand over past 6 weeks. The pain
seems especially severe at night often awakening her from sleep.The most
likelly cause is?
A. Carpal tunnel syndrome
B. Sensory peripheral neuropathy
C. Rheumatoid arthritis without complication
D. Rheumatoid vasculitis
E. Atlanto-axial sublaxation of cervical spine

21. A 19-year-old man has suffered from moderate mental retardation since
childhood. The patient is illiterate, can take care of himself, do simple
household work and other kinds of easy work under supervision. What does
his rehabilitation [tertiary prevention] require?
A. All the below mentioned
B. Supervision of a social worker
C. Supervision of relatives (if any)
D. None of the above mentioned
E. Physical work under supervision

22. A 38 y.o. patient was urgently admitted to the hospital with complaints of
sudden weakness, dizziness, loss of consciousness, body weight loss, nausea,
vomiting, severe pain in epigastric area, diarrhea, skin hyperpigmentation.
What is the most probable diagnosis?
A. Addisonic crisis
B. Acute gastroenteritis
C. Scleroderma
D. Pellagra
E. Meningoencephalitis

23. An unconscious patient presents with moist skin, shallow breathing. There are
signs of previous injection on the shoulders and hips. BP- 110/70 mm Hg.
Tonus of skeletal muscles and reflexes are increased. Cramps of muscles of the
extremities are seen. What is the most likely disorder?
A. Hypoglycemic coma
B. Hyperosmolar coma
C. Stroke
D. Hyperlactacidotic coma
E. Hyperglycemic coma

24. A patient was admitted to the hospital on the 7th day of the disease with
complaints of high temperature, headache, pain in the muscles, especially in
calf muscles. Dermal integuments and scleras are icteric. There is
hemorrhagic rash on the skin. Urine is bloody. The patient was fishing two
weeks ago. What is the most likely diagnosis?
A. Leptospirosis
B. Trichinellosis
C. Salmonellosis
D. Yersiniosis
E. Brucellosis

25. A 60-year-old woman, mother of 6 children, developed a sudden onset of


upper abdominal pain radiating to the back, accompanied by nausea,
vomiting, fever and chills. Subsequently, she noticed yellow discoloration of
her sclera and skin. On physical examination the patient was found to be
febrile with temp of 38,9°C, along with right upper quadrant tenderness. The
most likely diagnosis is:
A. Choledocholithiasis
B. Benign biliary stricture
C. Carcinoma of the head of the pancreas
D. Choledochal cyst
E. Malignant biliary stricture
26. A 42-year-old woman suffers from bronchial asthma, has an acute attack of
bronchial asthma. What medication from the listed below is contraindicated
to render a first aid?
A. Euphylinum
B. Morphinum hydrochloride
C. Strophanthin hydrochloride
D. Izardin
E. Corazolum

27. 4 days ago a 32-year-old patient caught a cold: he presented with sore throat,
fatigue. The next morning he felt worse, developed dry cough, body
temperature rose up to 38,2°C, there appeared muco-purulent expectoration.
Percussion revealed vesicular resonance over lungs, vesicular breathing
weakened below the angle of the right scapula, fine sonorous and sibilant
wheezes. What is the most likely diagnosis?
A. Focal right-sided pneumonia
B. Acute bronchitis
C. Bronchial asthma
D. Pulmonary carcinoma
E. Pulmonary gangrene

28. A 62-year-old patient complaining of enlargement of cervical, supraclavicular


and axillary lymph nodes, subfebrile temperature for the last 3 months has
been admitted to a hospital. In blood: WBCs - 64cdot109/l, lymphocytes -
72%. What method of study should be used to specify the diagnosis?
A. Myelogram
B. Lymphoscintigraphy
C. Lymphography
D. X-rays
E. Thermography

29. A 38 y.o. woman complains of a purulent discharge from the left nostril. The
body temperature is 37,50C. The patient has been ill for a week and associates
her illness with common cold. There are a pain and tenderness on palpation of
her left cheek. The mucous membrane in the left nasal cavity is red and
turgescent. The purulent exudate is seen in the middle meatus in maxillary.
What is the most probable diagnosis?
A. Acute purulent maxillary sinusitis
B. Acute purulent ethmoiditis
C. Acute purulent frontitis
D. Acute purulent sphenoiditis
30. A 40-year-old female patient has been hospitalized for attacks of asphyxia,
cough with phlegm. She has a 4-year history of the disease. The first attack of
asphyxia occurred during her stay in the countryside. Further attacks occurred
while cleaning the room. After 3 days of inpatient treatment the patients
condition has significantly improved. What is the most likely etiological
factor?
A. Household allergens
B. Infectious
C. Pollen
D. Chemicals
E. Psychogenic

31. The complications of acute cholecystitis which require surgical intervention


are as follows EXCEPT:
A. Jaundice
B. Cholangitis conditioned by the presence of stones in the bile tract
C. Emphysematous gall-bladder
D. Empyema of the gall-bladder
E. Gall-bladder perforation

32. A 22-year-old girl has been complaining of having itching rash on her face for
2 days. She associates this disease with application of cosmetic face cream.
Objectively: apparent reddening and edema of skin in the region of cheeks,
chin and forehead; fine papulovesicular rash. What is the most likely
diagnosis?
A. Allergic dermatitis
B. Eczema
C. Neurodermatitis
D. Erysipelas
E. Dermatitis simplex

33. A schizophrenic patient considers himself to be "an outstanding scientist, a


brilliant composer and an unrivalled artist". He complains that "family and
friends are always jealous of him and want to poison him". Determine the
psychopathological syndrome:
A. Paranoiac
B. Hebephrenic
C. Manic
D. Paranoid
E. Paratrophic

34. A 42-year-old patient complains of back pain, darkened urine, general


weakness, dizziness that occurred after treating a cold with aspirin and
ampicillin. Objectively: the patient is pale, with subicteric sclerae. HR - 98
bpm. Liver - +2 cm, spleen - +3 cm. In blood: RBCs - 2,6×1012/l, Hb - 60 g/l,
CI - 0,9, WBCs - 9,4cdot109/l, basophils - 0,5%, eosinophils - 3%, stab
neutrophils - 6% segmented neutrophils - 58%, lymphocytes - 25%,
monocytes - 7%, ESR - 38 mm/hour, reticulocytes - 24%. Total bilirubin - 38
millimole/l. What complication occurred in the patient?
A. Acquired hemolytic anemia
B. Cholelithiasis
C. Paroxysmal nocturnal hemoglobinuria
D. Agranulocytosis
E. Toxic hepatitis

35. A hospital has admitted a 52-year-old patient with disseminated pulmonary


tuberculosis, complaints of acute pain in the right half of chest, that developed
after going upstairs to the 3rd floor; cough, dyspnea at rest, marked cyanosis.
What kind of complication should suspected first of all?
A. Spontaneous pneumothorax
B. Pulmonary failure
C. Cardiac failure
D. Pleuritis
E. Acute myocardial infarction

36. A 38-year-old male patient has been taking alcohol for 3 years. 3 days after a
regular drinking period he felt anxiety and fear. It appeared to him that he was
surrounded by spiders and worms, pursued by some "condemnatory voices".
His behaviour became aggressive. The patient demonstrated correct self-
awareness but impairment of temporal and spatial orientation. What is the
most likely diagnosis?
A. Delirium alcoholicum
B. Alcoholic encephalopathy
C. Pathologic intoxication
D. Alcoholic paranoia
E. Alcoholic hallucinosis

37. A painter working at a motorcar plant has been diagnosed with moderately
severe intoxication with amide compounds of benzene. The in-patient
treatment resulted in a considerable health improvement. What expert
decision should be made in this case?
A. The patient should be issued a sick list for out-patient treatment
B. The patient should be referred to the medio-social expert commission for
attributing the disability group because of an occupational disease
C. The patient may get back to work providing he will keep to hygiene and
sanitary regulations
D. The patient should be referred to the medio-social expert commission for
evaluation of percentage of work capacity loss

38. A 16-year-old patient who has a history of intense bleedings from minor cuts
and sores needs to have the roots of teeth extracted. Examination reveals an
increase in volume of the right knee joint, limitation of its mobility. There are
no other changes. Blood analysis shows an inclination to anaemia (Hb- 120
g/l). Before the dental intervention it is required to prevent the bleeding by
means of:
A. Cryoprecipitate
B. Dried blood plasma
C. Calcium chloride
D. Epsilon-aminocapronic acid
E. Fibrinogen

39. A 44-year-old patient complains about difficult urination, sensation of


incomplete urinary bladder emptying. Sonographic examination of the urinary
bladder near the urethra entrance revealed an oval well-defined
hyperechogenic formation 2x3 cm large that was changing its position during
the examination. What conclusion can be made?
A. Concrement
B. Urinary bladder polyp
C. Primary ureter tumour
D. Prostate adenoma
E. Malignant tumour of the urinary bladder

40. A 7-year-old boy had complained of headache, nausea, fatigue for 3 weeks. His
condition gradually deteriorated, headache and general weakness progressed.
The boy had bronchitis at the age of 3. His father has a history of pulmonary
tuberculosis. Objectively: body temperature 37,5°C, conscious, lies supine,
with the hip and knee flexed to 90 degrees, nuchal rigidity +6 cm, partial
ptosis of the right eyelid, the dilated right pupil. General hyperalgesia is
present. Liquor: transparent, pressure - 400 mm of water column, protein -
1,5%, cytosis - 610/3 with predominant lymphocytes, sugar - 1,22 mmol/l,
chlorides - 500 mmol/l. What is the most likely diagnosis?
A. Tuberculous meningitis
B. Epidemic cerebrospinal meningitis
C. Pneumococcal meningitis
D. Serous meningitis
E. Secondary purulent meningitis

41. A 35-year-old patient complains about pain and morning stiffness of hand
joints and temporomandibular joints that lasts over 30 minutes. She has had
these symptoms for 2 years. Objectively: edema of proximal interphalangeal
digital joints and limited motions of joints. What examination should be
administered?
A. Roentgenography of hands
B. Rose-Waaler reaction
C. Complete blood count
D. Immunogram
E. Proteinogram

42. A 69-year-old female patient complains of temperature rise up to 38,3°C,


haematuria. ESR - 55 mm/h. Antibacterial therapy turned out to be
ineffective. What diagnosis might be suspected?
A. Renal cancer
B. Urolithiasis
C. Chronic glomerulonephritis
D. Polycystic renal disease
E. Renal amyloidosis

43. A 47-year-old patient complains of insomnia, heaviness over his entire body,
constantly depressed mood. He considers himself good-for-nothing,
inadequate. Believes that he is a burden to his family, wants to die. The patient
is depressed, inactive, has a hypomimic face with sorrowful expression. He
speaks quietly and monotonely,gives short answers. What is the most likely
diagnosis?
A. Major depressive disorder
B. Late-onset schizophrenia
C. Neurotic depression
D. Atherosclerotic depression
E. Initial stage of Alzheimers disease

44. A patient, aged 16, complains of headache, mainly in the frontal and temporal
areas, superciliary arch, appearing of vomiting at the peak of headache, pain
during the eyeballs movement, joints pain. On examination: excited, to- 39°C,
Ps- 110/min. Tonic and clonus cramps. Uncertain meningeal signs. What is
the most likely diagnosis?
A. Influenza with cerebral edema manifestations
B. Adenovirus infection
C. Respiratory syncytial virus
D. Influenza, typical disease duration
E. Parainfluenza

45. A 64 y.o. patient has developed of squeering substernal pain which had
appeared 2 hours ago and irradiated to the left shoulder, marked weakness.
On examination: pale skin, cold sweat. Pulse- 108 bpm, AP- 70/50 mm Hg,
heart sound are deaf, vesicular breathing, soft abdomen, painless, varicouse
vein on the left shin, ECG: synus rhythm, heart rate is 100 bmp, ST-segment is
sharply elevated in II, III aVF leads. What is the most likely disorder?
A. Cardiogenic shock
B. Cardiac tamponade
C. Pulmonary artery thromboembolia
D. Cardiac asthma
E. Disquamative aortic aneurizm

46. A 64-year-old patient complains of severe pain in the right side of chest,
dyspnea, dry cough which appeared suddenly on exertion. Objectively: the
right side of the chest lags behind in the act of breathing. Percussion reveals
tympanic sound. Auscultation reveals pronouncedly diminished breath
sounds on the right. Ps- 100/min, weak, arrhythmic. AP- 100/50 mm Hg.
Cardiac sounds are decreased. What disease can be suspected in this patient?
A. Right-sided pneumothorax
B. Right-sided dry pleurisy
C. Right-sided hydrothorax
D. Right-sided pleuropneumonia
E. PATE

47. A 29-year-old female patient complains of dyspnea, heaviness and chest pain
on the right, body temperature rise up to 37,2°C. The disease is associated
with a chest trauma received 4 days ago. Objectively: skin is pale and moist.
Ps- 90 bpm, regular. Palpation reveals a dull sound on the right, auscultation
reveals significantly weakened vesicular breathing. In blood: RBCs -
2,8×1012/l, colour index - 0,9, Hb- 100 g/l, WBCs - 8,0cdot109/l, ESR - 17
mm/h. What results of diagnostic puncture of the pleural cavity can be
expected?
A. Haemorrhagic punctate
B. Exudate
C. Purulent punctate
D. Transudate
E. Chylous liquid

48. A 54-year-old drowned man was rescued from the water and delivered to the
shore. Objectively: the man is unconscious, pale, breathing cannot be
auscultated, pulse is thready. Resuscitation measures allowed to save the
patient. What complications may develop in the near future?
A. Pulmonary edema
B. Cardiac arrest
C. Bronchospasm
D. Respiratory arrest
E. Encephalopathy

49. An 18-year-old patient since childhood suffers from bleeding disorder after
minor injuries. His younger brother also has bleeding disorders with
occasional haemarthrosis. Which laboratory test will be informative for
diagnosis verification?
A. Clotting time
B. Determination of prothrombin time
C. Blood clot retraction
D. Fibrinogen rate
E. Thrombocyte count

50. A 28-year-old patient complains of periodic compressing heart pain. His


brother died at the age of 34 from a cardiac disease with similar symptoms.
Objectively: the patients skin is pale. Heart borders display no significant
deviations. Heart sounds are loud, there is a systolic murmur above all the
points with a peak above the aorta. Echocardioscopy reveals thickening of the
interventricular septum in the basal parts, reduction of left ventricular cavity.
What drug should be administered in order to prevent the disease
progression?
A. Metoprolol
B. Nitroglycerin
C. Digoxin
D. Captopril
E. Furosemide

51. A patient, aged 40, has been ill during approximately 8 years, complains of
pain in the lumbar part of the spine on physical excertion, in cervical and
thoracal part (especially when coughing), pain in the hip and knee joints on
the right. On examination: the body is fixed in the forward inclination with
head down, gluteal muscles atrophy. Spine roentgenography: ribs
osteoporosis, longitudinal ligament ossification. What is the most likely
diagnosis?
A. Ancylosing spondyloarthritis
B. Psoriatic spondyloarthropatia
C. Tuberculous spondylitis
D. Spondyloarthropatia on the background of Reiters disease
E. Spread osteochondrosis of the vertebral column

52. A worker, aged 38, working in the slate production during 15 years, complains
of expiratory exertional dyspnea, dry cough. On examination: deafening of the
percutory sounds in interscapular region, rough breath sounds, dry
disseminated rales. On fingers skin - greyish warts. Factory’s sectorial doctor
suspects asbestosis. Which method is the most informative for diagnosis
verification?
A. Thorax roentgenography
B. Spirography
C. Blood gases examination
D. Bronchoalveolar lavage
E. Bronchoscopy

53. A 37 y.o. woman is suffering from squeezing substernal pain on physical


exertion. On examination: BP- 130/80 mm Hg, heart rate=pulse rate 72 bpm,
heart boarders are dilated to the left side, aortic systolic murmur. ECG- signs
of the left venticle hypertrophy. What method of examination is the most
informative in this case?
A. Echocardiography
B. Sphygmography
C. X-ray
D. Phonocardiography
E. Coronarography

54. A 58-year-old woman complains of osteoarthrosis of knee-joint. For 2 weeks


she had been receiving an in-patient medical treatment. She was discharged
from the hospital in satisfactory condition with complaints of minor pain after
prolonged static work. Local hyperemia and exudative effects in the area of
joints are absent. What further tactics is the most expedient?
A. Outpatient treatment
B. Conducting arthroscopy
C. Orthopedist consultation
D. Refferral to MSEC
E. Repeated in-patient treatment

55. A 42-year-old patient applied to hospital with complaints of pain behind the
sternum with irradiation to the left scapula. The pain appears during
significant physical work, this lasts for 5-10 minutes and is over on rest. The
patient is sick for 3 weeks. What is the preliminary diagnosis?
A. IHD:First established angina pectoris
B. IHD:Stable angina pectoris of effort IV FC
C. IHD:Progressive angina pectoris
D. IHD:Variant angina pectoris (Prinzmetals)
E. IHD:Stable angina pectoris of effort I FC

56. Medical examination of a 19-year-old worker revealed generalized


lymphadenopathy mainly affecting the posterior cervical, axillary and ulnar
lymph nodes. There are multiple injection marks on the elbow bend skin. The
man denies taking drugs, the presence of injection marks ascribes to influenza
treatment. Blood count: RBCs- 3,2×1012/l, Hb- 100 g/l, WBCs- 3,1cdot109/l,
moderate lymphopenia. What study is required in the first place?
A. ELISA for HIV
B. Sternal puncture
C. Lymph node biopsy
D. X-ray of lungs
E. Immunogram

57. A 24-year-old patient felt sick in 16 hours after dried fish intake. There was
nausea, vomiting, weakness, flabbiness, double vision. On physical exam,
there was decrease of a muscle tone, anisocoria, flaccid swallowing and tendon
reflex. What is the most probable diagnosis?
A. Botulism
B. Salmonellosis
C. Acute gastritis
D. Food toxicoinfection
E. Acute encephalitis

58. A 72-year-old patient after operation due to holecystectomia was prescribed


gentamicin (80 mg every 8 hours) and cephalothin (2 g every 6 hours) due to
fever. In 10 days there was an increase of creatinine up to 310mumol/L. BP -
130/80 mm Hg, daily quantity of the urine is 1200 mL. Urine tests are without
pathology. Ultrasound: the size of kidneys is normal. What is the most
probable reason for renal failure?
A. Nephrotoxity of gentamicin
B. Cortical necrosis of kidneys
C. Hepatorenal syndrome
D. Unequal infusion of the liquid
E. Acute glomerulonephritis

59. A 40 y.o. patient was admitted to the gasteroenterology with skin itching,
jaundice, discomfort in the right subcostal area, generalized weakness. On
examination: skin is jaundice, traces of scratches, liver is +5 cm, splin is 6x8
cm. In blood: alkaline phosphatase - 2,0 mmol/(hourL), general bilirubin - 60
mkmol/L, cholesterol - 8,0 mmol/L. What is the leading syndrome in the
patient?
A. Cholestatic
B. Mesenchymal inflammatory
C. Liver-cells insufficiency
D. Asthenic
E. Cytolytic
60. A 55-year-old male had been treated at the surgical department for acute
lower-extremity thrombophlebitis. On the 7th day of treatment he suddenly
developed pain in the left part of chest, dyspnea and cough. Body temperature
was 36,1°C, respiratory rate - 36/min. The patient was also found to have
diminished breath sounds without wheezing. Ps- 140/min, thready. AP- 70/50
mm Hg. The ECG shows Q_III-S_1 syndrome. What is the most likely
diagnosis?
A. Pulmonary embolism
B. Cardiac asthma
C. Myocardial infarction
D. Bronchial asthma
E. Pneumothorax

61. A 26-year-old patient undergoes a course of treatment due to chronic


glomerulonephritis. The treatment was successful, normalization of all the
characteristics was recorded. What sanitorium and health resort treatment
could be recommended?
A. The south coast of the Crimea
B. Myrhorod
C. Truskavets
D. Not recommended
E. Morshyn

62. After a wasp-bite there was an itching of skin, hoarse voice, barking cough,
anxiety. On physical exam: there is edema of lips, eyelids, cyanosis. What
medicine is to be taken first?
A. Prednisolone
B. Lasix
C. Seduxen
D. Adrenalin
E. Euphylin

63. A 16-year-old adolescent was vaccinated with DTP. In eight days there was
stiffness and pain in the joints, subfebrile temperature, urticarial skin
eruption, enlargement of inguinal, cervical lymph nodes and spleen. What
kind of allergic reaction is observed?
A. Immunocomplex
B. Cytoxic
C. Hypersensitivity of delayed type
D. Hypersensitivity of immediate type

64. 2 weeks after recovering from angina a 29-year-old patient noticed face
edemata, weakness, decreased work performance. There was gradual progress
of dyspnea, edemata of the lower extremities, lumbar spine. Objectively: pale
skin, weakening of the heart sounds, anasarca. AP- 160/100 mm Hg. In urine:
the relative density - 1021, protein - 5 g/l, erythrocytes - 20-30 in the field of
vision, hyaline cylinders - 4-6 in the field of vision. What is the most likely
diagnosis?
A. Acute glomerulonephritis
B. Infectious allergic myocarditis
C. Myxedema
D. Essential hypertension
E. Acute pyelonephritis

65. A 56-year-old scientist experiences constricting retrosternal pain several times


a day while walking for 100-150 m. The pain lasts for up to 10 minutes and can
be relieved by nitroglycerine. Objectively: the patient is overweight, heart
borders exhibit no abnormalities, heart sounds are rhythmic, Ps- 78 bpm, AP-
130/80 mm Hg. ECG contains low amplitude of T wave in V_4-5. What
disease might be suspected?
A. Stable FC III stenocardia
B. Stable FC IV stenocardia
C. Stable FC I stenocardia
D. Instable stenocardia
E. Stable FC II stenocardia

66. In autumn a 25-year-old patient developed stomach ache arising 1,5-2 hours
after having meals and at night. He complains of pyrosis and constipation.
The pain is getting worse after consuming spicy, salty and sour food, it can be
relieved by means of soda and hot-water bag. The patient has been suffering
from this disease for a year. Objectively: furred moist tongue. Abdomen
palpation reveals epigastrial pain on the right, resistance of abdominal
muscles in the same region. What is the most likely diagnosis?
A. Duodenal ulcer
B. Stomach ulcer
C. Chronic pancreatitis
D. Chronic cholecystitis
E. Diaphragmatic hernia

67. A 51 y.o. woman complains of dull pain in the right subcostal area and
epigastric area, nausea, appetite decline during 6 months. There is a history of
gastric peptic ulcer. On examination: weight loss, pulse is 70 bpm, AP is
120/70 mm Hg. Diffuse tenderness and resistance of muscles on
palpation.There is a hard lymphatic node 1x1cm in size over the left clavicle.
What method of investigation will be the most useful?
A. Esophagogastroduodenoscopy with biopsy
B. Ureatic test
C. Stomach X-ray
D. Ultrasound examination of abdomen
E. pH-metry

68. On the 20th of June a townsman was brought to clinic. The disease broke out
acutely, starting with fever, rise in temperature to 38-390C. There also was
weakness, acute headache, nausea, vomiting, pain all over the body, sleep
disorder. On physical examination: hyperemia of skin of face, neck, thorax.
Meningeal signs are positive. 12 days ago the patient returned from the
Siberia, from the forest. What is the most probable diagnosis?
A. Tick-borne encephalitis
B. Influenza
C. Pseudotuberculosis
D. Epidemic typhus
E. Omsk hemorrhagic fever

69. A 37-year-old patient has sudden acute pain in the right epigastric area after
having fatty food. What method of radiological investigation is to be used on
the first stage of examining the patient?
A. Ultrasonic
B. Magnetic-resonance
C. Thermographic
D. Roentgenological
E. Radionuclid

70. A man, aged 68, complains of tiredness, sweating, enlargement of cervical,


submaxillary and axillary lymph nodes. Blood test: WBC- 35x109/L,
lymphocytes - 60%, Botkin and Gumprecht bodies, level of haemoglobin and
quantity of thrombocytes is normal. Myelogram showed 40% of lymphocytes.
What is the most probable diagnosis?
A. Chronic lympholeucosis
B. Lymphogranulomatosis
C. Tuberculous lymphadenitis
D. Acute leucosis
E. Chronic myeloleucosis

71. A 38-year-old patient is under observation having polyneuritic syndrome with


considerable loss of weight, fever, rise in BP. Blood test:: considerable
inflammatory changes. What examination is the most expedient to make the
diagnosis?
A. Muscular biopsy with histological investigation of the material
B. Determination of antinuclear antibodies
C. Blood culture
D. Determination of HLA antigens
E. Electromyography

72. A 32-year-old male patient has been suffering from pain in the sacrum and
coxofemoral joints, painfulness and stiffness in the lumbar spine for a year.
ESR- 56 mm/h. Roentgenography revealed symptoms of bilateral sacroileitis.
The patient is the carrier of HLA B27 antigen. What is the most likely
diagnosis?
A. Ankylosing spondylitis
B. Coxarthrosis
C. Reiters disease
D. Spondylosis
E. Rheumatoid arthritis

73. A 58-year-old female patient complains about periodical headache, dizziness


and ear noise. She has been suffering from diabetes mellitus for 15 years.
Objectively: heart sounds are rhythmic, heart rate is 76/min, there is diastolic
shock above aorta, AP is 180/110 mm Hg. In urine: OD- 1,014. Daily loss of
protein with urine is 1,5 g. What drug should be chosen for treatment of
arterial hypertension?
A. Inhibitor of angiotensin converting enzyme
B. beta-blocker
C. Thiazide diuretic
D. alpha-blocker
E. Calcium channel antagonist

74. A 26 y.o. male patient with postoperative hypothyroidism take thyroxine 100
mg 2 times a day. He has developed tachycardia, sweating, irritability, sleep
disorder. Determine further treatment tactics
A. To decrease thyroxine dosage
B. To add mercasolil to the treatment
C. To administer sedatives
D. To increase thyroxine dosage
E. To administer betablockers

75. A 28-year-old man was discharged from the hospital after having an out-of -
hospital pneumonia. He has no complaints. On physical exam: his
temperature is -36,60C, RR-18/min, Ps - 78 bpm, BP- 120/80 mm Hg. During
ausculation there is harsh respiration to the right of the lower part of the lung.
Roentgenologically: infiltrative changes are absent, intensification of the
pulmonary picture to the right in the lower lobe. How long should the doctor
keep the patient under observation?
A. 12 months
B. 3 months
C. Permanently
D. 6 months
E. 1 month

76. A 20-year-old adolescent lives in the nidus of tuberculous infection. The


tuberculine Mantoux test with 2 TU was determined as hyperergic. What signs
determine the hyperergic test of this adolescent?
A. 6 mm papula, necrosis
B. 4 mm papula
C. 12 mm hyperemia
D. 20 mm papula
E. 24 mm hyperemia

77. A survey radiograph of a miner (24 years of service record, the dust
concentration in the workplace is at the rate of 260-280 mg/m3 with 15% of
free silica) shows lung changes that are typical for pneumoconiosis. What type
of pneumoconiosis is it?
A. Anthracosilicosis
B. Silicatosis
C. Carboconiosis
D. Anthracosilicatosis
E. Silicosis

78. A patient complains of pathological lump, appearing in the right inguinal


region on exercise. The lump is round-shaped, 4 cm in diameter, on palpation:
soft elastic consistency, is positioned near the medial part of Pouparts
ligament. The lump is situated inwards from the spermatic cord. What is the
most probable preliminary diagnosis?
A. Right-sided direct inguinal hernia
B. Right-sided femoral hernia
C. Lipoma of the right inguinal area
D. Varicose veins of the right hip
E. Right-sided oblique inguinal hernia

79. A 35-year-old man was operated on peptic ulcer of the stomach. Mass deficit
of the body is 10 kg. The level of glucose after operation in the undiluted
cellular blood on an empty stomach is 6,7 mmol. During repeated examination
- 11,1 mmol (after meal), level of HbA1c - 10%. Could you please make an
interpretation of the given data?
A. Diabetes mellitus
B. Norm
C. Postoperative hyperinsulinemia
D. Disordered tolerance to glucose
E. Diabetes mellitus risk group

80.A 52 y.o. woman complains of weakness, painful itching after washing and
bathing, sensation of heaviness in the head. On examination: hyperemia of
skin of face, neck, extremities. BP- 180/100 mm Hg. Spleen is 4 cm below the
rib arch edge. What is the most probable diagnosis?
A. Erythremia
B. Essential hypertension
C. Allergic dermatitis
D. Systemic sclerodermia
E. Dermatomyositis

81. A 37-year-old patient was brought to resuscitation unit. General condition of


the patient is very severe. Sopor. The skin is grey, moist. Turgor is decreased.
Pulse is rapid, intense. BP - 160/110 mm Hg, muscle tonus is increased.
Hyperreflexia. There is an ammonia odor in the air. What is the presumptive
diagnosis?
A. Uraemic coma
B. Hyperglycemic coma
C. Alcoholic coma
D. Hypoglycemic coma
E. Cerebral coma

82. A 57-year-old man complains of shortness of breath, swelling on shanks,


irregularity in cardiac work, pain in the left chest half with irradiation to the
left scapula.Treatment is uineffective. On physical exam: hearts sounds are
diminished, soft systolic murmur on the apex. Ps - 100/min, arrhythmical, BP
- 115/75 mm Hg. The liver is +2 cm, painful. Roentgenoscopy: enlargement of
heart shadow to all sides, pulsation is weak. Electrocardiogram (ECG):
leftventricled extrasystolia, decreased voltage. What method of investigation is
necessary to do to determine the diagnosis?
A. Echocardiography
B. Coronarography
C. X-ray kymography
D. Veloergometria
E. ECG in the dynamics

83. A 55 y.o. patient complains of distended abdomen and rumbling, increased


flatulance, liguid foamy feces with sour smell following the dairy products
consumption. What is the correct name of this syndrome?
A. Syndrome of fermentative dyspepsia
B. Syndrome of fatty dyspepsia
C. Syndrome of decayed dyspepsia
D. Dyskinesia syndrome
E. Malabsorption syndrome

84. A 54-year-old patient has an over 20-year history of femoral osteomyelitis.


Over the last month she has developed progressing edemata of the lower
extremities. Urine test reveals: proteinuria at the rate of 6,6 g/l; in blood:
dysproteinemia in form of hypoalbuminemia, increase in alpha_2- and
gamma-globulin rate, ESR - 50 mm/h. What is the most likely diagnosis?
A. Secondary renal amyloidosis
B. Chronic glomerulonephritis
C. Systemic lupus erythematosus
D. Acute glomerulonephritis
E. Myelomatosis

85. In an inhabited locality there is an increase of diphtheria during the last 3


years with separate outbursts in families. What measure can effectively
influence the epidemic process of diphtheria and reduce the morbidity rate to
single cases?
A. Immunization of the population
B. Hospitalization of patients
C. Early diagnostics
D. Disinfection in disease focus
E. Detection of carriers

86. A 14-year-old victim was drawn out of the water in winter after 15 minutes of
being in the water. The victim shows no vital signs. What measures are to be
taken?
A. To release respiratory tract from water, to create drain position
and to take on measures to restore respiration and blood
circulation
B. To transport the victim to the nearest warm room to carry out reanimation
measures
C. Not to carry out reanimation measures
D. Not to waste time on the release of respiratory tract from water, to take on
cardiopulmonary reanimation
E. To transport the victim to the nearest hospital to carry out reanimation
measures

87. An electro-gas welding operator working at a machine workshop performs


welding and cutting of metal, which is accompanied by intense UV-radiation.
His welding station is equipped with effective mechanical ventilation. What
occupational disease is most likely to develop in an electro-gas welding
operator?
A. Photoelectric ophthalmia
B. Vegetative-vascular dystonia
C. Pneumoconiosis
D. Chronic overheating
E. Heatstroke

88. A woman complains of high temperature to 380C, mild pain in the throat
during 3 days. On examination: angle lymphatic nodes of the jaw are 3 cm
enlarged, palatinel tonsils are enlarged and coated with grey plaque which
spreads to the uvula and frontal palatinel arches. What is the most probable
diagnosis?
A. Larynx dyphtheria
B. Agranulocytosis
C. Oropharyngeal candidosis
D. Infectious mononucleosis
E. Vincents angina

89. A 48-year-old male patient complains of constant pain in the upper abdomen,
mostly on the left, that is getting worse after taking meals; diarrhea, weight
loss. The patient is an alcohol abuser. 2 years ago he had acute pancreatitis.
Blood amylase is 4 g/hcdotl. Coprogram shows steatorrhea, creatorrhea.
Blood glucose is 6,0 mmol/l. What treatment is indicated for this patient?
A. Panzinorm forte
B. Gastrozepin
C. No-spa
D. Contrycal
E. Insulin

90. A 60-year-old female patient had been admitted to a hospital for acute
transmural infarction. An hour later the patients contition got worse. She
developed progressing dyspnea, dry cough. Respiratory rate - 30/min, heart
rate - 130/min, AP- 90/60 mm Hg. Heart sounds were muffled, there was also
diastolic shock on the pulmonary artery. The patient presented with medium
moist rales in the lower parts of lungs on the right and on the left. Body
temperature - 36,4°C. What drug should be given in the first place?
A. Promedol
B. Digoxin
C. Dopamine
D. Aminophylline
E. Heparin
91. A 62-year-old male has been hospitalized in the intensive care unit with a
continuous attack of retrosternal pain that cannot be relieved by nitroglycerin.
Objectively: AP- 80/60 mm Hg, heart rate - 106/min, breathing rate - 22/min.
Heart sounds are muffled, a gallop rhythm is present. How would you explain
the AP drop?
A. Reduction in cardiac output
B. Blood depositing in the abdominal cavity
C. Internal haemorrhage
D. Adrenergic receptor block
E. Reduction in peripheral resistance

92. A 18 y.o. male patient complains of pain in knee and ankle joints, temperature
elevation to 39,50C. He had a respiratory disease 1,5 week ago. On
examination: temperature- 38,50C, swollen knee and ankle joints, pulse- 106
bpm, rhythmic, AP- 90/60 mm Hg, heart borders without changes, sounds are
weakened, soft systolic apical murmur. What indicator is connected with
possible etiology of the process?
A. Antistreptolysine-0
B. Creatinkinase
C. Seromucoid
D. Rheumatic factor
E. 1-antitrypsine

93. A 30 y.o. male patient complains of itching of the skin which intensifies in the
evening. He has been ill for 1,5 months. On examination: there is rash with
paired papules covered with bloody crusts on the abdomen, hips, buttocks,
folds between the fingers, flexor surfaces of the hand. There are traces of line
scratches. What additional investigations are necessary to make diagnosis?
A. Examination of rash elements scrape
B. Examination for helmints
C. Serologic blood examination
D. Determination of dermographism
E. Blood glucose

94. A 50-year-old locksmith has a long-term record of work under the effect of
mercury vapors with concentration exceeding MPC by 5-10 times. Clinical
examination revealed the lability of vasomotors of skin, pulse and arterial
pressure; total hyperhydrosis; asymmetric innervation of facial and lingual
muscles, positive subcortical reflexes, intention tremor. Against the
background of increased emotional excitability the patient presents with lack
of self-confidence, shyness. A dentist found him to have parodontosis, chronic
stomatitis. What disease can be suspected?
A. Chronic mercury intoxication
B. Parkinsons syndrome
C. Vascular encephalopathy
D. Acute mercury intoxication
E. Residual effects of neuroinfection

95. 4 hours after having meals a patient with signs of malnutrition and
steatorrhea experiences stomach pain, especially above navel and to the left of
it. Diarrheas take turns with constipation lasting up to 3-5 days. Palpation
reveals moderate painfulness in the choledochopancreatic region. The
amylase rate in blood is stable. X-ray reveals some calcifications located above
navel. What is the most likely diagnosis?
A. Chronic pancreatitis
B. Chronic gastroduodenitis
C. Zollinger-Ellison syndrome
D. Chronic calculous cholecystitis
E. Duodenal ulcer

96. A 58 y.o. male patient is examined by a physician and suffers from general
weakness, fatigue, mild pain in the left subcostal area, sometimes frequent
painful urination. Moderate splenomegaly has been revealed. Blood test:
neutrophilic leukocytosis with the progress to myelocyte; basophil- 2%;
eosinophil- 5%. There is a urate crystales in urine, erythrocyte- 2-3 in the field
of vision. What is the preliminary diagnosis?
A. Chronic myeloleucosis
B. Lymphogranulomatosis
C. Leukemoid reaction
D. Hepar cirrhosis
E. Urolithiasis

97. A 56-year-old patient with diffuse toxic goiter has ciliary arrhythmia with
pulse rate of 110 bpm, arterial hypertension, AP- 165/90 mm Hg. What
preparation should be administered along with mercazolil?
A. Propranolol
B. Radioactive iodine
C. Verapamil
D. Corinfar
E. Procaine hydrochloride

98. A 54-year-old patient complains of weakness, weight loss despite the


unchanged appetite, frequent urination, skin itch for six months. Some time
ago the patient underwent treatment for furunculosis. She hasnt been
examined recently. Objectively: malnutrition, dry skin with signs of
scratching. Small lymph nodes can be palpated in the axillary regions.
Changes in the internal organs are absent. What testing must be administered
in the first place?
A. Blood sugar test on an empty stomach
B. Endoscopy of stomach
C. Blood sterility testing
D. Lymph node biopsy
E. Complete blood count

99. A 43 y.o. woman complains of severe pain in the right abdominal side
irradiating in the right supraclavicular area, fever, dryness and bitterness in
the mouth. There were multiple vomitings without relief. Patient relates the
onset of pain to the taking of fat and fried food. Physical examination: the
patient lies on the right side, pale, dry tongue, tachycardia. Right side of
abdomen is painful during palpation and somewhat tense in right
hypochondrium. What is the most likely diagnosis?
A. Perforative ulcer
B. Acute appendicitis
C. Right-sided renal colic
D. Acute cholecystitis
E. Acute bowel obstruction

100. Several hours before, a 28-year-old patient suddenly developed acute


headache and repeated vomiting, then lost consciousness. Objectively: focal
neurological symptoms were not found. Pronounced meningeal symptoms
were revealed. AP - 120/80 mm Hg. According to clinical and liquorological
findings the patient was diagnosed with subarachnoid haemorrhage. After
administration of dehydrants the patients condition somewhat improved.
What is the main component of further emergency care?
A. Coagulants
B. Fibrinolytics
C. Corticosteroids
D. Anticoagulants
E. Antiaggregants

101. A 24-year-old man on the 5th day of acute respiratory disease with high
grade temperature started having strong headaches, systemic dizziness,
sensation of double vision, paresis of mimic muscles to the right, tickling by
swallowing. Diagnosis: Acute viral encephalitis. Determine the basic direction
of the emergent therapy
A. Zovirax
B. Hemodesis
C. Cephtriaxon
D. Glucocorticoids
E. Lasix

102. On the 5th day of a respiratory disease accompanied by fever a 24-year-


old man developed progressing headaches, systemic dizziness, double vision,
facial muscles paresis on the right, choking from swallowing. He was
diagnosed with acute viral encephalitis. Identify the main tendency of the
emergency treatment:
A. Zovirax
B. Lasix
C. Haemodezum
D. Glucocorticoids
E. Ceftriaxone

103. A 30-year-old patient was delivered to the admission ward of the


infectious disease department. The disease had started acutely on the
background of normal temperature with the appearance of frequent, liquid,
profuse stool without pathological impurities. Diarrhea was not accompanied
by abdominal pain. 12 hours later there appeared recurrent profuse vomiting.
The patient rapidly developed dehydration. What is the most likely diagnosis?
A. Cholera
B. Salmonellosis
C. Campylobacteriosis
D. Shigellosis
E. Staphylococcal food toxicoinfection

104. A 65 y.o. woman complains of complicated mouth opening following


foot trauma 10 days ago. Next day she ate with difficulties, there were muscles
tension of back, the back of the head and abdomen. On the third day there was
tension of all muscle groups, generalized convulsions every 10-15 min. What is
the most probable diagnosis?
A. Tetanus
B. Epilepsy
C. Meningoencephalitis
D. Tetania
E. Hemorrhagic stroke

105. Gastric juice analysis of a 42-year-old male patient revealed absence of


free hydrochloric acid at all stages. Endoscopy revealed pallor, thinning of
gastric mucosa, smoothed folds. Microscopically the atrophy of glands with
intestinal metaplasia was found. What disease is this situation typical for?
A. Chronic type A gastritis
B. Stomach cancer
C. Chronic type C gastritis
D. Chronic type B gastritis
E. Menetrier disease

106. A 45-year-old female patient complains of frequent liquid stools with a


lot of mucus, pus and blood; pain across the abdomen, loss of 7 kg within 6
months. She has a 1-year history of non-specific ulcerative colitis. What group
of drugs should be preferred for this patient?
A. Corticosteroids
B. Nitrofurans
C. Polyenzymes
D. Antibacterial
E. Sulfonamides

107. A 54-year-old male patient complains of aching pain in the lumbar


region, that is getting worse after standing in an upright position, physical
exercise, supercooling. The patient also reports of experiencing weakness in
the afternoon. Pain in the lumbar region, said about 10 years old. Objectively:
pale skin, to- 37,2°C, AP- 180/100 mm Hg, minor costovertebral angle
tenderness (Pasternatsky symptom). In blood: RBCs - 3,5×1012/l, WBCs -
6,5cdot109/l, ESR - 22 mm/h. In urine: the relative density - 1010, leukocytes
- 12-15 in the field of vision, erythrocytes - 2-3 in the field of vision. Urine
bacterial count - 100000 in 1 ml. What is the most likely diagnosis?
A. Chronic pyelonephritis
B. Chronic glomerulonephritis
C. Amyloidosis
D. Nephrolithiasis
E. Polycystic renal disease

108. A 67-year-old male complains of dyspnea on exertion, attacks of


retrosternal pain, dizziness. He has no history of rheumatism. Objectively:
pale skin, acrocyanosis. There are rales in the lower parts of lungs. There is
systolic thrill in the II intercostal space on the right, coarse systolic murmur
conducted to the vessels of neck. AP- 130/90 mm Hg, heart rate - 90/min,
regular rhythm. The liver extends 5 cm under the edge of costal arch, shin
edemata are present. Specify the assumed valvular defect:
A. Aortic stenosis
B. Mitral insufficiency
C. Tricuspid regurgitation
D. Ventricular septal defect
E. Pulmonary artery stenosis

109. A 24-year-old female teacher complains of dizziness and heart pain


irradiating to the left nipple. Pain is not associated with physical activity and
cannot be relieved by nitroglycerin, it abates after taking Valocordin and lasts
an hour or more. The patient has a nearly 2-year history of this disease.
Objectively: Ps- 76 bpm. AP- 110/70 mm Hg. Heart borders are normal, heart
sounds are clear. The ECG shows respiratory arrhythmia. Radiograph of the
cervicothoracic spine shows no pathology. Lungs, abdomen are unremarkable.
What changes in blood formula can be expected?
A. No changes
B. Leukemic hiatus
C. Increased ESR
D. Leukocytosis
E. Thrombocytopenia

110. A 51-year-old female patient complains of frequent defecation and


liquid blood-streaked stools with mucus admixtures, diffuse pain in the
inferolateral abdomen, 6 kg weight loss over the previous month. Objectively:
body temperature - 37,4°C, malnutrition, skin is pale and dry. Abdomen is
soft, sigmoid is painful and spasmodic, makes a rumbling sound. Liver is
dense, painful, extends 3 cm below the costal margin. What is the most likely
diagnosis?
A. Non-specific ulcerative colitis
B. Helminthic invasion
C. Sprue
D. Bacillary dysentery
E. Intestinal enzymopathy

111. A 18 y.o. female student complains of dyspnea during the intensive


exertion. The condition became worse half a year ago. On examination: pulse
rate is 88 bpm, accelerated, AP- 180/20 mm Hg, pale skin, heart borders are
dilated to the left and up. There is systolic-diastolic murmur in the 2hd
intercostal space, S_2 at pulmonary artery is accentuated. ECG has revealed
both ventricles hypertrophy. Thoracic X-ray has revealed pulsation and
protrusion of the left ventricle, lung trunk. What doctors tactics should be?
A. Cardiosurgeon consultation
B. Dispensary observation
C. Continuation of investigation
D. Exemption from physical exercises
E. Administration of therapeutic treatment

112. A 49-year-old male patient complains of dyspnea of combined nature,


cough, shin edemata, abdomen enlargement due to ascites. He has a 20-year
history of chronic bronchitis. For the last 3 years he has been disabled (group
II) because of cardiac changes. Objectively: mixed cyanosis, edemata. Ps -
92/min, rhythmic, AP - 120/70 mm Hg, respiration rate - 24/min. There is
accentuation of the second sound above the pulmonary artery. Auscultation
reveals the box resonance above the lungs. There are also dry rales over the
entire surface of lungs. What is the mechanism of heart changes development
in this patient?
A. Euler-Liljestrand reflex
B. Kitaevs reflex
C. Cardiovascular reflex
D. Respiratory reflex
E. Bainbridge reflex

113. A 24-year-old man on the second day of the disease with a sudden
onset complains of a strong headache in temples and in the area of orbits, dull
pain in the body, dry painful cough. His temperature is 390C. Adynamic.
Mucous membrane of oropharynx is "flaming", rales are not ausculated. What
is the most probable diagnosis?
A. Influenza
B. Parainluenza
C. Pneumonia
D. Meningococcus infection
E. Respiratory mycoplasmosis

114. A 32-year-old female complains of dizziness, headache, palpitation,


tremor. For the last several months she has been under outpatient observation
for the increased arterial pressure. Since recently such attacks have become
more frequent and severe. Objectively: skin is covered with clammy sweat,
tremor of the extremities is present. HR- 110/min, AP- 220/140 mm Hg.
Heart sounds are muffled. Blood test results: WBCs - 9,8cdot109/l, ESR - 22
mm/h. Blood glucose - 9,8 millimole/l. What disease is the most likely cause
of this crisis?
A. Pheochromocytoma
B. Preeclampsia
C. Essential hypertension
D. Primary hyperaldosteronism
E. Diabetic glomerulosclerosis

115. A 43-year-old female patient complains of unstable defecation with


frequent constipations, abdominal swelling, headache, sleep disturbance.
Body weight is unchanged. What disease are these clinical presentations
typical for?
A. Irritable colon syndrome
B. Chronic atrophic gastritis
C. Colorectal cancer
D. Chronic enteritis
E. Chronic pancreatitis
116. A 43-year-old man who often contacts with ethyl gasoline was admitted
to a hospital with complaints of general weakness, dizziness, memory
impairment, sleepiness at daytime and insomnia at night, sensation of a hair
in the mouth, colicky pains in the right subcostal region. What is the most
likely diagnosis?
A. Chronic tetraethyl lead intoxication
B. Chronic mercury intoxication
C. Alcoholic delirium
D. Chronic manganese intoxication
E. Chronic lead intoxication

117. A 35-year-old patient has been in the intensive care unit for acute renal
failure due to crush for 4 days. Objectively: the patient is inadequate.
Breathing rate - 32/min. Over the last 3 hours individual moist rales can be
auscultated in lungs. ECG shows high T waves, right ventricular extrasystoles.
CVP - 159 mm Hg. In blood: the residual nitrogen - 62 millimole/l, K+- 7,1
millimole/l, Cl-- 78 millimole/l, Na+- 120 millimole/l, Ht - 0,32, Hb - 100 g/l,
blood creatinine - 0,9 millimole/l. The most appropriate method of treatment
would be:
A. Hemodialysis
B. Ultrafiltration
C. Hemosorption
D. Plasma sorption
E. Plasma filtration

118. A 45-year-old man was brought to clinic with complaints of the pain
that started suddenly in the left chest part and epigastric area, shortness of
breath, nausea, one-time vomiting. The acute pain started after weight-lifting.
On physical exam: shallow breathing, RR - 38/min, left chest part is behind
during respiration, by percussion - tympanitic sound, respiration is not
ausculated. Ps - 110 bpm, of weak filling. BP - 100/60 mm Hg, insignificant
displacement of heart to the right, sounds are dull. What examination is the
most expedient to do first?
A. Roentgenography
B. Ultrasound of the abdominal cavity
C. Bronchoscopy
D. Electrocardiography
E. Esophagogastroscopy

119. A 35 y.o. woman is suspected of aplastic anemia. The bone marrow


punction has been administered with the diagnostic purpose. What changes in
the marrow punctatum are suggested?
A. Replacement of marrow elements with adipose tissue
B. Prevalence of megaloblasts
C. Replacement of marrow elements with fibrous tissue
D. Presence of blast cells
E. Absolute lymphocytosis

120. A 47-year-old woman underwent a thyroid gland resection on account


of nodular euthyroid goiter. What preparations are most likely to prevent the
disease recurrence?
A. Thyroid hormones
B. Thyrotropin
C. Mercazolil
D. Antistruminum (potassium iodide)
E. Radioactive iodine

121. A 55 y.o. male patient complains of weakness during 2 months, pain in


the right side of the thorax, cough, blood-streaked sputum. On X-ray:
intensive triangle shadow in the area of lower lobe that is connected to
mediastinum. What is the most likely disorder in the lungs?
A. Central cancer of lungs
B. Pleuropneumonia
C. Bronchiectasia
D. Tuberculosis of lungs
E. Pulmonary infarction

122. A 60 y.o. patient experiences acute air insufficiency following of the


venoectomy due to subcutaneous vein thrombophlebitis 3 days ago. Skin
became cianotic, with grey shade. Marked psychomotor excitement,
tachypnea, substernal pain. What postoperative complication has occurred?
A. Thromboembolia of pulmonary artery
B. Valvular pneumothorax
C. Hypostatic pneumonia
D. Hemorrhagia
E. Myocardial infarction

123. A 19-year-old woman complains of pain in the abdomen and joints,


asks for more analgetics and somnifacient injections. The patient was
examined. Gynecological and urological pathologies are absent. There are
signs of previous punctures along superficial veins of the extremities. The
patient does not explain the origin of punctures. Tendon reflexes of upper and
lower extremities are the same, quick. Photoreaction of the pupil of the eye is
weak. The tongue is grey coated. During communication the patient in
affectively not even-tempered. There is diarrhea without pathologic
inclusions. What tactics is necessary to improve the condition of this patient?
A. Consultation of an expert in narcology
B. Additional consultation of surgeon
C. Prescription of medications the patient asks for
D. Treatment with antibiotics
E. Consultation of infectious diseases doctor

124. A patient has an over a year-old history of fast progressive rheumatoid


arthritis. X-raying confirms presence of marginal erosions. What basic drug
would be the most appropriate in this case?
A. Methotrexate
B. Diclofenac sodium
C. Aspirin
D. Chloroquine
E. Prednisolone

125. A female rheumatic patient experiences diastolic thoracic wall tremor


(diastolic thrill), accentuated S_1 at apex, there is diastolic murmur with
presystolic intensification, opening snap, S_2 accent at pulmonary artery.
What rind of heart disorder is observed?
A. Mitral stenosis
B. Mitral valve insufficiency
C. Opened arterial duct
D. Aortic valve insufficiency
E. Pulmonary artery stenosis

126. A 23-year-old patient complains of a dull ache, sensation of heaviness


and distention in the epigastrium immediately after meals, foul-smelling
eructation; dry mouth, empty stomach nausea, diarrhea. Objectively: the skin
is pale, the patient is of thin build. Abdomen is soft on palpation, there is
epigastric pain. The liver does not extend beyond the costal arch. In blood: Hb
- 110 g/l, RBCs - 3,4×1012/l, WBC count is normal. ESR - 16 mm/h. What is
the most informative study that will allow make a diagnosis?
A. Esophageal gastroduodenoscopy
B. X-ray of digestion organs
C. pH-metry
D. Duodenal probing
E. Study of gastric juice

127. A 49-year-old patient complains of deglutition problems, especially


with solid food, hiccups, voice hoarseness, nausea, regurgitation, significant
weight loss (15 kg within 2,5 months). Objectively: body weight is reduced.
Skin is pale and dry. In lungs: vesicular breathing, heart sounds are loud
enough, heart activity is rhythmic. The abdomen is soft, painless on palpation.
Liver is not enlarged. What study is required to make a diagnosis?
A. Esophageal duodenoscopy along with biopsy
B. X-ray in Trendelenburgs position
C. Study of gastric secretion
D. Clinical blood test
E. X-ray of digestive tract organs

128. A 60-year-old patient has been admitted to a hospital with complaints


of dyspnea, tightness in the right subcostal area, abdomen enlargement. These
presentations have been progressing for a year. Heart auscultation reveals
presystolic gallop rhythm. Objectively: swelling of the neck veins, ascites,
palpable liver and spleen. What disease requires differential diagnostics?
A. Constrictive pericarditis
B. Pulmonary embolism
C. Lung cancer with invasion to the pleura
D. Hepatocirrhosis
E. Chronic pulmonary heart

129. A 40-year-old patient, the forester, complains of severe headache, body


temperature rise up to 39,5°C, trembling limbs. From the patients history we
know that he had seriously cut his hand during the dissection of a killed fox.
Objectively: depressed mood. The patient asks not to turn on the light or open
the door. Any noise causes apparent motor excitation. When he saw a carafe of
water, he developed convulsive throat spasms. What tactics should an
emergency doctor choose?
A. Deliver the patient to the infectious disease hospital
B. Deliver the patient to the neurological department
C. Let him stay at home and consult a psychiatrist
D. Deliver the patient to the psychiatric hospital
E. Deliver the patient to the resuscitation department

130. A 28-year-old woman has a 12-year history of chronic


glomerulonephritis with latent course. Over the past six months she has
developed general weakness, loss of appetite, low work performance, nausea.
The patient complains of headache, pain in the joints. On examination:
anemia, blood urea - 34,5 millimole/l, blood creatinine - 0,766 millimole/l,
hyperkalemia. What complication has developed?
A. Chronic renal insufficiency
B. Nephrotic syndrome
C. Acute renal insufficiency
D. Renal amyloidosis
E. Pyelonephritis
131. A 70 y.o. male patient with mild headaches complains of speech
disorder, weakness in right limbs. There was a history of miocardial infarction
and arrhythmia. On nu eroligical examination there are elements of motor
aphasia, central paresis of VII and XII cranial nerves pairs on the right side,
cental type of hemiparesis and hemihyperesthisia on the same side. What is
the most probable diagnosis?
A. Ischemic stroke
B. Transitory ischemic attack
C. Hemorrhagic stroke
D. Epidural hematoma
E. Cerebral tumor

132. After treating a field with pesticides a machine operator presents with
great weakness, headache, nausea, vomiting, diarrhea, visual impairment,
watery eyes. Objectively: the patient is excited, hypersalivation, hyperhidrosis,
muscle fibrillation of tongue and eyelids are oberved. Pupils are narrowed,
there is tachycardia, lung auscultation reveals moist small and medium
bubbling rales. In blood: changed level of cholinesterase activity. What is the
most likely diagnosis?
A. Intoxication with organophosphorous pesticides
B. Intoxication with organomercurial pesticides
C. Intoxication with organochlorine pesticides
D. Intoxication with arsenic-containing pesticides
E. Intoxication with carbamic acid derivatives

133. A 40-year-old man is ill with autoimmune hepatitis. Blood test: A/G
ratio 0,8, bilirubin - 42mumol/L, transaminase : ALT- 2,3 mmol g/L, AST -
1,8 mmol g/L. What is the most effective means in treatment from the given
below?
A. Glucocorticoids, cytostatics
B. Hemosorbtion, vitamin therapy
C. Hepatoprotectors
D. Antibacterial medication
E. Antiviral medications

134. A farmer hurt his right foot during working in a field and came to the
emergency station. He doesnt remember when he got last vaccination and he
has never served in the army. Examination of his right foot revealed a
contaminated wound up to 5-6 cm long with uneven edges. The further
treatment tactics will be:
A. To make an injection of tetanus anatoxin and antitetanus serum
B. To make an injection of tetanus anatoxin
C. Surgical d-bridement only
D. To administer an antibiotic
E. To make an injection of antitetanus serum

135. A 35-year-old patient has been admitted to a hospital for pain in the left
sternoclavicular and knee joints, lumbar area. The disease has an acute
character and is accompanied by fever up to 38°C. Objectively: the left
sternoclavicular and knee joints are swollen and painful. In blood: WBCs -
9,5cdot109/l, ESR - 40 mm/h, CRP - 1,5 millimole/l, fibrinogen - 4,8 g/l, uric
acid - 0,28 millimole/l. Examination of the urethra scrapings reveals
chlamydia. What is the most likely diagnosis?
A. Reiters syndrome
B. Gout
C. Rheumatoid arthritis
D. Bechterews disease
E. Rheumatic arthritis

136. A 20 daily y.o. female patient is suffering from chronic bronchitis.


Recently there has been production about 0,5 L of purulent sputum with
maximum discharge in the morning. Fingers are like "drum sticks", there are
"watching glass" nails. What is the most probable diagnosis?
A. Bronchiectasia
B. Tuberculosis
C. Chronic bronchitis
D. Pneumonia
E. Gangrene of lungs

137. Topographic percussion of lungs in a patient who got a serious job-


related barotrauma revealed that the lower lungs borders were located one rib
below normal, there was a significant increase in both lungs height and
Kronigs isthmus. What disease should be suspected in the first place?
A. Pulmonary emphysema
B. Chronic bronchitis
C. Pneumothorax
D. Bronchial asthma
E. Exudative pleuritis

138. An 18 y.o. girl complains of weakness, dizziness, loss of appetite,


menorrhagia. There are many-coloured petechiae on the skin of the upper
extremities. Blood test: Hb- 105 g/l; RBC- 3,21012/L; C.I.- 0,95; thromb.-
20109/L. The sedimentation time according to Lee White is 5; hemorrhagia
duration according to Duke is 8, "pinch and tourniquet" test is positive. What
is the most probable diagnosis?
A. Idiopathic thrombocytopenic purpura
B. Hemorrhagic diathesis
C. Hemophilia
D. Iron deficiency anemia
E. Marchiafava-Michelis disease

139. A 28 y.o. male patient was admitted to the hospital because of high
temperature 390C, headache, generalized fatigue, constipation, sleep disorder
for 9 days. There are sporadic roseolas on the abdomen, pulse- 78 bpm, liver is
enlarged for 2 cm. What is the most probable diagnosis?
A. Abdominal typhoid
B. Leptospirosis
C. Sepsis
D. Typhus
E. Brucellosis

140. A 50-year-old patient complains about having pain attacks in the right
subcostal area for about a year. He pain arises mainly after taking fattening
food. Over the last week the attacks occurred daily and became more painful.
On the 3rd day of hospitalization the patient presented with icteritiousness of
skin and scleras, light-colored feces and dark urine. In blood: neutrophilic
leukocytosis - 13,1cdot109/l, ESR- 28 mm/h. What is the most likely
diagnosis?
A. Chronic calculous cholecystitis
B. Chronic cholangitis, exacerbation stage
C. Hypertensive dyskinesia of gallbladder
D. Chronic recurrent pancreatitis
E. Fatty degeneration of liver

141. A 20 y.o. patient with bronchial asthma experiences dyspnea attacks 3-


4 times a week. Nocturnal attacks are 1 time a week. FEV1- 50% of necessary
figures, during the day its variations is 25%. What is the severity of bronchial
asthma condition?
A. Moderate severity condition
B. Serious condition
C. Intermittent flow
D. Asthmatic status
E. Mild condition

142. A 40 y.o. man complains of headache in occipital area. On physical


examination: the skin is pale; face and hand edema, BP- 170/130 mm Hg. On
Ech°CG: concentric hypertrophy of the left ventricle. Ultrasound examination
of the kidneys reveals thinned cortical layer. Urine analysis shows proteinuria
of 3,5 g/day. What is the probable diagnosis?
A. Essential arterial hypertension
B. Polycystic disease of the kidneys
C. Cushings disease
D. Chronic pyelonephritis
E. Chronic glomerulonephritis

143. After a serious nervous stress a 35-year-old patient has developed on


the dorsal surface of hands redness and swelling that were later replaced by
small inflammatory nodules, vesicles and following erosion with a significant
serous discharge. The process is accompanied by severe itching. What is the
most likely diagnosis?
A. True eczema
B. Microbal eczema
C. Allergic dermatitis
D. Simple contact dermatitis
E. Toxicoderma

144. A 36-year-old patient complains of skin rash that appeared a week ago
and doesnt cause any subjective problems. Objectively: palm and sole skin is
covered with multiple lenticular disseminated papules not raised above the
skin level. The papules are reddish, dense on palpation and covered with
keratinous squamae. What is the provisional diagnosis?
A. Secondary syphilis
B. Palmoplanar rubrophytosis
C. Palm and sole callosity
D. Verrucosis
E. Palmoplanar psoriasis

145. In the morning a patient had nausea, abdominal discomfort, single


vomiting, dry mouth. In the evening, the patient presented with the increasing
general weakness, double vision, difficult swallowing of solid food.
Objectively: ptosis, mydriasis, anisocoria, absence of gag and pharyngeal
reflex, dry mucous membranes. The previous evening the patient had dinner
with canned food and alcohol. What is the presumptive diagnosis?
A. Botulism
B. Poliomyelitis
C. Intoxication with unknown poison
D. Food toxicoinfection
E. Acute ischemic stroke
146. A 30-year-old patient complains of paroxysmal abdominal pain,
frequent liquid stools up to 10 times a day. Throughout the first 3 days he had
a fever, since the 2nd day of disease there were scant liquid stools mixed with
mucus. On palpation: tenderness of all colon segments. Sigmoid colon was
found spastic. What is your provisional diagnosis?
A. Acute dysentery
B. Intestinal amebiasis
C. Cholera
D. Balantidiasis
E. Salmonellosis

147. A 38-year-old woman experiences episodic increases in arterial


pressure up to 240/120 mm Hg, which is accompanied by nausea, vomiting,
tachycardia, increased sweating, hyperglycemia. The attack is usually followed
by the excessive urination. Renal sonography reveals an additional formation
adjacent to the upper pole of the right kidney and possibly belonging to the
adrenal gland. What laboratory test will allow to clarify the diagnosis?
A. Determination of urinary excretion of catecholamines and
vanillylmandelic acid
B. Estimation of glomerular filtration rate by measuring endogenous creatinine
clearance
C. Blood test for insulin and C-peptide
D. Blood test for thyroxine and thyrotrophic hormone
E. Blood test for renin level

148. A 32-year-old patient has a 3-year history of asthma attacks, that can
be hardly stopped with berotec. Over a few last months he has experienced
pain in the joints and sensitivity disorder of legs and feet skin. Ps - 80/min,
AP - 210/100 mm Hg. In blood: eosinophilia at the rate of 15%. What disease
can be suspected in this case?
A. Periarteritis nodosa
B. Dermatomyositis
C. Wegeners disease
D. Systemic lupus erythematosus
E. Systemic scleroderma

149. A 46-year-old patient complains of sudden palpitation, which is


accompanied by pulsation in the neck and head, fear, nausea. The palpitation
lasts for 15-20 minutes and is over after straining when holding her breath.
What kind of cardiac disorder may be suspected?
A. An attack of supraventricular paroxysmal tachycardia
B. An attack of atrial flutter
C. An attack of ventricular paroxysmal tachycardia
D. An attack of ciliary arrhythmia
E. An attack of extrasystolic arrhythmia

150. A 5-grade pupil complains about extensive skin rash accompanied by


intensive itch, especially at night. Objectively: there are small red papules set
mostly in pairs in the region of interdigital folds on both hands, on the flexor
surface of radicarpal articulations, abdomen and buttock skin as well as
internal surface of thighs. In the centre of some papules vesicles or
serohaemorrhagic crusts can be seen. There are multiple excoriations. What is
the most likely diagnosis?
A. Scabies
B. Ringworm of body
C. Dermatitis
D. Toxicoderma
E. Eczema

151. A welder at work got the first-degree burns of the middle third of his
right shin. 5 days later the skin around the burn became edematic and itchy.
Objectively: on a background of a well-defined erythema there is polymorphic
rash in form of papules, vesicles, pustules, erosions with serous discharge.
What is the most likely diagnosis?
A. Microbal eczema
B. Occupational eczema
C. Streptococcal impetigo
D. True eczema
E. Toxicoderma

152. A 58-year-old patient has a 3-year history diabetes mellitus type II. He
has been keeping to a diet and regularly taking glyburide. He has been
delivered to a hospital on an emergency basis for acute abdomen. Objectively:
the patient is of supernutrition type. The skin is dry. In the lungs vesicular
breathing can be auscultated. Heart sounds are regular, 90/min. AP- 130/70
mm Hg. The symptom of "wooden belly" is visible. Blood sugar - 9,8
millimole/l. The patients has indication for laparotomy. What is the most
appropriate way of further treatment of diabetes?
A. To administer short insulin
B. To administer Semilong to be taken in the morning and insulin - in the
evening
C. To continue taking glyburide
D. To administer 1 tablet of Glurenorm three times a day
E. To administer 1 tablet of Maninil three times a day

153. A 56 y.o. man, who has taken alcoholic drinks regularly for 20 years,
complains of intensive girdle pain in the abdomen. Profuse nonformed stool
2-3- times a day has appeared for the last 2 years, loss of weight for 8 kg for 2
years. On examination: abdomen is soft, painless. Blood amylase - 12g/L.
Feces examination-neutral fat 15 g per day, starch grains. What is the most
reasonable treatment at this stage?
A. Pancreatine
B. Aminocapron acid
C. Contrykal
D. Levomicytine
E. Imodium

154. A 43-year-old female patiet complains of eruption on her right leg skin,
pain, weakness, body temperature rise up to 38°C. The disease is acute.
Objectively: there is an edema on the right leg skin in the region of foot, a
well-defined bright red spot in form of flame tips which feels hot. There are
isolated vesicles in focus. What is your provisional diagnosis?
A. Erysipelas
B. Contact dermatitis
C. Microbial eczema
D. Toxicoderma
E. Haemorrhagic vasculitis

155. A 45-year-old patient complains of some painless nodular elements


tending to peripheral growth and fusion. He has a 2-year history of this
disease. Aggravation takes place mainly in spring. In anamnesis: the patients
father had similar skin lesions. Objectively: pathological elements looke like
guttate and nummular nodules, plaques covered with white scales. What is
your provisional diagnosis?
A. Psoriasis
B. Neurodermitis
C. Seborrheic eczema
D. Pityriasis rosea
E. Lichen ruber planus

156. A 47-year-old patient came to see a doctor on the 7th day of disease.
The disease developed very fast: after the chill body temperature rose up to
40°C and lasted up to 7 hours, then it dropped abruptly, which caused profuse
sweat. There were three such attacks occuring once in two days. Two days ago
the patient arrived from Africa. Objectively: pale skin, subicteric sclera,
significantly enlarged liver and spleen. What is the cause of fever attacks in
this disease?
A. Erythrocytic schizogony
B. Endotoxin of a causative agent
C. Gametocytes
D. Tissue schizogony
E. Exotoxin of a causative agent

157. On the 2nd day of illness a 27-year-old patient complains of unbearable


headache, repeated vomiting. Objectively: the patient is in a grave condition.
He is conscious but adynamic. Lies in a forced position with his head thrown
back. There is no skin rash. Nuchal muscles are evidently rigid, there are
Kernigs and Brudzinskis signs. to- 39,5°C, Ps- 120/min, AP- 130/80 mm Hg.
The leading syndrome of this disease is caused by:
A. Liquor hypertension
B. Hyperthermy
C. Affection of the cranial nerve nuclei
D. Liquor hypotension
E. Haemorrhages into the adrenal glands

158. A 43 y.o. woman complains of shooting heart pain, dyspnea,


irregularities in the heart activity, progressive fatigue during 3 weeks. She had
acute respiratory disease a month ago. On examination: AP- 120/80 mm Hg,
heart rate 98 bpm, heart boarders +1,5 cm left side, sounds are muffled, soft
systolic murmur at apex and Botkins area; sporadic extrasystoles. Liver isnt
palpated, there are no edema. Blood test: WBC- 6,7x109/L, sedimentation
rate- 21 mm/hour. What is the most probable diagnosis?
A. Acute myocarditis
B. Ichemic heart disease, angina pectoris
C. Climacteric myocardiodystrophia
D. Rheumatism, mitral insufficiency
E. Hypertrophic cardiomyopathy

159. A 37-year-old woman is sick with bronchial asthma for 15 years.


Recenlty asthmatic attacks occur 4-5 times per week, night attacks -2-3 times
per month. To stop attacks, the patient takes salbutamol. On physical exam:
condition is relatively satisfactory. RR - 20/min, Ps is 76 bpm, BP - 120/80
mm Hg. Respiration in lungs is vesicular. Cardiac sounds are muted, rhythm
is normal. What medication should be prescribed to prevent attacks of
bronchial asthma on the first stage?
A. Cromoglycat sodium
B. Regular dose of salbutamol
C. Tabletted corticosteroids
D. Injection of corticosteroids
E. Inhalation corticosteroids

160. A 52 y.o. male patient has become ill gradually. There is pain in the left
side of the thorax during 2 weeks, elevation of temperature till 38-39°C. On
examination: left chest side falls behind in breathing movement no voice
tremor over the left lung. Dullness that is more intensive in lower parts of this
lung. Right heart border is deviated outside. Sharply weakened breathing over
the left lung, no rales. Heart sounds are mufflet, tachycardia. What is the most
probable diagnosis?
A. Exudative pleuritis
B. Infarction-pneumonia
C. Atelectasis of lung
D. Spotaneous pneumothorax
E. Cirrhotic tuberculosis

161. A 50-year-old patient was hospitalized in severe condition with


complaints of chills, high grade temperature, dryness in the mouth, multiple
vomiting, pain in the epigastrium, frequent watery, foamy, dirty green color
stool of unpleasant odor. The tongue and the skin are dry. BP - 80/40 mm Hg.
What first aid is necessary for the patient?
A. Intravenous injection of sodium solutions
B. To prescribe polyglucin
C. Fresh-frozen plasma transfusion
D. Sympathomimetics
E. Hemosorbtion

162. A 42-year-old female patient suffers from micronodular cryptogenic


cirrhosis. Over the last week her condition has deteriorated: she developed
convulsions, mental confusion, progressing jaundice. What study may give
reasons for such aggravation?
A. Determination of serum ammonia
B. Determination of cholesterol ethers
C. Determination of ALAT and ASAT
D. Determination of alkaline phosphatase
E. Determination of alpha-phetoprotein

163. A patient has chronic heart failure of the II stage. He takes furosemide
regularly three times a week. He had developed bronchopneumonia and had
been administered combined pharmacotherapy. On the fifth day of therapy
the patient complained of hearing impairment. What drug coadministered
with furosemide might have caused the hearing loss?
A. Gentamicin
B. Mucaltin
C. Nystatin
D. Linex
E. Tavegil
164. A 37-year-old woman complains of generalized fatigue, irritability,
dysphagia, chalk hunger. On physical exam: t- 36,50C, respirations - 20/min,
Ps - 96 bpm, BP - 110/70 mm Hg. Satisfactory nourishment. The skin and
visible mucous membranes are pale. Blood test: Hb -70g/L, erythrocytes -
3,41012/L, CI - 0,7, reticulocytes - 2%, leucocytes - 4,7109/L, eosinophilis. -
2%, band neutrophils - 3%, segmented neutrophils - 64%, lymphocytes - 26%,
monocytes - 5%, ESR - 15 mm/min. Serum ferrum - 7,3mumol/L, total protein
- 70g/L. Deficit of what factor caused the development of the disease?
A. Ferrum
B. Vitamin B12
C. Folic acid
D. Protein
E. Vitamin B6

165. A 28-year-old patient has been hospitalized for the pain in the
epigastric region. He has a 10-year history of duodenal ulcer (DU). Recently,
the pain character has changed: it became permanent, persistent, irradiating
to the back. There are general weakness, dizziness, fatigue. The patient has
put off weight. Objectively: HR- 68/min, AP- 120/80 mm Hg. What is most
likely cause of deterioration?
A. Penetration
B. Perforation of duodenal wall
C. Haemorrhage
D. Exacerbation of duodenal ulcer
E. Stenosis development

166. A 57-year-old male patient complains of dyspnea on exertion, heaviness


in the right hypochondrium and shin edemata towards evening. Objectively:
temperature - 38,1°C, HR- 20/min, HR=Ps=92/min, AP- 140/90 mm Hg.
There is apparent kyphoscoliosis. In the lungs single dry rales can be
auscultated. Heart sounds are muffled, rhythmic. ECG: Rv1+Sv5=15 mm. X-
ray picture shows the bulging of pulmonary artery cone, right ventricle
enlargement. What is the most likely cause of this condition?
A. Pulmonary heart
B. Mitral stenosis
C. Primary pulmonary hypertension
D. Atherosclerotic cardiosclerosis
E. Dilatation cardiomyopathy

167. A 24-year-old emotionally-labile woman presents with irritation,


depressed mood, palpitation, shooting pain in the heart area, generalized
fatigue following the divorce. On examination: palm hyperhydrosis, pulse
rate- 72-78 bpm, labile, heart without changes. ECG is normal. What is the
most probable pathology in this case?
A. Neurasthenia
B. Compulsive neurosis
C. Ipochondric neurosis
D. Schizophrenia
E. Depressive neurosis

168. A 30-year-old woman with a long history of chronic pyelonephritis


complains about considerable weakness, sleepiness, decrease in diuresis down
to 100 ml per day. AP- 200/120 mm Hg. In blood: creatinine - 0,62
millimole/l, hypoproteinemia, albumines - 32 g/l, potassium - 6,8 millimole/l,
hypochromic anemia, increased ESR. What is the first step in the patient
treatment tactics?
A. Haemodialysis
B. Antibacterial therapy
C. Haemosorption
D. Blood transfusion
E. Enterosorption

169. A patient had macrofocal myocardial infarction. He is overweight for


36%, AP is 150/90 mm Hg, blood sugar- 5,9 mmol/L, general cholesterol- 4,9
mmol/L, uric acid- 0,211 mmol/L. Which risk factor should be urgently
eradicated during the secondary prevention?
A. Obesity
B. Arterial hypertension
C. Hypercholesterolemia
D. Hyperuricemia
E. Hyperglycemia

170. A 36-year-old female patient complains of bruises on the body, gingival


haemorrhage, general weakness. A month ago she had a severe domestic
poisoning with some pesticide (the patient can not remember the name). She
has a 7-year record of working in contact with petroleum products,
particularly benzene. In blood: RBCs - 3,2×1012/l, WBCs - 2,7cdot109/l,
thrombocytes - 70cdot109/l. What is the most likely pathology?
A. Benzene intoxication
B. Chronic fatigue Syndrome
C. Organochlorine pesticide Intoxication
D. Organophosphorus pesticide intoxication
E. Mercury-containing pesticide intoxication

171. While staying in a stuffy room a 19-year-old emotionally labile girl


developed severe weakness, dizziness, blackout, nausea and loss of
consciousness without convulsions. Objectively: the patient is unconscious,
the skin is pale, extremities are cold. AP- 90/60 mm Hg, Ps- 96/min,
deficient, breathing is shallow. Pupillary and tendon reflexes are present.
There are no pathological signs. What is the most likely diagnosis?
A. Syncope
B. Hysterical neurosis
C. Transient ischemic attack
D. Vegetovascular paroxysm
E. Epileptic attack

172. A patient complains of frequent, bulky, frothy stools with greenish


mucus, cramping pain in the umbilical region, abdominal murmur, body
temperature at the rate of 39°C. The patient associates the disease with
consumption of soft-boiled eggs. What is the most likely pathogen?
A. Salmonella
B. Shigella
C. Vibrio cholerae El Tor
D. Enteropathogenic E.Coli
E. Yersinia

173. A 40-year-old female patient complains of headache, dizziness, muscle


weakness, occasional cramps in the extremities. She has been taking
antihypertensive medications for 10 years. AP- 180/100 mm Hg. Blood
potassium - 1,8 millimole/l, sodium - 4,8 millimole/l. In urine: alkaline
reaction, the relative density - 1012, protein and sugar are not found, WBCs -
3-4 in the field of vision, RBCs - 1-2 in the field of vision. Conns syndrome is
suspected. Which drug should be chosen for the treatment of arterial
hypertension?
A. Spironolactone
B. Enalapril
C. Clonidine
D. Hydrochlorothiazide
E. Propanolol

174. An 18-year-old patient presents no problems. Percussion reveals that


heart borders are displaced to the right and left by 1 cm, there is a coarse
systolic murmur with its epicenter within the 4th intercostal space on the left.
What is the most informative examination to confirm the clinical diagnosis?
A. Ventriculography
B. PCG
C. ECG
D. Echocardiography
E. Polycardiography
175. A 56-year-old patient complains of having persistent chest pain on the
right for the last 2 months. The pain is not associated with respiration. He also
complains of cough with blood-streaked sputum, weakness, decreased
performance, fatigue. Chest radiograph shows a globular shade of 4x6 cm
connected to the root of the lung in the lower part of the right lung. What is
the most likely diagnosis?
A. Peripheral lung cancer
B. Pneumonia
C. Tuberculoma
D. Metastasis
E. Lung abscess

176. A 18-year-old patient had subtotal strumectomy due to malignant


capillary cystadenoma of the thyroid gland. In 2 months there was a suspicion
of metastasis presence in the lungs. What rontgenological method is to be
used first?
A. Roentgenography of lungs
B. Angiopneumonography
C. Roentgenoscopy of lungs
D. Bronchography
E. Bronchoscopy

177. A 58-year-old patient was diagnosed basal-cell skin cancer, 1st stage.
Tumor is up to 1 cm in size and with up to 0,5 cm deep infiltration in tissues.
Tumor is localized in the right nasolabial area. Choose the most optimal
method of treatment
A. Short-distance roentgenotherapy
B. Long-distance roentgenotherapy
C. Chemotherapy
D. Surgical treatment
E. Long-distance gamma therapy

178. A 27-year-old patient complains of nasal haemorrhages, multiple


bruises on the anterior surface of the trunk and extremities, sudden weakness.
In blood: Hb- 74 g/l, reticulocytes - 16%, RBCs - 2,5×1012/l, platelets -
30cdot109/l, ESR- 25 mm/h. What is the most effective measure for the
treatment of thrombocytopenia?
A. Splenectomy
B. Cytostatics
C. Vitamin B12
D. Iron preparations
E. Hemotransfusion
179. 2 days ago a patient presented with acute pain in the left half of chest,
general weakness, fever and headache. Objectively: between the 4 and 5 rib on
the left the skin is erythematous, there are multiple groups of vesicles 2-4 mm
in diameter filled with transparent liquid. What disease are these symptoms
typical for?
A. Herpes zoster
B. Herpetiform Duhrings dermatosis
C. Herpes simplex
D. Pemphigus
E. Streptococcal impetigo

180. A woman while working in vegetable garden developed severe pain in


the loin. Lasagues and Nery tension signs are obviously marked on the right.
Lumbar lordosis is smoothed, movements are harshly restrained in lumbar
part of the spine. Right ankle (Achilles) reflex is absent. What kind of disease
can it be?
A. Lumbar-sacral radiculitis
B. Renal colic
C. Neuritis of femoral nerve
D. Lumbalgia
E. Hepatic colic

181. After lifting a load a patient felt undurable pain in the loin. He was
diagnosed with acute lumbosacral radiculitis. Which of the following is
contraindicated for this patient?
A. Warming procedures
B. Analgetics
C. Dehydrating drugs
D. Vitamins of B group
E. Intravenous injection of aminophylline

182. An unconscious 35-year-old patient has been delivered by an


ambulance to the intensive care unit. Objectively: the patient is in semicoma.
Moderate mydriasis is present. The reaction of pupils to light is reduced. The
reaction to verbal instructions is missing. AP is150/100 mm Hg, there is
tachycardia. Blood contains methanol. What antidote should be administered?
A. Ethanol
B. Naloxone
C. Thiamine chloride
D. Unithiol
E. Tavegil
183. A female, aged 20, after smoking notices a peculiar inebriation with the
feeling of burst of energy, elation, irreality and changing of surroundings: the
world gets full of bright colours, the objects change their dimensions, peoples
faces get cartoon features, loss of time and space judgement. What is the most
likely diagnosis?
A. Cocainism
B. Cannabism
C. Barbiturism
D. Morphinism
E. Nicotinism

184. A 75 y.o patient can not tell the month, date and season of the year.
After long deliberations she manages to tellher name. She is in irritable and
dissatisfied mood. She always carries a bundle with belongings with her, hides
a parcel with bread, shoes in her underwear in her bosom as well as
"invaluable books". What is the most probable diagnosis?
A. Senile dementia
B. Dissociated personality (psychopathy)
C. Presenile melancholia
D. Atherosclerotic (lacunar) dementia
E. Behaviour disorder

185. While lifting a heavy load a 39-year-old patient suddenly felt a severe
headache, pain in the interscapular region, and started vomiting. Objectively:
the pulse is rhythmic, 60/min, AP- 180/100 mm Hg. The patient is agitated.
He presents with photophobia, hyperacusis. There are positive Kernigs and
Brudzinskis signs on both sides. In blood: WBCs - 10cdot109/l. CSF is bloody,
cytosis is 240/3. What is the most likely diagnosis?
A. Subarachnoid haemorrhage
B. Ischemic stroke
C. Acute hypertonic encephalopathy
D. Sympathoadrenal crisis
E. Meningococcal meningitis

186. A 26-year-old male patient complains of piercing pain during


breathing, cough, dyspnea. Objectively: to- 37,3°C, respiration rate - 19/min,
heart rate = Ps- 92/min; AP- 120/80 mm Hg. Vesicular respiration. In the
inferolateral parts of chest auscultation in both inspiration and expiration
phase revealed noise that was getting stronger at phonendoscope pressing and
can be still heard after cough. ECG showed no pathological changes. What is
the most likely diagnosis?
A. Acute pleuritis
B. Intercostal neuralgia
C. Spontaneous pneumothorax
D. Pericarditis sicca
E. Subcutaneous emphysema

187. A 45-year-old patient, a sailor, was hospitalized on the 2nd day of the
disease. A week ago he returned from India. Complains of body temperature
of 41°C, severe headache, dyspnea, cough with frothy rusty sputum.
Objectively: the patient is pale, mucous membranes are cyanotic, breathing
rate is 24/min, tachycardia is present. In lungs: diminished breath sounds,
moist rales over both lungs, crepitation. What is the most likely diagnosis?
A. Pneumonic plaque
B. Sepsis
C. Influenza
D. Miliary tuberculosis
E. Ornithosis

188. HIV displays the highest tropism towards the following blood cells:
A. T-helpers
B. T-killers
C. Erythrocytes
D. Thrombocytes
E. T-suppressors

189. A 25-year-old patient complains of general weakness, dry cough,


sweating, subfebrile temperature. Objectively: lung auscultation reveals
vesicular resiration with no wheezing. Fluorogram shows focal shadows of
high intensity in the 1-2 segments of the right lung. Mantoux test gave a
reaction of 16 mm of induration. What clinical form of tuberculosis is most
likely?
A. Focal
B. Miliary
C. Disseminated
D. Infiltrative
E. Tuberculoma

190. A 22-year-old patient is a clerk. His working day runs in a conditioned


room. In summer he was taken by an acute disease with the following
symptoms: fever, dyspnea, dry cough, pleural pain, myalgia, arthralgia.
Objectively: moist rales on the right, pleural friction rub. X-ray picture showed
infiltration of the inferior lobe. In blood: WBC - 11cdot109/l, stab neutrophils
- 6%, segmented neutrophils - 70%, lymphocytes - 8%, ESR - 42 mm/h. What
is the ethiological factor of pneumonia?
A. Legionella
B. Pneumococcus
C. Streptococcus
D. Mycoplasm
E. Staphylococcus

191. A 53-year-old female patient complains of cardiac pain and rhythm


intermissions. She has experienced these presentations since childhood. The
patients father had a history of cardiac arrhythmias. Objectively: the patient is
in grave condition, Ps- 220 bpm, AP- 80/60 mm Hg. ECG results: heart rate -
215/min, extension and deformation of QRS complex accompanied by
atrioventricular dissociation; positive P wave. Some time later heart rate
reduced down to 45/min, there was a complete dissociation of P wave and
QRST complex. Which of the following will be the most effective treatment?
A. Implantation of the artificial pacemaker
B. beta-adrenoreceptor blocking agents
C. Calcium antagonists
D. Cardiac glycosides
E. Cholinolytics

192. A 49-year-old patient complains of dyspnea, cough. There are no


sputum discharges. He has repeatedly used salbutamol and intal but with no
effect. Objectively: he is only able to sit while leaning on the table. Cyanosis of
face, acrocyanosis are present. Breathing is shallow, laboured, in some parts it
cannot be auscultated; there are diffuse rales, expiration is significantly
prolonged. Heart sounds are muffled, tachycardia is present. Ps - 112/min,
AP- 110/70 mm Hg. Liver is located near the costal arch. There are no
peripheral edemata. What is your provisional diagnosis?
A. Status asthmaticus
B. Bronchiale asthma, moderate gravity
C. Cardiac asthma
D. Foreign object aspiration
E. Chronic obstructive bronchitis

193. Thrombosis of the coronary artery caused myocardial infarction. What


mechanisms of injury will be the dominating ones in this disease?
A. Calcium mechanisms
B. Electrolytoosmotic mechanisms
C. Protein mechanisms
D. Lipid mechanisms
E. Acidotic mechanisms

194. A female patient consulted a dermatologist about the rash on the trunk
and extremities. Objectively: interdigital folds, flexor surfaces of wrists and
navel region are affected with pairs of nodulo-cystic eruptions and crusts. The
rash is accompanied by skin itch that is getting stronger at night. What
external treatment should be administered?
A. 20% benzyl benzoate emulsion
B. 5% tetracycline ointment
C. 2% sulfuric paste
D. 5% sulfuric ointment
E. 5% naphthalan ointment

195. A 54 y.o. male patient suffers from dyspnea during mild physical
exertion, cough with sputum which is excreted with diffculty. On examination:
diffuse cyanosis. Is Barrel-chest. Weakened vesicular breathing with
prolonged expiration and dry whistling rales. AP is 140/80 mm Hg, pulse is
92 bpm, rhythmic. Spirography: vital capacity (VC)/predicted vital capacity-
65%, FEV1/FVC– 50%. Determine the type of respiratory insufficiency (RI)
A. RI of mixed type with prevailing obstruction
B. RI of restrictive type
C. RI of mixed type with prevailing restriction
D. There is no RI
E. RI of obstructive type

196. An ambulance had been called to a 48-year-old man. According to his


relatives, the patient had had three attacks of unconsciousness accompanied
by convulsions within 24 hours. On examination the doctor witnessed the
following attack: the patient lost consciousness and fell to the floor, developed
tonic, then clonic convulsions of trunk and extremities. The attack lasted 1
minute and ended with involuntary urination. Specify the kind of attack:
A. Epileptic seizure
B. Coma
C. Attack of hysteria
D. Syncope
E. Vegetative crisis

197. A patient is 60 years old, retired, worked as deputy director of a


research institute. Behavioural changes appeared 2 years ago after the death
of her husband: she stopped looking after herself and leaving the house; then
she refused to clean the apartment and cook. Mental status: temporal
disorientation. The patient does not understand many of the questions, is
confused; does not know how to cook soup or fasten a button. Her speech is
characterized by stumbling and logoclonia. She does not recognize doctors,
fellow patients. She cries a lot but can not explain the reason for tears. What is
the mechanism of this pathology?
A. Atrophy of the cerebral cortex
B. Atherosclerotic changes in cerebral vessels
C. Impaired conversion of dopamine to noradrenaline
D. Disorder of melatonin metabolism
E. Serotonin deficiency

198. A 26-year-old patient has abused alcohol since the age of 16, needs a
morning-after drink to cure hangover. He takes alcohol nearly every day, "a
little at a time". Twice a week he gets severely drunk. The patient works as a
motor mechanic, over the last 2 years work conflicts have become more
frequent. What medical and tactical actions should be taken in this case?
A. Voluntary consultation and treatment at an addiction clinic
B. Referral to treatment at an activity therapy centre
C. Compulsory treatment
D. Referral to medical-social expert commission for assessment of his working
ability
E. Consultation with a psychologist

199. A 26-year-old patient with left lower lobe pneumonia experiences an


acute chest pain on the left during coughing. Objectively: diffuse cyanosis,
extension of the left side of chest. Percussion reveals high tympanitis.
Auscultation reveals no respiratory murmurs above the left side of chest.
There is a deviation of the right cardiac border towards the midclavicular line.
What examination will be the most informative?
A. X-Ray
B. Bronchography
C. Bronchoscopy
D. Pneumotachometry
E. Spirography

200. A male patient presents with swollen ankles, face, eyelids, elevated AP-
160/100 mm Hg, pulse- 54 bpm, daily loss of albumine with urine- 4g. What
therapy is pathogenetic in this case?
A. Corticosteroids
B. NSAID
C. Diuretics
D. Calcium antagonists
E. Antibiotics

201. After myocardial infarction, a 50-year-old patient had an attack of


asthma. Objectively: bubbling breathing with frequency of 32/min, cough with
a lot of pink frothy sputum, acrocyanosis, swelling of the neck veins. Ps-
108/min, AP- 150/100 mm Hg. Heart sounds are muffled. Mixed moist rales
can be auscultated above the entire lung surface. What drug would be most
effective in this situation?
A. Nitroglycerin intravenously
B. Dopamine intravenously
C. Aminophylline intravenously
D. Pentamin intravenously
E. Strophanthin intravenously

202. During dynamic investigation of a patient the increase of central


venous pressure is combined with the decrease of arterial pressure. What
process is proved by such combination?
A. Increase of bleeding speed
B. Shunting
C. Developing of cardiac insufficiency
D. Depositing of blood in venous channel
E. Presence of hypervolemia

203. A male patient complains of heartburn which gets stronger while


bending the body, substernal pain during swallowing. There is a hiatus hernia
on X-ray. What disorder should be expected at gastroscopy?
A. Gastroesophageal reflux
B. Duodenal peptic ulcer
C. Gastric peptic ulcer
D. Chronic gastritis
E. Acute erosive gastritis

204. A 43 y.o. male complains of stomach pain, which relieves with


defecation, and is accompanied by abdominal winds, rumbling, the feeling of
incomplete evacuation or urgent need for bowel movement, constipation or
diarrhea in alternation. These symptoms have lasted for over 3 months. No
changes in laboratory tests. What is the most likely diagnosis?
A. Irritable bowel syndrome
B. Spastic colitis
C. Chronic enterocolitis, exacerbation phase
D. Atonic colitis
E. Colitis with hypertonic type dyskinesia

205. Against the background of angina a patient has developed pain in


tubular bones. Examination revealed generalized enlargement of lymph
nodes, hepatolienal syndrome, sternalgia. In blood: RBCs - 3,6*10^12/l, Hb-
87 g/l, thrombocytes - 45*10^9/l, WBCs - 13*10^9/l, blasts - 87%, stab
neutrophils - 1%, segmented neutrophils - 7%, lymphocytes - 5%, ESR - 55
mm/h. What is the most likely diagnosis?
A. Acute leukemia
B. Erythremia
C. Chronic myeloid leukemia
D. Multiple myeloma
E. Chronic lymphocytic leukemia

206. A 43-year-old male patient undergoing treatment for peptic ulcer


complains of weakness, dizziness, coffee-ground vomiting, melena. After
administration of haemostatics the patient’s condition has not improved, fresh
blood has shown up in the vomit, skin bruises of different sizes have appeared.
In blood: thrombocytes - 50cdot109/l, Lee-White clotting time - 35 minutes,
APTT - 80 seconds. In this case it is most rational to administer the following
preparation:
A. Fresh frozen plasma
B. Fibrinogen
C. Vikasol
D. Rheopolyglucinum
E. Heparin

207. A 38-year-old patient complains of inertness, subfebrile temperature,


enlargement of lymph nodes, nasal haemorrhages, bone pain. Objectively: the
patients skin and mucous membranes are pale, palpation revealed enlarged
painless lymph nodes; sternalgia; liver was enlarged by 2 cm, spleen - by 5 cm,
painless. In blood: erythrocytes - 2,7*10^12/l, Hb- 84 g/l, leukocytes -
58*10^9/l, eosinophils - 1%, stab neutrophils - 2%, segmented neutrophils -
12%, lymphocytes - 83%, lymphoblasts - 2%, smudge cells; ESR- 57 mm/h.
What is the most likely diagnosis?
A. Chronic lymphatic leukemia
B. Lymphogranulomatosis
C. Acute lymphatic leukemia
D. Chronic myeloleukemia
E. Acute myeloleukemia

208. A 24-year-old patient complains about putting on weight, limosis.


Objectively: the patients constitution is of hypersthenic type, body weight
index is 33,2 kg/m2, waist circumference is 100 cm. Correlation of waist
circumference to the thigh circumference is 0,95. What is the most likely
diagnosis?
A. Alimentary constitutional obesity of the I stage, abdominal type
B. Alimentary constitutional obesity of the III stage, gynoid type
C. Hypothalamic Itsenko-Cushing obesity of the II stage, gynoid type
D. Alimentary constitutional obesity of the II stage, abdominal type
E. Hypothalamic Itsenko-Cushing obesity of the I stage, abdominal type

209. A 58-year-old patient complains about sensation of numbness, sudden


paleness of II-IV fingers, muscle rigidness, intermittent pulse. The patient
presents also with polyarthralgia, dysphagia, constipations. The patients face
is masklike, solid edema of hands is present. The heart is enlarged;
auscultation revealed dry rales in lungs. In blood: ESR- 20 mm/h, crude
protein - 85/l, gamma-globulines - 25%. What is the most likely diagnosis?
A. Systemic scleroderma
B. Rheumatoid arthritis
C. Dermatomyositis
D. Systemic lupus erythematosus
E. Raynauds disease

210. A 45-year-old man has been exhibiting high activity for the last 2
weeks, he became talkative, euphoric, had little sleep, claimed being able "to
save the humanity" and solve the problem of cancer and AIDS, gave money
the starangers. What is the most likely diagnosis?
A. Maniacal onset
B. Agitated depression
C. Panic disorder
D. Schizo-affective disorder
E. Catatonic excitation

211. A 35-year-old patient complains of heartburn, sour eructation, burning,


compressing retrosternal pain and pain along the esophagus rising during
forward bending of body. The patient hasnt been examined, takes Almagel on
his own initiative, claims to feel better after its taking. Make a provisional
diagnosis:
A. Gastroesophageal reflux disease
B. Cardiospasm
C. Functional dyspepsia
D. Gastric ulcer
E. Duodenal ulcer

212. As a result of lifting a load a 62-year-old female felt acute pain in the
lumbar region, in a buttock, posterolateral surface of her right thigh, external
surface of the right shin and dorsal surface of foot. Objectively: weakness of
the anterior tibial muscle, long extensor muscle of the right toes, short
extensor muscle of the right toes. Low Achilles reflex on the right. Positive
Lasegues sign. What examination method would be the most effective for
specification of the diagnosis of discogenic compression of L_5 root?
A. Magnetic resonance scan
B. Electromyography
C. Spinal column X-ray
D. Angiography
E. Lumbar puncture

213. A 45-year-old female patient complaining of general weakness, nausea


and vomiting hass been delivered to a hospital by the ambulance. Recently
there has been a lack of appetite, weight loss. Objectively: hyperpigmentation
of skin, blood pressure at the rate of 70/45 mm Hg, bradycardia. Additional
studies revealed the reduced concentration of aldosterone and cortisol in
blood, decreased excretion of 17-ketosteroids and 17-oxyketosteroids in the
urine, hyponatremia, chloropenia, hypokalemia. What therapeutic measures
are required?
A. To administer glucocorticoids, mineralocorticoids, and a diet with
a high content of cooking salt
B. To administer aldosterone
C. To administer insulin
D. To prescribe a diet with a high content of cooking salt
E. To administer prednisolone

214. A 23-year-old female patient has a mental disease since the age of 18,
the course of disease has no remission periods. At a hospital the patient
mostly presents with non-purposeful foolish excitation: she makes stereotypic
grimaces, exposed, masturbating in front of a loud laugh, repeating the
stereotypical abusive shouts. The patient should be assigned:
A. Neuroleptics
B. Antidepressants
C. Nootropics
D. Mood stabilizers
E. Tranquilizers

215. A 40-year-old patient is registered in a narcological dispensary.


Somatically: skin is dramatically hyperemic, sclera are injected, hyperhidrosis
is present. AP- 140/100 mm Hg, heart rate - 100/min. Mental state:
autopsychic orientation is intact, allopsychic orientation is distorted. The
patient presents with motor anxiety. There is a look of fear on his face. He
refuses to talk about his problems and asks to release him immediately,
because he "may be killed." This state developed a day after a regular drinking
bout. What is your provisional diagnosis?
A. Delirium tremens
B. Alcoholic hallucinosis
C. Alcoholic paranoid
D. Organic delirium
E. Paranoia
216. A patient is being prepared for the operation on account of varix
dilatation of lower extremities veins. Examination of the patients soles
revealed flour-like desquamation along the skin folds. All the toenails are
greyish-yellow, thickened and partially decayed. What dermatosis should be
suspected?
A. Rubromycosis
B. Pityriasis versicolor
C. Microsporia
D. Microbial eczema
E. Candidosis

217. A 14-year-old patient with signs of internal haemorrhage has been


taken to a hospital after a fight. He has had haemophilia A since childhood. He
has been diagnosed with retroperitoneal hematoma. What should be
administered in the first place?
A. Cryoprecipitate
B. Dried plasma
C. Fresh blood
D. Platelet concentrate
E. Aminocapronic acid

218. A 58-year-old patient complains of a headache in the occipital region,


nausea, choking, opplotentes. The presentations appeared after a physical
exertion. Objectively: the patient is excited. Face is hyperemic. Skin is pale.
Heart sounds are regular, the 2nd aortic sound is accentuated. AP- 240/120
mm Hg, HR- 92/min. Auscultation reveals some fine moist rales in the lower
parts of the lungs. Liver is not enlarged. ECG shows signs of hypertrophy and
left ventricular overload. What is the most likely diagnosis?
A. Complicated hypertensic crisis, pulmonary edema
B. Uncomplicated hypertensic crisis
C. Community-acquired pneumonia
D. Acute myocardial infarction, pulmonary edema
E. Bronchial asthma exacerbation

219. A 37-year-old patient complains of pain in the lumbar spine that is


getting stronger during walking; restricted mobility, edema of the right side of
abdomen. He has a history of focal tuberculosis. X-ray picture shows the
destruction of the adjacent surfaces of the 1-2 vertebral bodies of the lumbar
spine, vertebral body height is decreased, intervertebral foramen is
undetectable. Abdominal ultrasound reveals a 15x20 cm formation in the
retroperitoneal space, there are echo signals of fluid presence. What is the
most likely diagnosis?
A. Tuberculous spondylitis of the lumbar spine
B. Spinal metastases
C. Fracture of the 1-2 vertebral bodies of the lumbar spine
D. Spondylolisthesis of the lumbar spine
E. Osteochondrosis

220. A 43-year-old female patient was delivered to the hospital in grave


condition. She has a history of Addisons disease. The patient had been
regularly taking prednisolone but a week before she stopped taking this drug.
Objectively: sopor, skin and visible mucous membranes are pigmented, skin
and muscle turgor is decreased. Heart sounds are muffled, rapid. AP- 60/40
mm Hg, heart rate - 96/min. In blood: Na - 120 millimole/l, K - 5,8
millimole/l. Development of this complication is primarily caused by the
deficit of the following hormone:
A. Cortisol
B. Corticotropin (ACTH)
C. Noradrenaline
D. Adrostendion
E. Adrenaline

221. Survey radiograph of a 52-year-old worker of an agglomeration plant


(28 years of experience, the concentration of metal dust is 22-37 mg/m3)
shows mildly pronounced interstitial fibrosis with diffused contrast well-
defined small nodular shadows. The patient has no complaints. Pulmonary
function is not compromised. What is the provisional diagnosis?
A. Siderosis
B. Silicatosis
C. Anthracosis
D. Silicosis
E. Anthraco-silicatosis

222. A 60-year-old patient complains of nearly permanent sensation of


heaviness and fullness in the epigastrium, that increases after eating, foul-
smelling eructation, occasional vomiting with food consumed 1-2 days ago,
weight loss. 12 years ago he was found to have an ulcer of pyloric channel. The
patient has taken ranitidine for periodic hunger pain. The patient’s condition
has been deteriorating over the last 3 months. Objectively: splashing sound in
the epigastrium is present. What kind of complication is it?
A. Pyloric stenosis
B. Functional pyloric spasm
C. Malignization of gastric ulcer
D. Foreign body in the stomach (bezoar)
E. Penetration of gastric ulcer
223. A 52-year-old patient works as a secretary and has 30 year record of
service. She complains of spasms in her right hand during working and
inability to type and write. Up to 80% of her work involves hand load. The
patient has been presenting with these symptoms for 2 years. Objectively: the
right hand is tense, there is an increase in muscle tone, attempts to write cause
spasms. Examination revealed no pathological changes of CNS. What is the
most likely diagnosis?
A. Spastic form of coordination neurosis
B. Paretic form of coordination neurosis
C. Neuralgic form of coordination neurosis
D. Hysteric neurosis
E. Chronic manganese intoxication

224. A 22-year-old vegetarian patient with signs of malnutrition consulted a


doctor about smell and taste distortion, angular stomatitis. Objectively:
expressively blue sclerae. The patient was diagnosed with iron deficiency
anemia. What is the dominating clinical syndrome?
A. Sideropenic
B. Haemologic
C. Myelodysplastic
D. Haemolytic
E. Anaemic

225. A patient complains of retrosternal pain, difficult swallowing, over 10


kg weight loss within three months, general weakness. In blood: hypochromic
anaemia, neutrophilic leukocytosis. In feces: weakly positive Gregersens
reaction. On esophagram a filling defect with ill-defined serrated edges shows
up along a large portion of the esophagus. What is the most likely diagnosis?
A. Esophageal carcinoma
B. Sideropenic dysphagia
C. Esophageal achalasia
D. Benign tumour
E. Peptic ulcer

226. An emergency doctor has diagnosed a 32-year-old woman with


generalized convulsive status epilepticus. The deterioration in the patients
condition is caused by a sudden gap in the epilepsy treatment. Specify the
doctors further tactics:
A. Hospitalization in the intensive care unit
B. Outpatient monitoring by a neurosurgeon
C. Hospitalization in the department of neurosurgery
D. Hospitalization in the department of neurology
E. Outpatient monitoring by a neuropathologist
227. A 42-year-old female lives in the basement, is unemployed,
undernourished. She complains of having general weakness, hair loss, brittle
nails for six months, likes to eat chalk. Objectively: the patient is emaciated,
pale, has dry skin. Peripheral lymph nodes are not enlarged. Liver is +1,5 cm.
In blood: RBCs - 1,8×1012/l, Hb- 62 g/l, colour index - 0,78, reticulocytes -
0,5o/_oo, ESR- 18 mm/h. Leukogram exhibits no pathology. What is a
provisional diagnosis?
A. Nutritional iron deficiency anaemia
B. Chronic hepatitis
C. Acquired haemolytic anaemia
D. Congenital haemolytic anaemia
E. B12-deficiency anaemia

228. A 20-year-old patient complains of breath shortness, continuous dull


heart pain, irritability. Objectively: general condition is satisfactory, the pulse
is labile, AP- 130/60 mm Hg. ECG shows repolarization disorder. The patient
has been diagnosed with cardiac-type neurocirculatory dystonia. The patient
should receive treatment under the following conditions:
A. Outpatient treatment
B. Inpatient treatment at the cardiology department
C. Inpatient treatment at the therapeutic department
D. Inpatient treatment at the cardiac surgery department
E. Inpatient treatment at the psychiatric department

229. A 38-year-old male works within the range of ionizing radiation. At a


routine medical examination he presents no problems. In blood: RBCs -
4,5×1012/l, Hb- 80 g/l, WBCs - 2,8cdot109/l, thrombocytes - 30cdot109/l.
Decide if this person can work with sources of ionizing radiation:
A. Working with radioactive substances and other sources of ionizing
radiation is contraindicated
B. The patient can only work with radioactive substances of low activity
C. The patient is allowed to work with radioactive substances
D. The patient can be allowed to work after an extended medical examination
E. The patient is allowed to work with radioactive substances for the limited
period of time

230. A patient who undergoes treatment at a tuberculosis clinic has


complained of having progressing headache for the last 3 weeks. Neurological
examination reveals rigidity of occipital muscles, no focal symptoms. What is
your provisional diagnosis?
A. Tuberculous meningitis
B. Brain tumour
C. Convexital arachnoiditis
D. Myelitis
E. Chorea minor

231. A patient with chronic suppurative otitis has developed severe


headache, vomiting, body temperature rise. The meningeal symptoms are
present. There are no focal neurological symptoms. The further tactics of a
doctor should be:
A. Urgent hospitalization and diagnostic lumbar puncture
B. Referral for a consultation with otolaryngologist
C. Regular medical check-up
D. Skull radiography
E. Administration of anti-inflammatory drugs

232. Routine examination of a 16-year-old boy revealed the presence of


three heart sounds on auscultation. The third sound is low and occurs in early
diastole, there is no additional murmur. In history: pneumonia six months
ago. The patient presents no problems. Examination revealed hyposthenia,
underdevelopment of muscles. Laboratory and instrumental studies reveald
no peculiarities. What is the origin of the additional heart sound?
A. Physiological III sound
B. The sound of the tricuspid valve opening
C. Protodiastolic gallop rhythm
D. The sound of the mitral valve opening
E. Pericardial diastolic sound

233. A patients condition is getting worse towards evening: she becomes


excited, complains of "internal anxiety", "a weight on her heart", foreboding of
evil - "something evil will happen to me or my family". The patient is sad,
melancholic, has poor appetite and sleep disorders. Specify the kind of mental
disorder:
A. Anxious depression
B. Endogenous depression
C. Somatized depression
D. Hypochondriac depression
E. Agitated depression

Krok 2 – 2014 Gynecology Base


1. A 52-year-old woman suffering from obesity, complains of bloody discharges
from sexual paths during 4 days. Last normal menses were 2 years ago.
Histological investigation of biopsy of the endometrium has revealed
adenomatous hyperplasia. What reason from the mentioned below caused the
development of disease?
A. Excessive transformation of preandrogens from adipose tissues
B. The increased contents of follicle-stimulating hormone
C. Supersecretion of androgens by the cortex of paranephroses.
D. Hypersecretion of estrogens by tissues of the organism.
E. Poor aromatization of preandrogens due to hypothyroidism

2. A 40-year-old woman complains of colic pains in the lower part of abdomen


and abundant bloody discharges from genital tract. Last 2 years she had
menses for 15-16 days, abundant, with clots, painful. Had 2 medical abortions.
In bimanual investigation: from the canal of the cervix uteri - a fibromatous
node, 3 cm in diameter, on the thin stem. Discharges are bloody,
moderate.Choose the correct tactics.
A. Operation: untwisting of born node
B. Phase by phase vitamin therapy
C. Hysterectomy without ovaries
D. Supravaginal ablation of the uterus without ovaries
E. Hormonal hemostasis

3. A 40-year-old woman complains of yellow color discharges from the vagina.


Bimanual examination: no pathological changes. Smear test: Trichomonas
vaginalis and mixed flora. Colposcopy: two hazy fields on the front labium,
with a negative Iodum probing. What is your tactics?
A. Treatment of specific colpitis with the subsequent biopsy
B. Cervix ectomy
C. Cryolysis of cervix uteri
D. Diathermocoagulation of the cervix uteri
E. Specific treatment of Trichomonas colpitis

4. A 32 y.o. woman consulted a gynecologist about having abundant long menses


within 3 months. Bimanual investigation: the body of the uterus is enlarged
according to about 12 weeks of pregnancy, distorted, tuberous, of dense
consistence. Appendages are not palpated. Histological test of the uterus body
mucosa: adenocystous hyperplasia of endometrium. Optimal medical tactics:
A. Surgical treatment
B. Radial therapy
C. Phase by phase vitamin therapy
D. Hormonetherapy
E. Phytotherapy

5. A woman complains of having slight dark bloody discharges and mild pains in
the lower part of abdomen for several days. Last menses were 7 weeks ago.
The pregnancy test is positive. Bimanual investigation: the body of the uterus
indicates for about 5-6 weeks of pregnancy, it is soft, painless. In the left
appendage there is a retort-like formation, 7х5 cm large, mobile, painless.
What examination is necessary for detection of fetus localization?
A. Ultrasound
B. Colposcopy
C. Cystoscopy
D. Hysteroscopy
E. Hromohydrotubation

6. A woman was hospitalised with full-term pregnancy. Examination: the uterus


is tender, the abdomen is tense, cardiac tones of the fetus are not auscultated.
What is the most probable complication of pregnancy?
A. Premature detachment of normally posed placenta
B. Premature labor
C. Acute hypoxia of a fetus
D. Hydramnion
E. Back occipital presentation

7. By the end of the 1st period of physiological labor clear amniotic fluid came
off. Contractions lasted 35-40 sec every 4-5min. Heartbeat of the fetus was
100 bpm. The BP was 140/90 mm Hg. What is the most probable diagnosis?
A. Acute hypoxia of the fetus
B. Premature labor
C. Back occipital presentation
D. Hydramnion
E. Premature detachment of normally posed placenta

8. A pregnant woman in her 40th week of pregnancy undergoes obstetric


examination: the cervix of uterus is undeveloped. The oxytocin test is
negative. Examination at 32 weeks revealed: AP 140/90 mm Hg, proteinuria 1
g/l, peripheral edemata. Reflexes are normal. Choose the most correct tactics:
A. Labour stimulation after preparation
B. Complex therapy of gestosis for 2 days
C. Absolute bed rest for 1 month
D. Caesarian section immediately
E. Complex therapy of gestosis for 7 days

9. Which gestational age gives the most accurate estimation of weeks of


pregnancy by uterine size?
A. Less that 12 weeks
B. Between 21 and 30 weeks
C. Between 12 and 20 weeks
D. Between 31 and 40 weeks
E. Over 40 weeks

10. A 26 year old woman had the second labour within the last 2 years with
oxytocin application. The childs weight is 4080 g. After the placent birth there
were massive bleeding, signs of hemorrhagic shock. Despite the injection of
contractive agents, good contraction of the uterus and absence of any cervical
and vaginal disorders, the bleeding proceeds. Choose the most probable cause
of bleeding:
A. Atony of the uterus
B. Hypotonia of the uterus
C. Hysterorrhexis
D. Injury of cervix of the uterus
E. Delay of the part of placenta

11. A woman is admitted to maternity home with discontinued labor activity and
slight bloody discharges from vagina. The condition is severe, the skin is pale,
consciousness is confused. BP is 80/40 mm Hg. Heartbeat of the fetus is not
heard. There was a Cesarian section a year ago. Could you please determine
the diagnosis?
A. Hysterorrhesis
B. Cord presentation
C. Expulsion of the mucous plug from cervix uteri
D. Premature expulsion of amniotic fluid
E. Placental presentation

12. On the first day after labour a woman had the rise of temperature up to 39°C.
Rupture of fetal membranes took place 36 hours before labour. Examination
of the bacterial flora of cervix of the uterus revealed hemocatheretic
streptococcus of A group. The uterus body is soft, tender. Discharges are
bloody, with admixtures of pus. Specify the most probable postnatal
complication:
A. Metroendometritis
B. Infective contamination of the urinary system
C. Apostasis of sutures after the episiotomy
D. Thrombophlebitis of veins of the pelvis
E. Infectious hematoma

13. Rise in temperature up to 39°C was registered the next day after a woman had
labor. Fetal membranes rupture took place 36 hours prior to labors. The
examination of the bacterial flora of cervix uteri revealed the following:
haemolytic streptococcus of group A. The uterus tissue is soft, tender.
Discharges are bloody, with mixing of pus. Establish the most probable
postnatal complication.
A. Metroendometritis
B. Infective contamination of the urinary system
C. Apostatis of stitches after the episiotomy
D. Thrombophlebitis of veins of the pelvis
E. Infected hematoma

14. A woman of a high-risk group (chronic pyelonephritis in anamnesis) had


vaginal delivery. The day after labour she complained of fever and loin pains,
frequent urodynia. Specify the most probable complication:
A. Infectious contamination of the urinary system
B. Infectious hematoma
C. Apostasis of sutures after episiotomy
D. Endometritis
E. Thrombophlebitis of veins of the pelvis

15. 13 months after the first labor a 24-year-old patient complained of


amenorrhea. Pregnancy ended in Caesarian section because of premature
detachment of normally positioned placenta which resulted in blood loss at
the rate of 2000 ml owing to disturbance of blood clotting. Choose the most
suitable investigation:
A. Estimation of gonadotropin rate
B. Progesteron assay
C. USI of small pelvis organs
D. Computer tomography of head
E. Estimation of testosteron rate in blood serum

16. In 13 months after the first labor a 24-year-old woman complains of


amenorrhea. Cesarian section was conducted as a result of premature
detachment of normally posed placenta. Hemorrhage has made low fidelity of
2000 ml due to breakdown of coagulation of blood. Choose the most suitable
investigation.
A. Determination of the level of gonadotropin
B. Progesteron test
C. Determination of the contents of testosteron-depotum in blood serum.
D. Computer tomography of the head
E. Ultrasound of organs of a small pelvis

17. In the woman of 24 years about earlier normal menstrual function, cycles
became irregular, according to tests of function diagnostics - anovulatory. The
contents of prolactin in blood is boosted. Choose the most suitable
investigation:
A. Computer tomography of the head
B. USI of organs of small pelvis
C. Determination of the contents of testosteron-depotum in blood serum
D. Progesterone assay
E. Determination of the level of gonadotropins

18. A woman in her 39th week of pregnancy, the second labour, has regular birth
activity. Uterine contractions take place every 3 minutes. What criteria
describe the beginning of the II labor stage the most precisely?
A. Cervical dilatation by no less than 4 cm
B. Cervical smoothing over 90%
C. Presenting part is in the lower region of small pelvis
D. Rupture of fetal bladder
E. Duration of uterine contractions over 30 seconds

19. A 20-year-old woman is having timed labor continued for 4 hours. Light
amniotic fluid came off. The fetus head is pressed to the orifice in the small
pelvis. The anticipated fetus mass is 4000,0 gpm 200,0 g. Heartbeat of the
fetus is normal. Intrinsic examination: cervix is absent, disclosure – 2 cm, the
fetal membranes are not present. The head is in 1-st plane of the pelvis, a
sagittal suture is in the left slanting dimension. What is the purpose of
glucose-calcium-hormone - vitaminized background conduction?
A. Prophylaxes of weakness of labor activity
B. Fetus hypoxia prophylaxes
C. Treatment of weakness of labor activity.
D. Antenatal preparation
E. Labor stimulation

20. A 24 years old primipara was hospitalised with complaints about discharge of
the amniotic waters. The uterus is tonic on palpation. The position of the fetus
is longitudinal, it is pressed with the head to pelvic outlet. Palpitation of the
fetus is rhythmical, 140 bpm, auscultated on the left below the navel. Internal
examination: cervix of the uterus is 2,5 cm long, dense, the external os is
closed, light amniotic waters out of it. Point a correct component of the
diagnosis:
A. Antenatal discharge of the amniotic waters
B. Early discharge of the amniotic waters
C. The end of the 1st stage of labour
D. Pathological preterm labour
E. The beginning of the 1st stage of labour

21. A 34 y.o. woman in her 29-th week of pregnancy, that is her 4-th labor to
come, was admitted to the obstetric department with complaints of sudden
and painful bloody discharges from vagina that appeared 2 hours ago. The
discharges are profuse and contain grumes. Cardiac funnction of the fetus is
rhytmic, 150 strokes in the minute, uterus tone is normal. The most probable
provisional diagnosis will be:
A. Placental presentation
B. Disseminated intravascular coagulation syndrome
C. Vasa previa
D. Detachment of normally located placenta
E. Bloody discharges

22. A 29 year old patient underwent surgical treatment because of the benign
serous epithelial tumour of an ovary. The postoperative period has elapsed
without complications. What is it necessary to prescribe for the
rehabilitational period:
A. Hormonotherapy and proteolytic enzymes
B. Magnitotherapy and vitamin therapy
C. The patient does not require further care
D. Antibacterial therapy and adaptogens
E. Lasertherapy and enzymotherapy

23. A 34-year-old woman with 10-week pregnancy (the second pregnancy) has
consulted gynaecologist to make a record in patient chart. There was a
hydramnion previous pregnancy, the birth weight of a child was 4086 g. What
tests are necessary first of all?
A. The test for tolerance to glucose
B. Fetus cardiophonography
C. Ultrasound of the fetus
D. Determination of the contents of alpha fetoprotein
E. Bacteriological test of discharge from the vagina

24. A 26 y.o. woman complains of sudden pains in the bottom of abdomen


irradiating to the anus, nausea, giddiness, bloody dark discharges from sexual
tracts for one week, the delay of menses for 4 weeks. Signs of the peritoneum
irritation are positive. Bimanual examination: borders of the uterus body and
its appendages are not determined because of sharp painfullness. The
diverticulum and painfullness of the back and dextral fornixes of the vagina
are evident. What is the most probable diagnosis?
A. Broken tubal pregnancy
B. Acute right-side adnexitis
C. Apoplexy of the ovary
D. Torsion of the crus of the ovary tumour
E. Acute appendicitis
25. At the gynaecological department there is a patient of 32 years with the
diagnosis: "acute bartholinitis". Body temperature is 38,2°C, leucocytes count
10,4×109/L, the ESR is 24 mm/hour. In the area of big gland of the
vestibulum - the dermahemia, the sign of the fluctuation, sharp tenderness
(pain). What is the most correct tactics of the doctor?
A. Surgical dissecting, a drainage of an abscess of the gland,
antibiotics
B. Antibiotics, Sulfanilamidums
C. Antibiotic therapy
D. Antibiotics, detoxication and biostimulants.
E. Surgical dissection, drainage of the abscess of the gland

26. A primagravida with pregnancy of 37-38 weeks complains of headache,


nausea, pain in epigastrium. Objective: the skin is acyanotic. Face is hydropic,
there is short fibrillar twitching of blepharons, muscles of the face and the
inferior extremities. The look is fixed. AP- 200/110 mm Hg; sphygmus of 92
bpm, intense. Respiration rate is 32/min. Heart activity is rhythmical.
Appreciable edemata of the inferior extremities are present. Urine is cloudy.
What medication should be administered?
A. Droperidolum of 0,25% - 2,0 ml
B. Dibazolum of 1% - 6,0 ml
C. Hexenalum of 1% - 2,0 ml
D. Pentaminum of 5% - 4,0 ml
E. Papaverine hydrochloride of 2% - 4,0 ml

27. An onset of severe preeclampsia at 16 weeks gestation might be caused by:


A. Hydatidiform mole
B. Anencephaly
C. Maternal renal disease
D. Interventricular defect of the fetus
E. Twin gestation

28. A woman had the rise of temperature up to 39°C on the first day after labour.
The rupture of fetal membranes took place 36 hours before labour. The
investigation of the bacterial flora of cervix of the uterus revealed
hemocatheretic streptococcus of group A. The uterus body is soft, tender.
Discharges are bloody, mixed with pus. Specify the most probable postnatal
complication:
A. Metroendometritis
B. Infection of the urinary system
C. Apostatis of junctures after the episiotomy
D. Thrombophlebitis of pelvic veins
E. Infected hematoma

29. A 24 y.o. patient 13 months after the first labour consulted a doctor about
amenorrhea. Pregnancy has concluded by a Cesarean section concerning to a
premature detachment of normally posed placenta hemorrhage has made low
fidelity 2000 ml owing to breakdown of coagulability of blood. Choose the
most suitable investigation:
A. Determination of the level of Gonadotropins
B. Progesteron assay
C. USI of organs of a small pelvis
D. Computer tomography of the head
E. Determination of the contents of Testosteron-Depotum in Serum of blood

30. A 34 year old woman in the 10th week of gestation (the second pregnancy)
consulted a doctor of antenatal clinic in order to be registered there. In the
previous pregnancy hydramnion was observed, the childs birth weight was
4086 g. What examination method should be applied in the first place?
A. The test for tolerance to glucose
B. US of fetus
C. Bacteriological examination of discharges from vagina
D. Determination of the contents of fetoproteinum
E. A cardiophonography of fetus

31. A 10 y.o. boy was ill with angina 2 weeks ago, has complaints of joint pain and
stiffness of his left knee and right elbow. There was fever (38,50) and ankle
disfunction, enlargement of cardiac dullness by 2 cm, tachycardia, weakness
of the 1st sound, gallop rhythm, weak systolic murmur near apex. What
diagnosis corresponds with such symptoms?
A. Acute rheumatic fever
B. Reiters disease
C. Reactive arthritis
D. Systemic lupus erythematosis
E. Juvenile rheumatoid arthritis

32. The disease began acutely. The frequent watery stool developed 6 hours ago.
The bodys temperature is normal. Then the vomiting was joined. On
examination: his voice is hoarse, eyes are deeply sunken in the orbits. The
pulse is frequent. Blood pressure is low. There is no urine. What is the
preliminary diagnosis?
A. Cholera
B. Typhoid fever
C. Salmonellosis
D. Toxic food-borne infection
E. Dysentery

33. At term of a gestation of 40 weeks height of standing of a uterine fundus is less


then assumed for the given term. The woman has given birth to the child in
weight of 2500 g, a length of a body 53 cm, with an assessment on a scale of
Apgar of 4-6 points. Labor were fast. The cause of such state of the child were:
A. Chronic fetoplacental insufficiency
B. Delay of an intra-uterine fetation
C. Infection of a fetus
D. Prematurity
E. Placental detachment

34. A pregnant woman may be diagnosed with hepatitis if it is confirmed by the


presence of elevated:
A. SGOT (ALT)
B. WBCs
C. Sedimentation rates
D. Alkaline phosphatase
E. BUN

35. A woman, aged 40, primigravida, with infertility in the medical history, on the
42-43 week of pregnancy. Labour activity is weak. Longitudinal presentation
of the fetus, I position, anterior position. The head of the fetus is engaged to
pelvic inlet. Fetus heart rate is 140 bmp, rhythmic, muffled. Cervix dilation is
4 cm. On amnioscopy: greenish colour of amniotic fluid and fetal membranes.
Cranial bones are dense, cranial sutures and small fontanel are diminished.
What should be tactics of delivery?
A. Caesarean section
B. Medication sleep, amniotomy, labour stimulation
C. Fetal hypoxia treatment, in the II period - forceps delivery
D. Amniotomy, labour stimulation, fetal hypoxia treatment
E. Fetal hypoxia treatment, conservative delivery

36. An endometrial adenocarcinoma that has extended to the uterine serosa


would be classified as stage:
A. IIIA
B. IIA
C. IC
D. IIB
E. IVAB
37. Which of the methods of examination is the most informative in the
diagnostics of a tube infertility?
A. Laparoscopy with chromosalpingoscopy
B. Hysterosalpingography
C. Pertubation
D. Transvaginal echography
E. Bicontrast pelviography

38. A pregnant woman (35 weeks), aged 25, was admitted to the hospital because
of bloody discharges. In her medical history there were two artificial
abortions. In a period of 28-32 weeks there was noted the onset of
hemorrhage and USD showed a placental presentation. The uterus is in
normotonus, the fetus position is transversal (Ist position). The heartbeats is
clear, rhythmical, 140 bpm. What is the further tactics of the pregnant woman
care?
A. To perform a delivery by means of Cesarean section
B. To introduct the drugs to increase the blood coagulation and continue
observation
C. To keep the intensity of hemorrhage under observation and after the bleeding
is controlled to prolong the pregnancy
D. Stimulate the delivery by intravenous introduction of oxytocin
E. To perform the hemotransfusion and to prolong the pregnancy

39. A woman, primagravida, consults a gynecologist on 05.03.2012. A week ago


she felt the fetus movements for the first time. Last menstruation was on
10.01.2012. When should she be given maternity leave?
A. 8 August
B. 22 August
C. 5 September
D. 11 July
E. 25 July

40. Condition of a parturient woman has been good for 2 hours after live birth:
uterus is thick, globe-shaped, its bottom is at the level of umbilicus, bleeding
is absent. The clamp put on the umbilical cord remains at the same level,
when the woman takes a deep breath or she is being pressed over the
symphysis with the verge of hand, the umbilical cord drows into the vagina.
Bloody discharges from the sexual tracts are absent. What is the doctors
further tactics?
A. To do manual removal of afterbirth
B. To do curettage of uterine cavity
C. To introduct oxitocine intravenously
D. To apply Abduladze method
E. To apply Credes method
41. The woman who has delivered twins has early postnatal hypotonic uterine
bleeding reached 1,5% of her bodyweight. The bleeding is going on.
Conservative methods to arrest the bleeding have been found ineffective. The
conditions of patient are pale skin, acrocyanosis, oliguria. The woman is
confused. The pulse is 130 bpm, BP– 75/50 mm Hg. What is the further
treatment?
A. Uterine extirpation
B. Uterine vessels ligation
C. Putting clamps on the uterine cervix
D. Inner glomal artery ligation
E. Supravaginal uterine amputation

42. A 26 y.o. woman complains of a mild bloody discharge from the vagina and
pain in the lower abdomen. She has had the last menstruation 3,5 months ago.
The pulse is 80 bpm. The blood pressure (BP) is 110/60 mm Hg and body
temperature is 36,6°C. The abdomen is tender in the lower parts. The uterus is
enlarged up to 12 weeks of gestation. What is your diagnosis?
A. Inevitable abortion
B. Incomplete abortion
C. Disfunctional bleeding
D. Complete abortion
E. Incipient abortion

43. 18 y.o. woman complains of pain in the lower abdomen. Some minutes before
she has suddenly appeared unconscious at home. The patient had no menses
within last 3 months. On examination: pale skin, the pulse- 110 bpm, BP-
80/60 mm Hg. The Schyotkins sign is positive. Hb- 76 g/L. The vaginal
examination: the uterus is a little bit enlarged, its displacement is painful.
There is also any lateral swelling of indistinct size. The posterior fornix of the
vagina is tendern and overhangs inside. What is the most probable diagnosis?
A. Impaired extrauterine pregnancy
B. Acute appendicitis
C. Twist of cystoma of right uterine adnexa
D. Ovarian apoplexy
E. Acute salpingoophoritis

44. A 20 y.o. pregnant woman with 36 weeks of gestation was admitted to the
obstetrical hospital with complains of pain in the lower abdomen and bloody
vaginal discharge. The general condition of the patient is good. Her blood
pressure is 120/80 mm Hg. The heart rate of the fetus is 140 bpm, rhythmic.
Vaginal examination: the cervix of the uterus is formed and closed. The
discharge from vagina is bloody up to 200 ml per day. The head of the fetus is
located high above the minor pelvis entry. A soft formation was defined
through the anterior fornix of the vagina. What is the probable diagnosis?
A. Placental presentation
B. Premature placental separation
C. Threatened premature labor
D. Incipient abortion
E. Uterine rupture

45. In the gynecologic office a 28 y.o. woman complains of sterility within three
years. The menstrual function is not impaired. There were one artificial
abortion and chronic salpingo-oophoritis in her case history. Oral
contraceptives were not used. Her husbands analysis of semen is without
pathology. What diagnostic method will you start from the workup in this case
of sterility?
A. Hysterosalpingography
B. Ultra sound investigation
C. Hysteroscopia
D. Diagnostic scraping out of the uterine cavity
E. Hormone investigation

46. A 28-year-old patient underwent endometrectomy as a result of incomplete


abortion. Blood loss was at the rate of 900 ml. It was necessary to start
hemotransfusion. After transfusion of 60 ml of erythrocytic mass the patient
presented with lumbar pain and fever which resulted in hemotransfusion
stoppage. 20 minutes later the patients condition got worse: she developed
adynamia, apparent skin pallor, acrocyanosis, profuse perspiration. to-
38,5°C, Ps- 110/min, AP- 70/40 mm Hg. What is the most likely diagnosis?
A. Hemotransfusion shock
B. DIC syndrome
C. Septic shock
D. Hemorrhagic shock
E. Anaphylactic shock

47. A 58-year-old female patient came to the antenatal clinic complaining of


bloody light-red discharges from the genital tracts. Menopause is 12 years.
Gynaecological examination revealed age involution of externalia and vagina;
uterine cervix was unchanged, there were scant bloody discharges from
uterine cervix, uterus was of normal size; uterine appendages were not
palpable; parametria were free. What is the most likely diagnosis?
A. Uterine carcinoma
B. Cervical carcinoma
C. Granulosa cell tumor of ovary
D. Atrophic colpitis
E. Abnormalities of menstrual cycle of climacteric nature
48. The results of a separate diagnostic curettage of the mucous of the uterus
cervix and body made up in connection with bleeding in a postmenopausal
period: the scrape of the mucous of the cervical canal revealed no pathology,
in endometrium - the highly differentiated adenocarcinoma was found.
Metastases are not found. What method of treatment is the most correct?
A. Surgical treatment and hormonotherapy
B. Surgical treatment and radial therapy
C. Surgical treatment + chemotherapy
D. Radial therapy

49. A 27 y.o. woman complains of having the disorders of menstrual function for 3
months, irregular pains in abdomen. On bimanual examination: in the dextral
appendage range of uterus there is an elastic spherical formation, painless, 7
cm in diameter. USI: in the right ovary - a fluid formation, 4 cm in diameter,
unicameral, smooth. What method of treatment is the most preferable?
A. Prescription of an estrogen-gestogen complex for 3 months with
repeated examination
B. Dispensary observation of the patient
C. Chemotherapeutic treatment
D. Anti-inflammatory therapy
E. Operative treatment

50. A 40 year old patient complains of yellowish discharges from the vagina.
Bimanual examination revealed no pathological changes. The smear contains
Trichomonas vaginalis and blended flora. Colposcopy revealed two hazy fields
on the frontal labium, with a negative Iodine test. Your tactics:
A. Treatment of specific colpitis and subsequent biopsy
B. Cervix ectomy
C. Cryolysis of cervix of the uterus
D. Diathermocoagulation of the cervix of the uterus
E. Specific treatment of Trichomonas colpitis

51. A 26-year-old secundipara at 40 weeks of gestation arrived at the maternity


ward after the beginning of labor activity. 2 hours before, bursting of waters
occurred. The fetus was in a longitudinal lie with cephalic presentation.
Abdominal circumference was 100 cm, fundal height - 42 cm. Contractions
occurred every 4-5 minutes and lasted 25 seconds each. Internal obstetric
examination revealed cervical effacement, opening by 4 cm. Fetal bladder was
absent. Fetal head was pressed against the pelvic inlet. What complication
arose in childbirth?
A. Early amniorrhea
B. Secondary uterine inertia
C. Clinically narrow pelvis
D. Discoordinated labor
E. Primary uterine inertia

52. A 28-year-old parturient complains about headache, vision impairment,


psychic inhibition. Objectively: AP- 200/110 mm Hg, evident edemata of legs
and anterior abdominal wall. Fetus head is in the area of small pelvis. Fetal
heartbeats is clear, rhythmic, 190/min. Internal examination revealed
complete cervical dilatation, fetus head was in the area of small pelvis. What
tactics of labor management should be chosen?
A. Forceps operation
B. Cesarean
C. Conservative labor management with episiotomy
D. Stimulation of labor activity
E. Embryotomy

53. A 28 year old woman had the second labour and born a girl with
manifestations of anemia and progressing jaundice. The childs weight was 3
400 g, the length was 52 cm. The womans blood group is B (III) Rh-, the
fathers blood group is A (III) Rh+, the childs blood group is B (III) Rh+. What
is the cause of anemia?
A. Rhesus incompatibility
B. Antigen A incompatibility
C. Antigen AB incompatibility
D. Intrauterine infection
E. Antigen B incompatibility

54. A 48 year old female patient complains about contact haemorrhage. Speculum
examination revealed hypertrophy of uterus cervix. It resembles of
cauliflower, it is dense and can be easily injured. Bimanual examination
revealed that fornices were shortened, uterine body was nonmobile. What is
the most probable diagnosis?
A. Cervical carcinoma
B. Metrofibroma
C. Cervical pregnancy
D. Cervical papillomatosis
E. Endometriosis

55. A 37 y.o. primigravida woman has been having labor activity for 10 hours.
Labor pains last for 20-25 seconds every 6-7 minutes. The fetus lies in
longitude, presentation is cephalic, head is pressed upon the entrance to the
small pelvis. Vaginal examination results: cervix of uterus is up to 1 cm long,
lets 2 transverse fingers in. Fetal bladder is absent. What is the most probable
diagnosis?
A. Primary uterine inertia
B. Discoordinated labor activity
C. Pathological preliminary period
D. Secondary uterine inertia
E. Normal labor activity

56. Laparotomy was performed to a 54 y.o. woman on account of big formation in


pelvis that turned out to be one-sided ovarian tumor along with considerable
omental metastases. The most appropriate intraoperative tactics involves:
A. Ablation of omentum, uterus and both ovaries with tubes
B. Biopsy of omentum
C. Ablation of an ovary and omental metastases
D. Ablation of omentum and both ovaries with tubes
E. Biopsy of an ovary

57. A parturient complains about pain in the mammary gland. Palpation revealed
a 3×4 cm large infiltration, soft in the centre. Body temperature is 38,5°C.
What is the most probable diagnosis?
A. Acute purulent mastitis
B. Pleuritis
C. Birth trauma
D. Retention of milk
E. Pneumonia

58. A 43 y.o. patient complains of formation and pain in the right mammary
gland, rise of temperature up to 37,2°C during the last 3 months. Condition
worsens before the menstruation. On examination: edema of the right breast,
hyperemia, retracted nipple. Unclear painful infiltration is palpated in the
lower quadrants. What is the most probable diagnosis?
A. Cancer of the right mammary gland
B. Tuberculosis of the right mammary gland
C. Right-side chronic mastitis
D. Right-side acute mastitis
E. Premenstrual syndrome

59. A 14 year old girl complains of profuse bloody discharges from genital tracts
during 10 days after suppresion of menses for 1,5 month. Similiar bleedings
recur since 12 years on the background of disordered menstrual cycle. On
rectal examination: no pathology of the internal genitalia. In blood: Hb - 70
g/l, RBC- 2,3×1012/l, Ht - 20. What is the most probable diagnosis?
A. Juvenile bleeding, posthemorrhagic anemia
B. Polycyst ovarian syndrome
C. Werlholfs disease
D. Hormonoproductive ovary tumor
E. Incomplete spontaneous abortion

60. A 33-year-old woman was urgently brought to clinic with complaints of the
pain in the lower part of the abdomen, mostly on the right, irradiating to
rectum, she also felt dizzy. The above mentioned complaints developed
acutely at night. Last menses were 2 weeks ago. On physical exam: the skin is
pale, Ps - 92 bpm, t- 36,6°C, BP- 100/60 mm Hg. The abdomen is tense,
slightly tender in lower parts, peritoneal symptoms are slightly positive. Hb-
98 g/L. What is the most probable diagnosis?
A. Apoplexy of the ovary
B. Renal colic
C. Intestinal obstruction
D. Acute appendicitis
E. Abdominal pregnancy

61. A secundipara has regular birth activity. Three years ago she had cesarean
section for the reason of acute intrauterine hypoxia. During parodynia she
complains of extended pain in the area of postsurgical scar. Objectively: fetus
pulse is rhythmic - 140 bpm. Vaginal examination shows 5 cm cervical
dilatation. Fetal bladder is intact. What is the tactics of choice?
A. Cesarean section
B. Obstetrical forceps
C. Augmentation of labour
D. Waiting tactics of labor management
E. Vaginal delivery

62. A 54-year-old female patient consulted a doctor about bloody discharges from
the genital tracts after 2 years of amenorrhea. USI and bimanual examination
revealed no genital pathology. What is the tactics of choice?
A. Fractional biopsy of lining of uterus and uterine mucous
membranes
B. Contracting drugs
C. Hysterectomy
D. Estrogenic haemostasia
E. Styptic drugs

63. Examination of a just born placenta reveals defect 2x3 cm large. Hemorrhage
is absent. What tactic is the most reasonable?
A. Manual uretus cavity revision
B. Prescription of uterotonic medicines
C. Parturient supervision
D. Instrumental uterus cavity revision
E. External uterus massage

64. A 27 y.o. gravida with 17 weeks of gestation was admitted to the hospital.
There was a history of 2 spontaneous miscarriages. On bimanual examination:
uterus is enlarged to 17 weeks of gestation, uterus cervix is shortened, isthmus
allows to pass the finger tip. The diagnosis is isthmico-cervical insufficiency.
What is the doctors tactics?
A. To place suture on the uterus cervix
B. To perform amniocentesis
C. To interrupt pregnancy
D. To administer tocolytic therapy
E. To administer hormonal treatment

65. A 27-year-old woman presents at the maternity welfare centre because of


infertility. She has had sexual life in marriage for 4 years, doesnt use
contraceptives. She hasnt get pregnant. On examination: genital development
is without pathology, uterine tubes are passable, basal (rectal) temperature is
one-phase during last 3 menstrual cycles. What is the infertility cause?
A. Anovular menstrual cycle
B. Abnormalities in genital development
C. Chronic adnexitis
D. Immunologic infertility
E. Genital endometriosis

66. A 43 y.o. woman complains of contact hemorrhages during the last 6 months.
Bimanual examination: cervix of the uterus is enlarged, its mobility is
reduced. Mirrors showed the following: cervix of the uterus is in the form of
cauliflower. Chrobak and Schiller tests are positive. What is the most probable
diagnosis?
A. Cancer of cervix of the uterus
B. Cervical pregnancy
C. Leukoplakia
D. Nascent fibroid
E. Polypus of the cervis of the uterus

67. A 26-year-old woman gave birth to a child 6 months ago. She applied to
gynecologist complaining of menstruation absence. The child is breast-fed.
Vagina exam: uterus is of normal form, dense consistence. What is the most
probable diagnosis?
A. Physiological amenorrhea
B. Sheehans syndrome
C. Gestation
D. Pseudoamenorrhea
E. Ashermans syndrome

68. A primagravida in her 20th week of gestation complains about pain in her
lower abdomen, blood smears from the genital tracts. The uterus has an
increased tonus, the patient feels the fetus movements. Bimanual examination
revealed that the uterus size corresponded the term of gestation, the uterine
cervix was contracted down to 0,5 cm, the external orifice was open by 2 cm.
The discharges were bloody and smeary. What is the most likely diagnosis?
A. Incipient abortion
B. Abortion in progress
C. Missed miscarriage
D. Incomplete abortion
E. Risk of abortion

69. Full-term pregnancy. Body weight of the pregnant woman is 62 kg. The fetus
has the longitudinal position, the fetal head is pressed against the pelvic inlet.
Abdominal circumference is 100 cm. Fundal height is 35 cm. What is the
approximate weight of the fetus?
A. 3 kg 500 g
B. 4 kg
C. 3 kg
D. 4 kg 500 g
E. 2 kg 500 g

70. A patient was admitted to the hospital with complaints of periodical pain in
the lower part of abdomen that gets worse during menses, weakness, malaise,
nervousness, dark bloody smears from vagina directly before and after
menses. Bimanual examination revealed that uterus body is enlarged,
appendages cannot be palpated, posterior fornix has tuberous surface.
Laparoscopy revealed: ovaries, peritoneum of rectouterine pouch and
pararectal fat have "cyanotic eyes". What is the most probable diagnosis?
A. Disseminated form of endometriosis
B. Chronic salpingitis
C. Ovarian cystoma
D. Tuberculosis of genital organs
E. Polycystic ovaries

71. A gravida with 7 weeks of gestation is referred for the artificial abortion. On
operation while dilating cervical canal with Hegar dilator No.8 a doctor
suspected uterus perforation. What is immediate doctors tactics to confirm
the diagnosis?
A. Probing of uterus cavity
B. Ultrasound examination
C. Bimanual examination
D. Laparoscopy
E. Metrosalpingography

72. A pregnant woman in her 8th week was admitted to the hospital for artificial
abortion. In course of operation during dilatation of cervical canal of uterus by
means of Hegars dilator No.8 the doctor suspected uterus perforation. What is
the immediate tactics for confirmation of this diagnosis?
A. Uterine probing
B. US examination
C. Metrosalpingography
D. Laparoscopy
E. Bimanual examination

73. A 59 year old female patient applied to a maternity welfare clinic and
complained about bloody discharges from the genital tracts. Postmenopause is
12 years. Vaginal examination revealed that external genital organs had signs
of age involution, uterus cervix was not erosive, small amount of bloody
discharges came from the cervical canal. Uterus was of normal size, uterine
appendages were unpalpable. Fornices were deep and painless. What method
should be applied for the diagnosis specification?
A. Separated diagnosic curretage
B. Culdoscopy
C. Puncture of abdominal cavity through posterior vaginal fornix
D. Laparoscopy
E. Extensive colposcopy

74. A 25-year-old woman complains of profuse foamy vaginal discharges, foul,


burning and itching in genitalia region. She has been ill for a week.
Extramarital sexual life. On examination: hyperemia of vaginal mucous,
bleeding on touching, foamy leucorrhea in the urethral area. What is the most
probable diagnosis?
A. Trichomonas colpitic
B. Bacterial vaginosis
C. Chlamydiosis
D. Gonorrhea
E. Vagina candidomicosis
75. A 26 year old woman who delivered a child 7 months ago has been suffering
from nausea, morning vomiting, sleepiness for the last 2 weeks. She suckles
the child, menstruation is absent. She hasnt applied any contraceptives. What
method should be applied in order to specify her diagnosis?
A. Ultrasonic examination
B. Speculum examination
C. Palpation of mammary glands and pressing-out of colostrum
D. Roentgenography of small pelvis organs
E. Bimanual vaginal examination

76. A newborns head is of dolichocephalic shape, that is front-to-back elongated.


Examination of the occipital region revealed a labour tumour located in the
middle between the prefontanel and posterior fontanel. Specify the type of
fetal presentation:
A. Posterior vertex presentation
B. Presentation of the bregma
C. Face presentation
D. Brow presentation
E. Anterior vertex presentation

77. A woman consulted a doctor on the 14th day after labour about sudden pain,
hyperemy and induration of the left mammary gland, body temperature rise
up to 39°C, headache, indisposition. Objectively: fissure of nipple,
enlargement of the left mammary gland, pain on palpation. What pathology
would you think about in this case?
A. Lactational mastitis
B. Phlegmon of mammary gland
C. Fibrous adenoma of the left mammary gland
D. Lacteal cyst with suppuration
E. Breast cancer

78. A young woman applied to gynecologist due to her pregnancy of 4-5 weeks.
The pregnancy is desirable. Anamnesis stated that she had rheumatism in the
childhood. Now she has combined mitral heart disease with the priority of
mitral valve deficiency. When will she need the inpatient treatment (what
periods of pregnancy)?
A. 8-12 weeks, 28–32 weeks, 37 weeks
B. 12-16 weeks, 27-28 weeks, 37-38 weeks
C. 16 weeks, 34 weeks, 39-40 weeks
D. 6-7weeks, 16 weeks, 38 weeks
E. 10-12 weeks, 24 weeks, 37-38 weeks
79. A woman in the first half of pregnancy was brought to clinic by an ambulance.
Term of pregnancy is 36 weeks. She complains of intensive pain in the
epigastrium, had vomiting for 2 times. Pain started after the patient had eaten
vinaigrette. Swelling of lower extremities. BP - 140/100 mm Hg. Urine became
curd after boiling. What is the most probable diagnosis?
A. Preeclampsia
B. Food toxicoinfection
C. Exacerbation of pyelonephritis
D. Dropsy of pregnant women
E. Nephropathy of the 3rd degree

80.A 13 year old girl consulted the school doctor on account of moderate bloody
discharge from the genital tracts, which appeared 2 days ago. Secondary
sexual characters are developed. What is the most probable cause of bloody
discharge?
A. Menarche
B. Werlhofs disease
C. Haemophilia
D. Juvenile hemorrhage
E. Endometrium cancer

81. In 10 min after childbirth by a 22-year-old woman, the placenta was


spontaneousely delivered and 100 ml of blood came out. Woman weight - 80
kg, infant weight - 4100 g, length - 53 cm. The uterus contracted. In 10
minutes the hemorrhage renewed and the amount of blood constitued 300 ml.
What amount of blood loss is permissible for this woman?
A. 400 ml
B. 650 ml
C. 300 ml
D. 1000 ml
E. 500 ml

82. A pregnant woman was registered in a maternity welfare clinic in her 11th
week of pregnancy. She was being under observation during the whole term,
the pregnancy course was normal. What document must the doctor give the
pregnant woman to authorize her hospitalization in maternity hospital?
A. Exchange card
B. Medical certificate
C. Sanitary certificate
D. Appointment card for hospitalization
E. Individual prenatal record
83. After examination a 46-year-old patient was diagnosed with left breast cancer
T2N2M0, cl. gr. II-a. What will be the treatment plan for this patient?
A. Radiation therapy + operation + chemotherapy
B. Operation only
C. Radiation therapy only
D. Chemotherapy only
E. Operation + radiation therapy

84. Immediately after delivery a woman had haemorrhage, blood loss exceeded
postpartum haemorrhage rate and was progressing. There were no symptoms
of placenta detachment. What tactics should be chosen?
A. Manual removal of placenta and afterbirth
B. Instrumental revision of uterine cavity walls
C. Intravenous injection of methylergometrine with glucose
D. Removal of afterbirth by Credes method
E. Uterus tamponade

85. A 30 y.o. primigravida woman has got intensive labor pain every 1-2 minutes
that lasts 50 seconds. The disengagement has started. The perineum with the
height of 4 cm has grown pale. What actions are necessary in this situation?
A. Episiotomy
B. Expectant management
C. Perineotomy
D. Perineum protection
E. Vacuum extraction of fetus

86. A 30-year-old gravida consulted a gynecologist about bright red bloody


discharges from the vagina in the 32 week of gestation. She was hospitalized
with a suspicion of placental presentation. Under what conditions is it rational
to conduct the internal examination in order to make a diagnosis?
A. In the operating room prepared for the operation
B. In the admission ward of maternity hospital
C. The examination is not to be conducted because of risk of profuse
haemorrhage
D. In the delivery room keeping to all the aseptics regulations
E. In the examination room of antenatal clinic

87. A 28 y.o. primagravida, pregnancy is 15-16 weaks of gestation, presents to the


maternity clinics with dull pain in the lower part of the abdomen and in
lumbar area. On vaginal examination: uterus cervix is 2,5 cm, external
isthmus allows to pass the finger tip. Uterus body is enlarged according to the
pregnancy term. Genital discharges are mucous, mild. What is the diagnosis?
A. Threatened spontaneous abortion
B. Hydatid molar pregnancy
C. Placenta presentation
D. Spontaneous abortion which has begun
E. Stopped pregnancy

88. A primapara with pelvis size 25-28-31-20 cm has active labor activity. Waters
poured out, clear. Fetus weight is 4500 g, the head is engaged to the small
pelvis inlet. Vastens sign as positive. Cervix of uterus is fully dilated. Amniotic
sac is absent. The fetus heartbeat is clear, rhythmic, 136 bpm. What is the
labor tactics?
A. Caesarean section
B. Obstetrical forseps
C. Vacuum extraction of the fetus
D. Conservative tactics of labor
E. Stimulation of the labor activity

89. Internal obstetric examination of a parturient woman revealed that the


sacrum hollow was totally occupied with fetus head, ischiadic spines couldnt
be detected. Sagittal suture is in the straight diameter, occipital fontanel is
directed towards symphysis. In what plane of small pelvis is the presenting
part of the fetus?
A. Plane of pelvic outlet
B. Wide pelvic plane
C. Plane of pelvic inlet
D. Over the pelvic inlet
E. Narrow pelvic plane

90. A 30 y.o. woman has the 2-nd labour that has been lasting for 14 hours.
Hearbeat of fetus is muffled, arrhythmic, 100/min. Vaginal examination:
cervix of uterus is completely opened, fetus head is level with outlet from
small pelvis. Saggital suture is in the straight diameter, small crown is near
symphysis. What is the further tactics of handling the delivery?
A. Use of obstetrical forceps
B. Use of cavity forceps
C. Cesarean section
D. Stimulation of labour activity by oxytocin
E. Cranio-cutaneous (Ivanovs) forceps

91. During examination of a patient, masses in the form of condyloma on a broad


basis are found in the area of the perineum. What is the tactics of the doctor?
A. To send a woman into dermatological and venerological centre
B. Cryodestruction of condyloms
C. Chemical coagulator treatment
D. Antiviral treatment
E. Surgical ablation of condyloms

92. A woman at 30 weeks pregnant has had an attack of eclampsia at home. On


admission to the maternity ward AP is 150/100 mm Hg. Predicted fetal weight
is 1500 g. There is face and shin pastosity. Urine protein is 0,66o/oo.
Parturient canal is not ready for delivery. An intensive complex therapy has
been started. What is the correct tactics of this case management?
A. Delivery by cesarean section
B. Continue therapy and prolong pregnancy for 3-4 weeks
C. Treat preeclampsia and achieve the delivery by way of conservative
management
D. Labor induction by intravenous oxytocin or prostaglandins
E. Continue therapy and prolong pregnancy for 1-2 weeks

93. A 28 year old woman has bursting pain in the lower abdomen during
menstruation; chocolate-like discharges from vagina. It is known from the
anamnesis that the patient suffers from chronic adnexitis. Bimanual
examination revealed a tumour-like formation of heterogenous consistency
7*7 cm large to the left from the uterus. The formation is restrictedly movable,
painful when moved. What is the most probable diagnosis?
A. Endometrioid cyst of the left ovary
B. Fibromatous node
C. Tumour of sigmoid colon
D. Exacerbation of chronic adnexitis
E. Follicular cyst of the left ovary

94. Vaginal inspection of a parturient woman revealed: cervix dilation is up to 2


cm, fetal bladder is intact. Sacral cavity is free, sacral promontory is reachable
only with a bent finger, the inner surface of the sacrococcygeal joint is
accessible for examination. The fetus has cephalic presentation. Sagittal
suture occupies the transverse diameter of pelvic inlet, the small fontanel to
the left, on the side. What labor stage is this?
A. Cervix dilatation stage
B. Prodromal stage
C. Placental stage
D. Stage of fetus expulsion
E. Preliminary stage
95. A 68-year-old patient consulted a doctor about a tumour in her left mammary
gland. Objectively: in the upper internal quadrant of the left mammary gland
there is a neoplasm up to 2,5 cm in diameter, dense, uneven, painless on
palpation. Regional lymph nodes are not enlarged. What is the most likely
diagnosis?
A. Cancer
B. Lipoma
C. Fibroadenoma
D. Cyst
E. Mastopathy

96. A 40-year-old female patient has been observing profuse menses accompanied
by spasmodic pain in the lower abdomen for a year. Bimanual examination
performed during menstruation revealed a dense formation up to 5 cm in
diameter in the cervical canal. Uterus is enlarged up to 5-6 weeks of
pregnancy, movable, painful, of normal consistency. Appendages are not
palpable. Bloody discharges are profuse. What is the most likely diagnosis?
A. Nascent submucous fibromatous node
B. Cervical myoma
C. Algodismenorrhea
D. Abortion in progress
E. Cervical carcinoma

97. A 29-year-old patient complains of sterility. Sexual life is for 4 years being
married, does not use contraception. There was no pregnancy before. On
physical examination, genitals are developed normally. Uterine tubes are
passable. Rectal temperature during three menstrual cycles is monophase.
What is the most probable reason for sterility?
A. Anovulatory menstrual cycle
B. Genital endometriosis
C. Anomalies of genitals development
D. Chronic adnexitis
E. Immunologic sterility

98. A 45 y.o. woman complains of contact bleedings during 5 months. On


speculum examination: hyperemia of uterus cervix, looks like cauliflower,
bleeds on probing. On bimanual examination: cervix is of densed consistensy,
uterus body isnt enlarged, mobile, nonpalpable adnexa, parametrium is free,
deep fornixes. What is the most likely diagnosis?
A. Cancer of cervix of uterus
B. Cancer of body of uterus
C. Cervical pregnancy
D. Polypose of cervix of uterus
E. Fibromatous node which is being born
99. 10 minutes after delivery a woman discharged placenta with a tissue defect
5×6 cm large. Discharges from the genital tracts were profuse and bloody.
Uterus tonus was low, fundus of uterus was located below the navel.
Examination of genital tracts revealed that the uterine cervix, vaginal walls,
perineum were intact. There was uterine bleeding with following blood
coagulation. Your actions to stop the bleeding:
A. To make manual examination of uterine cavity
B. To administer uterotonics
C. To introduce an ether-soaked tampon into the posterior fornix
D. To apply hemostatic forceps upon the uterine cervix
E. To put an ice pack on the lower abdomen

100. On the 5th day after labor body temperature of a 24-year-old parturient
suddenly rose up to 38,7°C. She complains about weakness, headache,
abdominal pain, irritability. Objectively: AP- 120/70 mm Hg, Ps- 92 bpm, to-
38,7°C. Bimanual examination revealed that the uterus was enlarged up to 12
weeks of pregnancy, it was dense, slightly painful on palpation. Cervical canal
lets in 2 transverse fingers, discharges are moderate, turbid, with foul smell.
In blood: skeocytosis, lymphopenia, ESR - 30 mm/h. What is the most likely
diagnosis?
A. Endometritis
B. Metrophlebitis
C. Lochiometra
D. Parametritis
E. Pelviperitonitis

101. A 20 y.o. patient complains of amenorrhea. Objectively: hirsutism,


obesity with fat tissue prevailing on the face, neck, upper part of body. On the
face there are acne vulgaris, on the skin - striae cutis distense. Psychological
and intellectual development is normal. Gynecological condition: external
genitals are moderately hairy, acute vaginal and uterine hypoplasia. What
diagnosis is the most probable?
A. Itsenko-Cushing syndrome
B. Turners syndrome
C. Shichans syndrome
D. Babinski-Froehlich syndrome
E. Stein-Leventals syndrome

102. A 27 y.o. woman suffers from pyelonephritits of the only kidney. She
presents to the maternity welfare centre because of suppresion of menses for
2,5 months. On examination pregnancy 11 weeks of gestation was revealed. In
urine: albumine 3,3 g/L, leucocytes cover the field of vision. What is doctors
tactics in this case?
A. Immediate pregancy interruption
B. Pregnancy interruption at 24-25 weeks
C. Maintenance of pregnancy till delivery term
D. Pregnancy interruption after urine normalization
E. Maintenance of pregnancy till 36 weeks

103. An 18-year-old primigravida in her 27-28 week of gestation underwent


an operation on account of acute phlegmonous appendicitis. In the
postoperative period it is necessary to take measures for prevention of the
following pregnancy complication:
A. Noncarrying of pregnancy
B. Late gestosis
C. Fetus hypotrophy
D. Intestinal obstruction
E. Premature placenta detachment

104. A 24-year-old female patient complains of acute pain in the lower


abdomen that turned up after a physical stress. She presents with nausea,
vomiting, dry mouth and body temperature 36,6°C. She has a right ovarian
cyst in history. Bimanual examination reveals that uterus is dense, painless, of
normal size. The left fornix is deep, uterine appendages arent palpable, the
right fornix is contracted. There is a painful formation on the right of uterus.
Its round, elastic and mobile. It is 7×8 cm large. In blood: leukocytosis with
the left shit. What is the most likely diagnosis?
A. Ovarian cyst with pedicle torsion
B. Acute metritis
C. Extrauterine pregnancy
D. Right-sided pyosalpinx
E. Subserous fibromyoma of uterus

105. A parturient woman is 23 years old. Vaginal obstetric examination


reveals full cervical dilatation. There is no fetal bladder. Fetal head is in the
plane of pelvic outlet. Sagittal suture is in mesatipellic pelvis, anterior fontanel
is closer to pubes. The fetal head diameter in such presentation will be:
A. Suboccipito-bregmaticus
B. Suboccipitio-frontalis
C. Mento-occipitalis
D. Fronto-occipitalis recta
E. Biparietal
106. A pregnant 26-year-old woman was admitted to a hospital for
abdominal pain and bleeding from the genital tract. Bimanual examination
revealed that uterus was the size of 9 weeks of pregnancy, the cervical canal let
a finger through. Fetal tissues could be palpated in the orifice. There was
moderate vaginal bleeding. What is the tactics of choice?
A. Instrumental extraction of fetal tissue
B. Therapy for the maintenance of pregnancy
C. Administration of hormones
D. Surveillance
E. Hemostatic and antianemic therapy

107. A 42-year-old woman has had hyperpolymenorrhea and progressing


algodismenorrhea for the last 10 years. Gynaecological examination revealed
no changes of uterine cervix; discharges are moderate, of chocolate colour,
uterus is slightly enlarged and painful, appendages are not palpable, the
fornices are deep and painless. What is the most likely diagnosis?
A. Uterine endometriosis
B. Uterine carcinoma
C. Endomyometritis
D. Adnexal endmetriosis
E. Subserous uterine fibromyoma

108. On the tenth day after discharge from the maternity house a 2-year-old
patient consulted a doctor about body temperature rise up to 39°C, pain in the
right breast. Objectively: the mammary gland is enlarged, there is a
hyperemized area in the upper external quadrant, in the same place there is an
ill-defined induration, lactostasis, fluctuation is absent. Lymph nodes of the
right axillary region are enlarged and painful. What is the most likely
diagnosis?
A. Lactational mastitis
B. Erysipelas
C. Abscess
D. Dermatitis
E. Tumour

109. During the dynamic observation over a parturient woman in the second
stage of labor it was registered that the fetal heart rate fell down to 90-
100/min and didnt come to normal after contractions. Vaginal examination
revealed the complete cervical dilatation, the fetal head filling the entire
posterior surface of the pubic symphysis and sacral hollow; the sagittal suture
lied in the anteroposterior diameter of the pelvic outlet, the posterior
fontanelle was in front under the pubic arch. What plan for further labour
management should be recommended?
A. Application of forceps minor
B. Episiotomy
C. Caesarean section
D. Application of cavity forceps
E. Stimulation of labour activity by intravenous injection of oxytocin

110. A 27-year-old sexually active female complains of numerous vesicles on


the right sex lip, itch and burning. Eruptions regularly turn up before
menstruation and disappear 8-10 days later. What is the most likely
diagnosis?
A. Herpes simplex virus
B. Primary syphilis
C. Bartholinitis
D. Cytomegalovirus infection
E. Genital condylomata

111. A 36-year-old female pesented to a gynecological hospital with a


significant bleeding from the genital tract and a 1-month delay of
menstruation. Bimanual examination revealed soft barrel-shaped cervix.
Uterus was of normal size, somewhat softened. Appendages were
unremarkable on both sides. Speculum examination revealed that the cervix
was cyanotic, enlarged, with the the external orifice disclosed up to 0,5 cm.
Urine hCG test was positive. What is the most likely diagnosis?
A. Cervical pregnancy
B. Uterogestation
C. Threatened miscarriage
D. Ectopic pregnancy
E. Abortion in progress

112. A 26-year-old woman complains of having bloody discharges from the


genitals for the last 14 days, abdominal pain, general fatiguability, weakness,
weight loss, body temperature rise, chest pain, obstructed respiration. 5 weeks
ago she underwent induced abortion in the 6-7 week of gestation. Objectively:
the patient is pale and inert. Bimanual examination revealed that the uterus
was enlarges up to 8-9 weeks of gestation. In blood: Hb- 72 g/l. Urine test for
chorionic gonadotropin gave the positive result. What is the most likely
diagnosis?
A. Chorioepithelioma
B. Uterus perforation
C. Uterine carcinoma
D. Uterine fibromyoma
E. Metroendometritis
113. A 28-year-old patient complains of discomfort, acute pain in the lower
third of the left labia majora. The disease began suddenly after menstruation.
Objectively: body temperature is 38°C. The left labia majora has a formation
to 3 cm diameter, with hyperemic surface, extremely painful to the touch, with
symptoms of fluctuation. What is the most likely diagnosis?
A. Acute bartholinitis
B. Vulvar fibroid
C. Vulvar cancer
D. Bartholin gland cyst
E. Hypertrophy of the labia

114. A 28-years-old woman complains of nausea and vomiting about 10


times per day. She has been found to have body weight loss and xerodermia.
The pulse is 100 bpm. Body temperature is 37,2°C. Diuresis is low. USI shows
5-6 weeks of pregnancy. What is the most likely diagnosis?
A. Moderate vomiting of pregnancy
B. I degree preeclampsia
C. Food poisoning
D. Premature abortion
E. Mild vomiting of pregnancy

115. A 40 week pregnant secundipara is 28 years old. Contractions are very


active. Retraction ring is at the level of navel, the uterus is hypertonic, in form
of hourglass. On auscultation the fetal heart sounds are dull, heart rate is
100/min. AP of the parturient woman is 130/80 mm Hg. What is the most
likely diagnosis?
A. Risk of hysterorrhexis
B. Attack of eclampsia
C. Disturbed labour
D. Mazolysis
E. Complete hysterorrhexis

116. After delivery and revision of placenta there was found the defect of
placental lobule. General condition of woman is normal, uterus is firm, there
is moderate bloody discharge. Speculum inspection of birth canal shows
absence of lacerations and raptures. What action is nesessary?
A. Manual exploration of the uterine cavity
B. External massage of uterus
C. Urine drainage, cold on the lower abdomen
D. Introduction of hemostatic medications
E. Introduction of uterine contracting agents
117. A 25 y.o. patient complains of body temperature rise up to 37°C, pain at
the bottom of her abdomen and vaginal discharges. Three days ago, when she
was in her 11th week of pregnancy, she had an artificial abortion. Objectibely:
cervix of uterus is clean, uterus is a little bit enlarged in size, painful.
Appendages cannot be determined. Fornixes are deep, painless. Vaginal
discharges are sanguinopurulent. What is the most probable diagnosis?
A. Postabortion endometritis
B. Postabortion uterus perforation
C. Parametritis
D. Hematometra
E. Pelvic peritonitis

118. A 25 y.o. pregnant woman in her 34th week was taken to the maternity
house in grave condition. She complains of headache, visual impairment,
nausea. Objectively: solid edemata, AP- 170/130 mm Hg. Suddenly there
appeared fibrillary tremor of face muscles, tonic and clonic convulsions,
breathing came to a stop. After 1,5 minute the breathing recovered, there
appeared some bloody spume from her mouth. In urine: protein - 3,5 g/L.
What is the most probable diagnosis?
A. Eclampsia
B. Cerebral hemorrhage
C. Epilepsy
D. Cerebral edema
E. Stomach ulcer

119. A 51-year-old patient complains of having intensive bloody discharges


from vagina for 15 days after delay of menstruation for 2,5 months. In
anamnesis: disorders of menstrual function during a year, at the same time
she felt extreme irritability and had sleep disorders. US examination results:
uterus corresponds with age norms, appendages have no pecularities,
endometrium is 14 mm thick. What is the doctors tactics?
A. Diagnostic curettage of uterine cavity
B. TORCH-infection test
C. Hysterectomy
D. Conservative treatment of bleeding
E. Supravaginal amputation of uterus without appendages

120. An 18 y.o. patient complains of painfulness and swelling of mammary


glands, headaches, irritability, edemata of lower extremities. These symptoms
have been present since the begin of menarche, appear 3-4 days before regular
menstruation. Gynecological examination revealed no pathology. What is the
most probable diagnosis?
A. Premenstrual syndrome
B. Neurasthenia
C. Mastopathy
D. Disease of cardiovascular system
E. Renal disease

121. A 22-year-old female patient complains of dull pain in her right iliac
area that she has been experiencing for a week, morning sickness and
gustatory change. She has a history of menstruation delay for 3 weeks.
Objectively: AP- 80/50 mm Hg, pulse is 78 bpm, body temperature is 37°C.
Bimanual examination reveals that uterus is enlarged, soft, mobile and
painless. Uterine appendages are palpable on the right, there is a dense,
elastic and moderately painful formation 3x4 cm large. What is the most likely
diagnosis?
A. Progressing fallopian pregnancy
B. Right ovarian cyst
C. Acute appendicitis
D. Uterogestation
E. Interrupted fallopian pregnancy

122. A 30 y.o. parturient woman was taken to the maternity house with
complaints of having acute, regular labour pains that last 25-30 seconds every
1,5-2 minutes. Labour activity began 6 hours ago. Uterus is in higher tonus,
head of the fetus is above the opening into the small pelvis. Fetal heartbeat is
136/min. P.V: cervical dilatation is 4 cm, uterine fauces is spasming at a
height of parodynia. Head is level with opening into the small pelvis, it is
being pushed off. What is the most probable diagnosis?
A. Discoordinated labour activity
B. Pathological preliminary period
C. Normal labour activity
D. Primary powerless labour activity
E. Secondary powerless labour activity

123. A primigravida woman appealed to the antenatal clinic on the 22.03.03


with complaints of boring pain in the lower part of abdomen. Anamnesis
registered that her last menstruation was on the 4.01.03. Bimanual
examination revealed that uterine servix is intact, external fauces is closed,
uterus is enlarged up to the 9-th week of pregnancy, movable, painless. What
complication can be suspected?
A. Risk of abortion in the 9-th week of pregnancy
B. Hysteromyoma
C. Vesicular mole
D. Abortion that started in the 9-th week of pregnancy
124. A 25-year-old female patient complains about having amenorrhea for 3
years. She associates it with difficult labour complicated by massive
hemorrhage. She also complains of loss of weight, hair fragility and loss, lack
of appetite and depression. Objective examination reveals no pathological
changes of uterus and its appendages. What is the disease pathogenesis?
A. Hypoproduction of gonadotropin
B. Hyperproduction of estrogens
C. Hypoproduction of progesterone
D. Hyperproduction of prolactin
E. Hyperproduction of androgens

125. A 30-year-old patient consulted a doctor about menstruation absence


for 2 years after labour, loss of hair, body weight loss. The labour was
complicated by a haemorrhage caused by uterus hypotonia. Objectively: the
patient is asthenic, external genitals are hypoplastic, the uterus body is small
and painless. The appendages are not palpaple. What is the most likely
diagnosis?
A. Sheehans syndrome
B. Turners syndrome
C. Galactorrhea-amenorrhea syndrome
D. Exhausted overy syndrome
E. Ovarian amenorrhea

126. A 28-year-old patient has been admitted to the gynecological


department three days after a casual coitus. She complains about pain in her
lower abdomen and during urination, profuse purulent discharges from the
vagina, body temperature rise up to 37,8°C. The patient was diagnosed with
acute bilateral adnexitis. Supplemental examination revealed: the 4th degree
of purity of the vaginal secretion, leukocytes within the whole visual field,
diplococcal bacteria located both intra- and extracellularly. What is the
etiology of acute adnexitis in this patient?
A. Gonorrheal
B. Chlamydial
C. Staphylococcal
D. Trichomonadal
E. Colibacterial

127. A 25-year-old woman came to a maternity welfare clinic and


complained about being unable to conceive within 3 years of regular sexual
life. Examination revealed weight gain, male pattern of hair distribution on
the pubis, excessive pilosis of thighs. Ovaries were dense and enlarged, basal
temperature was monophase. What is the most likely diagnosis?
A. Sclerocystosis of ovaries
B. Adrenogenital syndrome
C. Gonadal dysgenesis
D. Premenstrual syndrome
E. Tubo-ovaritis

128. A 28 year old patient complained about prolongation of intermenstrual


periods up to 2 months, hirsutism. Gynaecological examination revealed that
the ovaries were enlarged, painless, compact, uterus had no pecularities.
Pelvic ultrasound revealed that the ovaries were 4-5 cm in diameter and had
multiple enlarged follicles on periphery. Roentgenography of skull base
showed that sellar region was dilated. What is the most probable diagnosis?
A. Stein-Leventhal syndrome
B. Sheehans syndrome
C. Morgagni-Stewart syndrome
D. Premenstrual syndrome
E. Algodismenorrhea

129. A 32-year-old gravida complains of episodes of unconsciousness,


spontaneous syncopes that are quickly over after a change of body position. A
syncope can be accompanied by quickly elapsing bradycardia. There are no
other complications of gestation. What is the most likely reason for such
condition?
A. Postcava compresseion by the gravid uterus
B. Vegetative-vascular dystonia (cardiac type)
C. Psychosomatic disorders
D. Pressure rise in the veins of extremities
E. Pressure fall in the veins of extremities

130. A woman consulted a therapeutist about fatigability, significant weight


loss, weakness, loss of appetite. She has had amenorrhea for 8 months. A year
ago she born a full-term child. Haemorrhage during labour made up 2 l. She
got blood and blood substitute transfusions. What is the most probable
diagnosis?
A. Sheehans syndrome
B. Shereshevsky-Turners syndrome
C. Vegetovascular dystonia
D. Homological blood syndrome
E. Stein-Leventhal syndrome

131. A 54-year-old female patient consulted a gynaecologist about bloody


discharges from the vagina for 1 month. Last menstruation was 5 years ago.
Gynaecological examination revealed no pathological changes. What is the
tactics of choice?
A. Diagnostic fractional curettage of uterine cavity
B. USI
C. Colposcopy
D. Cytosmear
E. Symptomatic therapy

132. A 28-year-old female patient complains of having haemorrhage from


the genital tracts for 1 month. 6 months ago she had natural delivery and gave
birth to a girl weighing 3100 g. Objectively: the uterus is enlarged to 9-10
weeks, mobile, painless, of heterogenous consistency. Examination reveals
vaginal cyanosis, anaemia and body temperature rise up to 37,8°C. There is a
significant increase in hCG concentration in the urine. What is your
provisional diagnosis?
A. Uterine chorionepithelioma
B. Endometritis
C. Uterine fibromyoma
D. Pregnancy
E. Hydatidiform mole

133. An ambulance delivered a 21-year-old woman to the gynaecological


department with complaints of colicky abdominal pain and bloody discharges
from the genital tracts. Bimanual examination revealed that uterus was soft,
enlarged to the size of 6 weeks of gestation, a gestational sac was palpated in
the cervical canal. Uterine appendages werent palpable. Fornices are free,
deep and painless. Discharges from the genital tracts are bloody and profuse.
What is the most likely diagnosis?
A. Abortion in progress
B. Interrupted fallopian pregnancy
C. Threat of abortion
D. Cervical pregnancy
E. Incipient abortion

134. A 33 y.o. woman survived two operations on account of extrauterine


pregnancy, both uterine tubes were removed. She consulted a doctor with a
question about possibility of having a child. What can be advised in this case?
A. Extracorporal fertilization
B. Induction of ovulation
C. Substitutional maternity
D. Insemination with her husbands semen
E. Artifical fertilization with donors semen

135. On the fifth day after a casual sexual contact a 25-year-old female
patient consulted a doctor about purulent discharges from the genital tracts
and itch. Vaginal examination showed that vaginal part of uterine cervix was
hyperemic and edematic. There was an erosive area around the external
orifice of uterus. There were mucopurulent profuse discharges from the
cervical canal, uterine body and appendages exhibited no changes.
Bacterioscopic examination revealed bean-shaped diplococci that became red
after Grams staining. What is the most likely diagnosis?
A. Acute gonorrheal endocervicitis
B. Candidal vulvovaginitis
C. Bacterial vaginism
D. Clamydial endocervicitis
E. Trichomonal colpitis

136. A 26 year old woman complains about edemata, swelling and


painfulness of mammary glands, headache, tearfulness, irritability. These
signs turn up 5 days before menstruation and disappear after its start. What
clinical syndrome is it?
A. Premenstrual syndrome
B. Stein-Leventhal syndrome
C. Adrenogenital syndrome
D. Postcastration syndrome
E. Climacteric syndrome

137. A 49-year-old woman complains about headache, head and neck going
hot, increased perspiration, palpitation, arterial pressure rise up to 170/100
mm Hg, irritability, insomnia, tearfulness, memory impairment, rare and
scarce menses, body weight increase by 5 kg over the last half a year. What is
the most likely diagnosis?
A. Climacteric syndrome
B. Arterial hypertension
C. Postcastration syndrome
D. Premenstrual syndrome
E. Vegetative-vascular dystonia

138. A 30-year-old female patient has been delivered to the gynaecological


department with complaints of acute pain in the lower abdomen and body
temperature 38,8°C. In history: sexual life out of wedlock and two artificial
abortions. Gynaecological examination reveals no changes of uterine. The
appendages are enlarged and painful on both sides. Vaginal discharges are
purulent and profuse. What study is required to confirm a diagnosis?
A. Bacteriological and bacterioscopic analysis
B. Colposcopy
C. Laparoscopy
D. Hysteroscopy
E. Curettage of uterine cavity
139. A parturient woman is 27 year old, it was her second labour, delivery
was at term, normal course. On the 3rd day of postpartum period body
temperature is 36,8°C, Ps - 72/min, AP - 120/80 mm Hg. Mammary glands
are moderately swollen, nipples are clean. Abdomen is soft and painless.
Fundus of uterus is 3 fingers below the umbilicus. Lochia are bloody,
moderate. What is the most probable diagnosis?
A. Physiological course of postpartum period
B. Lactostasis
C. Postpartum metroendometritis
D. Subinvolution of uterus
E. Remnants of placental tissue after labour

140. A parturient woman is 25 years old, it is her second day of postpartum


period. It was her first full-term uncomplicated labour. The lochia should be:
A. Bloody
B. Mucous
C. Sanguino-serous
D. Purulent
E. Serous

141. A 32-year-old patient consulted a doctor about being inable to get


pregnant for 5-6 years. 5 ago the primipregnancy ended in artificial abortion.
After the vaginal examination and USI the patient was diagnosed with
endometrioid cyst of the right ovary. What is the optimal treatment method?
A. Surgical laparoscopy
B. Conservative therapy with estrogen-gestagenic drugs
C. Sanatorium-and-spa treatment
D. Hormonal therapy with androgenic hormones
E. Anti-inflammatory therapy

142. A woman is 34 years old, it is her tenth labor at full term. It is known
from the anamnesis that the labor started 11 hours ago, labor was active,
painful contractions started after discharge of waters and became continuous.
Suddenly the parturient got knife-like pain in the lower abdomen and labor
activity stopped. Examination revealed positive symptoms of peritoneum
irritation, ill-defined uterus outlines. Fetus was easily palpable, movable. Fetal
heartbeats wasnt auscultable. What is the most probable diagnosis?
A. Rupture of uterus
B. II labor period
C. Discoordinated labor activity
D. Uterine inertia
E. Risk of uterus rupture
143. Examination of placenta revealed a defect. An obstetrician performed
manual investigation of uterine cavity, uterine massage. Prophylaxis of
endometritis in the postpartum period should involve following actions:
A. Antibacterial therapy
B. Contracting agents
C. Intrauterine instillation of dioxine
D. Instrumental revision of uterine cavity
E. Haemostatic therapy

144. A 10 week pregnant woman was admitted to a hospital for recurrent


pain in the lower abdomen, bloody discharges from the genital tracts. The
problems turned up after ARVI. The woman was registered for antenatal care.
Speculum examination revealed cyanosis of vaginal mucosa, clean cervix,
open cervical canal discharging blood and blood clots; the lower pole of the
gestational sac was visible. What tactics should be chosen?
A. Curettage of the uterus
B. Expectant management, surveillance
C. Pregnancy maintenance therapy
D. Hysterectomy
E. Antiviral therapy

145. A 30 year old patient complains about inability to become pregnant


over 3 years of married life. The patient is of supernutrition type, she has hair
along the median abdominal line, on the internal thigh surface and in the
peripapillary area. Menses started at the age of 16, they are infrequent and
non-profuse. US revealed that the uterus was of normal size, ovaries were
4?5?5 cm large and had a lot of cystic inclusions. What is the most probable
diagnosis?
A. Polycystic ovaries
B. Ovarian cystoma
C. Menstrual irregularity
D. Bilateral ovarian tumours
E. Chronic oophoritis

146. A 29-year-old patient complains of absent menstruation for a year,


milk discharge from the nipples when pressed, loss of lateral visual fields. X-
ray shows an expansion of the sella turcica. What is the most likely cause of
this condition?
A. Pituitary tumour
B. Pregnancy
C. Functional disorder of the hypothalamic-pituitary-ovarian system
D. Mammary tumour
E. Ovarian tumor

147. A patient with fibromyoma of uterus sized up to 8-9 weeks of


pregnancy consulted a gynaecologist about acute pain in the lower abdomen.
Examination revealed pronounced positive symptoms of peritoneal irritation,
high leukocytosis. Vaginal examination revealed that the uterus was enlarged
corresponding to 9 weeks of pregnancy due to the fibromatous nodes, one of
which was mobile and extremely painful. Appendages were not palpable.
There were moderate mucous discharges. What is the optimal treatment
tactics?
A. Urgent surgery (laparotomy)
B. Fractional diagnostic curettage of the uterine cavity
C. Surveillance and spasmolytic therapy
D. Surgical laparoscopy
E. Surveillance and antibacterial therapy

148. A pregnant woman was delivered to the gynecological unit with


complaints of pain in the lower abdomen and insignificant bloody discharges
from the genital tracts for 3 hours. Last menstruation was 3 months ago.
Vaginal examination showed that body of womb was in the 10th week of
gestation, a fingertip could be inserted into the external orifice of uterus,
bloody discharges were insignificant. USI showed small vesicles in the uterine
cavity. What is the most likely diagnosis?
A. Grape mole
B. Abortion in progress
C. Threat of spontaneous abortion
D. Incomplete abortion
E. Incipient abortion

149. A 49-year-old patient undergoes regular medical check-up for uterine


fibromyoma. Within the last year the uterus has enlarged up to 20 weeks of
gestation. What is the rational way of treatment?
A. Surgical treatment
B. Treatment with prostaglandin inhibitors
C. Further surveillance
D. Hormonal therapy
E. Embolization of uterine arteries

150. A female patient complains of being unable to get pregnant for 5 years.
A complete clinical examination brought the following results: hormonal
function is not impaired, urogenital infection hasnt been found, on
hysterosalpingography both tubes were filled with the contrast medium up to
the isthmic segment, abdominal contrast was not visualized. The patients
husband is healthy. What tactics will be most effective?
A. In-vitro fertilization
B. Laparoscopic tubal plasty
C. ICSI within in-vitro fertilization program
D. Insemination with husbands sperm
E. Hydrotubation

151. A 19-year-old primiparous woman with a body weight of 54,5 kg gave


birth at 38 weeks gestation to a full-term live girl after a normal vaginal
delivery. The girls weight was 2180,0 g, body length - 48 cm. It is known from
history that the woman has been a smoker for 8 years, and kept smoking
during pregnancy. Pregnancy was complicated by moderate vomiting of
pregnancy from 9 to 12 weeks pregnant, edemata of pregnancy from 32 to 38
weeks. What is the most likely cause of low birth weight?
A. Fetoplacental insufficiency
B. Third trimester preeclampsia
C. Womans age
D. Low weight of the woman
E. First trimester preeclampsia

152. A primigravida is 22 years old. She has Rh(-), her husband has Rh(+).
Antibodies to Rh werent found at 32 weeks of pregnancy. Redetermination of
antibodies to Rh didnt reveal them at 35 weeks of pregnancy as well. How
often should the antibodies be determined hereafter?
A. Once a week
B. Once in three weeks
C. Once in two weeks
D. Montly
E. There is no need in further checks

153. A maternity house has admitted a primagravida complaining of


irregular, intense labour pains that have been lasting for 36 hours. The woman
is tired, failed to fall asleep at night. The fetus is in longitudinal lie, with
cephalic presentation. The fetus heartbeat is clear and rhythmic, 145/min.
Vaginal examination revealed that the uterine cervix was up to 3 cm long,
dense, with retroflexion; the external orifice was closed; the discharges were of
mucous nature. What is the most likely diagnosis?
A. Pathological preliminary period
B. Physiological preliminary period
C. Secondary uterine inertia
D. Uterine cervix dystocia
E. Primary uterine inertia
154. A 14-year-old girl complains of pain in vaginal area and lower abdomen
that last for 3-4 days and have been observed for 3 months about the same
time. Each time pain is getting worse. Objectively: mammary glands are
developed, hairiness corresponds to the age. The virginal membrane is intact,
cyanotic and protruded. She has never had menstruation. She has been
diagnosed with primary amenorrhea. What is the reason of amenorrhea?
A. Hymen atresia
B. Sexual development delay
C. Babinski-Frohlich syndrome
D. Turners syndrome
E. Pregnancy

155. A multigravida with Rh-isosensitization was found to have a decrease


in anti-Rh titer from 1:32 to 1:8 at 33-34 weeks of gestation. Ultrasound
revealed double contour of head, ebnlargement of fetal liver, placental
thickness of 50 mm. The patient has indication for:
A. Premature delivery
B. Plasmapheresis
C. Administration of anti-Rh gamma globulin
D. Repeated (after 2 weeks) USI
E. Course of desensitizing therapy

156. A 13-year-old girl was admitted to the gynecological department with


heavy bleeding, which appeared after a long delay of menstruation. Shortly
before, the girl suffered a serious psychotrauma. Her menarche occurred at
the age of 11, she has a 30-day cycle with 5 to 6 days of moderate, painless
bleeding. The patient is somatically healthy, of normosthenic constitution
with height of 160 cm, weight of 42 kg. The patient is pale. Rectoabdominal
examination revealed that the uterus was of normal size and consistency,
anteflexio-versio, the appendages were not changed. What is the most likely
diagnosis?
A. Juvenile bleeding
B. Hysteromyoma
C. Amenorrhea
D. Girl is healthy
E. Ovarian cyst

157. A 38-year-old female patient complains about hot flashes and feeling of
intense heat arising up to 5 times a day, headaches in the occipital region
along with high blood pressure, palpitations, dizziness, fatigue, irritability,
memory impairment. 6 months ago the patient underwent extirpation of the
uterus with its appendages. What is the most likely diagnosis?
A. Post-castration syndrome
B. Premenstrual syndrome
C. Secondary psychogenic amenorrhea
D. Physiological premenopause
E. Early pathological menopause

158. A 27-year-old patient complains of irritability, tearfulness, depression,


and sometimes aggressiveness, headache, nausea, vomiting, swelling of the
mammary glands. The mentioned problems arise 5-6 days before
menstruation and gradually progress until menstruation, 3 days after it the
problems disappear. What is the most likely diagnosis?
A. Premenstrual syndrome
B. Secondary psychogenic amenorrhea
C. Premature pathological climacterium
D. Preclimacterium syndrome
E. Algomenorrhea

159. A 23-year-old primigravida at 39 weeks gestation has been admitted to


the maternity ward with irregular contractions. The intensity of uterine
contractions is not changing, the intervals between them stay long. Bimanual
examination reveals that the cervix is centered, soft, up to 1,5 cm long. There
is no cervical dilatation. What diagnosis should be made?
A. Pregnancy I, 39 weeks, preliminary period
B. Pregnancy I, 39 weeks, labor I, period 1, the active phase
C. Pregnancy I, 39 weeks, pathological preliminary period
D. Pregnancy I, 39 weeks, birth I, 1 period, the acceleration phase
E. Pregnancy I, 39 weeks, labor I, 1 period, the latent phase

160. 20 minutes after a normal delivery at 39 weeks a puerpera had a single


temperature rise up to 38°C. Objectively: the uterus is dense, located between
the navel and the pubis, painless. Lochia are bloody, of small amount. Breasts
are moderately soft and painless. What is the optimal tactics?
A. Further follow-up
B. Appointment antipyretic
C. Expression of breast
D. Manual examination of the uterine cavity
E. Antibiotic therapy

161. On the 10th day postpartum a puerperant woman complains of pain


and heaviness in the left breast. Body temperature is 38,8°C, Ps - 94 bpm. The
left breast is edematic, the supero-external quadrant of skin is hyperemic.
Fluctuation symptom is absent. The nipples discharge drops of milk when
pressed. What is a doctors further tactics?
A. Antibiotic therapy, immobilization and expression of breast milk
B. Physiotherapy
C. Opening of the abscess and drainage of the breast
D. Compress to both breasts
E. Inhibition of lactation

162. On the 10th day postpartum a puerperant woman complains of pain


and heaviness in the left mammary gland. Body temperature is 38,8°C, Ps- 94
bpm. The left mammary gland is edematic, the supero-external quadrant of
skin is hyperemic. Fluctuation symptom is absent. The nipples discharge
drops of milk when pressed. What is a doctors further tactics?
A. Antibiotic therapy, immobilization and expression of breast milk
B. Physiotherapy
C. Opening of the abscess and drainage of the mammary gland
D. Compress to both mammary glands
E. Inhibition of lactation

163. A 30-year-old female patient complains of milk discharge from the


mammary glands, 5-month absence of menstruation. She had one
physiological labour four years ago. Objectively: mammary glands are
normally developed. Bimanual examination reveals that the uterus is
decreased in size, the ovaries are of normal size. MRI-scan shows no cerebral
pathologies. Concentration of thyroid-stimulating hormone is normal. The
serum prolactin level is increased. What is the most likely diagnosis?
A. Hyperprolactinemia
B. Hypothyroidism
C. Pituitary adenoma
D. Sheehan syndrome
E. Polycystic ovary syndrome

164. During self-examination a 22-year-old patient revealed a mammary


tumour. Palpation revealed a firm, painless, mobile formation up to 2 cm,
peripheral lymph nodes were not changed. USI results: in the superior
external quadrant of the right mammary gland there was a big formation of
increased echogenicity, sized 18x17 mm. The patient was provisionally
diagnosed with fibroadenoma. What is a doctors further tactics?
A. Surgical removal of the tumour prior to pregnancy
B. Dynamic follow-up
C. Radical mastectomy
D. Nonsteroid anti-inflammatory drugs, oral contraceptives
E. Surgical treatment after pregnancy
165. A 25-year-old female has a self-detected tumor in the upper outer
quadrant of her right breast. On palpation there is a painless, firm, mobile
lump up to 2 cm in diameter, peripheral lymph nodes are not changed. In the
upper outer quadrant of the right breast ultrasound revealed a massive
neoplasm with increased echogenicity sized 21x18 mm. What is the most likely
diagnosis?
A. Fibroadenoma
B. Diffuse mastopathy
C. Mastitis
D. Mammary cancer
E. Lactocele

166. A 49-year-old female patient complains of itching, burning in the


external genitals, frequent urination. The symptoms have been present for the
last 7 months. The patient has irregular menstruation, once every 3-4 months.
Over the last two years she has had hot flashes, sweating, sleep disturbance.
Examination revealed no pathological changes of the internal reproductive
organs. Complete blood count and urinalysis showed no pathological changes.
Vaginal smear contained 20-25 leukocytes per HPF, mixed flora. What is the
most likely diagnosis?
A. Menopausal syndrome
B. Trichomonas colpitis
C. Cystitis
D. Vulvitis
E. Bacterial vaginosis

167. 2 weeks after labour a parturient woman developed breast pain being
observed for 3 days. Examination revealed body temperature at the rate of
39°C, chills, weakness, hyperaemia, enlargement, pain and deformity of the
mammary gland. On palpation the infiltrate was found to have an area of
softening and fluctuation. What is the most likely diagnosis?
A. Infiltrative-purulent mastitis
B. Phlegmonous mastitis
C. Serous mastitis
D. Mastopathy
E. Lactostasis

168. Preventive examination of a 50-year-old woman revealed a dense


tumour of the right mammary gland up to 5 cm in diameter without distinct
outlines. The skin over the tumour looked like lemon peel. Palpation revealed
a lymph node in the axillary region. What is the most likely diagnosis?
A. Breast cancer
B. Mastitis
C. Breast lipoma
D. Lactocele
E. Diffuse mastopathy

169. A 20-year-old female consulted a gynecologist about not having


menstrual period for 7 months. History abstracts: early childhood infections
and frequent tonsillitis, menarche since 13 years, regular monthly menstrual
cycle of 28 days, painless menstruation lasts 5-6 days. 7 months ago the
patient had an emotional stress. Gynecological examination revealed no
alterations in the uterus. What is the most likely diagnosis?
A. Secondary amenorrhea
B. Spanomenorrhea
C. Cryptomenorrhea
D. Primary amenorrhea
E. Algomenorrhea

170. A 48-year-old female has been admitted to the gynecology department


for pain in the lower right abdomen and low back pain, constipations.
Bimanual examination findings: the uterus is immobile, the size of a 10-week
pregnancy, has uneven surface. Aspirate from the uterine cavity contains
atypical cells. What diagnosis can be made?
A. Hysterocarcinoma
B. Colon cancer
C. Chorionepithelioma
D. Cervical cancer
E. Metrofibroma

Krok 2 – 2014 Hygiene Base


1. A number of viable fetuses per 1000 women at the age between 15 and 44 is
determined by:
A. Genital index
B. Perinatal rate
C. Obstetric rate
D. Reproductive level
E. Birth rate

2. Point out the unit for statistical observation for the determination of blood
sugar level influence on a wound surface healing during postoperative period.
A. The patient in a postoperative period
B. Blood test
C. The patient who was discharged to outpatient treatment
D. The patient who has a wound surface
E. Blood sugar level
3. What methods of the collecting of the information is preferable for study of
housing conditions of students of medical HIGH SCHOOL for a training
period?
A. Questioning
B. Interviewing
C. A method of the directed selection
D. Statistical
E. Selecting of materials

4. What information gathering method is preferable to study housing conditions


of medical students during training period?
A. Questionaire
B. Interviewing
C. Directed selection method
D. Statistical
E. Materials selection

5. Choose a method of a graphic representation of monthly information about


number of the registered cases of acute intestinal infection and their
comparisons to the average monthly values, obtained for 5 previous years:
A. The linear diagram
B. The radial diagram
C. The figured diagram
D. The curvilinear disgram
E. The sector diagram

6. The parameter of infantile mortality for the last year was - 16,3, in present
year - 15,7. Name a kind of the diagram that can be used for a graphic
representation of it:
A. Stylar
B. Intrastylar
C. Linear
D. Sector
E. Radial

7. The average body lenth of newborn boys is 50,9 cm at a sigma 1,66; and
average mass - 3432 at a sigma 5,00. What criterion is necessary in order to
compare degree of variability of these signs?
A. Coefficient of variation
B. Amplitude
C. Coefficient of association
D. Sigma
E. Limit

8. What method is applied to establish rate of correlation between age of men


and their mortality due to myocardial infarction?
A. Method of grade correlation (Spirman)
B. Method of graduated correlation (Armler)
C. The quadrate method (Pirson)
D. Correlation ratio
E. The Indirect method (Student)

9. Indicate the registration medical document for the patient, who 21.02. was
addressed to the doctor with diagnosis ARVD for the first time in this year:
A. The statistical coupon is to be filled in and it is necessary to deliver
on a sign (+)
B. The statistical coupon for registration of final diagnosis is not necessary
C. It is necessary to fill in the emergency notice on a case of a contagion
D. The necessary registration form is not indicated
E. The statistical coupon is to be filled in, but a sign (+) is not necessary to be put
in

10. Define the basic registration document at the profound study of a case rate
with temporary lost labor ability at the industrial enterprise:
A. A card of the personal account of a case rate
B. "The Report on reasons of a temporary lost labor ability"
C. A ambulatory medical card
D. The inpatient medical record
E. The sick-leave certificate

11. Head of a department and a trade-union group have appealed to the head of a
hospital about dismissal of the senior nurse who has 17-year record of service.
The facts of charge were confirmed and recognized by the nurse herself. The
nurse lives with a daughter (who is divorced and unemployed) and a 9-month-
old grandson. Make an administrative decision:
A. To continue the worker in office with a warning of dismissal in case
of repeated violation of labor discipline
B. To issue the sick leave
C. To discharge the worker, i.e. to satisfy demands of the collective
D. To embark other officials or public organizations with this problem
12. Head of a department and a trade-union group have appealed to the head of a
hospital about dismissal of the senior nurse who has 17 year record of service.
The facts of charge were confirmed and recognized by the nurse herself. This
nurse lives with a daughter (who is divorced and unemployed) and a 9-month-
old grandson. Make an administrative decision:
A. To continue the worker in office with a warning of dismissal in case
of repeated violation of labor discipline
B. To shift the solution of this problem on other officials or public organizations
C. To discharge the worker, i.e. to satisfy demands of the collective
D. To issue the sick list

13. A child is 6 years old. Within one year of observation he had URI that lasted 8
days. Physical state is satisfactory. Specify hi health group:
A. I
B. III (a)
C. II
D. III (b)
E. III (c)

14. A 38 year old man was admitted to a hospital from his working place on July
19 because of hip fracture. He was invalid till November 19. Requires
prolongation of treatment. Who decides on the issue of further temporary
invalidity?
A. Specialized (traumatologic) MSEC
B. The head physician of a polyclinic
C. Regional MSEC
D. Interregional general MSEC
E. DCC

15. What is the maximum duration of medical certificate in case of tuberculosis?


A. 2 months
B. Week
C. Month
D. 10 months
E. 2 weeks

16. An engineer-chemist at the age of 47 often fells ill with an occupational skin
disease. Who makes a decision to transfer him to other job accepts?
A. DCC
B. The attending physician
C. A head physician
D. The chief of shop
E. MSEC
17. A patient with high temperature came to a first-aid post in the evening. The
fact of temporary disability was established. Indicate the order of examination
in this case:
A. The night duty doctor should issue a medical certificate, which will
be subsequently used for issuing a sick list from the date of the
previous day
B. The sick list for 3 days should be issued
C. Any document shouldnt be issued
D. The sick list for 1 day should be issued
E. The sick list for up to 3 days should be issued

18. A 5 tons milk batch was sampled. The lab analysis revealed: fat content 2%,
specific density - 1,04 g/cm3, acidity - 210?, reductase probe - weak-positive.
What way is the product to be used in?
A. Sell but inform customers about milk quality
B. Sell without limitations
C. Do the product away
D. Discard for animal feeding
E. Technical utilization

19. A sample of milk was taken for testing from a 5 ton milk batch. Lab analysis
showed the following: fat content 2%, specific density- 1,04 g/cm3, acidity
21°C, reductase probe – weak positive. What way the product is to be used in?
What would you advise?
A. Sell but inform customers about milk quality
B. Utilize technically
C. Write the product off for animal feeding
D. Sell without limitations
E. Annihilate the product

20. The student has the following devices: Geiger counter, Ebert counter, Krotovs
apparatus, Mischuk device, Ebert device. What device can he use to assess air
germ pollution?
A. Krotovs apparatus
B. Eberts counter
C. Mischuks device
D. Eberts device
E. Geigers counter

21. Thyreotoxicosis patient is in the two-place hospital ward of therapeutic


department. The area of the ward is 18 m2, height is 3 m, ventilation rate is
2,5/h. Air temperature is 20°C, relative humidity is 45%, air movement
velocity is 0,3 m/sec, light coefficient is 1/5, noise level constitutes 30 dB.
Make a hygienic assessment of these conditions.
A. Discomfortable microclimate
B. All conditions are OK
C. Poor lighting
D. Non-effective ventilation
E. High level of noise

22. A 9 y.o. girl has an average height and harmonic growth development. She was
ill with acute respiratory infection for five times. Define the group of her
health.
A. 2nd group
B. 3rd group
C. 1st group
D. 4th group
E. 5th group

23. A student lives in the modern house in the flat with a complete set of sanitary
equipment (WC, bath, shower, local water heater). How much water
consumption has he got?
A. 160-200 L/day
B. 50-100 L/day
C. 500 -600 L/day
D. 300-400 L/day
E. 10-15 L/day

24. A 30-year-old patient with complaints of occipital headache, disturbed sleep


with nightmares came to a policlinic. BP was 150/95 mm Hg. He was
diagnosed with hypertensic crisis. The patient should be registered in the
following dispensary group for arterial hypertension surveillance:
A. In the second
B. In the third
C. In the fifth
D. In the first
E. In the fourth

25. A young patient who came to a policlinic was diagnosed with the 1 stage of
hypertension. How often should he undergo the medical check-up?
A. Twice a year
B. 3 times a year
C. Once a year
D. 4 times a year
E. 5 times a year

26. 25 children at the age of 2-3 years who dont attend any child welfare
institutions should be observed by a district pediatrician within the current
year. How many initial visits of this group of children should be planned?
A. 50
B. 40
C. 200
D. 100
E. 20

27. Studying of pulmonary tuberculosis incidence provided data about patients


socioeconomic living conditions and bad habits. What method allows to
estimate the impact of these factors on tuberculosis incidence?
A. Calculation of correlation coefficient
B. Standardized index calculation
C. Calculation of reliability coefficient
D. Calculation of correspondence index
E. Calculation of regression coefficient

28. What juice should be included in a complex drug and dietary therapy for
patients with gastric ulcer or duodenal ulcer and increased gastric juice acidity
in order to accelerate the ulcer healing?
A. Potato, potato and carrot
B. Apple, birch and apple
C. Cabbage, cabbage and carrot
D. Celery, parsley
E. Pumpkin

29. A teacher of a secondary school was diagnosed with pulmonary tuberculosis.


What is the maximum duration of his medical certificate?
A. Ten months
B. Two months
C. A month
D. Five months
E. Four months

30. An employee of a private company was ill with acute respiratory viral
infection. Consulted a district doctor, who determined the fact of temporary
loss of working ability, but refused to issue a sick-list, arguing that the patient
worked in the private and not state-owned company. Should the sick-list be
issued to the employees of private companies?
A. Issued regardless of companys ownership
B. Issued only on condition of payment guarantee by the companys proprietor
C. Issued only to empties of state-owned companies
D. Issued a medical certificate of a set form
E. Issued a medical certificate of a free form

31. Clinic of a research instutute for occupational diseases examined a worker


who works at a concentration plant and diagnosed him with chronic dust
bronchitis. The case is investigated by a commission including the
representatives of: the plant, clinic, territorial SES, department of Social
Insurance Fund, trade union. According to the "regulation on investigation
of…", the commission should be headed by the representative of the following
authority:
A. Territorial SES
B. Social Insurance Fund
C. Clinic
D. Trade union
E. Plant

32. Basing upon the data of laboratory assessment of sanitary state of soil in a
certain territory, the soil was found to be low-contaminated according to the
sanitary indicative value; contaminated according to the coli titer; low-
contaminated according to the anaerobe titer (Cl. Perfringens). This is
indicative of:
A. Fresh fecal contamination
B. Old fecal contamination
C. Insufficient intensity of soil humification
D. Constant entry of organic protein contaminations
E. Insufficient insolation and aeration of soil

33. A maternity hospital registered 616 live births, 1 stillbirth, 1 death on the 5th
day of life over a 1 year period. What index allows the most precise estimation
of this situation?
A. Perinatal mortality
B. Crude mortality rate
C. Neonatal mortality
D. Natural increase
E. Natality

34. In one of the surgical departments the quality assurance testing of sterilization
of surgical instruments was performed. After an instrument had been treated
with 1% phenolphthalein, the solution turned pink. This indicates that the
instrument has:
A. Synthetic detergent residues
B. Drugs residues
C. Residual blood
D. Residual tissue
E. Disinfectant residues

35. A patient complained about problems with pain and tactile sensitivity, pain in
the nail bones at the end of the working day. He works at a plant with
mechanical devices. What pathology can be suspected?
A. Vibration disease
B. Noise disease
C. Hypovitaminosis of B1
D. Overwork symptoms
E. Caisson disease

36. Bacterial analysis of air in a living space in winter period by means of Krotovs
apparatus revealed that total number of microorganisms in 1 m3 of air was
7200. What is the permissible number of microorganisms for the air to be
characterized as "pure"?
A. Up to 4500
B. Up to 3500
C. Up to 2500
D. Up to 5500
E. Up to 7500

37. Maximum permissible concentration of carbon dioxide in the air is considered


to be a sanitary index of air purity in a classroom. What concentration of
carbon dioxide in the air is accepted as maximum permissible?
A. 0,1%
B. 0,15%
C. 0,05%
D. 0,2%
E. 0,3%

38. Study of actual diet of an adult revealed the following: proteins make up 16%
of energy value of daily ration, fats - 25%, carbohydrates - 59%. Evaluate
compliance of protein, fat and carbohydrate share in the energy value of daily
ration with the recommended shares of these nutrients?
A. Carbohydrate share is insufficient, there is excess of proteins
B. Carbohydrate share is excessive
C. Nutrient content complies with the recommended shares of energy value
D. Fat share is insufficient
E. Carbohydrate share is insufficicent
39. A patient who has been consuming refined foodstuffs for a long time
complains about headache, fatiguability, depression, insomnia, irritability.
Objectively: muscle asthenia, pain and cramps in the gastrocnemius muscles,
during walking the patient lands onto his heel first, then on the external edge
of foot. Cardiovascular system exhibits tachycardia, hypoxia, dystrophic
changes of myocardium. There are also gastrointestinal disorders. What is the
most likely diagnosis?
A. Hypovitaminosis B1
B. Hypovitaminosis B6
C. Hypovitaminosis B15
D. Hypovitaminosis B2
E. Hypovitaminosis B12

40. A patient who had eaten mushrooms in the morning was delivered to the
infectious diseases hospital at night. The disease development was rapid. The
patient presented with stomach pain, frequent diarrhea, intractable vomiting,
burning thirst, headache and dizziness. He died on the third day. What
mushrooms are most likely to have caused mycetismus?
A. Deadly amanita
B. Russules
C. Fly agarics
D. Morels
E. Sulfur-tufts

41. Periodical survey of a worker of a chemicals plant revealed a malignant


neoplasm on the urinary bladder. This occupational disease was the most
probably caused by contact with the following industrial poison:
A. Benzidine
B. Vinyl chloride
C. Asbestos
D. Arsenic
E. Nickel carbonyl

42. A 37 year old patient applied to a local therapeutist. As a result of exacerbation


of chronic obstructive bronchitis the patient had been temporarily disabled for
117 days within 1 year. What tactics will be legally correct?
A. The patient should be referred to the medicosocial expertise
B. The patient should be referred to the medical consultation comission for
extension of medical certificate
C. The patient shoul be referred to the sanatorium-and-spa treatment
D. The therapeutist should issue a new medical certificate
E. The therapeutist should extend a medical certificate
43. A 46-year-old patient once took part in elimination of breakdown at an atomic
power plant. Currently he is being treated at an in-patient hospital. He was
diagnosed with progressing vegetative insufficiency. This disease relates to the
following group of ionizing radiation effects:
A. Somato-stochastic
B. Heterosis
C. Genetic
D. Somatic
E. Hormesis

44. In treatment and prevention establishments, regardless of their organisational


and proprietary form, the rights of patients should be observed. Which of
these rights is the most significant?
A. The right to the protection of the patients interests
B. The right to the free choice
C. The right to be heard
D. The right to the protection from incompetence
E. The right to the information

45. A military unit stopped for 3-days rest in an inhabited locality after a long
march. The sanitary-epidemiological reconnaissance found several water
sources. It is necessary to choose the source complying with the hygienic
standards for drinking water in the field conditions:
A. Artesian well water
B. River water
C. Melt snow water
D. Rain water
E. Spring water

46. A district doctor was commisioned with a task to work out a plan of
treatment-and-prophylaxis actions for the population of his district. What
actions of secendary prophylaxis must he include into this plan?
A. Prevention of disease complications
B. Improvement of populations living conditions
C. Rehabilitation actions
D. Disease prevention
E. Elimination of disease causes

47. The district pediatrician is charged with the analysis of infant mortality. What
is taken for the unit of observation in infant mortality investigation?
A. A baby dead at the age up to 12 months
B. A baby dead at birth
C. A baby dead at the age over 28 days
D. A baby dead at the age up to 1 months
E. A baby dead at the age up to 6 days

48. Chief district pediatrician has to carry out analysis of infant mortality rate.
What should he take as a unit of the observation?
A. Child death case at the age up to 1 year
B. Child death case after 28 days of life
C. Child death case at the age up to the first month
D. Child death case during first 7 days of life
E. Child death case on labor

49. Deputy of chief medical officer carried out a study of morbidity rate for
population which had been served at the polyclinics within the last 5 years.
What statistical values can help in calculation of morbidity rates?
A. Relative values
B. Dynamic series
C. Average values
D. Standard values
E. Absolute values

50. There were registered 500 cases of urolithiasis per 10000 inhabitants. What
kind of statictical indices is presented?
A. Prevalence rate
B. Index of visualization
C. Index of compliance
D. Incidence rate
E. Correlation coefficient

51. At year-end hospital administration has obtained the following data: annual
number of treated patients and average annual number of beds used for
patients treatment. What index of hospital work can be calculated on the base
of this data?
A. Bed turnover
B. Average bed idle time
C. Average annual bed occupancy
D. Bed resources of the hospital
E. Average duration of patients presence in the hospital
52. Examination of a 13-year-old boy reveals that his body length is 147 cm (+2),
body weight - 38 kg (+1,5), circumference of chest - 72 cm (+0,2). Estimate
the harmonicity of the childs physical development:
A. Disharmonious
B. Sharply disharmonious
C. Supernormal
D. Harmonious
E. Above the average

53. A childrens health camp received a party of tinned food. External examination
of the tins revealed that they had deep dents, could be easily concaved when
pressed and wouldnt immediately return to the initial state; rust was absent;
the tins were greased with inedible fat. Specify the bloat type:
A. Physical
B. Biological
C. Physicochemical
D. Combined
E. Chemical

54. On physiologic-sanitary examination of railway department work it was


revealed that loaders work is of III degree of difficulty. They unload vagons
with sand, manually break coagulated mass by shovel and shift it. What
criteria was used to evaluate work of loaders?
A. Maximun load weigh which is shifted
B. Value of static loading for the shift
C. Time of passive observation, % to the shift duration
D. Intellectual efforts
E. Time of active activities, % to the shift duration

55. Workers of fishery are subjected to low temperatures of the air (from 5 till
15°C). Diseases of what organs and systems are the most frequent among
workers of such enterprises?
A. Respiratory system
B. Cardiovascular system
C. Liver
D. Gastrointestinal tract
E. Blood

56. During the periodic medical examination an assembly fitter (works on


soldering details) didnt report any health problems. Closer examination
revealed signs of asthenic-vegetative syndrome. Blood included red blood cells
with basophilic aggregations and a somewhat higher number of reticulocytes,
urine had a high concentration of delta-aminolevulinic acid. The complex of
symptoms indicates the initial stage of chronic intoxication with:
A. Lead
B. Ethanol
C. Mercury
D. Manganese
E. Tin

57. Some of the population of a city district have uneven teeth color. The
individuals have white spots, transverse brown stripes on the incisors.
Occurrence of these symptoms is associated with the quality of drinking water
from a deep well. Which of the following components of water can be the
cause of the disease?
A. F
B. Mg
C. Ca
D. J
E. Fe

58. An outpatient hospital made record of 11600 diseases within one year. Among
them influenza and ARD make up 5800, circulatory system diseases - 3480,
digestion diseases - 1300, other diseases - 1020. What relative index can be
calculated according this data?
A. Extensive
B. Intensive
C. Correlation
D. Visualization

59. Prevalence of a disease in region N amounted 1156 occurences per 1000 of


inhabitants. What of the mentioned indices characterizes the disease
prevalence?
A. Intensive
B. Standardized
C. Ratio
D. Extensive
E. Visual index

60. A pupil of the 8th form after trauma has acute atrophy of the left arm muscles,
tonus of which is distinctly decreased, active movements are only in the left
joint, pupils foot is deformed. Function of support of the left leg is absent,
support function of the right leg is preserved. The boy wears an orthopedic
footwear. What group of physical training does the boy belong to?
A. Special
B. Preparatory
C. Additional
D. Basic
E. Other

61. District physician was charged with plan drafting concerning medical and
preventive measures among the population in the area he is assigned to. What
measures must he include in this plan as regards primary prevention of
illness?
A. Prevention of disease onset
B. Measures to increase patients life quality
C. Referral of patients to sanatorium
D. Measures to improve patients life conditions
E. Prevention of disease complications

62. A therapeutist needs to analyze adult health in the area of service. Which
groups of indicators will be included into this analysis?
A. Demographic, sickness rates, disability
B. Birth rates, sickness rates, disability
C. Sickness rates, death rates, physical development
D. Demographic, sickness rates, physical development
E. Sickness rates, disability, death rates

63. A district pediarician has carried out infant mortality rate analysis in his area.
What data has been used?
A. Mortality of children under 1 y.o. structured by age, sex, causes
B. Mortality of children under 1 y.o., natimortality
C. Mortality of district adolescents
D. Mortality of district newborn
E. Hospital mortality of children, structured by age

64. At the radiological unit of a hospital gamma-devices of radiotherapy of "Agat"


type and other closed sources of ionizing radiation are used for treatment of
malignant neoplasms. What measures are to be taken to protect personnel
during working with radioactive sources of such type?
A. Reduction of working time and screening of the source
B. Capsulation of devices and organization of room ventilation
C. The increase of distance to the source and individual hygiene compliance
D. Systematical cleansing of surfaces from the radioactive contamination and
shortening of working time
E. Screening of the source and the use of means of individual protection of
respiration organs
65. A patient undergoes inpatient treatment with the diagnosis of acute
pancreatitis. To spare pancreas as much as possible the doctor prescribed for
him starvation for 1-3 days. What products is the patient allowed to eat during
recovery period after cancelling of starvation?
A. Potato and carrot mash
B. Boiled meat
C. Grape juice
D. Milk
E. Broth

66. A local doctor has to prepare a report about the health condition of the
population of his region. What medical indexes of population health condition
should he use?
A. Morbidity, disabilities, demographic, physical development
B. Way of life, genetic, pollution
C. Average longevity
D. Average treatment duration, complications
E. Social welfare, satisfaction of life quality

67. To study physical development of children and adolescents, anthropometric


studies are widely used. Choose a physiometric method of study from the
below given.
A. Determination of vital capacity of lungs
B. Determination of vertebra form
C. Determination of body weight
D. Measurement of growth
E. Determination of thorax form

68. Study of morbidity with temporary disability among workers of a machine


building plant revealed that average duration of a case was 20 days. What
diseases influenced upon the index value?
A. Chronic
B. Acute
C. Preexisting diseases
D. Hard to determine
E. Subacute

69. A patient consulted a doctor about acure respiratory viral infection. The
patient was acknowledged to be off work. The doctor issued him a medical
certificate for 5 days. The patient is not recovering. What measures should the
doctor take in order to legalize the further disability of patient?
A. To prolong the medical certificate at his own discretion but no
more than for 10 days in total
B. To send the patient to the medical consultative commission
C. To send the patient to the medical social expert comission
D. To prolong the medical certificate at his own discretion but no more than for 6
days in total
E. To prolong the medical certificate together with department superintendent

70. A patient is on the sick leave for 4 months continuously from the date of
injury. The treatment is going to last for 1-2 months more. Who has the right
to extend the duration of medical certificate for this patient?
A. Medical advisory commission after medico-social expert
commission examination
B. District doctor by agreement with a department chief
C. Medico-social expert commission
D. Medical superintendent
E. Medical advisory commission after inpatient treatment

71. Working conditions of a building company worker are characterized by


cooling microclimate effect, silica-containing dust, caustic alkali (quicklime)
and noise. What medical expert should be the chief of the commission that
periodically examines the workers of the mentioned category?
A. Therapeutist
B. Ophthalmologist
C. Dermatologist
D. Otolaryngologist
E. Neurologist

72. It is determined that 30 of a 100 women with risk factor had preterm labor,
and of a 100 women without risk factor 5 women had preterm labor. What
method of statistic data processing should the doctor use in order to estimate
reliability of differences between the compared groups?
A. Students criterion calculation
B. Standardization method
C. Correlation analysis
D. Average computing
E. Relative numbers calculation

73. 350 workers of a metalurgical plant had to undergo a yearly preventive


examination. A territorial polyclinic carried out preventive examination of 325
workers. As a result of it, 1 worker was recognized as temporarily disabled, 15
workers underwent further rehabilitation at an after-work sanatorium, 10
workers were provided with diet meal. What index characterizing the
preventive work of the polyclinic should be applied in this case?
A. Coverage of preventive medical examinations
B. Frequency of case detection during examinations
C. Percentage of people who were provided with diet meal
D. Percentage of temporarily disabled people
E. Percentage of people who underwent rehabilitation at an after-work
sanatorium

74. District doctor of rural medical department was called to a 42-year-old


patient. While examining the patient, the doctor suspected the dysentery.
What document must the doctor issue?
A. Urgent notification of infectious disease
B. Infectious disease report
C. Statistical coupon of final (precise) diagnosis
D. Abstract of outpatient medical card
E. Report addressed to Head of the village

75. During coal extraction in a mine the concentration of coal dust in the working
area is 450 mg/m3 (MPC is 10 mg/m3). What occupational respiratory
disease may develop in miners?
A. Anthracosis
B. Talcosis
C. Byssinosis
D. Allergic nasopharyngitis
E. Siderosis

76. A district doctor keeps the record of reconvalescents after infectious diseases,
people who are disposed to frequent and long-lasting diseases, patients with
chronic pathologies. What category of patients should belong to the III health
group?
A. People with chronic diseases
B. Reconvalescents after infectious diseases and patients with chronic
pathologies
C. All above mentioned categories
D. People disposed to frequent and long-lasting diseases
E. People with chronic pathologies and disposed to frequent and long-lasting
diseases

77. Over a current year among workers of an institution 10% havent been ill a
single time, 30% have been ill once, 15% - twice, 5% - 4 times, the rest - 5 and
more times. What is the percentage of workers relating to the I health group?
A. 55%
B. 40%
C. 10%
D. 60%
E. 22%

78. During a regular medical examination at a metallurgical plant 20% of workers


were found overweight (body weight was 5-14% higher than normal), and had
early signs of obesity (grade I-II) with Quetelet index from 26 to 30. What
products share must be reduced in the diet of this group of people in the first
place in order to normalize their body weight?
A. Bakery products
B. Vegetables
C. Milk and dairy products
D. Meat and fish products
E. Fruit

79. Chief physician of a polyclinic encharged a district doctor with a task to


determine the pathological prevalence of disease N in his district. What
document allows to estimate the disease prevalence in the population of a
medical district?
A. Prophylactic examinations register
B. Statistic coupons (+)
C. Statistic coupons (+) and (-)
D. Vouchers for medical appointments
E. Statistic coupons (-)

80.A 48-year-old male in-patient undergoes treatment for essential hypertension


of II-B stage. It is known from history that he works in a design engineering
office. His job involves neuro-emotional stress. Which of these foodstuffs do
not stimulate the central nervous system and can be recommended for the
patient?
A. Whole milk
B. Carbonated beverages
C. Mushroom broths
D. Meat broths
E. Vegetable broths

81. In a forest summer camp children have variable procedures to harden their
organisms. What procedure has the most hardening power?
A. Contrast shower
B. Bath with hydromassage
C. Hygienic shower
D. Morning exercises on the fresh air
E. Walking on the fresh air

82. There is a dynamic growth of number of congenital abnormalities such as


central paralysis, newborns blindness, idiocy among the population that lives
near to pesticides production enterprise. Compounds of which pollutant can
cause the development of this pathology?
A. Mercury
B. Cadmium
C. Strontium
D. Iron
E. Chrome

83. Ats planned to construct multifield a new hospital in one of the cental city
districts. What building type is the most appropriate in this case?
A. Centralized and blocked
B. Decentralized
C. Centralized
D. Mixed
E. Blocked

84. In an urban settlement situated on the riverbank, an outbreak of hepatitis A


was registered. The disease might have water origin. This assumption can be
confirmed by growth of the following values of water quality:
A. Number of coli-phages
B. Oxidability
C. Index of fecal coli-forms
D. Presence of benign leptospirosis pathogen
E. Escherichia coli index

85. During inspection of sanitary conditions of studying at a technical university it


was necessary to evaluate the visual regimen of students, who study from 9
a.m to 3 p.m. What index of natural light will be the most informative?
A. Natural light coefficient
B. Presence of mixed (superolateral) light
C. Depth of study room
D. Light coefficient
E. Time of the room insolation

86. 200 patients suffering from essential hypertension were examined in order to
obtain data about patients arterial pressure and age. What statistic value
should be applied in order to measure relation between these characteristics?
A. Correlation coefficient
B. Representation error
C. Sygmal deviation
D. Students coefficient
E. Coefficient of variation

87. A doctor of the general practice has registered the following death causes for
the previous year: the first place was taken by cardiovascular diseases (60%),
the second - by tumors (18%), then - traumas (8,3%) etc. What diagrams will
provide the most substantial information about the registered ocurrences?
A. Pie diagram
B. Cartogram
C. Circle diagram
D. Column diagram
E. Line diagram

88. On medical observation a doctor identified girl (162 cm tall and 59 kg weight)
who complained loss of ability to see surrounding objects clearly in the
evening. On examination: dry skin, hyperkeratosis. Her daily ration includes
the following vitamines: vitamine A– 0,5 mg, vit.B1– 2,0 mg, vit.B2– 2,5 mg,
vit.B6– 2 mg, vit.C?– 70 mg. What is the hypovitaminosis type?
A. A-hypovitaminosis
B. B2-hypovitaminosis
C. C-hypovitaminosis
D. B6-hypovitaminosis
E. B1-hypovitaminosis

89. As a result of prophylactic medical examination a 35 year old woman was


diagnosed with alimentary and constitutive obesity of the III degree. It is
known from her anamnesis that the patient doesnt observe rules of rational
nutrition: she often overeats, the last food intake is usually 10-15 minutes
before going to bed, prefers fattening and rich in carbohydrates food. What is
the main alimentary risk factor of obesity development?
A. Energetic unprofitableness of nutrition
B. Excess of carbohydrates
C. Lack of cellulose
D. Violation of dietary pattern
E. Excess of fats

90. The air of a foundry workers working zone contains condensation aerosol with
dust particles sized 2 nm (90%), 2-5 nm (2%), over 5 nm (6%), below 2 nm
(about 2%). Characterize the dust dispersivity:
A. Fine-dispersed
B. Ultrafine-dispersed
C. Mist
D. Median-dispersed
E. Coarsely dispersed

91. The major repair of a hospital included renewal of colour design of hospital
premises because it is of great psychological and aesthetical importance; and
so the walls of patient wards will be painted under consideration of:
A. Windows orientation
B. Diseases of patients who will be staying in these wards
C. Hospital profile
D. Wall reflection coefficient
E. Creation of cozy atmosphere

92. Atmospheric air of an industrial centre is polluted with the following wastes of
metallurgical plants: sulphuric, nitric, metal, carbon oxides that have negative
influence upon the inhabitants health. The effect of these hazards can be
characterized as:
A. Combined
B. Adjacent
C. Mixed
D. Complex
E. Associated

93. Study of morbidity rate in a city N revealed that population of different


administrative districts differed in age structure. What statistic method allows
to eliminate influence of this factor upon morbidity indices?
A. Standardization
B. Correlative regressive analysis
C. Wilcoxons t-criterion
D. Analysis of dynamic series
E. Calculation of average values

94. A driver had been fixing a car in a closed garage and afterwards complained
about headache, dizziness, nausea, muscle asthenia, sleepiness. Objectively:
pulse and respiratory rate elevation, excitement, hypertension, delirium of
persecution. What is the most likely diagnosis?
A. Intoxication with carbon oxide
B. Posttraumatic encephalopathy
C. Intoxication with ethyl gasoline
D. Hypertensive crisis
E. Asthenovegetative syndrome
95. An outbreak of food poisoning was recorded in an urban settlement. The
illness was diagnosed as botulism on the grounds of clinical presentations.
What foodstuffs should be chosen for analysis in the first place in order to
confirm the diagnosis?
A. Tinned food
B. Potatoes
C. Boiled meat
D. Cabbage
E. Pasteurized milk

96. A worker who undergoes regular medical check-up for duodenal ulcer
received a subsidized 24-day sanatorium voucher from his plant. The term of
annual leave of a worker is 24 calender days, it will take 4 days more to get to
the sanatorium and back home. What is the procedure of obtaining a 4-day
sick-leave?
A. Medical Expert Commission issues a 4-day sick list
B. The doctor in charge issues a health certificate and sanatorium patients file for
28 days
C. Medical Expert Commission issues a 4-day health certificate
D. Medical Expert Commission issues a 28-day sick list
E. The doctor in charge issues a 4-day sick list

97. A 42 year old metalworker has been working at the turning machine for
production of heavy large-size parts for 5 years. His work requires using of
hand and pedal levers that involves considerable physical force. What means
for osteoarthrosis prevention should be recommended?
A. To limit physical work
B. To administer protein-and-carbohydrate diet
C. To improve health at the Black sea coast
D. To go in for weightlifting
E. To administer protein-and-vitamin diet

98. Administration of a plant producing red lead paint intends to form a group of
medical specialists for periodical medical examinations. What specialist must
be obligatory included into this group?
A. Neuropathologist
B. Psychiatrist
C. Otolaryngologyst
D. Dermatologist
E. Gynaecologist

99. The total area of a ward at the therapeutical department is 28 m2. What is the
maximum number of beds that can be exploited in this ward?
A. 4
B. 1
C. 3
D. 5
E. 2

100. The amount of ultraviolet radiation dose was measured in minutes.


What device was applied for measurement of the biodose?
A. Gorbachevs biodosimeter
B. Actinometer
C. Catathermometer
D. Radiometer
E. UV-meter

101. The institutions which take part in medical examinations can be


prevention and treatment facilities, medical board of Ministry of Defense,
medical board of Ministry of Home Affairs, medico-social expert
commissions, forensic medical boards etc. What institutions are responsible
for temporary disability examination?
A. Prevention and treatment facilities
B. Medico-social expert commissions
C. Sanitary-and-prophylactic institutions
D. Medical boards of Ministry of Defense
E. Medical boards of Ministry of Home Affairs

102. The institutions which take part in medical examinations can be


prevention and treatment facilities, medical board of Ministry of Defense,
medical board of Ministry of Home Affairs, medico-social expert
commissions, forensic medical boards etc. What institutions are responsible
for temporary disability examination?
A. Prevention and treatment facilities
B. Medico-social expert commissions
C. Medical boards of the Ministry of Home Affairs
D. Medical boards of the Ministry of Defense
E. Sanitary-and-prophylactic institutions

103. A mother who is on partially paid maternity leave got sick and was
hospitalized. What document is to be issued to a working father who will be
taking care of a child during his mothers illness?
A. Sick-leave
B. Certificate of mothers illness
C. Extract from the medical card of out- or in-patient
D. Free-form certificate
E. Certificate of childs care necessity

104. Clinical and statistical study was devoted to the effect of a new
pharmacological medication upon the patients with coronary heart disease.
What parametric criterion (coefficient) can be used for estimation of results
validity?
A. Students coefficient (t)
B. Conformity coefficient
C. Sign criterion
D. Wilcoxons t-criterion
E. Kolmogorov-Smirnovs criterion

105. It is planned to make complete isolation boxes in the infectious


department in order to prevent nosocomial airborne infections. The boxes
consist of a tambour, a ward and a lock chamber. What structure should be
also included in a complete isolation box?
A. Bathroom unit
B. Doctors consulting room
C. Manipulation room
D. Patients examination room
E. Nursing room

106. Educational rooms are illuminated with various lighting fittings. What
type of lighting fittings is the most appropriate in respect of hygienic norms?
A. Indirect light fittings
B. Ambient light fittings
C. Combined light fittings
D. Direct light fittings
E. Semi-reflected light fittings

107. In terms of megacalorie (1000 kcal = 4184 kJ) the ration of an adult
includes 30 g of proteins, 37 g of fats, 137 g of carbohydrates, 15 mg of vitamin
C, 0,6 mg of thiamine (vitamin B1). The ration is UNBALANCED as to the
contents of:
A. Vitamin C
B. Carbohydrates
C. Thiamine
D. Proteins
E. Fats

108. A city somatic hospital with 300 beds consists of the main building
which houses the therapeutic and surgical departments. Several separate
buildings house the maternity, pediatric and radiologic departments that are
connected to the main building by underground walkways and above-ground
covered skybridges. Specify the building system of the hospital:
A. Central-unit
B. Combined
C. Decentralized
D. Centralized
E. Free

109. A heat station working on solid fuel is located in a residential district.


On cloudy foggy days in december there was an increase in diseases with
upper airway affection and signs of general intoxication. There were also
mortal cases among the elderly people. What is the most likely factor that
provoked toxic effect?
A. Suspended materials
B. Calm
C. High air humidity
D. Low air temperature
E. Temperature gradient

110. Indices that characterize population health include demographic


indices. What environment is used for calculation of these indices?
A. Population number
B. Number of hospitalized people
C. Number of population being liable to preventive examination
D. Number of patients
E. Employment number

111. While making sanitary examination of burn unit for adults it was stated
that wards for 4 persons are of 28 m2 square. What should be the minimum
ward area in this unit?
A. 40 m2
B. 28 m2
C. 52 m2
D. 30 m2
E. 24 m2

112. The results of 5 year monitoring allowed to estimate the level of


environmental influence upon health indices of popultaion. What statistic
method should be chosen?
A. Calculation of correlation coefficient
B. Calculation of coefficient of difference validity
C. Calculation of dynamic indices
D. Calculation of regression coefficient
E. Calculation of conformity coefficient

113. In order to improve organism tolerance of boarding-school pupils a


doctor developed a program. The program is based upon the following
principles: graduality, consistency, individuality, coomplexity. What of the
main principles of organism tempering wasnt taken into account?
A. Systematicness
B. Increase of resistance
C. Increase of influence force
D. Autodefense increase
E. Increase of influence intensity

114. A student analyzes noise level of cold-pressing process. What device


should be applied for this hygienic study?
A. Noise and vibration analyzer
B. Actinometer
C. Pyranometer
D. Noise analyzer
E. Sound tester

115. Environmental pollution is prevented by mechanical separation of


nontoxic solid domestic waste. Specify the method which can be used for
mechanical utilization of these wastes:
A. Compressing of wastes into building blocks
B. Burning as power-plant fuel
C. Waste neutralization in biothermal boxes
D. Burial of wastes
E. Hydrolysis

116. An emergency situation at a chemical plant caused acute occupational


intoxication. A doctor who revealed the case of "acute occupational disease
(intoxication)" must notify the following authority:
A. Sanitary and epidemiological station
B. Ministry of Health of Ukraine
C. Trade union committee of the plant
D. Plant administration
E. Medical unit of the plant

117. At a machine-building plant the casts are cleaned by means of abrasion


machines that are a source of local vibration. What are the most efficient
preventive measures for preventing harmful effect of vibration on workers
organisms?
A. Use of gloves that reduce vibration
B. Giving sanitary instructions to the workers
C. Hand massaging
D. Preliminary and periodical medical examinations
E. Warm hand baths

118. In order to study impact of microclimate upon the human organism it


is necessary to make systematic observation of air temperature over 3 days.
Choose a device that will allow to make the most precise temperature records:
A. Thermograph
B. Assmann psychrometer
C. Mercury thermometer
D. Alcohol thermometer
E. Augusts psychrometer

119. The Carpathian region is characterized by constant high humidity of


atmospheric air (over 80%). Inhabitants of this region feel severe cold in
corresponding season at a medium low temperature. Its caused by heat
emission by:
A. Convection
B. Vaporization
C. Radiation
D. Conduction

120. Carpathian region is characterized by permanently high (over 80%) air


humidity. In the cold season the population of this region feels very cold at
moderately low temperatures. This is due to an increase in the heat transfer
by:
A. Convection
B. Evaporation
C. Radiation
D. Conduction
E. Emission

121. Poorly refined wastes of an industrial plant are usually thrown into the
river that supplies drinking water. It causes perishing of some
microorganisms, disturbs processes of water self-purification and worsens its
quality that can have negative influence upon peoples health. How is this
effect of environmental factors called?
A. Indirect
B. Associated
C. Direct
D. Complex
E. Combined

122. In order to reduce weed growth on agricultural land, some herbicides


have been used for a long time. In terms of environmental stability these
herbicides are rated as stable. Specify the most likely route of their entry into
the human body:
A. Soil-plants-humans
B. Soil-animals-humans
C. Soil-insects-humans
D. Soil-protozoa-humans
E. Soil-microorganisms-humans

123. In a city with population 400000 people 5600 fatal cases were
recorded, including 3300 cases because of blood circulation diseases, 730 -
because of tumors. What index will allow to characterize mortality from blood
circulation diseases in this city?
A. Intensive index
B. Correlation index
C. Relative intensity index
D. Extensive index
E. Visuality index

124. A selective population research study was aimed at exploring the effect
of air emissions from a metallurgical plant on the obstructive bronchitis
morbidity in a city. The calculated correlation coefficient was +0,79. Evaluate
the strength and direction of the relationship:
A. Direct, strong
B. Direct, average
C. Inverse, average
D. Inverse, strong

125. Statistic of patients of common medical practice displays constant


increase in elderly and old people number. What kind of pathology is expected
to prevail in the morbidity structure of population in question?
A. Chronic pathology
B. Infectious pathology
C. Non-epidemic pathology
D. Occupational pathology
E. Acute pathology
126. A general practitioner visited a 2-year-old child and diagnosed him
with measles. The child attends a nursery, has a 5-year-old sister. What
document must be filled in for the effective antiepidemic measures in the
given health locality?
A. Emergency notification on infectious disease (form No. 058/o)
B. Sick leave
C. Infants record (report form No. 112/o)
D. Carers leave certificate
E. House call record (form No. 031/o)

127. In the current year general practitioners of the municipal polyclinic


have referred 11 patients with coronary artery disease to the in-patient
hospital. In 3 cases the diagnosis wasn’t confirmed. What managerial decision
should be made in such case?
A. Analysis of each case of diagnostic divergence
B. Analysis of diagnostic examination quality
C. Analysis of material and technical basisof the polyclinic
D. Analysis of doctors skill level
E. Analysis of medical check-up quality

128. Hygienic expertise of a sample taken from the batch of grain revealed
that 2% of grains were infected with microscopic Fusarium fungi. On the
ground of laboratory analyses this batch of grain should be:
A. Sold without restrictions
B. Tested for toxicity
C. Used for ethanol production
D. Destroyed
E. Used for forage production

129. An employee has been diseased for 4 months, further treatment is


necessary, the patient is incapacitated. Who is authorized to provide further
disability examination of this patient?
A. Medical and social expert board
B. Deputy chief responsible for disability examination
C. Physician in charge and chief of department
D. Medical consultative board
E. Chief physician of a medical facility

130. Among the inhabitants of a workmens settlement located near an


industrial plant the cases of nervous and endocrine system diseases as well as
renal diseases became more frequent. Blood analyses revealed a decrease in
sulfhydryl groups. These pathologies might have been caused by the following
substance released into the environment:
A. Mercury
B. Boron
C. Lead
D. Chromium
E. Cadmium

131. Examination of a 43-year-old man objectively revealed pallor of skin


and mucous membranes, loss of tongue papillae, transverse striation of
fingernails, cracks in the mouth corners, tachycardia. Blood test results: Hb-
90 g/l, anisocytosis, poikilocytosis. The most likely causative agent of this
state is inadequate intake of:
A. Iron
B. Copper
C. Magnesium
D. Selene
E. Zinc

132. During the medical examination a port crane operator complained of


dizziness, nausea, sense of pressure against tympanic membranes, tremor,
dyspnoea, cough. He works aloft, the work is connected with emotional stress.
Workers are affected by vibration (general and local), noise, ultrasound,
microclimate that warms in summer and cools in winter. What factor are the
workers complaints connected with?
A. Infrasound
B. Intensity of work
C. Altitude work
D. Noise
E. Vibration

133. It is required to analyze the level of daylight illumination in a ward of


therapeutics unit. What device should be applied to estimate the level of
daylight illumination?
A. Illuminometer
B. Anemometer
C. Actinometer
D. Psychrometer
E. Katathermometer

134. During the medical examination at school the schoolchildren had to


undergo plantography. After the analysis of footprints platypodia was found in
30% of pupils. What is the percentage of the flatfoot isthmus?
A. 65%
B. 30%
C. 55%
D. 45%
E. 50%

135. A factorys sectorial doctor selects a group of persons who often fall ill
for thorough monitoring. At the same time he takes into consideration the
number of etiologically related cases with temporary disability in each of the
employees over the last year. An employee falls into this group if the number
of sickness cases is:
A. 4 or more
B. 2 or more
C. 6 or more
D. 3 or more
E. 1 or more

136. Production areas of a greenhouse complex have the following


microclimate parameters: air temperature - 42°C, humidity - 98%, air velocity
- 0,05 mps, temperature of enclosing surfaces - 15°C. Characterize the
microclimate of production areas:
A. Overheated
B. Comfortable
C. Uncomfortable
D. Satisfactory
E. Cooling

137. Sanitary-veterinary examination of a cow carcass revealed measle


contamination (2-3 measles per 10 cm2). What tactics should be chosen in
respect of this meat consumption?
A. The whole carcass should be technically disposed
B. Meat should be disinfected by boiling
C. Meat should be disinfected by freezing
D. Meat can be sold without any restrictions
E. The carcass should be used for the production of canned meat

138. A factory worker has ARD complicated by acute bronchitis. He receives


treatment in the outpatient setting. The attending doctor has issued him a
medical certificate for 5 days and then extended its duration by 5 more days.
Patient cant get down to work because of his health status. Who should extend
the duration of medical certificate for this patient?
A. A department chief
B. Deputy medical superintendent in charge of temporary disability examination
C. Medical advisory commission
D. Deputy medical superintendent in charge of medical treatment
E. Medical superintendent

139. Preventive examination of an 11 year old boy helped to determine his


habitus type. It was established that the childs shoulders were deviated and
brought forward, with forward flexion of head, the thorax was flattened,
abdomen was convex. The childs backbone had signs of deepened cervical and
lumbar curvatures. What habitus is it?
A. Kyphosis
B. Corrected
C. Normal
D. Lordosis
E. Round-shouldered

140. A plot of land with total area of 2,0 hectare was intended for building of
a hospital. The maximal capacity of the hospital will be:
A. 100 beds
B. 800 beds
C. Over 1000 beds
D. 200 beds
E. 400 beds

141. During hygienic examination of a hospital it was established that the


area for each bed in a double ward was: in the infectious department for
children - 7 m2, in the infectious department for adults - 8 m2, in the burns
department - 9 m2, in the radiological department - 10 m2, in the critical care
department - 13 m2. In which wards the area for each bed doesnt correspond
with hygienic requirements?
A. In burns wards
B. In infectious wards for adults
C. In infectious wards for children
D. In radiological wards
E. In critical care wards

142. According to the report of water quality control, drinking city water has
the following characteristics: turbidity - 1,5 mg/m3, odour - 3 points, metallic
taste - 2 points, pale yellow colour, colour index - 20o, temperature - 12o.
Which of these factors doesnt comply with hygienic requirements?
A. Odour
B. Taste
C. Colour index
D. Turbidity
E. Temperature
143. A 2 year old child has been ill with acute respiratory viral infection of
upper thrice a year - in February, in April and in December. How should these
occurences be recorded?
A. It is necessary to fill in 3 statistic talons signed (+)
B. It is necessary to fill in 1 statistic talon signed (+)
C. It is necessary to fill in 3 statistic talons signed (-)
D. It is necessary to fill in 1 statistic talon signed (+) and 2 statistic talons signed
(-)
E. It is necessary to fill in 1 statistic talon signed (-)

144. A worker diagnosed with "acute dysentery" was sent to the infectious
department by a doctor of aid post. What document should be used for
registration of this disease?
A. Urgent report on infectious disease
B. Outpatients card
C. Statistic coupon for registration of final diagnoses
D. Inpatients card
E. Statistic card of the patient who left in-patient hospital

145. Researchers studied disease incidence of influenza and acute


respiratory viral infection within the last 5 years. What kind of graphic
presentation should be used for the best visualization of this data?
A. Linear diagram
B. Bar diagram
C. Pie diagram
D. Histogram
E. Radial diagram

146. The correlation between the service record and eosinophil


concentration in blood was studied in workers at dyeing shops of textile
factories. What index will be the most informative for the analysis of this data?
A. Correlation factor
B. Standardized index
C. Sign index
D. Fitting criterion
E. Students criterion

147. The objective of a statistical study was to find out the extent of seeking
medical care by the population. For this purpose 300 residents of the area
were interviewed. Information was collected by means of a special
questionnaire. What method of collecting information was used by
researchers?
A. Anamnestic
B. Immediate examination
C. Immediate registration
D. Doing extracts

148. Analysis of organization of medical care in a regional centre has shown


that every year about 12% of patients receive inpatient care for diseases that
dont require round-the-clock monitoring and intensive care. What are the
most appropriate organizational changes required to address this problem?
A. Development of medical care forms replacing the in-patient care
B. Restructuring of specialized care
C. Upgrading of hospital facilities
D. Changes to the statute of outpatient clinics
E. Development of primary care

149. A citys population is 400000 inhabitants in 2005 there were registered


5600 deaths, including 3300 cases caused by cardiovascular diseases, 730 - by
tumours. Which of the following indicators allows to characterize the share of
the circulatory system diseases as the cause of death in the city?
A. Extensive index
B. Index of relative intensity
C. Ratio
D. Demonstrativeness index
E. Intensive index

150. It is planned to organize a rural outpatient clinic. The patients will be


able to visit the doctors of the following specialities:
A. Therapeutist, dentist, pediatrician, obstetrician-gynecologist
B. Therapeutist, pediatrician, neurologist
C. Pediatrician, therapeutist, ophthalmologist
D. Obstetrician-gynaecologist, therapeutist
E. Pediatrician, obstetrician-gynaecologist, ophthalmologist

151. In a rural health district a child died in the first month of life. In order
to analyze this situation it was necessary to complete expert evaluation of
medical records. Which medical document was analyzed in the first place?
A. Child developmental history record
B. Record of vaccinations
C. Childs medical record
D. Medical record of an outpatient
E. Neonatal record
152. A department chief of an in-patient hospital is going to inspect resident
doctors as to observation of medical-technological standards of patient
service. What documentation should be checked for this purpose?
A. Health cards of in-patients
B. Annual report of a patient care institution
C. Treatment sheets
D. Statistic cards of discharged patients
E. Registry of operative interventions

153. The head of prenatal care department intends to complete expert


evaluation of compliance with medical and technological standards of
pregnancy follow-up. What documents must be verified for this purpose?
A. Individual records of pregnant and postpartum women
B. Neonatal record
C. Medical records of outpatients
D. Prenatal records
E. Labour and delivery record

154. Workers of a laboratory producing measuring devices (manometers,


thermometers etc) complain about a mettalic taste in mouth, stomatitis,
dyspepsia, sleep disturbance, unsteady walk, abrupt decrease in cardiac
activity. These presentations must have been caused by the intoxication with
the following substance:
A. Mercury
B. Tetraethyl lead
C. Manganese
D. Lead
E. Toluol

155. While asessing the health status of graduates of a secondary school, the
doctor found one of them to have grade 3 tonsillar hypertrophy, chronic
rhinitis and vegetative-vascular dystonia. The organism functionality is
reduced. This student belongs to the following health group:
A. III
B. I
C. V
D. IV
E. II
156. At first appointment with an obstetrician-gynaecologist a pregnant
woman is referred to other medical specialists. She must be obligatory
examined by the following specialists:
A. Therapeutist and dentist
B. Dentist and phthisiatrician
C. Dentist and cardiologist
D. ENT and ophthalmologist
E. Therapeutist and endocrinologist

157. Estimation of physical development of a child involved dynamometry


and estimation of body weight and length, annual gain in body length, chest
circumference, number of permanent teeth, secondary sexual characters, lung
vital capacity. Which of the mentioned indices relates to the physiometric
ones?
A. Lung vital capacity, dynamometry
B. Body length and weight, chest circumference
C. Number of permanent teeth
D. Annual gain in body length
E. Secondary sexual characters

158. Estimation of community health level involved analysis of a report on


diseases registered among the population of district under charge (reporting
form 12). What index is calculated on the grounds of this report?
A. Common morbidity rate
B. Index of hospitalized morbidity
C. Index of basic non-epidemic morbidity
D. Index of pathological affection
E. Index of morbidity with temporary disability

159. A 50-year-old male suburbanite underwent treatment in rural


outpatient clinic for pneumonia. The treatment didnt have effect and the
disease got complicated by exudative pleuritis. What prevention and
treatment facility should the patient be referred to for further aid?
A. Central district hospital
B. Regional hospital
C. Municipal hospital
D. Tuberculosis dispensary
E. Phthisio-pulmonological dispensary

160. A 46-year-old patient was issued a 10-day sick list because of


exacerbation of chronic cholecystitis. The patients general condition got
better, but the clinical manifestations of the disease are still present. What
authority is entitled to extend the sick list?
A. Medical Expert Commission
B. Family doctor
C. Deputy head doctor for medical-labour expertise
D. Head doctor
E. Deputy head doctor for terapeutic management

161. A 59-year-old male patient with essential hypertension of stage II is


registered with the dispensary department of a polyclinic. The patient
regularly takes ACE inhibitors and calcium antagonists. How often should a
therapeutist examine this patient (except for exacerbation periods)?
A. Every 3 months
B. Every 4 months
C. Every 9 months
D. Once a year
E. Every 6 months

162. A municipal hospital reported on the number of operated patients


including fatal outcomes following the operations. Which index of hospital
work can be calculated on the ground of this data?
A. Postoperative lethality
B. Standardized lethality
C. Total lethality
D. Index of late hospitalization since a disease incursion

163. It is planned to build a multi-disciplinary hospital with 500 beds in a


town. Specify the location of a polyclinic within the medical centre:
A. At the main entrance
B. In the centre of the territory near medical buildings
C. In the garden and park area
D. It is not allowed to place the polyclinic within the centre territory
E. In the service zone

164. Evaluation results of sanitary and hygiene conditions in a 4-bed ward


were as follows: ward area - 30 m2, height - 3,2 m, temperature - 20°C,
humidity - 55%, air velocity - 0,1 m/s, window-to-floor area ratio - 1:5,
daylight ratio - 0,6%, concentration of carbon dioxide in the air - 0,1%. Which
of the given indicators does not meet hygienic requirements?
A. Daylight ratio
B. Concentration of carbon dioxide in the air
C. Ward area
D. Air velocity
E. Window-to-floor area ratio
165. Within the structure of the regions population the share of persons
aged 0 to 14 years is 25%, the share of persons aged 50 years and older is 30%.
What concept most accurately describes this demographic situation?
A. Regressive type of population age structure
B. Stationary type of population age structure
C. Immigration of population
D. Progressive type of population age structure
E. Cohort reproduction

Krok 2 – 2014 Pediatrics Base


1. An 8 year old child has low-grade fever, arthritis, colicky abdominal pain and
a purpuric rash llocalized on the lower extremities. laboratory studies reveal a
guaiac-positive stool, urinalysis with red blood cell (RBC) casts and mild
proteinuria, and a normal platelet count. The most likely diagnosis is:
A. Henoch-Schonleins vasculitis
B. Rocky Mountain spotted fever
C. Systemic lupus erythematosus (SLE)
D. Idiopathic thrombocytopenic purpura
E. Poststreptococcal glomerulonephritis

2. A young man has painful indurations in the peripapillary regions of both


mammary glands. The most reasonable action will be:
A. To leave these indurations untouched
B. To administer steroids locally
C. To cut and drain them
D. To remove them
E. To take an aspirate for bacterial inoculation and cytology

3. A 9 year old girl with a history of intermittent wheezing for several years is
brought to the pediatrician. The child has been taking no medications for
some time. Physical examination reveals agitation and perioral cyanosis.
Intercostal and suprasternal retractions are present. The breath sounds are
quiet, and wheezing is audible bilaterally. The child is admitted to the
hospital. Appropriate interventions might include all of the following
EXCEPT:
A. Prescribe nebulized cromolyn sodium
B. Prescribe intravenous corticosteroids
C. Prescribe nebulized metaproterenol
D. Prescribe intravenous aminophylline
E. Administer supplemental oxygen
4. Routine examination of a child with a history of bronchial asthma reveals AP
of 140/90 mm Hg. The most likely cause of the hypertension is:
A. Renal disease
B. Chronic lung disease
C. Theophylline overdose
D. Coarctation of the aorta
E. Obesity

5. Patient with thyreotoxicosis is in the 2 beds hospital ward of therapeutic


department. The area of the ward is 18 m2, height 3 m, ventilation rate 2,5/hr.
Air temperature - 20°C, relative humidity - 45%, air movement velocity - 0,3
m/s, light coefficient - 1/5, noise level - 30 dB. Do hygienic evaluation of the
conditions meet the standards?
A. Discomfortable microclimate
B. Poor lighting
C. Non-effective ventilation
D. High level of noise
E. All conditions meet the requirements

6. The child is 11 m.o. He suffers from nervous-arthritic diathesis. The increased


synthesis of what acid is pathogenic at nervous-arthritic diathesis?
A. Uric acid
B. Phosphoric acid
C. Sulfuric acid
D. Hydrochloric acid
E. Acetic acid

7. A 10-year-old child complains of fever (temperature is 39°C), frequent painful


urination [pollakiuria]. Urine test: proteinuria [0,066 g/L], leukocytouria
[entirely within eyeshot], bacteriuria [105 colony forming units/mL]. What is
the most probable diagnosis?
A. Acute pyelonephritis
B. Acute glomerulonephritis
C. Acute cystitis
D. Urolithiasis
E. Dysmetabolic nephropathy

8. A 8-year-old boy has suffered from tonsilitis. In 2 weeks he started


complaining of migratory joint pain, edema of joints, restriction of
movements, fever. On examination, an acute rheumatic heart disease, activity
of the III-rd degree, primary rheumocarditis, polyarthritis; acute course of
disease, cardiovascular failure IIA. What medication is to be prescribed?
A. Prednisone
B. Cefazolin
C. Diprazinum
D. Erythromycin
E. Delagil

9. The 10 y.o. boy has complains on headache, weakness, fever 40°C, vomiting,
expressed dyspnea, pale skin with flush on right cheek, lag of right hemithorax
respiratory movement, dullness on percussion over low lobe of right lung,
weakness of vesicular respiration in this zone. The abdomen is painless and
soft at palpation. Which disease lead to these symptoms and signs?
A. Pneumonia croupousa
B. Acute cholecystitis
C. Flu
D. Intestinal infection
E. Acute appendicitis

10. A patient with acute respiratory viral infection (3rd day of disease) complains
of pain in lumbar region, nausea, dysuria, oliguria. Urinalysis - hematuria
(100-200 RBC in eyeshot spot), specific gravity - 1002. The blood creatinin
level is 0,18 millimole/l, potassium level - 6,4 millimole/l. Make the diagnosis:
A. Acute interstitial nephritis
B. Acute glomerylonephritis
C. Acute renal failure
D. Acute cystitis
E. Acute renal colic

11. A neonate was born from the 1st gestation on term. The jaundice was revealed
on the 2nd day of life, then it became more acute. The adynamia, vomiting and
hepatomegaly were observed. Indirect bilirubin level was 275 mumol/L, direct
bilirubin level – 5 mumol/L, Hb- 150 g/l. Mothers blood group - 0(I), Rh+,
childs blood group - A(II), Rh+. What is the most probable diagnosis?
A. Hemolytic disease of the neonate (??0 incompatibility), icteric type
B. Physiological jaundice
C. Hemolytic disease of the neonate (Rh - incompatibility)
D. Jaundice due to conjugation disorder
E. Hepatitis

12. A baby boy was born in time, it was his mothers 1st pregnancy. The jaundice
was revealed on the 2nd day of life, then it progressed. The adynamia,
vomiting and hepatomegaly were presented. The indirect bilirubin level was
275 mcmol/L, the direct bilirubin level - 5 mcmol/L, Hb- 150 g/L. Mothers
blood group - 0(I), Rh+, childs blood group - A(II), Rh+. Make a diagnosis.
A. Hemolytic disease of newborn (Rh incompatibility), icteric type
B. Hemolytic disease of newborn (Rh - incompatibility)
C. Hepatitis
D. Jaundice due to conjugation disorder
E. Physiological jaundice

13. A 3 month old infant suffering from acute segmental pneumonia has dyspnea
(respiration rate - 80 per minute), paradoxical breathing, tachycardia, total
cyanosis. Respiration and pulse - ratio is 1:2. The heart dullness under normal
size. Such signs characterise:
A. Respiratory failure of III degree
B. Respiratory failure of I degree
C. Myocarditis
D. Congenital heart malformation
E. Respiratory failure of II degree

14. The 7 m.o. infant is suffering from acute pneumonia which was complicated
by cardiovascular insufficiency and respiratory failure of II degree. The
accompanied diagnosis is malnutrition of II degree. Choose the best variant of
therapy:
A. Ampiox and Amicacin
B. Gentamycin and Macropen
C. Ampiox and Polymixin
D. Macropen and Penicillin
E. Penicillin and Ampiox

15. A 3 year old child has been suffering from fever, cough, coryza, conjunctivitis
for 4 days. He has been taking sulfadimethoxine. Today it has fever up to 39°C
and maculopapular rash on its face. Except of rash the childs skin has no
changes. What is your diagnosis?
A. Measles
B. Pseudotuberculosis
C. Rubella
D. Allergic rash
E. Scarlet fever

16. A 2 year old girl has been ill for 3 days. Today she has low grade fever, severe
catarrhal presentations, slight maculopapular rash on her buttocks and
enlarged occipital lymph nodes. What is your diagnosis?
A. Rubella
B. Scarlet fever
C. Adenoviral infection
D. Pseudotuberculosis
E. Measles
17. A 3 year old boy fell ill abruptly: fever up to 39°C, weakness, vomitng.
Haemorrhagic rash of various size appeared on his lower limbs within 5 hours.
Meningococcemia with infective - toxic shock of the 1 degree was diagnosed.
What medications should be administered?
A. Chloramphenicol succinate and prednisone
B. Penicillin and immunoglobulin
C. Ampicillin and immunoglobulin
D. Chloramphenicol succinate and interferon
E. Penicillin and prednisone

18. A 7 year old girl has mild form of varicella. Headache, weakness, vertigo,
tremor of her limbs, ataxia, then mental confusion appeared on the 5th day of
illness. Meningeal signs are negative. Cerebrospinal fluid examination is
normal. How can you explain these signs?
A. Encephalitis
B. Meningitis
C. Myelitis
D. Neurotoxic syndrome
E. Meningoencephalitis

19. A 7 y.o. girl fell ill abruptly: fever, headache, severe sore throat, vomiting.
Minute bright red rash appear in her reddened skin in 3 hours. It is more
intensive in axillae and groin. Mucous membrane of oropharynx is hyperemic.
Greyish patches is on the tonsills. Submaxillary lymph nodes are enlarged and
painful. What is your diagnosis?
A. Scarlet fever
B. Rubella
C. Measles
D. Pseudotuberculosis
E. Enteroviral infection

20. An 8-year-old boy fell ill acutely: he presents with fever, weakness, headache,
abdominal pain, recurrent vomiting, then diarrhea and tenesmus. Stools occur
12 times daily, are scanty, contain a lot of mucus, pus, streaks of blood. His
sigmoid gut is tender and hardened. What is your diagnosis?
A. Dysentery
B. Cholera
C. Escherichiosis
D. Staphylococcal gastroenteritis
E. Salmonellosis
21. The child has complains of the "night" and "hungry" abdominal pains. At
fibroscopy in area a bulbus ofa duodenum the ulcerrative defect of 4 mms
diameter is found, the floor is obtected with a fibrin, (H.p +). Administer the
optimum schemes of treatment:
A. Omeprasole - Trichopolum - Claritromicin
B. Maalox - Ranitidin
C. De-nol
D. Vicalinum - Ranitidin
E. Trichopolum

22. A woman delivered a child. It was her fifth pregnancy but the first delivery.
Mothers blood group is A(II)Rh-, newborns - A(II)Rh+. The level of indirect
bilirubin in umbilical blood was 58 micromole/l, haemoglobin - 140 g/l, RBC-
3,8×1012/l. In 2 hours the level of indirect bilirubin turned 82 micromole/l.
The hemolytic disease of newborn (icteric-anemic type, Rh-incompatibility)
was diagnosed. Choose the therapeutic tactics:
A. Replacement blood transfusion (conservative therapy)
B. Blood transfusion (conservative therapy)
C. Conservative therapy
D. Symptomatic therapy
E. Antibiotics

23. A mother with an infant visited the pediatrician for expertise advice. Her baby
was born with body weight 3,2 kg and body length 50 cm. He is 1 year old
now. How many teeth the baby should have?
A. 8
B. 12
C. 10
D. 20
E. 6

24. A mother consulted a pediatrician about her son. Her son was born with body
mass of 3 kg and length of 48 cm. He is 1 year old now. What is the required
normal mass?
A. 10,5 kg
B. 9,0 kg
C. 12,0 kg
D. 15,0 kg
E. 11,0 kg

25. 6 m.o. infant was born with bodys mass 3 kg and length 50 cm. He is given
natural feeding. How many times per day the infant should be fed?
A. 5
B. 4
C. 6
D. 7
E. 8

26. Infant is 6,5 months now and is given natural feeding since birth. Body mass
was 3,5 kg, with length 52 cm at birth. How many times per day the
supplement (up feeding) should be given?
A. 2
B. 1
C. 4
D. 0
E. 3

27. A 2 month old healthy infant with good appetite is given artificial feeding
since he turned 1 month old. When is it recommended to start the corrective
feeding (fruit juice)?
A. 4,0 months
B. 2,0 months
C. 1,5 months
D. 3,0 months
E. 1,0 months

28. An infant was born with body mass 3 kg and body length 50 cm. Now he is 3
years old. His brother is 7 years old, suffers from rheumatic fever. Mother
asked the doctor for a cardiac check up of the 3-year-old son. Where is the left
relative heart border located?
A. 1 cm left from the left medioclavicular line
B. 1 cm right from the left medioclavicular line
C. 1 cm left from he left parasternal line
D. 1 cm right from the left parasternal line
E. Along the left medioclavicular line

29. A boy of 7 y.o. had an attack of asthma and distant whistling rales after
playing with a dog. In the medical hystory: atopic dermatitis caused by eating
eggs, chicken, beef. What group of allergens is the reason of the development
of bronchial astma attacks?
A. Epidermal
B. Chemical
C. Pollen
D. Dust
E. Itch mite
30. A 14-year-old boy has rheumatism. Over the last 2 years he has had 3
rheumatic attacks. What course of rheumatism does the patient have?
A. Prolonged
B. Subacute
C. Persistent-reccurent
D. Latent
E. Acute

31. The patient with aquired heart failure has diastolic pressure of 0 mm Hg.
What heart failure does the child have?
A. Aortal insufficiency
B. Aortal stenosis
C. Mitral stenosis
D. Mitral insufficiency
E. Rheumatism

32. A 12 year old child has the ulcer disease of stomach. What is the etiology of
this disease?
A. Intestinal bacillus
B. Influenza
C. Salmonella
D. Helicobacter pylory
E. Lambliosis

33. A nine year old child is at a hospital with acute glomerulonephritis. Clinical
and laboratory examinations show acute condition. What nutrients must not
be limited during the acute period of glomerulonephritis?
A. Carbohydrates
B. Liquid
C. Fats
D. Proteins
E. Salt

34. An 18-month-old child was taken to a hospital on the 4-th day of the disease.
The disease began acutely with temperature 39, weakness, cough,
breathlessness. He is pale, cyanotic, has had febrile temperature for over 3
days. There are crepitative fine bubbling rales on auscultation. Percussion
sound is shortened in the right infrascapular region. X-ray picture shows non-
homogeneous segment infiltration 8-10 mm on the right, the intensification of
lung pattern. Your diagnosis:
A. Segmentary pneumonia
B. Bronchiolitis
C. Interstitial pneumonia
D. Grippe
E. Bronchitis

35. A 9-year-old girl has attacks of abdominal pain after fried food. No fever. She
has pain in Cera point. The liver is not enlarged. Portion B [duodenal probe] -
50 ml. What is your diagnosis?
A. Biliary tracts dyskinesia, hypotonic type
B. Hepatocirrhosis
C. Chronic duodenum
D. Peptic ulcer
E. Acute colitis

36. A baby was born at 36 weeks of gestation. Delivery was normal, by natural
way. The baby has a large cephalohematoma. The results of blood count are:
Hb- 120g/l, Er- 3,5×1012/l, total serum bilirubin - 123 mmol/l, direct
bilirubin - 11 mmol/l, indirect - 112 mmol/l. What are causes of
hyperbilirubinemia in this case?
A. Erythrocyte hemolysis
B. Mechanical obstruction of the bile outflow
C. Disturbance of the conjugative function of liver
D. Intravascular hemolysis
E. Bile condensing

37. A 4-month-old girl with blond hair and blue eyes has "mousy" odor of sweat
and urine, delayed psychomotoric development. The most typical laboratory
data for this disorder is:
A. Positive urine ferric chloride test
B. High level of oxyproline in urine
C. High concentration of chlorides in sweat
D. Low level of thyroid gland hormones in blood
E. High level of glycosaminoglycanes in urine

38. A neonate is 5 days old. What vaccination dose of BCG vaccine (in mg) is
necessary for vaccination of this child?
A. 0,05 mg
B. 0,075 mg
C. 0,2 mg
D. 0,1 mg
E. 0,025 mg
39. 7 y.o. boy with chronic sinusitis and rercurent pulmonary infections has chest
X-ray demonstrating a right-sided cardiac silhouette. What is the most likely
diagnosis?
A. Kartagener syndrome
B. Bronchiolitis obliterans
C. Cystic fibrosis (mucoviscidosis)
D. Laryngotracheomalacia
E. alpha-antitrypsin deficiency

40. A 2,9-kg term male infant is born to a mother who developed polyhydramnios
at 34 weeks gestation. At birth, the Apgar scores were 9 and 9. The infant
develops choking and cyanosis with the first feed. In addition, is unable to
place a nasogastric tube. What is the most likely diagnosis?
A. Esophageal atresia
B. Laryngomalacia
C. Respiratory distress syndrome
D. Tracheal atresia
E. Choanal atresia

41. Full term newborn has developed jaundice at 10 hours of age. Hemolytic
disease of newborn due to Rh-incompatibility was diagnosed. 2 hours later the
infant has indirect serum bilirubin level increasing up to 14 mmol/L. What is
most appropriate for treatment of hyperbilirubinemia in this infant?
A. Exchange blood transfusion
B. Infusion therapy
C. Phenobarbital
D. Phototherapy
E. Intestinal sorbents

42. A 4 year old girl was playing with her toys and suddenly she got an attack of
cough, dyspnea. Objectively: respiration rate - 45/min, heart rate - 130/min.
Percussion revealed dullness of percutory sound on the right in the lower
parts. Auscultation revealed diminished breath sounds with bronchial
resonance on the right. X-ray pictue showed shadowing of the lower part of
lungs on the right. Blood analysis revealed no signs of inflammation. The child
was diagnosed with foreign body in the right bronchus. What complication
caused such clinical presentations?
A. Atelectasis
B. Emphysema
C. Bronchitis
D. Pneumonia
E. Pneumothorax
43. A man, 42 years old, died in a road accident after the hemorrhage on the spot,
because of acute hemorrhagic anemia. What minimum percent of the whole
blood volume could result in death by acute hemorrhage?
A. 25-30%
B. 10-14%
C. 35-50%
D. 15-20%
E. 6-9%

44. A 6 week old child is admitted because of tachypnea. Birth had been
uneventful, although conjunctivitis developed on the third day of life and
lasted for about 2 weeks. Physical examination reveals tachypnea, bilateral
inspiratory crackles and single expiratory wheezing. Bilateral pneumonia is
evident on chest X-ray. The child is afebrile and has no history of fever. White
blood cell count is 15×109/l, with 28% of eosinophils. The most likely cause of
this childs symptoms is:
A. Clamydia trachomanis
B. Varicella
C. Mycoplasma pneumoniae
D. Pneumocystis carinii
E. Visceral larva migrans

45. A 6 y.o. asthmatic child was taken to the emergency hospital because of severe
coughing and wheezing for the last 24 hours. Physical examination reveals
that the child is excitable, has intercostal and suprasternal retractions,
expiratory wheezing throughout all lung fields, RR- 60/min. Initial treatment
may include the prescription of:
A. Subcutaneous epinephrine
B. Parenteral phenobarbital
C. N-acetyl cysteine and cromolyn by inhalation
D. Parenteral gentamicyn
E. Intravenous fluids in the first 2 h to compensate water deficiency

46. A full term infant was born after a normal pregnancy, delivery, however, was
complicated by marginal placental detachment. At 12 hours of age the child,
although appearing to be in good health, passes a bloody meconium stool. For
determining the cause of the bleeding, which of the following diagnostic
procedures should be performed first?
A. Barium enema
B. An upper gastrointestinal series
C. Platelet count, prothrombin time, and partial thromboplastin time
D. An Apt test
E. Gastric lavage with normal saline
47. In the 43rd week of gestation a long, thin infant was delivered. He is apneic,
limp, pale, and covered with "pea soup" amniotic fluid. The first step in the
resuscitation of this infant at delivery should be:
A. Suction of the trachea under direct vision
B. Artificial ventilation with endotracheal tube
C. Catheterization of the umbilical vein
D. Administration of 100% oxygen by mask
E. Artificial ventilation with bag and mask

48. A newborn infant has mild cyanosis, diaphoresis, poor peripheral pule,
hepatomegaly and cardiomegaly. Respiratory rate is 60 breaths per minute,
and heart rate is 230 beats per minute. The child most likely has congestive
heart failure caused by:
A. Paroxysmal atrial tachycardia
B. A large atrial septal defect and valvular pulmonary stenosis
C. Atrial flutter and partial atrioventricular block
D. A ventricular septal defect and transposition of the great vessels
E. Hypoplastic left heart syndrome

49. A 6-year-old boy was brought to the emergency room with a 3-hour history of
fever up to 39,5°C and sore throat. The child looks alert, anxious and has a
mild inspiratory stridor. You should immediately:
A. Prepare to establish an airway
B. Admit the child and place him in a mist tent
C. Order a chest x-ray and lateral view of the neck
D. Obtain an arterial blood gas and start an IV line
E. Examine the throat and obtain a culture

50. A 7 d.o. boy is admitted to the hospital for evaluation of vomiting and
dehydration. Physical examination is otherwise normal except for minimal
hyperpigmentation of the nipples. Serum sodium and potassium
concentrations are 120 meq/L and 9 meq/L respectively. The most likely
diagnosis is:
A. Congenital adrenal hyperplasia
B. Pyloric stenosis
C. Panhypopituitarism
D. Hyperaldosteronism
E. Secondary hypothyroidism

51. A 7 y.o. boy has crampy abdominal pain and a rash on the back of his legs and
buttocks as well as on the extensor surfaces of his forearms. Laboratory
analysis reveals proteinuria and microhematuria. He is most likely to be
affected by:
A. Anaphylactoid purpura
B. Polyarteritis nodosa
C. Dermatomyositis
D. Systemic lupus erythematosus
E. Poststreptococcal glomerulonephritis

52. A 5-year-old boy was progressively getting worse compared to the previous 2
months. A chest x-ray has shown right middle lobe collapse. A tuberculin skin
test was strongly positive. What is the most characteristic finding in primary
tuberculosis?
A. Hilar or paratracheal lymph node enlargement
B. Cavity formation
C. Hematogenous dissemination leading to extrapulmonary tuberculosis
D. Miliary tuberculosis
E. Atelectasis with obstructive pneumonia

53. A girl is 12-year-old. Yesterday she was overcooled. Now she is complaining on
pain in suprapubic area, frequent painful urination by small portions,
temperature is 37,8°C. Pasternatsky symptom is negative. Urine analysis:
protein - 0,033 g/L, WBC- 20-25 in f/vis, RBC- 1-2 in f/vis. What diagnosis is
the most probable?
A. Acute cystitis
B. Dysmetabolic nephropathy
C. Acute pyelonephritis
D. Urolithiasis
E. Acute glomerulonephritis

54. The girl of 11 y.o. She is ill for 1 month. She has "butterfly"-type rash on face
(spots and papules), pain and swelling of small joints on arms and legs, signs
of stomatitis (small-sized ulcers in mouth). CBC: Hb– 80 g/L, RBC–
2,9×1012/L, WBC– 15×109/L, ESR- 40 mm/hour. Urinalysis: protein– 0,33
g/L. What is the most probable diagnosis?
A. Systemic lupus erythematosus
B. Periarteriitis nodosa
C. Juvenile rheumatoid arthritis, systemic type
D. Acute rheumatic fever
E. Dermatomyositis

55. An infant aged 1 year on the third day of common cold at night developed
inspiratory stridor, hoarse voice and barking cough. Physical examination
revealed suprasternal and intercostal chest retractions. There is a bluish skin
discoloration moistly seen over the upper lip. The respiratory rate is 52 per
min and pulse- 122 bpm. The body temperature is 37,5°C. What disease does
the infant have?
A. Acute infectious croup due to viral laryngotracheitis
B. Acute bronchiolitis with respiratory distress
C. Acute epiglottitis
D. Acute laryngitis
E. Bronchopneumonia without complications

56. A newborn aged 3 days with hyperbilirubinemia (428 mkmol/L) developed


following disorders. From beginning there were severe jaundice with poor
suckling, hypotomia and hypodynamia. Little bit later periodical excitation,
neonatal convulsions and neonatal primitive reflexes loss are noted. Now
physical examination reveals convergent squint, rotatory nystagmus and
setting sun eye sign. How to explain this condition?
A. Encephalopathy due to hyperbilirubinemia
B. Spastic cerebral palsy
C. Brain tumour
D. Skull injury
E. Hydrocephalus

57. A child is 2 years old. The child complains of hoarse voice, dyspnea with
obstructed inspiration. The disease started 3 days ago from dry cough and
nose stuffiness. Objectively: general condition is unbalanced, stridor is
present. The childs skin is pale. Body temperature is 37,7°C. The palatine
arches are hyperemic. There is no deposit. Heart sounds are rhythmic.
Auscultation of lungs reveals rough breathing sounds, crepitation is absent.
Parainfluenza virus has been detected in nasopharynx lavage. What is the
most likely diagnosis?
A. Acute laryngotracheitis
B. Foreign body
C. Epiglottitis
D. Diphtheria
E. Laryngospasm

58. A 3-year-old child has been admitted to a hospital because of ostealgia and
body temperature rise up to 39°C. Objectively: the patient is in grave
condition, unable to stand for ostealgia, there is apparent intoxication, lymph
nodesare enlarged up to 1,5 cm. Liver can be palpated 3 cm below the costal
margin, spleen - 2 cm below the costal margin. In blood: RBCs - 3,0×1012/l,
Hb- 87 g/l, colour index - 0,9, thrombocytes – 190×109/l, WBCs - 3,2×109/l,
eosinophils - 1, stab neutrophils - 1, segmented neutrophils - 0, lymphocytes -
87, monocytes - 2, ESR - 36 mm/h. What examination should be conducted in
order to specify the diagnosis?
A. Sternal puncture
B. Computer tomography
C. Lymph node puncture
D. Ultrasound
E. Lymph node biopsy

59. Apgar test done on a newborn girl at 1st and 5th minute after birth gave the
result of 7-8 scores. During the delivery there was a short-term difficulty with
extraction of shoulder girdle. After birth the child had the proximal extremity
dysfunction and the arm couldnt be raised from the side. The shoulder was
turned inwards, the elbow was flexed, there was also forearm pronation,
obstetric palsy of brachial plexus. What is the clinical diagnosis?
A. Duchenne-Erb palsy
B. Trauma of right hand soft tissues
C. Right hand osteomyelitis
D. Trauma of thoracic spine
E. Intracranial haemorrhage

60. Examination of a 9-month-old girl revealed skin pallor, cyanosis during


excitement. Percussion revealed transverse dilatation of cardiac borders.
Auscultation revealed continuous systolic murmur to the left of the breastbone
in the 3-4 intercostal space. This murmur is conducted above the whole
cardiac region to the back. What congenital cardiac pathology can be
suspected?
A. Defect of interventricular septum
B. Pulmonary artery stenosis
C. Coarctation of aorta
D. Defect of interatrial septum
E. Fallots tetrad

61. A worker was temporarily off work because of illness for 16 days, was under
out-patient treatment. The doctor in charge issued a sick-list first for 5 days,
then prolonged it for 10 days. Who can further prolong the sick-list of this
patient?
A. The doctor in charge of the case together with the head of
department
B. The doctor in charge of the case with the permission of the head of
department
C. The head of department
D. Deputy head physician on the working ability expertise
E. Working ability expertise committee
62. A 13 y.o. patient was treated in dermatological hospital for atopic dermatitis
exacerbation. He was discharged in the condition of clinical remission. What
recommendations should the doctor give to prevent exacerbations?
A. Use of neutral creams to protect skin
B. Frequent skin washing with detergents
C. Systematic skin disinfection
D. Avoidance of skin insolation
E. Systematic use of local corticosteroids

63. On the 21 day after appearance of vesiculous chickenpox rash a 7-year-old


child developed ataxia, nystagmus, intention tremor, muscle hypotonia.
Liquor analysis shows a low-grade lymphocytic pleocytosis, slightly increased
protein rate. What complication is it?
A. Encephalitis
B. Purulent meningitis
C. Acute nephritis
D. Postherpetic neuralgia
E. Pneumonitis

64. An 8-year-old boy suffering from haemophilia was undergoing transfusion of


packed red cells. Suddenly he felt pain behind the breastbone and in the
lumbar area, dyspnea, cold sweat. Objectively: pale skin, heart rate - 100/min,
AP - 60/40 mm Hg; oliguria, brown urine. For the treatment of this
complication the following drug should be administered:
A. Prednisolone
B. Adrenaline
C. Analgine
D. Aminophylline
E. Lasix

65. A 3-year-old child has been diagnosed with type I diabetes mellitus,
hyperosmolar coma. The laboratory confirmed the diagnosis. Which
laboratory findings are characteristic for such condition?
A. High hyperglycemia without ketonemia
B. Hyperglycemia and glucosuria
C. Hyperglycemia and high indicators of acid-base balance
D. Hyperglycemia and ketonuria
E. Hyperglycemia and ketonemia

66. A 3-year-old child was playing in a playpen when he suddenly developed


paroxysmal cough and shortness of breath. Objectively: dry cough, mixed
dyspnea. Lung auscultation revealed some wheezes. Breathing sounds on the
right are diminished. The child doesnt mix with other children. Immunization
is age-appropriate. What pathological condition can be suspected?
A. Foreign body in the respiratory tracts
B. Pneumonia
C. Pertussis
D. Bronchial asthma
E. Acute respiratory viral infection

67. A 10-year-old child has been folowed-up for the dilated cardiomyopathy. The
child presents with dyspnea, cardialgia. There are dense, nonmobile edemata
on the lower extremities and sacrum. Ps- 120/min. The cardiac borders are
extended transversely. Heart sounds are muffled, there is blowing systolic
murmur at the apex and over the xiphoid process. Liver is 3 cm enlarged,
urine output is reduced. The blood total protein - 58.6 g/l. In urine: protein -
0,025 g/l, WBCs - 2-4 in the field of vision, RBCs - 2-3 in the field of vision.
What is the main mechanism of edema syndrome development:
A. Venous congestion of greater circulation
B. Secondary nephropathy development
C. Hypoproteinemia
D. Venous congestion of lesser circulation
E. Peripheral circulation disorder

68. After objective clinical examination a 12 year old child was diagnosed with
mitral valve prolapse. What complementary instrumental method of
examination should be applied for the diagnosis confirmation?
A. Echocardiography
B. Phonocardiography
C. Veloergometry
D. ECG
E. Roentgenography of chest

69. A full-term child survived antenatal and intranatal hypoxia, it was born in
asphyxia (2-5 points on Apgar score). After birth the child has progressing
excitability, there are also vomiting, nystagmus, spasms, strabismus,
spontaneous Moros and Babinskys reflexes. What localization of intracranial
hemorrhage is the most probable?
A. Subarachnoid hemorrhage
B. Subdural hemorrhage
C. Hemorrhages into the brain ventricles
D. Periventricular hemorrhages
E. Small cerebral tissue hemorrhages
70. A 15 y.o. boy was twice attacked by bees, as a result he had severe anaphylactic
shock. What is the most effective prophylaxis method?
A. Desensibilisation by means of bee venom extract
B. Prescription of corticosteroids for summer
C. Limitation of outside staying during summer months
D. Protective clothing
E. Long-term prophylactic treatment with antihistamines

71. A 9-year-old boy has been suffering from bronchoectasis since he was 3.
Exacerbations occur quite often, 3-4 times a year. Conservative therapy results
in short periods of remission. The disease is progressing, the child has
physical retardation. The childs skin is pale, acrocyanotic, he has "watch glass"
nail deformation. Bronchography revealed saccular bronchiectases of the
lower lobe of his right lung. What is the further treatment tactics?
A. Surgical treatment
B. Sanatorium-and-spa treatment
C. Tempering of the childs organism
D. Further conservative therapy
E. Physiotherapeutic treatment

72. A child with tetralogy of Fallot is most likely to exhibit:


A. Increased pressure in the right ventricle
B. Normal oxygen tension (PaO2) in the left ventricle
C. Increased pulse pressure
D. Increased pulmonary blood flow
E. Normal pressure gradient across the pulmonary valve

73. A 2-months-old child after preventive vaccination had a prolonged


hemorrhage from the vaccination place and due to those an intramuscular
hematoma. During examination of the child a considerable rise of
prothrombin consumption and a significant prolongation of the activated
partial thromboplastic time were found. What is the most probable diagnosis?
A. Hemophilia
B. Henoch-Schoenlein disease
C. Inborn afibrinogenemia
D. Hemorrhagic disease of the neonate
E. Werlhofs disease

74. A 10 y.o. boy with hemophilia has signs of acute respiratory viral infection
with fever. What of the mentioned antifebrile medications are contraindicated
to this patient?
A. Acetylsalicylic acid
B. Paracetamol
C. Panadol extra
D. Analgin
E. Pipolphen

75. A 7-year-old child is sick for 2 weeks with running nose, was taking nasal
drops. The boy suffers with alimentary allergy. He applied to doctor due to
suppurative and bloody discharges from nose, maceration of ala nasi and
upper lip. Rhinoscopy results: there are whitish-greyish areas at nasal septum.
Mucous membrane of oropharynx is not changed. What is the most probable
disease?
A. Diphtheria of the nose
B. Allergic rhinitis
C. Sinusitis (maxillar sinus))
D. Adenovirus
E. Rhinovirus

76. A 10-year-old boy underwent treatment in cardiological department for


rheumatism, I acute attack of rheumatic fever, active phase, II degree. The
patient was discharged in satisfactory condition. Which drug should be chosen
for prevention of rheumatism recurrence?
A. Bicillinum-5
B. Erythromycin
C. Oxacillin
D. Ampicillin
E. Bicillinum-1

77. A child is 4 years old, has been ill for 5 days. There are complaints of cough,
skin rash, to- 38,2°C, face puffiness, photophobia, conjunctivitis. Objectively:
there is bright, maculo-papulous, in some areas confluent rash on the face,
neck, upper chest. The pharynx is hyperemic. There are seropurulent
discharges from the nose. Auscultation revealed dry rales in lungs. What is the
most likely diagnosis?
A. Measles
B. Adenoviral infection
C. Rubella
D. Enterovirus exanthema
E. Scarlet fever

78. A 10 month old boy has been ill for 5 days after consumption of unboiled milk.
Body temperature is 38-39°C, there is vomiting, liquid stool. The child is pale
and inert. His tongue is covered with white deposition. Heart sounds are
muffled. Abdomen is swollen, there is borborygmus in the region of ubbilicus,
liver is enlarged by 3 cm. Stool is liquid, dark-green, with admixtures of
mucus, 5 times a day. What is the most probable diagnosis?
A. Salmonellosis
B. Acute shigellosis
C. Rotaviral infection
D. Staphylococcal enteric infection
E. Escherichiosis

79. A 3 year old child with weight deficiency suffers from permanent moist cough.
In history there are some pneumonias with obstruction. On examination:
distended chest, dullness on percussion over the lower parts of lungs. On
auscultation: a great number of different rales. Level of sweat chloride is 80
millimol/l. What is the most probable diagnosis?
A. Mucoviscidosis (cystic fibrosis)
B. Bronchiectasis
C. Pulmonary hypoplasia
D. Bronchial asthma
E. Recurrent bronchitis

80.A 12 y.o. child with acute glomerulonephritis presented with hypertensive


syndrom during first days of the disease. What is the role of angiotesin II in
the pathogenesis?
A. Intensifies production and secretion of aldosterone
B. Increases erythropoetin production
C. Increases renine level
D. Increases heart output
E. Infibits deppresive action of prostaglandins

81. A full-term infant is 3 days old. On the different parts of skin there are
erythemas, erosive spots, cracks, areas of epidermis peeling. The infant has
scalded skin syndrome. Nikolskys symptom is positive. General condition of
the infant is grave. Anxiety, hyperesthesia, febrile temperature are evident.
What is the most probable diagnosis?
A. Exfoliative dermatitis
B. Impetigo neonatorum
C. Mycotic erythema
D. Phlegmon of newborn
E. Fingers pseudofurunculosis

82. District pediatrician examines a healthy carried 1-month-old child. The child
is breast-fed. Prophylaxis of what disease will the doctor recommend to do
first?
A. Rachitis
B. Parathropy
C. Hypotrophia
D. Anemia
E. Spasmophilia

83. A 7-year-old boy has been managed for a month. Immediately after
hospitalization there were apparent edemata, proteinuria - 7,1 g/l, daily urine
protein - 4,2 g. Biochemical blood test shows persistent hypoproteinemia
(43,2 g/l), hypercholesterolemia (9,2 millimole/l). The patient is most likely
have the following type of glomerulonephritis:
A. Nephrotic
B. Isolated urinary
C. Combined
D. Hematuric
E. Nephritic

84. A 3 y.o. girl has had a temperature rise up to 38°C, rhinitis, dry superficial
cough, flabbiness, appetite loss. Palpation didnt reveal any changes over her
lungs. Percussion sound has a wooden resonance, auscultation revealed
puerile breathing, no rales. In blood: leukopenia, lymphocytosis, increased
ESR. What is the most probable diagnosis?
A. Acute simple tracheitis
B. Recurrent bronchitis, acute condition
C. Bilateral microfocal pneumonia
D. Acute simple bronchitis
E. Acute obstructive bronchitis

85. A 5-year-old girl with the transitory immunodeficiency according to T-system


has a clinical picture of a right-sided pneumonia during 2 months. How
pneumonia progress can be described?
A. Delaying
B. Chronic
C. Recidivating
D. Wavelike
E. Acute

86. Mother of a 10-month-old baby reports significant pallor, poor appetite,


enlarged abdomen in the baby. As a neonate, the child underwent treatment
in the in-patient hospital for jaundice and anemia. Objectively: the skin is pale
and jaundiced, teeth are absent, abdomen is enlarged, spleen is palpable.
Blood test results: Hb - 90 g/l, RBC - 3,0×1012/l, color index - 0,9,
microspherocytosis, reticulocytosis up to 20%, serum bilirubin - 37 mmol/l,
unconjugated bilirubin - 28 mmol/l. What type of anemia has occurred in the
patient?
A. Hemolytic anemia
B. Protein-deficiency anemia
C. Hereditary elliptocytosis
D. B12-deficiency anemia
E. Iron-deficiency anemia

87. A 12 y.o. girl took 2 pills of aspirine and 4 hours later her body temperature
raised up to 39-40°C. She complains of general indisposition, dizziness,
sudden rash in form of red spots and blisters. Objectively: skin lesions
resemble of second-degree burns, here and there with erosive surface or
epidermis peeling. Nikolskys symptom is positive. What is the most probable
diagnosis?
A. Acute epidermal necrolisis
B. Polymorphous exudative erythema
C. Duhrings disease
D. Bullous dermatitis
E. Pemphigus vulgaris

88. A 5-year-old child had an attack of palpitation with nausea, dizziness,


generalized fatigue. On ECG: tachycardia with heartbeat rate of 220/min.
Ventricle complexes are deformed and widened. P wave is absent. What
medication is to be prescribed to provide first aid?
A. Lydocain
B. Novocainamides
C. Strophantin
D. Isoptin
E. Seduxen

89. Examination of a 4 month old child revealed some lemon-yellow squamae


with fatty crusts on the scalp. What is the most probable diagnosis?
A. Gneiss
B. Pseudofurunculosis
C. Infantile eczema
D. Milk crust
E. Strophulus

90. A lumbar puncture was performed for a newborn suspected of having an


intracranial birth injury. Bloody cerebrospinal fluid was obtained. What
hemorrhage occurred in this case?
A. Subarachnoid
B. Supratentorial
C. Subtentorial
D. Cephalohematoma
E. Epidural

91. A neonate from gestation with severe gestosis of the second half was born on
the 41st week with 2400 g birth weight and 50 cm long. On physical
examination: skin is flaccid, subcutaneous fatty cellular tissue is thin, muscle
hypotonia, new-born period reflexes are decreased. Internal organs are
without pathological changes. How would you estimate this child?
A. Term infant with pre-natal growth retardation
B. Premature infant
C. Postmature infant
D. Term infant with normal body weight
E. Immature infant

92. A child was taken to a hospital with focal changes in the skin folds. The child
was anxious during examination, examination revealed dry skin with solitary
papulous elements and ill-defined lichenification zones. Skin eruption was
accompanied by strong itch. The child usually feels better in summer, his
condition is getting worse in winter. The child has been artificially fed since he
was 2 months old. He has a history of exudative diathesis. Grandmother by his
mothers side has bronchial asthma. What is the most likely diagnosis?
A. Atopic dermatitis
B. Urticaria
C. Seborrheal eczema
D. Contact dermatitis
E. Strophulus

93. A boy, aged 9, was examined: height - 127 cm (-0,36), weight - 28,2 kg
(+0,96), chest circumference - 64,9 cm (+0,66), lung vital capacity - 1520 ml
(-0,16). What is the complex assessment of the childs physical development?
A. Harmonious
B. Apparently disharmonious
C. Disharmonious
D. Excessive
E. Below the average

94. A child is 7 months old. Birth weight was 3450, the child is breastfed.
Supplemental feeding was introduced on time. Determine the daily protein
requirements for the child:
A. 3,0 g/kg
B. 2,0 g/kg
C. 3,5 g/kg
D. 4,0 g/kg
E. 2,5 g/kg

95. 2 weeks after recovering from angina an 8-year-old boy developed edemata of
face and lower limbs. Objectively: the patient is in grave condition, AP-
120/80 mm Hg. Urine is of dark brown colour. Oliguria is present. On urine
analysis: relative density - 1,015, protein - 1,2 g/l, RBCs are leached and cover
the whole vision field, granular casts - 1-2 in the vision field, salts are
represented by urates (big number). What is the most likely diagnosis?
A. Acute glomerulonephritis with nephritic syndrome
B. Acute glomerulonephritis with nephrotic syndrome, hematuria and
hypertension
C. Nephrolithiasis
D. Acute glomerulonephritis with isolated urinary syndrome
E. Acute glomerulonephritis with nephrotic syndrome

96. A 14 year old child suffers from vegetovascular dystonia of pubertal period. He
has got sympathoadrenal atack. What medicine should be used for attack
reduction?
A. Obsidan
B. No-shpa
C. Aminophylline
D. Corglicone
E. Amysyl

97. A child is 9 months old. The patients body temperature is 36,7°C, the skin is
pale, humid, there is pain in leg muscles. There is no extremities mobility,
sensitivity is present. The child has been diagnosed with poliomyelitis. The
causative agent of this disease relates to the following family:
A. Picornavirus
B. Adenovirus
C. Rotavirus
D. Paramyxovirus
E. Tohovirus

98. A 4 month old child fell seriously ill: body temperature rose up to 38,5°C, the
child became inert and had a single vomiting. 10 hours later there appeared
rash over the buttocks and lower limbs in form of petechiae, spots and
papules. Some haemorrhagic elements have necrosis in the centre. What is the
most probable disease?
A. Meningococcemia
B. Influenza
C. Rubella
D. Haemorrhagic vasculitis
E. Scarlet fever

99. A 5-year-old child had strong headache, vomiting, ataxy, dormancy,


discoordination of movements, tremor of the extremities on the 8th day of the
disease. It was followed by rise in body temperature, vesiculosis rash mainly
on the skin of the body and the hairy part of the head. At the second wave of
the fever a diagnosis of encephalitis was given. What disease complicated
encephalitis in this case?
A. Chicken pox
B. Herpetic infection
C. German measles
D. Measles
E. Enterovirus infection

100. A 13 year old girl was admitted to the cardiological department because
of pain in the muscles and joints. Examination of her face revealed an
edematic erythema in form of butterfly in the region of nose bridge and
cheeks. What is the most probable diagnosis?
A. Systemic lupus erythematosus
B. Periarteritis nodosa
C. Dermatomyositis
D. Rheumatism
E. Rheumatoid arthritis

101. A 4 y.o. boy was admitted to the hospital with complaints of dyspnea,
rapid fatigability. His anamnesis registers frequent respiratory diseases. On
percussion: heart borders are dilatated to the left and upwards. On
auscultation: amplification of the SII above pulmonary artery, a harsh
systolodyastolic "machine" murmur is auscultated between the II and the III
rib to the left of breast bone, this murmur is conducted to all other points
including back. AP is 100/20 mm Hg. What is the most probable diagnosis?
A. Opened arterial duct
B. Isolated stenosis of pulmonary arterial orifice
C. Interventricular septal defect
D. Interatrial septal defect
E. Valvar aortic stenosis

102. A 12 year old girl complains about abrupt weakness, nausea, dizziness,
vision impairment. The day before she ate home-made stockfish, beef.
Examination revealed skin pallor, a scratch on the left knee, dryness of
mucous membranes of oral pharynx, bilateral ptosis, mydriatic pupils. The girl
is unable to read a simple text (mist over the eyes). What therapy would be the
most adequate in this case?
A. Parenteral introduction of polyvalent antibotulinic serum
B. Parenteral introduction of antitetanus serum
C. Parenteral introduction of antibiotics
D. Parenteral disintoxication
E. Gastric lavage

103. A child from the first non-complicated pregnancy but complicated


labor had cephalhematoma. On the second day there developed jaundice. On
the 3th day appeared changes of neurologic status: nystagmus, Graefes sign.
Urea is yellow, feces- golden-yellow. Mothers blood group is A(II)Rh-, child-
A(II)Rh+. On the third day childs Hb- 200 g/L, RBC- 6,1×1012/L, bilirubin in
blood - 58 mk mol/L due to unconjugated bilirubin, Ht- 0,57. What is the
childs jaundice explanation?
A. Brain delivery trauma
B. Hemolytic disease of newborn
C. Physiologic jaundice
D. Bile ducts atresia
E. Fetal hepatitis

104. A full-term baby (the 1st uncomplicated pregnancy, difficult labour)


had a cephalogematoma. On the 2nd day there was jaundice, on the third the
following changes in neurological status appeared: nystagmus, Graefe
syndrome. Urine was yellow, feces were of golden-yellow colour. Mothers
blood group is A(II)Rh-, the babys one - A(II)Rh+. On the third day the childs
Hb was 200g/l, RBCs - 6,1×1012/l, blood bilirubin - 58 micromole/l at the
expense of unbound fraction. What caused the jaundice in the child?
A. Craniocerebral birth trauma
B. Biliary atresia
C. Fetal hepatitis
D. Physiological jaundice
E. Neonatal anaemia

105. After birth a child was pale and had arrhythmical breathing. Oxygen
therapy didnt have any effect. Pulse was weak and rapid. It was difficult to
measure arterial pressure accurately. There were no edemata. What is the
most likely reason for these symptoms?
A. Asphyxia
B. Congestive heart failure
C. Intrauterine sepsis
D. Congenital pneumonia
E. Intracranial haematoma
106. A child was delivered severely premature. After the birth the child has
RI symptoms, anasarca, fine bubbling moist rales over the lower lobe of the
right lung. Multiple skin extravasations, bloody foam from the mouth have
occured after the 2 day. On chest X-ray: atelectasis of the lower lobe of the
right lung. In blood: Hb-100 g/L, Ht- 0,45. What is the most probable
diagnosis?
A. Edematous-hemorrhagic syndrome
B. Pulmonary edema
C. Disseminated intravascular clotting syndrome
D. Hyaline membrane disease
E. Congenital pneumonia

107. An infant is 2 days old. He was born full-term with signs of intrauterine
infection, and therefore receives antibiotics. Neonates should be given
antibiotics at longer intervals and lower doses compared to older children and
adults because:
A. Neonates have lower glomerular filtration
B. Neonates have a reduced activity of glucuronyl transferase
C. Neonates have higher hematocrit
D. Neonates have a decreased blood pH
E. Neonates have lower concentration of protein and albumin in blood

108. An infant is 2 d.o. It was full-term born with signs of intrauterine


infection, thats why it was prescribed antibiotics. Specify, why the gap
between antibiotic introductions to the new-born children is longer and
dosage is smaller compared to the older children and adults?
A. The newborns have a lower level of glomerular filtration
B. The newborns have reduced activity of glucuronil transferase
C. The newborns have bigger hematocrit
D. The newborns have diminished blood pH
E. The newborns have lower concentration of protein and albumins in blood

109. A 10-year-old child is sick with chronic viral hepatitis B with marked
activity of the process. Total bilirubin – 70 mumol/L, direct - 26mumol/L,
indirect – 44 mumol/L. AST - 6,2 mmol/L, ALT - 4,8 mmol/L. What
mechanism underlies the transaminase level increase of this patient?
A. Cytolysis of hepatocytes
B. Intrahepatic cholestasis
C. Failure of bilirubin conjugation
D. Failure of the synthetical function of the liver
E. Hypersplenism
110. A 12-year-old girl applied to doctor with complaints of swelling on the
front part of the neck. The doctor diagnosed hyperplasia of the thyroid gland
of the second degree, euthyroidism. Ultrasound suspected autoimmune
thyroiditis. Blood was taken for titre of antibodies to thyroglobulin. What titre
of antibodies will be diagnostically important?
A. 1:100
B. 1:250
C. 1:150
D. 1:50
E. 1:200

111. A 14-year-old girl has been presenting with irritability and tearfulness
for about a year. A year ago she was also found to have diffuse enlargement of
the thyroid gland (II grade). This condition was regarded as a pubertal
manifestation, the girl didnt undergo any treatment. The girls irritability
gradually gave place to a complete apathy. The girl got puffy face, soft tissues
pastosity, bradycardia, constipations. Skin pallor and gland density
progressed, the skin became of a waxen hue. What disease may be suspected?
A. Autoimmune thyroiditis
B. Subacute thyroiditis
C. Juvenile basophilism
D. Diffuse toxic goiter
E. Thyroid carcinoma

112. In the anamnesis of a 2-year-old girl there are recurrent pneumonias


with signs of obstruction. There are heterogeneous moist and dry rales,
respiration is weakened. Dense, viscous secretion is difficult to hawk. There
are "drumsticks", physical retardation. What is the most probable diagnosis?
A. Mucoviscidosis, pulmonary form
B. Bronchial asthma
C. Pulmonary tuberculosis
D. Congenital pulmonary polycystosis
E. Recidivating bronchitis

113. On the 3rd day of life a baby presented with haemorrhagic rash, bloody
vomit, black stool. Examination revealed anaemia, extended coagulation time,
hypoprothrombinemia, normal thrombocyte rate. What is the optimal
therapeutic tactics?
A. Vitamin K
B. Calcium gluconate
C. Epsilon-aminocapronic acid
D. Sodium ethamsylate
E. Fibrinogen

114. A 2 month old full-term child was born with weight 3500 g and was on
the mixed feeding. Current weight is 4900 g. Evaluate the current weight of
the child:
A. Corresponding to the age
B. Hypotrophy of the I grade
C. 150 g less than necessary
D. Hypotrophy of the II grade
E. Paratrophy of the I grade

115. A 2 m.o. breast-fed child suffers from cheek skin hyperemia, sporadic
papulous elements on the skin of the chest and back following the apple juice
introduction. The child is restless. What is the initial pediatritians tactics?
A. Clarify mothers diet and exlude obligate allergens
B. Apply ointment with corticosteroids to affected skin areas
C. Administer general ultraviolet irradiation
D. Refer to prescribe dermathologist
E. Treat with claritine

116. A 5 month old boy was born prematurely, he didnt suffer from any
disease at the infant age and later on. Examination at an outpatients hospital
revealed paleness of skin, sleepiness. Blood count: Hb - 95 g/l, erythrocytes -
3,5×1012/l, reticulocytes – 9 o/oo, colour index - 0,7, osmotic stability of
erythrocytes - 0,44-0,33%, serum iron - 4,9 micromole/l. What is the most
probable cause of anemia?
A. Iron deficit
B. Hemogenesis immaturity
C. Erythrocyte hemolysis
D. B12 deficit
E. Infectious process

117. A 7 y.o. child had elevation of temperature tol 40°C in anamnesis. For
the last 3 months he presents fusiform swelling of fingers, ankle joints and
knee joint, pain in the upper part of the sternum and cervical part of the spinal
column. What is the most probable diagnosis?
A. Juvenile rheumatic arthritis
B. Osteoarthrits
C. Toxic synovitis
D. Rheumatism
E. Septic arthritis
118. An 8 year old girl complains about joint pain, temperature rise up to
38°C, dyspnea. Objectively: the left cardiac border is deviated by 2,5 cm to the
left, tachycardia, systolic murmur on the apex and in the V point are present.
Blood count: leukocytes – 20×109/l, ESR - 18 mm/h. What sign gives the
most substantial proof for rheumatism diagnosis?
A. Carditis
B. Leukocytosis
C. Arthralgia
D. Fever
E. Accelerated ESR

119. A 5 y.o. child with stigmas of dysembryogenesis (small chin, thick lips,
opened mouth, hyperthelorismus) has systolic murmur in the second
intercostal to the right of the sternum. The murmur passes to the neck and
along the sternum left edge. The pulse on the left brachial artery is weakened.
BP on the right arm is 110/60 mm Hg, on the left - 100/60 mm Hg. ECG
results: hypertrophy of the right ventricle. What defect is the most probable?
A. Aortic stenosis
B. Coarctation of the aorta
C. Open aortic duct
D. Defect of interventricular septum
E. Defect of interatrial septum

120. A 1,5-year-old child fell ill acutely with high temperature 38°C,
headache, fatigue. The temperature declined on the fifth day, muscular pain in
the right leg occured in the morning, there were no movements and tendon
reflexes, sensitivity was reserved. What is the initial diagnosis?
A. Polyomyelitis
B. Viral encephilitis
C. Osteomyelitis
D. Hip joint arthritis
E. Polyartropathy

121. A 3-year-old child has been delivered to a hospital in soporose state


with considerable amyotonia, inhibition of tendon and periosteal reflexes.
Miosis and asthenocoria are also present. Corneal reflexes are preserved.
Pulse is rapid and weak. AP- 80/50 mm Hg. The parents suspect the child of
accidental taking some tablets. Such clinical presentations are typical for
intoxication with the following tableted drugs:
A. Tranquilizers
B. Beta-2-adrenoceptor agonists
C. Antihypertensive drugs
D. Antropine drugs
E. Barbiturates
122. A 2 m.o. child with birth weight 5100 g has jaundice, hoarse cry,
umbilical hernia, physical development lag. Liver is +2 cm enlarged, spleen is
not enlarged. In anamnesis: delayed falling-away of umbilical cord rest. In
blood: Hb- 120 g/L, erythrocytes - 4,5×1012/L, ESR- 3 mm/h. Whole serum
bilirubin is 28 mcmole/L, indirect - 20 mcmole/L, direct - 8 mcmole/L. What
is the most probable diagnosis?
A. Congenital hypothyreosis
B. Hemolitic anemia
C. Cytomegalovirus infection
D. Conjugated jaundice
E. Congenital hepatitis

123. A 5-year-old child developed an acute disease starting from body


temperature rise up to 38,5°C, running nose, cough and conjunctivitis. On the
4th day the child presented with maculo-papular rash on face. Body
temparature rose again up to 39,2°C. Over the next few days the rash spread
over the whole body and extremities. Mucous membrane of palate was
hyperemic, there was whitish deposition on cheek mucous membrane next to
molars. What is your provisional diagnosis?
A. Measles
B. Acute viral respiratory infection
C. Enterovirus diseases
D. Rubella
E. Yersinia

124. A 3 year old child fell acutely ill, body temperature rose up to 39,5°C,
the child became inert, there appeared recurrent vomiting, headache.
Examination revealed positive meningeal symptoms, after this lumbal
puncture was performed. Spinal fluid is turbid, runs out under pressure,
protein concentration is 1,8 g/l; Pandy reaction is +++, sugar concentration is
2,2 millimole/l, chloride concentration - 123 millimole/l, cytosis is 2,35×109
(80% of neutrophils, 20% of lymphocytes). What is the most probable
diagnosis?
A. Purulent meningitis
B. Brain tumour
C. Serous tuberculous meningitis
D. Serous viral meningitis
E. Subarachnoid haemorrhage

125. A 13 y.o. girl complains of having temperature rises up to febrile figures


for a month, joint ache, periodical skin rash. Examination revealed steady
enhancing of ESR, LE-cells. What is the most probable diagnosis?
A. Systematic lupus erythematosus
B. Rheumatics
C. Systematic scleroderma
D. Juvenile rheumatoid arthritis
E. Acute lymphoblast leukosis

126. A 7-year-old child was brought to a doctor for a check. The child has a
4-year history of bronchial asthma, asthma attacks occur mainly in spring and
summer. Allergy tests revealed hypersensitivity to poplar seed tufts, field
herbs. What recommendation should be given?
A. Specific hyposensitization
B. Phytotherapy
C. Needle reflexotherapy
D. Physiotherapy
E. Treatment at a health resort

127. A 9-month-old child presents with fever, cough, dyspnea. The


symptoms appeared 5 days ago after a contact with a person having ARVI.
Objectively: the child is in grave condition. Temperature of 38°C, cyanosis of
nasolabial triangle is present. RR- 54/min, nasal flaring while breathing.
There was percussion dullness on the right below the scapula angle, and
tympanic sound over the rest of lungs. Auscultation revealed bilateral fine
moist rales predominating on the right. What is the most likely diagnosis?
A. Acute pneumonia
B. Acute bronchiolitis
C. Acute laryngotracheitis
D. ARVI
E. Acute bronchitis

128. An 8 y.o. boy complains of constant cough along with discharge of


greenish sputum, dyspnea during physical activities. At the age of 1 year and 8
months he fell ill for the first time with bilateral pneumonia that had
protracted course. Later on there were recurrences of the disease 5-6 times a
year, during the remission periods there was constant productive cough. What
examination results will be the most important for making a final diagnosis?
A. Bronchography
B. Bacterial inoculation of sputum
C. Roentgenography of thorax organs
D. Bronchoscopy
E. Spirography

129. A mother of a 5 y.o. girl consulted a doctor about doughters involuntary


urination at night, nightmares, sleep disorders, slow gaining of body weight.
Objectively: malnutrition, intellectual development is good, the girl can read
and explains common situations quite adultly. Her skin is very pale, liver is
enlarged in size. Her mother suffers from holetithiasis. What type of diathesis
is the most probable in the childs case?
A. Gouty diathesis
B. Lymphohypoplastic diathesis
C. Exudative diathesis
D. Urine acid diathesis
E. Allergic diathesis

130. A 10 year old girl complains about abdominal pain that is arising and
getting worse after eating rough or spicy food. She complains also about sour
eructation, heartburn, frequent constipations, headache, irritability. She has
been suffering from this for 12 months. Objectively: the girls diet is adequate.
Tongue is moist with white deposit at the root. Abdomen is soft, painful in its
epigastric part. What study method will help to make a diagnosis?
A. Esophagogastroduodenoscopy
B. Biochemical blood analysis
C. Fractional examination of gastric juice
D. Intragastral pH-metry
E. Contrast roentgenoscopy

131. A 40 h.o. child age has hyperosthesia, CNS depression, dyspepsia.


Sepsis is suspected. What should the differential diagnosis be made with?
A. Hypoglycemia
B. Hypomagnesemia
C. Hyperbilirubinemia
D. Hypocalcemia
E. Hyperkaliemia

132. Examination of a full-term 6-day-old infant revealed that different


areas of skin had erythemas, flaccid bubbles, eroded surface, cracks, peeling of
the epidermis looking like being scalded with boiling water. There was positive
Nikolskys symptom. General condition of the child was serious. The child was
restless, hypersensitive, febrile. What is the most likely diagnosis in this case?
A. Ritters exfoliative dermatitis
B. Neonatal pemphigus
C. Epidermolysis
D. Neonatal phlegmon
E. Fingers pseudofurunculosis

133. A 1,5 y.o. child fell seriously ill: chill, body temperature rise up to
40,1°C, then rapid dropping to 36,2°C, skin is covered with voluminous
hemorrhagic rash and purple cyanotic spots. Extremities are cold, face
features are sharpened. Diagnosis: meningococcosis, fulminant form,
infection-toxic shock. What antibiotic must be used at the pre-admission
stage?
A. Soluble Levomycetine succinate
B. Lincomycin
C. Penicillin
D. Gentamycin
E. Sulfamonometoxin

134. A 10 year old boy suffers from chronic viral hepatitis type B with
maximal activity. What laboratory test can give the most precise characteristic
of cytolysis degree?
A. Transaminase test
B. Takata-Ara test
C. Weltmans coagulation test
D. Prothrombin test
E. Test for whole protein

135. A 6 y.o child complains of thirst, polyuria, increased appetite for 2


months with weight loss for 3 kg. There has been nocturnal enuresis during
last week. On examination: hyperglycemia 14 mol/L. The diagnosis is diabetis
mellitus I type. What is the genesis of this disease?
A. Autoimmune
B. Neurogenic
C. Virus-bacterial
D. Viral
E. Bacterial

136. A 10 y.o. child who is at oligoanuretic stage of acute renal insufficiency


has got sensations of pricking in the mucous membrane of oral cavity and
tongue, extremities numbness, reduced reflexes, respiratory disturbance,
arrhythmia. What are these symptoms caused by?
A. Hyperkaliemia
B. Hyperazotemia
C. Alkalosis
D. Acidosis
E. Hyponatremia

137. Examination of a 12 year old child revealed diffuse thyroid enlargement


of the II degree. Heart auscultation revealed dullness of heart sounds, heart
rate was 64/min. The child has frequent constipations, anemia. Concentration
of thyreoglobulin antibodies is increased. What disease might have caused
such symptoms?
A. Autoimmune thyroiditis
B. Thyroid carcinoma
C. Endemic goiter
D. Thyroid hyperplasia
E. Diffuse toxic goiter

138. An 8-year-old girl has been admitted to the cardiology department.


Objectively: there is a skin lesion over the extensor surfaces of joints with
atrophic cicatrices, depigmentation, symmetrical affection of skeletal muscles
(weakness, edema, hypotrophy). What disease are these changes most typical
for?
A. Dermatomyositis
B. Systemic scleroderma
C. Systemic lupus erythematosus
D. Reiters disease
E. Nodular periarteritis

139. An 8-year-old child with a 3-year history of diabetes was hospitalized in


hyperglycemic coma. Specify the initial dose of insulin to be administered:
A. 0,1-0,2 U/kg of body weight per hour
B. 0,05 U/kg of body weight per hour
C. 0,3-0,4 U/kg of body weight per hour
D. 0,4-0,5 U/kg of body weight per hour
E. 0,2-0,3 U/kg of body weight per hour

140. A 12-year-old girl undergoes regular gastroenterological check-ups for


duodenal ulcer, biliary dyskinesia. What is the recommended frequency of
anti-relapse treatment?
A. Twice a year
B. Every 3 months
C. Every two months
D. Once a year
E. Three times a year

141. A 13 y.o. teenager who suffers from hemophilia A was taken to the
hospital after a fight at school. His diagnosis is right-sided hemarthros of knee
joint, retroperitoneal hematoma. What should be primarily prescribed?
A. Fresh frozen plasma
B. Washed thrombocytes
C. Aminocapronic acid
D. Placental albumin
E. Dry plasma

142. A 3 m.o. child fell seriously ill, body temperature rised up to 37,8°C,
there is semicough. On the 3-rd day the cough grew worse, dyspnea appeared.
On percussion: tympanic sound above lungs, on auscultation: a lot of fine
moist and wheezing rales during expiration. What is the most probable
diagnosis?
A. Acute respiratory viral infection, bronchiolitis
B. Acute respiratory viral infection, bronchitis with asthmatic component
C. Acute respiratory viral infection, focal pneumonia
D. Acute respiratory viral infection, bronchopneumonia
E. Acute respiratory viral infection, bronchitis

143. On the 1st day of life a full-term girl (2nd labour) weighing 3500g, with
Apgar score of 8 points, presented with jaundice. Indirect bilirubin of blood -
was 80 micromole/l, 6 hours later - 160 micromole/l. What is the optimal
method of treatment?
A. Exchange blood transfusion
B. Phototherapy
C. Phenobarbital treatment
D. Enterosorbents
E. Infusion therapy

144. A child was born at a gestational age of 34 weeks in grave condition.


The leading symptoms were respiratory distress symptoms, namely sonorous
and prolonged expiration, involving additional muscles into respiratory
process. The Silverman score at birth was 0 points, in 3 hours it was 3 points
with clinical findings. Which diagnostic study will allow to diagnose the form
of pneumopathy?
A. X-ray of chest
B. Determination of blood gas composition
C. Clinical blood test
D. Proteinogram
E. Immunoassay

145. A 10-year-old girl consulted a doctor about thirst, frequent urination,


weight loss. She has been observing these symptoms for about a month.
Objectively: no pathology of internal organs was revealed. What laboratory
analysis should be carried out in the first place?
A. Blood glucose analysis on an empty stomach
B. Acetone in urine test
C. Glucose in urine test on the base of daily diuresis
D. Glucose tolerance test
E. Glucosuric profile

146. A 6-year-old child complains of frequent liquid stool and vomiting. On


the 2nd day of desease the child presented with inertness, temperature rise up
to 38,2°C, Ps- 150 bpm, scaphoid abdomen, palpatory painful sigmoid colon,
defecation 10 times a day with liquid, scarce stool with mucus and streaks of
green. What is a provisional diagnosis?
A. Shigellosis
B. Escherichiosis
C. Salmonellosis
D. Intestinal amebiasis
E. Yersiniosis

147. A 4-year-old boy had untimely vaccination. He complains of painful


swallowing, headache, inertness, fever. Objectively: the child is pale, has
enlarged anterior cervical lymph nodes, swollen tonsils with cyanotic
hyperemia, tonsils are covered with gray-white pellicles which cannot be easily
removed. When the pellicles are forcibly removed, the tonsils bleed. What is
the most likely diagnosis?
A. Oropharyngeal diphtheria
B. Infectious mononucleosis
C. Follicular tonsillitis
D. Lacunar tonsillitis
E. Pseudomembranous tonsillitis

148. After a 10-year-old child had been bitten by a bee, he was delivered to a
hospital. There were lip, face and neck edemata. The patient felt hot and short
of breath. Objectively: breathing was laboured and noisy. There were foamy
discharges from the mouth, cough. The skin was pale and cold. There was
bradypnoea. Heart sounds were muffled and arrhythmic. Thready pulse was
present. What diagnosis was made by the expert in resuscitation?
A. Anaphylactic shock
B. Bronchial asthma
C. Cerebral coma
D. Acute cardiovascular collapse
E. Quinckes edema

149. A 13-year-old girl complains of fever up to 37,4°C during the last 2


months after recovering from ARVI. Objectively: malnutrition, diffuse grade II
enlargement of the thyroid gland feeling dense on palpation, exophthalmos,
tachycardia. What kind of pathological syndrome is it?
A. Thyrotoxicosis
B. Hypoparathyroidism
C. Thymomegaly
D. Hyperparathyroidism
E. Hypothyroidism

150. A 3-year-old girl presents with pertussis-like cough with thick sputum.
There have been persistent changes in lungs since the age of 6 months when
she was first diagnosed with acute pneumonia. Chloride concentration in the
perspiration is 112 mEq/l. The child has been diagnosed with mucoviscidosis.
What is the basis for autosomal recessive disease - mucoviscidosis?
A. Inadequate transport of sodium and chloride ions
B. ?1-antitrypsin deficiency
C. Pulmonary cysts
D. Pulmonary artery hypoplasia
E. Deposition of calcium triphosphates and carbotates in the alveoles

151. A newborn has purulent discharges from the umbilical wound, the skin
around the navel is swollen. The babys skin is pale, with a yellow-gray tint,
generalized hemorrhagic rash is present. What is the most likely diagnosis?
A. Sepsis
B. Omphalitis
C. Hemolytic disease of the newborn
D. Hemorrhagic disease of the newborn
E. Thrombocytopathy

152. From urine of a 14-year-old boy with the exacerbation of secondary


obstructive pyelonephritis Pseudomonas aeruginosa was isolated with a titer
of 1000000 microbes per 1 ml. Which antibiotic is most advisable to be
administered in this case?
A. Ciprofloxacin
B. Azithromycin
C. Chloramphenicol
D. Ampicillin
E. Cefazolinum

153. A 14-year-old boy with a history of chronic tonsillitis and sinusitis has
developed a feeling of heart irregularities and additional pulse. HR- 83/min.
ECG results: regular impulses with no visible P wave that occur every two
sinus contractions, QRS complex is dramatically deformed and prolonged to
over 0,11 s, T wave is discordant followed by a complete compensatory pause.
Specify the arrhythmia type:
A. Trigeminal extrasystole
B. Left bundle branch block
C. Partial AV-blockade
D. Bigeminal extrasystole
E. Complete AV-block

154. An 8-year-old girl periodically has sudden short-term heart pain,


sensation of chest compression, epigastric pain, dizziness, vomiting.
Objectively: the patient is pale, respiratory rate - 40/min, jugular pulse is
present. Ps- 185 bpm, of poor volume. AP- 75/40 mm Hg. ECG taken during
an attack shows ectopic P waves, QRS wave is not deformed. At the end of an
attack a compensatory pause is observed. The most likely cause of the attack
is:
A. Paroxysmal atrial tachycardia
B. Paroxysmal ventricular tachycardia
C. Atrial fibrillation
D. Complete AV-block
E. Sinus tachycardia

155. A 10-year-old child with a history of nonrheumatic carditis has periodic


attacks manifested by heart pain, dyspnea, pallor, high blood pressure, a
dramatic increase in heart rate up to 180/min. What drug would be most
effective to treat this patient?
A. Obsidan
B. Procainamide
C. Verapamil
D. Ajmaline
E. Lidocaine

156. A 1-month-old child became restless and presented with an increase in


head sweating. Its known from the history that the child has been fed with
cows milk since birth (September 5). Examination revealed craniotabes. A
doctor administered a course of UV radiation. Decide, if the child needs
ergocalciferol:
A. 2-2,5 months after the UVR withdrowal
B. In combination with UVR
C. A month after the UVR withdrowal
D. Immediately after the UVR withdrowal
E. Does not need

157. 15 minutes after the second vaccination with DTP vaccine a 4-month-
old boy exhibited the symptoms of Quinckes edema. What medication should
be given for emergency aid?
A. Prednisolone
B. Adrenalin
C. Heparin
D. Furosemide
E. Seduxen

158. A baby is 3 months old. The mother consulted a pediatrician about lack
of breast milk. After several test weighings it was found that the child had to
receive supplementary feeding. What is the optimal milk formula for this
child?
A. Malysh
B. Milk formula No. 3
C. Malutka
D. Whole cows milk
E. Milk formula No. 2

159. Examination of a newborn revealed skin redness that appeared


immediately after birth and reached the maximum intensity on the second day
of life. What is your provisional diagnosis?
A. Simple erythema
B. Transient erythema
C. Toxic erythema
D. Erythema nodosum
E. Annular erythema

160. A child is 2 days old. He was born with a weight of 2900 g, body length
of 50 cm. On examination the skin is intensely red, elastic, with preserved
turgor. Puerile respiration is present. Respiration rate - 40/min, cardiac
sounds are rhythmic, sonorous. HR- 138/min. The abdomen is soft. The liver
extends 2 cm below the costal margin. Diuresis is sufficient. Stool is in form of
meconium. What is the most likely diagnosis?
A. Physiological erythema of the newborn
B. Neonatal phlegmon
C. Exfoliative Ritters dermatitis
D. Erysipelas
E. Toxic erythema of the newborn

161. A full-term baby was born with body weight of 3200 g, body length of
50 cm, Apgar score - 8-10 points. What is the optimum time for the first
breast-feeding?
A. First 30 minutes
B. First 6 hours
C. First 48 hours
D. After 48 hours
E. First 24 hours
162. A 3-year-old child has been taken to a pediatrician. He has no recent
history of any diseases. Objective examination revealed no pathology of the
internal organs. The child needs the routine immunization against the
following disease:
A. Poliomyelitis
B. Type B hepatitis
C. Measles, rubella, parotitis
D. Diphtheria and tetanus
E. Pertussis

163. An 11-year-old girl has been immunized according to her age and in
compliance with the calendar dates. What vaccinations should the children
receive at this age?
A. Diphtheria and tetanus
B. TB
C. Hepatitis B
D. Pertussis
E. Polio

164. A 6-year-old child has duodenal ulcer. What antibacterial drug should
be co-administered together with metronidazole and De-Nol in order to
eradicate Helicobacter pylori infection?
A. Amoxicillin
B. Oleandomycin
C. Sulfadimethoxinum
D. Biseptol
E. Tetracycline

165. A baby born after fast labour has palsy of hand muscles. Grasp reflex is
absent, as well as hand-to-mouth reflex. Hand sensitivity is absent. What is
the most likely diagnosis?
A. Dejerine-Klumpke palsy
B. Duchenne-Erbs palsy
C. Muscle paresis
D. Bernard-Horner syndrome
E. Total lesion of the brachial plexus

166. A child is 12 years old. He complains of a dull aching pain in the


epigastrium and right hypochondrium, that is getting worse after taking fatty
or fried food, headache, weakness, nausea, low-grade fever. Abdominal
palpation reveals a marked resistance of muscles in the right hypochondrium,
positive Kerrs, Ortners, Murphys symptoms. What is the most likely
diagnosis?
A. Chronic cholecystitis
B. Acute gastritis
C. Acute pancreatitis
D. Acute appendicitis
E. Viral hepatitis

167. A 3-month-old girl presents with rhinitis, dyspnea, dry cough. These
manifestations has been observed for two days. Objectively: the child has pale
skin, acrocyanosis, shallow respiration at the rate of 80/min. Percussion
reveals handbox resonance over the whole surface of lungs, massive fine rales.
What is the most likely diagnosis?
A. Acute bronchiolitis
B. Acute bronchitis
C. Mucoviscidosis
D. Pneumonia
E. Foreign body of the airway

168. During the first home visit to a full-term boy after his discharge from
the maternity hospital a pediatrician revealed a symmetrical swelling of
mammae without skin changes over them, swelling of the scrotum. The body
temperature was of 36,5°C. The baby was calm, sucked the mothers breast
actively. What condition should you think of?
A. Hormonal crisis of the newborn
B. Congenital adrenal dysfunction
C. Sclerema
D. Neonatal mastitis
E. Necrotic neonatal phlegmon

169. A full-term neonate weighing 4500 g was born asphyxiated with Apgar
score of 4-6 points. During the delivery shoulder dystocia occurred.
Neurologic assessment revealed non-focal neurologic symptoms, total flaccid
paresis of the upper extremities since the arm was atonic and pronated.
Grasping, Babkins and Moros reflexes were absent. What segments of spinal
cord had been affected?
A. CV - ThI
B. CIII - CIV
C. ThVI - ThV
D. ThI - ThV
E. CI - CII
170. A newborn (mothers I pregnancy) weighing 3500 g presents with
jaundice, lethargy, reduced reflexes. Objectively: second grade jaundice of skin
with saffron tint, liver - +2 cm, spleen - +1 cm. Urine and feces are yellow.
Blood count: Hb- 100 g/l, RBCs - 3,2×1012/l, WBCs - 18,7×109/l, mothers
blood type - A(I) Rh(+), babys blood type - A(II) Rh(-), bilirubin - 170 mmol/l,
indirect fraction. ALT, AST rates are normal. What disease is the child most
likely to have?
A. Hemolytic disease of newborn, AB0-conflict
B. Hemolytic disease of newborn, Rh-conflict
C. Physiologic jaundice
D. Biliary atresia
E. Perinatal hepatitis

171. A 10-year-old girl was admitted to a hospital with carditis


presentations. It is known from the anamnesis that two weeks ago she had
exacerbation of chronic tonsillitis. What is the most likely etiological factor in
this case?
A. Streptococcus
B. Pneumococcus
C. Staphylococcus
D. Klebsiella
E. Proteus

172. All the joints on the left elbow of a newborn are extended, the whole
arm hangs vertically along the trunk with the forearm pronated. Active
movements in the elbow joint are absent but present in the shoulder joint. The
hand is flattened, atrophied, cold to the touch, hangs passively. Grasp reflex
and hand-mouth reflex on the affected side are missing. Haemogram values
are normal. What is the most likely diagnosis?
A. Inferior distal obstetrical paralysis
B. Complete obstetrical paralysis
C. Hypoxic-ischemic encephalopathy
D. Osteomyelitis
E. Proximal obstetrical paralysis

173. Head circumference of a 1-month-old boy with signs of excitement is 37


cm, prefontanel is 2x2 cm large. After feeding the child regurgitates small
portions of milk; stool is normal in respect of its volume and composition.
Muscle tonus is within norm. What is the most likely diagnosis?
A. Pylorospasm
B. Microcephaly
C. Craniostenosis
D. Meningitis
E. Pylorostenosis
174. 10 days after birth, a newborn developed a sudden fever up to 38,1°C.
Objectively: the skin in the region of navel, abdomen and chest is
erythematous; there are multiple pea-sized blisters with no infiltration at the
base; single bright red moist erosions with epidermal fragments on the
periphery. What is your provisional diagnosis?
A. Epidemic pemphigus of newborn
B. Streptococcal impetigo
C. Atopic dermatitis
D. Vulgar impetigo
E. Syphilitic pemphigus

175. On the second day after preventive vaccination a 2-year-old boy


presented with abdominal pain without clear localization, body temperature
rose up to 38°C. On the third day the child got red papular haemorrhagic
eruption on the extensor surfaces of limbs and around the joints. Knee joints
were edematic and slightly painful. Examination of other organs and systems
revealed no pathological changes. What is the most likely diagnosis?
A. Haemorrhagic vasculitis
B. Thrombocytopenic purpura
C. Urticaria
D. DIC syndrome
E. Meningococcemia

176. On the 6th day of life a child got multiple vesicles filled with
seropurulent fluid in the region of occiput, neck and buttocks. General
condition of the child is normal. What disease should be suspected?
A. Vesiculopustulosis
B. Miliaria
C. Impetigo neonatorum
D. Impetigo
E. Epidermolysis bullosa

177. A patient is 14 years old. Cytochemical study of punctate revealed 40%


of blasts, there was negative reaction to peroxidase and with Sudan black,
positive reaction to glycogen. Specify the form of acute leukemia:
A. Lymphoblastic
B. Monoblastic
C. Undifferentiated
D. Promyelocytic
E. Myeloblastic
178. Six months ago, a 5-year-old child was operated for CHD. For the last 3
weeks he has complained of fever, heart pain, aching muscles and bones.
Examination results: "white-coffee" skin colour, auscultation revealed systolic
murmur in the region of heart along with a noise in the III-IV intercostal
space. Examination of fingertips revealed Janeway lesions. What is your
provisional diagnosis?
A. Infectious endocarditis
B. Nonrheumatic carditis
C. Sepsis
D. Acute rheumatic fever
E. Typhoid fever

179. The condition of a 3-year-old child with acute non-rheumatic


myocarditis has suddenly deteriorated: he presents with anxiety, acrocyanosis,
peripheral edemata, dyspnea. Auscultation of lungs reveals fine moist rales on
both sides mainly in the lower parts. AP- 65/40 mm Hg. HR- 150/min, heart
sounds are muffled, arrhythmic (extrasystole). Liver is +4 cm. Oliguria is
present. The child has been diagnosed with acute heart failure. Which method
of examination is most informative for assessing the childs status dynamics?
A. Echocardiography
B. 24-hour monitoring of heart rhythm
C. Diuresis monitoring
D. ECG
E. Monitoring of K+, Na+ concentration in blood

180. A hospital admitted an 11-year-old boy diagnosed with medium-severe


asthma, exacerbation period. In order to arrest the attacks the boy was
administered broncholytic nebulizer therapy. During the day the childs
condition stabilized. What is the most appropriate method for further
monitoring of respiratory function in this patient?
A. Peak flowmetry
B. Pneumotachometry
C. Veloergometry
D. Bronchodilatation tests
E. Spirometry

181. A full-term newborn was born with body weight of 4000 g, body length
of 57 cm. Reaction to the postnatal check was absent. There was diffuse
cyanosis, heart rate of 80/min. What resuscitation measures should be taken?
A. Start ALV with a mask
B. Intubate the child and start ALV
C. Give an injection of naloxone
D. Start tactile stimulation
E. Give 100% oxygen
182. A 2-year-old child in a satisfactory condition periodically presents with
moderate proteinuria, microhematuria. USI results: the left kidney is
undetectable, the right one is enlarged, there are signs of double pyelocaliceal
system. What study is required to specify the diagnosis?
A. Excretory urography
B. Retrograde urography
C. Radioisotope renal scan
D. Doppler study of renal vessels
E. Micturating cystography

183. An 8-year-old boy has a 2-year history of blotchy itchy rash appearing
after eating citrus fruit. The first eruption occurred at the age of 6 months
after the introduction of juices to the babys diet. Father has a history of
bronchial asthma, mother - that of allergic rhinitis. What is the most likely
diagnosis?
A. Atopic dermatitis
B. Psoriasis
C. Urticaria
D. Quinckes edema
E. Pityriasis Rosea

184. An 8-year-old child was hospitalized for fever up to 39,8°C, inertness,


moderate headache, vomiting. Examination revealed meningeal symptoms.
Lumbar puncture was performed. The obtained fluid had raised opening
pressure, it was transparent, with the cell count of 450 cells per 1mcL (mainly
lymphocytes - 90%), glucose level of 2,6 mmol/l. What causative agent might
have caused the disease in the child?
A. Enterovirus
B. Kochs bacillus
C. Pneumococcus
D. Staphylococcus
E. Meningococcus

185. A 3-year-old child with ARVI had been administered biseptol,


paracetamol, nazoferon. On the third day of treatment the babys condition
deteriorated: he developed sore throat, stomatitis, conjunctivitis,
hypersalivation, painful dark red spots on the neck, face, chest and legs, then
the spots were replaced with vesicles. Examination revealed lesions of mucous
membranes around the mouth and anus. What is your provisional diagnosis?
A. Stevens-Johnson syndrome
B. Serum sickness
C. Bullous dermatitis
D. Atopic dermatitis
E. Chickenpox

186. A 12-year-old child had three attacks of acute rheumatic fever


accompanied by carditis. Examination revealed the symptoms of chronic
tonsillitis, mitral insufficiency, carious teeth. What is the optimal method of
secondary prophylaxis?
A. Year-round bicillin prophylaxis till the age of 25
B. Year-round bicillin prophylaxis for 3 years
C. Course of cardiotrophic drugs twice a year
D. Tonsillectomy
E. Oral cavity sanitation

187. A 7-year-old female child has developed an acute condition. She


complains of a headache, two onsets of vomiting. Objectively: deferred
reactions, body temperature - 39,3°C, pronounced hyperesthesia, nuchal
rigidity, positive superior and inferior Brudzinskis signs, symmetric Kernigs
sign. What is the provisional diagnosis?
A. Meningitis
B. Food toxicoinfection
C. Toxic encephalopathy
D. Encephalitis
E. Craniocerebral trauma

Krok 2 – 2014 Surgery Base


1. A 76 y.o. woman complains of progressing swallowing disorder, mostly she
has had problems with solid food for the last 6 weeks. Sometimes she has
regurgitation of solid masses. Swallowing is not painful. She lost 6 kg. 10 years
ago she had myocardiac infarction, she takes constantly aspirine and
prolonged nitrates. She consumes alcochol in moderate proportions, smokes.
Objectively: icteric skin, neck has no pecularities, lymph nodes are not
enlarged. Thorax has no changes, cardiovascular system has no evident
changes. Liver is +3 cm. What is the preliminary diagnosis?
A. Cancer of esophagus
B. Diffuse constriction of esophagus
C. Diaphragmatic hernia
D. Myasthenia
E. Esophageal achalasia

2. A 65 y.o. man who has problems with urination as a result of benign prostate
gland adenoma dveloped fever and chill, hypotension, sinus tachycardia. Skin
is warm and dry. Clinical blood analysis revealed absolute granulocytopenia.
These hemodynamic changes are most likely to be caused by:
A. Endotoxemia with activation of complement system
B. Secondary endothelial changes as a result of bacterial lesion
C. Secondary circulation insufficiency with retained systolic function as a result
of peripheral vasoconstriction
D. Secondary reflex vasodilatation as a result of lowered cardiac output
E. Reflex vagus stimulation with lowered cardiac output

3. A 60 y.o. man complains of sense of heaviness in the region of scrotum.


Objectively: scrotum edema in the left part. Testicle is of normal size, but
there is a soft, scrotum limited edema over it that can be pressed and
disappears when the patient lies down. What is the preliminary diagnosis?
A. Varicocele
B. Ectopic testicle
C. Varicosity of subcutaneous veins
D. Inguinal hernia
E. Inguinal lymphadenopathy

4. A patient was delivered to a surgical department after a road accident with a


closed trauma of chest and right-sided rib fracture. The patient was diagnosed
with right-sided pneumothorax, it is indicated to perform drainage of pleural
cavity. Pleural puncture should be made in:
A. In the 2nd intercostal space along the middle clavicular line
B. In the 7th intercostal space along the scapular line
C. In the point of the greatest dullness on percussion
D. In the projection of pleural sinus
E. In the 6th intercostal space along the posterior axillary line

5. A 40 year old woman has changes of mammary gland. What are the most
often symtomps that precede the malignization?
A. Skin induration with inverted nipple
B. Bloody discharges from the nipple
C. Pure discharges from the nipple
D. Painful movable induration
E. Painless movable induration

6. Name a statistical observation unit for determination of influence amount of


bloodsugar on the healing of wounds surface in a postoperative period:
A. The patient in a postoperative period
B. Blood analysis
C. The patient who was discharged on an after-care
D. The patient who has a wound surface
E. An amount of bloodsugar

7. A 20-year-old patient was delivered to the hospital in summer from the street
with haemorrage from the brachial artery. First medical aid involved
application of a tourniquet for provisional arrest of bleeding. What is the
maximal exposure of the tourniquet?
A. 120 minutes
B. 60 minutes
C. 180 minutes
D. 15 minutes
E. 30 minutes

8. A 9 y.o. child with diagnosis "chronic tonsillitis" stands dispanserization


control. Within 1 year of observation there was one exacerbation of disease.
Physical condition is satisfactory. The general state is not infringed. Define
group of health:
A. III (a)
B. III (c)
C. I-st
D. II-d
E. III (b)

9. A 38 year old man, previously in good health, suddenly develops severe


abdominal pain radiating from the left loin to groin and accompanied by
nausea, perspiration and the need for frequent urination. He is restless,
tossing in bed but has no abnormal findings. The most likely diagnosis is:
A. Leftsided renal colic
B. Sigmoid diverticulitis
C. Retroperitoneal haemorrhage
D. Torsion of the left testicle
E. Herpes zoster

10. A 40 year old woman has a self-detected hard breast mass. The procedure of
choice for confirming the diagnosis is:
A. Excision biopsy
B. Thermography
C. Aspiration biopsy with cytology
D. Ultrasonography
E. Mammography

11. During investigation for chronic, severe, epigastric pain, a 40 year old
alcoholic man is found to have multiple areas of narrowing alternating with
dilatation ("chain of lakes" appearance) of the main pancreatic duct. The
operation of choice is:
A. Lateral pancreaticojejunostomy
B. Sphincterotomy
C. Distal pancreaticojejunostomy
D. Distal pancreatectomy
E. Total pancreatectomy

12. The treatment of choice for duodenal obstruction caused by secondary


duodenal hematoma that developed a few days after blunt abdominal injury
is:
A. Nasogastric decompression and parenteral alimentation
B. Immediate exploration
C. Tube duodenostomy
D. Retrocolic gastrojejunostomy
E. Duodenojejunostomy

13. An anestesiologist gives narcosis to the patient, he uses a non-reversive


contour. Anesthetic is halothane. Air temperature in the operation room is
21°C, humidity 50%, level of noise 30 dB. What occupational hazard is the
principal one under these conditions?
A. Air pollution with anesthetic
B. Improper occupational microclimate
C. Mental overfatigue
D. Compelled working pose
E. High level of noise

14. A patient suddenly felt an acute chest pain irradiating to the left arm.
Objectively: the patient is excited, with pale skin. Breathing rate - 38/min, AP
- 180/110 mm Hg. Later the patient lost consciousness and fell down. Pulse on
the great vessels was absent, the pupils were equally dilated. What is the most
likely diagnosis?
A. Clinical death
B. Coma
C. Disorder of the cerebral circulation
D. Heart attack
E. Agonal state

15. A 37-year-old patient has come to clinic being wounded in the area of
umbilicus an hour ago.On physical exam, there is a prick and cut painful
wound of about 0,5x1 cm around umbilicus on the abdominal skin with slight
bleeding.How would you help this patient?
A. Laparotomy, abdominal cavity organs inspection. Primary surgical
processing of the wound
B. Aseptic bandage
C. Suture on the wound
D. Drainage of the wound with rubber strip
E. Inspection of the wound with canal probe

16. If a child has adherent fingers on his right hand, then what will be your
diagnosis?
A. Syndactyly
B. Ectrodactyly
C. Ectromelia
D. Polydactyly
E. Macrodactyly

17. A 34-year-old patient was bitten by a dog 3 hours ago.There is a wound by dog
teeth without bleeding in the left arm. What surgical help would you provide
to this patient?
A. Clean wound with detergent water and apply anti-septic
B. Aseptic bandage
C. Complete suture of the wound
D. Incomplete suture of the wound
E. Cream bandage

18. Purulent mediastinitis is diagnosed at a 63-year-old patient. What diseases


from the stated below CANNOT cause the purulent mediastinitis?
A. Cervical lymphadenitis
B. Deep neck phlegmon
C. Perforation of the thoracic part of the easophagus
D. Iatrogenic injury of the trachea
E. Perforation of the cervical part of the easophagus

19. A 63 year old patient was diagnosed with purulent mediastinitis. What of the
below listed diseases are NOT the cause of purulent mediastinitis?
A. Cervical lymphadenitis
B. Perforation of the cervical part of the oesophagus
C. Iatrogenic injury of the trachea
D. Perforation of the thoracic part of the oesophagus
E. Deep neck phlegmon

20. A 36 year old patient was diagnosed with right-sided pneumothorax. What
method of treatment is indicated to the patient?
A. Surgical treatment: drainage of the pleural cavity
B. Symptomatic therapy
C. Thoracotomy
D. Pleural puncture
E. Antiinflammation therapy

21. The diagnosis of a right-sided pneumothorax is made to a 36- year-old


patient. What method of treatment is indicated to the patient?
A. Drainage of the pleural cavity
B. Symptomatic therapy
C. Antiinflammation therapy
D. Pleural puncture
E. Thoracotomy

22. A 16 year old patient with complaints of frequent pain in the abdomen was
diagnosed with melanoma, examination revealed also pigmentation of the
mucosa and skin, polyp in the stomach and large intestine. It is know that the
patients mother has an analogous pigmentation and has been often treated for
anemia. What disease is suspected?
A. Peytz-Egerss polyposis
B. Chrons disease
C. Adolescent polyposis
D. Hirschprungs disease
E. Tuberculosis of the intestine

23. A 41 year old patient was admitted to the intensive care unit with
haemorrhagic shock due to gastric bleeding. He has a history of hepatitis B
during the last 5 years. The source of bleeding are esophageal veins. What is
the most effective method for control of the bleeding?
A. Introduction of obturator nasogastric tube
B. Hemostatic therapy
C. Administration of plasma
D. Operation
E. Intravenous administration of pituitrin

24. It is suspected that a 34 year old patient has an abscess of Douglas pouches.
What diagnostic method is to be chosen?
A. Digital examination of rectum
B. R-scopy of abdominal cavity
C. Laparoscopy
D. Rectoromanoscopy
E. Percussion and auscultation of stomach
25. A patient has restrained umbilateral hernia complicated by phlegmon hernia,
it is necessary to take following actions:
A. Herniotomy by Mayo-Sapezhko
B. Herniotomy by Lekser
C. Herniotomy by Grenov
D. Herniotomy by Mayo
E. Herniotomy by Sapezhko

26. A 52 year old man has recurrent transient ischemic attacks. Auscultation of
the carotid arteries detected murmur. What diagnostic method is to be applied
in the first place?
A. Ultrasound dopplerography
B. MRI of the brain
C. Electroencephalography
D. Cerebral angiography
E. CT of the brain

27. For the persons who live in a hot area after an accident at a nuclear object, the
greatest risk within the first decade is represented by cancer of:
A. Thyroid gland
B. Lungs
C. Reproduction system organs
D. Skin
E. Breast

28. A 10 year old boy complains about pain in his left eye and strong photophobia
after he had injured his left eye with a pencil at school. Left eye examination:
blepharospasm, ciliary and conjunctival congestion, cornea is transparent,
other parts of eyeball have no changes. Visus 0,9. Right eye is healthy, Visus
1,0. What additional method would you choose first of all?
A. Staining test with 1% fluorescein
B. Gonioscopia
C. Cornea sensation-test
D. X-ray examination of orbit
E. Tonometria

29. A patient had disorder of nasal respiration, mucopurulent discharges from


nose, headache for 3 weeks. At anterior rhinoscopy in middle nasal meathus
the stria of pus, edema, hyperemia of the mucosa of the nose have been
determined. What diagnostic method is necessary to administer first of all?
A. An X-ray of paranasal sinuses
B. General blood test
C. Bacteriology analysis of the nasal mucous
D. CT of a skull
E. Punction of the maxillar sinus

30. A patient with acute purulent otitis media complicated by mastoiditis was
admitted to a hospital. Roentgenogram of mastoid processes showed the
shadiowing of the cellular system on the lesion, absence of bone septa was
present. What are the necessary therapeutic actions at the second stage of
mastoiditis?
A. Mastoidotomy
B. Paracentesis of the drum
C. Tympanoplasty
D. Cateterization of the Eustachian tube
E. Radical operation on the middle ear

31. A 35-year-old woman was admitted to resuscitation department in asthmatic


status. What is the most trustworthy criterion of breath effectiveness?
A. PaCO2 and PaO2
B. Respiration rate
C. Respiratory volume
D. Minute respiratory volume
E. Determination of “dead” space

32. During dynamic investigation of a patient the increase of central venous


pressure is combined with the decrease of arterial pressure. What process is
proved by such combination?
A. Developing of cardiac insufficiency
B. Depositing of blood in venous channel
C. Presence of hypervolemia
D. Increase of bleeding speed
E. Shunting

33. A 22 year old patient was admitted to trauma center with complaints of pain
in the left ankle joint that was getting worse during moving and weight
bearing. On the clinical examination it was found that the patient had the
closed fracture of medial malleolus without displacement. In which position
should the foot be fixed in plaster cast?
A. At right angle with varus positioning of the foot
B. In position of planter flexion of foot
C. In position of supination
D. In position of dorsal flexion of foot
E. In position of pronation
34. A 74 y.o. patient has been ill with benign prostate hyperplasy for the last 5
years. 4 days ago, after alcochol consumption, there was an acute retention of
urination. At the pre-admission stage his urinary bladder was catheterized
with metallic catheter. Examination revealed: right epididymis is enlarged,
thick and painful, there are purulent discharges from urethra. What way of
emergency care must be chosen?
A. Trocar or open epicystostomy
B. Placing of intraprostatic stent
C. Introduction of permanent urethral catheter
D. Transuretral resection or prostatectomy
E. Microwave thermotherapy of prostate

35. A rounded well-defined shadow was found in the costo-vertebral angle on the
chest roentgenogram of an otherwise healthy 9 year old girl. Make a
preliminary diagnosis:
A. Ganglioneuroma
B. Ganglioneuroblastoma
C. Sarcoma of the vertebra
D. Sympatogonioma
E. Sympatoblastoma

36. A 4 y.o. child attends the kindergarten. Complains of poor appetite, fatigue.
Objective examination: skin and mucous membrane are pale, child is asthenic.
In the hemogram: hypochromatic anemia 1st, leucomoide reaction of the
eosinophile type. What pathology must be excluded first of all?
A. Helminthic invasion
B. Hypoplastic anemia
C. Atrophic gastritis
D. Duodenal ulcer
E. Lymphoprolipherative process

37. A 33 year old male patient was brought to Emergency Department with the
signs of cardiovascular collapse: BP - 60/30 mm Hg, Ps - 140 bpm, the skin is
pale and moist, diuresis 20 ml/h, Hb - 80 g/l, red blood cell count -
2,5×1012/l. The reduction of blood volume averages:
A. 30-40%
B. 25-30%
C. 15-20%
D. 10-15%
E. 20-25%
38. A 19 year old girl was admitted to emergency department: unconsciousness,
cyanosis, myotic pupils are present, superficial breathing is 12/min. BP is
90/60 mm Hg, Ps- 78/min. Choose the action necessary in this clinical
situation:
A. Controlled respiration
B. Oxygen inhalation
C. Cordiamine injection
D. Caffeine injection
E. Gastric lavage

39. What preparations are used for prevention of fungal infection?


A. Fluconozol, Orungol, Nisoral
B. Cytosar, Cormyctin, Lomycitin
C. Rubomycin, Bleomycin, Mytomycin C
D. Captopril, Enalapril
E. Isoniazid, Ftibazid, Pyrazinamid

40. What developes most often after accidental intake of Hydrochloric acid?
A. Cardiac insufficiency
B. Deyladss syndrome
C. Acute pancreatitis
D. Cushings syndrome
E. Kutlings syndrome

41. 30 y.o. woman, had mild trauma of 5th finger of the left hand 15 days ago. She
has treated her self at home. She presents to the hospital due to deterioration
of the condition and temperature rise. Objectively: hyperemia and swelling on
the ventral surface of finger. Restricted movements of the finger. X-ray of the
left hand: an early stage of osteomyolitis of the fifth finger could not be
excluded. The diagnosis: panaris of 5th finger of the left hand. What form of
panaris has occurred in the patient?
A. Bony
B. Paronychia
C. Joints type
D. Tendon type
E. Hypodermic

42. A 36 y.o. patient is diagnosed with right sided pneumothorax. What method of
treatment is indicated to the patient?
A. Surgical drainage of the pleural cavity
B. Symptomatic therapy
C. Antiinflammation therapy
D. Pleural puncture
E. Thoracotomy

43. A 28 year old woman was admitted to the emergency room with a slightly
reddened, painful "knot" 8 cm above the medial malleolus. Examination in the
standing position demonstrates a distended vein above and below the mass.
There are no other abnormalities on physical examination. The most likely
diagnosis is:
A. Superficial venous thrombosis
B. Insect bite
C. Early deep vein thrombosis
D. Cellulitis
E. Subcutaneous hematoma

44. A 30 year old man complains of acute pain in his right ear, hearing loss, high
temperature for three days. Objectively: right ear whispering language - 0,5
mm, external ear is intact, otoscopically - eardrum protrusion, hyperemia and
swelling, loss of landmarks. What disease is it?
A. Acute purulent otitis media
B. Chronic purulent otitis media
C. Eustachian tube disfunction
D. Acute mastoiditis
E. Chronic secretory otitis media

45. A 22 y.o. man complains of acute throat pain, increasing upon swallowing
during 3 days. Body temperature 38,3°C, neck lymph nodules are slightly
enlarged and painful. Pharyngoscopically - tonsilar hyperemia, enlargement
and edema, tonsils are covered by round yellow fibrinous patches around
crypts openings. Beta-haemolytic streptococcus in swab analysis. What is the
diagnosis?
A. Acute membranous tonsilitis
B. Pharyngeal diphtheria
C. Acute follicular tonsilitis
D. Infectious mononucleosis
E. Pharyngeal candidosis

46. A patient is staying in the hospital with the diagnosis of abdominal typhus.
During the 3-d week from the beginning of the disease the patient stopped
keeping diet and confinement to bed. As a result the body temperature and
rapid pulse decreased and melena appeared. What kind of complications
should we think about first of all?
A. Intestinal haemorrhage
B. Meningitis
C. Hepatite
D. Nephroso-nephritis
E. Thrombophlebitis

47. A 45-year-old woman, mother of four children, comes to the emergency room
complaining of a sudden onset of the epigastric and right upper quadrant
pain, radiating to the back, accompanied by vomiting. On examination,
tenderness is elicited in the right upper quadrant, bowel sounds are decreased,
and laboratory data shows leukocytosis, normal serum levels of amylase,
lipase, and bilirubin. The most likely diagnosis is:
A. Acute cholecystitis
B. Perforated peptic ulcer disease
C. Sigmoid diverticulitis
D. Acute pancreatitis
E. Myocardial infarction

48. During an operation for presumed appendicitis the appendix was found to be
normal; however, the terminal ileum is evidently thickened and feels rubbery,
its serosa is covered with grayish-white exudate, and several loops of
apparently normal small intestine are adherent to it. The most likely diagnosis
is:
A. Crohns disease of the terminal ileum
B. Ulcerative colitis
C. Perforated Meckels diverticulum
D. Ileocecal tuberculosis
E. Acute ileitis

49. A 50 year old woman with a 2-year history of mild, diffuse, tender thyroid
enlargement complains of 10 pound weight gain and fatigue. What is the most
probable diagnosis?
A. Hashimotos thyroiditis
B. Riedels thyroiditis
C. Suppurative thyroiditis
D. Papillary thyroid carcinoma
E. Subacute thyroiditis

50. A severely traumatized patient who has been receiving prolonged parenteral
alimentation develops diarrhea, mental depression, alopecia and perioral and
periorbital dermatitis. Administration of which of the following trace elements
is most likely to reverse these complications?
A. Zinc
B. Copper
C. Selenium
D. Iodine
E. Silicon

51. A 38 y.o. woman was hospitalized to the surgical unit with vomiting and acute
abdominal pain irradiating to the spine. On laparocentesis hemmorhagic fluid
is obtained. What disease should be suspected?
A. Acute pancreatitis
B. Perforated gastric ulcer
C. Acute appendicitis
D. Renal colic
E. Acute enterocolitis

52. The 67 y.o. patient had 5 recurrent fractures of the lower extremities without
considerable cause within 5 years. O-shaped deformity of the legs in the knee
joints has appeared. The skull, pelvis and lower extremities X-Ray shows the
thickening of flat bones. In the long bones there is a hyperostosis along the
bone axis.The blood test has not revealed any inflammation activity. Serum
calcium is normal. What disease do you consider in this case?
A. Pagets disease
B. Hyperparathyoid dystrophy
C. Myeloma
D. Mottled disease (marble disease)
E. Chronic osteomyelitis

53. A 33 y.o. patient was admitted to the reception room of the Central District
Hospital. He complains of a severely painful swelling localized on posterior
neck, fever up to 38,4°C and general weakness. In anamnesis: diabetes
mellitus within 5 years. On physical examination on the posterior neck surface
there is an infiltrate elevated above surrounding skin. The tissues affected by
swelling are tense and blue reddish discoloration in central area. There are
also several purulent necrotic pustules which are connected with each other
and form a large skin necrosis. A thinned necrotic skin of this swelling has
holes looking like sieve, pus discharges through out. What disease should a
doctor consider first of all?
A. Carbuncle
B. Skin abscess
C. Acute skin cellulitis
D. Furuncle
E. Carbuncle associated with anthrax

54. A 19 y.o. man was admitted to the reception department in 20 minutes after
being wounded with the knife to the left chest. The patient is confused. The
heart rate is 96 bpm and BP- 80/60 mm Hg. There are the dilated neck veins,
sharply diminished apical beat and evident heart enlargement What kind of
penetrative chest wound complications has developed in patient?
A. Pericardium tamponade
B. Valve-likes pneumothorax
C. Open pneumothorax
D. Massive hemothorax
E. Closed pneumothorax

55. A 35 y.o. patient complains of a difficult swallowing, pain behind the


breastbone. He can eat only liquid food. While swallowing sometimes he has
attacks of cough and dyspnea. Above mentioned complaints are progressing.
It is known that the patient has had a chemical burn of esophagus one month
ago. What complication does the patient have?
A. Corrosive esophagitis and stricture
B. Esophageal diverticula
C. Esophagitis
D. Cardiac achalasia
E. Cardiac insufficiency

56. Survey radiograph of chest of a 62 year old smoker who often suffers from
"pneumonias" showed a triangle shadow in the right lung, its vertex is
pointing towards the lung root. It also showed deviation of heart and
mediastinum shadows towards the lesion. What is the most probable
diagnosis?
A. Cenral cancer of lung
B. Lung cyst
C. Lung abscess
D. Peripheral cancer of lung
E. Atelectasis

57. A patient operated for acute paraproctitis undergoes antibacterial and


detoxification therapy, the local course of the disease has the positive
dynamics. Since the operation the patient has had chills, pyrexia, tachycardia,
euphoria for five days. The doctor suspected sepsis. What study will confirm
the diagnosis?
A. Blood culture for a pathogen
B. Determining the rate of average-weight molecules
C. Liver ultrasound
D. X-ray of lungs
E. Determining the rate of microbial contamination of wound

58. A 44-year-old patient has been admitted to a hospital with complaints of dull,
aching pain in the left lumbar region, the admixture of pus in the urine.
Examination revealed a grade II staghorn calculus on the left. What method of
treatment is indicated for this patient?
A. Surgery
B. Distance lithotripsy
C. Ascending litholysis
D. Conservative therapy
E. Contact lithotripsy

59. An emergency team has delivered to a hospital an unconscious patient found


lying in the street in winter. Objectively: the patient is pale, with superficial
respiration; bradycardia with heartrate 54/min, to- 35°C. AP- 100/60 mm Hg.
Palpation of chest and abdomen revealed no peritoneal symptoms. There is a
smell of alcohol from the patients mouth. What is the most likely diagnosis?
A. Hypothermia
B. Apparent death
C. Acute cardiovascular insufficiency
D. Frostbite of trunk and extremities

60. A 48-year-old patient got a job-related injury of a hypodermic varicose vein on


his shin that was accompanied by the intensive phleborrhagia. Choose the
optimal variant of first aid:
A. Pressure bandage and limb strapping
B. Maximal limb flexion in knee joint
C. Application of Esmarchs tourniquet beneath the injury
D. Application of Esmarchs tourniquet above the injury
E. Occlusion of femoral artery in a typical place

61. A 24-year-old patient got a puncture injury below the Pouparts ligament
accompanied by intense arterial bleeding. The best method to temporarily
stop the bleeding in the patient would be:
A. Compression band
B. Maximum limb bending
C. Esmarchs tourniquet
D. Compressing a blood vessel with a clamp
E. Wound suturing

62. While making a round, a doctor had noticed the edema of the right shoulder of
a 26-day-old child with diagnosis of "umbilical sepsis". Active movements in
the shoulder joint were absent, right hand was hanging down. Mother stated
that her childs condition had worsened for the last 24 hours what resulted in
childs refusal to be breast-fed, restlessness, weeping while swaddling, rise in
body temperature up to 38,8°C. What is the most probable preliminary
diagnosis?
A. Epiphysial osteomyelitis of the humerus
B. Fracture of the clavicle
C. Traumatic brachioplexitis
D. Fracture of the humerus
E. Phlegmon of the shoulder

63. A 65-year-old patient complains of dull pain in the rectum during and after
defecation, discharge of mucus and small amount of blood mixed up with
mucus and faeces. The discharged blood is of dark red color, sometimes with
small amount of clots. The patient is sick for 8 months, has lost some weight.
On digital examination, there is a round constriction of the rectum with
infiltrate at a height of 4-5 cm from the anus. What is the most probable
diagnosis?
A. Cancer of the medium-ampullar section of the rectum
B. Crohns disease
C. Non-specific ulcer colitis
D. Cicatricial stenosis of the rectum
E. Chronic paraproctitis

64. A 52 y.o. patient fell from 3 m height on the flat ground with the right lumbar
area. He complains of pain in this area. There is microhematuria in the urea.
Excretory urography revealed that kidneys functioning is satisfactory. What is
the most probable diagnosis?
A. Kidneys contusion
B. Paranephral hematoma
C. Kidneys abruption
D. Subcapsular kidneys rupture
E. Multiple kidneys ruptures

65. A female patient has been suffering from pain in the right subcostal area,
bitter taste in the mouth, periodical bile vomiting for a month. The patient put
off 12 kg. Body temperature in the evening is 37,6°C. Sonography revealed
that bile bladder was 5,5?2,7 cm large, its wall - 0,4 cm, choledochus - 0,8 cm
in diameter. Anterior liver segment contains a roundish hypoechoic formation
up to 5 cm in diameter and another two up to 1,5 cm each, walls of these
formations are up to 0,3 cm thick. What is the most likely diagnosis?
A. Alveolar echinococcus of liver
B. Liver abscess
C. Paravesical liver abscesses
D. Cystous liver cancer
E. Liver cancer
66. A 36-year-old woman applied to doctor with complaints of the enlargement of
the right mammary gland. On physical exam: the right mammary gland is
enlarged, hardened; nipple with areola is pasty and hydropic, looks like
"lemon peel". There palpates a lymph node in the right axilla, 1,5 cm in
diameter, dense consistence, not mobile. What is the most probable
diagnosis?
A. Hydropic and infiltrative cancer of the mammary gland
B. Diffusive mastopathy
C. Erysipelatouslike cancer of the mammary gland
D. Pagets cancer
E. Acute mastitis

67. A 43 year old patient had cholecystectomy 6 years ago because of chronic
calculous cholecystitis. Lately he has been suffering from pain in the right
subcostal area and recurrent jaundice. Jaundice hasnt gone for the last 2
weeks. Stenosing papillitis 0,5 cm long has been revealed. What is the best
way of treatment?
A. To perform endocsopic papillosphincterotomy
B. To perform external choledoch drainage
C. To treat conservatively: antibiotics, spasmolytics, antiinflammatory drugs
D. To perform transduodenal papillosphincterotomy
E. To perform choledochoduodenostomy

68. A 49-year-old male patient who had been scheduled for a surgery for gastric
cancer underwent preoperative infusion therapy. Up to 3,0 liters of liquid was
introduced into the right cubital vein. The following day, he felt a dragging
pain in the right shoulder. Objectively: on the inner surface of the shoulder
there is a longitudinal hyperemic zone, edema of skin, a tender cord. What
complication occurred in the patient?
A. Acute thrombophlebitis
B. Venepuncture and edema of paravenous tissue
C. Acute lymphangitis
D. Paravenous tissue phlegmon
E. Paravenous tissue necrosis

69. A 54-year-old patient applied with complaints of acute pain in the shoulder-
joint. 10 minutes earlier he fell in the street with his arms stretched. Doctor of
the traumatological unit noticed an acute deformation of the right shoulder-
joint, the impossibilty of active movement in the right extremity.
Roentgenologically: uncoincidence of articulating surfaces. What is the most
probable diagnosis?
A. Dislocation of the clavicle
B. Contusion of the humerus
C. Dislocation of the humerus
D. Dislocation of the scapula
E. Fracture of the humerus

70. A 62-year-old patient has been delivered to the surgical department with
complaints of sudden pain in the umbilical region irradiating to the back and
groin, which was accompanied by a syncope. Objectively: the patient is in
grave condition, umbilical region is tender on palpation, bowel sounds are
diminished. AP drop is present. USI reveals: free fluid in the abdomen,
thickening of the wall of the abdominal aorta. The most likely diagnosis is:
A. Rupture of abdominal aortic aneurism
B. Acute pancreatitis
C. Stomach ulcer perforation
D. Peritonitis
E. Acute appendicitis

71. Parents of a 2-year-old boy applied to clinic complaining of right testicle


absence in the scrotum of a boy. While examining the boy, hypoplasia of the
right half of the scrotum was revealed, absence of the testicle. Testicle is
miniaturized, it palpitates along the inguinal canal but it could not be moved
down to scrotum. What is the most probable diagnosis?
A. Right-sided cryptorchism, inguinal form
B. Ectopia of the right testicle, pubic form
C. Left-sided monorchism
D. Retraction of the right testicle (pseudocryptorchism)
E. Right-sided cryptorchism, abdominal form

72. A 62-year-old patient complains of the pain behind the sternum, bad passing
of solid and liquid food, bad breath, increased salivation weight, loss of 15 kg
during the period of 2 months. Appetite is retained. On physical exam: face
features are sharpened. The skin is pale, with sallow tint, its turgor is
decreased. The liver is not enlarged. Blood Hb - 86g/L. Gregersen reaction is
positive. What kind of pathology caused the given clinical situation?
A. Esophagus cancer
B. Chronic non-specific esophagitis
C. Cicatricial constriction of esophagus
D. Benign growth of esophagus
E. Achalasia of esophagus

73. A patient suffers from suddenly arising crampy pain in the right loin area. 2
hours after the pain had started, hematuria took place. Loin X-ray: no
pathological shadows. Ultrasound: pyelocaliectasis on the right, the left part is
normal. What is the most probable diagnosis?
A. Stone of the right kidney, renal colic
B. Tumour of the right kidney pelvis
C. Intestine invagination
D. Acute appendicitis
E. Twist of the right ovary cyst

74. A patient complains of an extremely intense pain in epigastrium. He has


peptic ulcer disease of duodenum for 10 years. The patient is in the forced
position being on the right side with legs abducted to stomach. Abdomen has
acute tenderness in the epigastrium. Guarding contraction of the abdominal
wall muscles is observed. What is the preliminary diagnosis?
A. Perforation of ulcer
B. Thrombosis of mesenteric vessels
C. Acute condition of peptic ulcer disease
D. Acute pancreatitis
E. Penetration of ulcer into pancreas

75. A 30-year-old patient complains of pain, hyperemia along subcutaneous veins,


rise in body temperature. While examining the large shin subcutaneous vein,
there is hyperemia, pain by pressing. Homanss and Lusess symptoms are
negative. What is the preliminary diagnosis?
A. Acute thrombophlebitis of subcutaneous veins
B. Lymphostasis
C. Acute ileophemoral phlebothrombosis
D. Embolism of aorta
E. Thrombosis of aorta

76. A 30-year-old man has suffered from traffic accident. Consciousness is absent.
Pulse on the carotid artery is undeterminable, no respiration. There is a wide
leather belt on mans waist. What measures are to be taken?
A. Conduct an artificial ventilation of lungs and closed-chest cardiac
massage after having released from the belt
B. Start immediate artificial ventilation of lungs and closed-chest cardiac
massage
C. Do not touch the victim until the arrival of the police
D. Put the victim with his back on the shield
E. Turn the victim to the right side

77. An emergency team deliverd a 83-year-old patient complaining of inability of


her right leg to support the body after falling on her right side. Objectively: the
patient lies on a gurney, her right leg is rotated outwards, the outside edge of
foot touches the bed. There is positive straight leg raising sign. What is your
provisional diagnosis?
A. Femoral neck fracture
B. Femoral diaphysis fracture
C. Hip joint contusion
D. Cotyloid cavity fracture
E. Hip dislocation

78. A 50-year-old patient complains of bursting pain in the left lower limb that is
getting worse on exertion, swelling in the region of shin and foot. Objectively:
left shin and foot are doughy, skin of the lower shin is indurated and has a
bronze tint, subcutaneous veins are dilated, there is an ulcer with necrotic
masses. What is the most likely diagnosis?
A. Postthrombophlebitic syndrome
B. Gangrene of the lower extremity
C. Acute arterial thrombosis
D. Chronic arterial insufficiency
E. Deep vein thrombosis of the lower limbs

79. 3 days ago a 29-year-old patient presented with pulsating pain in the region of
rectum, fever, general weakness. Objectively: local tenderness in the anal
region in the 6 oclock position. Digital investigation of rectum revealed a
painful infiltration reaching the pectinate line. What is the most likely
diagnosis?
A. Acute periproctitis
B. Acute haemorrhoids
C. Acute prostatitis
D. Rectum tumour
E. Acute anal fissure

80.A 65-year-old patient complains of gradual worsening of the left eye vision
during 10 months. On physical examination: acuty of vision of the left eye is
0,01, not correctable. The eye is quiet, pupil of the eye is grayish, reflex from
the eye-ground is absent. Intraocular pressure is 18 mm/Hg. What is the most
probable preliminary diagnosis?
A. Senile cataract
B. Disorder of blood circulation in retina vessels
C. Open-angle glaucoma
D. Leukoma of the cornea
E. Exfoliation of the retina

81. A 36 y.o. patient was admitted to the hospital with sharp pain in substernal
area following occasional swallowing of a fish bone. On esophagoscopy the
foreign body wasnt revealed. The pain increased and localized between
scapulas. In a day temperature elevated, condition became worse, dysphagia
intensified. What complication has developed?
A. Perforation of esophagus with mediastinitis
B. Obstruction of esophagus
C. Esophageal hemorrhage
D. Pulmonary atelectasis
E. Aspirative pneumonia

82. A 19 year old patient was admitted to a hospital with acute destructive
appendicitis. He suffers from hemophilia of B type. What antihemophilic
medications should be included in pre- and post-operative treatment plan?
A. Fresh frozen plasma
B. Native plasma
C. Dried plasma
D. Cryoprecipitate
E. Fresh frozen blood

83. On the 4th day after recovering from a cold a patient was hospitalized with
complaints of solitary spittings of mucoid sputum. On the 2nd day there was a
single discharge of about 250 ml of purulent blood-streaked sputum.
Objectively: the patients condition is moderately severe. Respiratory rate - 28-
30/min, Ps- 96 bpm, AP- 110/70 mm Hg. Respiration above the left lung is
vesicular, weak above the right lung. There are moist rales of different types
above the lower lobe and amphoric breath near the angle of scapula. What is
the most likely diagnosis?
A. Acute pulmonary abscess
B. Acute focal pneumonia
C. Pyopneumothorax
D. Pleural empyema
E. Exudative pleuritis

84. A 24 y.o. male patient was transferred to the chest surgery department from
general surgical department with acute post-traumatic empyema of pleura. On
the X-ray: wide level horizontal of fluid on the right. What method of
treatment should be prescribed?
A. Punction and drainage of pleural cavity
B. Pneumoectomy
C. Lobectomy
D. Thoracoplasty
E. Decortication of pleura

85. A patient complains of intense pressing pain in the pharynx, mainly to the
right, impossibility to swallow even liquid food. The illness started 5 days ago.
The patients condition is grave. Body temperature - 38,9°C, speech is difficult,
voice is constrained, difficulties in opening the mouth. Submaxillary glands to
the right are painful, enlarged. What is the most probable diagnosis?
A. Peritonsillar abscess
B. Vincents disease
C. Phlegmonous tonsillitis
D. Diphtheria
E. Pharyngeal tumour

86. In a 65 y.o. female patient a tumor 13x8 cm in size in the umbilical area and
above is palpated, mild tenderness on palpation, unmovable, pulsates. On
ausculation: systolic murmur. What is the most probable diagnosis?
A. Abdominal aortic aneurism
B. Arterio-venous aneurism
C. Stomach tumor
D. Tricuspid valve insufficiency
E. Mitral insufficiency

87. An hour before an elective surgery, a 56-year-old patient of the surgical


department got a dramatic increase in blood pressure, tachycardia, hand
tremor. The patient is confused, anxious, depressed, fearful, is pessimistic
about the operation outcome, refuses the surgery. What tactics should be
chosen by a surgeon?
A. Start the surgery after correction of blood pressure
B. Predict the psychological state of the patient
C. Organize monitoring of the patient by his family members
D. Organize monitoring of the patient by medical personnel and mental health
counselor
E. Isolate the patient

88. A patient with suspicion of pelvic bones fraction is under examination


conducted by the doctor who presses alae ilii in medial direction with his both
hands. What causes painful syndrome of the patient?
A. Disorder of continuity of the pelvic ring
B. Fracture of the sacrum
C. Retroperitoneal hematoma
D. Fracture of sciatic bones
E. Traumatic injury of the intrapelvic organs

89. A 30-year-old man was injured in a fire and got thermic burns of III-A and
III-B degree that affected 20% of the total skin. AP - 110/70 mm Hg, heart rate
-120/min. What transfusion means should be used for blind infusion before
transportation?
A. Saline solutions
B. Albumin
C. 10% glucose solution
D. Polyglycine
E. Fresh frozen plasma

90. A 48-year-old male patient complains of pain in the lower extremities,


especially when walking, intermittent claudication, numbness in the fingers,
cold extremities, inability to walk more than 100 meters. Sleeps with his leg
lowered. The patient has been a smoker since he was 16, abuses alcohol, has a
history of CHD. The left lower extremity is colder than the right one, the skin
of extremities is dry, pedal pulse cannot be palpated, femoral pulse is
preserved. What is the most likely diagnosis?
A. Obliterating endarteritis
B. Raynauds disease
C. Deep thrombophlebitis
D. Diabetic angiopathy
E. Leriche syndrome

91. A 24 y.o. woman consulted a doctor about continued fever, night sweating.
She lost 7 kg within the last 3 months. She had casual sexual contacts.
Objectively: enlargement of all lymph nodes, hepatolienal syndrome. Blood
count: leukocytes - 2,2×109/L. What disease can be suspected?
A. HIV-infection
B. Infectionous mononucleosis
C. Chroniosepsis
D. Lymphogranulomatosis
E. Tuberculosis

92. Development of chronic venous insufficiency of lower extremities depends on


the functional condition of so-called musculovenous pump. This term refers to
the following group of muscles:
A. Shin muscles
B. Abdominal wall muscles
C. Thigh muscles
D. Foot muscles
E. Buttock region muscles

93. A 27-year-old patient was brought to clinic with a crashed wound of the
posterior surface of the right shin in 2 hours after the accident happened.
During surgical treatment pieces of dirty clothes and gravel were removed.
What actions from the given below are the decisive ones to prevent an
anaerobic infection?
A. Radical surgical treatment
B. Introduction of the medical dose of the anti-gangrene serum
C. Hyperbaric oxygenation
D. Rational antibiotic therapy
E. Introduction of the preventive dose of an anti-gangrene serum

94. A 32-year-old patient lives in an area endemic for echinococcosis. In the last 6
months he reports of pain in the right subcostal area, fever. He is suspected to
have liver echinococcosis. What study would be the most informative in this
case?
A. USI
B. Liver scanning
C. Biochemical laboratory examination
D. Survey radiography of abdominal cavity
E. Angiography

95. In morgue there are dead bodies with the following causes of death:
electrotrauma; rupture of the spleen with acute anemia. There is one unknown
person; one ethyl alcohol poisoned person and one drowned man. What dead
body should the blood group be determined for?
A. All dead bodies of the unknown persons
B. Body of person with a sudden death
C. Body of drowned man
D. Body of poisoned person
E. Body of person with internal hemorrhage

96. A 50 y.o. patient was admitted to the hospital with complaints of blood in
urine. There was no pain or urination disorders, hematuria has lasted for 3
days. Objectively: kidneys are not palpable, suprapubic region has no
pecularities, external genitals have no pathology. Rectal examination revealed:
prevesical gland is not enlarged. Cytoscopy revealed no changes. What disease
would you think about first of all?
A. Cancer of kidney
B. Tuberculosis of urinary bladder
C. Kidney dystopy
D. Necrotic papillitis
E. Varicocele

97. A patient complains of nycturia, constant boring pain in perineum and


suprapubic region, weak urine jet, frequent, obstructed and painful urination.
He has been ill for several months, pain in perineum appeared after getting
problems with urination. Rectal examination revealed that prostate is
enlarged (mostly owing to the right lobe), dense, asymmetric, its central
sulcus is flattened, the right lobe is dense, painless, tuberous. What disease
can it be?
A. Cancer of prostate
B. Chronic congestive prostatitis
C. Urolithiasis, stone of the right lobe of prostate
D. Prostate sclerosis
E. Prostate tuberculosis

98. The patient complains of a painful swelling in the chin region, malaise,
headache. Examination reveals an acutely inflamed cone-shaped dense node.
The skin over it is tense, red. In the center of the node there is an ulcer with
overhanging edges and a necrotic core of a dirty-green colour. Submandibular
lymph nodes on the right are enlarged and painful. What is the most likely
diagnosis?
A. Furuncle
B. Parasitic sycosis
C. Carbuncle
D. Tuberculosis
E. Tertiary syphilis (gummatous form)

99. On the 5th day after a surgery for colon injury a patient complains of bursting
pain in the postoperative wound, weakness, drowsiness, headache, fever up to
40°C. Objectively: the skin around the wound is swollen, there is gas
crepitation. The wound discharges are scarce foul-smelling, of dark-gray
colorl. What is the most likely diagnosis?
A. Anaerobic clostridial wound infection
B. Postoperative wound infection
C. Abscess
D. Erysipelas
E. Phlegmon

100. A childrens surgical unit admitted a 1-month-old boy who had been
prenatally diagnosed with the left-sided pyelectasis. Such studies as drip
infusion urography, cystography and USI allowed to reveal initial
hydronephrosis. There is no information confirming the secondary
pyelonephritis. What tactics of this patient management is most advisable?
A. 6-month surveillance
B. Anderson-Hynes operation
C. Antibacterial therapy
D. There is no need in further surveillance and treatment
E. Urgent nephrostomy
101. A 60 y.o. man complains of significant pain in the right eye,
photophobia, lacrimation, reduced vision of this eye, headache of the right
part of the head. Pain occured 2 days ago. On examination: Vis OD- 0,03,
congested injection of the eye ball, significant cornea edema, front chamber is
deep, pupil is narrow, athrophic iris, there is optic nerve excavation on the eye
fundus, intraocular pressure- 38 mm Hg. Vis OS- 0,8 unadjustable. The eye is
calm, healthy. Intraoccular pressure- 22 mm Hg. What is the most probable
diagnosis?
A. Acute glaucoma attack
B. Eye nerves neuritis
C. Maculodystrophy
D. Right eyes uveitis
E. Right eyes keratitis

102. A 54-year-old female patient has been admitted to a hospital 12 days


after the beginning of acute pancreatitis. Objectively: the patient is in grave
condition. The body temperature is hectic. Ps - 112 bpm. The abdomen is
swollen. Epigastrium palpation reveals a very painful infiltration falling in the
localization of pancreas. Abdominal cavity contains fluid. There is an edema of
the left lumbar region. In blood: WBCs – 18×109/l. What is the required
tactics of the patients treatment?
A. Surgical treatment
B. Massive antibacterial therapy
C. Peritoneal dialysis
D. Further conservative treatment
E. Increase in antienzymes

103. A 37-year-old male patient has a history of diabetes of moderate


severity. On the left side of face the patient has a carbuncle. What severe
complication might have occurred in the patient?
A. Cavernous sinus thrombosis
B. Endarteritis
C. Lymphangitis
D. Thrombophlebitis
E. Thromboembolism

104. To replace the blood loss replacement 1000 ml of the same group of
Rhesus-compatible donated blood was transfused to the patient. The blood
was conserved by sodium citrate. At the end of hemotransfusion there
appeared excitement, pale skin, tachycardia, muscles cramps in the patient.
What complication should be suspected?
A. Citrate intoxication
B. Allergic reaction
C. Citrate shock
D. Anaphylactic shock
E. Pyrogenous reaction

105. A 20 y.o. patient suddely felt ill 12 hours ago. There was pain in
epigactric area, nausea, sporadic vomiting. He had taken alcohol before. In
few hours the pain localized in the right iliac area. On examination: positive
rebound tenderness symptoms. WBC- 12,2×109/L. What is the most probable
diagnosis?
A. Acute appendicitis
B. Perforated ulcer
C. Acute cholecystitis
D. Rightside kidney colic
E. Acute pancreatitis

106. A patient, aged 58, was fishing in the winter. On return home after
some time felt some pain in the feet. Consulted a doctor. On examination: feet
skin was pale, then after rewarming became red, warm to the touch. Edema is
not significant, limited to the toes. All types of sensitivity are preserved. No
blisters. What degree of frostbite is observed?
A. I degree
B. III degree
C. II degree
D. IV degree
E. V degree

107. After an accident a patient complains of pain in the hip joint.


Objectively: the leg is in the position of flexion, adduction and internal
rotation, significantly contracted. There is elastic resistance to passive
adduction or abduction of the extremity. Major trochanter is located high
above the Roser-Nelaton line. A significant lordosis is present. What is your
provisional diagnosis?
A. Iliac dislocation of hip
B. Femoral neck fracture with a displacement
C. Inferoposterior dislocation of hip
D. Pertrochanteric fracture of hip
E. Cotyloid cavity fracture with a central dislocation of hip

108. Three weeks after acute angina the patient is still weak, inert,
subfebrile, his retromaxillary lymph nodes are enlarged. Tonsils are flabby,
stick together with arches, there are purulent plugs in lacunae. What is the
most probable diagnosis?
A. Chronic tonsillitis
B. Paratonsillitis
C. Tonsillar tumour
D. Chronic pharyngitis
E. Acute lacunar tonsillitis

109. A 43 year old patient had right-sided deep vein thrombosis of


iliofemoral segment 3 years ago. Now he is suffering from the sense of
heaviness, edema of the lower right extremity. Objectively: moderate edema of
shin, brown induration of skin in the lower third of shin, varix dilatation of
superficial shin veins are present. What is the most probable diagnosis?
A. Postthrombophlebitic syndrome, varicose form
B. Parkes-Weber syndrome
C. Acute thrombophlebitis of superficial veins
D. Acute thrombosis of right veins
E. Lymphedema of lower right extremity

110. A child undergoes in-patient treatment for acute staphylococcal


destruction of the right lung. Unexpectedly he developed acute chest pain on
the right, dyspnea, cyanosis. The right side of chest lags behind in the
respiratory act. Percussion reveals dullness in the lower parts on the right,
bandbox resonance in the upper parts. Borders of the relative cardiac dullness
are shifted to the left. What complication has most likely developed?
A. Right-sided pyopneumothorax
B. Spontaneous pneumothorax
C. Pleural empyema
D. Exudative pleuritis
E. Right lung abscess

111. A 98 y.o. male patient complains of pain in the left lower limb which
intensifies on walking, feeling of cold and numbness in both feet. He has been
ill for 6 years. On examination: pale dry skin, hyperkeratosis. Hairy covering is
poorly developed on the left shin. "Furrow symptom " is positive on the left.
Pulse on foot arteries and popliteal artery isnt palpated, on the femoral artery
its weak. On the right limb the artery pulsation is reserved. What is the most
probable diagnosis?
A. Arteriosclerosis obliterans
B. Obliterating endarteritis
C. Raynaulds disease
D. Buergers disease (thromboangiitis obliterans)
E. Hemoral arthery thombosis

112. Examination of a dead man who died from hanging revealed that
cadaver spots disappeared when pressed upon and restored after 50 seconds,
rigor mortis was moderately expressed only in the masticatory muscles and
the muscles of neck and fingers. Body temperature was 31°C. Specify the time
of death:
A. 6-7 hours
B. 8-10 hours
C. 10-18 hours
D. 1-2 hours
E. 16-24 hours

113. Examination of the corpse of a man who died from hanging reveals:
death spots disappear when pressed upon and restore after 50 seconds, rigor
mortis is moderately expressed only in the masticatory muscles as well as neck
and finger muscles, body temperature is of 31°C. The time of death:
A. 6-7 hours ago
B. 10-18 hours ago
C. 16-24 hours ago
D. 1-2 hours ago
E. 8-10 hours ago

114. After contusion of the right eye a patient complains of sudden loss of
vision with remaining light perception. Objectively: the eye is not irritated.
The cornea is transparent. Pupil reacts to light. The pupil area is black. The
fundus reflex is absent. What is the most likely cause of vision loss?
A. Hemophthalmia
B. Acute occlusion of retinal vessels
C. Optic nerve avulsion
D. Retinal detachment
E. Traumatic cataract

115. A 65-year-old patient complains of pain in the lumbar spine, moderate


disuria. He has been suffering from these presentations for about half a year.
Prostate volume is 45 cm3 (there are hypoechogenic nodes in both lobes,
capsule invasion). The rate of prostate-specific antigen is 60 ng/l. Prostate
biopsy revealed an adenocarcinoma. Which of the supplemental examination
methods will allow to determine the stage of neoplastic process in this
patient?
A. Computer tomography of pelvis
B. Excretory urography
C. Roentgenography of lumbar spine
D. Bone scintigraphy
E. Roentgenography of chest

116. X-ray picture of chest shows a density and an abrupt decrease in the
upper lobe of the right lung. The middle and lower lobe of the right lung
exhibit significant pneumatization. The right pulmonary hilum comes up to
the dense lobe. In the upper and middle parts of the left pulmonary field there
are multiple focal shadows. In the basal region of the left pulmonary field
there are clear outlines of two annular shadows with quite thick and irregular
walls. What disease is this X-ray pattern typical for?
A. Fibro-cavernous pulmonary tuberculosis
B. Pancoast tumour
C. Abscessing pneumonia
D. Atelectasis of the right upper lobe
E. Peripheral cancer

117. A patient complains of impaired far vision. Previously his eyes often
turned red and hurt. Objectively: the eyes are not irritated, the cornea is
transparent, anterior chambers are median deep, their liquid is transparent.
The iris of the right eye has not changed in colour, its pattern is unchanged.
The pupil is of irregular shape, scalloped. Biomicroscopy of the crystalline lens
reveals the areas of opacity and vacuoles. Make a diagnosis:
A. Complicated cataract of the right eye
B. Diabetic cataract of the right eye
C. Radiation cataract of the right eye
D. Tetanic cataract of the right eye
E. Senile cataract of the right eye

118. A 26-year-old patient complains of experiencing pain in the right hand


for 4 days. The pain arose at the site of corns on the palmar surface at the base
of the II and III fingers. Some time later the dorsum of hand became
edematic. I and III fingers are half-bent in the interphalangeal joints, the hand
looks like "rake". What is the most likely diagnosis?
A. Phlegmon of the second interdigital space of the right hand
B. Tendovaginitis
C. Corn abscess of the right hand
D. Adenophlegmon of the right hand
E. U-shaped phlegmon of the right hand

119. A 30-year-old patient had deep burn covering 30% of body 30 days ago.
Now he presents with continued fever, loss of appetite, night sweats. Burned
surface weakly granulates. What is the stage of burn disease?
A. Septicotoxemia
B. Convalescence
C. Secondary burn shock
D. Primary burn shock
E. Acute burn toxemia
120. A 29 y.o. patient was admitted to the hospital with acute girdle pain in
epigastric area, vomiting in 1 hour after the meal. On examination: pale,
acrocyanosis. Breathing is frequent, shallow. Abdominal muscles are tensed,
positive Schotkin-Blumbergs symptom. What is the maximal term to make a
diagnosis?
A. In 2 hours
B. In 3 hours
C. In 6 hours
D. In 0.5 hours
E. In 1 hour

121. A surgeon examined a 42-year-old patient and diagnosed him with


right forearm furuncle at the purulo-necrotic stage. The furuncle was lanced.
At the hydration stage the wound dressing should enclose the following
medication:
A. Hypertonic solution
B. Dimexide
C. Ichthyol ointment
D. Vishnevsky ointment
E. Chloramine

122. A 30 year old patient undergoes treatment because of Werlhofs disease.


Objectively: the patient is pale, there are petechial haemorrhages on the
extension surfaces of forearms. Ps is 92 bpm, AP is 100/60 mm Hg. The lower
edge of spleen is at a level with umbilicus. Blood count: erythrocytes:
2,8×1012/l, Hb - 90 g/l, Ht - 0,38, thrombocytes – 30×109/l. The patient is
being prepared for splenectomy. What transfusion medium should be chosen
in the first place for the preoperational preparation?
A. Thrombocytic mass
B. Erythrocytic suspension
C. Washed erythrocytes
D. Stored blood
E. Native erythrocytic mass

123. A 19 year old boy was admitted to a hospital with closed abdominal
trauma. In course of operation multiple ruptures of spleen and small intestine
were revealed. AP is falling rapidly, it is necessary to perform
hemotransfusion. Who can specify the patients blood group and rhesus
compatibility?
A. A doctor of any speciality
B. A traumatologist
C. An anaesthesilogist
D. A laboratory physician
E. A surgeon
124. A 36-year-old man was delivered to the surgical department an hour
after a road accident. His condition is getting worse: respiratory insufficiency
is progressing, there are cardiac abnormalities. Clinical and roentgenological
investigations revealed mediastinal displacement. What process has caused
this complication?
A. Valvular pneumothorax
B. Closed pneumothorax
C. Open pneumothorax
D. Subcutaneous emphysema
E. Mediastinitis

125. A victim of the car crash was brought to neurosurgery due to


craniocerebral trauma. During examination symptoms of focused injury of the
cerebrum are found, cerebrum contusion is suspected. What method of
examination is to be used?
A. Computed tomography of the cerebrum
B. Eye-ground examination
C. Antiography of vessels of the cerebrum
D. X- ray of cranium bones
E. Ultrasound examination of vessels of the cerebrum

126. A 35 y.o. female patient was admitted to the surgical department with
symptoms of ulcerative gastric hemorrhage. Its been the third hemorrhage for
the last 2 years. After conservative treatment vomiting with blood stopped,
hemoglobin elevated from 60 till 108 g/L. General condition became better.
But profuse vomiting with blood reoccured in 2-3- hours. Hemoglobin
decreased to 93,1 g/L then to 58,1 g/L. What is the tactics of treatment?
A. Urgent surgery
B. Conservative treatment with following surgery
C. Taylors treatment
D. Deferred surgery
E. Conservative treatment

127. A 39-year-old patient complains of a tumour on the anterior surface of


her neck. The tumour has been observed for 2 years. It is nonmobile and has
enlarged recently. The patient has a changed tone of voice, a sense of pressure.
Objectively: in the left lobe of the thyroid gland a 3 cm node is palpable; it is
very dense, tuberous, painless. Cervical lymph nodes are enlarged. Functional
status of the thyroid gland is unchanged. What is the most likely diagnosis?
A. Thyroid gland cancer
B. Nodular hyperthyroid goiter
C. Nodular euthyroid goiter
D. Chronic lymphomatous Hashimotos thyroiditis
E. Chronic fibrous Riedels thyroiditis

128. An unconscious victim in severe condition is brought to clinic. It is


known that the patient touched the bare wire with his hand and during 5
minutes was under the influence of an alternating current with voltage of 220
V. Physical exam: skin is pale, cold by touch. Breath is weakened, BP - 90/50
mm Hg, Ps - 60 bpm, arrhythmical. There are fields of necrosis of the skin on
the right hand and on the right foot. What is the preliminary diagnosis?
A. Electroburn of the right foot and right hand
B. High voltaged electroburn of the right foot and shank
C. Electrotrauma, acute cardiovascular failure
D. Vascular collapse due to electric current lesion

129. A 47-year-old patient complains about cough with purulent sputum,


pain in the lower part of the left chest, periodical body temperature rise. She
has been suffering from these presentations for about 10 years. Objectively:
"drumstick" distal phalanges. What examination would be the most
informative for making a diagnosis?
A. Bronchography
B. Pleural puncture
C. Bacteriological analysis of sputum
D. Bronchoscopy
E. Survey radiograph of lungs

130. Blood typing resulted in positive isohemagglutination reaction with


standard sera of A(II) and B(III) groups and negative reaction with sera of
0(I) and AB(IV) groups. What is this result indicative of?
A. Faulty standard sera
B. The first blood group
C. The third blood group
D. The fourth blood group
E. The second blood group

131. A victim of a road accident, aged 44, is operated on account of


intraperitoneal haemorrhage. In which case can the patients blood from the
abdominal cavity be used for autotransfusion?
A. Stomach rupture
B. Liver rupture
C. Bladder rupture
D. Splenic rupture
E. Small intestines rupture
132. A man with a stab-wound of the right foot applied to doctor. He
stepped on the board with nail two hours ago. In the patient medical chart it is
stated that 3 years ago he passed the whole course of vaccination against
tetanus. What is the tactics of doctor to prevent tetanus in this case?
A. Do not conduct specific prophylaxis
B. Administer 1,0 ml of tetanus toxoid
C. Administer 1 ml tetanus toxoid and 3000 U of anti-tetanic serum
D. Administer 3000 U of anti-tetanic serum
E. Administer 0,5 ml of tetanus toxoid

133. A 42-year-old patient with acute haemorrhage and class III blood loss
underwent blood transfusion and got 1,8 l of preserved blood and erythromass
of the same group and Rh. After the transfusion the patient complained of
unpleasant retrosternal sensations, his arterial pressure dropped to 100/60
mm Hg, there appeared convulsions. Blood serum calcium was at the rate of
1,7 millimole/liter. What is the mechanism of this complication development?
A. Citrate binds calcium ions, hypocalcemia impairs myocardial
function
B. Citrate causes the development of metabolic acidosis
C. Citrate binds potassium causing severe hypokalemia
D. The increased citrate rate causes convulsions
E. Citrate is cardiotoxic and nephrotoxic

134. A 40-year-old woman with acute gastrointestinal hemorrhage was


transfused 400 ml of the preserved blood after having conducted all
compatibility tests. After hemotransfusion the condition of the patient became
worse. She started having headache, muscles pain, her temperature has risen
up to 38,8°C. What is the reason for such condition of the patient?
A. Pyrogenic reaction of medium severity
B. Allergic reaction
C. Air embolism
D. Bacterial and toxic shock development
E. Hemotransfusional shock development

135. A 27-year-old patient with a knife stomach wound has been delivered to
a hospital 4 hours after injury. Objectively: the patient is in grave condition.
Ps- 120 bpm, weak. AP- 70/40 mm Hg. Laparotomy revealed a lot of liquid
blood in the abdominal cavity. The patient has been found to have bleeding
from the mesenteric vessels of the small intestine. Damage to hollow organs
has not been revealed. What is the best way to restore the blood loss?
A. Autoblood reinfusiont
B. Transfusion of washed erythrocytes
C. Rheopolyglucinum transfusion
D. Transfusion of fresh frozen plasma
E. Erythromass transfusion

136. A patient presented to a hospital with a carbuncle of the upper lip. The
body temperature is 39°C. There is a pronounced edema of the upper lip and
eyelids. What is the surgeons tactics of choice?
A. Hospitalize in the surgical unit
B. Disclose the carbuncle and administer out-patient treatment
C. Administer out-patient course of antibiotics
D. Administer physiotherapy
E. Disclose the carbuncle and administer antibiotics

137. A patient, aged 25, suffering from stomach ulcer. Had a course of
treatment in the gastroenterological unit. 2 weeks later developed constant
pain, increasing and resistant to medication. The abdomen is painful in
epigastric area, moderate defence in pyloroduodenal area. Which
complication development aggravated the patients state?
A. Malignisation
B. Stenosis
C. Perforation
D. Penetration
E. Haemorrhage

138. A 62-year-old male patient complains of intense pain in the left leg that
suddenly arose three hours before, leg numbness and coldness. During the
year there has been pain in the leg while walking, hypersensitivity to cooling.
Objectively: the left foot and shin have marbled skin, subcutaneous veins are
collapsed. The foot is cold, active movements of the foot and toes are
preserved. Pulse is present only on the femoral artery. There is rough systolic
murmur above the artery. Make a provisional diagnosis:
A. Acute occlusion of the left femoral artery
B. Stenosis of the left popliteal artery
C. Acute arterial thrombosis ileofemoralny
D. Acute thrombophlebitis
E. Occlusive disease

139. A patient complains about pyrosis and permanent pain behind his
breastbone. When he bends forward after eating there appears regurgitation.
Roentgenological examination revealed extrasaccular cardiofunctional hernia
of esophageal opening of diaphragm. Esophagoscopy revealed signs of reflux-
esophagitis. What is the necessary tretment tactics?
A. Operation in a surgical department
B. Treatment at a health resort
C. Conservative treatment in the therapeutic hospital
D. Conservative treatment in an outpatients clinic
E. Conservative treatment in a surgical department

140. A patient aged 18 with a cranial injury was in comatose state during
several hours. In post-comatose period gets tired quickly, non-productive in
dialog - in the beginning answers 2-3 questions, then gets tired and can not
understand the point of the question. Which psychotropic should be given to
the patient to prevent psychoorganic syndrome?
A. Nootropics
B. Antidepressants
C. Stimulators
D. Neuroleptics
E. Tranquillisers

141. A 45-year-old male patient with acute abscess of the left lung has
suddenly developed acute chest pain and dyspnea while coughing, tachycardia
has increased. The control Ro-gram shows left lung collapse, the air in the left
pleural cavity and a horizontal fluid level. What is the mechanism of this
complication?
A. Abscess burst into the pleural cavity
B. Bullae rupture of the left lung
C. Atelectasis of the left lung
D. Acute cardiovascular insufficiency
E. Inflammation spread to the visceral pleura

142. Examination of a 38-year-old patient who had been hit with a blunt
object on the left side of chest revealed a fracture of the X rib with fragments
displacement, parietal pneumothorax. The patient complains of pain in the
left subcostal area. Objectively: the patient is pale, AP- 80/40 mm Hg, Ps-
138/min, of poor volume. USI reveals fluid in the left abdomen. Splenic
rupture is present. What treatment tactics should be chosen?
A. Drainage of the left pleural cavity followed by laparotomy
B. Immediate laparotomy and alcohol-novocaine block of the X rib
C. Immediate upper median laparotomy followed by drainage of the left pleural
cavity
D. Anti-schock actions followed by laparotomy after the arterial pressure rise
E. Left-sided thoracotomy immediately followed by laparotomy

143. On the 2nd day after a surgery for toxic mixed goiter IV a 35-year-old
patient complains of heart pain. ECG shows prolonged QT intervals.
Chvosteks and Trousseau symptoms cannot be clearly defined. The patient is
provisionally diagnosed with latent tetany. What study will allow to confirm
the diagnosis?
A. Determination of blood calcium and phosphor
B. Determination of potassim
C. Determination of thyroid hormones
D. Determination of sodium
E. Determination of thyrotropic hormone

144. A patient with bilateral hydrothorax has repeatedly undergone pleural


puncture on both sides. After a regular puncture the patients condition has
become worse: he presents with fever, chest pain. The next day, the attending
physician performing pleural puncture revealed some pus on the right. What
is the mechanism of acute right-sided empyema development?
A. Contact-and-aspiration
B. Hematogenous
C. Lymphogenous
D. Implantation
E. Aerial

145. A 25 y.o. patient was admitted with chest trauma. Clinical and X-ray
examination have revealed tense pneumothorax on the left. What emergency
treatment should be undertaken?
A. Pleural cavity drainage
B. Analgetics
C. Oxigenotherapy
D. Intravenous infusions
E. Intubation

146. A 38 y.o. patient lifted a heavy object that resulted in pain in the
lumbar part of spine irradiating to the posterior surface of his left leg. The
pain increases during changing body position and also in the upright position.
Examination revealed positive symptoms of tension. What is the preliminary
diagnosis?
A. Pathology of intercostal disks
B. Myelopathy
C. Arachnomyelitis
D. Spinal cord tumor
E. Polyneuritis

147. Two hours ago a 38-year-old patient got pain in his right shin. He was
diagnosed with popliteal artery embolism, acute arterial insufficiency of grade
I. What is the most appropriate therapeutic tactics?
A. Embolectomy
B. Resection of the popliteal artery
C. Amputation at the middle of shin
D. Bypass grafting
E. Destruction of the embolus by the catheter

148. A 38-year-old patient has suddenly developed pain in the left side of his
chest, suffocation. Objectively: moderately grave condition, Ps- 100/min, AP-
90/60 mm Hg, breath sounds on the left cannot be auscultated. Chest
radiography shows the collapse of the left lung up to 1/2. What kind of
treatment should be administered?
A. Passive thoracostomy
B. Rest, resolution therapy
C. Operative therapy
D. Active thoracostomy
E. Pleural puncture

149. A 47-year-old female patient complains of leg heaviness, fatigue when


standing and walking. This feeling disappears when she takes a horizontal
position. Objectively: dilatation of the superficial veins of the left shin and
thigh with pigmentation and trophic skin disorders. What functional test
should the examination be started with?
A. Trendelenburgs test
B. Pratt test 2
C. Sheinis test
D. Perthes test
E. Pratt test 1

150. A neonatologist examining a full-term mature baby revealed the


shortening and external rotation of the newborns lower extremity. Clinical
examination revealed positive Ortolani sign, symptom of non-vanishing pulse,
additional skin folds on the internal surface of thigh. What is the most likely
diagnosis?
A. Congenital hip dislocation
B. Fracture of the femoral shaft
C. Varus deformity of the femoral neck
D. Dysplasia of the hip joint
E. Femoral neck fracture

151. A child is being discharged from the surgical department after


conservative treatment of invagination. What recommendations should doctor
give to mother to prevent this disease recurrence?
A. Strict following of feeding regimen
B. Hardening of the child
C. Feces observation
D. Common cold prophilaxis
E. Gastro-intestinal disease prevention

152. A 10-year-old child has been admitted to a hospital with a closed


craniocerebral injury with suspected cerebral edema. The patient is in grave
condition, unconscious. The dyspnea, tachycardia, hypertension are present.
Muscle tone is increased, there is nystagmus, pupillary and oculomotor
reactions are disturbed. The mandatory component of intensive care is
dehydration. What diuretic is adequate in this case?
A. Furosemide
B. Spironolactone
C. Moduretic
D. Mannitol
E. Hydrochlorthiazide

153. A 40-year-old patient underwent an operation for a lumbar phlegmon.


Body temparature rose again up to 38°C, he got intoxication symptoms, there
was an increase of leukocyte number in blood. The wound that was nearly free
from necrotic tissues and full of granulations started to discharge pus, the
granulations turned pale. What complication dveloped in this patient?
A. Sepsis
B. Erysipelas
C. Erysipeloid
D. Allergic reaction
E. Putrid phlegmon

154. After a car accident a 37-year-old patient has an acute pain and
bleeding in the middle third of his right thigh. Objectively: there is a wound on
the anterior surface of the right thigh with massive bleeding, abnormal
mobility at the level of the middle third of the thigh. The first aid is to be
started with:
A. Digital occlusion of the femoral artery
B. Venipuncture and intravenous infusion of polyglycine
C. Tourniquet application
D. Injection of pain-killers
E. Immobilization of the extremity with a transportation splint

155. A 42-year-old builder consulted a doctor about a foot injury with a nail
that he got in the morning of the same day. The wound was washed with
water. Three years ago he was vaccinated against tetanus. Examination
established satisfactory condition of the patient. The left foot was slightly
edematic, there was a stab wound on the sole. In order to prevent tetanus it is
primarily required to:
A. Give an intravenous injection of 0,5 ml of tetanus anatoxin
B. Administer a course of antibiotic therapy
C. Give an intravenous injection of 3000 IU of antitetanus serum
D. Give an intravenous injection of 1 ml of tetanus anatoxin, 3000 IU of
antitetanus serum
E. Treat the wound with suds

156. A 9-year-old boy fell from a tree and hit the occipital region, there was
a momentary loss of consciousness. Objectively: the childs condition is
satisfactory, he complains of the headache and dizziness. The X-ray of skull
reveals a comminuted depressed fracture of occipital bone in the region of
inion. What treatment is indicated for the patient?
A. Surgical intervention
B. Hemostatic therapy
C. Anti-inflammatory therapy
D. Therapeutic lumbar punctures
E. Complex conservative treatment

157. A 55-year-old patient complains of severe itching, burning and pain in


the eyes, skin redness in the outer corners of the palpebral fissure. Objectively:
skin around the outer corners of the palpebral fissure is macerated,
eczematous, there are single moist cracks. Palpebral conjunctiva is hyperemic,
quaggy. There are minor discharges in form of stringing mucus. What is the
most likely diagnosis?
A. Chronic conjunctivitis
B. Blepharitis
C. Atopic eyelid dermatitis
D. Acute conjunctivitis
E. Sty

158. A 40-year-old female patient has a history of rheumatism. She


complains about acute pain in her left eye, especially at night, vision
impairment, photophobia, lacrimation. The patient cannot suggest any
reasons for the disease. Objectively: weak pericorneal injection, flattening of
iris relief, iris discoloration. What is the most likely diagnosis?
A. Iridocyclitis
B. Keratitis
C. Iritis
D. Choroiditis
E. Acute attack of glaucoma
159. 14 days ago a 49-year-old patient was operated on for perforated
appendicitis and disseminated fibrinopurulent peritonitis. The postoperative
period was uneventful. On the 9th day the patient presented with low-grade
fever, abdominal pain, frequent liquid stools. USI of the left mesogastrium
reveals a fluid formation 9x10 cm large. In blood: leukocytosis with the left
shift. What is your provisional diagnosis?
A. Interloop abscess
B. Liver abscess
C. Abdominal cavity tumour
D. Left kidney cyst
E. Spleen abscess

160. A patient, aged 81, complains of constant urinary excretion in drops,


feeling of fullness in thelower abdomen. On examination: above pubis there is
a spherical protrusion, over which there is a dullness of percussion sound,
positive suprapubic punch. What symptom is observed in this patient?
A. Paradoxal ischuria
B. Dysuria
C. Urinary incontinence
D. Enuresis
E. Pollakiuria

161. A 67 y.o. patient complains of dyspnea, breast pain, common weakness.


He has been ill for 5 months. Objectively: to- 37,3°C, Ps- 96/min. Vocal
tremor over the right lung cannot be determined, percussion sound is dull,
breathing cannot be auscultated. In sputum: blood diffusively mixed with
mucus. What is the most probable diagnosis?
A. Lung cancer
B. Bronchoectatic disease
C. Macrofocal pneumonia
D. Focal pulmonary tuberculosis
E. Exudative pleuritis

162. A 52 year old patient complains about pain in the right part of her
chest, dyspnea, cough with a lot of foul-smelling albuminoid sputum in form
of "meat slops". Objectively: the patients condition is grave, cyanosis is
present, breathing rate is 31/min, percussion sound above the right lung is
shortened, auscultation revealed different rales. What is the most probable
diagnosis?
A. Lung gangrene
B. Lung abscess
C. Multiple bronchiectasis
D. Chronic pneumonia
E. Pleura empyema
163. 3 hours ago a 65-year-old female patient felt sharp abdominal pain
irradiating to the right scapula, there was a single vomiting. She has a history
of rheumatoid arthritis. Objectively: pale skin, AP- 100/60 mm Hg, Ps-
60/min. Abdomen is significantly painful and tense in the epigastrium and
right subcostal areat, there are positive symptoms of parietal peritoneum
irritation over the right costal arch, that is tympanitis. What is the tactics of an
emergency doctor?
A. To take the patient to the surgical hospital
B. To perform gastric lavage
C. To take the patient to the rheumatological department
D. To inject spasmolytics
E. To inject pain-killers and watch the dynamics

164. A 15 year old patient suffers from headache, nasal haemorrhages, sense
of lower extremity coldness. Objectively: muscles of shoulder girdle are
developed, lower extremities are hypotrophied. Pulsation on the pedal and
femoral arteries is sharply dampened. AP is 150/90 mm Hg, 90/60 on the
legs. Systolic murmur can be auscultated above carotid arteries. What is the
most probable diagnosis?
A. Aorta coarctation
B. Aorta aneurism
C. Aortal insufficiency
D. Coarctation of pulmonary artery
E. Aortal stenosis

165. A 15 y.o. patient has developmental lag, periodical skin yellowing.


Objectively: spleen is 16?12?10 cm large, holecistolithiasis, skin ulcer on the
lower third of his left crus. Blood count: erythrocytes - 3,0×1012/L, Hb- 90
g/L, C.I.- 1,0, microspherocytosis, reticulocytosis. Blood bilirubin - 56
mmole/L, indirect bilirubin - 38 mmole/L. Choose the way of treatment:
A. Splenectomy
B. Omentohepatopexy
C. Portocaval anastomosis
D. Spleen transplantation
E. Omentosplenopexy

166. A 72-year-old patient complains of pain and bleeding during


defecation. Digital rectal investigation revealed a tumour of the anal canal.
After verification of the diagnosis the patient was diagnosed with squamous
cell carcinoma. The secondary (metastatic) tumour will be most probably
found in:
A. Lungs
B. Brain
C. Pelvic bones
D. Liver
E. Mediastinum

167. A patient with frostbite of both feet was delivered to the admission
ward. What actions should be taken?
A. To apply a bandage, to introduce vasodilating medications
B. To administer cardiac medications
C. To rub feet with snow
D. To apply an alcohol compress
E. To put feet into hot water

168. 10 years ago a patient had a fracture in the middle one-third of his left
femoral bone, and during the last 7 years he has been having acute
inflammation in the area of old fracture accompanied by formation of a fistula
through which some pus with small fragments of bone tissue is discharged.
After a time the fistula closes. What complication of the fracture is it?
A. Chronic osteomyelitis
B. False joint
C. Trophic ulcer
D. Bone tuberculosis
E. Soft tissue phlegmon

169. A 21-year-old male patient got a deep cut wound in his right thigh. In
the emergency room a surgeon on duty performed primary debridement of the
wound and primary wound closure with a suture. After 4 days, there appeared
pain, redness, edema, purulent discharge from the wound gap, body
temperature rose up to 39°C. What kind of wound complication can you think
of and what actions should be taken?
A. Wound abscess, remove the sutures and drain the wound
B. Tetanus, active-passive immunization against tetanus
C. Lymphangitis, apply a hot compress
D. Infiltration, apply a hot compress
E. Erysipelas, prescribe antibiotics

170. A patient has a stab wound on his right foot. On the fourth day after
injury the patients body temperature rose up to 38°C, inguinal lymph nodes
became enlarged and painful, skin over them reddened. What complication
might be suspected?
A. Lymphadenitis
B. Phlegmon
C. Erysipelas
D. Tetanus
E. Lymphangitis

171. During a surgery on a 30-year-old patient a dark ileo-ileal


conglomerate was discovered, the intussusceptum intestine was considered to
be unviable. The intussuscipiens intestine was dilated to 7-8 cm, swollen, full
of intestinal contents and gases. What pathology led to the surgery?
A. Invagination (combined) obstruction
B. Spastic obstruction
C. Obturation obstruction
D. Strangulation obstruction
E. Paralytic obstruction

172. A 52 year old patient complains about headache, weakness of his upper
left extremity. Neurological symptoms become more intense during physical
stress of the left extremity. Pulsation on the arteries of the left extremity is
sharply dampened but it remains unchanged on the carotid arteries. What is
the most probable diagnosis?
A. Occlusion of the left subclavicular artery, steal syndrome
B. Raynauds syndrome
C. Occlusion of brachiocephalic trunk
D. Takayasus disease
E. Thoracal outlet syndrome

173. A 43-year-old patient had been admitted to a hospital with clinical


presentations of ischiorectal periproctitis. On the 12th day of treatment the
patients condition deteriorated: there was an increase in the rate of
intoxication and hepatic failure, the body temperature became hectic, AP was
100/60 mm Hg. USI of liver revealed a hydrophilic formation. In blood: WBCs
- 19,6×109/l, RBCs.- 3,0×1012/l, Hb- 98 g/l. What complication was
developed?
A. Liver abscess
B. Pylephlebitis
C. Liver necrosis
D. Budd-Chiari syndrome
E. Hepatic cyst

174. A 57 year old patient abruptly lost the sight of one eye. Examination
revealed that his sight made up 0,02 excentrically, eye fundus has
haemorrhages of different forms and sizes ("squashed tomato" symptom).
Disc of optic nerve is hyperemic. In anamnesis general vascular pathology is
recorded. Direct-acting anticoagulants were administered. What is the most
probable diagnosis?
A. Thrombosis of central vein of retina
B. Hypertensive angiopathy
C. Embolism of central artery of retina
D. Diabetic retinopathy
E. Hypertensive angioneuropathy

175. Half an hour ago a 67-year-old patient with a hernia picked up a heavy
object, which caused acute pain in the region of hernia protrusion, the hernia
couldnt be reduced. Objectively: the hernia in the right inguinal region is
roundish, tight, moderately painful; during palpation it was reduced back to
the cavity, the pain was gone. Specify the further medical tactics:
A. Inpatient surveillance
B. Immediate laparotomy
C. Immediate hernioplasty
D. Planned hernioplasty a month later
E. Planned hernioplasty a year later

176. A 55 year old patient felt suddenly sick in a hospital corridor, he was
immediately examined by a doctor. Examination revealed that the patients
skin was pale, autonomous respiration was absent, pulse on carotid arteries
couldnt be felt, pupils were mydriatic. What action should be taken at the
beginning of cardiac resuscitation?
A. Precordial thump
B. Defibrillation
C. Closed-chest cardiac massage
D. Mouth-to-mouth ventilation
E. Restoration of airway patency

177. A 30-year-old patient has been admitted to the intensive care unit for
multiple bee stings. The skin is covered with cold sweat. The pulse is felt just
on the carotid arteries, 110 bpm, respiration is 24/min, rhythmic, weakened.
Which drug should be given in the first place?
A. Adrenalin hydrochloride intravenously
B. Adrenalin hydrochloride intramuscularly
C. Prednisolone intravenously
D. Dopamine intravenously
E. Tavegil intravenously

178. A 25-year-old victim of a road accient complains of chest pain, dyspnea.


Objectively: the patient is in a grave condition, Ps- 120/min, AP- 90/70 mm
Hg.There is pathological mobility of fragments of III-V ribs on the right.
Percussion reveals a box sound over the right lung, breathing sounds cannot
be auscultated on the right. What examination should be administered in the
first place?
A. X-ray of chest organs
B. Pleural puncture
C. Bronchoscopy
D. USI of chest organs
E. Thoracoscopy

179. After the pneumatic dilatation of oesophageal structure a patient


developed acute retrosternal pain getting worse when throwing the head back
and swallowing. Objectively: dilatation of the neck veins, dropped beat pulse,
signs of purulent intoxication, oliguria, emphysema of the upper portion of
chest. What disease can be suspected?
A. Suppurative mediastinitis
B. Pleural empyema
C. Spontaneous pneumothorax
D. Acute myocardial infarction
E. Thrombosis of the superior vena cava

180. A 6-year-old girl drank some coloured fizzy drink which gave her a
feeling of pressure in the throat. 30 minutes later the childs lips got swollen,
then edema gradually spread over the whole face, laryngeal breathing became
difficult. The child is excited. Ps- 120/min, breathing rate - 28/min, breathing
is noisy, indrawing of intercostal spaces is observed. What basic aid is most
appropriate for the restoration of laryngeal breathing?
A. Corticosteroids
B. Antibacterial drugs
C. Conicotomy
D. Sedative drugs
E. Tracheostomy

181. An 8-month-old baby has had problems with nasal breathing and
muco-purulent discharge from the nose for a week. Examination reveals a
rhinedema, muco-purulent discharges from the middle nasal meatus as well
as on the back of pharynx. What disease are these symptoms most typical for?
A. Ethmoiditis
B. Sphenoiditis
C. Frontitis
D. Hemisinusitis
E. Maxillary sinusitis

182. A 28-year-old female patient has been admitted to a hospital. She


states to be ill for 12 years. On examination she has been diagnosed with
bronchiectasis with affection of the left lower lobe of lung. What is the optimal
treatment tactics for this patient?
A. Left lower lobectomy
B. Bronchopulmonary lavage
C. Antibiotic therpy
D. Left-sided pneumoectomy
E. Active drainage of the left pleural cavity

183. 4 weeks after myocardial infarction a 56-year-old patient developed


acute heart pain, pronounced dyspnea. Objectively: the patients condition is
extremely grave, there is marked cyanosis of face, swelling and throbbing of
neck veins, peripheral pulse is absent, the carotid artery pulse is rhythmic, 130
bpm, AP is 60/20 mm Hg. Auscultation of heart reveals extremely muffled
sounds, percussion reveals heart border extension in both directions. What is
the optimal treatment tactics for this patient?
A. Pericardiocentesis and immediate thoracotomy
B. Conservative treatment, infusion of adrenomimetics
C. Pleural cavity drainage
D. Oxygen inhalation
E. Puncture of the pleural cavity on the left

184. Esophagus wall of a 72 year old patient with severe concomitant


pathology was injured during urgent fibroesophagogastroscopy. This resulted
in progressing of acute respiratory failure and collapse of the left lung. What
aid should be rendered?
A. Drainage of pleural cavity by Bullaux method, mediastinum
drainage, antibacterial therapy
B. Endoscopic closure of esophagus wound, drainage
C. Left-sided thoracotomy, closure of esophagus and mediastinum wound
D. Buelaus drainage of pleural cavity, antibacterial therapy
E. Left-sided thoracotomy, closure of esophagus wound

185. A 17-year-old patient complains of pain in the area of the left knee
joint. Soft tissues of thigh in the affected region are infiltrated, joint function
is limited. X-ray picture of the distal metaepiphysis of the left femur shows a
destruction focus with periosteum detachment and Codmans triangle found at
the defect border in the bone cortex. X-ray of chest reveals multiple small
focal metastases. What treatment is indicated?
A. Palliative chemotherapy
B. Radioiodine therapy
C. Disarticulation of the lower extremity
D. Amputation of the lower extremity
E. Distance gamma-ray therapy
186. A 35-year-old victim of a road accident has got an injury of the right
side of his chest. Objectively: respiration rate - 28-30/min, respiration is
shallow, restricted respiratory excursion and acrocyanosis are present. Ps- 110
bpm, AP- 90/60 mm Hg. Respiratory sounds over the right lung cannot be
auscultated. Chest radiograph shows fractures of the VI-VII ribs on the right,
the right pleural cavity contains both air and fluid, with the fluid at about the
level of the V rib, the shadow of the mediastinum is displaced to the left. What
first aid should be provided to the victim?
A. Puncture of the pleural cavity
B. Vagosympathetic blockade
C. Antibiotic administration
D. Artificial ventilation of lungs
E. Urgent thoracotomy

187. On the first day after a surgery for diffuse toxic goiter a patient
developed difficulty breathing, cold sweats, weakness. Objectively: pale skin,
body temperature - 38,5°C, RR - 25/min, Ps- 110/min, AP- 90/60 mm Hg.
What early postoperative complication occurred in the patient?
A. Thyrotoxic crisis
B. Compression of the trachea by the hematoma
C. Postoperative tetany
D. Hypothyroid crisis
E. Acute thyroiditis

188. A 20-year-old patient complains of pain in the left lumbar region,


arterial pressure rise up to 160/110 mm Hg. USI revealed that the structure
and size of the right kidney were within age norms, there were signs of 3
degree hydronephrotic transformation of the left kidney. Doppler examination
revealed an additional artery running to the lower pole of the kidney.
Excretory urogram shows a narrowing in the region of ureteropelvic junction.
Specify the therapeutic tactics:
A. Surgical intervention
B. Kidney catheterization
C. Administration of beta-blockers
D. Administration of spasmolytics
E. Administration of ACE inhibitors

189. A 49-year-old male patient consulted a doctor about difficult


swallowing, voice hoarseness, weight loss. These symptoms have been
gradually progressing for the last 3 months. Objectively: the patient is
exhausted, supraclavicular lymph nodes are enlarged. Esophagoscopy
revealed no esophageal pathology. Which of the following studies is most
appropriate in this case?
A. Computed tomography of chest and mediastinum
B. X-ray of lungs
C. Radioisotope investigation of chest
D. Ultrasound investigation of mediastinum
E. Multiplanar imaging of esophagus

190. A 36-year-old male patient complains of having headache, obstructed


nasal breathing, purulent nasal discharge for 2 weeks. A month before, he had
his right maxillary premolar filled. Radiolography revealed an intense opacity
of the right maxillary sinus. Diagnostic puncture revealed a large amount of
thick malodorous crumbly pus. What is the most likely diagnosis?
A. Chronic suppurative odontogenic sinusitis
B. Chronic purulent sinusitis
C. Tumor of the maxillary sinus
D. Chronic atrophic sinusitis
E. Acute purulent sinusitis

191. A patient with autoimmune thyroiditis accompanied by multinodular


goiter underwent the right lobe ectomy and subtotal resection of the left lobe.
What drug should be administered to prevent postoperative hypothyroidism?
A. L-thyroxine
B. Iodomarin
C. Insulin
D. Lithium drugs
E. Merkazolil

192. A hospital admitted a patient with coarse breathing (obstructed


inspiration), skin cyanosis, tachycardia and arterial hypertension. He has a
histrory of bronchial asthma. An hour ago he was having salbutamol
inhalation and forgot to remove a cap that was aspired while taking a deep
breath. What measures should the doctor take?
A. To perform the Heimlich manoever
B. To use an inhalation of a2-adrenoceptor agonist
C. To make a subcutaneous injection of dexamethasone
D. To perform conicotomy immediately
E. To send for an anesthesiologist and wait for him

193. An emergency physician arrived to provide medical care for a hangman


taken out of the loop by his relatives. The doctor revealed no pulse in the
carotid arteries, lack of consciousness, spontaneous breathing and corneal
reflexes; cadaver spots on the back and posterior parts of extremities. A
person can be declared dead if the following sign is present:
A. Cadaver spots
B. Lack of corneal reflexes
C. Unconsciousness
D. Pulselessness
E. Lack of spontaneous breathing

194. An emergency physician arrived to provide medical care for a hangman


taken out of the loop by his relatives. The doctor revealed no pulse in the
carotid arteries, absence of consciousness, spontaneous breathing and corneal
reflexes; cadaver spots on the back and posterior parts of extremities. A
person can be declared dead if the following sign is present:
A. Cadaver spots
B. Pulselessness
C. Unconsciousness
D. Absence of spontaneous breathing
E. Absence of corneal reflexes

195. A 69-year-old male patient has been hospitalized with hypothermia.


Objectively: the patient is pale, has shallow breathing. AP is 100/60 mm Hg,
Ps is 60/min. Palpation of the abdomen and chest reveals no pathological
signs. The body temperature is of 34,8°C. The patients breath smells of
alcohol. Give treatment recommendations:
A. Warm bath + intravenous administration of warm solutions
B. Antishock therapy
C. Forced diuresis
D. Body warming with hot-water bags
E. Rubbing with alcohol and snow

196. A patient undergoing treatment for the left-sided destructive


pneumonia presents with deterioration of his general condition, progressing
dyspnea, cyanosis. Objectively: the left side of chest is not involved in
respiration, breathing sounds cannot be auscultated. Radiograph shows a
shadow reaching the 5 rib with a horizontal fluid level and a radiolucency
above it, the mediastinum is displaced to the right. What is the medical
tactics?
A. Thoracostomy
B. Infusion and antibacterial therapy
C. Emergency bronchoscopy
D. Open thoracotomy
E. Endotracheal intubation
197. A 26-year-old male patient consulted a doctor abut sore throat, fever up
to 38,2°C. A week before, the patient had quinsy, didnt follow medical
recommendations. On examination, the patient had forced position of his
head, trismus of chewing muscles. Left peritonsillar region is markedly
hyperemic, swollen. What is the provisional diagnosis?
A. Left-sided peritonsillar abscess
B. Phlegmonous tonsillitis
C. Tonsil tumour
D. Pharyngeal diphtheria
E. Meningitis

198. A 26-year-old patient consulted a doctor abut sore throat, fever up to


38,2°C. A week ago, the patient had angina, didnt follow medical
recommendations. On examination, the patient had forced position of his
head, trismus of chewing muscles. Left peritonsillar region is markedly
hyperemic, swollen. What is the provisional diagnosis?
A. Left-sided peritonsillar abscess
B. Phlegmonous angina
C. Tonsil tumour
D. Diphtheria of the pharynx
E. Meningitis

199. A 77-year-old male patient complains of inability to urinate, bursting


pain above the pubis. The patient developed acute condition 12 hours before.
Objectively: full urinary bladder is palpable above the pubis. Rectal prostate is
enlarged, dense and elastic, well-defined, with no nodes. Interlobular sulcus is
distinct. Ultrasonography results: prostate volume is 120 cm3, it projects into
the bladder cavity, has homogeneous parenchyma. Prostate-specific antigen
rate is of 5 ng/ml. What is the most likely disease that caused acute urinary
retention?
A. Prostatic hyperplasia
B. Sclerosis of the prostate
C. Acute prostatitis
D. Tuberculosis of the prostate
E. Prostate carcinoma

200. A 4 month old child was admitted to a surgical department 8 hours


after the first attack of anxiety. The attacks happen every 10 minutes and last
for 2-3 minutes, there was also one-time vomiting. Objectively: the childs
condition is grave. Abdomen is soft, palpation reveals a tumour-like formation
in the right iliac area. After rectal examination the doctors finger was stained
with blood. What is the most probable diagnosis?
A. Ileocecal invagination
B. Gastrointestinal haemorrhage
C. Helminthic invasion
D. Pylorostenosis
E. Wilms tumour

201. A 3-year-old male patient consulted a family doctor 2 months after he


had been operated for an open fracture of brachial bone. Objectively: the
patients condition is satisfactory, in the region of the operative wound there is
a fistula with some purulent discharge, redness, fluctuation. X-ray picture
shows brachial bone destruction with sequestra. What complication arose in
the postoperative period?
A. Posttraumatic osteomyelitis
B. Wound abscess
C. Hematogenic osteomyelitis
D. Posttraumatic phlegmon
E. Suture sinus

202. A 40-year-old female patient complains of having a bulge on the


anterior surface of neck for 5 years. Objectively: Ps- 72 bpm, arterial pressure
- 110/70 mm Hg, in the right lobe of thyroid gland palpation reveals a mobile
4x2 cm node, the left lobe is not palpable, the basal metabolic rate is 6%. What
is the most likely diagnosis?
A. Nodular euthyroid goiter
B. Mixed euthyroid goiter
C. The median cervical cyst
D. Nodular hyperthyroid goiter
E. Riedels thyroiditis

203. Forensic medical expertise of corpse of a newborn revealed: body


weight 3500 g, body length 50 cm, the umbilical cord was smooth, moist,
glossy, without any signs of drying. Hydrostatic tests were positive. The test
results are the evidence of:
A. Live birth
B. Primary atelectasis
C. Stillbirth
D. Secondary atelectasis
E. Hyaline membrane disease

204. A week ago a 65-year-old patient suffered an acute myocardial


infarction, his general condition deteriorated: he complains of dyspnea at rest,
pronounced weakness. Objectively: edema of the lower extremities, ascites is
present. Heart borders are extended, paradoxical pulse is 2 cm displaced from
the apex beat to the left. What is the most likely diagnosis?
A. Acute cardiac aneurysm
B. Acute pericarditis
C. Recurrent myocardial infarction
D. Cardiosclerotic aneurysm
E. Pulmonary embolism

205. A 36-year-old injured has been taken to the emergency station with
open tibial fractures. Examination reveals bleeding: pulsating blood spurts out
of the wound. What medical manipulations are required at this stage of
medical care?
A. Apply a tourniquet on the thigh proximal to the source of bleeding
and transport the patient to the operating room
B. Transport the patient to the operating room
C. Apply a tourniquet on the thigh distal to the source of bleeding and transport
the patient to the operating room
D. Immobilize the fracture and transport the patient to the operating room
E. Stop the bleeding by a compressive bandage and transport the patient to the
operating room

206. A 75-year-old male patient complains of slight pain in the right iliac
region. The abdominal pain arose 6 days ago and was accompanied by nausea.
Surgical examination revealed moist tongue, Ps- 76 bpm. AP- 130/80 mm Hg.
Abdomen was soft, slightly painful in the right iliac region on deep palpation,
the symptoms of the peritoneum irritation were doubtful. In blood: RBCs -
4,0×1012/l, Hb- 135 g/l, WBCs - 9,5×109/l, stab neutrophils - 5%,
segmentonuclear - 52%, lymphocytes - 38%, monocytes - 5%, ESR - 20 mm/h.
Specify the doctors further tactics:
A. Emergency operation for acute appendicitis
B. Refer the patient to a district therapist
C. Administration of additional examination: abdominal ultrasound, x-ray
contrast study of the gastrointestinal tract
D. Hospitalization, dynamic surveillance
E. Send the patient home

207. A 56-year-old patient was undergoing a surgery for suture repair of


perforated ulcer. During the operation the cardiomonitor registered
ventricular fibrillation. The first-priority measure should be:
A. Electrical defibrillation
B. Injection of calcium chloride
C. Injection of lidocaine
D. Injection of adrenalin
E. Injection of atropine
208. A 78-year-old patient complains of severe pain in the lower abdomen,
perineum and rectum; intense urge to urinate and inability to urinate without
assistance. Abdomen palpation reveals a moderately painful globular
formation above the pubis, there is percussion dullness over the formation.
What is the most likely diagnosis?
A. Acute urinary retention
B. Chronic incomplete urinary retention
C. Cystitis
D. Paradoxical ischuria
E. Chronic urinary retention

209. A 24-year-old patient had been delivered to the thoracic department


with a chest injury, a fracture of the IV, V, VI ribs on the right. Plan
radiography shows the fluid level in the pleural cavity reaching the III rib on
the right. Puncture blood contained clots. What is the optimal treatment
tactics?
A. Emergency thoracotomy
B. Thoracentesis and thoracostomy
C. Pleural puncture
D. Hemostatic therapy
E. Medical thoracoscopy
Krok 2 – 2019 (General Medicine)

Krok 2 Telegram Channel

Results
1 of 200 questions answered correctly

Your time: 01:26:32

You have reached 1 of 200 points, (0.5%)

RESTART QUIZ VIEW QUESTIONS

1. Question
A 24- year-old pregnant woman on her 37th week of pregnancy has been brought to
the maternity obstetric service with complaints of weak fetal movements. Fetal
heartbeats are 95/min. on vaginal examination the uterine cervix is tilted backwards,
2cm long, external orifice allows inserting a fingertip. Biophysical profile of the fetus
equals 4 points. What tactics of pregnancy management should be chosen?

1. Urgent delivery via a cesarean section

2. Urgent preparation of the uterine cervix for delivery

3. Treatment of placental dysfunction and repeated analysis of the fetal biophysical profile on the
next day

4. Doppler measurement of blood velocity in the umbilical artery

5. Treatment of fetal distress; if ineffective, then elective cesarean section on the next day

2. Question
A 52-year-old woman has been suffering for 2 years from dull, occasionally
exacerbating pain in her right subcostal area, occurring after eating high-fat foods,
bitter taste in her mouth in the morning, constipations, and flatulence. Objectively
she has excess weight, her body temperature is 36.9°C; there is a coating on the root of
her tongue; the abdomen is moderately distended and painful in the area of
gallbladder projection. What examination would be the most helpful for diagnosis-
making?

1. Duodenal intubation

2. Ultrasound

3. Cholecystography

4. Duodenoscopy
5. Liver scanning

3. Question
An 18-year-old young man complains of pain in his knee and elbow joints and body
temperature upto 39.5 o One week and a half earlier he developed sore throat. On
examination his body temperature is 38.5 oC. Swelling of the knee and elbow joints is
observed. Pulse is 106/min, rhythmic. Blood pressure is 90/60 mmHg. Cardiac
borders are unchanged, heart sounds are weakened, at the cardiac apex there is a soft
systolic murmur. What factors would be the most indicative of the likely disease
etiology?

1. C reactive protein

2. Creatine kinase

3. Anti streptolysin O

4. Seromucoid

5. Rheumatoid factor

4. Question
In the air of the feed kitchen at the poultry factory, at the area where formula feed is
being mixed, the dust concentration reaches 200 mg/m3. Air microflora is represented
predominantly by Asperquillus and Mucor fungi. What effect determines pathogenic
properties of the dust?

1. Allergenic

2. Mutagenic

3. Fibrogenic

4. Toxic

5. Teratogenic

5. Question
A 22-day old infant developed subcutaneous red nodes from 1.0 to 1.5 cm in size on the
scalp; later the nodes suppurated. Temperature increased upto 37.7 oC, intoxication
symptoms appeared, regional lymph nodes enlarged. Complete blood count: anemia,
leucocytosis, neutrocytosis, increased ESR. What diagnosis can be made?

1. Pseudofurunculosis

2. Vesiculopustulosis

3. Pemphigus
4. Scalp phlegmon

6. Question
A 19-year-old young man complains of cough with expectoration of purulent sputum
in the amount of 100 mL per day, haemoptysis, dyspnea, increased body temperature
up to 37.8°C, general weakness, weight loss. The patient’s condition lasts for 4 years.
Exacerbations occur 2-3 times a year. The patient presents with malnutrition, pale
skin, cyanosis of the lips, drumstick (clubbed) fingers. Tympanic percussion sound in
the lungs, weakened repiration, numerous various moist crackles in the lower
pulmonary segments on the left can be observed. In blood: erythrocytes – 3.2 x 1012/L,
leukocytes – 8.4 x 109/L, ESR – 56 mm/hour. On X-ray: lung fields are
emphysematous, the left pulmonary root is deformed and dilated. What is the most
likely diagnosis?

1. Multiple bronchiectasis of the left lung

2. Suppuration of the cyst in the left lung

3. Chromic abscess of the left lung

4. Chronic left-sided pneumonia

5. Left-sided pulmonary cystic dysplasia

7. Question
2 hours after a traffic accident a 28 year old man in a grave condition was brought to
a hospital. The patient complains of abdominal pain. He received a blow to the
abdomen wit the steering wheel. Objective examination revealed the following: the
abdomen does not participate in respiration, is tense and acutely painful on
palpation; the abdominal muscles are defensively tense, peritoneal irritation signs
are positive, hepatic dullness is absent. BP is 90/60 mmHg, heart rate is 120/min.
What further treatment tactics should be chosen?

1. Laparocentesis

2. Ultrasound investigation

3. Cold to the abdomen

4. Laparotomy

5. Laparoscopy

8. Question
A 32-year-old man complains of pain in his legs that intensifies during walking,
intermittent claudication, numbness of his toes, extremity coldness and inability to
walk more than 100 meters. When he sleeps, his leg usually hangs down. The patient
has been smoking since he was 16. He drinks alcohol in excess. The left leg is colder
than the right one; the skin of the extremities is dry. No pulse can be detected on the
pedal arteries, while pulsation on the femoral arteries is retained. What is the most
likely diagnosis?

1. Raynaud disease

2. Deep thrombophlebitis

3. Leriche Syndrome (Aortoiliac occlusive disease)

4. Obliterating endarteritis

5. Diabetic angiopathy

9. Question
A 48-year-old woman complains of disturbed menstrual cycle: her periods last 7-9
days and are excessively profuse throughout the last half-year. She notes occasional
hot flashes in her head, insomnia, irritability, and headaches. Her skin is of normal
color. Blood pressure- 150/90 mm Hg, pulse – 90/min., rhythmic. The abdomen is soft
and painless. Bimanual examination shows no uterine enlargement, the appendages
cannot be detected. The vaginal fornices are free. What is the most likely diagnosis?

1. Premenstrual syndrome

2. Uterine myoma

3. Climacteric syndrome

4. Stein- Leventhal syndrome(polycystic ovary syndrome)

5. Adrenogenital syndrome

10. Question
In the process of hiring, a prospective employee has undergone preventive medical
examination and was declared fit to work in this manufacturing environment. What
type of preventive medical examination was it?

1. Scheduled

2. Specific

3. Comprehensive

4. Periodical

5. Preliminary

11. Question
A 38-year-old woman after physical overexertion suddenly developed palpitations,
dyspnea and dull pain in the cardiac area. For 10 years she has been registered for
regular check-ups due to rheumatism and mitral valve disease with non-distributed
blood circulation. Objectively her pulse is 96/min of unequal strength. Blood pressure
is 110/70 mmHg, heart rate is 120/min. ECG registers small unevenly sized waves in
place of P waves, R-R intervals are of unequal length. What is the most likely
diagnosis?

1. Atrial flutter

2. Paroxysmal ventricular tachycardia

3. Respiratory arrhythmia

4. Atrial fibrillation

5. Paroxysmal supraventricular tachycardia

12. Question
Having examined a 52 year old patient, the doctor diagnosed him with obesity (body
mass index- 34 kg/m2, waist circumference – 112 cm) and arterial hypertension (
170/105 mm Hg). 2- hour postprandial blood sugar is 10.8 mmol/L. What biochemical
blood analysis needs to be conducted to diagnosis the patient with metabolic
syndrome X?

1. Electrolytes

2. Calcium and phosphorus

3. Creatinine and urea

4. Lipid profile

5. Bilirubin

13. Question
A 24-year-old woman, a kindergarten teacher, has been sick for 2 days already.
Disease onset was acute. She presents with elevated body temperature upto 38.0°C ,
pain attacks in her lower left abdomen, liquid stool in small amounts with blood and
mucus admixtures 10 times a day. Pulse – 98/min., blood pressure – 110/70mm Hg.
Her tongue is moist and coated with white deposits. The abdomen is soft, the sigmoid
colon is painful and spastic. Make the provisional diagnosis:

1. Shigellosis

2. Yersiniosis

3. Escherichiosis
4. Rotavirus infection

5. Salmonellosis

14. Question
A 52-year-old man for the last 3 years has been suffering from difficult swallowing of
solid food, burning retrosternal pain that aggravated during eating, loss of body
mass and occasional vomiting with undigested food. Esophageal X-ray shows S-
shaped deformation of the esophagus and its dilation; at the cardiac orifice the
esophagus is constricted; Esophageal mucosa is smooth, without signs of peristalsis.
Make the provisional diagnosis:

1. Esophageal diverticulum

2. Reflux esophagitis

3. Esophageal achalasia

4. Diaphragmatic hernia

5. Esophageal carcinoma

15. Question
A 38-year-old woman complains of weakness, sleepiness, pain in the joints, weight
gain despite low appetite, and constipations. She presents with dry and thickened
skin, puffy and amimic face, narrowed palpebral fissures, thick tongue and deep
hoarse voice. Her heart sounds are weak, pulse is 56/min. Low levels of free T4 are
observed. This patient needs to take the following on a regular basis:

1. Lithium carbonate

2. Mercazolil (Thiamazole)

3. Calcium gluconate

4. Thyroxine

5. Furosemide

16. Question
A 20-year-old man was hospitalized on the 9th day of the disease. He attributes his
disease to eating of insufficiently thermally processed pork. At its onset this condition
manifested as periorbital edemas and fever. Objectively his body temperature is
38.5°C. The face is puffy and the eyelids are markedly swollen. Palpation of
gastrocnemius muscles is sharply painful. Blood test shows hypereosinophilia. What
is the etiology of this disease?

1. Trichinella
2. Echinococci

3. Ascarididae

4. Leptospira

5. Trichuris

17. Question
A 45-year-old man with thrombophlebitis of the deep veins in this legs suddenly after
physical exertion developed sharp pain in his thorax on the right, dyspnea and
hemoptysis. Objectively his condition is severe; he presents with acrocyanosis,
shortening of pulmonary percussion sound on the right and weakened respiration.
Respiration is 30/min, blood pressure is 110/80 mmHg. ECg shows sinus tachycardia,
heart rate is 120/min, electrical axis of the heart deviates to the right, SI-QIII. What is
the most likely diagnosis?

1. Spontaneous pneumothorax

2. Right sided exudative pleurisy

3. Cancer of the right lung

4. Community acquired right sided pneumonia

5. Pulmonary embolism

18. Question
A 25-year-old woman complains of fatigue, dizziness, hemorrhagic rashes on the skin.
She has been presenting with these signs for a month. Blood test: erythrocytes – 1.0 x
1012/L, Hb – 37 g/L, color index – 1.1, leukocytes – 1.2 x 109/L, platelets – 42 x 109/L.
What analysis would be the most advisable for diagnosis making in this case?

1. US of the gastrointestinal tract

2. Splenic biopsy

3. Coagulation studies

4. Sternal puncture (bone marrow biopsy)

5. Liver biopsy

19. Question
A 35 year old pregnant woman with degree 1 essential hypertension, developed
edemas and headache at the 33 week of her pregnancy. Objectively her general
condition is satisfactory, blood pressure – 160/100 mmHg, normal uterine tone. Fetal
heart rate is 140/min and rhythmic. She was diagnosed with daily proteinuria – 4
g/L, daily diuresis – 1100 mL. Creatinine – 80 mcmol/L, urea – 7 mmol/L, platelets –
100 x 109/L. What complication of pregnancy occurred?

1. Moderate preeclampsia

2. Hypertensive crisis

3. Mild preeclampsia

4. Renal failure

5. Severe preeclampsia

20. Question
A woman in her early to mild thirties has lost her consciousness 3-5 minutes ago. On
examination: the skin is pale, no pulse over the carotid arteries, no spontaneous
respiration, pupils are dilated; the patient is nonresponsive, presents with atony. The
patient’s condition can be determined as:

1. Syncope

2. Comatose state

3. Brain death

4. Clinical death

5. Natural death

21. Question
2 hours after eating unknown mushroom’s, a 28-year-old man sensed a decrease in
his mobility and deterioration of his ability to focus. This condition was then followed
by a state a agitated and aggression. On examination he is disoriented, and his speech
is illegible. 4 hours later he developed fetor hepaticus and lost his consciousness.
What syndrome can be observed in this patient?

1. Acute hepatic failure

2. Hepatolienal syndrome

3. Portal hypertension

4. Cholestatic syndrome

5. Cytolytic syndrome

22. Question
A 32-year-old woman complains of tumor like formation on the anterior surface of
her neck that appeared 2 year ago. Within the last 3 months the tumor has been
rapidly growing. It hinders swallowing and impairs speech; the tumor causes a
sensation of pressure. Objectively the skin moisture is normal. Pulse is 80/min,
rhythmic, blood pressure is 130/80 mmHg. In the right lobe of the thyroid gland there
is a dense lumpy node 3.0×3.5 cm that moves during swallowing. Scanning image
shows a ‘cold nodule’ in the thyroid gland. Make the provisional diagnosis:

1. Thyroid adenoma

2. Autoimmune thyroiditis

3. Thyroid cyst

4. Nodular goiter

5. Thyroid cancer

23. Question
A 51-year-old man complains of vomiting with blood. He has been drinking alcohol
excessively. Health disorder has been observed since he was 40, when he first
developed jaundice. On examination the skin and visible mucosa are icteric, with a
stellate vascular pattern. The patient is malnourished and presents with abdominal
distension, umbilical hernia and ascites. The edge of the liver is tapered and painless,
+3cm, the spleen is +2cm. Blood test: Hb- 80g/L, leukocytes – 3 x 109/L, platelets – 85
x 109/L. What is the cause of portal hypertension in this patient?

1. Thrombosis of the splenic vein

2. Hepatic cirrhosis

3. Budd-Chiari syndrome

4. Hemochromatosis

5. Constrictive pericarditis

24. Question
A 25-year-old woman has been suffering form diabetes mellitus since she was 9. She
was admitted into a nephology unit with significant edemas of the face, arms and
legs. BP – 200/110 mmHg, Hb – 90 g/L, blood creatinine – 850 mcmol/L, urine
proteins – 1.0 g/L, leukocytes – 10-15 in the vision field. Glomerular filtration rate –
10 mL/min. What tactics should the doctor choose?

1. Dietotherapy

2. Transfer into the endocrinology clinic

3. Active conservative therapy for diabetic nephropathy


4. Transfer into the hemodialysis unit

5. Renal transplantation

25. Question
A 62-year-old patient has been hospitalized with complaints of pain in the thorax on
the right during breathing, dyspnea and dry cough. Ten days ago he slipped and fell
hitting his right side. On examination: the patient lies on the left side. The right side of
the thorax lags during breathing. On the right there are crepitation and pain in the
III-IV ribs. Dullness of percussion sound and sharply diminished breath sounds can
be observed. On X-ray; signs of exudate, fracture of the III-IV ribs. On pleurocentesis;
blood is detected. Choose the further tactics:

1. Transfer to a thoracic surgery department

2. Perform repeated pleural taps

3. Prescribe conservative therapy

4. Apply a fixation bandage to the rib cage

5. Refer to a traumatologist

26. Question
A 45-year-old man developed constricting retrosternal pain that occurs during walks
at the distance of 200m. Objectively: heart rate is 80/min, BP is 160/90 mmHg.
During cardiopulmonary exercise test at 50 W there is a depression of S-T segment by
3 mm below the isoline in V3-V4. What is the provisional diagnosis?

1. Alcoholic myocardiodystrophy

2. Exertional angina pectoris, functional class IV

3. Exertional angina pectoris, functional class II

4. Somatoform autonomic dysfunction, hypertension type

5. Exertional angina pectoris, functional class III

27. Question
A 14-year-old girl came to the general practitioner with complaints of weakness, loss
of appetite, headache, rapid fatigability. Her last menstruation was profuse and
lasted for 14 days after the previous delay of 2 months. Objectively; the skin is pale,
heart rate is 90/min, BP is 110/70 mmHg, Hb is 88 g/L. Rectal examination; the uterus
and its appendages are without changes, no discharge from the genital tracts. What
complication occurred in the patient?
1. Posthemorrhagic anaemia

2. Somatoform autonomic dysfunction of hypotonic type

3. Dysmenorrhea

4. Gastritis

5. Migraine

28. Question
A 23-year-old man complains of severe pain in his left knee joint. Objectively the left
knee joint is enlarged, with hyperemic skin, painful on palpation. Complete blood
count: erythrocytes – 3.8 x 1012/L, Hb- 122g/L, leukocytes – 7.4 x 109/L, platelets – 183
x 109/L. Erythrocyte sedimentation rate – 10 mm/hour. Bleeding time (Duke method)
– 4 min., Lee- White coagulation time – 24 min. Partial thromboplastin time
(activated) – 89 seconds. Rheumatoid factor – negative. What is the most likely
diagnosis?

1. Hemorrhagic vasculitis (Henoch – Schonlein purpura), articular form

2. Werlhof disease (immune thrombocytopenia)

3. Thrombocytopathy

4. Hemophilia, hemarthrosis

5. Rheumatoid arthritis

29. Question
A boy had a foreign body removed from under his nail plate. 3 days later he developed
a sharp throbbing pain at the end of his distal phalanx, which intensifies when the
phalanx is pressed, hyperemia of the nail fold, elevated body temperature upto 38.5
o
C, and nail plate discoloration. Make the diagnosis:

1. Abscess

2. Erysipelas

3. Erysipeloid

4. Sublingual panaritium

5. Paronychia

30. Question
A 57 year old woman complains of weakness, dyspnea, loss of appetite and liquid
feces. She has been suffering from this condition for 2 years. Objectively she presents
with pale skin, subicteric sclerae and bright red fissured tongue. Lymph nodes are not
enlarged. Pulse – 100/min. BP – 105/70 mmHg. Liver +3cm, the spleen cannot be
palpated. Blood test: erythrocytes – 1.2 x 1012/L, Hb – 56 g/L, color index – 1.4,
macrocytes, leukocytes – 2.5 x 109/L, eosinophils – 1%, juvenile – 1%, metamyelocytes
– 1%, band neutrophils – 8%, segmented neutrophils – 47%, lymphocytes – 38%,
monocytes – 4%, reticulocytes – 0.1%, platelets – 100 x 109/L, ESR – 30 mm/hour,
indirect bilirubin – 26 mmol/L. What changes can be expected in the bone marrow
puncture material?

1. Prevalence of megaloblasts

2. Increased number of sideroblasts

3. Prevalence of lymphoid tissue

4. Erythtroid hyperplasia

5. Present of blast cells

31. Question
In April during the medical examination of various population groups, 27% of
individuals presented with low working ability and rapid fatigability. The following
symptoms were observed in the affected individuals: Swollen friable gingiva that
bleeds when pressed, hyperkeratosis follicularis not accompanied by skin dryness.
These symptoms most likely result from the following pathology:

1. Polyhypovitaminosis

2. B1-hypovitaminosis

3. A-hypovitaminosis

4. C-hypovitaminosis

5. Parodontosis

32. Question
A pregnant woman is 28 years old. Anamnesis accelerated labor complicated by the II
degree cervical rupture. The following two pregnancies resulted in spontaneous
abortions at the terms of 12 and 14 weeks. On mirror examination: the uterine cervix
is scarred from previous ruptures at 9 and 3 hours, the cervical canal is gaping. On
vaginal examination: the cervix is 2 cm long, the external orifice is open 1 cm wide, the
internal orifice is half open; the uterus is enlarged to the 12th week of pregnancy, soft,
mobile, painless, the appendages are without changes. What diagnosis can be made?

1. Threatened spontaneous abortion

2. Incipient abortion, habitual non carrying of pregnancy

3. Cervical hysteromyoma, habitual non-carrying of pregnancy


4. Isthmico-cervical insufficiency, habitual non carrying of pregnancy

5. Cervical pregnancy, 12 weeks

33. Question
A 10 year old boy with symptoms of arthritis and myocarditis was brought to a
hospital. Based on clinical examination the provisional diagnosis of juvenile
rheumatoid arthritis was made. What symptom is the most contribute for the
diagnostics of this disease?

1. Regional hyperemia of the joints

2. Enlarged heart

3. Increased heart rate

4. Reduced mobility of the joints in the morning

5. Affection of the large joints

34. Question
A child is 1 year old. After solid food was introduced into the diet, within the last
several months the child developed loss of appetite, diarrhea with large amount of
feces, and occasional vomiting. Body temperature remains normal. Body weight is 7
kg. The child is very pale, has leg edemas and extremely distended abdomen. Feces
analysis detects high levels of fatty acids and soaps. Diagnosis of celiac disease was
made and gluten free diet was prescribed. What should be excluded from the diet in
this case?

1. Milk and dairy products

2. Fruits

3. Animal protein

4. Cereals – Wheat, oats

5. Easily digestible carbohydrates

35. Question
A 40-year man claims that his wife is cheating on him and presents a ‘proof’ of her
infidelity. He repeatedly initiated scandals with his wife at home and at work,
demanding that she confess her infidelity, insulted her and threatened to kill her.
What preventive measures should be taken against socially dangerous actions on his
part?

1. Outpatient treatment
2. Consultation with the general practitioner

3. Consultation with the psychologist

4. Family counselling

5. Consultation with the psychiatrist

36. Question
A 35-year-old man suffers from insulin-dependent diabetes mellitus and chronic
cholecystitis. He takes NPH insulin; 20 units in the morning and 12 units in the
evening. After a meal he developed pain in the right subcostal area, nausea, vomiting,
sleepiness and increased polyuria. What prehospital measures will be the most
effective for prevention of crisis within the next several hours?

1. Decrease carbohydrates in the diet

2. Exclude fats from the diet

3. Take cholagogues

4. Take analgesics

5. Change insulin regimen

37. Question
A 43-year-old man complains of a protrusion in the right inguinal region, that
enlarges due to strain. He has been presenting with this condition for 6 months.
Within this period the protrusion has grown. Objectively in the right inguinal region
an elastic protrusion 8×5 cm is visible. On palpation it disappears, leaving an empty
space 4×4 cm between the pedicles of the Poupart ligament. “Cough push” sign is
positive over this opening. Make the diagnosis:

1. Right-sided inguinal lymphadenitis

2. Right-sided reducible inguinal hernia

3. Right-sided reducible femoral hernia

4. Right-sided reducible arcuate line hernia

5. Cyst of the right spermatic cord

38. Question
A 27-year-old man complains of pain in his leg joints, purulent discharge from the
eyes and painful burning sensations during urination. Disease onset was acute. He
has a history of influenza. The patient smokes and drinks alcohol in excess. In his line
of work he is often away on business trips. What is the most likely etiological factor of
this disease?
1. Candida

2. Chlamydia

3. Streptococci

4. Adenovirus

5. Staphylococci

39. Question
A 39-year-old man, a battery attendant, suddenly developed weakness, loss of
appetite, nonlocalized colicky abdominal pains and nausea. Objectively, his skin is
gray; there is a pink-gray stripe on his gums; the stomach is soft and sharply painful.
Blood test detected erythrocytes with basophilic stippling and anemia. The patient
has a history of peptic ulcer disease of the stomach. Constipation tends to occur every
3-4 days. What is the most likely provisional diagnosis?

1. Saturnism (Lead poisoning)

2. Perforation of gastric ulcer

3. Acute cholecystitis

4. Chronic alcoholism

5. Acute appendicitis

40. Question
A 35 year old man complains of rapidly increasing fatigue, palpitations, ‘visual snow’,
and dizziness. He has a history of peptic ulcer of the stomach. Objectively the skin is
pale. Vesicular respiration is observed in the lungs. Systolic murmur is detected over
the cardiac apex, heart rate is 100/min., BP is 100/70 mmHg. The epigastrium is
slightly tender on palpation. Blood test: erythrocytes – 3.2 x 1012/L, Hb – 100 g/L,
color index – 0.95. What type of anemia is the most likely present in this case?

1. Chronic iron-deficiency anemia

2. Hypoplastic anemia

3. Hemolytic anemia

4. Post-haemorrhagic anemia

5. Sideroblastic anemia

41. Question
A 30-year-old woman made an appointment with the family doctor for scheduled
vaccination of her 2-year-old child. What type of healthcare provide such medical
services?

1. Palliative care

2. Tertiary healthcare

3. Secondary healthcare

4. Emergency aid

5. Primary healthcare

42. Question
For three years a 31-year-old woman has been complaining of pain and swelling of
her radiocarpal and metacarpophalangeal articulations and their reduced mobility
in the morning, which persisted upto 1.5 hours. Two weeks ago she developed pain,
swelling and reddening of her knee joints, her body temperature increased upto 37.5 o
The treatment was untimely. Examination of the internal organs revealed no
pathologic alterations. Diagnosis of rheumatoid arthritis was made. What changes
are most likely to be visible on the arthrogram?

1. Numerous marginal osteophytes

2. Epiphyseal osteolysis

3. Cyst in the subchondral bone

4. Joint space narrowing, usuration

5. Joint space narrowing, subchondral osteosclerosis

43. Question
A 2 years old child with persisting cough and sub febrile body temperature after a
case of URTI developed dyspnea, cyanosis of the nasolabial triangle, percussion
dullness and weakened respiration in the lower lobe of the right lung and a slight
mediastinal displacement to the left. What pulmonary pathology is likely to cause this
clinical presentation?

1. Pleurisy

2. Emphysema

3. Pneumonia

4. Atelectasis

5. Bronchitis
44. Question
Employees work in conditions of high dust concentration. Certain chemical (silicon
dioxide content) and physical properties of dust aerosols contribute to the
development of occupational dust induced diseases. What is the main physical
property of dust aerosols?

1. Magnetization

2. Ionization

3. Electric charge

4. Dispersion

5. Solubility

45. Question
A 50-year-old patient was brought to a hospital with complaints of blood in urine.
Urination is painless and undisturbed. Macrohematuria had been observed for 3
days. Objectively: kidneys cannot be palpated, suprapubic area is without
alterations, external genitalia are non-pathologic. On rectal investigation: prostate is
not enlarged, painless, has normal structure. Cystoscopy revealed no changes. What
is the most likely diagnosis?

1. Dystopic kidney

2. Necrotic papillitis

3. Renal carcinoma

4. Varicocele

5. Bladder tuberculosis

46. Question
After eating shrimps, a 25 year old man suddenly developed skin itching, some areas
of his skin became hyperemic or erupted into vesicles. Make the diagnosis:

1. Urticaria pigmentosa

2. Hemorrhagic vasculitis (Henoch-Schonlein purpura)

3. Acute urticaria

4. Scabies

5. Psoriasis

47. Question
A 45-year-old man, a farmer, presents with acute onset of a disease. He complains of
headache, high temperature, pain in the gastrocnemius muscles, icteric face, and
dark urine. Objectively: body temperature – 38°C, blood pressure – 100/70 mm Hg,
conjunctival haemorrhages, hepatosplenomegaly, and oliguria. What is the most
likely provisional diagnosis?

1. Leptospirosis

2. Trichinosis

3. Pseudotuberculosis

4. Viral hepatitis

5. Brucellosis

48. Question
After a pain attack in the right subcostal area, a 58-year-old woman with
overnutrition developed icteric skin and sclera, light colored feces, and dark urine.
Her abdomen is distended and painful on palpation in the right subcostal area.
Palpation detects liver enlarged by 2-3 cm. Blood test: Total bilirubin – 90 mcmol/L,
conjugated bilirubic – 60 mcmol/L. What method of examination will be most
informative for diagnosis clarification?

1. Intravenous cholegraphy

2. US of the hepatopancreatobiliary zone

3. Percutaneous transhepatic cholegraphy

4. Retrograde cholangiopancreatography

5. Infusion cholegraphy

49. Question
Indicators of work of impatient departments in the city hospitals for the past year
were analysed. After that the meeting was held at the central city hospital and based
on the results of the analysis, a decision was made to decrease the number of beds in
the impatient departments and instead open daycare units with partial
hospitalization in the city polyclinics. What is the main goal of this decision?

1. Decrease of the length of impatient stay

2. Decreased mortality during impatient treatment

3. Intensification of the bed fund usage

4. Rational use of the bed fund

5. Optimization of the average bed occupancy rate per year


50. Question
A woman with atopic bronchial asthma was found to have one allergen to dog hair
+++. Carpets are removed from the apartment, the apartment was renovated, and air
conditioner was installed. However, recurrent asphyxia attacks still occurs every
night, despite the patient undergoing pathogenetic therapy. What long term
treatment tactics can help this patient to decrease her sensitivity to the allergen?

1. Antihistamine therapy

2. Continuation of prior treatment

3. Specific hyposensitization

4. Buteryko breathing technique

5. Referral for speleotherapy

51. Question
In the factory cafeteria there was an outbreak of food poisoning. Clinical presentation
indicates staphylococcal etiology of this disease. 15 people are sick. To confirm the
diagnosis of food poisoning, samples need to be sent to the laboratory. What samples
should be obtained for analysis?

1. Saliva

2. Urine

3. Vomit masses

4. Blood for hemoculture

5. Blood

52. Question
A 47-year-old man developed the signs of decompensated laryngeal stenosis against
the background of acute phlegmonous laryngitis. He presents with inspiratory
dyspnea at rest, forced position, cyanotic skin covered in cold sweat, tachycardia,
deficient pulse, and low blood pressure. What urgent treatment tactics should be
chosen?

1. Oxygen therapy

2. Intravenous administration of dehydrating agents

3. Tracheostomy

4. Administration of glucocorticoid hormones


5. Oral administration of hypersensitization substances and broncholytics

53. Question
A 28-year-old man complains of skin rash and itching on the both of his hands. The
condition persists for 1.5 years. The exacerbation of his condition he ascribes to the
occupational contact with formaldehyde resins. Objectively the lesion foci are
symmetrically localized on both hands. Against background of erythema with blurred
margins there are papulae, vesicles, erosions, crusts and scales. What is the most
likely pathology?

1. Allergic dermatitis

2. Erythema multiforme

3. Simple contact dermatitis

4. Idiopathic eczema

5. Occupational eczema

54. Question
During medical examination of high and middle school students, the doctors were
assessing correlation between biological and calendar age of the school students
based on the following criteria; height growth rate per year, ossification of the carpal
bones, the number of permanent teeth. What additional development criterion should
be assessed at this age?

1. Vital capacity of lungs

2. Hand strength

3. Development of secondary sex characteristics

4. Body mass

5. Chest circumference

55. Question
A 63-year-old man complains of unmotivated weakness and pressing and bursting
sensation in the left subcostal area. According to him, these signs have been present
for a year already. Previously he was healthy. He took part in containment measures
during the accident at the Chornobyl Nuclear power plant. Objectively the skin is
pale, peripheral lymph nodes are not enlarged, the liver is +3cm, the spleen is +10cm .
Complete blood count: erythrocytes – 3.1 x 1012/L, Hb- 100g/L, leukocytes – 46 x 109/L,
blasts – 2%, promyclocytes – 10%, myclocytes – 18%, band neutrophils – 27%,
segmented neutrophils – 10%, lymphocytes- 12%, eosinophils – 6%,basocytes – 3%,
monocytes – 2%, erythrocyte sedimentation rate – 20 mm/hour. What is the most
likely diagnosis?
1. Acute leukemia

2. Hepatic cirrhosis

3. Hemolytic anemia

4. Chronic lymphatic leukemia

5. Chronic myeloleukemia

56. Question
The 5-year-old child has been ill for 2 weeks. Cough attacks developed first and were
then followed by reprises. During coughing the child’s face turns red and cervical
veins bulge. The cough attacks induce vomiting, X-ray shows intensified bronchial
pattern. Blood test: leukocytes – 16 x 109/L, lymphocytes – 72%, erythrocyte
sedimentation rate – 4mm/hour. What is the most likely diagnosis?

1. Pneumonia

2. Adenovirus infection

3. Pertussis

4. Obstructive bronchitis

5. Foreign body

57. Question
The burns unit received a patient, who 6 hours ago during a fire received flame burns.
On the patient’s body there is gray-brown area of necrosis that covers ¾ of the body
perimeter. Occasionally there are small blisters with haemorrhagic contents and
patches of shredded epidermis. What local therapy is necessary in this case?

1. Decompression necrectomy

2. Necrectomy with dermal autograft

3. Necrectomy with xenotransplantation

4. Chemical necrolysis

5. Blister puncture

58. Question
A man was brought into the admission room after an overexposure to cold. He
complains of sharp pain in the small of his back and elevated body temperature upto
38 o He took some aspirin. Blood test: leukocyutes – 10.5 x 1012/L, eosinophils – 5%,
band neutrophils – 8%, segmented neutrophils – 51%, lymphocytes – 32%, monocytes
– 4%, erythrocyte sedimentation rate – 28 mm/hour. Urinalysis: Protein – 0.6 g/L,
leukocytes – cover the whole vision field, large amount of mucus. What is the most
likely diagnosis?

1. Subacute malignant glomerulonephritis

2. Tubulointerstitial nephritis

3. Acute glomerulonephritis

4. Acute pyelonephritis

5. Chronic pyelonephritis

59. Question
A 10-year-old boy, who was outdoors in windy and cold weather, developed moderate
pain and tingling in his fingers and toes. When he returned home, his parents noticed
that the tips of his fingers and toes were white and their sensitivity was lost. As the
affected areas were warming up, the fingers and toes developed tingling and painful
sensations. Skin pallor changed into redness, tingling stopped, mild itching and
swelling of fingers appeared. Determine the frostbite degree in this child:

1. Frostbite of the II degree

2. Frostbite of the I degree

3. Frostbite of the IV degree

4. Perniosis

5. Frostbite of the III degree

60. Question
A patient is being treated in the tuberculosis clinic. Throughout the last 3 weeks he
has been suffering from headaches of increasing intensity. Neurological examination
detects nuchal rigidity without focal signs. Make the provisional diagnosis:

1. Convexital arachnoiditis

2. Tuberculous meningitis

3. Myelitis

4. Brain tumor

5. Chorea minor

61. Question
A 59-year-old man complains of pain in his left eye and left side of his head,
significant vision impairment of the left eye, nausea, and vomiting. Visual acuity of
the right eye is 1.0. Visual acuity of the left eye is 0.03, attempts at correction bring no
improvement. Right eye intraocular pressure – 21 mm Hg, left eye intraocular
pressure – 65 mm Hg. Congestive injection is observed on the sclera of the left eye. The
cornea is thick and swollen. The anterior chamber is shallow, moist, and clear. The
pupil is dilated and unresponsive to the light, the fundus of the eye is not visible. What
is the most likely diagnosis?

1. Acute attack of glaucoma of the left eye

2. Acute iridocyclitis of the left eye

3. Stage II intraocular tumor of the left eye

4. Panophthalmitis of the left eye

5. Endophthalmitis of the left eye

62. Question
A middle school teacher with 4 year long record of work was issued a medical
certificate for pregnancy and childbirth leave. What amount of pay will she receive
for the duration of her leave in this case?

1. 50% of average salary

2. 80% of average salary

3. 70% of average salary

4. 60% of average salary

5. 100% of average salary

63. Question
A district doctor has diagnosed one of his patients with dysentery. What accounting
document reflects this type of morbidity?

1. Control card of a patient registered for regular check-ups

2. Certificate of temporary disability

3. Statistical report

4. Urgent report

5. Report on a major non-epidemic disease

64. Question
The body of a 24-year-old woman with suspected poisoning has been found on the
street. Forensic medical examination was requested on the site and the body.
According to the Criminal Procedure Code currently in force in Ukraine, forensic
medical examination is required when it is necessary to determine the:

1. Cause of death

2. Mode of death

3. Time of death

4. Manner of death

5. Mechanism of death

65. Question
A 1-year old child with a case of URTI suddenly developed noisy respirations with
difficult inspiration, intercostal retractions and barking cough on the 2nd night after
the disease onset. What is the most likely diagnosis?

1. Bronchial asthma

2. Acute bronchiolitis

3. Acute pulmonary inflammation

4. Stenosing laryngotracheobronchitis

5. Acute bronchitis

66. Question
A 48-year-old woman developed insomnia, depressive mood, anxiety, fears and
suicidal thoughts after the death of her husband that occurred one month ago. During
her stay in the hospital she speaks in a low voice, is depressed, anxious, avoids
sleeping, refuses to eat. What medications should be prescribed in this case?

1. Anticonvulsants

2. Antipsychotics

3. Antidepressants

4. Group B vitamins

5. Nootropics

67. Question
A 3-week-old infant developed large, flaccid vesicles with purulent contents on the
skin of chest and abdomen. The vesicles rupture quickly. Make the provisional
diagnosis:

1. Pemphigus neonatorum

2. Pemphigus syphiliticus

3. Toxic erythema

4. Pseudofurunculosis

5. Vesiculopustulosis

68. Question
Having studied the relationship between the distance from villages to the local
outpatient clinics and frequency of visits to the clinics among the rural population of
this area, it was determined that the rank correlation coefficient in this case equals
-0.9. how can this relationship be characterized?

1. Moderate inverse relationship

2. Strong direct relationship

3. Moderate direct relationship

4. Strong inverse relationship

69. Question
A 55-year-old woman complaints of pain and popping sounds in her left knee joint,
which occur when she climbs the stairs. Occasionally during movements her joint
becomes “stuck”, 5 years ago she suffered a trauma of her left knee. Complete blood
count and biochemical blood analysis show normal results. X- ray shows marked
osteosclerosis and osteophytes. The joint space is narrowed. Make the provisional
diagnosis:

1. Rheumatoid arthritis

2. Gouty arthritis

3. Osteoarthritis

4. Psoriatic arthritis

5. Reactive arthritis

70. Question
A 6-month-old child on breastfeeding is hospitalized in the inpatient department.
After the child recovers, the doctor recommends the mother to start introducing solid
food to the child’s diet. What products should be introduced to the child’s diet first?

1. Buckwheat porridge

2. Grated apple

3. Vegetable puree

4. Fermented dairy products

5. Semolina porridge

71. Question
A 32-year-old woman complains of body weight loss despite her increased appetite,
nervousness, and tremor if the extremities. Objectively: the skin is moist; the thyroid
gland is diffusely enlarged, painless, soft and mobile. Blood test: increased level of T3,
T4 and thyroid-stimulating hormone(THS). What is the most likely diagnosis?

1. Thyroid adenoma

2. Autoimmune(Hashimoto’s) thyroiditis

3. Diffuse toxic goiter

4. Thyroid carcinoma

5. Diffuse nontoxic goiter

72. Question
A 35-year-old man complains of marked dyspnea and cardiac pain. He describes his
disease to the case of influenza that he had 2 weeks ago. Objectively he leans forward
when sitting. The face is swollen, cyanotic, cervical veins are distended. Heart
borders are extended on both sides, heart sounds are muffled, heart rate = Ps =
118/min, BP is 90/60 mmHg. Blood test: ESR is 46 mm/hour. ECG shows low voltage.
X-ray shows trapezoidal cardiac silhouette and signs of pulmonary congestion.
Choose the treatment tactics:

1. Pericardial puncture

2. Pericardectomy

3. Glucocorticosteriods

4. Diuretics

5. Antibiotics

73. Question
A 32-year-old woman complaints of episodes of intense fear that occur without visible
cause and last for 10-20 minutes; the episodes are characterized by raped pulse,
sweating, labored breathing and vertigo. Specify the likely diagnosis:

1. Manic syndrome

2. Paranoid syndrome

3. Claustrophobia

4. Simple schizophrenia

5. Panic disorder

74. Question
A 40-year-old man with Bekhterev disease (ankylosing spondylitis) complains of
elevated body temperature upto 37.8°C, back pain and stiffness, especially observed
during the second half of the night. This condition has been lasting for 2 years.
Objectively: reduced spinal mobility, painful sacroiliac joint. erythrocyte
sedimentation rate – 45 mm/hour. X-ray shows narrowing of the intervertebral disc
space and of the sacroiliac joint. What eye pathology is often associated with this type
of disease progression ?

1. Optic nerve atrophy

2. Blepharitis

3. Cataract

4. Retinal detachment

5. Iridocyclitis

75. Question
On laboratory investigation of a pork sample there is 1 dead trichinella detected in 24
sections. This meat should be:

1. Frozen until the temperature of -10 oC is reached in the deep layers, with subsequent exposure
to cold for 15 days.

2. Processed and sold through public catering network

3. Allowed for sale with no restriction

4. Sent for technical disposal

5. Processed for boiled sausage production

76. Question
A 3-month-old child with signs of rickets presents with positive Chvostek, Trousseau
and Maslov signs. One day ago the parents witnessed a cyanotic attack in their child
– the child broke into a cold sweat, the eyes bulged and respiratory arrest occurred.
One minute later the child drew in a loud breath and child’s condition normalized
again. What is the cause of the described signs of the disease?

1. Decrease of blood calcium levels

2. Increase of blood phosphorus levels

3. Metabolic acidosis

4. Increase of blood calcium levels

5. Decrease of blood phosphorus levels

77. Question
A 57-year-old woman during a regular ultrasound examination presented with a
space-occupying heterogeneous lesion in the right kidney. What is the most
informative method of renal tumor diagnostic?

1. Excretory urography

2. Spiral computed tomography

3. Three glass urine test

4. Retrograde pyelography

5. Radioisotope renography

78. Question
A 59-year-old woman was brought into the rheumatology unit. Extremely severe case
of scleroderma is suspected. Objectively she presents with malnourishment “mask-
like” face, and acro-osteolysis. Blood: erythrocytes – 2.2 x 109 /L, erythrocyte
sedimentation rate – 40 mm/hour. Urine: elevated levels of free oxyproline. Name one
of the most likely pathogenetic links in this case:

1. Formation of antibodies to transversely striated muscles

2. Formation of antibodies to native DNA

3. Formation of antibodies to vessel wall

4. Formation of antibodies to blood corpuscles

5. Formation of antibodies to collagen

79. Question
During examination a 4-month-old child with meningococcemia presents with
acrocyanosis, cold extremities, tachypnea and thread pulse, anuria and sopor. What
clinical syndrome is it?

1. Neurotoxicosis

2. Toxic shock syndrome

3. Acute renal failure

4. Encephalic syndrome

5. Exicosis

80. Question
A 45-year-old man diagnosed with acute pulmonary abscess suddenly developed
sharp pain in his chest on the right and dyspnea upto 30/min. Examination detects
facial cyanosis and shallow rapid respirations. Auscultation reveals acutely
weakened respiration throughout the whole right lung; percussion reveals a
vesiculotympanititc (bandbox) resonance at the lung apex and dullness in the lower
lobe. What complication developed in this patient?

1. Esophageal perforation

2. Pneumothorax

3. Pyopneumothorax

4. Acute mediasinitis

5. Pleuropneumonia

81. Question
After a long drive with the window open a man developed facial asymmetry. He
cannot close his right eye, his right nasolabial fold is smoothed out, movements of
expression are absent on the right, there is a disturbance of gustatory sensation in the
tongue on the right. No other neurological pathologies were detected. What disease
can be provisionally diagnosed in this patient?

1. Neuropathy of the trigeminal nerve

2. Trigeminal ganglionitis

3. Ischemic stroke

4. Neuropathy of the facial nerve

5. Neuropathy of the oculomotor nerve

82. Question
A 57-year-old patient complains of dyspnea at rest. The patient presents with
orthopnoea, acrocyanosis, bulging cervical veins. On percussion: dull sound over the
lower lung segments. On auscultation: no respiratory sounds. Heart rate is 92/min.
Right-sided cardiac dilatation is observed. The liver is +7cm. Shins are swollen.
Pleural effusion is suspected. What indicator would confirm the presence of
transudate in this case?

1. Presence of atypical cells

2. Total protein content in the pleural fluid below 25 g/L

3. Positive Rivalta’s test

4. Specific gravity exceeding 1015

5. Total protein content in the pleural fluid exceeding 30g/L

83. Question
A chronic alcoholic was hospitalized into the therapeutic inpatient unit due to
pneumonia. On the day 5 of his hospitalization he became disoriented in time and
space, developed fear-inducing visual hallucinations and motor agitation. Full body
tremor and tremor of the limbs are observed. X-ray and physical examination detect
the signs of his convalescence from pneumonia. What tactics should be chosen
regarding his patient?

1. Transfer into the neuroresuscitation department

2. Transfer into the inpatient narcology department

3. Compulsory medical treatment for alcoholism

4. Discharge from the hospital

5. Continue the treatment in the therapeutic department

84. Question
A 25-year-old woman was brought into gynecological department with profuse
bloody discharge from her genital tracts. She is 12 weeks pregnant; the pregnancy is
planned. Within the last 3 days she was experiencing pains in her lower abdomen that
eventually started resembling cramps, she developed bleeding. Her skin is pale, pulse-
88/min., blood pressure- 100/60 mm Hg, body temperature – 36.8°C . Vaginal
examination: the uterus size corresponds with 11 weeks of pregnancy, the cervical
canal allows inserting 1 finger and contains fragments of the fertilized ovum, the
discharge is bloody and profuse. What is the most likely diagnosis?

1. 12-week pregnancy, spontaneous abortion in progress

2. Full-term pregnancy, term labor


3. 12-week pregnancy, threatened spontaneous abortion

4. Disturbed menstrual cycle, amenorrhea

5. Disturbed menstrual cycle, hyperpoylme-norrhea

85. Question
In the impatient gynecological unit within a year 6500 women underwent treatment.
They spent there a total of 102000 bed-days. What indicator of gynecological unit
work can be calculated based on these data?

1. Planned bed occupancy rate per year

2. Number of beds by hospital department

3. Bed turnover rate

4. Average bed occupancy rate per year

5. Average length of inpatient stay

86. Question
A 1.5-month-old child on breastfeeding presents from birth with daily vomiting,
irregular liquid foamy feces and meteorism, which are resistant to antibacterial and
probiotic therapy; no increase of body mass is observed. The child’s condition
improved, when breastmilk was substituted with ‘NAN low lactose’ formula. What
pathology is it?

1. Intestinal lambliasis (Giardiasis)

2. Lactase deficiency

3. Functional dyspepsia

4. Infectious enteritis

5. Drug induced enteritis

87. Question
A 53-year-old man complains of general weakness, loss of appetite, a painful vesicles
appearing on his skin. The disease onset occurred suddenly, after hyperinsolation one
week ago. Examination detects isolated vesicles with wrinkled opercula and
occasional painful erosions on the skin of the patient’s torso and limbs. Nikolsky sign
is positive. What is the most likely diagnosis?

1. Herpes

2. Nonacantholytic pemphigus
3. Toxicodermia

4. Duhring’s disease (Dermatitis herpetiformis)

5. Acantholytic pemphigus

88. Question
A 58 year old woman came to the gynecological clinic. She complains of bloody
discharge from her genital tracts. Menopause is 8 years. Gynecological examination:
the uterus is slightly enlarged, dense to touch, with limited mobility; the uterine
appendages cannot be detected; parametrium is free. Fractional curettage of the
uterine cavity yields a significant amount of medullary substance in the scrape. What
is the most likely diagnosis?

1. Hormone producing ovarian tumor

2. Uterine corpus cancer

3. Adenomyosis

4. Uterine cervix cancer

5. Chorioepithelioma

89. Question
A 40-year-old victim of a traffic accident sustained the following injuries: closed
diaphyseal femur fracture, brain concussion, multiple rib fractures,
hemopneumothorax, degloving shin injuries. What injuries require the most urgent
attention?

1. Degloving shin injuries

2. Multiple rib fractures, hemopneumothorax

3. All injuries are equivalent

4. Closed diaphyseal femur fracture

5. Brain concussion

90. Question
A 37 year old man suddenly developed acute headache accompanied by nausea,
vomiting, and impaired consciousness. Objectively blood pressure is 190/120 mmHg,
the face is hyperemic. Patient’s consciousness is clouded, his answers to the questions
are short, monosyllabic. Movement and sensory disturbances are absent. Meningeal
signs are positive. Cerebrospinal fluid contains blood. What provisional diagnosis
can be made?
1. Meningitis

2. Subarachnoid haemorrhage

3. Encephalitis

4. Cerebral vascular embolism

5. Ischemic stroke

91. Question
A 72-year-old man with pneumonia complains of marked dyspnea, chest pain, severe
cough with expectoration, temperature is 39.5-40.0 oC, no urination for a whole day.
Objectively the patient is conscious. Respiratory rate is 36/min. Over the right lower
pulmonary lobe percussion sound is dull; on auscultation there is bronchial
respiration and numerous moist crackles. BP is 80/60 mmHg. Heart rate is 120/min.
Heart sounds are muffled, there is tachycardia. What tactics should the family doctor
choose in the management of this patient?

1. Hospitalization into the neurology unit

2. Outpatient treatment

3. Hospitalization into the pulmonology unit

4. Hospitalization into the intensive care unit

5. Treatment in the day patient facility

92. Question
A 28-year-old woman complaining of irregular menstruations and infertility came to
gynecological clinic. Menstruations occur since the age of 15, irregular, with delays
up to 2 months. On examination she presents with marked hirsutism and excessive
body weight. On vaginal examination the uterus is reduced in size and painless. The
ovaries on the both sides are dense and enlarged. Ultrasound shows microcystic
changes in the ovaries, the ovaries are 5×4 cm and 4.5× 4 cm in size with dense
ovarian capsule. Basal body temperature is monophasic . What is the most likely
diagnosis?

1. Krukenberg tumor

2. Ovarian carcinoma

3. Bilateral adnexitis

4. Polycystic ovary syndrome

5. Endometrioid cysts

93. Question
10 hours after birth a child developed jaundice, hypotonia, hyporeflexia and moderate
hepatosplenomegaly. Feces and urine are of normal color. Umbilical cord blood
bilirubin is 51 mcmol/L due to unconjugated bilirubin levels. In venous blood:
erythrocytes – 3.5 x 1012/L, Hb – 140 g/L, reticulocytes – 1.5%, bilirubin – 111
mcmol/L, conjugated – 11 mcmol/L, ALT – 40 U/L, AST – 30 U/L. mother’s blood
group is A(II) Rh(-), child’s blood group is A(II) Rh(+). What laboratory test can
confirm the diagnosis?

1. Viral hepatitis markers analysis

2. Coombs test

3. Measurement of erythrocyte osmotic resistance

4. Measurement of glucose 6 phosphate dehydrogenase levels in erythrocytes

5. Erythrocytometry

94. Question
A 27 year old woman, a teacher in the elementary school, complains of frequent
stools, upto 3 times per day, with lumpy feces and large amount of mucus, abdominal
pain that gradually abates after a defecation, irritability. Her skin is pale and icteric.
Pulse is 74/min., rhythmic, can be characterized as satisfactory. Blood pressure is
115/70 mmHg. The abdomen is soft, moderately tender along the colon on palpation.
Fiberoptic colonoscopy detects no changes. What disease can be suspected?

1. Chronic enteritis

2. Crohn disease (regional enteritis)

3. Irritable bowel syndrome

4. Whipple disease

5. Chronic non-ulcerative colitis

95. Question
A woman has been provisionally diagnosed with pheochromocytoma. At the stage of
intermission her BP is within norm; there is tendency towards tachycardia. No urine
pathologies. The decision has been made to perform a provocative test with
histamine. What drug should be kept close at hand for emergency aid in case of
positive test result?

1. Mesaton (Phenylephrine)

2. Prednisolone

3. Nifedipine
4. Pipolphen (Promethazine)

5. Phentolamine

96. Question
A 3-year-old child has been brought to a hospital with complaints of pain in the legs,
fever, and loss of appetite. Objectively: pale skin and mucosa, haemorrhagic rash.
Lymph nodes are enlarged, painless, dense and elastic, not matted together. Bones,
joints, and abdomen are painful. The liver and spleen are enlarged. Hemogram: Hb-
88 g/L, color index – 1.3, platelets – 80 x 109/L, leukocytes – 25.8 x 109/L,
lymphoblasts – 70% , ESR- 52mm/hour. Make the provisional diagnosis:

1. Haemorrhagic vasculitis( Henoch-Schonlein purpura)

2. Acute leukaemia

3. Acute rheumatic fever

4. Thrombocytopenic purpura

5. Infectious mononucleosis

97. Question
A 45-year-old woman complains of intolerable paroxysmal facial pain on the left with
attacks that last for 1-2 minutes. Attacks are provoked by chewing. The disease onset
was two months ago after the overexposure to cold. Objectively: pain at the exit points
of the trigeminal nerve on the left. Touching near the wing of the nose on the left
induces a pain attack with tonic spasm of the facial muscles. What is the most likely
diagnosis?

1. Facial migraine

2. Temporomandibular joint arthritis

3. Trigeminal neuralgia

4. Maxillary sinusitis

5. Glossopharyngeal neuralgia

98. Question
A 22-year-old postparturient woman on the 12th day after normal child birth informs
of elevated body temperature up to 39° C for the last 3 days and pain in her right
mammary gland. The right mammary gland is enlarged, hot to touch, tense,
hyperemic, and painful. Palpation reveals there a dense infiltration 8×8 cm with a
fluctuation in its center. What is the most likely diagnosis?

1. Postpartum period, day 12. Right-sided lactostasis.


2. Postpartum period, day 12. Right-sided serous mastitis.

3. Postpartum period, day 12. Right-sided gangrenous mastitis.

4. Postpartum period, day 12. Right-sided phlegmonous mastitis

5. Postpartum period, day 12. Right-sided infiltrative-purulent mastitis

99. Question
A surgery unit received a person with an incised stab wound on the upper third of the
right thigh. Examination detects an incised stab wound 3.0×0.5×2.0 cm in size on the
inner surface of the upper third of the right thigh. Bright-red blood flows from deep
within the wound in a pulsing stream. Characterize this type of bleeding:

1. Venous

2. Mixed

3. Capillary

4. Arterial

5. Parenchymatous

100. Question
To assess the effectiveness of medical technologies and determine the power and
direction of their effect on the public health indicators, the research was conducted to
study the immunization rate of children and measles incidence rate by district. What
method of statistical analysis should be applied in this case?

1. Calculation of standardized ratio

2. Calculation of morbidity index among the nonvaccinated

3. Calculation of correlation coefficient

4. Calculation of matching factor

5. Calculation of statistical significance of the difference between two estimates

101. Question
A patient has gradually lost consciousness. The skin is pale and dry. There is a smell
of ammonia from the mouth. Respirations are deep and noisy. Heart sounds are
muffled, pericardial friction rub is present. Blood pressure is 180/130 mmHg. Blood
test: Hb – 80 g/L, leukocytes – 12 x 109/L, blood glucose – 6.4 mmol/L, urea – 50
mmol/L, creatinine – 1200 mcmol/L, blood osmolarity – 350 mOsmol/L. No urinary
excretion. Make the diagnosis:
1. Acute disturbance of cerebral circulation

2. Uremic coma

3. Hyperglycemic coma

4. Hyperosmolar coma

5. Acute renal failure

102. Question
A young man has made an appointment with the dermatologist. He complains of a
painful facial rash in the beard and moustache area. This condition has been
persisting for several weeks already. After shaving, the patient’s condition
aggravates. The diagnosis of sycosis is made. What primary morphological elements
can be observed in the rash in this case?

1. Nodes, nodules

2. Maculae, nodes

3. Pustules, papulae

4. Phlyctenae, maculae

5. Pustules, bumps

103. Question
A 46-year-old woman came to the maternity clinic with complaints of moderate blood
discharge from the vagina, which developed after the menstruation delay of 1.5
months. On vaginal examination; the cervix is clean; the uterus is not enlarged,
mobile, painless; appendages without changes. Make the diagnosis:

1. Ectopic pregnancy

2. Adenomyosis

3. Dysfunctional uterine bleeding

4. Cancer of uterine body

5. Submucous uterine myoma

104. Question
A 13-year-old girl for a month has been complaining of fatigability, dull pain in her
right subcostal area, abdominal distension and constipation. Abdominal palpation
reveals positive Kehr, Murphy and Ortner signs, while Desjasrdins and Mayo-Robson
points are painless. Total bilirubin is 14.7 mcmol/L, predominantly indirect, ALT – 20
U/L, AST – 40 U/L, amylase – 6.3 mmol/L. Echocholecytography shows practically no
contraction of gallbladder. Make the provisional diagnosis?

1. Acute pancreatitis

2. Hypokinetic biliary dyskinesia

3. Chronic hepatitis

4. Chronic pancreatitis

5. Hyperkinetic biliary dyskinesia

105. Question
After a surgery for a left thigh phlegmon the disease progression was complicate by
sepsis. On the 7th day after the surgery, there are marked signs of a generalized
inflammatory reaction, in blood there are signs of toxic anemia and progressing
hypoprotieinemia, bilirubin levels are 40 mcmol/L, AST and ALT exceed the norm by
2.5 times. Oliguria persists (700 mL of urine per day). Name the phase of sepsis
progression:

1. Anabolic phase

2. Stress phase

3. Catabolic phase

4. Recovery phase

5. Mixed phase

106. Question
A 10 year old boy was brought into the hospital with complaints of expiratory
dyspnea, respiration are 30/min. He explains his state by a change in the weather
conditions. For the last 4 years the body has been registered for regular check-ups due
to his diagnosis of third degree persistent bronchial asthma. To provide emergency
aid for this child, first he needs to be given:

1. Euphylline (Aminophylline)

2. Salbutamol or short acting B2 agonists

3. Dexamethasone

4. Adrenaline

5. Claritin (Loratadine)

107. Question
A 72 year old man complains of lower extremity edema, sensation of heaviness in the
right subcostal area, dyspnea at rest. For over 25 years he has been suffering from
COPD. Objectively: Orthopnea, jugular venous distension, diffuse cyanosis,
acrocyanosis. Barrel chest is observed, on ‘percussion there is a vesiculotympanitic
(bandbox) resonance, sharply weakened vesicular respiration on both sides, moist
crepitant crackles in the lower segments of the lungs. Heart sounds are weakened, the
II heart sound is accentuated over the pulmonary artery. The liver is +3 cm. What
complicated the clinical course of COPD in this patient?

1. Diffuse pneumosclerosis

2. Pulmonary embolism

3. Acute left ventricular failure

4. Chronic pulmonary heart

5. Community acquired pneumonia

108. Question
A 35 year old woman complains of high body temperature and pain in the upper outer
quadrant of her right buttock, which developed after an injection. She has been
presenting with this condition for 3 days. At the site of injection the skin is hyperemic;
there is painful infiltrate with an area of softening in its center. The woman is
diagnosed with a post injection abscess of the right buttock. What tactics should the
surgeon choose in this case?

1. Abscess incision, sanation and drainage of the cavity

2. Abscess puncture, pus removal followed by application of antiseptics

3. Hospitalization, prescription of antibiotics, UHF

4. Antipyretic agents, massage and application of dry heat to the right buttock

5. 10-15 minutes of low intensity laser radiation directed at the right buttock

109. Question
On the day 4 after the caesarean section a woman developed fever with body
temperature upto 39oC and abdominal pain. Pulse – 104/min. She vomited twice. The
patient is sluggish, her tongue is dry and has gray coating. The abdomen is distended.
Signs of peritoneal irritation are positive in all segments. Peristalsis cannot be
auscultated. No passage of gas occurs. Uterine fundus is located at the level of the
navel. The uterus is painful on palpation. The discharge is moderate and contains
blood and pus. What is the most likely diagnosis?

1. Metroendometritis
2. Progressive thrombophlebitis

3. Pelvic peritonitis

4. Parametritis

5. Diffuse peritonitis

110. Question
A newborn girl has apgar score of 7-8 points at the 1-5 minutes after birth. During the
labor there was a brief difficulty with extraction of the shoulder girdle. After birth the
baby presents with disturbed function of the proximal segment and forced position of
the right arm. The shoulder is rotated inwards, the elbow is extended, the forearm is
pronated and the whole upper limb resembles an arm of a doll. What is the most likely
clinical diagnosis in this case?

1. Osteomyelitis of the right arm

2. Intracranial haemorrhage

3. Thoracic spine trauma

4. Soft tissue injury of the right arm

5. Erb-Duchenne palsy

111. Question
A 38-year-old woman developed a medical condition 7 days after her return from
Bangladesh. Periodical elevation of temperature was accompanied by chills and
excessive sweating. She was diagnosed with tropical malaria. Next day her condition
further deteriorated: body temperature – 38 oC, inertness, periodical loss of
consciousness, generalized seizures, tachycardia, hypotension and icteric skin. What
complication can be suspected in this case?

1. Cerebral coma

2. Acute hepatic failure

3. Acute heart failure

4. Purulent meningitis

5. Serious meningitis

112. Question
A 72 year old man diagnosed with ischemic heart disease presents with diffuse
cardiosclerosis, permanent tachysystolic artrial fibrillation, heart failure IIa, FC III.
Objective examination of the vital signs: blood pressure is 135/80 mmHg, heart rate is
160/min, pulse is 125/min. Left ventricular ejection fraction is 32%, What drug is
indicated in this case and should be prescribed to the patient?

1. Verapamil

2. Procainamide (Novocainamide)

3. Isadrine (Isoprenaline)

4. Digoxin

5. Ivabradine

113. Question
A 16 year old patient has made an appointment with an otolaryngologist. He
complains of elevated body temperature and sore throat. Disease onset was 2 days
ago, after the patient are two portions of ice cream. Pharygoscopy shows hyperemic
mucosa of the palatine tonsils with purulent exudate in the lucanae. Make the
provisional diagnosis:

1. Follicular tonsillitis

2. Diphtheria

3. Acute pharyngitis

4. Pseudomembranous (Vincent’s) tonsillitis

5. Lacunar tonsillitis

114. Question
A 23-year-old man has accidentally swallowed brake fluid. After that he has been
presenting with anuria for 5 days already; his creatinine levels elevated up to
0.569mmol/L . What treatment tactics should be chosen in this case?

1. Plasmapheresis

2. Hemodialysis

3. Diuretics

4. Detoxication therapy

5. Antidotal therapy

115. Question
A 34 year old man complains of pale edema of the face, feet, shins and lumbar area,
elevated BP upto 160/100 mmHg, and general weakness. He has a clinical history of
nonspecific ulcerative colitis. Objectively: pulse – 84/min, rhythmic, blood pressure –
165/100 mmHg; edemas all over the body; the skin is pale and dry, with low turgor.
The kidneys cannot be palpated, on an attempt to palpate them they are painless.
Blood test: erythrocytes – 3.0 x 1012/L, Hb – 100 g/L, erythrocyte sedimentation rate –
50 mm/hour. Urinalysis: proteins – 3.5 g/L, erythrocytes – 7-10 in the vision field,
leukocytes – 5-6 in the vision field. Daily proteinuria – 6 grams. What analysis should
be conducted additionally to verify the diagnosis?

1. Renal ultrasound

2. Radioisotopic examination of kidneys

3. Gingival biopsy for the diagnosis of amyloid disease

4. Survey and excretory urography

5. Urinalysis for Bence-Jones protein

116. Question
An 18-year-old girl was brought into the gynaecology department with complaints of
elevated body temperature up to 37.8°C, sharp pain in her lower abdomen, more
intense on the right and difficult defecation. Vaginal examination detected a painful
dense elastic formation 5×6 cm in the area of her right ovary. Pregnancy test is
negative. What is the most likely diagnosis?

1. Appendicitis

2. Ectopic pregnancy

3. Torsion of ovarian tumor pedicle

4. Ovarian apoplexy

5. Ovarian cyst rupture

117. Question
A 48 year old woman has been hospitalized due to development of tachysystolic atrial
fibrillation. She has lost 5 kg of body weight within 2 months. On palpation, there is
node in the left lobe of the thyroid gland. What pathology resulted in the development
of this condition?

1. Toxic nodular goitre

2. Autoimmune thyroiditis

3. Atherosclerotic cardiosclerosis

4. Chronic thyroiditis

5. Non toxic nodular goitre


118. Question
A 8 year old girl complains of frequent painful urination in small amounts and
urinary incontinence. The signs have been present for 2 days already. She complains
her disease by overexposure to cold. Costovertebral angle tenderness is absent.
Complete blood count is without pathologies. Urine test: leukocytes: 20-30 in the
vision field, erythrocytes: 40-50 in the vision field, unchanged, bacteriuria. What is
the most likely diagnosis?

1. Glomerulonephritis

2. Vulvitis

3. Urolithiasis

4. Pyelonephritis

5. Cystitis

119. Question
A 39-year-old man suffers from chronic rheumatic heart disease. He complains of
dyspnea during physical exertion, cough with expectoration and palpitations.
Auscultation detects intensified I heart sound and diastolic murmur; the sound of
opening mitral value can be auscultated at the cardiac apex. The II heart sound is
accentuated over the pulmonary artery. The patient is cyanotic, X-ray shows dilated
pulmonary root and enlargement of the right ventricle and left atrium. What is the
most likely diagnosis?

1. Patent ductus arteriosus

2. Coarctation of the aorta

3. Mitral stenosis

4. Pulmonary artery stenosis

5. Aortic stenosis

120. Question
During medical examination a cadet in the naval college was detected to have a
painless dense ulcer 1.5×0.5 in size in his perianal area at the 2 O’clock position. The
ulcer floor resembles ‘old fat’. What is the provisional diagnosis?

1. Anal cancer

2. Hard syphilitic chancre of the rectum

3. Anal crypt suppuration

4. Rectal fistula
5. Rectal fissure

121. Question
A 35-year-old patient developed an epileptic attack with tonoclonic spasms that lasted
for 3 minutes. After the attack the patient fell asleep but in 5 minutes the second
attack occurred. The first step of emergency aid would be to:

1. Introduce diazepam intravenously

2. Ensure patency of airways

3. Administer chloral hydrate via an enema

4. Prescribe antiepileptic drugs

5. Take blood from the vein for analysis

122. Question
At night a 2-year-old child with upper respiratory tract infection suddenly developed
dyspnea with labored inspiration. Objectively the skin is pale, perioral cyanosis and
slight acrocyanosis are observed. Breathing is loud, respiration rate is 32/min.
Jugular, supra and infraclavicular fossae retract during breathing. Respiration is
coarse on auscultation. Heart sounds are clear and sonorous, heart rate is 120/min.
What condition was complicated by the development of the upper respiratory tract
infection?

1. Bornchiolitis

2. Bronchial asthma

3. Stenosing laryngotracheitis

4. Obstructive bronchitis

5. Airway foreign body

123. Question
A newborn has Apgar score of 9. When should this infant be put to the breast?

1. On the 3rd day

2. After 2 hours

3. After 12 hours

4. In the delivery room

5. On the 2nd day


124. Question
A 38-year-old patient has been brought by an ambulance to the surgical department
with complaints of general weakness, indisposition, black stool. On examination the
patient is pale, there are dotted haemorrhages on the skin of his torso and extremities.
On digital investigation there are black feces on the glove. Blood test: Hb – 108 g/L,
thrombocytopenia. Anamnesis states that a similar condition was observed 1 year
ago. Make the diagnosis:

1. Non specific ulcerative colitis

2. Thrombocytopenia purpura

3. Hemophilia

4. Ulcerative bleeding

5. Rectal tumor

125. Question
A 30 year old woman came to the gynaecological department. She complains of sharp
pain in her lower abdomen and temperature of 38.8 o She has a history of
extramarital sexual activity and 2 artificial abortions. On gynaecological
examination the uterus is unchanged. The appendages are bilaterally enlarged and
painful. Profuse purulent discharge is being produced from the vagina. What
examination needs to be conducted to clarify the diagnosis?

1. Colposcopy

2. Laparoscopy

3. Bacteriological and bacterioscopic analysis

4. Hysteroscopy

5. Curettage of the uterine cavity

126. Question
A 62-year-old woman was brought into the admission room with complaints of severe
burning retrosternal pain and asphyxia. She has a 10 year long history of essential
hypertension. Objectively her condition is moderately severe. She presents with skin
pallor, cyanotic lips and vesicular respiration over her lungs. The II heart sound is
accentuated over the aorta. Blood pressure – 210/120 mmHg, heart rate (pulse) –
76/min. ECG shows elevation of ST segment in the lead I, AVL and V5-V6. What is the
most likely diagnosis?

1. Hypertensive crisis complicated with acute left ventricular failure

2. Uncomplicated hypertensive crisis


3. Hypertensive crisis complicated with acute myocardial infarction

4. Pulmonary embolism

5. Hypertensive crisis complicated with instable angina pectoris

127. Question
A 13-year-old girl for the last two weeks has been complaining of dyspnea and shin
and foot edemas that appear after physical exertion. In the morning the edemas
significantly decrease. Clinical examination revealed enlarged liver and coarse
systolic murmur over the heart area. Blood test and urinalysis are without changes.
What is the most likely cause of edemas in this child?

1. Nephrotic syndrome

2. Acute pyelonephritis

3. Heart failure

4. Angioneurotic edema

5. Hepatic cirrhosis

128. Question
An 18-year-old patient always obeys others and adapts his needs to the demands of
the people on whom he depends. He excessively defers to their wishes and makes them
responsible for his well being, cannot defend his interests and needs support from
other people. Such psychic profile has been formed in the childhood, remains
unchanged, and hinders adaptation. What psychic disorder is observed in this
patient?

1. Markedly accentuated personality

2. Anxiety (avoidant) personality disorder

3. Psychopathy-like state

4. Dependent personality disorder

5. Anankastic personality disorder

129. Question
A 30-year-old man was brought to the neurosurgical department with complaints of
constant headaches, nausea, vomiting, fever and weakness of the right-side limbs.
Anamnesis states that one month ago the patient had a surgery for left-sided
suppurative otitis and mastoiditis. He has been undergoing treatment in an ENT
department. Approximately 2 weeks ago the temperature increased, and the patient
developed headaches. Objectively: heart rate – 98/min., BP – 140/90 mm Hg,
temperature- 38.3°C. neurologically manifested stiff neck: bilateral Kerning’s
symptom, unsteadiness during Romberg’s maneuver. Computer tomography of the
brain revealed a three- dimensional growth with a capsule in the left hemisphere.
Make the diagnosis:

1. Cerebral abscess

2. Hemorrhage

3. Arnold-Chiari malformation

4. Echinococcus

5. Hydrocephalus

130. Question
A 73-year-old woman came to the family physician for one of her regular follow–up
examinations. Three months ago, she was found to have type 2 diabetes mellitus. She
was keeping to her diet and exercise plan and taking phytopreparations. On
examination her fasting glucose was within the range of 7.8-8.6 mmol/L, HbA1c –
7.9%. Height – 164 cm, weight – 83 kg. What blood sugar controlling medicine should
she be prescribed first in the course of her pharmacological therapy?

1. Glibenclamide

2. Metformin

3. Gliclazide

4. Glimepiride

5. Insulin

131. Question
A 42-year-old man, a worker at the meat processing factory, developed an itching
spot on his lower jaw, which gradually transformed into a slightly painful carbuncle
3 cm in diameter, surrounded by a painless swelling that reaches the clavicle.
Temperature is subfebrile, under 37.8 o The doctor suspects anthrax. What drug
should this man be prescribed for treatment?

1. Biseptol (Co-trimoxazole)

2. Levomycetin (Chloramphenicol)

3. Azidothymidin (Zidovudine)

4. Penicillin

5. Interferon alpha
132. Question
At night a 63-year-old woman suddenly developed an asphyxia attack. She has a 15
year long history of essential hypertension and has a myocardial infarction 2 year
ago. Objectively her position in bed is orthopneic, the skin is pale, the patient is
covered with cold sweat, acrocyanosis is observed. Pulse – 10/min. blood pressure –
210/130 mmHg, respiration rate – 38/min. Pulmonary percussion sound is clear, with
slight dullness in the lower segments; throughout the lungs single dry crackles can be
heard that become bubbling and non-resonant in the lower segments. What is the
most likely complication in this patient?

1. Pulmonary embolism

2. Acute left ventricular failure

3. Bronchial asthma attack

4. Paroxysmal tachycardia

5. Acute right ventricular failure

133. Question
A 25-year-old man was hospitalized with complaints of pain in his lower abdomen
and right lumbar area that appeared one hour ago. Patient’s general state is
moderately severe. Body temperature – 38.2 oC, heart rate – 102/min. Tongue is dry.
The abdomen is painful on deep palpation in the right iliac area and in the petit
triangle. Aure-Rozanov and Gabay signs are positive. Make the provisional diagnosis:

1. Intestinal obstruction

2. Right sided renal colic

3. Acute appendicitis

4. Acute cholecystitis

5. Cecal tumor

134. Question
In a rural health care area there is an increasing cervical cancer morbidity observed.
The decision is made to conduct a medical examination of the women living in this
locality. What type of medical examination is it?

1. Prelimiary

2. Regular

3. Screening

4. Target
5. Complex

135. Question
A 45 year old woman came to the maternity clinic with complaints of periodical pains
in her mammary glands that start 1 day before menstruation and stop after the
menstruation begins. Palpation of the mammary glands detects diffuse nodes
predominantly in the upper outer quadrants. What is the most likely diagnosis?

1. Fibrocystic mastopathy

2. Hyperprolactinemia

3. Mastitis

4. Breast cyst

5. Breast cancer

136. Question
During analysis of morbidity in the city, it was determined that age structure of
population is different in each district. What statistical method allows to exclude this
factor, so that it would not skew the morbidity data?

1. Analysis of average values

2. Dynamic time series analysis

3. Wilcoxon signed rank test

4. Correlation regression analysis

5. Standardization

137. Question
An 11 year old boy for a month has been presenting with increasing pain in the right
femur. In the painful area there is a non mobile painful tumor with unclear margins.
The child complains of general indisposition, weakness, increased body temperature
up to 39oC. X-ray shows widened medullary cavity, small foci of cancellous bone
destruction, and onion like lamellar exfoliation of the cortical layer. What is the most
likely pathology resulting in such clinical presentation?

1. Fibrosarcoma

2. Osteogenic sarcoma

3. Ewing sarcoma

4. Juxtacortical sarcoma
5. Chondrosarcoma

138. Question
During regular preventive gynaecological examination a 30 year old woman was
detected to have dark blue punctulated ‘perforations’ on the vaginal portion of the
uterine cervix. The doctor suspects endometriosis of the vaginal portion of the uterine
cervix. What investigation method would be most informative for diagnosis
confirmation?

1. Hysteroscopy

2. Colposcopy, target biopsy of the cervix

3. Curettage of the uterine cavity

4. US of the lesser pelvis

5. Hormone testing

139. Question
A 42 year old man, a dispatcher, suffers from peptic ulcer disease of the duodenum.
The disease is of moderate severity. He wants to be assigned a disability group. Make
the conclusion regarding his working ability:

1. Capable of working, employable

2. First group of disability

3. Third group of disability

4. Capable of working, non-employable

5. Second group of disability

140. Question
A 27 year old woman complains of foul smelling discharge from her genital tracts,
pain in her lower abdomen and elevated temperature. The complaints arose 2 days
ago. She has a history of surgical abortion at the term of 8 weeks one week ago.
Mirror examination: the uterine cervix is clear, external orifice produces foul smelling
discharge. Vaginal examination: the uterus lies in anteflextion, is mobile, painful, and
slightly enlarged. The appendages are without changes. Make the provisional
diagnosis:

1. Salpingoophoritis

2. Acute respiratory disease

3. Postabortal endometritis
4. Appendicitis

5. Enterocolitis

141. Question
The dermatologist has an appointment with a 30-year-old man that complains of
severely itching rashes that especially disturb him at night. The rashes developed 2
week ago, after he had returned from a travel. Objectively on the lateral surfaces of
his fingers, hands, writs, elbows, lower abdomen, genitals and thighs there are paired
papulovesicles, single pustules, and scratch marks. What disease can be suspected?

1. Eczema

2. Scabies

3. Pyoderma

4. Dermatitis

5. Shingles

142. Question
A 20 year old man, a calibrator of dosimetry equipment, committed a gross violation
of safety regulations, when he put two ampoules with cobal-60, each with the
radioactivity of 7 MCi, in the pockets of his trousers and jacket. He has been keeping
the ampoules in his pocket for 8 hours. The tissue at the distance of 0.5 cm from the
source received the dose of 30 Gy (3000 R), while the tissues at the distance of 20 cm –
2 R. Did this man develop radiation sickness?

1. Yes, he developed chronic radiation syndrome

2. Yes, he developed a moderate form of acute radiation syndrome

3. Yes, he developed a severe form of acute radiation syndrome

4. Yes, he developed a mild form of acute radiation syndrome

5. No, he did not

143. Question
A 7-year-old boy has severe pulmonary mucoviscidosis (cystic fibrosis). He complains
of dyspnea and blood expectoration. Objectively he presents with lagging physical
development, acrocyanosis, hepatomegaly, drumstick fingers, and nail plates
resembling a “clock face”. Provisional diagnosis of chronic examination would be the
most informative for diagnosis confirmation?

1. Chest X-ray
2. Electrocardiography

3. Rheography of the pulmonary artery

4. Doppler echocardiography

5. Ultrasound of the liver

144. Question
A 48-year-old woman has arrived to the surgical unit with wounds in her thigh. On
examination the wound surface has dirty-gray coating with unpleasant sweet smell.
Wound content resembles raspberry jelly. Skin tissues around the wound are glossy
and turgid. Palpation reveals moderate crepitation in the tissues. What microflora is
the most likely to cause such inflammation?

1. Blue push bacillus

2. Staphylococci

3. Anaerobic non-clostridial

4. Streptococci

5. Anaerobic clostridial

145. Question
A woman with the pregnancy term of 8 weeks complains of elevated temperature upto
37.6°C, skin rash that can be characterized as macular exanthema, enlargement of
posterior cervical and occipital lymph nodes, small amount of bloody discharge from
the genital tracts. She was examined by the infectious diseases specialist and
diagnosed with rubella. What tactics should the obstetrician-gynaecologist choose?

1. Prescription of hemostatic therapy

2. Prescription of antiviral therapy

3. Abortion

4. Prescription of antibacterial therapy

5. Treatment of incipient abortion

146. Question
A 58-year-old man complains of weakness and tumor like formations that appeared
on the anterior surface of his neck and in the inguinal region. Palpation detects soft
painless mobile cervical and inguinal lymph nodes up to 2 cm in diameter. The liver
protrudes by 2 cm from the edge of the costal margin, the lower splenic pole is at the
umbilical level. In blood: erythrocytes – 3.5 x 1012/L, Hb – 88 g/L, leukocytes – 86 x

9
109/L, band neutrophils – 1%, segmented neutrophils – 10%, lymphocytes – 85%,
eosinophils – 2%, basocytes – 0%, monocytes – 2%, erythrocyte sedimentation rate –
15 mm/hour, Gumprecht shadows. What is the most likely diagnosis?

1. Acute leukemia

2. Chronic myeloleukemia

3. Chronic lymphatic leukemia

4. Lymphocytic leukemoid reaction

5. Lymphogranulomatosis

147. Question
A 65-year-old man complains of dyspnea, severe cough with expectoration of small
amount of blood streaked sputum, weight loss, body temperature 37.2 oC, loss of
appetite and weakness. He has been suffering from this condition for years. The
patient’s condition deteriorated one year ago, dyspnea developed 3 weeks ago. The
patient is a lifelong smoker. He is a carpenter by occupation. Objectively he is of
normal body type but emaciated. Right side of the chest is retracted, excursions are
limited, accessory muscles take part in the respiration, respiratory rate is 22/min.
Percussion detects dull sound over the right upper segment. Chest X-ray shows
shrunken right upper lobe with homogeneous shadow connected to the root of the
lung; the root is deformed; mediastinal organs are displaced to the right. What is the
most likely diagnosis?

1. Pulmonary sarcoidosis

2. Fibrosing alveolitis

3. Obstructive atelectasis

4. Pulmonary tuberculosis

5. Complete right-sided pneumothorax

148. Question
A 30-year-old man came to the family physician. 2 months ago he underwent a
surgery for open fracture of the humerus. On examination the patient’s condition is
satisfactory; in the area of postoperative wound there is a fistula that discharges a
small amount of pus; the area itself is red; fluctuation is detected. X-ray shows
destruction of the humerus with sequestra. What complication did the patient develop
during the postoperative period?

1. Suture sinus

2. Post traumatic osteomyelitis


3. Hematogenous osteomyelitis

4. Post traumatic phlegmon

5. Wound suppuration

149. Question
A woman is 40 weeks pregnant. The fetus is in the longitudinal lie and cephalic
presentation. Pelvic size: 26-29-31-20. Expected weight of the fetus is 4800 gram. The
labor contractions has been lasting for 12 hours, within the last 2 hours they were
extremely painful, the parturient woman is anxious. The waters broke 4 hours ago.
On external examination the contraction ring is located 2 finger widths above the
navel, Henkel-Vasten sign is positive. Fetal heart rate is 160/min, muffled. On internal
examination the uterine cervix is fully open, the head is engaged and pressed to the
entrance into the lesser pelvis. What is the most likely diagnosis?

1. Anatomically contracted pelvis

2. Complete uterine rupture

3. Hyperactive uterine contractions

4. Threatened uterine rupture

5. Abruption of normally positioned placenta

150. Question
During regular medical examination a lyceum student presents with signs of cheilitis
that manifests as epithelial maceration in the area of lip seal. The lips are bright-red,
with single vertical cracks covered with brown-red scabs. These clinical signs are
most likely caused by insufficient content of the following in this diet:

1. Ascorbic acid

2. Calciferol

3. Thiamine

4. Retinol

5. Riboflavin

151. Question
A 13-year-old girl has 30% of excessive body mass, she started to gain weight at the
age of 3. She has a family history of obesity. Her height and sexual development are
normal for her age. The appetite is excessive. She complains of periodical headaches.
Blood pressure – 120/80 mmHg. Subcutaneous fat is evenly distributed, she has no
stretch marks. There is juvenile acne on her face. What type of obesity is it?
1. Hypothalamic obesity

2. Hypothalamic syndrome of puberty

3. Alimentary constitutive obesity

4. Adrenal obesity

5. Hypothyroid obesity

152. Question
After overexposure to cold a 45 year old woman developed acute pain in her
suprapubic and lumbar areas during urination, sharp pains at the end of urination,
false urges to urinate. Urine is turbid with blood streaks. The doctor suspects urinary
tract infection. What results of laboratory analysis would be the most indicative of
such infection?

1. Daily proteinuria over 3.0

2. Leukocyturia gross hematuria

3. Gross hematuria

4. Increased blood creatinine and blood urea

5. Daily proteinuria under 3.0

153. Question
A 26 year old woman is suspected to suffer from systemic lupus erythematosus due to
systemic lesions of skin, vessels, joints, serous tunics and heart that developed after
photosensitization. The following is detected: LE cells, antibodies to native DNA,
isolated anti-centromere antibodies, rheumatoid factor is 1:100, Wassermann
reaction is positive, circulating immune complex is 120 units. What immunological
indicators are considered to be specific to this disease?

1. Increased circulating immune complex

2. DNA antibodies

3. Immunoglobulin A

4. Anti centromere antibodies

5. Rheumatoid factor

154. Question
A 45-year-old man underwent a cardiac surgery one week ago. His general state has
been deteriorating since then; dyspnea at rest, retrosternal pain that irradiates to the
neck, marked weakness. Objectively his body temperature is hectic. His cardiac
borders are expanded, apical beat is weakened. Auscultation detects pericardial
friction rub. What is the most likely diagnosis?

1. Acute myogenic dilation of the heart

2. Acute cardiac aneurysm

3. Acute pericarditis

4. Myocardial infarction

5. Pulmonary embolism

155. Question
A 40 year old man, a welder, uses manganese electrodes in his line of work (18 year of
experience). He complains of difficulties with writing, bad mood, inertness, gait
abnormalities, problems with speech, and hand tremors. Objectively the following is
observed in the patient: hypomimia, increased muscle tone of plastic type, and quiet
monotonous speech, tremor of the tongue, pill-rolling tremor of the fingers, and
retropulsion. What syndrome developed in this patient due to manganese poisoning?

1. Polyneuritic syndrome

2. Vestibular syndrome

3. Meningism

4. Parkinsonism

5. Hypothalamic syndrome

156. Question
During an outdoors school event in hot weather, a 10 year old girl lost her
consciousness. Body temperature – 36.7 o Objectively her skin is pale and cold to
touch, her pupils are dilated. Blood pressure – 90/50 mmHg. Heart rate – 58/min.
What pathology, occurred in this case?

1. Sympathicotonic collapse

2. Paralytic collapse

3. Syncope

4. Sunstroke

157. Question
A 37-year-old patient complains of pain in the spinal column, reduced mobility. The
condition persists for 7 years. ‘Sway back’ is observed, there is no movement in all
spinal regions. X-ray shows ‘bamboo spine’ vertebral column. What is the most likely
diagnosis?

1. Osteochondrosis

2. Tuberculous spondylitis

3. Spondylolisthesis

4. Ankylosing spondyloarthritis

5. Spondylitis deformans

158. Question
A 5-year-old child was brought to the ENT department by an ambulance. The child
presents with cough and difficult respiration. From the patient’s history it is known
that the child was playing with a toy construction set, when suddenly started
coughing and developed labored breathing. Examination detects periodical cough,
labored expiration and respiratory lag in the left side of the child’s thorax.
Auscultation: diminished respiration on the left. Percussion: tympanitis. X-ray shows
a displacement of the mediastinal organs to the right. Make the diagnosis:

1. A foreign body in the trachea

2. A foreign body in the right bronchus, valvular bronchostenosis

3. A foreign body in the left bronchus, valvular bronchostenosis

4. A foreign body in the left bronchus, complete bronchostenosis

5. A foreign body in the right bronchus, partial bronchostenosis

159. Question
A woman undergoing in patient treatment for viral hepatitis type B developed
headache, nausea, recurrent vomiting, memory lapses, flapping tremor of her hands
and rapid pulse. Sweet smell from her mouth is detected. Body temperature is 37.6 oC,
heart rate is 89/min. What complication developed in the patient?

1. Hypoglycemic shock

2. Meningoencephalitis

3. Acute liver failure

4. Gastrointestinal haemorrhage

5. Ischemic stroke

160. Question
A patient has the second and third degree burns of the 15% of the body surface. On the
20th day after the trauma the patient presents with sharp increase of body presents
with sharp increase of body temperature, general weakness, rapid vesicular
respiration; facial features are sharpened, BP is 90/50 mmHg, heart rate is 112/min.
What complication is it?

1. Pneumonia

2. Sepsis

3. Anaerobic infection

4. Purulent bronchitis

5. Acute intoxication

161. Question
A 39-year-old man suffers from chronic adrenal insufficiency and receives
replacement glucocorticoid therapy (hydrocortisone – 15 mg/day). He is to undergo
elective surgery for calculous cholecystitis. What medication adjustment should be
made on the day of the surgery to prevent the development of acute adrenal
insufficiency?

1. Add a mineralocorticoid

2. Cancel the drug for the day of the surgery

3. Add an antibiotic

4. Increase the dosage by 2-3 times

5. Prescribe a large volume intravenous fluid infusion

162. Question
A 15 year old girl complains of dizziness and sensation of lack of air that she develops
in emotionally straining situations. Relief occurs after she takes corvalol. Objectivrly:
hyperhidrosis and marble-like pattern of the skin of her palms and feet. Clinical and
instrumental examination revealed no organic changes in the central nervous,
cardiovascular, and respiratory systems. What provisional diagnosis can be made?

1. Acute epiglottitis

2. Stenosing laryngotracheitis

3. Obstructive bronchitis

4. Bronchial asthma

5. Somatoform autonomic dysfunction


163. Question
A 23-year-old man complains of facial edemas, headache, dizziness, low urinary
output and urine discoloration (dark red). These complaints arose after a case of
acute tonsillitis. On examination there are facial edemas, the skin is pale, temperature
is 37.4 oC, heart rate is 86/min, blood pressure is 170/100 mmHg. Heart sounds are
muffled, the II heart sound is accentuated over the aorta. What etiological factor is
the most likely in this case?

1. Streptococcus viridans

2. Staphylococcus saprophyticus

3. Beta hemolytic streptococcus

4. Staphylococcus aureus

5. Streptococcus pyogenes

164. Question
Increased general morbidity of the local population is observed in the area near a
factory, where atmosphere is being intensively polluted with sulphurous gas. What
effect does polluted air have on human body in this case?

1. Selective

2. Chronic specific

3. Chronic non specific

4. Acute specific

5. Acute non specific

165. Question
A 43-year-old man, a coal face worker with 15 year long record of work, complains of
cough, thoracic pain and dyspnea. The cough is mild, usually dry, occurs mostly in
the morning. The pain is localized in the interscapular region and aggravates during
a deep intake of breath. Dyspnea occurs during physical exertion. Vesicular
respiration in the lungs is weakened. Heart sounds are rhythmic, heart rate is
86/min, blood pressure is 135/80 mmHg. The abdomen is soft and painless. X-ray
shows micronodular pulmonary fibrosis. Make the provisional diagnosis.

1. Berylliosis

2. Byssinosis

3. Siderosis

4. Metal pneumoconiosis
5. Carboconiosis

166. Question
A newborn with gestational age of 31 weeks presents with hypotonia and depressed
consciousness. Hematocrit is 35%, general cerebrospinal fluid analysis shows
increased content of erythrocytes and protein, and low glucose. These data
correspond with clinical presentation of:

1. Sepsis

2. Intrauterine infection

3. Meningitis

4. Intracranial haemorrhage

5. Anemia

167. Question
Mother of a 5-year-old child noticed on the head of her child a round ‘bald’ spot 3 cm
in diameter. All the hairs in the focus are broken off at the lengths of 5-6 mm. The day
before the child was petting a stray cat. Make the diagnosis:

1. Deep trichophytosis

2. Alopecia areata

3. Microsporia

4. Psoriasis

5. Superficial trichophytosis

168. Question
Clinical trials have proved the ‘Lipoflacon’ drug to be effective for treatment of
unstable angina pectoris in the control group and experimental group of patients.
Neither patients nor researchers knew who belonged to which group. Name this type
of study:

1. Double blind study

2. Multicenter study

3. Total blind study

4. Simple blind study

5. Triple blind study


169. Question
A 7 year old girl has been twice treated with antibacterial agents for urinary tract
infection. US shows no severe renal defects. The child presents with recurrence of
leukocyturia and bacteriuria, elevated body temperature upto 38.5 oC, and pain in
her left lumbar area. What examination should be conducted first to clarify the cause
of urinary infection recurrence?

1. Micturating cystourethrography

2. Immunogram

3. Retrograde pyelography

4. Excretory urography

5. Radioisotope renography

170. Question
A 45 year old woman is registered for regular check ups due to Werlhof disease
(immune thrombocytopenia). Complete blood count: Hb – 100 g/L, erythrocytes – 2.8
x 1012/L, platelets – 90.0 x 109/L, leukocytes – 8.4 x 109/L, erythrocyte sedimentation
rate – 13 mm/hour. Examination detects a single small hematoma on the anterior
surface of the thigh, developed after the patient accidently stumbled on a table. What
treatment tactics should be chosen in this case?

1. Urgently start a hemostatic therapy, followed by a planned hospitalization into the haematology
unit.

2. Urgent hospitalization into the haematology unit

3. Administer thrombolytic mass, continue with treatment in the haematology unit

4. Urgent hospitalization into the general care unit

5. Continue the supervision by the hospital haematologist

171. Question
A 9-month-old infant presents with delayed tooth eruption and fontanel closure,
weakness and excessive sweating. What type of hypovitaminosis is the most likely in
this child?

1. Hypovitaminosis A

2. Hypovitaminosis C

3. Hypovitaminosis B1

4. Hypovitaminosis D
5. Hypovitaminosis B6

172. Question
At the railroad crossing a passenger train collided with a bus. In this collision 26 bus
passengers died, another 18 passengers received mechanical injuries of varying
severity. Where will be professional medical aid provided for the victims of this
accident. Who will provide this aid?

1. At the site of the accident; specialized second response emergency teams

2. At the site of the accident; first response emergency teams

3. In medico prophylactic institutions; general physicians and surgeons

4. In medico-prophylactic institutions; specialized second response emergency teams

5. In medical institutions; all listed type of healthcare workers

173. Question
A 3 year old child presents with dyspnea that abates in the sitting position, occasional
loss of consciousness and seizures, delayed physical development, cyanosis,
drumstick fingers. Echocardioscopy detects aortic dextroposition, ventricular septal
defect, pulmonary artery stenosis, and right ventricular hypertrophy. What is the
most likely diagnosis?

1. Ventricular septal defect

2. Coarctation of the aorta

3. Transposition of the great vessels

4. Tetrad of Fallot

5. Acquired valvular disease

174. Question
A 46-year-old woman has Diarrhea with abdominal distension, loss of body mass,
and large amounts of porridge like foul smelling stool without blood steaks or
tenesmus. Objectively examination detects moderate tenderness in the mesogastrium
and left abdominal flank. Feces analysis detects steatorrhea with neutral fat and
creatorrhea. What prescription would be the most advisable in this case?

1. Metronidazole and Loperamide

2. Antacids and antispasmodics

3. Cholinergic antagonists and antibacterial agents


4. Cholinergic antagonists

5. Multi enzyme preparations

175. Question
3 hours after a trauma, a young man developed bradycardia of 46/min, anisocoria
D>S, hemi hyperreflexia S>D, hemiphypesthesia on the left, and a convulsive
disorder. The character of this process needs to be clarified. What method of
examination will be most accurate for this purpose?

1. Lumbar puncture

2. Electroencephalography

3. Echoencephalography

4. Brain CT

5. Skull X-ray

176. Question
On the 15th day after a small trauma of the right foot, the patient developed
indisposition, fatigability, irritability, headache, elevated body temperature, and
sensation of constriction, tension and twitching in the muscles of the right shin. What
disease can be suspected?

1. Acute thrombophlebitis

2. Tetanus

3. Anaerobic gas gangrene

4. Thrombophlebitis of the popliteal artery

5. Erysipelas

177. Question
Human body receives from the atmosphere a number of chemicals. What type of
action results in the combined effects that is less than the sum of isolated effects of
these chemicals on the body?

1. Antagonism

2. Complex action

3. Isolated action

4. Potentiation
5. Synergistic action

178. Question
It is the 3rd day after the first normal term labor; the infant is rooming in with the
mother and is on breastfeeding. Objectively; the mother’s general condition is
satisfactory. Temperature is 36.4 oC, heart rate is 80/min, BP is 120/80 mmHg.
Mammary glands, are soft and painless; lactation is moderate, unrestricted milk
flow. The uterus is dense, the uterine fundus is located 3 finger widths below the
navel. Lochia are sanguine-serous, moderate in volume. Assess the dynamics of
uterine involution:

1. Pathologic involution

2. Physiological involution

3. Lochiometra

4. Hematometra

5. Subinvolution

179. Question
A 7-year-old boy has been an inpatient for 1.5 months. He had been brought to the
hospital with complaints of edemas all over his body, low urine output and headache.
Clinical urinalysis: proteins – 7.1 g/L, leukocytes – 1-2 in the vision field, erythrocytes
– 3-4 in the vision field. During the course of treatment, the edemas gradually
dissipated, headache abated, diuresis normalized. Daily urine proteins – 3g/L.
Biochemical blood test; total protein – 43.2 g/L, urea – 5.2 mmol/L, cholesterol – 9.2
mmol/L. What glomerulonephritis syndrome is the most likely to be present in the
patient?

1. Isolated urinary

2. Hematuric

3. Nephrotic

4. Mixed

5. Nephritic

180. Question
A 17-year-old girl has made an appointment with doctor. She plans to begin her sex
life. No signs of gynecological pathology were detected. In the family history the
patient’s grandmother had cervical cancer. The patient was consulted about the
maintenance of her reproductive health. What recommendation will be most helpful
for prevention of invasive cervical cancer?
1. Vaccination against human papillomavirus (HPV)

2. Vitamins, calcium, omega 3

3. Antiviral and antibacterial drugs

4. Immunomodulators

5. Timely treatment of sexually transmitted diseases

181. Question
A 20-year-old student after failing an exam developed complaints of a sensation of a
round foreign body in her throat, difficult swallowing. She fixates on her condition,
limits her diet, often cries, seeks attention, exhibits demonstrative attitude. She is
highly susceptible to psychotherapeutic suggestion. What psychiatric diagnosis can
be made in this case?

1. Paranoid personality disorder

2. Obsessive neurosis

3. Hypochondriacal neurosis

4. Hysterical neurosis

5. Depressive neurosis

182. Question
A 26-year-old man complains of chills, rhinitis, dry cough and fever up to 38 o
Examination shows him to be in a moderately severe condition; there are small pale
pink non-merging spots on the skin of his back, abdomen and extremities. Palpation
reveals enlarged occipital and axillary lymph nodes. No information about
vaccination history could be obtained. What is the likely etiology of this disease?

1. Streptococcus

2. Epstein-barr virus

3. Mumps virus

4. Rubella virus

5. Neisseria meningitis

183. Question
A 30-year-old maltigravida has been in labour for 18 hours. 2 hour ago the pushing
stage began. Fetal heart rate is clear, rhythmic, 136/min. Vaginal examination
reveals complete cervical dilation, the fetal head in the pelvic outlet plane. Sagittal
suture is in line with obstetric conjugate, the occipital fontanel is near the pubis. The
patient has been diagnosed with primary uterine inertia. What is the further tactics of
labor management?

1. Skin head Ivanov’s forceps

2. Labour stimulation

3. Cesarean section

4. Outlet forceps

5. Vacuum extraction of the fetus

184. Question
A 26-year-old woman presents with amenorrhea. 10 months ago she gave birth for a
second time. In her early postpartum period she developed a massive hypotonic
haemorrhage. No breastfeeding. Lately she has been presenting with loss of weight,
loss of hair and indisposition. Gynecological examination revealed atrophy of the
external genitals, the uterus is abnormally small, no uterine appendages can be
detected. What is the most likely diagnosis?

1. Physiological amenorrhea

2. Suspected progressing ectopic pregnancy

3. Stein-Leventhal syndrome (Polycystic ovary syndrome)

4. Galactorrhea-amenorrhea syndrome

5. Sheehan syndrome (Postpartum pituitary gland necrosis)

185. Question
A woman has been working as a polisher for a year and half. Her workstation is
equipped with a grinding machine (grinding wheels). She complains of white
discoloration of her fingers and toes that appears when she is nervous. Objectively
there are no changes in the coloration of the distal segments of her limbs. Grip
strength measured with a dynamometer is 25kg, algesimetry findings are 0.1; 0.3;
0.5. Cold stimulus is extremely positive on the upper and lower limbs. Internal organs
are without pathogens. Make the diagnosis:

1. Raynaud disease

2. Raynaud syndrome

3. Vibration disease

4. Syringomyelia

5. Polyneuritis
186. Question
A 78 year old man with a prostate adenoma underwent a herniotomy for a direct
inguinal hernia. After the surgery he presents with absent urination. Enlarged
urinary bladder is detectable above the patient’s pubis. What measures should be
taken in this case?

1. Prescribe proserin (neostigmine) intramuscularly

2. Bladder catheterization

3. Prescribe antispasmodics subcutaneously

4. Apply cold to the urinary bladder area

5. Prescribe processing of the postoperative wound with UHF field

187. Question
A 34 year old man on the 3rd day of ceftriaxone treatment for acute otitis (daily
dosage – 2 grams) developed Diarrhea occurring 5-6 times per day. Feces are without
mucus or blood admixtures. Temperature is 36.6 o Gregersen reaction (Occult blood in
feces) is negative. Stool culture detected no pathogenic germs. What is the most likely
cause of Diarrhea in this case?

1. Crohn’s disease (Regional enteritis)

2. Intestinal dysbiosis

3. Ulcerative colitis

4. Antibiotic associated diarrhea

5. Bacterial overgrowth syndrome

188. Question
A 23 year old woman came to the gynaecological clinic. She complains of pain,
itching, and burning in her vulva, general weakness, indisposition, elevated body
temperature upto 37.2 oC, and headache. On examination in the vulva there are
multiple vesicles upto 2-3 mm in diameter with clear contents against the background
of hyperemia and mucosal edema. Make the provisional diagnosis:

1. Cytomegalovirus infection

2. Vulvar cancer

3. Primary syphilis

4. Papillomavirus infection
5. Genital herpes infection

189. Question
A woman came to the doctor with complaints of increased body temperature upto 37.8
oC and moderately sore throat for the last 3 days. Objectively; mandibular lymph

nodes are enlarged upto 3cm. Palatine tonsils are hypertrophied and covered with
gray coating that spreads to the uvula and anterior pillars of the fauces. What is the
most likely diagnosis?

1. Infectious mononucleosis

2. Pseudomembranous (Vincent’s) tonsillitis

3. Oropharyngeal diphtheria

4. Agranulocytosis

5. Oropharyngeal candidiasis

190. Question
Heart X-ray of a 31-year-old man has been revealed the following: with tightly filled
opacified esophagus there is a marginal filling defect in its middle third on the
posterior wall; the defects is 1.8×1.3 cm in size with clear oval border. Mucosal folds
are retained and envelop the defect; wall peristalsis and elasticity are not affected.
There are no complaints regarding the condition of the patient’s alimentary canal.
Make the provisional diagnosis:

1. Achlasia cardiae

2. Barrett esophagus

3. Diverticulum

4. Esophageal tumor

5. Esophageal burns

191. Question
A 38-year-old woman works in flax processing, she dries flax. She came to the hospital
complaining of difficult breathing, constricting sensation in her chest and cough
attacks. These signs appear on the first day of her working week and gradually
diminish on the following days. What respiratory disease is likely in this case?

1. Byssinosis

2. Allergic rhinopharyngitis

3. Bronchial asthma
4. Silicosis

5. Asthmatic bronchitis

192. Question
A 10 year old girl exhibits high level of physical development, her body length
increased by 10 cm within a year (which is double the norm of her age group), the
number of permanent teeth corresponds with the age norm (20), the development of
her secondary sex characteristics is 3 years ahead of her age (Ma, P, Ax, menarche).
Development rate ahead of her biological age can occur due to:

1. Certain components of her diet

2. Sports training

3. Endocrine disorders

4. Acceleration

5. Deficient hygienic education

193. Question
A 45 year old man was brought by an ambulance into the emergency hospital. He
complains of sudden pain in the lumbar area, frequent painful urination and
vomiting. Examination detects pain in the lumbar area, costovertebral angle
tenderness, pain on palpation of kidneys and along the ureter on the right. Urine test:
proteins, fresh erythrocytes, leukocytes. Make the provisional diagnosis:

1. Acute renal failure

2. Acute glomerulonephritis

3. Polycystic kidney disease

4. Acute pyelonephritis

5. Urolithiasis, renal colic

194. Question
A 65 year old woman on abdominal palpation presents with a tumor in the umbilical
region and above it; the tumor is 13×8 in size, moderately painful, non-mobile,
pulsing. On auscultation systolic murmur can be observed. What is the most likely
diagnosis?

1. Asteriovenous aneurysm

2. Abdominal aortic aneurysm

3. Bicuspid insufficiency
4. Gastric tumor

5. Tricuspid insufficiency

195. Question
Clinical statistical investigation was performed to determine effectiveness of a new
pharmacological preparation for patients with ischemic heart disease. What
parametric test (coefficient) can be used to estimate the reliability of the results?

1. Wilcoxon signed rank test

2. Kolmogorov-Smirnov test

3. Sign test

4. Matching factor

5. Student’s t-distribution

196. Question
A 33-year-old man developed multiple rashes on the skin of his torso and extensor
surfaces of his upper and lower limbs. The rashes itch and occasionally fuse together
and form plaques. The elements of rash are covered with silver-white fine scales that
easily flake off when scratched. Grattage test results in three sequential phenomena:
Stearin spot, terminal film and punctate haemorrhage. What diagnosis can be
suspected?

1. Secondary popular syphilid

2. Lichen ruber planus

3. Pyoderma

4. Psoriasis

5. Parapsoriasis

197. Question
Disease onset was acute. A child developed general weakness, pain in the joints and
elevated temperature. Later these signs became accompanied by itching skin rash
manifested as erythematous spots 2-5 mm in size. The rash gradually turned
hemorrhagic. Large joints are painful and swollen; pain attacks periodically occur in
the paraumbilical area; there are signs of intestinal haemorrhage. What is the most
likely diagnosis?

1. Streptococcal impetigo

2. Hemorrhagic meningoencephalitis
3. Rheumatism

4. Hemorrhagic vasculitis (Henoch-Schonlein purpura)

5. Scarlet fever

198. Question
A prurient woman is 30-year-old, stage I of the labour is ongoing. The fetus is in the
cephalic presentation. Auscultation of the fetal heart sounds detects bradycardia.
Evaluation of cardiotocogram yielded the following date: decreased of basal heart
rate down to 90/min, variability – monotonous (2 and less); late decelerations with
amplitude of 50/min. Make the diagnosis and choose the obstetrical tactics necessary
in this case:

1. Fetal distress. Forceps delivery

2. Fetal distress. Stimulation of uterine contraction

3. Fetal distress. Vacuum extraction delivery

4. Normal condition of the fetus. Vaginal birth.

5. Fetal distress. Urgent caesarean section delivery

199. Question
A 56 year old woman was diagnosed with stage 2 hypertension of the 2nd She belong
to the group of moderate risk and has bronchial asthma. What group of drugs is
CONTRAINDICATED to this patient ?

1. Imidazoline receptor antagonists

2. B blockers

3. Calcium antagonist

4. Angiotensin converting enzyme inhibitors

5. Diuretics

200. Question
A 16 year old girl has primary amenorrhea, no pubic hair growth, normally
developed mammary glands; her genotype is 46 XY; uterus and vagina are absent.
What is your diagnosis?

1. Testicular feminization syndrome

2. Mayer-Rokitansky-Kuster-hauser syndrome

3. Cushing disease
4. Cushing syndrome

5. Sheehan syndrome

Kiev eneutron.info@gmail.com

Created by Eneutron Team


Krok 2 – 2020 (General Medicine)

Krok 2 Telegram Channel

Results
2 of 130 questions answered correctly

Your time: 01:36:54

You have reached 2 of 130 points, (1.54%)

RESTART QUIZ VIEW QUESTIONS

1. Question
A 65-year old man underwent a left hemicolectomy due to a malignant tumor in the
descending colon. On the 4th day after the surgery he developed pain and edema in his
left shin. The Homans sign is positive on the left. What postoperative complication
developed in this patient?

1. Acute disturbance of the cerebral blood flow

2. Acute postoperative thrombosis of the deep veins in the left shin

3. Postoperative allergic reaction

4. Acute postoperative lymphangitis of the left shin

5. Acute postoperative thrombosis of the popliteal artery on the left

2. Question
A 31-year-old drug-addicted person complains of a cough with bloody expectorations,
dyspnea, persistent fever, and leg edemas. The jugular veins are distended. There is a
coarse pansystolic murmur detected above the base of the xiphoid process and in the
second intercostal space on the left, close to the edge of the sternum. Heart sounds are
clear, arrythmia is detected, heart rate is 128/min, pulse-82/min, blood pressure is
100/70 mmHg. What is the most likely diagnosis?

1. Infective endocarditis

2. Pulmonary embolism

3. Community acquired pneumonia

4. Coarctation of the aorta

5. Lutembacher syndrome
3. Question
A 55-year-old woman came to a gynecologist with complaints of leukorrhea and
bloody discharge from the vagina after 5 years of menopause. Anamnesis states no
pregnancies. Bimanual examination: the uterus and uterine appendages are without
changes. During diagnostic curettage of the uterine cavity the physician scrapped off
a sample of encephaloid matter. What is the most likely diagnosis in this case?

1. Cervical carcinoma

2. Subserous uterine myoma

3. Adenomyosis

4. Ovarian carcinoma

5. Endometrial carcinoma

4. Question
A multigravida, labor II, 36-37 weeks of gestation, has gone into labor. Her waters
broke 8 hours ago, the labor activity continues for the last 4 hours, it is regular, with
contractions that last 35 seconds and occur every 3 –4 minutes. The child is in the
cephalic presentation, with the head pressed to the entrance into the lesser pelvis. The
parturient woman complains of a sudden sharp abdominal pain. Her pulse is100/min,
blood pressure is 110/70- 100/70 mmHg. The uterus is tense and does not relax
between the contractions. Fetal heartbeat is muffled- 100/min. The amniotic fluid is
blood-colored and continues to leak. What is the most likely diagnosis?

1. Partial placenta previa

2. Cervical rupture

3. Rupture of the umbilical vessels

4. Uterine rupture

5. Premature detachment of the normally positioned placenta

5. Question
A 56-year-old woman has been working as a disinfector for 19 years. She complains of
general weakness, nausea, bitter taste in her mouth, heavy sensation in her right
subcostal area, and rapid fatigability. Objectively, her body temperature is 37.1 C, the
sclerae are icteric, and the liver is enlarged. Total bilirubin is 40 mcmol/L. What is the
likely diagnosis?

1. Acute cholecystitis

2. Occupational toxic hepatitis


3. Biliary dyskinesia

4. Chronic pancreatitis

5. Chronic cholecystitis

6. Question
A 16-year-old boy developed dizziness. His heart rate is 35/min, blood pressure is
85/45mmHg, heart borders are not enlarged. Heart sounds are loud and clear. ECG
shows P waves disconnected from QRS complex, dissociation and different rhythm of
atria and ventricles is accompanied by varying location of P wave in relation to QRST
complex. This presentation is the most characteristic of the following disease:

1. Sinus bradycardia

2. Complete atrioventricular block (III block)

3. Atrioventricular dissociation

4. Extrasystole

5. Atrioventricular block (II degree)

7. Question
A 48-year old woman was delivered into the surgical unit with wounds in her thigh.
On examination the wound surface has a dirty-gray coating with unpleasant sweet
smell. The wound content resembles a raspberry jelly. Skin tissues around the wound
are glossy and turgid. Palpation reveals moderate crepitation in the tissues. What
bacteria is the most likely to cause such inflammation?

1. Anaerobic clostridial

2. Staphylococci

3. Blue pus bacillus

4. Anaerobic non-clostridial

5. Streptococci

8. Question
A district doctor has diagnosed one of his patients with dysentery. What accounting
document reflects this type of morbidity?

1. Urgent report

2. Report on a major non-epidemic disease

3. Certificate of temporary disability


4. Statistical report

5. Control card of a patient registered for regular check-ups

9. Question
A 45-year old man has been suffering from ankylosing spondylitis for 15 years. For
the last 3 years he has been noticing facial swelling and edemas of the limbs.
Objectively he assumes a “beggar’s” position. X-ray shows “bamboo spine” changes in
the thoracic and lumbar segments. Heart ultrasound shows aortic regurgitation.
Complete blood count: Hb-106g/L; leukocytes- 8.9 x 109 /L; ESR-40mm/hour. Daily
proteinuria-9.6 grams per 24 hours. Blood creatinine-230 mcmol/L. What is the cause
of kidney failure in this case?

1. Medicine side effects

2. Renal amyloidosis

3. Pyelonephritis

4. Concomitant heart disease

5. Urolithiasis

10. Question
A 45-year old man came to the hematologist with complaints of general weakness,
elevated body temperature, excessive sweating, enlarged cervical lymph nodes.
Objectively, his body temperature is 37.5 C, the skin is pale and dry, the posterior
cervical lymph nodes are dense and elastic, upto 2 cm in diameter, mobile. There are
no peculiarities in the patient’s heart and lungs. Hepatosplenomegaly was detected.
What examination is necessary to determine the scope of pathologic process?

1. Ultrasound of the cervical lymph nodes

2. Bone scintigraphy

3. Abdominal X-ray

4. Complete blood count

5. Computed tomography

11. Question
A 35-year old woman had acute onset of the disease that started with fever up to 39.0C
and cough. 3days later a dyspnea at rest increased to 35/minute. Downward from her
right shoulder blade angle, percussion detects a dull sound. No vocal fremitus,
respiratory sounds cannot be auscultated. What is the treatment tactics?

1. Pleural tap
2. Oxygen therapy

3. Artificial lung ventilation

4. Physiotherapy

5. Antibiotic therapy

12. Question
A woman complains of frequent watery stool (up to 9-10 times per day) with mucus
and blood admixtures, dull pain in the hypogastrium, weight loss of 4kg within the
last year. Objectively, malnutrition, dry skin, low turgor, aphthous stomatitis. The
abdomen is soft, the sigmoid colon is spastic and painful on palpation. Occult blood
test is positive. Fibro colonoscopy; edema, hyperemia, mucosal granulation,
pseudopolyps, small ulcers with irregular edged. Make the diagnosis:

1. Nonspecific ulcerative colitis

2. Irritable bowel syndrome

3. Crohn's disease (regional enteritis)

4. Colon cancer

5. Chronic enterocolitis

13. Question
A patient is 45 year old. He was referred for a consultation with a psychiatrist due to
complains of abdominal pain and discomfort that occur in emotional staining
situation. Objectively, no changes of the gastrointestinal tract was detected. The
complaints emerged over 10 years ago against the background of severe alcohol
poisoning. The patient has been repeatedly visiting gastroenterologists, who were
unable to find any significant changes in the patient. The prescribed therapy was
ineffective. What is the likely conclusion?

1. Functional dyspepsia

2. No disorder

3. Chronic alcoholism

4. Somatoform autonomic dysfunction

5. Organic brain disorder

14. Question
A 54-year-old woman complains of fogged vision in her right eye, rainbow circles in
her vision, headache, and nausea. Within the last month she twice experienced a
similar condition, but back the all the signs eventually disappeared and her sight was
restored. Currently, all the signs have beed persisting for over 2 days. Objectively, the
patient has eyelid edema, congestive injection of the eyeball, corneal opacity, shallow
anterior chamber of the eye, and dilated pupil that is unresponsive to the light. Her
intraocular pressure is 48mmHg. Make the diagnosis:

1. Cyclitis

2. Iritis

3. Iridocyclitis

4. Keratitis

5. Glaucoma

15. Question
On the 5th day after giving birth a post parturient woman complains of a pain in her
left mammary gland and body temperature up to 38.1C. Examination shows that her
mammary gland is enlarged and painful on palpation, the nipple is edematous and
has fissures, the upper external quadrant of the gland is hyperemic. Name the
measures that would have prevented the development of this complication in the
patient

1. Constant expression of breast milk

2. Feeding no longer that 10 minutes through an overlay

3. Feeding on schedule

4. Stop breastfeeding when fissures appear

5. Feeding on demand, expression of breast milk, prevention of nipple fissures

16. Question
A woman with the pregnancy term of 8 weeks complains of elevated temperature up
to 37.6 C, skin rash that can be characterized as macular exanthema, enlargement of
posterior cervical and occipital lymph nodes, small amount of bloody discharge from
the genital tracts. She was examined by the infectious diseases specialist and
diagnosed with rubella. What tactics should be obstetrician- gynecologist choose?

1. Treatment of insipient abortion

2. Abortion

3. Prescription of hemostatic therapy

4. Prescription of antiviral therapy

5. Prescription of antibacterial therapy


17. Question
After a long drive with the window open a man developed a facial asymmetry; he
cannot close his right eye, his right nasolabial fold is smoothed out, movements of
expression are absent on the right, there is a disturbance of taste sensation in the
tongue on the right. No other neurological abnormalities were detected. What disease
can be provisionally diagnosed in this case?

1. Ischemic stroke

2. Neuropathy of the trigeminal nerve

3. Neuropathy of the facial nerve

4. Trigeminal ganglionitis

5. Neuropathy of the oculomotor nerve

18. Question
A 5-year-old child had acute onset of the disease that manifested in body temperature
up to 39.5 C, marked chills, weakness, inertness, skin pallor, and headache. 8 hours
later a hemorrhagic rash developed on the skin of the buttocks and les. The child is
sluggish, the body temperature has dropped, blood pressure is 80/40, respirations
are 28-30/min, diuresis is decreased. Make the provisional diagnosis:

1. Measles

2. Reye syndrome

3. Hemorrhagic vasculitis (Henoch-Schonlein purpura)

4. Meningococcemia

5. thrombocytopenic purpura

19. Question
A 17-year-old girl complains of a pain in her knee and ankle joints and body
temperature up to 39c . 2 weeks ago, she had a case of acute tonsillitis. Objectively,
her joints are swollen, sharply painful and their mobility is reduced. On the skin of
her truck and limbs there are barely visible circle-shaped pale pink spots. Heart rate
is 95/min, blood pressure is 90/60 mmHg, heart sounds are weakened, there is a soft
systolic noise over the apex. Make the provisional diagnosis:

1. Systemic scleroderma

2. Reactive arthritis

3. Acute rheumatic fever

4. Rheumatoid arthritis
5. Erythema nodosum

20. Question
After semolina was introduced into the diet, a1-year old child for 2 months has been
presenting with loss of appetite, irritability, loss of body mass, and loss of previously
learned skills. The feces are copious and foul smelling. The skin is pale and dry, the
hair is brittle. The abdomen is distended, while the limbs are thin. Stool test shows
high levels of fatty acids. What is the most likely diagnosis?.

1. Lactase deficiency

2. Celiac disease

3. Irritable bowel syndrome

4. Functional diahhrea

5. Mucoviscidosis

21. Question
A patient was brought into the pulmonology department with complaints of
inspiratory dyspnea and dry cough at the highest point of inhalation. On the
examination the following is observed: pale skin, cyanotic lips, “Hippocratic fingers”.
Auscultation detects Velcro-type crackles (like opening a Velcro fastener). X- ray
shows a “ground glass opacity” pattern. What is the most likely diagnosis?

1. Exogenous allergic alveolitis

2. Hand-Schuller-Christian disease

3. Idiopathic pulmonary hemosiderosis

4. Idiopathic pulmonary fibrosis

5. Pulmonary histiocytosis X

22. Question
A 38-year-old woman has been suffering from glomerulonephritis for 20 years. For
approximately 16 years she has been presenting with progressing renal parenchymal
arterial hypertension that became refractory and accompanied by leg edemas. She
receives a combination of 100mg losartan and 20mg lercanidipine with insufficient
antihypertensive effect. What medicine can she be recommended for intensification of
the antihypertensive effect of her therapy?

1. Doxazosin

2. Bisoprolol
3. Torasemide

4. Urapidil

5. Lisinopril

23. Question
A 26-year-old man complains of chills, rhinitis, dry cough, and fever up to 38C.
Examination shows him to be in a moderately severe condition; there are small pale
pink non-merging spots on the skin of his back, abdomen, and extremities. Palpation
reveals enlarged occipital and axillary lymph nodes. No information about
vaccination history could be obtained. What is the likely etiology of this disease?

1. Epstein-Barr virus

2. Streptococcus

3. Rubella virus

4. Mumps virus

5. Meningococcus

24. Question
A 20-year-old woman complains of headaches, vertigo, tearfulness, vomiting, pain in
the area of the heart, and tachycardia. The signs appear 6-7 days before menstruation
and disappear in the first days of menstruation. Make the diagnosis

1. Metabolic craniopathy

2. Diencephalic syndrome

3. Stein-leventhal syndrome

4. Premenstrual syndrome

5. Algomenorrhea

25. Question
During examination of a patient, the doctor detected in him disorders of the eyes
(hemeralopia, Bitot’s spots), skin and skin appendages, mucosa, ang gastrointestinal
tract. He was provisionally diagnosed with Prasad’s syndrome. What causes the
development of this pathology?

1. Vanadium deficiency

2. Copper deficiency

3. Zinc deficiency
4. Manganese deficiency

5. Iron deficiency

26. Question
A 19-year-old girl complains of moderate, itching and hair loss on her head.
Objectively, on the skin of her occipital region there is a single round erythematous
focus 3 cm in diameter with clear margins. Asbestos- like scales can be observed on
the surface of the lesion. The hair in the focus of the lesion is broken off at the length of
6-8 cm. What is the most likely diagnosis?

1. Trichophytosis

2. Microsporia

3. Scabies

4. Seborrhea

5. Psoriasis

27. Question
On ultrasound of the thyroid gland. A 47-year-old woman presents with a hypoechoic
node 1.6 cm in diameter with blurred margins and intranodular
hypervascularization. The doctor suspects thyroid carcinoma. What method should
be used to verify the diagnosis?

1. Case monitoring

2. Thyroid scintigraphy

3. Fine-needle aspiration biopsy

4. Position emission tomography (PET)

5. Determine TSH levels in the blood

28. Question
A 55-year old woman complaints of thyroid gland enlargement that can be observed
throughout the last two years and a discomfort during swallowing. Objectively, she
has signs of hypothyroidism. The thyroid gland on palpation is dense, non-fused with
the surrounding tissues and mobile on swallowing. The regional lymph nodes are not
enlarged. In the serum there are antithyroid antibodies detected. What is the most
likely diagnosis?

1. Midline cervical cyst

2. Endemic goiter
3. Thyroid cancer

4. Hashimoto’s thyroiditis

5. Acute thyroiditis

29. Question
A 39-year-old man, a battery attendant suddenly developed weakness, loss of
appetite, nonlocalized colicky abdominal pains, and nausea. Objectively, his skin, its
grey, there is pink-gray stripe on his gums; the abdomen is soft and sharply painful.
Blood test detected erythrocytes with basophilic stippling and anemia. The patient
has a history of peptic ulcer disease of the stomach. Constipations occur each 3-4
days. What is the most likely provisional diagnosis?

1. Acute cholecystitis

2. Performation of gastric ulcer

3. Acute appendicitis

4. Saturnism (lead poisoning)

5. Chronic alcoholism

30. Question
A 58-year old man complains of weakness and tumor-like formations that appeared
on the anterior surface of his neck and in the inguinal region. Palpation detects soft
painless mobile cervical and inguinal lymph nodes up to 2cm in diameter. The liver
protrudes by 2cm from the edge of the costal margin, the lower splenic pole is at the
umbilical level. In blood: erythrocytes- 3.5 x 1012/L, Hb- 88 g/L, leukocytes- 86 x 109/L,
band neutrophils- 1%, segmented neutrophils-10%, lymphocytes- 85%, eosinophils-2%,
basophils-0%, monocytes-2%, erythrocyte sedimentation rate- 15 mm/hour,
Gumprecht shadows. What is the most likely diagnosis?

1. Lymphogranulomatosis

2. Acute leukemia

3. Lymphocytic leukemoid reaction

4. Chronic myeloid leukemia

5. Chronic lymphocytic leukemia

31. Question
The director of a medical facility draws up a financial plan for the next year. To
improve the economic well-being of his establishment, he decided to increase the
amount of medical services provided. How will it change the fixed cost per unit of
service?
1. Decrease

2. There is no correlation between these variables

3. Increase

4. No changes

5. Fluctuate

32. Question
A 9-year-old boy fell off a tree and hit the back of his head. A brief loss of
consciousness was observed. The child’s condition is satisfactory, he has a headache
and vertigo. Skull X-ray scans shows depressed fracture of the occipital bone in the
area of the external occipital protuberance. What treatment tactics is indicated for
this patient?

1. Hemostatic therapy

2. Complex conservative treatment

3. Anti-inflammatory therapy

4. Lumbar puncture to relieve the pressure

5. Surgical intervention

33. Question
A 45-year old man with thrombophlebitis of the deep veins in his legs suddenly after
physical exertion developed sharp pain in his thorax on the right, dyspnea, and
hemoptysis. Objectively his condition is severe; he presents with acrocyanosis,
shortening of pulmonary percussion sound on the right, and weakened respiration.
Respiration is 30/min, blood pressure is 110/80mm Hg. ECG shows sinus tachycardia,
his heart rate is 120/min, the electrical axis of the heart deviates to the right. S1-Q3.
What is the most likely diagnosis?

1. Cancer of the right lung

2. Spontaneous pneumothorax

3. Community acquires right sided pneumonia

4. Pulmonary embolism

5. Right-sided exudative pleurisy

34. Question
A 45-year old man underwent a cardiac surgery one week ago. His general state has
been deteriorating since then: dyspnea at rest, retrosternal pain that irradiates to the
neck, marked weakness. Objectively, his body temperature is hectic. His cardiac
borders are expanded, apical beat is weakened. Auscultation detects pericardial
friction rub. What is the most likely diagnosis?

1. Acute cardiac aneurism

2. Acute pericarditis

3. Myocardial infarction

4. Pulmonary embolism

5. Acute myogenic dilation of the heart

35. Question
Examination detected vesicles with seropurulent content on the neck, back of the
head, and buttocks of an infant on the 4th day of life. The patient’s condition is
satisfactory, the child is active, all newborn reflexes can be fully induced, the
umbilical cord is at the stage of mummification, the umbilical area is without any
peculiarities. What disease can be suspected?

1. Neonatal pemphigus

2. Phlegmon

3. Miliaria

4. Epidermolysis bullosa

5. Vesiculopustulosis

36. Question
A woman complains of a severe pain in her throat on the left, difficult swallowing and
mouth opening, elevated body temperature, and general malaise. The onset of the
disease was 4 days ago after a case of tonsillitis. Examination detects a trismus of the
masticatory muscles; the left tonsil is displaced toward the midline; the anterior
palatal arch is infiltrated and protruding. The regional lymph nodes on the right are
enlarged and painful on palpation. Make the diagnosis:

1. Tonsillar tumor

2. Infectious mononucleosis

3. Peritonsillar abscess

4. Lacunar tonsillitis

5. Acute pharyngitis
37. Question
A 27-year-old man complains of pain in his leg joints, purulent discharge from the
eyes, and painful burning sensations during urination. The disease onset was acute.
He has a history of influenza. The patient smokes and drinks alcohol in excess. In his
line of work, he is often away on business trips. What is the most likely etiological
factor of this disease?

1. Adenovirus

2. Streptococci

3. Candida

4. Staphylococci

5. Chlamydia

38. Question
A 26-year-old woman was hospitalized into the gynecological department with the
complains of body temperature up to 38.2 C, fever, general weakness, and dirty-red
blood discharge from her genital tracts. She is hemodynamically stable. Two days
ago, she underwent a medical abortion on the 8th week of pregnancy. Ultrasound
detects the remains of the fertilized egg in her uterine cavity. What are the tactics of
the patient management in this case?

1. Revision of the uterine cavity with vacuum aspirator. Antibiotic therapy

2. Pipelle biopsy

3. Laparotomy, extirpation of the uterus and tubes. Abdominal drainage

4. Laparotomy, supravaginal uterine amputation. Abdominal drainage

5. Uterine cavity treatment with antibiotic solutions

39. Question
A 35-year old forestry officer was delivered to the hospital on the 7th day after the
onset of the disease. He complains of chills, elevated body temperature up to 40.0 C,
sharp headache, and myalgias. On examination his face is puffy and hyperemic, the
tongue is dry, “chalk-dusted”. In the left inguinal area, a sharply painful
conglomeration of enlarged lymph nodes can be palpated. The skin over the
conglomeration is hyperemic and tense. What etiotropic therapy should be prescribed
to this patient?

1. Ketoconazole

2. Administration of heterologous serum


3. Streptomycin

4. Human immunoglobulin intravenously

5. Ribavirin

40. Question
A 48-year old woman has been hospitalized due to development of tachysystolic atrial
fibrillation. She has lost 5 kg of body weight within 2 months. On palpation there is a
node in the left lobe of the thyroid gland. What pathology resulted in the development
of this condition?

1. Chronic thyroiditis

2. Toxic nodular goiter

3. Nontoxic nodular goiter

4. Atherosclerotic cardiosclerosis

5. Autoimmune thyroiditis

41. Question
A pregnant woman of 32 weeks of gestation underwent dopperometry of umbilical
circulation, which revealed a reverse diastolic component. Objectively, the height of
the uterus is 27 cm above the pubic bone, head of the fetus is mobile and located above
the entrance to the lesser pelvis. Fetal heartbeat is 136/min. Vaginal examination
shows that the uterine cervix is closed, its length is 3 cm. What tactics should the
obstetrician choose?

1. Urgent cesarean section

2. Fetal biophysical profile

3. Ultrasound photometry of the fetus

4. Labor induction with oxytocin

5. Repeated dopplerometry next day

42. Question
A newborn has a round red formation in the suprapubic region. Examination shows
that urine is being discharged in pulses from the two orifices located in the lower part
of this formation. Name this developmental anomaly:

1. Vesico-umbilical fistula

2. Bladder exstrophy
3. Bladder diverticulum

4. Bladder agenesis

5. Urachal cyst

43. Question
A 2.5-year –old child is ill for the second day. The onset of the disease was associated
with the temperature up to 37.8 C, a single bout of vomiting, a watery diarrhea up to 5
times per day. During the second day, vomiting occurred twice, body temperature is
38.0C, the child has low appetite, watery diarrhea continues. The treatment of the
child should start with the following:

1. Prescribe loperamide

2. Prescribe nifuroxazide

3. Prescribe oral rehydration

4. Prescribe polymyxin

5. Prescribe ceftriaxone

44. Question
A 28-year old man, a teacher, after an emotional stress developed painful muscle
spasms in his right hand that occur during writing; now he has to hold the pen
between the second and the third fingers. He has no problems with typing or writing
on the black board, no other motor disturbances or neurological pathologies are
detected. What is the most likely diagnosis?

1. Parkinsonism

2. Cortical agraphia

3. Writer's cramp

4. neuropathy of the right ulnar nerve

5. Neuropathy of the right radial nerve

45. Question
Children from a certain township presents with brittle teeth, malocclusion, dental
enamel erosions, and dental pigmentation that looks like yellow- brown spots. What
is the likely cause of this presentation?

1. High levels of nitrates

2. High levels if fluorine in water


3. Low levels of fluorine in water

4. Low levels of sulfates in water

5. High levels of sulfates in water

46. Question
The 5-year old child has been ill for 2 weeks. Cough attacks developed first and were
then followed by reprises. During coughing the child’s face turns red and cervical
veins bulge. The cough attacks induce vomiting. X-ray shows intensified bronchial
pattern. Blood test: leukocytes-16 x 109/L, lymphocytes-72%, erythrocyte
sedimentation rate-4mm/hour. What is the most likely diagnosis?

1. Pneumonia

2. Pertussis

3. Adenovirus infection

4. Obstructive bronchitis

5. Foreign body

47. Question
A 13-year- old girl for a month has been complaining of fatigability, dull pain in her
right subcostal area, abdominal distension, and constipations. Abdominal palpation
reveals positive Kehr, Murphy, and Ortner signs, while Desjardins and Mayo-Robson
points are painless. Total bilirubin is 14.7 mcmol/L, predominantly indirect, ALT-20
U/L, AST- 40 U/L, amylase – 6.3 mmol/L. Echo cholecystography shows practically
no contraction of the gallbladder. Make the provisional diagnosis:

1. Chronic hepatitis

2. Hypokinetic biliary dyskinesia

3. Hyperkinetic biliary dyskinesia

4. Acute pancreatitis

5. Chronic pancreatitis

48. Question
During regular examination of a 2-year old boy, he presents with enlarged left
kidney, painless on palpation. The right kidney was undetectable on palpation.
Excretory urography shows no contrast on the right. Cytoscopy detected hemi
atrophy of the urinary bladder trigone, the right ureteral orifice is not detected. What
pathology is it?
1. Agenesis of the right kidney

2. Hypoplasia of the right kidney

3. Agenesis of the right ureter

4. Ectopic right ureteral orifice

5. Dystopia of the right kidney

49. Question
On laboratory investigation of a pork sample there is 1 dead trichinella detected in 24
sections. This meat should be:

1. Frozen until the temperature of -10 degree C is reached in the deep layers, with subsequent
exposure to cold for 15 days

2. Handed over for technical disposal

3. Allowed for sale with no restrictions

4. Processed and sold through public catering network

5. Processed for boiled sausage production

50. Question
A 45-year-old woman complains of increasing body weight throughout the last year.
Examination revealed moon face syndrome, brittle hair, hirsutism, stretch marks on
the abdomen, and disproportionally thin limbs. The patient’s height is 162 cm, her
body weight is 94 kg, her body mass index is 35.8 kg/m2. What type of obesity it?

1. Cerebral

2. Dysplastic

3. Gynoid

4. Android

5. Alimentary-constitutive

51. Question
A 25-year-old woman was brought into the gynecological department with profuse
bloody discharge from her genital tracts. She is 12 weeks pregnant, the pregnancy is
planned. Within the last 3 days she was experiencing pains in her lower abdomen that
eventually started resembling cramps, ahe developed bleeding. Her skin is pale,
pulse-88/min, blood pressure-100/60mmHg, body temperature –36.8C. Vaginal
examination; the uterus size corresponds to 11 weeks of gestation, the cervical canal
allows inserting 1 finger and contains fragments of the fertilized ovum, the discharge
is bloody and profuse. What is the most likely diagnosis?

1. Full-term pregnancy, term labor

2. 12-week pregnancy, spontaneous abortion in progress

3. 12-week pregnancy, threatened spontaneous abortion

4. Disturbed menstrual cycle, hyperpolymenorrhea

5. Disturbed menstrual cycle, amenorrhea

52. Question
A 68-year old woman with congestive heart failure and left ventricular ejection
fraction of less than 40% receives the following pharmacotherapy scheme: ramipril,
torasemide, bisoprolol, clopidogrel and digoxin. During one of her regular
examination, frequent polymorphic ventricular extrasystole were detected in the
patient. What medicine should be removed from her therapy scheme?

1. Clopidogrel

2. Digoxin

3. Ramipril

4. Bisoprolol

5. Torasemide

53. Question
40% of workers who, who polish the art glass, using an abrasive disk, and have a long
record of employment, are diagnosed with ulnar neuritis, 21%- with vegetative
polyneuritis, and 12% – with vegetomyofascitis of the upper limbs. These pathologies
are associated with the following harmful factor:

1. Noise

2. Vibration

3. Electromagnetic field

4. Dust

5. Microclimate

54. Question

nd
A 23-year-old woman with type 1 diabetes mellitus during the 2nd week of community-
acquired pneumonia developed nausea and vomiting. In the evening she has lost her
consciousness and was hospitalized. Objectively, the patient’s skin is pale and dry.
Her respiration is long, the tongue is dry, with brown deposit. Her heart rate is
129/min, blood pressure is 85/50mmHg. Palpation of the patient’s abdomen provokes
no response. the liver is +3 cm. Acetone test is markedly positive, blood glucose is 26
mmol/l. Make the provisional diagnosis:

1. Hepatic coma

2. Hyperosmolar coma

3. Infectious toxic shock

4. Lacticaemic coma

5. Ketoacidotic coma

55. Question
A 45-year old woman was hospitalized with complaints of periodical severe
headaches against the background of elevated blood pressure up to 180/90 mmHg,
muscle weakness, and frequent urination (at night as well). Her anamnesis states
that despite combining various antihypertensive drugs and adjusting the dosage her
arterial hypertension cannot be corrected with drugs. The patient’s blood serum
potassium levels are 2.0 mmmol/L, blood serum sodium level are 160mmmol/L.
Ultrasound imaging detects three dimensional formations approximately 1.0 cm in
diameter in the both adrenal glands. Selective endovascular blood sampling from the
suprarenal vein was performed which revealed significant increase of cortisol and
aldosterone levels. Make the diagnosis:

1. Cushing's disease

2. Aldosteroma

3. Androsteroma

4. Pheochromocytoma

5. Cushing's syndrome

56. Question
During a regular examination, an 8-year old girl with type 1 diabetes mellitus
presents with a swelling on the anterior surface of her hip. The swelling is 3 cm in
diameter, dense, painless on palpation. The skin over this formation has normal color
and temperature. Localization of the swelling matches the place where the girl
usually receives her insulin injections. What is the most likely cause of this clinical
presentation?
1. Formation of post-injection infiltration

2. Allergic response

3. Development of atrophic lipodystrophy

4. Formation of post-injection abscess

5. Development of hypertrophic lipodystrophy

57. Question
A 73-year-old woman came to the family physician for one of her regular follow-up
examinations. Three months ago she was found to have type 2 diabetes mellitus. She
was keeping to her diet and exercise plan and taking Phyto preparations. On
examination her fasting glucose was within the range of 7.8-8.6mmol/L,HbA1c- 7.9%.
height –164cm, weight –83kg. What blood glucose controlling medicine should be
prescribed first in the course of her pharmacological therapy?

1. metformin

2. Gliclazide

3. Insulin

4. Glimepiride

5. Glibenclamide

58. Question
After playing with” mosaics”, a two-year-old child suddenly developed cough,
stridorous respiration, urges to vomit, and cyanosis against the background of
relative stomatic health. What should the doctor suspect first when examining the
child?

1. Acute obstructive bronchitis

2. Foreign body aspiration

3. Acute laryngotracheitis

4. Pertussis

5. Pneumonia

59. Question
A 19-year-old young man was diving and hit his head on the bottom of the pool. He
complains of pain in the neck, head movements are limited and painful. During
examination his head is bowed forward and to the right and the patient supports it
with his hands. Palpation detects tense neck muscles and protruding spinous process
of the IV cervical vertebra(C4). When pressure is applied to this process and to the
head (axial load), the pain intensifies. Make the provisional diagnosis:

1. Uncomplicated cervical fracture

2. Complicated cervical fracture

3. Spinal root injury

4. Cervical contusion

5. Neck muscle injury

60. Question
An unconscious patient was delivered to a hospital by an ambulance. Objectively, his
body temperature is 39 C, he presents with convulsion and red dry skin. It is known
that the patient works as a stoker in the boiler room. What is the likely diagnosis?

1. food poisoning

2. Hypertensive urgency

3. CO poisoning

4. Heat stroke

5. Acute viral respiratory infection

61. Question
A woman came to the gynecologist to plan her pregnancy. She was advised to
increase her intake of the products rich in folic acid, particularly soy beans, bread
made of coarsely ground flour, fruits, leafy green vegetables. Such changes in her diet
will work toward the prevention of:

1. Non- closure of the neural tube in the fetus

2. Non-closure of the fontanel

3. Polyhydramnios

4. Iron- deficiency anemia in the pregnant woman

5. Rickets

62. Question
A 10-year-old boy came to the polyclinic with complaints of stuffy nose. It is known
that these signs occur in the child periodically (in spring and autumn). He has a
history of atopic dermatitis. The father of the child had bronchial asthma. Objectively,
the boy’s face is pale and slightly swollen. Respirations are 22/min. Auscultation
detects vesicular respiration over the lungs. Rhinoscopy shows swollen and pale
nasal mucosa. What disease can be suspected?

1. Recurrent respiratory disease

2. Acute maxillary sinusitis

3. Acute rhinitis

4. Acute adenoiditis

5. Allergic rhinitis

63. Question
What modern organizational method can provide the patients in the remote
settlements with timely access to quality medical aid and such medical services as
consulting, diagnostics, and treatment, especially in the situations when time and
distance are crucial?

1. Mobile communication

2. Ambulance services

3. Air medical services

4. Telemedicine

5. Field medical services

64. Question
A surgery unit received a person with an incised stab wound on the upper third of the
right thigh. Examination detects an incised stab wound 3.0×0.5×2.0 cm in size on the
inner surface of the upper third of the right thigh. Bright red blood flow from deep
within the wound in a pulsing stream. Characterize this type of bleeding:

1. Arterial

2. Venous

3. Parenchymatous

4. Mixed

5. Capillary

65. Question
A patient has gradually lost his consciousness. The skin is pale and dry. There is smell
of ammonia from the mouth. Respiration is deep and noisy. Heart sound muffled,
pericardial friction rub are present, blood pressure is 180/130mmHg. Blood test : Hb
– 80g/l, leukocytes- 12 x 109/l , blood glucose-6.4mmol/l ,urea – 50mmol/l, creatine-
1200mmol/l, blood osmolarity – 350 mOsmol/kg H2O.No urinary excretion. Make the
diagnosis:

1. Hyperglycemic coma

2. Uremic coma

3. Hyperosmolar coma

4. Acute disturbance of cerebral circulation

5. Acute renal failure

66. Question
Disease onset was acute. A child developed general weakness, pain in the joints, and
fever. Later these signs became accompanied by itching skin rash manifested as
erythematous spots 2-5 mm in size. The rash gradually turned hemorrhagic. Large
joints are painful and swollen ;pain attacks periodically occur in paraumbilical area;
there are signs of intestinal hemorrhage. What is the most likely diagnosis?

1. Hemorrhagic vasculitis (Henoch-Schonlein purpura)

2. Rheumatism

3. Scarlet fever

4. Streptococcal impetigo

5. Hemorrhagic meningoencephalitis

67. Question
A 72-year-old man with pneumonia complains of marked dyspnea, chest pain, severe
cough with expectoration, temperature is 39.5-40 C, no urination for the last 24
hours. Objectively, the patient is conscious. Respiratory rate is 36/min. Percussion
sound is dull over the right lower pulmonary lobe; on auscultation there is bronchial
respiration and numerous moist crackles. Blood pressure is 80/60 mmHg. Heart rate
is 120/min. Heart sounds are muffled, there is tachycardia. What tactics should the
family doctor choose for the management for this patient?

1. Hospitalization into the pulmonary unit

2. Outpatient treatment

3. Treatment in the day patient facility


4. Hospitalization into the intensive care unit

5. Hospitalization into the neurology unit

68. Question
A 6-year-old girl complains of body temperature up to 39 C, rhinitis, dry cough,
dyspnea. She has been presenting with these signs for 5 days already. On examination
her condition is of moderate severity. Her dyspnea is of mixed genesis. Respirations
are 28/min. Percussion produces a dull sound in the right lower segments; in the same
area auscultation detects weakened respiration and fine vesicular wet crackles;
coarse respiration can be detected on the left. Make the provisional diagnosis:

1. Right-sided community acquired pneumonia

2. Acute simple bronchitis

3. Acute obstructive bronchitis

4. Acute bronchiolitis

5. Stenosing laryngotracheitis

69. Question
A 48-year-old woman complains of disturbed menstrual cycle; her periods last for 7-9
days and are excessively profuse throughout the last half-year. She notes occasional
hot flashes in her head, insomnia, irritability, and headaches. Her skin is of normal
color. Blood pressure – 150/90mmHg, pulse –90/min, rhythmic. The abdomen is soft
and painless. Bimanual examination shows no uterine enlargement, the appendages
cannot be detected. The vaginal fornixes are free. What is the most likely diagnosis?

1. Stein- leventhal syndrome (polycystic ovary syndrome)

2. Climacteric syndrome

3. Adrenogenital syndrome

4. Uterine myoma

5. Premenstrual syndrome

70. Question
A 25-year old man was hospitalized with complaints of pain in his lower abdomen and
right lumbar area that appeared one hour ago. The patients general state is
moderately severe. Body temperature- 38.2C, heart rate-102/min. The tongue is dry.
The abdomen is painful on deep palpation j the right iliac area and in the petit
triangle. Aure-Rozanov and Gabay signs are positive. Make the provisional diagnosis:

1. Right-sided renal colic


2. Acute cholecystitis

3. Cecal tumor

4. Intestinal obstruction

5. Acute appendicitis

71. Question
A 57-year-old man, a miner, complaints of a pain in his chest, dyspnea on physical
exertion, excessive sweating, constant sub febrile temperature, and cough that
produces blood-steaked sputum. He has been smoking for approximately 40 years (2
packs a day) and frequently has ”pneumonias”. Survey chest X-ray shows a
triangular shadow in the middle lobe of the right lung. One of the apices of the shadow
points to the lung root. Cardiac and mediastinal shadows are displaced toward the
affected area. Make the provisional diagnosis:

1. Chronic bronchitis

2. Right-sided pleuropneumonia

3. Cancer of the right lung

4. Tuberculosis of the right lung

5. Pneumoconiosis

72. Question
A full-term newborn (born with the body weight of 3900 g at gestational age of 39
weeks) on the first day of his life developed respiratory disturbances: dyspnea,
arrhythmic respiration, cyanosis attacks. On examination there is paradoxical
respiration observed and left side of the chest lags behind in the act of breathing. On
auscultation the respiration is weakened in the lungs on the left. Neurologist
diagnosed the patient with left- sided Erb-Duchenne palsy. Complete blood count
shows no changes. What is the most likely diagnosis?

1. Transient tachypnea of the newborn

2. Respiratory distress syndrome

3. Left-sided diaphragm paresis

4. Congenital pneumonia

5. Left-sided pneumonia

73. Question
A 65-year old man with acute anterior myocardial infarction developed an asphyxia
attack. Examination detected diffuse cyanosis. In the lungs there are numerous
heterogenous wet crackles. Heart rate is 100/min. Blood pressure is 120/100 mm Hg.
What complication occurred in this patient?

1. Hypertensive crisis

2. Pulmonary embolism

3. Pulmonary edema

4. Ventricular septal rupture

5. Cardiogenic shock

74. Question
On the 3rd day after the artificial abortion the woman was hospitalized into the
gynecological department in a severe condition with signs of intoxication, abdominal
pain, and purulent discharge from the vagina. Objectively, the patient’s condition is
severe, her body temperature is 38.8 C, pulse is 100/min, blood pressure is 110/70
mmHg, the uterus is soft, the uterine fundus is located at the level of navel, there are
positive signs of peritoneal irritation. What is the most likely diagnosis?

1. Uterine perforation

2. Ectopic pregnancy

3. Acute metro endometritis

4. Acute suppurative salpingo-oophoritis

5. Pelviperitonitis

75. Question
The body of a citizen was found at the place of his dwelling. On his face, neck, and
hands there were detected irregular-shaped wounds, varying from 2×3 cm to 4×5 cm
in size. The skin and underlying tissues are absent in the wounds. The margins of the
wounds are uneven, with major and minor scalloping along the edges and no signs of
bleeding. What is the initiating mechanism of these wounds?

1. Multiple stab and incised wounds

2. Local effect of cold

3. Pellet gunshot wound

4. Blast injury

5. Animal bite wounds


76. Question
A 22-year old post parturient woman on the 12th day after the normal childbirth
informs of fever up to 39 degree C for the last three days and pain in her right
mammary gland. The right mammary gland is enlarged, hot to touch, tense,
hyperemic, and painful. Palpation reveals there a dense infiltration 8*8 cm with a
fluctuation in its center. What is the most likely diagnosis?

1. Postpartum period, day 12. Right sided infiltrative-purulent mastitis

2. Postpartum period, day 12. Right sided serous mastitis

3. Postpartum period, day 12. Right sided phlegmonous mastitis

4. Postpartum period, day 12. Right sided gangrenous mastitis

5. Postpartum period, day 12. Right sided lactostasis

77. Question
An 18-year-old patient complains of skin rash. The patient has been suffering from
this condition for 5 years. The first instance of this disease occurred after a car
accident. Objectively, the patient presents with a papular rash covered in silvery
scales, “thimble” sign (small pits on the nails) affected joints. What is the most likely
diagnosis?

1. Psoriasis

2. Rheumatism

3. Onychomycosis

4. Panaritium

5. Lupus erythematosus

78. Question
2 hours after eating unknown mushrooms, a 28-year-old man sensed a decrease in his
mobility and deterioration of his ability to focus. This condition was then followed by
a state of agitation and aggression. On examination, he is disoriented and his speech
is illegible. 4 hours later he developed fetor hepaticus and lost his consciousness.
What syndrome can be observed in this patient?

1. Acute hepatic failure

2. Cholestatic syndrome

3. Portal hypertension

4. Hepatolienal syndrome
5. Cytolytic syndrome

79. Question
A patient 1 year had a Q wave myocardial infarction of the posterior wall of the left
ventricle. For the last 2 weeks he has been suffering from daily attacks of atrial
fibrillation and bradycardia episodes, accompanied by bouts of vertigo. What tactics
is the most advisable in this case?

1. Prescription of bisoprolol

2. Prescription of amiodarone

3. Prescription of procainamide

4. Prescription of digoxin

5. Pacemaker implantation

80. Question
A 20-year old patient complains of a severe headache, double vision, weakness, fever,
irritability. Objectively: body temperature is 38.1 C, the patient is reluctant to
contact, sensitive to stimuli. There is ptosis of the left eyelid, exotropia, anisocoria S
>D, and pronounced meningeal syndrome. On the lumbar puncture the cerebrospinal
fluid flowed out under a pressure of 300 mmHg, the fluid is clear, slightly opalescent.
24 hours later there appeared a fibrin film. Protein- 1.4 g/L, lymphocytes- 600 per
mm^3, glucose- 0.3 mmol/L. What is the provisional diagnosis?

1. Meningococcal meningitis

2. Lymphocytic (Armstrong’s) meningitis

3. Tuberculous meningitis

4. Syphilitic meningitis

5. Mumps meningitis

81. Question
A pregnant woman at 32 weeks of gestation with the risk of preterm labor undergoes
a treatment to prevent fetal respiratory distress syndrome. What medicine was she
prescribed?

1. Progesterone

2. Misoprostol

3. Oxytocin
4. Gynipral ( hexoprenaline)

5. Dexamethasone

82. Question
A 45-year old woman undergoes an inpatient treatment. She complains of elevated
body temperature up to 39.0 C, pain in her right lumbar area, turbid urine with
blood. CT scan shows an area of low density within the parenchyma, no difference
between the cortical and medullary layers, an increased density of the perinephric fat
due to edema. What is the diagnosis?

1. Para nephritis

2. Renal carcinoma

3. Pyelonephritis

4. Renal abscess

5. Glomerulonephritis

83. Question
A 56-year old woman was diagnosed with stage 2 hypertension of the 2nd degree. She
belongs to the group of moderate risk and has bronchial asthma. What group of drugs
is CONTRAINDICATED to this patient?

1. Imidazoline receptor antagonist

2. Diuretics

3. Beta blockers

4. Calcium antagonists

5. Angiotensin converting enzyme inhibitors

84. Question
A 21-year-old man was hospitalized on the 2nd day of the disease. His general
condition is severe, body temperature is 39C. On his skin there are numerous
irregular-shaped hemorrhghic elements. The diagnosis of meningococcemia was
made. The next day his body temperature suddenly decreased, blood pressure was
80/40mmHg, pulse was 120/min. Acrocyanosis was detected. What complication did
the patient develop?

1. Acute heart failure

2. Cerebral coma
3. Acute hemorrhage

4. Acute liver failure

5. Acute adrenal insufficiency

85. Question
What should be prescribed as secondary prevention drugs for a patient with atrial
fibrillation after an ischemic stroke caused by cardiac embolism?

1. Oral anticoagulants

2. Calcium antagonists

3. Nootropics

4. Beta blockers

5. Aspirin or clopidogrel

86. Question
A 7-year-old boy after a fall from a height presents with rapid and shallow breathing
and cyanotic face. The right half of his thorax is distended and takes no part in the
respiration. Percussion detects tympanitis in the affected area, while auscultation
detects no breathing there. What pathology is the most likely cause of this clinical
presentation? What instrumental examination would be most informative in this
case?

1. Right-sided hemothorax. Survey X-ray of the chest

2. Airway foreign body. Diagnostic therapeutic bronchoscopy

3. Right-sided tension pneumothorax. Chest X-ray

4. Mediastinitis. Survey X-ray of the chest

5. Tension cyst of the right lung. Tracheobronchoscopy

87. Question
A 23-year old woman without visible cause developed a conflicting behavior at the
work place. She accused the management of underestimating her, claimed that she
can be a deputy director, because she speaks four languages, is very attractive, and
can make useful connections for the company. She has been dressing extravagantly,
flirting with her colleges, and singing loudly in her office. Infact, she has only the
training of a computer operator and speaks no foreign languages. What is the likely
clinical diagnosis?

1. Mild mental retardation


2. Manic episode

3. Schizophrenia

4. Depressive disorder

5. Epilepsy

88. Question
A 30-year-old woman made an appointment with the family doctor for scheduled
vaccination of her 2-year-old child. What type of healthcare provides such medical
services?

1. Primary healthcare

2. Emergency aid

3. Palliative care

4. Tertiary healthcare

5. Secondary healthcare

89. Question
A 19-year-old young man complains of cough with expectoration of purulent sputum
in amount of 100 ml per day, hemoptysis, dyspnea, increased body temperature up to
37.8 C, general weakness, weight loss. The patients condition lasts for 4 years.
Exacerbations occur 2-3 times a year. The patient presents with malnutrition, pale
skin, cyanosis of the lips, drumstick (clubbed fingers). Tympanic percussion sound in
the lungs, weakened respiration, and various numerous moist crackles in the lower
pulmonary segments on the left can be observed in this patient. in blood:
erythrocytes-3.2 x 1012/L, leukocytes- 8.4 x 109/L, ESR-56 mm/hour. On X-ray: lung
fields are emphysematous, the left pulmonary root is deformed and dilated. What is
the most likely diagnosis?

1. Left-sided pulmonary cystic dysplasia

2. Suppuration of the cyst in the left lung

3. Chronic abscess of the left lung

4. Chronic left-sided pneumonia

5. Multiple bronchiectasis of the left lung

90. Question
For 20 years the role of excessive weight in ischemic heart disease development
among the working age male production over 40 was studied. It was determined that
overweight men developed ischemic heart disease more often. What type of
epidemiological study is it?

1. Case report

2. Vase series report

3. Experimental study

4. Case-control study

5. Cohort study

91. Question
A 3-month-old child with sighs of rickets presents with positive Chvostek, Trousseau,
and Maslov signs. One days ago, the parents witnessed a cyanotic attack in their
child- the child broke into a cold sweat, the eyes bulged, and respiratory arrest
occurred. One minute later the child drew in a loud breath and child’s condition
normalized again. What is the cause the described signs of the disease?

1. Decrease of blood phosphorus levels

2. Decrease of blood calcium levels

3. Increase of blood calcium levels

4. Metabolic acidosis

5. Increase of blood phosphorus levels

92. Question
You witnessed a car accident. When examining the place of the accident you noticed a
man of about 30 years, who was hit by the car. He is unconscious. On his neck on the
left there is a profuse hemorrhage with bright-red blood. How to stop this
hemorrhage?

1. Apply a plaster cast

2. Put him in a stable position

3. Apply a neck brace

4. Digital occlusion, Mikulich method

5. Maximal hyperextension of the neck

93. Question
During regular medical examination a lyceum student present with signs of cheilitis
that manifests as epithelial maceration in the area of lip seal. The lips are bright-red
with single vertical cracks covered with brown-red scabs. These clinical signs are
mostly likely caused by insufficient content of the following in the diet:

1. Thiamine

2. Ascorbic acid

3. Riboflavin

4. Calciferol

5. Retinol

94. Question
A 45-year-old man with a history of myocardial infarction developed a brief attack of
palpitation, accompanied by the sensations of lack of air, fear, and vertigo. His blood
pressure is 90/60mmHg. ECG during the attack shows extended QRS complex (0.13
seconds) with heart rate of 160/min, discordant shift of ST segment and T wave,
dissociation of atrial and ventricular rhythm. What disturbances of cardiac rhythm it
is?

1. Atrial fibrillation

2. Paroxysmal supraventricular tachycardia

3. Ventricular fibrillation

4. Paroxysmal ventricular tachycardia

5. Frequent ventricular extrasystole

95. Question
A 12-year-old girl after a case of respiratory infection developed dyspnea at rest,
paleness of skin. Heart rate is 110/min, BP is 90/55 mmHg. Heart sounds are muffled.
Borders of relative heart dullness: right- the left parasternal line, upper- the III rib,
left- 1,0 cm outwards from the midclavicular line. Make the provisional diagnosis:

1. Infectious myocarditis

2. Hypertrophic cardiopathy

3. Exudative pericarditis

4. Somatoform autonomic dysfunction

5. Functional cardiomyopathy

96. Question
A 37-year old woman received an occupational trauma that resulted in a severe vision
impairment. Now she needs to be trained for another occupation. What type of
rehabilitation should be doctor choose for the patient in this case?

1. Medical rehabilitation

2. Target rehabilitation

3. Occupational rehabilitation

4. Psychological rehabilitation

5. Social rehabilitation

97. Question
A 38-year-old patient has been delivered by an ambulance toa surgical department
with complaints of general weakness, indisposition, black stool. On examination the
patient is pale, there are dotted hemorrhages on the skin of his torso and extremities.
On digital investigation there are black feces on the glove. Blood test: Hb-108g/L,
thrombocytopenia. Anamnesis states that similar condition was observed I year ago.
Make the diagnosis:

1. Rectal tumor

2. Ulcerative bleeding

3. Nonspecific ulcerative colitis

4. Thrombocytopenic purpura

5. Hemophilia

98. Question
Forensic autopsy of a body of a 59-year-old man, who died suddenly at home without
signs of violent death, shows pink skin and mucosa, liquid bright-red blood, and
bright-red plethoric internal organs. Forensic toxicology testing detected 1.44% of
ethanol in the blood and carboxyhemoglobin levels of 55%. What is the cause of death?

1. Arsenic poisoning

2. Alcohol poisoning

3. Aniline poisoning

4. Potassium cyanide poisoning

5. Carbon monoxide poisoning

99. Question
A 20-year old man suffers from headache, general weakness, and face edema that
appears in the morning. 18 days earlier he had a case of tonsillitis. Objectively, his
skin is pale, there are edema under his eyes. Hear rate is 60/min, blood pressure is
185/100 mmHg. The sign of costovertebral angle tenderness (punch sign in the lumbar
region) is negative. Urinalysis: color of “meat slops”, protein-4.5 g/, altered
erythrocytes- 40-45 in the vision field, leukocytes- 5-6 in the vision field. 24-hour
diuresis is 400 mL. What is the most likely diagnosis?

1. Acute glomerulonephritis

2. Acute pyelonephritis

3. Renal amyloidosis

4. Urolithiasis

5. Systemic lupus erythematosus

100. Question
A 3-year-old child has been brought to a hospital with pain in the legs, fever, and loss
of appetite. Objectively: pale skin and mucosa, hemorrhagic rash. Lymph nodes are
enlarged, painless, dense and elastic, not matted together. Bones, joints and abdomen
are painful. The liver and spleen are enlarged. Hemogram: Hb-88g/L, color index- 1.3,
platelets- 80 x 109/L, leukocytes- 25.8 x 109/L, lymphoblasts- 70%, ESR-52 mm/hour.
Make the provisional diagnosis:

1. Acute rheumatic fever

2. Thrombocytopenic purpura

3. Infectious mononucleosis

4. Hemorrhagic vasculitis (Henoch-Schonlein purpura)

5. Acute leukemia

101. Question
A 35-year-old woman came to the family doctor with complaints of profuse
menstruations that last up to 10 days. Gynecological examinations shows that the
uterine cervix is without changes, the uterus is in anterflexio, has normal size , is
mobile and painless. The uterine appendages on the both sides are without
peculiarities. The family doctor made the provisional diagnosis of the abnormal
uterine bleeding. What instrumental method of examination needs to be performed
first to diagnose this pathology?

1. Laparoscopy

2. Transabdominal ultrasound
3. Culdoscopy

4. Transvaginal ultrasound

5. Colposcopy

102. Question
A 34-year-old multipara was brought to the labor ward with regular labor activity.
Her pelvic size 26-29-32-22 cm. Vaginal examination shows 6 cm cervical dilation, the
amniotic sac is unbroken. The fetus is in the breech presentation, with buttocks
pressed to the entrance into the lesser pelvis. The promontory cannot be reached, no
exostoses. Feta heart rate is 140/min, expected fetal weight is 2800g. What labor
tactics should be chosen?

1. External obstetric version of fetus

2. Urgent cesarean section

3. Classic combined external-internal version of the fetus

4. Fetal extraction from the pelvic end

5. Delivery through the natural birth canal

103. Question
A 17-year old girl has been suffering from hepatic cirrhosis for 3 years. Lately her
periods of excitation have been intermittent with depression, she does not sleep
enough. Objectively her condition is severe, the girl is sluggish, gives one-word
responses, has tremor in her extremities, her skin is icteric, with single hemorrhagic
rashes. Name the likely complication of her disease:

1. Kidney failure

2. Hepatic encephalopathy

3. Bipolar affective disorder

4. Sepsis

5. Reye syndrome

104. Question
After eating shrimps, a 25-year old man suddenly developed skin itching, some areas
of his skin became hyperemic or erupted into vesicles. Make the diagnosis:

1. Acute urticaria

2. Urticaria pigmentosa
3. Psoriasis

4. Scabies

5. Hemorrhagic vasculitis (Henoch-Schonlein purpura)

105. Question
A 48-year-old man came to a doctor with complaints of vomiting that brings no relief
and a burning pain in his left subcostal region that irradiates to the left lumbar
region. These signs appeared after a meal. The Ortner’s and Mayo-Robson’s signs are
positive. In the blood; leukocytosis and increased ESR. In the urine; elevated diastase
levels. Make the diagnosis:

1. Chronic pancreatitis in the exacerbation stage

2. Chronic gastritis in the exacerbation stage

3. Chronic cholecystitis in the exacerbation stage

4. Gastric ulcer Performation

5. Renal colic

106. Question
A patient underwent suture plication of the perforated duodenal ulcer. On the 3rd day
after the operation he started producing a large amount of discharge from the
abdominal drain tube. The discharge contains bile and has high amylase levels. What
complication occurred in the patient?

1. Acute postoperative pancreatitis

2. Suture incompetence of the ulcerative defect

3. Acute cholecystitis

4. Hemorrhage from the ulcer

5. Early postoperative adhesive obstruction

107. Question
An 18-year old patient always obeys others and adapts his needs to the demands of
the people on whom he depends. He excessively defers to their wishes and makes them
responsible for his wellbeing, cannot defend his interests and needs support from
other people. Such psychic profile has been formed in the childhood, remains
unchanged, and hinders adaptation. What psychic disorder is observed in this
patient?

1. Psychopathy-like state
2. Markedly accentuated personality

3. Dependent personality disorder

4. Anxiety (avoidant) personality disorder

5. Anankastic personality disorder

108. Question
A 22-year-old infant developed subcutaneous red nodes from 1.0 to 1.5 cm in size on
the scalp; later the nodes enlarged. Complete blood count: anemia, leukocytosis,
neutrocytosis, raised ESR. What diagnosis will you make?

1. Pseudo furunculosis

2. Vesiculopustulosis

3. Pemphigus

4. Scalp phlegmon

109. Question
In a pediatric clinic, located in a rural area, there are 9 children, who simultaneously
fell ill. The following signs were detected: low physical activity, acrocyanosis of the
nasolabial triangle and fingertips, mucosal cyanosis, tachycardia, dyspnea. It was
determined that all the sick children were fed with a formula that was dissolved in the
water taken from a dug well. Laboratory analysis revealed high levels of
methemoglobin in the blood of the children. These signs can be caused by increased
content of a certain element in the water. Name this element:

1. Sulfates

2. Selenium

3. Methyl mercury

4. Nitrates

5. Lead

110. Question
A 35-year old woman complaints of a pain in her right axillary region. She has been
suffering from this condition for a week. Her body temperature is 38 C. in the right
axillary region there are 2 formations, 2 cm in size each. The skin over the formations
is dark red and thin, palpation produces a yellow-white discharge from the fistular
openings. What is the most likely diagnosis?

1. Furuncle
2. Hydradenitis

3. Folliculitis

4. Lymphadenitis

5. Carbuncle

111. Question
A child was born at 40 weeks of gestation with the weight of 3700g. The child’s Apgar
score is 7/9. The baby was put to breast immediately after birth and suckled actively.
On the 3rd day of life, the child’s weight decreased to 3600g. What transitory
condition is observed in this child?

1. Toxic erythema

2. Transient dysbiosis

3. Uric acid infarction

4. Physiological jaundice

5. Physiological weight loss

112. Question
An 8-day-old boy was delivered to the hospital on the second day after the onset of the
disease. His parents complain of his fussiness, regurgitation, body temperature up to
38.5 C, red skin with infiltration in the lumbar area. His medical history has no
peculiarities. The child is in the severe condition, inert, pale, suckles poorly. In the
lumbar area, on the sacrum and buttocks there is a tense infiltration with hyperemic
and cyanotic areas and with a soft spot 8×7 cm in its center. the stool is 10 times in 24
hours, with green and mucous admixtures. What is the most likely diagnosis?

1. Adiponecrosis

2. Congenital soft-tissue tumor

3. Erysipelas

4. Phlegmon of the newborn

5. Hemangioma

113. Question
A man works in casting of nonferrous metals and alloys for 12 years. In the air of
working area there was registered high content of heavy metals, carbon monoxide,
and nitrogen. During periodic health examination the patient presents with
asthenovegetative syndrome, sharp abdominal pains, constipations, pain in the
hepatic area. In urine: aminolaevulinic acid and coproporphyrin are detected. In
blood: reticulocytosis, low hemoglobin level. Such intoxication is caused by:

1. Carbon monoxide

2. Zinc

3. Lead and lead salts

4. Tin

5. Nitric oxide

114. Question
A 40-year-old victim of a traffic accident sustained the following injuries: closed
diaphyseal femur fracture, brain concussion, multiple rib fractures,
hemopneumothorax, degloving shin injuries. What injuries require the most urgent
attention?

1. Multiple rib fractures, hemopneumothorax

2. Degloving shin injuries

3. All injuries are equivalent

4. Closed diaphyseal femur fracture

5. Brain concussion

115. Question
After the pregnant woman’s water broke, it was noted that they are significantly
contaminated with meconium. Upon birth, the baby is not breathing, remains inert,
the skin is cyanotic and covered in meconium, heart rate is 98/min. What
resuscitation measures should be taken after the baby is born?

1. Tactile stimulation of the newborn

2. Artificial pulmonary ventilation with a mask and Ambu bag

3. Direct laryngoscopy, intubation, sanation of the trachea

4. Give adrenaline intravenously

5. Sanation of the upper respiratory tracts with a rubber balloon

116. Question
A 32-year-old woman complains of general weakness, low-grade, fever persisting for
4 months, lumbar pain, and dysuria. Anamnesis includes frequent acute respiratory
diseases, overexposure to cold, low-calorie diet, a case of pulmonary tuberculosis in
childhood. Clinical urine analysis; pH 4.8 leukocytosis, hematuria. Complete blood
count: leukocytosis, lymphocytosis, raised ESR, urography concludes; dilation of
renal pelvis and calyceal system of both kidneys, foci of the calcification in the
projection of right kidney parenchyma. What is the most likely diagnosis?

1. Right renal carcinoma

2. Acute glomerulonephritis

3. Chronic pyelonephritis

4. Nephrotuberculosis

5. Right renal cyst

117. Question
A 34-year-old woman after rapidly changing her position from horizontal to vertical
suddenly paled, fell down, her skin became moist, her limbs are cold, her pupils are
dilated. The pulse is rapid and thready, blood pressure is 50/25 mmHg. What
condition has likely developed in the patient?

1. Collapse

2. Morgagni-Adams-stokes syndrome

3. Ventricular fibrillation

4. Coma

5. Shock

118. Question
A 20-year old woman, gravida 2, para1 has been in labor for 4 hours. Her condition is
satisfactory. Moderately painful contractions occur every 3 minutes and last for 35-
40 seconds. The waters have not burst yet. The fetus is in longitudinal position. Fetal
heartbeats are 136/min., clear and rhythmic. Major segment of the fetal head is
engaged to the pelvic inlet. Vaginal examination shows smooth cervix of 6 cm,
amniotic sac is intact, sagital suture is in the left oblique diameter, occipital fontanel
is on the right near the symphasis pubis. What stage of labor is it?

1. Latent phase of the first stage of normal labor

2. Precursors of childbirth

3. The second stage of normal labor


4. Active phase of the first stage of normal labor

5. Preliminary stage

119. Question
A 52-year-old woman has been suffering for 2 years from dull, occassionally
exacerbating pain in her right subcostal area, occuring after eating high-fat foods,
bitter taste in her mouthin the morning, constipations, and flatulence. Objectively,
she is obese, her body temperature is 36.9 C; there is a coating on the root of her
tongue; abdomen is moderately distended and painful in the area of the gallbladder
projection. What examination would be most helpful for diagnosis making?

1. Duodenal probe

2. Cholecystography

3. Duodenoscopy

4. Ultrasound

5. Liver scanning

120. Question
Employees work in conditions of high dust concentration. Certain chemical (silicon
dioxide content) and physical properties of dust aerosols contribute to the
development of occupational dust-induced diseases. What is the main physical
property of dust aerosols?

1. Ionization

2. Magnetization

3. Solubility

4. Electric charge

5. Dispersion

121. Question
During winter epidemics of influenza caused predominantly by virus
A/California/04/2009 (H1N1), on the 2nd day after the disease onset a 30-year old
hospitalized man presented with high fever, dry cough, myalgia, headache, and
general weakness. What should be prescribed as an etiotropic treatment in this case?

1. Immunoglobulin

2. Interferon inducers
3. Acyclovir

4. Antibiotics

5. Neuraminidase inhibitors (oseltamivir)

122. Question
3 hours after a trauma, a young man developed bradycardia of 46/min, anisocoria D
>S, hemi-hyperreflexia S >D, hemi hypesthesia on the left, and a convulsive disorder.
The character of this process needs to be clarified. What method of examination will
be the most accurate for this purpose?

1. Lumbar puncture

2. Skull X-ray

3. Echoencephalography

4. Head CT

5. Electroencephalography

123. Question
A family doctor performed an external obstetrical examination on a pregnant woman
and determined that her uterine fundus is located at the level of the navel. What is the
most likely term of pregnancy in this woman?

1. 8 weeks

2. 32 weeks

3. 16 weeks

4. 40 weeks

5. 24 weeks

124. Question
When planning treatment of a patient, it was decided to use a medicine with evidence
level A. what trials produce the evidence that allows to classify the medicine level as
A?

1. Data obtained from many non-randomized trials

2. Data obtained from one randomized clinical trial

3. Expert consensus

4. Data obtained from several randomized clinical trials


5. Case-control studies

125. Question
A 30-year old woman complains of itching skin, predominantly in the evening and at
night. The condition lasts for 2 weeks already. On the skin of interdigital folds,
mammary glands, abdomen, buttocks, and thighs there are numerous fine papular
and papulovesicular rashes located in pairs, excoriations. There is no rash on the face
and neck. Similar rash is observed in the husband of the patient. What is the most
likely diagnosis?

1. Scabies

2. Epidermophytosis

3. Herpes

4. Eczema

5. Neurodermatitis

126. Question
A 58-year old woman has type2 diabetes mellitus that is compensated with diet and
metformin. She prepares for cholecystectomy. Objectively, her height is 164 cm,
weight is 90 kg, heart rate is 72/min, blood pressure is 130/80 mmHg. Her abdomen is
soft, painful in the right subcostal region. The liver is not enlarged. Fasting glucose-
6.2 mmol/L. Glycated hemoglobin- 6.5%. what further tactics of sugar lowering
therapy should be chosen in this case?

1. Continue the present scheme of therapy

2. Prescription of glurenorm (gliquidone)

3. Prescription of a long-acting insulin

4. Prescription of a short-acting insulin

5. Prescription of an insulin mixture

127. Question
A dweller of the northern Dnieper area, a fisherman, for the last several days has been
complaining of a discomfort in his right subcostal region, periodical episodes of
diahhrea, intermittent with constipation, frequent skin rashes. Abdominal ultrasound
shows enlarged liver and pancreatic head. Make the provisional diagnosis:

1. Taeniasis

2. Opisthrochiasis
3. Ornithosis

4. Onchocerciasis

5. Trichinellosis

128. Question
A 43-year old man complains of facial edema, dyspnea, and difficult swallowing that
appeared 3 hours after he was stung by an insect. Objectively, the skin of his eyelids,
cheeks and lips is acutely hyperemic and edematous. What medicines should he be
prescribed for emergency aid?

1. Diuretics

2. Nonsteroidal anti-inflammatory drugs

3. Antihistamines

4. Glucocorticoids

5. Muscle relaxants

129. Question
A 23-year-old woman came to the gynecological clinic. she complains of pain, itching,
and burning in her vulva, general weakness, indisposition, elevated body
temperature up to37.2 C, and headache. On examination in the vulva there are
multiple vesicles up to 2-3 mm in diameter with clear contents against the
background of hyperemia and mucosal edema. Make the provisional diagnosis:

1. Primary syphilis

2. Genital herpes infection

3. Cytomegalovirus infection

4. Papillomavirus infection

5. Vulvar cancer

130. Question
A 75-year-old man in a severe condition suffers from dyspnea at rest, marked
weakness, and arrhythmia. Abdominal aortic pulsation is observed, further on is a
systolic murmur detected. Palpation reveals a volumetric formation in the
mesogastrium. Blood pressure is 70/40mmHg. There is no pulsation over the femoral
arteries. Oliguria is detected. Which diagnosis is the correct one?

1. Dissecting aortic aneurysm


2. Acute pericarditis

3. Cardio sclerotic aneurysm

4. Pancreatic cyst

5. Acute cardiac aneurysm

Kiev eneutron.info@gmail.com

Created by Eneutron Team


Dr.HUR A. Salman - OdNMU

KROK REVIEW
- First of all
I’d like to thank all the sources , sites , books and persons that I used their material for this preparation , and I
know it is wrong that I used a lot of pictures under copy right without permission , therefore; I’m asking
apologize from all that resources and persons whom made that efforts for humanity
please anyone (sites and persons) object on this , don’t hesitate to contact me , directly I will remove his work
with pleasure and respecting his rights .
Unfortunately I don’t have enough place and time to mention you all ,
Here I’m repeating my thanks and appreciation for serving medicine and health overall the world .

- DEDICATION:
I dedicate this simple works for all humanity, asking to stop wars , destruction and killing people and living
Lovely , Peacefully with happiness and as one human being , we are all same source , beginning and same
end just differ in between , life like 2 brackets ( ) one begin other one end , so try to fill in-between these
brackets with humanity , love , charity , saving the world .
- Thanks God , and all who supported me .

- How to prepare Krok test


krok is MCQ based exam (200Q/4h ) in clinical subjects ( Internal , surgery , pediatrics , Gyne & OB) with
other subjects like Hygiene & nutrition , psychiatry , social medicine , occupational disease , infectious
disease , pediatrics surgery )
this exam based on translated questions from Russian to English therefore; a lot of mistakes and non-usable
English words , some non medical terms as well as printing mistakes.
- If we ask any foreign students about this exam , he will answer better to cancel this exam from protocol of
international faculty , because it’s never assess the student knowledge , the information that used in test old ,
non updated diseases and old protocols and based on SSRI period.
- Unfortunately very old information make student confused with the international parameters, procedures
and new guidelines and modalities.

- Another thing why it is just one attempt ?? and you should repeat all the course again on account of some
miserable questions ??
So the best solution for this problem is to the exam selective as before , obligatory for Ukrainian citizen and
selective for foreign , because no benefit of this exam
Or 2nd option to change the discipline to make real international not just on papers
- May be this information will not change anything , but that notes for next courses

- About IFOM exam , should not be applied unless they change the syllabus and remove all that old books ,
procedures , examination , and all protocols , old medical terms , and unusable words , as well as the
combination Latin roots that they make it , as they want to make it formal exam , it is not like krok just keep
without understand , it is hard exam and depend on updated infromations

- Actually students can Help to update books and protocols , espcailly the INTERNATIONAL
- So what student should do in this case , study the old questions and keep it without understanding
unfortunately especially for Hygiene , non medical questions , why doctor should keep dimension of room
or how to calculate air ventilation and which place should build hospital , this subject should be selective ,
Dr.HUR A. Salman - OdNMU
doctor after graduate work with patient and need to learn examination diagnosis of disease and treatment
better than learn numbers and old protocols
- So I suggest on you to analyze the questions use the following steps :
1- for medical Q try to find signs or symptoms that related to the disease , whatever was diagnosis or
treatment and tactic
- 2- for non-understandable Q try to use Excluding the options , because a lot of Q they used 4 options so far
from the answer and the last one will the correct whatever it is right information or wrong .
- 3- use KEY way to match the Q with the answer , if clinical or other
- Sometimes there is more than one right answer but you should choose the more specific one
for ex give you information about extrauterine pregnancy and cervical pathology and they put in the answer
– ectopic pregnancy and cervical pregnancy ,, both of them right logically but you should vhosse the
cervical pregnancy it is more specific .
- For Hygiene most Q not understandable , some of them keep it , others if you translate it you can understand
, others you find the answer in the Q same words but not always , so I think just go a head and do the easiest
way you like .
- Note: MOST information here based on Ukrainian information and protocols , so try to keep this
information temporarily because it differ completely from your state exam or another country ..

- I advice you don’t use this information furthermore due to most of these information not based on clinical
and practical measurement , most of it theoretical and hypothetical information .

- Before Finally I wish all pass safely and wish all best of luck , hoping to see all best doctors and have a
good futures , caring all people , saving the humanity , especially children and poor people and all who on
need , repairing the world which destroyed by wars.

- Remember KROK does’t make you a DOCTOR !


And never assess you
Please Don’t hesitate to contact me for adding some information or correct anything .
If you feel some information not well explained or there is any comments , please I’m listening and I will
correct it again and re share that information
I will be happy with your feedback
Finally if you find this file good and can help others , please share it

All answers is A with


Yellow color : the answers
Green color : key words of Q
Blue color : this is the most important words in Q (additional Keys)
Grey color : this is my note and additional info
Red color : this repeated Q or high-light
Violant color : critical notes

Yours
Dr. HUR A. SALMAN
OdNMU / Odessa
20/5/2018
E-mail: hur.amer@gmail.com
Facebook: Hr Salman
Contact app : +964-790-489-6865
Dr.HUR A. Salman - OdNMU
Krok 2 – 2014 INTERNAL MEDICINE Base

Krok 2 Medicine
1
A 47 y.o. woman complains of having paroxysmal headaches for the last 5 years. The pain is
one-sided, intense, localised in frontal region of head, accompanied by nausea and stomach
discomfort, begins one of a sudden. Onset is usually preceded by vision reduction. Anamnesis
gives evidence of periodical AP rise, but at the moment the woman doesn't take any medicines.
Inbetween the onsets of headache her state is satisfactory. Objectively: high-calorie diet (body
weight index - 29), AP- 170/95 mm Hg. Neurologic state has no pecularities. What is the most
probable diagnosis?
A Migraine → this is common type without aura , while classic type with aura
B Chronic subdural hematoma
C Epilepsy
D Benign intracranial hypertension
E Hypertensive encephalopathy

2
A 7 y.o. boy suddenly felt pain in his right knee, it became edematic. The day before he took part
in a cross-country race. Family anamnesis has no data about hemophilia and bleeding
sickness. Objectively: body temperature is 37,5oС. The knee is painful, hot to the touch,
edematic with local tissue tension over it. Blood count: Нb- 123 g/L, leukocytes - 5,6*109/L,
thrombocytes - 354*109/L, prothrombin time - 12 seconds (normally 10-15 seconds), partly
activated thromboplastin time - 72 seconds (normally 35-45 seconds). Hemorrhage time is
normal, VIII:C factor is 5% of norm. What is the most probable diagnosis?
A Hemophilia A → deficiency of blood clotting Factor VIII (8) .
B Hemophilia B → deficiency of blood clotting Factor VIII (9) .
C Schoenlein-Henoch disease
D Vitamin K deficiency
E Thrombocytopenia
Dr.HUR A. Salman - OdNMU
3- On the 3rd day after the acute anterior myocardial infarction a 55 y.o. patient complains of dull
ache behind his breast bone, that can be reduced by bending forward, and of dyspnea.
Objectively: AP- 140/180 mm Hg, heart sounds are dull. ECG results: atrial fibrillation with
frequence of ventricular contractions at the rate of 110/min, pathological Q wave and S-T
segment raising in the right chest leads. The patient refused from thrombolisis. What is the most
probable diagnosis?
A Acute pericarditis→(usually evidenced by a pericardial friction rub), and/or a pericardial effusion.
B Pulmonary embolism → Hemoptysis , Dyspnea , chest pain (pleuritic /infacrvtion , Central /RV
ischemia )
C Tietze's syndrome → benign inflammation of one or more of the costal cartilages
D Dissecting aortic aneurysm
E Dressler's syndrome → is a secondary form of pericarditis that occurs in the setting of injury to
the heart or the pericardium (the outer lining of the heart). It consists of fever, pleuritic pain,
pericarditis and/or a pericardial effusion.

ST Elevation
1- acute MI
2- acute Pericarditis
3- cardiac aneurism
(Ventricular)

4- A 54 y.o. man was admitted to the hospital with complaints of sudden intense headache in
occipital region and vomiting. In the medical history: moderate arterial hypertension, the patient
was taking hydrochlorothiazide. Three days ago he consulted a therapeutist about intense
headache that was suppressed by an analgetic. Objectively: consciousness is confused, left
pupil is mydriatic. Evident photophobia and tension of neck muscles. Left-side hemiparesis with
increased muscle tonus and reflexes. Body temperature is low, rash is absent. AP- 230/130 mm
Hg, Ps- 50 bpm, BR- 12/min. What is your preliminary diagnosis?
A Acute subdural hematoma (SDH) is not only the most common type of intracranial mass lesion, occurring in about
one third of those with severe head injuries (Glasgow Coma Scale [GCS] score <9), but also associated with high mortality
and morbidity rates, the emergent management of acute SDH is critical
B Myasthenia
C Disseminated sclerosis
D Migraine
E Acute bacterial meningitis
Dr.HUR A. Salman - OdNMU

5-A 51 y.o. women was taken to the emergency department in convulsive status epilepticus. The
first means of medical management should be:
A Ensuring that the airway is open and the patient is oxygenating → ABC ..
B Inserting a tongue blade
C Administering an instravenous bolus of 50% dextrose
D Injecting 5 mg of diazepam followed by a loading dose of phenytoin
E Inducing pentobarbital coma
Status epilepticus (SE) is a single epileptic seizure lasting more than five minutes or two or
more seizures within a five-minute period without the person returning to normal between them

6-A 38 y.o. man complains of having occasional problems with swallowing of both hard and fluid
food for many months. Sometimes he feels intense pain behind his breast bone, epecially after
hot drinks. There are asphyxia onsets at night. He has not put off weight. Objectively: his
general condition is satisfactory, skin is of usual colour. Examination revealed no changes of
gastrointestinal tract. X-ray picture of thorax organs presents esophagus dilatation with level of
fluid in it. What is the preliminary diagnosis?
A Esophagus achalasia → is a failure of smooth muscle fibers to relax, which can cause a sphincter
to remain closed and fail to open when needed. 
B Myastenia
C Cancer of esophagus
D Esophagus candidosis
E Gastroesophageal reflux
Dr.HUR A. Salman - OdNMU
7- A 35 y.o. woman consulted a doctor about occasional pains in paraumbilical and iliac region that
reduce after defecation or passage of gases. Defecation takes place up to 6 times a day, stool
is not solid, with some mucus in it. Appetite is normal, she has not put off weight. First such
symptoms appeared 1,5 year ago, but colonoscopy data reveals no organic changes.
Objectively: abdomen is soft, a little bit painful in the left iliac region. Blood and urine are normal.
What is the preliminary diagnosis?
A Irritable bowels syndrome → (IBS) is a group of symptoms—including abdominal pain and changes in
the pattern of bowel movements without any evidence of underlying damage. These symptoms occur over a
long time, often years. It has been classified into four main types depending on whether diarrhea is
common, constipation is common, both are common, or neither occurs very often
B Celiac disease
C Crohn's disease
D Pseudomembranous colitis
E Dispancreatism

8-The physician must undertake measures for primary prophylaxis of iron deficiency anemia.
Which of the following categories of patient are subject to such primary prophylactic measures?
A Pregnant women
B Patients after 60
C All children
D Patients after operation
E Workers of industrial enterprises

9-A patient with unstable angina pectoris was given the following complex treatment:
anticoagulants , nitrates, α-adrenoblockers. However on the third day of treatment the pain still
romains. Which in vestigation shoud be carried out to establish diagnosis?
A Coronarography → X-ray imaging (Fluoroscopy) usually at femoral A. catheterization with
contrast injected and tracethe contrast → to see the coronary arteries stenosis or occlusion
Nowadays you can do diagnostic Coronarography by CT-scan
B Stress-echocardiogram
C Test with dosed physical exercises
D Esophageal electrocardiac stimulator
E Myocardial scintigraphy
Dr.HUR A. Salman - OdNMU

10-The 28 y.o. woman applied to doctor because of limited loss of the hair. In the anamnesis - she
had frequent headache indisposition, arthromyalgia, fever, irregular casual sexual life, drug
user. RW is negative. What examination must be done first?
A Examination for HIV
B Examination for neuropathology
C Examination for gonorrhea
D Examination for fungi
E Examination for trichomoniasis

11-A 35 y.o. woman was admitted to thoracic surgery department with fever up to $40^0C$, onset
of pain in the side caused by deep breathing, cough with considerable quantity of purulent
sputum and blood with bad smell. What is the most likely diagnosis?
A Abscess of the lung
B Complication of liver echinococcosis
C Bronchiectatic disease
D Actinomycosis of lungs
E Pulmonary tuberculosis
Dr.HUR A. Salman - OdNMU
12-A 67 y.o. patient complains of palpitation, dizziness, noise in ears, feeling of shortage of air.
Objectively: pale, damp skin. Vesicular respiration, respiratory rate- 22 per min, pulse- 200 bpm,
AP- 100/70 mm Hg. On ECG: heart rate- 200 bmp, ventricular complexes are widened,
deformed, location of segments ST and of wave T is discordant. The wave Р is not changed,
superimposes QRST, natural conformity between Р and QRS is not present. What kind of
arrhythmia is present?
A Paroxismal ventricular tachycardia→ due to QRS complex abnormaities
(ventricular/supraventricular)
B Sinus tachycardia→ should have no changes in P , QRS , only rhthy 
C Atrial flutter →no p wave
D Ventricular extrasystole → no ectopic beat.
E Atrial tachycardia → QRS not deform

13-Male 30 y.o., noted growing fingers and facial scull, changed face. Complains of poor eyesight,
weakness, skin darkening, loss of body weight. X-ray shows broadening of sella turcica, thinning
of tuberculin sphenoidale, signs of increased intracranial pressure. What diagnosis can you
make?
A Adenoma of hypophysis → Pitutary adenoma (micro/Macro) here because of widening sella
turscia (where pituitary lying) so is big size (Macroadenoma>10 mm )
B Encephalitis of truncus
C Optico - hiasmatic arachnoiditis
D Adrenal gland tumor
E Tumor of pondo-cerebellar corner
Dr.HUR A. Salman - OdNMU
14-A patient complains of a tormental (agonizing) cough with expectoration of up to 600 ml/daily
purulent chocolate color sputum with a decay smell. Onset of illness was abrupt, to-
39oC, fever of irregular type. There is the area of darkening with a cavity in a center on
X-ray film, with irregular contours and level of liquid. What disease is the question?
A Gangrene of lung → also called necrotizing pneumonia .
B Tuberculosis
C Bronchiectatic illness
D Pneumonia complicated by an abscess
E Lobar pneumonia

pulmonary gangrene differs by a terminal expression of signs. The state of the patients is critical. The patient
is adynamic, exhausted, with edemas on legs. Dyspnea in rest, hemodynamic disturbances are evident.
Dirty-grey or brown sputum with detrites, pieces of necrotic parenchyma and threads of blood excretes out
with the cough up to 1 l. Early pleural complications are usual and represent with pulmonary bleeding,
which may be profuse. Often it is associated with vital organ dysfunction and loss of consciousness.

15-A 24 y.o. patient complains of nausea, vomiting, headache, shortness of breath. He had an acute
nephritis being 10 y.o. Proteinuria was found out in urine. Objectively: a skin is grey-pale, the
edema is not present. Accent of II tone above aorta. BP 140/100-180/100 mm Hg. Blood level of
residual N2- 6,6 mmol/L, creatinine- 406 mmol/L. Day's diuresis- 2300 ml, nocturia. Specific
density of urine is 1009, albumin- 0,9 g/L, WBC- 0-2 in f/vis. RBC.- single in f/vis., hyaline casts
single in specimen. Your diagnosis?
A Chronic nephritis with violation of kidney function → impaired renal function due to  Urea and
creatinine (Normal refrences you will get it at the end of booklet)
B Feochromocitoma → pheochromocytoma : Catecholamine secreting tumor of the supra-renal
gland .
C Hypertensive illness of the II degree → his hypertension due to impaired renal function.
D Nephrotic syndrome → NO PROTEINURIA
E Stenosis of kidney artery → he have hx of nephritis , no stenosis info like Doppler U/S

16-A 33 y.o. male patient was admitted to a hospital. A patient is pale, at an attempt to stand up he
complains of strong dizziness. There was vomiting like coffee-grounds approximately hour ago.
BP- 90/60 mm Hg., pulse- 120 b/min. In anamnesis, a patient has suffered from ulcer of the
stomach, painless form during 4 years. An ulcer was exposed at gastrofiberoscopy. Your
diagnosis:
A Ulcer of stomach, complicated with bleeding→ hemorrhagic ulcer .
B Ulcer of duodenum, complicated with bleeding
C Erosive gastritis
D Acute pleurisy
E Acute myocardial infarction, abdominal form
Dr.HUR A. Salman - OdNMU

17-A 48-year-old patient complains of heaviness in the right hypochondrium, itching of the skin.He
had been treated in infectious diseases hospital repeatedly due to icterus and itch. On physical
exam: meteorism, ascitis, dilation of abdominal wall veins, protruded umbilicus, spleen
enlargement. What can be diagnosed in this case?
A Liver cirrhosis
B Cancer of the liver
C Cancer of the head of pancreas
D Gallstones
E Viral hepatitis B
Dr.HUR A. Salman - OdNMU
18-A 25-year-old man has facial edema, moderate back pains. His temperature is 37,5oС, BP
180/100 mm Hg, hematuria [up to 100 in v/f], proteinuria [2,0 g/L], hyaline casts - 10 in v/f.,
specific gravity -1020. The onset of the disease is probably connected with acute tonsillitis that
started 2 weeks ago. What is the most probable diagnosis?
A Acute glomerulonephritis → after 2 week of tonsilits he got that symptoms (post-streptococcal) ,
so it is acute
B Acute pyelonephritis → usually WBC appear in urine or BACTERIA
C Cancer of the kidney
D Urolithiasis → UT stone , no renal colic
E Chronic glomerulonephritis

19
In the development of the inflammation processes glucocorticoids reduce the level of certain
most important active enzyme. It results also in the reducing of the synthesis of prostaglandins
and leucotrienes which have a key role in the development of inflammation processes. What is
the exact name of this enzyme?
A Phospholipase A2 → check the enzyme function down 
B Arachidonic acid
C Lipoxygenasе
D Cyclooxygenase – 1
E Cyclooxygenase – 2
Dr.HUR A. Salman - OdNMU

20
A 30 y.o. female with rheumatoid arthritis of five years duration complains of pain in the frist
three fingers of her right hand over past 6 weeks. The pain seems especially severe at night
often awakening her from sleep.The most likelly cause is?
A Carpal tunnel syndrome
B Atlanto-axial sublaxation of cervical spine
C Sensory peripheral neuropathy
D Rheumatoid vasculitis
E Rheumatoid arthritis without complication

21-A 19-year-old man has suffered from moderate mental retardation since childhood. The patient
is illiterate, can take care of himself, do simple household work and other kinds of easy work
under supervision. What does his rehabilitation [tertiary prevention] require?
A All the above mentioned → All down mentioned ☺☺ I NEED HELP FROM ALL
B Supervision of a social worker
C Physical work under supervision
D Supervision of relatives (if any)
E None of the above mentioned

22- A 38 y.o. patient was urgently admitted to the hospital with complaints of sudden weakness,
dizziness, loss of consciousness, body weight loss, nausea, vomiting, severe pain in epigastric
area, diarrhea, skin hyperpigmentation. What is the most probable diagnosis?
A Addisonic crisis
Dr.HUR A. Salman - OdNMU
B Acute gastroenteritis
C Meningoencephalitis
D Scleroderma
E Pellagra

23- An unconscious patient presents with moist skin, shallow breathing. There are signs of
previous injection on the shoulders and hips. BP- 110/70 mm Hg. Tonus of skeletal muscles and
reflexes are increased. Cramps of muscles of the extremities are seen. What is the most likely
disorder?
A Hypoglycemic coma
B Hyperglycemic coma
C Hyperosmolar coma
D Hyperlactacidotic coma
E Stroke
Dr.HUR A. Salman - OdNMU
24-A patient was admitted to the hospital on the 7th day of the disease with complaints of high
temperature, headache, pain in the muscles, especially in calf muscles. Dermal integuments and
scleras are icteric. There is hemorrhagic rash on the skin. Urine is bloody. The patient was
fishing two weeks ago. What is the most likely diagnosis?
A Leptospirosis → Leptospirosis is an acute infectious zoonotic disease characterized by the primary
involvement of the kidneys, liver, nervous system, and circulatory organs. The causative agent of
leptospirosis is Leptospira interrogans.
B Yersiniosis → Y.enterocolitica - gastroenteritis F/N/V/abd pain
C Salmonellosis → S.Typhi /Typhoid –paratyphoid
D Brucellosis CULFF muscle ach
E Trichinellosis
IMPORTANT one
in ukraine

25- A 60-year-old woman, mother of 6 children, developed a sudden onset of upper abdominal
pain radiating to the back, accompanied by nausea, vomiting, fever and chills. Subsequently, she
noticed yellow discoloration of her sclera and skin. On physical examination the patient was
found to be febrile with temp of 38,9oC, along with right upper quadrant tenderness. The
most likely diagnosis is:
A Choledocholithiasis → common bile duct stones
in Q he gave you typical symptoms of cholangitis (Charchot triad = fever , upper quadrant pain and
jaunidce) , but this symptoms could share the CBD stone because stone in duct and cause
inflammation (cholangitis) and obstruction. , but no fever → then should be calculus cholangitis.
B Benign biliary stricture
C Malignant biliary stricture
D Carcinoma of the head of the pancreas
E Choledochal cyst
Dr.HUR A. Salman - OdNMU
26-A 42-year-old woman suffers from bronchial asthma, has an acute attack of bronchial asthma.
What medication from the listed below is contraindicated to render a first aid?
A Euphylinum → Euphyllin = Aminophylline / Theophyllin bronchodilator
B Izardin
C Corazolum
D Morphinum hydrochloride
E Strophanthin hydrochloride

27-4 days ago a 32-year-old patient caught a cold: he presented with sore throat, fatigue. The next
morning he felt worse, developed dry cough, body temperature rose up to 38,2oC, there
appeared muco-purulent expectoration. Percussion revealed vesicular resonance over lungs,
vesicular breathing weakened below the angle of the right scapula, fine sonorous and sibilant
wheezes. What is the most likely diagnosis?
A Focal right-sided pneumonia
B Bronchial asthma
C Acute bronchitis
D Pulmonary carcinoma
E Pulmonary gangrene

28-A 62-year-old patient complaining of enlargement of cervical, supraclavicular and axillary


lymph nodes, subfebrile temperature for the last 3 months has been admitted to a hospital. In
blood:WBCs – 64x109/l, lymphocytes - 72%. What method of study should be used to specify the
diagnosis?
A Myelogram → bone marrow puncture → CLL (chronic Lyphoid Leukemia)
B Lymphography
C Lymphoscintigraphy
D X-rays
E Thermography
Dr.HUR A. Salman - OdNMU
29-A 38 y.o. woman complains of a purulent discharge from the left nostril. The body temperature
is 37,5oC. The patient has been ill for a week and associates her illness with common cold.
There are a pain and tenderness on palpation of her left cheek. The mucous membrane in the
left nasal cavity is red and turgescent. The purulent exudate is seen in the middle meatus in
maxillary. What is the most probable diagnosis?
A Acute purulent maxillary sinusitis
B Acute purulent frontitis → upper meatus of noise
C Acute purulent ethmoiditis → middle nasal meatus
D Acute purulent sphenoiditis
E-

30- A 40-year-old female patient has been hospitalized for attacks of asphyxia, cough with phlegm.
She has a 4-year history of the disease. The first attack of asphyxia occurred during her stay in
the countryside. Further attacks occurred while cleaning the room. After 3 days of inpatient
treatment the patient's condition has significantly improved. What is the most likely etiological
factor?
A Household allergens
B Pollen
C Infectious
D Chemicals
E Psychogenic

31- The complications of acute cholecystitis which require surgical intervention are as follows
EXCEPT:
A Jaundice
B Empyema of the gall-bladder
C Emphysematous gall-bladder
D Gall-bladder perforation
E Cholangitis conditioned by the presence of stones in the bile tract

32-A 22-year-old girl has been complaining of having itching rash on her face for 2 days. She
associates this disease with application of cosmetic face cream. Objectively: apparent
reddening and edema of skin in the region of cheeks, chin and forehead; fine papulovesicular
rash. What is the most likely diagnosis?
A Allergic dermatitis
B Dermatitis simplex
C Eczema
Dr.HUR A. Salman - OdNMU
D Erysipelas
E Neurodermatitis

33- A schizophrenic patient considers himself to be "an outstanding scientist, a brilliant composer
and an unrivalled artist". He complains that "family and friends are always jealous of him and
want to poison him". Determine the psychopathological syndrome:
A Paranoiac → part of schizophrenia
B Paranoid This man
C Manic want
D Paratrophic poisoning me
E Hebephrenic

34- A 43-year-old alcohol abuser had not consumed alcohol for the last two days. In the evening he
claimed to see rats and feel like they bite his feet. The patient is disoriented, agitated, all the
time attempts to run somewhere. Specify the psychopathological syndrome:
A Delirious
B Amential → Mental impairment; state of being mentally handicapped.
C Oneiroid→ dream-like disturbances of one's consciousness by vivid scenic hallucinations
D Choreatic → jerky,involuntary movements, chiefly of the face and extremities.
E Ganser's syndrome

35- A 42-year-old patient complains of back pain, darkened urine, general weakness, dizziness that
occurred after treating a cold with aspirin and ampicillin. Objectively: the patient is pale, with
subicteric sclerae. HR - 98 bpm. Liver - +2 cm, spleen - +3 cm. In blood: RBCs -
2,6x1012/l, Hb - 60 g/l, CI - 0,9, WBCs - 9,4x109/l, basophils - 0,5%, eosinophils - 3%, stab
neutrophils - 6% segmented neutrophils - 58%, lymphocytes - 25%, monocytes - 7%, ESR - 38
mm/hour, reticulocytes - 24%. Total bilirubin - 38 millimole/l. What complication occurred in the
patient?
Dr.HUR A. Salman - OdNMU
A Acquired hemolytic anemia → Drug (Penicillin) induced immune mediated hemolytic anemia
B Toxic hepatitis
C Cholelithiasis → stones of common bile duct .
D Agranulocytosis → low or abscence granulocytes (a major class of white blood cells that includes
neutrophils, basophils, and eosinophils) drops below 500 cells/mm³ of blood.
E Paroxysmal nocturnal hemoglobinuria → spontaneous complement activation.

36-A hospital has admitted a 52-year-old patient with disseminated pulmonary tuberculosis,
complaints of acute pain in the right half of chest, that developed after going upstairs to the 3rd
floor; cough, dyspnea at rest, marked cyanosis. What kind of complication should suspected
first of all?
A Spontaneous pneumothorax
B Cardiac failure
C Pulmonary failure
D Pleuritis
E Acute myocardial infarction

37- A 38-year-old male patient has been taking alcohol for 3 years. 3 days after a regular drinking
period he felt anxiety and fear. It appeared to him that he was surrounded by spiders and
worms, pursued by some "condemnatory voices". His behaviour became aggressive. The
patient demonstrated correct self-awareness but impairment of temporal and spatial orientation.
What is the most likely diagnosis?
A Delirium alcoholicum → check Q 34 
B Alcoholic paranoia
C Alcoholic hallucinosis
D Alcoholic encephalopathy
E Pathologic intoxication
Dr.HUR A. Salman - OdNMU
38-A painter working at a motorcar plant has been diagnosed with moderately severe intoxication
with amide compounds of benzene. The in-patient treatment resulted in a considerable health
improvement. What expert decision should be made in this case?
A The patient should be issued a sick list for out-patient treatment → because patient improved so
not need to stay in-patient , discharge him and give him sick leave to continue treatment as a follow
up in out-patient.
B The patient may get back to work providing he will keep to hygiene and sanitary regulations
C The patient should be referred to the medio-social expert commission for evaluation of
percentage of work capicty loss
D The patient should be referred to the medio-social expert commission for attributing the
disability group because of an occupational disease
E-

39-A 16-year-old patient who has a history of intense bleedings from minor cuts and sores needs
to have the roots of teeth extracted. Examination reveals an increase in volume of the right knee
joint, limitation of its mobility. There are no other changes. Blood analysis shows an inclination
to anaemia (Hb- 120 g/l). Before the dental intervention it is required to prevent the bleeding by
means of:
A Cryoprecipitate → factor VIII (8) (Hemoph A) , vWF
B Epsilon-aminocapronic acid → fibrinolytic inhibitor, give at Bleeding associated with
fibrinolysis. Prevent conversion of plasminogen to plasmin that important to convert fibrinogen to
fibrin the lsat process in clot formation
C Fibrinogen → clotting Factor (I) Fibrinogen is used to treat bleeding episodes in people with a
congenital fibrinogen deficiency
D Dried blood plasma → don’t search this ☺
E Calcium chloride → Tx. of k+ , Mg+ ,Ca+, calcium channel blocker toxicity & to prevent
Hypotension from CCB

40- A 44-year-old patient complains about difficult urination, sensation of incomplete urinary
bladder emptying. Sonographic examination of the urinary bladder near the urethra entrance
revealed an oval well-defined hyperechogenic formation 2x3 cm large that was changing its
position during the examination. What conclusion can be made?
A Concrement = stones , when pt. change position it will move inside urinary bladder
B Malignant tumour of the urinary bladder → here you can see formation (hypoechoic (grey color
in U/S) or homogenous mass without acoustic shadow) not changing position of patient during
ultrasound exam , in addition to that he will tell you the general symptoms of CA like , subfebrile T
for long time , weight loss , LN 
C Urinary bladder polyp → hypoechoic formation (dark color in U/S) , not changing position
D Prostate adenoma
E Primary ureter tumour
Dr.HUR A. Salman - OdNMU

A B C
There pic is ultrasound examination for , stone , Tumor, polyp

41- A 7-year-old boy had complained of headache, nausea, fatigue for 3 weeks. His condition
gradually deteriorated, headache and general weakness progressed. The boy had bronchitis at
the age of 3. His father has a history of pulmonary tuberculosis. Objectively: body temperature
37,5oC, conscious, lies supine, with the hip and knee flexed to 90 degrees, nuchal rigidity +6cm,
partial ptosis of the right eyelid, the dilated right pupil. General hyperalgesia is present. Liquor:
transparent, pressure - 400 mm of water column, protein - 1,5%, cytosis - 610/3 with predominant
lymphocytes, sugar - 1,22 mmol/l, chlorides - 500 mmol/l. What is the most likely diagnosis?
A Tuberculous meningitis
B Secondary purulent meningitis
C Epidemic cerebrospinal meningitis
D Serous meningitis
E Pneumococcal meningitis
Differential diagnosis of meningitis
Tuberculosis Purulent bacterial Subarachnoid
Signs Meningismus Viral meningitis
meningitis meningitis hemorrhage
Colorless, Colorless, bloody, after
Color, Colorless, White-yellow
transparent or xanthochromi settling –
transparence transparent or green, muddy
opalescent cor opalescent xanthochromic
Pressure(mm. H2
250-500
O), flow out below 180-200 200-300 250-500 250-400
jet, sometimes rare
speed (drops 50-80 60-90 60-90 > 70 or jet
drops
per 1 minute)
It is hard to count
in the first days,
cytosis (in 1 mkl.) 2-12 20-800 200-700 500-1000 andmore
from 5-7 day15-
120
cytogram from 5-7 day
lymphocytes, % 80-85 80-100 40-60 0-30 lymphocytes
neutrophyls, % 15-20 0-20 20-50 30-100 prevail
protein, g/l 0.16-0.33 0.33-1.0 1.0-3.3 0.66-16.0 0.66-16.0
sedimentation
– +(++) +++(++++) +++(++++) +++
tests (Pandy)
cellular-proteinon
the low level (from protein- cellular-proteinon
Dissociations Absent –
8-10 day – protein- cellular the high level
cellular)
Often rough
Fibrin pellicle – - in 3-5 % Often as a sediment Rare
in30-40 %
For 2- Normal or slightly
glucose, mmol/l 2.2-3.3 2.2-3.3 3 weeks1.0-2.0
normal
less than normal
Dr.HUR A. Salman - OdNMU
42- A 35-year-old patient complains about pain and morning stiffness of hand joints and
temporomandibular joints that lasts over 30 minutes. She has had these symptoms for 2 years.
Objectively: edema of proximal interphalangeal digital joints and limited motions of joints. What
examination should be administered?
A Roentgenography of hands → X ray of Hand – Rheumatoid arthritis
B Complete blood count
C Rose-Waaler reaction →if +ve = presence of rheumatic factor
D Immunogram
E Proteinogram

43-A 69-year-old female patient complains of temperature rise up to 38,3oC, haematuria. ESR- 55
mm/h. Antibacterial therapy turned out to be ineffective. What diagnosis might be suspected?
A Renal cancer
B Polycystic renal disease → no fever , usually congenital .
C Renal amyloidosis → No protein info.
D Urolithiasis → no fever , colic pain radiation to pubis and inguinal
E Chronic glomerulonephritis → no previous infection , no protein .

44- Examination of a 35-year-old patient with rheumatism revealed that the right heart border was
1 cm displaced outwards from the right parasternal line, the upper border was on the level with
inferior margin of the 1st rib, the left border was 1 cm in from the left midclavicular line.
Auscultation revealed atrial fibrillation, loud apical first sound, diastolic shock above the
pulmonary artery. Echocardiocopy revealed abnormal pattern of the mitral valve motion. What
heart disease is characterized by these symptoms?
A Mitral stenosis → check down .
B Mitral valve prolapse
C Mitral valve insufficiency
D Aortic stenosis
E Tricuspid valve insufficiency
mitral stenosis — is a narrowing of the heart's mitral valve. This abnormal valve doesn't open
properly, blocking blood flow into the main pumping chamber of your heart (left ventricle). Mitral
valve stenosis can make you tired and short of breath, among other problems.

- The main cause of mitral valve stenosis is an infection called rheumatic fever, which is related to
strep infections. Rheumatic fever — now rare in the United States, but still common in developing
countries — can scar the mitral valve. Left untreated, mitral valve stenosis can lead to serious heart
complications.
Dr.HUR A. Salman - OdNMU

45- A 54-year-old male patient works as an engineer. At the age of 35, he got infected with syphilis
and treated it with "traditional remedies". About 5 years ago, he became forgetful, unable to cope
with work, told cynical jokes, bought useless things, collected cigarette butts in the street.
Objectively: the patient is indifferent, has slow speech, dysarthria, can make only primitive
judgments, is unabe to perform simple arithmetic operations or explain simple metaphors. The
patient is untidy, takes no interest in anything, passive. Considers himself to be completely
healthy. Qualify mental condition of the patient:
A Total dementia
B Lacunar (dysmnestic) dementia
C Somnolentia
D Korsakoff's (amnesic) syndrome
E Hysterical pseudodementia

Who am I ?

46- A 47-year-old patient complains of insomnia, heaviness over his entire body, constantly
depressed mood. He considers himself good-for-nothing, inadequate. Believes that he is a burden
to his family, wants to die. The patient is depressed, inactive, has a hypomimic face with sorrowful
expression. He speaks quietly and monotonely,gives short answers. What is the most
likely diagnosis?
A Major depressive disorder
B Atherosclerotic depression
C Initial stage of Alzheimer's disease
D Late-onset schizophrenia
E Neurotic depression
Dr.HUR A. Salman - OdNMU

Hope you not getting


this DEPRESSSION
before KROK

47- A patient, aged 16, complains of headache, mainly in the frontal and temporal areas,
superciliary arch, appearing of vomiting at the peak of headache, pain during the eyeballs
movement, joint's pain. On examination: excited, to- 39oC, Ps- 110/min. Tonic and clonus cramps.
Uncertain meningeal signs. What is the most likely diagnosis?
A Influenza with cerebral edema manifestations → pt had symptoms of Influenza (virus) then
appeared neurological symptoms (Tonic- clonic) and no meningeal symptoms, could be viral
encephalitis as a complication
B Influenza, typical disease duration
C Respiratory syncytial virus
D Parainfluenza
E Adenovirus infection

48-A 64 y.o. patient has developed of squeering substernal pain which had appeared 2 hours ago
and irradiated to the left shoulder, marked weakness. On examination: pale skin, cold sweat.
Pulse- 108 bpm, AP- 70/50 mm Hg, heart sound are deaf, vesicular breathing, soft abdomen,
painless, varicouse vein on the left shin, ECG: synus rhythm, heart rate is 100 bmp,
ST-segment is sharply elevated in II, III aVF leads. What is the most likely disorder?
A Cardiogenic shock → pt. had shock symptoms also have heart abnormalities , sure he got this
shcok due to throms that came from leg and make IHD.
B Cardiac asthma
C Pulmonary artery thromboembolia→ BE CAREFUL , here also you can think by this because he
have varicouse vein in left leg , so may this thromb can go to pulmonary artery , BUT , why pt. got
shock ?? so go back to A and click it ☺
D Disquamative aortic aneurizm
E Cardiac tamponade → Beck’s triad (Bp, JVP, muffled heart sound)

49-A 64-year-old patient complains of severe pain in the right side of chest, dyspnea, dry cough
which appeared suddenly on exertion. Objectively: the right side of the chest lags behind in the
act of breathing. Percussion reveals tympanic sound. Auscultation reveals pronouncedly
diminished breath sounds on the right. Ps- 100/min, weak, arrhythmic. AP- 100/50 mm Hg.
Cardiac sounds are decreased. What disease can be suspected in this patient?
A Right-sided pneumothorax → air in pleura .
B Right-sided hydrothorax → fluid in pleura .
C Right-sided dry pleurisy → inflammation of the pleura .
D Right-sided pleuropneumonia → is inflammation of the lungs and pleura,
E PATE → pulmonary atery thrombo-Embolism. → Virchow triad
Lag= late , slower tha other side
Dr.HUR A. Salman - OdNMU

50-A 29-year-old female patient complains of dyspnea, heaviness and chest pain on the right, body
temperature rise up to 37,2oC. The disease is associated with a chest trauma received 4days ago.
Objectively: skin is pale and moist. Ps- 90 bpm, regular. Palpation reveals a dull sound
on the right, auscultation reveals significantly weakened vesicular breathing. In blood: RBCs -
2,8x1012/l, colour index - 0,9, Hb- 100 g/l, WBCs - 8,0x109/l, ESR - 17 mm/h. What results of
diagnostic puncture of the pleural cavity can be expected?
A Haemorrhagic punctuate → due to hemorrhage after trauma inside rt. Side chest
B Chylous liquid → pus
C Exudate → extravascular fluid with high protein .. like in infection, TB…
D Transudate → extravascular fluid with low protein like in heart failure…
E Purulent punctuate → pus

51-A 54-year-old drowned man was rescued from the water and delivered to the shore. Objectively:
the man is unconscious, pale, breathing cannot be auscultated, pulse is thready. Resuscitation
measures allowed to save the patient. What complications may develop in the near future?
A Pulmonary edema
B Respiratory arrest
C Encephalopathy
D Cardiac arrest
E Bronchospasm
Dr.HUR A. Salman - OdNMU

52-An 18-year-old patient since childhood suffers from bleeding disorder after minor injuries. His
younger brother also has bleeding disorders with occasional haemarthrosis. Which laboratory
test will be informative for diagnosis verification?
A Clotting time → is the time required for a sample of blood to coagulate
In Q they mean to confirm Hemophilia (BOY)  , his brother also have same disease this guide us
to hemophilia , so will see  APPT (active partial thromboplastin time)
B Fibrinogen rate
C Blood clot retraction
D Thrombocyte count
E Determination of prothrombin time

53-A 28-year-old patient complains of periodic compressing heart pain. His brother died at the age
of 34 from a cardiac disease with similar symptoms. Objectively: the patients skin is pale. Heart
borders display no significant deviations. Heart sounds are loud, there is a systolic murmur
above all the points with a peak above the aorta. Echocardioscopy reveals thickening of the
interventricular septum in the basal parts, reduction of left ventricular cavity. What drug should
be administered in order to prevent the disease progression?
A Metoprolol → selective β 1 blocker
B Digoxin
C Nitroglycerin
D Captopril
E Furosemide

54- A patient, aged 40, has been ill during approximately 8 years, complains of pain in the lumbar
part of the spine on physical excertion, in cervical and thoracal part (especially when coughing),
pain in the hip and knee joints on the right. On examination: the body is fixed in the forward
inclination with head down, gluteal muscles atrophy. Spine roentgenography: ribs osteoporosis,
longitudinal ligament ossification. What is the most likely diagnosis?
A Ancylosing spondyloarthritis
Dr.HUR A. Salman - OdNMU
B Tuberculous spondylitis
C Psoriatic spondyloarthropatia
D Spondyloarthropatia on the background of Reiter's disease
E Spread osteochondrosis of the vertebral column

A B C
** radiological findings: A/ Bmboo spine –B/ Dagger sign – C/ sacroiliatis

55-A worker, aged 38, working in the slate production during 15 years, complains of expiratory
exertional dyspnea, dry cough. On examination: deafening of the percutory sounds in
interscapular region, rough breath sounds, dry disseminated rales. On fingers' skin - greyish
warts. Factory's sectorial doctor suspects asbestosis. Which method is the most informative for
diagnosis verification?
A Thorax roentgenography → chest x ray → INTERSTITIAL PNEUMOCLEROSIS
B Bronchoscopy
C Spirography
D Bronchoalveolar lavage
E Blood gases examination
** Dx of asbestosis → in sputum : asbestos bodies , in skin : asbestos warts (containing asbestos)
Dr.HUR A. Salman - OdNMU
56- A 37 y.o. woman is suffering from squeezing substernal pain on physical exertion. On
examination: AP- 130/80 mm Hg, heart rate=pulse rate 72 bpm, heart boarders are dilated to
the left side, aortic systolic murmur. ECG- signs of the left venticle hypertrophy. What method of
examination is the most informative in this case?
A Echocardiography
B Phonocardiography → PCG is a plot of high-fidelity recording of the sounds and murmurs made
by the heart with the help of this machine
C Coronarography → Q 9
D Sphygmography → mechanical device used to measure blood pressure
E X-ray

57-A 58-year-old woman complains of osteoarthrosis of knee-joint. For 2 weeks she had been
receiving an in-patient medical treatment. She was discharged from the hospital in satisfactory
condition with complaints of minor pain after prolonged static work. Local hyperemia and
exudative effects in the area of joints are absent. What further tactics is the most expedient?
A Outpatient treatment→ because she became better after hospital tx. ; now she need just follow up
after tx. escpcially she have OA this is degenerative disease (her age).
B Repeated in-patient treatment
C Conducting arthroscopy
D Refferral to MSEC → MSEC = Medico-social Expert commission. / this for determination of
disability degree and need for sick leave in case of employee
E Orthopedist consultation
B,C,E → no indication and not need because she already became better after in-pt treatment.

58-A 36-year-old female has a 7-year history of pollen allergy. Over the last 2 years in August and
September (during ragweed flowering), the patient has had 2-3 asthma attacks that could be
treated with one dose of salbutamol. Objectively: body temperature - 36,5oC, respiratory rate
- 18/min, Ps - 78/min, AP - 115/70 mm Hg. There is vesicular breathing above the lungs.
Cardiac sounds are sonorous, of regular rhythm. What drug would be most effective to prevent
asthma attacks during the critical season for the patient?
A Intalum inhalation → Cromoglicic acid or cromoglicate) is a mast cell stabilizer, cromolyn
sodium. This drug prevents the release of inflammatory chemicals such
as histamine from mast cells.
It is not histamine antagonist / bronchodilator →therefore NOT used in ASTHMATIC ATTACK
B Berotec inhalation → Albuterol—Salbutamol : Bronchodilators, Adrenergic B blocker
C Atrovent inhalation → ipratropium / is a bronchodilator that relaxes muscles in the airways and
increases air flow to the lungs also to prevent bronchospasm, or narrowing airways in the lungs, in
people with bronchitis, emphysema, or COPD
Dr.HUR A. Salman - OdNMU
D Suprastin administration → Chloropyramine Antiallergic agent Histamine H₁-receptor
antagonist (H₁-antihistaminic)
E Theopecum administration → Etofylline and Theophylline / Antiasthmatic agent
Bronchodilator / Non-selective phosphodiesterase inhibitor / Cardiac stimulant

59- A 42-year-old patient applied to hospital with complaints of pain behind the sternum with
irradiation to the left scapula. The pain appears during significant physical work, this lasts for
5-10 minutes and is over on rest. The patient is sick for 3 weeks. What is the preliminary
diagnosis?
A IHD:First established angina pectoris → beginning of angina , because symptoms start 3 wks and
mild form , it resolved by rest .
B IHD:Variant angina pectoris (Prinzmetal's)
C IHD:Stable angina pectoris of effort I FC
D IHD:Stable angina pectoris of effort IV FC
E IHD:Progressive angina pectoris
IHD = Ischemic Heart Disease .

60-Medical examination of a 19-year-old worker revealed generalized lymphadenopathy mainly


affecting the posterior cervical, axillary and ulnar lymph nodes. There are multiple injection marks
on the elbow bend skin. The man denies taking drugs, the presence of injection marks ascribes to
influenza treatment. Blood count: RBCs- 3,2x1012/l, Hb- 100 g/l, WBCs- 3,1x109/l, moderate
lymphopenia. What study is required in the first place?
A ELISA for HIV
B Immunogram
Dr.HUR A. Salman - OdNMU
C Sternal puncture
D X-ray of lungs
E Lymph node biopsy

61-A 25-year-old female patient complains of marked weakness, sleepiness, blackouts, dizziness,
taste disorder. The patient has a history of menorrhagia. Objectively: the patient has marked
weakness, pale skin, cracks in the corners of mouth, peeling nails, systolic apical murmur. Blood
test results: RBC - 3,4x1012/l, Hb - 70 g/l, color index - 0,75, platelets – 140x109/l, WBC - 6,2x109/l.
What is the most likely diagnosis?
AChronic posthemorrhagic anemia → repeated excessive blood loss in each menstrual cycle leading
to anemia , as the pt. had cracks of mouth, peeling nails = these symptoms need time to appear ,
that my explaining  for this they chose chronic not acute and here at least 1-2 or more menstrual
cycle to appear that sx.
B Acute leukemia
C Acute posthemorrhagic anemia → check above , actually the didn’t said about duration and
amount , but we can determine it from symptoms , in acute not appear corner of mouth , peeling
nail , at least 2 or 3 menstural bleeding till that appear = and that’s make it chronic
D B12-deficiency anemia → no info , no Macrocytosis.
E Werlhof's disease → idiopathic Thrombocytopenic purpura .

62-A 24-year-old patient felt sick in 16 hours after dried fish intake. There was nausea, vomiting,
weakness, flabbiness, double vision. On physical exam, there was decrease of a muscle tone,
anisocoria, flaccid swallowing and tendon reflex. What is the most probable diagnosis?
A Botulism
B Food toxicoinfection STOCK-FISH
C Acute gastritis → BOTULISM
D Acute encephalitis
E Salmonellosis
Dr.HUR A. Salman - OdNMU
63- A 72-year-old patient after operation due to holecystectomia was prescribed gentamicin (80
mg every 8 hours) and cephalothin (2 g every 6 hours) due to fever. In 10 days there was an
increase of creatinine up to 310 mu*mol/L. BP - 130/80 mm Hg, daily quantity of the urine is
1200 mL. Urine tests are without pathology. Ultrasound: the size of kidneys is normal. What is the
most probable reason for renal failure?
A Nephrotoxity of gentamicin
B Acute glomerulonephritis
C Cortical necrosis of kidneys
D Unequal infusion of the liqiud
E Hepatorenal syndrome

64-A 40 y.o. patient was admitted to the gasteroenterology with skin itching, jaundice, discomfort
in the right subcostal area, generalized weakness. On examination: skin is jaundice, traces of
scratches, liver is +5 cm, splin is 6x8 cm. In blood: alkaline phosphatase - 2,0 mmol/(hour*L),
general bilirubin - 60 mkmol/L, cholesterol - 8,0 mmol/L. What is the leading syndrome in the
patient?
A Сholestatic → is a condition where bile cannot flow from the liver to the duodenum. The two
basic distinctions are an obstructive type of cholestasis where there is a mechanical blockage in the
duct system that can occur from a gallstone or malignancy, and metabolic types of cholestasis
which are disturbances in bile formation that can occur because of genetic defects or acquired as a
side effect of many medications.
B Сytolytic
C Mesenchymal inflammatory
D Asthenic
E Liver-cells insufficiency

65-A 55-year-old male had been treated at the surgical department for acute lower-extremity
thrombophlebitis. On the 7th day of treatment he suddenly developed pain in the left part of
chest, dyspnea and cough. Body temperature was 36,1oC, respiratory rate - 36/min. The
patient was also found to have diminished breath sounds without wheezing. Ps- 140/min,
thready. AP- 70/50 mm Hg. The ECG shows QІІІ-S1 syndrome. What is the most likely
diagnosis?
A Pulmonary embolism → Virchow’s triad = (vessels wall injury – stasis – hypercoagulability )
B Myocardial infarction
Dr.HUR A. Salman - OdNMU
C Cardiac asthma
D Bronchial asthma
E Pneumothorax

66 A 51-year-old female is a weaving factory worker with 15 years of service record. During a
regular preventive examination she complained of frequent headaches, poor sleep, tingling in
the heart, irritability, rapid fatigability, hearing impairment. For years, the noise level has
exceeded the maximum allowable concentration by 10-15 dB. A year ago, the patient underwent
a course of treatment for essential hypertension. Specify the most likely diagnosis:
A Noise disease
B Essential hypertension
C Neurasthenia
D Asthenic-vegetative syndrome
E Arteriosclerotic encephalopathy

67- A 26-year-old patient undergoes a course of treatment due to chronic glomerulonephritis. The
treatment was successful, normalization of all the characteristics was recorded. What sanitorium
and health resort treatment could be recommended?
A The south coast of the Crimea → each student who pass krok can go there ☺☺☺ nice place
B Not recommended
C Morshyn
D Myrhorod
E Truskavets
Dr.HUR A. Salman - OdNMU

68-After a wasp-bite there was an itching of skin, hoarse voice, barking cough, anxiety. On physical
exam: there is edema of lips, eyelids, cyanosis. What medicine is to be taken first?
A Prednisolone → for stop allergic reaction.
B Adrenalin
C Euphylin
D Lasix
E Seduxen

69-A 16-year-old adolescent was vaccinated with DTP. In eight days there was stiffness and pain in
the joints, subfebrile temperature, urticarial skin eruption, enlargement of inguinal, cervical
lymph nodes and spleen. What kind of allergic reaction is observed?
A Immunocomplex
B Hypersensitivity of immediate type
C Cytoxic
D Hypersensitivity of delayed type
E-
Dr.HUR A. Salman - OdNMU
70-2 weeks after recovering from angina a 29-year-old patient noticed face edemata, weakness,
decreased work performance. There was gradual progress of dyspnea, edemata of the lower
extremities, lumbar spine. Objectively: pale skin, weakening of the heart sounds, anasarca. AP-
160/100 mm Hg. In urine: the relative density - 1021, protein - 5 g/l, erythrocytes - 20-30 in the
field of vision, hyaline cylinders - 4-6 in the field of vision. What is the most likely diagnosis?
A Acute glomerulonephritis → post streptocococcus (angina=Tonsillitis)
B Essential hypertension
C Acute pyelonephritis
D Infectious allergic myocarditis
E Myxedema

71 -A 56-year-old scientist experiences constricting retrosternal pain several times a day while
walking for 100-150 m. The pain lasts for up to 10 minutes and can be relieved by nitroglycerine.
Objectively: the patient is overweight, heart borders exhibit no abnormalities, heart sounds are
rhythmic, Ps- 78 bpm, AP- 130/80 mm Hg. ECG contains low amplitude of T wave in
V4-5. What disease might be suspected?
A Stable FC III stenocardia → 150 -100 m
B Instable stenocardia
C Stable FC I stenocardia → severe physical exertion
D Stable FC II stenocardia → 500 – 700 m
E Stable FC IV stenocardia → at rest
Dr.HUR A. Salman - OdNMU
72-In autumn a 25-year-old patient developed stomach ache arising 1,5-2 hours after having meals
and at night. He complains of pyrosis and constipation. The pain is getting worse after
consuming spicy, salty and sour food, it can be relieved by means of soda and hot-water bag.
The patient has been suffering from this disease for a year. Objectively: furred moist tongue.
Abdomen palpation reveals epigastrial pain on the right, resistance of abdominal muscles in the
same region. What is the most likely diagnosis?
A Duodenal ulcer
B Chronic cholecystitis
C Diaphragmatic hernia
D Stomach ulcer
E Chronic pancreatitis

73-A 51 y.o. woman complains of dull pain in the right subcostal area and epigastric area, nausea,
appetite decline during 6 months. There is a history of gastric peptic ulcer. On examination:
weight loss, pulse is 70 bpm, AP is 120/70 mm Hg. Diffuse tenderness and resistance of
muscles on palpation. There is a hard lymphatic node 1x1cm in size over the left clavicle. What
method of investigation will be the most useful?
A Esophagogastroduodenoscopy with biopsy
B Ultrasound examination of abdomen
C pH-metry
D Ureatic test
E Stomach X-ray
Dr.HUR A. Salman - OdNMU
74- On the 20th of June a townsman was brought to clinic. The disease broke out acutely, starting
with fever, rise in temperature to 38-39oC. There also was weakness, acute headache,
nausea, vomiting, pain all over the body, sleep disorder. On physical examination: hyperemia of
skin of face, neck, thorax. Meningeal signs are positive. 12 days ago the patient returned from
the Siberia, from the forest. What is the most probable diagnosis?
A Tick-borne encephalitis → Siberia risk of Infection TBE
B Influenza
C Omsk hemorrhagic fever
D Pseudotuberculosis
E Epidemic typhus

75-A 37-year-old patient has sudden acute pain in the right epigastric area after having fatty food.
What method of radiological investigation is to be used on the first stage of examining the
patient?
A Ultrasonic → for checking any RUQ pathology , liver or Gall bladder to exclude , Gall stones
(Cholelithiasis) ? or cholestasis?
B Roentgenological
C Radionuclid
D Magnetic-resonance
E Thermographic

76-A man, aged 68, complains of tiredness, sweating, enlargement of cervical, submaxillary and
axillary lymph nodes. Blood test: WBC- 35*109/L, lymphocytes - 60%, Botkin and Gumprecht
bodies, level of haemoglobin and quantity of thrombocytes is normal. Myelogram showed 40% of
lymphocytes. What is the most probable diagnosis?
A Chronic lympholeucosis → Chronic Lymphocytic Leukemia CLL most common in elderly (40% >
60 y.o) male 2:1 female , SMUDGE CELL = GUMPRECHT BODY
B Chronic myeloleucosis
C Lymphogranulomatosis
D Acute leucosis
E Tuberculous lymphadenitis
Dr.HUR A. Salman - OdNMU
DO YOU KNOW !!
Gumprecht body it is same
(smudge cell)

77-A 38-year-old patient is under observation having polyneuritic syndrome with considerable loss
of weight, fever, rise in BP. Blood test:: considerable inflammatory changes. What examination
is the most expedient to make the diagnosis?
A Muscular biopsy with histological investigation of the material
B Determination of antinuclear antibodies
C Electromyography
D Blood culture
E Determination of HLA antigens
** Guillain–Barré syndrome or Acute idiopathic polyneuritis
- rapid-onset muscle weakness caused by the immune system damaging the peripheral nervous
system.
- 30% of cases are provoked by Campylobacter jejuni bacteria, which cause diarrhea. A further 10%
are attributable to cytomegalovirus (CMV, HHV-5).
- Two other herpesviruses (Epstein–Barr virus/HHV-4 and varicella zoster virus/HHV-3) and the
bacterium Mycoplasma pneumoniae have been associated with GBS

2 3
1
Dr.HUR A. Salman - OdNMU
78-A 32-year-old male patient has been suffering from pain in the sacrum and coxofemoral joints,
painfulness and stiffness in the lumbar spine for a year. ESR- 56 mm/h. Roentgenography
revealed symptoms of bilateral sacroileitis. The patient is the carrier of HLA B27 antigen. What
is the most likely diagnosis?
A Ankylosing spondylitis → check Q 54
B Coxarthrosis
C Rheumatoid arthritis
D Reiter's disease
E Spondylosis

79-A 58-year-old female patient complains about periodical headache, dizziness and ear noise.
She has been suffering from diabetes mellitus for 15 years. Objectively: heart sounds are
rhythmic, heart rate is 76/min, there is diastolic shock above aorta, AP is 180/110 mm Hg. In
urine: OD- 1,014. Daily loss of protein with urine is 1,5 g. What drug should be chosen for
treatment of arterial hypertension?
A Ihibitor of angiotensin converting enzyme → ACE inhibitor .
B $\beta$-blocker
C Calcium channel antagonist
D Thiazide diuretic
E $\alpha$-blocker
As the pt. had DM & renal disease so
kidney will release Angiotensin I by
Renin then converted to Angiotensin II
in lung , that made vasoconstriction
and lead to Hypertension , so if we
want to  Bp , we should stop the
action of Angiotensin

80-A 26 y.o. male patient with postoperative hypothyroidism take thyroxine 100 mg 2 times a day.
He has developed tachycardia, sweating, irritability, sleep disorder. Determine further treatment
tactics.
A To decrease thyroxine dosage → because pt have symptoms of hyperthyroidism , need to
decrease drug doses.
B To increase thyroxine dosage
C To administer betablockers
D To add mercasolil to the treatment
E To administer sedatives

81-A 28-year-old man was discharged from the hospital after having an out-of hospital pneumonia
He has no complaints. On physical exam: his temperature is 36,6oC, RR-18/min, Ps – 78 bpm, BP-
120/80 mm Hg. During ausculation there is harsh respiration to the right of the lower part of the
lung. Roentgenologically: infiltrative changes are absent, intensification of the pulmonary picture
to the right in the lower lobe. How long should the doctor keep the patient under observation?
A 12 months → each 1 year need one time observation (community acquired Pneumonia).
B 1 month
C 3 months
Dr.HUR A. Salman - OdNMU
D 6 months
E Permanently

82-A 20-year-old adolescent lives in the nidus of tuberculous infection. The tuberculine Mantoux
test with 2 TU was determined as hyperergic. What signs determine the hyperergic test of this
adolescent?
A 6 mm papula, necrosis
B 20 mm papula
C 24 mm hyperemia
D 4 mm papula
E 12 mm hyperemia

83-A survey radiograph of a miner (24 years of service record, the dust concentration in the
workplace is at the rate of 260-280 mg/m3 with 15% of free silica) shows lung changes that
are typical for pneumoconiosis. What type of pneumoconiosis is it?
A Anthracosilicosis
B Carboconiosis
C Silicatosis
D Anthracosilicatosis
E Silicosis

84-A patient complains of pathological lump, appearing in the right inguinal region on exercise.
The lump is round-shaped, 4 cm in diameter, on palpation: soft elastic consistency, is positioned
near the medial part of Poupart's ligament. The lump is situated inwards from the spermatic
cord. What is the most probable preliminary diagnosis?
Dr.HUR A. Salman - OdNMU
A Right-sided direct inguinal hernia
B Right-sided oblique inguinal hernia
C Right-sided femoral hernia
D Varicose veins of the right hip
E Lipoma of the right inguinal area

85-A 35-year-old man was operated on peptic ulcer of the stomach. Mass deficit of the body is 10
kg. The level of glucose after operation in the undiluted cellular blood on an empty stomach is
6,7 mmol. During repeated examination - 11,1 mmol (after meal), level of HbA1c - 10%. Could
you please make an interpretation of the given data?
A Diabetes mellitus
B Disordered tolerance to glucose
C Diabetes mellitus risk group
D Norm
E Postoperative hyperinsulinemia
Dr.HUR A. Salman - OdNMU
86-A 52 y.o. woman complains of weakness, painful itching after washing and bathing, sensation
of heaviness in the head. On examination: hyperemia of skin of face, neck, extremities. АP-
180/100 mm Hg. Speeln is 4 cm below the rib arch edge. What is the most probable diagnosis?
A Erythremia
B Essential hypertension
C Dermatomyositis
D Allergic dermatitis
E Systemic sclerodermia

87 -A 37-year-old patient was brought to resuscitation unit. General condition of the patient is very
serious. Sopor. The skin is grey, moist. Turgor is decreased. Pulse is rapid, intense. BP -
160/110 mm Hg, muscle tonus is increased. Hyperreflexia. There is an ammonia odor in the air.
What is the presumptive diagnosis?
A Uraemic coma → the condition of having "urea in the blood". Urea is one of the primary
components of urine. It can be defined as an excess of amino acid and protein metabolism end
products, such as urea and creatinine, in the blood that would be normally excreted in the
urine. Uremic syndrome can be defined as the terminal clinical manifestation of kidney failure (also
called renal failure).
B Alcoholic coma
C Hyperglycemic coma
D Hypoglycemic coma
E Cerebral coma

88-A 57-year-old man complains of shortness of breath, swelling on shanks, irregularity in cardiac
work, pain in the left chest half with irradiation to the left scapula. Treatment is uineffective. On
physical exam: heart's sounds are diminished, soft systolic murmur on the apex. Ps - 100/min,
arrhythmical, BP - 115/75 mm Hg. The liver is +2 cm, painful. Roentgenoscopy: enlargement of
heart shadow to all sides, pulsation is weak. Electrocardiogram (ECG): left ventricled extrasystolia,
decreased voltage. What method of investigation is necessary to do to determine the diagnosis?
A Echocardiography
B Veloergometria → Cardiac stress test, testing of the heart ("treadmill test")
C X-ray kymography
D ECG in the dynamics
E Coronarography
Dr.HUR A. Salman - OdNMU
89- A 55 y.o. patient complains of distended abdomen and rumbling, increased winds evacuation,
liguid foamy feces with sour smell following the diary products consumption. What is the correct
name of this syndrome?
A Syndrome of fermentative dyspepsia
B Syndrome of decayed dyspepsia
C Syndrome of fatty dyspepsia
D Dyskinesia syndrome
E Malabsorption syndrome

90- A 54-year-old patient has an over 20-year history of femoral osteomyelitis. Over the last month
she has developed progressing edemata of the lower extremities. Urine test reveals: proteinuria
at the rate of 6,6 g/l; in blood: dysproteinemia in form of hypoalbuminemia, increase in α2- and γ-
globulin rate, ESR - 50 mm/h. What is the most likely diagnosis?
A Secondary renal amyloidosis
B Acute glomerulonephritis
C Myelomatosis
D Chronic glomerulonephritis
E Systemic lupus erythematosus
Dr.HUR A. Salman - OdNMU
91- In an inhabited locality there is an increase of diphtheria during the last 3 years with separate
outbursts in families. What measure can effectively influence the epidemic process of
diphtheria and reduce the morbidity rate to single cases?
A Immunization of the population
B Hospitalization of patients
C Detection of carriers
D Early diagnostics
E Disinfection in disease focus
** DTP vaccine from 3 months age 3 times with 30 days interval (3, 4, 5 months), revaccination in
18 months (DTaP), 6, 14, 18 years (DT), later – every 10 years.

92-A 14-year-old victim was drawn out of the water in winter after 15 minutes of being in the
water. The victim shows no vital signs. What measures are to be taken?
A To release respiratory tract from water, to create drain position and to take on measures to
restore respiration and blood circulation
B Not to waste time on the release of respiratory tract from water, to take on cardiopulmonary
reanimation
C To transport the victim to the nearest hospital to carry out reanimation measures
D Тo transport the victim to the nearest warm room to carry out reanimation measures
E Not to carry out reanimation measures

93-An electro-gas welding operator working at a machine workshop performs welding and cutting
of metal, which is accompanied by intense UV-radiation. His welding station is equipped with
effective mechanical ventilation. What occupational disease is most likely to develop in an
electro-gas welding operator?
A Photoelectric ophthalmia
B Heatstroke
C Vegetative-vascular dystonia
D Chronic overheating
E Pneumoconiosis
Dr.HUR A. Salman - OdNMU
94- A woman complains of high temperature to 38oC,
mild pain in the throat during 3 days. On
examination: angle lymphatic nodes of the jaw are 3 cm enlarged, palatinel tonsils are enlarged
and coated with grey plaque which spreads to the uvula and frontal palatinel arches. What is the
most probable diagnosis?
A Larynx dyphtheria
B Infectious mononucleosis
C Vincent's angina
D Agranulocytosis
E Oropharyngeal candidosis

95- A 48-year-old male patient complains of constant pain in the upper abdomen, mostly on the
left, that is getting worse after taking meals; diarrhea, weight loss. The patient is an alcohol abuser.
2 years ago he had acute pancreatitis. Blood amylase is 4 g/h*l. Coprogram shows steatorrhea,
creatorrhea. Blood glucose is 6,0 mmol/l. What treatment is indicated for this patient?
A Panzinorm forte → Pancreatin = treatment for chronic pancreatitis , the pt. had acute since 2
years with alcohol intake , no he got chronic pancreatitis.
B Insulin
C Gastrozepin → Pirenzepine Treatment of peptic ulcer / Gastric acid secretion inhibitor
D Contrycal → tx. of ACUTE PANREATITIS .
E No-spa → Drotaverine an antispasmodic drug, used to enhance cervical dilation during
childbirth. It is structurally related to papaverine, is a selective inhibitor of phosphodiesterase 4,
and has no anticholinergic effects
Dr.HUR A. Salman - OdNMU
96- A 60-year-old female patient had been admitted to a hospital for acute transmural infarction.
An hour later the patient's contition got worse. She developed progressing dyspnea, dry cough.
Respiratory rate - 30/min, heart rate - 130/min, AP- 90/60 mm Hg. Heart sounds were muffled,
there was also diastolic shock on the pulmonary artery. The patient presented with medium
moist rales in the lower parts of lungs on the right and on the left. Body temperature -
$36,4^oC$. What drug should be given in the first place?
A Promedol → Trimeperidine (Promedol) is an opioid analgesic that is an analogue of prodine. It
was developed in the early 1950s in the USSR during research into the related drug pethidine.
It is opioids, such as analgesia and sedation, along with side effects such
as nausea, itching, vomiting and respiratory depression
B Aminophylline
C Dopamine
D Heparin
E Digoxin

97- A 62-year-old male has been hospitalized in the intensive care unit with a continuous attack of
retrosternal pain that cannot be relieved by nitroglycerin. Objectively: AP- 80/60 mm Hg, heart
rate - 106/min, breathing rate - 22/min. Heart sounds are muffled, a gallop rhythm is present.
How would you explain the AP drop?
A Reduction in cardiac output
B Reduction in peripheral resistance
C Blood depositing in the abdominal cavity
D Adrenergic receptor block
E Internal haemorrhage

98-A 35-year-old female reports heart pain (aching and drilling) occurring mainly in the morning
in autumn and spring and irradiating to the neck, back and abdomen; rapid heartbeat; low vitality.
Occurrence of this condition is not associated with physical activity. In the evening, the patient's
condition improves. Study of somatic and neurological status, and ECG reveal no pathology.
What pathology is most likely to have caused these clinical presentations?
Dr.HUR A. Salman - OdNMU
A Somatization depression → 
B Resting stenocardia
C Pseudoneurotic schizophrenia
D Neurocirculatory asthenia
E Hypochondriacal depression

99-A 18 y.o. male patient complains of pain in knee and ankle joints, temperature elevation to
39,5oC. He had a respiratory disease 1,5 week ago. On examination: temperature- 38,5oC, swollen
knee and ankle joints, pulse- 106 bpm, rhythmic, AP- 90/60 mm Hg, heart borders without
changes, sounds are weakened, soft systolic apical murmur. What indicator is connected with
possible etiology of the process?
A Antistreptolysine-0 → check down ..
B 1-antitrypsine →it is a genetic disorder that may result in lung disease or liver disease. Onset of
lung problems is typically between 20 and 50 years old. This may result in shortness of breath,
wheezing, or an increased risk of lung infections. Complications may include COPD , cirrhosis
, neonatal jaundice, or panniculitis.
C Creatinkinase → is assayed in blood tests as a marker of damage of CK-rich tissue such as
in myocardial infarction (heart attack), rhabdomyolysis (severe muscle breakdown), muscular
dystrophy, autoimmune myositides, and acute kidney injury
D Rheumatic factor
E Seromucoid → mucoprotien →comprises a group of glycoprotein = indicator of inflammatory
process , degeneration or trauma →  ,BUT in hepatic and endocrine insufficiency will 
Dr.HUR A. Salman - OdNMU

100-A 30 y.o. male patient complains of itching of the skin which intensifies in the evening. He has
been ill for 1,5 months. On examination: there is rash with paired papules covered with bloody
crusts on the abdomen, hips, buttocks, folds between the fingers, flexor surfaces of the hand.
There are traces of line scratches. What additional investigations are necessary to make
diagnosis?
A Examination of rash elements scrape → Scabie
B Determination of dermographism
C Serologic blood examination
D Blood glucose
E Examination for helmints

101
A 50-year-old locksmith has a long-term record of work under the effect of mercury vapors with
concentration exceeding MPC by 5-10 times. Clinical examination revealed the lability of
vasomotors of skin, pulse and arterial pressure; total hyperhydrosis; asymmetric innervation of
facial and lingual muscles, positive subcortical reflexes, intention tremor. Against the background
of increased emotional excitability the patient presents with lack of self-confidence, shyness. A
dentist found him to have parodontosis, chronic stomatitis. What disease can be suspected?
Dr.HUR A. Salman - OdNMU
A Chronic mercury intoxication
B Residual effects of neuroinfection
C Parkinson's syndrome
D Acute mercury intoxication
E Vascular encephalopathy

102-4 hours after having meals a patient with signs of malnutrition and steatorrhea experiences
stomach pain, especially above navel and to the left of it. Diarrheas take turns with constipation
lasting up to 3-5 days. Palpation reveals moderate painfulness in the choledochopancreatic
region. The amylase rate in blood is stable. X-ray reveals some calcifications located above
navel. What is the most likely diagnosis?
A Chronic pancreatitis
B Chronic gastroduodenitis
C Duodenal ulcer
D Zollinger-Ellison syndrome
E Chronic calculous cholecystitis
Dr.HUR A. Salman - OdNMU
103-A 58 y.o. male patient is examined by a physician and suffers from general weakness, fatigue,
mild pain in the left subcostal area, sometimes frequent painful urination. Moderate
splenomegaly has been revealed. Blood test: neutrophilic leukocytosis with the progress to
myelocyte; basophil- 2%; eosinophil- 5%. There is a urate crystales in urine, erythrocyte- 2-3
in the field of vision. What is the preliminary diagnosis?
A Chronic myeloleucosis
B Leukemoid reaction
C Lymphogranulomatosis
D Hepar cirrhosis
E Urolithiasis
CML , is a cancer of the white blood cells. It is a form of leukemia characterized by the increased
and unregulated growth of predominantly myeloid cells in the bone marrow and the accumulation
of these cells in the blood. CML is a clonal bone marrow stem cell disorder in which a proliferation
of mature granulocytes(neutrophils, eosinophils and basophils) and their precursors is found. It is
a type of myeloproliferative neoplasm associated with a characteristic chromosomal
translocation called the Philadelphia chromosome.

104- A 56-year-old patient with diffuse toxic goiter has ciliary arrhythmia with pulse rate of 110
bpm, arterial hypertension, AP- 165/90 mm Hg. What preparation should be administered along
with mercazolil?
A Propranolol → non-selective β blocker used to treat high blood pressure, a number of types
of irregular heart rate, thyrotoxicosis, capillary hemangiomas, performance anxiety, and essential
tremors , also B blocker can  production of T3 and T4 same time
B Radioactive iodine
C Procaine hydrochloride
D Verapamil
E Corinfar
Dr.HUR A. Salman - OdNMU
105-A 54-year-old patient complains of weakness, weight loss despite the unchanged appetite,
frequent urination, skin itch for six months. Some time ago the patient underwent treatment for
furunculosis. She hasn't been examined recently. Objectively: malnutrition, dry skin with signs of
scratching. Small lymph nodes can be palpated in the axillary regions. Changes in the internal
organs are absenr. What testing must be administered in the first place?
A Blood sugar test on an empty stomach → Fast blood sugar . signs of DM
B Complete blood count
C Endoscopy of stomach
D Lymph node biopsy
E Blood sterility testing

106-A 43 y.o. woman complains of severe pain in the right abdominal side irradiating in the right
supraclavicular area, fever, dryness and bitterness in the mouth. There were multiple vomitings
without relief. Patient relates the onset of pain to the taking of fat and fried food. Physical
examination: the patient lies on the right side, pale, dry tongue, tachycardia. Right side of
abdomen is painful during palpation and somewhat tense in right hypochondrium. What is the
most likely diagnosis?
A Perforative ulcer
B Acute cholecystitis
C Acute bowel obstruction
D Acute appendicitis
E Right-sided renal colic

107-Several hours before, a 28-year-old patient suddenly developed acute headache and repeated
vomiting, then lost consciousness. Objectively: focal neurological symptoms were not found.
Pronounced meningeal symptoms were revealed. AP - 120/80 mm Hg. According to clinical and
liquorological findings the patient was diagnosed with subarachnoid haemorrhage. After
administration of dehydrants the patient's condition somewhat improved. What is the main
component of further emergency care?
A Coagulants
B Anticoagulants
C Antiaggregants
D Fibrinolytics
E Corticosteroids
Dr.HUR A. Salman - OdNMU

108
On the 5th day of a respiratory disease accompanied by fever a 24-year-old man developed
progressing headaches, systemic dizziness, double vision, facial muscles paresis on the right,
choking from swallowing. He was diagnosed with acute viral encephalitis. Identify the main
tendency of the emergency treatment:
A Zovirax → antiviral
B Glucocorticoids
C Ceftriaxone
D Lasix
E Haemodezum

109- A 24-year-old man on the 5th day of acute respiratory disease with high grade temperature
started having strong headaches, systemic dizziness, sensation of double vision, paresis of
mimic muscles to the right, tickling by swallowing. Diagnosis: Acute viral encephalitis. Determine
the basic direction of the emergent therapy.
A Zovirax
B Glucocorticoids
C Cephtriaxon
D Lasix
E Hemodesis
** same above

110- A 30-year-old patient was delivered to the admission ward of the infectious disease
department. The disease had started acutely on the background of normal temperature with the
Dr.HUR A. Salman - OdNMU
appearance of frequent, liquid, profuse stool without pathological impurities. Diarrhea was not
accompanied by abdominal pain. 12 hours later there appeared recurrent profuse vomiting. The
patient rapidly developed dehydration. What is the most likely diagnosis?
A Cholera
B Shigellosis
C Staphylococcal food toxicoinfection
D Salmonellosis
E Campylobacteriosis

111- A 65 y.o. woman complains of complicated mouth opening following foot trauma 10 days ago.
Next day she ate with difficulties, there were muscles tension of back, the back of the head and
abdomen. On the third day there was tension of all muscle groups, generalized convulsions
every 10-15 min. What is the most probable diagnosis?
A Tetanus
B Tetania
C Meningoencephalitis
D Hemorrhagic stroke
E Epilepsy
Dr.HUR A. Salman - OdNMU
112- Gastric juice analysis of a 42-year-old male patient revealed absence of free hydrochloric acid
at all stages. Endoscopy revealed pallor, thinning of gastric mucosa, smoothed folds.
Microscopically the atrophy of glands with intestinal metaplasia was found. What disease is this
situation typical for?
A Chronic type A gastritis
B Chronic type B gastritis
C Chronic type C gastritis
D Menetrier disease
E Stomach cancer

113-A 45-year-old female patient complains of frequent liquid stools with a lot of mucus, pus and
blood; pain across the abdomen, loss of 7 kg within 6 months. She has a 1-year history of
non-specific ulcerative colitis. What group of drugs should be preferred for this patient?
A Corticosteroids
B Antibacterial
C Sulfonamides
D Nitrofurans
E Polyenzymes

114-A 54-year-old male patient complains of aching pain in the lumbar region, that is getting worse
after standing in an upright position, physical exercise, supercooling. The patient also reports of
experiencing weakness in the afternoon. Pain in the lumbar region, said about 10 years old.
Objectively: pale skin, to- 37,2oC, AP- 180/100 mm Hg, minor costovertebral angle tenderness
(Pasternatsky symptom). In blood: RBCs - 3,5x1012/l, WBCs - 6,5x109/l, ESR - 22 mm/h. In urine:
the relative density - 1010, leukocytes - 12-15 in the field of vision, erythrocytes - 2-3 in the field of
Dr.HUR A. Salman - OdNMU
vision. Urine bacterial count - 100000 in 1 ml. What is the most likely diagnosis?
A Chronic pyelonephritis
B Nephrolithiasis → renal stones
C Polycystic renal disease → multiple renal cysts (congenital)
D Chronic glomerulonephritis → no previous inf. , no proteinuria and other symptoms.
E Amyloidosis → no protein pathology

115- A 67-year-old male complains of dyspnea on exertion, attacks of retrosternal pain, dizziness.
He has no history of rheumatism. Objectively: pale skin, acrocyanosis. There are rales in the
lower parts of lungs. There is systolic thrill in the II intercostal space on the right, coarse systolic
murmur conducted to the vessels of neck. AP- 130/90 mm Hg, heart rate - 90/min, regular
rhythm. The liver extends 5 cm under the edge of costal arch, shin edemata are present.
Specify the assumed valvular defect:
A Aortic stenosis
B Pulmonary artery stenosis
C Mitral insufficiency
D Ventricular septal defect
E Tricuspid regurgitation
Dr.HUR A. Salman - OdNMU

116- A 24-year-old female teacher complains of dizziness and heart pain irradiating to the left
nipple. Pain is not associated with physical activity and cannot be relieved by nitroglycerin, it
abates after taking Valocordin and lasts an hour or more. The patient has a nearly 2-year history of
this disease. Objectively: Ps- 76 bpm. AP- 110/70 mm Hg. Heart borders are normal, heart sounds
are clear. The ECG shows respiratory arrhythmia. Radiograph of the cervicothoracic spine
shows no pathology. Lungs, abdomen are unremarkable. What changes in blood formula can be
expected?
A No changes → no organic diseases could seen , so the blood analyze will be without changes
B Leukocytosis
C Thrombocytopenia
D Leukemic hiatus
E Increased ESR

117-A 51-year-old female patient complains of frequent defecation and liquid blood-streaked stools
with mucus admixtures, diffuse pain in the inferolateral abdomen, 6 kg weight loss over the
previous month. Objectively: body temperature - 37,4oC, malnutrition, skin is pale and dry.
Abdomen is soft, sigmoid is painful and spasmodic, makes a rumbling sound. Liver is dense,
painful, extends 3 cm below the costal margin. What is the most likely diagnosis?
A Non-specific ulcerative colitis
B Bacillary dysentery
C Sprue
D Intestinal enzymopathy
E Helminthic invasion

118- A 18 y.o. female student complains of dyspnea during the intensive exertion. The condition
became worse half a year ago. On examination: pulse rate is 88 bpm, accelerated, AP- 180/20
mm Hg, pale skin, heart borders are dilated to the left and up. There is systolic-diastolic murmur
in the 2hd intercostal space, S2 at pulmonary artery is accentuated. ECG has revealed both
ventricles hypertrophy. Thoracic X-ray has revealed pulsation and protrusion of the left
Dr.HUR A. Salman - OdNMU
ventricle, lung trunk. What doctor's tactics should be?
A Cardiosurgeon consultation
B Dispensary observation
C Administration of therapeutic treatment
D Continuation of investigation
E Exemption from physical exercises

119- A 49-year-old male patient complains of dyspnea of combined nature, cough, shin edemata,
abdomen enlargement due to ascites. He has a 20-year history of chronic bronchitis. For the
last 3 years he has been disabled (group II) because of cardiac changes. Objectively: mixed
cyanosis, edemata. Ps - 92/min, rhythmic, AP - 120/70 mm Hg, respiration rate - 24/min. There
is accentuation of the second sound above the pulmonary artery. Auscultation reveals the box
resonance above the lungs. There are also dry rales over the entire surface of lungs. What is
the mechanism of heart changes development in this patient?

A Euler-Liljestrand reflex → Euler-Liljestrand mechanism (also called alveolar-vascular reflex


according to Paul H. Rossier ), also known clinically as hypoxic pulmonary vasoconstriction
(HPV) , describes the relationship between ventilation ( ventilation ) and blood flow ( Perfusion ) of
the lung , described as the ventilation / perfusion ratio (or V / Q quotient ). The term reflex is
misleading because reflexes are strictly neuronal mediated. Better is the use of the term hypoxic
pulmonary vasoconstriction (HPV)
B Kitaev's reflex
C Bainbridge reflex
D Cardiovascular reflex
E Respiratory reflex

120
A 24-year-old man on the second day of the disease with a sudden onset complains of a strong
headache in temples and in the area of orbits, dull pain in the body, dry painful cough. His
temperature is 39oC. Adynamic. Mucous membrane of oropharynx is "flaming", rales are
not ausculated. What is the most probable diagnosis?
A Influenza
B Parainluenza
C Respiratory mycoplasmosis
D Pneumonia
E Meningococcus infection
Dr.HUR A. Salman - OdNMU
121-A 32-year-old female complains of dizziness, headache, palpitation, tremor. For the last several
months she has been under outpatient observation for the increased arterial pressure. Since
recently such attacks have become more frequent and severe. Objectively: skin is covered with
clammy sweat, tremor of the extremities is present. HR- 110/min, AP- 220/140 mm Hg. Heart
sounds are muffled. Blood test results: WBCs - 9,8x109/l, ESR - 22 mm/h. Blood
glucose - 9,8 millimole/l. What disease is the most likely cause of this crisis?

A Pheochromocytoma → is a neuroendocrine tumor of the medulla of the adrenal glands


(originating in the chromaffin cells), or extra-adrenal chromaffin tissue that failed to involute after
birth, that secretes high amounts of catecholamines, mostly norepinephrine, plus epinephrine to a
lesser extent
B Essential hypertension
C Preeclampsia
D Primary hyperaldosteronism
E Diabetic glomerulosclerosis

122- A 76-year-old male consulted a therapist about slow discharge of urine with a small jet. The
patient reported no cardiac problems. Examination revealed atrial fibrillation with a heart rate of
72/min and without pulse deficit. There are no signs of heart failure. ECG confirms the presence
of atrial fibrillation. From history we know that the arrhythmia was detected three years ago.
What tactics for the treatment of atrial fibrillation in the patient should be chosen?
A Does not require treatment → as he had Af since 3 y ago , and no symptoms of HF , not need any
drug or treatment and will stay under follow up .
B Digoxin
C Verapamil
D Obzidan
E Ajmaline

123-A 43-year-old female patient complains of unstable defecation with frequent constipations,
abdominal swelling, headache, sleep disturbance. Body weight is unchanged. What disease are
these clinical presentations typical for?
A Irritable colon syndrome → review Q 7
B Chronic enteritis
C Chronic pancreatitis
D Chronic atrophic gastritis
E Colorectal cancer
Dr.HUR A. Salman - OdNMU
124-A 43-year-old man who often contacts with ethyl gasoline was admitted to a hospital with
complaints of general weakness, dizziness, memory impairment, sleepiness at daytime and
insomnia at night, sensation of a hair in the mouth, colicky pains in the right subcostal region.
What is the most likely diagnosis?
A Chronic tetraethyl lead intoxication
B Alcoholic delirium
C Chronic mercury intoxication
D Chronic manganese intoxication
E Chronic lead intoxication

125- A 35-year-old patient has been in the intensive care unit for acute renal failure due to crush
for 4 days. Objectively: the patient is inadequate. Breathing rate - 32/min. Over the last 3 hours
individual moist rales can be auscultated in lungs. ECG shows high T waves, right ventricular
extrasystoles. CVP - 159 mm Hg. In blood: the residual nitrogen - 62 millimole/l, K+- 7,1
millimole/l, Cl- - 78 millimole/l, Na+- 120 millimole/l, Ht - 0,32, Hb - 100 g/l, blood creatinine
- 0,9 millimole/l. The most appropriate method of treatment would be:
A Hemodialysis
B Plasma sorption
C Hemosorption
D Plasma filtration
E Ultrafiltration

126- A 45-year-old man was brought to clinic with complaints of the pain that started suddenly in
the left chest part and epigastric area, shortness of breath, nausea, one-time vomiting. The acute
pain started after weight-lifting. On physical exam: shallow breathing, RR - 38/min, left chest
part is behind during respiration, by percussion - tympanitic sound, respiration is not ausculated.
Ps - 110 bpm, of weak filling. BP- 100/60 mm Hg, insignificant displacement of heart to the right,
sounds are dull. What examination is the most expedient to do first?
A Roentgenography → X ray , PNEUMOTHORAX ?
B Electrocardiography
C Bronchoscopy
D Esophagogastroscopy
E Ultrasound of the abdominal cavity
Dr.HUR A. Salman - OdNMU

127- A 35 y.o. woman is suspected of aplastic anemia. The bone marrow punction has been
administered with the diagnostic purpose. What changes in the marrow punctatum are suggested?
A Replacement of marrow elements with adipose tissue
B Replacement of marrow elements with fibrous tissue
C Prevalence of megaloblasts
D Presence of blast cells
E Absolute lymphocytosis

128-A 58-year-old female patient complains of spontaneous bruises, weakness, bleeding gums,
dizziness. Objectively: the mucous membranes and skin are pale with numerous hemorrhages
of various time of origin. Lymph nodes are not enlarged. Ps is 100/min, AP - 110/70 mm Hg.
There are no changes of internal organs. Blood test results: RBC - 3,0x1012/l, Нb - 92 g/l, colour
index - 0,9, anisocytosis, poikilocytosis, WBC – 10x109/l, eosinophils - 2%, stab neutrophils - 12%,
segmented neutrophils - 68%, lymphocytes - 11%, monocytes - 7%, ESR - 12 mm/h. What
laboratory test is to be determined next for making a diagnosis?
A Platelets→ need to exclude thrombocytopenia
B Reticulocytes
C Clotting time
D Osmotic resistance of erythrocytes
E Fibrinogen

129-A 47-year-old woman underwent a thyroid gland resection on ccount of nodular euthyroid
goiter. What preparations are most likely to prevent the disease recurrence?
A Thyroid hormones
B Mercazolil
C Thyrotropin
D Antistruminum (potassium iodide)
E Radioactive iodine
Dr.HUR A. Salman - OdNMU
130-A 55 y.o. male patient complains of weakness during 2 months, pain in the right side of the
thorax, cough, blood-streaked sputum. On X-ray: intensive triangle shadow in the area of lower
lobe that is connected to mediastinum. What is the most likely disorder in the lungs?

A Central cancer of lungs → mostly in SMOKER , follow infilitration on xray and choose the
distention if going towards the hilum or lung root or mediastinum , so this is CENTRAL
and if the infiltration triangle opposite from hilum or mediastinum towards the periphery and
costophrenic angle so this will be Peripheral .
B Tuberculosis of lungs
C Bronchiectasia
D Pulmonary infarction
CENTRAL vs PERIPHRAL lung CA
E Pleuropneumonia

In KROK , by xray will tell


you the shadow infiltration In KROK , by xray will
from one lobe (periphery) tell you the shadow
or costophrenic angle to triangle of one lobe
the hilum or mediastinum connected to the
(CENTRAL) mediastinum or hilum

** choose the distention , where the infiltration going (towards → )

131-A 60 y.o. patient experiences acute air insufficiency following of the venoectomy due to
subcutaneous vein thrombophlebitis 3 days ago. Skin became cianotic, with grey shade. Marked
psychomotor excitement, tachypnea, substernal pain. What postoperative complication has
occured?
A Thromboembolia of pulmonary artery
B Hemorrhagia
C Hypostatic pneumonia
D Myocardial infarction
E Valvular pneumothorax
Dr.HUR A. Salman - OdNMU

132-A 19-year-old woman complains of pain in the abdomen and joints, asks for more analgetics
and somnifacient injections. The patient was examined. Gynecological and urological pathologies
are absent. There are signs of previous punctures along superficial veins of the extremities. The
patient does not explain the origin of punctures. Tendon reflexes of upper and lower extremities are
the same, quick. Photoreaction of the pupil of the eye is weak. The tongue is grey coated. During
communication the patient in affectively not even-tempered. There is diarrhea without pathologic
inclusions. What tactics is necessary to improve the condition of this patient?
A Consultation of an expert in narcology
B Prescription of medications the patient asks for
C Additional consultation of surgeon
D Treatment with antibiotics
E Consultation of infectious diseases doctor

133-A patient has an over a year-old history of fast progressive rheumatoid arthritis. X-raying
confirms presence of marginal erosions. What basic drug would be the most appropriate in this
case?
Dr.HUR A. Salman - OdNMU
A Methotrexate → known as amethopterin, is a chemotherapy agent and immune system
suppressant . It is used to treat cancer, autoimmune diseases, ectopic pregnancy, and for
medical abortions. Types of cancers it is used for include breast cancer, leukemia, lung ca,
lymphoma, and osteosarcoma. Types of autoimmune diseases it is used for
include psoriasis, rheumatoid arthritis, and Crohn's disease.
B Chloroquine → antimalaria , antirheumatoid.
C Prednisolone → steroid
D Diclofenac sodium → NSAIDs Voltarin
E Aspirin → NSAID acetyl-salicylic acid

134-A female rheumatic patient experiences diastolic thoracic wall tremor (diastolic thrill),
accentuated S1 at apex, there is diastolic murmur with presystolic intensification, opening snap, S2
accent at pulmonary artery. What rind of heart disorder is observed?
A Mitral stenosis → check Q 144
B Aortic valve insufficiency
C Pulmonary artery stenosis
D Mitral valve insufficiency
E Opened arterial duct

135-A 23-year-old patient complains of a dull ache, sensation of heaviness and distention in the
epigastrium immediately after meals, foul-smelling eructation; dry mouth, empty stomach
nausea, diarrhea. Objectively: the skin is pale, the patient is of thin build. Abdomen is soft on
palpation, there is epigastric pain. The liver does not extend beyond the costal arch. In blood:
Hb - 110 g/l, RBCs - 3,4x1012/l, WBC count is normal. ESR - 16 mm/h. What is the most
informative study that will allow make a diagnosis?
A Esophageal gastroduodenoscopy → Review Q 73
B X-ray of digestion organs
C Study of gastric juice
D pH-metry
E Duodenal probing

136-A 49-year-old patient complains of deglutition problems, especially with solid food, hiccups,
voice hoarseness, nausea, regurgitation, significant weight loss (15 kg within 2,5 months).
Objectively: body weight is reduced. Skin is pale and dry. In lungs: vesicular breathing, heart
sounds are loud enough, heart activity is rhythmic. The abdomen is soft, painless on palpation.
Liver is not enlarged. What study is required to make a diagnosis?
A Esophageal duodenoscopy along with biopsy → Review Q 73
B Clinical blood test
C X-ray of digestive tract organs
D X-ray in Trendelenburg's position
E Study of gastric secretion

137- A 60-year-old patient has been admitted to a hospital with complaints of dyspnea, tightness in
the right subcostal area, abdomen enlargement. These presentations have been progressing
for a year. Heart auscultation reveals presystolic gallop rhythm. Objectively: swelling of the neck
veins, ascites, palpable liver and spleen. What disease requires differential diagnostics?
A Constrictive pericarditis
B Hepatocirrhosis
C Lung cancer with invasion to the pleura
D Chronic pulmonary heart
Dr.HUR A. Salman - OdNMU
E Pulmonary embolism

138-A 40-year-old patient, the forester, complains of severe headache, body temperature rise up to
39,5oC, trembling limbs. From the patient's history we know that he had seriously cut his
hand during the dissection of a killed fox. Objectively: depressed mood. The patient asks not to
turn on the light or open the door. Any noise causes apparent motor excitation. When he saw a
carafe of water, he developed convulsive throat spasms. What tactics should an emergency
doctor choose?
A Deliver the patient to the infectious disease hospital → rabie’s
B Deliver the patient to the resuscitation department
Dr.HUR A. Salman - OdNMU
C Deliver the patient to the neurological department
D Deliver the patient to the psychiatric hospital
E Let him stay at home and consult a psychiatrist

I HATE
WATER

139- A 28-year-old woman has a 12-year history of chronic glomerulonephritis with latent course.
Over the past six months she has developed general weakness, loss of appetite, low work
performance, nausea. The patient complains of headache, pain in the joints. On examination:
anemia, blood urea - 34,5 millimole/l, blood creatinine - 0,766 millimole/l, hyperkalemia. What
complication has developed?
A Chronic renal insufficiency
B Acute renal insufficiency
C Nephrotic syndrome
D Renal amyloidosis
E Pyelonephritis
Dr.HUR A. Salman - OdNMU
140-A 72-year-old male had had a moderate headache. Two days later, he developed the
progressing speech disorders and weakness in the right extremities. The patient has a history
of myocardial infarction, arrhythmia. Study of the neurologic status revealed elements of motor
aphasia, central paresis of the VII I XII cranial nerves on the right, central hemiparesis on the
same side and hyperaesthesia. What is the most likely diagnosis?
A Ischemic stroke
B Hemorrhagic stroke
C Transient ischemic attack
D Epidural hematoma
E Brain tumor

141 SAME Q UP 
A 70 y.o. male patient with mild headaches complains of speech disorder, weakness in right
limbs. There was a history of miocardial infarction and arrhythmia. On nu eroligical examination
there are elements of motor aphasia, central paresis of VII and XII cranial nerves pairs on the
right side, cental type of hemiparesis and hemihyperesthisia on the same side. What is the most
probable diagnosis?
A Ischemic stroke
B Hemorrhagic stroke
C Transitory ischemic attack
D Epidural hematoma
E Cerebral tumor

142-After treating a field with pesticides a machine operator presents with great weakness,
headache, nausea, vomiting, diarrhea, visual impairment, watery eyes. Objectively: the patient is
excited, hypersalivation, hyperhidrosis, muscle fibrillation of tongue and eyelids are oberved.
Pupils are narrowed, there is tachycardia, lung auscultation reveals moist small and medium
bubbling rales. In blood: changed level of cholinesterase activity. What is the most likely diagnosis?
A Intoxication with organophosphorous pesticides → check down 
B Intoxication with organochlorine pesticides
C Intoxication with organomercurial pesticides
D Intoxication with arsenic-containing pesticides → check down 
E Intoxication with carbamic acid derivatives
Dr.HUR A. Salman - OdNMU

143-A 40-year-old man is ill with autoimmune hepatitis. Blood test: А/G ratio 0,8, bilirubin – 42
mu*mol/L, transaminase: ALT- 2,3 mmol g/L, АSТ - 1,8 mmol g/L. What is the most effective
means in treatment from the given below?
A Glucocorticoids, cytostatics → Glucocorticoids = immunosuppressant , cytostatics= is the
inhibition of cell growth and multiplication. to prevent liver cell damage.
B Antibacterial medication
C Hepatoprotectors
D Antiviral medications
E Hemosorbtion, vitamin therapy

144- A farmer hurt his right foot during working in a field and came to the emergency station. He
doesn't remember when he got last vaccination and he has never served in the army.
Examination of his right foot revealed a contaminated wound up to 5-6 cm long with uneven
edges. The further treatment tactics will be:
A To make an injection of tetanus anatoxin and antitetanus serum
B To make an injection of tetanus anatoxin
C To make an injection of antitetanus serum
D Surgical d-bridement only
E To administer an antibiotic
** Tetanus Toxoid is administered to the animal to help build an immunity to Tetanus. After the
injection, the Toxoid will start to take effect approximately 10 to 25 days, which is the time it takes
Dr.HUR A. Salman - OdNMU
tetanus to show up in the animal if present. The injection will stay in the system up to 2 to 3
months.
Tetanus Antitoxin is administered to the animal if tetanus is present in the animal already or you
suspect the animal of getting it. This antitoxin takes effect almost immediately after the injection
and only stays in the system for up to 10 days.

145- A 35-year-old patient has been admitted to a hospital for pain in the left sternoclavicular and
knee joints, lumbar area. The disease has an acute character and is accompanied by fever up to
38oC. Objectively: the left sternoclavicular and knee joints are swollen and painful. In blood: WBCs
- 9,5x109/l, ESR - 40 mm/h, CRP - 1,5 millimole/l, fibrinogen - 4,8 g/l, uric acid - 0,28 millimole/l.
Examination of the urethra scrapings reveals chlamydia. What is the most likely diagnosis?
A Reiter's syndrome → Reactive Arthritis
B Rheumatic arthritis
C Gout
Or Tell you ,
D Bechterew's disease
there is a
E Rheumatoid arthritis CHLAMYDIA
infection
Dr.HUR A. Salman - OdNMU
146- A 20 daily y.o. female patient is suffering from chronic bronchitis. Recently there has been
production about 0,5 L of purulent sputum with maximum discharge in the morning. Fingers are
like "drum sticks", there are "watching glass" nails. What is the most probable diagnosis?
A Bronchiectasia → Abnormal permanent dilation of the bronchial tubes , Treatment surgery
B Pneumonia
C Chronic bronchitis
D Gangrene of lungs
E Tuberculosis

147-Topographic percussion of lungs in a patient who got a serious job-related barotrauma


revealed that the lower lungs borders were located one rib below normal, there was a significant
increasein both lungs height and Kronig's isthmus. What disease should be suspected in the first
place?
A Pulmonary emphysema Barotrauma: is physical damage to body
B Exudative pleuritis tissues caused by a difference in pressure
C Chronic bronchitis between a gas space inside, or in contact
D Bronchial asthma with the body
E Pneumothorax
Dr.HUR A. Salman - OdNMU
148- An 18 y.o. girl complains of weakness, dizziness, loss of appetite, menorrhagia. There are
many-coloured petechiae on the skin of the upper extremities. Blood test: Hb- 105 g/l; RBC-
3,2x1012/L; C.I.- 0,95; thromb.- 20x109/L. The sedimentation time according to Lee
White is 5'; hemorrhagia duration according to Duke is 8', "pinch and tourniquet" test is
positive. What is the most probable diagnosis?
A Idiopathic thrombocytopenic purpura → platelets 
B Hemophilia →  APTT , platelets normal
C Hemorrhagic diathesis
D Iron deficiency anemia
E Marchiafava-Micheli's disease →haemoglobinuria paroxysmalis nocturia

BLEEDING TIME
Lee White = 3 -5 min
<3 = hypocoagulate ,
>5 Hypercoagulate
DUKE = 2-6 min

149
A 28 y.o. male patient was admitted to the hospital because of high temperature 39oC,
headache, generalized fatigue, constipation, sleep disorder for 9 days. There are sporadic
roseolas on the abdomen, pulse- 78 bpm, liver is enlarged for 2 cm. What is the most probable
diagnosis?
A Abdominal typhoid → Salmonella typhii
B Typhus → Rickettsia typhii
C Sepsis
D Brucellosis
E Leptospirosis
Dr.HUR A. Salman - OdNMU
150-A 50-year-old patient complains about having pain attacks in the right subcostal area for
about a year. He pain arises mainly after taking fattening food. Over the last week the attacks
occurred daily and became more painful. On the 3rd day of hospitalization the patient presented
with icteritiousness of skin and scleras, light-colored feces and dark urine. In blood: neutrophilic
leukocytosis - 13,1x109/l, ESR- 28 mm/h. What is the most likely diagnosis?
A Chronic calculous cholecystitis
B Chronic recurrent pancreatitis
C Fatty degeneration of liver
D Chronic cholangitis, exacerbation stage → he didn’t have acute cholangitis in hx.
** Exacerbation: A worsening. In medicine, exacerbation may refer to an increase in the severity of
a disease or its signs and symptoms.
E Hypertensive dyskinesia of gallbladder

151- A 20 y.o. patient with bronchial asthma experiences dyspnea attacks 3-4 times a week.
Nocturnal attacks are 1 time a week. FEV1- 50% of necessary figures, during the day it's variations
is 25%. What is the severity of bronchial asthma condition?
A Moderate severity condition → check down
B Mild condition
C Serious condition
D Asthmatic status
E Intermittent flow
Classification of COPD
Stage and severity Signs

І, mild - FEVІ < 80% , FEV1/FVC < 70% - As a rule chronic cough with sputum

II, moderate - 50%< FEVІ < 80% - FEV1/FVC < 70% - Symptoms are more significant,
presence of dyspnea during physical activity and exacerbation
III, severe 30%< FEVІ < 50% FEV1/FVC < 70% - Symptoms cause worsening of life
quality
IV, very severe - FEVІ < 30% FEV1/FVC < 70% and CRF
Dr.HUR A. Salman - OdNMU
152-A 40 y.o. man complains of headache in occipital area. On physical examination: the skin is
pale; face and hand edema, BP- 170/130 mm Hg. On EchoCG: concentric hypertrophy of the
left ventricle. Ultrasound examination of the kidneys reveals thinned cortical layer. Urine
analysis shows proteinuria of 3,5 g/day. What is the probable diagnosis?
A Essential arterial hypertension
B Chronic pyelonephritis
C Chronic glomerulonephritis
D Polycystic disease of the kidneys
E Cushing's disease

153-A 27-year-old patient has a severe headache, nausea and vomiting. Objectively: body
temperature is 38,9oC, there is a haemorrhagic stellate rash on the legs. The patient takes
meningeal pose in bed. Meningeal symptoms are strongly positive. Deep reflexes are brisk,
uniform. Pathological reflexes are absent. It has been suspected that the patient has epidemic
cerebrospinal meningitis. Which of additional tests should be performed in the first place to
verify the diagnosis?
A Lumbar puncture
B Echoencephalography
C Rheoencephalography
D Electroencephalography
E Survey craniogram
Dr.HUR A. Salman - OdNMU

154-After a serious nervous stress a 35-year-old patient has developed on the dorsal surface of
hands redness and swelling that were later replaced by small inflammatory nodules, vesicles and
following erosion with a significant serous discharge. The process is accompanied by severe
itching. What is the most likely diagnosis?
A True eczema → emotion – stress
B Allergic dermatitis → allergic reaction .
C Microbal eczema → infection – burn – injury – trauma.
D Simple contact dermatitis
E Toxicoderma
Dr.HUR A. Salman - OdNMU

155-A 36-year-old patient complains of skin rash that appeared a week ago and doesn't cause any
subjective problems. Objectively: palm and sole skin is covered with multiple lenticular
disseminated papules not raised above the skin level. The papules are reddish, dense on palpation
and covered with keratinous squamae. What is the provisional diagnosis?
A Secondary syphilis
B Verrucosis
C Palmoplanar psoriasis
D Palmoplanar rubrophytosis
E Palm and sole callosity
Dr.HUR A. Salman - OdNMU
156-In the morning a patient had nausea, abdominal discomfort, single vomiting, dry mouth. In
the evening, the patient presented with the increasing general weakness, double vision, difficult
swallowing of solid food. Objectively: ptosis, mydriasis, anisocoria, absence of gag and
pharyngeal reflex, dry mucous membranes. The previous evening the patient had dinner with
canned food and alcohol. What is the presumptive diagnosis?
A Botulism
B Food toxicoinfection
C Intoxication with unknown poison
D Acute ischemic stroke
E Poliomyelitis

157-A 30-year-old patient complains of paroxysmal abdominal pain, frequent liquid stools up to 10
times a day. Throughout the first 3 days he had a fever, since the 2nd day of disease there
were scant liquid stools mixed with mucus. On palpation: tenderness of all colon segments.
Sigmoid colon was found spastic. What is your provisional diagnosis?
A Acute dysentery
B Intestinal amebiasis
C Salmonellosis
D Cholera → normal tempreture , severe dehydration
E Balantidiasis → protozoan infection caused by infection with Balantidium coli , intestinal
mucosa , or systemic in nature and include either diarrhea or constipation.

** Dysentery is a type of gastroenteritis that results in diarrhea with blood. Other symptoms may
include fever, abdominal pain, and a feeling of incomplete defecation.
Dr.HUR A. Salman - OdNMU

158-A 38-year-old woman experiences episodic increases in arterial pressure up to 240/120 mm


Hg, which is accompanied by nausea, vomiting, tachycardia, increased sweating, hyperglycemia.
The attack is usually followed by the excessive urination. Renal sonography reveals an additional
formation adjacent to the upper pole of the right kidney and possibly belonging to the adrenal
gland. What laboratory test will allow to clarify the diagnosis?
A Determination of urinary excretion of catecholamines and vanillylmandelic acid →
Pheochromocytoma
B Blood test for insulin and C-peptide
C Estimation of glomerular filtration rate by measuring endogenous creatinine clearance
D Blood test for thyroxine and thyrotrophic hormone
E Blood test for renin level
Review Q 121

159-A 32-year-old patient has a 3-year history of asthma attacks, that can be hardly stopped with
berotec. Over a few last months he has experienced pain in the joints and sensitivity disorder of
legs and feet skin. Ps - 80/min, AP - 210/100 mm Hg. In blood: eosinophilia at the rate of 15%.
What disease can be suspected in this case?
A Periarteritis nodosa  → Polyarteritis nodosa (or periarteritis nodosa) is a
vasculitis of medium-sized arteries, which become swollen and damaged from attack by
rogue immune cells. Polyarteritis nodosa is also called Kussmaul disease.
B Systemic lupus erythematosus
C Systemic scleroderma
D Dermatomyositis
E Wegener's disease
Dr.HUR A. Salman - OdNMU
160-3 hours before, a 68-year-old male patient got a searing chest pain radiating to the neck and
left forearm, escalating dyspnea. Nitroglycerin failed to relieve pain but somewhat reduced
dyspnea. Objectively: there is crimson cyanosis of face. Respiratory rate is 28/min. The patient
has vesicular breathing with isolated sibilant rales. Heart sounds are muffled, with a gallop
rhythm. Ps - 100/min, AP - 100/65 mmHg. ECG shows negative T-wave in V2-V6 leads. What
drug can reduce the heart's need for oxygen without aggravating the disease?
A Isosorbide dinitrate → pt got myocardial infarction .
B Corinfar
C Atenolol
D Streptokinase
E Aminophylline

161
A 46-year-old patient complains of sudden palpitation, which is accompanied by pulsation in the
neck and head, fear, nausea. The palpitation lasts for 15-20 minutes and is over after straining
when holding her breath. What kind of cardiac disorder may be suspected?
A An attack of supraventricular paroxysmal tachycardia
B An attack of ventricular paroxysmal tachycardia
C An attack of atrial flutter
D An attack of ciliary arrhythmia
E An attack of extrasystolic arrhythmia
Dr.HUR A. Salman - OdNMU

162-A 5-grade pupil complains about extensive skin rash accompanied by intensive itch, especially
at night. Objectively: there are small red papules set mostly in pairs in the region of interdigital
folds on both hands, on the flexor surface of radicarpal articulations, abdomen and buttock skin
as well as internal surface of thighs. In the centre of some papules vesicles or serohaemorrhagic
crusts can be seen. There are multiple excoriations. What is the most likely diagnosis?
A Scabies
B Dermatitis
C Ringworm of body
D Toxicoderma
E Eczema
Dr.HUR A. Salman - OdNMU

163-A welder at work got the first-degree burns of the middle third of his right shin. 5 days later
the skin around the burn became edematic and itchy. Objectively: on a background of a well-
defined erythema there is polymorphic rash in form of papules, vesicles, pustules, erosions with
serous discharge. What is the most likely diagnosis?
A Microbal eczema → post infection or injury , burn
B True eczema
C Toxicoderma
D Occupational eczema
E Streptococcal impetigo

164
A 58-year-old patient has a 3-year history diabetes mellitus type II. He has been keeping to a
diet and regularly taking glyburide. He has been delivered to a hospital on an emergency basis
for acute abdomen. Objectively: the patient is of supernutrition type. The skin is dry. In the lungs
vesicular breathing can be auscultated. Heart sounds are regular, 90/min. AP- 130/70 mm Hg.
The symptom of "wooden belly" is visible. Blood sugar - 9,8 millimole/l. The patients has
indication for laparotomy. What is the most appropriate way of further treatment of diabetes?
A To administer short insulin
B To continue taking glyburide
C To administer Semilong to be taken in the morning and insulin - in the evening
D To administer 1 tablet of Glurenorm three times a day
E To administer 1 tablet of Maninil three times a day
Dr.HUR A. Salman - OdNMU
165-A 56 y.o. man, who has taken alcoholic drinks regularly for 20 years, complains of intensive
girdle pain in the abdomen. Profuse nonformed stool 2-3- times a day has appeared for the last
2 years, loss of weight for 8 kg for 2 years. On examination: abdomen is soft, painless. Blood
amylase - 12g/L. Feces examination-neutral fat 15 g per day, starch grains. What is the most
reasonable treatment at this stage?
A Pancreatine → pansinorm forte = creon = chronic pancreatitis .
B Contrykal
C Aminocapron acid
D Levomicytine
E Imodium

166-A 43-year-old female patiet complains of eruption on her right leg skin, pain, weakness, body
temperature rise up to 38oC. The disease is acute. Objectively: there is an edema on the
right leg skin in the region of foot, a well-defined bright red spot in form of flame tips which feels
hot. There are isolated vesicles in focus. What is your provisional diagnosis?
A Erysipelas → Erysipelas should not be confused with erysipeloid, a skin infection caused
by Erysipelothrix . it is characterized clinically by shiny, raised, indurated, and tender plaques with
distinct margins. High fever, chills, and malaise frequently accompany erysipelas. There is also a
bullous form of erysipelas.
B Microbial eczema
C Contact dermatitis
D Toxicoderma
E Haemorrhagic vasculitis

167- A 45-year-old patient complains of some painless nodular elements tending to peripheral
growth and fusion. He has a 2-year history of this disease. Aggravation takes place mainly in
spring. In anamnesis: the patient's father had similar skin lesions. Objectively: pathological
elements looke like guttate and nummular nodules, plaques covered with white scales. What
is your provisional
diagnosis?
A Psoriasis
B Lichen ruber planus
C Neurodermitis
D Pityriasis rosea
E Seborrheic eczema
Dr.HUR A. Salman - OdNMU

168
A 47-year-old patient came to see a doctor on the 7th day of disease. The disease developed
very fast: after the chill body temperature rose up to 40oC and lasted up to 7 hours, then it
dropped abruptly, which caused profuse sweat. There were three such attacks occuring once in
two days. Two days ago the patient arrived from Africa. Objectively: pale skin, subicteric sclera,
significantly enlarged liver and spleen. What is the cause of fever attacks in this disease?
A Erythrocytic schizogony → Malaria
B Tissue schizogony
C Exotoxin of a causative agent
D Endotoxin of a causative agent
E Gametocytes
Dr.HUR A. Salman - OdNMU

169-On the 2nd day of disease a 27-year-old patient complains of unbearable headache, repeated
vomiting. Objectively: the patient is in a grave condition. He is conscious but adynamic. Lies in a
forced position with his head thrown back. There is no skin rash. Nuchal muscles are evidently
rigid, there are Kernig's and Brudzinski's signs. to - 39,5oC, Ps -120/min, AP - 130/80 mm Hg. The
leading syndrome of this disease is caused by:
A Liquor hypertension → intracranial hypertension . causing meningeal irritation. Also the pt have
hx of hypertension
B Liquor hypotension → decrease pressure.
C Affection of the cranial nerve nuclei
D Haemorrhages in the adrenal glands
E Hyperthermy
Dr.HUR A. Salman - OdNMU
170 Same Q above
On the 2nd day of illness a 27-year-old patient complains of unbearable headache, repeated
vomiting. Objectively: the patient is in a grave condition. He is conscious but adynamic. Lies in a
forced position with his head thrown back. There is no skin rash. Nuchal muscles are evidently
rigid, there are Kernig's and Brudzinski's signs. to- 39,5oC, Ps- 120/min, AP- 130/80 mm Hg. The
leading syndrome of this disease is caused by:
A Liquor hypertension
B Liquor hypotension
C Affection of the cranial nerve nuclei
D Haemorrhages into the adrenal glands
E Hyperthermy

171-A 43 y.o. woman complains of shooting heart pain, dyspnea, irregularities in the heart activity,
progressive fatigue during 3 weeks. She had acute respiratory disease a month ago. On
examination: AP- 120/80 mm Hg, heart rate 98 bpm, heart boarders +1,5 cm left side, sounds
are muffled, soft systolic murmur at apex and Botkin's area; sporadic extrasystoles. Liver isn't
palpated, there are no edema. Blood test: WBC- 6,7x109/L, sedimentation rate- 21 mm/hour. What
is the most probable diagnosis?
A Acute myocarditis → previous respiratory infection then he got heart symptoms .. mostly viral .
B Climacteric myocardiodystrophia
C Ichemic heart disease, angina pectoris
D Rheumatism, mitral insufficiency
E Hypertrophic cardiomyopathy

172-A 63-year-old female complains of general weakness, a feeling of heaviness, compression in


the epigastrium, postprandial fullness, nausea, belching after meals. These symptoms have
been observed for about 15 years. Objectively: body temperature is 36,4oC, respiratory
rate - 20/min, Ps - 88/min, blood pressure - 115/75 mm Hg. Skin and mucous membranes are
pale. Blood test results: RBC - 2,0x1012/l, Hb - 100 g/l. Tests revealed parietal-cell antibodies. What
is the most likely reason for the development of anemia in this patient?
A Production of antibodies to intrinsic factor→ pernicious anemia due to Gastritis A –autoimmune.
B Disruption of hemoglobin synthesis
C Disruption of erythropoietin synthesis
D Impaired iron absorption
E Increased loss of iron

173-A 37-year-old woman is sick with bronchial asthma for 15 years. Recenlty asthmatic attacks
occur 4-5 times per week, night attacks -2-3 times per month. To stop attacks, the patient takes
salbutamol. On physical exam: condition is relatively satisfactory. RR - 20/min, Ps is 76 bpm, BP -
120/80 mm Hg. Respiration in lungs is vesicular. Cardiac sounds are muted, rhythm is normal.
What medication should be prescribed to prevent attacks of bronchial asthma on the first stage?
A Cromoglycat sodium → review Q 58
B Regular dose of salbutamol
C Inhalation corticosteroids
D Tabletted corticosteroids
E Injection of corticosteroids

174- a 52 y.o. male patient has become ill gradually. There is pain in the left side of the thorax
during 2 weeks, elevation of temperature till 38-39oC. On examination: left chest side falls behind
in breathing movement no voice tremor over the left lung. Dullness that is more intensive in lower
parts of this lung. Right heart border is deviated outside. Sharply weakened breathing over the
Dr.HUR A. Salman - OdNMU
left lung, no rales. Heart sounds are mufflet, tachycardia. What is the most probable diagnosis?
A Exudative pleuritic → Pleurisy is inflammation of the pleura.
B Spotaneous pneumothorax
C Atelectasis of lung
D Cirrhotic tuberculosis
E Infarction-pneumonia
Causes of transudtive and exudative effusions
Transudate Congestive Liver Nephroticsyndrome Myxoedema Constrictive Meigs’
heart failure cirrhosis syndrome
pericarditis

Exudates Pneumonia Tuberculosis Lung infarction Lung cancer Connective Acute


tissue pancreatitis
disease

Characteristics of transudate and exudate


Test Exudate Transudate
Fluid/serum protein ratio 0,5 <0,5
Fluid/serum LDH ratio >0,6 <0,6
Specific gravity 1,016 <1,016
Protein ≥3g/dl <3g/dl
Appearance Cloudy viscous Clear, thin non-clotting

175-A 50-year-old patient was hospitalized in severe condition with complaints of chills, high grade
temperature, dryness in the mouth, multiple vomiting, pain in the epigastrium, frequent watery,
foamy, dirty green color stool of unpleasant odor. The tongue and the skin are dry. BP - 80/40
mm Hg. What first aid is necessary for the patient?
A Intravenous injection of sodium solutions → to prevent dehydration.
B Fresh-frozen plasma transfusion
C To prescribe polyglucin
D Sympathomimetics
E Hemosorbtion

176-A 42-year-old female patient suffers from micronodular cryptogenic cirrhosis. Over the last
week her condition has deteriorated: she developed convulsions, mental confusion, progressing
jaundice. What study may give reasons for such aggravation?
A Determination of serum ammonia
B Determination of cholesterol ethers
C Determination of alpha-phetoprotein
D Determination of ALAT and ASAT
E Determination of alkaline phosphatase
Dr.HUR A. Salman - OdNMU

177-A patient has chronic heart failure of the II stage. He takes furosemide regularly three times a
week. He had developed bronchopneumonia and had been administered combined
pharmacotherapy. On the fifth day of therapy the patient complained of hearing impairment.
What drug coadministered with furosemide might have caused the hearing loss?
A Gentamicin
B Linex
C Nystatin
D Tavegil
E Mucaltin

178-A 60-year-old female patient complains of recurrent pain in the proximal interphalangeal and
wrist joints, their periodic swelling and reddening that have been observed for 4 years. X-ray
picture represents changes in form of osteoporosis, joint space narrowing and single usuras.
What is the most likely diagnosis?
A Rheumatoid arthritis → usura mean erosions 
B Osteoarthritis
C Gout
D Pseudogout
E Multiple myeloma
Dr.HUR A. Salman - OdNMU
179-A 37-year-old woman complains of generalized fatigue, irritability, dysphagia, chalk hunger.
On physical exam: t- 36,5oC, respirations - 20/min, Ps - 96 bpm, BP - 110/70 mm Hg. Satisfactory
nourishment. The skin and visible mucous membranes are pale. Blood test: Hb -70g/L,
erythrocytes - 3,4x1012/L$, CI - 0,7, reticulocytes - 2%, leucocytes - 4,7x109/L, eosinophilis. - 2%,
band neutrophils - 3%, segmented neutrophils - 64%, lymphocytes – 26%, monocytes - 5%, ESR -
15 mm/min. Serum ferrum - 7,3 mu*mol/L, total protein - 70g/L. Deficit of what factor caused the
development of the disease?
A Ferrum → Adult males have serum ferritin values averaging about 100 ug/L, while adult females have
levels averaging 30 ug/L. As iron stores are depleted, the serum ferritin falls to <15 ug/L. Such levels are
virtually always diagnostic of absent body iron stores.
B Vitamin B6
C Vitamin B12
D Protein
E Folic acid

180-A 28-year-old patient has been hospitalized for the pain in the epigastric region. He has a
10-year history of duodenal ulcer (DU). Recently, the pain character has changed: it became
permanent, persistent, irradiating to the back. There are general weakness, dizziness, fatigue.
The patient has put off weight. Objectively: HR- 68/min, AP- 120/80 mm Hg. What is most likely
cause of deterioration?
A Penetration
B Haemorrhage
C Perforation of duodenal wall
D Exacerbation of duodenal ulcer
E Stenosis development
** Duodenal ulcer pain often awakens the patient at night. About 50-80% of patients with duodenal
ulcers experience nightly pain, as opposed to only 30-40% of patients with gastric ulcers and 20-
40% of patients with non ulcer dyspepsia (NUD). Pain typically follows a daily pattern specific to
the patient. Pain with radiation to the back is suggestive of a posterior penetrating gastric ulcer
complicated by pancreatitis.

181-A 57-year-old male patient complains of dyspnea on exertion, heaviness in the right
hypochondrium and shin edemata towards evening. Objectively: temperature - 38,1oC, HR-
20/min, HR=Ps=92/min, AP- 140/90 mm Hg. There is apparent kyphoscoliosis. In the lungs
single dry rales can be auscultated. Heart sounds are muffled, rhythmic. ECG: Rv1+Sv5=15
mm. X-ray picture shows the bulging of pulmonary artery cone, right ventricle enlargement. What
is the most likely cause of this condition?
A Pulmonary heart → Cor pulmonale is a right-sided heart failure caused by arterial pulmonary
hypertension. Pulmonary hypertension is defined as an elevated mean pulmonary artery pressure
of ≥25 mm Hg at rest as assessed by right heart catheterization
B Atherosclerotic cardiosclerosis
C Dilatation cardiomyopathy
D Mitral stenosis
E Primary pulmonary hypertension
*** Acute cor pulmonale is a form of acute right heart failure produced by a sudden increase in
resistance to blood flow in the pulmonary circulation: it is observed in pulmonary embolism (PE)
and acute respiratory distress syndrome. Chronic cor pulmonale is a right heart failure caused by
chronic pulmonary arterial hypertension resulting from chronic lung disease, pulmonary vascular
disorders, or neuromuscular and skeletal diseases causes.
Dr.HUR A. Salman - OdNMU
** Right-sided ventricular disease caused by a primary abnormality of the left side of the heart or
congenital heart disease is not considered cor pulmonale.

182-A 30-year-old female patient has been delivered to a hospital for sudden dyspnea progessing
to asthma, sensation of having a "lump in the throat", hand tremor, fear of death. The attack has
developed for the first time and is associated with a strong emotion. There is no previous
history. Objectvely: respiratory rate - 28/min, Ps - 104/min, rhythmic, AP - 150/85 mm Hg. The
patient has rapid superficial vesicular breathing with extended expiration. Percussion findings:
heart borders are not changed. Cardiac sounds are loud, rhythmic. What is the most likely
diagnosis?
A Neurocirculatory asthenia → Da Costa's syndrome, which was colloquially known as soldier's
heart, is a syndrome with a set of symptoms that are similar to those of heart disease, though
a physical examination does not reveal any physiological abnormalities. In modern times, Da
Costa's syndrome is considered the manifestation of an anxiety disorder, and treatment is primarily
behavioral, involving modifications to lifestyle and exercise.
B Bronchial asthma
C Hypertensive crisis
D Cardiac asthma
E Thyrotoxic crisis

183-A 24-year-old emotionally-labile woman presents with irritation, depressed mood, palpitation,
shooting pain in the heart area, generalized fatigue following the divorce. On examination: palm
hyperhydrosis, pulse rate- 72-78 bpm, labile, heart without changes. ECG is normal. What is the
most probable pathology in this case?
A Neurasthenia → is a term that was first used at least as early as 1829 to label a mechanical
weakness of the nerves
B Ipochondric neurosis
Dr.HUR A. Salman - OdNMU
C Compulsive neurosis
D Schizophrenia
E Depressive neurosis

184-A 30-year-old woman with a long history of chronic pyelonephritis complains about
considerable weakness, sleepiness, decrease in diuresis down to 100 ml per day. AP- 200/120 mm
Hg. In blood: creatinine - 0,62 millimole/l, hypoproteinemia, albumines - 32 g/l, potassium - 6,8
millimole/l, hypochromic anemia, increased ESR. What is the first step in the patient treatment
tactics?
A Haemodialysis → review Q 125
B Antibacterial therapy
C Enterosorption
D Haemosorption
E Blood transfusion

185-A patient had macrofocal myocardial infarction. He is overweight for 36%, AP is 150/90 mm
Hg, blood sugar- 5,9 mmol/L, general cholesterol- 4,9 mmol/L, uric acid- 0,211 mmol/L. Which
risk factor should be urgently eradicated during the secondary prevention?
A Obesity
B Arterial hypertension
C Hyperglycemia
D Hypercholesterolemia
E Hyperuricemia

186-A 36-year-old female patient complains of bruises on the body, gingival haemorrhage, general
weakness. A month ago she had a severe domestic poisoning with some pesticide (the patient
can not remember the name). She has a 7-year record of working in contact with petroleum
products, particularly benzene. In blood: RBCs - 3,2x1012/l, WBCs - 2,7x109/l, thrombocytes –
70x109/l. What is the most likely pathology?
A Benzene intoxication
B Organophosphorus pesticide intoxication
C Organochlorine pesticide Intoxication
D Mercury-containing pesticide intoxication
E Chronic fatigue Syndrome

187-While staying in a stuffy room a 19-year-old emotionally labile girl developed severe weakness,
dizziness, blackout, nausea and loss of consciousness without convulsions. Objectively: the
patient is unconscious, the skin is pale, extremities are cold. AP- 90/60 mm Hg, Ps- 96/min,
deficient, breathing is shallow. Pupillary and tendon reflexes are present. There are no
pathological signs. What is the most likely diagnosis?
A Syncope → she got suffocation then loss of consciousness 
B Vegetovascular paroxysm
C Epileptic attack
D Hysterical neurosis
E Transient ischemic attack
** stuffy : (of a place) lacking fresh air or ventilation. Like overcrowded office

188-A patient complains of frequent, bulky, frothy stools with greenish mucus, cramping pain in
the umbilical region, abdominal murmur, body temperature at the rate of 39oC. The patient
associates the disease with consumption of soft-boiled eggs. What is the most likely pathogen?
Dr.HUR A. Salman - OdNMU
A Salmonella → half cooked egg or unboiled milk .. they like salmonella , don’t mix when you see
greenish mucus stool you will think about shigellosis (dysentri)
B Yersinia
C Shigella
D Enteropathogenic E.Coli
E Vibrio cholerae El Tor

189- A 40-year-old female patient complains of headache, dizziness, muscle weakness, occasional
cramps in the extremities. She has been taking antihypertensive medications for 10 years. AP-
180/100 mm Hg. Blood potassium - 1,8 millimole/l, sodium - 4,8 millimole/l. In urine: alkaline
reaction, the relative density - 1012, protein and sugar are not found, WBCs - 3-4 in the field of
vision, RBCs - 1-2 in the field of vision. Conn's syndrome is suspected. Which drug should be
chosen for the treatment of arterial hypertension?
A Spironolactone
B Propanolol
C Enalapril
D Hydrochlorothiazide
E Clonidine
Dr.HUR A. Salman - OdNMU

190-An 18-year-old patient presents no problems. Percussion reveals that heart borders are
displaced to the right and left by 1 cm, there is a coarse systolic murmur with its epicenter within
the 4th intercostal space on the left. What is the most informative examination to confirm the
clinical diagnosis?
A Ventriculography
B ECG
C PCG
D Echocardiography
E Polycardiography

191
A 56-year-old patient complains of having persistent chest pain on the right for the last 2
months. The pain is not associated with respiration. He also complains of cough with
blood-streaked sputum, weakness, decreased performance, fatigue. Chest radiograph shows a
Dr.HUR A. Salman - OdNMU
globular shade of 4x6 cm connected to the root of the lung in the lower part of the right lung.
What is the most likely diagnosis?
A Peripheral lung cancer → review Q 130
B Metastasis
C Lung abscess
D Pneumonia
E Tuberculoma

192-A 18-year-old patient had subtotal strumectomy due to malignant capillary cystadenoma of
the thyroid gland. In 2 months there was a suspicion of metastasis presence in the lungs. What
rontgenological method is to be used first?
A Roentgenography of lungs → chest xray …………… vey old , where is CT scan ?
B Roentgenoscopy of lungs
C Angiopneumonography
D Bronchography
E Bronchoscopy
** strumectomy : Surgical removal of all or a portion of a goitrous tumor.

193- A 58-year-old patient was diagnosed basal-cell skin cancer, 1st stage. Tumor is up to 1 cm in
size and with up to 0,5 cm deep infiltration in tissues. Tumor is localized in the right nasolabial
area. Choose the most optimal method of treatment.
A Short-distance roentgenotherapy → focal radiotherapy
B Long-distance roentgenotherapy
C Long-distance gamma therapy
D Chemotherapy
E Surgical treatment
Dr.HUR A. Salman - OdNMU

194-A 27-year-old patient complains of nasal haemorrhages, multiple bruises on the anterior
surface of the trunk and extremities, sudden weakness. In blood: Hb- 74 g/l, reticulocytes - 16%,
RBCs - 2,5x1012/l, platelets – 30x109/l, ESR- 25 mm/h. What is the most effective measure for the
treatment of thrombocytopenia?
A Splenectomy
B Iron preparations
C Hemotransfusion
D Cytostatics
E Vitamin B12

195- 2 days ago a patient presented with acute pain in the left half of chest, general weakness, fever
and headache. Objectively: between the 4 and 5 rib on the left the skin is erythematous, there
are multiple groups of vesicles 2-4 mm in diameter filled with transparent liquid. What diease are
these symptoms typical for?
A Herpes zoster
B Pemphigus
C Herpes simplex
D Streptococcal impetigo
E Herpetiform Duhring's dermatosis
Dr.HUR A. Salman - OdNMU

196- A woman while working in vegetable garden developed severe pain in the loin. Lasague's and
Nery tension signs are obviously marked on the right. Lumbar lordosis is smoothed, movements
are harshly restrained in lumbar part of the spine. Right ankle (Achilles) reflex is absent. What
kind of disease can it be?
A Lumbar-sacral radiculitis → inflammation of the nerve root of Lumbo-sacral spine / SCIATICA
B Lumbalgia → low back pain at lumber region
C Hepatic colic
D Renal colic
E Neuritis of femoral nerve

Radiculitis symptoms start off with radicular pain (pain that is felt down the pathway of the nerve
that is affected by its origin in the spine). The pain is also usually accompanied with sensations of
numbness, tingling, and muscle weakness

197
After lifting a load a patient felt undurable pain in the loin. He was diagnosed with acute
lumbosacral radiculitis. Which of the following is contraindicated for this patient?
A Warming procedures
B Dehydrating drugs
C Analgetics
D Vitamins of B group
E Intravenous injection of aminophylline
Dr.HUR A. Salman - OdNMU

198- An unconscious 35-year-old patient has been delivered by an ambulance to the intensive care
unit. Objectively: the patient is in semicoma. Moderate mydriasis is present. The reaction of
pupils to light is reduced. The reaction to verbal instructions is missing. AP is150/100 mm Hg,
there is tachycardia. Blood contains methanol. What antidote should be administered?
A Ethanol
B Unithiol
C Thiamine chloride
D Tavegil
E Naloxone
TREAT ALCOHOL BY ALCOHOL (methanol→ Ethanol)

199-After lifting a load, a 36-year-old male patient has experienced a severe pain in the lumbar
region, which spread to the right leg and was getting worse when he moved his foot or coughed.
Objectively: the long back muscles on the right are strained. Achilles jerk is reduced on the right.
There is a pronounced tenderness of paravertebral points in the lumbar region. The straight leg
raise (Lasegue's sign ) is positive on the right. What additional tests should be performed in the
first place?
A Radiography of the spinal column → x ray of lumbar region
B Computed tomography
C Magnetic resonance tomography → choose this when you find nerve root compression
D Electromyography
E Lumbar puncture
** I Hope you will not get this Q , because actually MRI best choice for spinal cord
and nerve roots also disc pathology

200
A female, aged 20, after smoking notices a peculiar inebriation with the feeling of burst of
energy, elation, irreality and changing of surroundings: the world gets full of bright colours, the
objects change their dimensions, people's faces get cartoon features, loss of time and space
judgement. What is the most likely diagnosis?
A Cocainism
B Morphinism
C Barbiturism
D Nicotinism
E Cannabism
Dr.HUR A. Salman - OdNMU

201
A 75 y.o patient can not tell the month, date and season of the year. After long deliberations she
manages to tell her name. She is in irritable and dissatisfied mood. She always carries a bundle
with belongings with her, hides a parcel with bread, shoes in her underwear in her bosom as well
as "invaluable books". What is the most probable diagnosis?
A Senile dementia → age
B Atherosclerotic (lacunar) dementia
C Presenile melancholia
D Behaviour disorder
E Dissociated personality (psychopathy)
** bosom a woman's chest.

202
A 17-year-old male patient consulted a therapist about malaise, chills, runny nose, aching
muscles and joints, nausea and diarrhea. The patient asks to prescribe him a lot of painkillers
and sedatives (tramadol or solpadein that help the best, and diazepam). Pharyngeal mucosa is
pale pink, clean. Auscultation reveals vesicular breathing. Tachycardia is present. The pupils
are dilated, there is sluggish response to light. There are injection marks on the forearm skin.
During examination, the patient's manner is vulgar, irritable, rude and untruthful. Make a
diagnosis:
A Opioid addiction
B Painkillers addiction
C Sedative drug addiction
D Acute respiratory disease
E Food-born toxic infection
Dr.HUR A. Salman - OdNMU

203
While lifting a heavy load a 39-year-old patient suddenly felt a severe headache, pain in the
interscapular region, and started vomiting. Objectively: the pulse is rhythmic, 60/min, AP-
180/100 mm Hg. The patient is agitated. He presents with photophobia, hyperacusis. There are
positive Kernig's and Brudzinski's signs on both sides. In blood: WBCs – 10x109/l. CSF
is bloody, cytosis is 240/3. What is the most likely diagnosis?
A Subarachnoid haemorrhage → meningeal symptoms +ve and CSF bloody , signs of anemia or
internal hemorrhage .
B Sympathoadrenal crisis
C Acute hypertonic encephalopathy
D Meningococcal meningitis
E Ischemic stroke

204-A 26-year-old male patient complains of piercing pain during breathing, cough, dyspnea.
Objectively: to- 37,3oC, respiration rate - 19/min, heart rate = Ps- 92/min; AP- 120/80
mm Hg. Vesicular respiration. In the inferolateral parts of chest auscultation in both inspiration
and expiration phase revealed noise that was getting stronger at phonendoscope pressing and
can be still heard after cough. ECG showed no pathological changes. What is the most likely
giagnosis?
A Acute pleuritis → review Q 174 also check 
B Intercostal neuralgia
C Subcutaneous emphysema
D Spontaneous pneumothorax
E Pericarditis sicca
Dr.HUR A. Salman - OdNMU
Acute pleuritis is inflammation of the pleura.
Classification:
◼ Dry pleurisy (pleuritis sicca)
◼ Pleurisy with effusion (pleuritis exudativa)
The character of the inflammatory effusion may be different: serous, serofibrinous, purulent,
and haemorrhagic.
Etiology and pathogenesis
 Serous and serofibrinous pleurisy (tuberculosis in 70-90 per cent of cases, pneumonia,
certain infections, and also rheumatism in 10-30 per cent of cases)
 Purulent process (pneumococci, streptococci, staphylococci, and other microbes)
 Haemorrhagic pleurisy (tuberculosis of the pleura, bronchogenic cancer of the lung with
involvement of the pleura, and also in injuries to the chest)
205-A 45-year-old patient, a sailor, was hospitalized on the 2nd day of the disease. A week ago he
returned from India. Complains of body temperature of 41oC, severe headache, dyspnea,
cough with frothy rusty sputum. Objectively: the patient is pale, mucous membranes are
cyanotic, breathing rate is 24/min, tachycardia is present. In lungs: diminished breath sounds,
moist rales over both lungs, crepitation. What is the most likely diagnosis?
A Pneumonic plaque
B Miliary tuberculosis
C Influenza
D Ornithosis
E Sepsis
Dr.HUR A. Salman - OdNMU
206-HIV displays the highest tropism towards the following blood cells:
A T-helpers
B T-suppressors
C T-killers
D Thrombocytes
E Erythrocytes

207-A 25-year-old patient complains of general weakness, dry cough, sweating, subfebrile
temperature. Objectively: lung auscultation reveals vesicular resiration with no wheezing.
Fluorogram shows focal shadows of high intensity in the 1-2 segments of the right lung. Mantoux
test gave a reaction of 16 mm of induration. What clinical form of tuberculosis is most likely?
A Focal
B Infiltrative
C Disseminated
D Tuberculoma
E Miliary

208-A 22-year-old patient is a clerk. His working day runs in a conditioned room. In summer he
was taken by an acute disease with the following symptoms: fever, dyspnea, dry cough, pleural
pain, myalgia, arthralgia. Objectively: moist rales on the right, pleural friction rub. X-ray picture
showed infiltration of the inferior lobe. In blood: WBC – 11x109/l, stab neutrophils - 6%, segmented
neutrophils - 70%, lymphocytes - 8%, ESR - 42 mm/h. What is the ethiological factor of
pneumonia?
A Legionella
B Mycoplasm
C Streptococcus
D Staphylococcus
E Pneumococcus
Dr.HUR A. Salman - OdNMU
209-A 43-year-old female patient complains of dyspnea, swelling of legs, abdomen enlargement,
pricking heart pain. She has a history of tuberculous bronchadenitis, quinsies. The patient's
condition deteriorated 6 months ago. Objectively: cyanosis, bulging neck veins, vesicular
breathing. Heart borders are not displaced. Heart sounds are muffled, Ps is 106/min, liver is +4
cm, ascites is present. Low voltage on the ECG has been revealed. Radiograph shows a thin
layer of calcium deposits along the left contour of heart. What treatment should be
recommended to the patient?
A Treatment by a cardiac surgeon → Constrictive pericarditis - ** review Q 137
B Digitalis preparations
C Anti-TB drugs
D Diuretics
E Vasodilators, nitrates

210- A 53-year-old female patient complains of cardiac pain and rhythm intermissions. She has
experienced these presentations since childhood. The patient's father had a history of cardiac
arrhythmias. Objectively: the patient is in grave condition, Ps- 220 bpm, AP- 80/60 mm Hg. ECG
results: heart rate - 215/min, extension and deformation of QRS complex accompanied by
atrioventricular dissociation; positive P wave. Some time later heart rate reduced down to
45/min, there was a complete dissociation of P wave and QRST complex. Which of the
following will be the most effective treatment?
A Implantation of the artificial pacemaker → 3rd degree AV block
B $\beta$-adrenoreceptor blocking agents
C Cholinolytics
D Calcium antagonists
E Cardiac glycosides

211
A 26-year-old female patient has an 11-year history of rheumatism. Four years ago she
suffered 2 rheumatic attacks. Over the last 6 months there have been paroxysms of atrial
fibrillation every 2-3 months. What option of antiarrhythmic therapy or tactics should be
proposed?
A Prophylactic administration of cordarone → amiodarone
B Immediate hospitalization
C Defibrillation
D Lidocaine administration
E Heparin administration
Dr.HUR A. Salman - OdNMU
212- A 49-year-old patient complains of dyspnea, cough. There are no sputum discharges. He has
repeatedly used salbutamol and intal but with no effect. Objectively: he is only able to sit while
leaning on the table. Cyanosis of face, acrocyanosis are present. Breathing is shallow,
laboured, in some parts it cannot be auscultated; there are diffuse rales, expiration is
significantly prolonged. Heart sounds are muffled, tachycardia is present. Ps - 112/min, AP-
110/70 mm Hg. Liver is located near the costal arch. There are no peripheral edemata. What is
your provisional diagnosis?
A Status asthmaticus → review Q 26 – 38 -151 -173
B Chronic obstructive bronchitis
C Bronchiale asthma, moderate gravity
D Foreign object aspiration
E Cardiac asthma

213-Thrombosis of the coronary artery caused myocardial infarction. What mechanisms of injury
will be the dominating ones in this disease?
A Calcium mechanisms
B Electrolytoosmotic mechanisms
C Acidotic mechanisms
D Protein mechanisms
E Lipid mechanisms

214- On the second day of the disease a 22-year-old male patient complains of high-grade fever,
headache in the region of forehead and superciliary arches, and during eye movement; aching
muscles and joints. Objectively: body temperature is 39oC. Face is hyperemic, sclerae are
injected. The mucous membrane of the soft palate and posterior pharyngeal wall is bright
hyperemic and has petechial hemorrhages. What changes in the hemogram are typical for this
disease?
A Leukopenia → symptoms of viral infection = WBC 
B Leukocytosis → this mostly for bacterial infection.
C Neutrocytosis
D Anemia
E Accelerated ESR
Dr.HUR A. Salman - OdNMU
215- A female patient consulted a dermatologist about the rash on the trunk and extremities.
Objectively: interdigital folds, flexor surfaces of wrists and navel region are affected with pairs of
nodulo-cystic eruptions and crusts. The rash is accompanied by skin itch that is getting stronger
at night. What external treatment should be administered?
A 20% benzyl benzoate emulsion →Scabies = Review Q 100
B 5% sulfuric ointment
C 2% sulfuric paste
D 5% naphthalan ointment
E 5% tetracycline ointment

216- A 54 y.o. male patient suffers from dyspnea during mild physical exertion, cough with sputum
which is excreted with diffculty. On examination: diffuse cyanosis. Is Barrel-chest. Weakened
vesicular breathing with prolonged expiration and dry whistling rales. AP is 140/80 mm Hg, pulse
is 92 bpm, rhythmic. Spirography: vital capacity (VC)/predicted vital capacity- 65%, FEV1/FVC–
50%. Determine the type of respiratory insufficiency (RI).
A RI of mixed type with prevailing obstruction→ they mean mix type (obstructive and restrictive)
but mainly obstruction
B RI of restrictive type
C RI of obstructive type
D RI of mixed type with prevailing resriction
E There is no RI

217-An ambulance had been called to a 48-year-old man. According to his relatives, the patient had
had three attacks of unconsciousness accompanied by convulsions within 24 hours. On
examination the doctor witnessed the following attack: the patient lost consciousness and fell to
the floor, developed tonic, then clonic convulsions of trunk and extremities. The attack lasted 1
minute and ended with involuntary urination. Specify the kind of attack:
A Epileptic seizure
B Syncope
C Vegetative crisis
D Coma
E Attack of hysteria

218-A patient is 60 years old, retired, worked as deputy director of a research institute.
Behavioural changes appeared 2 years ago after the death of her husband: she stopped looking after
herself and leaving the house; then she refused to clean the apartment and cook. Mental status:
temporal disorientation. The patient does not understand many of the questions, is confused;
does not know how to cook soup or fasten a button. Her speech is characterized by stumbling
and logoclonia. She does not recognize doctors, fellow patients. She cries a lot but can not
explain the reason for tears. What is the mechanism of this pathology?
A Atrophy of the cerebral cortex → this pathology mostly related to age
B Atherosclerotic changes in cerebral vessels
C Serotonin deficiency
D Impaired conversion of dopamine to noradrenaline
E Disorder of melatonin metabolism unfortunately from I
will BECOME CRAZY
with age .. I don’t want
loose my BRAIN  
Dr.HUR A. Salman - OdNMU

219-A 26-year-old patient has abused alcohol since the age of 16, needs a morning-after drink to
cure hangover. He takes alcohol nearly every day, "a little at a time". Twice a week he gets
severely drunk. The patient works as a motor mechanic, over the last 2 years work conflicts
have become more frequent. What medical and tactical actions should be taken in this case?
A Voluntary consultation and treatment at an addiction clinic
B Compulsory treatment
C Referral to treatment at an activity therapy centre
D Referral to medical-social expert commission for assessment of his working ability
E Consultation with a psychologist

220-A 47-year-old female patient has an 8-year history of ulcerative colitis, has been treated with
glucocorticoids. She complains of cramping pain in the umbilical region and left iliac region which
has significantly increased during the past 2 weeks, diarrhea with mucus and blood 4-6 times a
day, elevated body temperature up to 38-39oC, headache and pain in the knee joints.
Objectively: the patient is in moderate condition, Ps - 108/min, AP - 90/60 mm Hg; heart and
lungs are unremarkable; the tongue is moist; abdominal muscle tone is significantly decreased;
peristaltic noises are absent. What complication developed in the patient?
A Toxic dilatation of the colon
B Perforation of the colon
C Enterorrhagia
D Stricture of the colon
E Colon carcinoma

221- A 26-year-old patient with left lower lobe pneumonia experiences an acute chest pain on the
left during coughing. Objectively: diffuse cyanosis, extension of the left side of chest. Percussion
reveals high tympanitis. Auscultation reveals no respiratory murmurs above the left side of
chest. There is a deviation of the right cardiac border towards the midclavicular line. What
examination will be the most informative?
A X-Ray
B Bronchoscopy
C Bronchography
D Pneumotachometry
E Spirography
Dr.HUR A. Salman - OdNMU
222-A male patient presents with swollen ankles, face, eyelids, elevated AP- 160/100 mm Hg,
pulse-54 bpm, daily loss of albumine with urine- 4g. What therapy is pathogenetic in this case?
A Corticosteroids
B Diuretics
C NSAID
D Calcium antagonists
E Antibiotics

223-After myocardial infarction, a 50-year-old patient had an attack of asthma. Objectively:


bubbling breathing with frequency of 32/min, cough with a lot of pink frothy sputum, acrocyanosis,
swelling of the neck veins. Ps- 108/min, AP- 150/100 mm Hg. Heart sounds are muffled. Mixed
moist rales can be auscultated above the entire lung surface. What drug would be most effective in
this situation?
A Nitroglycerin intravenously
B Pentamin intravenously
C Strophanthin intravenously
D Dopamine intravenously
E Aminophylline intravenously

224
During dynamic investigation of a patient the increase of central venous pressure is combined
with the decrease of arterial pressure. What process is proved by such combination?
A Increase of bleeding speed ???
B Developing of cardiac insufficiency → this more logic for answer as in surgical Base Q
C Shunting
D Depositing of blood in venous channel
E Presence of hypervolemia

225
A male patient complains of heartburn which gest stronger while bending the body, substernal
pain during swallowing. There is a hiatus hernia on X-ray. What disoeder should be expected at
gastroscopy?
A Gastroesophageal reflux
B Chronic gastritis
C Gastric peptic ulcer
D Acute erosive gastritis
E Duodenal peptic ulcer

226
A 43 y.o. male complains of stomach pain, which relieves with defecation, and is accompanied
by abdominal winds, rumbling, the feeling of incomplete evacuation or urgent need for bowel
movement, constipation or diarrhea in alternation. These symptoms have lasted for over 3
months. No changes in laboratory tests. What is the most likely diagnosis?
A Irritable bowel syndrome → check Q 7
B Spastic colitis
C Colitis with hypertonic type dyskinesia
D Chronic enterocolitis, exacerbation phase
E Atonic colitis
Dr.HUR A. Salman - OdNMU

227
Against the background of angina a patient has developed pain in tubular bones. Examination
revealed generalized enlargement of lymph nodes, hepatolienal syndrome, sternalgia. In blood:
RBCs - 3,6x1012/l, Hb- 87 g/l, thrombocytes – 45x109/l, WBCs – 13x109/l, blasts - 87%, stab
neutrophils - 1%, segmented neutrophils - 7%, lymphocytes - 5%, ESR - 55 mm/h. What is the most
likely diagnosis?
A Acute leukemia
B Erythremia
C Chronic lymphocytic leukemia → Q 76
D Chronic myeloid leukemia
E Multiple myeloma

228
A 49-year-old female patient with schizophrenia is all the time listening to something, insists that
"there is a phone in her head" as she hears the voice of her brother who tells her to go home.
The patient is anxious, suspicious, looks around all the time. Specify the psychopathological
syndrome:
A Hallucinatory
B Generalized anxiety disorder
C Paranoiac
D Paraphrenic
E Depressive

229
A 43-year-old male patient undergoing treatment for peptic ulcer complains of weakness,
dizziness, coffee-ground vomiting, melena. After administration of haemostatics the patient's
condition has not improved, fresh blood has shown up in the vomit, skin bruises of different
sizes have appeared. In blood: thrombocytes – 50x109/l, Lee-White clotting time - 35
minutes, APTT - 80 seconds. In this case it is most rational to administer the following
preparation:
A Fresh frozen plasma
B Heparin
C Fibrinogen
D Rheopolyglucinum
E Vikasol

230
A 38-year-old patient complains of inertness, subfebrile temperature, enlargement of lymph
nodes, nasal haemorrhages, bone pain. Objectively: the patient's skin and mucous membranes
are pale, palpation revealed enlarged painless lymph nodes; sternalgia; liver was enlarged by 2
cm, spleen - by 5 cm, painless. In blood: erythrocytes - 2,7x1012/l, Hb- 84 g/l, leukocytes –
58x109/l, eosinophils - 1%, stab neutrophils - 2%, segmented neutrophils - 12%, lymphocytes - 83%,
lymphoblasts - 2%, smudge cells; ESR- 57 mm/h. What is the most likely diagnosis?
A Chronic lymphatic leukemia → CLL , Review 76
B Chronic myeloleukemia
C Acute lymphatic leukemia
Dr.HUR A. Salman - OdNMU
D Acute myeloleukemia
E Lymphogranulomatosis

231
A 30-year-old male patient complains of inertness, low-grade fever, bleeding gums, frequent
quinsies, aching bones. Objectively: the patient has pale skin and mucous membranes,
sternalgia, +2 cm liver, +5 cm painless spleen. Blood test results: RBC - 2,7x1012/l, Нb
- 80 g/l, WBC – 3x109/l, eosinophils - 4%, basophils - 5%, blasts - 4%, stab neutrophils
- 2%, segmented neutrophils - 17%, lymphocytes - 29%, myelocytes - 25%, promyelocytes -
12%, monocytes - 2%, platelets – 80x109/l, ESR - 57 mm/h. What test should be performed to
verify the diagnosis?
A Sternal puncture
B Trephine biopsy
C Lymph node biopsy
D Lumbar puncture
E Chest X-ray

232
A 24-year-old patient complains about putting on weight, limosis. Objectively: the patient's
constitution is of hypersthenic type, body weight index is 33,2 kg/m2, waist circumference is
100 cm. Correlation of waist circumference to the thigh circumference is 0,95. What is the most
likely diagnosis?
A Alimentary constitutional obesity of the I stage, abdominal type
B Hypothalamic Itsenko-Cushing obesity of the II stage, gynoid type
C Alimentary constitutional obesity of the III stage, gynoid type
D Alimentary constitutional obesity of the II stage, abdominal type
E Hypothalamic Itsenko-Cushing obesity of the I stage, abdominal type

233
A 47-year-old male patient has been lately complaining of compressing chest pain that occurs
when he walks a distane of 700-800 m. Once a week, he drinks 2 liters of beer. Rise in arterial
pressure has been observed for the last 7 years. Objectively: Ps - 74/min, AP - 120/80 mm Hg.
The bicycle ergometry performed at workload of 75 watts shows 2 mm ST-segment
depression in V4-V6 leads. What is the most likely diagnosis ?
A Exertional stenocardia, II functional class → walk 700 – 800 m(>500m) (according to krok)
B Exertional stenocardia, III functional class → walk 100 – 150 m
C Exertional stenocardia, IV functional class → dyspnea at rest
D Vegetative-vascular dystonia of hypertensive type
E Alcoholic cardiomyopathy
Review Q 71

234
A 58-year-old patient complains about sensation of numbness, sudden paleness of II-IV fingers,
muscle rigidness, intermittent pulse. The patient presents also with polyarthralgia, dysphagia,
constipations. The patient's face is masklike, solid edema of hands is present. The heart is
enlarged; auscultation revealed dry rales in lungs. In blood: ESR- 20 mm/h, crude protein - 85/l,
γ-globulines - 25%. What is the most likely diagnosis?
A Systemic scleroderma
Dr.HUR A. Salman - OdNMU
B Dermatomyositis
C Rheumatoid arthritis
D Systemic lupus erythematosus
E Raynaud's disease

235
A 45-year-old man has been exhibiting high activity for the last 2 weeks, he became talkative,
euphoric, had little sleep, claimed being able "to save the humanity" and solve the problem of
cancer and AIDS, gave money the starangers. What is the most likely diagnosis?
A Maniacal onset
B Panic disorder
C Agitated depression
D Schizo-affective disorder
E Catatonic excitation

236
A patient had four generalized convulsive seizures within a day. Between the seizures the
patient did not come to waking consciousness (was in a coma or stupor). Specify his state:
A Status epilepticus
B Frequent generalized seizures
C Frequent jacksonian seizures
D Hysterical attacks
E Frequent complex partial seizures

237
A 35-year-old patient complains of heartburn, sour eructation, burning, compressing retrosternal
pain and pain along the esophagus rising during forward bending of body. The patient hasn't
been examined, takes Almagel on his own initiative, claims to feel better after its taking. Make a
provisional diagnosis:
A Gastroesophageal reflux disease
B Functional dyspepsia
C Cardiospasm
D Gastric ulcer
E Duodenal ulcer

238
As a result of lifting a load a 62-year-old female felt acute pain in the lumbar region, in a buttock,
posterolateral surface of her right thigh, external surface of the right shin and dorsal surface of
foot. Objectively: weakness of the anterior tibial muscle, long extensor muscle of the right toes,
short extensor muscle of the right toes. Low Achilles reflex on the right. Positive Lasegue's sign.
What examination method would be the most effective for specification of the diagnosis of
discogenic compression of L5 root?
A Magnetic resonance scan
B Spinal column X-ray
C Electromyography
D Angiography
E Lumbar puncture
Dr.HUR A. Salman - OdNMU

239
A 45-year-old female patient complaining of general weakness, nausea and vomiting hass been
delivered to a hospital by the ambulance. Recently there has been a lack of appetite, weight
loss. Objectively: hyperpigmentation of skin, blood pressure at the rate of 70/45 mm Hg,
bradycardia. Additional studies revealed the reduced concentration of aldosterone and cortisol
in blood, decreased excretion of 17-ketosteroids and 17-oxyketosteroids in the urine,
hyponatremia, chloropenia, hypokalemia. What therapeutic measures are required?
A To administer glucocorticoids, mineralocorticoids, and a diet with a high content of cooking salt
B To prescribe a diet with a high content of cooking salt
C To administer prednisolone
D To administer aldosterone
E To administer insulin

240
A 23-year-old female patient has a mental disease since the age of 18, the course of disease
has no remission periods. At a hospital the patient mostly presents with non-purposeful foolish
excitation: she makes stereotypic grimaces, exposed, masturbating in front of a loud laugh,
repeating the stereotypical abusive shouts. The patient should be assigned:
A Neuroleptics
B Antidepressants
C Tranquilizers
D Nootropics
E Mood stabilizers

241
A 40-year-old patient is registered in a narcological dispensary. Somatically: skin is dramatically
hyperemic, sclera are injected, hyperhidrosis is present. AP- 140/100 mm Hg, heart rate -
100/min. Mental state: autopsychic orientation is intact, allopsychic orientation is distorted. The
patient presents with motor anxiety. There is a look of fear on his face. He refuses to talk about
his problems and asks to release him immediately, because he "may be killed." This state
developed a day after a regular drinking bout. What is your provisional diagnosis?
A Delirium tremens
B Organic delirium
C Paranoia
D Alcoholic hallucinosis
E Alcoholic paranoid

242
During the preventive examination a 17-year-old young man reports no health problems.
Objectively: the patient is undernourished, asthenic; blood pressure is 110/70 mm Hg, Ps -
80/min. Heart borders are within normal range. Auscultation reveals three apical heart sounds,
murmurs are absent. ECG shows no pathological changes, PCG registers the S3 occurring
0,15 seconds after the S2. How can you interpret these changes?
A Physiologic S3
B Fout-ta-ta-rou (three-component rhythm)
C Protodiastolic gallop rhythm
Dr.HUR A. Salman - OdNMU
D Presystolic gallop rhythm
E Physiologic S4

243
A patient is being prepared for the operation on account of varix dilatation of lower extremities
veins. Examination of the patient's soles revealed flour-like desquamation along the skin folds.
All the toenails are greyish-yellow, thickened and partially decayed. What dermatosis should be
suspected?
A Rubromycosis
B Pityriasis versicolor
C Candidosis
D Microsporia
E Microbial eczema

244
A 14-year-old patient with signs of internal haemorrhage has been taken to a hospital after a
fight. He has had haemophilia A since childhood. He has been diagnosed with retroperitoneal
hematoma. What should be administered in the first place?
A Cryoprecipitate
B Aminocapronic acid
C Dried plasma
D Platelet concentrate
E Fresh blood

245
A 58-year-old patient complains of a headache in the occipital region, nausea, choking,
opplotentes. The presentations appeared after a physical exertion. Objectively: the patient is
excited. Face is hyperemic. Skin is pale. Heart sounds are regular, the 2nd aortic sound is
accentuated. AP- 240/120 mm Hg, HR- 92/min. Auscultation reveals some fine moist rales in
the lower parts of the lungs. Liver is not enlarged. ECG shows signs of hypertrophy and left
ventricular overload. What is the most likely diagnosis?
A Complicated hypertensic crisis, pulmonary edema
B Acute myocardial infarction, pulmonary edema
C Bronchial asthma exacerbation
D Uncomplicated hypertensic crisis
E Community-acquired pneumonia

246
A 37-year-old patient complains of pain in the lumbar spine that is getting stronger during
walking; restricted mobility, edema of the right side of abdomen. He has a history of focal
tuberculosis. X-ray picture shows the destruction of the adjacent surfaces of the 1-2 vertebral
bodies of the lumbar spine, vertebral body height is decreased, intervertebral foramen is
undetectable. Abdominal ultrasound reveals a 15x20 cm formation in the retroperitoneal space,
there are echo signals of fluid presence. What is the most likely diagnosis?
A Tuberculous spondylitis of the lumbar spine
B Fracture of the 1-2 vertebral bodies of the lumbar spine
C Spinal metastases
Dr.HUR A. Salman - OdNMU
D Spondylolisthesis of the lumbar spine
E Osteochondrosis

247
A 43-year-old female patient was delivered to the hospital in grave condition. She has a history
of Addison's disease. The patient had been regularly taking prednisolone but a week before she
stopped taking this drug. Objectively: sopor, skin and visible mucous membranes are pigmented,
skin and muscle turgor is decreased. Heart sounds are muffled, rapid. AP- 60/40 mm Hg, heart
rate - 96/min. In blood: Na - 120 millimole/l, K - 5,8 millimole/l. Development of this complication
is primarily caused by the deficit of the following hormone:
A Cortisol
B Corticotropin (ACTH)
C Adrenaline
D Noradrenaline
E Adrostendion

248
In a cold weather, the emergency room admitted a patient pulled out of the open water. There
was no respiratory contact with the water. The patient is excited, pale, complains of pain,
numbness of hands and feet, cold shiver. Respiratory rate is 22/min, AP - 120/90 mm Hg, Ps -
110/min, rectal temperature is 34,5oC. What kind of warming is indicated for this patient?
A Passive warming
B Infusion of 37oC solutions
C Hot compresses
D Warm bath
E Hemodialysis with blood warming

249
Survey radiograph of a 52-year-old worker of an agglomeration plant (28 years of experience,
the concentration of metal dust is 22-37 mg/m3) shows mildly pronounced interstitial fibrosis
with diffused contrast well-defined small nodular shadows. The patient has no complaints.
Pulmonary function is not compromised. What is the provisional diagnosis?
A Siderosis
B Silicosis
C Anthraco-silicatosis
D Silicatosis
E Anthracosis

250
A 60-year-old patient complains of nearly permanent sensation of heaviness and fullness in the
epigastrium, that increases after eating, foul-smelling eructation, occasional vomiting with food
consumed 1-2 days ago, weight loss. 12 years ago he was found to have an ulcer of pyloric
channel. The patient has taken ranitidine for periodic hunger pain. The patient's condition has
been deteriorating over the last 3 months. Objectively: splashing sound in the epigastrium is
present. What kind of complication is it?
A Pyloric stenosis
B Penetration of gastric ulcer
Dr.HUR A. Salman - OdNMU
C Functional pyloric spasm
D Foreign body in the stomach (bezoar)
E Malignization of gastric ulcer

251
A 52-year-old patient works as a secretary and has 30 year record of service. She complains
of spasms in her right hand during working and inability to type and write. Up to 80% of her work
involves hand load. The patient has been presenting with these symptoms for 2 years.
Objectively: the right hand is tense, there is an increase in muscle tone, attempts to write cause
spasms. Examination revealed no pathological changes of CNS. What is the most likely diagnosis?
A Spastic form of coordination neurosis
B Neuralgic form of coordination neurosis
C Paretic form of coordination neurosis
D Hysteric neurosis
E Chronic manganese intoxication

252
Examination of an electric welder with 15 years of service record revealed dry rales in the lower
lung fields. Radiograph shows diffuse nodules sized 3-4 mm in the middle and lower lung fields.
What disease can be suspected?
A Heavy-metal coniosis
B Silicosis
C Silicatosis
D Carbon pneumo coniosis
E Bronchitis

253
A 22-year-old vegetarian patient with signs of malnutrition consulted a doctor about smell and
taste distortion, angular stomatitis. Objectively: expressively blue sclerae. The patient was
diagnosed with iron deficiency anemia. What is the dominating clinical syndrome?
A Sideropenic
B Anaemic
C Haemologic
D Haemolytic
E Myelodysplastic

254
A patient complains of retrosternal pain, difficult swallowing, over 10 kg weight loss within three
months, general weakness. In blood: hypochromic anaemia, neutrophilic leukocytosis. In feces:
weakly positive Gregersen's reaction. On esophagram a filling defect with ill-defined serrated
edges shows up along a large portion of the esophagus. What is the most likely diagnosis?
A Esophageal carcinoma
B Benign tumour
C Esophageal achalasia
D Peptic ulcer
E Sideropenic dysphagia
Dr.HUR A. Salman - OdNMU

255
A 12-year-old boy periodically has short episodes (10-15 seconds) of a brief loss of awareness
with a dazed look and eyes stare in an upright position, blank expression of face, absence of
motions and subsequent amnesia. Specify the described state:
A Absence seizure
B Obnubilation
C Trance
D Fugue
E Sperrung

256
A 19-year-old male patient complains of intense pain in the left knee joint. Objectively: the left
knee joint is enlarged, the overlying skin is hyperemic, the joint is painful on palpation. Blood test
results: RBC - 3,8x1012/l, Hb - 122 g/l, lymphocytes - 7,4x109/l, platelets – 183x109/l. ESR - 10
mm/h. Duke bleeding time is 4 minutes, Lee-White clotting time - 24 minutes. A-PTT is 89 s.
Rheumatoid factor is negative. What is the most likely diagnosis?
A Hemophilia, hemarthrosis
B Werlhof's disease
C Rheumatoid arthritis
D Thrombocytopathy
E Hemorrhagic vasculitis, articular form
Dr.HUR A. Salman - OdNMU

257
Explosion of a tank with benzene at a chemical plant has killed and wounded a large number of
people. There are over 50 victims with burns, mechanical injuries and intoxication. Specify the
main elements of medical care and evacuation of population in this situation:
A Sorting, medical assistance, evacuation
B Sorting, evacuation, treatment
C Medical assistance, evacuation, isolation
D Isolation, rescue activity, recovery
E Sorting, recovery, rescue activity

258-An emergency doctor has diagnosed a 32-year-old woman with generalized convulsive status
epilepticus. The deterioration in the patient's condition is caused by a sudden gap in the epilepsy
treatment. Specify the doctor's further tactics:
A Hospitalization in the intensive care unit
B Hospitalization in the department of neurology
C Hospitalization in the department of neurosurgery
D Outpatient monitoring by a neuropathologist
E Outpatient monitoring by a neurosurgeon

259-A 63-year-old male patient with persistent atrial fibrillation complains of moderate dyspnea.
Objectively: peripheral edemata are absent, vesicular breathing is present, heart rate - 72/min,
AP - 140/90 mm Hg. What combination of drugs will be most effective for the secondary
prevention of heart failure?
A Beta-blockers, ACE inhibitors
B Beta-blockers, cardiac glycosides
C Cardiac glycosides, diuretics
D Cardiac glycosides, ACE inhibitors
E Diuretics, beta-blockers

260
A 57-year-old male patient had an attack of retrosternal pain that lasted more than 1,5 hours.
Objectively: the patient is inert, adynamic, has pale skin, cold extremities, poor volume pulse,
heart rate - 120/min, AP - 70/40 mm Hg. ECG shows ST elevation in II, III, aVF leads. What
condition are these changes typical for?
Dr.HUR A. Salman - OdNMU
A Cardiogenic shock
B Arrhythmogenic shock
C Perforated gastric ulcer
D Acute pericarditis
E Acute pancreatitis

261
A 42-year-old female lives in the basement, is unemployed, undernourished. She complains of
having general weakness, hair loss, brittle nails for six months, likes to eat chalk. Objectively: the
patient is emaciated, pale, has dry skin. Peripheral lymph nodes are not enlarged. Liver is +1,5
cm. In blood: RBCs - 1,8x1012/l, Hb- 62 g/l, colour index - 0,78, reticulocytes - 0,5 o/oo, ESR- 18
mm/h. Leukogram exhibits no pathology. What is a provisional diagnosis?
A Nutritional iron deficiency anaemia
B Chronic hepatitis
C B12-deficiency anaemia
D Acquired haemolytic anaemia
E Congenital haemolytic anaemia

262
A 20-year-old patient complains of breath shortness, continuous dull heart pain, irritability.
Objectively: general condition is satisfactory, the pulse is labile, AP- 130/60 mm Hg. ECG shows
repolarization disorder. The patient has been diagnosed with cardiac-type neurocirculatory
dystonia. The patient should receive treatment under the following conditions:
A Outpatient treatment
B Inpatient treatment at the therapeutic department
C Inpatient treatment at the cardiology department
D Inpatient treatment at the cardiac surgery department
E Inpatient treatment at the psychiatric department

263
A 45-year-old male patient complains of acute pain in his right side irradiating to the right thigh
and crotch. The patient claims also to have frequent urination with urine which resembles a meat
slops. The patient has no previous history of this condition. There is costovertebral angle
tenderness on the right (positive Pasternatsky's symptom). What is the most likely diagnosis?
Dr.HUR A. Salman - OdNMU
A Urolithiasis
B Acute appendicitis
C Acute pyelonephritis
D Acute cholecystitis. Renal colic
E Acute pancreatitis

264
A 38-year-old male works within the range of ionizing radiation. At a routine medical examination
he presents no problems. In blood: RBCs - 4,5x1012/l, Hb- 80 g/l, WBCs - 2,8x109/l, thrombocytes
– 30x109/l. Decide if this person can work with sources of ionizing radiation:
A Working with radioactive substances and other sources of ionizing radiation is contraindicated
B The patient is allowed to work with radioactive substances
C The patient can only work with radioactive substances of low activity
D The patient can be allowed to work after an extended medical examination
E The patient is allowed to work with radioactive substances for the limited period of time

265
A patient who undergoes treatment at a tuberculosis clinic has complained of having
progressing headache for the last 3 weeks. Neurological examination reveals rigidity of occipital
muscles, no focal symptoms. What is your provisional diagnosis?
A Tuberculous meningitis
B Chorea minor
C Brain tumour
D Myelitis
E Convexital arachnoiditis

266-A patient with chronic suppurative otitis has developed severe headache, vomiting, body
temperature rise. The meningeal symptoms are present. There are no focal neurological
symptoms. The further tactics of a doctor should be:
A Urgent hospitalization and diagnostic lumbar puncture
B Skull radiography
C Regular medical check-up
D Administration of anti-inflammatory drugs
E Referral for a consultation with otolaryngologist

267-A 28-year-old male patient complains of sour regurgitation, cough and heartburn that occurs
every day after having meals, when bending forward or lying down. These problems have been
observed for 4 years. Objective status and laboratory values are normal. FEGDS revealed
endoesophagitis. What is the leading factor in the development of this disease?
A Failure of the lower esophageal sphincter
B Hypersecretion of hydrochloric acid
C Duodeno-gastric reflux
D Hypergastrinemia
E Helicobacter pylori infection

268-Routine examination of a 16-year-old boy revealed the presence of three heart sounds on
Dr.HUR A. Salman - OdNMU
auscultation. The third sound is low and occurs in early diastole, there is no additional murmur.
In history: pneumonia six months ago. The patient presents no problems. Examination revealed
hyposthenia, underdevelopment of muscles. Laboratory and instrumental studies reveald no
peculiarities. What is the origin of the additional heart sound?
A Physiological III sound
B The sound of the mitral valve opening
C Protodiastolic gallop rhythm
D Pericardial diastolic sound
E The sound of the tricuspid valve opening

269
A patient's condition is getting worse towards evening: she becomes excited, complains of
"internal anxiety", "a weight on her heart", foreboding of evil - "something evil will happen to me
or my family". The patient is sad, melancholic, has poor appetite and sleep disorders. Specify
the kind of mental disorder:
A Anxious depression
B Somatized depression
C Endogenous depression
D Hypochondriac depression
E Agitated depression

270
A 30-year-old male patient had been admitted to the TB hospital because of the following
changes detected by fluorography: an ill-defined shadow of low intensity up to 1 cm in diameter
in the S1 of the right lung. CT scan showed a destruction area in the center of the shadow.
Sputum analysis revealed MTB. The patient was diagnosed with focal tuberculosis. What
phases of tuberculosis are the identified changes typical for?
A Infiltration and disintegration
B Infiltration and dissemination
C Resorption and scarring
D Disintegration and dissemination
E Calcification and resorption

271
A 43-year-old female complains of significant weakness, sore throat, occurrence of multiple
unexplained bruises on her skin. These symptoms have been present for a week, the disease is
associated with quinsy which she had some time before. Objectively: body temperature -
38,9oC, respiratory rate - 24/min, Ps - 110/min, AP - 100/65 mm Hg. The patient has pale
Dr.HUR A. Salman - OdNMU
skin, petechial rash on the extremities, enlarged lymph nodes. Blood test results: Hb - 80 g/l,
RBC - 2,2x1012/l; WBC - 3,5x109/l; blasts - 52%; eosinophils - 2%; stab neutrophils - 3%;
segmented neutrophils - 19%; lymphocytes - 13%; monocytes - 1%; platelets – 35x109/l. ESR - 47
mm/h. What test is required to specify the diagnosis?
A Immunophenotyping → is a technique used to study the protein expressed by cells. This
technique is commonly used in basic science research and laboratory diagnostic purpose. This can
be done on tissue section (fresh or fixed tissue), cell suspension, etc. An example is the detection
of tumor marker, such as in the diagnosis of leukemia.
B Protein electrophoresis
C Lymph node biopsy
D Determination of anti-platelet antibody titer
E Cytogenetic study

272-A 47-year-old male patient complains of compressive chest pain that occurs both at rest and
during light physical activity; irregular heartbeat. These problems arose 3 months ago. The
patient's brother died suddenly at the age of 30. Objectively: Ps - 84/min, arrhythmic, AP -
130/80 mm Hg. ECG confirms signs of left ventricular hypertrophy, abnormal Q-waves in V4-V6
leads. EchoCG reveals that interventricular septum is 1,7 cm, left ventricular wall thickness is
1,2 cm. What is the most likely diagnosis?
A Hypertrophic cardiomyopathy
B Neurocirculatory asthenia
C Exertional angina
D Myocarditis
E Pericarditis
Dr.HUR A. Salman - OdNMU

273- A 38-year-old male patient complains of marked dyspnea that escalates with physical
exertion.
The problems, namely acute chest pain on the left and cough, arose unexpectedly 2 hours
before at work. The pain abated, but there were progressing dyspnea, dizziness, pallor, cold
sweat, cyanosis. Auscultation reveals the absence of vesicular breath sounds, radiograph
shows a shadow on the left. What pathology can be suspected?
A Left-sided spontaneous pneumothorax
B Pulmonary infarction
C Pleurisy
D Left-sided pneumonia
E Lung abscess

274-A 67-year-old male patient complains of rash, severe pain in the subscapular region on the
right.Objectively: skin in the right subscapular region is covered with linearly arranged pink-red
edematous lesions that are somewhat infiltrated, and have clear boundaries. On the lesion
surface there are vesicles with transparent exudate. What is the most likely diagnosis?
A Herpes zoster
B Duhring dermatitis
C Erysipelas
D Atopic dermatitis
E Impetigo

275-A 64-year-old male patient has a 35-year history of chronic pancreatitis. In the last 5 years, he
claims to observe the pain abatement, bloating, frequent bowel movements up to 3-4 times a
day, grayish, glossy stool with undigested food rests, the progressive loss of body weight.
Change of symptoms in the patient is due to overlay of:
A Exocrine pancreatic insufficiency → enzymes deficiency
B Endocrine pancreatic insufficiency
C Lactase deficiency syndrome
D Irritable bowel syndrome
E Chronic enterocolitis

276-During the doctor's round, a 56-year-old male patient with decompensated cirrhosis
complains of dizziness, palpitations, moving black specks seen before the eyes, general weakness.
The patient is pale, Ps- 110/min, AP- 90/50 mm Hg. What complication is most likely to have
occurred in the patient?
A Bleeding from esophageal varices
Dr.HUR A. Salman - OdNMU
B Hepatocellular insufficiency
C Hepatic encephalopathy
D Acute coronary syndrome
E Paroxysmal tachycardia

277- 5 days before, a 26-year-old female patient developed an acute condition. Objectively: marked
headache, vomiting, weakness, poor appetite, temperature up to 39oC. Objectively: the
patient is in a moderately grave condition, excited. The face is hyperemic, sclerae are injected.
The tongue is coated with brown fur. The trunk and limbs are covered with plentiful roseolous
and petechial rash. Hepatosplenomegaly is present. Complement binding reaction with
Rickettsia prowazekii is positive with the titer of 1:640. What drug should be administered?
A Doxycycline→ epidemic tuyphus.
B Chloramphenicol
C Penicillin
D Streptomycin
E Metronidazole

278-A 39-year-old female patient complains of dyspnea when walking, palpitation, edemata in the
evening. The patient's height is 164 cm, weight - 104 kg. Objectively: overnutrition. Heart sounds
are weak, and tachycardia is present. The menstrual cycle is not broken. Blood sugar is 5,6
mmol/l, ACTH-response tests revealed no alterations. X-ray of the Turkish saddle revealed no
pathology. What disease is it?
A Alimentary obesity
B Climax
C Pituitary obesity
D Diabetes mellitus
E Cushing's syndrome (primary hypercortisolism)

I LIKE FOOD
Dr.HUR A. Salman - OdNMU

279-A 26-year-old male patient complains of a rash on the upper lip skin, which arose on a
background of influenza with high-grade fever and is accompanied by pain and burning. The
rash has been present for 3 days. Objectively: the skin of the upper lip is edematic and
erythematous, grouped vesicles are filled with serous fluid and have a rough surface. What is
the most likely diagnosis?
A Herpetic vesicular dermatitis
B Eczema
C Contact dermatitis
D Dermatitis herpetiformis
E Erythema multiforme

280-A 15-year-old patient consulted a dermatologist about a painful lump in the armpit.
Objectively:there is a walnut-sized node, lymphadenitis, infiltration of the surrounding tissues. The
patienthas been diagnosed with hidradenitis. What is the most likely causative agent of this
disease?
A Staphylococci
B Streptococci
C Proteus vulgaris
D Pseudomonas aeruginosa
E Mixed infection

281-A 36-year-old female patient complains of intense pain in the knee joints and neck. In the
morning she experiences pain in the interscapular region and leg joints; pain subsides after
warm-up gymnastics. The patient is overnourished, there is a clicking sound in the knees when
squatting, the knees are somewhat disfigured, painful on palpation. Blood test results: ESR- 18
mm/h, WBC- 8,0*109/l. Radiography reveals subchondral sclerosis in the left knee.
What is the basis of this pathology?
A Degenerative processes in cartilage
B Autoimmune process in the synovium
C Deposition of urates (tophi) in the articular tissues
Dr.HUR A. Salman - OdNMU
D Beta-haemolytic streptococcus
E Hemarthrosis

282-A 33-year-old female complains of escalating spastic pain in the abdomen after the
psycho-emotional stress. The patient has intermittent bowel movements, that is 2-3 bowel
movements after waking up alternate with constipation lasting for 1-2 days. Objectively: body
weight is unchanged, there is moderate pain on palpation of the sigmoid colon. Hb- 130 g/l,
WBC- 5,2*109/l, ESR- 9 mm/h. Proctosigmoidoscopy causes pain due to spastic bowel
condition, intestinal mucosa is not changed. In the lumen there is a lot of mucus. What is the
most likely diagnosis?
A Irritable bowel syndrome → Review Q 7
B Crohn's disease
C Non-specific ulcerative colitis
D Acute bowel ischemia
E Malabsorption syndrome

283- After a holiday in the Crimea, a 36-year-old female patient presents with severe pain in the
elbow joints, dyspnea and weakness. The body temperature is of 37,6oC, the skin is pale,
there is erythema of cheeks and nose, lower lip ulceration. Visual inspection reveals no
changes in the joints, the right elbow movement is limited. There is murmur and pleural friction in
the lungs below the right angle of the scapula. Cardiac sounds are muffled, there is tachycardia,
gallop rhythm, Ps- 114/min. AP- 100/60. What is the most likely diagnosis?
A SLE
B Rheumatic heart disease
C Rheumatoid arthritis
D Infectious allergic myocarditis
E Dry pleurisy
Dr.HUR A. Salman - OdNMU

284- A 63-year-old male patient complains of cough with expectoration of mucous blood-streaked
sputum, asthma, low-grade fever, general weakness. These presentations have been observed
for 3 months. The patient has been a smoker since childhood. Objectively: to- 37,4oC,
respiratory rate is 26/min, Ps- 82/min, rhythmic. AP- 130/85 mm Hg. There is limited breathing
movement in the right side of chest cavity, as well as percussive dullness and diminished breath
sounds. Radiograph shows a homogeneous opacity of the pulmonary field on the right with the
mediastinum displacement to the affected side. What is the most likely diagnosis?

A Central lung cancer → Review Q 130


B Pleural effusion
C Pleuropneumonia
D Pulmonary tuberculosis
E Bronchiectasis

285-A 33-year-old male patient developed a condition that had a stormy clinical course: chills,
fever up to 39oC, vomiting, epigastric pain, diarrhea with watery smelly feces. 6 hours before, he
ate a raw egg, fried potatoes with stewed meat, drank some juice. What pathogen is likely to
have caused this condition?
A Salmonella
B Colibacillus
C Campylobacter
D Shigella
E Vibrio cholerae

286-A 53-year-old male has been admitted to a hospital for an attack of renal colic which has
repeatedly occurred throughout the year. Objectively: in the region of auricles and the right
elbow some nodules can be seen that are covered with thin shiny skin. Ps- 88/min, AP- 170/100
mm Hg. There is bilateral costovertebral angle tenderness (positive Pasternatsky's symptom).
The patient has been scheduled for examination. What laboratory value would be most helpful
for making a diagnosis?
A Uric acid
B Rheumatoid factor
C ESR
D Urine sediment
E Lactic acid

287-A 38-year-old male complains of tonic tension of the masticatory muscles, so that he cannot
open his mouth. 12 days before, he was bitten by an unknown dog. Objectively: there is
pronounced tension and twitching of the masticatory muscles. What is the most likely diagnosis?
A Tetanus
B Rabies
C Hysteria
D Trigeminal neuralgia
E Apyretic tetanus
Dr.HUR A. Salman - OdNMU

288-2 weeks after having quinsy, a 26-year-old male patient got facial edemata, moderate pain in
the sacrum. Objectively: body temperature is 37,5oC, AP- 100/80 mm Hg. Urinalysis results:
RBC- up to 100 fresh cells in per HPF, protein - 2,2 g/l, hyaline cylinders - up to 10 per HPF,
relative density - 1002. What is the most likely diagnosis?
A Acute glomerulonephritis
B Nephroma
C Acute pyelonephritis
D Urolithiasis
E Chronic glomerulonephritis

289-Two years ago, a 46-year-old patient was diagnosed with stage I silicosis. Currently the patient
complains of escalating dyspnea, pain in the infrascapular regions. Radiograph shows a diffuse
enhancement and distortion of lung markings, as well as multiple nodular shadows 2-4 mm in
diameter. There is interlobar pleural density on the right. Dense shadows are found in the hilar
regions. Specify the form of radiographic pulmonary fibrosis in this case:
A Nodular
B Interstitial
C Interstitial nodular
D Nodal
E Tumor-like
Dr.HUR A. Salman - OdNMU
290- A 39-year-old male patient complains of moderate pain and weakness in the shoulder, back
andpelvic girdle muscles, that has been progressing for the last 3 weeks; great difficulty in getting
out of bed, going up and down the stairs, shaving. Dermatomyositis has been suspected. Blood
test results: Hb- 114 g/l, WBC- 10,8*109/l, eosinophils - 9%, ESR -22 mm/h, C-reactive
protein - (++). The alteration in the following laboratory value wil be of decisive diagnostic
significance:
A Creatine phosphokinase
B Ceruloplasmin
C Sialic acids
D dsDNA antibodies
E Gamma-globulins

In many patients, the first sign of dermatomyositis is the presence of a symptomless, itchy or
burning rash. Cutaneous involvement may manifest as follows:
• Reddish or bluish-purple patches, on photo-exposed surfaces
• Purple eyelids, which are described as heliotrope, as they resemble the heliotrope flower.
• Eruption on the dorsal hands, particularly over the knuckles, which are known as Gottron
papules
Dr.HUR A. Salman - OdNMU
• Ragged cuticles and prominent blood vessels on nail folds
• The rash may also affect cheeks, nose, shoulders, upper chest and elbows
• Scaly scalp or diffuse hair loss
Less commonly there is poikiloderma i.e. the skin is atrophic (pale, thin skin), red (dilated blood
vessels) and brown (post-inflammatory pigmentation)
Muscle involvement manifests as the following:
• Proximal muscle weakness
• Muscle fatigue/weakness when climbing stairs, walking, rising from a seated position, combing
hair, or reaching for items above shoulders
• Difficulty swallowing (dysphagia)
• Muscle tenderness: May occur, but not a typical feature of dermatomyositis

291-A 42-year-old male patient wth essential hypertension presents with headache, palpitations,
unexplained fear. Objectively: Ps- 100/min, AP- 200/100 mm Hg, the left border of cardiac
dullness is displaced by 1,5 cm to the left, vesicular breathing is present. ECG shows sinus
tachycardia, signs of left ventricular hypertrophy. What drug should be administered as an
emergency?
A Obzidan → Propranolol hydrochloride .. non-selective β blocker used to treat high blood
pressure, a number of types of irregular heart rate, thyrotoxicosis, capillary
hemangiomas, performance anxiety, and essential tremors
B Dibazol
C Reserpine
D Magnesium sulfate
E Furosemide

292-A 65-year-old male patient complains of dyspnea that is getting worse with exertion, morning
cough with expectoration of mucous sputum. For about 15 years, he has been subject to regular
medical check-up for chronic bronchitis. The patient takes berodual (16 inhaled doses per day).
Objectively: body temperature is 36,8oC, RR- 24/min, Ps- 110/min, AP- 145/90 mm Hg.
Auscultation reveals a lot of dry rales above the lungs. FEV1- 65%. What is the optimal tactics
of further management of the patient?
A To administer inhalation corticosteroids
B To administer antibiotics
C To administer theophylline
D To increase the daily dose of berodual
E To include short-acting β2-agonists in the therapy

293-A 49-year-old countryman got an itching papule on the dorsum of his right hand. In the centre
there is a vesicle with serosanginous exudate. Within the next 2 days the patient developed a
painless edema of hand and forearm. On the 4th day the temperature rose to 38,5oC, in the
right axillary region a large painful lymph node was found. One day before the onset of the
disease the patient had examined a dead calf. What is the most likely diagnosis?
A Cutaneous anthrax
B Bubonic plague
C Carbuncle
D Lymphocutaneous tularemia
E Erysipelas
Dr.HUR A. Salman - OdNMU

294-A 38-year-old female suddenly developed acute inflammatory rash in form of roseolas,
papules, vesicles that are scattered on the skin of trunk in irregular and predominantly focal
manner. The rash appeared a few hours after visiting a restaurant. The patient complains of itching
skin. What is the most likely diagnosis?
A Toxicodermatosis → an inflammation of the skin caused by a toxic substance like food or
material .
B Atopic dermatitis
C Contact dermatitis
D Eczema
E-

295-A 41-year-old male patient was delivered to a hospital unconscious. During the previous 7
days he had been taking large doses of biseptolum for a cold. The night before, he began
complaining of dyspnea, especially when lying down, swollen legs, 2-day urinary retention. In the
morning he had seizures and lost consciousness. Objctively: noisy breathing at the rate of 30/min,
edematous legs and lumbar region, Ps- 50/min. Plasma creatinine is 0,586 mmol/l, plasma
potassium - 7,2 mmol/l. What treatment is necessary for this patient?
A Hemodialysis
B Large doses of verospiron
C Plasma volume expanders
D Glucocorticosteroids
E Heparin

296- A 24-year-old male patient had been diagnosed with class III diffuse toxic goiter. There is
moderate hyperthyroidism. A surgery was suggested, and the patient agreed to it. What
preoperative measures should be taken for prevention of thyrotoxic crisis in the postoperative
period?
A Administration of antithyroid drugs
B Minimally invasive surgical techniques
C Bed rest
D Detoxification therapy
E Administration of corticosteroids
Dr.HUR A. Salman - OdNMU
297- A 26-year-old male patient complains of pain in the right knee, which is getting worse in the
morning. Two weeks before, he consulted an urologist about prostatitis. Objectively:
conjunctivitis is present. There is also periarticular edema of the knee joint, redness of the
overlying skin. Rheumatoid factor was not detected. Until further diagnosis is specified, it would
be reasonable to start treatment with the following antibiotic:
A Tetracyclines → pt. have signs of reactive arthritis
B Cephalosporins
C Penicillins
D Aminoglycosides
E Lincosamides

298- For 3 days, a 28-year-old emale patient had had the body temperature increase up to
$38^oC$, weakness, poor appetite, nausea, a single vomiting. On the 4th day the temperature was
normal, the condition improved, but the jaundice developed. Objectively: moderate ictericity of
skin, +3 cm enlarged liver of elastic consistency. Ortner's, Kehr's and Voznesensky's symptoms
are negative. What test will verify the diagnosis?
A IgM Anti-HAV detection → Acute Hepatitis A infection .
B Complete blood count
C Ultrasound of the abdomen
D Total bilirubin
E AST activity

299- A 50-year-old male in a grave condition has been admitted to the intensive care unit. It is
known from life history that the patient works in agriculture, and 3 hours ago was engaged into
insecticide treatment of crops for control of colorado potato beetle. Condition on admission:
acrocyanosis, bronchorrhea, tachypnea, AP- 100/60 mm Hg, Ps- 44/min. What method of
efferent therapy would be most appropriate at this stage?
A Hemosorbtion
B Hemodialysis
C Plasmapheresis
D Lymphosorption
E Plasma dialysis
Dr.HUR A. Salman - OdNMU
300
A 46-year-old male patient complains of periodic epigastric pain that occurs at night. Objectively:
HR- 70/min, AP- 125/75 mm Hg, tenderness in the epigastric region is present. EGD confirms
duodenal ulcer of 0,6 cm in diameter. Test for H. Pylori is positive. Which of the given
antisecretory drugs will be a compulsory element of the treatment regimen?
A Omeprazole → proton pump inhibitor .
B Famotidine
C Pirenzepine
D Atropine
E Maalox

301-A 34-year-old male visited Tajikistan. After return, he complains of fever up to 40oC which
occurs every second day and is accompanied by chills, sweating. Hepatosplenomegaly is
present. Blood test results: RBC- 3*1012/l, Нb- 80 g/l, WBC- 4*109/l, eosinophils - 1%, stab
neutrophils - 5%, segmented neutrophils - 60%, lymphocytes - 24%, monocytes - 10%, ESR - 25
mm/h. What is the provisional diagnosis?
A Malaria
B Infectious mononucleosis
C Sepsis
D Typhoid fever
E Leptospirosis

302- A 49-year-old female patient has type 1 diabetes of moderate severity. The disease is
complicated by retinopathy and polyneuropathy. Besides that, repeated analyses of the daily
urinary excretion of albumin revealed microalbuminuria (200-300 mg/day). Glomerular filtration
rate is 105 ml/min. Blood pressure is within normal range. Normalization of the following
indicator should be the first-priority task in the secondary prevention of diabetic nephropathy:
A Glycosylated hemoglobin → HbA1c
B C-peptide
C Blood insulin
D Fasting glucose
E Glycemia 2 hours after a meal

303- A 21-year-old female patient has been hospitalized on an emergency basis because of severe
dyspnea, pain in the left side of chest. Body temperature is 38,8oC. The condition developed three
days ago. Respiratory rate is 42/min, auscultation reveals shallow breathing. There is percussive
dullness on the right starting from the middle of the blade, breath sounds cannot be heard. The left
border of heart is 3 cm displaced outwards. Embryocardia is present, HR is 110/min. The right
hypochondrium is painful on palpation. What urgent therapeutic measures should be taken in this
situation?
A Emergency puncture of the pleural cavity
B Administration of penicillin antibiotics
C Injection of Lasix
D Injection of cardiac glycosides
E Transferring the patient to the thoracic surgery department

304- After having the flu, a 39-year-old male patient with a history of Addison's disease developed
anbcondition manifested by weakness, depression, nausea, vomiting, diarrhea, hypoglycemia. AP-
75/50 mm Hg. Blood test results: low corticosterone and cortisol, 13-oxycorticosteroids,
17-oxycorticosteroids levels. What condition developed in the patient?
A Acute adrenal insufficiency
Dr.HUR A. Salman - OdNMU
B Acute gastritis
C Acute enterocolitis
D Collapse
E Diabetes mellitus

305- A 41-year-old patient cosulted a dermatologist about discoloration, thickening, brittleness of


toenails. These symptoms have been present for about five years. Objectively: nail plates in all
toes are thickened, of dirty yellow color, lustreless, crumble over the edge. Microscopy of the
nail plate material treated with alkali revealed mycelial filaments. Material inoculation onto
Sabouraud medium resulted in growth of the Trichophyton rubrum colony. What is the most likely
diagnosis?
A Rubromycosis of toenails
B Candidal onychia
C Psoriasis of the nails
D Nail dystrophy
E Epidermophytosis of nails

306
A 29-year-old female patient complains of dyspnea and palpitations on exertion. According to
her mother, as a child she had heart murmur, did not undergo any examinations. Objectively: the
patient has pale skin, Ps- 94/min, rhythmic. AP- 120/60 mm Hg. In the II intercostal space on
the left auscultation reveals a continuous rasping systolodiastolic murmur, diastolic shock above
the pulmonary artery. Blood and urine are unremarkable. What is the most likely diagnosis?
A Patent ductus arteriosus → PDA
B Atrial septal defect
C Ventricular septal defect
D Aortarctia
E Tetralogy of Fallot
Dr.HUR A. Salman - OdNMU
307
A 67-year-old female patient with hypertensive crisis has asthma, cough with expectoration of
frothy pink sputum, moist rales in the lungs. The patient stays in sitting position, respiratory rate
is 40/min, AP- 214/136 mm Hg, heart rate - 102/min. What is the most rational tactics of this
patient management?
A Intravenous administration of furosemide
B Urgent pneumography
C Bed rest, lying position
D Intravenous administration of a β-blocker
E Tactics can be determined after ECG and chest radiography

308
A 49-year-old male patient complains of retrosternal pain, heartburn, weight loss of 8 kg over
the last year, constipation, weakness. The patient has been a smoker for 20 years, and has a
10-year history of gastroesophageal reflux disease. The patient is asthenic, has dry skin. EGD
revealed an ulcer in the lower third of the esophagus and esophageal stricture accompanied by
edema, hyperemia and multiple erosions of the mucosa. What study is required for more
accurate diagnosis?
A Biopsy of the esophageal mucosa
B X-ray examination of the esophagus
C Respiratory test for Helicobacter pylori
D pH-metry of the esophagus and the stomach
E Fecal occult blood test

309
During an exam, a 22-year-old female student fainted. She grew up in a family with many
children, has a history of frequent acute respiratory infections. Objectively: the patient has pale
skin and mucous membranes, split-end hair, brittle nails. Blood test results: RBC-
2,7*1012/l, Hb- 75 g/l, color index - 0,7, WBC- 3,2*109/l, platelets – 210*109/l, ESR- 30 mm/h.
Blood serum iron is 6 mmol/l. What is the most likely diagnosis?
A Iron-deficiency anemia
B Acute leukemia
C $B_{12}$-deficiency anemia
D Vegetative-vascular dystonia
E Aplastic anemia

310
During the ultrasound study of carotid and vertebral arteries a 74-year-old patient developed a
condition manifested by dizziness, weakness, nausea, transient loss of consciousness.
Objectively: pale skin, AP- 80/60 mm Hg, Ps- 96/min of poor volume. ECG shows sinus
tachycardia, left ventricular hypertrophy. Focal neurological symptoms were not found. What is
the provisional diagnosis?
A Carotid sinus syncope
B Orthostatic syncope
C Morgagni-Adams-Stokes attack
D Complete atrioventricular block
E Acute cerebrovascular accident
Dr.HUR A. Salman - OdNMU

311
After a holiday in the Crimea, a 49-year-old male patient with a history of lung tuberculosis felt
increased weakness, periodic dizziness, easing bowel movements with abdominal pain, the
need for additional salting his meals. The patient has noted that his condition improves after
some sweet tea and validol taken sublingually. Objectively: there is an intense darkening of skin,
AP- 70/50 mm Hg, glycemia is 3,0 mmol/l. What is the possible cause of health deterioration:
A Chronic adrenal insufficiency
B Diabetes mellitus
C Coronary artery disease
D Chronic pancreatitis
E Pulmonary tuberculosis
Dr.HUR A. Salman - OdNMU

HELPFUL INFORMATION : JUST FOR NOTE


\
The list of biochemical and hematological tests used for the nutritional status
assessment (obligatory)

Normal concentration
Indices
(according to the CI)
Glucose in blood 3.89 – 6.1 mmol/l
in urine 0.72 mmol/day
Whole protein in blood serum 60 – 78 g/l
Albumin 0.494 – 0.86 mmol/l
Triglicerides 0.59 – 1.77 mmol/l
Alkaline phosphatase activity 0.5 – 1.3 mmol/(hourl)
Vitamin С in blood 34.1 – 90.9 μmol/l
in urine 113.5 – 170.3 μmol/day
Vitamin В2 in urine 0.82 – 2.73 μmol/day
Vitamin А in blood serum 0.52 – 2.09 mmol/l
Fine blood smear (blood formula)
Additional methods of research
Total nitrogen in daily urine 423.4 – 1213 mmol/day
Urea in daily urine 333 – 583 mmol/day
0.0 – 0.314 mmol/day – for males
Creatinine in daily urine
7.1 – 15.9 mmol/day – for females
Blood creatine 53 – 106.1 μmol/l
40 – 48 % - for males
Packed cell volume, PCV
36 – 42 % - for females
Total cholesterol in blood serum 2.97 – 8.79 mmol/l
Cholesterol fractions 2.97 – 8.79 mmol/l
Pyruvic acid in blood 0.034 – 0.102 mmol/l
Lactic acid in blood 0.33 – 2.22 mmol/l
Ketone bodies in urine 861 μmol/day
Inorganic phsphorus in blood serum 0.65 – 1.29 mmol/l
Calcium in blood serum 1.03 – 1. 27 mmol/l
Iron in blood serum 11.6 – 31.3 μmol/l
Hemoglobin 1.86 – 2.79 mmol/l
Pyridoxine according to the N-methyl-
51.1 – 87.6 μmol/day
nicotinamide content in urine
Dr.HUR A. Salman - OdNMU
Dr.HUR A. Salman - OdNMU
Dr.HUR A. Salman - OdNMU
Dr.HUR A. Salman - OdNMU
Dr.HUR A. Salman - OdNMU
Dr.HUR A. Salman - OdNMU
REALLY SORRY FOR LATE
I KNOW THE TIME ALREADY FINISHED
HOPE SUCCESS FOR ALL
SORRY FOR MISTAKES AND OLD INFORMATION
IF YOU READ THIS > MY RESPECT TO YOU
Dr.H.A.Salman - OdNMU

KROK REVIEW
- First of all
I’d like to thank all the sources , sites , books and persons that I used their material for this preparation
, and I know it is wrong that I used a lot of pictures under copy right without permission , therefore;
I’m asking apologize from all that resources and persons whom made that efforts for humanity
please anyone (sites and persons) object on this , don’t hesitate to contact me , directly I will remove
his work with pleasure and respecting his rights .
Unfortunately I don’t have enough place and time to mention you all ,
Here I’m repeating my thanks and appreciation for serving medicine and health overall the world .
- DEDICATION:
I dedicate this simple works for all humanity, asking to stop wars , destruction and killing people and
living Lovely , Peacefully with happiness and as one human being , we are all same source , beginning
and same end just differ in between , life like 2 brackets ( ) one begin other one end , so try to fill
inbetween
these brackets with humanity , love , charity , saving the world .
- Thanks God , and all who supported me .
- How to prepare Krok test
krok is MCQ based exam (200Q/4h ) in clinical subjects ( Internal , surgery , pediatrics , Gyne & OB)
with other subjects like Hygiene & nutrition , psychiatry , social medicine , occupational disease ,
infectious disease , pediatrics surgery )
this exam based on translated questions from Russian to English therefore; a lot of mistakes and
nonusable
English words , some non medical terms as well as printing mistakes.
- If we ask any foreign students about this exam , he will answer better to cancel this exam from
protocol of international faculty , because it’s never assess the student knowledge , the information
that used in test old , non updated diseases and old protocols and based on SSRI period.
- Unfortunately very old information make student confused with the international parameters,
procedures and new guidelines and modalities.
- Another thing why it is just one attempt ?? and you should repeat all the course again on account of
some miserable questions ??
So the best solution for this problem is to the exam selective as before , obligatory for Ukrainian
citizen and selective for foreign , because no benefit of this exam
Or 2nd option to change the discipline to make real international not just on papers
- May be this information will not change anything , but that notes for next courses
- About IFOM exam , should not be applied unless they change the syllabus and remove all that old
books , procedures , examination , and all protocols , old medical terms , and unusable words , as well
as the combination Latin roots that they make it , as they want to make it formal exam , it is not like
krok just keep without understand , it is hard exam and depend on updated infromations
- Actually students can Help to update books and protocols , espcailly the INTERNATIONAL
- So what student should do in this case , study the old questions and keep it without understanding
unfortunately especially for Hygiene , non medical questions , why doctor should keep dimension of
room or how to calculate air ventilation and which place should build hospital , this subject should be
selective , doctor after graduate work with patient and need to learn examination diagnosis of disease
and treatment better than learn numbers and old protocols
Dr.H.A.Salman - OdNMU
- So I suggest on you to analyze the questions use the following steps :
Dr.H.A.Salman - OdNMU

1- for medical Q try to find signs or symptoms that related to the disease , whatever was diagnosis or
treatment and tactic
- 2- for non-understandable Q try to use Excluding the options , because a lot of Q they used 4 options
so far from the answer and the last one will the correct whatever it is right information or wrong .
- 3- use KEY way to match the Q with the answer , if clinical or other
- Sometimes there is more than one right answer but you should choose the more specific one
for ex give you information about extrauterine pregnancy and cervical pathology and they put in the
answer – ectopic pregnancy and cervical pregnancy ,, both of them right logically but you should
vhosse the cervical pregnancy it is more specific .
- For Hygiene most Q not understandable , some of them keep it , others if you translate it you can
understand , others you find the answer in the Q same words but not always , so I think just go a head
and do the easiest way you like .
- Note: All information here based on Ukrainian information and protocols , so try to keep this
information temporarily because it differ completely from your state exam or another country ..
- I advice you don’t use this information furthermore due to most of these information not based on
clinical and practical measurement , most of it theoretical and hypothetical information .
- Before Finally I wish all pass safely and wish all best of luck , hoping to see all best doctors and have
a good futures , caring all people , saving the humanity , especially children and poor people and all
who on need , repairing the world which destroyed by wars.
- Remember KROK does’t make you a DOCTOR !
And never assess you
Please Don’t hesitate to contact me for adding some information or correct anything .
If you feel some information not well explained or there is any comments , please I’m listening and I will
correct it again and re share that information
I will be happy with your feedback
Finally if you find this file good and can help others , please share it
All answers is A with
Yellow color : the answers
Green color : key words of Q
Blue color : this is the most important words in Q (additional Keys)
Grey color : this is my note and additional info
Red color : this repeated Q or high-light
Violant color : important notes

Yours
Dr. H.A.SALMAN
OdNMU
7/6/2020
E-mail: hur.amer@gmail.com
F/b: Hr Salman
Mobile + Net : +964-790-489-6865
Dr.H.A.Salman - OdNMU

Krok 2 – 2014 Gynecology-OB Base


1. A 52-year-old woman suffering from obesity, complains of bloody discharges from sexual
paths during 4 days. Last normal menses were 2 years ago. Histological investigation of
biopsy of the endometrium has revealed adenomatous hyperplasia. What reason from
the mentioned below caused the development of disease?
A. Excessive transformation of preandrogens from adipose tissues → due to
obesity ( adipose tissue ) preandrogen will transform to androgen then to oestrogen causing that
symptoms amenorrhea and adenomatous hyperplasia of endometrium which causes of bloody
discharge.
B. The increased contents of follicle-stimulating hormone
C. Supersecretion of androgens by the cortex of paranephroses.
D. Hypersecretion of estrogens by tissues of the organism.
E. Poor aromatization of preandrogens due to hypothyroidism

2. A 40-year-old woman complains of colic pains in the lower part of abdomen and
abundant bloody discharges from genital tract. Last 2 years she had menses for 15-16
days, abundant, with clots, painful. Had 2 medical abortions. In bimanual investigation:
from the canal of the cervix uteri - a fibromatous node, 3 cm in diameter, on the thin
stem. Discharges are bloody, moderate. Choose the correct tactics.
A. Operation: untwisting of born node.  surgery to untwisted fibroid (node) it
suppose after untwisting to remove it . as we felt it during bimanual exam that mean it is
pudendated and twisited due to blood and clot.
B. Phase by phase vitamin therapy
C. Hysterectomy without ovaries
D. Supravaginal ablation of the uterus without ovaries
E. Hormonal hemostasis
Dr.H.A.Salman - OdNMU

on the Rt. Side .


3. A 40-year-old woman complains of yellow color discharges from the vagina. Bimanual
examination: no pathological changes. Smear test: Trichomonas vaginalis and mixed
flora. Colposcopy: two hazy fields on the front labium, with a negative Iodum probing.
What is your tactics?
A. Treatment of specific colpitis with the subsequent biopsy  colpitis = vaginitis
(colpa latin = vagina) and from discharge color yellow mean infection why biopsy check
below ↓↓
B. Cervix ectomy
C. Cryolysis of cervix uteri
D. Diathermocoagulation of the cervix uteri
E. Specific treatment of Trichomonas colpitis

Why biopsy ?
Because Negative iodum probing
(staining)

Schiller's test or Schiller's Iodine


test is a medical test in which iodine
solution is applied to the cervix in order
to diagnose cervical cancer
Schiller's iodine solution is applied
to the cervix under direct vision. Normal
cervical mucosa contains glycogen and
stains brown, whereas abnormal areas,
such as early cervical cancer, do not take
up the stain.
Lugol's iodine (Potassium Iodide)
can be used as an alternative.
Dr.H.A.Salman - OdNMU

4. A 32 y.o. woman consulted a gynecologist about having abundant long menses within 3
months. Bimanual investigation: the body of the uterus is enlarged according to about 12
weeks of pregnancy, distorted, tuberous, of dense consistence. Appendages are not
palpated. Histological test of the uterus body mucosa: adenocystous hyperplasia of
endometrium. Optimal medical tactics:
A. Surgical treatment
B. Radial therapy
C. Phase by phase vitamin therapy
D. Hormonetherapy
E. Phytotherapy  the treatment of medical conditions using plants, or substances made
from plants
Note:
Hysterectomy is indicated in women not wanting to preserve their fertility when
(i) progression to atypical hyperplasia occurs during follow-up, or
(ii) there is no histological regression of hyperplasia despite 12 months of treatment, or
(iii) there is relapse of endometrial hyperplasia after completing progestogen treatment, or
(iv) there is persistence of bleeding symptoms, or
(v) the woman declines to undergo endometrial surveillance or comply with medical treatment
Here according to Histopathology adenocystous End.Hyp. considered it as atypia which gives
indication for surgery , also Bulky uterus ( enlarged about 12week pregnancy ), distorded,
tuberous, dense, in addition to 3 months abundant long menses as a bleeding which also
another indication of surgical treatment . as appendages not palpated that’s mean it is normal
and not need for surgery, don’t forget the pt. is young 32y.
Dr.H.A.Salman - OdNMU

5. A woman complains of having slight dark bloody discharges and mild pains in the lower
part of abdomen for several days. Last menses were 7 weeks ago. The pregnancy test is
positive. Bimanual investigation: the body of the uterus indicates for about 5-6 weeks of
pregnancy, it is soft, painless. In the left appendage there is a retort-like formation, 7х5
cm large, mobile, painless. What examination is necessary for detection of fetus
localization?
A. Ultrasound  safe and non-invasive technique for diagnosis of the pregnancy, through
it we can now the number of gestational sac – viability of fetus – heart pulsation and later
on movement of fetus, also site of placenta and amount of liquor ( fetus fluid) also we can
assess the cervical ineteral os and cerival competency in additions to another parameters
we can check.
B. Colposcopy  vaginal scope
C. Cystoscopy  urinary bladder scope
D. Hysteroscopy  uterus scope - contraindicated during pregnancy
E. Hromohydrotubation  Hydrotubation is the trans-cervical flushing of fluid through the
fallopian tubes, commonly used in combination with laparoscopy for diagnostic purposes to
determine the tubal patency in infertility investigation. Hydrotubation-only procedure can be
done in women that refuse diagnostic laparoscopy

pict. 1 Pict. 2
Picture 1 = normal 7 weeks gestational sac and embryonic pole inside with head and body and
may have heart beat
picture 2 = Lt. adnexa have cystic structure called corpus luteum which the remenant of the
Graffian follicle and produce progesterone during early pregnancy which necessary to the fetus
till the placenta take the role .
6. A woman was hospitalised with full-term pregnancy. Examination: the uterus is tender,
the abdomen is tense, cardiac tones of the fetus are not auscultated. What is the most
probable complication of pregnancy?
A. Premature detachment of normally posed placenta  Placental abruption,
resulted in abd. Pain and fetal heart not heard may indicate a bleeding and fetal
compromised to due detached placenta.
Dr.H.A.Salman - OdNMU

B. Premature labor
C. Acute hypoxia of a fetus
D. Hydramnion
E. Back occipital presentation

7. By the end of the 1st period of physiological labor clear amniotic fluid came off.
Contractions lasted 35-40 sec every 4-5min. Heartbeat of the fetus was 100 bpm. The BP
was 140/90 mm Hg. What is the most probable diagnosis?
A. Acute hypoxia of the fetus decreased HR, increased BP
B. Premature labor
C. Back occipital presentation
D. Hydramnion
E. Premature detachment of normally posed placenta

8. A pregnant woman in her 40th week of pregnancy undergoes obstetric examination: the
cervix of uterus is undeveloped. The oxytocin test is negative. Examination at 32 weeks
revealed: AP 140/90 mm Hg, proteinuria 1 g/l, peripheral edemata. Reflexes are normal.
Choose the most correct tactics:
A. Labour stimulation after preparation 
one of the important indication of Labour
induction is Hypertensive disorder during
pregnancy and this pt. have hx. At exam.
32 wk. also she have pictures goes with
preeclampsia
2nd thing she is almost reaching full term
(4oth week)
B. Complex therapy of gestosis for 2 days
C. Absolute bed rest for 1 month
D. Caesarian section immediately
E. Complex therapy of gestosis for 7 days
Dr.H.A.Salman - OdNMU

9. Which gestational age gives the most accurate estimation of weeks of pregnancy by
uterine size?
A. Less that 12 weeks  through measuring fundal heigh.
B. Between 21 and 30 weeks
C. Between 12 and 20 weeks
D. Between 31 and 40 weeks
E. Over 40 weeks

10. A 26 year old woman had the second labour within the last 2 years with oxytocin
application. The childs weight is 4080 g. After the placent birth there were massive
bleeding, signs of hemorrhagic shock. Despite the injection of contractive agents, good
contraction of the uterus and absence of any cervical and vaginal disorders, the bleeding
proceeds. Choose the most probable cause of bleeding:
A. Atony of the uterus  fetal wight large which is risk of atony , bleeding contioud after
injection of contractive agent .
B. Hypotonia of the uterus
C. Hysterorrhexis
D. Injury of cervix of the uterus
E. Delay of the part of placenta

11. A woman is admitted to maternity home with discontinued labor activity and slight
bloody discharges from vagina. The condition is severe, the skin is pale, consciousness is
confused. BP is 80/40 mm Hg. Heartbeat of the fetus is not heard. There was a Cesarian
section a year ago. Could you please determine the diagnosis?
A. Hysterorrhesis  = rupture of uterus (laceration) , the pt. have risk factor of
previous C/S the discontinuation of labor activity and bleeding with signs of shock gives
picture of uterine rupture.
B. Cord presentation
C. Expulsion of the mucous plug from cervix uteri
D. Premature expulsion of amniotic fluid
E. Placental presentation
Dr.H.A.Salman - OdNMU

12. On the first day after labour a woman had the rise of temperature up to 39°C. Rupture of
fetal membranes took place 36 hours before labour. Examination of the bacterial flora of
cervix of the uterus revealed hemocatheretic streptococcus of A group. The uterus body
is soft, tender. Discharges are bloody, with admixtures of pus. Specify the most probable
postnatal complication:
A. Metroendometritis  infection of uterus with endometrium by bacterial
flora reveal Group A SC with pus, common after delivery and due to long
premature rupture of membrane .
B. Infective contamination of the urinary system
C. Apostasis of sutures after the episiotomy
D. Thrombophlebitis of veins of the pelvis
E. Infectious hematoma

13. Rise in temperature up to 39°C was registered the next day after a woman had labor.
Fetal membranes rupture took place 36 hours prior to labors. The examination of the
bacterial flora of cervix uteri revealed the following: haemolytic streptococcus of group
A. The uterus tissue is soft, tender. Discharges are bloody, with mixing of pus. Establish
the most probable postnatal complication.
A. Metroendometritis  same question above
B. Infective contamination of the urinary system
C. Apostatis of stitches after the episiotomy
D. Thrombophlebitis of veins of the pelvis
E. Infected hematoma

14. A woman of a high-risk group (chronic pyelonephritis in anamnesis) had vaginal


delivery. The day after labour she complained of fever and loin pains, frequent urodynia.
Specify the most probable complication:
A. Infectious contamination of the urinary system  in the history pt. have
Chronic PN also due to hx of vaginal delivery , compilcation happened
specifically UTI (urodynia = pain accompanying urination)
B. Infectious hematoma
C. Apostasis of sutures after episiotomy
D. Endometritis
E. Thrombophlebitis of veins of the pelvis

15. 13 months after the first labor a 24-year-old patient complained of amenorrhea.
Pregnancy ended in Caesarian section because of premature detachment of normally
positioned placenta which resulted in blood loss at the rate of 2000 ml owing to
disturbance of blood clotting. Choose the most suitable investigation:
Dr.H.A.Salman - OdNMU

A. Estimation of gonadotropin rate the 1st symptom of pregnancy , therefore


here there is suspected pregnant βhCG (β human chorionic gonadotropin)
indicated to confirm the presence of pregnancy .
B. Progesteron assay
C. USI of small pelvis organs
D. Computer tomography of head
E. Estimation of testosteron rate in blood serum

16. In 13 months after the first labor a 24-year-old woman complains of amenorrhea.
Cesarian section was conducted as a result of premature detachment of normally posed
placenta. Hemorrhage has made low fidelity of 2000 ml due to breakdown of
coagulation of blood. Choose the most suitable investigation.
A. Determination of the level of gonadotropin  same Q above
B. Progesteron test
C. Determination of the contents of testosteron-depotum in blood serum.
D. Computer tomography of the head
E. Ultrasound of organs of a small pelvis

17. In the woman of 24 years about earlier normal menstrual function, cycles became
irregular, according to tests of function diagnostics - anovulatory. The contents of
prolactin in blood is boosted. Choose the most suitable investigation:
A. Computer tomography of the head here we need to check the Pituitary
gland due to hyperprolactinemia and this may lead to micro adenoma of the
pituitary gland which can be detected by brain tomography imaging.
in fact the CT scan have less benefit to diagnose pituitary Microadenoma ,
MRI preferred with dynamic study ( to trace the contrast enhancing during
the imaging )
B. USI of organs of small pelvis
C. Determination of the contents of testosteron-depotum in blood serum
D. Progesterone assay
E. Determination of the level of gonadotropins
Dr.H.A.Salman - OdNMU

18.A woman in her 39th week of pregnancy, the second labour, has regular birth activity.
Uterine contractions take place every 3 minutes. What criteria describe the beginning of
the II labor stage the most precisely?
A. Cervical dilatation by no less than 4 cm
B. Cervical smoothing over 90%
C. Presenting part is in the lower region of small pelvis
D. Rupture of fetal bladder
E. Duration of uterine contractions over 30 seconds

19. A 20-year-old woman is having timed labor continued for 4 hours. Light amniotic fluid
came off. The fetus head is pressed to the orifice in the small pelvis. The anticipated fetus
mass is 4000,0 gpm 200,0 g. Heartbeat of the fetus is normal. Intrinsic examination:
cervix is absent, disclosure – 2 cm, the fetal membranes are not present. The head is in 1-
st plane of the pelvis, a sagittal suture is in the left slanting dimension. What is the
purpose of glucose-calcium-hormone - vitaminized background conduction?
A. Prophylaxes of weakness of labor activity above mentioned symptoms of
weak labor activity , therefore need some prophylaxis preparations.
B. Fetus hypoxia prophylaxes
C. Treatment of weakness of labor activity.
D. Antenatal preparation
E. Labor stimulation
Dr.H.A.Salman - OdNMU

20. A 24 years old primipara was hospitalised with complaints about discharge of the
amniotic waters. The uterus is tonic on palpation. The position of the fetus is
longitudinal, it is pressed with the head to pelvic outlet. Palpitation of the fetus is
rhythmical, 140 bpm, auscultated on the left below the navel. Internal examination:
cervix of the uterus is 2,5 cm long, dense, the external os is closed, light amniotic waters
out of it. Point a correct component of the diagnosis:
A. Antenatal discharge of the amniotic waters this is soviet term (premature
rupture of membrane )
(PROM) is a rupture (breaking open) of the membranes (amniotic sac) before labor begins. If
PROM occurs before 37 weeks of pregnancy, it is called preterm premature rupture of
membranes (PPROM).
B. Early discharge of the amniotic waters
C. The end of the 1st stage of labour
D. Pathological preterm labour
E. The beginning of the 1st stage of labour

21. A 34 y.o. woman in her 29-th week of pregnancy, that is her 4-th labor to come, was
admitted to the obstetric department with complaints of sudden and painful bloody
discharges from vagina that appeared 2 hours ago. The discharges are profuse and
contain grumes. Cardiac funnction of the fetus is rhytmic, 150 strokes in the minute,
uterus tone is normal. The most probable provisional diagnosis will be:
A. Placental presentation  grume :mean thick, clotted like : grumous blood,
bloody discharge give signs of placenta position
B. Disseminated intravascular coagulation syndrome
C. Vasa previa
D. Detachment of normally located placenta
E. Bloody discharges
Dr.H.A.Salman - OdNMU

22. A 29 year old patient underwent surgical treatment because of the benign serous
epithelial tumour of an ovary. The postoperative period has elapsed without
complications. What is it necessary to prescribe for the rehabilitational period:
A. Hormonotherapy and proteolytic enzymes this treatment usually used in
Cancer like ovarian Ca , also hormone therapy using in recurrent of tumor
after surgery
anyhow for your info. Just keep this question because it is best option 
B. Magnitotherapy and vitamin therapy
C. The patient does not require further care
D. Antibacterial therapy and adaptogens
E. Lasertherapy and enzymotherapy
Benign serous tumours are loculated, have a single layer of flattened or cuboidal epithelium
and the absence of mitoses. Papillae are sometimes present on the external or internal surfaces.
Low grade
The "low grade" classification of serous tumors includes benign and borderline tumors, as well as low grade
malignant tumors. Benign serous tumors are distinguished from borderline tumors by the absence of cellular
stratification. Stromal invasion distinguishes borderline tumors from low grade malignant tumors.[1] Surgery is
curative for benign tumors, and likely curative for other low grade tumors.

- Benign serous tumors include


- serous cystadenomas,
- cystadenofibromas, and
- adenofibromas

High grade
High grade serous tumors often involve both ovaries. The tumors are solid and cystic
with haemorrhage and necrosis. They are morphologically heterogenous. Serous carcinomas often have
bulky peritoneal and omental metastases, and spread to the lymph nodes is frequent.

23. A 34-year-old woman with 10-week pregnancy (the second pregnancy) has consulted
gynaecologist to take a record in patient chart. There was a hydramnion previous
pregnancy, the birth weight of a child was 4086 g. What tests are necessary first of all?
A. The test for tolerance to glucose  signs of gestational diabetes : increased
fetal liquor (amnion)- hydroamnion also big fetal size / weight ( large for
gestational age)
B. Fetus cardiophonography
C. Ultrasound of the fetus
D. Determination of the contents of alpha fetoprotein
E. Bacteriological test of discharge from the vagina
glucose is given and blood samples taken afterward to determine how quickly it is cleared from the blood. The
test is usually used to test for diabetes, insulin resistance, impaired beta cell function, and sometimes reactive
hypoglycemia and acromegaly, or rarer disorders of carbohydrate metabolism.
= oral glucose tolerance test (OGTT),
Dr.H.A.Salman - OdNMU

24. A 26 y.o. woman complains of sudden pains in the bottom of abdomen irradiating to the
anus, nausea, giddiness, bloody dark discharges from sexual tracts for one week, the
delay of menses for 4 weeks. Signs of the peritoneum irritation are positive. Bimanual
examination: borders of the uterus body and its appendages are not determined because
of sharp painfullness. The diverticulum and painfullness of the back and dextral fornixes
of the vagina are evident. What is the most probable diagnosis?
A. Broken tubal pregnancy  = ruptured uterine tube due to ectopic tubul
pregnancy. Signs of pregnant (no mens 4 week), bloody discharge, +ve
peritoneal irritation sign . why ectopic , uterus not palpated
B. Acute right-side adnexitis
C. Apoplexy of the ovary
D. Torsion of the crus of the ovary tumour
E. Acute appendicitis

25. At the gynaecological department there is a patient of 32 years with the diagnosis:
"acute bartholinitis". Body temperature is 38,2°C, leucocytes count 10,4×109/L, the ESR
is 24 mm/hour. In the area of big gland of the vestibulum - the dermahemia, the sign of
the fluctuation, sharp tenderness (pain). What is the most correct tactics of the doctor?
A. Surgical dissecting, a drainage of an abscess of the gland, antibiotics due to
abscess formation we need surgery and drian the abscess and we have to give
antibiotics after as a complementray treatment and preventive.
Be aware from E ( no antibiotic)
B. Antibiotics, Sulfanilamidums
C. Antibiotic therapy
D. Antibiotics, detoxication and biostimulants.
E. Surgical dissection, drainage of the abscess of the gland

Bartholinitis is inflammation of one or both of the


two Bartholin's glands, which are located one on
either side of the opening of the vagina, behind the
labia (lips). The inflammation is sometimes due to
germs picked up during sex, but in many cases the
inflammation is not sexually transmitted

Bartholinitis Fiboisis  duct get blocked


exudate pent up inside  Abscess

26. A primagravida with pregnancy of 37-38 weeks complains of headache, nausea, pain in
epigastrium. Objective: the skin is acyanotic. Face is hydropic, there is short fibrillar
twitching of blepharons, muscles of the face and the inferior extremities. The look is
fixed. AP- 200/110 mm Hg; sphygmus of 92 bpm, intense. Respiration rate is 32/min.
Dr.H.A.Salman - OdNMU

Heart activity is rhythmical. Appreciable edemata of the inferior extremities are present.
Urine is cloudy. What medication should be administered?
A. Droperidolum of 0,25% - 2,0 ml  old medication is an antidopaminergic drug used as
an antiemetic (that is, to prevent or treat nausea) and as an antipsychotic. Droperidol is also often
used as a sedative in intensive-care treatment.
For this case patient have Pre-eclampsia / eclampsia ( pregnant + muscle
spasm + hypertension + edema)
B. Dibazolum of 1% - 6,0 ml
C. Hexenalum of 1% - 2,0 ml
D. Pentaminum of 5% - 4,0 ml
E. Papaverine hydrochloride of 2% - 4,0 ml

Sphygmic = of or relating to the circulatory pulse = Heart Rate


Another thing about Droperidol :
side effect: Dysphoria, sedation, hypotension resulting from peripheral alpha adrenoceptor
blockade, prolongation of QT interval which can lead to torsades de pointes, and extrapyramidal
side effects such as dystonic reactions/neuroleptic malignant syndrome
Indications
Need in neuroleptanalgesia, psychomotor excitement in the postoperative period, poisoning,
preparing patients to narcosis, to surgical interferences, hallucinations, hypertonic crises,
traumas, myocardial infarction, shock.

27. An onset of severe preeclampsia at 16 weeks gestation might be caused by:


A. Hydatidiform mole
B. Anencephaly
C. Maternal renal disease
D. Interventricular defect of the fetus
E. Twin gestation
Dr.H.A.Salman - OdNMU

28. A woman had the rise of temperature up to 39°C on the first day after labour. The
rupture of fetal membranes took place 36 hours before labour. The investigation of the
bacterial flora of cervix of the uterus revealed hemocatheretic streptococcus of group A.
The uterus body is soft, tender. Discharges are bloody, mixed with pus. Specify the most
probable postnatal complication:
A. Metroendometritis  repeated same Q 12
B. Infection of the urinary system
C. Apostatis of junctures after the episiotomy
D. Thrombophlebitis of pelvic veins
E. Infected hematoma

29. A 24 y.o. patient 13 months after the first labour consulted a doctor about amenorrhea.
Pregnancy has concluded by a Cesarean section concerning to a premature detachment
of normally posed placenta hemorrhage has made low fidelity 2000 ml owing to
breakdown of coagulability of blood. Choose the most suitable investigation:
A. Determination of the level of Gonadotropins repeated same Q 15
B. Progesteron assay
C. USI of organs of a small pelvis
D. Computer tomography of the head
E. Determination of the contents of Testosteron-Depotum in Serum of blood

30. A 34 year old woman in the 10th week of gestation (the second pregnancy) consulted a
doctor of antenatal clinic in order to be registered there. In the previous pregnancy
hydramnion was observed, the childs birth weight was 4086 g. What examination
method should be applied in the first place?
A. The test for tolerance to glucose repeated same Q 23
B. US of fetus
C. Bacteriological examination of discharges from vagina
D. Determination of the contents of fetoproteinum
E. A cardiophonography of fetus

31. A 10 y.o. boy was ill with angina 2 weeks ago, has complaints of joint pain and stiffness of
his left knee and right elbow. There was fever (38,50) and ankle disfunction,
enlargement of cardiac dullness by 2 cm, tachycardia, weakness of the 1st sound, gallop
rhythm, weak systolic murmur near apex. What diagnosis corresponds with such
symptoms?
A. Acute rheumatic fever
B. Reiters disease
C. Reactive arthritis
D. Systemic lupus erythematosis
E. Juvenile rheumatoid arthritis
Dr.H.A.Salman - OdNMU

32. The disease began acutely. The frequent watery stool developed 6 hours ago. The bodys
temperature is normal. Then the vomiting was joined. On examination: his voice is
hoarse, eyes are deeply sunken in the orbits. The pulse is frequent. Blood pressure is low.
There is no urine. What is the preliminary diagnosis?
A. Cholera
B. Typhoid fever
C. Salmonellosis
D. Toxic food-borne infection
E. Dysentery
Dr.H.A.Salman - OdNMU

33. At term of a gestation of 40 weeks height of standing of a uterine fundus is less then
assumed for the given term. The woman has given birth to the child in weight of 2500 g,
a length of a body 53 cm, with an assessment on a scale of Apgar of 4-6 points. Labor
were fast. The cause of such state of the child were:
A. Chronic fetoplacental insufficiency
B. Delay of an intra-uterine fetation
C. Infection of a fetus
D. Prematurity
E. Placental detachment

34. A pregnant woman may be diagnosed with hepatitis if it is confirmed by the presence
of elevated:
A. SGOT (ALT)  one of the liver’s function enzymes.
increased ALT indicated one of the followings :
 extensive liver damage from toxins like alcohol or over-the-counter painkillers
 acute hepatitis
 gallbladder disease
 cancer
 in pregnant women, preeclampsia or HELLP syndrome, which is defined by its
characteristics — hemolysis, elevated liver enzymes, and low platelet count.
B. WBCs
C. Sedimentation rates
D. Alkaline phosphatase
E. BUN

35. A woman, aged 40, primigravida, with infertility in the medical history, on the 42-43
week of pregnancy. Labour activity is weak. Longitudinal presentation of the fetus, I
position, anterior position. The head of the fetus is engaged to pelvic inlet. Fetus heart
rate is 140 bmp, rhythmic, muffled. Cervix dilation is 4 cm. On amnioscopy: greenish
colour of amniotic fluid and fetal membranes. Cranial bones are dense, cranial sutures
and small fontanel are diminished. What should be tactics of delivery?
Dr.H.A.Salman - OdNMU

A. Caesarean section  a lot of factors indicated for C/S full term gestation ,
labour activity weak , No cervical dilation, rupture of amniotic fluid also age
of pt.,and Primigravia ( first pregnancy).
B. Medication sleep, amniotomy, labour stimulation
C. Fetal hypoxia treatment, in the II period - forceps delivery
D. Amniotomy, labour stimulation, fetal hypoxia treatment
E. Fetal hypoxia treatment, conservative delivery

36. An endometrial adenocarcinoma that has extended to the uterine serosa would be
classified as stage:
A. IIIA
B. IIA
C. IC
D. IIB
E. IVAB

37. Which of the methods of examination is the most informative in the diagnostics of a
tube infertility?
A. Laparoscopy with chromosalpingoscopy  laparoscopy is performed
for diagnostic or therapeutic purposes in women with infertility, chromopertubation (instillation
of dye through the fallopian tubes) is often performed to assess tubal patency.
B. Hysterosalpingography
C. Pertubation
Dr.H.A.Salman - OdNMU

D. Transvaginal echography
E. Bicontrast pelviography

38. A pregnant woman (35 weeks), aged 25, was admitted to the hospital because of bloody
discharges. In her medical history there were two artificial abortions. In a period of 28-
32 weeks there was noted the onset of hemorrhage and USD showed a placental
presentation. The uterus is in normotonus, the fetus position is transversal (Ist position).
The heartbeats is clear, rhythmical, 140 bpm. What is the further tactics of the pregnant
woman care?
A. To perform a delivery by means of Cesarean section  above causes all
indicated to do C/S.
B. To introduct the drugs to increase the blood coagulation and continue observation
C. To keep the intensity of hemorrhage under observation and after the bleeding is
controlled to prolong the pregnancy
D. Stimulate the delivery by intravenous introduction of oxytocin
E. To perform the hemotransfusion and to prolong the pregnancy
USD = it is not dollar  , they mean Ultrasonography (ultrasound examination).
39. A woman, primagravida, consults a gynecologist on 05.03.2012. A week ago she felt
the fetus movements for the first time. Last menstruation was on 10.01.2012. When
should she be given maternity leave?
A. 8 August
B. 22 August
C. 5 September
D. 11 July
E. 25 July

+7d / - 3m / +1 y
Dr.H.A.Salman - OdNMU

The Naegele's formula is simple arithmetic method for calculating the EDD (estimated date of
delivery) based on the LMP (last menstrual period). To the date of the first day of the LMP (e.g.
22nd June 2008): add seven days (i.e 29th) subtract 3 months (i.e March)
Be aware from LMP months in case of ( January – February – March) we don’t add 1 to year
because the delivery will be in the same year
40. Condition of a parturient woman has been good for 2 hours after live birth: uterus is
thick, globe-shaped , its bottom is at the level of umbilicus, bleeding is absent. The clamp
put on the umbilical cord remains at the same level, when the woman takes a deep
breath or she is being pressed over the symphysis with the verge of hand, the umbilical
cord drows into the vagina. Bloody discharges from the sexual tracts are absent. What is
the doctors further tactics?
A. To do manual removal of afterbirth  that mean the placenta still inside
when we press the cord goes inside , one of the last stage of delivery
procedure to evacuate the uterus from the placenta and cord by pull it out
manually to prevent retained product of conception.
B. To do curettage of uterine cavity
C. To introduct oxitocine intravenously
D. To apply Abduladze method
E. To apply Credes method

41. The woman who has delivered twins has early postnatal hypotonic uterine bleeding
reached 1,5% of her bodyweight. The bleeding is going on. Conservative methods to
arrest the bleeding have been found ineffective. The conditions of patient are pale skin,
acrocyanosis, oliguria. The woman is confused. The pulse is 130 bpm, BP– 75/50 mm
Hg. What is the further treatment?
A. Uterine extirpation
B. Uterine vessels ligation
C. Putting clamps on the uterine cervix
D. Inner glomal artery ligation
E. Supravaginal uterine amputation
Extirpation = mean removal of uterus
= Hysterectomy  types of
hysterectomy ( sub-total, total , and
Radical)
Dr.H.A.Salman - OdNMU

42. A 26 y.o. woman complains of a mild bloody discharge from the vagina and pain in the
lower abdomen. She has had the last menstruation 3,5 months ago. The pulse is 80 bpm.
The blood pressure (BP) is 110/60 mm Hg and body temperature is 36,6°C. The
abdomen is tender in the lower parts. The uterus is enlarged up to 12 weeks of gestation.
What is your diagnosis?
A. Inevitable abortion  in this Q there is wrong information regarding
inevitable abortion because the important note are not mentioned which is
cervical is opened. See down
B. Incomplete abortion  no tissue piece were discharged + blood – cervix should dilated
C. Disfunctional bleeding  they mean dysfunction bleeding and this not in pregnancy.
D. Complete abortion  cervix full dilation 10mm and full component of gestation out.
E. Incipient abortion  initial stage of abortion
Inevitable abortion is an early pregnancy with vaginal bleeding and dilatation of the
cervix. Typically, the vaginal bleeding is worse than with a threatened abortion, and more
cramping is present. No tissue has passed yet

43. 18 y.o. woman complains of pain in the lower abdomen. Some minutes before she has
suddenly appeared unconscious at home. The patient had no menses within last 3
months. On examination: pale skin, the pulse- 110 bpm, BP- 80/60 mm Hg. The
Schyotkins sign is positive. Hb- 76 g/L. The vaginal examination: the uterus is a little bit
enlarged, its displacement is painful. There is also any lateral swelling of indistinct size.
The posterior fornix of the vagina is tendern and overhangs inside. What is the most
probable diagnosis?
A. Impaired extrauterine pregnancy  no menses gives indicate pregnancy,
and uterus little enlarged gives indicate that gestational sac extrauterine.
B. Acute appendicitis
C. Twist of cystoma of right uterine adnexa
D. Ovarian apoplexy
E. Acute salpingoophoritis
Dr.H.A.Salman - OdNMU

Blumberg's sign (also referred to as rebound tenderness, Shyotkin-Blumberg sign) is a clinical sign
that is elicited during physical examination of a patient's abdomen by a doctor or other health care
provider. It is indicative of peritonitis.

44. A 20 y.o. pregnant woman with 36 weeks of gestation was admitted to the obstetrical
hospital with complains of pain in the lower abdomen and bloody vaginal discharge. The
general condition of the patient is good. Her blood pressure is 120/80 mm Hg. The heart
rate of the fetus is 140 bpm, rhythmic. Vaginal examination: the cervix of the uterus is
formed and closed. The discharge from vagina is bloody up to 200 ml per day. The head
of the fetus is located high above the minor pelvis entry. A soft formation was defined
through the anterior fornix of the vagina. What is the probable diagnosis?
A. Placental presentation  same Q 21
B. Premature placental separation
C. Threatened premature labor
D. Incipient abortion
E. Uterine rupture

45. In the gynecologic office a 28 y.o. woman complains of sterility within three years. The
menstrual function is not impaired. There were one artificial abortion and chronic
salpingo-oophoritis in her case history. Oral contraceptives were not used. Her husbands
analysis of semen is without pathology. What diagnostic method will you start from the
workup in this case of sterility?
A. Hysterosalpingography
B. Ultra sound investigation
C. Hysteroscopia
D. Diagnostic scraping out of the uterine cavity
E. Hormone investigation

46. A 28-year-old patient underwent endometrectomy as a result of incomplete abortion.


Blood loss was at the rate of 900 ml. It was necessary to start hemotransfusion. After
transfusion of 60 ml of erythrocytic mass the patient presented with lumbar pain and
fever which resulted in hemotransfusion stoppage. 20 minutes later the patients
condition got worse: she developed adynamia, apparent skin pallor, acrocyanosis,
Dr.H.A.Salman - OdNMU

profuse perspiration. to- 38,5°C, Ps- 110/min, AP- 70/40 mm Hg. What is the most likely
diagnosis?
A. Hemotransfusion shock  patient got shock after blood transfusion .
B. DIC syndrome
C. Septic shock
D. Hemorrhagic shock
E. Anaphylactic shock

47. A 58-year-old female patient came to the antenatal clinic complaining of bloody light-
red discharges from the genital tracts. Menopause is 12 years. Gynaecological
examination revealed age involution of externalia and vagina; uterine cervix was
unchanged, there were scant bloody discharges from uterine cervix, uterus was of
normal size; uterine appendages were not palpable; parametria were free. What is the
most likely diagnosis?
A. Uterine carcinoma this is non – specific symptoms of uterine ca except
cervix in unchanged therefore it is not cervical ca it is uterine ca, another
sign the age of patient ,and involution of genitalia , bloody discharge,
menopause.
B. Cervical carcinoma
C. Granulosa cell tumor of ovary
D. Atrophic colpitis.
E. Abnormalities of menstrual cycle of climacteric nature

48. The results of a separate diagnostic curettage of the mucous of the uterus cervix and
body made up in connection with bleeding in a postmenopausal period: the scrape of the
mucous of the cervical canal revealed no pathology, in endometrium - the highly
differentiated adenocarcinoma was found. Metastases are not found. What method of
treatment is the most correct?
A. Surgical treatment and hormonotherapy due to no metastasis the protocol
will be removing of the tumor then hormone therapy .
Patient Menopause:
For this standard just keep it because this is wide variety of protocols.
B. Surgical treatment and radial therapy
C. Surgical treatment + chemotherapy
D. Radial therapy

49. A 27 y.o. woman complains of having the disorders of menstrual function for 3
months, irregular pains in abdomen. On bimanual examination: in the dextral
appendage range of uterus there is an elastic spherical formation, painless, 7 cm in
diameter. USI: in the right ovary - a fluid formation, 4 cm in diameter, unicameral,
smooth. What method of treatment is the most preferable?
Dr.H.A.Salman - OdNMU

A. Prescription of an estrogen-gestogen complex for 3 months with repeated


examination 
Estrogen and Progestin (Hormone Replacement Therapy)
combination therapy, this form of HT combines doses of estrogen and
progesterone (progestin is a synthetic form of progesterone). This type of HT
is used if a woman still has her uterus.
- gestogen = progestogen
- Dextra appendage = Right Adnexa
- USI = ultrasound investigation
- Unicameral = single cavity

\in brief : this patient have menstrual disorders 3 months due to Rt. Ovarian
cyst  treatment give combined oral contraceptives (COC )

B. Dispensary observation of the patient


C. Chemotherapeutic treatment
D. Anti-inflammatory therapy
E. Operative treatment

50. A 40 year old patient complains of yellowish discharges from the vagina. Bimanual
examination revealed no pathological changes. The smear contains Trichomonas
vaginalis and blended flora. Colposcopy revealed two hazy fields on the frontal labium,
with a negative Iodine test. Your tactics:
A. Treatment of specific colpitis and subsequent biopsy  repeated Q 3
B. Cervix ectomy
C. Cryolysis of cervix of the uterus
D. Diathermocoagulation of the cervix of the uterus
E. Specific treatment of Trichomonas colpitis
Colpitis = vaginitis
51. A 26-year-old secundipara at 40 weeks of gestation arrived at the maternity ward after
the beginning of labor activity. 2 hours before, bursting of waters occurred. The fetus was
in a longitudinal lie with cephalic presentation. Abdominal circumference was 100 cm,
fundal height - 42 cm. Contractions occurred every 4-5 minutes and lasted 25 seconds
each. Internal obstetric examination revealed cervical effacement, opening by 4 cm. Fetal
bladder was absent. Fetal head was pressed against the pelvic inlet. What complication
arose in childbirth?
Dr.H.A.Salman - OdNMU

A. Early amniorrhea Be aware this is not amenorrhea ( absence of


mensturation ) this term latin – combination of amnion ( liquor = fetal fluid)
+ suffix – orrhea which mean discharge
Early amniorrhea they mean early rupture of amniotic membrane
Be aware this is not PROM = premature rupture of membrane which is
happened at 37 weeks
B. Secondary uterine inertia
C. Clinically narrow pelvis
D. Discoordinated labor
E. Primary uterine inertia

52. A 28-year-old parturient complains about headache, vision impairment, psychic


inhibition. Objectively: AP- 200/110 mm Hg, evident edemata of legs and anterior
abdominal wall. Fetus head is in the area of small pelvis. Fetal heartbeats is clear,
rhythmic, 190/min. Internal examination revealed complete cervical dilatation, fetus
head was in the area of small pelvis. What tactics of labor management should be
chosen?
A. Forceps operation
B. Cesarean
C. Conservative labor management with episiotomy
D. Stimulation of labor activity
E. Embryotomy

53. A 28 year old woman had the second labour and born a girl with manifestations of
anemia and progressing jaundice. The childs weight was 3 400 g, the length was 52 cm.
The womans blood group is B (III) Rh-, the fathers blood group is A (III) Rh+, the childs
blood group is B (III) Rh+. What is the cause of anemia?
A. Rhesus incompatibility
B. Antigen A incompatibility
C. Antigen AB incompatibility
D. Intrauterine infection
Dr.H.A.Salman - OdNMU

E. Antigen B incompatibility

54. A 48 year old female patient complains about contact haemorrhage. Speculum
examination revealed hypertrophy of uterus cervix. It resembles of cauliflower, it is
dense and can be easily injured. Bimanual examination revealed that fornices were
shortened, uterine body was nonmobile. What is the most probable diagnosis?
A. Cervical carcinoma
B. Metrofibroma
C. Cervical pregnancy
D. Cervical papillomatosis
E. Endometriosis

55. A 37 y.o. primigravida woman has been having labor activity for 10 hours. Labor pains
last for 20-25 seconds every 6-7 minutes. The fetus lies in longitude, presentation is
cephalic, head is pressed upon the entrance to the small pelvis. Vaginal examination
results: cervix of uterus is up to 1 cm long, lets 2 transverse fingers in. Fetal bladder is
absent. What is the most probable diagnosis?
Dr.H.A.Salman - OdNMU

A. Primary uterine inertia


B. Discoordinated labor activity
C. Pathological preliminary period
D. Secondary uterine inertia
E. Normal labor activity
the failure of the myometrium to ever establish
a contractile pattern adequate to expel neonates
from the uterus

56. Laparotomy was performed to a 54 y.o. woman on account of big formation in pelvis
that turned out to be one-sided ovarian tumor along with considerable omental
metastases. The most appropriate intraoperative tactics involves:
A. Ablation of omentum, uterus and both ovaries with tubes
B. Biopsy of omentum
C. Ablation of an ovary and omental metastases
D. Ablation of omentum and both ovaries with tubes
E. Biopsy of an ovary

57.A parturient complains about pain in the mammary gland. Palpation revealed a 3×4 cm
large infiltration, soft in the centre. Body temperature is 38,5°C. What is the most
probable diagnosis?
A. Acute purulent mastitis  pain – infiltration and fever signs of mastitis
B. Pleuritis
C. Birth trauma
D. Retention of milk
E. Pneumonia
Dr.H.A.Salman - OdNMU

58. A 43 y.o. patient complains of formation and pain in the right mammary gland, rise of
temperature up to 37,2°C during the last 3 months. Condition worsens before the
menstruation. On examination: edema of the right breast, hyperemia, retracted nipple.
Unclear painful infiltration is palpated in the lower quadrants. What is the most
probable diagnosis?
A. Cancer of the right mammary gland
B. Tuberculosis of the right mammary gland
C. Right-side chronic mastitis
D. Right-side acute mastitis
E. Premenstrual syndrome

59. A 14 year old girl complains of profuse bloody discharges from genital tracts during 10
days after suppresion of menses for 1,5 month. Similiar bleedings recur since 12 years on
the background of disordered menstrual cycle. On rectal examination: no pathology of
the internal genitalia. In blood: Hb - 70 g/l, RBC- 2,3×1012/l, Ht - 20. What is the most
probable diagnosis?
A. Juvenile bleeding, posthemorrhagic anemia this is
Abnormal uterine bleeding (AUB) in adolescents is defined as excessive bleeding
occurring between menarche and 19 years of age. During the first 12–18 months
afterthe onset of menstruation
Dr.H.A.Salman - OdNMU

B. Polycyst ovarian syndrome


C. Werlholfs disease
D. Hormonoproductive ovary tumor
E. Incomplete spontaneous abortion

60. A 33-year-old woman was urgently brought to clinic with complaints of the pain in the
lower part of the abdomen, mostly on the right, irradiating to rectum, she also felt dizzy.
The above mentioned complaints developed acutely at night. Last menses were 2 weeks
ago. On physical exam: the skin is pale, Ps - 92 bpm, t- 36,6°C, BP- 100/60 mm Hg. The
abdomen is tense, slightly tender in lower parts, peritoneal symptoms are slightly
positive. Hb- 98 g/L. What is the most probable diagnosis?
A. Apoplexy of the ovary  is a sudden rupture in the ovary, commonly at the site of a cyst,
accompanied by hemorrhage in the ovarian tissue and/or intraperitoneal bleeding
B. Renal colic
C. Intestinal obstruction
D. Acute appendicitis
E. Abdominal pregnancy

61. A secundipara has regular birth activity. Three years ago she had cesarean section for the
reason of acute intrauterine hypoxia. During parodynia she complains of extended pain
in the area of postsurgical scar. Objectively: fetus pulse is rhythmic - 140 bpm. Vaginal
examination shows 5 cm cervical dilatation. Fetal bladder is intact. What is the tactics of
choice?
A. Cesarean section  the previous C/S gives indicate for current C/S and pain
at scar site also , in addition to cervix not fully dilated (5cm) also give
indicate C/S.
B. Obstetrical forceps
C. Augmentation of labour
D. Waiting tactics of labor management
E. Vaginal delivery
Labor pain = parodynia
rhythmic uterine contractions that under normal conditions increase in intensity, frequenc
y, and duration, culminating in vaginal delivery of the infant.
62. A 54-year-old female patient consulted a doctor about bloody discharges from the
genital tracts after 2 years of amenorrhea. USI and bimanual examination revealed no
genital pathology. What is the tactics of choice?
Dr.H.A.Salman - OdNMU

A. Fractional biopsy of lining of uterus and uterine mucous membranes


patient age - post menopause, bloody discharge = do biopsy
B. Contracting drugs
C. Hysterectomy
D. Estrogenic haemostasia
E. Styptic drugs

63. Examination of a just born placenta reveals defect 2x3 cm large. Hemorrhage is absent.
What tactic is the most reasonable?
A. Manual uretus cavity revision  after placenta delivered at the end stage of
labor, we have to check the placenta for any defect which mean there is still
part of it retained inside uterus , here as no hemorrhage we can explore and
remove the remnant of placenta manually if present .
B. Prescription of uterotonic medicines
C. Parturient supervision
D. Instrumental uterus cavity revision
E. External uterus massage

64. A 27 y.o. gravida with 17 weeks of gestation was admitted to the hospital. There was a
history of 2 spontaneous miscarriages. On bimanual examination: uterus is enlarged to
17 weeks of gestation, uterus cervix is shortened, isthmus allows to pass the finger tip.
The diagnosis is isthmico-cervical insufficiency. What is the doctors tactics?
A. To place suture on the uterus cervix
B. To perform amniocentesis
C. To interrupt pregnancy
D. To administer tocolytic therapy
E. To administer hormonal treatment
Cervical weakness, also called cervical incompetence or cervical insufficiency, is a medical
condition of pregnancy in which the cervix begins to dilate (widen) and efface (thin) before the
pregnancy has reached term.
Dr.H.A.Salman - OdNMU

65. A 27-year-old woman presents at the maternity welfare centre because of infertility.
She has had sexual life in marriage for 4 years, doesnt use contraceptives. She hasnt get
pregnant. On examination: genital development is without pathology, uterine tubes are
passable, basal (rectal) temperature is one-phase during last 3 menstrual cycles. What is
the infertility cause?
A. Anovular menstrual cycle  means no ovulations
B. Abnormalities in genital development
C. Chronic adnexitis
D. Immunologic infertility
E. Genital endometriosis

66. A 43 y.o. woman complains of contact hemorrhages during the last 6 months.
Bimanual examination: cervix of the uterus is enlarged, its mobility is reduced. Mirrors
showed the following: cervix of the uterus is in the form of cauliflower. Chrobak and
Schiller tests are positive. What is the most probable diagnosis?
A. Cancer of cervix of the uterus  cauliflower is the key in krok
B. Cervical pregnancy
C. Leukoplakia
D. Nascent fibroid
E. Polypus of the cervis of the uterus
. Cervical Ca  Q 54
Chrobak and Schiller tests : explained at question Q3
67. A 26-year-old woman gave birth to a child 6 months ago. She applied to gynecologist
complaining of menstruation absence. The child is breast-fed. Vagina exam: uterus is of
normal form, dense consistence. What is the most probable diagnosis?
A. Physiological amenorrhea  when the puerperal
women on breastfeed no menses can happened due to
secretions of prolactin & oestrogen. In this case called
physiological & considered as natural contraceptive.
B. Sheehans syndrome= postpartum pituitary gland necrosis
C. Gestation
D. Pseudoamenorrhea
E. Ashermans syndrome = scar tissue (adhesions) form
inside the uterus and/or the cervix
Dr.H.A.Salman - OdNMU

68. A primagravida in her 20th week of gestation complains about pain in her lower
abdomen, blood smears from the genital tracts. The uterus has an increased tonus, the
patient feels the fetus movements. Bimanual examination revealed that the uterus size
corresponded the term of gestation, the uterine cervix was contracted down to 0,5 cm,
the external orifice was open by 2 cm. The discharges were bloody and smeary. What is
the most likely diagnosis?
A. Incipient abortion  = imminent abortion
An impending abortion characterised by copious vaginal bleeding, uterine
contractions, and cervical dilation.
= threatened abortion is defined as a painless vaginal bleeding occurring
before 24 weeks of pregnancy
B. Abortion in progress  abortion already started and cervical more dilated
C. Missed miscarriage fetal death inside uterus and abortion will happen
D. Incomplete abortion started and pass some tissue of conception
E. Risk of abortion have some factors related to fetus and mother may lead to abortion.

69. Full-term pregnancy. Body weight of the pregnant woman is 62 kg. The fetus has the
longitudinal position, the fetal head is pressed against the pelvic inlet. Abdominal
circumference is 100 cm. Fundal height is 35 cm. What is the approximate weight of the
fetus?
A. 3 kg 500 g  estimated fetal weight = Fundal height x Abdominal
circumference = 35 x 100 = 3500 g which is 3 kg + 500 g.
try another ex. AC = 110 , FH = 33  FW = 33 x 110 = 3630 g = 3 kg + 630 g.
B. 4 kg
C. 3 kg
D. 4 kg 500 g
E. 2 kg 500 g

70. A patient was admitted to the hospital with complaints of periodical pain in the lower
part of abdomen that gets worse during menses, weakness, malaise, nervousness, dark
bloody smears from vagina directly before and after menses. Bimanual examination
revealed that uterus body is enlarged, appendages cannot be palpated, posterior fornix
has tuberous surface. Laparoscopy revealed: ovaries, peritoneum of rectouterine pouch
and pararectal fat have "cyanotic eyes". What is the most probable diagnosis?
A. Disseminated form of endometriosis  because it reached outside uterus
and vaginal posterior fornix.
B. Chronic salpingitis
C. Ovarian cystoma
D. Tuberculosis of genital organs
E. Polycystic ovaries
Dr.H.A.Salman - OdNMU

71. A gravida with 7 weeks of gestation is referred for the artificial abortion. On operation
while dilating cervical canal with Hegar dilator No.8 a doctor suspected uterus
perforation. What is immediate doctors tactics to confirm the diagnosis?
A. Probing of uterus cavity
B. Ultrasound examination
C. Bimanual examination
D. Laparoscopy Indications: suspicion of submucosal
E. Metrosalpingography myoma, malformations of a uterus, genital
Hegar dilators infantility; to m it is shown at differentiation
of a tumor of a uterus and ovary, before a
scraping of walls of a uterus, at the
infertility caused by an atresia of the
channel of a neck of uterus
Diagrammatic representation of uterine
probing: the back wall of a vagina is
delayed by means of a mirror, the front lip
of a neck of uterus is taken by bullet
nippers, the probe is entered into the
cervical canal.

72. A pregnant woman in her 8th week was admitted to the hospital for artificial abortion.
In course of operation during dilatation of cervical canal of uterus by means of Hegars
dilator No.8 the doctor suspected uterus perforation. What is the immediate tactics for
confirmation of this diagnosis?
A. Uterine probing  see Q above 71
B. US examination
C. Metrosalpingography
D. Laparoscopy
E. Bimanual examination
Dr.H.A.Salman - OdNMU

73. A 59 year old female patient applied to a maternity welfare clinic and complained
about bloody discharges from the genital tracts. Postmenopause is 12 years. Vaginal
examination revealed that external genital organs had signs of age involution, uterus
cervix was not erosive, small amount of bloody discharges came from the cervical canal.
Uterus was of normal size, uterine appendages were unpalpable. Fornices were deep and
painless. What method should be applied for the diagnosis specification?
A. Separated diagnostic curettage 
B. Culdoscopy
C. Puncture of abdominal cavity through posterior vaginal fornix
D. Laparoscopy
E. Extensive colposcopy

74. A 25-year-old woman complains of profuse foamy vaginal discharges, foul, burning
and itching in genitalia region. She has been ill for a week. Extramarital sexual life. On
examination: hyperemia of vaginal mucous, bleeding on touching, foamy leucorrhea in
the urethral area. What is the most probable diagnosis?
A. Trichomonas colpitic
B. Bacterial vaginosis
C. Chlamydiosis
D. Gonorrhea
E. Vagina candidomicosis

Trichomonas colpitis
= Trichomonas vaginitis
Leucorrhea = white discharge
Dr.H.A.Salman - OdNMU

75.A 26 year old woman who delivered a child 7 months ago has been suffering from
nausea, morning vomiting, sleepiness for the last 2 weeks. She suckles the child,
menstruation is absent. She hasnt applied any contraceptives. What method should be
applied in order to specify her diagnosis?
A. Ultrasonic examination  the women have sypmtoms of pregnancy and
could be pregnant , ultrasound to check the presence of gestational sac
,although she is on breastfeeding (suckles the child) which considered as a
natural contraceptive but still there is a possibility to be pregnant .
B. Speculum examination
C. Palpation of mammary glands and pressing-out of colostrum
D. Roentgenography of small pelvis organs
E. Bimanual vaginal examination

76. A newborns head is of dolichocephalic shape, that is front-to-back elongated.


Examination of the occipital region revealed a labour tumour located in the middle
between the prefontanel and posterior fontanel. Specify the type of fetal presentation:
A. Posterior vertex presentation
B. Presentation of the bregma
C. Face presentation
D. Brow presentation
E. Anterior vertex presentation
Dr.H.A.Salman - OdNMU

77. A woman consulted a doctor on the 14th day after labour about sudden pain, hyperemy
and induration of the left mammary gland, body temperature rise up to 39°C, headache,
indisposition. Objectively: fissure of nipple, enlargement of the left mammary gland,
pain on palpation. What pathology would you think about in this case?
A. Lactational mastitis
B. Phlegmon of mammary gland
C. Fibrous adenoma of the left mammary gland
D. Lacteal cyst with suppuration
E. Breast cancer

78. A young woman applied to gynecologist due to her pregnancy of 4-5 weeks. The
pregnancy is desirable. Anamnesis stated that she had rheumatism in the childhood.
Now she has combined mitral heart disease with the priority of mitral valve deficiency.
When will she need the inpatient treatment (what periods of pregnancy)?
A. 8-12 weeks, 28–32 weeks, 37 weeks keep it . that’s it .
B. 12-16 weeks, 27-28 weeks, 37-38 weeks
C. 16 weeks, 34 weeks, 39-40 weeks
D. 6-7weeks, 16 weeks, 38 weeks
E. 10-12 weeks, 24 weeks, 37-38 weeks

79. A woman in the first half of pregnancy was brought to clinic by an ambulance. Term of
pregnancy is 36 weeks. She complains of intensive pain in the epigastrium, had vomiting
for 2 times. Pain started after the patient had eaten vinaigrette. Swelling of lower
extremities. BP - 140/100 mm Hg. Urine became curd after boiling. What is the most
probable diagnosis?
A. Preeclampsia
B. Food toxicoinfection
C. Exacerbation of pyelonephritis
D. Dropsy of pregnant women
E. Nephropathy of the 3rd degree
Dr.H.A.Salman - OdNMU

80. A 13 year old girl consulted the school doctor on account of moderate bloody discharge
from the genital tracts, which appeared 2 days ago. Secondary sexual characters are
developed. What is the most probable cause of bloody discharge?
A. Menarche is the first menstrual cycle, or first menstrual bleeding, in
female humans. See the age of girl and the symptoms.
B. Werlhofs disease
C. Haemophilia
D. Juvenile hemorrhage
E. Endometrium cancer

81.In 10 min after childbirth by a 22-year-old woman, the placenta was spontaneousely
delivered and 100 ml of blood came out. Woman weight - 80 kg, infant weight - 4100 g,
length - 53 cm. The uterus contracted. In 10 minutes the hemorrhage renewed and the
amount of blood constitued 300 ml. What amount of blood loss is permissible for this
woman?
A. 400 ml the total amount during the delivery of placenta and later
hemorrhge amount ( 100+300 = 400ml).
Ex. Amount of blood during placenta out 150ml and later 350 total will be
500ml.
B. 650 ml
C. 300 ml
D. 1000 ml
E. 500 ml

82. A pregnant woman was registered in a maternity welfare clinic in her 11th week of
pregnancy. She was being under observation during the whole term, the pregnancy
course was normal. What document must the doctor give the pregnant woman to
authorize her hospitalization in maternity hospital?
A. Exchange card = information female consultation about pregnancy.
Dr.H.A.Salman - OdNMU

B. Medical certificate
C. Sanitary certificate
D. Appointment card for hospitalization
E. Individual prenatal record

83. After examination a 46-year-old patient was diagnosed with left breast cancer
T2N2M0, cl. gr. II-a. What will be the treatment plan for this patient?
A. Radiation therapy + operation + chemotherapy  staging below, due to
Large tumour T2 and Lymph node fixed involoed ipsilateral so need
radiation to control the spreading of Ca then surgery for the primary tumour
then chemotherapy to prevent metastasis and activity after surgery
B. Operation only
C. Radiation therapy only
D. Chemotherapy only
E. Operation + radiation therapy
Dr.H.A.Salman - OdNMU

84. Immediately after delivery a woman had haemorrhage, blood loss exceeded
postpartum haemorrhage rate and was progressing. There were no symptoms of placenta
detachment. What tactics should be chosen?
A. Manual removal of placenta and afterbirth review Q 40
B. Instrumental revision of uterine cavity walls
C. Intravenous injection of methylergometrine with glucose
D. Removal of afterbirth by Credes method
E. Uterus tamponade

85. A 30 y.o. primigravida woman has got intensive labor pain every 1-2 minutes that lasts
50 seconds. The disengagement has started. The perineum with the height of 4 cm has
grown pale. What actions are necessary in this situation?
A. Episiotomy  in Ukraine the separate Episiotomy from Perineotomy , while
other sources saying it is synoyme
Dr.H.A.Salman - OdNMU

B. Expectant management
C. Perineotomy
D. Perineum protection
E. Vacuum extraction of fetus

Episiotomy, also known as perineotomy, is a surgical incision of the perineum and the posterior vaginal
wall generally done by a midwife or obstetrician. Episiotomy is usually performed during second stage
of labor to quickly enlarge the opening for the baby to pass through

86. A 30-year-old gravida consulted a gynecologist about bright red bloody discharges
from the vagina in the 32 week of gestation. She was hospitalized with a suspicion of
placental presentation. Under what conditions is it rational to conduct the internal
examination in order to make a diagnosis?
A. In the operating room prepared for the operation because the patient have
bright red bleeding that mean she may continue bleeding therefore we have
to exam her at the operation room and there is possibility of premature
labor through C/S.
B. In the admission ward of maternity hospital
C. The examination is not to be conducted because of risk of profuse haemorrhage
D. In the delivery room keeping to all the aseptics regulations
E. In the examination room of antenatal clinic

87. A 28 y.o. primagravida, pregnancy is 15-16 weaks of gestation, presents to the


maternity clinics with dull pain in the lower part of the abdomen and in lumbar area. On
vaginal examination: uterus cervix is 2,5 cm, external isthmus allows to pass the finger
tip. Uterus body is enlarged according to the pregnancy term. Genital discharges are
mucous, mild. What is the diagnosis?
A. Threatened spontaneous abortion  review Q 42 is when vaginal bleeding occurs,
and the diagnostic for a spontaneous abortion has not been met. In a threatened abortion, the
cervical os is closed
Dr.H.A.Salman - OdNMU

B. Hydatid molar pregnancy


C. Placenta presentation
D. Spontaneous abortion which has begun
E. Stopped pregnancy

88. A primapara with pelvis size 25-28-31-20 cm has active labor activity. Waters poured
out, clear. Fetus weight is 4500 g, the head is engaged to the small pelvis inlet. Vastens
sign as positive. Cervix of uterus is fully dilated. Amniotic sac is absent. The fetus
heartbeat is clear, rhythmic, 136 bpm. What is the labor tactics?

A. Caesarean section If a woman has a narrow pelvis, then with a fetal weight of
more than 3 kg 600 g, only a caesarean section is performed.
B. Obstetrical forseps
C. Vacuum extraction of the fetus
D. Conservative tactics of labor
E. Stimulation of the labor activity
Vasten's symptom is the ratio of the head of the child to the pelvis of a woman. It is
determined by the location of the head relative to the symphysis. A manual determination is
made. If the child’s head sticks out and is above the symphysis, then delivery is not possible in
a natural way.

89. Internal obstetric examination of a parturient woman revealed that the sacrum hollow
was totally occupied with fetus head, ischiadic spines couldnt be detected. Sagittal suture
is in the straight diameter, occipital fontanel is directed towards symphysis. In what
plane of small pelvis is the presenting part of the fetus?
A. Plane of pelvic outlet Plane of obstetric outlet (plane of least pelvic dimensions):
passes from the lower border of the symphysis pubis anteriorly, to the ischial spines laterally, to
the tip of the sacrum posteriorly
B. Wide pelvic plane
C. Plane of pelvic inlet
D. Over the pelvic inlet
E. Narrow pelvic plane
Dr.H.A.Salman - OdNMU

90. A 30 y.o. woman has the 2-nd labour that has been lasting for 14 hours. Hearbeat of
fetus is muffled, arrhythmic, 100/min. Vaginal examination: cervix of uterus is
completely opened, fetus head is level with outlet from small pelvis. Saggital suture is in
the straight diameter, small crown is near symphysis. What is the further tactics of
handling the delivery?
A. Use of obstetrical forceps
B. Use of cavity forceps
C. Cesarean section
D. Stimulation of labour activity by oxytocin
E. Cranio-cutaneous (Ivanovs) forceps

91. During examination of a patient, masses in the form of condyloma on a broad basis are
found in the area of the perineum. What is the tactics of the doctor?
A. To send a woman into dermatological and venerological centre
B. Cryodestruction of condyloms
C. Chemical coagulator treatment
D. Antiviral treatment
E. Surgical ablation of condyloms

in the form of a large, cauliflower-like, exophytic mass lesion in the perianal region is a rarely
encountered pathology in general surgery practice. Anogenital condylomata are known to be present in
at least 0.5–1% of sexually active population aged 15–25 years
Dr.H.A.Salman - OdNMU

92. A woman at 30 weeks pregnant has had an attack of eclampsia at home. On admission
to the maternity ward AP is 150/100 mm Hg. Predicted fetal weight is 1500 g. There is
face and shin pastosity. Urine protein is 0,66o/oo. Parturient canal is not ready for
delivery. An intensive complex therapy has been started. What is the correct tactics of
this case management?
A. Delivery by cesarean section Eclampsia cases = delivery C/S.
B. Continue therapy and prolong pregnancy for 3-4 weeks
C. Treat preeclampsia and achieve the delivery by way of conservative management
D. Labor induction by intravenous oxytocin or prostaglandins
E. Continue therapy and prolong pregnancy for 1-2 weeks
Pastosity = Edema
93. A 28 year old woman has bursting pain in the lower abdomen during menstruation;
chocolate-like discharges from vagina. It is known from the anamnesis that the patient
suffers from chronic adnexitis. Bimanual examination revealed a tumour-like formation
of heterogenous consistency 7*7 cm large to the left from the uterus. The formation is
restrictedly movable, painful when moved. What is the most probable diagnosis?
A. Endometrioid cyst of the left ovary
B. Fibromatous node
C. Tumour of sigmoid colon
D. Exacerbation of chronic adnexitis
E. Follicular cyst of the left ovary

94. Vaginal inspection of a parturient woman revealed: cervix dilation is up to 2 cm, fetal
bladder is intact. Sacral cavity is free, sacral promontory is reachable only with a bent
finger, the inner surface of the sacrococcygeal joint is accessible for examination. The
fetus has cephalic presentation. Sagittal suture occupies the transverse diameter of pelvic
inlet, the small fontanel to the left, on the side. What labor stage is this?
A. Cervix dilatation stage review Q 12 stages of labor.
B. Prodromal stage
C. Placental stage
D. Stage of fetus expulsion
E. Preliminary stage
Dr.H.A.Salman - OdNMU

95. A 68-year-old patient consulted a doctor about a tumour in her left mammary gland.
Objectively: in the upper internal quadrant of the left mammary gland there is a
neoplasm up to 2,5 cm in diameter, dense, uneven, painless on palpation. Regional
lymph nodes are not enlarged. What is the most likely diagnosis?
A. Cancer age of patient.
B. Lipoma
C. Fibroadenoma
D. Cyst
E. Mastopathy
Uneven = not smooth – not equal.
96. A 40-year-old female patient has been observing profuse menses accompanied by
spasmodic pain in the lower abdomen for a year. Bimanual examination performed
during menstruation revealed a dense formation up to 5 cm in diameter in the cervical
canal. Uterus is enlarged up to 5-6 weeks of pregnancy, movable, painful, of normal
consistency. Appendages are not palpable. Bloody discharges are profuse. What is the
most likely diagnosis?
A. Nascent submucous fibromatous node
B. Cervical myoma  uterus not enlarged , in Q uterus enlarged.
C. Algodismenorrhea  Oligomenorrhea is a condition in which you have infrequent menstrual
periods. It occurs in women of childbearing age
D. Abortion in progress  no pregnancy
E. Cervical carcinoma  no symptoms of CA
Nascent = budding ( in the beginning of formation).
Dr.H.A.Salman - OdNMU

97. A 29-year-old patient complains of sterility. Sexual life is for 4 years being married,
does not use contraception. There was no pregnancy before. On physical examination,
genitals are developed normally. Uterine tubes are passable. Rectal temperature during
three menstrual cycles is monophase. What is the most probable reason for sterility?
A. Anovulatory menstrual cycle review Q 65
B. Genital endometriosis
C. Anomalies of genitals development
D. Chronic adnexitis
E. Immunologic sterility

98. A 45 y.o. woman complains of contact bleedings during 5 months. On speculum


examination: hyperemia of uterus cervix, looks like cauliflower, bleeds on probing. On
bimanual examination: cervix is of densed consistensy, uterus body isnt enlarged,
mobile, nonpalpable adnexa, parametrium is free, deep fornixes. What is the most likely
diagnosis?
A. Cancer of cervix of uterus review Q 54
B. Cancer of body of uterus
C. Cervical pregnancy
D. Polypose of cervix of uterus
E. Fibromatous node which is being born

99. 10 minutes after delivery a woman discharged placenta with a tissue defect 5×6 cm
large. Discharges from the genital tracts were profuse and bloody. Uterus tonus was low,
fundus of uterus was located below the navel. Examination of genital tracts revealed that
the uterine cervix, vaginal walls, perineum were intact. There was uterine bleeding with
following blood coagulation. Your actions to stop the bleeding:
A. To make manual examination of uterine cavity review Q 63
B. To administer uterotonics
C. To introduce an ether-soaked tampon into the posterior fornix
D. To apply hemostatic forceps upon the uterine cervix
E. To put an ice pack on the lower abdomen

100. On the 5th day after labor body temperature of a 24-year-old parturient suddenly rose
up to 38,7°C. She complains about weakness, headache, abdominal pain, irritability.
Objectively: AP- 120/70 mm Hg, Ps- 92 bpm, to- 38,7°C. Bimanual examination revealed
that the uterus was enlarged up to 12 weeks of pregnancy, it was dense, slightly painful
on palpation. Cervical canal lets in 2 transverse fingers, discharges are moderate, turbid,
with foul smell. In blood: skeocytosis, lymphopenia, ESR - 30 mm/h. What is the most
likely diagnosis?
Dr.H.A.Salman - OdNMU

A. Endometritis  infection of the endometrium (lining of the uterus ) the


inner layer. Common after delivery .
B. Metrophlebitis
C. Lochiometra
D. Parametritis
E. Pelviperitonitis

101. A 20 y.o. patient complains of amenorrhea. Objectively: hirsutism, obesity with fat
tissue prevailing on the face, neck, upper part of body. On the face there are acne
vulgaris, on the skin - striae cutis distense. Psychological and intellectual development is
normal. Gynecological condition: external genitals are moderately hairy, acute vaginal
and uterine hypoplasia. What diagnosis is the most probable?
A. Itsenko-Cushing syndrome= (moon face) caused by a pituitary tumor leading to
excessive production of ACTH (adrenocorticotropic hormone). Excessive ACTH stimulates the
adrenal cortex to produce high levels of cortisol, producing the disease state.
B. Turners syndrome
C. Shichans syndrome = Sheehan syndrome , or postpartum hypopituitarism, is a rare
complication of postpartum hemorrhage.
D. Babinski-Froehlich syndrome  Adiposogenital Dystrophy, rare childhood
metabolic disorder characterized by obesity, growth retardation, and retarded development of
the genital organs
E. Stein-Leventals syndrome  polycystic ovarian syndrome.(PCOS).
condition that affects a woman's hormone levels. Women with PCOS produce higher-than-normal
amounts of male hormones. characterized by an elevated level of male hormones (androgens) and
infrequent or absent ovulation (anovulation)

102. A 27 y.o. woman suffers from pyelonephritits of the only kidney. She presents to the
maternity welfare centre because of suppresion of menses for 2,5 months. On
examination pregnancy 11 weeks of gestation was revealed. In urine: albumine 3,3 g/L,
leucocytes cover the field of vision. What is doctors tactics in this case?
A. Immediate pregancy interruption because the pt. have single kidney and
there is pyelonephritis and UTI , so doctor will terminate the pregnancy for
mother safety .
B. Pregnancy interruption at 24-25 weeks
C. Maintenance of pregnancy till delivery term
D. Pregnancy interruption after urine normalization
E. Maintenance of pregnancy till 36 weeks

103. An 18-year-old primigravida in her 27-28 week of gestation underwent an operation on


account of acute phlegmonous appendicitis. In the postoperative period it is necessary to
take measures for prevention of the following pregnancy complication:
Dr.H.A.Salman - OdNMU

A. Noncarrying of pregnancy  this is an old protocol when you have operation


of appendicitis you have to terminate the pregnancy, nowadays , you can do
the operation especially through laparoscopy
B. Late gestosis
C. Fetus hypotrophy
D. Intestinal obstruction
E. Premature placenta detachment

104. A 24-year-old female patient complains of acute pain in the lower abdomen that
turned up after a physical stress. She presents with nausea, vomiting, dry mouth and
body temperature 36,6°C. She has a right ovarian cyst in history. Bimanual examination
reveals that uterus is dense, painless, of normal size. The left fornix is deep, uterine
appendages arent palpable, the right fornix is contracted. There is a painful formation on
the right of uterus. Its round, elastic and mobile. It is 7×8 cm large. In blood:
leukocytosis with the left shit. What is the most likely diagnosis?
A. Ovarian cyst with pedicle torsion Ovarian torsion. Cysts that enlarge can cause
the ovary to move, increasing the chance of painful twisting of your ovary (ovarian torsion).
Symptoms can include an abrupt onset of severe pelvic pain, nausea and vomiting
B. Acute metritis
C. Extrauterine pregnancy
D. Right-sided pyosalpinx
E. Subserous fibromyoma of uterus

105. A parturient woman is 23 years old. Vaginal obstetric examination reveals full cervical
dilatation. There is no fetal bladder. Fetal head is in the plane of pelvic outlet. Sagittal
suture is in mesatipellic pelvis, anterior fontanel is closer to pubes. The fetal head
diameter in such presentation will be:
A. Suboccipito-bregmaticus
B. Suboccipitio-frontalis
C. Mento-occipitalis
D. Fronto-occipitalis recta
E. Biparietal
Dr.H.A.Salman - OdNMU

106. A pregnant 26-year-old woman was admitted to a hospital for abdominal pain and
bleeding from the genital tract. Bimanual examination revealed that uterus was the size
of 9 weeks of pregnancy, the cervical canal let a finger through. Fetal tissues could be
palpated in the orifice. There was moderate vaginal bleeding. What is the tactics of
choice?
A. Instrumental extraction of fetal tissue
B. Therapy for the maintenance of pregnancy
C. Administration of hormones
D. Surveillance
E. Hemostatic and antianemic therapy

107. A 42-year-old woman has had hyperpolymenorrhea and progressing algodismenorrhea


for the last 10 years. Gynaecological examination revealed no changes of uterine cervix;
discharges are moderate, of chocolate colour, uterus is slightly enlarged and painful,
appendages are not palpable, the fornices are deep and painless. What is the most likely
diagnosis?
A. Uterine endometriosis review Q 70
B. Uterine carcinoma
C. Endomyometritis
D. Adnexal endmetriosis
E. Subserous uterine fibromyoma

108. On the tenth day after discharge from the maternity house a 2-year-old patient
consulted a doctor about body temperature rise up to 39°C, pain in the right breast.
Objectively: the mammary gland is enlarged, there is a hyperemized area in the upper
external quadrant, in the same place there is an ill-defined induration, lactostasis,
fluctuation is absent. Lymph nodes of the right axillary region are enlarged and painful.
What is the most likely diagnosis?
A. Lactational mastitis  review Q 77 not the age is wrong 2 years old ? may be
20y.
B. Erysipelas
C. Abscess
D. Dermatitis
E. Tumour
Dr.H.A.Salman - OdNMU

109. During the dynamic observation over a parturient woman in the second stage of labor
it was registered that the fetal heart rate fell down to 90-100/min and didnt come to
normal after contractions. Vaginal examination revealed the complete cervical dilatation,
the fetal head filling the entire posterior surface of the pubic symphysis and sacral
hollow; the sagittal suture lied in the anteroposterior diameter of the pelvic outlet, the
posterior fontanelle was in front under the pubic arch. What plan for further labour
management should be recommended?
A. Application of forceps minor
B. Episiotomy
C. Caesarean section
D. Application of cavity forceps
E. Stimulation of labour activity by intravenous injection of oxytocin

 Maternal factors for use of forceps:


1. Maternal exhaustion.
2. Prolonged second stage of labour.
3. Maternal illness such as heart disease, hypertension, glaucoma, aneurysm, or other conditions that
make pushing difficult or dangerous.
4. Hemorrhaging.
5. Analgesic drug-related inhibition of maternal effort (especially with epidural/spinal anaesthesia).

 Fetal factors for use of forceps:


1. Non-reassuring fetal heart tracing.
2. Fetal distress.
3. After-coming head in breech delivery.

110. A 27-year-old sexually active female complains of numerous vesicles on the right sex
lip, itch and burning. Eruptions regularly turn up before menstruation and disappear 8-
10 days later. What is the most likely diagnosis?
A. Herpes simplex virus
B. Primary syphilis
C. Bartholinitis
D. Cytomegalovirus infection
E. Genital condylomata
Dr.H.A.Salman - OdNMU

111. A 36-year-old female pesented to a gynecological hospital with a significant bleeding


from the genital tract and a 1-month delay of menstruation. Bimanual examination
revealed soft barrel-shaped cervix. Uterus was of normal size, somewhat softened.
Appendages were unremarkable on both sides. Speculum examination revealed that the
cervix was cyanotic, enlarged, with the the external orifice disclosed up to 0,5 cm. Urine
hCG test was positive. What is the most likely diagnosis?
A. Cervical pregnancy
B. Uterogestation
C. Threatened miscarriage
D. Ectopic pregnancy
E. Abortion in progress

112. A 26-year-old woman complains of having bloody discharges from the genitals for the
last 14 days, abdominal pain, general fatiguability, weakness, weight loss, body
temperature rise, chest pain, obstructed respiration. 5 weeks ago she underwent induced
abortion in the 6-7 week of gestation. Objectively: the patient is pale and inert. Bimanual
examination revealed that the uterus was enlarges up to 8-9 weeks of gestation. In blood:
Hb- 72 g/l. Urine test for chorionic gonadotropin gave the positive result. What is the
most likely diagnosis?
A. Chorioepithelioma a malignant, trophoblastic cancer, usually of the placenta. It is
characterized by early hematogenous spread to the lungs. It belongs to the malignant end of the
spectrum in gestational trophoblastic disease (GTD). It is also classified as a germ cell tumor and may
Dr.H.A.Salman - OdNMU

arise in the testis or ovary


there is sign and symptoms of malignancy see above.
B. Uterus perforation
C. Uterine carcinoma
D. Uterine fibromyoma
E. Metroendometritis

113. A 28-year-old patient complains of discomfort, acute pain in the lower third of the left
labia majora. The disease began suddenly after menstruation. Objectively: body
temperature is 38°C. The left labia majora has a formation to 3 cm diameter, with
hyperemic surface, extremely painful to the touch, with symptoms of fluctuation. What is
the most likely diagnosis?
A. Acute bartholinitis review Q 25
B. Vulvar fibroid
C. Vulvar cancer
D. Bartholin gland cyst
E. Hypertrophy of the labia

114. A 28-years-old woman complains of nausea and vomiting about 10 times per day. She
has been found to have body weight loss and xerodermia. The pulse is 100 bpm. Body
temperature is 37,2°C. Diuresis is low. USI shows 5-6 weeks of pregnancy. What is the
most likely diagnosis?
A. Moderate vomiting of pregnancy
B. I degree preeclampsia
C. Food poisoning
D. Premature abortion
E. Mild vomiting of pregnancy

115. A 40 week pregnant secundipara is 28 years old. Contractions are very active.
Retraction ring is at the level of navel, the uterus is hypertonic, in form of hourglass. On
Dr.H.A.Salman - OdNMU

auscultation the fetal heart sounds are dull, heart rate is 100/min. AP of the parturient
woman is 130/80 mm Hg. What is the most likely diagnosis?
A. Risk of hysterorrhexis risk of uterine rupture.
B. Attack of eclampsia
C. Disturbed labour
D. Mazolysis
E. Complete hysterorrhexis

116. After delivery and revision of placenta there was found the defect of placental lobule.
General condition of woman is normal, uterus is firm, there is moderate bloody
discharge. Speculum inspection of birth canal shows absence of lacerations and raptures.
What action is nesessary?
A. Manual exploration of the uterine cavity review Q 40 , Q 84
B. External massage of uterus
C. Urine drainage, cold on the lower abdomen
D. Introduction of hemostatic medications
E. Introduction of uterine contracting agents

117. A 25 y.o. patient complains of body temperature rise up to 37°C, pain at the bottom of
her abdomen and vaginal discharges. Three days ago, when she was in her 11th week of
pregnancy, she had an artificial abortion. Objectibely: cervix of uterus is clean, uterus is
a little bit enlarged in size, painful. Appendages cannot be determined. Fornixes are
deep, painless. Vaginal discharges are sanguinopurulent. What is the most probable
diagnosis?
A. Postabortion endometritis sign of endometrium infection after abortion.
B. Postabortion uterus perforation
C. Parametritis
D. Hematometra
E. Pelvic peritonitis

118. A 25 y.o. pregnant woman in her 34th week was taken to the maternity house in grave
condition. She complains of headache, visual impairment, nausea. Objectively: solid
edemata, AP- 170/130 mm Hg. Suddenly there appeared fibrillary tremor of face
muscles, tonic and clonic convulsions, breathing came to a stop. After 1,5 minute the
breathing recovered, there appeared some bloody spume from her mouth. In urine:
protein - 3,5 g/L. What is the most probable diagnosis?
A. Eclampsia  review Q 92
B. Cerebral hemorrhage
C. Epilepsy
D. Cerebral edema
E. Stomach ulcer
Dr.H.A.Salman - OdNMU

119. A 51-year-old patient complains of having intensive bloody discharges from vagina for
15 days after delay of menstruation for 2,5 months. In anamnesis: disorders of menstrual
function during a year, at the same time she felt extreme irritability and had sleep
disorders. US examination results: uterus corresponds with age norms, appendages have
no pecularities, endometrium is 14 mm thick. What is the doctors tactics?
A. Diagnostic curettage of uterine cavity  review Q48 , Q 73
B. TORCH-infection test
C. Hysterectomy
D. Conservative treatment of bleeding
E. Supravaginal amputation of uterus without appendages

120. An 18 y.o. patient complains of painfulness and swelling of mammary glands,


headaches, irritability, edemata of lower extremities. These symptoms have been present
since the begin of menarche, appear 3-4 days before regular menstruation. Gynecological
examination revealed no pathology. What is the most probable diagnosis?
A. Premenstrual syndrome
B. Neurasthenia
C. Mastopathy
D. Disease of cardiovascular system
E. Renal disease
Dr.H.A.Salman - OdNMU

121. A 22-year-old female patient complains of dull pain in her right iliac area that she has
been experiencing for a week, morning sickness and gustatory change. She has a history
of menstruation delay for 3 weeks. Objectively: AP- 80/50 mm Hg, pulse is 78 bpm,
body temperature is 37°C. Bimanual examination reveals that uterus is enlarged, soft,
mobile and painless. Uterine appendages are palpable on the right, there is a dense,
elastic and moderately painful formation 3x4 cm large. What is the most likely
diagnosis?
A. Progressing fallopian pregnancy
B. Right ovarian cyst
C. Acute appendicitis
D. Uterogestation
E. Interrupted fallopian pregnancy

122. A 30 y.o. parturient woman was taken to the maternity house with complaints of
having acute, regular labour pains that last 25-30 seconds every 1,5-2 minutes. Labour
activity began 6 hours ago. Uterus is in higher tonus, head of the fetus is above the
opening into the small pelvis. Fetal heartbeat is 136/min. P.V: cervical dilatation is 4 cm,
uterine fauces is spasming at a height of parodynia. Head is level with opening into the
small pelvis, it is being pushed off. What is the most probable diagnosis?
A. Discoordinated labour activity discoordination means a disorder in coordinated
contractions of different parts of an uterus. It is known that this pathology may result in a high
incidence of operative delivery, birth injury, pathological blood losses, postpartum septic
complications, maternal and perinatal morbidity
B. Pathological preliminary period
C. Normal labour activity
D. Primary powerless labour activity
E. Secondary powerless labour activity
Dr.H.A.Salman - OdNMU

123. A primigravida woman appealed to the antenatal clinic on the 22.03.03 with
complaints of boring pain in the lower part of abdomen. Anamnesis registered that her
last menstruation was on the 4.01.03. Bimanual examination revealed that uterine servix
is intact, external fauces is closed, uterus is enlarged up to the 9-th week of pregnancy,
movable, painless. What complication can be suspected?
A. Risk of abortion in the 9-th week of pregnancy  in comparing of LMP and
clinic visit date , it is suppose that the gestaional sac small for age, gives
indicate of risk of abortion .
B. Hysteromyoma
C. Vesicular mole
D. Abortion that started in the 9-th week of pregnancy

124. A 25-year-old female patient complains about having amenorrhea for 3 years. She
associates it with difficult labour complicated by massive hemorrhage. She also
complains of loss of weight, hair fragility and loss, lack of appetite and depression.
Objective examination reveals no pathological changes of uterus and its appendages.
What is the disease pathogenesis?
A. Hypoproduction of gonadotropin the symptoms mentioned above indicate
of decreased levels of hormone
B. Hyperproduction of estrogens
C. Hypoproduction of progesterone
D. Hyperproduction of prolactin
E. Hyperproduction of androgens

125. A 30-year-old patient consulted a doctor about menstruation absence for 2 years after
labour, loss of hair, body weight loss. The labour was complicated by a haemorrhage
caused by uterus hypotonia. Objectively: the patient is asthenic, external genitals are
hypoplastic, the uterus body is small and painless. The appendages are not palpaple.
What is the most likely diagnosis?
A. Sheehans syndrome is postpartum hypopituitarism caused by necrosis of the
pituitary gland. It is usually the result of severe hypotension or shock caused by massive
hemorrhage during or after delivery. Patients with SS have varying degrees of anterior
pituitary hormone deficiency
B. Turners syndrome
C. Galactorrhea-amenorrhea syndrome
D. Exhausted overy syndrome
E. Ovarian amenorrhea
See diagram below
Dr.H.A.Salman - OdNMU

126. A 28-year-old patient has been admitted to the gynecological department three days
after a casual coitus. She complains about pain in her lower abdomen and during
urination, profuse purulent discharges from the vagina, body temperature rise up to
37,8°C. The patient was diagnosed with acute bilateral adnexitis. Supplemental
examination revealed: the 4th degree of purity of the vaginal secretion, leukocytes within
the whole visual field, diplococcal bacteria located both intra- and extracellularly. What
is the etiology of acute adnexitis in this patient?
Dr.H.A.Salman - OdNMU

A. Gonorrheal is a sexually transmitted disease (STD) that can infect both men and women. It
can cause infections in the genitals, rectum, and throat. It is a very common infection, especially
among young people ages 15-24 years
B. Chlamydial
C. Staphylococcal
D. Trichomonadal
E. Colibacterial

127. A 25-year-old woman came to a maternity welfare clinic and complained about being
unable to conceive within 3 years of regular sexual life. Examination revealed weight
gain, male pattern of hair distribution on the pubis, excessive pilosis of thighs. Ovaries
were dense and enlarged, basal temperature was monophase. What is the most likely
diagnosis?
A. Sclerocystosis of ovaries = Ovarian sclerocytosis is the process of their
regeneration, accompanied by the formation of small cystic formations up to 1 cm
in size.
When sclerocystic ovaries are enlarged, and sealed shells are formed on
their surface.
Sometimes this syndrome is called
Stein- Leventhal syndrome. Typically, sclerocystic disease is formed as a result
of polycystic ovarian disease as the disorder progresses.
B. Adrenogenital syndrome
C. Gonadal dysgenesis
D. Premenstrual syndrome
E. Tubo-ovaritis

128. A 28 year old patient complained about prolongation of intermenstrual periods up to 2


months, hirsutism. Gynaecological examination revealed that the ovaries were enlarged,
painless, compact, uterus had no pecularities. Pelvic ultrasound revealed that the ovaries
were 4-5 cm in diameter and had multiple enlarged follicles on periphery.
Dr.H.A.Salman - OdNMU

Roentgenography of skull base showed that sellar region was dilated. What is the most
probable diagnosis?
A. Stein-Leventhal syndrome  see Q above Polycystic Ovarian syndrome.
B. Sheehans syndrome
C. Morgagni-Stewart syndrome
D. Premenstrual syndrome
E. Algodismenorrhea

129. A 32-year-old gravida complains of episodes of unconsciousness, spontaneous


syncopes that are quickly over after a change of body position. A syncope can be
accompanied by quickly elapsing bradycardia. There are no other complications of
gestation. What is the most likely reason for such condition?
A. Postcava compresseion by the gravid uterus is compression of the abdominal
aorta and inferior vena cava by the gravid uterus when a pregnant woman lies on her back, i.e.
in the supine position
B. Vegetative-vascular dystonia (cardiac type)
C. Psychosomatic disorders
D. Pressure rise in the veins of extremities
E. Pressure fall in the veins of extremities

130. A woman consulted a therapeutist about fatigability, significant weight loss, weakness,
loss of appetite. She has had amenorrhea for 8 months. A year ago she born a full-term
child. Haemorrhage during labour made up 2 l. She got blood and blood substitute
transfusions. What is the most probable diagnosis?
A. Sheehans syndrome review Q 125
B. Shereshevsky-Turners syndrome
C. Vegetovascular dystonia
D. Homological blood syndrome
E. Stein-Leventhal syndrome
Dr.H.A.Salman - OdNMU

131. A 54-year-old female patient consulted a gynaecologist about bloody discharges from
the vagina for 1 month. Last menstruation was 5 years ago. Gynaecological examination
revealed no pathological changes. What is the tactics of choice?
A. Diagnostic fractional curettage of uterine cavity review Q48 , Q 73, Q 119
B. USI
C. Colposcopy
D. Cytosmear
E. Symptomatic therapy

132. A 28-year-old female patient complains of having haemorrhage from the genital tracts
for 1 month. 6 months ago she had natural delivery and gave birth to a girl weighing
3100 g. Objectively: the uterus is enlarged to 9-10 weeks, mobile, painless, of
heterogenous consistency. Examination reveals vaginal cyanosis, anaemia and body
temperature rise up to 37,8°C. There is a significant increase in hCG concentration in the
urine. What is your provisional diagnosis?
A. Uterine chorionepithelioma = Chorioepithelioma review Q112
B. Endometritis
C. Uterine fibromyoma
D. Pregnancy
E. Hydatidiform mole

133. An ambulance delivered a 21-year-old woman to the gynaecological department with


complaints of colicky abdominal pain and bloody discharges from the genital tracts.
Bimanual examination revealed that uterus was soft, enlarged to the size of 6 weeks of
gestation, a gestational sac was palpated in the cervical canal. Uterine appendages
werent palpable. Fornices are free, deep and painless. Discharges from the genital tracts
are bloody and profuse. What is the most likely diagnosis?
A. Abortion in progress review Q 42
B. Interrupted fallopian pregnancy
C. Threat of abortion
D. Cervical pregnancy
E. Incipient abortion

134. A 33 y.o. woman survived two operations on account of extrauterine pregnancy, both
uterine tubes were removed. She consulted a doctor with a question about possibility of
having a child. What can be advised in this case?
A. Extracorporal fertilization
B. Induction of ovulation
C. Substitutional maternity
Dr.H.A.Salman - OdNMU

D. Insemination with her husbands semen


E. Artifical fertilization with donors semen

135. On the fifth day after a casual sexual contact a 25-year-old female patient consulted a
doctor about purulent discharges from the genital tracts and itch. Vaginal examination
showed that vaginal part of uterine cervix was hyperemic and edematic. There was an
erosive area around the external orifice of uterus. There were mucopurulent profuse
discharges from the cervical canal, uterine body and appendages exhibited no changes.
Bacterioscopic examination revealed bean-shaped diplococci that became red after
Grams staining. What is the most likely diagnosis?
A. Acute gonorrheal endocervicitis review Q 126
B. Candidal vulvovaginitis
C. Bacterial vaginism
D. Clamydial endocervicitis
E. Trichomonal colpitis

136. A 26 year old woman complains about edemata, swelling and painfulness of mammary
glands, headache, tearfulness, irritability. These signs turn up 5 days before
menstruation and disappear after its start. What clinical syndrome is it?
A. Premenstrual syndrome see Q 120
B. Stein-Leventhal syndrome
C. Adrenogenital syndrome
D. Postcastration syndrome
E. Climacteric syndrome
Dr.H.A.Salman - OdNMU

137. A 49-year-old woman complains about headache, head and neck going hot, increased
perspiration, palpitation, arterial pressure rise up to 170/100 mm Hg, irritability,
insomnia, tearfulness, memory impairment, rare and scarce menses, body weight
increase by 5 kg over the last half a year. What is the most likely diagnosis?
A. Climacteric syndrome is a set of symptoms caused by the decline of ovarian hormone
levels, which alters brain neurotransmission and provokes musculoskeletal pains, mood
disorders, poor sleep quality and hot flushes
=menapousal syndrome :
recurring symptoms experienced by some women during the climacteric period; they include hot flashes, c
hills, headache, irritability, and depression.
B. Arterial hypertension
C. Postcastration syndrome
D. Premenstrual syndrome
E. Vegetative-vascular dystonia

138. A 30-year-old female patient has been delivered to the gynaecological department with
complaints of acute pain in the lower abdomen and body temperature 38,8°C. In history:
sexual life out of wedlock and two artificial abortions. Gynaecological examination
reveals no changes of uterine. The appendages are enlarged and painful on both sides.
Vaginal discharges are purulent and profuse. What study is required to confirm a
diagnosis?
A. Bacteriological and bacterioscopic analysis lab test to determined the
infection and need the culture to identified the source and causative agent ,
which is mostly affecting the appendages like – salpingo-oopheritis ..
B. Colposcopy
C. Laparoscopy
D. Hysteroscopy
E. Curettage of uterine cavity

139. A parturient woman is 27 year old, it was her second labour, delivery was at term,
normal course. On the 3rd day of postpartum period body temperature is 36,8°C, Ps -
72/min, AP - 120/80 mm Hg. Mammary glands are moderately swollen, nipples are
clean. Abdomen is soft and painless. Fundus of uterus is 3 fingers below the umbilicus.
Lochia are bloody, moderate. What is the most probable diagnosis?
A. Physiological course of postpartum period
B. Lactostasis
C. Postpartum metroendometritis
D. Subinvolution of uterus
E. Remnants of placental tissue after labour
Dr.H.A.Salman - OdNMU

140. A parturient woman is 25 years old, it is her second day of postpartum period. It was
her first full-term uncomplicated labour. The lochia should be:
A. Bloody see above – bloody = red color lasting 3-4 days
B. Mucous
C. Sanguino-serous
D. Purulent
E. Serous

141. A 32-year-old patient consulted a doctor about being inable to get pregnant for 5-6
years. 5 ago the primipregnancy ended in artificial abortion. After the vaginal
examination and USI the patient was diagnosed with endometrioid cyst of the right
ovary. What is the optimal treatment method?
A. Surgical laparoscopy
B. Conservative therapy with estrogen-gestagenic drugs
C. Sanatorium-and-spa treatment
D. Hormonal therapy with androgenic hormones
E. Anti-inflammatory therapy
Dr.H.A.Salman - OdNMU

142. A woman is 34 years old, it is her tenth labor at full term. It is known from the
anamnesis that the labor started 11 hours ago, labor was active, painful contractions
started after discharge of waters and became continuous. Suddenly the parturient got
knife-like pain in the lower abdomen and labor activity stopped. Examination revealed
positive symptoms of peritoneum irritation, ill-defined uterus outlines. Fetus was easily
palpable, movable. Fetal heartbeats wasnt auscultable. What is the most probable
diagnosis?
A. Rupture of uterus review Q 11
B. II labor period
C. Discoordinated labor activity
D. Uterine inertia
E. Risk of uterus rupture

143. Examination of placenta revealed a defect. An obstetrician performed manual


investigation of uterine cavity, uterine massage. Prophylaxis of endometritis in the
postpartum period should involve following actions:
A. Antibacterial therapy
B. Contracting agents
C. Intrauterine instillation of dioxine
D. Instrumental revision of uterine cavity
E. Haemostatic therapy

144. A 10 week pregnant woman was admitted to a hospital for recurrent pain in the lower
abdomen, bloody discharges from the genital tracts. The problems turned up after ARVI.
The woman was registered for antenatal care. Speculum examination revealed cyanosis
of vaginal mucosa, clean cervix, open cervical canal discharging blood and blood clots;
the lower pole of the gestational sac was visible. What tactics should be chosen?
A. Curettage of the uterus
B. Expectant management, surveillance
C. Pregnancy maintenance therapy
D. Hysterectomy
E. Antiviral therapy
ARVI = Acute Respiratory viral infection.
145. A 30 year old patient complains about inability to become pregnant over 3 years of
married life. The patient is of supernutrition type, she has hair along the median
abdominal line, on the internal thigh surface and in the peripapillary area. Menses
started at the age of 16, they are infrequent and non-profuse. US revealed that the uterus
was of normal size, ovaries were 4?5?5 cm large and had a lot of cystic inclusions. What
is the most probable diagnosis?
Dr.H.A.Salman - OdNMU

A. Polycystic ovaries  review Q 102 , Q 127


B. Ovarian cystoma
C. Menstrual irregularity
D. Bilateral ovarian tumours
E. Chronic oophoritis

146. A 29-year-old patient complains of absent menstruation for a year, milk discharge
from the nipples when pressed, loss of lateral visual fields. X-ray shows an expansion of
the sella turcica. What is the most likely cause of this condition?
A. Pituitary tumour this pt. have hyperprolactinemia in addition to expansion
of pituitary gland this may indicate pituitary Microadenoma ?
B. Pregnancy
C. Functional disorder of the hypothalamic-pituitary-ovarian system
D. Mammary tumour
E. Ovarian tumor
By the way here in this question the information regarding the diagnosis of expansion of sella
tursica is imaginary , the best way to diagnose the sella tursica and pituitary adenomas is MRI
imaging with contrast.
147. A patient with fibromyoma of uterus sized up to 8-9 weeks of pregnancy consulted a
gynaecologist about acute pain in the lower abdomen. Examination revealed pronounced
positive symptoms of peritoneal irritation, high leukocytosis. Vaginal examination
revealed that the uterus was enlarged corresponding to 9 weeks of pregnancy due to the
fibromatous nodes, one of which was mobile and extremely painful. Appendages were
not palpable. There were moderate mucous discharges. What is the optimal treatment
tactics?
A. Urgent surgery (laparotomy)
B. Fractional diagnostic curettage of the uterine cavity
C. Surveillance and spasmolytic therapy
D. Surgical laparoscopy
E. Surveillance and antibacterial therapy

148. A pregnant woman was delivered to the gynecological unit with complaints of pain in
the lower abdomen and insignificant bloody discharges from the genital tracts for 3
hours. Last menstruation was 3 months ago. Vaginal examination showed that body of
womb was in the 10th week of gestation, a fingertip could be inserted into the external
orifice of uterus, bloody discharges were insignificant. USI showed small vesicles in the
uterine cavity. What is the most likely diagnosis?
A. Grape mole Hydatiform mole – review Q 27
B. Abortion in progress
C. Threat of spontaneous abortion
D. Incomplete abortion
E. Incipient abortion
Dr.H.A.Salman - OdNMU

149. A 49-year-old patient undergoes regular medical check-up for uterine fibromyoma.
Within the last year the uterus has enlarged up to 20 weeks of gestation. What is the
rational way of treatment?
A. Surgical treatment
B. Treatment with prostaglandin inhibitors
C. Further surveillance
D. Hormonal therapy
E. Embolization of uterine arteries
Uterine fibroids

150. A female patient complains of being unable to get pregnant for 5 years. A complete
clinical examination brought the following results: hormonal function is not impaired,
urogenital infection hasnt been found, on hysterosalpingography both tubes were filled
with the contrast medium up to the isthmic segment, abdominal contrast was not
visualized. The patients husband is healthy. What tactics will be most effective?
A. In-vitro fertilization review Q 134 ( there is bilateral blockage of uterine
tube at the level after isthmus )
B. Laparoscopic tubal plasty
C. ICSI within in-vitro fertilization program
D. Insemination with husbands sperm
E. Hydrotubation

151. A 19-year-old primiparous woman with a body weight of 54,5 kg gave birth at 38 weeks
gestation to a full-term live girl after a normal vaginal delivery. The girls weight was
2180,0 g, body length - 48 cm. It is known from history that the woman has been a
smoker for 8 years, and kept smoking during pregnancy. Pregnancy was complicated by
moderate vomiting of pregnancy from 9 to 12 weeks pregnant, edemata of pregnancy
from 32 to 38 weeks. What is the most likely cause of low birth weight?
A. Fetoplacental insufficiency review Q 33
Dr.H.A.Salman - OdNMU

B. Third trimester preeclampsia


C. Womans age
D. Low weight of the woman
E. First trimester preeclampsia

152. A primigravida is 22 years old. She has Rh(-), her husband has Rh(+). Antibodies to
Rh werent found at 32 weeks of pregnancy. Redetermination of antibodies to Rh didnt
reveal them at 35 weeks of pregnancy as well. How often should the antibodies be
determined hereafter?
A. Once a week
B. Once in three weeks
C. Once in two weeks
D. Montly
E. There is no need in further checks

153. A maternity house has admitted a primagravida complaining of irregular, intense


labour pains that have been lasting for 36 hours. The woman is tired, failed to fall asleep
at night. The fetus is in longitudinal lie, with cephalic presentation. The fetus heartbeat is
clear and rhythmic, 145/min. Vaginal examination revealed that the uterine cervix was
up to 3 cm long, dense, with retroflexion; the external orifice was closed; the discharges
were of mucous nature. What is the most likely diagnosis?
A. Pathological preliminary period
B. Physiological preliminary period
C. Secondary uterine inertia
D. Uterine cervix dystocia
E. Primary uterine inertia
Dr.H.A.Salman - OdNMU

154. A 14-year-old girl complains of pain in vaginal area and lower abdomen that last for 3-
4 days and have been observed for 3 months about the same time. Each time pain is
getting worse. Objectively: mammary glands are developed, hairiness corresponds to the
age. The virginal membrane is intact, cyanotic and protruded. She has never had
menstruation. She has been diagnosed with primary amenorrhea. What is the reason of
amenorrhea?
A. Hymen atresia
B. Sexual development delay
C. Babinski-Frohlich syndrome
D. Turners syndrome
E. Pregnancy
Dr.H.A.Salman - OdNMU

155. A multigravida with Rh-isosensitization was found to have a decrease in anti-Rh titer
from 1:32 to 1:8 at 33-34 weeks of gestation. Ultrasound revealed double contour of
head, ebnlargement of fetal liver, placental thickness of 50 mm. The patient has
indication for:
A. Premature delivery early delivery due to RH- isosensitization .
B. Plasmapheresis
C. Administration of anti-Rh gamma globulin
D. Repeated (after 2 weeks) USI
E. Course of desensitizing therapy

156. A 13-year-old girl was admitted to the gynecological department with heavy bleeding,
which appeared after a long delay of menstruation. Shortly before, the girl suffered a
serious psychotrauma. Her menarche occurred at the age of 11, she has a 30-day cycle
with 5 to 6 days of moderate, painless bleeding. The patient is somatically healthy, of
normosthenic constitution with height of 160 cm, weight of 42 kg. The patient is pale.
Rectoabdominal examination revealed that the uterus was of normal size and
consistency, anteflexio-versio, the appendages were not changed. What is the most likely
diagnosis?
A. Juvenile bleedingreview Q 59
B. Hysteromyoma
C. Amenorrhea
D. Girl is healthy
E. Ovarian cyst

157. A 38-year-old female patient complains about hot flashes and feeling of intense heat
arising up to 5 times a day, headaches in the occipital region along with high blood
pressure, palpitations, dizziness, fatigue, irritability, memory impairment. 6 months ago
the patient underwent extirpation of the uterus with its appendages. What is the most
likely diagnosis?
A. Post-castration syndrome these symptoms appeared after removing the
uterus and its appendages ( tubes and ovaries).
B. Premenstrual syndrome
C. Secondary psychogenic amenorrhea
D. Physiological premenopause
E. Early pathological menopause

158. A 27-year-old patient complains of irritability, tearfulness, depression, and sometimes


aggressiveness, headache, nausea, vomiting, swelling of the mammary glands. The
mentioned problems arise 5-6 days before menstruation and gradually progress until
menstruation, 3 days after it the problems disappear. What is the most likely diagnosis?
A. Premenstrual syndrome review Q 120
B. Secondary psychogenic amenorrhea
Dr.H.A.Salman - OdNMU

C. Premature pathological climacterium


D. Preclimacterium syndrome
E. Algomenorrhea

159. A 23-year-old primigravida at 39 weeks gestation has been admitted to the maternity
ward with irregular contractions. The intensity of uterine contractions is not changing,
the intervals between them stay long. Bimanual examination reveals that the cervix is
centered, soft, up to 1,5 cm long. There is no cervical dilatation. What diagnosis should
be made?
A. Pregnancy I, 39 weeks, preliminary period
B. Pregnancy I, 39 weeks, labor I, period 1, the active phase
C. Pregnancy I, 39 weeks, pathological preliminary period
D. Pregnancy I, 39 weeks, birth I, 1 period, the acceleration phase
E. Pregnancy I, 39 weeks, labor I, 1 period, the latent phase

160. 20 minutes after a normal delivery at 39 weeks a puerpera had a single temperature
rise up to 38°C. Objectively: the uterus is dense, located between the navel and the pubis,
painless. Lochia are bloody, of small amount. Breasts are moderately soft and painless.
What is the optimal tactics?
A. Further follow-up all symptoms above normal after delivery but she
resolved after a time for ex. Temp should decreased , uterus size should
decrease , locia should stop .
B. Appointment antipyretic
C. Expression of breast
D. Manual examination of the uterine cavity
E. Antibiotic therapy

161. On the 10th day postpartum a puerperant woman complains of pain and heaviness in
the left breast. Body temperature is 38,8°C, Ps - 94 bpm. The left breast is edematic, the
supero-external quadrant of skin is hyperemic. Fluctuation symptom is absent. The
nipples discharge drops of milk when pressed. What is a doctors further tactics?
A. Antibiotic therapy, immobilization and expression of breast milk patient
have acute mastitis so treatment plan give give antibiotic , fix breast and
remove milk.
B. Physiotherapy
C. Opening of the abscess and drainage of the breast
D. Compress to both breasts
E. Inhibition of lactation

162. On the 10th day postpartum a puerperant woman complains of pain and heaviness in
the left mammary gland. Body temperature is 38,8°C, Ps- 94 bpm. The left mammary
gland is edematic, the supero-external quadrant of skin is hyperemic. Fluctuation
symptom is absent. The nipples discharge drops of milk when pressed. What is a doctors
further tactics?
Dr.H.A.Salman - OdNMU

A. Antibiotic therapy, immobilization and expression of breast milk see Q


above
B. Physiotherapy
C. Opening of the abscess and drainage of the mammary gland
D. Compress to both mammary glands
E. Inhibition of lactation

163. A 30-year-old female patient complains of milk discharge from the mammary glands,
5-month absence of menstruation. She had one physiological labour four years ago.
Objectively: mammary glands are normally developed. Bimanual examination reveals
that the uterus is decreased in size, the ovaries are of normal size. MRI-scan shows no
cerebral pathologies. Concentration of thyroid-stimulating hormone is normal. The
serum prolactin level is increased. What is the most likely diagnosis?
A. Hyperprolactinemia increased serum prolactin in blood .
B. Hypothyroidism
C. Pituitary adenoma
D. Sheehan syndrome
E. Polycystic ovary syndrome

164. During self-examination a 22-year-old patient revealed a mammary tumour. Palpation


revealed a firm, painless, mobile formation up to 2 cm, peripheral lymph nodes were not
changed. USI results: in the superior external quadrant of the right mammary gland
there was a big formation of increased echogenicity, sized 18x17 mm. The patient was
provisionally diagnosed with fibroadenoma. What is a doctors further tactics?
A. Surgical removal of the tumour prior to pregnancy-> it is benign tumour and
can be remove before pregnancy because during pregnancy it may indcrease
in size and cause complications.
B. Dynamic follow-up
C. Radical mastectomy
D. Nonsteroid anti-inflammatory drugs, oral contraceptives
E. Surgical treatment after pregnancy

165. A 25-year-old female has a self-detected tumor in the upper outer quadrant of her right
breast. On palpation there is a painless, firm, mobile lump up to 2 cm in diameter,
peripheral lymph nodes are not changed. In the upper outer quadrant of the right breast
ultrasound revealed a massive neoplasm with increased echogenicity sized 21x18 mm.
What is the most likely diagnosis?
A. Fibroadenoma  look at age of patient , for young female commonly benign
tumors are common like fibro-adenoma , while for old age mostly
carcinoma.
B. Diffuse mastopathy
C. Mastitis
D. Mammary cancer
E. Lactocele
Dr.H.A.Salman - OdNMU

166. A 49-year-old female patient complains of itching, burning in the external genitals,
frequent urination. The symptoms have been present for the last 7 months. The patient
has irregular menstruation, once every 3-4 months. Over the last two years she has had
hot flashes, sweating, sleep disturbance. Examination revealed no pathological changes
of the internal reproductive organs. Complete blood count and urinalysis showed no
pathological changes. Vaginal smear contained 20-25 leukocytes per HPF, mixed flora.
What is the most likely diagnosis?
A. Menopausal syndrome
B. Trichomonas colpitis
C. Cystitis
D. Vulvitis
E. Bacterial vaginosis

167. 2 weeks after labour a parturient woman developed breast pain being observed for 3
days. Examination revealed body temperature at the rate of 39°C, chills, weakness,
hyperaemia, enlargement, pain and deformity of the mammary gland. On palpation the
infiltrate was found to have an area of softening and fluctuation. What is the most likely
diagnosis?
A. Infiltrative-purulent mastitis  review Q 57
B. Phlegmonous mastitis
C. Serous mastitis
D. Mastopathy
E. Lactostasis

168. Preventive examination of a 50-year-old woman revealed a dense tumour of the right
mammary gland up to 5 cm in diameter without distinct outlines. The skin over the
tumour looked like lemon peel. Palpation revealed a lymph node in the axillary region.
What is the most likely diagnosis?
A. Breast cancer in krok = skin lemon peel other details review Q 58
B. Mastitis
C. Breast lipoma
D. Lactocele
Dr.H.A.Salman - OdNMU

E. Diffuse mastopathy

169. A 20-year-old female consulted a gynecologist about not having menstrual period for 7
months. History abstracts: early childhood infections and frequent tonsillitis, menarche
since 13 years, regular monthly menstrual cycle of 28 days, painless menstruation lasts
5-6 days. 7 months ago the patient had an emotional stress. Gynecological examination
revealed no alterations in the uterus. What is the most likely diagnosis?
A. Secondary amenorrhea
B. Spanomenorrhea
C. Cryptomenorrhea
D. Primary amenorrhea
E. Algomenorrhea
Secondary amenorrhea can occur due to natural changes in the body. For example, the most common
cause of secondary amenorrhea is pregnancy. Breastfeeding and menopause are also common, but natural,
causes.
You are more likely to have absent periods if you:
 Are obese
 Exercise too much and for long periods of time
 Have very low body fat (less than 15% to 17%)
 Have severe anxiety or emotional distress
 Lose a lot of weight suddenly (for example, from strict or extreme diets or after gastric bypass surgery)

Other causes include:


 Brain (pituitary) tumors
 Drugs for cancer treatment
 Drugs to treat schizophrenia or psychosis
 Overactive thyroid gland
 Polycystic ovarian syndrome
 Reduced function of the ovaries

170. A 48-year-old female has been admitted to the gynecology department for pain in the
lower right abdomen and low back pain, constipations. Bimanual examination findings:
the uterus is immobile, the size of a 10-week pregnancy, has uneven surface. Aspirate
from the uterine cavity contains atypical cells. What diagnosis can be made?
A. Hysterocarcinoma  Uterine Carcinoma. same Q37
B. Colon cancer
C. Chorionepithelioma
D. Cervical cancer
E. Metrofibroma
Dr.Hur A.Salman - OdNMU
KROK REVIEW
- First of all
I’d like to thank all the sources , sites , books and persons that I used their material for this
preparation , and I know it is wrong that I used a lot of pictures under copy right without permission ,
therefore; I’m asking apologize from all that resources and persons whom made that efforts for
humanity
please anyone (sites and persons) object on this , don’t hesitate to contact me , directly I will remove
his work with pleasure and respecting his rights .
Unfortunately I don’t have enough place and time to mention you all ,
Here I’m repeating my thanks and appreciation for serving medicine and health overall the world .

- DEDICATION:
I dedicate this simple works for all humanity, asking to stop wars , destruction and killing people and
living Lovely , Peacefully with happiness and as one human being , we are all same source ,
beginning and same end just differ in between , life like 2 brackets ( ) one begin other one end , so
try to fill in-between these brackets with humanity , love , charity , saving the world .
- Thanks God , and all who supported me .

- How to prepare Krok test


krok is MCQ based exam (200Q/4h ) in clinical subjects ( Internal , surgery , pediatrics , Gyne & OB)
with other subjects like Hygiene & nutrition , psychiatry , social medicine , occupational disease ,
infectious disease , pediatrics surgery )
this exam based on translated questions from Russian to English therefore; a lot of mistakes and non-
usable English words , some non medical terms as well as printing mistakes.
- If we ask any foreign students about this exam , he will answer better to cancel this exam from
protocol of international faculty , because it’s never assess the student knowledge , the information
that used in test old , non updated diseases and old protocols and based on SSRI period.
- Unfortunately very old information make student confused with the international parameters,
procedures and new guidelines and modalities.

- Another thing why it is just one attempt ?? and you should repeat all the course again on account of
some miserable questions ??
So the best solution for this problem is to the exam selective as before , obligatory for Ukrainian
citizen and selective for foreign , because no benefit of this exam
Or 2nd option to change the discipline to make real international not just on papers

- May be this information will not change anything , but that notes for next courses

- About IFOM exam , should not be applied unless they change the syllabus and remove all that old
books , procedures , examination , and all protocols , old medical terms , and unusable words , as well
as the combination Latin roots that they make it , as they want to make it formal exam , it is not like
krok just keep without understand , it is hard exam and depend on updated information

- Actually students can Help to update books and protocols , especially the INTERNATIONAL
- So what student should do in this case , study the old questions and keep it without understanding
unfortunately especially for Hygiene , non medical questions , why doctor should keep dimension of
room or how to calculate air ventilation and which place should build hospital , this subject should be
selective , doctor after graduate work with patient and need to learn examination diagnosis of disease
and treatment better than learn numbers and old protocols
Dr.Hur A.Salman - OdNMU
- So I suggest on you to analyze the questions use the following steps :
1- for medical Q try to find signs or symptoms that related to the disease , whatever was diagnosis or
treatment and tactic
- 2- for non-understandable Q try to use Excluding the options , because a lot of Q they used 4 options
so far from the answer and the last one will the correct whatever it is right information or wrong .
- 3- use KEY way to match the Q with the answer , if clinical or other
- Sometimes there is more than one right answer but you should choose the more specific one
for ex give you information about extrauterine pregnancy and cervical pathology and they put in the
answer – ectopic pregnancy and cervical pregnancy ,, both of them right logically but you should
choose the cervical pregnancy it is more specific .
- For Hygiene most Q not understandable , some of them keep it , others if you translate it you can
understand , others you find the answer in the Q same words but not always , so I think just go ahead
and do the easiest way you like .
- Note: All information here based on Ukrainian information and protocols , so try to keep this
information temporarily because it differ completely from your state exam or another country ..

- I advice you don’t use this information furthermore due to most of these information not based on
clinical and practical measurement , most of it theoretical and hypothetical information .

- Before Finally I wish all pass safely and wish all best of luck , hoping to see all best doctors and have
a good futures , caring all people , saving the humanity , especially children and poor people and all
who on need , repairing the world which destroyed by wars.

- Remember KROK does’t make you a DOCTOR !


And never assess you
Please Don’t hesitate to contact me for adding some information or correct anything .
If you feel some information not well explained or there is any comments , please I’m listening and I will
correct it again and re share that information
I will be happy with your feedback

Finally if you find this file good and can help others , please share it

All answers is A with


Yellow color : the answers
Green color : key words of Q
Blue color : this is the most important words in Q (additional Keys)
Grey color : this is my note and additional info
Red color : this repeated Q or high-light
Violant color : critical notes

Yours
Dr. HUR A. SALMAN
OdNMU /Odessa
12/5/2018
E-mail: hur.amer@gmail.com
F/b: Hr Salman
Net : +964-790-489-6865
Dr.Hur A.Salman - OdNMU
Krok 2 – PEDIATRICS Base 2014
1- An 8 year old child has low-grade fever, arthritis, colicky abdominal pain and a purpuric rash
llocalized on the lower extremities. laboratory studies reveal a guaiac-positive stool, urinalysis with
red blood cell (RBC) casts and mild proteinuria, and a normal platelet count. The most likely
diagnosis is:
A Henoch-Schonlein's vasculitis  called also Henoch–Scholein Purpura or vasculitis
(HSP) ,or anaphylactoid purpura
B Systemic lupus erythematosus (SLE)  butterfly rash
C Rocky Mountain spotted fever  hemorrhagic fever/ Rickettsia rickettsii, rash appear in wrirst
and ankle then palms and soles then spread centrally to the trunk and face (mostly north and south
America)
D Idiopathic thrombocytopenic purpura   platelets
E Poststreptococcal glomerulonephritis  no sign of previous infection here, like tonsillitis. As well
as no glomerulonephritis
**stool guaiac-positive this test to detect blood with stool

2- A young man has painful indurations in the peripapillary regions of both mammary glands.
The most reasonable action will be:
A To leave these indurations untouched don’t touch my breast 
induration:The hardening of a normally soft tissue or organ, especially the skin, due to infla
mmation, infiltration of a neoplasm,or accumulation of blood.
maturity changes or one symptoms of breast malignization ??
B To remove them
C To cut and drain them
D To take an aspirate for bacterial inoculation and cytology
E To administer steroids locally

3- A 9 year old girl with a history of intermittent wheezing for several years is brought to the
pediatrician. The child has been taking no medications for some time. Physical examination
reveals agitation and perioral cyanosis. Intercostal and suprasternal retractions are present.
The breath sounds are quiet, and wheezing is audible bilaterally. The child is admitted to the
hospital. Appropriate interventions might include all of the following EXCEPT:

A Prescribe nebulized cromolyn sodium  Cromoglicic acid or cromoglicate) is a mast cell


stabilizer, cromolyn sodium. This drug prevents the release of inflammatory chemicals such
as histamine from mast cells.
It is not histamine antagonist / bronchodilator  therefore NOT used in ASTHMATIC ATTACK
Dr.Hur A.Salman - OdNMU
B Prescribe intravenous aminophylline
C Administer supplemental oxygen
D Prescribe intravenous corticosteroids
E Prescribe nebulized metaproterenol = Orciprenaline: ẞ 2 selective adrenergic agonist :
bronchodilator used in tx of asthma.
**All other choices BCDE could be used in asthmatic attacks in emergency situation, while A not.

4-Routine examination of a child with a history of bronchial asthma reveals AP of 140/90 mm Hg.
The most likely cause of the hypertension is:
A Renal disease  here as the pt. had bronchial asthma (hypersensitivity reaction type I) so the pt
could have due that glomerulonephritis (which is also hypersensitivity reaction Type III) , due to
this may induce hypertension .
B Theophylline overdose  cause HYPOTENSION , Tachycardia, K+, Ca 2
C Chronic lung disease  not direct effect.
D Coarctation of the aorta  HTN differs between upper and Lower extremities.
E Obesity  no info in Q

5-Patient with thyreotoxicosis is in the 2 beds hospital ward of therapeutic department. The area
of the ward is 18 m2, height 3 m, ventilation rate 2,5/hr. Air temperature - 200 , relative humidity
- 45%, air movement velocity - 0,3 m/s, light coefficient - 1/5, noise level - 30 dB. Do
hygienic evaluation of the conditions meet the standards?
A Discomfortable microclimate  local set of atmospheric conditions that differ from those in the
surrounding areas , not suitable due to air T low.
B Non-effective ventilation
C Poor lighting
D High level of noise
E All conditions meet the requirements
This Q from Hygiene.

6-The child is 11 m.o. He suffers from nervous-arthritic diathesis. The increased synthesis of
what acid is pathogenic at nervous-arthritic diathesis?
A Uric acid  is an altered reactivity of the nervous system caused by the inheritance of the
morphofunctional features of the organism that determine the breakdown of the breakdown and
synthesis of purine nucleotides
these people are sick with gout, obesity, nephritis, urolithiasis, diabetes, cholelithiasis, early
atherosclerosis. Uric acid is responsible for this.
B Acetic acid
C Phosphoric acid
D Hydrochloric acid
E Sulfuric acid
** Diathesis (medical) a hereditary or constitutional predisposition to a disease or other disorder

7-A 10-year-old child complains of fever (temperature is 39oC), frequent painful urination
[pollakiuria]. Urine test: proteinuria [0,066 g/L], leukocytouria [entirely within eyeshot],
bacteriuria [105 colony forming units/mL]. What is the most probable diagnosis?
A Acute pyelonephritis  there is symptoms of infection , so Bacteria & WBC in seen urine , in
addition to that very mild proteinuria comparing with GN very high
B Acute glomerulonephritis  mostly post streptococcal after URTI / tonsillitis
C Dysmetabolic nephropathy  renal disease due metabolic disorder.
D Acute cystitis  should tell site of pain in pubis area or lower abd. As well as Pasternatsky sign
will be – ve , which is more specific for pyelonephritis .
Dr.Hur A.Salman - OdNMU
E Urolithiasis  urinary tract stones , usu pain and hematuria .

8-A 8-year-old boy has suffered from tonsillitis. In 2 weeks he started complaining of migratory
joint pain, edema of joints, restriction of movements, fever. On examination, an acute rheumatic
heart disease, activity of the III-rd degree, primary rheumocarditis, polyarthritis; acute course of
disease, cardiovascular failure IIA. What medication is to be prescribed?
A Prednisone  steroids here due to severe carditis and HF IIA
B Cefazolin  I class cephalosporine G+ve , +/- G-ve
C Delagil  anti malarial drug.
D Diprazinum  Promethazine = calss I anti-Histamine
E Erythromycin  Macrolids antibiotics Group B streptococcal infection not in acute RF caused by
Group A (ẞ -hemolytics)

9-The 10 y.o. boy has complains on headache, weakness, fever 400 , vomiting, expressed dyspnea,
pale skin with flush on right cheek, lag of right hemithorax respiratory movement, dullness on
percussion over low lobe of right lung, weakness of vesicular respiration in this zone. The abdomen
is painless and soft at palpation. Which disease lead to these symptoms and signs?
A Pneumonia croupousa  new name Lobar Pneumonia
B Intestinal infection
C Acute appendicitis
D Acute cholecystitis
E Flu
** Lag mean lateness , slow ( Rt. Half of chest breathing later or slower than Lt in movement )

10-A patient with acute respiratory viral infection (3rd day of disease) complains of pain in
lumbar region, nausea, dysuria, oliguria. Urinalysis - hematuria (100-200 RBC in eyeshot spot),
specific gravity - 1002. The blood creatinin level is 0,18 millimole/l, potassium level - 6,4
millimole/l. Make the diagnosis:
A Acute interstitial nephritis
B Acute renal failure
C Acute glomerylonephritis
D Acute cystitis
E Acute renal colic

11-A neonate was born from the 1st gestation on term. The jaundice was revealed on the 2nd day
of life, then it became more acute. The adynamia, vomiting and hepatomegaly were observed.
Indirect bilirubin level was 275 mu*mol/L, direct bilirubin level – 5 mu*mol/L, Hb- 150 g/l.
Dr.Hur A.Salman - OdNMU
Mother's blood group - 0(I),Rh+, Rh+.
child's blood group - A(II), What is the most
probable diagnosis?
A Hemolytic disease of the neonate (АВ0 incompatibility), icteric type  be careful here ABO not
matched while Rh is matched both of them +ve , icterus type = mean Juandice (pre-hepatic)
indirect bilirubin will increase (unconjugated)
B Jaundice due to conjugation disorder  need to know about the enzyme glucuronyltransferase
C Hepatitis
D Physiological jaundice
E Hemolytic disease of the neonate (Rh - incompatibility)  doNOT CONFUSE…

12- same Q 11
12-A baby boy was born in time, it was his mother's 1st pregnancy. The jaundice was revealed on
the 2nd day of life, then it progressed. The adynamia, vomiting and hepatomegaly were presented.
The indirect bilirubin level was 275 mcmol/L, the direct bilirubin level - 5 mcmol/L, Hb- 150 g/L.
Mother's blood group - 0(I), Rh+, child's blood group - A(II), Rh+. Make a diagnosis.
A Hemolytic disease of newborn (АВ incompatibility), icteric type
B Jaundice due to conjugation disorder
C Hepatitis
D Physiological jaundice
E Hemolytic disease of newborn (Rh - incompatibility)

13-A 3 month old infant suffering from acute segmental pneumonia has dyspnea (respiration rate -
80 per minute), paradoxical breathing, tachycardia, total cyanosis. Respiration and pulse - ratio
is 1:2. The heart dullness under normal size. Such signs characterise:
Dr.Hur A.Salman - OdNMU
A Respiratory failure of III degree  RR 80/min
B Respiratory failure of I degree  RR 30/min
C Respiratory failure of II degree  RR 50/min
D Myocarditis
E Congenital heart malformation

14-The 7 m.o. infant is suffering from acute pneumonia which was complicated by cardiovascular
insufficiency and respiratory failure of II degree. The accompanied diagnosis is malnutrition of II
degree. Choose the best variant of therapy:
A Ampiox and Amicacin  ampiox (ampicillin+ oxacillin) , amicacin = aminoglycosides
B Macropen and Penicillin
C Penicillin and Ampiox
D Gentamycin and Macropen
E Ampiox and Polymixin  DON’T MIX

15-A 3 year old child has been suffering from fever, cough, coryza, conjunctivitis for 4 days. He has
been taking sulfadimethoxine. Today it has fever up to 39oC and maculopapular rash on its
face. Except of rash the child's skin has no changes. What is your diagnosis?
A Measles  maculopapular rash = raseola rash started at face and goes down = descending rash,
in addition to Koplik’s spot
B Allergic rash
C Rubella  maculopapular rash = raseola rash started at lower extremities and buttocks then
goes up = Ascending rash With occipital lymph node
D Scarlet fever
E Pseudotuberculosis

16-A 2 year old girl has been ill for 3 days. Today she has low grade fever, severe catarrhal
presentations, slight maculopapular rash on her buttocks and enlarged occipital lymph nodes.
What is your diagnosis?
A Rubella also called (German measls) ascending rash , occipital LN , Forschheimer spot in 20%
of pt (palatine petechial rash) , not specific for rubella also check Table down
B Scarlet fever  check  Q 19 also check Table down
C Measles also check Table down
D Adenoviral infection
E Pseudotuberculosis also check Table down
Dr.Hur A.Salman - OdNMU
Differential diagnosis of meningococcemia
Signs Measles Rubella Scarlet fever
Initial catarrhal signs from upper Increase of occipital Acutely - intoxication,
symptoms airways, conjunctives lymph nodes, small angina, regional
during 2-4 days, catarrhal signs and lymphadenitis
intoxication intoxication
Time of the on 4-5 days of the 1 day, seldom 2 1 day (in 20% - 2)
rashes' disease, with stages
beginning
Morphology maculopapulous small-papulous, small point-like
Sizes of middle, large small, middle small
elements
Localization 1 day - on the face 2 - on on whole body, mainly mainly on bending
the face, trunk; 3 - on the on unbending surfaces surfaces of limbs, down
face, trunk, limbs of the limbs the abdomen, lumbar
region, face, lateral
surfaces of the trunk, pale
nose-labial triangle
Brightness and bright red pale-rose bright
color of
elements
Further rashes pigmentation, slight disappear on 3-4 days gradually turn pale for 4-5
' development hulling days, small, lamellar
hulling
Catarrhal expressed in first 5-6 days small, short for 1-2 Not typical,
phenomena days
Oral mucous hyperemied, friable, clear, sometimes single marked off, bright
membranes enanthema, Koplick's elements of enanthema hyperemia, enanthema on
spots palate, angina
Intoxication significant, lasts 5-7 days small or being absent proportional to local signs,
short for 1-3 days
Other Complications increased and painful angina, changes on the
symptoms (respiratory, digestive, posterior neck and tongue (raid, from 4-5 days
nervous, urinary systems, occipital lymph nodes "strawberry"),
eye, ears, skin) complications on 2-3
weeks
Laboratory leucopenia, leucopenia, leucocytosis, shift to the
criteria lymphocytosis, lymphocytosis, left, neutrophyllosis,
aneosynophylia, increase of the enlarged ESR, in
serological reaction with plasmatic cells' pharyngeal, nasal swabs –
measles antigen (+) number, serological streptococci
reactions with rubella
antigen (+)
Dr.Hur A.Salman - OdNMU
Signs Pseudotubercullosis Meningococcemia Chickenpox
Initial symptoms acutely with many intoxication, develops Acutely, observing
symptoms (intoxication, very acutely, initial catarrh, intoxication, rash
intestinal changes, seldom measles-like rash
- catarrhal signs
Time of the on 2-8 day first hours of the On 1-2 days, appear next
rashes' disease 3-5 days as pushes
beginning
Morphology puncture-like, small spots, hemorrhagic "star-like" Polymorphic (spots,
erythema with necrosis in the papules, vesicles, crusts)
centre
Sizes of Small, middle, large from small to middle
elements significant
Localization "hood", "mitten", "socks" buttocks, lower limbs, On whole body, on hair
signs, in skin folds, less - on trunk, hands, part of the head, seldom -
bends, around joints face on palms and soles
Brightness and bright hemorrhagic, bright, Papules are pink, vesicles -
color of sometimes cyanotic on hyperemied base
elements
Further rashes' gradually disappear for 2- Small, disappear After desquamation of the
development 5 days, small, lamellar gradually, significant, crusts - a slight
shelling leave "dry" necrosis pigmentation
Catarrhal Not typical are absent, in 30-40% Moderate,
phenomena on previous 2-3 days -
nasopharyngitis
Oral mucous Possible hyperemia of the hyperemia and groiness On pink background -
membranes pharynx, tonsils, of back pharyngeal polymorphic elements
wall, hypertrophy of
follicles
Intoxication expressed, long-lasting sharply expressed Small or moderate
(2-3 weeks)
Other symptoms arthritis, myocarditis, meningitis, Seldom: generalized
diarrhea, hepatitis, encephalitis, arthritis, visceral forms,
abdominal syndrome, iridocyclitis, meningoencephalitis
lymphoproliferative endocarditis, aortitis,
symptom, kidneys, pneumonia, pleurisy
nervous system damage,
pneumonia
Laboratory leucocytosis, shift to the leucocytosis, shift to Leucopenia,
criteria left, high ESR, Indirect the left, lymphocytosis, serological:
hemagglutination reaction neutrophyllosis, high binding complement
with special diagnosticum ESR, in reaction with Chickenpox
(+), separation of Y. nasopharyngeal swab, antigen (+)
pseudotuberculosis from thick drop of blood -
excrements meningococci
Dr.Hur A.Salman - OdNMU
39oC,
17- A 3 year old boy fell ill abruptly: fever up to weakness, vomitng. Haemorrhagic rash of
various size appeared on his lower limbs within 5 hours. Meningococcemia with infective - toxic
shock of the 1 degree was diagnosed. What medications should be administered?
A Chloramphenicol succinate and prednisone  nitrobenzene derivate and broad-spectrum
antibiotic + steroid
B Penicillin and prednisone
C Penicillin and immunoglobulin
D Chloramphenicol succinate and interferon
E Ampicillin and immunoglobulin

18- A 7 year old girl has mild form of varicella. Headache, weakness, vertigo, tremor of her
limbs, ataxia, then mental confusion appeared on the 5th day of illness. Meningeal signs are
negative. Cerebrospinal fluid examination is normal. How can you explain these signs?
A Encephalitis  previous viral infection (Varicella) cause of chickenpox and herpes zoster , then
appear neurological symptoms = mean the infection reach Brain , also meningeal signs -ve
B Meningitis
C Meningoencephalitis  same details above just add to it meningeal signs +ve
D Myelitis
E Neurotoxic syndrome

19- A 7 y.o. girl fell ill abruptly: fever, headache, severe sore throat, vomiting. Minute bright red
rash appear in her reddened skin in 3 hours. It is more intensive in axillae and groin. Mucous
membrane of oropharynx is hyperemic. Greyish patches is on the tonsills. Submaxillary lymph
nodes are enlarged and painful. What is your diagnosis?
A Scarlet fever  with features above , strawberry tongue check Q 16
B Measles check Q 16
C Rubella
D Pseudotuberculosis
E Enteroviral infection

20-An 8-year-old boy fell ill acutely: he presents with fever, weakness, headache, abdominal pain,
recurrent vomiting, then diarrhea and tenesmus. Stools occur 12 times daily, are scanty, contain a
lot of mucus, pus, streaks of blood. His sigmoid gut is tender and hardened.What is your diagnosis?
A Dysentery  shiegellosis / endo + exotoxin , no invation of blood stream
B Salmonellosis  Typhoid fever / Endotoxin only / can invade blood stream.
C Cholera  rice watery stool./ dehydration , Temp. normal
D Staphylococcal gastroenteritis food borne intoxication, 2-8h after eating /enterotoxin of S.
aureus
E Escherichiosis  acute intestinal infection caused by E.choli , usu affected 1 year old babies.
Dr.Hur A.Salman - OdNMU
21-The child has complains of the "night" and "hungry" abdominal pains. At fibroscopy in area a
bulbus ofa duodenum the ulcerrative defect of 4 mms diameter is found, the floor is obtected
with a fibrin, (H.p +). Administer the optimum schemes of treatment:
A Omeprasole - Trichopolum - Claritromicin  complex of ulcer tx . 
B De-nol
C Maalox - Ranitidin
D Vicalinum - Ranitidin
E Trichopolum

22-A woman delivered a child. It was her fifth pregnancy but the first delivery. Mother's blood
group is A(II)Rh-, newborn's - A(II)Rh+. The level of indirect bilirubin in umbilical blood was 58
micromole/l, haemoglobin - 140 g/l, RBC- 3,8x1012/l. In 2 hours the level of indirect bilirubin
turned 82 micromole/l. The hemolytic disease of newborn (icteric-anemic type, Rh-
incompatibility) was diagnosed. Choose the therapeutic tactics:
A Replacement blood transfusion (conservative therapy)
B Conservative therapy
C Blood transfusion (conservative therapy)
D Symptomatic therapy
E Antibiotics
** review Q 11 – 12

23-A mother with an infant visited the pediatrician for expertise advice. Her baby was born with
body weight 3,2 kg and body length 50 cm. He is 1 year old now. How many teeth the baby should
have?
A 8  there is a formula (N-4) n=age in months ; 1 year =12 month (12-4)=8 , if baby age 14month
the answer will be 14-4= 10 , and so on …
B 10
C 12
D 20
E6
24-A mother consulted a pediatrician about her son. Her son was born with body mass of 3 kg and
length of 48 cm. He's 1 year old now. What is the required normal mass?
A 10,5 kg  check down for new Q
B 9,0 kg
C 11,0 kg
D 12,0 kg
E 15,0 kg
** according to the chart baby should gain 7150 g by 1 year
May be this mistake not 3 kg may be 3.5kg according to 1 y old should be triple birth weight .
Dr.Hur A.Salman - OdNMU
Or use this rule : in 1 y baby gain 3+1/2 times his original weight
if 3 k ( 3 x 3.5 = 10.5 kg) . e.g if 3.5 x 3.5 = 12.25 kg +/-
So if add 3000 + 7150 will be 10150 ≠ 10.5 Kg !! any how keep it
General Trends in Weight and Height Gain During Infancy
Age Weight gain (grams) Height gain (cm)
Monthly For the whole period Monthly For the whole period
1. 600 600 3 3
2. 800 1400 3 6
3. 800 2200 3 9
4. 750 2950 2.5 11.5
5. 700 3650 2.5 14
6. 650 4300 2.5 16.5
7. 600 4900 2 18.5
8. 550 5450 2 20.5
9. 500 5950 2 22.5
10. 450 6400 1-1.5 23.5-24
11. 400 6800 1-1.5 24.5-25
12. 350 7150 1-1.5 25.5-27

25- 6 m.o. infant was born with body's mass 3 kg and length 50 cm. He is given natural feeding.
How many times per day the infant should be fed?
A 5  6 m and above
B 7  first 2 months (1-2)
C 6  3-5 m
D 8  shortly after birth
E4
Number of daily feedings:
First 2 months of life: 7 feedings per day every 3 hours with night break in 6 hrs.
3-5 months of life: 6 feedings per day every 3,5 hours with night break in 6,5 hrs.
After 6 months: 5 feedings per day every 4 hours with night break in 8 hrs.
26-Infant is 6,5 months now and is given natural feeding since birth. Body mass was 3,5 kg, with
length 52 cm at birth. How many times per day the supplement (up feeding) should be given?
A2
B3
C1
D0
E4
27-A 2 month old healthy infant with good appetite is given artificial feeding since he turned 1
month old. When is it recommended to start the corrective feeding (fruit juice)?
A 4,0 months
B 1,5 months
C 2,0 months
D 3,0 months
E 1,0 months
** I think this wrong Q , because artificial feed should start at 6 months and above. some books said
from 4-6m .. I hope they not use this Q , anyhow keep it .
Dr.Hur A.Salman - OdNMU
28-An infant was born with body mass 3 kg and body length 50 cm. Now he is 3 years old. His
brother is 7 years old, suffers from rheumatic fever. Mother asked the doctor for a cardiac check up
of the 3-year-old son. Where is the left relative heart border located?
A 1 cm left from the left medioclavicular line  check Table 
B 1 cm right from the left medioclavicular line
C Along the left medioclavicular line
D 1 cm left from he left parasternal line
E 1 cm right from the left parasternal line

AGE , 3

Lt border

29-A boy of 7 y.o. had an attack of asthma and distant whistling rales after playing with a dog. In
the medical hystory: atopic dermatitis caused by eating eggs, chicken, beef. What group of
allergens is the reason of the development of bronchial astma attacks?
A Epidermal  contact
B Dust
C Pollen
D Itch mite
E Chemical
30-A 14-year-old boy has rheumatism. Over the last 2 years he has had 3 rheumatic attacks. What
course of rheumatism does the patient have?
A Prolonged  due to exacerbations and remissions 3 times during last 2 yreas .
B Acute
C Subacute
D Latent
E Persistent-reccurent
31-The patient with aquired heart failure has diastolic pressure of 0 mm Hg. What heart failure
does the child have?
A Aortal insufficiency  better without discuss !! even there is a relation .
B Mitral stenosis
C Aortal stenosis
D Mitral insufficiency
E Rheumatism

32-A 12 year old child has the ulcer disease of stomach. What is the etiology of this disease?
A Intestinal bacillus hope not face this !!
B Helicobacter pylory  what’s about this ??
Dr.Hur A.Salman - OdNMU
C Salmonella
D Lambliosis
E Influenza

33-A nine year old child is at a hospital with acute glomerulonephritis. Clinical and laboratory
examinations show acute condition. What nutrients must NOT be limited during the acute period
of glomerulonephritis?
A Carbohydrates  McDonald, KFC, Dominos, SALO …… Free 
B Salt
C Liquid
D Proteins
E Fats

34-An 18-month-old child was taken to a hospital on the 4-th day of the disease. The disease
began acutely with temperature 39, weakness, cough, breathlessness. He is pale, cyanotic, has had
febrile temperature for over 3 days. There are crepitative fine bubbling rales on auscultation.
Percussion sound is shortened in the right infrascapular region. X-ray picture shows non-
homogeneous segment infiltration 8-10 mm on the right, the intensification of lung pattern. Your
diagnosis:
A Segmentary pneumonia signs of focal pneumonia in one segment , therefore called segmentary
B Grippe  this mean influenza
C Bronchitis
D Bronchiolitis
E Interstitial pneumonia

35- A 9-year-old girl has attacks of abdominal pain after fried food. No fever. She has pain in
Cera point. The liver is not enlarged. Portion B [duodenal probe] - 50 ml. What is your diagnosis?
A Biliary tracts dyskinesia, hypotonic type decrease motility of biliary duct. check 
B Hepatocirrhosis
C Acute colitis
D Chronic duodenum
E Peptic ulcer
** Cera point they translated it from = Kehr's point / symptom is strengthening of pain at pressure
on the area of gall-bladder, especially on deep inhalation.
** During palpation painfulness in the place of crossing of right costal arc with the external edge of
direct muscle of stomach can be observed (the Kehr's point). By superficial and deep palpation of
right hypochondrium, as a rule, painfulness, increased gall-bladder is exposed, that can be
important as a symptom, and sometimes determining for the diagnosis.
Dr.Hur A.Salman - OdNMU

36-A baby was born at 36 weeks of gestation. Delivery was normal, by natural way. The baby has a
large cephalohematoma. The results of blood count are: Hb- 120g/l, Er- 3,5x1012/l, total serum
bilirubin - 123 mmol/l, direct bilirubin - 11 mmol/l, indirect - 112 mmol/l. What are causes of
hyperbilirubinemia in this case?
A Erythrocyte hemolysis  total bilirubin indirect bilirubin  , RBC/Hb  (causing pre-hepatic
Jaundice) .
B Intravascular hemolysis   
C Disturbance of the conjugative function of liver
D Bile condensing
E Mechanical obstruction of the bile outflow  Direct bilirubin 

37-A 4-month-old girl with blond hair and blue eyes has "mousy" odor of sweat and urine, delayed
psychomotoric development. The most typical laboratory data for this disorder is:

A Positive urine ferric chloride test  aminoaciduria like Phenylketomiuria (usu. congenital)
B High level of oxyproline in urine
C High level of glycosaminoglycanes in urine
D High concentration of chlorides in sweat
E Low level of thyroid gland hormones in blood

38-A neonate is 5 days old. What vaccination dose of BCG vaccine (in mg) is necessary for
vaccination of this child?
A 0,05 mg
B 0,025 mg
C 0,075 mg
D 0,1 mg
E 0,2 mg
Dr.Hur A.Salman - OdNMU

39-7 y.o. boy with chronic sinusitis and rercurent pulmonary infections has chest X-ray
demonstrating a right-sided cardiac silhouette. What is the most likely diagnosis?
A Kartagener syndrome  also called Primary Ciliary Dyskinesia !! attention.
B Cystic fibrosis (mucoviscidosis)
C Bronchiolitis obliterans
D Laryngotracheomalacia
E $\alpha$-antitrypsin deficiency

** note : in some Q did not write situs inversus totalis (all organs are in opposite to
its natural location) but you see  Dextrocardia (only heart) become to the right.

40-A 2,9-kg term male infant is born to a mother who developed polyhydramnios at 34 weeks'
gestation. At birth, the Apgar scores were 9 and 9. The infant develops choking and cyanosis
with the first feed. In addition, is unable to place a nasogastric tube. What is the most likely
diagnosis?
A Esophageal atresia  congenital closing of esophagus. 
Dr.Hur A.Salman - OdNMU
B Choanal atresia
C Laryngomalacia
D Tracheal atresia
E Respiratory distress syndrome

41-Full term newborn has developed jaundice at 10 hours of age. Hemolytic disease of newborn
due to Rh-incompatibility was diagnosed. 2 hours later the infant has indirect serum bilirubin
level increasing up to 14 mmol/L. What is most appropriate for treatment of hyperbilirubinemia in
this infant?
A Exchange blood transfusion , review Q 22
B Phototherapy
C Phenobarbital
D Intestinal sorbents
E Infusion therapy

42-A 4 year old girl was playing with her toys and suddenly she got an attack of cough, dyspnea.
Objectively: respiration rate - 45/min, heart rate - 130/min. Percussion revealed dullness of
percutory sound on the right in the lower parts. Auscultation revealed diminished breath sounds
with bronchial resonance on the right. X-ray pictue showed shadowing of the lower part of lungs
on the right. Blood analysis revealed no signs of inflammation. The child was diagnosed with
foreign body in the right bronchus. What complication caused such clinical presentations?
A Atelectasis  is the collapse or closure of a lung resulting in reduced or absent gas exchange. It
may affect part or all of a lung.
B Emphysema
Dr.Hur A.Salman - OdNMU
C Pneumothorax
D Bronchitis
E Pneumonia

43-A man, 42 years old, died in a road accident after the hemorrhage on the spot, because of
acute hemorrhagic anemia. What minimum percent of the whole blood volume could result in
death by acute hemorrhage?
A 25-30%
B 6-9%
C 10-14%
D 15-20%
E 35-50%
44-A 6 week old child is admitted because of tachypnea. Birth had been uneventful, although
conjunctivitis developed on the third day of life and lasted for about 2 weeks. Physical examination
reveals tachypnea, bilateral inspiratory crackles and single expiratory wheezing. Bilateral
pneumonia is evident on chest X-ray. The child is afebrile and has no history of fever. White blood
cell count is 15x109/l, with 28% of eosinophils. The most likely cause of this child's symptoms is:

A Clamydia trachomanis  neonatal pneumonia, and neonatal conjunctivitis


B Pneumocystis carinii  opportunistic infection in person infected with HIV.
C Mycoplasma pneumoniae  walking pneumonia , leading pn in children  primary atypical pn.
Related to Cold agglutinin disease is an autoimmune disease characterized by the presence of high
concentrations of circulating antibodies, usually IgM, directed against red blood cells
D Visceral larva migrans  toxocara canis – Dog Ascaris
E Varicella  varicella-zoster virus (VZV) is one of 8 herpes viruses known to infect humans. It
causes chickenpox (varicella), a disease most commonly affecting children, teens, and young adults,
and herpes zoster (shingles) in older adults.

45-A 6 y.o. asthmatic child was taken to the emergency hospital because of severe coughing and
wheezing for the last 24 hours. Physical examination reveals that the child is excitable, has
intercostal and suprasternal retractions, expiratory wheezing throughout all lung fields, RR-
60/min. Initial treatment may include the prescription of:
A Subcutaneous epinephrine  Emenfency/ status asthmaticus , α & β agonist: prevent airway
obstruction bronchodilator and resistance and cardiovascular collapse   foce of Myocardial
contraction , also if you saw Corticosteriod instead epinephrine in asthmaticus , click it
B Parenteral phenobarbital
C Intravenous fluids in the first 2 h to compensate water deficiency
D N-acetyl cysteine and cromolyn by inhalation
E Parenteral gentamicyn
Dr.Hur A.Salman - OdNMU

46-A full term infant was born after a normal pregnancy, delivery, however, was complicated by
marginal placental detachment. At 12 hours of age the child, although appearing to be in good
health, passes a bloody meconium stool. For determining the cause of the bleeding, which of the
following diagnostic procedures should be performed first?
A Barium enema check down .
B An Apt test  test used to differentiate fetal or neonatal blood from maternal blood found in a
newborn's stool or vomit, or from maternal vaginal blood
C Gastric lavage with normal saline
D An upper gastrointestinal series
E Platelet count, prothrombin time, and partial thromboplastin time

47
In the 43rd week of gestation a long, thin infant was delivered. He is apneic, limp, pale, and
covered with "pea soup" amniotic fluid. The first step in the resuscitation of this infant at delivery
should be:
A Suction of the trachea under direct vision  there is meconium , be careful from ASPIRATED
PNEUMONIA
B Artificial ventilation with bag and mask
C Artificial ventilation with endotracheal tube
D Administration of 100% oxygen by mask
E Catheterization of the umbilical vein
Dr.Hur A.Salman - OdNMU
48-A newborn infant has mild cyanosis, diaphoresis, poor peripheral pule, hepatomegaly and
cardiomegaly. Respiratory rate is 60 breaths per minute, and heart rate is 230 beats per minute.
The child most likely has congestive heart failure caused by:
A Paroxysmal atrial tachycardia  Sudden increased HR at atrial level ( in ECG you see HR 150-
250/ min. P wave not seen , QRS normal without changes, non-specific ST and T wave changes .
B A ventricular septal defect and transposition of the great vessels
C Atrial flutter and partial atrioventricular block
D Hypoplastic left heart syndrome
E A large atrial septal defect and valvular pulmonary stenosis

49-A 6-year-old boy was brought to the emergency room with a 3-hour history of fever up to
39,5oC and sore throat. The child looks alert, anxious and has a mild inspiratory stridor. You
should immediately:
A Prepare to establish an airway  Emergency ABC , read down 
B Obtain an arterial blood gas and start an IV line
C Order a chest x-ray and lateral view of the neck
D Examine the throat and obtain a culture
E Admit the child and place him in a mist tent
Stridor  (creaking or grating noise) is a high-pitched breath sound resulting from turbulent air
flow in the larynx or lower in the bronchial tree.
Stridor is a physical sign which is caused by a narrowed or obstructed airway. It can be
inspiratory, expiratory or biphasic, although it is usually heard during inspiration. Inspiratory
stridor often occurs in children with croup. It may be indicative of serious airway obstruction from
severe conditions such as epiglottitis, a foreign body lodged in the airway, or a laryngeal tumor.
Stridor should always command attention to establish its cause. Visualization of the airway by
medical experts equipped to control the airway may be needed.

50-A 7 d.o. boy is admitted to the hospital for evaluation of vomiting and dehydration. Physical
examination is otherwise normal except for minimal hyperpigmentation of the nipples. Serum
sodium and potassium concentrations are 120 meq/L and 9 meq/L respectively. The most likely
diagnosis is:
A Congenital adrenal hyperplasia  take attention his age 7 days , mean congenital pathology , as
well as other sx like hyper pigmentation,  Na , K , check down 
B Pyloric stenosis
C Secondary hypothyroidism
D Panhypopituitarism
E Hyperaldosteronism
Dr.Hur A.Salman - OdNMU
51 -A 7 y.o. boy has crampy abdominal pain and a rash on the back of his legs and buttocks as well
as on the extensor surfaces of his forearms. Laboratory analysis reveals proteinuria and
microhematuria. He is most likely to be affected by:
A Anaphylactoid purpura  also called Hencoh Scholen purpura , review Q 1
B Systemic lupus erythematosus  butterfly rash.
C Poststreptococcal glomerulonephritis  no previous strep. Infection , URTI/Tonsillits
D Polyarteritis nodosa
E Dermatomyositis

52-A 5-year-old boy was progressively getting worse compared to the previous 2 months. A chest
x-ray has shown right middle lobe collapse. A tuberculin skin test was strongly positive. What is
the most characteristic finding in primary tuberculosis?
A Hilar or paratracheal lymph node enlargement
B Atelectasis with obstructive pneumonia
C Cavity formation
D Miliary tuberculosis
E Hematogenous dissemination leading to extrapulmonary tuberculosis

53-A girl is 12-year-old. Yesterday she was overcooled. Now she is complaining on pain in
suprapubic area, frequent painful urination by small portions, temperature is 37,8oC. Pasternatsky
symptom is negative. Urine analysis: protein - 0,033 g/L, WBC- 20-25 in f/vis, RBC- 1-2 in f/vis.
What diagnosis is the most probable?
A Acute cystitis  acute inflammation or urinary bladder .
B Dysmetabolic nephropathy
C Acute glomerulonephritis
D Acute pyelonephritis
E Urolithiasis
Review Q 7

54-The girl of 11 y.o. She is ill for 1 month. She has "butterfly"-type rash on face (spots and
papules), pain and swelling of small joints on arms and legs, signs of stomatitis (small-sized
ulcers in mouth). CBC: b– 80 g/L, RBC– 2,9x1012/L, WBC– 15x109/L, ESR- 40 mm/hour.
Urinalysis: protein– 0,33 g/L. What is the most probable diagnosis?
A Systemic lupus erythematosus  SLE
B Juvenile rheumatoid arthritis, systemic type
C Periarteriitis nodosa
Dr.Hur A.Salman - OdNMU
D Acute rheumatic fever
E Dermatomyositis

55-An infant aged 1 year on the third day of common cold at night developed inspiratory stridor,
hoarse voice and barking cough. Physical examination revealed suprasternal and intercostal chest
retractions. There is a bluish skin discoloration moistly seen over the upper lip. The respiratory rate
is 52 per min and pulse- 122 bpm. The body temperature is 37,5oC. What disease does the infant
have?
A Acute infectious croup due to viral laryngotracheitis para-inflaunza virus .
B Acute laryngitis
C Bronchopneumonia without complications
D Acute bronchiolitis with respiratory distress
E Acute epiglottitis
Dr.Hur A.Salman - OdNMU
56-A newborn aged 3 days with hyperbilirubinemia (428 mkmol/L) developed following isorders.
From beginning there were severe jaundice with poor suckling, hypotomia and hypodynamia. Little
bit later periodical excitation, neonatal convulsions and neonatal primitive reflexes loss are noted.
Now physical examination reveals convergent squint, rotatory nystagmus and setting sun eye sign.
How to explain this condition?
A Encephalopathy due to hyperbilirubinemia  Kernicterus
B Skull injury
C Brain tumour
D Hydrocephalus
E Spastic cerebral palsy

57
A child is 2 years old. The child complains of hoarse voice, dyspnea with obstructed inspiration.
The disease started 3 days ago from dry cough and nose stuffiness. Objectively: general
condition is unbalanced, stridor is present. The child's skin is pale. Body temperature is
37,7oC. The palatine arches are hyperemic. There is no deposit. Heart sounds are
rhythmic. Auscultation of lungs reveals rough breathing sounds, crepitation is absent.
Parainfluenza virus has been detected in nasopharynx lavage. What is the most likely
diagnosis?
A Acute laryngotracheitis  croup , review Q 55
B Epiglottitis
C Foreign body
D Diphtheria
E Laryngospasm

58-A 3-year-old child has been admitted to a hospital because of ostealgia and body temperature
rise up to 39oC. Objectively: the patient is in grave condition, unable to stand for ostealgia,
there is apparent intoxication, lymph nodes are enlarged up to 1,5 cm. Liver can be palpated 3
cm below the costal margin, spleen - 2 cm below the costal margin. In blood: RBCs -
3,0x1012/l, Hb- 87 g/l, colour index - 0,9, thrombocytes – 190x109/l, WBCs - 3,2x109/l, eosinophils
- 1, stab neutrophils - 1, segmented neutrophils - 0, lymphocytes - 87, monocytes - 2, ESR - 36
Dr.Hur A.Salman - OdNMU
mm/h. What examination should be conducted in order to specify the diagnosis?
A Sternal puncture  this child might have Acute Lymphocytic Leukemia ALL
B Ultrasound
C Lymph node puncture
D Lymph node biopsy
E Computer tomography

59-Apgar test done on a newborn girl at 1st and 5th minute after birth gave the result of 7-8 cores.
During the delivery there was a short-term difficulty with extraction of shoulder girdle. After birth
the child had the proximal extremity dysfunction and the arm couldn't be raised from the side. The
shoulder was turned inwards, the elbow was flexed, there was also forearm pronation, obstetric
palsy of brachial plexus. What is the clinical diagnosis?
A Duchenne-Erb palsy  involve C5-C6 nerve roots (proximal) or Upper part of brachial plexus
B Trauma of thoracic spine
C Right hand osteomyelitis
D Intracranial haemorrhage
E Trauma of right hand soft tissues

60-Examination of a 9-month-old girl revealed skin pallor, cyanosis during excitement.


Dr.Hur A.Salman - OdNMU
Percussion revealed transverse dilatation of cardiac borders. Auscultation revealed continuous
systolic murmur to the left of the breastbone in the 3-4 intercostal space. This murmur is
conducted above the whole cardiac region to the back. What congenital cardiac pathology can be
suspected?
A Defect of interventricular septum  VSD , whole cardiac murmur = pancardiac murmur
B Defect of interatrial septum
C Coarctation of aorta
D Fallot's tetrad
E Pulmonary artery stenosis

61-A worker was temporarily off work because of illness for 16 days, was under out-patient
Dr.Hur A.Salman - OdNMU
treatment. The doctor in charge issued a sick-list first for 5 days, then prolonged it for 10 days.
Who can further prolong the sick-list of this patient?
A The doctor in charge of the case together with the head of department  HYGIENE  anyhow
will explain it  Doctor usu can give 5 days and have the right to prolong it another 5 days (will be
10) together with head of department .
B Working ability expertise committee
C The doctor in charge of the case with the permission of the head of department  DON’T MIX.
D Deputy head physician on the working ability expertise
E The head of department
So funny hygiene pediatrics ( don’t be surprised ) they will do new department

62-A 13 y.o. patient was treated in dermatological hospital for atopic dermatitis exacerbation. He
was discharged in the condition of clinical remission. What recommendations should the doctor
give to prevent exacerbations?
A Use of neutral creams to protect skin
B Frequent skin washing with detergents
C Systematic use of local corticosteroids
D Systematic skin disinfection
E Avoidance of skin insolation

63-On the 21 day after appearance of vesiculous chickenpox rash a 7-year-old child developed
ataxia, nystagmus, intention tremor, muscle hypotonia. Liquor analysis shows a low-grade
lymphocytic pleocytosis, slightly increased protein rate. What complication is it?

A Encephalitis  connect it , hx. of viral inf. Then neurological sx. appeared


B Purulent meningitis  no meningeal irritation symptoms also purulent mean bacterial , should
tell you previous bacterial inf. Or Neutrophilic pleocytosis.
C Pneumonitis
D Acute nephritis
E Postherpetic neuralgia
Liquor mean CSF cerebrospinal fluid .
Pleocytosis mean increased cell in CSF  if bacterial inf mean Neutophil  or called
(PMN cell = polymorphonuclear cell ) and  if viral or TB Lymphocyte 

64-An 8-year-old boy suffering from haemophilia was undergoing transfusion of packed red cells.
Suddenly he felt pain behind the breastbone and in the lumbar area, dyspnea, cold sweat.
Objectively: pale skin, heart rate - 100/min, AP - 60/40 mm Hg; oliguria, brown urine. For the
treatment of this complication the following drug should be administered:
A Prednisolone  post transfusion hypersensitivity , give corticosteroid to prevent anaphylactic
and eliminate hypersensitivity reaction
B Lasix
C Adrenaline
D Aminophylline
E Analgine

65-A 3-year-old child has been diagnosed with type I diabetes mellitus, hyperosmolar coma. The
laboratory confirmed the diagnosis. Which laboratory findings are characteristic for such
condition?
A High hyperglycemia without ketonemia  no Aceton smell
B Hyperglycemia and ketonemia
C Hyperglycemia and glucosuria
Dr.Hur A.Salman - OdNMU
D Hyperglycemia and ketonuria
E Hyperglycemia and high indicators of acid-base balance

66-A 3-year-old child was playing in a playpen when he suddenly developed paroxysmal cough
and shortness of breath. Objectively: dry cough, mixed dyspnea. Lung auscultation revealed some
wheezes. Breathing sounds on the right are diminished. The child doesn't mix with other children.
Immunization is age-appropriate. What pathological condition can be suspected?
A Foreign body in the respiratory tracts  Easy .. He swallowed the pain ,, but How ?!!
B Pneumonia
C Acute respiratory viral infection
D Pertussis
E Bronchial asthma

67-A 10-year-old child has been folowed-up for the dilated cardiomyopathy. The child presents
with dyspnea, cardialgia. There are dense, nonmobile edemata on the lower extremities and
sacrum. Ps- 120/min. The cardiac borders are extended transversely. Heart sounds are muffled,
there is blowing systolic murmur at the apex and over the xiphoid process. Liver is 3 cm enlarged,
urine output is reduced. The blood total protein - 58.6 g/l. In urine: protein - 0,025 g/l, WBCs -
2-4 in the field of vision, RBCs - 2-3 in the field of vision. What is the main mechanism of edema
syndrome development:
A Venous congestion of greater circulation
B Venous congestion of lesser circulation
C Peripheral circulation disorder
D Secondary nephropathy development
E Hypoproteinemia
Dr.Hur A.Salman - OdNMU

68-After objective clinical examination a 12 year old child was diagnosed with mitral valve
prolapse. What complementary instrumental method of examination should be applied for the
diagnosis confirmation?
A Echocardiography  one of best method for valve demonstration
B Roentgenography of chest
C Phonocardiography
D ECG
E Veloergometry
69-A full-term child survived antenatal and intranatal hypoxia, it was born in asphyxia (2-5 points
on Apgar score). After birth the child has progressing excitability, there are also vomiting,
nystagmus, spasms, strabismus, spontaneous Moro's and Babinsky's reflexes. What localization of
intracranial hemorrhage is the most probable?
A Subarachnoid hemorrhage  birth trauma
B Small cerebral tissue hemorrhages
C Subdural hemorrhage
D Periventricular hemorrhages
E Hemorrhages into the brain ventricles

70-A 15 y.o. boy was twice attacked by bees, as a result he had severe anaphylactic shock. What is
the most effective prophylaxis method?
A Desensibilisation by means of bee venom extract
B Prescription of corticosteroids for summer
C Long-term prophylactic treatment with antihistamines
Dr.Hur A.Salman - OdNMU
D Limitation of outside staying during summer months
E Protective clothing

71
A 9-year-old boy has been suffering from bronchoectasis since he was 3. Exacerbations occur
quite often, 3-4 times a year. Conservative therapy results in short periods of remission. The
disease is progressing, the child has physical retardation. The child's skin is pale, acrocyanotic,
he has "watch glass" nail deformation. Bronchography revealed saccular bronchiectases of the
lower lobe of his right lung. What is the further treatment tactics?
A Surgical treatment
B Further conservative therapy
C Physiotherapeutic treatment
D Sanatorium-and-spa treatment
E Tempering of the child's organism

72-A child with tetralogy of Fallot is most likely to exhibit:


A Increased pressure in the right ventricle .. check down ..
B Increased pulmonary blood flow
C Increased pulse pressure
D Normal pressure gradient across the pulmonary valve
E Normal oxygen tension (PaO2) in the left ventricle

Exhibit = present
Dr.Hur A.Salman - OdNMU

73-A 2-months-old child after preventive vaccination had a prolonged hemorrhage from the
vaccination place and due to those an intramuscular hematoma. During examination of the child
a considerable rise of prothrombin consumption and a significant prolongation of the activated
partial thromboplastic time were found. What is the most probable diagnosis?
A Hemophilia =  PTT , PT normal
B Werlhof's disease  Immune thrombocytopenia (ITP) = PT, PTT normal
C Henoch-Schoenlein disease  HSP vasculitis.
D Hemorrhagic disease of the neonate
E Inborn afibrinogenemia  rare .. A deficiency or absence of FIBRINOGEN in the blood

** Activated partial thromboplastic time : PTT

74-A 10 y.o. boy with hemophilia has signs of acute respiratory viral infection with fever. What of
the mentioned antifebrile medications are contraindicated to this patient?
A Acetylsalicylic acid  APSIRIN .. is contraindicated / it may cause Reye Syndrom = rapidly
progressive encephalopathy
B Analgin
C Pipolphen
D Paracetamol
E Panadol extra

75-A 7-year-old child is sick for 2 weeks with running nose, was taking nasal drops. The boy
suffers with alimentary allergy. He applied to doctor due to suppurative and bloody discharges
from nose, maceration of ala nasi and upper lip. Rhinoscopy results: there are whitish-greyish areas
at nasal septum. Mucous membrane of oropharynx is not changed. What is the most probable
disease?
A Diphtheria of the nose  info 
B Adenovirus
C Rhinovirus
D Allergic rhinitis
E Sinusitis (maxillar sinus))
Dr.Hur A.Salman - OdNMU

76-A 10-year-old boy underwent treatment in cardiological department for rheumatism, I acute
attack of rheumatic fever, active phase, II degree. The patient was discharged in satisfactory
condition. Which drug should be chosen for prevention of rheumatism recurrence?
A Bicillinum-5  Benzylpenicillin benzathine
B Bicillinum-1
C Erythromycin
D Ampicillin
E Oxacillin
77-A child is 4 years old, has been ill for 5 days. There are complaints of cough, skin rash, to-
38,2oC, face puffiness, photophobia, conjunctivitis. Objectively: there is bright, maculo-papulous,
in some areas confluent rash on the face, neck, upper chest. The pharynx is hyperemic. There are
seropurulent discharges from the nose. Auscultation revealed dry rales in lungs. What is the most
likely diagnosis?
A Measles  Descending rash = from up to down.
B Adenoviral infection
C Scarlet fever
D Rubella
E Enterovirus exanthema
Review Q 15 + 16

78-A 10 month old boy has been ill for 5 days after consumption of unboiled milk. Body
temperature is 38-39oC, there is vomiting, liquid stool. The child is pale and inert. His tongue is
covered with white deposition. Heart sounds are muffled. Abdomen is swollen, there is
borborygmus in the region of umbilicus, liver is enlarged by 3 cm. Stool is liquid, dark-green, with
admixtures of mucus, 5 times a day. What is the most probable diagnosis?
A Salmonellosis  ** BE CAREFUL : most student see Liquid Stool , Dark green , going Directly
to Shigellosis , Here because of milk the coosed Sallmonella
please read all Q attentively and accurately – check 
B Staphylococcal enteric infection
C Escherichiosis
D Acute shigellosis  BE CAREFUL Q 147
E Rotaviral infection
Borborygmus : intestinal rumbling caused by moving gas
Dr.Hur A.Salman - OdNMU

79-A 3 year old child with weight deficiency suffers from permanent moist cough. In history there
are some pneumonias with obstruction. On examination: distended chest, dullness on
percussion over the lower parts of lungs. On auscultation: a great number of different rales.
Level of sweat chloride is 80 millimol/l. What is the most probable diagnosis?
A Mucoviscidosis (cystic fibrosis)  sweat chloride test is a common and simple test used to
evaluate a patient who is suspected of having cystic fibrosis (CF), the most common lethal genetic
disease affecting Caucasians. CF is often clinically suspected when there is poor growth during
infancy or recurrent serious intestinal or respiratory diseases in a toddler or young child. The
genetic defect in cystic fibrosis affects the way chloride moves in and out of cells, and sweat
contains chloride in the form of sodium chloride (salt). Measurement of the chloride in sweat has
been the standard method for diagnosing CF for over 40 years. Because cystic fibrosis is so
common, many states include testing for the CF gene as part of the Newborn Screen; however,
sweat testing is still required to confirm the diagnosis.
B Bronchial asthma
C Recurrent bronchitis
D Bronchiectasis
E Pulmonary hypoplasia
** normal sweat chloride values are 10-35 milliequ/L.
** Cystic fibrosis usually have a sweat chloride value > 60 milliequ/L.
Dr.Hur A.Salman - OdNMU

80-A 12 y.o. child with acute glomerulonephritis presented with hypertensive syndrom during
first days of the disease. What is the role of angiotesin II in the pathogenesis?
A Intensifies production and secretion of aldosterone.
B Increases heart output
C Infibits deppresive action of prostaglandins
D Increases erythropoetin production
E Increases renine level

81-A full-term infant is 3 days old. On the different parts of skin there are erythemas, erosive
spots, cracks, areas of epidermis peeling. The infant has scalded skin syndrome. Nikolsky's
symptom is positive. General condition of the infant is grave. Anxiety, hyperesthesia, febrile
Dr.Hur A.Salman - OdNMU
temperature are evident. What is the most probable diagnosis?
A Exfoliative dermatitis  peeling
B Phlegmon of newborn  abscesses of newborn
C Finger's pseudofurunculosis  2 know this 1- GOD 2- who put this option.
D Impetigo neonatorum  review Q 177
E Mycotic erythema

82-District pediatrician examines a healthy carried 1-month-old child. The child is breast-fed.
Prophylaxis of what disease will the doctor recommend to do first?
A Rachitis  Rickets = weak or soft bones in children. Symptoms include bowed legs, stunted
growth, bone pain, large forehead, and trouble sleeping. Complications may include bone
fractures, muscle spasms, an abnormally curved spine, or intellectual disability.
The most common cause is vitamin D deficiency
B Anemia
C Hypotrophia
D Spasmophilia
E Parathropy

83-A 7-year-old boy has been managed for a month. Immediately after hospitalization there were
apparent edemata, proteinuria - 7,1 g/l, daily urine protein - 4,2 g. Biochemical blood test shows
persistent hypoproteinemia (43,2 g/l), hypercholesterolemia (9,2 millimole/l). The patient is most
likely have the following type of glomerulonephritis:
A Nephrotic  all criteria described in Q 
B Nephritic  Hematuria , Oligouria, Edema , HTN …..
C Isolated urinary
D Hematuric
E Combined
84-A 3 y.o. girl has had a temperature rise up to 38o , rhinitis, dry superficial cough, flabbiness,
appetite loss. Palpation didn't reveal any changes over her lungs. Percussion sound has a wooden
resonance, auscultation revealed puerile breathing, no rales. In blood: leukopenia, lymphocytosis,
increased ESR. What is the most probable diagnosis?
A Acute simple tracheitis  inflammation of the Trachea , around the age 3
B Acute obstructive bronchitis
Dr.Hur A.Salman - OdNMU
C Recurrent bronchitis, acute condition
D Acute simple bronchitis
E Bilateral microfocal pneumonia

85-A 5-year-old girl with the transitory immunodeficiency according to T-system has a clinical
picture of a right-sided pneumonia during 2 months. How pneumonia progress can be described?
A Delaying  due to immunodeficiency
B Recidivating  regression 
C Chronic
D Wavelike
E Acute
86-Mother of a 10-month-old baby reports significant pallor, poor appetite, enlarged abdomen in
the baby. As a neonate, the child underwent treatment in the in-patient hospital for jaundice and
anemia. Objectively: the skin is pale and jaundiced, teeth are absent, abdomen is enlarged,
spleen is palpable. Blood test results: Hb - 90 g/l, RBC - 3,0x1012/l, color index - 0,9,
microspherocytosis, reticulocytosis up to 20\%, serum bilirubin - 37 mmol/l, unconjugated
bilirubin - 28 mmol/l. What type of anemia has occurred in the patient?
A Hemolytic anemia  Q 11,22
B Iron-deficiency anemia
C Protein-deficiency anemia
D $B_{12}$-deficiency anemia
E Hereditary elliptocytosis

87-A 12 y.o. girl took 2 pills of aspirine and 4 hours later her body temperature raised up to
39-40o . She complains of general indisposition, dizziness, sudden rash in form of red spots and
blisters. Objectively: skin lesions resemble of second-degree burns, here and there with erosive
surface or epidermis peeling. Nikolsky's symptom is positive. What is the most probable diagnosis?

A Acute epidermal necrolisis  check 


B Pemphigus vulgaris  rare chronic blistering skin disease and the most common form
of pemphigus , age: middle + 50-60 y.(type II hypersensitivity).
C Polymorphous exudative erythema
D Bullous dermatitis  Bullous pemphigoid
E Duhring's disease  Dermatitis herpetiformis
Dr.Hur A.Salman - OdNMU

88-A 5-year-old child had an attack of palpitation with nausea, dizziness, generalized fatigue. On
ECG: tachycardia with heartbeat rate of 220/min. Ventricle complexes are deformed and
widened. P wave is absent. What medication is to be prescribed to provide first aid?

A Lydocain  Lidocaine  .. Anti-arrhythmic drug class IB [Na+ channel blocker] .. this pt. had Vf
(Ventricular fibrillation )
B Isoptin  verapamil=class IV anti-arrhythmic Ca+ channel blocker .
C Seduxen  diazepam = A benzodiazepine with anticonvulsant, anxiolytic, sedative, muscle
relaxant, and amnesic properties and a long duration of action
D Novocainamides  Procainamide , Anti-arrhythmic class IA (Na+) channel block (intermediate
association/dissociation) and K+ channel blocking effect; affects QRS complex =
tx. ventricular arrhythmias: ventricular ectopy and tachycardia and supraventricular
arrhythmias: atrial fibrillation, and re-entrant and automatic supraventricular tachycardia
E Strophantin  Strophanthin K = is a cardiac glycoside which works as an inhibitor of Na+ /K+-
ATPase . This inhibition has an inotropic effect on the cardiac muscles increasing their force by
approximately 100%.

89- Examination of a 4 month old child revealed some lemon-yellow squamae with fatty crusts on
the scalp. What is the most probable diagnosis?
A- Gneiss  Seborrheic dermatitis, is an inflammatory skin reaction to increased activity of oil
glands of nurslings. Almost half of the newborn babies suffer from this disease. In ordinary
parlance, people call this disease “cradle cap” or a milky crust. It usually occurs in first 3 months
after the birth of a baby.
B Milk crust
C Strophulus
D Pseudofurunculosis
E Infantile eczema
Dr.Hur A.Salman - OdNMU
90-A lumbar puncture was performed for a newborn suspected of having an intracranial birth
injury. Bloody cerebrospinal fluid was obtained. What hemorrhage occurred in this case?
A Subarachnoid
B Cephalohematoma
C Epidural
D Supratentorial
E Subtentorial

91-A neonate from gestation with severe gestosis of the second half was born on the 41st week
with 2400 g birth weight and 50 cm long. On physical examination: skin is flaccid, subcutaneous
fatty cellular tissue is thin, muscle hypotonia, new-born period reflexes are decreased. Internal
organs are without pathological changes. How would you estimate this child?
A Term infant with pre-natal growth retardation  also called Intrauterine growth retardation
(IUGR)
term : mean births that happen after 37 weeks of pregnancy. Prenatal : mean before delivery , the
baby got growth retardation/ low birth weight (<2500 g)
B Premature infant check table down 
C Immature infant  a term sometimes applied to an infant who weighs less than 1134 g and who
Is significantly underdeveloped at birth.
D Post mature infant
E Term infant with normal body weight
** Emerging complications in the first half (early) pregnancy are called toxicosis, and in the second
(late) – gestosis.
Classification of prematurity.
The grade of the Term of the gestation Weight, gr
prematurity
I 35 – 37 weeks 2001 – 2500
II 32 – 34 weeks 1501 – 2000
III 29 – 31 weeks 1001 – 1500
IV Under 29 weeks Less than 1000
Dr.Hur A.Salman - OdNMU
92-A child was taken to a hospital with focal changes in the skin folds. The child was anxious
during examination, examination revealed dry skin with solitary papulous elements and ill-defined
lichenification zones. Skin eruption was accompanied by strong itch. The child usually feels
better in summer, his condition is getting worse in winter. The child has been artificially fed since
he was 2 months old. He has a history of exudative diathesis. Grandmother by his mother's
side has bronchial asthma. What is the most likely diagnosis?
A Atopic dermatitis  Allergic . family hx of atopia (Grandmother with asthma )
B Contact dermatitis
C Seborrheal eczema
D Strophulus
E Urticaria
93-A boy, aged 9, was examined: height - 127 cm (-0,36), weight - 28,2 kg (+0,96), chest
circumference - 64,9 cm (+0,66), lung vital capacity - 1520 ml (-0,16). What is the complex
assessment of the child's physical development?
A Harmonious  check down . you will understand it .
B Disharmonious
C Apparently disharmonious
D Excessive
E Below the average
The main criterions of assessment of physical development are:
1. weight;
2. height (stature, head-to-heel length);
3. head circumference (HC); •
4. chest circumference;
5. proportionality of these measurements.
**NOTE: all result between (+1 and – 1) will be Harmonious,
If result between (+2 to +1 and -2 to - 1 ) disharmonious
if more than +2 or more than -2 will be apparently (severe) disharmony
>-2 -2 -1 0 +1 +2 >+2

Severe Dis Har Dis Harmonus Dis Severe Dis Har


Ha Ha
Others using another way to correlate height to age if suitable so it is harmonious .
94-A child is 7 months old. Birth weight was 3450, the child is breastfed. Supplemental feeding
was introduced on time. Determine the daily protein requirements for the child:
A 3,0 g/kg
B 2,0 g/kg
C 2,5 g/kg
D 3,5 g/kg
E 4,0 g/kg
The daily requirement infants
in basic food ingredients (1 kg) in mixed feeding
• Protein 4 months to 2.0 - 2.5 g 3.5 - 4.0 g
• 4-9 months 3.0 - 3.5 g 3.5 - 4.0 g
• 9-12 months 3.0 - 3.5 g 3.5 - 4.0 g
• Fats up to 4 months 6.5 - 6.0 g 6.5 - 6.0 g
• 4-9 months 6.0 - 5.5 g 6.0 - 5.5 g
• 9-12 months 5.5 - 5.0 g 5.5 - 5.0 g
• Carbohydrate during the year 12.0 - 14.0 g
Dr.Hur A.Salman - OdNMU
as the question answer they used only the 1st
value just 3 not as shown in table up
(3-3.5g/kg) anyhow , this just if they renew the Q

95- 2 weeks after recovering from angina an 8-year-old boy developed edemata of face and lower
limbs. Objectively: the patient is in grave condition, AP- 120/80 mm Hg. Urine is of dark brown
colour. Oliguria is present. On urine analysis: relative density - 1,015, protein - 1,2 g/l, RBCs are
leached and cover the whole vision field, granular casts - 1-2 in the vision field, salts are
represented by urates (big number). What is the most likely diagnosis?
A Acute glomerulonephritis with nephritic syndrome check table 
B Acute glomerulonephritis with nephrotic syndrome
C Acute glomerulonephritis with nephrotic syndrome, hematuria and hypertension
D Acute glomerulonephritis with isolated urinary syndrome
E Nephrolithiasis
Dr.Hur A.Salman - OdNMU
96-A 14 year old child suffers from vegetovascular dystonia of pubertal period. He has got
sympathoadrenal atack. What medicine should be used for attack reduction?

A Obsidan  Propranolol hydrochloride .. non-selective β blocker used to treat high blood


pressure, a number of types of irregular heart rate, thyrotoxicosis, capillary
hemangiomas, performance anxiety, and essential tremors
B No-shpa  antispasmine
C Amysyl  may be they mean Amaryl Tablets (Glimepiride) hypoglycemic agent
D Aminophylline  Bronchodilator
E Corglicone  Corglycon Cardiac glycosides Acute & chronic heart failure (intolerance of
digitalis).
sympathoadrenal crises can be called diseases of the cardiovascular system, hormonal dysfunction
caused by age-related changes in the body during adolescence or menopause, during pregnancy.
Typically, the crisis begins suddenly, very quickly "dispersed" and in seconds reaches its peak.
pressure may spike to 200/100-120 mm Hg. art., pulse up to 120 to 150 and even higher.
They mean Addisonian crisis

97-A child is 9 months old. The patient's body temperature is 36,7oC, the skin is pale, humid,
there is pain in leg muscles. There is no extremities mobility, sensitivity is present. The child has
been diagnosed with poliomyelitis. The causative agent of this disease relates to the following
family:
A Picornavirus
B Paramyxovirus
C Tohovirus
D Adenovirus
E Rotavirus

98-A 4 month old child fell seriously ill: body temperature rose up to 38,5oC, the child became
inert and had a single vomiting. 10 hours later there appeared rash over the buttocks and lower
limbs in form of petechiae, spots and papules. Some haemorrhagic elements have necrosis in
the centre. What is the most probable disease?
A Meningococcemia  star-like rash petechial (satellite)
B Rubella
C Influenza
D Haemorrhagic vasculitis
E Scarlet fever
Dr.Hur A.Salman - OdNMU

99-A 5-year-old child had strong headache, vomiting, ataxy, dormancy, discoordination of
movements, tremor of the extremities on the 8th day of the disease. It was followed by rise in
body temperature, vesiculosis rash mainly on the skin of the body and the hairy part of the head.
At the second wave of the fever a diagnosis of encephalitis was given. What disease
complicated encephalitis in this case?
A Chicken pox  Varicella zoster virus VZV, in adult called Shingles , Inflammation of the brain,
or encephalitis, can occur in immunocompromised individuals check Q 16
B Measles
C German measles
D Enterovirus ifection
E Herpetic infection
Dr.Hur A.Salman - OdNMU

100-A 13 year old girl was admitted to the cardiological department because of pain in the muscles
and joints. Examination of her face revealed an edematic erythema in form of butterfly in the
region of nose bridge and cheeks. What is the most probable diagnosis?
A Systemic lupus erythematosus  SLE Review Q 54
B Rheumatism
C Dermatomyositis
D Rheumatoid arthritis
E Periarteritis nodosa

101-A 4 y.o. boy was admitted to the hospital with complaints of dyspnea, rapid fatigability. His
anamnesis registers frequent respiratory diseases. On percussion: heart borders are dilatated
to the left and upwards. On auscultation: amplification of the SII above pulmonary artery, a harsh
systolodyastolic "machine" murmur is auscultated between the II and the III rib to the left of
breast bone, this murmur is conducted to all other points including back. AP is 100/20 mm Hg.
What is the most probable diagnosis?
A Opened arterial duct  Patent Ductus Arteriosis PDA (machinar Murmur)
B Interventricular septal defect
C Isolated stenosis of pulmonary arterial orifice
D Interatrial septal defect
E Valvar aortic stenosis
Dr.Hur A.Salman - OdNMU

102-A 12 year old girl complains about abrupt weakness, nausea, dizziness, vision impairment.
The day before she ate home-made stockfish, beef. Examination revealed skin pallor, a scratch on
the left knee, dryness of mucous membranes of oral pharynx, bilateral ptosis, mydriatic pupils.
The girl is unable to read a simple text (mist over the eyes). What therapy would be the most
adequate in this case?
A Parenteral introduction of polyvalent antibotulinic serum
B Parenteral disintoxication
C Parenteral introduction of antibiotics
D Gastric lavage
E Parenteral introduction of antitetanus serum

BOUTILISM 
103-A child from the first non-complicated pregnancy but complicated labor had
cephalohematoma. On the second day there developed jaundice. On the 3th day appeared changes
of neurologic status: nystagmus, Graefe's sign. Urea is yellow, feces- golden-yellow. Mother's blood
Dr.Hur A.Salman - OdNMU
group isА(II)Rh-, А(II)Rh+.
child- On the third day child's Hb- 200 g/L, RBC- 6,1x1012/L, bilirubin
in blood - 58 mk mol/L due to unconjugated bilirubin, Ht- 0,57. What is the child's jaundice
explanation?
A Brain delivery trauma
B Physiologic jaundice
C Hemolytic disease of newborn  BE CARFUL
D Bile ducts atresia
E Fetal hepatitis

104-A full-term baby (the 1st uncomplicated pregnancy, difficult labour) had a cephalogematoma.
On the 2nd day there was jaundice, on the third the following changes in neurological status
appeared: nystagmus, Graefe syndrome. Urine was yellow, feces were of golden-yellow colour.
Mother's blood group is A(II)Rh-, the baby's one - A(II)Rh+. On the third day the child's Hb was
200g/l, RBCs - 6,1x1012/l, blood bilirubin - 58 micromole/l at the expense of unbound fraction.
What caused the jaundice in the child?
A Craniocerebral birth trauma  Same Q up 
B Physiological jaundice
C Neonatal anaemia
D Biliary atresia
E Fetal hepatitis

105-After birth a child was pale and had arrhythmical breathing. Oxygen therapy didn't have any
effect. Pulse was weak and rapid. It was difficult to measure arterial pressure accurately. There
were no edemata. What is the most likely reason for these symptoms?
A Asphyxia
B Congestive heart failure
C Intracranial haematoma
D Intrauterine sepsis
E Congenital pneumonia

106-A child was delivered severely premature. After the birth the child has RI symptoms,
anasarca, fine bubbling moist rales over the lower lobe of the right lung. Multiple skin
extravasations, bloody foam from the mouth have occurred after the 2 day. On chest X-ray:
atelectasis of the lower lobe of the right lung. In blood: Hb-100 g/L, Ht- 0,45. What is the most
probable diagnosis?
A Edematous-hemorrhagic syndrome  "Finkelstein's disease or Seidlmayer syndrome is a skin
Dr.Hur A.Salman - OdNMU
condition that affects children under the age of two with a recent history of upper respiratory
illness, a course of antibiotics, or both. The disease was first described in 1938 by Finkelstein and
later by Seidlmayer as "Seidlmayer cockade purpura
B Disseminated intravascular clotting syndrome
C Pulmonary edema
D Hyaline membrane disease
E Congenital pneumonia

107-An infant is 2 days old. He was born full-term with signs of intrauterine infection, and
therefore receives antibiotics. Neonates should be given antibiotics at longer intervals and lower
doses compared to older children and adults because:
A Neonates have lower glomerular filtration  most antibiotic clearance through kidney and
Neonate still have Low GFR , so need to lower doseof drug and increase intervals .
B Neonates have lower concentration of protein and albumin in blood
C Neonates have a reduced activity of glucuronyl transferase
D Neonates have a decreased blood pH
E Neonates have higher hematocrit

108-An infant is 2 d.o. It was full-term born with signs of intrauterine infection, that's why it was
prescribed antibiotics. Specify, why the gap between antibiotic introductions to the new-born
children is longer and dosage is smaller compared to the older children and adults?
A The newborns have a lower level of glomerular filtration  Same Q above
B The newborns have lower concentration of protein and albumins in blood
C The newborns have reduced activity of glucuronil transferase
D The newborns have diminished blood pH
E The newborns have bigger hematocrit

109-A 10-year-old child is sick with chronic viral hepatitis B with marked activity of the process.
Total bilirubin – 70 mu*mol/L, direct - 26mu*mol/L, indirect – 44 mu*mol/L. АS - 6,2 mmol/L,
АL - 4,8 mmol/L. What mechanism underlies the transaminase level increase of this patient?
A Cytolysis of hepatocytes  hepatocyte necrosis due to Viral infection
B Failure of the synthetical function of the liver
C Hypersplenism
D Intrahepatic cholestasis
E Failure of bilirubin conjugation
110-A 12-year-old girl applied to doctor with complaints of swelling on the front part of the neck.
The doctor diagnosed hyperplasia of the thyroid gland of the second degree, euthyroidism.
Ultrasound suspected autoimmune thyroiditis. Blood was taken for titre of antibodies to
Dr.Hur A.Salman - OdNMU
thyroglobulin. What titre of antibodies will be diagnostically important?
A 1:100
B 1:50
C 1:150
D 1:200
E 1:250

Indication for antithyroglobulin antibody test:

1. If the patient has following symptoms:


1. If there is weight gain without any explanation.
2. Patients have a history of fatigue.
3. History of constipation.
4. In the case of dry skin.
Normal
 Titer = <1:100
 A small normal population may have antithyroglobulin antibody.
 5 to 10% normal population may show low titer

111-A 14-year-old girl has been presenting with irritability and tearfulness for about a year. A year
ago she was also found to have diffuse enlargement of the thyroid gland (II grade). This
condition was regarded as a pubertal manifestation, the girl didn't undergo any treatment. The
Dr.Hur A.Salman - OdNMU
girl's irritability gradually gave place to a complete apathy. The girl got puffy face, soft tissues
pastosity, bradycardia, constipations. Skin pallor and gland density progressed, the skin became
of a waxen hue. What disease may be suspected?
A Autoimmune thyroiditis  Hashimato’s Thyroiditis (Hypothyroidism) symptoms
autoimmune thyroiditis is the release and entering of thyroid antigens into the blood as the result
of inflammatory processes and traumas combined with surgical operations on thyroid gland. It
has been found the presence of antibodies to thyroglobulins, colloidal component of thyroid
gland and microsomal fraction. However the presence of antithyroid Ab not always results in the
damage of the thyroid. The cytotoxic properties of these antibodies manifest only after their
interaction with Т-lymphocytes and HLA antigens.
B Diffuse toxic goiter  type of hyperthyroidism
C Thyroid carcinoma  no specific symptoms for CA also age young
D Subacute thyroiditis  it is self-limiting, no specific treatment, 15-20% of patients presenting
with thyrotoxicosis and 10% of patients presenting with hypothyroidism. And can be euthyroid
E Juvenile basophilism  PITUITARY BASOPHILISM IN THE JUVENILE
it is type of Acanthosis nigricans is characterized by the formation of hyperpigmented
** Pastosity = mean swelling and edema

112-In the anamnesis of a 2-year-old girl there are recurrent pneumonias with signs of
obstruction. There are heterogeneous moist and dry rales, respiration is weakened. Dense, viscous
secretion is difficult to hawk. There are "drumsticks", physical retardation. What is the most
probable diagnosis?
A Mucoviscidosis, pulmonary form
B Recidivating bronchitis
C Bronchial asthma
D Congenital pulmonary polycystosis
E Pulmonary tuberculosis
Review Q 79

113- On the 3rd day of life a baby presented with haemorrhagic rash, bloody vomit, black stool.
Examination revealed anaemia, extended coagulation time, hypoprothrombinemia, normal
thrombocyte rate. What is the optimal therapeutic tactics?
A Vitamin K  Hemorrhagic Disease of Newborn due to Vit K deficiency
Vitamin K is also a byproduct of certain types of bacteria that live in your intestines and colon (gut
flora). Like (Lactobacillus)also found in breast-fed babies doesn’t synthesize vitamin K
B Sodium ethamsylate  increasing capillary endothelial resistance and promoting platelet
adhesion .
C Epsilon-aminocapronic acid  fibrinolytic inhibitor, give at Bleeding associated with
fibrinolysis. Prevent conversion of plasminogen to plasmin that important to convert fibrinogen to
fibrin the lsat process in clot formation
D Fibrinogen  Fibrinogen is used to treat bleeding episodes in people with a congenital
fibrinogen deficiency.
E Calcium gluconate
** clotting time (coagulation time) the time required for blood to clot in a glass tube

114-A 2 month old full-term child was born with weight 3500 g and was on the mixed feeding.
Current weight is 4900 g. Evaluate the current weight of the child:
A Corresponding to the age  review Q 24 (for 2 months add 1400g ) 3500 + 1400 = 4900g so the
infant growing normally .
B 150 g less than necessary
Dr.Hur A.Salman - OdNMU
C Hypotrophy of the I grade  underweight
D Hypotrophy of the II grade
E Paratrophy of the I grade  mean obesity, when his weight greater than normal
115-A 2 m.o. breast-fed child suffers from cheek skin hyperemia, sporadic papulous elements on
the skin of the chest and back following the apple juice introduction. The child is restless. What is
the initial pediatritian's tactics?
A Clarify mother's diet and exlude obligate allergens  child got hypersensitivity from juice …
you should explain mother about child diet , and should not give any food before 4 – 6 months and
should not get any allergens food .
B Refer to prescribe dermathologist
C Administer general ultraviolet irradiation
D Treat with claritine
E Apply ointment with corticosteroids to affected skin areas

116-A 5 month old boy was born prematurely, he didn't suffer from any disease at the infant age
and later on. Examination at an outpatient's hospital revealed paleness of skin, sleepiness. Blood
count: Hb - 95 g/l, erythrocytes - 3,5x1012/l, reticulocytes – 9 o/o, colour index - 0,7, osmotic
stability of erythrocytes - 0,44-0,33%, serum iron - 4,9 micromole/l. What is the most probable
cause of anemia?
A Iron deficit  pt. have sx. of Iron Deficiency anemia (RBC/Hb reticluocyte CI mean
hypochromic )
normal Iron in blood serum 11.6 – 31.3 μmol/l (or they will write you normal value at end)
B Hemogenesis immaturity
C Infectious process
D Erythrocyte hemolysis
E B12 deficit  also called cyanocobalamine: type of megaloblastic anemias (big cell size),
( Pernicious anemia)

117-A 7 y.o. child had elevation of temperature to 40oC in anamnesis. For the last 3 months he
presents fusiform swelling of fingers, ankle joints and knee joint, pain in the upper part of the
sternum and cervical part of the spinal column. What is the most probable diagnosis?
A Juvenile rheumatic arthritis  mean young RA .
B Rheumatism
C Toxic synovitis  inflammation of synovial fluid of joints
D Septic arthritis
E Osteoarthrits  usually old age

118-An 8 year old girl complains about joint pain, temperature rise up to 38oC, dyspnea.
Objectively: the left cardiac border is deviated by 2,5 cm to the left, tachycardia, systolic murmur
on the apex and in the V point are present. Blood count: leukocytes – 20x109/l, ESR - 18 mm/h.
What sign gives the most substantial proof for rheumatism diagnosis?
A Carditis  one of the major criteria of Bens Jones of Acute Rheumatic fever.
B Arthralgia
C Leukocytosis  minor criteria
D Fever
E Accelerated ESR

119-A 5 y.o. child with stigmas of dysembryogenesis (small chin, thick lips, opened mouth,
hyperthelorismus) has systolic murmur in the second intercostal to the right of the sternum. The
murmur passes to the neck and along the sternum left edge. The pulse on the left brachial artery
Dr.Hur A.Salman - OdNMU
is weakened. BP on the right arm is 110/60 mm Hg, on the left - 100/60 mm Hg. ECG results:
hypertrophy of the right ventricle. What defect is the most probable?
A Aortic stenosis
B Defect of interventricular septum
C Defect of interatrial septum
D Coarctation of the aorta
E Open aortic duct

Don’t afraid from MURMURS  

120-A 1,5-year-old child fell ill acutely with high temperature 38oC, headache, fatigue. The
temperature declined on the fifth day, muscular pain in the right leg occured in the morning, there
were no movements and tendon reflexes, sensitivity was reserved. What is the initial diagnosis?
A Polyomyelitis  is an acute infectious disease that is caused by one of three types of poliovirus
and is characterized by the large range of clinical forms (from abortive to paralytic one).
B Viral encephilitis
C Polyartropathy
D Osteomyelitis
E Hip joint arthritis
Dr.Hur A.Salman - OdNMU
Forms of poliomyelitis without the CNS damage:
I. Innaparant (virus carrying).
ІІ. A ortive s all ill ess .
Forms of poliomyelitis with the CNS damage:
І. o paralyti or e i geal.
ІІ. Paralyti :
1. Spinal (neck, pectoral, lumbar, limited or widespread).
2. pontinus.
3. Bulbar.
4. Pontospinal.
5. Bulbospinal.
6. Bulbopontospinal.

Spinal form of poliomyelitis

Differential diagnostics of poliomyelitis with similar forms of enterovirus infection


Signs poliomyelitis poliomyelitis like forms of enterovirus
infection

Latent period 5-35 days 2-10 days

Toxic syndrome severe Mild or moderate

fever high moderate

Catarrhal signs mild typical (herpangina)

Preparalytic period 2-3 days 5-7 days


duration

Skin rashes absent Often present

paralysis peripheral paralysis, stable peripheral paresis, usually disappears

the tendon reflexes absent Decreased or normal

Muscular atrophy typical Rare, some muscles

Renewal of function Less damaged motoneurons in Practically complete in 3-4 wks


a year

CSF changes As in serous meningitis Not typical

Virological studies Poliovirus Coxackie A, Е


Dr.Hur A.Salman - OdNMU
121-A 3-year-old child has been delivered to a hospital in soporose state with considerable
amyotonia, inhibition of tendon and periosteal reflexes. Miosis and asthenocoria are also
present. Corneal reflexes are preserved. Pulse is rapid and weak. AP- 80/50 mm Hg. The
parents suspect the child of accidental taking some tablets. Such clinical presentations are
typical for intoxication with the following tableted drugs:
A Tranquilizers  anxiolytics, antipsychotics. Mood stabilizers : drugs which is designed for the
treatment of anxiety, fear, tension, agitation, and disturbances of the mind, specifically to reduce
states of anxiety and tension
B Antropine drugs
C Antihypertensive drugs
D Barbiturates
E Beta-2-adrenoceptor agonists
** Soporose: characterized by or manifesting morbid sleep or sleepiness
**Amyotonia : deficiency of muscle tone.

122-A 2 m.o. child with birth weight 5100 g has jaundice, hoarse cry, umbilical hernia, physical
development lag. Liver is +2 cm enlarged, spleen is not enlarged. In anamnesis: delayed
falling-away of umbilical cord rest. In blood: Hb- 120 g/L, erythrocytes - 4,5x1012/L,
ESR- 3 mm/h. Whole serum bilirubin is 28 mcmole/L, indirect - 20 mcmole/L, direct - 8
mcmole/L. What is the most probable diagnosis?
A Congenital hypothyreosis  congenital Hypothyroidism
B Congenital hepatitis
C Hemolitic anemia
D Conjugated jaundice
E Cytomegalovirus infection
** Lag : mean delay , physical development lag :mean delay in growth (retardation)

123-A 5-year-old child developed an acute disease starting from body temperature rise up to
38,5oC, running nose, cough and conjunctivitis. On the 4th day the child presented with
maculo-papular rash on face. Body temparature rose again up to 39,2oC. Over the next few
days the rash spread over the whole body and extremities. Mucous membrane of palate was
hyperemic, there was whitish deposition on cheek mucous membrane next to molars. What is
your provisional diagnosis?
A Measles  Koplik’s spot
B Acute viral respiratory infection
C Yersinia
D Enterovirus diseases
E Rubella
Review Q 15 – 16 / 77
Dr.Hur A.Salman - OdNMU
124-A 3 year old child fell acutely ill, body temperature rose up to 39,5oC,
the child became inert,
there appeared recurrent vomiting, headache. Examination revealed positive meningeal
symptoms, after this lumbal puncture was performed. Spinal fluid is turbid, runs out under
pressure, protein concentration is 1,8 g/l; Pandy reaction is +++, sugar concentration is 2,2
millimole/l, chloride concentration - 123 millimole/l, cytosis is 2,35x109 (80% of neutrophils, 20%
of lymphocytes). What is the most probable diagnosis?

A Purulent meningitis  Neutrophil / Polymorphonuclear cell / PMN and hx of purulent infect.


B Serous viral meningitis  Lymphocyte predominant and sugar normal , hx of viral infection.
C Serous tuberculous meningitis  Lymphocyte predominant and sugar low , hx of TB
D Subarachnoid haemorrhage  Hx of trauma , hemorrhage also no infection , no T
E Brain tumour

Differential diagnosis of meningitis


Purulent
Tuberculosis bacterial Subarachnoid
Signs Meningismus Viral meningitis
meningitis meningitis hemorrhage

Colorless, Colorless, White-yellow bloody, after


Color, Colorless,
transparent or xanthochromic or green, muddy settling –
transparence transparent
opalescent or opalescent xanthochromic
Pressure(mm. H 250-500
below 180-
2O), flow out 200-300 250-500 250-400
200 jet, sometimes
speed (drops 60-90 60-90 rare drops
> 70 or jet
50-80
per 1 minute)
It is hard to
500- count in the
cytosis (in 1 mk
2-12 20-800 200-700 1000 andmore first days,
l.)
from 5-7
day15-120

cytogram from 5-7 day


lymphocytes, % 80-85 80-100 40-60 0-30 lymphocytes
neutrophyls, % 15-20 0-20 20-50 30-100 prevail

protein, g/l 0.16-0.33 0.33-1.0 1.0-3.3 0.66-16.0 0.66-16.0


sedimentation +++(++++)
– +(++) +++(++++) +++
tests (Pandy)
cellular-
proteinon the cellular-
Dissociations Absent low level (from protein-cellular proteinon the –
high level
8-10 day –
protein-cellular)
Often rough Often as a
Fibrin pellicle – - in 3-5 % sediment Rare
in30-40 %
Normal or
For 2- slightly less than
glucose, mmol/l 2.2-3.3 2.2-3.3 normal
3 weeks1.0-2.0 normal
Dr.Hur A.Salman - OdNMU
125-A 13 y.o. girl complains of having temperature rises up to febrile figures for a month, joint
ache, periodical skin rash. Examination revealed steady enhancing of ESR, LE-cells. What is the
most probable diagnosis?
A Systematic lupus erythematosus  Review Q 54
B Juvenile rheumatoid arthritis
C Systematic scleroderma
D Acute lymphoblast leukosis
E Rheumatics

126-A child is 1 year old. Ater the recent introduction of complementary feeding the child has
presented with loss of appetite, diarrhea with large amounts of feces and occasional vomiting,
body temperature is normal. Objectively: body weight is 7 kg, the child is very pale, there are
edemata of both legs, abdomen is significantly enlarged. Coprogram shows many fatty acids
and soaps. The child has been diagnosed with celiac disease and administered the gluten-free
diet. What is to be excluded from the ration?
A Cereals - wheat and oats
B Milk and dairy products
C Fruit
D Animal protein
E High digestible carbohydrates
** Coprogram: mean general stool test 
autoimmune disorder that primarily affects
the small intestine. Classic symptoms include
gastrointestinal problems such as chronic
diarrhoea, abdominal distention, malabsorption,
loss of appetite and among children failure to
grow normally.
This often begins between six months and two
years of age
Coeliac disease is caused by a reaction to gluten,
which are various proteins found in wheat and in
other grains such as barley and rye. Moderate
quantities of oats, free of contamination with
other gluten-containing grains, are usually
tolerated.

127-A 7-year-old child was brought to a doctor for a check. The child has a 4-year history of
bronchial asthma, asthma attacks occur mainly in spring and summer. Allergy tests revealed
hypersensitivity to poplar seed tufts, field herbs. What recommendation should be given?
A Specific hyposensitization
B Physiotherapy
C Treatment at a health resort
D Phytotherapy
E Needle reflexotherapy

128- A 9-month-old child presents with fever, cough, dyspnea. The symptoms appeared 5 days
ago after a contact with a person having ARVI. Objectively: the child is in grave condition.
Temperature of 38oC, cyanosis of nasolabial triangle is present. RR- 54/min, nasal flaring
while breathing. There was percussion dullness on the right below the scapula angle, and
tympanic sound over the rest of lungs. Auscultation revealed bilateral fine moist rales
Dr.Hur A.Salman - OdNMU
predominating on the right. What is the most likely diagnosis?
A Acute pneumonia  complication from ARVI , from percussion we can determined that is focal
pneumonia
B ARVI  be careful , he have it already but then complicated.
C Acute laryngotracheitis  Croup , stridor cough.
D Acute bronchitis
E Acute bronchiolitis
ARVI mean acute respiratory viral infection .

DIAGNOSTIC CRITERIA OF PNEUMONIA


Clinical  Increasing of body temperature above 38°C, hyperthermia during 3-5 days and more; dry cough
at first, then moist; signs of intoxication and respiratory failure; at palpation increased voice fremitus, at
percussion over the affected area of lung a shortened tympanitis, at auscultation - hard breathing, first dry,
then moist sonorous small and medium bubble wheezing, possible crepitation over the size of lesions,
enforced bronchophonia.

X-Ray - Infiltrative shadows in the form of foci of different size and intensity, darkening of one or
several segments, lobe or several foci.

Differential diagnosis of focal pneumonia, bronchitis and bronchiolitis

Functional changes in lungs X-ray changes in lungs


Percussi- Type of Rales Root of Vascular Local focal
Diseases on sighs breath the lungs pattern shadows
Focal Lung Harsh, Local fine Widening Non Mostly Different
pneumonia sound focally moist or structive on intensive size and
with box weak with crepitation both sides or on the density
inflection bronchial on the side of side of lesion sometimes
inflection lesion confluent
Simple Lung Harsh Disseminat Widening Intensive Absent
bronchitis sound ed moist nonstructive on the
with box and on both sides both sides
inflection dry rales
Obstructive Bandbox Harsh Disseminat Widening Intensive Absent
bronchitis sound ed dry rale nonstructive on the both
s on both sides sides,swelling
of lungs
Bronchioli- Bandbox Harsh Disseminat Widening Intensive on Absent
tis sound ed moist nonstructive the both sides,
fine rales on both sides sharp swelling
of lungs
Dr.Hur A.Salman - OdNMU
129-An 8 y.o. boy complains of constant cough along with discharge of greenish sputum, dyspnea
during physical activities. At the age of 1 year and 8 months he fell ill for the first time with
bilateral pneumonia that had protracted course. Later on there were recurrences of the disease
5-6 times a year, during the remission periods there was constant productive cough. What
examination results will be the most important for making a final diagnosis?
A Bronchography x ray with contrast media injected through the bronchus. , pt have symptoms of
bronchitis
B Roentgenography of thorax organs
C Bacterial inoculation of sputum
D Bronchoscopy
E Spirography

130- A mother of a 5 y.o. girl consulted a doctor about doughter's involuntary urination at night,
nightmares, sleep disorders, slow gaining of body weight. Objectively: malnutrition, intellectual
development is good, the girl can read and explains common situations quite adultly. Her skin is
very pale, liver is enlarged in size. Her mother suffers from holetithiasis. What type of diathesis
is the most probable in the child's case?
A Gouty diathesis  keep it , just connect mother had GB stone, so the daughter have gout 
B Urine acid diathesis
C Exudative diathesis
D Allergic diathesis
E Lymphohypoplastic diathesis
Holetithiasis = choletithiasis

131-A 10 year old girl complains about abdominal pain that is arising and getting worse after eating
rough or spicy food. She complains also about sour eructation, heartburn, frequent constipations,
headache, irritability. She has been suffering from this for 12 months. Objectively: the girl's diet is
adequate. Tongue is moist with white deposit at the root. Abdomen is soft, painful in its epigastric
part. What study method will help to make a diagnosis?
A Esophagogastroduodenoscopy  to exclude any upper GIT pathology like GERD, Gastritis ,
peptic ulcer , esopheageal sphincter (achalasia) and so on ……
B Intragastral pH-metry
C Fractional examination of gastric juice
D Contrast roentgenoscopy
E Biochemical blood analysis
Dr.Hur A.Salman - OdNMU

132-A 40 h.o. child age has hyperosthesia, CNS depression, dyspepsia. Sepsis is suspected. What
should the differential diagnosis be made with?
A Hypoglycemia
B Hypocalcemia
C Hyperbilirubinemia
D Hyperkaliemia
E Hypomagnesemia

133-Examination of a full-term 6-day-old infant revealed that different areas of skin had
erythemas, flaccid bubbles, eroded surface, cracks, peeling of the epidermis looking like being
scalded with boiling water. There was positive Nikolsky's symptom. General condition of the child
was serious. The child was restless, hypersensitive, febrile. What is the most likely diagnosis in this
case?
A Ritter's exfoliative dermatitis
Dr.Hur A.Salman - OdNMU
B Neonatal phlegmon
C Finger's pseudofurunculosis
D Neonatal pemphigus
E Epidermolysis

Scalded skin syndrome (Ritter disease): A relatively rare syndrome caused by exfoliative toxin takes
the form of superficial fragile blisters that burst, leaving a tender base. The patient often is febrile
and occasionally has mucopurulent eye discharge. Place special emphasis in making this diagnosis
because it can often be mistaken for erythema multiforme and/or toxic epidermal necrolysis, which
are treated with corticosteroids. Misdiagnosis delays treatment and allows exfoliation to progress,
and corticosteroid therapy could potentiate bacterial superinfection. Although the mortality rate is
low in children with this entity, most fatalities are caused by delays in diagnosis.

Examination shows superficial fragile blisters that burst, leaving a tender base. Skin sloughs easily
when touched, called Nikolsky sign (see ) Fever is often present. A mucopurulent eye discharge
may be present. Misdiagnosis must be avoided.

Nikolsky sign.

Therapy for this, as with any S aureus toxin–mediated disease, should be aimed at eradicating the
focus of infection and ending toxin production. Administer large doses of intravenous
antistaphylococcal agents, such as oxacillin (150 mg/kg/d) or a first-generation cephalosporin,
such as cefazolin (100 mg/kg/d)
Dr.Hur A.Salman - OdNMU
40,1o
134-A 1,5 y.o. child fell seriously ill: chill, body temperature rise up to , then rapid dropping
to 36,2o , skin is covered with voluminous hemorrhagic rash and purple cyanotic spots.
Extremities are cold, face features are sharpened. Diagnosis: meningococcosis, fulminant
form, infection-toxic shock. What antibiotic must be used at the pre-admission stage?
A Soluble Levomycetine succinate  Chloramphenicol
B Penicillin
C Lincomycin
D Gentamycin
E Sulfamonometoxin

135-A 10 year old boy suffers from chronic viral hepatitis type B with maximal activity. What
laboratory test can give the most precise characteristic of cytolysis degree?
A Transaminase test  liver enzymes Alanine transaminase (ALT/ SGPT) and Aspartate
transaminase (AST/ SGOT)
B Weltman's coagulation test
C Takata-Ara test
D Prothrombin test
E Test for whole protein

136-A 6 y.o child complains of thirst, polyuria, increased appetite for 2 months with weight loss
for 3 kg. There has been nocturnal enuresis during last week. On examination: hyperglycemia 14
mol/L. The diagnosis is diabetis mellitus I type. What is the genesis of this disease?
A Autoimmune
B Viral
C Bacterial
D Neurogenic
E Virus-bacterial

137-A 10 y.o. child who is at oligoanuretic stage of acute renal insufficiency has got sensations of
pricking in the mucous membrane of oral cavity and tongue, extremities numbness, reduced
reflexes, respiratory disturbance, arrhythmia. What are these symptoms caused by?
A Hyperkaliemia   K+
Dr.Hur A.Salman - OdNMU
B Hyponatremia   Na+
C Hyperazotemia   nitrogen waste in blood
D Acidosis  if metabolic PH , HCO3 , PCO2 
E Alkalosis  if metabolic PH , HCO3 , PCO2

138-Examination of a 12 year old child revealed diffuse thyroid enlargement of the II degree.
Heart auscultation revealed dullness of heart sounds, heart rate was 64/min. The child has
frequent constipations, anemia. Concentration of thyreoglobulin antibodies is increased. What
disease might have caused such symptoms?
A Autoimmune thyroiditis  review Q 111
B Diffuse toxic goiter
C Thyroid carcinoma
D Thyroid hyperplasia
E Endemic goiter

139- An 8-year-old girl has been admitted to the cardiology department. Objectively: there is a
skin lesion over the extensor surfaces of joints with atrophic cicatrices, depigmentation,
symmetrical affection of skeletal muscles (weakness, edema, hypotrophy). What disease are these
changes most typical for?
A Dermatomyositis  is an idiopathic inflammatory myopathy (IIM) with characteristic cutaneous
findings. It is a systemic disorder that most frequently affects the skin and muscles, but may also
affect the joints, the esophagus, the lungs, and, less commonly, the heart. Calcinosis is a
complication of dermatomyositis that is observed most often in children and adolescents. An
association between dermatomyositis and cancer has long been recognized.

B Systemic scleroderma CREST syndrome .


C Nodular periarteritis  vasculitis .
D Systemic lupus erythematosus  SLE explained above.
E Reiter's disease  Reactive Arthritis.
Dr.Hur A.Salman - OdNMU
Dermatomyositis.
# Bohan and Peter suggested 5 subsets of myositis, as follows:
- Dermatomyositis
- Polymyositis
- Myositis with malignancy
- Childhood dermatomyositis/polymyositis
- Myositis overlapping with another collagen-vascular disorder

Clinical Presentation
Skin eruption/hair loss

A B
A : is diffuse alopecia with a scaly scalp dermatosis is common in dermatomyositis.
- Muscle involvement
B: Juvenile dermatomyositis ( Children)  Calcinosis is a complication of it .

The characteristic and possibly pathognomonic cutaneous features of dermatomyositis are the
heliotrope rash and Gottron papules.
This rash is rarely observed in other disorders; therefore, its presence is highly suggestive of
dermatomyositis.

C D
C is Heliotrope rash in a woman with dermatomyositis.
D & E - Gottron papules
Gottron papules are found over bony prominences, particularly the metacarpophalangeal joints,
the proximal interphalangeal joints, and/or the distal interphalangeal joints
Dr.Hur A.Salman - OdNMU
Also include malar erythema, poikiloderma extensor surfaces of the arm, the vee of the neck or the
upper part of the back (Shawl sign) or on the lateral thighs (Holster sign).
in a photosensitive distribution (Patients rarely complain of photosensitivity), violaceous erythema
on the extensor surfaces, and periungual telangiectases and cuticular changes.
nail-fold telangiectasia are present in this patient.

E
In some patients, particularly those with anti-synthetase antibodies, ulceration over the knuckles
occurs These lesions may require surgical intervention.

F G
F : Ulceration over the dorsal and lateral fingers in a patient with dermatomyositis.
G : Calcifying panniculitis in a patient with dermatomyositis.

Other cutaneous findings


panniculitis and urticaria, as well as changes of hyperkeratosis of the palms known as mechanic's
hands. Other findings include cutaneous mucinosis, follicular hyperkeratosis, hyperpigmentation,
ichthyosis, white plaques on the buccal mucosa, cutaneous vasculitis, and a flagellate erythem
- joint swelling, changes associated with Raynaud phenomenon, and abnormal findings on
cardiopulmonary examination. The arthritis is nondeforming.

140- An 8-year-old child with a 3-year history of diabetes was hospitalized in hyperglycemic coma.
Specify the initial dose of insulin to be administered:
A 0,1-0,2 U/kg of body weight per hour
B 0,05 U/kg of body weight per hour
C 0,2-0,3 U/kg of body weight per hour
D 0,3-0,4 U/kg of body weight per hour
E 0,4-0,5 U/kg of body weight per hour

141- A 12-year-old girl undergoes regular gastroenterological check-ups for duodenal ulcer,
Dr.Hur A.Salman - OdNMU
biliary dyskinesia. What is the recommended frequency of anti-relapse treatment?
A Twice a year  or tell you 1 time each 6 months.
B Every two months
C Every 3 months
D Once a year
E Three times a year

142- A 13 y.o. teenager who suffers from hemophilia A was taken to the hospital after a fight at
school. His diagnosis is right-sided hemarthros of knee joint, retroperitoneal hematoma. What
should be primarily prescribed?
A Fresh frozen plasma  this given in Hemophilia B not A !!!
B Aminocapronic acid
C Washed thrombocytes
D Placental albumin
E Dry plasma
NOTE
Hemophilia A deficiency of F VIII preferably treated by Cryoprecipitate .
and for Hemophilia B treated by fresh frozen plasma using

143- A 3 m.o. child fell seriously ill, body temperature rised up to 37,8oC, there is semicough. On
the 3-rd day the cough grew worse, dyspnea appeared. On percussion: tympanic sound above
lungs, on auscultation: a lot of fine moist and wheezing rales during expiration. What is the most
probable diagnosis?
A Acute respiratory viral infection, bronchiolitis  review Q 128.
B Acute respiratory viral infection, bronchopneumonia
C Acute respiratory viral infection, bronchitis
D Acute respiratory viral infection, bronchitis with asthmatic component
E Acute respiratory viral infection, focal pneumonia

Bronchiolitis: is blockage of the small airway in the lungs due to a viral infection. It usually only
occurs in children less than two years of age. Symptoms may include fever, cough, runny
nose, wheezing, and breathing problems. More severe cases may be associated with nasal flaring,
grunting, or the skin between the ribs pulling in with breathing. If the child has not been able to
feed properly, signs of dehydration may be present.
respiratory syncytial virus (72% of cases) or human rhinovirus (26% of cases)

144-On the 1st day of life a full-term girl (2nd labour) weighing 3500g, with Apgar score of 8
points, presented with jaundice. Indirect bilirubin of blood - was 80 micromole/l, 6 hours later -
160 micromole/l. What is the optimal method of treatment?
A Exchange blood transfusion  review 22 , 41
B Phototherapy
C Infusion therapy
D Phenobarbital treatment
E Enterosorbents
Dr.Hur A.Salman - OdNMU

145
A child was born at a gestational age of 34 weeks in grave condition. The leading symptoms
were respiratory distress symptoms, namely sonorous and prolonged expiration, involving
additional muscles into respiratory process. The Silverman score at birth was 0 points, in 3
hours it was 3 points with clinical findings. Which diagnostic study will allow to diagnose the form
of pneumopathy?
A X-ray of chest
B Clinical blood test
C Determination of blood gas composition
D Proteinogram
E Immunoassay
Dr.Hur A.Salman - OdNMU
146-A 10-year-old girl8 consulted a doctor about thirst, frequent urination, weight loss. She has
been observing these symptoms for about a month. Objectively: no pathology of internal organs
was revealed. What laboratory analysis should be carried out in the first place?
A Blood glucose analysis on be an empty stomach  Fast blood sugar (FSB), signs of Diabetes?
B Glucose in urine test on the base of daily diuresis
C Acetone in urine test
D Glucose tolerance test
E Glucosuric profile

147- A 6-year-old child complains of frequent liquid stool and vomiting. On the 2nd day of
desease the child presented with inertness, temperature rise up to 38,2oC, Ps- 150 bpm, scaphoid
abdomen, palpatory painful sigmoid colon, defecation 10 times a day with liquid, scarce stool
with mucus and streaks of green. What is a provisional diagnosis?
A Shigellosis
B Salmonellosis
C Escherichiosis
D Intestinal amebiasis
E Yersiniosis
Review Q 78 see Difference between shig. And Sallmon.
 CLINICAL PICTURE
- The transmission of the infection is realized through contaminated food-
stuffs and water. Infection of food-stuffs, water, different objects happens
due to direct contamination by infected excrements, through dirty hands
and also with participation of flies.
- Incubation period is from 2 to 5 days, rarely – 7 days.
- Symptoms begin with sudden onset of high-grade fever, abdominal cramps &
watery diarrhea
- Subsequently the diarrhea became mucoid, of small volume & mixed with
blood. This is accompanied by abdominal pain, tenesmus & urgency. Fecal
incontinence may occur.
- Physical signs are those of dehydration beside fever, lower abdominal
tenderness & normal or increased bowel sounds
148-A 4-year-old boy had untimely vaccination. He complains of painful swallowing, headache,
inertness, fever. Objectively: the child is pale, has enlarged anterior cervical lymph nodes,
swollen tonsils with cyanotic hyperemia, tonsils are covered with gray-white pellicles which
cannot be easily removed. When the pellicles are forcibly removed, the tonsils bleed. What is
the most likely diagnosis?
A Oropharyngeal diphtheria
B Lacunar tonsillitis
C Pseudomembranous tonsillitis
D Infectious mononucleosis
E Follicular tonsillitis
Dr.Hur A.Salman - OdNMU

149-After a 10-year-old child had been bitten by a bee, he was delivered to a hospital. There were
lip, face and neck edemata. The patient felt hot and short of breath. Objectively: breathing was
laboured and noisy. There were foamy discharges from the mouth, cough. The skin was pale
and cold. There was bradypnoea. Heart sounds were muffled and arrhythmic. Thready pulse
was present. What diagnosis was made by the expert in resuscitation?
A Anaphylactic shock
B Quincke's edema  Angioedema.
C Bronchial asthma
D Acute cardiovascular collapse
E Cerebral coma

150- A 13-year-old girl complains of fever up to 37,4oC during the last 2 months after recovering
from ARVI. Objectively: malnutrition, diffuse grade II enlargement of the thyroid gland feeling
dense on palpation, exophthalmos, tachycardia. What kind of pathological syndrome is it?
A Thyrotoxicosis  is the condition that occurs due to excessive thyroid hormone of any cause and
therefore includes hyperthyroidism.
B Hypothyroidism
C Hypoparathyroidism
D Hyperparathyroidism
E Thymomegaly
Dr.Hur A.Salman - OdNMU
151-A 3-year-old girl presents with pertussis-like cough with thick sputum. There have been
persistent changes in lungs since the age of 6 months when she was first diagnosed with acute
pneumonia. Chloride concentration in the perspiration is 112 mEq/l. The child has been
diagnosed with mucoviscidosis. What is the basis for autosomal recessive disease -
mucoviscidosis?
A Inadequate transport of sodium and chloride ions  Cystic Fibrosis .
B α1-antitrypsin deficiency
C Deposition of calcium triphosphates and carbotates in the alveoles
D Pulmonary cysts
E Pulmonary artery hypoplasia
Review Q 79 / 112

152-A newborn has purulent discharges from the umbilical wound, the skin around the navel is
swollen. The baby's skin is pale, with a yellow-gray tint, generalized hemorrhagic rash is present.
What is the most likely diagnosis?
A Sepsis  severe infection that has spread via the bloodstream .
B Hemorrhagic disease of the newborn
C Hemolytic disease of the newborn
D Thrombocytopathy
E Omphalitis

153-From urine of a 14-year-old boy with the exacerbation of secondary obstructive pyelonephritis
Pseudomonas aeruginosa was isolated with a titer of 1000000 microbes per 1 ml. Which
antibiotic is most advisable to be administered in this case?
A Ciprofloxacin  good in Urinary tract infection. (P.aeruginosae , E.coli , Klebsiella)
B Ampicillin
C Cefazolinum
D Azithromycin
E Chloramphenicol

154-A 14-year-old boy with a history of chronic tonsillitis and sinusitis has developed a feeling of
heart irregularities and additional pulse. HR- 83/min. ECG results: regular impulses with no
visible P wave that occur every two sinus contractions, QRS complex is dramatically deformed and
prolonged to over 0,11 s, T wave is discordant followed by a complete compensatory pause. Specify
the arrhythmia type:
A Trigeminal extrasystole  each 2 wave normal the 3rd one extrasystole
B Bigeminal extrasystole  each 1 wave normal the 2nd one extrasystole
C Partial AV-blockade
D Complete AV-block
E Left bundle branch block  wide QRS, [ V1 QS (or rS) W , V6 R notched (M)

Extrasystole
(Ectopic)
Dr.Hur A.Salman - OdNMU
155-An 8-year-old girl periodically has sudden short-term heart pain, sensation of chest
compression, epigastric pain, dizziness, vomiting. Objectively: the patient is pale, respiratory
rate - 40/min, jugular pulse is present. Ps- 185 bpm, of poor volume. AP- 75/40 mm Hg. ECG
taken during an attack shows ectopic P waves, QRS wave is not deformed. At the end of
an attack a compensatory pause is observed. The most likely cause of the attack is:
A Paroxysmal atrial tachycardia  PAT = Sudden atrial increased heart rate. , attention QRS not
deformed
B Sinus tachycardia
C Paroxysmal ventricular tachycardia  p wave normal , QRS deformed .
D Complete AV-block
E Atrial fibrillation

156-A 10-year-old child with a history of nonrheumatic carditis has periodic attacks manifested by
heart pain, dyspnea, pallor, high blood pressure, a dramatic increase in heart rate up to 180/min.
What drug would be most effective to treat this patient?
A Obsidan  review Q 96
B Procainamide
C Lidocaine
D Verapamil
E Ajmaline

157-A 1-month-old child became restless and presented with an increase in head sweating. It's
known from the history that the child has been fed with cow's milk since birth (September 5).
Examination revealed craniotabes. A doctor administered a course of UV radiation. Decide, if
the child needs ergocalciferol:
A 2-2,5 months after the UVR withdrowal  UVR = Ultra-violate radiation , here the child have
thinning of bone (may be richets?) Vit D deficiency ? you can give ergocalciferol after 2 – 2.5
Dr.Hur A.Salman - OdNMU
months after stop exposure to UV radiation .
cholecalciferol (vitamin D3) is produced naturally by the skin when exposed to ultraviolet light.
B Does not need
C In combination with UVR
D Immediately after the UVR withdrowal
E A month after the UVR withdrowal

** Craniotabes is softening or thinning of the skull in infants and children, which may be normally
present in newborns
** Ergocalciferol, also known as vitamin D2 and calciferol, is a type of vitamin D found in food and
used as a dietary supplement.. As a supplement it is used to prevent and treat vitamin D deficiency.

158-15 minutes after the second vaccination with DTP vaccine a 4-month-old boy exhibited the
symptoms of Quincke's edema. What medication should be given for emergency aid?
A Prednisolone  to relieve hypersensitivity reaction after DTP vaccine
B Heparin
C Adrenalin
D Furosemide
E Seduxen

159-A baby is 3 months old. The mother consulted a pediatrician about lack of breast milk. After
several test weighings it was found that the child had to receive supplementary feeding. What is
the optimal milk formula for this child?
A Malysh
B Milk formula № 2
C Milk formula № 3
D Whole cow's milk
E Malutka

160-Examination of a newborn revealed skin redness that appeared immediately after birth and
reached the maximum intensity on the second day of life. What is your provisional diagnosis?
A Simple erythema  redness of the skin, may be the result of increased temperature from climatic
conditions, local inflammation, or infection. It may also appear as a sign of skin irritation, allergy, or
other dermatoses.
B Toxic erythema
C Transient erythema
D Erythema nodosum
E Annular erythema

161-A child is 2 days old. He was born with a weight of 2900 kg, body length of 50 cm. On
examination the skin is intensely red, elastic, with preserved turgor. Puerile respiration is
present. Respiration rate - 40/min, cardiac sounds are rhythmic, sonorous. HR- 138/min. The
Dr.Hur A.Salman - OdNMU
abdomen is soft. The liver extends 2 cm below the costal margin. Diuresis is sufficient. Stool is
in form of meconium. What is the most likely diagnosis?
A Physiological erythema of the newborn as the child have normal parameters and condition just
this redness is physiologically appeared and dissolve without any
B Toxic erythema of the newborn
C Neonatal phlegmon
D Erysipelas
E Exfoliative Ritter's dermatitis  Q 81 , 133

162-A full-term baby was born with body weight of 3200 g, body length of 50 cm, Apgar score - 8-
10 points. What is the optimum time for the first breast-feeding?
A First 30 minutes
B First 6 hours
C First 24 hours
D First 48 hours
E After 48 hours

163-A 3-year-old child has been taken to a pediatrician. He has no recent history of any diseases.
Objective examination revealed no pathology of the internal organs. The child needs the routine
immunization against the following disease:
A Poliomyelitis  review Q 120
B Diphtheria and tetanus
C Measles, rubella, parotitis
D Pertussis
E Type B hepatitis

164-An 11-year-old girl has been immunized according to her age and in compliance with the
calendar dates. What vaccinations should the children receive at this age?
A Diphtheria and tetanus
B TB
C Polio
D Hepatitis B
E Pertussis

165-A 6-year-old child has duodenal ulcer. What antibacterial drug should be co-administered
together with metronidazole and De-Nol in order to eradicate Helicobacter pylori infection?
A Amoxicillin
B Tetracycline
C Oleandomycin
D Biseptol
E Sulfadimethoxinum

166-A baby born after fast labour has palsy of hand muscles. Grasp reflex is absent, as well as
hand-to-mouth reflex. Hand sensitivity is absent. What is the most likely diagnosis?
A Dejerine-Klumpke palsy  distal brachial plexus paralysis (C7-8-T1)
B Duchenne-Erb's palsy  proximal brachial plexus paralysis (C5-6)
C Total lesion of the brachial plexus  both proximal and distal (A+B)
D Muscle paresis
E Bernard-Horner syndrome  oculosympathetic paresis
Dr.Hur A.Salman - OdNMU

167-A child is 12 years old. He complains of a dull aching pain in the epigastrium and right
hypochondrium, that is getting worse after taking fatty or fried food, headache, weakness,
nausea, low-grade fever. Abdominal palpation reveals a marked resistance of muscles in the
right hypochondrium, positive Kerr's, Ortner's, Murphy's symptoms. What is the most likely
diagnosis?
A Chronic cholecystitis  chronic inflammation of Gall bladder .
B Acute appendicitis
C Viral hepatitis
D Acute gastritis
E Acute pancreatitis

168-A 3-month-old girl presents with rhinitis, dyspnea, dry cough. These manifestations has been
observed for two days. Objectively: the child has pale skin, acrocyanosis, shallow respiration at
the rate of 80/min. Percussion reveals handbox resonance over the whole surface of lungs,
massive fine rales. What is the most likely diagnosis?
A Acute bronchiolitis  review Q 128 , 143
B Pneumonia
C Mucoviscidosis
D Foreign body of the airway
E Acute bronchitis

169-During the first home visit to a full-term boy after his discharge from the maternity hospital a
pediatrician revealed a symmetrical swelling of mammae without skin changes over them,
swelling of the scrotum. The body temperature was of 36,5oC. The baby was calm, sucked
the mother's breast actively. What condition should you think of?
A Hormonal crisis of the newborn
Dr.Hur A.Salman - OdNMU
B Neonatal mastitis
C Sclerema
D Necrotic neonatal phlegmon
E Congenital adrenal dysfunction
Mammae = breast .
170-A full-term neonate weighing 4500 g was born asphyxiated with Apgar score of 4-6 points.
During the delivery shoulder dystocia occurred. Neurologic assessment revealed non-focal
neurologic symptoms, total flaccid paresis of the upper extremities since the arm was atonic and
pronated. Grasping, Babkin's and Moro's reflexes were absent. What segments of spinal cord
had been affected?
A V - ThI  ( C5 -6-7-8-T1 ) total brachial plexus paralysis , proximal and distal . ERB’s +
KLUMPKE . Review Q 128 , 143
B I - II
C III - IV
D ThI - ThV
E ThVI - ThVІІ

171-A newborn (mother's I pregnancy) weighing 3500 g presents with jaundice, lethargy, reduced
reflexes. Objectively: second grade jaundice of skin with saffron tint, liver - +2 cm, spleen - +1
cm. Urine and feces are yellow. Blood count: Hb- 100 g/l, RBCs - 3,2x1012/l, WBCs -
18,7x109/l, mother's blood type - 0(I) Rh(+), baby's blood type - А(II) Rh(-), bilirubin - 170
mmol/l, indirect fraction. ALT, AST rates are normal. What disease is the child most likely to
have?
A Hemolytic disease of newborn, AB0-conflict
B Perinatal hepatitis
C Hemolytic disease of newborn, Rh-conflict
D Biliary atresia
E Physiologic jaundice
Review Q 11

172-A 10-year-old girl was admitted to a hospital with carditis presentations. It is known from the
anamnesis that two weeks ago she had exacerbation of chronic tonsillitis. What is the most
likely etiological factor in this case?
A Streptococcus
B Staphylococcus
C Pneumococcus
D Klebsiella
E Proteus

173-All the joints on the left elbow of a newborn are extended, the whole arm hangs vertically
along the trunk with the forearm pronated. Active movements in the elbow joint are absent but
present in the shoulder joint. The hand is flattened, atrophied, cold to the touch, hangs passively.
Grasp reflex and hand-mouth reflex on the affected side are missing. Haemogram values are
normal. What is the most likely diagnosis?
A Inferior distal obstetrical paralysis  Review Q 128 , 143
B Osteomyelitis
C Proximal obstetrical paralysis
D Complete obstetrical paralysis
E Hypoxic-ischemic encephalopathy
Dr.Hur A.Salman - OdNMU
174-Head circumference of a 1-month-old boy with signs of excitement is 37 cm, prefontanel is
2x2 cm large. After feeding the child regurgitates small portions of milk; stool is normal in respect
of its volume and composition. Muscle tonus is within norm. What is the most likely diagnosis?
A Pylorospasm  child physical development normal .
B Meningitis  inflammation of meningea.
C Pylorostenosis  child physical development retarded . throwing vomiting / projectile
D Microcephaly  small head .
E Craniostenosis  premature closure of cranial suture due to abnormalities of skull development

175- 10 days after birth, a newborn developed a sudden fever up to 38,1oC. Objectively: the skin
in the region of navel, abdomen and chest is erythematous; there are multiple pea-sized blisters
with no infiltration at the base; single bright red moist erosions with epidermal fragments on the
periphery. What is your provisional diagnosis?

A Epidemic pemphigus of newborn  Is an acute contagious disease of staphylococcal nature.


Pathogenesis: the main pathogenic factor is sensitivity of the skin to infections, prematurity,
pregnancy toxicosis, birth injury. The source of infection is often the medical staff, mother and the
patients themselves
Clinical features. Emergence of small vesicles with thin tensed cover and serous-yellow
contents, tendency towards fusion and formation of large vesicles. Fever is possible. Vesicles are
converted into pustules, open, erosion slowly epithilizes. It is necessary to differentiate epidemic
pemphigus form syphilitic pemphigus. The latter arises on palms and soles, on an infiltrated base,
characterized by the presence of T. palladium in the contents of the vesicles, positive compliment
fixation test in child and mother.

First, the symptoms of the disease often occurs in infants born in 4-10 days, contagious. Most
damage occurred in the face, hands and other exposed parts, but also occur widely in the trunk and
limbs, palms and soles, often without damage, and sometimes damage also occurred in the mucosa
or spread finger nail bed caused by inflammation or suppurative paronychia. The initial stage of the
disease symptoms was not obvious, with the progress of the disease may include fever, body
temperature as high as 39 diarrhea, pneumonia, nephritis, meningitis or septicemia, resulting in
death in children.
Second, the lesions began to tip to the great big big red spots, blisters appeared on it quickly. Rapid
expansion of blisters, from pea to a large walnut or larger, blister and thin and break. A day or two,
become turbid fluid bullae or blister before the end of some yellow pus, but most of the bullae will
fester. Bullous be very full, after expanding and relaxation. After the blisters rupture, exposing
flushing, smooth erosion surface, formed after the thin crust. Bullae can also occur elsewhere, but
Dr.Hur A.Salman - OdNMU
as you can into a piece of the general erosion of pemphigus.
Treatment. Antibiotics, antistaphylococcal serum, transfusion of plasma, locally: aniline stains,
creams with antibodies.

B Syphilitic pemphigus  check 


C Streptococcal impetigo  non-follicular pustule on a hyperemic edematous base, small red spot
on the surface of which a vesicle of the size of a pinhead to a lentil forms in a few hours
- localization of the process is the face and the sides of the trunk and limbs
D Vulgar impetigo
E Atopic dermatitis  allergy
Naval region mean umbilicus

176-On the second day after preventive vaccination a 2-year-old boy presented with abdominal
pain without clear localization, body temperature rose up to 38oC. On the third day the child got
red papular haemorrhagic eruption on the extensor surfaces of limbs and around the joints.
Knee joints were edematic and slightly painful. Examination of other organs and systems
revealed no pathological changes. What is the most likely diagnosis?
A Haemorrhagic vasculitis  Henoch –Scholen Purpura check Q1
B Thrombocytopenic purpura
C Meningococcemia
D Urticaria
E DIC syndrome

177-On the 6th day of life a child got multiple vesicles filled with seropurulent fluid in the region of
occiput, neck and buttocks. General condition of the child is normal. What disease should be
suspected?
A Vesiculopustulosis  Is a disease of the newborn, characterized by multiple pustules, emerging
in the openings of ducts of eccrine sweat glands. In pathogenesis the main role is played by
maceration of the skin, prematurity, artificial feeding. Bathing is restricted during the disease.
Pustules are cleaned with aniline stains. The skin around the pustules is cleaned by disinfectant
solutions
B Impetigo neonatorum  
C Miliaria  
D Impetigo
E Epidermolysis bullosa  Q 175
Dr.Hur A.Salman - OdNMU

178-A patient is 14 years old. Cytochemical study of punctate revealed 40% of blasts, there was
negative reaction to peroxidase and with Sudan black, positive reaction to glycogen. Specify the
form of acute leukemia:
A Lymphoblastic  check table down  >> esp. red mark .. ::::
B Myeloblastic
C Monoblastic
D Promyelocytic
E Undifferentiated

All other options explained at table 


Dr.Hur A.Salman - OdNMU

179-Six months ago, a 5-year-old child was operated for CHD. For the last 3 weeks he has
complained of fever, heart pain, aching muscles and bones. Examination results: "white-coffee"
skin colour, auscultation revealed systolic murmur in the region of heart along with a noise in the
III-IV intercostal space. Examination of fingertips revealed Janeway lesions. What is your
provisional diagnosis?
A Infectious endocarditis
B Sepsis
C Nonrheumatic carditis
D Acute rheumatic fever
E Typhoid fever

Janeway lesions : are non-tender, mall erythematous or hemorrhagic macular or nodular lesions
on the palms or soles only a few millimeters in diameter that are indicative of infective endocarditis

Pathologically, the lesion is described to be a micro abscess of the dermis with marked necrosis and
inflammatory infiltrate not involving the epidermis. They are caused by septic emboli which
deposit bacteria, forming microabscesses. Janeway lesions are distal, flat, ecchymotic, and painless.

Osler's nodes : are painful, red, raised lesions found on the hands and feet. They are associated
with a number of conditions, including infective endocarditis, and are caused by immune
complex deposition. Their presence is one definition of Osler's sign
Dr.Hur A.Salman - OdNMU

180- The condition of a 3-year-old child with acute non-rheumatic myocarditis has suddenly
deteriorated: he presents with anxiety, acrocyanosis, peripheral edemata, dyspnea.
Auscultation of lungs reveals fine moist rales on both sides mainly in the lower parts. AP- 65/40
mm Hg. HR- 150/min, heart sounds are muffled, arrhythmic (extrasystole). Liver is +4 cm.
Oliguria is present. The child has been diagnosed with acute heart failure. Which method of
examination is most informative for assessing the child's status dynamics?
A Echocardiography  for checking Ejection fraction , wall thickness (like Hypertrophied) , check
valve , (pt. with symptoms of RT side HF) , cardiac Temponade & pericardial effusion
B ECG
C Diuresis monitoring
D Monitoring of K+, Na+ concentration in blood
E 24-hour monitoring of heart rhythm
Dr.Hur A.Salman - OdNMU
181-A hospital admitted an 11-year-old boy diagnosed with medium-severe asthma, exacerbation
period. In order to arrest the attacks the boy was administered broncholytic nebulizer therapy.
During the day the child's condition stabilized. What is the most appropriate method for further
monitoring of respiratory function in this patient?
A Peak flowmetry
B Spirometry
C Pneumotachometry
D Bronchodilatation tests
E Veloergometry

182-A full-term newborn was born with body weight of 4000 g, body length of 57 cm. Reaction to
the postnatal check was absent. There was diffuse cyanosis, heart rate of 80/min. What
resuscitation measures should be taken?
A Start ALV with a mask
B Give 100% oxygen
C Intubate the child and start ALV
D Start tactile stimulation
E Give an injection of naloxone
ALV = Artificial Lung Ventilation
Dr.Hur A.Salman - OdNMU

183 -A 2-year-old child in a satisfactory condition periodically presents with moderate proteinuria,
microhematuria. USI results: the left kidney is undetectable, the right one is enlarged, there are
signs of double pyelocaliceal system. What study is required to specify the diagnosis?
A Excretory urography  x ray with I.V contrast media better on (Fluorography)
B Micturating cystography
C Retrograde urography
D Doppler study of renal vessels
E Radioisotope renal scan
Dr.Hur A.Salman - OdNMU
184-An 8-year-old boy has a 2-year history of blotchy itchy rash appearing after eating citrus fruit.
The first eruption occurred at the age of 6 months after the introduction of juices to the baby's
diet. Father has a history of bronchial asthma, mother - that of allergic rhinitis. What is the most
likely diagnosis?
A Atopic dermatitis  Review Q 92 Same Q also check 29
B Psoriasis
C Pityriasis Rosea
D Urticaria
E Quincke's edema

185-An 8-year-old child was hospitalized for fever up to 39,8oC, inertness, moderate headache,
vomiting. Examination revealed meningeal symptoms. Lumbar puncture was performed. The
obtained fluid had raised opening pressure, it was transparent, with the cell count of 450 cells
per 1mcL (mainly lymphocytes - 90%), glucose level of 2,6 mmol/l. What causative agent might
have caused the disease in the child?
A Enterovirus  viral meningitis , sugar normal , Review Q 124
B Meningococcus
C Koch's bacillus
D Staphylococcus
E Pneumococcus

186-A 3-year-old child with ARVI had been administered biseptol, paracetamol, nazoferon. On
the third day of treatment the baby's condition deteriorated: he developed sore throat, stomatitis,
conjunctivitis, hypersalivation, painful dark red spots on the neck, face, chest and legs, then the
spots were replaced with vesicles. Examination revealed lesions of mucous membranes around
the mouth and anus. What is your provisional diagnosis?
A Stevens-Johnson syndrome  type of skin reaction- together with toxic epidermal necrolysis
(TEN) -- due to drugs that used
B Atopic dermatitis
C Chickenpox
D Serum sickness
E Bullous dermatitis

187-A 12-year-old child had three attacks of acute rheumatic fever accompanied by carditis.
Examination revealed the symptoms of chronic tonsillitis, mitral insufficiency, carious teeth.
What is the optimal method of secondary prophylaxis?
A Year-round bicillin prophylaxis till the age of 25
B Course of cardiotrophic drugs twice a year
Dr.Hur A.Salman - OdNMU
C Year-round bicillin prophylaxis for 3 years
D Tonsillectomy
E Oral cavity sanitation

188-A 7-year-old female child has developed an acute condition. She complains of a headache,
two onsets of vomiting. Objectively: deferred reactions, body temperature - 39,3oC, pronounced
hyperesthesia, nuchal rigidity, positive superior and inferior Brudzinski's signs, symmetric
Kernig's sign. What is the provisional diagnosis?
A Meningitis  check Q 124
B Food toxicoinfection
C Craniocerebral trauma
D Toxic encephalopathy
E Encephalitis

189
A 7-year-old child complains of itching, papular erythematous rash, dry skin. Objectively: there is
lichenification in the popliteal fossae and antecubital spaces. What immunologic indicator if
found in the blood serum will verify the diagnosis (atopic dermatitis)?
A Total IgE  check Q 29-92
B Secretory IgA
C IgM
D IgG
E IgD

190-A 7-year-old patient presents with body temperature rise up to 39oC, dry cough, pain in the
lateral abdomen. Objectively: there is cyanosis of the nasolabial triangle, inspiratory dyspnea
with accessory muscle recruitment. Percussion reveals pulmonary dullness; among auscultation
findings there are diminished breath sounds, crepitant rales. Respiratory rate is of 50/min, HR-
120/min. Evaluate the grade of respiratory failure in the patient:
A II RR = 50 /min
B I  RR = 30 /min
C III RR = 80 /min
D IV
E0

191-An 8-year-old boy was brought to the admission department by his parents. Parents report
that he has had pain in the right knee for the last 9 months, recently mother has noticed some
limitation of motion in his right leg, and morning stiffness that doesn't last till the evening. What
is the most likely diagnosis?
A Juvenile rheumatoid arthritis  check Q 117
B Rheumatism
C Osteomyelitis of the knee joint
D Reactive arthritis
E Traumatic arthritis

192-A 9-year-old patient has measles. On the 6th day after the rash appeared, the boy developed a
condition manifested by dyspnea, barking cough, stenotic respiration. Objectively: the rash on
the face, neck and torso turned brown. There is a branny desquamation. Respiratory rate is
22/min. What complication should be diagnosed?
A Laryngotracheitis  check Q 55 - 128
B Bronchitis
Dr.Hur A.Salman - OdNMU
C Pneumonia
D Pharyngitis
E Quinsy

193-A 13-year-old boy with hypertrophic cardiomyopathy complains of dyspnea on minimal


exertion. EhoCG reveals asymmetric left ventricular hypertrophy, signs of pulmonary hypertension,
dilatation of the left atrium. EF is 64%. The revealed alterations are indicative of:
A Diastolic heart failure  Hypertrohy cardiomyopathy = Filling problem
B Systolic heart failure  Dilated cardiomyopathy = Pumping problem
C Primary pulmonary hypertension
D Primary arterial hypertension
E Symptomatic arterial hypertension
Dr.Hur A.Salman - OdNMU
194-A 6-year-old boy complains of paroxysmal pain that occurs after a mental stress, consuming
cold drinks or ice cream. After clinical and instrumental examination the boy has been
diagnosed with hypertensive biliary dyskinesia. The drugs of the following groups should be
administered in the first place:
A Antispasmodics and choleretics  to relieve spasm and abnormal motility of biliary ducts and
Choleretics = increase bile secretion by hepatic cell , e.g bile salts and secretin / drug:
Ursodeoxycholic acid (UDCA) Ursodiol
B Choleretics and cholekinetics
C Sedatives and cholekinetics
D Antioxidants
E Antibiotics

195-A 5-year-old girl has had thirst, polyuria, increased appetite for two months. At the same
time, there is a 3 kg decrease in body weight. During the last week, these presentations got
accompanied by nocturnal enuresis. Examination revealed hyperglycemia at the rate of 14
mmol/l. The child has been diagnosed with type I diabetes. What is the most likely genesis of this
disease?
A Autoimmune
B Viral
C Bacterial
D Neurogenic
E Viral and bacterial

196-A 9-year-old girl has been admitted to a hospital for an elevated body temperature (39,8oC),
painful dry cough, abdominal pain on the right. Examination reveals dullness on percussion on the
right, diminished breath sounds, crepitus. What study is required to make a diagnosis?
A Radiography of the chest cavity = Chest x ray , to exclude pneumonia ?
B USI of the chest cavity
C Pleural puncture
D Bronchoscopy
E Bronhography

197-A 3-month-old infant has occipital alopecia, restless sleep, excessive sweating. What disease
can you think of?
A Rickets  check Q 82
B Spasmophilic diathesis
C Anemia
D Phosphate diabetes
E Chondrodystrophy

198-A 12-year-old boy presents with nausea, frequent repeated vomiting that first occurred after
eating canned vegetables. Objectively: the patient has dry mucous membranes, muscular
hypotonia, anisocoria, mydriasis, dysphagia and dysarthria. What is the most likely diagnosis?
A Botulism
B Shigellosis
C Salmonellosis
D Cholera
E Yersiniosis
Dr.Hur A.Salman - OdNMU

199-At the first minute of life a full-term infant born with umbilical cord entangled around his
neck has total cyanosis, apnea, HR- 80/min, hypotonia and areflexia. There are no signs of
meconium aspiration. After the airway suctioning the newborn did not start breathing. What is the
next action of the doctor?
A ALV with a 100% O2 mask  signs of asphyxia due to umbilical cord
B Intravenous administration of adrenaline
C Intravenous administration of etamsylate
D Tracheal intubation and ALV
E Stimulation of the skin along the spine

200-Examination of an 11-year-old boy revealed frequent nosebleeds, fatigue when walking,


underdevelopment of the lower half of the body, increased blood pressure in the upper
extremities and decreased pressure in the lower ones, extension of the left heart border,
blowing systolic murmur in the interscapular region. ECG shows the horizontal axis of heart.
Radiography reveals left cardiomegaly, costal usuration. What is the most likely diagnosis?
A Aortarctia  aortic coarctation : mean stenosis of aortic arch from outside , location after
branches
B Aortic stenosis
C Patent ductus arteriosus
D Ventricular septal defect
E Atrial septal defect
Dr.Hur A.Salman - OdNMU
201-A 6-year-old boy had had a quinsy. 9 days later, there appeared edemata of the face,
extremities and trunk, general health condition deteriorated. Urine became turbid. Objectively:
expressive edemata, ascites. AP- 100/55 mm Hg, diuresis - 0,2 l of urine per day. Results of
the biochemical blood analysis: total protein - 50 g/l, cholesterol - 11,28 mmol/l, urea - 7,15
mmol/l, creatinine - 0,08 mmol/l. Urinalysis results: leukocytes - 3-5 per HPF, red blood cells are
absent. What is the provisional diagnosis?
A Acute glomerulonephritis
B Acute pyelonephritis
C Urolithiasis
D Acute renal failure
E Chronic glomerulonephritis
Quinsy : peritonsillar abscess

202-A 13-year-old boy has had abdominal pain, bloating, nausea, liquid fatty gray stool with
putrid smell for the last 3 years. Palpation reveals epigastric tenderness, as well as tenderness in
theDesjardins' pancreatic point, Chauffard's triangle; there is positive Mayo-Robson's sign. Failure
of exocrine pancreatic function has been suspected. What is the most informative method for
evaluating the state of exocrine pancreatic function?
A Fecal elastase-1 determination
B Blood serum trypsin determination
C Sonography of the pancreas
D Blood and urine amylase determination
E Scatological study
* Pancreatic elastase 1 is a serine endopeptidase, a specific type of protease that has the amino
acid serine at its active site*
Determination of fecal elastase-1 is highly sensitive in the diagnosis of severe and moderate
exocrine pancreatic insufficiency and is of significantly higher sensitivity than fecal chymotrypsin
estimation
Main indications:
-Diagnosis/exclusion of exocrine pancreatic insufficiency caused by e.g. Chronic Pancreatitis,
Cystic Fibrosis, Diabetes Mellitus, Cholelithiasis (Gallstones), “Failure to Thrive”, Pancreatic
Cancer, Papillary Stenosis
-Follow-up monitoring of patients with mild or moderate pancreatic insufficiency
-Diagnosis/exclusion of pancreatic involvement in association with gastrointestinal symptoms,
abdominal pain or osteoporosis, for example.
Dr.Hur A.Salman - OdNMU
203-An 11-year-old boy complains of general weakness, fever up to 38,2oC, pain and swelling of
the knee joints, feeling of irregular heartbeat. 3 weeks ago, the child had quinsy. Knee joints are
swollen, the overlying skin and skin of the knee region is reddened, local temperature is
increased, movements are limited. Heart sounds are muffled, extrasystole is present,
auscultation reveals apical systolic murmur that is not conducted to the left inguinal region. ESR
is 38 mm/h. CRP is 2+, antistreptolysin O titre - 400. What is the most likely diagnosis?
A Acute rheumatic fever
B Vegetative dysfunction
C Non-rheumatic carditis
D Juvenile rheumatoid arthritis
E Reactive arthritis
Quinsy = Peritonsillar abscess

204
A 38-year-old male patient complains of marked dyspnea that escalates with physical exertion.
The problems, namely acute chest pain on the left and cough, arose unexpectedly 2 hours
before at work. The pain abated, but there were progressing dyspnea, dizziness, pallor, cold
sweat, cyanosis. Auscultation reveals the absence of vesicular breath sounds, radiograph
shows a shadow on the left. What pathology can be suspected?
A Left-sided spontaneous pneumothorax
B Pulmonary infarction
C Pleurisy
D Left-sided pneumonia
E Lung abscess
Dr.Hur A.Salman - OdNMU

205-A 67-year-old male patient complains of rash, severe pain in the subscapular region on the
right. Objectively: skin in the right subscapular region is covered with linearly arranged pink-red
edematous lesions that are somewhat infiltrated, and have clear boundaries. On the lesion
surface there are vesicles with transparent exudate. What is the most likely diagnosis?

A Herpes zoster = HZV also called Shingles in adult


B Duhring dermatitis
C Erysipelas
D Atopic dermatitis
E Impetigo  review Q 177

206
A 64-year-old male patient has a 35-year history of chronic pancreatitis. In the last 5 years, he
claims to observe the pain abatement, bloating, frequent bowel movements up to 3-4 times a
day, grayish, glossy stool with undigested food rests, the progressive loss of body weight.
Change of symptoms in the patient is due to overlay of:
A Exocrine pancreatic insufficiency
B Endocrine pancreatic insufficiency
C Lactase deficiency syndrome
D Irritable bowel syndrome
E Chronic enterocolitis
Dr.Hur A.Salman - OdNMU

207-During the doctor's round, a 56-year-old male patient with decompensated cirrhosis
complains of dizziness, palpitations, moving black specks seen before the eyes, general weakness.
The patient is pale, Ps- 110/min, AP- 90/50 mm Hg. What complication is most likely to have
occurred in the patient?
A Bleeding from esophageal varices
B Hepatocellular insufficiency
C Hepatic encephalopathy
D Acute coronary syndrome
E Paroxysmal tachycardia
Dr.Hur A.Salman - OdNMU
208-5 days before, a 26-year-old female patient developed an acute condition. Objectively: marked
headache, vomiting, weakness, poor appetite, temperature up to 39oC. Objectively: the
patient is in a moderately grave condition, excited. The face is hyperemic, sclerae are injected.
The tongue is coated with brown fur. The trunk and limbs are covered with plentiful roseolous
and petechial rash. Hepatosplenomegaly is present. Complement binding reaction with
Rickettsia prowazekii is positive with the titer of 1:640. What drug should be administered?
A Doxycycline  antibiotic , pt have Epidemic Typhus
B Chloramphenicol
C Penicillin
D Streptomycin
E Metronidazole

209-A 39-year-old female patient complains of dyspnea when walking, palpitation, edemata in
the evening. The patient's height is 164 cm, weight - 104 kg. Objectively: overnutrition. Heart
sounds are weak, and tachycardia is present. The menstrual cycle is not broken. Blood sugar is 5,6
mmol/l, ACTH-response tests revealed no alterations. X-ray of the Turkish saddle revealed no
pathology. What disease is it?
A Alimentary obesity  overnutrition (eating much) , no Hormonal problems .
B Climax  the highest or most intense point in the development or/ age related
C Pituitary obesity  normal Pituitary
D Diabetes mellitus  normal glucose
E Cushing's syndrome (primary hypercortisolism)  normal ACTH/ Adrenal

210-A 26-year-old male patient complains of a rash on the upper lip skin, which arose on a
background of influenza with high-grade fever and is accompanied by pain and burning. The
rash has been present for 3 days. Objectively: the skin of the upper lip is edematic and
erythematous, grouped vesicles are filled with serous fluid and have a rough surface. What is
the most likely diagnosis?
A Herpetic vesicular dermatitis
B Eczema
C Contact dermatitis
D Dermatitis herpetiformis
E Erythema multiforme
Dr.Hur A.Salman - OdNMU
211
A 15-year-old patient consulted a dermatologist about a painful lump in the armpit. Objectively:
there is a walnut-sized node, lymphadenitis, infiltration of the surrounding tissues. The patient
has been diagnosed with hidradenitis. What is the most likely causative agent of this disease?
A Staphylococci  Hidradenitis= inflammation of sweat gland
B Streptococci
C Proteus vulgaris
D Pseudomonas aeruginosa
E Mixed infection
Armpit = Axilla

Rest of Q it is not pediatrics , I didn’t do it

SORRY FOR LATE

THANKS FOR TRUST


Dr.H.A.Salman - OdNMU
KROK REVIEW
- First of all
I’d like to thank all the sources , sites , books and persons that I used their material for this preparation
, and I know it is wrong that I used a lot of pictures under copy right without permission , therefore;
I’m asking apologize from all that resources and persons whom made that efforts for humanity
please anyone (sites and persons) object on this , don’t hesitate to contact me , directly I will remove
his work with pleasure and respecting his rights .
Unfortunately I don’t have enough place and time to mention you all ,
Here I’m repeating my thanks and appreciation for serving medicine and health overall the world .

- DEDICATION:
I dedicate this simple works for all humanity, asking to stop wars , destruction and killing people and
living Lovely , Peacefully with happiness and as one human being , we are all same source , beginning
and same end just differ in between , life like 2 brackets ( ) one begin other one end , so try to fill in-
between these brackets with humanity , love , charity , saving the world .
- Thanks God , and all who supported me .

- How to prepare Krok test


krok is MCQ based exam (200Q/4h ) in clinical subjects ( Internal , surgery , pediatrics , Gyne & OB)
with other subjects like Hygiene & nutrition , psychiatry , social medicine , occupational disease ,
infectious disease , pediatrics surgery )
this exam based on translated questions from Russian to English therefore; a lot of mistakes and non-
usable English words , some non medical terms as well as printing mistakes.
- If we ask any foreign students about this exam , he will answer better to cancel this exam from
protocol of international faculty , because it’s never assess the student knowledge , the information
that used in test old , non updated diseases and old protocols and based on SSRI period.
- Unfortunately very old information make student confused with the international parameters,
procedures and new guidelines and modalities.

- Another thing why it is just one attempt ?? and you should repeat all the course again on account of
some miserable questions ??
So the best solution for this problem is to the exam selective as before , obligatory for Ukrainian
citizen and selective for foreign , because no benefit of this exam
Or 2nd option to change the discipline to make real international not just on papers

- May be this information will not change anything , but that notes for next courses

- About IFOM exam , should not be applied unless they change the syllabus and remove all that old
books , procedures , examination , and all protocols , old medical terms , and unusable words , as well
as the combination Latin roots that they make it , as they want to make it formal exam , it is not like
krok just keep without understand , it is hard exam and depend on updated infromations

- So what student should do in this case , study the old questions and keep it without understanding
unfortunately especially for Hygiene , non medical questions , why doctor should keep dimension of
room or how to calculate air ventilation and which place should build hospital , this subject should be
selective , doctor after graduate work with patient and need to learn examination diagnosis of disease
and treatment better than learn numbers and old protocols
Dr.H.A.Salman - OdNMU
- So I suggest on you to analyze the questions use the following steps :
1- for medical Q try to find signs or symptoms that related to the disease , whatever was diagnosis or
treatment and tactic
- 2- for non-understandable Q try to use Excluding the options , because a lot of Q they used 4 options
so far from the answer and the last one will the correct whatever it is right information or wrong .
- 3- use KEY way to match the Q with the answer , if clinical or other
- Sometimes there is more than one right answer but you should choose the more specific one
for ex give you information about extrauterine pregnancy and cervical pathology and they put in the
answer – ectopic pregnancy and cervical pregnancy ,, both of them right logically but you should
vhosse the cervical pregnancy it is more specific .
- For Hygiene most Q not understandable , some of them keep it , others if you translate it you can
understand , others you find the answer in the Q same words but not always , so I think just go a head
and do the easiest way you like .
- Note: All information here based on Ukrainian information and protocols , so try to keep this
information temporarily because it differ completely from your state exam or another country ..

- I advice you don’t use this information furthermore due to most of these information not based on
clinical and practical measurement , most of it theoretical and hypothetical information .

- Before Finally I wish all pass safely and wish all best of luck , hoping to see all best doctors and have
a good futures , caring all people , saving the humanity , especially children and poor people and all
who on need , repairing the world which destroyed by wars.

- Remember KROK does’t make you a DOCTOR !


And never assess you

Please Don’t hesitate to contact me for adding some information or correct anything .
If you feel some information not well explained or there is any comments , please I’m listening and I will
correct it again and re share that information
I will be happy with your feedback

Finally if you find this file good and can help others , please share it

All answers is A with


Yellow color : the answers
Green color : key words of Q
Blue color : this is the most important words in Q (additional Keys)
Grey color : this is my note and additional info
Red color : this repeated Q or high-light
Violant color : critical notes

Yours
Dr. H.A.SALMAN
OdNMU
5/5/2018
E-mail: hur.amer@gmail.com
F/b: Hr Salman
Net : +964-790-489-6865
Dr.H.A.Salman - OdNMU
Krok 2 – 2014 Surgery Base
1. A 76 y.o. woman complains of progressing swallow ing disorder, mostly she has had problems with solid
food for the last 6 weeks. Sometimes she has regurgitation of solid masses. Swallowing is not painful. She lost
6 kg. 10 years ago she had myocardiac infarction, she takes constantly aspirine and prolonged nitrates. She
consumes alcochol in moderate proportions, smokes.
Objectively: icteric skin, neck has no pecularities, lymph nodes are not enlarged. Thorax has no changes,
cardiovascular system has no evident changes. Liver is +3 cm. What is the preliminary diagnosis?
A. Cancer of esophagus
B. Diffuse constriction of esophagus
C. Diaphragmatic hernia
D. Myasthenia
E. Esophageal achalasia

2. A 65 y.o. man who has problems with urination as a result of benign prostate gland adenoma dveloped
fever and chill, hypotension, sinus tachycardia. Skin is warm and dry. Clinical blood analysis revealed
absolute granulocytopenia. These hemodynamic changes are most likely to be caused by:

A. Endotoxemia with activation of complement system


B. Secondary endothelial changes as a result of bacterial lesion
C. Secondary circulation insufficiency with retained systolic function as a result of peripheral
vasoconstriction
D. Secondary reflex vasodilatation as a result of lowered cardiac output
E. Reflex vagus stimulation with lowered cardiac output

Agranulocytosis, or granulopenia, acute condition involving a severe and dangerous leukopenia


(lowered WBC count), most commonly of neutrophils causing a neutropenia in the circulating blood. It is a severe
lack of one major class of infection-fighting white blood cells. People with this condition are at very high risk of
serious infections due to their suppressed immune system.
absolute granulocytopenia : there is no granulocyte at all even may be zero ( Neutro, Baso, esinoph ) and due to
decrease that cell need to immune activity there for we choose complement system activation ( phagocytosis , in
inflammation , bacteria membrane attack.

3. A 60 y.o. man complains of sense of heaviness in the region of scrotum. Objectively: scrotum edema in the
left part. Testicle is of normal size, but there is a soft, scrotum limited edema over it that can be pressed and
disappears when the patient lies down. What is the preliminary diagnosis?
A. Varicocele
B. Ectopic testicle
C. Varicosity of subcutaneous veins
Dr.H.A.Salman - OdNMU
D. Inguinal hernia
E. Inguinal lymphadenopathy

note: varicocele is an enlargement of the veins within the loose bag of skin that holds testicles (scrotum)
ectopic mean not in the origin place (undescending testis) and this mostly in children
C: the varcis will be superficially and obvious , you can see the dilated & tortuous vein
D: inguinal hernia will start at inguinal canal and may descend to scrotal
E: LAP will not reaching scrotum and not disappear during lie down .

4. A patient was delivered to a surgical department after a road accident with a closed trauma of chest and
right-sided rib fracture. The patient was diagnosed with right-sided pneumothorax, it is indicated to perform
drainage of pleural cavity. Pleural puncture should be made in:

A. In the 2nd intercostal space along the middle clavicular line


B. In the 7th intercostal space along the scapular line
C. In the point of the greatest dullness on percussion
D. In the projection of pleural sinus
E. In the 6th intercostal space along the posterior axillary line

** for pneumothorax (air) we do pucture at 2nd intercostal while in hydrothorax (fluid, blood , pus ) we do 6th
ICS (air is light going up - X - fluid is heavy going down )

5. A 40 year old woman has changes of mammary gland. What are the most often symtomps that precede the
malignization?

A. Skin induration with inverted nipple


B. Bloody discharges from the nipple
C. Pure discharges from the nipple
D. Painful movable induration
E. Painless movable induration
Dr.H.A.Salman - OdNMU

6. Name a statistical observation unit for determination of influence amount of Blood sugar on the healing of
wounds surface in a postoperative period:

A. The patient in a postoperative period  if you want to make statistic to know the relation between glucose
and wound healing un pt Post Op period , off corse you will name the statistic , pt in Post Op period.
B. Blood analysis
C. The patient who was discharged on an after-care
D. The patient who has a wound surface
E. An amount of blood sugar

7. A 20-year-old patient was delivered to the hospital in summer from the street with haemorrage from the
brachial artery. First medical aid involved application of a tourniquet for provisional arrest of bleeding. What
is the maximal exposure of the tourniquet?
A. 120 minutes  at summer
B. 60 minutes
C. 180 minutes
D. 15 minutes
E. 30 minutes  this in winter

8. A 9 y.o. child with diagnosis "chronic tonsillitis" stands dispanserization control. Within 1 year of
observation there was one exacerbation of disease. Physical condition is satisfactory. The general state is not
infringed. Define group of health:
A. III (a)
B. III (c)
C. I-st
D. II-d
E. III (b)
** This Q is not surgery

9. A 38 year old man, previously in good health, suddenly develops severe abdominal pain radiating from the
left loin to groin and accompanied by nausea, perspiration and the need for frequent urination. He is restless,
tossing in bed but has no abnormal findings. The most likely diagnosis is:
A. Left sided renal colic
B. Sigmoid diverticulitis
C. Retroperitoneal haemorrhage
D. Torsion of the left testicle
E. Herpes zoster
Dr.H.A.Salman - OdNMU
10. A 40 year old woman has a self-detected hard breast mass. The procedure of choice for confirming the
diagnosis is:

A. Excision biopsy
B. Thermography
C. Aspiration biopsy with cytology
D. Ultrasonography
E. Mammography
Types of biopsy:
1- The shave biopsy is the least invasive of all three techniques. Your doctor will remove the outermost layers of
skin. You will not need stitches.
2- Punch biopsies are most often used for deeper skin spots or sores. Your doctor removes a small round piece of
skin (usually the size of a pencil eraser) using a sharp, hollow instrument. If a large sample is taken, the area may be
closed with stitches.
3- An excisional biopsy is done to remove the entire lesion. A numbing medicine is injected into the area. Then the
entire lump, spot, or sore is removed, going as deep as needed to get the whole area. The area is closed with
stitches. Pressure is applied to the area to stop any bleeding. If a large area is biopsied, a skin graft or flap of normal
skin may be used to replace the skin that was removed.
4- An incisional biopsy takes a piece of a larger growth for examination. The area is injected with a numbing
medicine. A piece of the growth is cut and sent to the lab for examination. You may have stitches, if needed. The
rest of the growth can be treated after the diagnosis is made.

11. During investigation for chronic, severe, epigastric pain, a 40 year old alcoholic man is found to have
multiple areas of narrowing alternating with dilatation ("chain of lakes" appearance) of the main pancreatic
duct. The operation of choice is:
A. Lateral pancreaticojejunostomy
B. Sphincterotomy
C. Distal pancreaticojejunostomy
D. Distal pancreatectomy
E. Total pancreatectomy

12. The treatment of choice for duodenal obstruction caused by secondary duodenal hematoma that
developed a few days after blunt abdominal injury is:
A. Nasogastric decompression and parenteral alimentation
B. Immediate exploration
C. Tube duodenostomy
D. Retrocolic gastrojejunostomy
E. Duodenojejunostomy
Dr.H.A.Salman - OdNMU

13. An anestesiologist gives narcosis to the patient, he uses a non-reversive contour. Anesthetic is halothane.
Air temperature in the operation room is 21°C, humidity 50%, level of noise 30 dB. What occupational hazard
is theprincipal one under these conditions?
A. Air pollution with anesthetic
B. Improper occupational microclimate
C. Mental overfatigue
D. Compelled working pose
E. High level of noise

14. A patient suddenly felt an acute chest pain irradiating to the left arm. Objectively: the patient is excited,
with pale skin. Breathing rate - 38/min, AP - 180/110 mm Hg. Later the patient lost consciousness and fell
down. Pulse on the great vessels was absent, the pupils were equally dilated. What is the most likely
diagnosis?
A. Clinical death
B. Coma
C. Disorder of the cerebral circulation
D. Heart attack
E. Agonal state

15. A 37-year-old patient has come to clinic being wounded in the area of umbilicus an hour ago. On physical
exam, there is a prick and cut painful wound of about 0,5x1 cm around umbilicus on the abdominal skin with
slight bleeding. How would you help this patient?

A. Laparotomy, abdominal cavity organs inspection. Primary surgical


processing of the wound
B. Aseptic bandage
Dr.H.A.Salman - OdNMU
C. Suture on the wound
D. Drainage of the wound with rubber strip
E. Inspection of the wound with canal probe

16. If a child has adherent fingers on his right hand, then what will be your diagnosis?
A. Syndactyly = adherent fingers
B. Ectrodactyly = split hand - cleft
C. Ectromelia = long bones missing or underdeveloped
D. Polydactyly = multiple finger > 5
E. Macrodactyly = big fingers
A B D
17. A 34-year-old patient was bitten by a dog 3 hours ago. There is a wound by dog teeth without bleeding in
the left arm. What surgical help would you provide to this patient?

A. Clean wound with detergent water and apply anti-septic


B. Aseptic bandage
C. Complete suture of the wound
D. Incomplete suture of the wound
E. Cream bandage

18. Purulent mediastinitis is diagnosed at a 63-year-old patient. What diseases from the stated below
CANNOT cause the purulent mediastinitis?
A. Cervical lymphadenitis  inflammation of Cervical Lymph nodes
B. Deep neck phlegmon
C. Perforation of the thoracic part of the easophagus
D. Iatrogenic injury of the trachea
E. Perforation of the cervical part of the easophagus

19. repeated 18
20. A 36 year old patient was diagnosed with right-sided pneumothorax. What method of treatment is
indicated to the patient?
A. Surgical treatment: drainage of the pleural cavity
B. Symptomatic therapy
C. Thoracotomy  opening of thorax
D. Pleural puncture  this can be right but just keep what they answered above 
E. Antiinflammation therapy

21. repeated 20
The diagnosis of a right-sided pneumothorax is made to a 36- year-old patient. What method of treatment is
indicated to the patient?
A. Drainage of the pleural cavity
B. Symptomatic therapy
C. Antiinflammation therapy
D. Pleural puncture
E. Thoracotomy

22. A 16 year old patient with complaints of frequent pain in the abdomen was diagnosed with melanoma,
examination revealed also pigmentation of the mucosa and skin, polyp in the stomach and large intestine. It
is know that the patients mother has an analogous pigmentation and has been often treated for anemia. What
disease is suspected?
A. Peytz-Egerss polyposis
B. Chrons disease
C. Adolescent polyposis
D. Hirschprungs disease
E. Tuberculosis of the intestine
Dr.H.A.Salman - OdNMU
23. A 41 year old patient was admitted to the intensive care unit with haemorrhagic shock due to gastric
bleeding. He has a history of hepatitis B during the last 5 years. The source of bleeding are esophageal veins.
What is the most effective method for control of the bleeding?
A. Introduction of obturator nasogastric tube  this can stop esophageal bleeding
B. Hemostatic therapy
C. Administration of plasma
D. Operation
E. Intravenous administration of pituitrin

24. It is suspected that a 34 year old patient has an abscess of Douglas pouches. What diagnostic method is to
be chosen?
A. Digital examination of rectum
B. R-scopy of abdominal cavity
C. Laparoscopy
D. Rectoromanoscopy
E. Percussion and auscultation of stomach

25. A patient has restrained umbilateral hernia complicated by phlegmon hernia, it is necessary to take
following actions:
A. Herniotomy by Mayo-Sapezhko
B. Herniotomy by Lekser
C. Herniotomy by Grenov
D. Herniotomy by Mayo
E. Herniotomy by Sapezhko
Dr.H.A.Salman - OdNMU

Mayo Method Sapezhko method

Meyo method defect of anterior abdominal wall in the umbilical ring is sutured by U-shaped stitches in
transversal direction.
Sapezhko proposed to form duplication of the abdominal white line by stitches in longitudinal direction

26. A 52 year old man has recurrent transient ischemic attacks. Auscultation of the carotid arteries detected
murmur. What diagnostic method is to be applied in the first place?
A. Ultrasound dopplerography
B. MRI of the brain
C. Electroencephalography
D. Cerebral angiography
E. CT of the brain

27. For the persons who live in a hot area after an accident at a nuclear object, the greatest risk within the
first decade is represented by cancer of:
A. Thyroid gland
B. Lungs
C. Reproduction system organs
D. Skin
E. Breast

28. A 10 year old boy complains about pain in his left eye and strong photophobia after he had injured his left
eye with a pencil at school. Left eye examination: blepharospasm, ciliary and conjunctival congestion, cornea
is transparent, other parts of eyeball have no changes. Visus 0,9. Right eye is healthy, Visus 1,0. What
additional method would you choose first of all?
A. Staining test with 1% fluorescein 
B. Gonioscopia
C. Cornea sensation-test
D. X-ray examination of orbit
E. Tonometria
Dr.H.A.Salman - OdNMU

This is a test that uses orange dye (fluorescein) and a blue light to detect foreign bodies in the eye. This test can
also detect damage to the cornea
Blepharospasm is any abnormal contraction or twitch of the eyelid.

29. A patient had disorder of nasal respiration, mucopurulent discharges from nose, headache for 3 weeks. At
anterior rhinoscopy in middle nasal meathus the stria of pus, edema, hyperemia of the mucosa of the nose
have been determined. What diagnostic method is necessary to administer first of all?
A. An X-ray of paranasal sinuses  sinusitis
B. General blood test
C. Bacteriology analysis of the nasal mucous
D. CT of a skull
E. Punction of the maxillar sinus

30. A patient with acute purulent otitis media complicated by mastoiditis was admitted to a hospital.
Roentgenogram of mastoid processes showed the shadiowing of the cellular system on the lesion, absence of
bone septa was present. What are the necessary therapeutic actions at the second stage of mastoiditis?
A. Mastoidotomy  opening of mastoid air cell (lower occipital region )
B. Paracentesis of the drum
C. Tympanoplasty
D. Cateterization of the Eustachian tube
E. Radical operation on the middle ear

31. A 35-year-old woman was admitted to resuscitation department in asthmatic status. What is the most
trustworthy criterion of breath effectiveness?
A. PaCO2 and PaO2 measuring of blood gases: partial alveolar pressure of CO2 and pressure of O2
B. Respiration rate
C. Respiratory volume
Dr.H.A.Salman - OdNMU
D. Minute respiratory volume
E. Determination of “dead” space

32. During dynamic investigation of a patient the increase of central venous pressure is combined with the
decrease of arterial pressure. What process is proved by such combination?
A. Developing of cardiac insufficiency   CVP -  ABP
B. Depositing of blood in venous channel
C. Presence of hypervolemia
D. Increase of bleeding speed
E. Shunting

33. A 22 year old patient was admitted to trauma center with complaints of pain in the left ankle joint that
was getting worse during moving and weight bearing. On the clinical examination it was found that the
patient had the closed fracture of medial malleolus without displacement. In which position should the foot
be fixed in plaster cast?
A. At right angle with varus positioning of the foot
B. In position of planter flexion of foot
C. In position of supination
D. In position of dorsal flexion of foot
E. In position of pronation

34. A 74 y.o. patient has been ill with benign prostate hyperplasy for the last 5 years. 4 days ago, after
alcochol consumption, there was an acute retention of urination. At the pre-admission stage his urinary
bladder was catheterized with metallic catheter. Examination revealed: right epididymis is enlarged, thick
and painful, there are purulent discharges from urethra. What way of emergency care must be chosen?

A. Trocar or open epicystostomy  suprapubic catheter , we put through U.B when contraindication through
urethra like obstruction or laceration or infection with discharge.
B. Placing of intraprostatic stent
C. Introduction of permanent urethral catheter
D. Transuretral resection or prostatectomy
E. Microwave thermotherapy of prostate

Trocar
Dr.H.A.Salman - OdNMU
35. A rounded well-defined shadow was found in the costo-vertebral angle on the chest roentgenogram of an
otherwise healthy 9 year old girl. Make a preliminary diagnosis:
A. Ganglioneuroma
B. Ganglioneuroblastoma
C. Sarcoma of the vertebra
D. Sympatogonioma
E. Sympatoblastoma

Ganglioneuroma is a rare and benign tumor of the


autonomic nerve fibers arising from neural crest
sympathogonia, which are completely undifferentiated
cells of the sympathetic nervous system. However,
ganglioneuromas themselves are fully differentiated
neuronal tumors that do not contain immature
elements.

36. A 4 y.o. child attends the kindergarten. Complains of poor appetite, fatigue. Objective examination: skin
and mucous membrane are pale, child is asthenic. In the hemogram: hypochromatic anemia 1st, leucomoide
reaction of the eosinophile type. What pathology must be excluded first of all?
A. Helminthic invasion  worms
B. Hypoplastic anemia
C. Atrophic gastritis
D. Duodenal ulcer
E. Lymphoprolipherative process

37. A 33 year old male patient was brought to Emergency Department with the signs of cardiovascular
collapse: BP - 60/30 mm Hg, Ps - 140 bpm, the skin is pale and moist, diuresis 20 ml/h, Hb - 80 g/l, red
blood cell count - 2,5×1012/l. The reduction of blood volume averages:
A. 30-40%  stage 3 Hypovolumic shock
B. 25-30%
C. 15-20%
D. 10-15%
E. 20-25%
Dr.H.A.Salman - OdNMU
Stage 1 Stage 2 Stage 3 Stage 4
30–40% (1500–2000
Blood loss Up to 15% (750 mL) 15–30% (750–1500 mL) Over 40% (over 2000 mL)
mL)
Normal (Maintained
Blood pressure Increased diastolic BP Systolic BP < 100 Systolic BP < 70
by vasoconstriction)
Slight tachycardia (> 100 Tachycardia (> 120 Extreme tachycardia (> 140 bpm) with weak
Heart rate Normal
bpm) bpm) pulse
Respiratory
Normal Increased (> 20) Tachypneic (> 30) Extreme tachypnea
rate
Mental status Normal Slight anxiety, restless Altered, confused Decreased LOC, lethargy, coma
Skin Pallor Pale, cool, clammy Increased diaphoresis Extreme diaphoresis; mottling possible
Capillary refill Normal Delayed Delayed Absent
Urine output Normal 20–30 mL/h 20 mL/h Negligible

38. A 19 year old girl was admitted to emergency department: unconsciousness, cyanosis, myotic pupils are
present, superficial breathing is 12/min. BP is 90/60 mm Hg, Ps- 78/min. Choose the action necessary in this
clinical situation:
A. Controlled respiration -- ABC emergency (Airway, Breathing , Circulation….)
B. Oxygen inhalation
C. Cordiamine injection
D. Caffeine injection
E. Gastric lavage

39. What preparations are used for prevention of fungal infection?


A. Fluconozol, Orungol, Nisoral
B. Cytosar, Cormyctin, Lomycitin  chemotherapy , nifidipin
C. Rubomycin, Bleomycin, Mytomycin C  chemotherapy / anti cancer
D. Captopril, Enalapril  ACE inhibitor – hypotensive agents
E. Isoniazid, Ftibazid, Pyrazinamid  anti TB

40. What developes most often after accidental intake of Hydrochloric acid?
A. Cardiac insufficiency …. Just God know why !! just keep .. don’t think and wasting time !
B. Deyladss syndrome
C. Acute pancreatitis
D. Cushings syndrome
E. Kutlings syndrome

41. 30 y.o. woman, had mild trauma of 5th finger of the left hand 15 days ago. She has treated her self at
home. She presents to the hospital due to deteriorat ion of the condition and temperature rise. Objectively:
hyperemia and swelling on the ventral surface of finger. Restricted movements of the finger. X-ray of the
left hand: an early stage of osteomyolitis of the fifth finger could not be excluded. The diagnosis: panaris of
5th finger of the left hand. What form of panaris has occurred in the patient?
A. Bony
B. Paronychia  soft tissue infection around a fingernail that begins as cellulitis but that may progress to abscess
C. Joints type
D. Tendon type
E. Hypodermic
Dr.H.A.Salman - OdNMU

Panaris is a general term used to refer


to "all acute inflammations (soft tissue
parts) of fingers, whatever their nature,
extent and depth

Also called Felon : An infection inside the tip of the finger can form an enclosed pocket of pus (or abscess) that is
very painful as it expands. A felon is a fingertip abscess deep in the palm side of the finger. It usually is caused by
bacterial infection, but a herpes virus called herpetic whitlow and, more rarely, fungi also can cause felons.

42. repeated 20
42. A 36 y.o. patient is diagnosed with right sided pneumothorax. What method of treatment is indicated to
the patient?
A. Surgical drainage of the pleural cavity
B. Symptomatic therapy
C. Antiinflammation therapy
D. Pleural puncture
E. Thoracotomy

43. A 28 year old woman was admitted to the emergency room with a slightly reddened, painful "knot" 8 cm
above the medial malleolus. Examination in the standing position demonstrates a distended vein above and
below the mass. There are no other abnormalities on physical examination. The most likely diagnosis is:
A. Superficial venous thrombosis
B. Insect bite
C. Early deep vein thrombosis
D. Cellulitis
E. Subcutaneous hematoma

44. A 30 year old man complains of acute pain in his right ear, hearing loss, high temperature for three days.
Objectively: right ear whispering language - 0,5 mm, external ear is intact, otoscopically - eardrum
protrusion, hyperemia and swelling, loss of landmarks. What disease is it?
A. Acute purulent otitis media  infection with pus of middle ear –bacterial
B. Chronic purulent otitis media
C. Eustachian tube disfunction
D. Acute mastoiditis
E. Chronic secretory otitis media
Dr.H.A.Salman - OdNMU

45. A 22 y.o. man complains of acute throat pain, increasing upon swallowing during 3 days. Body
temperature 38,3°C, neck lymph nodules are slightly enlarged and painful. Pharyngoscopically - tonsilar
hyperemia, enlargement and edema, tonsils are covered by round yellow fibrinous patches around crypts
openings. Beta-haemolytic streptococcus in swab analysis. What is the diagnosis?
A. Acute membranous tonsilitis
B. Pharyngeal diphtheria
C. Acute follicular tonsilitis
D. Infectious mononucleosis
E. Pharyngeal candidosis

46. A patient is staying in the hospital with the diagnosis of abdominal typhus. During the 3-d week from the
beginning of the disease the patient stopped keeping diet and confinement to bed. As a result the body
temperature and rapid pulse decreased and melena appeared. What kind of complications should we think
about first of all?
A. Intestinal haemorrhage

B. Meningitis
C. Hepatite
D. Nephroso-nephritis
E. Thrombophlebitis
Dr.H.A.Salman - OdNMU
47. A 45-year-old woman, mother of four children, comes to the emergency room complaining of a sudden
onset of the epigastric and right upper quadrant pain, radiating to the back, accompanied by vomiting. On
examination, tenderness is elicited in the right upper quadrant, bowel sounds are decreased, and laboratory
data shows leukocytosis, normal serum levels of amylase, lipase, and bilirubin. The most likely diagnosis is:
A. Acute cholecystitis
B. Perforated peptic ulcer disease no sx. Or hx of ulcer
C. Sigmoid diverticulitis  pain & tenderness in Lt iliac fossa
D. Acute pancreatitis  normal s. amylase & Lipase
E. Myocardial infarction  no sx of MI or risk factors

48. During an operation for presumed appendicitis the appendix was found to be normal; however, the
terminal ileum is evidently thickened and feels rubbery, its serosa is covered with grayish-white exudate, and
several loops of apparently normal small intestine are adherent to it. The most likely diagnosis is:
A. Crohns disease of the terminal ileum
B. Ulcerative colitis
C. Perforated Meckels diverticulum
D. Ileocecal tuberculosis
E. Acute ileitis

49. A 50 year old woman with a 2-year history of mild, diffuse, tender thyroid enlargement complains of 10
pound weight gain and fatigue. What is the most probable diagnosis?

A. Hashimotos thyroiditis  autoimmune thyroiditis – Hypothyroidism


B. Riedels thyroiditis
C. Suppurative thyroiditis
D. Papillary thyroid carcinoma
E. Subacute thyroiditis
Dr.H.A.Salman - OdNMU

50. A severely traumatized patient who has been receiving prolonged parenteral alimentation develops
diarrhea, mental depression, alopecia and perioral and periorbital dermatitis. Administration of which of the
following trace elements is most likely to reverse these complications?
A. Zinc
B. Copper  Wilson’s disease
C. Selenium   risk of DM II , risk of bladder CA , peripheral artery diseases,
D. Iodine  goiter , hypothyroidism
E. Silicon  mild: bruising, and stomach/skin irritation.

51. A 38 y.o. woman was hospitalized to the surgical unit with vomiting and acute abdominal pain irradiating
to the spine. On laparocentesis hemmorhagic fluid is obtained. What disease should be suspected?
A. Acute pancreatitis
B. Perforated gastric ulcer
C. Acute appendicitis
D. Renal colic
E. Acute enterocolitis

52. The 67 y.o. patient had 5 recurrent fractures of the lower extremities without considerable cause within 5
years. O-shaped deformity of the legs in the knee joints has appeared. The skull, pelvis and lower extremities
X-Ray shows the thickening of flat bones. In the long bones there is a hyperostosis along the bone axis. The
blood test has not revealed any inflammation activity. Serum calcium is normal. What disease do you
consider in this case?
A. Pagets disease  cotton wool Appearance in x ray .
B. Hyperparathyoid dystrophy
C. Myeloma
D. Mottled disease (marble disease)
E. Chronic osteomyelitis
Dr.H.A.Salman - OdNMU
53. A 33 y.o. patient was admitted to the reception room of the Central District Hospital. He complains of a
severely painful swelling localized on posterior neck, fever up to 38,4°C and general weakness. In anamnesis:
diabetes mellitus within 5 years. On physical examination on the posterior neck surface there is an infiltrate
elevated above surrounding skin. The tissues affected by swelling are tense and blue reddish discoloration in
central area. There are also several purulent necrotic pustules which are connected with each other and form
a large skin necrosis. A thinned necrotic skin of this swelling has holes looking like sieve, pus discharges
through out. What disease should a doctor consider first of all?

A. Carbuncle
B. Skin abscess
C. Acute skin cellulitis
D. Furuncle
E. Carbuncle associated with anthrax

`
54. A 19 y.o. man was admitted to the reception department in 20 minutes after being wounded with the
knife to the left chest. The patient is confused. The heart rate is 96 bpm and BP- 80/60 mm Hg. There are the
dilated neck veins, sharply diminished apical beat and evident heart enlargement What kind of penetrative
chest wound complications has developed in patient?
A. Pericardium tamponade  Beck’s Traids
B. Valve-likes pneumothorax
C. Open pneumothorax
D. Massive hemothorax
E. Closed pneumothorax
Dr.H.A.Salman - OdNMU
55. A 35 y.o. patient complains of a difficult swallowing, pain behind the breastbone. He can eat only liquid
food. While swallowing sometimes he has attacks of cough and dyspnea. Above mentioned complaints are
progressing. It is known that the patient has had a chemical burn of esophagus one month ago. What
complication does the patient have?
A. Corrosive esophagitis and stricture
B. Esophageal diverticula
C. Esophagitis
D. Cardiac achalasia
E. Cardiac insufficiency

56. Survey radiograph of chest of a 62 year old smoker who often suffers from "pneumonias" showed a
triangle shadow in the right lung, its vertex is pointing towards the lung root. It also showed deviation of
heart and mediastinum shadows towards the lesion. What is the most probable diagnosis?
A. Cenral cancer of lung
B. Lung cyst
C. Lung abscess
D. Peripheral cancer of lung
E. Atelectasis
Dr.H.A.Salman - OdNMU
57. A patient operated for acute paraproctitis undergoes antibacterial and detoxification therapy, the local
course of the disease has the positive dynamics. Since the operation the patient has had chills, pyrexia,
tachycardia, euphoria for five days. The doctor suspected sepsis. What study will confirm the diagnosis?
A. Blood culture for a pathogen  when you have sepsis mean you have previous definite pathogen causing
sepsis , you can detect it by culture bacteriologically .
B. Determining the rate of average-weight molecules
C. Liver ultrasound
D. X-ray of lungs
E. Determining the rate of microbial contamination of wound
58. A 44-year-old patient has been admitted to a hospital with complaints of dull, aching pain in the left
lumbar region, the admixture of pus in the urine. Examination revealed a grade II staghorn calculus on the
left. What method of treatment is indicated for this patient?
A. Surgery
B. Distance lithotripsy
C. Ascending litholysis
D. Conservative therapy
E. Contact lithotripsy

59. An emergency team has delivered to a hospital an unconscious patient found lying in the street in winter.
Objectively: the patient is pale, with superficial respiration; bradycardia with heartrate 54/min, to- 35°C. AP-
100/60 mm Hg. Palpation of chest and abdomen revealed no peritoneal symptoms. There is a smell of
alcohol from the patients mouth. What is the most likely diagnosis?
A. Hypothermia
B. Apparent death
C. Acute cardiovascular insufficiency
D. Frostbite of trunk and extremities

60. A 48-year-old patient got a job-related injury of a hypodermic varicose vein on his shin that was
accompanied by the intensive phleborrhagia. Choose the optimal variant of first aid:
A. Pressure bandage and limb strapping
B. Maximal limb flexion in knee joint
C. Application of Esmarchs tourniquet beneath the injury
D. Application of Esmarchs tourniquet above the injury
E. Occlusion of femoral artery in a typical place
** (shin) : prominent anterior edge of the tibia or the leg.
Dr.H.A.Salman - OdNMU
Phleborrhagia  veinous hemorrhage
61. A 24-year-old patient got a puncture injury below the Pouparts ligament accompanied by intense arterial
bleeding. The best method to temporarily stop the bleeding in the patient would be:
A. Compression band
B. Maximum limb bending
C. Esmarchs tourniquet
D. Compressing a blood vessel with a clamp
E. Wound suturing
** Pouparts ligament is inguinal ligament
62. While making a round, a doctor had noticed the edema of the right shoulder of a 26-day-old child with
diagnosis of "umbilical sepsis". Active movements in the shoulder joint were absent, right hand was hanging
down. Mother stated that her childs condition had worsened for the last 24 hours what resulted in
childs refusal to be breast-fed, restlessness, weeping while swaddling, rise in body temperature up to 38,8°C.
What is the most probable preliminary diagnosis?
A. Epiphysial osteomyelitis of the humerus
B. Fracture of the clavicle
C. Traumatic brachioplexitis
D. Fracture of the humerus
E. Phlegmon of the shoulder

63. A 65-year-old patient complains of dull pain in the rectum during and after defecation, discharge of
mucus and small amount of blood mixed up with mucus and faeces. The discharged blood is of dark red
color, sometimes with small amount of clots. The patient is sick for 8 months, has lost some weight.
On digital examination, there is a round constriction of the rectum with infiltrate at a height of 4-5 cm from
the anus. What is the most probable diagnosis?
A. Cancer of the medium-ampullar section of the rectum 
B. Crohns disease
C. Non-specific ulcer colitis
D. Cicatricial stenosis of the rectum
E. Chronic paraproctitis

Fig. 1. Anatomy of the rectum: scheme. At about


8-10 cm from internal anal sphincter, on the right
lateral wall there is a Kohlrausch valve (2)
corresponding to the peritoneal fold. It
represents the upper limit of mid-rectum. On the
left lateral wall there are the inferior (1) and
superior (3) Houston valves. Inferior Houston
valve (1) represents the upper limit of the lower
rectum while the recto-sigmoid junction (4) is not
definitely recognized on imaging. It is located 10
cm from the anus
Dr.H.A.Salman - OdNMU
64. A 52 y.o. patient fell from 3 m height on the flat ground with the right lumbar area. He complains of pain
in this area. There is microhematuria in the urea. Excretory urography revealed that kidneys functioning is
satisfactory. What is the most probable diagnosis?
A. Kidneys contusion  called a kidney bruise, occurs following blunt trauma or direct impact to the lower back.
This trauma leads to bleeding inside of the kidney. It may also cause pain, tenderness, and discoloration of the
skin.
B. Paranephral hematoma
C. Kidneys abruption
D. Subcapsular kidneys rupture
E. Multiple kidneys ruptures

65. A female patient has been suffering from pain in the right subcostal area, bitter taste in the mouth,
periodical bile vomiting for a month. The patient put off 12 kg. Body temperature in the evening is 37,6°C.
Sonography revealed that bile bladder was 5,5 x 2,7 cm large, its wall - 0,4 cm, choledochus - 0,8 cm
in diameter. Anterior liver segment contains a roundish hypoechoic formation up to 5 cm in diameter and
another two up to 1,5 cm each, walls of these formations are up to 0,3 cm thick. What is the most likely
diagnosis?
A. Alveolar echinococcus of liver  liver Hydatid Cysts (H.C) (daughter cysts)
B. Liver abscess
C. Paravesical liver abscesses
D. Cystous liver cancer
E. Liver cancer

Ultrasound CT-scan

66. A 36-year-old woman applied to doctor with complaints of the enlargement of the right mammary gland.
On physical exam: the right mammary gland is enlarged, hardened; nipple with areola is pasty and hydropic,
looks like "lemon peel". There palpates a lymph node in the right axilla, 1,5 cm in diameter, dense onsistence,
not mobile. What is the most probable diagnosis?
A. Hydropic and infiltrative cancer of the mammary gland
B. Diffusive mastopathy
Dr.H.A.Salman - OdNMU
C. Erysipelatouslike cancer of the mammary gland
D. Pagets cancer  Paget disease of the nipple
E. Acute mastitis  inflammation of the Breast

67. A 43 year old patient had cholecystectomy 6 years ago because of chronic calculous cholecystitis. Lately
he has been suffering from pain in the right subcostal area and recurrent jaundice. Jaundice hasnt gone for
the last 2 weeks. Stenosing papillitis 0,5 cm long has been revealed. What is the best way of treatment?
A. To perform endocsopic papillosphincterotomy  reparing the stenosis and oening the papilla by the
endoscopy ( ERCP)
B. To perform external choledoch drainage
C. To treat conservatively: antibiotics, spasmolytics, antiinflammatory drugs
D. To perform transduodenal papillosphincterotomy
E. To perform choledochoduodenostomy

68. A 49-year-old male patient who had been scheduled for a surgery for gastric cancer underwent
reoperative infusion therapy. Up to 3,0 liters of liquid was introduced into the right cubital vein. The
following day, he felt a dragging pain in the right shoulder. Objectively: on the inner surface of the shoulder
there is a longitudinal hyperemic zone, edema of skin, a tender cord. What complication occurred in the
patient?
A. Acute thrombophlebitis  blood clot causing inflammation of the vein due to long standing the IV line to the
cubital vein (3L) infusion big amounts of fluid
B. Venepuncture and edema of paravenous tissue
C. Acute lymphangitis
D. Paravenous tissue phlegmon
E. Paravenous tissue necrosis
Dr.H.A.Salman - OdNMU

This example how long


standing I.V line causing
thrombs and led to vein
inflammation 
theombophlebitis in hand
viens
cubital vein in Elbow

69. A 54-year-old patient applied with complaints of acute pain in the shoulder joint. 10 minutes earlier he
fell in the street with his arms stretched. Doctor of the traumatological unit noticed an acute deformation of
the right shoulderjoint, the impossibilty of active movement in the right extremity. Roentgenologically:
uncoincidence of articulating surfaces. What is the most probable diagnosis?
A. Dislocation of the clavicle
B. Contusion of the humerus
C. Dislocation of the humerus
D. Dislocation of the scapula
E. Fracture of the humerus
Note** roentgen = x ray , radiography
un coincidence  = mean mismatch (thanks google) translated from Ru

70. A 62-year-old patient has been delivered to the surgical department with complaints of sudden pain in
the umbilical region irradiating to the back and groin, which was accompanied by a syncope. Objectively: the
patient is in grave condition, umbilical region is tender on palpation, bowel sounds are diminished. AP drop
is present. USI reveals: free fluid in the abdomen, thickening of the wall of the abdominal aorta. The most
likely diagnosis is:
A. Rupture of abdominal aortic aneurism
B. Acute pancreatitis
C. Stomach ulcer perforation
D. Peritonitis
E. Acute appendicitis
Dr.H.A.Salman - OdNMU

71. Parents of a 2-year-old boy applied to clinic complaining of right testicle absence in the scrotum of a boy.
While examining the boy, hypoplasia of the right half of the scrotum was revealed, absence of the testicle.
Testicle is miniaturized, it palpitates along the inguinal canal but it could not be moved down to scrotum.
What is the most probable diagnosis?
A. Right-sided cryptorchism, inguinal form  undescending testis
B. Ectopia of the right testicle, pubic form
C. Left-sided monorchism
D. Retraction of the right testicle (pseudocryptorchism)
E. Right-sided cryptorchism, abdominal form

72. A 62-year-old patient complains of the pain behind the sternum, bad passing of solid and liquid food, bad
breath, increased salivation weight, loss of 15 kg during the period of 2 months. Appetite is retained. On
physical exam: face features are sharpened. The skin is pale, with sallow tint, its turgor is decreased. The liver
is not enlarged. Blood Hb - 86g/L. Gregersen reaction is positive. What kind of pathology caused the given
clinical situation?
A. Esophagus cancer
B. Chronic non-specific esophagitis
C. Cicatricial constriction of esophagus
D. Benign growth of esophagus
E. Achalasia of esophagus
Dr.H.A.Salman - OdNMU
** review Q 1
Gregersen reaction: Feces test for latent blood

73. A patient suffers from suddenly arising crampy pain in the right loin area. 2 hours after the pain had
tarted, hematuria took place. Loin X-ray: no pathological shadows. Ultrasound: pyelocaliectasis on the right,
the left part is normal. What is the most probable diagnosis?
A. Stone of the right kidney, renal colic
B. Tumour of the right kidney pelvis
C. Intestine invagination
D. Acute appendicitis
E. Twist of the right ovary cyst
** pyelocaliectasis : dilation of the renal pelvis and calices. usually due to obstruction or infection.

74. A patient complains of an extremely intense pain in epigastrium. He has peptic ulcer disease of
duodenum for 10 years. The patient is in the forced position being on the right side with legs abducted to
stomach. Abdomen has acute tenderness in the epigastrium. Guarding contraction of the abdominal
wall muscles is observed. What is the preliminary diagnosis?
A. Perforation of ulcer
B. Thrombosis of mesenteric vessels
C. Acute condition of peptic ulcer disease
D. Acute pancreatitis
E. Penetration of ulcer into pancreas

75. A 30-year-old patient complains of pain, hyperemia along subcutaneous veins, rise in body temperature.
While examining the large shin subcutaneous vein, there is hyperemia, pain by pressing. Homanss and
Lusess symptoms are negative. What is the preliminary diagnosis?
A. Acute thrombophlebitis of subcutaneous veins
B. Lymphostasis
C. Acute ileophemoral phlebothrombosis
D. Embolism of aorta
E. Thrombosis of aorta
** Homans' sign or the dorsiflexion sign is considered a sign of deep vein thrombosis (DVT)
as discomfort behind the knee on forced dorsiflexion of the foot.
Dr.H.A.Salman - OdNMU
Lowenberg's sign : found in patients with DVT of the lower leg , The sign is positive when pain is elicited
rapidly when a blood pressure cuff is placed around the calf and inflated to 80mmHg. this sign is neither
sensitive nor specific for the presence of thrombosis

76. A 30-year-old man has suffered from traffic accident. Consciousness is absent. Pulse on the carotid artery
is undeterminable, no respiration. There is a wide leather belt on mans waist. What measures are to be
taken?
A. Conduct an artificial ventilation of lungs and closed-chest cardiac massage after having released from the belt
 start CPR – ABC
B. Start immediate artificial ventilation of lungs and closed-chest cardiac
massage
C. Do not touch the victim until the arrival of the police
D. Put the victim with his back on the shield
E. Turn the victim to the right side

77. An emergency team deliverd a 83-year-old patient complaining of inability of her right leg to support the
body after falling on her right side. Objectively: the patient lies on a gurney, her right leg is rotated outwards,
the outside edge of foot touches the bed. There is positive straight leg raising sign. What is your provisional
diagnosis?
A. Femoral neck fracture  straight leg raising sign not specific for neck fracture
B. Femoral diaphysis fracture
C. Hip joint contusion
D. Cotyloid cavity fracture
E. Hip dislocation

78. A 50-year-old patient complains of bursting pain in the left lower limb that is getting worse on exertion,
swelling in the region of shin and foot. Objectively: left shin and foot are doughy, skin of the lower shin is
indurated and has a bronze tint, subcutaneous veins are dilated, there is an ulcer with necrotic masses. What
is the most likely diagnosis?
A. Postthrombophlebitic syndrome
B. Gangrene of the lower extremity
C. Acute arterial thrombosis
Dr.H.A.Salman - OdNMU
D. Chronic arterial insufficiency
E. Deep vein thrombosis of the lower limbs

79. 3 days ago a 29-year-old patient presented with pulsating pain in the region of rectum, fever, general
weakness. Objectively: local tenderness in the anal region in the 6 oclock position. Digital investigation of
rectum revealed a painful infiltration reaching the pectinate line. What is the most likely diagnosis?
A. Acute periproctitis
B. Acute haemorrhoids
C. Acute prostatitis
D. Rectum tumour
E. Acute anal fissure

N.B: Paraproctitis is a purulent inflammation of the cellular tissues surrounding the rectum. The most
frequent cause is penetration of bacterial flora from the rectum into the surrounding cellular tissues, which
may occur through an anal fissure. The inflammation is sometimes limited to the formation of an abscess, and
in some cases it spreads for a considerable distance and may be complicated by sepsis.
Dr.H.A.Salman - OdNMU

80.A 65-year-old patient complains of gradual worsening of the left eye vision during 10 months. On physical
examination: acuty of vision of the left eye is 0,01, not correctable. The eye is quiet, pupil of the eye is grayish,
reflex from the eye-ground is absent. Intraocular pressure is 18 mm/Hg. What is the most probable
peliminary diagnosis?
A. Senile cataract  is an age-related, vision-impairing disease characterized by gradual progressive
thickening of the lens of the eye. It is the world’s leading cause of treatable blindness.
B. Disorder of blood circulation in retina vessels
C. Open-angle glaucoma
D. Leukoma of the cornea
E. Exfoliation of the retina

A C E

81. A 36 y.o. patient was admitted to the hospital with sharp pain in substernal area following occasional
swallowing of a fish bone. On esophagoscopy the foreign body wasnt revealed. The pain increased and
localized between scapulas. In a day temperature elevated, condition became worse, dysphagia intensified.
What complication has developed?
A. Perforation of esophagus with mediastinitis
B. Obstruction of esophagus
C. Esophageal hemorrhage
D. Pulmonary atelectasis
E. Aspirative pneumonia

82. A 19 year old patient was admitted to a hospital with acute destructive appendicitis. He suffers from
hemophilia of B type. What antihemophilic medications should be included in pre- and post-operative
treatment plan?
A. Fresh frozen plasma  good for Factor XI (9) deficiency (Hemoph B)
B. Native plasma
C. Dried plasma
D. Cryoprecipitate  factor VIII (8) , vWF ( Hemoph A )
E. Fresh frozen blood
Dr.H.A.Salman - OdNMU

83. On the 4th day after recovering from a cold a patient was hospitalized with complaints of solitary
spittings of mucoid sputum. On the 2nd day there was a single discharge of about 250 ml of purulent blood-
streaked sputum. Objectively: the patients condition is moderately severe. Respiratory rate - 28-30/min, Ps-
96 bpm, AP- 110/70 mm Hg. Respiration above the left lung is vesicular, weak above the right lung. There are
moist rales of different types above the lower lobe and amphoric breath near the angle of scapula. What is
the most likely diagnosis?
A. Acute pulmonary abscess
B. Acute focal pneumonia
C. Pyopneumothorax
D. Pleural empyema
E. Exudative pleuritis

84. A 24 y.o. male patient was transferred to the chest surgery department from general surgical department
with acute post-traumatic empyema of pleura. On the X-ray: wide level horizontal of fluid on the right. What
method of treatment should be prescribed?
A. Punction and drainage of pleural cavity 
B. Pneumoectomy  surgical removing of lung
C. Lobectomy  removing of one or more lobe of lung
D. Thoracoplasty  1- plastic surgery of the thorax, or
**surgical removal of several ribs or a part of them to permit thecollapse of a diseased lung, used in cases of p
ulmonary tuberculosisand bronchiectasis
E. Decortication of pleura  removing the layer of Pleura
Dr.H.A.Salman - OdNMU

85. A patient complains of intense pressing pain in the pharynx, mainly to the right, impossibility to swallow
even liquid food. The illness started 5 days ago. The patients condition is grave. Body temperature - 38,9°C,
speech is difficult, voice is constrained, difficulties in opening the mouth. Submaxillary glands to the right are
painful, enlarged. What is the most probable diagnosis?
A. Peritonsillar abscess  = Synonyms Quinsy , abcess formation around tonsillis
B. Vincents disease  painful ulcers and an acute destructive inflammation
of the gums called necrotizing gingivitis
C. Phlegmonous tonsillitis
D. Diphtheria
E. Pharyngeal tumour

86. In a 65 y.o. female patient a tumor 13x8 cm in size in the umbilical area and above is palpated, mild
tenderness on palpation, unmovable, pulsates. On ausculation: systolic murmur. What is the most probable
diagnosis?
A. Abdominal aortic aneurism
B. Arterio-venous aneurism
C. Stomach tumor
D. Tricuspid valve insufficiency
E. Mitral insufficiency

87. An hour before an elective surgery, a 56-year-old patient of the surgical department got a dramatic
increase in blood pressure, tachycardia, hand tremor. The patient is confused, anxious, depressed, fearful, is
pessimistic about the operation outcome, refuses the surgery. What tactics should be chosen by a surgeon?
A. Start the surgery after correction of blood pressure  protocol before each operation
B. Predict the psychological state of the patient
C. Organize monitoring of the patient by his family members
Dr.H.A.Salman - OdNMU
D. Organize monitoring of the patient by medical personnel and mental health
counselor
E. Isolate the patient

88. A patient with suspicion of pelvic bones fraction is under examination conducted by the doctor who
presses alae ilii in medial direction with his both hands. What causes painful syndrome of the patient?
A. Disorder of continuity of the pelvic ring  mean there is fracture
B. Fracture of the sacrum
C. Retroperitoneal hematoma
D. Fracture of sciatic bones
E. Traumatic injury of the intrapelvic organs
** ala ilii  iliac wings = iliac crest

89. A 30-year-old man was injured in a fire and got thermic burns of III -A and III-B degree that affected
20% of the total skin. AP - 110/70 mm Hg, heart rate -120/min. What transfusion means should be used for
blind infusion before transportation?
A. Saline solutions = normal saline (Salty)
B. Albumin
C. 10% glucose solution
D. Polyglycine
E. Fresh frozen plasma

90. A 48-year-old male patient complains of pain in the lower extremities, especially when walking,
intermittent claudication, numbness in the fingers, cold extremities, inability to walk more than 100 meters.
Sleeps with his leg lowered. The patient has been a smoker since he was 16, abuses alcohol, has a history of
CHD. The left lower extremity is colder than the right one, the skin of extremities is dry, pedal pulse cannot
be palpated, femoral pulse is preserved. What is the most likely diagnosis?
A. Obliterating endarteritis  severe proliferating endarteritis (inflammation of the intima or inner lining of
an artery) that results in an occlusion of the lumen of the artery.  pic A
typical of vasculitis associated with rheumatological disease
B. Raynauds disease  capillary vasospasm
C. Deep thrombophlebitis  inflammation of deep veins
D. Diabetic angiopathy  disease of vessles due to diabetes complication
E. Leriche syndrome  distal aorto-iliac occlusion

91. A 24 y.o. woman consulted a doctor about continued fever, night sweating. She lost 7 kg within the last 3
months. She had casual sexual contacts. Objectively: enlargement of all lymph nodes, hepatolienal syndrome.
Blood count: leukocytes - 2,2×109/L. What disease can be suspected?
A. HIV-infection  Sexual Transmitted disease
B. Infectionous mononucleosis
C. Chroniosepsis
D. Lymphogranulomatosis
E. Tuberculosis
Dr.H.A.Salman - OdNMU
** Hepatolienal syndrome = hepato-splenomegally
Causal here mean = irregular

92. Development of chronic venous insufficiency of lower extremities depends on the functional condition of
so-called musculovenous pump. This term refers to the following group of muscles:
A. Shin muscles  Leg muscles
B. Abdominal wall muscles
C. Thigh muscles
D. Foot muscles
E. Buttock region muscles

93. A 27-year-old patient was brought to clinic with a crashed wound of the posterior surface of the right shin
in 2 hours after the accident happened. During surgical treatment pieces of dirty clothes and gravel were
removed. What actions from the given below are the decisive ones to prevent an anaerobic infection?

A. Radical surgical treatment  Surgery to remove a large amount of damaged or affected tissue
and/or adjoining areas of lymphatic drainage to obtain acomplete cure. This is in contrast to conservative su
rgery. Usually this using in neoplasms
B. Introduction of the medical dose of the anti-gangrene serum
C. Hyperbaric oxygenation
D. Rational antibiotic therapy
E. Introduction of the preventive dose of an anti-gangrene serum

94. A 32-year-old patient lives in an area endemic for echinococcosis. In the last 6 months he reports of pain
in the right subcostal area, fever. He is suspected to have liver echinococcosis. What study would be the most
informative in this case?
A. USI  Ultrasongraphy Imaging – liver echinococcosis = Liver Hydatid cysts
C. Biochemical laboratory examination
D. Survey radiography of abdominal cavity
E. Angiography

95. In morgue there are dead bodies with the following causes of death: electrotrauma; rupture of the spleen
with acute anemia. There is one unknown person; one ethyl alcohol poisoned person and one drowned man.
What dead body should the blood group be determined for?
A. All dead bodies of the unknown persons  to help recognizing that unknown person by asking his family or
relatives
B. Body of person with a sudden death
C. Body of drowned man
D. Body of poisoned person
Dr.H.A.Salman - OdNMU
E. Body of person with internal hemorrhage

96. A 50 y.o. patient was admitted to the hospital with complaints of blood in urine. There was no pain or
urination disorders, hematuria has lasted for 3 days. Objectively: kidneys are not palpable, suprapubic region
has no pecularities, external genitals have no pathology. Rectal examination revealed: prevesical gland is not
enlarged. Cytoscopy revealed no changes. What disease would you think about first of all?
A. Cancer of kidney  old age hematuria without apperant pathology can suspect CA
B. Tuberculosis of urinary bladder
C. Kidney dystopy
D. Necrotic papillitis
E. Varicocele

97. A patient complains of nycturia, constant boring pain in perineum and suprapubic region, weak urine jet,
frequent, obstructed and painful urination. He has been ill for several months, pain in perineum appeared
after getting problems with urination. Rectal examination revealed that prostate is enlarged (mostly owing to
the right lobe), dense, asymmetric, its central sulcus is flattened, the right lobe is dense, painless, tuberous.
What disease can it be?
A. Cancer of prostate
B. Chronic congestive prostatitis
C. Urolithiasis, stone of the right lobe of prostate
D. Prostate sclerosis
E. Prostate tuberculosis
Dr.H.A.Salman - OdNMU
98. The patient complains of a painful swelling in the chin region, malaise, headache. Examination reveals an
acutely inflamed cone-shaped dense node. The skin over it is tense, red. In the center of the node there is an
ulcer with overhanging edges and a necrotic core of a dirty-green colour. Submandibular lymph nodes on the
right are enlarged and painful. What is the most likely diagnosis?
A. Furuncle
B. Parasitic sycosis
C. Carbuncle
D. Tuberculosis
E. Tertiary syphilis (gummatous form)

99. On the 5th day after a surgery for colon injury a patient complains of bursting pain in the postoperative
wound, weakness, drowsiness, headache, fever up to 40°C. Objectively: the skin around the wound is swollen,
there is gas crepitation. The wound discharges are scarce foul-smelling, of dark-gray colorl. What is the most
likely diagnosis?
A. Anaerobic clostridial wound infection  gas gangrene
B. Postoperative wound infection
C. Abscess
D. Erysipelas
E. Phlegmon
Dr.H.A.Salman - OdNMU
100. A childrens surgical unit admitted a 1-month-old boy who had been prenatally diagnosed with the left-
sided pyelectasis. Such studies as drip infusion urography, cystography and USI allowed to reveal initial
hydronephrosis. There is no information confirming the secondary pyelonephritis. What tactics of this
patient management is most advisable?
A. 6-month surveillance follow up 6 months to exclude pyelonephritis .
B. Anderson-Hynes operation
C. Antibacterial therapy
D. There is no need in further surveillance and treatment
E. Urgent nephrostomy

101. A 60 y.o. man complains of significant pain in the right eye, photophobia, lacrimation, reduced vision of
this eye, headache of the right part of the head. Pain occured 2 days ago. On examination: Vis OD- 0,03,
congested injection of the eye ball, significant cornea edema, front chamber is deep, pupil is narrow,
athrophic iris, there is optic nerve excavation on the eye fundus, intraocular pressure- 38 mm Hg. Vis OS- 0,8
unadjustable. The eye is calm, healthy. Intraoccular pressure- 22 mm Hg. What is the most probable
diagnosis?
A. Acute glaucoma attack
B. Eye nerves neuritis
C. Maculodystrophy
D. Right eyes uveitis
E. Right eyes keratitis
** Normal intraocular pressures average between 12-22 mm Hg

102. A 54-year-old female patient has been admitted to a hospital 12 days after the beginning of acute
pancreatitis. Objectively: the patient is in grave condition. The body temperature is hectic. Ps - 112 bpm. The
abdomen is swollen. Epigastrium palpation reveals a very painful infiltration falling in the localization of
pancreas. Abdominal cavity contains fluid. There is an edema of the left lumbar region. In blood: WBCs –
18×109/l. What is the required tactics of the patients treatment?
A. Surgical treatment  because this condition referring to pancreatic abscess as a complication
B. Massive antibacterial therapy
C. Peritoneal dialysis
D. Further conservative treatment
E. Increase in antienzymes
** grave = severe
hectic fever = intermittent fever = there is a big swing in the body temperature and the temperature rises with
chill and rigor, then it persists for few hours and suddenly falls with profuse sweating.
Dr.H.A.Salman - OdNMU

Hectic fever Seen Pyogenic infection this can


guide us to abscess formation

103. A 37-year-old male patient has a history of diabetes of moderate severity. On the left side of face the
patient has a carbuncle. What severe complication might have occurred in the patient?
A. Cavernous sinus thrombosis  CST most commonly results from contiguous spread of infection from a
nasal furuncle (50%), sphenoidal or ethmoidal sinuses (30%) and dental infections (10%)
B. Endarteritis  inflammation of the arterial intema
C. Lymphangitis  inflammation of the lymph vessels
D. Thrombophlebitis  venous inflammation due to thrombs
E. Thromboembolism  presence of Embloi (movable thromb) = TE
Dr.H.A.Salman - OdNMU

104. To replace the blood loss replacement 1000 ml of the same group of Rhesus-compatible donated blood
was transfused to the patient. The blood was conserved by sodium citrate. At the end of hemotransfusion
there appeared excitement, pale skin, tachycardia, muscles cramps in the patient. What complication should
be suspected?
A. Citrate intoxication
B. Allergic reaction
C. Citrate shock
D. Anaphylactic shock
E. Pyrogenous reaction

105. A 20 y.o. patient suddely felt ill 12 hours ago. There was pain in epigactric area, nausea, sporadic
vomiting. He had taken alcohol before. In few hours the pain localized in the right iliac area. On examination:
positive rebound tenderness symptoms. WBC- 12,2×109/L. What is the most probable diagnosis?
A. Acute appendicitis
B. Perforated ulcer
C. Acute cholecystitis
D. Rightside kidney colic
E. Acute pancreatitis
Dr.H.A.Salman - OdNMU

106. A patient, aged 58, was fishing in the winter. On return home after some time felt some pain in the feet.
Consulted a doctor. On examination: feet skin was pale, then after rewarming became red, warm to the touch.
Edema is not significant, limited to the toes. All types of sensitivity are preserved. No blisters. What degree of
frostbite is observed?
A. I degree
B. III degree
C. II degree
D. IV degree
E. V degree

1- Frostbite of the I stage


 Duration of cold action is short. Paleness changes into hyperemia. Sensitiveness is saved. Moves of
fingers of hand and feet are active. Edema is progressing. Pain is severe. Full recovery in 5-7 days.

2- Frostbite of the II stage


 During first days severe pain and itching. Edema of tissues is bigger than injured area. Main sign is
formation of blisters with plasma like transparent liquid. Blisters appear on 2 day after the injury.
Derivates of skin are saved that is why full restoring of skin cover is possible and it takes 1-2 weeks.
Nails will be restored. Scars are not permanent.

Frostbite of the I-II stage

3- Frostbite of the III stage


Dr.H.A.Salman - OdNMU
- Necrosis of all skin layers sometimes even injury of subcutaneous tissue. Blisters with hemorrhage
liquid. Decreasing of sensitiveness. Skin in reactive period is red and cyanotic. Cold on touch. If
proper treatment with autoplastic is not used, big connective tissue scars appear.
4- Frostbite of the IV stage
 Injury includes destroying of bones and joints. Temperature of skin in bеfore reactive period is
extremely decreased. Edema appears during first 1-2 hours, than - dry or wet gangrene of injured
areas. Demarcation line appears after 1-2 weeks. Edema takes much larger area than gangrene.

Frostbite of the III-IV stage

107. After an accident a patient complains of pain in the hip joint. Objectively: the leg is in the position of
flexion, adduction and internal rotation, significantly contracted. There is elastic resistance to passive
adduction or abduction of the extremity. Major trochanter is located high above the Roser-Nelaton line .
A significant lordosis is present. What is your provisional diagnosis?
A. Iliac dislocation of hip
B. Femoral neck fracture with a displacement
C. Inferoposterior dislocation of hip
D. Pertrochanteric fracture of hip
E. Cotyloid cavity fracture with a central dislocation of hip
Dr.H.A.Salman - OdNMU

108. Three weeks after acute angina the patient is still weak, inert , subfebrile, his retromaxillary lymph
nodes are enlarged. Tonsils are flabby, stick together with arches, there are purulent plugs in lacunae. What is
the most probable diagnosis?
A. Chronic tonsillitis
B. Paratonsillitis
C. Tonsillar tumour
D. Chronic pharyngitis
E. Acute lacunar tonsillitis
Angina = tonsillitis

109. A 43 year old patient had right-sided deep vein thrombosis of iliofemoral segment 3 years ago. Now he is
suffering from the sense of heaviness, edema of the lower right extremity. Objectively: moderate edema of
shin, brown induration of skin in the lower third of shin, varix dilatation of superficial shin veins are present.
What is the most probable diagnosis?
A. Postthrombophlebitic syndrome, varicose form
B. Parkes-Weber syndrome  is a rare congenital condition characterized by a large number of abnormal
blood vessels
C. Acute thrombophlebitis of superficial veins
D. Acute thrombosis of right veins
E. Lymphedema of lower right extremity

110. A child undergoes in-patient treatment for acute staphylococcal destruction of the right lung.
Unexpectedly he developed acute chest pain on the right, dyspnea, cyanosis. The right side of chest lags
behind in the respiratory act. Percussion reveals dullness in the lower parts on the right, bandbox resonance
Dr.H.A.Salman - OdNMU
in the upper parts. Borders of the relative cardiac dullness are shifted to the left. What complication has most
likely developed?
A. Right-sided pyopneumothorax  pus and air in pleural cavity
B. Spontaneous pneumothorax
C. Pleural empyema
D. Exudative pleuritis
E. Right lung abscess

111. A 98 y.o. male patient complains of pain in the left lower limb which intensifies on walking, feeling of
cold and numbness in both feet. He has been ill for 6 years. On examination: pale dry skin, hyperkeratosis.
Hairy covering is poorly developed on the left shin. "Furrow symptom " is positive on the left. Pulse on foot
arteries and popliteal artery isnt palpated, on the femoral artery its weak. On the right limb the artery
pulsation is reserved. What is the most probable diagnosis?
A. Arteriosclerosis obliterans
B. Obliterating endarteritis
C. Raynaulds disease
D. Buergers disease (thromboangiitis obliterans)  young men , SMOKER
E. Hemoral arthery thombosis

112. Examination of a dead man who died from hanging revealed that cadaver spots disappeared when
pressed upon and restored after 50 seconds, rigor mortis was moderately expressed only in the masticatory
muscles and the muscles of neck and fingers. Body temperature was 31°C. Specify the time of death:
A. 6-7 hours
Dr.H.A.Salman - OdNMU
B. 8-10 hours
C. 10-18 hours
D. 1-2 hours
E. 16-24 hours

Body cooling : rate of cooling is not uniform but is almost proportional to the d/f in temperature between
the body and surroundings. Average heat loss is roughly 1°С per hour and the body attains
environmental temperature in about 16-20 hours after death.

Post-mortem Lividity. It begins as mottled patches within 1-3 hours. These patches increase in size, coalesce
in 3-6 hours when the lividity is fully developed and fixed in about 6-12 hours. Despite any stage of its
development, it has a definite duration.
Rigor Mortis. It commences 2-3 hours after death, develops in 8-12 hours, persists for 2-3 days, and passes
off 3-4 days after death.
Decomposition Changes.
Bad odor is noticed after 20-30 hours have elapsed since death.
Greenish color of the right iliac region is presented 2-3 days since death. Presence of the green coloration
over the entire abdomen, genitals, extremities and face indicates 4-5 days after death. Full discoloration of
the body suggests 1-2 weeks post-mortem interval. In 1-3 months, fulldecomposition of the body occurs.
Adipocere formation and mummification take place in 3-10 months.
Contents of the Stomach and bowels. Milk, tea, coffee leave stomach fairly rapidly (15-20 min.) mixed diets
(meat, vegetables) exit the stomach in 4-5 hours. Conditions like fear, shock or coma delay the emptying rate
and power of digestion.
Supervital reactions. The dead tissue of the human body are proved to react to the actions of some
mechanical and chemical stimuli. All those reactions can be detected during a limited time period. That is
why they can be used for calculating the post-mortem interval.

113. Examination of the corpse of a man who died from hanging reveals: death spots disappear when pressed
upon and restore after 50 seconds, rigor mortis is moderately expressed only in the masticatory muscles as
well as neck and finger muscles, body temperature is of 31°C. The time of death:
A. 6-7 hours ago
B. 10-18 hours ago
C. 16-24 hours ago
D. 1-2 hours ago
E. 8-10 hours ago
** same above

114. After contusion of the right eye a patient complains of sudden loss of vision with remaining light
perception. Objectively: the eye is not irritated. The cornea is transparent. Pupil reacts to light. The pupil area
is black. The fundus reflex is absent. What is the most likely cause of vision loss?
A. Hemophthalmia  intraocular hemorrhage , due to trauma (contusion)
B. Acute occlusion of retinal vessels
C. Optic nerve avulsion
D. Retinal detachment
E. Traumatic cataract
Dr.H.A.Salman - OdNMU

115. A 65-year-old patient complains of pain in the lumbar spine, moderate dysuria. He has been suffering
from these presentations for about half a year. Prostate volume is 45 cm3 (there are hypoechogenic nodes in
both lobes, capsule invasion). The rate of prostate-specific antigen is 60 ng/l. Prostate biopsy revealed an
adenocarcinoma. Which of the supplemental examination methods will allow to determine the stage of
neoplastic process in this patient?
A. Computer tomography of pelvis
B. Excretory urography
C. Roentgenography of lumbar spine
D. Bone scintigraphy
E. Roentgenography of chest
** note : for PROSTATE CA staging better do MRI , while here they used CT. , anyhow just keep it for exam.

116. X-ray picture of chest shows a density and an abrupt decrease in the upper lobe of the right lung. The
middle and lower lobe of the right lung exhibit significant pneumatization. The right pulmonary hilum comes
up to the dense lobe. In the upper and middle parts of the left pulmonary field there are multiple focal
shadows. In the basal region of the left pulmonary field there are clear outlines of two annular shadows with
quite thick and irregular walls. What disease is this X-ray pattern typical for?

A. Fibro-cavernous pulmonary tuberculosis  chronic destructive process, characterized by the presence of an old
fibrous cavern, expressed fibrosis and nidi of bronchogenic dissemination in lung tissue, surrounding the cavern, or in
other parts of the lungs; protracted undulant course with aggravations and remissions periods, constant or periodic
bacterial secretion. In the social aspect fibrous-cavernous lung tuberculosis patients are invalids, predominantly of the
2-nd group.
B. Pancoast tumour  or syndrome typically results when a malignant neoplasm of the superior sulcus of the lung
(lung ca) leads to destructive lesions of the thoracic inlet and involvement of the brachial plexus and cervical
sympathetic nerves (stellate ganglion)
C. Abscessing pneumonia
D. Atelectasis of the right upper lobe
E. Peripheral cancer
Dr.H.A.Salman - OdNMU
117. A patient complains of impaired far vision. Previously his eyes often mturned red and hurt. Objectively:
the eyes are not irritated, the cornea is transparent, anterior chambers are median deep, their liquid is
transparent. The iris of the right eye has not changed in colour, its pattern is unchanged. The pupil is of
irregular shape, scalloped. Biomicroscopy of the crystalline lens reveals the areas of opacity and vacuoles.
Make a diagnosis:
A. Complicated cataract of the right eye
B. Diabetic cataract of the right eye
C. Radiation cataract of the right eye
D. Tetanic cataract of the right eye
E. Senile cataract of the right eye

118. A 26-year-old patient complains of experiencing pain in the right hand for 4 days. The pain arose at the
site of corns on the palmar surface at the base of the II and III fingers. Some time later the dorsum of hand
became edematic. I and III fingers are half-bent in the interphalangeal joints, the hand looks like "rake".
What is the most likely diagnosis?
A. Phlegmon of the second interdigital space of the right hand
B. Tendovaginitis
C. Corn abscess of the right hand
D. Adenophlegmon of the right hand
E. U-shaped phlegmon of the right hand

119. A 30-year-old patient had deep burn covering 30% of body 30 days ago. Now he presents with continued
fever, loss of appetite, night sweats. Burned surface weakly granulates. What is the stage of burn disease?
Dr.H.A.Salman - OdNMU
A. Septicotoxemia
B. Convalescence
C. Secondary burn shock
D. Primary burn shock
E. Acute burn toxemia

four periods: I \burn shock, II — an acute burn toxaemia, III — a septicotoxemia (a burn infection), IV —
reconvalescence.

I. Burn shock is the first period of a burn disease. Shock duration (from several hours to several days) is
defined mainly by the area of defeat. Any burn wound is initially mikrobno zafyaznenny, however in the
period of burn shock influence of an infection is not expressed yet.
II. The acute burn toxaemia is the second period of a disease. It begins from 2 — 3 days, 7 — 8 days proceed
and it is characterized by dominance of the phenomena of the expressed intoxication.
III. The septicotoxemia period (a burn infection) conditionally begins from 10th days and is characterized by
dominance of an infectious factor during a disease. At negative dynamics of process development of the burn
cachexia leading in the subsequent to death of the patient is possible.
IV. The period of reconvalescence is characterized by gradual normalization of functions and systems of an
organism. It comes after healing of burn wounds, or after their operational closing.

It is considered that at a superficial burn of any degree of 15 — 20% of a body surface or at a deep burn more
than 10% of a body surface usually develop burn shock. Its degree depends on extensiveness of a burn: at the
total area of defeat to 20% little burn shock, from 20% to 60% — heavy usually develops and at more
extensive defeat — extremely heavy burn shock (V. K. Sologub and soavt., 1979).
JUST SEE THE YEAR OF THIS CLASSIFICATION  

120. A 29 y.o. patient was admitted to the hospital with acute girdle pain in epigastric area, vomiting in 1 hour
after the meal. On examination: pale, acrocyanosis. Breathing is frequent, shallow. Abdominal muscles are
tensed, positive Schotkin-Blumbergs symptom. What is the maximal term to make a diagnosis?
A. In 2 hours  signs of Peritonitis need early diagnosis .
B. In 3 hours
C. In 6 hours
D. In 0.5 hours
E. In 1 hour

121. A surgeon examined a 42-year-old patient and diagnosed him with right forearm furuncle at the purulo-
necrotic stage. The furuncle was lanced. At the hydration stage the wound dressing should enclose the
following medication:
A. Hypertonic solution
B. Dimexide
C. Ichthyol ointment
D. Vishnevsky ointment
E. Chloramine
Dr.H.A.Salman - OdNMU

Note:
Phases of wounds’ reparation
Rufanov differ 2 phases: hydration and dehydration.
Girgolav determined 3 period of wounds reparation:
1. preparing period.
2. the period of regeneration.
3. the period of stitch formation.
In present time the most popular classification is (Cusin, 1977 year):
1. the phase of inflammation (1-5 day). It has period of vessel’s changes and period of purifying of
the wound from necrosis.
2. the phase of regeneration (6-14 day).
3. the phase of formation and stitch reorganization (begin from 15-th day).

In the second phase of healing process of formation of granulative tissue plays the leading role. Despite it has
a protective function; the repeated inflammation cannot be totally excluded. In this period in case of absence
of complications, exudation decreases and necessity in hygroscopic bandage, using of hypertonic solutions
and draining disappears. Granulations are very tender and vulnerable, that’s why it is necessary to use
preparation on ointment basis, which prevent mechanical traumatization. Antibiotics (syntomycine,
tetracycline, hentamycine ointments etc.), stimulating substances (5% and 10% methyluracyl acid,
“Solcoseryl”, “Actovegyn”) are introduced into the content of ointments, emulsions and liniments.

122. A 30 year old patient undergoes treatment because of Werlhofs disease. Objectively: the patient is pale,
there are petechial haemorrhages on the extension surfaces of forearms. Ps is 92 bpm, AP is 100/60 mm Hg.
The lower edge of spleen is at a level with umbilicus. Blood count: erythrocytes: 2,8×1012/l, Hb - 90 g/l, Ht -
0,38, thrombocytes – 30×109/l. The patient is being prepared for splenectomy. What transfusion medium
should be chosen in the first place for the preoperational preparation?
A. Thrombocytic mass
B. Erythrocytic suspension
C. Washed erythrocytes
D. Stored blood
E. Native erythrocytic mass
Note :
Werlhofs disease immune thrombocytopenia (ITP) is a type of thrombocytopenic purpura defined as
isolated low platelet count (thrombocytopenia) with normal bone marrow and the absence of other causes of
thrombocytopenia.
 Vorob'ev A.I.; Gorodetskii V.M., 1980: Thrombocytic mass methods of preparation and efficacy of
clinical use. ( why somebody tell me ? 1980 !! ) ok pass it !

123. A 19 year old boy was admitted to a hospital with closed abdominal trauma. In course of operation
multiple ruptures of spleen and small intestine were revealed. AP is falling rapidly, it is necessary to perform
hemotransfusion. Who can specify the patients blood group and rhesus compatibility?
A. A doctor of any speciality
B. A traumatologist
C. An anaesthesilogist
D. A laboratory physician
E. A surgeon
 what he was thinking when he put this Q .. Really

124. A 36-year-old man was delivered to the surgical department an hour after a road accident. His condition
is getting worse: respiratory insufficiency is progressing, there are cardiac abnormalities. Clinical and
roentgenological investigations revealed mediastinal displacement. What process has caused this
complication?
Dr.H.A.Salman - OdNMU
A. Valvular pneumothorax  Valvular Pneumothorax = it is due to
a wound in the pulmonary pleura which allows of the passage of air from the lung into the pleural cavity but
closes under pressure and does not permit of the return of air into the lung.
B. Closed pneumothorax
C. Open pneumothorax
D. Subcutaneous emphysema
E. Mediastinitis
Roentgenological mean X ray

125. A victim of the car crash was brought to neurosurgery due to craniocerebral trauma. During examination
symptoms of focused injury of the cerebrum are found, cerebrum contusion is suspected. What method of
examination is to be used?
A. Computed tomography of the cerebrum  CT scan is the gold standard for Brain Trauma.
B. Eye-ground examination
C. Antiography of vessels of the cerebrum
D. X- ray of cranium bones
E. Ultrasound examination of vessels of the cerebrum

126. A 35 y.o. female patient was admitted to the surgical department with symptoms of ulcerative gastric
hemorrhage. Its been the third hemorrhage for the last 2 years. After conservative treatment vomiting with
Dr.H.A.Salman - OdNMU
blood stopped, hemoglobin elevated from 60 till 108 g/L. General condition became better. But profuse
vomiting with blood reoccured in 2-3- hours. Hemoglobin decreased to 93,1 g/L then to 58,1 g/L. What is the
tactics of treatment?
A. Urgent surgery  to prevent Hypovolemic shock due to repeated hemorrhage from ulcer.
B. Conservative treatment with following surgery
C. Taylors treatment
D. Deferred surgery
E. Conservative treatment

127. A 39-year-old patient complains of a tumour on the anterior surface of her neck. The tumour has been
observed for 2 years. It is nonmobile and has enlarged recently. The patient has a changed tone of voice, a
sense of pressure. Objectively: in the left lobe of the thyroid gland a 3 cm node is palpable; it is very dense,
tuberous, painless. Cervical lymph nodes are enlarged. Functional status of the thyroid gland is unchanged.
What is the most likely diagnosis?
A. Thyroid gland cancer
B. Nodular hyperthyroid goiter  in question gland function not changed
C. Nodular euthyroid goiter  No Lymphadenopathy
D. Chronic lymphomatous Hashimotos thyroiditis  autoimmune causing hypothyroidism
E. Chronic fibrous Riedels thyroiditis  systemic disease that can affect many organ systems called IgG4-
related disease. It is often a multi-organ disease affecting pancreas, liver, kidney, salivary and orbital tissues
and retroperitoneum. The hallmarks of the disease are fibrosis and infiltration by IgG4 secreting plasma cells
** characterized by a replacement of the normal thyroid parenchyma by a dense fibrosis that invades
adjacent structures of the neck and extends beyond the thyroid capsule. This makes the thyroid gland stone-
hard (woody) and fixed to adjacent structures. The inflammatory process infiltrates muscles and causes
symptoms of tracheal compression. Surgical treatment is required to relieve tracheal or esophageal
obstruction.

128. An unconscious victim in severe condition is brought to clinic. It is known that the patient touched the
bare wire with his hand and during 5 minutes was under the influence of an alternating current with voltage
of 220 V. Physical exam: skin is pale, cold by touch. Breath is weakened, BP - 90/50 mm Hg, Ps - 60 bpm,
arrhythmical. There are fields of necrosis of the skin on the right hand and on the right foot. What is the
preliminary diagnosis?
A. Electroburn of the right foot and right hand
B. High voltaged electroburn of the right foot and shank
C. Electrotrauma, acute cardiovascular failure
D. Vascular collapse due to electric current lesion

129. A 47-year-old patient complains about cough with purulent sputum, pain in the lower part of the left
chest, periodical body temperature rise. She has been suffering from these presentations for about 10 years.
Objectively: "drumstick" distal phalanges. What examination would be the most informative for making a
diagnosis?
A. Bronchography  x ray with contrast media injected through the bronchus.
B. Pleural puncture
C. Bacteriological analysis of sputum
D. Bronchoscopy
E. Survey radiograph of lungs
** the pt have Obstructive lung diseases symptoms , the difference between bronchograpy and bronchoscopy
by graphy (contrast x ray) can reach to all the small bronchiols inside lung , while in scope reaching the main
bronchi and may be the primary bronchiols , this procedure now replaced by CT scan
Dr.H.A.Salman - OdNMU

130. Blood typing resulted in positive isohemagglutination reaction with standard sera of A(II) and B(III)
groups and negative reaction with sera of 0(I) and AB(IV) groups. What is this result indicative of?
A. Faulty standard sera  it can’t be negative for AB and O in the same time, should one of them +ve
B. The first blood group
C. The third blood group
D. The fourth blood group
E. The second blood group

131. A victim of a road accident, aged 44, is operated on account of intraperitoneal haemorrhage. In which
case can the patients blood from the abdominal cavity be used for autotransfusion?
A. Stomach rupture
B. Liver rupture
C. Bladder rupture
D. Splenic rupture
E. Small intestines rupture

132. A man with a stab-wound of the right foot applied to doctor. He stepped on the board with nail two
hours ago. In the patient medical chart it is stated that 3 years ago he passed the whole course of vaccination
against tetanus. What is the tactics of doctor to prevent tetanus in this case?
A. Do not conduct specific prophylaxis  not need because he already recently vaccinated
B. Administer 1,0 ml of tetanus toxoid
C. Administer 1 ml tetanus toxoid and 3000 U of anti-tetanic serum
D. Administer 3000 U of anti-tetanic serum
E. Administer 0,5 ml of tetanus toxoid

133. A 42-year-old patient with acute haemorrhage and class III blood loss underwent blood transfusion and
got 1,8 l of preserved blood and erythromass of the same group and Rh. After the transfusion the patient
complained of unpleasant retrosternal sensations, his arterial pressure dropped to 100/60 mm Hg, there
Dr.H.A.Salman - OdNMU
appeared convulsions. Blood serum calcium was at the rate of 1,7 millimole/liter. What is the mechanism of
this complication development?
A. Citrate binds calcium ions, hypocalcemia impairs myocardial function  sodium citrate this compound add
to blood bag to prevent it from agglutination. In this pt. got complication after transfusion because citrate bind
Ca therefore Ca in blood  and symptoms of hypocalcaemia appeared like convulsions .
B. Citrate causes the development of metabolic acidosis
C. Citrate binds potassium causing severe hypokalemia
D. The increased citrate rate causes convulsions
E. Citrate is cardiotoxic and nephrotoxic

134. A 40-year-old woman with acute gastrointestinal hemorrhage was transfused 400 ml of the preserved
blood after having conducted all compatibility tests. After hemotransfusion the condition of the patient
became worse. She started having headache, muscles pain, her temperature has risen up to 38,8°C. What is
the reason for such condition of the patient?
A. Pyrogenic reaction of medium severity  pyrogenic mean temperature reaction , what’s this reaction mean I
don’t know just god and the person who put this st..id Q .
B. Allergic reaction  no specific symptoms for allergy
C. Air embolism
D. Bacterial and toxic shock development  no sign of shock , like BP , HR, weak pulse …...etc
E. Hemotransfusional shock development  no sign of shock , like BP , HR, weak pulse …...etc

135. A 27-year-old patient with a knife stomach wound has been delivered to a hospital 4 hours after injury.
Objectively: the patient is in grave condition. Ps- 120 bpm, weak. AP- 70/40 mm Hg. Laparotomy revealed a
lot of liquid blood in the abdominal cavity. The patient has been found to have bleeding from the mesenteric
vessels of the small intestine. Damage to hollow organs has not been revealed. What is the best way to restore
the blood loss?
A. Autoblood reinfusiont
B. Transfusion of washed erythrocytes
C. Rheopolyglucinum transfusion
D. Transfusion of fresh frozen plasma
E. Erythromass transfusion
** stomach bleeding  we can do autotrasfusion. We give this blood again for the same pt.

136. A patient presented to a hospital with a carbuncle of the upper lip. The body temperature is 39°C. There
is a pronounced edema of the upper lip and eyelids. What is the surgeons tactics of choice?
A. Hospitalize in the surgical unit
B. Disclose the carbuncle and administer out-patient treatment
C. Administer out-patient course of antibiotics
D. Administer physiotherapy
E. Disclose the carbuncle and administer antibiotics

137. A patient, aged 25, suffering from stomach ulcer. Had a course of treatment in the gastroenterological
unit. 2 weeks later developed constant pain, increasing and resistant to medication. The abdomen is painful
in epigastric area, moderate defence in pyloroduodenal area. Which complication development aggravated
the patients state?
A. Malignisation  they think that because resistance to treatment .
B. Stenosis
C. Perforation
D. Penetration
Dr.H.A.Salman - OdNMU
E. Haemorrhage

138. A 62-year-old male patient complains of intense pain in the left leg that suddenly arose three hours
before, leg numbness and coldness. During the year there has been pain in the leg while walking,
hypersensitivity to cooling. Objectively: the left foot and shin have marbled skin, subcutaneous veins are
collapsed. The foot is cold, active movements of the foot and toes are preserved. Pulse is present only on the
femoral artery. There is rough systolic murmur above the artery. Make a provisional diagnosis:
A. Acute occlusion of the left femoral artery  also called acute limb ischemia
B. Stenosis of the left popliteal artery
C. Acute arterial thrombosis ileofemoralny
D. Acute thrombophlebitis
E. Occlusive disease

139. A patient complains about pyrosis and permanent pain behind his breastbone. When he bends forward
after eating there appears regurgitation. Roentgenological examination revealed extrasaccular
cardiofunctional hernia of esophageal opening of diaphragm. Esophagoscopy revealed signs of
refluxesophagitis. What is the necessary tretment tactics?
A. Operation in a surgical department  to repair the hernia
B. Treatment at a health resort
C. Conservative treatment in the therapeutic hospital
D. Conservative treatment in an outpatients clinic
E. Conservative treatment in a surgical department
** pyrosis ; mean heartburn, a burning sensation in the upper abdomen. , breastbone ; mean sternum ,
Dr.H.A.Salman - OdNMU
140. A patient aged 18 with a cranial injury was in comatose state during several hours. In post-comatose
period gets tired quickly, non-productive in dialog - in the beginning answers 2-3 questions, then gets tired
and can not understand the point of the question. Which psychotropic should be given to the patient to
prevent psychoorganic syndrome?
A. Nootropics
B. Antidepressants
C. Stimulators
D. Neuroleptics
E. Tranquillisers
Psych Q should in Internal
** Nootropics , known as smart drugs and cognitive enhancers, are drugs, supplements, and other substances
that improve cognitive function, particularly executive functions, memory, creativity, or motivation, in
healthy individuals.
drugs e.g: CNS stimulants : Amphetamine e.g., dextroamphetamine ,
Methylphenidate – a Benzylpiperidine
Eugeroics (armodafinil and modafinil)
Xanthines (most caffeine)
Nicotine
Racetams, such as piracetam, oxiracetam, and aniracetam
Miscellaneous L-Theanine , Levodopa , Atomoxetine

141. A 45-year-old male patient with acute abscess of the left lung has suddenly developed acute chest pain
and dyspnea while coughing, tachycardia has increased. The control Ro-gram shows left lung collapse, the air
in the left pleural cavity and a horizontal fluid level. What is the mechanism of this complication?
A. Abscess burst into the pleural cavity  abscess ruptured (air fluid level) also called Empyema.
B. Bullae rupture of the left lung
C. Atelectasis of the left lung
D. Acute cardiovascular insufficiency
E. Inflammation spread to the visceral pleura

142. Examination of a 38-year-old patient who had been hit with a blunt object on the left side of chest
revealed a fracture of the X rib with fragments displacement, parietal pneumothorax. The patient complains
of pain in the left subcostal area. Objectively: the patient is pale, AP- 80/40 mm Hg, Ps- 138/min, of poor
volume. USI reveals fluid in the left abdomen. Splenic rupture is present. What treatment tactics should be
chosen?
A. Drainage of the left pleural cavity followed by laparotomy  first we do puncture to relief Pneumothorax
because it is life threatening , then surgical open the abdomen to repair the splenic rupture .
B. Immediate laparotomy and alcohol-novocaine block of the X rib
C. Immediate upper median laparotomy followed by drainage of the left pleural
cavity
D. Anti-schock actions followed by laparotomy after the arterial pressure rise
Dr.H.A.Salman - OdNMU
E. Left-sided thoracotomy immediately followed by laparotomy

143. On the 2nd day after a surgery for toxic mixed goiter IV a 35-year-old patient complains of heart pain.
ECG shows prolonged QT intervals. Chvosteks and Trousseau symptoms cannot be clearly defined. The
patient is provisionally diagnosed with latent tetany. What study will allow to confirm the diagnosis?
A. Determination of blood calcium and phosphor  to confirm the tetanus , should get hypocalcaemia because
we didn’t determined it through that symptoms (chvosteks and Trousseau) which is specific for Hypocalcaemia .
B. Determination of potassim
C. Determination of thyroid hormones
D. Determination of sodium
E. Determination of thyrotropic hormone

144. A patient with bilateral hydrothorax has repeatedly undergone pleural puncture on both sides. After a
regular puncture the patients condition has become worse: he presents with fever, chest pain. The next day,
the attending physician performing pleural puncture revealed some pus on the right. What is the mechanism
of acute right-sided empyema development?
A. Contact-and-aspiration  recurrent needle insertion and drainage caused infection then pus
B. Hematogenous
C. Lymphogenous
D. Implantation
E. Aerial

145. A 25 y.o. patient was admitted with chest trauma. Clinical and X-ray examination have revealed tense
pneumothorax on the left. What emergency treatment should be undertaken?
A. Pleural cavity drainage  tension Pntx need urgent puncture , check Q 124
Dr.H.A.Salman - OdNMU
B. Analgetics
C. Oxigenotherapy
D. Intravenous infusions
E. Intubation

146. A 38 y.o. patient lifted a heavy object that resulted in pain in the lumbar part of spine irradiating to the
posterior surface of his left leg. The pain increases during changing body position and also in the upright
position. Examination revealed positive symptoms of tension. What is the preliminary diagnosis?
A. Pathology of intercostal disks  could be disc herniated and prolapsed causing compression on spinal canal
and nerve root of the lt side that make the pain descended to the Lt lower limb, also called sciatica
B. Myelopathy
C. Arachnomyelitis
D. Spinal cord tumor
E. Polyneuritis

147. Two hours ago a 38-year-old patient got pain in his right shin. He was diagnosed with popliteal artery
embolism, acute arterial insufficiency of grade I. What is the most appropriate therapeutic tactics?
A. Embolectomy  removing the embolism , because it can circulate from lower limbs to the heart and brain
and causing occlusion of the blood supply causing acute MI or stroke
B. Resection of the popliteal artery
C. Amputation at the middle of shin
D. Bypass grafting
E. Destruction of the embolus by the catheter

148. A 38-year-old patient has suddenly developed pain in the left side of his chest, suffocation. Objectively:
moderately grave condition, Ps- 100/min, AP-90/60 mm Hg, breath sounds on the left cannot be uscultated.
Chest radiography shows the collapse of the left lung up to 1/2. What kind of treatment should be
dministered?
A. Passive thoracostomy  too much fluid not need suction (active drainage) , passive enough
B. Rest, resolution therapy
C. Operative therapy
D. Active thoracostomy
E. Pleural puncture
Dr.H.A.Salman - OdNMU

149. A 47-year-old female patient complains of leg heaviness, fatigue when standing and walking. This feeling
disappears when she takes a horizontal position. Objectively: dilatation of the superficial veins of the left shin
and thigh with pigmentation and trophic skin disorders. What functional test should the examination be
started with?
A. Trendelenburgs test
B. Pratt test 2
C. Sheinis test
D. Perthes test
E. Pratt test 1
Dr.H.A.Salman - OdNMU

150. A neonatologist examining a full-term mature baby revealed the shortening and external rotation of the
newborns lower extremity. Clinical examination revealed positive Ortolani sign, symptom of non-vanishing
pulse, additional skin folds on the internal surface of thigh. What is the most likely diagnosis?
A. Congenital hip dislocation
B. Fracture of the femoral shaft
C. Varus deformity of the femoral neck
D. Dysplasia of the hip joint
E. Femoral neck fracture

151. A child is being discharged from the surgical department after conservative treatment of invagination.
What recommendations should doctor give to mother to prevent this disease recurrence?
A. Strict following of feeding regimen
B. Hardening of the child
C. Feces observation
D. Common cold prophilaxis
E. Gastro-intestinal disease prevention

152. A 10-year-old child has been admitted to a hospital with a closed craniocerebral injury with suspected
cerebral edema. The patient is in grave condition, unconscious. The dyspnea, tachycardia, hypertension are
present. Muscle tone is increased, there is nystagmus, pupillary and oculomotor reactions are disturbed. The
mandatory component of intensive care is dehydration. What diuretic is adequate in this case?
A. Furosemide
B. Spironolactone
C. Moduretic
D. Mannitol
E. Hydrochlorthiazide
** for brain Edema better to use Mannitol , I don’t know why they answered Fursemide ??

153. A 40-year-old patient underwent an operation for a lumbar phlegmon. Body temparature rose again up
to 38°C, he got intoxication symptoms, there was an increase of leukocyte number in blood. The wound that
was nearly free from necrotic tissues and full of granulations started to discharge pus, the granulations
turned pale. What complication dveloped in this patient?
A. Sepsis
B. Erysipelas
C. Erysipeloid
D. Allergic reaction
E. Putrid phlegmon

154. After a car accident a 37-year-old patient has an acute pain and bleeding in the middle third of his right
thigh. Objectively: there is a wound on the anterior surface of the right thigh with massive bleeding,
abnormal mobility at the level of the middle third of the thigh. The first aid is to be started with:
Dr.H.A.Salman - OdNMU
A. Digital occlusion of the femoral artery
B. Venipuncture and intravenous infusion of polyglycine
C. Tourniquet application
D. Injection of pain-killers
E. Immobilization of the extremity with a transportation splint

Black arrow the occlusion of the FA , Red arrow : is the collateral circulation

155. A 42-year-old builder consulted a doctor about a foot injury with a nail that he got in the morning of the
same day. The wound was washed with water. Three years ago he was vaccinated against tetanus.
Examination established satisfactory condition of the patient. The left foot was slightly edematic, there was a
stab wound on the sole. In order to prevent tetanus it is primarily required to:
A. Give an intravenous injection of 0,5 ml of tetanus anatoxin
B. Administer a course of antibiotic therapy
C. Give an intravenous injection of 3000 IU of antitetanus serum
D. Give an intravenous injection of 1 ml of tetanus anatoxin, 3000 IU of
antitetanus serum
E. Treat the wound with suds

156. A 9-year-old boy fell from a tree and hit the occipital region, there was a momentary loss of
consciousness. Objectively: the childs condition is satisfactory, he complains of the headache and dizziness.
The X-ray of skull reveals a comminuted depressed fracture of occipital bone in the region of inion. What
treatment is indicated for the patient?
A. Surgical intervention
B. Hemostatic therapy
C. Anti-inflammatory therapy
D. Therapeutic lumbar punctures
E. Complex conservative treatment
Fell = fall
Inion = External occipital protuberance

157. A 55-year-old patient complains of severe itching, burning and pain in the eyes, skin redness in the outer
corners of the palpebral fissure. Objectively: skin around the outer corners of the palpebral fissure is
macerated, eczematous, there are single moist cracks. Palpebral conjunctiva is hyperemic, quaggy. There are
minor discharges in form of stringing mucus. What is the most likely diagnosis?
A. Chronic conjunctivitis
B. Blepharitis  Eyelid inflammation
C. Atopic eyelid dermatitis
D. Acute conjunctivitis
E. Sty  stye also called (hordeolum) , is a bacterial infection of an oil gland in the eyelid
Dr.H.A.Salman - OdNMU

158. A 40-year-old female patient has a history of rheumatism. She complains about acute pain in her left
eye, especially at night, vision impairment, photophobia, lacrimation. The patient cannot suggest any
reasons for the disease. Objectively: weak pericorneal injection, flattening of iris relief, iris discoloration.
What is the most likely diagnosis?
A. Iridocyclitis  inflammation of Iris and ciliary body
B. Keratitis
C. Iritis  inflammation of the iris only
D. Choroiditis  inflammation of the chorionic fluid of eye
E. Acute attack of glaucoma
** Iridocyclitis: pt complains of photophobia, lacrimation, dull pain and reduction of vision.
Clinical features:
— pericorneal vascular injection;
- hyperemia of the iris and the presence of the iris nodules;
- the pupil is constricted and react poorly to light;
- aggregates of cells adhere to the corneal endothelium (keratic precipitates).
** Uveitis; iritis and iridocyclitis (anterior uveitis), and choroiditis (posterior uveitis)

159. 14 days ago a 49-year-old patient was operated on for perforated appendicitis and disseminated
fibrinopurulent peritonitis. The postoperative period was uneventful. On the 9th day the patient presented
with low-grade fever, abdominal pain, frequent liquid stools. USI of the left mesogastrium reveals a fluid
formation 9x10 cm large. In blood: leukocytosis with the left shift. What is your provisional diagnosis?
A. Interloop abscess  fluid collection due to perforated appendix and peritonitis and that fluid accumulated
inside intestinal loops
B. Liver abscess
C. Abdominal cavity tumour
D. Left kidney cyst
E. Spleen abscess
Dr.H.A.Salman - OdNMU

160. A patient, aged 81, complains of constant urinary excretion in drops, feeling of fullness in the lower
abdomen. On examination: above pubis there is a spherical protrusion, over which there is a dullness of
percussion sound, positive suprapubic punch. What symptom is observed in this patient?
A. Paradoxal ischuria  mean sudden urine retention  in some Q , they put both of them in one question ,
ischuria and urine retention , but REMEMBER HERE , pt. have constant urinary excretion in drops , so the pt.
have enlarged prostate (Ischuria Basic symptom of III stage of BPH) that’s what’s I found through search in
Ukrainian info
B. Dysuria
C. Urinary incontinence
D. Enuresis
E. Pollakiuria

161. A 67 y.o. patient complains of dyspnea, breast pain, common weakness. He has been ill for 5 months.
Objectively: to- 37,3°C, Ps- 96/min. Vocal tremor over the right lung cannot be determined, percussion
sound is dull, breathing cannot be auscultated. In sputum: blood diffusively mixed with mucus. What is the
most probable diagnosis?
A. Lung cancer
B. Bronchoectatic disease
C. Macrofocal pneumonia
D. Focal pulmonary tuberculosis
E. Exudative pleuritis
** Symptoms related to the ca: The growth of the ca and invasion of lung tissues and surrounding tissue may
interfere with breathing, cough, shortness of breath, wheezing, chest pain, and coughing up blood
Dr.H.A.Salman - OdNMU
(hemoptysis). If the ca has invaded nerves, for example, it may cause shoulder painthat travels down the
outside of the arm (called Pancoast syndrome) or paralysis of the vocal cords leading to hoarseness. Invasion
of the esophagus may dysphagia. If a large airway is obstructed, collapse of a portion of the lung may occur
and cause infections (abscesses, pneumonia) in the obstructed area.
162. A 52 year old patient complains about pain in the right part of her chest, dyspnea, cough with a lot of
foul-smelling albuminoid sputum in form of "meat slops". Objectively: the patients condition is grave,
cyanosis is present, breathing rate is 31/min, percussion sound above the right lung is shortened,
auscultation revealed different rales. What is the most probable diagnosis?
A. Lung gangrene  also called necrotizing pneumonia .
B. Lung abscess
C. Multiple bronchiectasis
D. Chronic pneumonia
E. Pleura Empyema

pulmonary gangrene differs by a terminal expression of signs. The state of the patients is critical. The patient
is adynamic, exhausted, with edemas on legs. Dyspnea in rest, hemodynamic disturbances are evident. Dirty-
grey or brown sputum with detrites, pieces of necrotic parenchyma and threads of blood excretes out with the
cough up to 1 l. Early pleural complications are usual and represent with pulmonary bleeding, which may be
profuse. Often it is associated with vital organ dysfunction and loss of consciousness.

163. 3 hours ago a 65-year-old female patient felt sharp abdominal pain irradiating to the right scapula, there
was a single vomiting. She has a history of rheumatoid arthritis. Objectively: pale skin, AP- 100/60 mm Hg,
Ps- 60/min. Abdomen is significantly painful and tense in the epigastrium and right subcostal areat, there
are positive symptoms of parietal peritoneum irritation over the right costal arch, that is tympanitis. What is
the tactics of an emergency doctor?
A. To take the patient to the surgical hospital
B. To perform gastric lavage
C. To take the patient to the rheumatological department
D. To inject spasmolytics
E. To inject pain-killers and watch the dynamics

164. A 15 year old patient suffers from headache, nasal haemorrhages, sense of lower extremity coldness.
Objectively: muscles of shoulder girdle are developed, lower extremities are hypotrophied. Pulsation on the
pedal and femoral arteries is sharply dampened. AP is 150/90 mm Hg, 90/60 on the legs. Systolic murmur
can be auscultated above carotid arteries. What is the most probable diagnosis?
A. Aorta coarctation
B. Aorta aneurism
C. Aortal insufficiency
D. Coarctation of pulmonary artery
E. Aortal stenosis
Dr.H.A.Salman - OdNMU

165. A 15 y.o. patient has developmental lag, periodical skin yellowing. Objectively: spleen is 16x12x10 cm
large, holecistolithiasis, skin ulcer on the lower third of his left crus. Blood count: erythrocytes - 3,0×1012/L,
Hb- 90 g/L, C.I.- 1,0, microspherocytosis, reticulocytosis. Blood bilirubin – 56 mmole/L, indirect bilirubin -
38 mmole/L. Choose the way of treatment:
A. Splenectomy  this disease is Hereditary microspherocytosis.
B. Omentohepatopexy
C. Portocaval anastomosis
D. Spleen transplantation
E. Omentosplenopexy
Dr.H.A.Salman - OdNMU
166. A 72-year-old patient complains of pain and bleeding during defecation. Digital rectal investigation
revealed a tumour of the anal canal. After verification of the diagnosis the patient was diagnosed with
squamous cell carcinoma. The secondary (metastatic) tumour will be most probably found in:
A. Lungs
B. Brain
C. Pelvic bones
D. Liver
E. Mediastinum

167. A patient with frostbite of both feet was delivered to the admission ward. What actions should be taken?
A. To apply a bandage, to introduce vasodilating medications
B. To administer cardiac medications
C. To rub feet with snow
D. To apply an alcohol compress
E. To put feet into hot water

168. 10 years ago a patient had a fracture in the middle one-third of his left femoral bone, and during the last
7 years he has been having acute inflammation in the area of old fracture accompanied by formation of a
fistula through which some pus with small fragments of bone tissue is discharged. After a time the fistula
closes. What complication of the fracture is it?
A. Chronic osteomyelitis
B. False joint
C. Trophic ulcer
D. Bone tuberculosis
E. Soft tissue phlegmon

169. A 21-year-old male patient got a deep cut wound in his right thigh. In the emergency room a surgeon on
duty performed primary debridement of the wound and primary wound closure with a suture. After 4 days,
there appeared pain, redness, edema, purulent discharge from the wound gap, body temperature rose up to
39°C. What kind of wound complication can you think of and what actions should be taken?
A. Wound abscess, remove the sutures and drain the wound
B. Tetanus, active-passive immunization against tetanus
C. Lymphangitis, apply a hot compress
D. Infiltration, apply a hot compress
E. Erysipelas, prescribe antibiotics
Dr.H.A.Salman - OdNMU
170. A patient has a stab wound on his right foot. On the fourth day after injury the patients body
temperature rose up to 38°C, inguinal lymph nodes became enlarged and painful, skin over them reddened.
What complication might be suspected?
A. Lymphadenitis is the inflammation or enlargement of a lymph node
B. Phlegmon
C. Erysipelas
D. Tetanus
E. Lymphangitis inflammation of lymph vessels.

171. During a surgery on a 30-year-old patient a dark ileo-ileal conglomerate was discovered, the
intussusceptum intestine was considered to be unviable. The intussuscipiens intestine was dilated to 7-8 cm,
swollen, full of intestinal contents and gases. What pathology led to the surgery?
A. Invagination (combined) obstruction
B. Spastic obstruction
C. Obturation obstruction
D. Strangulation obstruction
E. Paralytic obstruction
** conglomerate = mass

172. A 52 year old patient complains about headache, weakness of his upper left extremity. Neurological
symptoms become more intense during physical stress of the left extremity. Pulsation on the arteries of the
left extremity is sharply dampened but it remains unchanged on the carotid arteries. What is the most
probable diagnosis?
A. Occlusion of the left subclavicular artery, steal syndrome
B. Raynauds syndrome
C. Occlusion of brachiocephalic trunk
D. Takayasus disease
E. Thoracal outlet syndrome
Dr.H.A.Salman - OdNMU

173. A 43-year-old patient had been admitted to a hospital with clinical presentations of ischiorectal
periproctitis. On the 12th day of treatment the patients condition deteriorated: there was an increase in the
rate of intoxication and hepatic failure, the body temperature became hectic, AP was 100/60 mm Hg. USI of
liver revealed a hydrophilic formation. In blood: WBCs - 19,6×109/l, RBCs.- 3,0×1012/l, Hb- 98 g/l. What
complication was developed?
A. Liver abscess
B. Pylephlebitis
C. Liver necrosis
D. Budd-Chiari syndrome
E. Hepatic cyst

174. A 57 year old patient abruptly lost the sight of one eye. Examination revealed that his sight made up 0,02
excentrically, eye fundus has haemorrhages of different forms and sizes ("squashed tomato" symptom).
Disc of optic nerve is hyperemic. In anamnesis general vascular pathology is recorded. Direct-acting
anticoagulants were administered. What is the most probable diagnosis?
A. Thrombosis of central vein of retina
B. Hypertensive angiopathy
C. Embolism of central artery of retina
D. Diabetic retinopathy
E. Hypertensive angioneuropathy
Dr.H.A.Salman - OdNMU

175. Half an hour ago a 67-year-old patient with a hernia picked up a heavy object, which caused acute pain in
the region of hernia protrusion, the hernia couldnt be reduced. Objectively: the hernia in the right inguinal
region is roundish, tight, moderately painful; during palpation it was reduced back to the cavity, the pain was
gone. Specify the further medical tactics:
A. Inpatient surveillance  for follow up , reducible hernia .
B. Immediate laparotomy
C. Immediate hernioplasty
D. Planned hernioplasty a month later
E. Planned hernioplasty a year later

176. A 55 year old patient felt suddenly sick in a hospital corridor, he was immediately examined by a doctor.
Examination revealed that the patients skin was pale, autonomous respiration was absent, pulse on carotid
arteries couldnt be felt, pupils were mydriatic. What action should be taken at the beginning of cardiac
resuscitation?
A. Precordial thump
B. Defibrillation
C. Closed-chest cardiac massage
D. Mouth-to-mouth ventilation
E. Restoration of airway patency
Dr.H.A.Salman - OdNMU
177. A 30-year-old patient has been admitted to the intensive care unit for multiple bee stings. The skin is
covered with cold sweat. The pulse is felt just on the carotid arteries, 110 bpm, respiration is 24/min,
rhythmic, weakened. Which drug should be given in the first place?
A. Adrenalin hydrochloride intravenously
B. Adrenalin hydrochloride intramuscularly
C. Prednisolone intravenously
D. Dopamine intravenously
E. Tavegil intravenously

178. A 25-year-old victim of a road accident complains of chest pain, dyspnea. Objectively: the patient is in a
grave condition, Ps- 120/min, AP- 90/70 mm Hg. There is pathological mobility of fragments of III-V ribs on
the right. Percussion reveals a box sound over the right lung, breathing sounds cannot be auscultated on the
right. What examination should be administered in the first place?
A. X-ray of chest organs
B. Pleural puncture
C. Bronchoscopy
D. USI of chest organs
E. Thoracoscopy

179. After the pneumatic dilatation of oesophageal structure a patient developed acute retrosternal pain
getting worse when throwing the head back and swallowing. Objectively: dilatation of the neck veins,
dropped beat pulse, signs of purulent intoxication, oliguria, emphysema of the upper portion of chest. What
disease can be suspected?
A. Suppurative mediastinitis  inflammation of the mediastinum – pus form
B. Pleural empyema
C. Spontaneous pneumothorax
D. Acute myocardial infarction
E. Thrombosis of the superior vena cava

180. A 6-year-old girl drank some coloured fizzy drink which gave her a feeling of pressure in the throat. 30
minutes later the childs lips got swollen, then edema gradually spread over the whole face, laryngeal
breathing became difficult. The child is excited. Ps- 120/min, breathing rate - 28/min, breathing is noisy,
indrawing of intercostal spaces is observed. What basic aid is most appropriate for the restoration of
laryngeal breathing?
A. Corticosteroids  pt got hypersensitivity reaction .
B. Antibacterial drugs
C. Conicotomy
D. Sedative drugs
E. Tracheostomy
Dr.H.A.Salman - OdNMU
181. An 8-month-old baby has had problems with nasal breathing and muco-purulent discharge from the
nose for a week. Examination reveals a rhinedema, muco-purulent discharges from the middle nasal meatus
as well as on the back of pharynx. What disease are these symptoms most typical for?
A. Ethmoiditis  inflammation of ethmoid sinus , it opening to nose through middle nasal meatus.
B. Sphenoiditis  sphenoid sinus inflammation
C. Frontitis  frontal sinus , it open to nose through upper nasal meatus . (superior)
D. Hemisinusitis  whole side of the sinus (Rt half or Lt half)
E. Maxillary sinusitis  maxillary sinus , open to nose through lower nasal meatus (inferior)

182. A 28-year-old female patient has been admitted to a hospital. She states to be ill for 12 years. On
examination she has been diagnosed with bronchiectasis with affection of the left lower lobe of lung. What is
the optimal treatment tactics for this patient?
A. Left lower lobectomy
B. Bronchopulmonary lavage
C. Antibiotic therpy
D. Left-sided pneumoectomy
E. Active drainage of the left pleural cavity

183. 4 weeks after myocardial infarction a 56-year-old patient developed acute heart pain, pronounced
dyspnea. Objectively: the patients condition is extremely grave, there is marked cyanosis of face, swelling and
throbbing of neck veins, peripheral pulse is absent, the carotid artery pulse is rhythmic, 130 bpm, AP is
60/20 mm Hg. Auscultation of heart reveals extremely muffled sounds, percussion reveals heart border
extension in both directions. What is the optimal treatment tactics for this patient?
A. Pericardiocentesis and immediate thoracotomy  puncture of pericardium (heart sheath) and drainage of
fluid (pericardial effusion) then open Thorax
B. Conservative treatment, infusion of adrenomimetics
C. Pleural cavity drainage
D. Oxygen inhalation
E. Puncture of the pleural cavity on the left
Dr.H.A.Salman - OdNMU

184. Esophagus wall of a 72 year old patient with severe concomitant pathology was injured during urgent
fibroesophagogastroscopy. This resulted in progressing of acute respiratory failure and collapse of the left
lung. What aid should be rendered?
A. Drainage of pleural cavity by Bullaux method, mediastinum drainage, antibacterial therapy  this mean
chest tube , drainage , then antibiotic , sorry google don’t know about this method, what I find Buelaus drainage
of pleural cavity ^-_-^ . just keep.
B. Endoscopic closure of esophagus wound, drainage
C. Left-sided thoracotomy, closure of esophagus and mediastinum wound
D. Buelaus drainage of pleural cavity, antibacterial therapy  TAKE ATTENTION !! Not this
E. Left-sided thoracotomy, closure of esophagus wound

185. A 17-year-old patient complains of pain in the area of the left knee joint. Soft tissues of thigh in the
affected region are infiltrated, joint function is limited. X-ray picture of the distal metaepiphysis of the left
femur shows a destruction focus with periosteum detachment and Codmans triangle found at the defect
border in the bone cortex. X-ray of chest reveals multiple small focal metastases. What treatment is
indicated?
A. Palliative chemotherapy  this pt. had Osteogenic carcinoma with metastasis , because of this they
recommended palliative therapy just to control disease and treat symptoms .
B. Radioiodine therapy
C. Disarticulation of the lower extremity
D. Amputation of the lower extremity
E. Distance gamma-ray therapy
Dr.H.A.Salman - OdNMU

186. A 35-year-old victim of a road accident has got an injury of the right side of his chest. Objectively:
respiration rate - 28-30/min, respiration is shallow, restricted respiratory excursion and acrocyanosis are
present. Ps- 110 bpm, AP- 90/60 mm Hg. Respiratory sounds over the right lung cannot be auscultated.
Chest radiograph shows fractures of the VI -VII ribs on the right, the right pleural cavity contains both air
and fluid, with the fluid at about the level of the V rib, the shadow of the mediastinum is displaced to the left.
What first aid should be provided to the victim?
A. Puncture of the pleural cavity  Tension Pneumothorax + pleural effusion ?
B. Vagosympathetic blockade
C. Antibiotic administration
D. Artificial ventilation of lungs
E. Urgent thoracotomy

187. On the first day after a surgery for diffuse toxic goiter a patient developed difficulty breathing, cold
sweats, weakness. Objectively: pale skin, body temperature - 38,5°C, RR - 25/min, Ps- 110/min, AP- 90/60
mm Hg. What early postoperative complication occurred in the patient?
A. Thyrotoxic crisis
B. Compression of the trachea by the hematoma
C. Postoperative tetany
D. Hypothyroid crisis
E. Acute thyroiditis
Dr.H.A.Salman - OdNMU
188. A 20-year-old patient complains of pain in the left lumbar region, arterial pressure rise up to 160/110
mm Hg. USI revealed that the structure and size of the right kidney were within age norms, there were signs
of 3 degree hydronephrotic transformation of the left kidney. Doppler examination revealed an additional
artery running to the lower pole of the kidney. Excretory urogram shows a narrowing in the region of
ureteropelvic junction. Specify the therapeutic tactics:
A. Surgical intervention  this case called obstructive uropathy (e.g: renal calculi) , for correction of the narrow
pelvi-uretric junction and to reduce hydronephrosis , also for checking the extra renal artery , because pt have
HTN , so maybe there is Renal artery stenosis RAS
B. Kidney catheterization
C. Administration of beta-blockers
D. Administration of spasmolytics
E. Administration of ACE inhibitors

189. A 49-year-old male patient consulted a doctor about difficult swallowing, voice hoarseness, weight loss.
These symptoms have been gradually progressing for the last 3 months. Objectively: the patient is
exhausted, supraclavicular lymph nodes are enlarged. Esophagoscopy revealed no esophageal pathology.
Which of the following studies is most appropriate in this case?
A. Computed tomography of chest and mediastinum  there is signs of tumor in mediastinum (may be laryngeal
CA?) , as esophagoscopy not showing pathology , may be this tumor from outside , casuing compressing on both
esophagus and larynx , anyhow , need CT with contrast to exclude tumors
B. X-ray of lungs  not specific for this pt. , due to no specific lung symptoms.
C. Radioisotope investigation of chest  not showing soft tissues clearly especially in mediastinum.
D. Ultrasound investigation of mediastinum  not useful for mediastinum except for pleural puncture.
E. Multiplanar imaging of esophagus  not so informative , due to normal esophagoscope.

190. A 36-year-old male patient complains of having headache, obstructed nasal breathing, purulent nasal
discharge for 2 weeks. A month before, he had his right maxillary premolar filled. Radiolography revealed an
intense opacity of the right maxillary sinus. Diagnostic puncture revealed a large amount of thick malodorous
crumbly pus. What is the most likely diagnosis?
A. Chronic suppurative odontogenic sinusitis
B. Chronic purulent sinusitis
C. Tumor of the maxillary sinus
D. Chronic atrophic sinusitis
E. Acute purulent sinusitis
Dr.H.A.Salman - OdNMU
191. A patient with autoimmune thyroiditis accompanied by multinodular goiter underwent the right lobe
ectomy and subtotal resection of the left lobe. What drug should be administered to prevent postoperative
hypothyroidism?
A. L-thyroxine  as the pt. had autoimmune thyroiditis (hashimato’s) so he already had hypothyroidism and
then he did thyroidectomy (sub total resection) sure he need Thyroxin to prevent post operation hypothyroidism
.
B. Iodomarin  potassium iodide (Oral anti-thyroid agent) , treatment of hyperthyroidism and
thyrotoxicosis and preoperatively to induce thyroid involution
C. Insulin
D. Lithium drugs  anti mania (bipolar disorder)
E. Merkazolil  methimazole – antithyroid drug.

192. A hospital admitted a patient with coarse breathing (obstructed inspiration), skin cyanosis, tachycardia
and arterial hypertension. He has a histrory of bronchial asthma. An hour ago he was having salbutamol
inhalation and forgot to remove a cap that was aspired while taking a deep breath. What measures should the
doctor take?
A. To perform the Heimlich manoever
B. To use an inhalation of a2-adrenoceptor agonist
C. To make a subcutaneous injection of dexamethasone
D. To perform conicotomy immediately
E. To send for an anesthesiologist and wait for him

193. An emergency physician arrived to provide medical care for a hangman taken out of the loop by his
relatives. The doctor revealed no pulse in the carotid arteries, lack of consciousness, spontaneous breathing
and corneal reflexes; cadaver spots on the back and posterior parts of extremities. A person can be declared
dead if the following sign is present:
A. Cadaver spots  also called livor mortis. , cadaver mean corpse
Dr.H.A.Salman - OdNMU
B. Lack of corneal reflexes
C. Unconsciousness
D. Pulselessness
E. Lack of spontaneous breathing

194. ** same up 
An emergency physician arrived to provide medical care for a hangman taken out of the loop by his relatives.
The doctor revealed no pulse in the carotid arteries, absence of consciousness, spontaneous breathing and
corneal reflexes; cadaver spots on the back and posterior parts of extremities. A person can be declared dead
if the following sign is present:
A. Cadaver spots
B. Pulselessness
C. Unconsciousness
D. Absence of spontaneous breathing
E. Absence of corneal reflexes

195. A 69-year-old male patient has been hospitalized with hypothermia. Objectively: the patient is pale, has
shallow breathing. AP is 100/60 mm Hg, Ps is 60/min. Palpation of the abdomen and chest reveals no
pathological signs. The body temperature is of 34,8°C. The patients breath smells of alcohol. Give treatment
recommendations:
A. Warm bath + intravenous administration of warm solutions
B. Antishock therapy
C. Forced diuresis
D. Body warming with hot-water bags
E. Rubbing with alcohol and snow

196. A patient undergoing treatment for the left-sided destructive pneumonia presents with deterioration of
his general condition, progressing dyspnea, cyanosis. Objectively: the left side of chest is not involved in
respiration, breathing sounds cannot be auscultated. Radiograph shows a shadow reaching the 5 rib with a
horizontal fluid level and a radiolucency above it, the mediastinum is displaced to the right. What is the
medical tactics?
A. Thoracostomy  this pt had hydropneumothorax (Air-Fluid level) make opening .
B. Infusion and antibacterial therapy
C. Emergency bronchoscopy
D. Open thoracotomy  don’t mix / this mean open chest
E. Endotracheal intubation
Dr.H.A.Salman - OdNMU

197. A 26-year-old male patient consulted a doctor abut sore throat, fever up to 38,2°C. A week before, the
patient had quinsy, didnt follow medical recommendations. On examination, the patient had forced position
of his head, trismus of chewing muscles. Left peritonsillar region is markedly hyperemic, swollen. What is the
provisional diagnosis?
A. Left-sided peritonsillar abscess  also called quinsy pus due to an infection behind the tonsils. it is most
often a complication of tonsillitis. The bacteria involved are similar to those that cause strep throat. Streptococcal
bacteria most commonly cause an infection in the soft tissue around the tonsils (usually just on one side). , they
already answered the question above 
B. Phlegmonous tonsillitis
C. Tonsil tumour
D. Pharyngeal diphtheria
E. Meningitis
Dr.H.A.Salman - OdNMU
198. A 26-year-old patient consulted a doctor abut sore throat, fever up to 38,2°C. A week ago, the patient
had angina, didnt follow medical recommendations. On examination, the patient had forced position of his
head, trismus of chewing muscles. Left peritonsillar region is markedly hyperemic, swollen. What is the
provisional diagnosis?
A. Left-sided peritonsillar abscess  see Q up 
B. Phlegmonous angina
C. Tonsil tumour
D. Diphtheria of the pharynx
E. Meningitis

199. A 77-year-old male patient complains of inability to urinate, bursting pain above the pubis. The patient
developed acute condition 12 hours before. Objectively: full urinary bladder is palpable above the pubis.
Rectal prostate is enlarged, dense and elastic, well-defined, with no nodes. Interlobular sulcus is
distinct. Ultrasonography results: prostate volume is 120 cm3, it projects into the bladder cavity, has
homogeneous parenchyma. Prostate-specific antigen rate is of 5 ng/ml. What is the most likely disease that
caused acute urinary retention?
A. Prostatic hyperplasia  take attention to PSA 5 (normal up tp 4 ) so not increased , while in prostate CA will
be highly increased , therefore this called benign prostatic hyperplasia BPH
B. Sclerosis of the prostate  this mean calcification , can see only through examination like US, CT
C. Acute prostatitis  no signs of infection.
D. Tuberculosis of the prostate
E. Prostate carcinoma  no symptoms of CA like subfebrile T, loosing weight m very high PSA …….

200. A 4 month old child was admitted to a surgical department 8 hours after the first attack of anxiety. The
attacks happen every 10 minutes and last for 2-3 minutes, there was a lso one-time vomiting. Objectively: the
childs condition is grave. Abdomen is soft, palpation reveals a tumour-like formation in the right iliac area.
After rectal examination the doctors finger was stained with blood. What is the most probable diagnosis?
A. Ileocecal invagination  also called intussusceptions.
B. Gastrointestinal haemorrhage
C. Helminthic invasion
D. Pylorostenosis
E. Wilms tumour
** pay attension to this child 4 months have attacks of anexity  愚蠢的問題. don’t translate
Dr.H.A.Salman - OdNMU
201. A 3-year-old male patient consulted a family doctor 2 months after he had been operated for an open
fracture of brachial bone. Objectively: the patients condition is satisfactory, in the region of the operative
wound there is a fistula with some purulent discharge, redness, fluctuation. X-ray picture shows brachial
bone destruction with sequestra. What complication arose in the postoperative period?
A. Posttraumatic osteomyelitis
B. Wound abscess
C. Hematogenic osteomyelitis
D. Posttraumatic phlegmon
E. Suture sinus

202. A 40-year-old female patient complains of having a bulge on the anterior surface of neck for 5 years.
Objectively: Ps- 72 bpm, arterial pressure - 110/70 mm Hg, in the right lobe of thyroid gland palpation
reveals a mobile 4x2 cm node, the left lobe is not palpable, the basal metabolic rate is 6%. What is the most
likely diagnosis?
A. Nodular euthyroid goiter
B. Mixed euthyroid goiter
C. The median cervical cyst
D. Nodular hyperthyroid goiter  no symptoms of Hyperthyroidism.
E. Riedels thyroiditis
** gland function looks normal  Euthyroid .
Noular goiter  it can be single node (nodular) or multiple (multinodular )
- accorging to function  it can be  hyperthyroidism , normal (euthyroid) , or  hypothyroidism.
** Basal metabolic rate (BMR) is the rate at which the body spends energy (Calories) with no activity .
Several factors affect BMR, including age, body size, sex, body temperature, fasting/starvation, stress,
menstruation, and thyroid function.
**Thyroxin is a key BMR regulator; the more thyroxine produced in the body, the higher the BMR.

203. Forensic medical expertise of corpse of a newborn revealed: body weight 3500 g, body length 50 cm, the
umbilical cord was smooth, moist, glossy, without any signs of drying. Hydrostatic tests were positive. The
test results are the evidence of:
A. Live birth  mean child born alive then died , (normal Weight, Height) .
B. Primary atelectasis
C. Stillbirth
D. Secondary atelectasis
E. Hyaline membrane disease
Dr.H.A.Salman - OdNMU
204. A week ago a 65-year-old patient suffered an acute myocardial infarction, his general condition
deteriorated: he complains of dyspnea at rest, pronounced weakness. Objectively: edema of the lower
extremities, ascites is present. Heart borders are extended, paradoxical pulse is 2 cm displaced from the apex
beat to the left. What is the most likely diagnosis?
A. Acute cardiac aneurysm
B. Acute pericarditis
C. Recurrent myocardial infarction
D. Cardiosclerotic aneurysm
E. Pulmonary embolism

205. A 36-year-old injured has been taken to the emergency station with open tibial fractures. Examination
reveals bleeding: pulsating blood spurts out of the wound. What medical manipulations are required at this
stage of medical care?
A. Apply a tourniquet on the thigh proximal to the source of bleeding and transport the patient to the operating
room arterial bleeding , emergency need to stop bleeding after tourniquet.
B. Transport the patient to the operating room
C. Apply a tourniquet on the thigh distal to the source of bleeding and transport
the patient to the operating room
D. Immobilize the fracture and transport the patient to the operating room
E. Stop the bleeding by a compressive bandage and transport the patient to the
operating room

206. A 75-year-old male patient complains of slight pain in the right iliac region. The abdominal pain arose 6
days ago and was accompanied by nausea. Surgical examination revealed moist tongue, Ps- 76 bpm. AP-
130/80 mm Hg. Abdomen was soft, slightly painful in the right iliac region on deep palpation, the symptoms
of the peritoneum irritation were doubtful. In blood: RBCs - 4,0×1012/l, Hb- 135 g/l, WBCs - 9,5×109/l, stab
neutrophils - 5%, segmentonuclear - 52%, lymphocytes - 38%, monocytes - 5%, ESR - 20 mm/h. Specify the
doctors further tactics:
A. Emergency operation for acute appendicitis
B. Refer the patient to a district therapist
C. Administration of additional examination: abdominal ultrasound, x-ray
contrast study of the gastrointestinal tract
D. Hospitalization, dynamic surveillance
E. Send the patient home
Dr.H.A.Salman - OdNMU
207. A 56-year-old patient was undergoing a surgery for suture repair of perforated ulcer. During the
operation the cardiomonitor registered ventricular fibrillation. The first-priority measure should be:
A. Electrical defibrillation
B. Injection of calcium chloride
C. Injection of lidocaine
D. Injection of adrenalin
E. Injection of atropine

208. A 78-year-old patient complains of severe pain in the lower abdomen, perineum and rectum; intense
urge to urinate and inability to urinate without assistance. Abdomen palpation reveals a moderately painful
globular formation above the pubis, there is percussion dullness over the formation. What is the most likely
diagnosis?
A. Acute urinary retention  Absence of urine discharge at intensive appeals on urination
B. Chronic incomplete urinary retention
C. Cystitis
D. Paradoxical ischuria
E. Chronic urinary retention
** can review Q 160 . same Q different answer
they only thing that I found that they choose Paradoxical ischuria when Incontinence of urine at the retention
of urination or dripping , when didn’t mansion it choose acute urinary retention
acutely both of them same meaning why they put them together !!! not sure 
another thing is paradoxical Ischuria is Basic symptom of III stage of BPH

209. A 24-year-old patient had been delivered to the thoracic department with a chest injury, a fracture of the
IV, V, VI ribs on the right. Plan radiography shows the fluid level in the pleural cavity reaching the III rib on
the right. Puncture blood contained clots. What is the optimal treatment tactics?
A. Emergency thoracotomy indication: 1- Penetrating thoracic injury , 2- Haemodynamic instability (SPB
<70mmHg despite vigorous fluid resuscitation) 3- Traumatic arrest with previously witnessed cardiac activity
(prehospital or ED) { this pt. have hemothorax }
B. Thoracentesis and thoracostomy
C. Pleural puncture
D. Hemostatic therapy
E. Medical thoracoscopy
Dr.H.A.Salman - OdNMU
Additional Q
210 - A 2-year-old boy has been admitted to the casualty department for the pain and deformity of his
right thigh. Radiograph shows a femoral fracture with longitudinal displacement. What method of
treatment is indicated for the patient?
A-Schede-type vertical suspension  see down 
B Closed reduction
C Intramedullary osteosynthesis
D Use of Ilizarov apparatus
E Periosteal osteosynthesis

211- A 57-year-old female patient complains of intense pain in the right hypochondrium irradiating to
the right supraclavicular region. Skin and sclerae are icteric. There is tension and tenderness in the right
hypochondrium on palpation. Body temperature is 38,8oC. Blood test results: WBC- 11,2*109/l, total bilirubin
- 112 mmol/l (conjugated - 86 mmol/l, unconjugated - 26 mmol/l). What is the most likely diagnosis?
A Cholangitis  inflammation/infection of the biliary ducts (Charcot triad ) pain, fever, jaundice , pay
attention to the conjugated (direct bilirubin) 
B Acute pancreatitis
C Acute appendicitis
D Pancreatic tumor
E Perforated duodenal ulcer

212- A 3-year-old girl has had an increase in body temperature up to 38,5oC for four days. The child
refuses to eat. Over the last two days, nose and mouth breathing has become difficult. Mesopharyngoscopy
reveals hyperthermia and enlargement of tonsils, as well as hyperemia and bulging of the posterior wall of the
oropharynx, which significantly narrows the oropharyngeal lumen. What complication of quinsy occurred in
the patient?
A- Retropharyngeal abscess  Abscess formation behind the pharynx.
B Paratonsillar abscess
C Parapharyngeal abscess
D Phlegmon of the mouth floor
E Laryngostenosis
Dr.H.A.Salman - OdNMU
213- A 22-year-old female patient has been delivered by an ambulance team to a surgical clinic with
symptoms of acute intestinal obstruction. It is known from the past history that 2 years ago she was
operated for acute destructive appendicitis. For two years, she has repeatedly complained of bloating and
abdominal pain. Which of the following etiological factors has led to the intestinal obstruction in the patient?
A- Abdominal adhesions
B Ileal diverticulum
C Dolichosigma
D Diet violation
E Helminthiasis

214- A 39-year-old female patient complains of rapid fatigability, drowsiness, dry skin, hair loss, swelling of
the face. A month ago, she underwent a surgery for thyrotoxicosis. The patient has the following gland
dysfunction:
A Thyroid (hypothyroidism), due to inadequate operative technique
B Pituitary, due to a tumor
C Adrenal
D Parathyroid, due to the gland removal during surgery
E Ovarian, due to a tumor

215- During dressing of a poorly-granulating wound Pseudomonas aeruginosa infection was revealed. What
medication would be optimal for the wound debridement?
A- Boric acid solution  antiseptic and some antibacterial activity.
B Biogenic stimulators
C Sulfonamides
D Salicylic acid
E Antibiotics

216- A 42-year-old female patient complains of a dull pain in her left side, low-grade fever, accelerated
painful urination in small portions. These presentations have been observed for three years. For a long time,
the patient has had cystitis with frequent exacerbations, there is pulmonary tuberculosis in the past history.
Urinalysis results: microscopic hematuria, leukocyturia. What is the most likely provisional diagnosis?
A- Renal tuberculosis
B Urolithiasis
C Chronic pyelonephritis
D Renal tumor
E Chronic cystitis

217- A 44-year-old male patient complains of severe non-localized abdominal pain, pain in the right shoulder
girdle, repeated vomiting, red urine. The onset of the disease is associated with alcohol consumption. The
face is hyperemic. AP- 70/40 mm Hg. Abdominal radiography reveals no pathological shadows.
Hemodiastase is 54 mg/h/l. Prothrombin is 46%. What is the provisional diagnosis?
A- Acute pancreatitis
B Acute myocardial infarction
C Perforated gastric ulcer
D Thrombosis of mesenteric vessels
E Aneurysm of the abdominal aorta
Dr.H.A.Salman - OdNMU
218- A 66-year-old female patient has been admitted to a hospital for massive gross hematuria with
release of shapeless blood clots, frequent painful urination. The patient also reports a moderate weight loss
within 3-4 months. Gross hematuria that was not accompanied by pain and dysuria first occurred three
months ago for no apparent reason, and after a few days the bleeding subsided independently. What is the
most likely diagnosis?
A- Bladder tumor
B Urolithiasis
C Renal tumor
D Chronic cystitis
E Acute cystitis

219- A 40-year-old male patient has had heaviness in the epigastric region for the last 6 months. He has not
undergone any examinations. The night before, he abused vodka. In the morning there was vomiting, and 30
minutes after physical activity the patient experienced dizziness and profuse hematemesis. What pathology
should be suspected in the first place?
A- Mallory-Weis's syndrome
B Menetrier's disease
C Gastric ulcer
D Perforated ulcer
E Zollinger-Ellison syndrome

220- same Q 160 -- 221-same Q 167


222- It has been suspected that a newborn has congenital diaphragmatic hernia (asphyctic incarceration).
What study will allow to confirm the diagnosis?
A- Plan radiography of the chest cavity
B Plan abdominal radiography
C Irrigography
Dr.H.A.Salman - OdNMU
D Pneumoirrigoscopy
E Fibroesophagogastroduodenoscopy

223- A 29-year-old unconscious patient has been delivered to a hospital. Objectively: skin and mucous
membranes are pale, cyanotic, breath sounds are dramatically diminished on the right and cannot be
auscultated in the lower parts, at the level of the 6 rib along the anterior axillary line there is a wound hole
with moderate bleeding and passage of air during inspiration. Radiography reveals a bullet in the pleural
cavity. What is the medical tactics of choice?
A- Emergency thoracotomy  review Q 209 , same
B Thoracoscopy with removal of bullet
C Converting a tension pneumothorax into a simple (open) pneumothorax
D Thoracostomy
E Tight bandage on a wound

224- As a result of a road accident a 45-year-old male patient got multiple injuries, namely closed fractures
of the right humerus and the left antebrachial bones with a displacement of bone fragments, a closed blunt
abdominal injury. The patient was delivered to the emergency department 30 minutes after the injury.
Objectively: the skin is pale. AP- 90/20 mm Hg, there is pain and deformation at the fracture sites. Abdomen
is tense, palpation causes severe pain, there is rebound tenderness (positive Blumberg's sign). What is the
treatment tactics of choice?
A- Urgent diagostic laparotomy  this pt may had appendix injury.
B Infusion therapy to stabilize blood pressure
C Fracture immobilization, analgesia
D Local anesthetic blockade of fractures
E Additional tests to specify the diagnosis
225- A 23-year-old male patient consulted a doctor about pain occurring in the lower third of the thigh
with weight bearing activities and unloaded. The patient denies any injuries to the region.Objectively: the
skin is of normal color, deep palpation reveals pastosity and tenderness, movements of the knee joint are
limited. Radiograph of the distal femoral metaepiphyseal region shows a zone of degradation and spicules. In
blood: immature cells are present, there are no signs of inflammation. What is the most likely diagnosis?
A- Osteosarcoma  cancer of bone
B Hyperparathyroid dystrophy
C Chronic osteomyelitis
D Multiple myeloma
E Marble bone disease
** pastosity = swelling/ edema
Dr.H.A.Salman - OdNMU
226- A 47-year-old female patient complains of having pain on swallowing and difficult passing of solid food
for two months. The patient has taken to the liquid and semi-liquid food. During the last week the liquid
food has barely passed through. General condition is satisfactory, the patient is undernourished, the appetite
is preserved, there is a fear of eating. What is the provisional diagnosis?
A- Esophageal carcinoma  review Q 1
B Esophageal stricture
C Esophageal foreign body
D Esophageal varices
E Esophageal achalasia
227- A 5-year-old boy has a history of repeated pneumonia, frequent acute respiratory viral diseases.
Objectively: exertional dyspnea, minor fatigabilty. There is a systolic murmur having its epicenter in the IV
intercostal space on the left. Left relative dullness is found along the midclavicular line. According to the
findings of instrumental methods of examination (electrocardiography, echocardiography), the patient has
been diagnosed with ventricular septal defect, subcompensation stage. What is the main method of
treatment?
A- Operative therapy  to correct VSD
B Phytotherapy
C Does not require treatment
D Conservative treatment
E Indomethacin
228- A 25-year-old patient works as a tractor driver. Four days ago, he got pain in the left axillary region,
general weakness, fever up to 38oC. He hadn't sought medical helf until a painful solid lump appeared in this
region. Objectively: in the left axilla there is a very painful cone-shaped mass sized 3x2,5 cm, with a
destruction in the center of the pointed vertex. The surrounding skin is hyperemic, there are purulent
discharges. What is the most likely diagnosis?
A- Hydradenitis  inflammation of sweat gland
B Carbuncle
C Furuncle
D Lymphadenitis
E Abscess
229- A 42-year-old male patient has been delivered to a hospital in a grave condition with dyspnea,
cough with expectoration of purulent sputum, fever up to 39,5oC. The first symptoms appeared 3 weeks ago.
Two weeks ago, a local therapist diagnosed him with acute right-sided pneumonia. Over the last 3 days, the
patient's condition deteriorated: there was a progress of dyspnea, weakness, lack of appetite. Chest
radiography confirms a rounded shadow in the lower lobe of the right lung with a horizontal fluid level, the
right sinus is not clearly visualized. What is the most likely diagnosis?
A- Abscess of the right lung  review Q 141
B Acute pleuropneumonia
C Right pulmonary empyema
D Atelectasis of the right lung
E Pleural effusion

230- A 27-year-old male patient consulted a doctor about pain in the lower third of the thigh with weight
bearing activities and unloaded. Two years ago, the patient underwent treatment in the casualty depatment
for the open fracture of the lower third of femur. The fracture healed slowly, the healing process was
accompanied by prulent inflammation. Objectively: edema of the lower third of the thigh, elevated local
temperature. Radiograph shows signs of destruction and sequestration. What is the most likely diagnosis?
A- Chronic post-traumatic osteomyelitis  review Q 201
B Osteosarcoma
C Tuberculosis of femur
D Hematogenous osteomyelitis
E Multiple myeloma
Krok 2 Medicine (англомовний варiант, iноземнi студенти) 2018 рiк 25

Laboratory values
Complete blood count
Laboratory value Normal reference range
Male: 4.3 − 5.9 · 1012 /L
Erythrocyte count
Female: 3.5 − 5.5 · 1012 /L
Male: 135-175 g/L
Hemoglobin, blood
Female: 120-160 g/L
Color index 0.85- 1.05
Reticulocyte count 0.5 - 1.5%
Platelet count 150 − 400 · 109 /L
Leukocyte count 4.0 − 9.0 · 109 /L
Basophils 0 − 0.065 · 109 /L (0-1%)
Eosinophils 0.02 − 0.30 · 109 /L (0.5 - 5.0%)
Band neutrophils 0.04 − 0.30 · 109 /L (1 - 6%)
Segmented neutrophils 2.0 − 5.50 · 109 /L (47 - 72%)
Monocytes 0.09 − 0.60 · 109 /L (3 - 11%)
Lymphocytes 1.2 − 3.0 · 109 /L (19 - 37%)
Erythrocyte sedimentation rate Male: 0 - 15 mm/h
(ESR) Female: 0 - 20 mm/h
Male: 41 - 53%
Hematocrit
Female: 36 - 46%
Biochemical blood analysis (Metabolic panel)
Total proteins 60-78 g/L
Albumin 35-50 g/L (52-65%)
Globulin: 23-35 g/L (35-48%)
α1 -Globulin 2-4 g/L (4.2-7.2%)
α2 -Globulin 5-9 g/L (6.8-12%)
β -Globulin 6-11 g/L (9.3-15%)
γ -Globulin 11-15 g/L (15-19%)
Immunoglobulins:
IgD 0 - 0.15 g/L
IgG 6.5-15 g/L
IgM 0.4-3.45 g/L
IgA 0.76-3.90 g/L
IgE 0-380 kU/L
Bilirubin:
Total 2-17 mcmol/L
Indirect (unconjugated) 2-17 mcmol/L
Direct (conjugated) 0-5 mcmol/L
Triglycerides 0.59-1.77 mmol/L
Total cholesterol 3.9-6.2 mmol/L
Lipoproteins:
high-density lipoproteins (β -Lipoproteins) <4.2 mmol/L
low-density lipoproteins (α-Lipoproteins) 0.8-1.8 mmol/L
Glucose, blood 3.3-6.1 mmol/L (fasting)
Glycated hemoglobin 6%
Iron, blood 9-30 mcmol/L
Potassium, plasma 3.5-5.0 mmol/L
Sodium, plasma 136-145 mmol/L
Calcium, plasma 0.75-2.5 mmol/L
Krok 2 Medicine (англомовний варiант, iноземнi студенти) 2018 рiк 26

Magnesium, plasma 0.75 - 1 mmol/L


Phosphorus (inorganic), serum 0.646-1.292 mmol/L
Chloride, blood 95-105 mmol/L
Urea, serum 3.33 - 8.32 mmol/L
Creatinine 53 - 106 mcmol/L
Male: 0.12-0.38 mcmol/L
Uric acid
Female: 0.12-0.46 mcmol/L
Lactate dehydrogenase (LDH) 45-90 U/L
α-Amylase (diastase), blood 25-125 U/L
Aspartate aminotransferase (AST) 15-40 U/L
Alanine aminotransferase (ALT) 10-40 U/L
Phosphatase (alkaline) 30-115 U/L
Coagulogram
Prothrombin time 80-100% (< 12 seconds)
Fibrinogen 2-4 g/L
Partial thromboplastin time (activated) < 28 seconds
Lee-White coagulation time 5-10 minutes
Bleeding time (Duke method) < 4 minutes
Acid-base balance
pH arterial blood 7.35-7.45
Pco2 arterial blood 33-45 mm Hg
Po2 arterial blood 75 - 105 mm Hg
Bicarbonate, blood (HCO3 ) 22-28 mmol/L
Other blood values
Cortisol, serum 230 -750 nmol/L
Osmolality, serum 275 - 295 mOsmol/kg H2 O
Thyroid-stimulating hormone, serum or plasma 0.5-5 mIU/L
Thyroxine (T4), serum 65-155 nmol/L
Triiodothyronine (T3), serum 1.77 - 2.43 nmol/L
Male: 15-200 mcg/L
Ferritin, serum
Female: 12-150 mcg/L
Thymol turbidity test <5U
C-reactive protein (CRP) <6 mg/dL
Antistreptolysin 0 (ASL0) 250 U
Antistreptohyaluronidase (ASH) 250 U
Urine
Urine specific gravity 1.016-1.022
Nechiporenko’s test:
leukocytes < 2 · 106 /л
erythrocytes < 1 · 106 /л
Proteins, total <45.0 - 75.0 mg/24 h (<0.033 g/L)
Calcium 2.5 - 7.5 mmol/24 h
Male: 97 - 137 mL/min
Creatinine clearance
Female: 88 - 128 mL/min
Oxalate 90 - 445 mcmol/L
Male: 27.7 - 79.7 mcmol/24 h
17-Ketosteroids (17-KS)
Female: 17.4 - 55.4 mcmol/24 h
17-Hydroxycorticosteroids (17-OCS) 5.5 - 27.6 mcmol/24 h
α-Amylase (diastase), urine 28-160 U/L

You might also like